Você está na página 1de 889

1600 FÍSICO-QUÍMICA APLICADA EXERCÍCIOS COMENTADOS - IME – ITA – OLIMPÍADA

1600
FÍSICO QUÍMICA APLICADA

EXERCÍCIOS COMENTADOS

PROFESSOR ALEXANDRE
VARGAS GRILLO

1
1600 FÍSICO-QUÍMICA APLICADA EXERCÍCIOS COMENTADOS - IME – ITA – OLIMPÍADA

INTRODUÇÃO DA OBRA

O principal objetivo de realizar esta obra foi compilar os principais Tópicos da Físico-Química, pesquisando os
exercícios de vestibulares mais difíceis deste país, como o IME (Instituto Militar de Engenharia), o ITA (Instituto
Tecnológico da Aeronáutica), além de exercícios á nível olímpico, tais como OQRJ (Olimpíada de Química do Rio de
janeiro e OBQ (Olimpíada Brasileira de Química).

A obra contempla mil e seiscentos exercícios totalmente gabaritado e comentado, contemplando os mais variados
tópicos da físico-química. Ela pode – e deve – ser utilizada tanto por alunos de ensino médio como também por
alunos de cursos superiores de química e engenharia, que desejam desenvolver um conhecimento mais prático neste
ramo tecnológico.

Este manual de exercícios de Físico-química é um livro auxiliar na forma e-book, que apresenta como maior objetivo
servir e mostrar ao leitor as mais diversas situações que um tópico de físico-química pode apresentar.

Os capítulos abordados estão em consonância com o edital de grandes instituições, como o IME (Instituto Militar de
Engenharia) e ITA (Instituto Tecnólogico da Aeronáutica), além de também apresentar exercícios dos mais distintos
núcleos de Olimpíadas do Brasil como um todo.

Apesar do esforço imenso que coloquei como desafiador, é de se esperar que uma obra dessa dimensão deva conter
inevitavelmente algum erro ou omissão. Diante disso, aceitarei com muito apreço que sejam encaminhadas todas
estas ideias para o seguinte endereço eletrônico: alexandre.grillo@ifrj.edu.br

Espero que este seja útil a todos que querem ou necessitem aprender todos estes fundamentos desta disciplina
fascinante que é a Química.

Janeiro de 2021

Professor Alexandre Vargas Grillo

2
1600 FÍSICO-QUÍMICA APLICADA EXERCÍCIOS COMENTADOS - IME – ITA – OLIMPÍADA

APRESENTAÇÃO DO AUTOR

Alexandre Vargas Grillo é Doutor em Engenharia de Materiais e Processos Químicos e Metalúrgicos pela PUC-Rio,

mestre na mesma área pela própria PUC-Rio e graduado em Engenharia Química pelo também pela PUC-Rio.

Atualmente atua como Professor do Instituto Federal do Rio de Janeiro – IFRJ – Campus Nilópolis, atuando no

departamento de Química, mais especificamente na Físico-Química em Nanotecnologia.

Atua como professor colaborador em pesquisas na área de Síntese de Nanopartículas pelo Departamento de

Engenharia Química e de Materiais – PUC-Rio, além de fazer parte da coordenação das Olimpíadas de Química do

Rio de Janeiro – OQRJ e das turmas Olímpicas de Química do IFRJ – Campus Nilópolis.

AGRADECIMENTOS

Dedico este trabalho primeiramente а Deus e também os meus guias espirituais presentes em todos os momentos da
minha vida.
À minha mãе Estela Vargas Grillo, meus pais Vincenzo Grillo e Jorge Luiz Zaupa e a minha irmã Denise Vargas Grillo.
Dedico “In Memorian” este trabalho ao meu grande mestre, JOÃO ROBERTO DA PACIÊNCIA NABUCO, por ter sido
um grande amigo, pai e também mestre da vida e da química,
Ao meu PAI, MESTRE, AMIGO E ETERNO ORIENTADOR, Professor Dr. Francisco José Moura.
À PUC-Rio, por ter me propiciado um curso de excelência em Engenharia tanto na graduação quanto na pós-
graduação.

Agradecido por tudo e meu muito obrigado.

3
1600 FÍSICO-QUÍMICA APLICADA EXERCÍCIOS COMENTADOS - IME – ITA – OLIMPÍADA

PREFÁCIO

Apaixonado pelo que faz. Foi assim que o notei na ligação em que fui convidado a fazer parte do prefácio
desta obra.
Mesmo sabendo da grande responsabilidade em dar início a um verdadeiro manual para aprovação nos
exames de maior nível de complexidade que conhecemos, entre IME, ITA e Olimpíadas de Química, aceitei o convite
bastante honrado e feliz. Afinal, há alguns anos, enquanto me preparava para o vestibular do IME, tive a oportunidade
de conhecer o autor desta obra enquanto ia muito além de ensinar estequiometria, propriedades coligativas ou
termodinâmica: ele ensinava dedicação, carinho, paixão pelo que faz e muito respeito.
Doutor em Engenharia de Materiais e de Processos Químicos e Metalúrgicos pela PUC-RIO e formado pela
mesma instituição em Engenharia Química, Alexandre Vargas Grillo é um dos responsáveis por aprovações e
premiações de centenas de alunos em concursos e exames de alto nível, como o Instituto Militar de Engenharia, o
Instituto Tecnológico de Aeronáutica e Olimpíadas Nacionais e Internacionais de Química.
Não fosse a leveza ao ministrar suas aulas e a sinceridade em tratar de assuntos importantes, é possível que
a centena de aprovações não contasse com a minha.
Conheci o Grillo, como é chamado carinhosamente, num pré-vestibular em Nova Iguaçu. Lembro de ter me
sentido à vontade para expressar minha dificuldade em Química logo na primeira semana de sua aula. “Mestre, não
sei o que é carbono”, eu disse. As possíveis reações, imagine você, poderiam ser diversas. Mas, ratificando o que
pensava sobre o mestre, seu perfil dentro e fora de sala de aula era de direcionar os estudos para todos os níveis da
Química. Do perito ao não-conheço-carbono: todos se sentiriam abraçados.
Na época, não imaginava que seria o primeiro ano de uma nova vida. Me mudei de Nova Iguaçu, decidi
estudar para o IME e, felizmente, voltamos a nos encontrar. Mais um ano, desta vez como meu professor da turma
IME, teria o Grillo como um mentor e amigo. Entre vários carbonos – estes já íntimos deste que vos escreve – posso
dizer que aprendi com ele que mudança é a parte fundamental do processo. Mesmo resistentes a isto, mudar pode
ser o primeiro passo para solução do problema.
O objetivo desta obra é ser um convite para mudança.
Embora este livro não substitua o autor, você, leitor, ávido por questões desafiadoras (como todo aprovado)
vai ter contato, em breve, com uma seleção única de 3000 questões na área que na minha modéstia opinião é a mais
complexa, que se trata da Físico-Química, com comentários de um especialista na área. Conjunto este que consegue
se comunicar com o leitor, atender suas dúvidas e anseios e conduzi-lo pelo caminho da aprovação – assim como o
autor o faria.
Mude. Respire novos ares. Respire esta obra.
BRYAN MAIA CORREA
PROFESSOR DE MATEMÁTICA
INSTITUTO MILITAR DE ENGENHARIA

4
1600 FÍSICO-QUÍMICA APLICADA EXERCÍCIOS COMENTADOS - IME – ITA – OLIMPÍADA

APRESENTAÇÃO DA OBRA

CAPÍTULO I. ESTEQUIOMETRIA INDUSTRIAL – página 06


CAPÍTULO II. ESTUDO DOS SISTEMAS GASOSOS – GASES IDEAIS E REAIS – página 104
CAPÍTULO III. ESTUDO DAS SOLUÇÕES QUÍMICAS E REAÇÕES ESTEQUIOMÉTRICAS COM SOLUÇÕES
QUÍMICAS – página 203
CAPÍTULO IV. PROPRIEDADES COLIGATIVAS – página 260
CAPÍTULO V. TERMODINÂMICA & TERMOQUÍMICA – página 322
CAPÍTULO VI. – CINÉTICA QUÍMICA – ESTUDO DA VELOCIDADE DAS REAÇÕES QUÍMICAS – página 452
CAPÍTULO VII. ESTUDO DO EQUILÍBRIO QUÍMICO – KC e KP – página 550
CAPÍTULO VIII. ESTUDO DO EQUILÍBRIO IÔNICO – KW, KA, KB, KPS E KH – página 618
CAPÍTULO IX. ELETROQUÍMICA – PILHA E ELETRÓLISE: ÍGNEA E AQUOSA – página 669
CAPÍTULO X. TESTE PARA PRATICAR – página 746
CAPÍTULO XI. TESTE PARA PRATICAR – GABARITO – página 790
CAPÍTULO XII. REFERÊNCIA BIBLIOGRÁFICA – página 872
CAPÍTULO XIII. APÊNDICE – página 876

APÊNDICE A – CONVERSÃO DE UNIDADE


APÊNDICE B – TABELA DE CONSTANTES QUÍMICAS
APÊNDICE C – CONSTANTE DOS GASES (R)
APÊNDICE D – CONSTANTE DE VAN DER WAALS (A E B)
APÊNDICE E - DISTINÇÃO ENTRE PILHA GALVÂNICA E ELETRÓLISE
APÊNDICE F – TABELA DE POTENCIAL PADRÃO
APÊNDICE G – CAPACIDADE CALORÍFICA MÉDIA À PRESSÃO CONSTANTE (CP)
APÊNDICE H - TABELA DE CONSTANTES CRIOSCÓPICAS E EBULIOSCÓPICAS
APÊNDICE I – TABELA PERIÓDICA DOS ELEMENTOS QUÍMICOS

5
1600 FÍSICO-QUÍMICA APLICADA EXERCÍCIOS COMENTADOS - IME – ITA – OLIMPÍADA

CAPÍTULO I

ESTEQUIOMETRIAL
INDUSTRIAL

PROFESSOR ALEXANDRE
VARGAS GRILLO

6
1600 FÍSICO-QUÍMICA APLICADA EXERCÍCIOS COMENTADOS - IME – ITA – OLIMPÍADA

Questão 01 - (GRILLO) A reação de transformação do dióxido de carbono em monóxido de carbono, representada


pela equação a seguir, é muito importante principalmente para processos metalúrgicos e químicos. A reação química
não balanceada é dada a seguir: C(s) + CO2(g) → CO(g). Este processo apresenta uma variação de entalpia na ordem
de (-174) kJ.mol-1 de carbono, caracterizando um processo exotérmico. A partir destas informações, determine os
seguintes itens expostos:
a) a partir de 88 gramas de dióxido de carbono, determine a massa de monóxido de carbono produzido;
b) o volume de monóxido de carbono nas CNTP (condições normais de temperatura e pressão);
c) o volume de monóxido de carbono nas CNATP (condições normais ambientais de temperatura e pressão).

Resolução: Equação química: C(s) + CO2(g) → 2 CO(g)


a) Cálculo da massa de monóxido de carbono: C(s) + CO2(g) → 2 CO(g)
1 mol de CO2(g) ----------------- 2 mol de CO(g)
1 mol x (44 g.mol-1) ----------- 2 mol x (28 g.mol-1)
88 g ------------------------------ mCO
mCO = 112 g

b) Cálculo do volume de monóxido de carbono nas CNTP: C(s) + CO2(g) → 2 CO(g)


1 mol de CO2(g) ---------------- 2 mol de CO(g)
1 mol x (44 g.mol-1) ---------- 2 mol x 22,4 L.mol-1
88 g ----------------------------- VCO
VCO = 89,6 litros de CO

c) Cálculo do volume de monóxido de carbono nas CNATP: C(s) + CO2(g) → 2 CO(g)


1 mol de CO2(g) ----------------- 2 mol de CO(g)
1 mol x (44 g.mol-1) ----------- 2 mol x 24,45 L.mol-1
88 g ------------------------------ VCO
VCO = 97,8 L de CO

Questão 02 - Considere um composto molecular que apresenta a seguinte composição química elementar em
massa/massa (m/m): 46,6% de C; 4,4% de H; 31,1% de N e 17,7% de O. Deseja-se estudar a combustão completa
de 1000 g do referido material, tendo-se como únicos produtos, dióxido de carbono, vapor d´água e dióxido de
nitrogênio. Determine o volume total de gás gerado, para uma combustão controlada realizada a 1 atm e 227oC.
Resolução: Primeiramente será necessário determinar a fórmula molecular do composto orgânico de interesse.
Considerando como base de cálculo 100 g da massa total do composto, a massa para cada elemento presente pode
ser determinado seguindo os seguintes passos:
Primeiro passo:
46,6
mC = ( 100 ) x 100 g = 46,6 g

4,4
mH = (100) x 100 g = 4,4 g

31,1
mN = ( ) x 100 g = 31,1 g
100
17,7
mO = ( 100 ) x 100 g = 17,7 g

Segundo passo: Em seguida, com base nas massas atômicas de cada elemento e na massa calculada acima, o
número de mols de cada elemento presente em 100 g do material orgânico pode ser prontamente calculado.
46,6
nC = ( 12 ) = 3,88 mol

4,4
nH = ( 1 ) = 4,4 mol

7
1600 FÍSICO-QUÍMICA APLICADA EXERCÍCIOS COMENTADOS - IME – ITA – OLIMPÍADA

31,1
nN = ( 14 ) = 2,22 mol

17,7
nO = ( ) = 1,11 mol
16

Terceiro passo: Como o oxigênio consiste no elemento presente em menor quantidade em termos molares, a fórmula
molecular do composto pode ser obtida dividindo-se os números de mols encontrados pelo número de mols de O.
Como resultado, tem-se o número de mols de um certo elemento para cada mol de O presente.
3,88
NC = ( ) ≅ 3,5
1,11
4,4
NH = ( )=4
1,11
2,22
NN = ( )=2
1,11
1,11
NO = ( )=1
1,11
Quarto passo:
Finalmente, como o número de mols de C não se apresenta como um número inteiro, todos os números de mols
encontrados devem ser multiplicados por dois (x 2). Logo, o composto orgânico de interesse deve apresentar a
seguinte fórmula molecular C7H8N4O2. De acordo com o enunciado, os produtos da combustão do material orgânico
são: CO2, H2O e NO2. Logo, o processo de combustão pode ser representado pela seguinte equação química
balanceada.
C7H8N4O2(s) + 12 O2(g) → 7 CO2(g) + 4 NO2(g) + 4 H2O(g)
Sabendo que 1000 g de material foram utilizados no experimento e que esta massa sofre combustão com 100% de
conversão, o número de mols de material que serão consumidos pode ser diretamente determinado.
1000 𝑔
𝑛𝐶7 𝐻8 𝑁4 𝑂2 = = 5,56 𝑚𝑜𝑙
180 𝑔. 𝑚𝑜𝑙 −1
Com base na estequiometria, cada mol de composto que reagem com O2 gera 15 mol de gases (7 mol de CO2, 4 mol
de NO2, 4 mol de H2O). Logo, através de uma regra de três simples com base no número de mols de C7H8N4O2
consumido, tem-se:
1 mol de C7H8N4O2 --------------- 15 mol
5,56 mols de C7H8N4O2 ---------- ngases
ngases = 83,4 mol

Considerando que a mistura de gases se comporta como ideal, o volume total de gás gerado durante a combustão a
227oC e 1 atm pode ser facilmente calculado:
ngases x R x T 83,4 x 0,08206 x (227 + 273) 83,4 x 0,08206 x 500
Vgases = = = = 3421,90 Litros
P 1 1
Questão 03 - A produção de cloreto de potássio e de cloreto de manganês, ambos em fase aquosa, pode ser
representada pela seguinte equação química balanceada: 2 KMnO4(aq) + 5 H2O2(aq) + 6 HCl(aq) → 2 MnCl2(aq) + 2 KCl(aq)
+ 5 O2(g) + 8 H2O(l).
Considere que no experimento foram utilizados 45,0 g de permanganato de potássio, 30 g de peróxido de hidrogênio
e 160 g de ácido clorídrico.
a) Identifique o reagente limitante, justificando sua resposta.
b) Calcule a massa dos dois cloretos de interesse formados.

8
1600 FÍSICO-QUÍMICA APLICADA EXERCÍCIOS COMENTADOS - IME – ITA – OLIMPÍADA

Resolução: Item a) A identificação do reagente limitante requer a determinação do número de mols de cada
reagente considerado.

m1 45,0 g
n1 = = = 0,28 mol
< MM >1 158 g. mol−1
m2 30,0 g
n2 = = = 0,88 mol
< MM >2 34 g. mol−1
m3 160,0 g
n3 = = = 4,38 mol
< MM >3 36,5 g. mol−1
Onde n1, n2 e n3 representam, respectivamente, o número de mols de permanganato de potássio (KMnO 4), peróxido
de hidrogênio (H2O2) e ácido clorídrico (HCl) inicialmente presentes. Dividindo-se os números de mols encontrados
pelo coeficiente estequiométrico de cada reagente, pode-se identificar o reagente limitante, como sendo aquele cujo
número de mols final apresenta a menor magnitude.
0,28mol
n1 ′ = = 0,14 mol (permanganato de potássio, reagente limitante)
2
0,88 mol
n2 ′ = = 0,176 mol
5
4,38 mol
n3 ′ = = 0,73 mol
6
Item b) Uma vez identificado o reagente limitante, as massas de cada cloreto produzido podem ser diretamente
determinadas mediante novamente a utilização da regra de três simples, com base na estequiometria do processo.
Para o MnCl2, tem-se:
2 KMnO4(aq) + 5 H2O2(aq) + 6 HCl(aq) → 2 MnCl2(aq) + 2 KCl(aq) + 5 O2(g) + 8 H2O(l)
2 mols de KMnO4(aq) --------------------------- 2 mols de MnCl2(aq)
2 x 158 g ---------------------------------------- 2 x 126 g
45 g ---------------------------------------------- mMnCl2
11340
𝑚𝑀𝑛𝐶𝑙2 = = 35,89 𝑔
316
Empregando-se o mesmo raciocínio para o cloreto de potássio, tem-se:
2 mols de KMnO4(aq) ------------------------ 2 mols de KCl(aq)
2 x (158) g ------------------------------------ 2 x (74,5) g
45 g -------------------------------------------- mKCl
6705
mKCl = = 21,22 g
316
Questão 04 - Calcule a massa de pirolusita que contém 85% (m/m) de dióxido de manganês (MnO2), necessária para
a produção de 250 g de óxido de alumínio e de manganês metálico, pelo processo de aluminotermia, que se
fundamenta na seguinte equação química balanceada: 3 MnO2(s) + 4 Al(s) → 3 Mn(s) + 2 Al2O3(s).

Resolução: Considerando a estequiometria do processo, tem-se que 3 mol de MnO2 geram 2 mol de Al2O3. Logo,
sabendo-se a massa de Al2O3, a massa requerida de MnO2 pode ser obtida mediante uma regra de três simples.
3 mols de MnO2(s) ---------------------------------------- 2 mols de Al2O3(s)
3 mols x 87 g.mol-1 --------------------------------------- 2 mols x 102 g.mol-1
mMnO2(s) ---------------------------------------------------- 250 g

250 𝑥 3 𝑥 87 65250
𝑚𝑀𝑛𝑂2 = = = 319,85 𝑔
2 𝑥 102 204
9
1600 FÍSICO-QUÍMICA APLICADA EXERCÍCIOS COMENTADOS - IME – ITA – OLIMPÍADA

A massa de pirolusita deve considerar o grau de pureza. No presente caso, sabe-se que 85% em massa do total da
mMnO2 319,85
amostra está associado ao MnO2, logo: mpirolusita = = = 376,29 g
0,85 0,85

Questão 05 - (GRILLO) Uma pequena amostra contendo 4,20 gramas de carbonato de magnésio foi tratada com
ácido clorídrico obtendo-se 476 mL de dióxido de carbono, medidos nas CNTP. Determine o rendimento desta reação.

Resolução: Equação química: MgCO3 + 2 HCl → MgCl2 + H2O + CO2


Cálculo do rendimento do processo (R):

1 mol de MgCO3 ------------ 1 mol de CO2


1 mol x 84 g.mol-1 ---------- 1 mol x 22,4 L.mol-1 x R
4,20 g ------------------------ 472 x 10-3 L
R = 0,421 (42,1%)

Questão 06 - (GRILLO)
a) Calcule a massa de sulfato de sódio formado a partir da reação de neutralização total, com 147 gramas de
ácido sulfúrico e 100 gramas de hidróxido de sódio.
b) Determine o reagente limitante.

Resolução: Item a) Equação química: H2SO4(aq) + 2 NaOH(aq) → Na2SO4(aq) + H2O(l)

Cálculo do número de mol de cada reagente:


m 147
Para o ácido sulfúrico: n = <MM> = 98
= 1,50 mol

m 100 2,5
Para o hidróxido de sódio: n = = = = 1,25 mol (reagente limitante)
<MM> 40 2

Item b) Cálculo da massa de sulfato de sódio produzido, a partir do reagente limitante: H2SO4(aq) + 2 NaOH(aq) →
Na2SO4(aq) + H2O(l)

2 mol de NaOH ------------------ 1 mol de Na2SO4


80 g -------------------------------- 142 g
100 g ------------------------------ mNa2SO4
mNa2SO4 = 177,5 g

b) Reagente limitante = hidróxido de sódio (NaOH).

Questão 07 - (GRILLO) Determine a massa de sulfato de cálcio obtida quando se tratam 370 gramas de hidróxido de
cálcio contendo 20% de impurezas por solução de ácido sulfúrico? Resolução: Cálculo do valor percentual referente
ao reagente na sua forma pura: 100% - 20% = 80%

Equação química balanceada: H2SO4(aq) + Ca(OH)2(aq) → CaSO4(aq) + 2 H2O(l)

Cálculo da massa de Ca(OH)2 puro: mCa(OH)2 = 370 g x 0,80 = 296 g

H2SO4(aq) + Ca(OH)2(aq) → CaSO4(aq) + 2 H2O(l)


74 gramas de Ca(OH)2 ------------ 136 gramas de CaSO4
296 gramas de Ca(OH)2 ----------- mCaSO4
mCaSO4 = 544 g

Questão 08 - Considere a reação de decomposição do nitrato de amônio conforme apresentada a seguir pela equação
química balanceada, NH4NO3(s) → N2(g) + O2(g) + H2O(g). Calcule o valor do volume gasoso a 227°C e 1,0 atm que será
produzido pela decomposição de 800 gramas de nitrato de amônio.

10
1600 FÍSICO-QUÍMICA APLICADA EXERCÍCIOS COMENTADOS - IME – ITA – OLIMPÍADA

Resolução: Equação química da decomposição do nitrato de amônio: 2 NH4NO3(s) → 2 N2(g) + O2(g) + 4 H2O(g)
m 800
Cálculo do número de mol de nitrato de amônio: n = <MM> = 80 = 10 mol

Cálculo do número de mol dos produtos gasosos: 2 NH4NO3(s) → 2 N2(g) + O2(g) + 4 H2O(g)
2 mols de NH4NO3 ------------- (2 + 1 + 4) mols de gases
10 mols de NH4NO3 ----------- ngases
ngases = 35 mol

Cálculo do volume dos gases, considerando comportamento ideal: pVgases = ngases.RT → 1 x Vgases = 35 x (0,08206)
x (227 + 273) → Vgases = 1436,05 L
Questão 09 - Os aromatizantes em sua grande maioria são cadeias carbônicas oxigenadas da família dos ésteres. O
butanoato de metila pode ser formado pela reação de esterificação entre um ácido carboxílico e um álcool, conforme
representado pela equação química: CH3CH2CH2COOH(aq) + CH3OH(aq) → CH3CH2CH2COOCH3(aq) + H2O(l). Calcule
o número de mol de butanoato de metila que pode ser obtido a partir de 4,75 g de ácido butírico e 2,75 g de metanol,
levando em consideração que há o consumo total do reagente limitante. Resolução: Considerando as massas de
ácido butírico e metanol informado no enunciado, o número de mols de cada reagente a ser consumido pode ser
diretamente determinado.
m1 4,75 g 4,75 g
n1 = = −1
= = 0,054 mol
< MM >1 (48 + 32 + 8)g. mol 88 g. mol−1
m2 2,75 g 2,75 g
n2 = = (12+16+4)g.mol−1 = = 0,086 mol
<MM>2 32 g.mol−1

Onde 𝑛1 representa o número de mols de ácido butírico, e 𝑛2 o número de mols de metanol. Como para cada mol de
ácido butírico, um mol de metanol é requerido pela estequiometria do processo, 0,054 mol de ácido requer a mesma
quantidade em mols do álcool. Como esta quantidade é inferior à quantidade de metanol inicialmente presente, este
pode ser dito em excesso, sendo, portanto, o reagente limitante definido pelo ácido butírico. Portanto, a massa de
éster formado deverá ser determinada com base no número de mols do reagente limitante (ácido butírico) consumido.
Considerando-se as massas moleculares de cada espécie, tal cálculo pode ser executado através de uma regra de
três simples.
Equação química: CH3CH2CH2COOH(aq) + CH3OH(aq) → CH3CH2CH2COOCH3(aq) + H2O(l)
1 mol de CH3CH2CH2COOH ------------ 1 mol de CH3CH2CH2COOCH3
88 g de ácido ------------------------------- 102 g de éster
4,75 g de ácido ----------------------------- méster
méster = 5,50 g de éster
m 5,50 g
Finalmente, o número de mols de éster pode ser facilmente determinado: néster = <MM>
éster
= 102g.mol−1 =
éster
0,054 mol
Questão 10 – Considere a seguinte reação química para a síntese do bromo em fase líquida a partir do dióxido de
titânio, representado pela equação química, 3 TiO2(s) + 4 BrF3(l) → 3 TiF4(s) + 2 Br2(l) + 3 O2(g). Se 12,0 g do óxido produz
0,056 g de oxigênio, determine o grau de pureza do TiO2 utilizado.
Resolução: Analisando-se a estequiometria da reação, percebe-se que para cada três mol de TiO2 consumidos
produzem um total de três mols de oxigênio. Com esta observação e com base na massa de oxigênio produzida,
pode-se determinar a quantidade de TiO2 que realmente reagiram, e, consequentemente, seu grau de pureza.
3 mol de TiO2(s) -------------------- 3 mol de O2(g)
3 x (79,87) g ------------------------ 3 mol x (32) g
mTiO2 --------------------------------- 0,056 g

11
1600 FÍSICO-QUÍMICA APLICADA EXERCÍCIOS COMENTADOS - IME – ITA – OLIMPÍADA

13,42
𝑚 𝑇𝑖𝑂2 = = 0,140 𝑔
96
0,14
O grau de pureza pode ser avaliado através da fração mássica de TiO2 na amostra, logo: (%) 𝑇𝑖𝑂2 = 12,0 =
0,0167 (1,67%)

Questão 11 – Calcule o número de átomos de hidrogênio presentes em 100 gramas do hidrocarboneto octano.

Resolução: Fórmula molecular do hidrocarboneto: C8H18 (<MM>C8H18 = 114 g.mol-1)


Cálculo do número de átomos de hidrogênio:
1 mol C8H18 ---------- 114 g ---------- 18 x 6,02 x 1023 átomos de hidrogênio
100 g ---------- X
X = 9,50 x 1024 átomos de hidrogênio

Questão 12 – (GRILLO) Considere o seguinte processo químico para a produção de sulfato de potássio, sulfato de
manganês II, sulfato de ferro III e água, conforme a equação química não-balanceada apresentada a seguir: KMnO4
+ FeSO4 + H2SO4 → K2SO4 + MnSO4 + Fe2(SO4)3 + H2O. Considere que foram colocados para reagir um quilograma
para cada reagente. A partir desta informação, determine a massa de todos os produtos formados.

Resolução: Realizando o balanceamento pelo método de oxirredução, temos: 2 KMnO4 + 10 FeSO4 + 8 H2SO4 →
K2SO4 + 2 MnSO4 + 5 Fe2(SO4)3 + 8 H2O, em que o permanganato de potássio é o agente oxidante e o sulfato de ferro
II é o agente redutor.

Cálculo do número de mol para cada reagente:

mKMnO4 1000 g 6,33


nKMnO4 = = −1
= = 3,16 mol
< MM >KMnO4 158 g. mol 2
mFeSO4 1000 g 6,58
nFeSO4 = = −1
= = 0,66 mol (reagente limitante)
< MM >FeSO4 152 g. mol 10
mH2 SO4 1000 g 10,20
nH2 SO4 = = −1
= = 1,27 mol
< MM >H2 SO4 98 g. mol 8

Cálculo da massa de todos os produtos formados:

10 mol de FeSO4 --------- 1 mol de K2SO4 ------- 2 mol de MnSO4 ----- 5 mol de Fe2(SO4)3 ----- 8 mol de H2O
1520 g -------------------- 174 g --------------------- 302 g ------------------- 2000 g ------------------- 128 g
1000 g -------------------- mK2SO4 -------------------- mMnSO4 ----------------- mFe2(SO4)3 ---------------- mH2O

As massas dos produtos são os seguintes:


mK2SO4 = 114,47 g (K2SO4 = sulfato de potássio)
mMnSO4 = 198,68 g (MnSO4 = Sulfato de Manganês II)
mFe2(SO4)3 = 1315,79 g [Fe2(SO4)3]= sulfato de ferro III)
mH2O = 84,21 g

Questão 13 – (OLIMPÍADA DE QUÍMICA DO DISTRITO FEDERAL) Um fermento utilizado industrialmente para


fabricar pão, devido a sua praticidade, é a mistura de dihidrogenopirofosfato de sódio (Na2H2P2O7) com NaHCO3, que
reage de acordo com a equação não balanceada: Na2H2P2O7 + NaHCO3 → Na4P2O7 + CO2 + H2O. Quando aquecidos
juntos, eles reagem liberando CO2, fazendo “crescer” a massa do pão. Calcule:
a) A massa necessária de NaHCO3 para produzir 5 mols de CO2;
b) A quantidade de matéria de Na2H2P2O7, necessária para reagir com 90 g de NaHCO3;
c) O rendimento percentual da reação, sabendo que 30 g de NaHCO3, produziu 25,70 g de Na4P2O7. Despreze
a parte fracionária, se houver.

12
1600 FÍSICO-QUÍMICA APLICADA EXERCÍCIOS COMENTADOS - IME – ITA – OLIMPÍADA

Resolução: Equação química: Na2H2P2O7 + 2 NaHCO3 → Na4P2O7 + 2 CO2 + 2 H2O

Item a) Cálculo do número de mol de bicarbonato de sódio, a partir da reação química: Na2H2P2O7 + 2 NaHCO3 →
Na4P2O7 + 2 CO2 + 2 H2O

2 mol de NaHCO3 --------------------- 2 mol de CO2


n ----------------------------------------- 5 mol de CO2
n = 5 mol de bicarbonato de sódio

Cálculo da massa, a partir do número de mol (n): m = n x < MM > = 5 x 84 = 420 g


90
Item b) Cálculo do número de mol de bicarbonato de sódio: 𝑛 = 84 = 1,07 𝑚𝑜𝑙

Cálculo do número de matéria de dihidrogenopirofosfato de sódio (Na2H2P2O7), a partir da reação química:

Na2H2P2O7 + 2 NaHCO3 → Na4P2O7 + 2 CO2 + 2 H2O


1 mol de Na2H2P2O7 ---------------------- 2 mol de NaHCO3
n --------------------------------------------- 1,07 mol de NaHCO3
n = 0,536 mol

Item c) Cálculo do rendimento da reação (R):

Na2H2P2O7 + 2 NaHCO3 → Na4P2O7 + 2 CO2 + 2 H2O


2 mol de NaHCO3 --------------------------- 1 mol de Na4P2O7
2 x (84) g ------------------------------------- 266 g x R
30 g -------------------------------------------- 25,70
4317,6
𝑅= = 0,541 (54,10%)
7980
Questão 14 – (IME) Uma amostra de 12,5 g de calcário (CaCO3, impuro) foi calcinada e o resíduo obtido adicionado
a 1 litro de água. Após a filtração borbulhou-se anidrido sulfúrico no meio, fazendo precipitar 13,6 g de sulfato de
cálcio. Qual a pureza do calcário?

Resolução: As equações químicas balanceadas estão apresentadas a seguir:


Equação 1: CaCO3(s) → CaO(s) + CO2(g)
Equação 2: CaO(s) + H2O(l) → Ca(OH)2(aq)
Equação 3: Ca(OH)2(aq) + SO3(g) → CaSO4(ppt) + H2O(l)

Somando as equações químicas, temos:


CaCO3(s) → CaO(s) + CO2(g)
CaO(s) + H2O(l) → Ca(OH)2(aq)
Ca(OH)2(aq) + SO3(g) → CaSO4(ppt) + H2O(l) +
CaCO3(s) + SO3(g) → CaSO4(ppt) + CO2(g), onde ppt significa precipitado.

Cálculo da pureza de carbonato de cálcio:


CaCO3(s) + SO3(g) → CaSO4(ppt) + CO2(g)
1 mol de CaCO3 -------------------- 1 mol de CaSO4
100 g --------------------------------- 136 g
p x 12,5 g ---------------------------- 13,6 g

1360
𝑝= = 0,80 (80%)
1700

13
1600 FÍSICO-QUÍMICA APLICADA EXERCÍCIOS COMENTADOS - IME – ITA – OLIMPÍADA

Questão 15 – O ouro é uma das substâncias metálicas que realizam poucas reações químicas e uma delas é com a
água régia, segundo a seguinte equação química não balanceada apresenta a seguir, Au + HCl + HNO3 → AuCl3 +
H2O + NO.

a) A partir de 1 quilograma de ouro, determine a massa e o número de mol de cloreto de ouro formado.
b) A partir da mesma quantidade de ouro, determine o volume de NO nas CNTP.

Resolução: Para este problema o balanceamento deverá ser feito através do método de oxirredução, onde o ouro
metálico é o agente redutor, sendo uma reação de oxidação e o ácido nítrico é o agente oxidante, sofrendo redução.
Então, a reação balanceada fica da seguinte forma:
Au(s) + 3 HCl(aq) + HNO3(aq) → AuCl3(aq) + 2H2O(l) + NO(g)

Item a) Cálculo da massa e do número de mol de cloreto de ouro:


1 mol de Au --------------------- 1 mol de AuCl3
1 mol x 197 g.mol-1 ------------ 1 mol x 303,5 g.mol-1
1000 g de Au ------------------- mAuCl3
mAuCl3 = 1540,61 g de AuCl3
1540,61
Cálculo do número de mol (AuCl3): n𝐴𝑢𝐶𝑙3 = 303,5
= 5,08 𝑚𝑜𝑙

Item b) Cálculo do volume de monóxido de nitrogênio nas CNTP: Au(s) + 3 HCl(aq) + HNO3(aq) → AuCl3(aq) + 2H2O(l) +
NO(g)

1 mol de Au ------------------- 1 mol de NO


1 mol x 197 g.mol-1 ---------- 1 mol x (22,4 L.mol-1)
1000 g de Au ------------------ VNO
VNO = 113,70 L

Questão 16 – (ITA) No processo de aquecimento, na presença de ar representado pela equação, Ca(HCO3)2(c) →


CaO(c) + 2 CO2(g) + H2O(g). Qual a perda de massa para cada grama do respectivo reagente no estado sólido?

Resolução: A reação química apresenta a decomposição do bicarbonato de cálcio, representado pela seguinte
equação química balanceada: Ca(HCO3)2(c) → CaO(c) + 2 CO2(g) + H2O(g).

Cálculo da perda de massa, em bases percentuais, a partir da relação estequiométrica:

1 mol de Ca(HCO3)2(c) ----------------- 1 mol de CaO(c)


162 g ------------------------------------- 56 gramas de CaO(c)
1 g ---------------------------------------- mCaO
mCaO = 0,346 g (34,6%)

Percentual de sobra de bicarbonato de cálcio = 100% - 34,6% = 65,4%.

Questão 17 - Considere o processo para a produção de cobre metálico, a partir do minério chamado calcosita:
Processo Químico 1: 2 Cu2S(s) + 3 O2(g) → 2 Cu2O(s) + 2 SO2(g)
Processo Químico 2: Cu2S(s) + 2 Cu2O(s) → 6 Cu(s) + SO2(g)
Calcule a quantidade de cobre, a partir 127,7 gramas do minério calcosita, com 75% de pureza.

Resolução: Somando as duas equações químicas apresentadas na questão indicadas pelos processos 1 e 2:
2Cu2S(s) + 3O2(g) → 2Cu2O(s) + 2SO2(g)
Cu2S(s) + 2Cu2O(s) → 6Cu(s) + SO2(g) +
3 Cu2S(s) + 3 O2(g) → 6 Cu(s) + 3 SO2(g) (Equação química global)

Dividindo a equação química por três, temos: Cu2S(s) + O2(g) → 2Cu(s) + SO2(g)

14
1600 FÍSICO-QUÍMICA APLICADA EXERCÍCIOS COMENTADOS - IME – ITA – OLIMPÍADA

Cálculo da massa de calcosita pura: mCu2S = 127,7 x 0,75 = 95,77 g

A partir da reação global, temos: Cu2S(s) + O2(g) → 2 Cu(s) + SO2(g)

1 mol de Cu2S(s) ------------------------------------------ 2 mol de Cu(s)


1 mol x (2 x 63,55 + 1 x 32) g.mol-1 ------------------ 2 mol x 63,55 g.mol-1
95,77 gramas --------------------------------------------- mCu(s)
mCu(s) = 76,51 g

Questão 18 - (OLIMPÍADA DE QUÍMICA DO DISTRITO FEDERAL) Em um motor movido a etanol ocorre uma reação
descrita pela seguinte equação: C2H6O(l) + O2(g) → CO2(g) + H2O(g). Considerando os dados, calcule o númeo de
moléculas de oxigênio necessárias para a combustão completa de 130 g de etanol. Resolução:

Equação química: C2H6O(l) + 3 O2(g) → 2 CO2(g) + 3 H2O(g)


130
Primeiramente será necessário calcular o número de mol de etanol: 𝑛𝑒𝑡𝑎𝑛𝑜𝑙 = 46 = 2,83 𝑚𝑜𝑙
Cálculo do número de mol de mol de oxigênio gasoso, a partir da reação química:

C2H6O(l) + 3 O2(g) → 2 CO2(g) + 3 H2O(g)


1 mol de C2H6O(l) ---------- 3 mol de O2(g)
2,83 mol de C2H6O(l) ------ nO2
nO2 = 8,49 mol

6,02 x 1023
Cálculo do número de moléculas de oxigênio gasoso: NO2 = 8,49 mol x moléculas =
1 mol
5,11 x 1024 moléculas

Questão 19 – (OLIMPÍADA BRASILEIRA DE QUÍMICA) Uma das razões do vasto uso da platina é a sua relativa
inércia química, entretanto, ela é “solúvel” na “água régia”, uma mistura de ácido nítrico e ácido clorídrico, segundo a
reação química (não balanceada) abaixo:
Pt(s) + HNO3(aq) + HCl(aq) → H2PtCl6(aq) + NO(g) + H2O(l)
Faça o balanceamento desta equação e responda as questões que se seguem:
a) Se você dispõe de 11,7 gramas de platina, quantos gramas de ácido cloroplatínico, H2PtCl6, poderá obter?
b) Que volume de óxido de nitrogênio, medido em CNTP, pode ser obtido a partir de 11,7 mg de Pt?
c) Quantos mililitros de ácido nítrico concentrado 10 mol.L-1 são necessários para reagir completamente com
11,7 gramas de Pt?
d) Se você tem 10,0 gramas de Pt e 180 mL de HCl de concentração 5,00 mol.L-1, mais excesso de ácido nítrico,
qual é o reagente limitante?

Resolução: Item a) O balanceamento desta reação química será feito pelo método redox. A platina (Pt) é o agente
redutor, sofrendo uma reação de oxidação e o ácido nítrico (HNO3) é o agente oxidante, sofrendo uma reação de
redução. A equação química balanceada fica da seguinte maneira:

3 Pt(s) + 4 HNO3(aq) + 18 HCl(aq) → 3 H2PtCl6(aq) + 4 NO(g) + 8 H2O(l).

Cálculo da massa de ácido cloroplatínico: 3 Pt(s) + 4 HNO3(aq) + 18 HCl(aq) → 3 H2PtCl6(aq) + 4 NO(g) + 8 H2O(l)

3 mol de Pt(s) ---------------------- 3 mol de H2PtCl6(aq)


585 g ------------------------------- 1230 g
11,7 g ------------------------------- m
m = 24,60 g de ácido cloroplatínico, H2PtCl6

Item b) Cálculo do volume de NO(g), nas CNTP: 3 Pt(s) + 4 HNO3(aq) + 18 HCl(aq) → 3 H2PtCl6(aq) + 4 NO(g) + 8 H2O(l)
3 mol de Pt(s) ---------------------- 4 mol de NO(g)
nPt ----------------------------------- nNO

15
1600 FÍSICO-QUÍMICA APLICADA EXERCÍCIOS COMENTADOS - IME – ITA – OLIMPÍADA

4.nPt = 3 x nNO
p x VNO mPt
Considerando que o monóxido de nitrogênio apresenta comportamento ideal, temos: 3 x RxT
=4x <MM>Pt
1 x VNO 11,7 x 10−3
3x =4x
0,08206 x (0 + 273) 195

VNO = 1,79 x 10−3 L

Item c) 3 Pt(s) + 4 HNO3(aq) + 18 HCl(aq) → 3 H2PtCl6(aq) + 4 NO(g) + 8 H2O(l)


3 mol de Pt(s) ------------ 4 mol de HNO3(aq)
nPt ------------------------- nHNO3
mPt
3 x [HNO3 ] x Vsolução = 4 x
< MM >Pt
11,7
3 x 10 x Vsolução = 4 x
195
11,7
Vsolução = 4 x = 0,008 L (8 mL)
30 x 195
mPt 10
Item d) Cálculo do número de mol para cada reagente químico: analisando a platina: nPt = = =
<MM>Pt 195
5,13 x 10−2
3
= 0,0171 mol

Analisando o ácido clorídrico, a partir da concentração da quantidade de matéria: nHCl = [HCl] x Vsolução = 5,0 x 0,180
= 0,90 mol
0,90
Observando a reação química, o ácido clorídrico apresenta 18 mols sendo consumido, logo: 𝑛𝐻𝐶𝑙 = 18
=
0,05 𝑚𝑜𝑙
Comparando os valores de cada número de mol, a platina é o reagente limitante.

Questão 20 - (IME) Uma solução aquosa de NaOH possui as seguintes características:


Fração molar de NaOH igual a 0,01.
Massa específica da solução igual a 1,04 g.mL-1 e
Um litro dessa solução neutraliza 2 litros de solução aquosa de ácido ortofosfórico. Calcule para a solução de H3PO4:
a) Molaridade;
b) Normalidade;
c) Concentração em g.L-1;
d) Molalidade.

Resolução: Para este problema será necessário primeiramente calcular a massa de hidróxido de sódio (soluto) e a
massa de água (solvente).

Dados da solução de hidróxido de sódio: X NaOH = 0,01


nNaOH
= 0,01
nTOTAL

nNaOH = 0,01 x nTOTAL

Sabendo que o somatório das frações molares é igual a 1, a fração molar da água será igual a:

XNaOH + XH2O = 1
XH2O = 1 - XNaOH
XH2O = 1 – 0,01 = 0,99

16
1600 FÍSICO-QUÍMICA APLICADA EXERCÍCIOS COMENTADOS - IME – ITA – OLIMPÍADA

𝑛𝐻2 𝑂
= 0,99
𝑛 𝑇𝑂𝑇𝐴𝐿

𝑛𝐻2 𝑂 = 0,99 𝑥 𝑛 𝑇𝑂𝑇𝐴𝐿

Logo, as equações são as seguintes:


nNaOH = 0,01 x nTOTAL (Equação 1)
nH2O = 0,99 x nTOTAL (Equação 2)
𝑚
Desenvolvendo a equação (1), temos: <𝑀𝑀>
𝑁𝑎𝑂𝐻
= 0,01 𝑥 𝑛 𝑇𝑂𝑇𝐴𝐿
𝑁𝑎𝑂𝐻

𝑚𝑁𝑎𝑂𝐻
= 0,01 𝑥 𝑛 𝑇𝑂𝑇𝐴𝐿
40

𝑚𝑁𝑎𝑂𝐻 = 0,40 𝑥 𝑛 𝑇𝑂𝑇𝐴𝐿 (Equação 3)

Desenvolvendo a equação (2), temos:


𝑚𝐻2 𝑂
= 0,99 𝑥 𝑛 𝑇𝑂𝑇𝐴𝐿
< 𝑀𝑀 >𝐻2 𝑜

𝑚𝐻2 𝑂
= 0,99 𝑥 𝑛 𝑇𝑂𝑇𝐴𝐿
18

𝑚𝐻2 𝑂 = 17,82 𝑥 𝑛 𝑇𝑂𝑇𝐴𝐿 (Equação 4)

Dividindo a equação (3) pela equação (4), temos:


𝑚𝑁𝑎𝑂𝐻 0,40 𝑥 𝑛
𝑚𝐻2𝑂
= 17,82 𝑥 𝑛𝑇𝑂𝑇𝐴𝐿
𝑇𝑂𝑇𝐴𝐿

𝑚𝑁𝑎𝑂𝐻
𝑚𝐻2𝑂
= 2,24 𝑥 10−2 (Equação 5)

Cálculo da massa da solução, a partir da densidade da solução:


1,04 gramas de solução ------------------------- 1 mL de solução (10-3 L)
msolução --------------------------------------------- 1 L de solução
msolução = 1040 gramas de solução, logo:

mNaOH + mH2O = 1040 g (Equação 6)

Cálculo da massa de água, substituindo a equação 5 na equação 6:


mNaOH + mH2O = 1040 g
2,24 x 10-3 x mH2O + mH2O = 1040 g
1,0224 x mH2O = 1040 g
mH2O = 1017 g

Logo, mNaOH = 22,78 g

Item a) Cálculo da concentração da quantidade de matéria do ácido ortofosfórico: H3PO4(aq) + 3NaOH(aq) → Na3PO4(aq)
+ 3H2O(l)
1 mol de H3PO4(aq) -------------- 3 mol de NaOH(aq)
nácido ------------------------------ nbase
nbase = 3 x nácido

17
1600 FÍSICO-QUÍMICA APLICADA EXERCÍCIOS COMENTADOS - IME – ITA – OLIMPÍADA

mNaOH
= 3 x [H3 PO4 ] x Vsolução
< MM >NaOH

22,78
= 3 x [H3 PO4 ] x 2
40,0

[H3 PO4 ] = 0,0950 mol. L−1

Item a) Cálculo da massa de ácido fosfórico, a partir de 22,78 g de NaOH:


1 mol de H3PO4(aq) ------------------- 3 mol de NaOH(aq)
98 g ------------------------------------- 3 x 40 g
mH3PO4 ---------------------------------- 22,78 g
mH3PO4 = 18,60 g
𝑚𝐻3𝑃𝑂4 18,60
Item b) Cálculo da concentração normal (N): 𝑁 = 𝑒𝑞𝑔 𝑥 𝑉𝑆𝑜𝑙𝑢çã𝑜
= 98 = 0,285 𝑒𝑞𝑔. 𝐿−1
𝐻3 𝑃𝑂4 ( )𝑥2
3

Item c) Cálculo da concentração comum do ácido fosfórico, a partir da concentração da quantidade de matéria: C =
[H3PO4] x <MM>H3PO4 = 0,095 x 98 = 9,31 g.L-1
18,60
Item d) Cálculo do número de mol de ácido fosfórico: 𝑛𝐻3 𝑃𝑂4 = = 0,190 𝑚𝑜𝑙
98

𝑛𝐻3𝑃𝑂4 0,190
Cálculo da molalidade (W) do ácido fosfórico: 𝑊 = 𝑠𝑜𝑙𝑣𝑒𝑛𝑡𝑒 = 1,017 = 0,187 𝑚𝑜𝑙. 𝑘𝑔−1
𝑚(𝑘𝑔)

Questão 21 – (IME) Determinar o volume máximo de ácido nítrico 0,5 mol.L-1 que pode ser obtido pela oxidação de
17 gramas de amônia.

Resolução: Cálculo do volume de ácido nítrico, a partir da relação estequiométrica: NH3 + 2 O2 → HNO3 + H2O

1 mol de NH3 ----------------- 1 mol de HNO3


17 g de NH3 ------------------- 1 mol de HNO3
17 g de NH3 ------------------- (M x V)ácido
0,5 x V ácido = 1
Vácido = 2,0 Litros

Questão 22 – (IME) Cinco gramas de um alcano deram, pela combustão, nove gramas de água. Qual a fórmula
molecular do hidrocarboneto?
3n+1
Resolução: Processo de combustão completa: Cn H2n+2 + 2
O2 → n CO2 + (n + 1)H2 O

1 mol de CnH2n+2 ---------------------------- (n +1) H2O


(12n + 2n + 2) mol de CnH2n+2 ------------ 18 x (n + 1) mol de H2O
5,0 g ------------------------------------------ 9,0 g

9 x (14n + 2) = 5 x 18 x (n + 1)
n = 2 [Alcano; etano (C2H6)]

Questão 23 – (IME) Certa massa de sódio reagiu com água, produzindo o composto A, o qual com ácido clorídrico
forneceu a substância B. Quando se tratou B com excesso de nitrato de prata, obteve-se um precipitado que, depois
de lavado e seco, apresentou uma massa de 14,35 gramas. Qual a massa de sódio usada?

Resolução: Organizando as equações químicas:


✓ Na + H2O → A

18
1600 FÍSICO-QUÍMICA APLICADA EXERCÍCIOS COMENTADOS - IME – ITA – OLIMPÍADA

✓ A + HCl → B
✓ B + AgNO3 → C

Completando as equações químicas:


Na + H2O → NaOH + ½ H2
NaOH + HCl → NaCl + H2O
NaCl + AgNO3 → AgCl + NaNO3

Somando as três equações químicas, temos:


Na + H2O → NaOH + ½ H2
NaOH + HCl → NaCl + H2O
NaCl + AgNO3 → AgCl + NaNO3 +
Equação química global: Na + HCl + AgNO3 → AgCl + NaNO3 + ½ H2

Cálculo da massa do precipitado do cloreto de prata: Na + HCl + AgNO3 → AgCl + NaNO3 + ½ H2


1 mol de Na ---------- 1 mol de AgCl
23 g -------------------- (108 + 35,5) g
mNa --------------------- 14,35 g
mNa = 2,3 g

Questão 24 – (IME) A um excesso de bicarbonato de potássio adicionou-se 125 mL de solução de ácido sulfúrico. O
volume do gás liberado, medido a seco a 200C e 765 mmHg foi de 2,5 litros . Calcular a concentração do ácido
expressa em número de mol por litro.

Resolução: Equação química: 2 KHCO3(s) + H2SO4(aq) → K2CO3(s) + 2 H2O(l) + 2 CO2(g)

Cálculo da concentração da quantidade de matéria do ácido sulfúrico:


2 KHCO3(s) + H2SO4(aq) → K2CO3(s) + 2 H2O(l) + 2 CO2(g)
1 mol de H2SO4 ---------- 2 mol de CO2
nH2SO4 ---------------------- nCO2
nCO2 = 2 x nH2SO4

p x VCO2
= 2 x [H2 SO4 ] x Vsolução
RxT
765
( ) x 2,50
760 = 2 x [H2 SO4 ] x 0,125
0,08206 x (20 + 273)

765 𝑥 2,50 1912,5


[H2 SO4 ] = = = 0,419 mol. L−1
760 x 0,08206 x 293 x 0,250 4568,28

Questão 25 – (IME) Uma mistura gasosa ideal de propano e ar é queimada a pressão constante, gerando 720 litros
de CO2 por hora, medidos a 20oC. Sabe-se que o propano e o ar encontram-se em proporção estequiométrica.
Determine a velocidade média de reação da mistura em relação ao ar, considerando a composição do ar 21% de
oxigênio e 79% de nitrogênio, em volume.

Resolução: Equação química da combustão completa do gás propano: C3H8(g) + 5O2(g) → 3CO2(g) + 4H2O(g)

Cálculo do número de mol de gás nitrogênio (N2):


5 mol de O2 --------------- 21%
nN2 mol de N2 ------------- 79%
nN2 = 18,81 mol de N2

19
1600 FÍSICO-QUÍMICA APLICADA EXERCÍCIOS COMENTADOS - IME – ITA – OLIMPÍADA

Equação de combustão completa do propano com a presença de gás nitrogênio: C3H8(g) + 5 O2(g) + 18,81 N2(g) →
18,81 N2(g) + 3 CO2(g) + 4 H2O(g)

Cálculo da velocidade em relação ao ar: C3H8(g) + 5 O2(g) + 18,81 N2(g) → 18,81 N2(g) + 3 CO2(g) + 4 H2O(g)
(5 + 18,81) mol de ar ---------- 3 mol de CO2
23,81 mol de ar ----------------- 3 mol de CO2
Var -------------------------------- 720 L.h-1
Var = 5714,4 L.h-1

Questão 26 – (ITA) Escreva a equação química balanceada da combustão completa do iso-octano com o ar
atmosférico. Considere que o ar é seco e composto por 21% de oxigênio gasoso e 79% de nitrogênio gasoso.

Resolução: Equação química da combustão completa do iso-octano: C8H18 + 25/2 O2 → 8 CO2 + 9 H2O

Levando em consideração que a composição do ar tenha somente O2(g) e N2(g), com 20,9% e 78,1%, respectivamente,
o número de mol de N2(g) será:

Cálculo do número de mol de nitrogênio (N2):


(25/2) mol de O2 ---------- 20,90% de O2
n mol de N2 ---------------- 78,10% de N2
nN2 = 47,02 mol de N2

Equação química de combustão completa do iso-octano com a presença do gás nitrogênio: C8H18 + (25/2) O2 + 47,02
N2→ 8 CO2 + 9 H2O + 47,02 N2

Questão 27 – (ITA) Escreva a reação de combustão completa de um hidrocarboneto genérico (CαHβ) com ar
atmosférico. Considere a presença do nitrogênio gasoso no balanço estequiométrico da reação e expresse os
coeficientes estequiométricos dessa reação em função de α e β.

β
Resolução: Equação química da combustão completa do hidrocarboneto genérico: Cα Hβ + (α + 4) O2 →
β
α CO2 + 2 H2 O

Cálculo do número de mol de nitrogênio gasoso: Levando em consideração que o ar tenha somente O2(g) e N2(g), com
composições de 20,9% e 78,1%, respectivamente, o número de mol de N2(g) será:

(α + β/4) mol de O2 ---------- 20,90% de O2(g)


n mol de N2 ------------------- 78,10% de N2(g)
nN2 = 3,74 x (α + β/4) mol de N2

A equação química de combustão do hidrocarboneto genérico (CαHβ) com a presença do nitrogênio gasoso ficará da
seguinte forma:
β β β β
Cα Hβ + (α + ) O2 + 3,74 (α + ) N2 → α CO2 + H2 O + 3,74 (α + ) N2
4 4 2 4

Questão 28 – (ITA) Três recipientes iguais de 4 litros de capacidade, chamados de 1, 2 e 3, mantidos na mesma
temperatura, contêm 180 mL de água. A cada um destes recipientes se junta, respectivamente, 0,10 mol e cada uma
das seguintes substâncias: óxido de cálcio, cálcio metálico e hidreto de cálcio. Após a introdução do respectivo sólido,
cada frasco é bem fechado. Escreva as equações químicas, completas e balanceadas, para cada uma das reações
que ocorre em cada recipiente.

Resolução:
Recipiente 1: CaO(s) + H2O(l) → Ca(OH)2
Recipiente 2: Ca(s) + 2 H2O(l) → Ca(OH)2(aq) + H2(g)
Recipiente 3: CaH2(s) + 2 H2O(l) → Ca(OH)2 + 2H2(g)

20
1600 FÍSICO-QUÍMICA APLICADA EXERCÍCIOS COMENTADOS - IME – ITA – OLIMPÍADA

Questão 29 – (ITA) Uma amostra de 1,222 gramas de cloreto de bário hidratado (BaCl2.nH2O) é aquecida até a
eliminação total da água de hidratação, resultante em uma massa de 1,042 gramas. Com base nas informações
fornecidas e mostrando os cálculos efetuados, determine:
a) O número de mol de cloreto de bário;
b) O número de mol de água;
c) A fórmula molecular do sal hidratado.

Resolução:
𝑚𝐵𝑎𝐶𝑙 1,042
Item a) Cálculo do número de mol de cloreto de bário: 𝑛𝐵𝑎𝐶𝑙2 = <𝑀𝑀> 2 = 209
= 4,98 𝑥 10−3 𝑚𝑜𝑙
𝐵𝑎𝐶𝑙2

Item b) Massa de água = massa do cloreto de bário hidratado – massa de cloreto de bário = 1,222 gramas – 1,042
gramas = 0,180 gramas.
𝑚H O 0,180
Item c) Cálculo do número de mol de água: 𝑛H2 O = <𝑀𝑀>2 = 18
= 10−2 𝑚𝑜𝑙
H2 O
Cálculo do n: BaCl2 n.H2O

1,222 gramas ----------------------- (209 + 18n)


1,042 gramas ----------------------- 209
n=2
Fórmula molecular: BaCl2 x 2.H2O

Questão 30 – (IME) O nitrogênio forma cinco diferentes óxidos. A análise centesimal de amostras desses óxidos
forneceu os resultados a seguir.

Percentagem Percentagem
em massa em massa
de nitrogênio de oxigênio
Óxido 1 63,63 36,37
Óxido 2 46,67 53,33
Óxido 3 36,84 63,16
Óxido 4 30,44 69,56
Óxido 5 25,93 74,04

Determine, a partir destes dados:


a) a fórmula mínima de cada um;
b) a(s) nomenclatura(s) correspondente(s) de cada óxido.

Resolução: Base de cálculo: 100 gramas para cada óxido.

Analisando o óxido 1:

63,63 63,63 g 4,545 mol


N: x 100 g = 63,63 g = = =2
100 14 g. mol−1 2,273 mol

36,37 36,37 g 2,273 mol


O: x 100 g = 36,37 g = = =1
100 16 g. mol−1 2,273 mol

Fórmula Molecular = N2O

Analisando o óxido 2:

21
1600 FÍSICO-QUÍMICA APLICADA EXERCÍCIOS COMENTADOS - IME – ITA – OLIMPÍADA

46,67 46,67 g 3,33 mol


N: x 100 g = 46,67 g = −1
= =1
100 14 g. mol 3,33 mol

53,33 53,33 g 3,33 mol


O: x 100 g = 53,33 g = −1
= =1
100 16 g. mol 3,33 mol

Fórmula Molecular = NO

Analisando o óxido 3:

36,84 36,84 g 2,63 mol 𝑥2


N: x 100 g = 36,84 g = = = 1 → 2
100 14 g. mol−1 2,63 mol

63,16 63,16 g 3,95 mol 𝑥2


O: x 100 g = 63,16 g = = = 1,50 → 3
100 16 g. mol−1 3,95 mol

Fórmula Molecular = N2O3

Analisando o óxido 4:

30,44 30,44 g 2,17 mol


N: x 100 g = 30,44 g = −1
= =1
100 14 g. mol 2,17 mol

69,56 69,56 g 4,34 mol


O: x 100 g = 69,56 g = −1
= =2
100 16 g. mol 2,17 mol

Fórmula Molecular = NO2

Analisando o óxido 5:

25,93 25,93 g 1,85 mol 𝑥2


N: x 100 g = 25,93 g = −1
= = 1→ 2
100 14 g. mol 1,85 mol

74,04 74,04 g 4,63 mol 𝑥2


O: x 100 g = 74,04 g = −1
= = 2,5 → 5
100 16 g. mol 1,85 mol

Fórmula Molecular = N2O5

Questão 31 - (IME) Em uma síntese, a partir de dois óxidos, obtém-se 8,2 g de nitrato de cálcio. Considerando a
conversão estequiométrica, determine:
a) quais são os óxidos;
b) as quantidades necessárias, em gramas, de cada reagente;
c) a massa de carbonato de cálcio necessária para se obter um dos óxidos para esta síntese.

Resolução: Os óxidos que reagem para a formação do sal inorgânico - nitrato de cálcio [Ca(NO3)2] são os seguintes:
óxido básico + óxido ácido →sal inorgânico. Logo: CaO + N2O5 → Ca(NO3)2

CaO → óxido de cálcio e N2O5 → pentóxido de dinitrogênio.

a) Cálculo da massa de cada reagente, a partir de 8,2 g de Ca(NO3)2: CaO + N2O5 → Ca(NO3)2

1 mol de CaO ------------- 1 mol de N2O5 ----------------- 1mol de Ca(NO3)2


56 g ------------------------- 108 g --------------------------- 164 g
mCaO ------------------------ mN2O5 --------------------------- 8,2 g

22
1600 FÍSICO-QUÍMICA APLICADA EXERCÍCIOS COMENTADOS - IME – ITA – OLIMPÍADA

mCaO = 2,8 g de CaO


mN2O5 = 5,4 g de N2O5

b) Para este item será necessário apresentar a decomposição do carbonato de cálcio: CaCO3(g) → CaO(s) + CO2(g)

1 mol de CaCO3 ------------ 1 mol de CaO


100 g de CaCO3 ------------ 56 g de CaO
mCaCO3 ------------------------ 2,8 g de CaO
mCaCO3 = 5,0 g

Questão 32 – Foi adicionado uma quantidade de massa de carbonato de amônio a 49 gramas de ácido fosfórico em
um determinado béquer.
a) Determine a massa de sal formado.
b) Nomeie o sal formado entre o carbonato de amônio com o ácido fosfórico.
c) Determine o volume de dióxido de carbono liberado nas condições ambientes de temperatura e pressão.
d) Determine o volume de dióxido de carbono liberado nas condições normais de temperatura e pressão.

Resolução: Equação química: 3 (NH4)2CO3 + 2 H3PO4 → 2 (NH4)3PO4 + 3 H2O + 3 CO2

a) O sal formado é o fosfato de amônio, cuja fórmula molecular é [(NH4)3PO4]. Cálculo da massa de sal formado:

3 (NH4)2CO3 + 2 H3PO4 → 2 (NH4)3PO4 + 3 H2O + 3 CO2


2 x (3 + 31 + 64) g ---------- 2 x (42 + 12 + 31 + 64) g
49 g --------------------------- Massa
Massa = 74,50 g

b) O sal formado é o fosfato de amônio, cuja fórmula molecular é [(NH4)3PO4].

c) Cálculo do volume de CO2 nas CNATP, sabendo que o volume é igual a 24,45 L.mol-1 (p = 1 atm e T = 25°C).
Equação química: 3 (NH4)2CO3 + 2 H3PO4 → 2 (NH4)3PO4 + 3 H2O + 3 CO2

2 mol de H3PO4 ------------------------------------------ 3 CO2


2.(3 + 31 + 64) g ----------------------------------------- 3 x (24,45 L)
49 g -------------------------------------------------------- VCO2
VCO2 = 18,34 L

d) Cálculo do volume de CO2 nas CNTP: 3 (NH4)2CO3 + 2 H3PO4 → 2 (NH4)3PO4 + 3 H2O + 3 CO2

2 mol de H3PO4 ------------------------------------------ 3 CO2


2 x (3 + 31 + 64) g --------------------------------------- 3 x (22,4 L)
49 g -------------------------------------------------------- VCO2
VCO2 = 16,8 L

Questão 33 – (ITA) Calcule o máximo de massa de água que se pode obter partindo de 8,0 gramas de hidrogênio e
32,0 gramas de oxigênio. Indique qual o reagente em excesso e quanto sobra do mesmo. Resolução:

Determinação do reagente em excesso e do regente limitante – cálculo do número de mol:

m O2 32
nO2 = = = 1,0 mol (reagente limitante)
< MM >O2 32

m H2 8
nH2 = = = 4,0 mol (reagente em excesso)
< MM >H2 2

Cálculo da massa de água produzida: H2 + ½ O2 → H2O

23
1600 FÍSICO-QUÍMICA APLICADA EXERCÍCIOS COMENTADOS - IME – ITA – OLIMPÍADA

½ mol de O2 ---------- 1 mol de H2O


(½ x 32)g ------------- 18 gramas de H2O
32 g -------------------- mH2O
mH2O = 36 g
Cálculo da massa de reagente em excesso: H2 + ½ O2 → H2O
1 mol de H2 ---------- ½ mol de O2
2,0 g H2 --------------- ½ x 32,0 g
mH2 -------------------- 32,0 g
mH2 = 4,0 g

Questão 34 – (GRILLO) A recuperação do ferro metálico, a um percentual de aproximadamente 86,30%, a partir de


um minério que contém 43,20% de trióxido de diferro (magnetita). Determine a massa de ferro metálico que pode ser
recuperada a partir de 2,00 kg de magnetita. Favor considerar que o ferro obtido é a partir da redução do minério com
monóxido de carbono. Resolução: Fe3O4(s) + 4 CO(g) → 3 Fe(s) + 4 CO2(g)

1 mol de Fe3O4 ------------- 3 mol de Fe x R


232 g ------------------------- 144,98 g
864 g ------------------------- mFe
mFe = 539,92 g

Questão 35 – (ITA – MODIFICADA) Um método de obtenção de carbono puro consiste na decomposição de certos
hidratos de carbono. Quantos gramas de carbono seriam obtidos pela decomposição de um quilograma de sacarose,
supondo que os únicos produtos sejam carbono e água? Calcule também a massa de água que seria formada.
Resolução: Equação química: C12H22O11 → 12 C + 11 H2O

1 mol de C12H22O11 ------------------------ 12 mol de C ------------------------ 11 mol de H2O


1 mol x 342 g.mol-1 de C12H22O11 ------- 12 mol x 12 g.mol-1 de C -------- 11 mol x 18 g,mol-1 de H2O
1000 g de C12H22O11 ----------------------- mCarbono ------------------------------ mH2O

mCarbono = 421,05 g e mH2O = 578,95 g

Questão 36 – (OLIMPÍADA DE QUÍMICA DO RIO GRANDE DO SUL) Incêndios podem ser naturais ou artificiais. Os
naturais são aqueles em que não há intervenção direta ou indireta do homem. Muitos extintores de incêndio utilizam
pó químico, em uma mistura de NaHCO3, KHCO3 e outros aditivos. O NaHCO3 submetido a altas temperaturas sofre
a reação de decomposição térmica a seguir: 2 NaHCO3(s) → Na2CO3(s) + CO2(g) + H2O(v). O gás carbônico e o vapor
d’água, produzidos pela reação, são capazes de extinguir o fogo.
a) Quais os nomes das substâncias NaHCO3 e Na2CO3, abordadas?
b) Segundo a equação química fornecida, calcule a massa, em gramas, de dióxido de carbono obtido caso haja
decomposição de 200 gramas de NaHCO3.

Resolução:

Item a) NaHCO3 = Bicarbonato de sódio e KHCO3 = Bicarbonato de potássio.

Item b) 2 NaHCO3(s) → Na2CO3(s) + CO2(g) + H2O(v)


2 mol de NaHCO3(s) ---------- 1 mol de CO2(g)
168 g ---------------------------- 44 g
200 g ---------------------------- mCO2
mCO2 = 52,38 g

Questão 37 – (OLIMPÍADA DE QUÍMICA DO RGS) O nitrato de sódio, também conhecido como salitre do Chile é
usado, entre outras coisas, para fabricação de fertilizantes e explosivos e para preservação da cor de produtos
embutidos, como carnes, salsichas, salames, presuntos, etc. Pode ser obtido pela reação do ácido nítrico com o
hidróxido de sódio. Baseado nessa reação:
a) Escrever a equação química de obtenção do nitrato de sódio.

24
1600 FÍSICO-QUÍMICA APLICADA EXERCÍCIOS COMENTADOS - IME – ITA – OLIMPÍADA

b) Calcular a massa, em gramas, de sal obtido, partindo-se de 4 gramas de base, com 80% de pureza.
c) Determinar o grau de pureza do nitrato de sódio, supondo-se que 400 gramas deste sal contêm 20 gramas
de impurezas.
d) Calcule a quantidade de matéria de ácido nítrico necessária para obter 510 gramas de sal.

Resolução: Item a) Equação química: HNO3(aq) + NaOH(aq) → NaNO3(aq) + H2O(l)

Item b) Cálculo da massa de hidróxido de sódio pura: mNaOH = 4,0 x 8,0 = 32,0 g

Cálculo da massa de nitrato de sódio:

1 mol de NaOH(aq) ----------- 1 mol de NaNO3(aq)


40 g de NaOH ---------------- 85,0 g de NaNO3
3,20 g de NaOH -------------- mNaNO3
mNaNO3 = 6,80 g

Item c) Cálculo do grau de pureza do nitrato de sódio (%): mNaOH = 40 g x 0,80 = 32,0 g

Cálculo da massa de nitrato de sódio: mNaNO3(puro) = 400 gramas – 20 gramas = 380 g


380 𝑔
Cálculo da pureza: (%)𝑝𝑢𝑟𝑜 = = 0,95 (95%)
400 𝑔

Item d) Equação química: HNO3(aq) + NaOH(aq) → NaNO3(aq) + H2O(l)

1 mol de HNO3(aq) --------------------- 1 mol de NaNO3(aq)


63 g de HNO3(aq) ----------------------- 85 g de NaNO3(aq)
Massa HNO3 ------------------------------ 510 g de NaNO3(aq)
Massa HNO3 = 378 g
mHN𝑂3 378
Cálculo do número de mol: nHN𝑂3 = = = 5,56 𝑚𝑜𝑙
<MM>HN𝑂3 63

Questão 38 – (IME) A reação de 124 gramas de fósforo branco com uma solução de ácido nítrico gera óxido nítrico
e 98 gramas de ácido fosfórico. Sabendo que o rendimento da reação é de 100%, determine o grau de pureza do
fósforo.

Resolução: Equação química: 20 HNO3 + 3 P4 + 8 H2O → 20 NO + 12 H3PO4. Cálculo do grau de pureza (P) do
fósforo branco:

3 mol de P4 ------------------------------ 12 mol de H3PO4


3 mol x (124) g.mol-1 ------------------ 12 mol x (98) g.mol-1
(p x 124 g) ------------------------------ 98 g de ácido fosfórico
Pureza (p) = 0,25 (25%)

Questão 39 – (IME) Determine o volume de cloro obtido, a 27,0°C e 738 mmHg, pela ação de excesso de ácido
clorídrico concentrado sobre 30,7 gramas de pirolusita com 85,0% em peso de MnO2. Considere o cloro com
comportamento ideal. Resolução: Equação química de formação do cloro gasoso:

MnO2 + 4 HCl → MnCl2 + 2 H2O + Cl2

1 mol − − − − − − − 87 g
85
30,7 x −−−−−n
100

nCl2 = 0,30 mol

25
1600 FÍSICO-QUÍMICA APLICADA EXERCÍCIOS COMENTADOS - IME – ITA – OLIMPÍADA

nxRxT
Cálculo do volume de gás cloro, a partir da aplicação da equação dos gases ideais: V= pideal
=
0,30 x 0,08206 x 300
738 = 7,60 L
( )
760
Questão 40 – (OLIMPÍADA PORTUGUESA DE QUÍMICA) Em muitas transformações químicas há liberação de um
gás, e nesses casos as determinações quantitativas têm de ser efetuadas com mais cuidado. Analise os exemplos
seguintes:
1. O carbonato de cálcio reage com ácido clorídrico liberando dióxido de carbono.
a) Escreva a equação química, devidamente acertada, que traduz a transformação.
b) Calcule a massa de CO2 que espera venha a ser produzida por reação de 15,0 g de CaCO3 com uma solução
aquosa contendo 4,5 g de HCl.
c) Calcule o rendimento da reação se forem obtidas 1,8 g de CO2 nas condições da reação descritas na linha
b).

Resolução: Equação química: CaCO3(s) + 2 HCl(aq) → CaCl2(aq) + H2O(l) + CO2(g)

a) Para a determinação da massa de dióxido de carbono, será necessário primeiramente determinar o número
de mol para cada reagente químico:

mCaCO3 15,0
nCaCO3 = = = 0,15 mol (reagente em excesso)
< MM >CaCO3 100
mHCl 4,5 0,123
nHCl = = = = 0,0616 mol (reagente limitante)
< MM >HCl 36,5 2
Através da reação, temos a seguinte relação estequiométrica:
2 mol de HCl(aq) ------------- 1 mol de CO2(g)
73 g --------------------------- 44 g
4,5 g -------------------------- mCO2
mCO2 = 2,71 g

b) Através da reação química, temos:


2 mol de HCl(aq) ------------ 1 mol de CO2(g)
73 g -------------------------- 44 g x Rendimento
4,5 g ------------------------- 1,8 g
127.8
Rendimento = = 0,664 (66,4%)
198

Questão 41 – (ITA) No processo de Mond de purificação do níquel, o metal impuro é tratado com CO, segundo a
𝐶𝑂
equação: Ni → Ni(CO)4. Determine o volume de CO medido nas CNTP, necessário para purificar 0,500 gramas de
níquel contendo 25% de impurezas.

Resolução: Equação química: Ni(s) + 4 CO(g) → Ni(CO)4

Cálculo do volume de CO, medido nas CNTP: 1 mol de Ni(s) ------------ 4 mol de CO(g)
58,7 g de Ni(s) ----------- 4 mol de CO x 22,4 L.mol-1
0,500 g x 0,75 ---------- VCO
VCO = 0,572 litros de CO

26
1600 FÍSICO-QUÍMICA APLICADA EXERCÍCIOS COMENTADOS - IME – ITA – OLIMPÍADA

Questão 42 – (ITA) Determine o menor volume de solução de ácido clorídrico 0,250 mol.L-1 necessário para dissolver
completamente 13,50 gramas de alumínio metálico granulado.

Resolução: Equação química: Al(s) + 3 HCl(aq) → AlCl3(aq) + 3/2 H2(g)

Relação estequiométrica entre o alumínio metálico e a solução aquosa de ácido clorídrico:


1 mol de Al ------ 3 mol de HCl
nAl ----------------- nHCl
3 x nAl = nHCl
massa
Cálculo do volume de ácido: 3 x <MM> = [Al]x Vsolução
Al
13,50
3x = 0,25 x Vsolução
27,0

Vsolução = 6,0 L

Questão 43 – (ITA) Deseja-se preparar 57 gramas de sulfato de alumínio Al2(SO4)3 a partir de alumínio sólido (Al),
praticamente puro, e ácido sulfúrico. O ácido sulfúrico disponível é uma solução aquosa 96% (m/m), com massa
específica de 1,84 g.cm-3.
a) Qual a massa, em gramas, de alumínio necessária para preparar a quantidade de Al2(SO4)3 especificada? Mostre
os cálculos realizados.
b) Qual a massa, em gramas, de ácido sulfúrico necessária para preparar a quantidade de Al2(SO4)3 especificada?
Mostre os cálculos realizados.
c) Nas condições normais de temperatura e pressão (CNTP), qual é o volume, em litros, de gás formado durante a
preparação da quantidade de Al2(SO4)3 especificada? Mostre os cálculos realizados.
d) Caso a quantidade especificada de Al2(SO4)3 seja dissolvida em água acidulada, formando 1 litro de solução, qual
a concentração de íons Al+3 e de íons SO4-2 existentes nesta solução?

Resolução: Item a) Cálculo da massa de alumínio (mAl): 2 Al(s) + 3 H2SO4(aq) → Al2(SO4)3(aq) + 3 H2(g)

2 mol de Al(s) ---------------------- 1 mol de Al2(SO4)3(aq)


2 mol x (27 g.mol-1) -------------- 1 mol x (342 g.mol-1)
mAl ---------------------------------- 57 g
Massa de alumínio = 9,0 g

Item b) Cálculo da massa de ácido sulfúrico (mH2SO4): 2 Al(s) + H2SO4(aq) → Al2(SO4)3(aq) + H2(g)

3 mol de H2SO4(aq) -------------------- 1 mol de Al2(SO4)3(aq)


3 mol. (98 g.mol-1) -------------------- 1 mol. (342 g.mol-1)
mH2SO4 ----------------------------------- 57 g
mH2SO4 = 49,0 g

Item c) Cálculo do volume de gás formado (VH2): 2 Al(s) + H2SO4(aq) → Al2(SO4)3(aq) + H2(g)

1 mol de Al2(SO4)3(aq) -------------------- 3 mol de H2 (g)


1 mol. (342 g.mol-1) ---------------------- 3 mol. (22,4 L.mol-1)
57 g ----------------------------------------- VH2
VH2 = 11,2 L
nAl (SO )
Item d) Cálculo da concentração da quantidade de matéria do sulfato de alumínio: [Al2 (SO4 )3 ] = V 2 4 3 =
solução
57
= 0,167 mol. L−1
342

Equação química: Al2(SO4)3(aq) → 2 Al+3(aq) + 3 (SO4)-3(aq)

Cálculo da concentração da quantidade de matéria dos íons:

27
1600 FÍSICO-QUÍMICA APLICADA EXERCÍCIOS COMENTADOS - IME – ITA – OLIMPÍADA

1 mol de Al2(SO4)3(aq) ---------- 2 mol de Al+3(aq) ---------- 3 mol de SO4-3


0,167 mol.L-1 ------------------- [Al+3] ----------------------- [SO4-3]
[Al+3] = 0,333 mol.L-1
[SO4-3] = 0,501 mol.L-1

Questão 44 – Calcule a massa de hidróxido de sódio que deve ser dissolvida em 0,500 mL de água para que a
solução resultante tenha um pH12 a 25°C.

Resolução: Cálculo do potencial hidroxiliônico, a 25°C: pH + pOH = 14


12 + pOH = 14
pOH = 2

Cálculo da concentração hidroxiliônica [OH-]: pOH = - log[OH-]


- log[OH-] = 2
[OH-] = 0,010 mol.L-1

Equação química de dissociação do hidróxido de potássio: KOH(aq) → K+(aq) + OH-(aq)

1 mol de NaOH(aq) ---------- 1 mol de Na+(aq) ---------------------- 1 mol de OH-(aq)


[NaOH] ----------------------- 1 mol de Na+(aq) ---------------------- 10-2 mol.L-1
[NaOH] = 10-2 mol.L-1
mKOH
Cálculo da massa de KOH, a partir da concentração da quantidade de matéria: [KOH] = <MM>
KOH x Vsolução

mNaOH = [NaOH] x < MM >KOH x Vsolução = 0,010 x 40 x 0,50 x 10−3 = 0,00020 g

Questão 45 – (ITA) 3,64 gramas de fosfeto de cálcio foram adicionados a uma certa quantidade de água. Após a
reação completa, todo o produto gasoso formado foi recolhido em um recipiente de 8,2 ml. Calcule o valor numérico
da pressão, em atm, exercida pelo produto gasoso a 27°C.

Resolução: Pela proporção estequiométrica, 1 mol de fosfeto de cálcio produz 2 mol de fosfina.
3,64
Cálculo do número de mol de fosfeto de cálcio: n = 182
= 0,02 mol

Equação química: Ca3P2 + 6 H2O → 3 Ca(OH)2 + 2 PH3

1 mol de Ca3P2 ---------- 2 mol de PH3


nCa3P2 ---------------------- nPH3
2 x nCa3P2 = nPH3
2 x 0,02 = nPH3
nPH3 = 0,04 mol
nxRxT 0,04 x 0,08206 x 300
Através da equação dos gases ideais, temos: p = = = 120,1 atm
VPH3 8,20 x 10−3

Questão 46 – (ITA) Considere uma mistura gasosa constituída de C3H8, CO e CH4. A combustão, em excesso de
oxigênio de 50 mL dessa mistura gasosa forneceu 70 mL de CO2(g). Determine o valor numérico do percentual de
C3H8 na mistura gasosa.

Resolução:

x C3H8 + 5x O2 → 3x CO2 + 4x H2O


y CO + 0,5y O2 → y CO2
z CH4 + 2 O2 → z CO2 + 2z H2O

28
1600 FÍSICO-QUÍMICA APLICADA EXERCÍCIOS COMENTADOS - IME – ITA – OLIMPÍADA

Volume da mistura: x + y + z = 50 ml

Volume de CO2 formado: 3x + y + z = 70 ml

Subtraindo uma equação da outra, temos: 2x = 20 ml


x = 10 ml
𝑥 10
Portanto, a fração de C3H8 na mistura é 50 = 50 = 0,20 (20%)

Questão 47 – (GRILLO) Foram tratados 16 gramas de uma soda comercial, composta basicamente por carbonato
de sódio e ácido clorídrico. Após um determinado tempo, houve liberação de um determinado gás que, medido na
pressão de 1,5 atm, preencheu um determinado recipiente de 2,2 litros na temperatura de 27ºC. Qual o grau de
pureza da soda? Resolução:
p x VCO2
Cálculo do número de mol de dióxido de carbono, a partir da equação dos gases ideais: nCO2 = =
RxT
1,50 x 2,20
0,08206 x (27+273)
= 0,134 mol

Cálculo da massa do número de massa de dióxido de carbono: mCO2 = nCO2 x <MM>CO2


mCO2 = 0,134 x 44 = 5,90 g

Cálculo da massa de soda comercial: 2 HCl + Na2CO3 → 2 NaCl + H2O + CO2


1 mol de Na2CO3 ----------------------- 1 mol de CO2
106 gramas de Na2CO3 ---------------- 44 gramas de CO2
mNa2CO3 ----------------------------------- 5,90 gramas de CO2
mNa2CO3 = 14,21 g
14,21 𝑔
Cálculo do grau percentual de pureza da soda comercial: %𝑁𝑎2 𝐶𝑂3 = 16 𝑔
= 0,8888 (88,88%)

Questão 48 – (GRILLO) Considere a seguinte reação de produção de ácido sulfúrico e monóxido de nitrogênio a
partir do ácido nítrico, conforme pode ser observado pela equação química não balanceada: As2S3 + HNO3 + H2O →
H2SO4 + H3AsO4 + NO. Partindo de 1000 gramas de ácido nítrico, calcule a massa de ácido sulfúrico produzido e o
volume de NO nas CNTP.

Resolução: O balanceamento desta reação deverá ser feita pelo métdo de oxidação-redução: 3 As2S3 + 28 HNO3 +
4 H2O → 9 H2SO4 + 6 H3AsO4 + 28 NO

Dados do problema: Massa de HNO3 = 1000 g; Massa de H2SO4 = ? e Volume de NO nas CNTP = ?

Cálculo da massa de H2SO4: 3 As2S3 + 28 HNO3 + 4 H2O → 9 H2SO4 + 6 H3AsO4 + 28 NO


28 mol de HNO3 ---------- 9 mol de H2SO4
28 x 63 g ------------------- 9 x 98 g
1000 g ---------------------- mH2SO4
mH2SO4 = 500 g

Cálculo do volume de NO nas CNTP: 3 As2S3 + 28 HNO3 + 4 H2O → 9 H2SO4 + 6 H3AsO4 + 28 NO


28 mol de HNO3 ---------------------------------------------- 28 x 22,4 L de NO
(1000/63) ------------------------------------------------------- VNO
VNO = 355,55 L
Questão 49 – (OLIMPÍADA DE QUÍMICA DO RIO GRANDE DO SUL) O cloreto de hidrogênio é um gás tóxico, que
pode ser fabricado por meio da reação entre os gases hidrogênio e cloro. Quando dissolvido em água, recebe o nome
de ácido clorídrico. Soluções aquosas de cloreto de hidrogênio, razoavelmente impuras, são vendidas comercialmente

29
1600 FÍSICO-QUÍMICA APLICADA EXERCÍCIOS COMENTADOS - IME – ITA – OLIMPÍADA

com o nome de ácido muriático e são frequentemente usadas para limpeza de pisos e paredes. Soluções de ácido
muriático podem ser neutralizadas por cal hidratada ou por soda cáustica. Baseado no texto acima faça as reações:
a) De obtenção do cloreto de hidrogênio.
b) Da dissolução do cloreto de hidrogênio em água.
c) Das duas reações de neutralização do ácido muriático.

Resolução:

Item a) Equação química para a obtenção do cloreto de hidrogênio: H2(g) + Cl2(g) → 2 HCl(g)
Item b) Equação de dissolução do cloreto de hidrogênio: HCl + H2O → Cl- + H3O+
Item c) Primeira reação de neutralização: 2 HCl + Ca(OH)2 → CaCl2 + 2 H2O
Segunda equação química: HCl + NaOH → NaCl + H2O

Questão 50 – (ITA) Quantos mL de solução 0,200 mol.L-1 de ácido sulfúrico são necessários para neutralizar 0,200
gramas de hidróxido de magnésio?

Resolução: Equação química: H2SO4(aq) + Mg(OH)2(aq) → MgSO4(aq) + H2O(l)


1 mol de H2SO4 ---------- 1 mol de Ca(OH)2
nH2SO4 ---------------------- nMg(OH)2
mMg(OH)2
[H2 SO4 ] x Vsolução =
< MM >Mg(OH)2

0,200
0,200 x Vsolução =
58

Vsolução = 0,01724 L (17,24 mL)

Questão 51 – (ITA) Uma amostra de hidróxido de cálcio impuro, pesando 2,00 gramas, foi neutralizada por 50,0 mL
de ácido sulfúrico 0,500 mol.L-1. Qual a pureza da amostra?

Resolução: Equação química: H2SO4(aq) + Ca(OH)2(aq) → CaSO4(aq) + H2O(l)

1 mol de H2SO4 ---------- 1 mol de Ca(OH)2


nH2SO4 ---------------------- nCa(OH)2
mCa(OH)2
[H2 SO4 ] x Vsolução =
< MM >Ca(OH)2

mCa(OH)2
0,500 x 50 x 10−3 =
74

mCa(OH)2 = 0,500 x 50 x 10−3 x 74 = 1,85 g

1,85 g
Cálculo da pureza de hidróxido de cálcio (%): %Ca(OH)2 = = 0,925 (92,50 %)
2,0 g

Questão 52 – (OLIMPÍADA BRASILEIRA DE QUÍMICA - MODIFICADA) Indique as funções orgânicas oxigenadas


e calcule a percentagem de carbono contida em uma molécula de aspartame?

30
1600 FÍSICO-QUÍMICA APLICADA EXERCÍCIOS COMENTADOS - IME – ITA – OLIMPÍADA

Resolução: Identificação das funções orgânicas oxigenadas: Através da figura abaixo, estão identificadas através
de círculos, as funções orgânicas.

Cálculo do percentual de carbono na molécula de aspartame (C14H18O5N2; 292 g,mol-1)

292 g,mol-1 ---------- 100%


168 g,mol-1 ---------- (%)C
(%)C = 57,53%

Questão 53 – (OLIMPÍADA MINEIRA DE QUÍMICA) A reação química que ocorre entre a glicose e o oxigênio produz
dióxido de carbono e água. A equação química que descreve esta reação é representado por: C6H12O6(s) + O2(g) →
CO2(g) + H2O(l).
a) Após o balanceamento da equação química, calcule a massa de oxigênio necessária para reagir
completamente com 25,0 gramas de glicose.
b) Calcule as massas de CO2 e H2O formados quando 2,50 mol de glicose reagem completamente com uma
quantidade suficiente de oxigênio disponível no meio reacional.

Resolução: Item a) Cálculo da massa de oxigênio: C6H12O6(s) + 6 O2(g) → 6 CO2(g) + 6 H2O(l)

1 mol de C6H12O6(s) -------------------- 6 mol de O2(g)


180 gramas ----------------------------- (6 x 32) gramas
25,0 gramas ---------------------------- mO2
4800
mO2 = 180 = 26,67 g de O2

Item b) Cálculo do número de mol de CO2 e H2O: C6H12O6(s) + 6 O2(g) → 6 CO2(g) + 6 H2O(l)

1 mol de C6H12O6(s) --------------- 6 mol de CO2(g) ---------- 6 mol de H2O(l)


2,50 mol de C6H12O6(s) ----------- nCO2 ------------------------ nH2O
nCO2 = 15,00 mol
nH2O = 15,00 mol

Cálculo da massa de cada produto:

Dióxido de carbono: mCO2 = nCO2 x < MM >CO2 = 15,0 x 44,0 = 660 g

31
1600 FÍSICO-QUÍMICA APLICADA EXERCÍCIOS COMENTADOS - IME – ITA – OLIMPÍADA

Água: mH2 O = nH2 O x < MM >H2 O = 15,00 x 18,00 = 270,0 g

Questão 54 – (GRILLO) Determine o volume de ar, nas CNTP, que fornece o oxigênio necessário a combustão não
catalisada de 15 mol de amônia.

Resolução: Cálculo do número de mol de gás nitrogênio, considerando que o ar apresenta em sua composição –
20% de Oxigênio e 80% de nitrogênio:

5/2 mol de O2(g) ----------- 20%


nN2 -------------------------- 80%
nN2 = 10 mol de N2(g)

Equação química balanceada com a presença do gás nitrogênio: 2 NH3(g) + 5/2 O2(g) + 10 N2(g) → 2 NO(g) + 3 H2O(g) +
10 N2(g).

Cálculo do volume de ar:


2 mol de NH3(g) ------------ 12,5 mol de ar
15 mol de NH3(g) ---------- nar
nar = 93,75 mol

Cálculo do volume de ar, nas CNTP: Var = nar x (22,4 L.mol-1) = 93,75 x 22,4 = 2100 L

Questão 55 – (ITA) Quando relâmpagos ocorrem na atmosfera, energia suficiente é fornecida para a iniciação da
reação de nitrogênio com oxigênio, gerando monóxido de nitrogênio, o qual, em seguida, interage com oxigênio,
gerando dióxido de nitrogênio, um dos responsáveis pela acidez de chuvas.
a) Escreva a equação química, balanceada, de cada uma das três transformações mencionadas no enunciado.
b) Descreva o método industrial utilizado para obter ácido nítrico. De sua descrição deve constar a matéria-
prima utilizada, as equações químicas balanceadas para reações que ocorrem durante cada etapa do processo e a
concentração (em % m/m) do ácido vendido comercialmente.
c) Cite três aplicações para o ácido nítrico.

Resolução: Item a) Equações químicas:


N2(g) + O2(g) → 2 NO(g)
2 NO(g) + O2(g) → 2 NO2(g)
2 NO2(g) + H2O(l) → HNO2(aq) + HNO3(aq)
3 NO2(g) + H2O(l) → 2 HNO3(aq) + NO(g)

Item b) Matérias-primas: Amônia (NH3), Ar atmosférico, oxigênio (O2), água (H2O) e platina (Pt).
Etapa I: 4 NH3(g) + 5 O2(g) → 4 NO(g) + 6 H2O(g)
Etapa II: 2 NO(g) + O2(g) → 2 NO2(g)
Etapa III: 3 NO2(g) + H2O(l) → 2 HNO3(aq) + NO(g)

Item c) Fertilizantes, corantes e nylon.

32
1600 FÍSICO-QUÍMICA APLICADA EXERCÍCIOS COMENTADOS - IME – ITA – OLIMPÍADA

Questão 56 - (ITA) Descreva um método de preparação do ácido nítrico economicamente viável e utilizado pelas
indústrias químicas modernas para a produção em grande escala. Utilize equações balanceadas para representar as
reações químicas que ocorrem com o emprego do método proposto.

Resolução: As equações químicas balanceadas estão apresentadas a seguir:

2 NH3(g) + 5/2 O2(g) → 2 NO(g) + 3 H2O(g)


5 NO(g) + ½ O2(g) → NO2(g)
3 NO2(g) + H2O → 2 HNO3 + NO(g)

Questão 57 – (OLIMPÍADA IBEROAMERICANA DE QUÍMICA) Nem todos os hidretos covalentes podem ser obtidos
por meio de processos exotérmicos, e muitos deles devem ser obtidos por outros métodos, tal como a protonação de
uma base de Brønsted. Desta forma, o PH3 pode ser sintetizado segundo a equação seguinte: Ca3P2 + 3 H2SO4 →
2PH3 + 3 CaSO4
a) Escreva os nomes, segundo a IUPAC, das espécies seguintes: PH3 e H2SO4.
b) Sabendo que o rendimento deste processo é de 80,00%, qual a massa de Ca 3P2 , com 90,00% de pureza, é
necessária para preparar 20,0 g de PH3?
c) Sabendo que o rendimento deste processo é de 80%, qual o volume de solução de H2SO4 de concentração 2,00
mol/L será necessário para preparar 20,0 g de PH3?
d) No laboratório, você dispõe de H2SO4 comercial com 98,08% de pureza e densidade 1,84 g/mL. Que volume de
H2SO4 comercial será necessário para preparar o volume da solução 2,00 mol/L calculado no item anterior.

Resolução: Item a) PH3 → Hidreto de fósforo ou também conhecido como fosfina; H2SO4 → Ácido sulfúrico.

Item b) Cálculo da massa de fosfeto de cálcio (Ca3P2): Ca3P2 + 3 H2SO4 → 2PH3 + 3 CaSO4
1 mol de Ca3P2 ------------------- 2 mol de PH3
182 g de Ca3P2 ------------------- 68 g de PH3 x (0,80)
0,90 x mCa3P2 --------------------- 20 g de PH3
mCa3P2 = 74,35 g

Item c) Ca3P2 + 3 H2SO4 → 2PH3 + 3 CaSO4


3 mol de H2SO4 ---------------- 2 mol de PH3 x 0,80
nH2SO4 ---------------------------- nPH3

1,60 x nH2SO4 = 3 x nPH3


1,60 x [H2SO4] x Vsolução = 3 x [mPH3 / <MM>]
1,60 x 2,0 x Vsolução = 3 x [20,0 / 34]
Vsolução = 0,551 L

Item d) O volume calculado do ácido sulfúrico no item (c) é igual a 0,551 litros, para uma concentração de quantidade
de matéria igual a 2,00 mol.L-1. Logo:

Informações do problema: 98,08 g de H2SO4 para cada 100 g de solução e 1,84 g de solução para cada 1 mL de
solução

Cálculo do volume de solução:


1,84 g de solução --------- 1 mL de solução
100 g de solução ---------- Vsolução
Vsolução = 54,35 mL (54,35 x 10-3 L)

H SO n mH2SO4
Cálculo da concentração da quantidade de matéria da H2SO4: [H2 SO4 ] = V 2 4 = <MM> x Vsolução
=
solução H2 SO4
98,08
= 18,41 mol. L−1
98 x 54,35 x−3 x10−3

Cálculo do volume de ácido sulfúrico para 18,41 mol.L-1: [H2 SO4 ] x Vsolução = 2 x 0,551

33
1600 FÍSICO-QUÍMICA APLICADA EXERCÍCIOS COMENTADOS - IME – ITA – OLIMPÍADA

18,41 x Vsolução = 2 x 0,551

Vsolução = 5,98 x 10−2 L

Questão 58 – (IME) Escreva a equação química, representativa, da preparação do cloro, em laboratório, figurando
entre os reagentes as substâncias H2SO4 e MnO2.

Resolução: Equação química: 2 H2SO4 + 2 NaCl + MnO2 → MnSO4 + NaSO4 + Cl2 + 2 H2O.

Questão 59 – (IME) Escreva as reações correspondentes à fabricação do ácido sulfúrico, pelo processo de contato,
a partir da pirita.

Resolução: As reações que correspondem a produção de ácido sulfúrico estão apresentadas pelas equações
químicas balanceadas a seguir:
2 FeS2 + 11/2 O2 → Fe2O3 + 4 SO2
SO2 + ½ O2 → SO3
SO3 + H2O → H2SO4

Questão 60 – (ITA) Explique o que se entende por chuva ácida. Quais são as causas desse problema? Quais as
formas de controlá-lo?

Resolução: Nos grandes centros metropolitanos, onde o tráfego é muito intenso, o ar é poluído, particularmente, com
CO, SO2, óxidos de nitrogênio, etc. O SO2 na atmosfera se oxida a SO3, que reagindo com a água produz H2SO4:
SO2 + 1/2SO2 → SO3
SO3 + H2O → H2SO4

Os óxidos do nitrogênio NO e NO2 formam o ácido nítrico (HNO3):


2 NO + O2 → 2NO2
3 NO2 + H2O → 2HNO3 + NO

Esses ácidos na água da chuva é que correspondem à chuva ácida.


Esses ácidos ocorrem também na atmosfera de regiões fabris, como consequência de despejos gasosos industriais.

As chuvas ácidas atacam a vegetação, tornam o solo ácido e causam sérios danos à Agricultura. Os monumentos de
mármore também são danificados, conforme as equações:
H2SO4 + CaCO3(mármore) → CaSO4 + H2O + CO2
2 NHO3 + CaCO3 → Ca(NO3)2 + H2O + CO2

A forma de controlar o problema é utilizar nas indústrias equipamentos especiais para eliminar os poluentes, como
precipitador Cotrell nas chaminés. Atualmente os automóveis têm catalisadores que atuam sobre os gases da
descarga, acelerando a transformação dos óxidos de nitrogênio em N2 e do CO em CO2.

Questão 61 – (IME) Foram misturados 40 gramas de hidrogênio (H2) com 40 gramas de oxigênio (O2), com a finalidade
de produzir água, segundo a equação H2(g) + O2(g) → H2O(g). Determine:
a) o reagente limitante;
b) a massa do produto formado;
c) a massa do reagente em excesso.

Resolução: Item a) Equação química: H2(g) + ½ O2(g) → H2O(g).


mH 40
Cálculo do número de mol de H2: nH2 = <MM>2 = 2
= 20 mol
H2

34
1600 FÍSICO-QUÍMICA APLICADA EXERCÍCIOS COMENTADOS - IME – ITA – OLIMPÍADA

mO 40
Cálculo do número de mol de O2: nO2 = <MM>2 = 32 = 1,25 mol
O2

1/2 mol de O2 -------------------- 1,25 mol


n O2 -------------------------------- 1,0 mol
nO2 = 2,50 mol

Reagente limitante: O2(g)


Reagente em excesso: H2(g)

Item b) Cálculo da massa de água formada:


½ mol de O2 --------------- 1 mol de H2O
(½ x 32)g de O2 ----------- (1 x 18) g de H2O
40g de O2 ------------------ MassaH2O
MassaH2O = 45 g

Item c) Quantidade que sobra de gás hidrogênio, H2:


1 mol de H2 -------------------- 1/2 mol de O2
2,0 g ----------------------------- ½ x 32 g
Massa ---------------------------- 40,0 g
Massa = 5,00 g (reagiu)

Não reagido (Sobra) = 40,00 – 5,00 = 35,0 g

Questão 62 – (IME) Uma fonte de vanádio é o mineral vanadinita, cuja fórmula é Pb5(VO4)3Cl. Determine:
a) a porcentagem em massa de vanádio nesse mineral;
b) a massa em gramas de vanádio numa amostra que contém 2,4 x 1024 átomos de cloro.

Resolução: Sendo a fórmula molecular da vanadinita Pb5(VO4)3Cl, será necessário o cálculo da sua massa molecular:

Item a) Massa molar da vanadinita: <MM>vanadinita = 1415,5 g.mol-1

< MM >vanádio 51,0 x 3


(%)V = ( ) x 100 = ( ) x 100 = 0,1081 (10,81%)
< MM >vanadinita 1415,5

Item b) 1 molécula ------ 1 átomo de Cl → número de mol de Pb5(VO4)3Cl = nº de mols Cl

1 mol ---------- 6,02 x 1023 átomos


nCl -------------- 2,40 x 1024 átomos
nCl = 3,99 mol de átomos de cloro

1 molécula de vanadinita apresenta 3 átomos de Vanádio


1 molécula de vanadinita apresenta 1 átomo de cloro
m
Conclusão: nvanádio = <MM>
vanádio
= 3 x ncloro
vanádio

mvanádio = 3 x ncloro x < MM >vanádio = 3 x 3,99 x 51 = 610,47 g

Questão 63 – (GRILLO) Calcule o volume de ar a 27°C e pressão de 850 mm Hg, necessário para a ustulação de
400 g de pirita com 90% de pureza, cuja equação química não-balanceada é a seguinte: FeS2 + O2 → Fe2O3 + SO2.

Resolução: Equação química: 4 FeS2 + 11 O2 → 2 Fe2O3 + 8 SO2

Considerando que o ar apresenta composição igual a 21% de oxigênio e 79% de gás nitrogênio, temos:

35
1600 FÍSICO-QUÍMICA APLICADA EXERCÍCIOS COMENTADOS - IME – ITA – OLIMPÍADA

11 mol de O2(g) ---------- 21%


nN2 ------------------------ 79%
nN2 = 41,38 mol de N2(g)

Equação química balanceada com a presença de gás nitrogênio: 4 FeS2 + 11 O2 + 41,38 N2(g) → 2 Fe2O3 + 8 SO2 +
41,38 N2(g).

Número de mol de ar = 11 mol de O2 + 41,38 mol de N2 = 52,38 mol de ar.

Cálculo do número de mol de ar: 4 FeS2 + 11 O2 + 41,38 N2(g)→ 2 Fe2O3 + 8 SO2 + 41,38 N2(g)

4 mol de FeS2 --------------------- 52,38 mol de ar


400 𝑥 0,90
( ) mol de FeS2 ---------- nar
120
nar = 39,28 mol de ar

39,28 x 0,08206 x (27+273)


Cálculo do volume de ar, considerando comportamento ideal: Var = 850 =
( )
760
39,28 x 0,08206 x 300 x 760
850
= 864,61 L

Questão 64 – (OLIMPÍADA DE QUÍMICA DO RIO GRANDE DO SUL) O sulfato de alumínio é o mais tradicional
agente coagulante usado no tratamento de água para consumo humano. É empregado na etapa de clarificação, onde
provoca a aglutinação da matéria em suspensão, formando flocos que ganham em densidade e sedimentam-se. Pode
ser obtido pela reação entre cloreto de alumínio e ácido sulfúrico.
a) Faça a reação balanceada de obtenção desse composto.
b) Partindo-se de 534 gramas de sal, com 75% de pureza, calcular a massa de sulfato de alumínio obtida,
considerando rendimento de 90%.
c) Calcule a quantidade de matéria do enxofre existente em 855 gramas de sulfato de alumínio puro.
d) Faça a reação balanceada de obtenção do sulfato de alumínio, substituindo o sal de alumínio por sua
respectiva base.

Resolução: Item a) Equação química de formação do sulfato de alumínio: 2 AlCl3 + 3 H2SO4 → Al2(SO4)3 + 6 HCl

Item b) Massa de sal puro de cloreto de alumínio = 534 gramas x 0,75 = 400,50 g

2 AlCl3 + 3 H2SO4 → Al2(SO4)3 + 6 HCl


2 mol de AlCl3 -------------- 1 mol de Al2(SO4)3
267 gramas ------------------ 342 gramas x (0,90)
400,50 gramas -------------- mAl2(SO4)3
mAl2(SO4)3 = 461,70 g

Item c) Cálculo do número de mol de enxofre:


1 mol de Al2(SO4)3 ---------- 342 gramas ---------- 3 mol de átomos de S
855 gramas ---------- nS
nS = 7,50 mol de átomos

Item d) Equação química: 3 H2SO4(aq) + 2 Al(OH)3(aq) → Al2(SO4)3 + 6 H2O(l).

36
1600 FÍSICO-QUÍMICA APLICADA EXERCÍCIOS COMENTADOS - IME – ITA – OLIMPÍADA

Questão 65 – (GRILLO) Uma determinada amostra com 6,0 g de mercúrio foi cuidadosamente adicionada a uma
solução com ácido nítrico diluído, em condições reacionais. Depois de completada a reação química, determine o
volume do gás que se desprendeu, medida nas condições normais de temperatura e pressão (CNTP).

Resolução: Equação química: 3 Hg + 8 HNO3 → 3 Hg(NO3)2 + 4 H2O + 2 NO

Cálculo do volume de monóxido de nitrogênio, nas CNTP: 3 Hg + 8 HNO3 → 3 Hg(NO3)2 + 4 H2O + 2 NO

3 mol de Hg ----------------------------------- 2 mol de NO


(3 x 200) g ------------------------------------- 2 x 22, 4 L
6,0 g ------------------------------------------ V
V = 448 L de NO

Questão 66 – (IME) O processo Solvay de produção de carbonato de sódio realiza-se mediante as reações abaixo:
CaCO3 → CaO + CO2
CaO + H2O → Ca(OH)2
NH3 + H2O → NH4OH
2 NH4OH + CO2 → (NH4)2CO3 + H2O
(NH4)2CO3 + CO2 + H2O → 2 NH4HCO3
NH4HCO3 + NaCl → NH4Cl + NaHCO3
2 NaHCO3 → Na2CO3 + CO2 + H2O
2 NH4Cl + Ca(OH)2 → 2 NH3 + CaCl2 + 2 H2O
A partir destas equações, determine:
a) a reação global que representa o processo;
b) a massa de cada reagente que é necessária para produzir 1.000 kg de carbonato de sódio.

Resolução: Item a) Analisando as equações químicas:


CaCO3 → CaO + CO2
CaO + H2O → Ca(OH)2
2 NH3 + 2 H2O → 2 NH4OH (Equação Multiplicada por dois)
2 NH4OH + CO2 → (NH4)2CO3 + H2O
(NH4)2CO3 + CO2 + H2O → 2 NH4HCO3
2 NH4HCO3 + 2 NaCl → 2 NH4Cl + 2 NaHCO3 (Equação Multiplicada por dois)
2 NaHCO3 → Na2CO3 + CO2 + H2O
2 NH4Cl + Ca(OH)2 → 2 NH3 + CaCl2 + 2 H2O +
CaCO3 + 2 NaCl → Na2CO3 + CaCl2

Item b) Cálculo da massa de cada reagente:

1 mol CaCO3 ---------- 2 mol de NaCl ---------- 1 mol de Na2CO3


100 g -------------------- 2 x 58,5 g ------------- 106 g
mCaCO3 ------------------ mNaCl ------------------- 1000 kg
mCaCO3 = 943,40 kg e mNaCl = 1103,77 kg

Questão 67 – (OLIMPÍADA DE QUÍMICA DO RIO GRANDE DO SUL) Uma amostra de 12,5 gramas de calcário
(CaCO3, impuro) foi calcinada e o resíduo obtido adicionado a 1,0 litro de água. Após filtração, borbulhou-se anidrido
sulfúrico na solução resultante, precipitando 13,6 gramas de sulfato de cálcio. Responda:
a) Escreva todas as reações que ocorrem.
b) Qual a pureza do calcário?

Resolução: Reação Química I: Calcinação do carbonato de cálcio: CaCO3(s) → CaO(s) + CO2(g)

Equação Química II: Diluição do resíduo (CaO): CaO(s) + H2O(l) → Ca(OH)2(aq)

Equação Química III: Anidrido Sulfúrico + hidróxido de cálcio: SO3(g) + Ca(OH)2(aq) → CaSO4 + H2O(l)

37
1600 FÍSICO-QUÍMICA APLICADA EXERCÍCIOS COMENTADOS - IME – ITA – OLIMPÍADA

Somando as equações químicas:


CaCO3(s) → CaO(s) + CO2(g)
CaO(s) + H2O(l) → Ca(OH)2(aq)
SO3(g) + Ca(OH)2(aq) → CaSO4 + H2O(l) +
CaCO3(s) + SO3(g) → CO2(g) + CaSO4 (Equação Química Global)

Questão 68 – (OLIMPÍADA IBEROAMERICANO DE QUÍMICA) Nem todos os hidretos covalentes podem ser obtidos
por meio de processos exotérmicos, e muitos deles devem ser obtidos por outros métodos, tal como a protonação de
uma base de Brønsted. Desta forma, o PH3 pode ser sintetizado segundo a equação seguinte: Ca3P2 + 3 H2SO4 → 2
PH3 + 3 CaSO4.
a) Escreva os nomes, segundo a IUPAC, das espécies seguintes: PH3 e H2SO4.
b) Sabendo que o rendimento deste processo é de 80,00%, qual a massa de Ca3P2, com 90,00% de pureza é
necessária para preparar 20,0 gramas de PH3?
c) Sabendo que o rendimento deste processo é de 80%, qual o volume de solução de H2SO4 de concentração 2,00
mol.L-1 será necessário para preparar 20,0 gramas de PH3? .
d) No laboratório, você dispõe de H2SO4 comercial com 98,08% de pureza e densidade 1,84 g.ml-1. Que volume de
H2SO4 comercial será necessário para preparar o volume da solução 2,00 mol.L-1 calculado no item anterior.

Resolução: Item a) PH3 = Hidreto de fósforo e H2SO4 = Ácido sulfúrico

Item b) Equação química: Ca3P2 + 3 H2SO4 → 2 PH3 + 3 CaSO4.


1 mol de Ca3P2 ---------- 2 mol de PH3
1 mol de Ca3P2 ---------- 2 mol de PH3
182 g ---------------------- 68 g x 0,80
0,90 x mCa3P2 ------------ 20,0 g
mCa3P2 = 74,35 g

Item c) Equação química: Ca3P2 + 3 H2SO4 → 2 PH3 + 3 CaSO4.

3 mol de H2SO4 ---------- 2 mol de PH3 x 0,80


nH2SO4 ---------------------- nPH3
1,60 x nH2SO4 = 3 x nPH3
mPH3
Cálculo do volume da solução de ácido sulfúrico: 1,60 x [H2 SO4 ] x Vsolução = 3 x <MM>PH3
20
1,60 x 2,0 x Vsolução =3x
34

Vsolução = 0,551 L

Item d) Cálculo da concentração da quantidade de matéria de ácido sulfúrico comercial: [H2 SO4 ]comercial =
CH2 SO4 10 x d x (%) 1804,67
<MM>H2 SO4
= <MM> = 98
= 18,41 mol. L−1
H2 SO4

comercial
Cálculo do volume de ácido sulfúrico comercial: [H2 SO4 ] x Vsolução = [H2 SO4 ]comercial x Vsolução

comercial
2,0 x 0,56 = 18,41 x Vsolução

comercial
Vsolução = 0,061 L (61 mL)

Questão 69 – (OLIMPÍADA MINEIRA DE QUÍMICA) A reação química que ocorre entre a glicose e o oxigênio produz
dióxido de carbono e água. A equação química que descreve esta reação é representada por: C6H12O6(s) + O2(g) →
CO2(g) + H2O(l).

38
1600 FÍSICO-QUÍMICA APLICADA EXERCÍCIOS COMENTADOS - IME – ITA – OLIMPÍADA

a) Após o balanceamento da equação química, calcule a massa de oxigênio necessária para reagir
completamente com 25,0 gramas de glicose.
b) Calcule as massas de CO2 e H2O formadas quando 2,50 mol de glicose reagem completamente com uma
quantidade suficiente de oxigênio disponível no meio reacional.

Resolução: Item a) Cálculo da massa de gás oxigênio: C6H12O6(s) + 6 O2(g) → 6 CO2(g) + 6 H2O(l)

1 mol de C6H12O6 ------------- 6 mol de O2


180 gramas --------------------- 192 gramas de O2(g)
25,0 gramas -------------------- mO2
mO2 = 26,67 g

Item b) Cálculo da massa CO2(g) e H2O(l):

1 mol de C6H12O6(s) -------------- 6 mol de CO2 ---------- 6 mol de H2O


2,50 mol de C6H12O6(s) ---------- nCO2 ---------------------- nH2O

nCO2 = 15,0 mol e nH2O = 15,0 mol

Massa de CO2: mCO2 = nCO2 x <MM>CO2 = 15 x 44 = 660 g

Massa de H2O: mH2O = nH2O x <MM>H2O = 15 x 18 = 270 g

Questão 70 – O carbonato de sódio empregado na fabricação de vidro é preparado a partir do carbonato de cálcio e
cloreto de cálcio, segundo a equação química não balanceada: CaCO3 + NaCl → Na2CO3 + CaCl2. Partindo-se de
1250 gramas de carbonato de cálcio (com grau de pureza de 80&) e 650 gramas de cloreto de sódio (com grau de
pureza 90%), determine a massa de carbonato de sódio, levando em consideração um rendimento igual a 80%.
mCaCO3 1250
Resolução: Cálculo do número de mol de carbonato de cálcio: nCaCO3 = <MM> = 100
= 12,50 mol
CaCO3

mNaCl 650 11,11


Cálculo do número de mol de cloreto de sódio: nNaCl = = = = 5,55 mol
<MM>NaCl 58,5 2

Massa de cloreto de sódio puro (reagente limitante): mNaCl = 650 x 0,90 = 585 g

Através da estequiometria da reação, temos: CaCO3 + 2 NaCl → Na2CO3 + CaCl2


2 mol de NaCl ----------------- 1 mol de Na2CO3
117 gramas de NaCl ---------- 106 gramas de Na2CO3
585 gramas de NaCl ---------- mNa2CO3
mNa2CO3 = 530 g

Considerando que o rendimento seja igual a 80%, temos:


530 gramas ---------- 100%
mNa2CO3 --------------- 80%
mNa2CO3 = 424 g
Questão 71 – (IME) Uma forma de sintetizar óxido nítrico em meio aquoso é reagir nitrito de sódio com sulfato
ferroso e ácido sulfúrico, produzindo, além do óxido nítrico, sulfato férrico e bissulfato de sódio. Partindo de 75,0
gramas de nitrito de sódio, 150,0 gramas de ácido sulfúrico e 152,0 gramas de sulfato ferroso e tendo a reação 90%
de rendimento, determine a massa de óxido nítrico obtida.

Resolução: Equação química balanceada pelo método redox: 2 NaNO2 + 2 FeSO4 + 3 H2SO4 → 2 NO + Fe2(SO4)3
+ 2 NaHSO4 + 2 H2O.

Determinação do reagente limitante:

39
1600 FÍSICO-QUÍMICA APLICADA EXERCÍCIOS COMENTADOS - IME – ITA – OLIMPÍADA

mNaNO2 75 1,09
nNaNO2 = = = = 0,545 mol
< MM >NaNO2 69 2

mFeSO4 152 1,0


nFeSO4 = = = = 0,50 mol (reagente limitante)
< MM >FeSO4 152 2

mH2 SO4 150 1,53


nH2 SO4 = = = = 0,51 mol
< MM >H2 SO4 98 3

Cálculo da massa de NO: 2 NaNO2 + 2 FeSO4 + 3 H2SO4 → 2 NO + Fe2(SO4)3 + 2 NaHSO4 + 2 H2O

3 mol de H2SO4 ---------------------- 2 mol de NO


(3 x 98) gramas ---------------------- (2 x 30) gramas x R
(3 x 98) gramas ---------------------- (2 x 30) gramas x 0,90
152 gramas --------------------------- mNO
mNO = 27,92 g

Questão 72 – (IME) Um calcário composto por MgCO3 e CaCO3 foi aquecido para produzir MgO e CaO. Uma amostra
de 2,00 gramas desta mistura de óxidos foi tratada com 100 cm³ de ácido clorídrico 1,00 mol.L -1. Sabendo-se que o
excesso de ácido clorídrico necessitou de 20,0 cm³ de solução de NaOH 1,00 mol.L-1 para ser neutralizado, determine
a composição percentual, em massa, de MgCO3 e CaCO3 na amostra original desse calcário.

Resolução: A reação de neutralização entre ácido clorídrico e hidróxido de sódio é a representada pela seguinte
equação química balanceada: HCl(aq) + NaOH(aq) → H2O(l) + NaCl(aq). Observando a estequiometria da reação, 1 mol
de ácido clorídrico consome 1 mol de hidróxido de sódio, 20.10-3 mol de NaOH (20,0 cm3 de solução de NaOH 1,00
mol.L-1) reagem com 20 x 10-3 mol de HCl, ou seja, houve reação de 8 x 10-2 mol do ácido com os óxidos de acordo
com as seguintes equações químicas:
2 HCl(aq) + CaO(s) → H2O(l) + CaCl2(aq)
2 HCl(aq) + MgO(s) → H2O(l) + MgCl2(aq)

Assim, sabe-se que 8 x 10-2 mol de HCl corresponderão à soma de 2x mol de CaO e 2y mol de MgO. Uma vez que 2
gramas da mistura dos óxidos correspondem a (56,1 g.mol-1.x mol de CaO) + (40,3 g.mol-1.y mol de MgO) então:
x = 4.10-2 - y
56,1 x (4.10-2 – y) + 40,3y = 2

Resolvendo a equação matemática acima, temos: y = 0,01544 mol (MgO) e x = 0,02456 mol (CaO).

Considerando a decomposição dos carbonatos, para cada 1 mol de carbonato será gerado 1 mol do óxido de acordo
com as seguintes equações químicas balanceadas:
CaCO3(s) ⎯⎯→ CaO(s) + CO2(g)
MgCO3(s) ⎯⎯→ MgO(s) + CO2(g)

Conclusão: Na mistura inicial há 0,01544 mol de MgCO3 (1,3016 g) e 0,02456 mol CaCO3 (2,4585 g), ou seja, 34,62%
de MgCO3 e 65,38% de CaCO3.

Questão 73 – (OLIMPÍADA NORTE – NORDESTE DE QUÍMICA) Os antiácidos são medicamentos usados para a
acidez estomacal e aliviar a dor de distúrbios estomacais e duodenais, causados principalmente pela ingestão de
frutas cítricas e cafeína. O “leite de magnésia” é um dos antiácidos mais utilizados e trata-se de uma suspensão de
hidróxido de magnésio que contém de 7,0 a 8,5 g do hidróxido em cada 100 g, segundo a United States Pharmacopeia.
Na determinação do teor de hidróxido de magnésio, a 1,0 g de amostra de leite de magnésia foi adicionado 50 mL de
ácido clorídrico padronizado 0,102 mol.L-1e duas gotas de indicador vermelho de metila. Esta solução foi titulada com
21,9 mL de solução padrão de hidróxido de sódio 0,100 mol.L-1. Com base nestas informações, resolva as questões
abaixo:
a) Equacione a reação balanceada de neutralização estomacal.
b) Calcule a porcentagem do hidróxido de magnésio, em massa, da amostra.

40
1600 FÍSICO-QUÍMICA APLICADA EXERCÍCIOS COMENTADOS - IME – ITA – OLIMPÍADA

c) Sabendo que o sulfato de alumínio é utilizado, também, como agente floculante, no tratamento de água e
efluentes, equacione a reação balanceada de obtenção do sulfato de alumínio a partir do hidróxido de alumínio.

Resolução: Equação química: 2 HCl + Mg(OH)2 → MgCl2 + 2 H2O


Item a) Cálculo do volume de HCl em excesso: [HCl]1 x V1 = [HCl]2 x V2
0,102 x V1 = 0,100 x 21,9
V1 = 21,47 mL

Cálculo do volume de HCl que reagiu: VHCl reagiu = 50,0 – 21,47 = 28,53 mL
Cálculo do número de mol de HCl que reagiu: nHCl = [HCl] x Vsolução = 0,102 x (28,53 / 1000) = 0,0029 mol

Através da estequiometria, temos:


2 mol de HCl ------------------ 1 mol de Mg(OH)2
0,0029 mol --------------------- nMg(OH)2
nMg(OH)2 = 0,0015 mol
Cálculo da massa de hidróxido de magnésio: mMg(OH)2 = [Mg(OH)2] x Vsolução
mMg(OH)2 = 0,0015 x 58,32 = 0,085 g

Cálculo do percentual:
1 g --------------- 100 %
0,085 g ---------- (%)
(%) = 8,5%

Item b) Equação química balanceada para a formação do sulfato de alumínio: 2 Al(OH)3 + 3 H2SO4 → Al2(SO4)3 + 6
H2O.

Questão 74 – (IME) Uma amostra de 1,850 g de uma mistura de CuCl2 e CuBr2 foi dissolvida em água e totalmente
misturada com uma porção de 1,800 g de AgCl. Após a reação, o sólido, que agora consiste numa mistura de AgCl e
AgBr, foi filtrado, lavado e secado, e a massa encontrada foi 2,052 g. Determine a porcentagem de CuBr 2 na mistura
original.

Resolução: Composição dos 1,850 gramas de amostra: x mol de CuCl2 e y mol de CuBr2.
134,5 x + 223,5y = 1,850 g

CuCl2 não reage com AgCl, mas reage com o CuBr2, conforme a seguinte equação química:
y CuBr2 + 2y AgCl → y CuCl2 + 2y AgBr

A massa de 2y AgBr = 2y (108 + 80)g = 376y

A massa de AgCl na mistura final corresponde ao AgCl inicial menos o AgCl que reagiu com o CuBr2:
Massa inicial de AgCl: 1,800 g.
Massa de AgCl que reagiu: 2y AgCl = 2y .143,5 g = 287.y
Massa de AgCl na mistura final: 1,800 - 287y

Mistura final, de AgBr + AgCl:

376 y + (1,800 – 287 y) = 2,052


376 y – 287 y = 2,052 - 1,800
89 y = 0,252
y = 2,83 x 10-3 mol

Massa inicial de CuBr2 = 2,83 x 10-3 mol x 223,5 g.mol-1 = 0,6325 gramas, presente em 1,850 gramas da mistura.
0,632 𝑔
Cálculo da percentagem de CuBr2: % = (1,850 𝑔) 𝑥 100% = 34,19 %

41
1600 FÍSICO-QUÍMICA APLICADA EXERCÍCIOS COMENTADOS - IME – ITA – OLIMPÍADA

Questão 75 – (IME) A combustão completa de 3,0 g de um certo composto orgânico X produz, exclusivamente, 6,6 g
de CO2 e 3,6 g de H2O. A 100°C, 5,3g de X (que se encontra no estado gasoso a esta temperatura) são misturados
com 14 g de N2 em um recipiente de volume 3,0 litros. A pressão medida no interior do recipiente, nestas condições,
é igual a 6,0 atm. Considere que os gases, no interior do recipiente, se comportam idealmente. Sabendo que a reação
de X com dicromato de potássio em ácido sulfúrico aquoso gera uma cetona, determine a composição centesimal do
composto X, suas fórmulas mínima, molecular e estrutural, e dê a sua nomenclatura IUPAC.

Resolução:

6,6 g de CO2:
44 gramas de CO2 ----------- 12 gramas de C
6,6 gramas de CO2 ---------- mC
mC = 1,8 gramas de C

3,6 g de H2O:
18 gramas de H2O ----------- 2 gramas de H
3,6 gramas de H2O ---------- mH
mH = 0,4 gramas de H

Logo, a massa de oxigênio em 3,0 g do cmposto é: 3,0 g – (1,8 + 0,4) = 0,80 g de O

Assim sendo, C = 1,80 g, H = 0,40 g e O = 0,80 g, o que conduz à fórmula mínima e molecular C3H8O.

Como a oxidação de X gera uma cetona, X é o propan–2–ol, então: C = 60,00%; H = 13,33%; O = 26,67%.

Fórmula molecular: C3H8O

Fórmula estrutural: (Nomenclatura: propan–2–ol)

Questão 76 – (IME) O alumínio é o metal mais empregado pelo homem depois do ferro. É o elemento metálico mais
abundante na crosta terrestre (8,29% em massa) e não existe naturalmente na forma livre, sendo o minério sílico -
aluminato seu composto mais importante. Apresenta propriedade anfotérica, isto é, reage tanto com ácidos quanto
com bases. Partindo da equação apresentada abaixo, responda o que se pede:
NaNO3(aq) + Al(s) + NaOH(aq) + H2O(l) → NH3(aq) + Na [Al(OH)4](aq)
a) A equação da semi-reação de oxidação iônica balanceada (carga e massa) com os menores coeficientes
inteiros possíveis.
b) A equação da semi-reação de redução iônica balanceada (carga emassa) com os menores coeficientes
inteiros possíveis.
c) A equação total balanceada (carga e massa) com os menores coeficientes inteiros possíveis.
d) O íon oxidante.
e) A fórmula do redutor.

Resolução: Os processos de oxidação e redução envolvidos são as seguintes:


Item a) Al(s) + 4 OH-(aq) → Al(OH)-4(aq) + 3 e-
Item b) NO-3(aq) + 6 H2O(l) + 8 e- → NH3(aq) + 9 OH-(aq)
Item c) 3 NaNO3(aq) +18 H2O(l) + 8 Al(s) + 5 NaOH(aq) → 3 NH3(aq) + 8 Na[Al(OH)4](aq)
Item d) NO− 3 (Nitrato)
Item e) Alumínio (Al)

Questão 77 – (OLIMPÍADA NORTE-NORDESTE DE QUÍMICA) As essências naturais de frutas geralmente são


formadas por misturas de substâncias voláteis, responsáveis pelo odor e sabor. Essas substâncias são chamadas de
flavorizantes e na indústria alimentícia, são produzidas em grande quantidade, substituindo as naturais. A produção

42
1600 FÍSICO-QUÍMICA APLICADA EXERCÍCIOS COMENTADOS - IME – ITA – OLIMPÍADA

da essência de abacaxi, por exemplo, usada em preparos para bolos, é obtida através da reação de esterificação,
realizada com aquecimento interno e sob refluxo. Com base nas estruturas abaixo, determine:

a) O nome sistemático e a função de cada substância orgânica envolvida na reação.


b) As reações de esterificação são reversíveis. Discuta os fatores que alteram o equilíbrio da reação,
deslocando-o para o lados dos produtos.
c) Considerando a utilização de 13,2 gramas do ácido carboxílico e um rendimento de 75%, qual a massa do
éster produzida na reação.
d) O octanoato de etila é um flavorizante utilizado na indústria alimentícia como essência de laranja. Equacione
a reação de produção do octanoato de etila.

Resolução: Item a) Considerando a seguinte equação química e discutindo cada espécie química, temos:

= Ácido butanoico → Fsunção orgânica = ácido carboxílico → Fórmula molecular =


C4H8O2.

= Etanol (Álcool Etílico) → Função orgânica = álcool → Fórmula molecular = C2H6O.

= Butanoato de etila → Função orgânica = éster → Fórmula molecular = C6H12O2.

Item b) Através do Princípio de Le Chatelier, temos as seguintes situações:


- Aumento da concentração da quantidade de matéria da solução aquosa de ácido butanoico;
- Aumento da concentração da quantidade de matéria da solução de álcool etílico (etanol).

Item c) Resolvendo o cálculo estequiométrico: C4H8O2(aq) + C2H6O(aq) → C6H12O2(aq) + H2O(l)


1 mol de C4H8O2 ---------- 1 mol de C6H12O2
88 gramas ------------------ 116 gramas x 0,75
13,2 gramas ---------------- méster
méster = 13,05 g

Item d) Equação química: C8H16O2(aq) + C2H6O(aq) → C10H20O2(aq) + H2O(l).

Questão 78 – (IME) Um composto cuja molécula contém apenas carbono, hidrogênio, oxigênio e nitrogênio foi
queimado em presença de O2, fornecendo uma mistura gasosa de CO2, H2O e N2. A água presente nesta mistura foi
condensada e correspondeu a 1/6 do total de mol. Verificou-se que o CO2 representava 80% em mol da fração não
condensada. Determine:
a) a fórmula mínima do composto, sabendo-se ainda que sua molécula contém tantos átomos de carbono
quanto de oxigênio;

43
1600 FÍSICO-QUÍMICA APLICADA EXERCÍCIOS COMENTADOS - IME – ITA – OLIMPÍADA

b) a fórmula molecular do composto, sabendo-se que 170,4 gramas do mesmo, no estado gasoso a 800 K e
0,64 atm, ocupam 82 litros;
c) a massa mínima de O2 necessária para a combustão completa de 213,0 gramas deste composto.

Resolução: Equação química: Composto + O2 → _ CO2 + _ H2O + _ N2

CO2: 0,80 . (5/6) ntotal


H2O: (1/5) . ntotal
N2: x.ntotal

Cálculo do número de mol total (ntotal): (4/6).nt + (1/6).nt + x.nt = nt

4/6 + 1/6 + x = 1
x = 1 – 5/6 = 1/6

Equação química: Composto + O2 → 4/6 CO2 + 1/6 H2O + 1/6 N2

Item a) Como a proporção entre os átomos C:O é de 1:1 temos: C4O4H2N2


mxRxT 170,4 x 0,082 x 800
Item b) Cálculo da massa molar, a partir da equação dos gases ideais: < MM >= pxV
= 0,64 x 82
=
−1
213 g. mol

Fórmula Molecular: (C2O2HN)x


71x = 213  x = 3
Composto: (C2O2HN)3 = C6O6H3N3

Item c) Equação de combustão completa: 2 C6H3N3O6 + (15/2) O2 → 12 CO2 + 3 H2O + 3 N2.


(2 x 213) g de C6H3N3O6 ------------------------ (15/2) x 32 g de O2
213 g de C6O6H3N3 ------------------------------- X
X = 120 g de O2.

Observação: A fórmula molecular aponta, como estrutura mais provável para o composto em questão, o TNB
(trinitrobenzeno), sendo um poderoso explosivo e na temperatura de 800 K, se decompõe de forma bastante violenta.

NO2

O2N NO2

Questão 79 – (OLIMPÍADA BRASILEIRA DE QUÍMICA) Uma das razões do vasto uso da platina é a sua relativa
inércia química; entretanto, ela é “solúvel” na “água régia”, uma mistura de ácido nítrico e ácido clorídrico, segundo a
reação química não balanceada abaixo: Pt(s) + HNO3(aq) + HCl(aq) → H2PtCl6 + NO(g) + H2O(l). Faça o balanceamento
desta equação e responda as questões que se seguem:
a) Se você dispõe de 11,7 gramas de platina, quantos gramas de ácido cloroplatínico poderá obter?
b) Que volume de óxido de nitrogênio, medido nas CNTP, pode ser obtido a partir de 11,7 mg de Pt?
c) Quantos mililitros de ácido nítrico de concentração 10,0 mol.L-1 são necessários para reagir completamente
com 11,7 g de Pt?
d) Se você tem 10,0 g de platina e 180 mL de HCl de concentração 5,00 mol.L-1, mais excesso de ácido nítrico,
qual é o reagente limitante?

44
1600 FÍSICO-QUÍMICA APLICADA EXERCÍCIOS COMENTADOS - IME – ITA – OLIMPÍADA

Resolução: Balanceamento da equação química pelo método de oxirredução: 3 Pt(s) + 4 HNO3(aq) + 18 HCl(aq) → 3
H2PtCl6 + 4 NO(g) + 8 H2O(l).

Item a) Cálculo da massa de H2PtCl6: 3 Pt(s) + 4 HNO3(aq) + 18 HCl(aq) → 3 H2PtCl6 + 4 NO(g) + 8 H2O(l)
3 mol de Pt(s) ------------------------ 3 mol de H2PtCl6
3 x 195 g ----------------------------- 3 x 410 g
11,7 g --------------------------------- mH2PtCl6
mH2PtCl6 = 24,60 g

Item b) Cálculo do volume de monóxido de nitrogênio (NO) nas CNTP: 3 Pt(s) + 4 HNO3(aq) + 18 HCl(aq) → 3 H2PtCl6 +
4 NO(g) + 8 H2O(l)
3 mol de Pt(s) ------------------------------------ 4 mol de NO
3 x (195 g) --------------------------------------- 4 x (22,4 L;mol-1)
11,7 x 10-3 g ------------------------------------- VNO
VNO = 1,79 x 10-3 L

Item c) Cálculo da massa de ácido nítrico (HNO3): 3 Pt(s) + 4 HNO3(aq) + 18 HCl(aq) → 3 H2PtCl6 + 4 NO(g) + 8 H2O(l)
3 mol de Pt ------------------------ 4 mol de HNO3
3 x 195 g -------------------------- 4 x 63 g
11,7 g ------------------------------ mHNO3
mHNO3 = 5,04 g
5,04
A partir da concentração da quantidade de matéria, temos: 10,0 = 63 x Vsolução
Vsolução = 0,008 L (8 mL)

Item d) Cálculo do número de mol para cada reagente:


m 10
Platina: nPt = <MM>
Pt
= 195 = 0,017 mol (reagente limitante)
Pt

0,90
Ácido nítrico: nHNO3 = [HNO3 ] x Vsolução = 5,0 x 0,18 = 4
= 0,225 mol

Questão 80 – (OLIMPÍADA BRASILEIRA DE QUÍMICA – SEGUNDA FASE - MODIFICADA) O aumento na venda


mundial de automóveis tem gerado uma série de preocupações relacionadas as emissões decorrentes da queima dos
combustíveis. Embora existam hoje protótipos movidos a hidrogênio ou eletricidade, praticamente todos os veículos
comercializados consomem algum combustível à base de carbono (álcool, gasolina, diesel, GNV, etc...). A combustão
destas substâncias produz altas quantidades de gás carbônico. O aumento da concentração deste gás na atmosfera
amplifica o efeito estufa dando origem ao chamado aquecimento global. Uma das vantagens da utilização do etanol,
derivado da cana-de-açúcar, como combustível é o fato de sua produção e seu consumo formarem um ciclo onde o
gás carbônico emitido em sua queima é consumido no processo de fotossíntese da cana-de açúcar, na produção.
Assim, contabilizando este ciclo, o etanol não é considerado um emissor do gás estufa. Outro problema decorrente
de tais emissões são as chuvas ácidas. Para diminuir as emissões causadoras destas, é importante remover o enxofre
presente nos combustíveis derivados de petróleo antes de sua comercialização. Um dos processos utilizados para a
remoção do enxofre do gás natural e de derivados de petróleo é a hidrodessulfurização (HDS) catalítica, conforme a
seguinte reação:
C2H5SH + H2 → C2H6 + H2S
(C2H5SH = etanotiol)
a) Qual a nomenclatura do H2S, quando no estado gasoso e quando em solução aquosa?
b) Demonstre a ionização total do H2S em etapas.
c) Qual o volume de H2 medido a 2 atm e 227°C necessário para produzir 0,30 mg de etano?

Resolução:
Item a) H2S (estado gasoso) = Sulfeto de hidrogênio
H2S (solução aquosa) = Ácido sulfídrico

45
1600 FÍSICO-QUÍMICA APLICADA EXERCÍCIOS COMENTADOS - IME – ITA – OLIMPÍADA

Item b) Primeira ionização do ácido sulfídrico: H2S(aq) + H2O(l) → HS-(aq) + H3O+(aq)


Segunda ionização do ácido sulfídrico: HS-(aq) + H2O(l) → S-2(aq) + H3O+(aq)

Item c) Através da equação química, temos: C2H5SH + H2 → C2H6 + H2S


1 mol de H2 ---------- 1 mol de C2H6
2 g --------------------- 30 g
mH2 -------------------- 0,30 x 10-3 g
mH2 = 2,0 x 10-5 g

Questão 81 – (IME) A reação no estado sólido de iodato de potássio com sacarose (C12H22O11) produz dióxido de
carbono, água e um sal. Ao se adicionar 0,1 L de uma solução 0,5 mol/L de nitrato de mercúrio II aos produtos,
observa-se a formação de um precipitado cuja solubilidade em água é desprezível. Determine a massa desse
precipitado, sabendo-se que a amostra de iodato de potássio reagiu totalmente, gerando 168,0 L de gás, nas
condições normais de temperatura e pressão.

Resolução: As equações químicas balanceadas estão apresentadas abaixo:

8 KIO3 + C12H22O11 → 12 CO2 + 11 H2O + 8 KI


8 KI + 4 Hg(NO3)2 → 8 KNO3 + 4 HgI2

12 mol de CO2 ------------- 8 mol de KI


12 x 22,4 L ----------------- 8 mol
168,0 L --------------------- n
n = 5 mol de KI

Como foi colocado apenas 0,05 mol de Hg(NO3)2 teremos o iodeto de potássio (KI) como reagente em excesso na
segunda reação química.

8 mol KI -------------- 4 mol Hg(NO3)2


n’ ---------------------- 0,05 mol
n’ = 0,10 mol de KI

A proporção entre o nitrato e o iodeto de mercúrio II é de um para um, logo, o número de mol produzidos de HgI2 será
0,05 mol.

Massa de iodeto de mercúrio: m = 0,05 x <MM>HgI2 = 0,05 x [200,6 + (126,9 x 2)] = 22,72 g

Questão 82 – (ITA) Considere reações de combustão do etanol:

a) Escreva a equação química balanceada para a reação com oxigênio puro.


b) Escreva a equação química balanceada para a reação com ar atmosférico.
c) Escreva a equação química balanceada para a reação com 50% da quantidade estequiométrica de ar
atmosférico.
d) Classifique as reações dos itens a), b) e c) em ordem crescente de variação de entalpia reacional.

Resolução:

Item a) Equação química: C2H6O + 3 O2(g) → 2 CO2(g) + 3 H2O(l)

Item b) Cálculo do número de mol de N2, a partir da composição do ar atmosférico: 79% de N2 e 21% de O2.

3 mol de O2 ----------- 21%


nN2 ---------------------- 79%
nN2 = 11,28 mol de N2

Equação química: C2H6O + 3 O2(g) + 11,28 N2(g) → 2 CO2(g) + 3 H2O(l) + 11,28 N2(g)

46
1600 FÍSICO-QUÍMICA APLICADA EXERCÍCIOS COMENTADOS - IME – ITA – OLIMPÍADA

Item c) A partir de 50% de quantidade estequiométrica de ar atmosférico, o combustível etanol sofre combustão de
forma incompleta, apreentando como produtos o carvão e o monóxido de carbono. A equação química é a seguinte:
C2H6O + 1,50 O2(g) + 5,64 N2(g) → C(s) + CO(g) + 5,64N2(g) + 3 H2O(l)
Item d) As três equações químicas balanceadas apresentadas nos itens anteriores são reações exotémicas. As
reações químicas dos itens a e b liberama mesma quantidade de calor, uma vez que são reações de combustão
completa. Já em relação a reação química c, vai haver menor liberação de quantidade de calor, por seu uma reação
de combustão incompleta. Por isso, a ordem é a seguinte: |∆Hc | < |∆Hb | = |∆Ha |

Questão 83 – (IME) Um determinado metal forma dois óxidos distintos, nos quais as percentagens em massa de
oxigênio são 32,0% e 44,0%. Determine a massa atômica do metal.

Resolução: Através da Lei das Proporções múltiplas, a massa do metal M foi fixada e através desta propriedade
teremos a relação da massa de oxigênio dos dois óxidos:

Informação referente ao óxido 1: 68 g de M e 32 g de O.

Informação referente ao óxido 2: 56 g de M e 44 g de O.


56 gramas de M ---------- 44 gramas de O
68 gramas de M ---------- X
X = 53,429 g de O.
Massa 32 g
Cálculo da razão entre as massas de Oxigênio: Massaóxido 1 = 53,43 g ≅ 0,60
óxido 2

Através desta relação, as fórmulas empíricas dos óxidos ficarão da seguinte maneira:
Óxido 1: MxO3
Óxido 2: MxO5

Observação: os índices do átomo de oxigênio são números ímpares, com isso, sabendo que o número de oxidação é
igual a (-2), as fórmulas empíricas dos dois óxidos ficarão da seguinte maneira:

Fórmula molecular do óxido 1 = M2O3


Fórmula molecular do óxido 2 = M2O5

Cálculo da massa molar do óxido 1:


100 gramas do óxido 1 ---------- 32 gramas de Oxigênio
X ------------------------------------ 48 gramas de Oxigênio
X = 150 g

Cálculo da massa molar do metal M, através do óxido 1: 2x + 3.(16) = 150


x = 51 g.mol-1

Então, a massa atômica do elemento é de aproximadamente 51 u, logo, o metal em questão é o vanádio (V).

Fórmula molecular do óxido 1 = V2O3 e fórmula molecular do óxido 2 = V2O5

Questão 84 – (OLIMPÍADA MINEIRA DE QUÍMICA) Quando o ácido salicílico (C7H6O3) reage com metanol (CH4O)
forma-se o salicilato de metila (C8H8O3). Esta substância é um éster de odor extremamente agradável com
propriedades analgésicas e usado topicamente em pomadas para luxações, entorses, etc.

47
1600 FÍSICO-QUÍMICA APLICADA EXERCÍCIOS COMENTADOS - IME – ITA – OLIMPÍADA

Com relação a esta reação química, responda:


a) Quantos gramas de ácido salicílico (C7H6O3) são necessários para formar 0,4 mol de salicilato de metila
(C8H8O3)?
b) Qual o rendimento teórico desta reação quando 20 gramas de ácido salicílico reagem com 10 gramas de
metanol?
c) Qual o reagente limitante na reação descrita no item b?
Resolução: Item a) Equação química: C7H6O3 + CH3OH → C8H8O3 + H2O
1 mol de C7H6O3 -------------------- 1 mol de C8H8O3
nC7H6O3 -------------------------------- 0,40 mol de C8H8O3
nC7H6O3 = 0,40 mol de C7H6O3

Cálculo da massa de ácido salicílico, a partir do número de mol: mC7H6O3 = nC7H6O3 x <MM>C7H6O3 = 0,40 x 138 = 55,2
g

Item b) Através da equação química balanceada, temos: C7H6O3 + CH3OH → C8H8O3 + H2O

138 gramas de C7H6O3 -------------------- 32 gramas de CH3OH x Rendimento


20 gramas de C7H6O3 ---------------------- 10 gramas de CH3OH
640
Rendimento = = 0,46 (46%)
1380

Item c) Cálculo do número de mol para cada reagente:

20
nácido = = 0,145 mol (reagente limitante)
138
10
nbase = = 0,3125 mol
32

Questão 85 – (IME) Aspirina é produzida pela adição de anidrido acético a uma solução aquosa de ácido salicílico. A
equação da reação não balanceada é C4H6O3(l) + C7H6O3(s) → C9H8O4(s) + H2O(l). Se forem usados 2,0 kg de anidrido
acético e 1,0 kg de ácido salicílico, determine:
a) Os coeficientes para balancear a equação.
b) O reagente limitante.
c) O rendimento teórico da aspirina, em gramas.
d) Se o rendimento percentual da reação é de 86%, quantos comprimidos de 0,50 gramas de aspirina poderiam
ser preparados a partir das quantidades usadas nessa reação?

Resolução: Item a) Equação química: C4H6O3(l) + 2 C7H6O3(s) → 2 C9H8O4(s) + 1 H2O(l)

Determinação do reagente limitante e do reagente em excesso:

manidrido 2000
nanidrido = = = 19,61 mol
< MM >anidrido 102

mácido 1000 7,25


nácido = = = = 3,62 mol (reagente limitante)
< MM >ácido 138 2

48
1600 FÍSICO-QUÍMICA APLICADA EXERCÍCIOS COMENTADOS - IME – ITA – OLIMPÍADA

Item b) C4H6O3(l) + 2 C7H6O3(s) → 2 C9H8O4(s) + 1 H2O(l)

2 mol de C7H6O3(s) ------------------- 2 mol de C9H8O4(s)


2 x (138) gramas --------------------- 2 x (180) gramas
1000 gramas -------------------------- maspirina
maspirina = 1,30 x 10³ gramas de aspirina

Item c) C4H6O3(l) + 2 C7H6O3(s) → 2 C9H8O4(s) + 1 H2O(l)


2 mol de C7H6O3(s) ------------------ 2 mol de C9H8O4(s)
2 x (138) gramas --------------------- 2 x (180) gramas
1000 gramas ------------------------- maspirina
maspirina = 1,30 x 10³ g de aspirina

maspirina = 1,30 x 10³ g de aspirina x rendimento = 1,30 x 10³ g de aspirina x 0,86 = 1118 g de aspirina
1118
Número de aspirinas (N): 0,50 = 2236 comprimidos

Questão 86 – (OLIMPÍADA PORTUGUESA DE QUÍMICA) O ácido clorídrico é altamente corrosivo e quando ingerido
corrói as mucosas, esôfago e estômago, causa disfagia, náuseas, falha circulatória e morte. O hidróxido de sódio é
cáustico e se ingerido provoca vômito, prostação e colapso. Um condenado a morte foi obrigado a beber soluções
concentradas destes dois venenos, mas conseguiu fazê-lo sem sofrer qualquer problema. Como?

Resolução: Observa-se que quando ocorre a reação entre um ácido com a base definida como reação de
neutralização, há a formação de sal e água. O condenado conseguiu sobreviver, colocando uma quantidade
estequiométrica de ambos os reagentes na medida certa. Trata-se de uma pessoa com muita sorte. Equação química
de neutralização: HCl(aq) + NaOH(aq) → NaCl(aq) + H2O(l).

Questão 87 – (OLIMPÍADA BAIANA DE QUÍMICA) Uma vela de massa 34,5 gramas é acesa e encoberta por um
bequer. Após algum tempo a chama apaga. Após essa queima a massa da vela foi 33,8 gramas. Considerando que
a combustão é total e que a vela é formada apenas de C30H62, responda:
a) Qual a massa de dióxido de carbono formada?
b) Qual a massa do reagente limitante?

Resolução: Item a) Cálculo da massa de vela queimada (Δm): Δm = 34,5 gramas – 33,8 gramas = 0,70 gramas.

Equação química combustão completa da vela: C30H62(s) + (91/2) O2(g) → 30 CO2(g) + 31 H2O(l)

1 mol de C30H62 ---------------------- 30 mol de CO2


422 gramas de C30H62 --------------- 1320 gramas de CO2
0,70 gramas de C30H62 -------------- mCO2
mCO2 = 2,19 g

Item b) O reagente limitante para esta situação é o oxigênio (O2). Cálculo da massa do reagente limitante:

C30H62(s) + (91/2) O2(g) → 30 CO2(g) + 31 H2O(l)


1 mol de C30H62 ---------------------- 91/2 mol de O2
422 gramas de C30H62 --------------- 1456 gramas de O2
0,70 gramas de C30H62 -------------- mO2
mO2 = 2,41 g

Questão 88 – (OLIMPÍADA DE QUÍMICA DO RIO GRANDE DO NORTE) O trifluoreto de cloro, ClF3, é um dos
compostos mais reativos que se conhece. Reage violentamente com muitas substâncias que, em geral, são inertes.
Durante a Segunda Guerra Mundial foi grande sua aplicabilidade, sendo usado em bombas incendiárias. A obtenção
desta substância pode ocorrer pelo aquecimento de Cl2 e F2 em um sistema fechado.
a) A partir dos reagentes citados no texto, escreva a equação equilibrada do processo de obtenção do trifluoreto
de cloro.

49
1600 FÍSICO-QUÍMICA APLICADA EXERCÍCIOS COMENTADOS - IME – ITA – OLIMPÍADA

b) Se forem misturados 0,71 g de Cl2 e 1,00 g de F2, qual seria a produção teórica de ClF3?

Resolução:

Item a) Equação química: Cl2(g) + 3 F2(g) → ClF3(g)

Item b) Cálculo do número de mol de cada reagente:

mCl2 0,71
nCl2 = = = 0,01 mol
< MM >Cl2 71

mF2 1,00 0,026


nF2 = = = = 0,00877 mol (reagente limitante)
< MM >F2 38 3

Através da reação química, temos a seguinte relação estequiométrica:


3 mol de F2 ----------------------- 1mol de ClF3
(3 x 38) g de F2 ------------------ 92,50 g de ClF3
1,00 gramas de F2 --------------- mClF3
mClF3 = 0,811 g

Questão 89 – (IME) Permanganato de potássio reage com cloreto de sódio em presença de ácido sulfúrico, resultando
em sulfato de manganês II, sulfato de potássio, sulfato de sódio, água e cloro gasoso. Calcule o rendimento da reação
quando 58,5 gramas de cloreto de sódio e 32,6 gramas do permanganato forem adicionadas a 80,4 gramas de ácido
sulfúrico, produzindo 34,4 gramas de gás. Resolução: Balanceamento pelo método de oxirredução: 2 KmnO4 + 10
NaCl + 8 H2SO4 → 2 MnSO4 + K2SO4 + 5 NaSO4 + 8 H2O + 5 Cl2

Calculando o número de mol dos reagentes, para a determinação do reagente limitante:

1 mol de KMnO4
32,6 KMnO4 x ( ) = 0,2 mol de KMnO4
158g de KMnO4

1 mol de NaCl
58,5 NaCl x ( ) = 1,0 mol de NaCl (reagente limitante)
58,5g de NaCl
1 mol de H2 SO4
80,4 g de H2 SO4 x ( ) = 0,8 mol de H2 SO4
98 g de H2 SO4

5 mol de Cl2 71,0 g de Cl2


Cálculo do rendimento: 1 mol de NaCl x 10 mol de NaCl
x 1,0 mol de Cl2
= 35,5 g de Cl2

34,4
Rendimento: 35,5 = 0,97(97,7%)

Questão 90 – (OLIMPÍADA DE QUÍMICA DO RIO DE JANEIRO) O ácido clorídrico é muitas vezes vendido na forma
de uma solução de baixa pureza e concentração variável para a limpeza de pisos, pedras e até mesmo metais. Nesta
forma, ele é conhecido como ácido muriático. Uma indústria que necessitava neutralizar seu estoque de 1,35
toneladas de ácido muriático, escolheu o carbonato de cálcio para efetuar a neutralização. Esta reação produz um
sal, água e dióxido de carbono. Considerando que a solução possui 110 gramas de ácido clorídrico para cada litro de
solução. A densidade desta a solução é de 1,2 g.mL-1.
a) Escreva a reação química balanceada para a neutralização do ácido muriático.
b) Qual é a massa do carbonato que será utilizada na neutralização, considerando que o mesmo possui 7,0%
de impurezas?

Resolução: Item a) Equação química: 2 HCl(aq) + CaCO3(s) → CaCl2(s) + H2O(l) + CO2(g)

Item b) Cálculo da massa de carbonato de cálcio puro: 2 HCl(aq) + CaCO3(s) → CaCl2(s) + H2O(l) + CO2(g)

50
1600 FÍSICO-QUÍMICA APLICADA EXERCÍCIOS COMENTADOS - IME – ITA – OLIMPÍADA

2 mol de HCl(aq) ----------- 1 mol de CaCO3(s)


73 g de HCl ---------------- 100 g de CaCO3
110 g de HCl --------------- mCaCO3
mCaCO3 = 150,68 g

Cálculo da massa de carbonato de cálcio puro: mpuro = 150,68 x (1,0 – 0,07) = 140,13 g

Questão 91 – (OLIMPÍADA NORTE – NORDESTE DE QUÍMICA) Devido os atentados terroristas ocorridos em Nova
Iorque, Madrid e Londres, os Estados Unidos e países da Europa têm aumentado o controle quanto a venda e
produção de compostos explosivos que possam ser usados na confecção de bombas. Dentre os compostos químicos
explosivos, a nitroglicerina é um dos mais conhecidos. É um líquido à temperatura ambiente, altamente sensível a
qualquer vibração, decompondo-se de acordo com a equação: 2 C3H5(NO3)3(l) → 3 N2(g) + ½ O2(g) + 6 CO2(g) + 5 H2O(g).
Considere-se uma amostra de 4,45 gramas de nitroglicerina, massa molar 227 g.mol-1, contida em um frasco fechado
com volume total de 100,0 mL. Calcule a pressão máxima no interior do frasco antes de seu rompimento,
considerando-se que a temperatura atinge 127°C. Resolução:
4,45
Cálculo do número de mol de nitroglicerina: n = 227
= 0,0196 mol

Cálculo do número de mol de gases produzidos, a partir de 0,0196 mol de nitroglicerina: 2 C3H5(NO3)3(l) → 3 N2(g) + ½
O2(g) + 6 CO2(g) + 5 H2O(g)

2 mol de C3H5(NO3)3 ------------------------------ 14,5 mol de gases


0,0196mol mol de C3H5(NO3)3 ------------------ ngases
ngases = 0,142 mol

0,142 x 0,08206 x (127+273)


Cálculo da pressão aplicando a equação dos gases ideais: p = 0,10
= 46,61 atm

Questão 92 – (IME - MODIFICADA) Uma massa x de CaCO3 reagiu com 50 mL de HCl 0,20 mol.L-1 aquoso, sendo
o meio reacional, posteriormente, neutralizado com 12 mL de NaOH aquoso. Sabe-se que 20 mL desta solução foram
titulados com 25 mL do HCl 0,20 mol.L-1. Determine a massa (x) de carbonato de cálcio.

Resolução: Equação química: HCl + NaOH → NaCl + H2O

Pela estequiometria, temos:


1 mol HCl ---------- 1 mol de NaOH
nHCl ------------------ nNaOH

nHCl = nNaOH
[HCl] x V = [NaOH] x V

(25 x 0,20)HCl = [NaOH] x 20


[NaOH] = 0,25 mol x L-1

Nos 50 mL de HCl com concentração igual a 0,20 mol/L, há 10,0 mmol de HCl. Destes 10 mmol, uma parte dele foi
neutralizado por 12 mol de NaOH, o que corresponde a (12 x 0,25) mmol, ou seja 3,0 mmol.

Assim, 7,0 mmol de ácido clorídrico reage estequiometricamente com o carbonato de cálcio.
Equação química: CaCO3(s) + 2 HCl(aq) → CaCl2(aq) + H2O(l) + CO2(g)

nHCl 7 mmol
Cálculo do número de mol do carbonato de cálcio: nCaCO3 = 2
= 2
= 3,5 mmol

Logo, a massa de carbonato de cálcio será igual a: mCaCO3 = n x <MM>CaCO3 = 3,5 x 10-3 x 100 g = 0,35 g

51
1600 FÍSICO-QUÍMICA APLICADA EXERCÍCIOS COMENTADOS - IME – ITA – OLIMPÍADA

Questão 93 – (IME) O elemento constituinte da substância simples A possui um nome que em grego significa verde.
Livre, como molécula, é um gás venenoso. Na crosta terrestre, encontra-se combinado a outros elementos, como
minerais em depósitos subterrâneos e em oceanos. É solúvel em água e também em éter. Quando A reage com
hidróxido de sódio em solução aquosa, produz a substância compota B, usada como agente alvejante e bactericida.
Quando A reage com sódio fundido, produz a substância composta C, que é essencial ao ser humano. A eletrólise de
C, em solução aquosa, produz no catodo de ferro a substância simples D. A substância simples E é o produto gasoso
da reação, sob aquecimento, entre sódio metálico e nitrato de sódio. Ao reagir E com D, produz-se a substância
composta F, utilizada na fabricação de ácido nítrico, corantes, explosivos, medicamentos, detergentes e, ainda, na
forma de seus sais, como fertilizante. Determine:
a) as fórmulas moleculares de B, C, E e F;
b) as equações químicas das reações de produção de B, E e F;
c) o nome e a fórmula do composto produzido pela reação de F com ácido nítrico em solução aquosa.

Resolução: Após a identificação de que o elemento em questão é o cloro, temos:

Item a) B = NaClO (Hipoclorito de sósio), C = NaCl (Cloreto de sódio), E = N2 (Gás nitrogênio), F = NH3 (Amônia).
Produção de hipoclorito de sódio: Cl2 + 2 NaOH → NaCl + NaClO + H2O
Item b) Produção de nitrogênio: 10 Na + 2NaNO3 → 6Na2O + N2
Produção de amônia: N2 + 3 H2 → 2NH3
Item c) Nitrato de amônio = NH4NO3

Questão 94 – Considere uma amostra gasosa de pentacloreto de fósforo pesando 2,69 gramas sendo colocado em
um recipiente de volume igual a um litro. A temperatura regulada deste recipiente foi igual a 250°C e a pressão
regulada foi de uma atmosfera. A reação reversível está descrita a seguir: PCl5(g) ⇄ PCl3(g) + Cl2(g). A partir de todas
estas informações, determine as pressões parciais de todos os participantes gasosos na reação química.

Resolução: Para este problema será necessário o uso da tabela de equilíbrio químico, para o cálculo do número de
mol total. Base de cálculo: n (mol) inicial de PCl5.

Tabela de equilíbrio químico:


PCl5(g) ⇄ PCl3(g) Cl2(g)
Início n 0 0
Reage nα nα nα
Equilíbrio n - nα nα nα

Cálculo do número de mol total (nTOTAL): nTOTAL = n - nα + nα + nα = n + nα

Cálculo do grau de dissociação (α), utilizando a equação dos gases ideais: pideal
T x VT = n x (1 + α) x R x T
m
pideal
T x VT = <MM> x (1 + α) x R x T (*)

2,69
Substituindo os valores na equação (*) acima, temos: 1 x 1 = 208,5 x (1 + α) x 0,08206 x (250 + 273)
α = 0,80 (80%)
Cálculo das pressões parciais:

n x (1 − α) (1 − 0,80)
PPCl5 = x pT = x 1,0 = 0,11 atm
n x (1 + α) (1 + 0,80)

52
1600 FÍSICO-QUÍMICA APLICADA EXERCÍCIOS COMENTADOS - IME – ITA – OLIMPÍADA

nα 0,80
PPCl3 = PCl2 = x pT = x 1,0 = 0,44 atm
n(1 + α) (1 + 0,80)

Questão 95 - (IME) Obtêm-se clorato de potássio pela passagem de cloro em uma solução quente de hidróxido de
potássio, produzindo-se também cloreto de potássio e água. Uma solução assim obtida foi evaporada à secura e
aquecida para a decomposição do clorato. Sabendo-se que o resíduo total de cloreto de potássio pesou 298 g, calcular
a massa de hidróxido de potássio usado. Resolução: primeira reação química: Obtenção de clorato de potássio e
cloreto de potássio.

Equação química: Cl2 + KOH → KClO3 + KCl + H2O

Para o balanceamento desta reação, será que ser feito pelo método da oxirredução, em que o cloro gasoso sofre
tanto oxidação quanto redução. Logo, os coeficientes estequiométricos serão os seguintes:

Equação química: 3 Cl2(g) + 6 KOH(aq) → KClO3(aq) + 5 KCl(aq) + 3 H2O(l).

Segunda reação química: Obtenção de cloreto de potássio, a partir da decomposição do clorato de potássio.

Equação química: KClO3(aq) → KCl(s) + 3/2 O2(g)
Somando as duas equações químicas, temos:

3 Cl2(g) + 6 KOH(aq) → KClO3(aq) + 5 KCl(aq) + 3 H2O(l)


KClO3(aq) → KCl(s) + 3/2 O2(g) +
3 Cl2(g) + 6 KOH(aq) → 6 KCl(aq) + 3 H2O(l) + + 3/2 O2(g) (Reação química global)

Observa-se que teremos a seguinte relação estequiométrica:


6 mol de KOH(aq) ------------------ 6 mol de KCl(aq)
6 x (39 + 16 + 1) g ---------------- 6 mol x (39 + 35,5) g
mKOH -------------------------------- 298 g
mKOH = 224 g

Questão 96 - (IME) Permanganato de potássio reage com cloreto de sódio em presença de ácido sulfúrico, resultando
em sulfato de manganês II, sulfato de potássio, sulfato de sódio, água e cloro gasoso. Calcule o rendimento da reação
quando 58,5 g de cloreto de sódio e 32,6 g do permanganato forem adicionadas a 80,4 g de ácido sulfúrico, produzindo
34,4 g de gás.

Resolução: Para realizar o balanceamento desta reação química, será necessário utilizar o balanceamento pelo
método redox, sendo o permanganato de potássio o agente oxidante e o cloreto de sódio o agente redutor. Com isso,
o balanceamento desta reação química vai ficar os seguintes coeficientes estequiométricos: 2 KMnO 4 + 10 NaCl + 8
H2SO4 → 2 MnSO4 + K2SO4 + 5 NaSO4 + 8 H2O + 5 Cl2

Determinação do reagente limitante:

mKMnO4 32,6 0,206


nKMnO4 = = = = 0,103 mol
< MM >KMnO4 158 2

mNaCl 58,5 1,0


nNaCl = = = = 0,10 mol (reagente limitante)
< MM >NaCl 58,5 10
mH2 SO4 80,4 0,820
nH2 SO4 = = = = 0,1025 mol
< MM >:H2 SO4 98 8

Cálculo do rendimento (R):


10 mol de NaCl ------------------------ 5 mol de Cl2
10 mol x (58,5) g ---------------------- 5 x (71) g x R

53
1600 FÍSICO-QUÍMICA APLICADA EXERCÍCIOS COMENTADOS - IME – ITA – OLIMPÍADA

58,5 g ----------------------------------- 34,4 g


R = 0,969 (≅ 97%)

Questão 97 - (ITA) Quantos litros de ar, contendo 20% em volume de oxigênio são necessários para queimar
completamente 5,6 litros de propano? (Todos os gases são medidos nas CNTP).

Resolução: Cálculo do número de mol de gás nitrogênio: C3 H8(g) + 5 O2(g) → 3 CO2(g) + 4 H2 O(l)
5 mol de O2(g) − − − − 20%
nN2 − − − − − − − − 80%
5 x 80
nN2 = = 20 mol
20

Equação química da combustão de gás propano com a presença do nitrogênio molecular:

C3 H8(g) + ⏟
5 O2(g) + 20 N2(g) → 3 CO2(g) + 4 H2 O(l) + 20 N2(g)
ar atmosférico
n=5+20=25 mol

Cálculo do volume de ar atmosférico nas CNTP:


1 mol de C2 H8 − − − − 25 mol de ar
1 mol x 22,4 L − − − − 25 mol x 22,4 L na CNTP
5,60 L − − − − − − − nar

5,60 x 25 x 22,4
nar = = 140 L
22,4

Questão 98 – (IME) O sulfato cúprico anidro é obtido a partir de uma solução aquosa de ácido sulfúrico 98% (em
massa), a quente, com cobre. Sabendo que a solução aquosa de ácido sulfúrico tem massa específica 1,84 g.cm -3 e
que o ácido sulfúrico é o reagente limitante, calcule a massa de sulfato cúprico obtida a partir da reação de 10,87 mL
da solução aquosa de ácido sulfúrico. Resolução: Equação química: Cu(s) + 2 H2SO4(aq) → CuSO4(aq) + 2 H2O(l) +
SO2(g)

Cálculo da massa de ácido sulfúrico, a partir da concentração mássica: C = 10 x d x (%)


mH2 SO4
= 10 x d x (%)
Vsolução

mH2 SO4 = 10 x d x (%)x Vsolução = 10 x 1,84 x 98 x 10,84 x 10−3 = 19,55 g

Pela estequiometria, temos: Cu(s) + 2 H2SO4(aq) → CuSO4(aq) + H2O(l) + SO2(g)

2 mol de H2SO4(aq) ------------- 1 mol de CuSO4(aq)


196 g ---------------------------- 160 g
19,55 g -------------------------- mCuSO4
mCuSO4 = 15,96 g

Questão 99 – (ITA) Partindo de 8,2 gramas de um brometo de alquila, obtém-se o respectivo composto de Grignard
que, por hidrólise, fornece 4,3 gramas de um hidrocarboneto. Determine o número de átomos de carbono que deve
possuir esse hidrocarboneto. Resolução:

Primeira reação química: CnH2n+1 – Br + Mg → CnH2n+1 MgBr

Segunda reação química: CnH2n+1 MgBr + H2O → CnH2n+2 + Mg(OH)Br

Pelas equações químicas, podemos relacionar estequiometricamente da seguinte forma: nhaleto de alquila = nhidrocarboneto

54
1600 FÍSICO-QUÍMICA APLICADA EXERCÍCIOS COMENTADOS - IME – ITA – OLIMPÍADA

8,2 4,3
{ }={ }
12n + 2n + 1 + 80 12n + 2n + 2

n = 6,07

Questão 100 - (OLIMPÍADA BRASILEIRA DE QUÍMICA) Uma amostra de dióxido de carbono, pesando 22 mg,
contém:
a) 3,01 x 1020 mol de CO2
b) 3,01 x 1023 moléculas
c) 6,02 x 1023 átomos de oxigênio
d) Ocupa o volume de 11,2 mL em CNTP
e) Ocupa o volume de 1,12 L em CNTP

Resolução: Alternativa D.

A relação numérica para o dióxido de carbono fica da seguinte maneira:

1 mol de CO2 ----- 44 g ----- 6,02 x 1023 moléculas ----- 22,4 L ----- 2 x 6,02 x 1023 átomos de O

a) Analisando a primeira alternativa:


1 mol de CO2 ------ 44 g de CO2
nCO2 ----------------- 22 x 10-3 g de CO2
nCO2 = 5,0 x 10-4 mol de CO2

b) Analisando a segunda alternativa:


44 g de CO2 --------------- 6,02 x 1023 moléculas de CO2
22 x 10-3 g de CO2 ------- X moléculas de CO2
X = 3,01 x 1020 moléculas de CO2

c) Analisando a terceira alternativa:


44 gramas de CO2 --------------- 2 x 6,02 x 1023 átomos de Oxigênio
22 x 10-3 gramas de CO2 ------- X átomos de Oxigênio
X = 6,02 x 1020 átomos de Oxigênio

d) Analisando a quarta alternativa:


44 gramas de CO2 --------------- 22,4 L de CO2 nas CNTP
22 x 10-3 gramas de CO2 ------- VCO2
VCO2 = 11,2 x 10-3 L (11,2 mL)

Questão 101 - (ITA) Num equipamento adequado para permitir adição de solução, assim como coleta e medida de
volume de gases, fez-se a seguinte experiência: Após colocar neste equipamento 100 cm3 de uma solução aquosa
contendo 1,06 gramas de carbonato de sódio por litro de solução, adiciona-se um excesso de solução de ácido
clorídrico. Admitindo que, nesta experiência, todo o gás que pudesse ser produzido pela reação entre as duas soluções
foi de fato coletado, qual o volume medido, em cm3, sabendo-se que a experiência foi realizada na temperatura de
27°C e pressão de 750 mmHg?
a) (0,10 x 22,4)
b) (1,10 x 24,9)
c) (100 x 0,0821)
d) (0,20 x 62,3)
e) (0,40 x 62,3)

Resolução: Alternativa E.

Em 1 litro de solução há 1,06 gramas de carbonato de sódio dissolvido. Logo, para um volume de 100 cm³ utilizado,
a massa de carbonato de sódio dissolvida será de 0,106 gramas.

55
1600 FÍSICO-QUÍMICA APLICADA EXERCÍCIOS COMENTADOS - IME – ITA – OLIMPÍADA

Cálculo da massa de dióxido de carbono: Na2CO3 + 2 HCl → 2 NaCl + H2O + CO2


1 mol de Na2CO3 ---------------------- 1 mol de CO2
106 gramas ----------------------------- 44 gramas
0,106 gramas --------------------------- mCO2
mCO2 = 0,044 g
0,044
Usando a equação dos gases ideais: 750 x V = 44
x 62,30 x (27 + 273)

V = (0,40 x 62,30) cm³

Questão 102 – (ITA) Considere a queima completa de vapores das quatro seguintes substâncias: metano, etano,
metanol e etanol. Os volumes de ar necessário para a queima de 1 litro de cada um destes vapores, todos a mesma
pressão e temperatura, são, respectivamente, V1, V2, V3 e V4. Assinale a alternativa que apresenta a comparação
correta entre os volumes de ar utilizado na combustão:
a) V2 > V4 > V1 > V3
b) V2 > V1 > V4 > V3
c) V4 > V2 > V3 > V1
d) V4 > V3 > V2 > V1
e) V4 = V3 > V2 = V1

Resolução: Alternativa A.

Considere que a composição do ar apresenta 20% de O2 e 80% de N2.

Analisando o gás metano (CH4): CH4(g) + 2 O2(g) → CO2(g) + 2 H2O(g)

Cálculo do número de mol de N2, a partir da composição do ar:


2 mol de O2 ---------- 20%
nN2 --------------------- 80%
nN2 = 8 mol de N2(g)

Equação química com a presença de N2(g): CH4(g) + 2 O2(g) + 8 N2(g) → CO2(g) + 2 H2O(g) + 8 N2(g)

Observando a estequiometria da reação, temos que para cada 1 litro de CH4(g) está para 10 Litros de ar.

Analisando o gás etano (C2H6): C2H6(g) + 7/2 O2(g) → 2 CO2(g) + 3 H2O(g)

Cálculo do número de mol de N2, a partir da composição do ar:


(7/2) mol de O2 ---------- 20%
nN2 ------------------------- 80%
nN2 = 14 mol de N2(g)

Equação química com a presença de N2(g): C2H6(g) + 7/2 O2(g) + 14 N2(g) → 2 CO2(g) + 3 H2O(g) + 14 N2(g)

Observando a estequiometria da reação, temos que para cada 1 litro de C2H6(g) está para 17,5 Litros de ar.

Analisando o gás metanol (CH4O): CH4O(g) + 3/2 O2(g) → CO2(g) + 2 H2O(g)

Equação química: CH4O(g) + 3/2 O2(g) → CO2(g) + 2 H2O(g)

Cálculo do número de mol de N2, a partir da composição do ar:


(3/2) mol de O2 ---------- 20%
nN2 ------------------------- 80%
nN2 = 6 mol de N2(g)

56
1600 FÍSICO-QUÍMICA APLICADA EXERCÍCIOS COMENTADOS - IME – ITA – OLIMPÍADA

Equação química com a presença de N2(g): CH4O(g) + 3/2 O2(g) + 6 N2(g) → CO2(g) + 2 H2O(g) + 6 N2(g)

Observando a estequiometria da reação, temos que para cada 1 litro de CH4(g) está para 7,5 Litros de ar.

Analisando o gás etanol (C2H6O):

Equação química balanceada: C2H6O(g) + 3 O2(g) → 2 CO2(g) + 3 H2O(g)

Cálculo do número de mol de N2, a partir da composição do ar:

3 mol de O2 -------------- 20%


nN2 ------------------------- 80%
nN2 = 12 mol de N2(g)

Equação química com a presença de N2(g): C2H6O(g) + 3 O2(g) + 12 N2(g) → 2 CO2(g) + 3 H2O(g) + 12 N2(g)

Observando a estequiometria da reação, temos que para cada 1 litro de CH4(g) está para 15 Litros de ar.

C H6 C H6 O CH4 CH4 O
Comparando os volumes, temos: V2 2 > V4 2 > V1 > V3

Questão 103 – (ITA) Qual a massa de nitrato de potássio que deve ser decomposta termicamente em nitrito de
potássio para que o volume de oxigênio, medido a 77°C e pressão de 700 mmHg, seja de 1m³?
a) 3,80 kg
b) 5,10 kg
c) 6,00 kg
d) 6,50 kg
e) 7,60 kg

Resolução: Alternativa D.

Equação química: KNO3(s) → KNO2(s) + ½ O2(g)


𝐩 𝐱 𝐕𝐎𝟐 𝐱 <𝐌𝐌>𝐎𝟐
Cálculo da massa de oxigênio, utilizando a equação dos gases ideais: 𝐦𝐎𝟐 = 𝐑𝐱𝐓
=
𝟕𝟎𝟎
( ) 𝐱 𝟏𝟎𝟎𝟎 𝐱 𝟑𝟐 𝟕𝟎𝟎 𝐱 𝟑𝟐𝟎𝟎𝟎 𝟐𝟐𝟒𝟎𝟎𝟎𝟎𝟎
𝟕𝟔𝟎
𝟎,𝟎𝟖𝟐𝟎𝟔 𝐱 (𝟕𝟕+𝟐𝟕𝟑)
= 𝟕𝟔𝟎 𝐱 𝟎,𝟎𝟖𝟐𝟎𝟔 𝐱 𝟑𝟓𝟎 = 𝟐𝟏𝟖𝟐𝟕,𝟗𝟔
= 𝟏𝟎𝟐𝟔, 𝟐𝟏 𝐠

Cálculo da massa de nitrato de potássio: KNO3(s) → KNO2(s) + ½ O2(g)

1 mol de KNO3(s) ---------------------------- ½ mol de O2(g)


1 x (1 x 39 + 1 x 14 + 3 x 16)g ------------ ½ x 32g de O2(g)
mKNO3 ------------------------------------------1026,21 g
mKNO3 = 6477,93 g (≈ 6,50 kg)

Questão 104 – (IME) A taxa de emissão de dióxido de carbono em função do consumo médio de certo combustível,
em um carro de testes, é apresentada a seguir.

57
1600 FÍSICO-QUÍMICA APLICADA EXERCÍCIOS COMENTADOS - IME – ITA – OLIMPÍADA

Para um consumo médio de 10 km.L-1, a massa total mensal de combustível consumida é 2175 kg. Dentre as opções
abaixo, pode-se afirmar que o combustível testado foi o:
a) metano
b) propano
c) butano
d) heptano
e) octano

Resolução: Alternativa C.

A partir do gráfico, o consumo médio mensal de 10 km.L-1, a taxa de CO2 é de 6600 kg.(mês)-1. Processo de combustão
3n+1
completa: Cn H2n+2 + O2 → n CO2 + (n + 1)H2 O
2

CnH2n+2 + (3n+1) O2 → n CO2 + (n+1) H2O


(14n+2) ----------------- 44n
2175 kg ----------------- 6600 kg
n=4
Logo, o combustível testado é o butano (C4H10).

Questão 105 – (IME) Em um recipiente fechado queima-se propano com 80% da quantidade estequiométrica de ar.
Admitindo que não haja hidrocarbonetos após a combustão, que todos os produtos da reação estejam na fase gasosa
e que a composição volumétrica do ar seja de uma parte de O2 para quatro partes de N2, calcule a porcentagem molar
no recipiente após a combustão (considere comportamento ideal para os gases).
a) 4,35
b) 4,76
c) 5,26
d) 8,70
e) 14,28

Resolução: Alternativa A.

Equação química da combustão completa do gás propano: C3H8(g) + 5O2(g) → 3CO2(g) + 4 H2O(g)
A quantidade estequiométrica de O2 seria 5 mol. Segundo o enunciado, foram utilizados 80% da quantidade
estequiométrica de O2, ou seja, 4 mol de O2 e 16 mol de N2, pois a proporção entre oxigênio e nitrogênio no ar é de
1:4.
A queima foi incompleta, mas não havia hidrocarboneto entre os produtos. Logo, formaram-se CO, CO2 e vapor
d’água, logo, a equação química de combustão nesse caso ficará da seguinte maneira:
2𝑥+𝑦+4
C3H8(g) + 2
O2(g) → x CO2(g) + y CO(g) + 4 H2O(g)

2𝑥+𝑦+4
x+y=3e 2
=4x=1ey=2

58
1600 FÍSICO-QUÍMICA APLICADA EXERCÍCIOS COMENTADOS - IME – ITA – OLIMPÍADA

A equação de combustão incompleta com os coeficientes estequiométricos é a seguinte: C3H8 + 4 O2 → CO + 2 CO2


+ 4 H2O

N2 não reage, e por isso no fim da combustão haverá também 16 mol de N2.
1
Cálculo percentual: (%)𝐶𝑂2 = 23 𝑥 100% = 4,35%

Questão 106 – (IME) Sabendo que 18,0 gramas de um elemento X reagem exatamente com 7,75 gramas de oxigênio
para formar um composto de fórmula X2O5, a massa de um mol de X é:
a) 99,2 gramas
b) 92,9 gramas
c) 74,3 gramas
d) 46,5 gramas
e) 18,6 gramas

Resolução: Alternativa B.

Cálculo da massa atômica (<MA>) do elemento X: 2 X + 5/2 O2(g) → X2O5

2 mol de X ----------------- 5/2 mol de O2(g)


(2 x <MA> ) -------------- 5/2 x 32 g
18,0 g ----------------------- 7,75 g
<M.A> = 92,90 g.mol-1

Questão 107 - (IME) Em 19,9 gramas de um sal de cálcio encontra-se 0,15 mol desse elemento. Qual a massa molar
do ânion trivalente que forma esse sal?
a) 139 g.mol-1
b) 278 g.mol-1
c) 63,3 g.mol-1
d) 126,6 g.mol-1
e) 95 g.mol-1

Resolução: Alternativa A. Cálculo da massa molar do sal (Ca3X2):

1 mol de sal (CaX2) ----------- 3 mol de átomos de Ca


(3 x 40 + 2.<MMx>) ---------- 3 mol de átomos de Ca
19,9 g --------------------------- 0,15 mol
<MMx> = 139 g.mol-1

Questão 108 – (IME) O osso humano é constituído por uma fase mineral e uma fase orgânica, sendo a primeira
correspondente a cerca de 70% da massa óssea do ser humano. Dentre os minerais conhecidos, a hidroxiapatita,
Ca10(PO4)6(OH)2, é o mineral de estrutura cristalina e estequiometria mais próxima à dos nanocristais constituintes da
fase mineral dos tecidos ósseos. Considere que os átomos de cálcio estão na fase mineral dos tecidos ósseos e que
o esqueleto de um indivíduo corresponde a um terço do seu peso. O número de átomos de cálcio em uma pessoa de
60 kg é:
a) 8,37 x 1024;
b) 2,52 x 1025;
c) 8,37 x 1025;
d) 1,20 x 1026;
e) 2,52 x 1026.

Resolução: Alternativa C.

A fase mineral corresponde a 70% da massa corpórea, logo: m = 0,70 x (60 kg) = 42 kg

59
1600 FÍSICO-QUÍMICA APLICADA EXERCÍCIOS COMENTADOS - IME – ITA – OLIMPÍADA

Massa de átomos de cálcio: mCa = 42 kg x 1/3 = 14 kg

Cálculo do número de átomos de cálcio:


1 mol de Ca10(PO4)6(OH)2 ---------- 1004 g ------------------------- 10 x 6,02 x 1023 átomos de cálcio
14000 g ------------------------ X
X = 8,39 x 1025 átomos de Cálcio.

Questão 109 – (IME) Em um recipiente fechado queima-se propano com 80% da quantidade estequiométrica de ar.
Admitindo que não haja hidrocarbonetos após a combustão, que todos os produtos da reação estejam na fase gasosa
e que a composição volumétrica do ar seja de uma parte de O2 para quatro partes de N2, calcule a porcentagem molar
de CO2 no recipiente após a combustão (considere comportamento ideal para os gases).
a) 4,35 %
b) 4,76 %
c) 5,26 %
d) 8,70 %
e) 14,28 %

Resolução: Alternativa A.

Equação química: C3H8(g) + 5 O2(g) → 3 CO2(g) + 4 H2O(g)

Cálculo do número de mol de O2:


5 mol de O2 ---------- 100%
nO2 --------------------- 80%
nO2 = 4 mol

Cálculo do número de mol de N2:


4 mol de O2 ----------- 1 parte
nN2 --------------------- 4 partes
nN2 = 16 mol

Equação química com a presença de N2(g): C3H8(g) + 4 O2(g) + 16 N2(g) → CO2(g) + 4 H2O(g) + 2 CO(g) + 16 N2(g)

Cálculo do numero de mol total dos gases presentes no recipiente: nTotal = nCO2 + nH2O + nCO + nN2 = 1 + 4 + 2 + 16 =
23 mol
𝑛𝐶𝑂2 1
Cálculo do percentual de CO2 no recipiente: (%)𝐶𝑂2 = 𝑛 = 23 = 0,0435 (4,35%)
𝑇𝑂𝑇𝐴𝐿

Questão 110 - (ITA) O hidrogênio produzido na reação de 5,59 gramas de ferro metálico com excesso de solução de
ácido clorídrico reduziria a seguinte massa de óxido cuproso:
a) 5,6 gramas
b) 7,9 gramas
c) 9,5 gramas
d) 14,2 gramas
e) 28,4 gramas

Resolução: Alternativa D. Equação química: Fe + 2 HCl → FeCl2 + H2


1 mol de Fe ------------- 1 mol de H2
55,90 g ------------------ 2,0 g de H2
5,59 g -------------------- mH2
mH2 = 0,20 g

Segunda equação química: Cu2O + H2 → 2 Cu(s) + H2O


1 mol ---------------------------------------- 1 mol de H2
(63,50 x 2 + 16) g ------------------------- 2,0 g

60
1600 FÍSICO-QUÍMICA APLICADA EXERCÍCIOS COMENTADOS - IME – ITA – OLIMPÍADA

Massa --------------------------------------- 0,20 g


Massa de Cu2O = 14,30 g

Questão 111 – (ITA) Uma amostra de óxido de crômio (III) contaminada com impurezas inertes é reduzida com
hidrogênio de acordo com a equação: Cr2O3 + 3 H2 → 2 Cr + 3 H2O. O volume de hidrogênio medido nas CNTP,
necessário para purificar 5 gramas de óxido de crômio (III) contendo 15% de impurezas inertes é igual a:
a) [(0,15 x 5,0 x 3 x 22,4) / 152] litros
b) [(0,85 x 5,0 x 3 x 22,4) / 152] litros
c) [(0,15 x 5,0 x 3 x 22,4) / 104] litros
d) [(0,85 x 5,0 x 3 x 22,4) / 104] litros
e) [(0,15 x 5,0 x 22,4) / 104] litros

Resolução: Alternativa B.

Percentual de pureza = 100% - 15% = 85% (0,85)

Massa (pura) de óxido de cromo III = 0,85 x 5g = 4,25 g

Equação química: Cr2O3 + 3 H2→ 2 Cr + 3 H2O

152 gramas de Cr2O3 ---------------- 6 gramas de H2


4,25 gramas de Cr2O3 --------------- y
y = 0,167 g de H2
mH 0,167
Cálculo do número de mol de H2: nH2 = <MM>2 = 2
= 0,083 mol
H2

1 mol de H2 nas CNTP --------------- 22,4 L


0,083 mol nas CNTP ----------------- Y

0,85 x 5,0 x 3 x 22,4


Y= = 1,87 L
152

Questão 112 – (ITA) Certa massa de nitrato de cobre Cu(NO3)2 foi calcinada em um ambiente aberto até resfriar um
resíduo com massa constante, que é sólido e preto. Formaram-se dois produtos gasosos, conforme a equação
química: 2 Cu(NO3)2(s) → 2 CuO(s) + 4 NO2(g) + O2(g). A massa do NO2 formado na reação de decomposição é igual a
18,4 gramas. Qual é o valor que mais se aproxima da massa inicial do nitrato de cobre?
a) 9,40 gramas
b) 37,50 gramas
c) 57,50 gramas
d) 123 gramas
e) 246 gramas

Resolução: Alternativa B. Equação química: 2 Cu(NO3)2(s) → 2 CuO(s) + 4 NO2(g) + O2(g).

2 mol de nitrato de cobre -------------------- 4 mol de óxido de nitrogênio


2 x (63,55 + 28 + 6 x 16) -------------------- 4 x (14 + 32) gramas
MassaCu(NO3)2 ---------------------------------- 18,40 g
MassaCu(NO3)2 = 37,5 g

Questão 113 – (ITA) O gás cloro pode ser obtido, de acordo com a reação: MnO2 + 4 HCl → MnCl2 + 2 H2O + Cl2.
Para produzirmos 3,00 litros desse gás nas condições normais, supondo a reação completa, será necessário o volume
de ácido clorídrico (densidade = 1,12 g.mL-1 contendo 40,0 % de HCl em massa).
a) 17,30 litros
b) 19,40 litros
c) 43,70 litros

61
1600 FÍSICO-QUÍMICA APLICADA EXERCÍCIOS COMENTADOS - IME – ITA – OLIMPÍADA

d) 48,5 litros
e) Nenhuma das respostas anteriores.

Resolução: Alternativa E.

Equação química: MnO2 + 4 HCl → MnCl2 + 2 H2O + Cl2


g
Cálculo da concentração comum do ácido clorídrico: 𝐶 = 10 x d x (%) = 10 x 1,12 x 40 = 448 L

448 mol
Cálculo da concentração da quantidade de matéria do ácido clorídrico: [HCl] = 36.5 = 12,27 L

Cálculo do volume de ácido clorídrico, em litros, através da estequiometria: MnO2 + 4 HCl → MnCl2 + 2 H2O + Cl2

4 mol de HCl ------------------------- 1 mol de Cl2


nHCl ------------------------------------ nCl2
nHCl = 4 x nCl2

𝐩𝐱𝐕
[𝐇𝐂𝐥] 𝐱 𝐕𝐬𝐨𝐥𝐮çã𝐨 = 𝟒 𝐱
𝐑𝐱𝐓
𝟏𝐱𝟑
𝟏𝟐, 𝟐𝟕 𝐱 𝐕𝐬𝐨𝐥𝐮çã𝐨 = 𝟒 𝐱 ( )
𝟎, 𝟎𝟖𝟐𝟎𝟔 𝐱 𝟐𝟕𝟑

𝐕𝐬𝐨𝐥𝐮çã𝐨 = 𝟒, 𝟑𝟔 𝐱 𝟏𝟎−𝟐 𝐋

Questão 114 – (ITA) Tratando 2,13 gramas do carbonato do metal M com ácido sulfúrico, em quantidade suficiente,
obtém-se 3,17 gramas do sulfato desse metal M pode ser:
a) Alumínio
b) Bário
c) Cálcio
d) Lítio
e) Sódio

Resolução: Alternativa D.

Cálculo da massa molar do metal M: MCO3 + H2SO4 → MSO4 + H2CO3


1 mol de MCO3 ---------------------------- 1 mol de MSO4
1 mol de (<MM>M + 60) g --------------- (<MM>M + 96) g
2,13 g --------------------------------------- 3,17 g
<MM>M = 13,73 g.mol-1

Pelo valor calculado, não há nenhum elemento com esta massa molar. Considerando que o metal M seja um elemento
do grupo 1, temos:
M2CO3 + H2SO4 → M2SO4 + H2CO3
1 mol de M2CO3 --------------------------- 1 mol de M2SO4
1 mol de (2 x <MM>M + 60)g ------------ (2 x <MM>M + 96)g
2,13 g --------------------------------------- 3,17 g
<MM>M = 6,86 g.mol-1

Com esta massa molar, trata-se do elemento químico Lítio (Li).

Questão 115 – (ITA) Foi preparada uma solução usando 490 gramas de ácido sulfúrico puro e água até completar
1,0 litro. Qual a massa de carbonato de sódio necessária para neutralizar 200 mL desta solução?
a) 53,0 gramas

62
1600 FÍSICO-QUÍMICA APLICADA EXERCÍCIOS COMENTADOS - IME – ITA – OLIMPÍADA

b) 100 gramas
c) 212 gramas
d) 10,6 gramas
e) 21,2 gramas

Resolução: Alternativa B.
mH SO
2 4 490
nH2SO4 <MM>H SO
Cálculo da concentração da quantidade de matéria de ácido sulfúrico: [H2 SO4 ] = V = 2 4
Vsolução
= 98
1,0
=
solução
5,0 mol. L−1
Equação química balanceada: H2SO4(aq) + Na2CO3(s) → Na2SO4(aq) + H2CO3(aq).
1 mol de H2SO4(aq) ---------- 1 mol de Na2CO3(s)
nácido --------------------------- nsal
nácido = nsal

[H2 SO4 ] x Vsolução = nNa2 CO3

mNa2 CO3
5 x 0,200 =
106

mNa2 CO3 = 106 g

Questão 116 – (ITA) A combustão de um composto X na presença de ar atmosférico ocorre com a formatação de
fuligem. Dos compostos abaixo, assinale a opção que contém o composto X que apresenta a maior tendência de
combustão fuliginosa.
a) C6H6
b) C2H5OH
c) CH4
d) CH3(CH2)6CH3
e) CH3OH
Resolução: Alternativa D.

A formação de fuligem ocorre quando há uma impossibilidade de ocorrer uma estequiometria de combustão definida
como incompleta, ou seja, combustão que consiste na formação de monóxido de carbono (CO) ou de carbono
(fuligem), devido a insuficiência de gás oxigênio (O2). Já a combustão completa é definida como a formação de dióxido
de carbono (CO2).

Analisando a combustão completa, formando dióxido de carbono (CO2):


✓ Analisando o benzeno: C6H6 + O2 → 6 CO2 + 3 H2O
✓ Analisando o etanol: C2H5OH + 7/2 O2 → 2 CO2 + 3 H2O
✓ Analisando o metano: CH4 + 2 O2 → CO2 + 2 H2O
✓ Analisando o octano: CH3(CH2)6CH3 + 25/2 O2 → 8 CO2 + 9 H2O
✓ Analisando o metanol: CH3OH + 2 O2 → CO2 + 2 H2O

Analisando a combustão incompleta, formando monóxido de carbono (CO):


✓ Analisando o benzeno: C6H6 + 9/2 O2 → 6 CO + 3 H2O
✓ Analisando o etanol: C2H5OH + 7/2 O2 → 2 CO + 3 H2O
✓ Analisando o metano: CH4 + 3/2 O2 → CO + 2 H2O
✓ Analisando o octano: CH3(CH2)6CH3 + 17/2 O2 → 8 CO + 9 H2O
✓ Analisando o metanol: CH3OH + 3/2 O2 → CO + 2 H2O

Analisando a combustão incompleta, formação de fuligem (C):


✓ Analisando o benzeno: C6H6 + 3/2 O2 → 6 C + 3 H2O
✓ Analisando o etanol: C2H5OH + 3 O2 → 2 C + 3 H2O
✓ Analisando o metano: CH4 + O2 → C + 2 H2O
✓ Analisando o octano: CH3(CH2)6CH3 + 9 O2 → 8 C + 9 H2O

63
1600 FÍSICO-QUÍMICA APLICADA EXERCÍCIOS COMENTADOS - IME – ITA – OLIMPÍADA

✓ Analisando o metanol: CH3OH + O2 → C + 2 H2O

Resposta: Para todos os tipos, o octano apresenta a maior tendência de combustão fuliginosa.

Questão 117 – (ITA) Adicionou-se um excesso de solução de sulfato de sódio a 500 mL de uma solução de nitrato
de chumbo, tendo se formado um precipitado de sulfato de chumbo de massa igual a 3,0 gramas. A concentração
inicial do sal de chumbo na solução era:
a) 2,0 x 10-3 mol.L-1
b) 1,0 x 10-2 mol.L-1
c) 2,0 x 10-2 mol.L-1
d) 5,0 x 10-2 mol.L-1
e) 2,0 x 10-1 mol.L-1

Resolução: Alternativa C.

Cálculo da concentração (mol.L-1) do sulfato de chumbo, a partir da estequiometria: Na2SO4 + Pb(NO3)2 → PbSO4 + 2 NaNO3

1 mol de Pb(NO3)2 --------------- 1 mol de PbSO4


nPb(NO3)2 ----------------------------- nPbSO4
nPb(NO3)2 = nPbSO4

3,0
[Pb(NO3 )2 ] x 0,50 =
303

[Pb(NO3 )2 ] = 1,98 x 10−2 mol. L−1

As questões 118 e 119 referem-se à combustão completa do pentanol gasoso representada pela equação: C5H11OH
+ x O2 → 5 CO2 + 6 H2O.

Questão 118 – (ITA) O coeficiente x da equação acima é:


a) 13/2
b) 14/2
c) 15/2
d) 16/2
e) Nenhuma das anteriores

Resolução: Alternativa D. O balanceamento da reação química da combustão do pentanol é feito pelo método das
tentativas. Logo, a equação química balanceada ficará da seguinte forma: C5H11OH + 15/2 O2 → 5 CO2 + 6 H2O.

Questão 119 – (ITA) Se nesta reação são consumidos 176 gramas de pentanol, o número de mol de água formada
será:
a) 6,0
b) 12,0
c) 18,0
d) 24,0
e) 30,0

Resolução: Alternativa B. Equação química: C5H11OH + 15/2 O2 → 5 CO2 + 6 H2O.


m
pentanol 176
Cálculo do número de mol (n) de pentanol: npentanol = <MM> = 88
= 2,0 mol
pentanol

1 mol de C5H11OH -------------- 6 mol de H2O


2 mol de C5H11OH -------------- nH2O
nH2O = 12 mol de água

64
1600 FÍSICO-QUÍMICA APLICADA EXERCÍCIOS COMENTADOS - IME – ITA – OLIMPÍADA

Questão 120 – (ITA) A 25°C, uma mistura de metano e propano ocupa um volume (V), sob uma pressão total de
0,080 atm. Quando é realizada a combustão completa desta mistura e apenas dióxido de carbono é coletado, verifica-
se que a pressão desse gás é de 0,12 atm, quando este ocupa o mesmo volume (V) e está sob a mesma temperatura
da mistura original. Admitindo que os gases têm comportamento ideal, assinale a opção que contém o valor correto
da concentração, em fração em mol, do gás metano na mistura original.
a) 0,01
b) 0,25
c) 0,50
d) 0,75
e) 1,00

Resolução: Alternativa D.

Informação do problema: nCH4 + nC3H8 = 0,080

Os dois gases realizam combustão completa, conforme pode ser observado nas seguintes equações químicas
balanceadas:
CH4(g) + 2 O2(g) → CO2(g) + 2 H2O(g)
C3H8(g) + 5 O2(g) → 3 CO2(g) + 4 H2O(g)

Analisando a reação de combustão do metano: CH4(g) + 2 O2(g) → CO2(g) + 2 H2O(g)


1 mol de CH4(g) ---------- 1 mol de CO2(g)
nCH4 ------------------------ nCO2
nCH4 = nCO2

Analisando a reação de combustão de propano: C3H8(g) + 5 O2(g) → 3 CO2(g) + 4 H2O(g)


1 mol de C3H8(g) ---------- 3 mol de CO2(g)
nCH4 ------------------------- nCO2
nCO2 = 3 x nC3H8
CH C3 H8
A segunda equação química será a seguinte: nCO24 + 3nCO 2
= 0,120

A partir deste momento há duas equações com duas incógnitas, logo:


nCH4 + nC3H8 = 0,080
nCH4 + 3 x nC3H8 = 0,120

Resolvendo este sistema, chegamos aos seguintes valores:nCH4 = 0,060 mol e nC3H8 = 0,020 mol.
0,060
Cálculo do percentual: (%) = 0,080 = 0,75 (75%)

Questão 121 – (ITA) Na temperatura ambiente, hidróxido de potássio sólido reage com o cloreto de amônio sólido,
com a liberação de um gás. Assinale a alternativa correta para o gás liberado nesta reação.
a) Cl2
b) H2
c) HCl
d) NH3
e) O2

Resolução: Alternativa D. A reação entre hidróxido de potássio e o cloreto de amônio terá como produto o cloreto de potássio e
o hidróxido de amônio. O hidróxido de amônio se decompõe dando origem a molécula de água e amônia, sendo esta última a
substância gasosa liberada. Equação química balanceada: KOH + NH4Cl → KCl + H2O + NH3

Questão 122 – (ITA) Através da fusão de mistura de SiO2(s) e Al2O3(s) em forno suficientemente aquecido é possível
produzir aluminossilicatos. Considere que seja produzido um aluminossilicato com a relação de massa {(g de Al2O3) /
(g de SiO2)} igual a 2,6. Qual das alternativas correspondente ao valor da quantidade {(mol de Al2O3) / (mol de SiO2)}
neste aluminossilicato?

65
1600 FÍSICO-QUÍMICA APLICADA EXERCÍCIOS COMENTADOS - IME – ITA – OLIMPÍADA

a) 0,59
b) 1,0
c) 1,5
d) 2,6
e) 4,4
mAl2O3
Resolução: Alternativa C. Informação do problema: mSiO2
= 2,60
nAl2 O3 x <MM>Al2O3
= 2,60
nSiO2 x < MM >SiO2

nAl2 O3 x 102
= 2,60
nSiO2 x 60

nAl2 O3
= 1,53
nSiO2

Questão 123 – (ITA) 100 mL de solução aquosa de NaOH 0,1 mol.L-1 foram misturados com 100 mL de solução
aquosa de H2SO4 0,3 mol.L-1. Quantos mol.L-1 de H+ existem na solução final?
a) 0,10
b) 0,20
c) 0,25
d) 0,30
e) 0,50

Resolução: Alternativa C.

Equação química balanceada de neutralização: 2 NaOH(aq) + H2SO4(aq) → Na2SO4(s) + 2 H2O(l).

Cálculo do número de mol de ácido sulfúrico: n = [H2SO4] x V = 0,30 mol.L-1 x 100 x 10-3 L = 0,03 mol

Cálculo do número de mol de hidróxido de sódio: n = [NaOH] x V = 0,10 mol.L-1 x 100 x 10-3 L = 0,01 mol

Para:
2 mol de NaOH ---------- 0,01 mol
1 mol de NaOH ---------- nNaOH
nNaOH = 0,005 mol de NaOH

Observando o número de mol de cada reagente, conclui-se que o reagente limitante é o hidróxido de sódio e o
reagente em excesso é o ácido sulfúrico.
nexcesso
H2 SO4 = 0,03 mol − 0,005 mol = 0,025 mol

nexcesso
H2 SO4 0,025
Cálculo da concentração da quantidade de matéria do ácido sulfúrico: [H2 SO4 ] = = =
Vsolução 0,20
0,125 mol. L−1

Cálculo da concentração da quantidade de matéria dos íon hidrogênio, a partir da seguinte estequiometria:
H2 O
H2 SO4(aq) → +
2H(aq) + SO4 −2
(aq)

1 mol de H2SO4(aq) --------------- 2 mol de H+


0,125 mol.L-1 -------------------- [H+]
[H+] = 0,250 mol.L-1

66
1600 FÍSICO-QUÍMICA APLICADA EXERCÍCIOS COMENTADOS - IME – ITA – OLIMPÍADA

Questão 124 – (ITA) Dispõe-se de 10,0 litros de uma solução 1,00 mol.L-1 de ácido clorídrico. Para que o pH desta
solução seja elevado para o valor 2,0; a massa de hidróxido de sódio sólido que deve ser adicionada, admitindo
desprezível a variação de volume, será:
a) 400 gramas
b) 396 gramas
c) 360 gramas
d) 320 gramas
e) 80 gramas

Resolução: Alternativa B.

Cálculo da concentração da quantidade de matéria de H+ do ácido clorídrico, para um pH = 2,0: pH = -log[H+]


2 = - log[H+]
[H+] = 0,01 mol.L-1
H2 O +
Cálculo da concentração da quantidade de matéria de HCl, a partir da reação de dissociação: HCl(aq) → H(aq) +
Cl−
(aq)
1 mol de HCl ---------- 1 mol de H+
[HCl] ------------------- 0,01 mol.L-1
[HCl] = 0,01 mol.L-1
Cálculo do volume de HCl: [HCl]1 x V1 = [HCl]2 x V2
1 x 10 = 0,01 x V2
V2 = 1000 L

Para que o pH fique igual a 2,0, hove a adição de 990 litros de ácido, ou seja: VHCl = 1000 L – 10 L = 990 L

Cálculo da massa de NaOH, a partir da estequiometria: HCl(aq) + NaOH(aq) → NaCl(aq) + H2O(l)

1 mol de HCl ---------- 1 mol de NaOH


nHCl --------------------- nNaOH
nHCl = nNaOH
mNaOH
= [HCl] x Vsolução
< MM >NaOH

mNaOH = < MM >NaOH x [HCl] x Vsolução = 40 x 10−2 x 990 = 396 g

Questão 125 – (ITA) Num recipiente inerte, hermeticamente fechado, estão presentes 100 gramas de ferro, 100
gramas de água e 100 gramas de oxigênio. Supondo que ocorre a reação 2 Fe(s) + 3 H2O(l) + 3/2 O2(g) → 2 Fe(OH)2(s)
e que a reação prossiga até o consumo completo do(s) reagente(s) minoritário(s), podemos prever que irá(irão) sobrar:
a) Fe e H2O
b) Fe e O2
c) H2O e O2
d) Fe
e) H2O

Resolução: Alternativa C.

Equação química: 2 Fe(s) + 3 H2O(l) + 3/2 O2(g) → 2 Fe(OH)2(s).

Cálculo do número de mol de cada reagente químico:

mFe 100 1,78 mol


nFe = = = = 0,89 mol (reagente limitante)
< MM >Fe 56 2

67
1600 FÍSICO-QUÍMICA APLICADA EXERCÍCIOS COMENTADOS - IME – ITA – OLIMPÍADA

mH2 O 100 5,55 mol


nH2 O = = = = 1,85 mol
< MM >H2 O 18 3

m O2 100 3,125 mol


nO2 = = = = 2,08 mol
< MM >O2 32 1,5

Pelos cálculos realizados, irão sobrar H2O e O2.

Questão 129 – (ITA) O volume de HCl gasoso, medido na pressão de 624 mmHg e temperatura igual a 27°C,
necessário para neutralizar completamente 500 cm3 de uma solução aquosa 0,200 mol.L-1 de NaOH é:
a) 0,27 litros
b) 1,5 litros
c) 3,0 litros
d) 6,0 litros
e) 27 litros

Resolução: Alternativa C.

Cálculo do número de mol de hidróxido de sódio, a partir da concentração da quantidade de matéria: nNaOH =
[NaOH] x Vsolução = 0,200 x 0,500 = 0,100 mol

Cálculo do número de mol de cloreto de hidrogênio: HCl(g) + NaOH(aq) → NaCl(aq) + H2O(l)

1 mol de HCl(g) ---------- 1 mol de NaOH(aq)


nHCl(g) ---------------------- 0,100 mol
nHCl(g) = 0,100 mol
𝑛𝐻𝐶𝑙 𝑥 𝑅 𝑥 𝑇
Cálculo do volume de cloreto de hidrogênio, considerando comportamento ideal:𝑉𝐻𝐶𝑙 = 𝑝𝑖𝑑𝑒𝑎𝑙
=
0,100 𝑥 0,08206 𝑥 (27+273) 0,100 𝑥 0,08206 𝑥 300 𝑥 760
624 = 624
= 3,00 𝐿
760

Questão 130 – (ITA) Qual é a relação entre as massas de gás oxigênio consumido na combustão completa de um
mol, respectivamente, de metano, etanol e octano?
a) 3 : 6 : 24
b) 3 : 6 : 25
c) 4 : 7 : 25
d) 6 : 9 : 27
e) 6 : 10 : 34

Resolução: Alternativa B.

Analisando a reação de combustão do metanol: CH4O + 3/2 O2 → CO2 + 2 H2O; são consumidos 3/2 mol de oxigênio.

Analisando a reação de combustão do etanol: C2H6O + 3 O2 → 2 CO2 + 3 H2O; são consumidos 3 mol de oxigênio.

Analisando a reação de combustão do octano: C8H18 + 25/2 O2 → 8 CO2 + 9 H2O. são consumidos 25/2 mol de
oxigênio.

Relação estequiométrica: nmetanol


O2 = netanol
O2 = noctano
O2

3 25
:3 ∶
2 2

68
1600 FÍSICO-QUÍMICA APLICADA EXERCÍCIOS COMENTADOS - IME – ITA – OLIMPÍADA

3 25
Multiplicando por dois, temos: (2 : 3 ∶ 2
)𝑥 2 = 3: 6 ∶ 25

Questão 131 – (ITA) O volume SO2 gasoso, medido nas CNTP, necessário para transformar completamente 250 cm3
de solução aquosa 0,100 mol.L-1 de NaOH em solução de Na2SO3, é:
a) 0,14 litros
b) 0,28 litros
c) 0,56 litros
d) 1,12 litros
e) 2,24 litros

Resolução: Alternativa B.
Cálculo do volume de dióxido de enxofre nas condições normais de temperatura e pressão (0°C e 1 atm): SO2(g) + 2
NaOH(aq) → Na2SO3(aq) + H2O(l)
1 mol de SO2 ---------- 2 mol de NaOH
nSO2 --------------------- nNaOH
2 x nSO2 = nNaOH
1 x VSO2
2x = 0,100 x 0,250
0,08206 x 273

VSO2 = 0,280 L

Questão 132 – (ITA) Um método de obtenção de prata pura e porosa consiste na decomposição térmica de seu
carbonato. Qual massa de prata seria obtida pela decomposição de um quilograma de Ag2CO3?
a) (1000 g / 275,8 g) x 107,9 g
b) (1000 g / 275,8 g/mol) x 215,8 g/mol
c) (275,8 g / 107,9 g/mol) x 1000 g/mol
d) (1000 g / 215,8 g) x 275,8 g
e) (275,8 g/mol / 1000 g) x 107,8 mol

Resolução: Alternativa B.

Cálculo da massa de prata metálica (Ag): Ag2CO3(s) → 2 Ag(s) + CO2(g) + ½ O2(g)

1 mol de AgCO3 --------------------- 2 mol de Ag


1 mol x 276 g ------------------------- 2 mol x 108 g
1000 g ---------------------------------- mAg

1000 x 215,74
mAg =
275,74

Questão 133 – (ITA) O volume, em litros, de uma solução 0,30 mol.L-1 de sulfato de alumínio que contém 3,0 mol de
cátion de alumínio, é:
a) 2,50
b) 3,30
c) 5,0
d) 9,0
e) 10,0

Resolução: Alternativa C.

Cálculo do número de mol de sulfato de alumínio: Al2(SO4)2(aq) → 2 Al+3(aq) + 3 SO4-2(aq)

1 mol de Al2(SO4)2(aq) ---------- 2 mol de Al+3(aq)


nAl2(SO4)2 -------------------------- 3 mol de Al+3

69
1600 FÍSICO-QUÍMICA APLICADA EXERCÍCIOS COMENTADOS - IME – ITA – OLIMPÍADA

nAl2(SO4)2 = 1,5 mol.


nAl2(SO4 )3 1,50
Cálculo do volume, a partir da concentração da quantidade de matéria: Vsolução = [Al2 (SO4 )3 ]
= 0,30
= 5,0 L

Questão 134 – (U.S. NATIONAL CHEMISTRY OLYMPIAD) 0,422 g of an element Z reacts with oxygen to form
0,797 g of the oxide Z2O3. What is the element Z?
a) Al
b) Sc
c) Cr
d) Ga

Resolução: Alternativa A.

Equação química: 2 Z + 3/2 O2 → Z2O3

Determinação da massa molar de Z:


2 mol de Z -------------- 1 mol de Z2O3
2 x <MM>Z --------------- 2 x <MM>Z + 48
0,422 g -------------------- 0,797 g
1,594 x <MM>Z = 0,844 x <MM>Z + 20,26
<MM>Z = 27,01 g/mol

Questão 135 – (ITA) Hematita (óxido férrico) e siderita (carbonato ferroso) são minérios importantes a partir dos quais
se obtém ferro metálico. As massas máximas, em Kg, de ferro que podem ser obtidas a partir de 1,00 Kg de hematita
e 1,00 Kg de siderita, supostas secas e puras são respectivamente:
a) 55,8 / (55,8 + 16,0); 2 x 55,8 / (2 x 55,8 + 180)
b) 2 x 55,8 / (2 x 55,8 + 48,0); 55,8 / (55,8 + 60,0)
c) 2 x 55,8 / (2 x 55,8 + 48,0); 2 x 55,8 / (2 x 55,8 + 180,0)
d) 55,8 / [2.(2 x 55,8 + 48,0)]; 55,8 / [2 x (55,8 + 60,0)]
e) (2 x 55,8 + 48,0) / (2 x 55,8); (2 x 55,8 + 180,0) / (2 x 55,8)

Resolução: Alternativa B.
Analisando a hematita: 2 Fe2O3 + 3 C → 4 Fe + 3 CO2
2 mol de Fe2O3 ----------- 4 mol de Fe
2 x (112 + 48) g ---------- 4 x (56) g
1 kg ------------------------ mFe

4 x 56 x 1 232
mFe = =
2 x 160 320
Analisando a siderita: FeCO3 + CO → Fe + 2 CO2
1 mol de FeCO3 --------------- 1 mol de Fe
1 x (56 + 12 + 48) g ---------- 56 g
1 kg ----------------------------- mFe

56 x 1 56
mFe = =
1 x 116 116
Questão 136 – (ITA) Determine o menor volume de solução de ácido clorídrico 0,250 mol.L-1 necessário para dissolver
completamente 13,50 gramas de alumínio metálico granulado.
a) 6,0 litros
b) 5,0 litros
c) 4,0 litros
d) 3,0 litros
e) 2,0 litros

70
1600 FÍSICO-QUÍMICA APLICADA EXERCÍCIOS COMENTADOS - IME – ITA – OLIMPÍADA

Resolução: Alternativa A.

Relação estequiométrica entre o alumínio metálico e a solução de ácido clorídrico, através da seguinte equação
química: Al(s) + 3 HCl(aq) → AlCl3(aq) + 3/2 H2(g).

1 mol de Al ------ 3 moles de HCl


nAl ----------------- nHCl
3 x nAl = nHCl
massa
Cálculo do volume de ácido, a partir da concentração da quantidade de matéria: 3 x <MM>Alumínio
= [HCl] x V
13,50
3x = 0,250 x V
27,00

V = 6,0 L

Questão 137 – (ITA) Uma amostra de ácido dicarboxílico com 0,104 gramas de massa é neutralizada com 20 cm3 de
uma solução aquosa 0,1 mol.L-1 em NaOH. Qual das opções abaixo contém a fórmula química do ácido constituinte
da amostra?
a) C2H2O4
b) C3H4O4
c) C4H4O4
d) C4H6O4
e) C5H8O4

Resolução: Alternativa B.

Ácido dicarboxílico + hidróxido → sal orgânico + água

Levando em consideração que a reação entre os reagentes seja de 1:1, temos:

1 mol de ácido dicarboxílico ---------------- 1 mol de hidróxido


nácido -------------------------------------------- nhidróxido
nácido = nhidróxido
mácido
= [hidróxido] x Vsolução
< MM >ácido

0,104
= 0,10 x 20 x 10−3
< MM >ácido
g
< MM >ácido = 52,0
mol

Esta massa molar do ácido dicarboxilíco refere-se à sua fórmula mínima, uma vez que nenhuma das alternativas
apresenta este valor. Na verdade, todas as alternativas apresentam uma massa molar maior que 52 g.mol-1. Por isso,
multiplicando por dois a sua massa molar, temos 104 g.mol-1. Analisando as alternativas, o ácido dicarboxílico será o
C3H4O4.

Questão 138 – (ITA) Quando carbeto de alumínio (Al4C3) é adicionado em um béquer contendo água líquida a 25ºC,
ocorre a formação de hidróxido de alumínio e a liberação de um gás. O gás formado é o:
a) H2
b) CO
c) CO2
d) CH4
e) C2H2

71
1600 FÍSICO-QUÍMICA APLICADA EXERCÍCIOS COMENTADOS - IME – ITA – OLIMPÍADA

Resolução: Alternativa D.
Equação química: Al4C3(s) + 12 H2O(l) → 4 Al(OH)3(s) + 3 CH4(g).

Questão 139 – (ITA) Ao colocar-se um pedaço de magnésio em uma solução de ácido clorídrico, verifica-se que
ocorre aumento da temperatura e desprendimento de gás. O gás que se desprende é, sobretudo:
a) Hidrogênio
b) Vapor de água
c) Vapor de magnésio
d) Mistura de vapores de magnésio e água
e) Mistura de vapores de magnésio e hidrogênio

Resolução: Alternativa A.
Equação química: Mg(s) + HCl(aq) → MgCl2(aq) + H2(g).

Questão 140 – (ITA) Colocando grãos de nitrato de potássio em um frasco com água nota-se que com o passar do
tempo o sólido desaparece dentro d’água. Qual das equações abaixo é a mais adequada para representar a
transformação que ocorreu dentro do frasco?
a) KNO3(c) → KNO3(l)
b) KNO3(c) + H2O(l) → KOH(aq) + KNO3 (aq)
c) KNO3(c) → K+(aq) + NO3-(aq)
d) KNO3(c) → K(l) + NO3(aq)
e) KNO3(c) + H2O(l) → KNO2(aq) + H2O2(aq)

Resolução: Alternativa C.
O processo que vai ocorrer trata-se de uma reação denominada como dissociação iônica. Define-se dissociação
iônica, também conhecida como dissociação eletrolítica, um processo em que compostos iônicos têm seus íons
separados. Sua representação é a seguinte: KNO3(s) → K+(aq) + NO3-(aq).

Questão 141 – (OLIMPÍADA BRASILEIRA DE QUÍMICA – PRIMEIRA FASE) Indique as funções orgânicas
oxigenadas e a percentagem de carbono contida em uma molécula de aspartame?

a) Éter, cetona e ácido carboxílico; 50%.


b) Éster, cetona e ácido carboxílico; 50%
c) Éster, ácido carboxílico e amida; 50%
d) Éster, ácido carboxílico e amida; 60%
e) Éster, cetona e ácido carboxílico; 60%

Resolução: Alternativa D. Identificação das funções orgânicas oxigenadas presentes na figura abaixo:

72
1600 FÍSICO-QUÍMICA APLICADA EXERCÍCIOS COMENTADOS - IME – ITA – OLIMPÍADA

Cálculo do percentual de carbono na molécula de aspartame: C14H18O5N2 (292 g,mol-1).


292 g,mol-1 ---------- 100%
168 g,mol-1 ---------- (%)C
(%)C = 57,53%

Questão 142 – (OLIMPÍADA DE QUÍMICA DO RIO DE JANEIRO) Um caminhão tanque transportando ácido sulfúrico
tombou na via Dutra (Rio – São Paulo) e derramou sobre a pista cinco toneladas do ácido. A polícia rodoviária federal
ao chegar ao local chamou o Ministério do Meio Ambiente temendo um impacto ambiental sem precedentes na região.
O Ministério já se dirigiu ao local levando carbonato de cálcio em pó, para neutralizar o ácido derramado. Admitindo
que o carbonato de cálcio esteja com 90% de pureza, qual deve ser a massa do mesmo utilizada?
a) 5668,93 kg
b) 5453,25 kg
c) 5389,74 kg
d) 4789,61 kg
e) 4591,82 kg

Resolução: Alternativa A.

Cálculo da massa de carbonato de cálcio: CaCO3 + H2SO4 → CaSO4 + H2O + CO2

1 mol de CaCO3 ---------- 1 mol de H2SO4


100 g ----------------------- 98 g
0,90. mCaCO3 --------------- 5 t
mCaCO3 = 5,66893 t (5668,93 kg)

Questão 143– (OLIMPÍADA DE QUÍMICA DO RIO DE JANEIRO) Qual o número de átomos de carbono contidos em
9,0 mg de glicose?
a) 1,8 x 1020 átomos
b) 1,8 x 1023 átomos
c) 3,0 x 1019 átomos
d) 3,0 x 1022 átomos
e) 1,26 x 1026 átomos

Resolução: Alternativa A.

180 g ----------------------- 6 x 6 x 1023 átomos de carbono


9,0 x 10-3 g ---------------- X
X = 1,8 x 1020 átomos de carbono

Questão 144 – (OLIMPÍADA DE QUÍMICA DO RIO DE JANEIRO) O ácido sulfúrico é um dos agentes da chuva
ácida. Ao precipitar a chuva ácida reage com monumentos de mármore (carbonato de cálcio), “destruindo-os”. Qual o
volume de gás produzido a 27°C e 1,00 atm quando 2,45 gramas de ácido sulfúrico precipitam na forma de chuva
ácida e reagem com estes monumentos?
a) 1,23 x 10³ mL
b) 615 mL
c) 560 mL
d) 111 mL
e) 55,3 mL

Resolução: Alternativa B. Cálculo da massa de dióxido de carbono: H2SO4 + CaCO3 → CaSO4 + H2O + CO2.
1 mol de H2SO4 ------------------------ 1 mol de CO2
98 g de H2SO4 -------------------------- 44 g de CO2
2,45 g de H2SO4 ------------------------ mCO2
mCO2 = 1,1 g

73
1600 FÍSICO-QUÍMICA APLICADA EXERCÍCIOS COMENTADOS - IME – ITA – OLIMPÍADA

Cálculo do volume de dióxido de carbono, considerando com comportamento ideal: 1,0 x VCO2 =
1,1
x (0,08206) x (27 + 273)
44
VCO2 = 0,025 x 0,08206 x 300 = 0,615 L (615 mL)

Questão 145 – (OLIMPÍADA DE QUÍMICA DO RIO DE JANEIRO) 2,7 gramas de alumínio são dissolvidos em 500
mL de uma solução aquosa 1,00 mol/L em ácido clorídrico. Todo o hidrogênio produzido é recolhido. Após a secagem,
o volume de hidrogênio à pressão de 1 atm e 25°C é:
a) 1,2 litros
b) 1,6 litros
c) 2,4 litros
d) 3,6 litros
e) 12 litros

Resolução: Alternativa D.

Equação química: Al(s) + 3 HCl(aq) → AlCl3(aq) + 3/2 H2(g)

Determinação do reagente limitante:


mAl 2,7
Cálculo do número de mol de alumínio metálico: nAl = <MM> = 27 = 0,10 mol (Reagente limitante)
Al

Cálculo do número de mol de HCl, a partir da concentração da quantidade de matéria: nHCl = [HCl] x Vsolução = 1,00 x
0,500 = 0,500 mol

3 mol de HCl ---------- 0,500 mol


1 mol de HCl ---------- nHCl
nHCl = 0,167 mol

Relação estequiométrica entre o alumínio metálico e o hidrogênio gasoso:


1 mol de Al ---------- 3/2 mol de H2
nAl --------------------- nH2
3
nH2 = x nAl
2
1x VH 3 2,7
Considerando que o gás hidrogênio apresenta comportamento ideal, temos: 0,08206 x (25+273)
2
= 2 x ( 27 )
VH2 = 3,67 L

Questão 146 – (PETROBRÁS – ENGENHEIRO DE PROCESSAMENTO JÚNIOR) A combustão completa de 5 litros


de octano é processada. Considerando que foi utilizado oxigênio em quantidade estequiométrica e qua a massa
específica do octano é igual a 0,70 g,mL-1, a soma das massas de gases liberados é, em kg, aproximadamente, igual
a:
a) 6
b) 16
c) 26
d) 36
e) 46

Resolução: Alternativa B.
moctano
Cálculo da massa de octano (moctano), a partir da massa específica (μ): μ = Voctano

g moctano
0,70 =
mL 5000 mL

74
1600 FÍSICO-QUÍMICA APLICADA EXERCÍCIOS COMENTADOS - IME – ITA – OLIMPÍADA

moctano = 3500 g

Cálculo da massa de dióxido de carbono e água: C8H18 + O2(g) → CO2(g) + H2O(g)


1 mol de C8H18 ----------- 1 mol de CO2 ---------- 1 mol de H2O
3500g de C8H18 ---------- mCO2 --------------------- mH2O
mCO2 =10807,02 g (10,81 kg)
mH2O = 4973,68 g (4,87 kg)

Cálculo da massa dos gases liberados: mTOTAL = mCO2 + mH2O = 10,81 kg + 4,87 kg = 15,78 kg

Questão 147 – (TÉCNICO DE OPERAÇÃO JÚNIOR – PETROBRÁS) A 200 mL de solução contendo 0,624 gramas
de BaCl2 foram adicionados 200 mL de solução contendo 0,568 gramas de Na2SO4. Considere a equação a seguir:
BaCl2(aq) + Na2SO4(aq) → BaSO4(s) + 2 NaCl(aq). A quantidade máxima de composto sólido formado é:
a) 0,699 gramas
b) 0,754 gramas
c) 0,855 gramas
d) 0,930 gramas
e) 0,992 gramas

Resolução: Alternativa A.
mBaCl2 0,624 g
Cálculo do número de mol do cloreto de bário: nBaCl2 = = =
<MM>BaCl2 208 g.mol−1
0,003 mol (reagente limitante)
mNa2 SO4 0,568 g
Cálculo do número de mol do sulfato de sódio: nNa2 SO4 = = = 0,004 mol
<MM>Na2 SO4 142 g.mol−1

Cálculo da massa de sulfato de bário formado, a partir do reagente limitante: BaCl2(aq) + Na2SO4(aq) → BaSO4(s) + 2
NaCl(aq)

1 mol de BaCl2 ----------- 1 mol de BaSO4


(137 + 71) g -------------- (137 + 32 + 64) g
0,624 g -------------------- mBaSO4
mBaSO4 = 0,699 g

Questão 148 – (TÉCNICO DE OPERAÇÃO JÚNIOR – PETROBRÁS) O silício pode ser obtido a partir de uma
matéria-prima muito abundante, o óxido de silício (SiO2). No processo, a matéria-prima é reduzida pela reação com
carbono, segundo a equação abaixo: SiO2(s) + C(s) → Si(s) + CO2(g). Se, em uma certa condição experimental, obteve-
se 787 kg de silício a partir de 2000 kg de SiO2, o rendimento percentual da reação foi de:
a) 42,4%
b) 60,0%
c) 84,4%
d) 91,8%
e) 100%

Resolução: Alternativa C.
Cálculo percentual do rendimento para o referido processo, a partir da seguinte equação química: SiO2(s) + C(s) → Si(s)
+ CO2(g)
1 mol de SiO2(s) ---------- 1 mol de Si(s)
(28 + 32) g ---------------- 28 x R
2000 kg -------------------- 787 kg
R = 0,8432 (84,32%)

75
1600 FÍSICO-QUÍMICA APLICADA EXERCÍCIOS COMENTADOS - IME – ITA – OLIMPÍADA

Questão 149 – (OLIMPÍADA DE QUÍMICA DO RIO GRANDE DO SUL) O zinco é um elemento importante para a
saúde, mas é importante também manter uma dieta balanceada desse elemento. Deficiência de zinco pode ocasionar
problemas de crescimento, desenvolvimento incompleto dos órgãos sexuais e dificuldades de cicatrização dos
ferimentos. Por outro lado, o excesso de zinco pode causar anemia e problemas renais. O zinco está presente nos
ovos, fígado e mariscos, numa concentração, em torno de 4 mg por 100 g. Quantos átomos de zinco aproximadamente
estão presentes em 1,7 kg de fígado?
a) 5 x 1020
b) 5 x 1021
c) 6 x 1018
d) 6 x 1019
e) 6 x 1020
Resolução: Alternativa E.

Cálculo da massa de zinco, a partir de 1,7 kg de fígado:


4 mg de Zn ---------- 100 g de fígado
mzn ------------------- 1700 g de fígado
mZN = 68 mg de zinco

Cálculo do número de átomos de zinco (XZN):


1 mol de Zn ---------- 68,5 g de Zn ---------------- 6 x 1023 átomos de Zn
68 x 10-3 g de Zn ---------------- xZn
XZN = 6,24 x 1020 átomos

Questão 150 – (OLIMPÍADA MINEIRA DE QUÍMICA - MODIFICADA) O acetileno é um gás muito utilizado como
combustível para produção de chamas de altas temperaturas (aproximadamente 3000°C) quando associado ao
oxigênio. Considerando a queima completa do acetileno gasoso (C2H2), qual é a quantidade de matéria desta
substância necessária para fornecer 315 gramas de água?
a) 43,8 mol
b) 3,50 mol
c) 17,5 mol
d) 7,00 mol
e) 6,00 mol

Resolução: Alternativa C.

Equação química: C2H2 + 5/2 O2 → 2 CO2 + H2O


mH O 315
Cálculo do número de mol de água: nH2 O = <MM>2 = 18
= 17,5 mol
H2 O

Cálculo do número de mol de acetileno, a partir da reação de combustão: C2H2 + 5/2 O2 → 2 CO2 + H2O
1 mol de C2H2 ---------- 1 mol de H2O
nC2H2 ---------------------- 17,5 mol de H2O
nC2H2 = 17,5 mol

Questão 151 – (OLIMPÍADA MINEIRA DE QUÍMICA - MODIFICADA) A cal viva, CaO, pode ser produzida em duas
etapas, conforme representado a seguir:
Ca(OH)2(aq) + CO2(g) → CaCO3(s) + H2O(l)
CaCO3(s) → CaO(s) + CO2(g)
Considerando que o rendimento percentual da primeira etapa seja de 80% e o rendimento percentual da segunda
etapa seja de 50%, qual é o rendimento percentual total esperado para se produzir CaO a partir de Ca(OH)2?
a) 80%
b) 65%
c) 55%
d) 40%
e) 30%

76
1600 FÍSICO-QUÍMICA APLICADA EXERCÍCIOS COMENTADOS - IME – ITA – OLIMPÍADA

Resolução: Alternativa D.

Determinação da equação química global:


Ca(OH)2(aq) + CO2(g) → CaCO3(s) + H2O(l) R1 = 80%
CaCO3(s) → CaO(s) + CO2(g) + R2 = 50%
Ca(OH)2(aq) → CaO(s) + H2O(l) RT = R1 x R2 = 0,80 x 0,50 = 0,40 (40%)

Questão 152 – (OLIMPÍADA TOCANTINENSE DE QUÍMICA) Reagiu-se excesso de zinco metálico com 100 mL de
solução de ácido clorídrico de pH = 2,0. Qual é a massa em gramas, de cloreto de zinco obtido?
a) 68,2
b) 1,36 x 10-1
c) 6,82 x 10-2
d) 13,6
e) 3,67 x 10-2

Resolução: Alternativa C.

Cálculo da concentração de [H+]: pH = -log[H+]


[H+] = 10-2 mol.L-1.

Esta solução foi preparada a partir de 100 mL.


1 L ------------------ 10-2 mol
0,100 mol ---------- [H+]
[H+] = 10-3 mol

Cálculo da concentração de HCl: HCl → H+ + Cl-

1 mol de HCl ------------------ 1,0 mol de [H+]


[HCl] --------------------------- 10-3 mol
[HCl] = 10-3 mol

Cálculo da massa de cloreto de zinco: Zn(s) + 2 HCl(aq) → ZnCl2(aq) + H2(g)


2 mol de HCl ---------- 1 mol de ZnCl2
na ------------------------ ns
msal
10−3 mol = 2 x
134,5 g. mol−1
msal = 0,06725 g

Questão 153 - (OLIMPÍADA DE QUÍMICA DO RIO DE JANEIRO) Um monitor de laboratório encontra um frasco com
rótulo danificado no armário de bases, não sendo possível a identificação do conteúdo. Para identificar a base contida
no frasco, o monitor procede com uma titulação usando uma solução de HCl 0,1 mol.L-1. Sabendo que para neutralizar
completamente uma amostra de 0,100 g de base sólida, dissolvida em 100 mL de água destilada, são necessários
17,85 mL da solução ácida, a base desconhecida poderia ser:
a) LiOH
b) NaOH
c) KOH
d) Ca(OH)2
e) Ba(OH)2

Resolução: Alternativa C.

77
1600 FÍSICO-QUÍMICA APLICADA EXERCÍCIOS COMENTADOS - IME – ITA – OLIMPÍADA

Relação estequiométrica química entre o ácido clorídrico e a base desconhecida, a partir da seguinte equação
química: HCl(aq) + X(OH)(aq) → XCl(aq) + H2O(l).
1 moles de HCl ------ 1 mol de XOH
nHCl --------------------- nXOH
nHCl = nXOH
0,100
Cálculo da massa molecular da base desconhecida: 0,100 x 17,85 x 10−3 = <MM>
base

< MM >base = 56,02 g. mol−1

Analisando a massa molecular de cada alternativa, hidróxido de potássio é a alternativa que apresenta esta massa
molecular.

Questão 154 - (OLIMPÍADA DE QUÍMICA DO RIO DE JANEIRO) Uma solução contendo 1,699 kg de nitrato de prata
reagiu com uma solução contendo 0,2925 kg de cloreto de sódio. A quantidade de cloreto de prata produzida foi:
a) 1 mol
b) 2 mol
c) 3 mol
d) 4 mol
e) 5 mol

Resolução: Alternativa E. Determinação do reagente limitante e do reagente em excesso:


1699 g
Cálculo do número de mol do nitrato de prata: nAgNO3 = = 10 mol
170 g.mol−1

292 g
Cálculo do número de mol do cloreto de sódio: nNaCl = = 5,0 mol (reagente limitante)
58,5 g.mol−1

Cálculo do número de cloreto de prata: AgNO3(aq) + NaCl(aq) → NaNO3(aq) + AgCl(aq)

1 mol de NaCl(aq) ---------- 1 mol de AgCl(aq)


5 mol de NaCl(aq) ---------- nAgCl(aq)
nAgCl(aq) = 5 mol

Questão 155 – (U.S. NATIONAL CHEMISTRY OLYMPIAD) Explosivos produzem, em geral, um grande volume de
gases como produtos. A nitroglicerina detona de acordo com a seguinte reação: 2 C3H5N3O9(l) → 6 CO2(g) + 3 N2(g) +
5 H2O(g) + ½ O2(g). Se 1,0 grama de nitroglicerina sofre uma explosão, o volume de gases produzidos, se a pressão
total é de 1 atm e a temperatura 500°C, será de:
a) 1 Litros
b) 2 Litros
c) 3 Litros
d) 4 Litros
e) 5 Litros

Resolução: Alternativa B.

Cálculo do número de mol de gases produzidos, a partir de 1,0 grama de nitroglicerina: 2 C3H5N3O9(l) → 6 CO2(g) + 3
N2(g) + 5 H2O(g) + ½ O2(g)
2 mol de C3H5N3O9(l) ----------- (6 + 3 + 5 + ½) mol de gases
2 x 227 g ------------------------- 14,5 mol de gases
1,0 g ------------------------------ ngases
ngases = 0,032 mol de gases

Cálculo do volume dos gases produzidos: 1 x Vgases = 0,032 x 0,08206 x (500 + 273)
Vgases = 0,032 x 0,08206 x 773

78
1600 FÍSICO-QUÍMICA APLICADA EXERCÍCIOS COMENTADOS - IME – ITA – OLIMPÍADA

Vgases = 2,03 L

Questão 156 – (OLIMPÍADA DE QUÍMICA DO RIO DE JANEIRO) Uma amostra contendo 800 gramas de metano
sofreu uma reação de combustão completa. Um dos óxidos produzidos reagiu completamente com 6,132 kg de óxido
de bário, produzindo um sal. Qual é o percentual de pureza do metano utilizado?
a) 50%
b) 80%
c) 60%
d) 70%
e) 90%

Resolução: Alternativa D.

Equação química: CH4(g) + O2(g) → CO2(g) + H2O(g)

Equação química: CO2(g) + Ba(OH)2 → BaCO3 + H2O

Cálculo da massa de CO2, na reação de produção de carbonato de bário: CO2(g) + Ba(OH)2 → BaCO3 + H2O

1 mol de CO2(g) ---------- 1 mol de Ba(OH)2


44 g ----------------------- 171 g
mCO2 ---------------------- 6,132 kg
mCO2 = 1,58 kg

Cálculo da massa de CH4, a partir da reação de combustão completa: CH4(g) + O2(g) → CO2(g) + H2O(g)

1 mol de CH4(g) ----------- 1 mol de CO2(g)


16 g ------------------------ 44 g
mCH4 ----------------------- 1,58 kg
mCH4 = 0,574 kg
574 𝑔
Cálculo do grau de pureza de gás metano: (%)𝐶𝐻4 = 800 𝑔
= 0,72 (72%)

Questão 157 – (OLIMPÍADA BRASILEIRA DE QUÍMICA) Hipoclorito de sódio pode ser obtido através da seguinte
reação: Cl2(g) + 2 NaOH(aq) → NaCl(aq) + NaOCl(aq) + H2O(l). Considerando a existência de cloro gasoso em excesso,
qual o volume de uma solução de NaOH de concentração 2 mol.L-1 necessário para produzir hipoclorito em quantidade
suficiente para preparar 2,0 L de uma solução 0,50 mol.L-1 de NaOCl?
a) 1,0 Litros
b) 2,0 Litros
c) 3,0 Litros
d) 4,0 Litros
e) 5,0 Litros

Resolução: Alternativa A.

Relação estequiométrica química entre o hidróxido de sódio e a solução de hipoclorito de sódio: Cl2(g) + 2 NaOH(aq) →
NaCl(aq) + NaOCl(aq) + H2O(l)
2 mol de NaOH --------- 1 mol de NaOCl
nNaOH ---------------------- nNaOCl
2 x nNaOCl = nNaOH

Cálculo do volume de hidróxido de sódio: 2 x [NaOCl] x Vsolução = [NaOH] x Vsolução


2 x (0,50 x 2,0) = 2,0 x Vsolução
Vsolução = 1,0 L

79
1600 FÍSICO-QUÍMICA APLICADA EXERCÍCIOS COMENTADOS - IME – ITA – OLIMPÍADA

Questão 158 – Tratando-se 16 gramas de uma soda comercial, através de uma reação química entre o carbonato de
sódio e do ácido clorídrico, havendo liberação de um determinado gás. Este gás foi coletado em uma pressão de 1,5
atm e volume igual a 2,2 litros a 30ºC. Assinale a opção mais próxima do grau de pureza da soda.
a) 88%
b) 89%
c) 90%
d) 91%
e) 92%

Resolução: Alternativa A.

pideal x V 1,50 x 2,20 1,50 x 2,20


Cálculo do número de mol de dióxido de carbono: nCO2 = RxT
= 0,08206 x (30+273) = 0,08206 x 303 =
0,133 mol
mCO2
Cálculo da massa do número de massa de dióxido de carbono, a partir do número de mol: nCO2 = 44
=
0,133 mol

mCO2 = 0,133 x 44 = 5,84 g

Cálculo da massa de soda comercial: 2 HCl + Na2CO3 → 2 NaCl + H2O + CO2

1 mol de Na2CO3 ---------------------- 1 mol de CO2


106 gramas de Na2CO3 --------------- 44 gramas de CO2
mNa2CO3 ---------------------------------- 5,84 gramas de CO2
mNa2CO3 = 14,07 g
14,07 g
Cálculo do grau percentual de pureza da soda comercial: (%)Na2 CO3 = = 0,8793 (87,93%)
16 g

Questão 159 – (ITA) A massa de 0,48 kg de carbonato de amônio reage com excesso de ácido o-fosfórico de acordo
com a reação: 3 (NH4)2CO3 + 2 H3PO4 → 2 (NH4)3PO4 + 3 CO2 + 3 H2O. O volume de gás carbônico liberado a 0°C e
2,0 atm é:
a) 16,8 litros
b) 22,4 litros
c) 11,2 litros
d) 61,1 litros
e) 56,0 litros

Resolução: Alternativa E.

Cálculo do número de mol de dióxido de carbono: : 3 (NH4)2CO3 + 2 H3PO4 → 2 (NH4)3PO4 + 3 CO2 + 3 H2O
3 mol de (NH4)2CO3 ---------- 3 mol de CO2
480
mol de (NH4)2CO3 ---------- nCO2
96
nCO2 = 5 mol
480
nCO2 x R x T ( ) x 0,08206 x 273
Cálculo do volume de CO2, considerando comportamento ideal: VCO2 = pideal
= 96
2
= 56 L

80
1600 FÍSICO-QUÍMICA APLICADA EXERCÍCIOS COMENTADOS - IME – ITA – OLIMPÍADA

Questão 160 – Considere a seguinte equação química balanceada: CaCO3 + 2 HCl → CaCl2 + H2O + CO2. Com base
na equação acima podemos afirmar que 200 gramas de carbonato de cálcio produziu a seguinte massa de cloreto de
cálcio e volume de dióxido de carbono nas CNTP, respectivamente, supondo-se que a reação tenha 100% de
rendimento:
a) 111 gramas e 22,4 litros
b) 222 gramas e 22,4 litros
c) 111 gramas e 44,8 litros
d) 222 gramas e 44,8 litros
e) 111 gramas e 67,2 litros

Resolução: Alternativa D.

Equação química: CaCO3 + 2 HCl → CaCl2 + H2O + CO2

1 mol de CaCO3 ---------- 1 mol de CaCl2 ---------- 1 mol de CO2


100 g de CaCO3 ---------- 111 g de CaCl2 ---------- 22,4 Litros de CO2
200 g de CaCO3 ---------- mCaCl2 --------------------- VCO2

mCaCl2 = 222 g de CaCl2 e VCO2 = 44,8 L de CO2

Questão 161 – (OLIMPÍADA MINEIRA DE QUÍMICA) O airbag é um equipamento de segurança obrigatório no Brasil.
A bolsa de airbag é feita de um material bastante reforçado e resistente, que costuma ser o polímero nylon. No interior
dessa bolsa há uma mistura de reagentes: azoteto de sódio, nitrato de potássio e dióxido de silício. No momento da
colisão, sensores localizados em pontos estratégicos do carro detectam a forte desaceleração do veículo e são
acionados, emitindo sinais para uma unidade de controle. O sensor é ligado a um filamento que fica em contato com
uma pastilha de azoteto de sódio, dentro do airbag. Ele emite uma descarga elétrica, que aquece o azoteto, fornecendo
a energia de ativação necessária para dar inicio a reação, que libera grande quantidade de gás nitrogênio e sódio a
partir da decomposição do NaN3, em frações de segundo. Sabendo que o volume total de uma bolsa de airbag é de
70,0 litros e, que nas condições normais de temperatura e pressão (CNTP) 1 mol de gás corresponde a um volume
de 22,4 litros, qual é a massa de azoteto de sódio necessária para inflar todo o dispositivo de segurança?
a) 304 g
b) 67,5 g
c) 405 g
d) 135 g

Resolução: Alternativa D.
Cálculo da massa de azoteto de sódio: NaN3(s) → Na(s) + 3/2 N2(g)
1 mol de NaN3 ---------- 3/2 mol de N2
65 g ----------------------- 3/2 x (22,4 L)
mNaN3 --------------------- 70 L
mNaN3 = 135,42 g

Questão 162 – (ITA) Na temperatura ambiente, hidróxido de potássio sólido reage com o cloreto de amônio sólido,
com a liberação de um gás. Assinale a alternativa correta para o gás liberado nesta reação. a) Cl2 b) H2 c) HCl d) NH3
e) O2

Resolução: Alternativa D. Na temperatura ambiente, a reação entre o hidróxido de potássio sólido com o cloreto de
amônio sólido há a liberação de amônia.

Questão 163 - (ITA) Considere a equação química não balanceada, que representa a reação do sulfeto de cádmio
em solução aquosa com de ácido nítrico: CdS + HNO3 → Cd(NO3)2 + NO + Y + H2O. Pode-se afirmar que, na equação
química não balanceada, a espécie Y é:
a) Cd(HSO4)2
b) CdSO4
c) SO3

81
1600 FÍSICO-QUÍMICA APLICADA EXERCÍCIOS COMENTADOS - IME – ITA – OLIMPÍADA

d) SO2
e) S

Resolução: Alternativa E. Para que a reação esteja balanceada, o elemento químico Y será o enxofre (S).

Questão 164 – (ITA) Considere que 20 gramas de tiossulfato de potássio com pureza de 95% reagem com ácido
clorídrico em excesso, formando 3,2 gramas de um sólido de coloração amarela. Assinale a alternativa que melhor
representa o rendimento desta reação: K2S2O3 + 2 HCl → S + H2O + SO2 + 2 KCl.
a) 100%
b) 95%
c) 80%
d) 70%
e) 65%

Resolução: Alternativa A. Observação quanto ao enunciado do problema: a coloração amarela se refere ao enxofre.

Cálculo do rendimento deste processo: K2S2O3 + 2 HCl → S + H2O + SO2 + 2 KCl

1 mol de K2S2O3 ------------------------ 1 mol de S


190,32 gramas de K2S2O3 ------------- 32,06 gramas de S x R
0,95 x 20 gramas de K2S2O3 ---------- 3,2 gramas de S
R = 0,999 (≅ 100%)

Questão 165 – (U.S. NATIONAL CHEMISTRY OLYMPIAD) 0,250 gramas de um elemento M reage com excesso
com flúor, para produzir 0,547 gramas de MF6. Qual é o elemento?
a) Cr
b) Mo
c) S
d) Te

Resolução: Alternativa B.

Equação química balanceada: M + 3 F2 → MF6


1 mol de M ----------- 1 mol de MF6
<MM>M --------------- (<MM>M + 6 x 19)
0,250 g ---------------- 0,547 g

0,547 x <MM>M = 0,250 x (<MM>M + 114)


0,297. <MM>M = 28,50
<MM>M = 95,96 g.mol-1

Com o auxílio da tabela periódica, o elemento químico que apresenta a massa molar igual a 95,95 g.mol-1 trata-se do
Molibdênio (Mo).

Questão 166 – O número de átomos presentes em 2,20 gramas de propano é aproximadamente igual a:
a) 9,0 x 1022
b) 6,0 x 1022
c) 3,0 x 1023
d) 9,0 x 1023
e) 9,0 x 1021

Resolução: Alternativa C.

44 gramas de C3H8 ------------------------ 11 mol x (6,02 x 1023 átomos / mol)


2,20 gramas de C3H8 ---------------------- X
X = 3,31 x 1023 átomos

82
1600 FÍSICO-QUÍMICA APLICADA EXERCÍCIOS COMENTADOS - IME – ITA – OLIMPÍADA

Questão 167 – (U.S. NATIONAL CHEMISTRY OLYMPIAD) Aspirina, C9H8O4, é preparada a partir do ácido salicílico,
C7H6O3, de acordo com a seguinte equação química: C7H6O3 + (CH3CO)2O → C9H8O4 + CH3COOH. Se o rendimento
desta reação é de 83%, qual a massa de ácido salicílico deve ser preparado a partir de 1,0 kg de aspirina?
a) 0.77 kg
b) 0.92 kg
c) 1.2 kg
d) 1.3 kg

Resolução: Alternativa B.

Cálculo da massa ácido salicílico, considerando um rendimento de 83%: C7H6O3 + (CH3CO)2O → C9H8O4 + CH3COOH
1 mol de C7H6O3 ---------------- 1 mol de C9H8O4
138 g ------------------------------ 180 g x 0,83
mC7H6O3 --------------------------- 1 kg
mC7H6O3 = 0,92 kg

Questão 168 – (OLIMPÍADA PERUANA DE QUÍMICA) O ácido sulfúrico pode ser formado a partir de oxigênio, água
e dióxido de enxofre. Suponha que se faz reagir 400 gramas do óxido, 175 gramas de oxigênio e 125 gramas de água,
até que se esgote o reagente limitante. Que massa de ácido se forma?
a) 700 g
b) 612 g
c) 472 g
d) 0g

Resolução: Alternativa B.
Cálculo do número de mol de cada reagente químico:
mO 175
Analisando o oxigênio: nO2 = <MM>2 = 32
= 5,47 mol
O2

Pela estequiometria, temos: ½ O2(g) + H2O(l) + SO2(g) → H2SO4


½ mol de O2 ---------- 5,47 mol
1 mol de O2 ----------- nO2
nO2 = 10,94 mol
mH O 125
Analisando a água: nH2 O = <MM>2 = 18
= 6,94 mol
H2 O

mSO 400
Analisando o dióxido de enxofre: nSO2 = <MM>2 = 64
= 6,25 mol (reagente limitante)
SO2

Cálculo da massa de ácido sulfúrico produzido, a partir do dióxido de enxofre: ½ O2(g) + H2O(l) + SO2(g) → H2SO4

1 mol de SO2 ----------------- 1 mol de H2SO4


6,25 mol de SO2 ------------- nH2SO4
nH2SO4 = 6,25 mol de ácido sulfúrico.

Cálculo da massa de H2SO4, a partir do número de mol: mH2 SO4 = nH2 SO4 x < MM >H2 SO4 = 6,25 x 98 =
612,5 g

Questão 169 – (U.S. NATIONAL CHEMISTRY OLYMPIAD) Fe2O3 reage com excesso de CO a altas temperaturas,
de acordo com a seguinte equação: Fe2O3 + 3 CO → 2 Fe + 3 CO2. Se 6,50 gramas de Fe2O3 reage com 3,85 gramas
de ferro, qual é o rendimento da reação?
a) 59,2%
b) 69,9%

83
1600 FÍSICO-QUÍMICA APLICADA EXERCÍCIOS COMENTADOS - IME – ITA – OLIMPÍADA

c) 76,3%
d) 84,7%

Resolução: Alternativa D. Equação química balanceada: Fe2O3 + 3 CO → 2 Fe + 3 CO2

1 mol de Fe2O3 ------------- 2 mol de Fe


160 g ------------------------- 112 g x R
6,50 g ------------------------ 3,85 g
R = 0,8461 (84,61%)

Questão 170 – (OLIMPÍADA BRASILEIRA DE QUÍMICA) Considere a equação química que representa a oxidação
de cloreto por permanganato, em meio ácido: MnO4- + H+ + Cl- → Mn2+ + Cl2 + H2O. Após o balanceamento desta
equação, o coeficiente estequiométrico para o íon MnO4- é 2 e para o H+ é:
a) 4
b) 8
c) 10
d) 14
e) 16

Resolução: Alternativa E.
Realizando o balanceamento pelo método de oxidação-redução (redox), a equação química balanceada fica da
seguinte maneira: 2 MnO4- + 16 H+ + 10 Cl- → 2 Mn+2 + 5 Cl2 + 8 H2O

Questão 171 - (OLIMPÍADA BRASILEIRA DE QUÍMICA – PRIMEIRA FASE) A partir da pirita também se pode obter
ácido sulfúrico, segundo a sequência de reações, cujas equações químicas não – balanceadas são mostradas a
seguir:
FeS2 + O2 → Fe2O3 + SO2
SO2 + O2 → SO3
SO3 + H2O → H2SO4
Após o balanceamento destas equações, pode-se deduzir que a quantidade de matéria (número de mol) de ácido
sulfúrico obtida a partir de 1 mol de FeS2, é igual a:
a) 1
b) 2
c) 3
d) 4
e) 8

Resolução: Alternativa B.

Cálculo do número de mol de dióxido de enxofre: 2 FeS2 + 11/2 O2 → Fe2O3 + 4 SO2


2 mol de FeS2 ---------- 4 mol de SO2
1 mol de FeS2 ---------- nSO2
nSO2 = 2 mol

Cálculo do número de mol de trióxido de enxofre: SO2 + ½ O2 → SO3


1 mol de SO2 ---------- 1 mol de SO3
2 mol de FeS2 --------- nSO3
nSO3 = 2 mol

Cálculo do número de mol de ácido sulfúrico: SO3 + H2O → H2SO4


1 mol de SO3 ---------- 1 mol de H2SO4
2 mol de FeS2 --------- nH2SO4
nH2SO4 = 2 mol

84
1600 FÍSICO-QUÍMICA APLICADA EXERCÍCIOS COMENTADOS - IME – ITA – OLIMPÍADA

Questão 172 – (OLIMPÍADA BRASILEIRA DE QUÍMICA – PRIMEIRA FASE) O gás SO2 é formado na queima de
combustíveis fósseis. Sua liberação na atmosfera é um grave problema ambiental, pois através de uma série de
reações ele irá se transformar em H2SO4(aq), um ácido muito corrosivo, no fenômeno conhecido como chuva ácida. A
sua formação pode ser simplificadamente representada por: S(s) + O2(g) → SO2(g). Quantas toneladas de dióxido de
enxofre serão formadas caso ocorra a queima de uma tonelada de enxofre?
a) 1 tonelada
b) 2 toneladas
c) 3 toneladas
d) 4 toneladas
e) 5 toneladas

Resolução: Alternativa B.

Cálculo da massa de dióxido de enxofre: S(s) + O2(g) → SO2(g)


1 mol de S --------------- 1 mol de SO2
32 g de S(s) --------------- 64 g de SO2(g)
1 tonelada de S ---------- mSO2
mSO2 = 2 toneladas

Questão 173 – (OLIMPÍADA BRASILEIRA DE QUÍMICA) Se uma hemoglobina tem 0,335% de ferro, qual o valor
aproximado para a menor massa molar possível para esta hemoglobina?
a) 1,0 x 104
b) 1,7 x 104
c) 2,5 x 104
d) 6,7 x 104
e) 7,0 x 104

Resolução: Alternativa B.
0,335% ---------- 56 g.mol-1 de Ferro
100% ------------- <MM>Hemoglobina
<MM>Hemoglobina = 16716,42 g.mol-1 (1,7x104 g.mol-1)

Questão 174 – (OLIMPÍADA DE QUÍMICA DO RGS) Reagiu-se excesso de zinco metálico com 100 mL de solução
de ácido clorídrico de pH = 2,0. Qual é a massa, em gramas, de cloreto de zinco obtida?
a) 68,2
b) 1,36 x 10-1
c) 6,82 x 10-2
d) 13,6
e) 3,67 x 10-2

Resolução: Alternativa C.

Cálculo da concentração da quantidade de matéria dos íons hidrogeniônico [H+]: 2 = −log [H + ]


[H + ] = 10−2 mol. L−1

Equação de dissociação do ácido clorídrico: HCl(aq) → H+(aq) + Cl-(aq).

1 mol de HCl ---------- 1 mol de H+


[HCl] ------------------- 10-2 mol.L-1
[HCl] = 10-2 mol.L-1

Cálculo da massa de HCl, a partir da concentração da quantidade de matéria: mHCl = [HCl] x <
MM >HCl x Vsolução = 10−2 x 36,5 x 0,100 = 3,65 x 10−2 g

Cálculo da massa de cloreto de zinco: Zn(s) + 2 HCl(aq) → ZnCl2(aq) + H2(g)

85
1600 FÍSICO-QUÍMICA APLICADA EXERCÍCIOS COMENTADOS - IME – ITA – OLIMPÍADA

2 mol de HCl -------------- 1 mol de ZnCl2


2 x 36,5 g ------------------ 136 g
3,65 x 10-2 g --------------- mZnCl2
mZnCl2 = 0,068 g de cloreto de zinco

Questão 175 – (U.S. NATIONAL CHEMISTRY OLYMPIAD) Cloro pode ser produzido através da reação do HCl com
MnO2. A reação está representada através da seguinte equação química: MnO2(s) + HCl(aq) → Cl2(g) + MnCl2(aq) + 2
H2O(l). Assumindo que a reação fica completa, qual a massa de HCl concentrado é necessário para produzir 2,50
gramas de Cl2(g)?
a) 5,15 gramas
b) 14,3 gramas
c) 19,4 gramas
d) 26,4 gramas

Resolução: Alternativa A.

Equação química: MnO2(s) + HCl(aq) → Cl2(g) + MnCl2(aq) + 2 H2O(l)


4 mol de HCl ------------ 1 mol de Cl2
4 x 36,5 g --------------- 1 x 71 g
mHCl ---------------------- 2,50 g
mHCl = 5,14 g

Questão 176 – (OLIMPÍADA DE QUÍMICA DO RGS) Na reação representada pela equação: MnO4- + C2O4-2 + H+ →
Mn+2 + CO2 + H2O, após o balanceamento, usando os menores números inteiros, o coeficiente do agente oxidante
será:
a) 2
b) 5
c) 3
d) 8
e) 1

Resolução: Alternativa A.

Realizando o balanceamento pelo método de oxidação-redução, temos: MnO4- + C2O4-2 + H+ → Mn+2 + CO2 + H2O

Balanceamento químico: 2 MnO4- + 5 C2O4-2 + 16 H+ → 2 Mn+2 + 10 CO2 + 8 H2O

Questão 177 – (ITA) Um estudante preparou uma mistura A, constituída dos seguintes sólidos: cloreto de sódio,
cloreto de potássio e cloreto de bário. Numa primeira experiência, foi preparada uma solução aquosa pela total
dissolução de 34,10 gramas da mistura A em água destilada, a 25ºC, à qual foi adicionada, a seguir, uma solução
aquosa de nitrato de prata em excesso, obtendo-se 57,40 gramas de um certo precipitado. Num segundo experimento,
foi preparada uma solução aquosa pela total dissolução de 6,82 gramas da mistura A em água destilada, a 25ºC, à
qual foi adicionada, a seguir, uma solução aquosa de sulfato de sódio em excesso, obtendo-se 4,66 gramas de um
outro precipitado. Qual das opções abaixo apresenta o valor correto da composição percentual, em massa, da mistura
A?
a) 17,2% de NaCl, 21,8% de KCl 61,0% de BaCl2
b) 21,8% de NaCl, 17,2% de KCl 61,0% de BaCl2
c) 61,0% de NaCl, 21,8% de KCl 17,2% de BaCl2
d) 21,8% de NaCl, 61,0% de KCl 17,2% de BaCl2
e) 61,0% de NaCl, 17,2% de KCl 21,8% de BaCl2

Resolução: Alternativa A.
Primeira Experiência: Nesta ocorrem as seguintes reações:
NaCl + AgNO3 → AgCl

86
1600 FÍSICO-QUÍMICA APLICADA EXERCÍCIOS COMENTADOS - IME – ITA – OLIMPÍADA

KCl + AgNO3 → KNO3 + AgCl


BaCl2 + 2AgNO3 → Ba(NO3)2 + 2AgCl

Resultando em: NaCl + KCl + BaCl2 + 4 AgNO3 → NaNO3 + KNO3 + Ba(NO3)2 + 4 AgCl

As substâncias a serem relacionadas são aquelas que fazem parte da mistura, pois estas são incógnitas, e o
precipitado, pois dele temos dados suficientes. O precipitado, cuja solubilidade é desprezível, deve ser um dos
produtos, que neste caso é o AgCl. Temos então a seguinte relação:
1 x 58,44 -------------------- 4 x 143,32
X --------------------- 57,4
X = 5,85 de NaCl

Cálculo da porcentagem:
34,10g -------- 100
5,85g --------- P
P = 17,15

Segunda Experiência: BaCl2 + 2KCl + 2Na2SO4 → 4NaCl + K2SO4 + BaSO4

Agora o precipitado é BaSO4 (Sulfato de bário)


BaCl2 BaSO4
208,23g --------------------------- 233,39g
X --------------------------- 4,66
X = 4,15g

Cálculo da porcentagem:
6,82 g ---------- 100
4,15g ---------- P
P  61%

Temos que o cloreto de potássio apresenta percentual igual a 21,8%.

Questão 178 – (ITA) Uma camada escura é formada sobre objetos de prata exposta a uma atmosfera poluída
contendo compostos de enxofre. Esta camada pode ser removida quimicamente envolvendo os objetos em questão
com uma folha de alumínio. A equação química que melhor representa a reação que ocorre neste caso é
a) 3Ag2S(s) + 2Al(s) → 6Ag(s) + Al2S3(s)
b) 3Ag2O(s) + 2Al(s) → 6Ag(s) + Al2O3(s)
c) 3Ag2H(s) + Al(s) → 3Ag(s) + AlH3(s)
d) 3Ag2SO4(s) + 2Al(s) → 6Ag(s) + Al2S3(s) + 6O2(g)
e) 3Ag2SO3(s) + 2Al(s) → 6Ag(s) + Al2S3(s) + 9/2 O2(g)

Resolução: Alternativa A.
O enunciado cita uma substância escura que se forma sobre objetos de prata, quando estes estão expostos a
ambientes poluídos com compostos de enxofre. A substância escura que deve ser removida da superfície do objeto
refere-se ao sulfeto de prata, o qual reagirá com o alumínio metálico.
Equação química: 3 Ag2S(s) + 2 Al(s) → 6 Ag(s) + Al2S3(s)

Questão 179 – (ITA) Em um copo de 500 mL são misturados 100 mL de ácido clorídrico 1,00 mol.L-1 em 100 mL de
hidróxido de sódio 0,50 mol.L-1. A solução resultante do copo é:
a) 1,0 x 10-7 mol.L-1 em OH-
b) 1,0 x 10-7 mol.L-1 em H+
c) 0,05 mol.L-1 em H+
d) 0,25 mol.L-1 em H+
e) 0,50 mol.L-1 em H+

87
1600 FÍSICO-QUÍMICA APLICADA EXERCÍCIOS COMENTADOS - IME – ITA – OLIMPÍADA

Resolução: Alternativa D.

Relação estequiométrica entre os reagentes: HCl(aq) + NaOH(aq) → NaCl(aq) + H2O(l)

1 mol de HCl(aq) ---------- 1 mol de NaOH(aq)


nHCl ------------------------- nNaOH
nHCl = nNaOH

Cálculo do número de mol de HCl: nHCl = 0,10 x 1,0 = 0,10 mol


Cálculo do número de mol de NaOH: nNaOH = 0,50 x 0,10 = 0,05 mol

Excesso de ácido: nHCl excesso = 0,10 - 0,05 = 0,05 mol

0.005 mol
Cálculo da concentração da quantidade de matéria em excesso: [HCl]excesso = 0,200 L
= 0,25 mol. L−1

Questão 180 – (OLIMPÍADA MINEIRA DE QUÍMICA) Na metalurgia de obtenção do zinco a partir da blenda, um
minério constituído de sulfeto de zinco, ocorre por meio de duas reações, representadas abaixo:
2 ZnS(s) + 3 O2(g) → 2 ZnO(s) + 2 SO2(g)
ZnO(s) + CO(g) → Zn(s) + CO2(g)
Qual quantidade de zinco a 90%(m/m) pode ser obtida a partir de 243,75 toneladas do minério blenda, a 80%(m/m)?
a) 147,15 toneladas
b) 130,80 toneladas
c) 117,72 toneladas
d) 163,50 toneladas

Resolução: Alternativa C

2 ZnS(s) + 3 O2(g) → 2 ZnO(s) + 2 SO2(g)


2 ZnO(s) + 2 CO(g) → 2 Zn(s) + 2 CO2(g) (Reação multiplicada por dois) +
2 ZnS(s) + 3 O2(g) + 2 CO(g) → 2 Zn(s) + 2 CO2(g) + 2 SO2(g) (Reação Global)

Cálculo da pureza de blenda = 243,75 t x 0,80 = 185 t.

2 ZnS(s) + 3 O2(g) + 2 CO(g) → 2 Zn(s) + 2 CO2(g) + 2 SO2(g)

2 mol de ZnS -------------- 2 mol de Zn


(2 x 97,4) g ---------------- 2 x 65,4 g
195 toneladas ------------- mZn
mZn = 130,93 x 0,90 = 117,84 t

Questão 181 – (OLIMPÍADA MINEIRA DE QUÍMICA) A um paciente, o qual se queixava de dor de cabeça, foi
receitado metamizol sódico (ou dipirona sódica), em uma sondagem de 250 mg, a cada 6 horas. A fórmula estrutural
deste analgésico é dada abaixo:

A quantidade de matéria do analgésico ingerida pelo paciente por dia é igual a:


a) 0,015 mol
b) 0,0015 mol

88
1600 FÍSICO-QUÍMICA APLICADA EXERCÍCIOS COMENTADOS - IME – ITA – OLIMPÍADA

c) 0,003 mol
d) 0,03 mol

Resolução: Alternativa C.
1 dia ---------- 24 horas ----------- massa
6 horas ------------- 250 mg
massa = 1000 mg

Fórmula molecular do metamizol sódico e sua massa molar: C13H16O4N3SNa (<MM> = 333 g.mol-1)
1 mol ---------- 333 gramas
X --------------- 1 grama
X = 0,0030 mol

Questão 182 – (OLIMPÍADA BRASILEIRA DE QUÍMICA) A pirita (FeS2) é um minério de ferro conhecido como ouro
de tolo em face de sua aparência. Quando queimada na presença de oxigênio do ar, a pirita é convertida nos óxidos
Fe2O3 e SO2. O ferro é então obtido do óxido de ferro em um alto forno. A massa de ferro (em kg) que pode ser obtida
a partir de 1 tonelada de pirita de pureza igual a 95% está entre:
a) 200 e 300 kg
b) 300 e 350 kg
c) 350 e 400 kg
d) 400 e 450 kg
e) 450 3 500 kg

Resolução: Alternativa D.

Equação química I: 2 FeS2(s) + 11/2 O2(g) → Fe2O3(s) + 4 SO2(g)


Equação química II: Fe2O3(s) → 2 Fe(s) + 3/2 O2(g)

Somando as duas reações químicas, temos:


2 FeS2(s) + 11/2 O2(g) → Fe2O3(s) + 4 SO2(g)
Fe2O3(s) → 2 Fe(s) + 3/2 O2(g) +
2 FeS2(s) + 4 O2(g) → 2 Fe(s) + 4 SO2(g)

Massa de pirita puro = 1 t x 0,95 = 0,95 toneladas

2 FeS2(s) + 4 O2(g) → 2 Fe(s) + 4 SO2(g)


240 gramas ------------- 112 gramas
0,95 toneladas ---------- mFe
mFe = 0,443 toneladas (443 kg)

Questão 183 – (IME) Em 33,65 gramas de um sal de magnésio está presente 1 mol deste elemento. Sendo trivalente
o ânion deste sal, é correto afirmar que a massa de 1 mol do ânion é:
a) 6,23 g
b) 14,01 g
c) 24,31 g
d) 42,03 g
e) 48,62 g

Resolução: Alternativa B.

Sal constituinte: Mg+2 e X-3


Massa molecular do sal: <MM>Mg3X2 = (72 + 2.<MM>X), onde <MM>X é a massa atômica do elemento X.

72 g ---------- (72 + 2 x <MM>X)


24 g ---------- 33,65 g

89
1600 FÍSICO-QUÍMICA APLICADA EXERCÍCIOS COMENTADOS - IME – ITA – OLIMPÍADA

<MM>X = 14,47 g.mol-1

Questão 184 – (OLIMPÍADA MINEIRA DE QUÍMICA) O TiO2, na forma de rutilo, é usado como pigmento em tintas
brancas. Essa substância pode ser obtida com rendimento de 100%, por meio da reação entre a ilmenita, FeTiO3, e
o cloro, em meio ácido, de acordo com a equação química não balanceada: ___ FeTiO3(s) + ___ HCl(aq) + ___ Cl2(g) →
___ FeCl3(aq) + ___ TiO2(s) + ___ H2O(l). Qual a quantidade de matéria de rutilo pode ser preparada a partir de 950
gramas de um minério que contém 80% (m/m) de ilmenita?
a) 6,25 mol
b) 6,50 mol
c) 5,00 mol
d) 5,50 mol

Resolução: Alternativa C.

Realizando o balanceamento por redox, a equação química fica da seguinte maneira: 3 FeTiO3(s) + 6 HCl(aq) + 3/2
Cl2(g) → 3 FeCl3(aq) + 3 TiO2(s) + 3 H2O(l).

Cálculo da massa de ilmenita pura: mFeTiO3 = 950 g x 0,80 = 760 g

Cálculo do número de mol de rutilo:


3 mol de FeTiO3 -------------------------- 3 mol de TiO2
3 x (152 g) --------------------------------- 3 mol de TiO2
760 g ---------------------------------------- nTiO2
nTiO2 = 5,00 mol

Questão 185 – (IME) CrI3 + CI2 + NaOH → NaIO4 + Na2CrO4 + NaCl + H2O. Assinale a alternativa que indica a soma
dos menores coeficientes inteiros capazes de balancear a equação química acima:
a) 73
b) 95
c) 173
d) 187
e) 217

Resolução: Alternativa D.

Balanceamento pelo método íon-elétron, temos:


x2
4 H2 O + Cr +3 → CrO−2 + −
4 + 8 H + 3e (Oxidação)
x6
4 H2 O + I − → IO−2 + −
4 + 8 H + 8e (Oxidação)
x27
Cl2 + 2e− → 2 Cl− (Redução)
32 H2 O + 2 Cr +3 + 6 I − + 27 Cl2 → 2 CrO−2 − −
4 + 6 IO4 + 54 Cl + 64 H
+

Como a reação ocorre em meio básico, iremos eliminar os íons H+ adicionando OH- em ambos os lados. Logo:

2 CrI3 + 27 Cl2 + 64 NaOH → 6 NaIO4 + 2 Na2CrO4 + 54 NaCl + 32 H2O (Soma dos coeficientes estequiométricos = 2
+ 27 + 64 + 6 + 2 + 54 + 32 = 187)

Questão 186 – (ITA) Uma mistura de 300 mL de metano e 700 mL de cloro foi aquecida no interior de um cilindro
provido de um pistão móvel sem atrito, resultando na formação de tetracloreto de carbono e cloreto de hidrogênio.
Considere todas as substâncias no estado gasoso e temperatura constante durante a reação. Assinale a opção que
apresenta os volumes corretos, medidos nas mesmas condições de temperatura e pressão, das substâncias
presentes no cilindro após reação completa.

VCH4 (mL) VCl2 (mL) VCCl4 (mL) VHCl (mL)


a) 0 0 300 700

90
1600 FÍSICO-QUÍMICA APLICADA EXERCÍCIOS COMENTADOS - IME – ITA – OLIMPÍADA

b) 0 100 300 600


c) 0 400 300 300
d) 125 0 175 700
e) 175 0 125 700

Resolução: Alternativa D.

Equação química balanceada: CH4(g) + 4 Cl2(g) → CCl4(g) + 4 HCl(g)

1 mol 4 mol 1 mol 4 mol


100 mL 400 mL 100 mL 400 mL
x 700 mL x y
700 x 100
O gás metano encontra-se em excesso, e, a quantidade que reage será a seguinte: x = 400
= 175 mL

Pela Lei de Avogadro, V  nmol.

Questão 187 – (OLIMPÍADA MARANHENSE DE QUÍMICA) Qual é a massa de carbonato de prata formada ao
misturarmos 100 mL de uma solução 0,1 mol.L-1 de nitrato de prata com 100 mL de uma solução 0,1 mol.L-1 de
carbonato de sódio?
a) 0,69 g
b) 1,38 g
c) 2,76 g
d) 1,72 g
Resolução: Alternativa B.

Determinação do reagente limitante e do reagente em excesso:

mol 0,010
nAgNO3 = 0,10 L x 0,10 = mol = 0,005 mol (reagente limitante)
L 2

nNa2 CO3 = 0,10 x 0,10 = 0,01 mol

Cálculo da massa de carbonato de prata: 2 AgNO3(aq) + Na2CO3(aq) → Ag2CO3(s) + 2 NaNO3(aq)


2 mol de AgNO3(aq) -------------------- 1 mol de Ag2CO3(s)
nAgNO3 ------------------------------------ nAg2CO3
nAgNO3 = 2 x nAg2 CO3

mAg2 CO3
[AgNO3 ] x Vsolução = 2 x
< MM >Ag2 CO3

[AgNO3 ] x Vsolução x < MM >Ag2 CO3 0,1 x 0,1 x 276


mAg2 CO3 = = = 1,38 g
2 2

Questão 188 – (OLIMPÍADA MARANHENSE DE QUÍMICA) Um método de análise desenvolvido por Lavoisier (1743
– 1794) e aperfeiçoado por Leibig (1803 – 1873) permitiu determinar a composição percentual dos hidrocarbonetos.
O procedimento baseia-se na combustão total – em excesso de oxigênio (O2) – da amostra analisada, em que todo
carbono é convertido em gás carbônico (CO2) e todo hidrogênio transformado em água (H2O). A queima de 0,50
gramas de um hidricarboneto, em presença de oxigênio em excesso, fornece 1,65 gramas de dióxido de carbono
(CO2) e 0,45 gramas de água (H2O). Considerando as informações acima, pode-se afirmar que as porcentagens em
massa de carbono (C) e hidrogênio (H) no hidrocarboneto são respectivamente:
a) 85% e 15%
b) 95% e 5%
c) 90% e 10%

91
1600 FÍSICO-QUÍMICA APLICADA EXERCÍCIOS COMENTADOS - IME – ITA – OLIMPÍADA

d) 91% e 9%
e) 93% e 12%

Resolução: Alternativa C.

Hidrocarboneto do tipo CxHy

𝒚 𝒚
Equação química balanceada: 𝑪𝒙 𝑯𝒚 + (𝒙 + 𝟒) 𝑶𝟐(𝒈) → 𝒙 𝑪𝑶𝟐(𝒈) + 𝟐 𝑯𝟐 𝑶(𝒈)

1 mol de CxHy -------------------- x mol de CO2 ---------- y/2 mol de H2O


(12x + y) mol de CxHy ---------- 44x ---------------------- 9y
0,50 g ----------------------------- 1,65 g ------------------- 0,45 g

Realização da primeira equação matemática, a partir da estequiometria entre o CxHy e o CO2:


1,65.(12x + y) = 0,50.44x
19,8x + 1,65y = 22x
Y = 1,33x (Equação I)

Realização da segunda equação matemática, a partir da estequiometria entre o CxHy e o H2O:


0,45 x (12x + y) = 0,50 x 9y
5,4x + 0,45y = 0,45y

Y = 1,33x (Equação II)

Tanto para a equação I quanto a equação II, a relação entre y e x é igual a y = 1,33x
Logo, para x = 3, y = 4.
Hidrocarboneto = C3H4
𝟑𝟔
Cálculo do percentual de carbono no hidrocarboneto: %(carbono) = 𝟒𝟎 = 𝟎, 𝟗𝟎 (𝟗𝟎%)
%(hidrogênio) = 100% - 90% = 10%

Questão 189 – (OLIMPÍADA MARANHENSE DE QUÍMICA) A reação de explosão da nitroglicerina acontece quando
este composto é submetido a uma onda de choques provocada por um detonador, causando sua decomposição de
acordo com a reação: 4 C3H5(NO3)3(l) → 6 N2(g) + O2(g) + 12 CO2(g) + 10 H2O(g). Considerando que esta reação ocorre
a 1,0 atm e a 298,15 K e que os gases gerados apresentam comportamento ideal, assinale a alternativa que
corretamente indica o volume total (em L) de gás produzido quando ocorre a explosão de quatro mol de nitroglicerina.
a) 509
b) 609
c) 709
d) 809
e) 909

Resolução: Alternativa C.
Observando a reação química a partir de 4 mol de nitroglicerina consumidos são formados 29 mol de gases. Com
isso, considerando que os gases apresentam comportamento ideal, o volume será calculado a partir da equação dos
gases ideais.

29 x 0,08206 x 273
V= = 709,16 L
1,0

Questão 190 – (OLIMPÍADA MARANHENSE DE QUÍMICA) O principal componente da cal, importante produto
industrial fabricado no Ceará, é o óxido de cálcio (CaO). A produção de CaO se processa de acordo com a seguinte
reação química: CaCO3(s) → CaO(s) + CO2(g). Considerando o comportamento ideal, assinale a alternativa que
expressa corretamente o volume (em L) de CO2 gerado na produção de 561 kg de CaO a 300 K e 1 atm.

92
1600 FÍSICO-QUÍMICA APLICADA EXERCÍCIOS COMENTADOS - IME – ITA – OLIMPÍADA

a) 22,4
b) 224
c) 2460
d) 24600
e) 246000

Resolução: Alternativa E.

Cálculo da massa de dióxido de carbono: CaCO3(s) → CaO(s) + CO2(g)

1 mol de CaO ---------------- 1 mol de CO2


56 gramas de CaO ---------- 44 gramas de CO2
561 kg ------------------------ mCO2
mCO2 = 440,58 kg
440580
Cálculo do volume de dióxido de carbono: VCO2 = 44 x 1
x 0,08206 x 300 = 246504,51 L

Questão 191 – (OLIMPÍADA DE QUÍMICA DO RIO GRANDE DO NORTE) A scheelita é um mineral de tungstato de
cálcio (CaWO4) encontrado em quantidades apreciáveis na região do Seridó do estado do Rio Grande do Norte, e se
constitui numa das principais fontes do metal do tungstênio, o qual é utilizado em filamentos de lâmpadas e na indústria
bélica. O método clássico da obtenção de tungstênio a partir da scheelita segue duas etapas principais: na primeira,
o mineral reage com excesso de HCl em altas temperaturas produzindo o óxido de tungstênio (IV) com rendimento
teórico de 65%, segundo a reação: CaWO4 + 2 HCl → WO3 + H2O + CaCl2. Em seguida, o óxido de tungstênio é
reduzido em presença de gás hidrogênio produzindo o metal na forma pura com rendimento médio de 98% a partir da
reação: WO3 + 3 H2 → W + 3 H2O. Quantas toneladas do metal tungstênio podem ser produzidas a partir de 65
toneladas de scheelita, imaginando que este mineral é constituído apenas por CaWO4?
a) 26,45
b) 27,00
c) 41,53
d) 50,52
e) 66,13

Resolução: Alternativa A.

Cálculo da massa de WO3, a partir da primeira reação química: CaWO4 + 2 HCl → WO3 + H2O + CaCl2

1 mol de CaWO4 ----------------- 1 mol de WO3


288 gramas de CaWO4 ---------- 232 gramas de WO3 x 0,65
65 t de CaWO4 ------------------- mWO3

232 x 65 x 0,65 9802


mW𝑂3 = = = 34,03 𝑡
288 288

Cálculo da massa de W (tungstênio), a partir da segunda reação química: WO3 + 3 H2 → W + 3 H2O

1 mol de WO3 -------------------- 1 mol de W


232 gramas de CaWO4 ---------- 184 gramas de W x 0,98
34,03 t de WO3 ------------------- mW
mW = 26,45 t

Questão 192 – (OLIMPÍADA CEARENSE DO ENSINO SUPERIOR DE QUÍMICA) Os íons tiossulfatos (S2O3-2)
desproporcionam-se em uma solução ácida produzindo o enxofre sólido (S) e o íon hidrogeno sulfito (HSO3-): 2 S2O3-
(aq) + 2 H3O (aq) → 2 HSO3 (aq) + 2 H2O(l) + 2S(s). Assumindo que a reação se completa e que uma solução contendo
2 + -

10,1 mL de íons HSO3 (aq) (55% m/m) cuja densidade é 1,45 g.cm-3, é obtida na reação, determine a massa de S2O3-
-

(aq) presente inicialmente na reação:


2

93
1600 FÍSICO-QUÍMICA APLICADA EXERCÍCIOS COMENTADOS - IME – ITA – OLIMPÍADA

a) 11,13 gramas
b) 14,64 gramas
c) 8,05 gramas
d) 0,750 gramas
e) 23,30 gramas

Resolução: Alternativa A.
Equação Iônica: 2 S2O3-2(aq) + 2 H3O+(aq) → 2 HSO3-(aq) + 2 H2O(l) + 2S(s)

Pela estequiometria, temos:


2 mol de S2O3-2 ------------- 2 mol de HSO3-
nS2O3-2 ------------------------ nHSO3-
n𝑆2 𝑂3−2 = n𝐻𝑆𝑂3−

Cálculo da concentração mássica, a partir da densidade da solução e do título: C = 10 x d x (%) =


g
10 x 1,45 x 55 = 797,50 L

C 797,50
Cálculo da concentração da quantidade de matéria do HSO3-: [HSO−
3] = = = 9,84 mol. L−1
<MM>HSO− 81
3

mS O−2
Cálculo da massa de tiossulfato: <MM>2 3
= Vsolução x [HSO−
3]
S2 O−2
3

mS2 O−2
3
= Vsolução x [HSO−
3 ] x < MM >S2 O−2
3
= 11,13 g

Questão 193 – (OLIMPÍADA CEARENSE DO ENSINO SUPERIOR DE QUÍMICA) Titânio metálico e suas ligas são
bastante utilizadas na indústria aeroespacial devido a vantagem de apresentarem pesos leves e alta resistência.
Titânio pode ser obtido a partir do TiCl4, que por sua vez é obtido através do TiO2, o qual constitui a maior fonte natural
do titânio. A reação de produção do tetracloreto de titânio é mostrada abaixo: TiO 2(s) + C(s) + Cl2(g) → TiCl4(g) + CO2(g)
+ CO(g). Em um frasco reacional contendo 5,00 g de TiO2, 4,50 g de C e 7,78 g de Cl2 e supondo o consumo completo
dos reagentes, qual a quantidade máxima de TiCl4 produzida?
a) 11,88 g
b) 53,30 g
c) 10,41 g
d) 12,48 g
e) 54,08 g

Resolução: Alternativa C. Equação química: 6 TiO2(s) + 8 C(s) + 12 Cl2(g) → 6 TiCl4(g) + 4 CO2(g) + 4 CO(g)
mTiO 5,0 0,0625
Determinação do reagente limitante: nTiO2 = <MM> 2 = 80
= 6
= 0,0104 mol
TiO2

mC 4,50 0,375
nC = = = = 0,0469 mol
< MM >C 12 8
mCl 7,78 0,1096
nCl2 = <MM>2 = 71
= 12
= 0,0091 mol (reagente limitante)
Cl2
Cálculo da massa de tetracloreto de titânio: 6 TiO2(s) + 8 C(s) + 12 Cl2(g) → 6 TiCl4(g) + 4 CO2(g) + 4 CO(g)

12 mol de Cl2 ---------------- 6 mol de TiCl4


852 gramas de Cl2 ----------- 1140 gramas de TiCl4
7,78 gramas de Cl2 ---------- mTiCl4
mTiCl4 = 10,41 g

Questão 194 – (OLIMPÍADA CEARENSE DO ENSINO SUPERIOR DE QUÍMICA) Considerando que o zinco metálico

94
1600 FÍSICO-QUÍMICA APLICADA EXERCÍCIOS COMENTADOS - IME – ITA – OLIMPÍADA

reage com soluções de ácidos, como por exemplo, o ácido clorídrico em água, usada para preparar o hidrogênio em
laboratório, de acordo com a seguinte reação:
Zn(s) + 2 HCl(aq) → ZnCl(aq) + H2(g). Ao dissolver 12,05 g de zinco, qual o volume em mililitros de ácido clorídrico 2,0
mol.L-1 necessários para converter completamente o zinco a cloreto de zinco?
a) 184 mL
b) 368 mL
c) 200 mL
d) 18,4 mL
e) 120 mL

Resolução: Alternativa A. Equação química: Zn(s) + 2 HCl(aq) → ZnCl(aq) + H2(g)

1 mol de Zn ---------- 2 mol de HCl


nZn --------------------- nHCl
2 x nZn = nHCl
mZn
[HCl] x Vsolução = 2 x
< MM >Zn
12,05
2 x Vsolução =2x
65,4

Vsolução = 0,184 L (184 mL)

Questão 195 – (OLIMPÍADA MARANHENSE DE QUÍMICA) Uma das formas de monitoramento da água de caldeiras
de indústrias, responsável pelo fornecimento de energia, é a medição do teor de íons cloreto (Cl-). Um método de
análise de cloreto é a adição à água de caldeira de uma solução de nitrato de prata (AgNO 3), segundo a equação:
Cl−(aq) + AgNO3(aq) → AgCl(s) + NO3−(aq). A análise de 10,0 ml de água de caldeira, contendo Cl−, com solução aquosa
de AgNO3, formou um precipitado de 0,287 g de AgCl. A concentração molar de Cl− presente na amostra de água de
caldeira é igual a:
a) 2
b) 0,2
c) 0,02
d) 0,002
e) 0,0002

Resolução: Alternativa B.
Cl−(aq) + AgNO3(aq) → AgCl(s) + NO3−(aq)

1 mol de Cl− ---------- 1 mol de AgCl


nCl− --------------------- nAgCl
nCl− = nAgCl
𝒎𝑨𝒈𝑪𝒍
[𝑪𝒍− ] 𝒙 𝑽𝒔𝒐𝒍𝒖çã𝒐 =
< 𝑴𝑴 >𝑨𝒈𝑪𝒍

𝟎, 𝟐𝟖𝟕
[𝑪𝒍− ] 𝒙 𝟏𝟎 𝒙 𝟏𝟎−𝟑 =
𝟏𝟒𝟑, 𝟓

𝒎𝒐𝒍
[𝑪𝒍− ] = 𝟎, 𝟐𝟎
𝑳

95
1600 FÍSICO-QUÍMICA APLICADA EXERCÍCIOS COMENTADOS - IME – ITA – OLIMPÍADA

Questão 196 – (OLIMPÍADA DE QUÍMICA DO RIO DE JANEIRO) O clorato de potássio, ao ser aquecido, se
decompõe gerando cloreto de potássio e gás oxigênio. Num experimento, foram aquecidos 25 g de clorato de potássio
e constatou-se que 75% do sal se decompuseram. Todo o gás liberado foi coletado sobre a água, o volume ocupado
foi de 2,25 L obtendo-se uma pressão de 1,25 atm. A temperatura do sistema era de 25°C. Considerando o oxigênio
um gás ideal, determine o grau de pureza do clorato de potássio.
a) 100 %
b) 84 %
c) 72 %
d) 61 %
e) 50 %

Resolução: Alternativa E.

Cálculo do número de mol de oxigênio coletado, a partir da utilização da equação dos gases ideais: 1,25 x 2,25 =
nO2 x 0,08206 x 298
2,8125
nO2 = = 0,115 mol
24,45

Cálculo da massa de clorato de potássio, a partir da estequiometria: KClO3(s) → KCl(s) + 3/2 O2(g)

1 mol de KClO3 ---------------------- 3/2 mol de O2


122,50 gramas de KClO3 ----------- 3/2 mol de O2 x (¾)
mKClO3 --------------------------------- 0,115 mol de O2
mKClO3 = 12,52 gramas de KClO3

𝟏𝟐,𝟓𝟐
Cálculo do grau de pureza do clorato de potássio (%) = 𝟐𝟓
= 𝟎, 𝟓𝟎 (𝟓𝟎%)

Questão 197 – (OLIMPÍADA DE QUÍMICA DO RIO DE JANEIRO) Uma amostra contendo 800 g de metano sofreu
uma reação de combustão completa. Um dos óxidos produzidos reagiu completamente com 6,132 kg de óxido de
bário, produzindo um sal. Qual é o percentual de pureza do metano utilizado?
a) 50%
b) 80%
c) 60%
d) 70%
e) 90%

Resolução: Alternativa B.

Equação química 1: CH4 + O2 → CO2 + H2O


Equação química 2: CO2 + BaO → BaCO3

Somando as duas equações químicas:


CH4 + 2 O2 → CO2 + 2 H2O
CO2 + BaO → BaCO3 +
CH4 + 2 O2 + BaO → BaCO3 + 2 H2O

Cálculo da massa de metano:


1 mol de CH4 ---------- 1 mol de BaO
16 gramas -------------- 153 gramas
mCH4 -------------------- 6152 gramas
mCH4 = 643,35 g
643,35
Cálculo da pureza (%): % = 800
= 0,801 (80%)

96
1600 FÍSICO-QUÍMICA APLICADA EXERCÍCIOS COMENTADOS - IME – ITA – OLIMPÍADA

Questão 198 - (ITA) Num garrafão de 3,50 litros de capacidade, contendo 1,50 litros de solução 1,0 mol.L-1 de ácido
sulfúrico, introduzem-se 32,7 g de aparas de zinco; fecha-se rapidamente com rolha de borracha. Supondo que a
temperatura do ambiente onde esta perigosa experiência está sendo feita seja de 20C, o incremento máximo de
pressão interna (P) do frasco, em atm, será de:
a) 0,41
b) 3,4
c) 5,6
d) 6,0
e) 12,0

Resolução: Alternativa D.

Equação química: Zn(s) + H2SO4(aq) → ZnSO4(aq) + H2(g)

Determinação do reagente limitante:

Cálculo do número de mol de ácido: nácido = [ácido] x Vsolução = 1,0 x 1,5 = 1,5 mol

m 32,7
Cálculo do número de mol do metal: nmetal = <MM>
metal
= 65,4
= 0,50 mol (reagente limitante)
metal

Cálculo do número de mol de gás hidrogênio produzido na reação: Zn(s) + H2SO4(aq) → ZnSO4(aq) + H2(g)
1 mol de Zn ----------------------- 1 mol de H2
0,50 mol ---------------------------- nH2
nH2 = 0,50 mol de gás hidrogênio
∆n x R x T 0,50 x 0,08206 x 293
Cálculo da variação máxima de pressão: ∆P = V
= 2
= 6,01 atm

Questão 199 - (IME) Oleum, ou ácido sulfúrico, é obtido através da absorção do trióxido de enxofre por ácido sulfúrico.
Ao se misturar oleum com água obtém-se ácido sulfúrico concentrado. Supondo que uma indústria tenha comprado
1000 kg de oleum com concentração em massa de trióxido de enxofre de 20% e de ácido sulfúrico de 80%, calcule a
quantidade de água que deve ser adicionada para que seja obtido ácido sulfúrico com concentração de 95% em
massa.
a) 42 kg
b) 300 kg
c) 100 kg
d) 45 kg
e) 104,5 kg
Resolução: Alternativa C.

Sabendo que 1000 kg de oleum apresentam 200 kg de SO3 + 800 kg de H2SO4 puro.

Cálculo da massa de água necessária para transformar o trióxido de enxofre em ácido sulfúrico:

Analisando o trióxido de enxofre:

1 mol de SO3 --------------- 1 mol de H2O


80 gramas de SO3 ---------- 18 gramas de H2O
200 kg de SO3 -------------- XH2O
XH2O = 45 kg

Analisando o ácido sulfúrico:

1 mol de SO3 --------------- 1 mol de H2SO4


80 gramas de SO3 ---------- 98 gramas de H2O

97
1600 FÍSICO-QUÍMICA APLICADA EXERCÍCIOS COMENTADOS - IME – ITA – OLIMPÍADA

200 kg de SO3 -------------- YH2SO4


YH2SO4 = 245 kg

Cálculo da massa de água necessária para a transformação desta massa de ácido sulfúrico puro em solução 95% em
massa:

1045 kg de H2SO4 --------------- 95%


msolução ----------------------------- 100%
msolução = {(1045 x 100%) / 95%} = 1100 kg
Cálculo da massa total de água = 45 kg + (1100 kg – 1045 kg) = 100 kg

Questão 200 – (OLIMPÍADA CEARENSE DO ENSINO SUPERIOR DE QUÍMICA) Ferro é um elemento muito
importante no corpo humano porque está presente nas células vermelhas do sangue e atua como transportador do
oxigênio para vários órgãos. Existem em torno de 2,6 x 1013 células vermelhas no sangue de um adulto que contém
um total de 2,9 g de ferro. Quantos átomos de ferro existem em cada célula vermelha do sangue?
a) 3,1 x 1010
b) 6,5 x 108
c) 1,2 x 109
d) 8,1 x 1012
e) 1,5 x 109

Resolução: Alternativa C.
mFe 2,9
Cálculo do número de mol de Ferro: nFe = = = 0,052 mol
<MM>Fe 56

Cálculo do número de átomos (X) de Ferro, correspondente a 0,052 mol: XFe = 0,052 x (6,02 x 1023) átomos de ferro
= 3,13 x 1022 átomos de Fe

Cálculo do número de átomos (X) de Ferro para uma célula:

3,13 x 1022 átomos de Ferro -------------------------- 2,6 x 1013 células


XFerro ----------------------------------------------------- 1 célula
XFerro = 1,20 x 109 átomos de ferro

Questão 201 – (U.S. NATIONAL CHEMISTRY OLYMPIAD) Lithium reacts with water to produce hydrogen gas and
lithium hydroxide. What volume of hydrogen collected over water at 22°C and 750 mm Hg pressure is produced by the
reaction of 0,208 g of Li? [VPH2O = 19.8 mm Hg]

a) 367 mL
b) 378 mL
c) 735 mL
d) 755 mL

Resolução: Alternativa B.

Equação química: Li(s) + H2O(l) → LiOH(aq) + ½ H2(g)

Sabendo que a pressão total fornecida pelo problema é de 750 mmHg, a pressão parcial de H2 será igual a: PTOTAL =
PVAPOR(H2O) + PH2

PH2 = 750 mmHg – 19,8 mmHg = 730,2 mmHg (0,961 atm)

Cálculo da massa de gás hidrogênio, a partir da estequiometria: Li(s) + H2O(l) → LiOH(aq) + ½ H2(g)

1 mol de Li --------------- ½ mol de H2


6,94 g Li ----------------- 1,0 g

98
1600 FÍSICO-QUÍMICA APLICADA EXERCÍCIOS COMENTADOS - IME – ITA – OLIMPÍADA

0,208 g Li --------------- mH2


mH2 = 2,99 x 10-2 g
2,99 x 10−2
( )x 0,08206 x (22+273) 2,99 x 10−2 x 0,08206 x 295
2,0
Cálculo do volume de gás hidrogênio: VH2 = 0,961
= 2,0 x 0,961
=
0,376 L (376 mL)

Questão 202 – (OLIMPÍADA DE QUÍMICA DO RIO DE JANEIRO) Uma amostra de 1,0 g de zinco metálico impura
foi dissolvida em excesso de ácido clorídrico, gerando um volume de 0,206 L de gás H 2, que foi contido em uma
bexiga. Assumindo que o experimento foi realizado nas CNTP, pode-se concluir que a pureza do zinco metálico, em
%, é de, aproximadamente:
a) 60
b) 70
c) 80
d) 90
e) 100

Resolução: Alternativa A. Equação química: Zn(s) + 2HCl(aq) → ZnCl2(aq) + H2(g)

1 mol de Zn ------------ 1 mol de H2


65,41 g ------------------ 22,4 L
p x 1,0 g ---------------- 0,206 L
p = 0,601 (60%)

Pureza do metal igual a 60%.

Questão 203 – (OLIMPÍADA DE QUÍMICA DO RIO DE JANEIRO) A aspirina é um medicamente muito utilizado pela
medicina, uma vez que é utilizado como antipirético e analgésico, e, comercialmente é conhecido como ácido
acetilsalicílico ou AAS. Sua fórmula molecular está representada pela seguinte fórmula molecular, C9H8O4. A partir
desta informação, assinale a alternativa aproximada que representa o número de moléculas deste ácido em 0,80
gramas.
a) 2,7 x 1020
b) 2,7 x 1021
c) 2,0 x 1020
d) 3,5 x 1019
e) 2,0 x 1021

Resolução: Alternativa B
Massa molar do analgésico: <MM> = (9 x 12) + (8 x 1) + (4 x 16) = 180 g.mol-1

Cálculo do número de moléculas em 0,80 g:


180 g ---------- 6,02 x 1023 moléculas
0,80 g --------- X
X = 2,67 x 1021 moléculas

Questão 204 – (OLIMPÍADA DE QUÍMICA DO RIO DE JANEIRO) Uma pequena amostra contendo 4,50 g de
carbonato de magnésio foi tratada com excesso de ácido clorídrico, obtendo-se 500 mL de dióxido de carbono medida
nas CNTP. A partir desta informação, determine o valor aproximado do rendimento da reação citada.
a) 15%
b) 20%
c) 35%
d) 42%
e) 65%

Resolução: Alternativa D

99
1600 FÍSICO-QUÍMICA APLICADA EXERCÍCIOS COMENTADOS - IME – ITA – OLIMPÍADA

A partir da relação estequiométrica, temos: MgCO3(aq) + 2 HCl(aq) → MgCl2(aq) + H2O(l) + CO2(g)


1 mol de MgCO3(aq) ------------------- 1 mol de CO2(g) x R
nsal ---------------------------------------- ngás
nsal x R = ngás

m p.VCO2
Considerando que o dióxido de carbono apresenta comportamento ideal, temos: R x (<MM>
sal
)= R.T
sal
4,50 1 x 0,500
Rx ( )=
84 0,08206 x 273

R = 0,4166 (≅ 42%)

Questão 205 – (U. S. NATIONAL CHEMISTRY OLYMPIAD) 100 litros de dióxido de carbono medido a 740 mmHg e
50°C é produzido pela combustão completa de uma amostra de pentano: 2 C5H12 + 16 O2 → 10 CO2 + 12 H2O. Qual
a massa de pentano reagido?
a) 342 g
b) 265 g
c) 64,4 g
d) 53,0 g

Resolução: Alternativa D.
mCO2
Cálculo da massa de dióxido de carbono, considerando comportamento ideal: p x VCO2 = .R xT
<MM>CO2
740 mCO2
( ) x 100 = x 0,08206 x (50 + 273)
760 44

mCO2 = 161,63 g

Cálculo da massa de pentano: 2 C5H12 + 16 O2 → 10 CO2 + 12 H2O


2 mol de C5H12 ----------------- 10 mol de CO2
142 g ----------------------------- 440 g
mC5H12 ---------------------------- 161,63 g
mC5H12 = 52,90 g

Questão 206 – (ITA) Qual é a fórmula molecular de uma substância gasosa que contém 46,1% de carbono e 53,9%
de nitrogênio em massa? Sabe-se que 2,60 gramas dessa substância ocupam um volume de 1,12 litros nas condições
normais de pressão e temperatura.

Resolução: Base de cálculo: 100 g da substância pura.

46,1 46,1 g 3,84 mol


Analisando em função do átomo de carbono: 100 x 100 g = g = 3,84 mol = 1
12
mol

53,9 53,9 g 3,85 mol


Analisando em função do átomo de nitrogênio: 100 x 100 g = g = 3,84 mol = 1
14
mol
Fórmula mínima: (CN)x

Determinação do valor do x:
28x − − − − − − − − 22,4 L
2,60 − − − − − − − − 1,12 L
2,60 x 22,4 58,24
x= = = 1,86
28 x 1,12 31,36

Valor mais próximo para x é igual a 2. Logo a fórmula molecular será C2N2 ou (CN)2.

100
1600 FÍSICO-QUÍMICA APLICADA EXERCÍCIOS COMENTADOS - IME – ITA – OLIMPÍADA

Questão 207 – Quantas moléculas existem em:


a) 4,0 g de hélio gasoso?
b) 159,8 g de bromo líquido?
c) 253,8 g de iodo sólido?
Em quais dos casos acima o volume da substância é 22,4 litros nas condições normais de temperatura e pressão?
Por quê?
Resolução:
a) Analisando o gás nobre Hélio: 4,0 g − − − − − 6,02 x 1023 moléculas
b) Analisando o bromo líquido: 2 x 79,90 g − − − 6,02 x 1023 moléculas
c) Analisando o iodo sólido: 2 x 126,9 g − − − − 6,02 x 1023 moléculas
Trata-se do gás nobre hélio, o que pode ser observado pela equação dos gases ideais a seguir: V =
1,0 x 0,08206 x 273
1,0
= 22,4 litros

Questão 208 – (ITA) Dentre as seguintes substâncias: CO2, NH3, HCl, NO2, H2O, CO e N2, a lei das proporções
múltiplas pode ser exemplificada usando o seguinte par:
a) H2O e HCl
b) NH3 e NO2
c) NO2 e CO2
d) CO e CO2
e) Nenhuma das respostas anteriores

Resolução: Alternativa D.
Lei de Dalton – Lei das proporções múltiplas: “Quando dois elementos se combinam para formar mais de um
composto, se a massa de um deles permanecer constante, a do outro variará numa relação de números inteiros
pequenos. Exemplo: CO (monóxido de carbono, 28 g x mol-1) e CO2 (dióxido de carbono, 44 g x mol-1)
CO = 12 g/mol de carbono e 16 g/mol de oxigênio
CO2 = 12 g/mol de carbono e 32 g/mol de oxigênio
32
O= = 2,0
16
Questão 209 – (ITA) Que massa de dissulfeto de carbono é obtida quando 1,0 x 1024 átomos de carbono reagem com
a quantidade estequiométrica de enxofre?
a) (10,0/6,023) x 76 g
b) (6,023/10,00) x 76 g
c) 1,00 x 1024 x 79 g
d) 6,023 x 1023 x 32 g
e) Nenhuma das respostas anteriores

Resolução: Alternativa A. Equação química: C(s) + S2(g) → CS2(g)

1 mol de C ----------------------------- 1 mol de CS2


6,02 x 1023 átomos de carbono ------ 76 g
1,00 x 1022 átomos de carbono ------ Massa
76 𝑥 10
Massa = 6,02 (g)

101
1600 FÍSICO-QUÍMICA APLICADA EXERCÍCIOS COMENTADOS - IME – ITA – OLIMPÍADA

Questão 210 – (ITA) O equivalente – grama do ferro (massa atômica = 55,85) na reação 2 FeCl2 + Cl2 → FeCl3, é:
a) 55,85 / 2
b) 55,85 x 2
c) 55,85
d) 55,85 / 3
e) Nenhuma das respostas anteriores

Resolução: Alternativa C.
O número de oxidação do ferro varia de +2 → +3
A variação do número de oxidação é ∆ = 3 – 2 = 1
<MM>Fe 55,85
Cálculo do equivalente-grama: Eqg Fe = ∆
= 1,0
= 55,85

Questão 211 – (ITA) Definir e classificar os óxidos dando um exemplo de cada caso.
Resposta: Óxidos são compostos binários que apresentam o oxigênio como o elemento químico mais eletronegativo.
a) óxido ácido: são compostos que reagindo com água formam ácidos. Exemplo: SO2(g) + H2 O(l) → H2 SO4(aq)

b) óxido básico: são compostos que reagindo com água formam hidróxidos. Exemplo: CaO(2) + H2 O(l) →
Ca(OH)2(aq)

c) óxido neutro: são compostos que não reagem com ácidos ou bases. Trata-se de um conjunto limitado de compostos.
CO (monóxido de carbono), NO (monóxido de nitrogênio e N2O (monóxido de dinitrogênio). Exemplo: SO2(g) +
H2 O(l) → H2 SO4(aq)

d) óxido anfótero: são compostos que apresentam propriedades ácidas e básicas. Exemplo: ZnO.
e) óxido misto: são compostos que apresentam a seguinte forma: X3O4. Exemplo: Fe3 O4 .
e) Peróxidos: são compostos em que o elemento oxigênio apresenta número de oxidação igual a -1. Exemplo:
H2 O2 (peróxido de hidrogênio ou conhecido comercialmente como água oxigenada).
f) Superóxidos: são compostos em que o elemento oxigênio apresenta número de oxidação igual a -1/2. Exemplo:
Na2 O4 (superóxido de sódio).
Questão 212 – (ITA) Determinar a fórmula mínima da substância pura que apresenta a seguinte composição
centesimal: 94,1% de oxigênio – 5,9% de hidrogênio, explicando detalhadamente o raciocínio adotado.

Resolução: Base de cálculo: 100 g da substância pura.

5,9 5,9 g 5,9 mol


Analisando em função do átomo de hidrogênio: 100 x 100 g = g = 5,88 mol ≅ 1
1
mol

94,1 94,1 g 5,88 mol


Analisando em função do átomo de oxigênio: 100 x 100 g = g = 5,88 mol = 1
16
mol

Como a proporção é 1:1, a fórmula molecular é igual a HO. Como não existe uma substância pura com esta fórmula
molecular, a mais próxima é H2O2 (peróxido de hidrogênio).

Questão 213 – (ITA) Determinar pelo método de óxido-redução, com explicação detalhada, os coeficientes da
seguinte equação química: KMnO4 + HCl → KCl + MnCl2 + H2 O + Cl2

102
1600 FÍSICO-QUÍMICA APLICADA EXERCÍCIOS COMENTADOS - IME – ITA – OLIMPÍADA

Resolução: Realizando o balanceamento pelo método REDOX: 2 KMnO4 + 16 HCl → 2 KCl + 2 MnCl2 +
8 H2 O + 5 Cl2

Agente redutor: HCl (ácido clorídrico)

Agente Oxidante: KMnO4 (permanganato de potássio)

Questão 214 – (ITA) As substâncias NH3, CaO e Na2CO3:


a) reagem com água dando soluções de hidróxidos
b) possuem estrutura cristalina
c) dão origem a soluções aquosas alcalinas
d) são óxidos básicos
e) Nenhuma das respostas anteriores

Resolução: Alternativa C. Analisando as reações químicas representadas por suas equações químicas:
I) Amônia: NH3(g) + H2 O(l) → NH4 OH(aq) (hidróxido de amônio)
II) Óxido de cálcio: CaO(s) + H2 O(l) → Ca(OH)2(aq) (hidróxido de cálcio)
H2 O(l)
III) ⏞ 2 Na+
Carbonato de sódio: Na2 CO3(g) → −2
(aq) + CO3(aq)
2 Na+ −2 +
(aq) + CO3(aq) + H2 O(l) → Na (aq) + OH −
⏟ (aq) + H2 CO3(aq)
𝑠𝑜𝑙𝑢çã𝑜 𝑏á𝑠𝑖𝑐𝑎
Todas as três soluções são alcalinas.

103
1600 FÍSICO-QUÍMICA APLICADA EXERCÍCIOS COMENTADOS - IME – ITA – OLIMPÍADA

Resolução: As equações
químicas balanceadas estão
apresentadas
abaixo:CAPÍTULO II

ESTUDO DOS SISTEMAS


GASOSOS – GASES IDEAIS E
REAIS

PROFESSOR ALEXANDRE
VARGAS GRILLO
104
1600 FÍSICO-QUÍMICA APLICADA EXERCÍCIOS COMENTADOS - IME – ITA – OLIMPÍADA

105
1600 FÍSICO-QUÍMICA APLICADA EXERCÍCIOS COMENTADOS - IME – ITA – OLIMPÍADA

Questão 215 – Explique a Lei de Boyle e Mariotte.

Resposta: A Lei de Boyle - Mariotte descreve o comportamento de um gás ideal apenas quando sua temperatura é
mantida constante (processo isotérmico). Quanto mais a pressão do sistema aumenta, o volume diminui. Logo as
grandezas volume e pressão são inversamente proporcionais. Então a lei de Boyle diz matematicamente que: pV = k,
onde k é uma constante que depende da massa, temperatura e da natureza desse gás.

Questão 216 – Explique a Lei de Charles e Gay Lussac.

Resposta: A Lei de Charles - Gay Lussac descreve o comportamento de um gás ideal apenas quando a pressão é
mantida constante (process isobárico). Para uma dada massa de gás à pressão constante, o volume ocupado pelo
gás é diretamente proporcional à temperatura absoluta. Um aumento da temperatura absoluta acarreta no aumento
𝑉
do volume ocupado pelo gás, de maneira que o quociente seja constante = 𝑐𝑜𝑛𝑠𝑡𝑎𝑛𝑡𝑒. A relação entre volume
𝑇

e temperatura foi inicialmente observada em 1787 por Jacques Charles e quantificada em 1802 por Joseph Gay-
Lussac, sendo conhecida como lei de Charles - Gay-Lussac.

Questão 217 – (ITA) Explique que tipos de conhecimentos teóricos ou experimentais, já obtidos por outros
pesquisadores, levaram A. Avogadro a propor a hipótese que leva o seu nome.

Resposta: Segundo Avogadro: “volumes iguais de quaisquer gases, à mesma temperatura e pressão, contêm o
mesmo número de moléculas”.
Avogadro sugeriu essa hipótese para tentar resolver um problema criado entre Dalton e Gay-Lussac: em 1808, Gay-
Lussac tornou público um trabalho que tinha por base o estudo das reações envolvendo gases, ou seja, a lei
volumétrica (combinações de volumes). Segundo ele, dois volumes de Hidrogênio estavam para um volume de
Oxigênio na formação de dois volumes de água todos em estado gasoso. No entanto, Dalton ao fazer o estudo desse
trabalho viu que a relação existente entre volumes era a mesma existente entre o número de partículas: havia duas
partículas hidrogênio reagindo com uma partícula de oxigênio e formando duas partículas de água. Dessa maneira,
esse raciocínio contrariava a ideia de Dalton (átomo indivisível), pois o número de átomos estava sendo igualado ao
número de partículas, isto é, um átomo de oxigênio não poderia ser dividido para formar duas partículas água. Assim,
Avogadro propôs que J. Dalton tinha confundido átomos com moléculas e que as moléculas de oxigênio deveriam ser
diatômicas. Dessa maneira uma molécula de oxigênio poderia formar duas moléculas de água, demonstrando o seu
trabalho a partir do modelo atômico de Dalton e das leis das combinações.

Questão 218 – Defina gases ideais.

106
1600 FÍSICO-QUÍMICA APLICADA EXERCÍCIOS COMENTADOS - IME – ITA – OLIMPÍADA

Resposta: Modelo idealizado, teórico, constituído por moléculas pontuais, que se movem de forma aleatória.
Obedece a equação dos gases ideais, definida da seguinte maneira: 𝑝𝑉 = 𝑛𝑅𝑇. Para este modelo, os gases se
comportam a baixas pressões ou a altas temperaturas.

Questão 219 – Defina constante dos gases. Mostre que a constante universal dos gases (R) vale 0,08206 atm.L.mol –
1.K –1. Apresente em uma tabela outros valores para a constante dos gases.

Resposta: A partir da equação dos gases ideais, temos: ptotal x Vtotal = ntotal x R x T. Isolando a constante dos
ptotal x Vtotal
gases: R = .
ntotal x T

Considerando que o processo ocorre nas condições normais de Temperatura e pressão (CNTP), ou seja, 1 atmosfera
e zero graus CelsiusDigite a equação aqui., o volume será igual a 22,4 L.mol-1.
ptotal x Vtotal 1 atm x 22,4 L atm x L
R= = = 0,08206
ntotal x T 1 mol x 273 K mol x K
Tabela dos principais valores para a constante dos gases ideais.
8,314 Pa.m³.mol-1.K-1
8,314 J.mol-1.K-1
1,987 cal.mol-1.K-1
0,08206 atm.L.mol-1.K-1
62,36 mmHg.L. mol-1.K-1

Questão 220 – (ITA) Partindo da lei dos gases perfeitos deduza uma expressão que fornece o valor da densidade de
um gás (d) em função da pressão (p), da temperatura (T) e de sua massa molar (<MM>).

Resposta: Recorrendo a equação de Clapeyron, temos: ptotal x Vtotal = ntotal x R x T


massa
ptotal x V = xRxT
< MM >
massa
ptotal x < MM > = xRxT
⏟V
densidade

p x<MM>
Logo a relação entre a densidade com a pressão é dada pela seguinte equação matemática: d = RxT

Questão 221 – Explique a Lei de Dalton das pressões parciais.

Resposta: Considere um recipiente de volume V constante que apresenta três gases distintos (A, B e C) à mesma
temperatura. Partindo da equação dos gases ideais, temos: ptotal x Vtotal = ntotal x R x T

107
1600 FÍSICO-QUÍMICA APLICADA EXERCÍCIOS COMENTADOS - IME – ITA – OLIMPÍADA

ptotal x (VA + VB + VC ) = (nA + nB + nC ) x R x T


nA x R x T nB x R x T nC x R x T
ptotal = + +
⏟ VA ⏟ VB ⏟ VC
PA PB PC

ptotal = PA + PB + PC

Enunciado sobre a Lei de Dalton das pressões parciais: “Quando pelo menos dois ou mais gases que não reagem
quimicamente são colocados num mesmo reservatório de volume V, a pressão exercida por cada gás na mistura é a
mesma que ele teria se estivesse sozinho no reservatório.” A pressão exercida por cada gás numa mistura é chamada
de pressão Parcial. “A pressão total exercida por uma mistura é a soma das pressões parciais”.

Questão 222 – Explique a Lei de Amagat dos volumes parciais.

Resposta: Considere um recipiente de volume V contendo três gases distintos (A, B e C), com temperatura constante
T.
pA x VA
Cálculo do número de mol para o gás A: nA = RxT
pB x VB
Cálculo do número de mol para o gás B: nB = RxT
pC x VC
Cálculo do número de mol para o gás C: nC =
RxT
p x Vtotal pA x VA pB x VB p xV
Na mistura gasosa, temos a seguinte relação: ntotal = nA + nB + nC = RxT
= RxT
+RxT
+ RC x TC
p
Considerando que o processo ocorra na mesma pressão e na mesma temperatura, temos: R x T x Vtotal =
pA p p
RxT
x VA + R xBT x VB + R xCT x VC
p 𝑝 p p
x Vtotal = x VA + x VB + xV
RxT RxT RxT RxT C
Vtotal = VA + VB + VC
Lei de Amagat dos volumes parciais: “Quando dois ou mais gases que não reagem quimicamente entre si são
colocados num mesmo reservatório, o volume exercido por cada gás na mistura é a mesma que ele teria se estivesse
sozinho no reservatório. O volume exercido por cada gás numa mistura é chamado de volume parcial.” O volume total
exercido por uma mistura é a soma dos volumes parciais”.

Questão 223 – Como fica a equação dos gases ideais que relaciona a densidade com o grau de dissociação?

Resposta: Considerando uma reação gasosa de decomposição hipotética do tipo: A(g) → B(g) + C(g). Base de cálculo:
n (mol) inicial do reagente gasoso A. Utilizando a tabela de equilíbrio químico, temos:
A(g) → B(g) C(g)
Início n 0 0

108
1600 FÍSICO-QUÍMICA APLICADA EXERCÍCIOS COMENTADOS - IME – ITA – OLIMPÍADA

Reage nα nα nα
Equilíbrio n - nα nα nα

Cálculo do número de mol total da mistura gasosa: ntotal = nA + nB + nC = n - nα + nα + nα = n.(1 + α)


Relação entre a densidade da mistura gasosa e o grau de dissociação: ptotal x V = 𝑛(1 + 𝛼) x R x T
𝑚
ptotal x < MM > = 𝑥 (1 + 𝛼) x R x T

𝑉
𝑑𝑒𝑛𝑠𝑖𝑑𝑎𝑑𝑒
p x𝑉
total
d = (1+𝛼) , onde: <MM> = massa molar do composto gasoso; α = grau de dissociação; R = constante dos
xRxT

gases e T = temperatura absoluta.

Questão 224 – Defina fator de compressibilidade. Qual é o significado físico para Z > 1 e Z < 1 e Z = 1.

Resposta: O fator de compressibilidade de um determinado gás, designado pela letra Z, é a razão entre o volume
molar do gás no comportamento real pelo volume molar do gás no comportamento ideal. O fator de compressibilidade
𝑅𝑒𝑎𝑙
𝑉𝑚𝑜𝑙𝑎𝑟
leva em consideração dois fatores: estrutura molecular e forças intermoleculares. Equação matemática: 𝑍 = 𝐼𝑑𝑒𝑎𝑙 ,
𝑉𝑚𝑜𝑙𝑎𝑟

na mesma pressão e temperatura.


Z > 1 → esta relação indica que o volume molar de um gás real apresenta volume maior do que o volume molar para
um gás ideal, portanto, as interações intermoleculares repulsivas são dominantes.
Z < 1 → esta relação indica que o volume molar de um gás ideal apresenta volume maior do que o volume molar para
um gás real, portanto, as interações intermoleculares atrativas são dominantes.
Z = 1 → indica exatamente que o gás apresenta comportamento idealizado, uma vez que, a partir da equação do gás
real: pV = ZnRT. Para Z = 1, a equação se reduz para pV = nRT. Para pressões baixas, lim 𝑍 = 1.
𝑝→0

Questão 225 – Explique o comportamento para os gases de Van der Waals.

Resposta: A equação de Van der Waals procurou adaptar a equação dos gases ideais para descrever o
comportamento de um gás real. Ele adicionou duas constantes (a e b), uma devido ao fato das partículas ocuparem
um volume finito (b) e outro devido às interações entre as partículas constituintes do gás (a). O modelo do gás ideal
assume que não existem interações entre as partículas de um gás. No entanto, as partículas de um gás real interagem
entre si através das forças de Van der Waals. Outra falha no modelo do gás ideal é a desconsideração total do
chamado “volume próprio das partículas”.
𝑎
Equação de Van der Waals: (𝑝𝑟𝑒𝑎𝑙 + 𝑉 2 ) 𝑥 (𝑉𝑚𝑜𝑙𝑎𝑟 − 𝑏) = 𝑅 𝑥 𝑇
𝑚𝑜𝑙𝑎𝑟

109
1600 FÍSICO-QUÍMICA APLICADA EXERCÍCIOS COMENTADOS - IME – ITA – OLIMPÍADA

Questão 226 – Enuncie a Teoria Cinética dos Gases.

Resposta: De acordo com a Teoria Cinética dos Gases, as moléculas e átomos de algumas substâncias estão em
constante estado de movimento de forma caótica e inteiramente livres de força de atração e repulsão, a partir de uma
temperatura acima do zero absoluto. Trata-se de uma teoria que o modelo gasoso é tratado de forma bem simples,
ou seja, o estudo e conhecimento de um gás ideal.

Questão 227 – Defina e deduza a velocidade média quadrática.

1
Resposta: A partir da equação fundamental da teoria cinética dos gases, temos: 𝒑𝑽 = 3 𝑥 𝑛 𝑥 < 𝑀𝑀 > 𝑥 𝑐 2 .

Esta equação representa a pressão que um determinado gás realiza com a parede de um determinado recipiente. A
pressão torna-se constante, com um número bastante grande de colisões na ordem de bilhões de colisões. Levando
em considerando que os gases apresentam comportamento ideal, temos: 𝐩𝐕 = nRT
Igualando as equações através do produto pV que se encontra em negrito em ambas as equações, temos:
1
𝐩𝐕 = x n x < MM > x c 2 e pV = nRT
3
1
x n x < MM > x c 2 = n x R x T
3
3xRxT
c2 =
< MM >
Em um gás ideal, a teoria cinética dos gases afirma que a uma dada temperatura temos a mesma energia cinética
para cada molécula. Desta forma, moléculas que apresentam maior massa molar, como por exemplo, moléculas
diatômicas de oxigênio (32,0 g.mol-1) apresentam menor velocidade que as de nitrogênio (28,0 g.mol-1). A partir da
velocidade média quadrática das moléculas é diretamente proporcional a temperatura e inversamente proporcional a
raiz quadrada da massa molar. Quanto maior a temperatura, maior a velocidade média quadrática das moléculas.
Questão 228 – Enuncie a Lei de Difusão de Graham.

Resposta: Lei de Graham → O cientista escocês Thomas Graham descobriu que, quanto menos denso um gás,
maior sua velocidade de difusão. A constatação de Graham ou Lei de Graham assim se enuncia: “Através de dois
gases distintos, nas mesmas condições de temperatura e pressão, a razão das velocidades de difusão é igual a raiz
quadrada do inverso das massas molares”. A partir da equação da velocidade média quadrática, temos:
3xRxT
Analisando o Gás I: cI2 = <MM>I
3xRxT
Analisando o Gás II: cII2 = <MM>
II
3xRxT
c2I <MM>I
Dividindo a equação da velocidade do gás I com o do gás II, temos: c2II
= 3xRxT
<MM>II

110
1600 FÍSICO-QUÍMICA APLICADA EXERCÍCIOS COMENTADOS - IME – ITA – OLIMPÍADA

3xRxT
cI2 < MM >I
=
cII2 3xRxT
< MM >II

cI < MM >II
=√
cII < MM >I

Questão 229 – Defina e apresente a equação da temperatura de Boyle.

Resposta: A temperatura de Boyle é uma temperatura característica para cada gás, com comportamento ideal, numa
faixa grande de pressão, que vai de pressões bem baixas até pressões acima de 200 atmosferas. A equação da
a
temperatura de Boyle (TB): T Boyle = b x R, em que a é a constante de van der Waals referente as interações

intermoleculares, b é o volume da partícula gasosa (co-volume) e R é a constante dos gases ideais.

Questão 230 – Duas salas de mesmo tamanho se comunicam por uma porta aberta. Entretanto, a média de
temperatura nas duas salas é mantida a valores distintos. Em qual sala há mais ar?

Resposta: Informações do problema:


Informações para a sala I: TI, PI, VI, nI
Informações para a sala II: TII, PII, VII, nII
Considerações a serem feitas para a resolução do problema:
✓ Levando em conta que a temperatura na sala I é maior que a sala II, logo: TI > TII.
✓ O problema menciona que as salas apresentam o mesmo volume, ou seja, VI = VII (processo isocórico ou
isovolumétrico).
✓ As salas apresentam o mesmo nível, logo: pI = pII (processo isobárico).

𝐶𝑜𝑛𝑠𝑡𝑎𝑛𝑡𝑒

pI VI
Analisando a sala I: = nI x TI
𝑅

𝐶𝑜𝑛𝑠𝑡𝑎𝑛𝑡𝑒
p⏞
II VII
Analisando a sala II: 𝑅
= nII x TII

Igualando as equações para as situações I e II: nI x TI = nII x TII


nI TII
=
nII TI

Como a temperatura na sala I é maior que na sala II (TI > TII ), isso vai implicar que o número de mol na sala I é
menor que na sala II (nI < nII ), logo a sala que apresenta menor temperatura apresentará maior ar.

111
1600 FÍSICO-QUÍMICA APLICADA EXERCÍCIOS COMENTADOS - IME – ITA – OLIMPÍADA

Questão 231 – Defina ponto crítico e apresente as equações para a determinação do volume crítico, temperatura
crítica e pressão crítica.

Resposta: Ponto crítico = Ponto que a uma certa temperatura, a densidade de ambas as fases passa a ser a mesma,
havendo uma anulação por completo da diferença entre líquido e vapor. Como consequência deste ponto, a
temperatura, a pressão e o volume molar correspondem a Temperatura crítica (Tc), Pressão crítica (Pc) e Volume
crítico (Vc). Estes valores são característicos para cada substância.
Expressão matemática para o volume crítico: Vc = 3 x b
8xa
Expressão matemática para a temperatura crítica: Tc = 27 x b x R
a
Expressão matemática para a pressão crítica: Tc = 27 x b2

Questão 232 – (ITA) Numa experiência de eletrólise da água formam-se 3,00 gramas de H2(g). Calcule o volume
ocupado por esta massa de hidrogênio suposta isenta de umidade, na temperatura de 300 K e sob a pressão de 684
mmHg.

Resolução: A eletrólise da água há a produção de gás hidrogênio, conforme pode ser observado pela seguinte
+
semirreação de redução: 2H(aq) 2e− → H2(g). Aplicando a equação dos gases ideias, e sabendo que uma
3,0
nH2 x R x T ( )x 0,08206 x 300
atmosfera corresponde a 760 mm Hg, tem-se: VH2 = = 2,0
684 = 41,03 L
p ( )
760

Questão 233 – (OLIMPÍADA MINEIRA DE QUÍMICA - MODIFICADA) O processo de produção de amônia (NH3), a
partir dos gases hidrogênio (H2) e nitrogênio (N2), foi de difícil desenvolvimento e demandou vários anos de estudo.
Este método recebeu o nome de processo Haber-Bosch, em homenagem aos seus idealizadores. A equação química
abaixo descreve a síntese da amônia: N2(g) + 3 H2(g) → 2 NH3(g). Supondo que os gases nitrogênio e hidrogênio sejam
colocados em um reator fechado, onde não ocorra nenhuma troca de matéria com a vizinhança e na presença de um
catalisador apropriado. Calcule a pressão que 17 gramas do gás amônia exercem em um reator de volume igual a 5
litros e operando a 427°C.
nRT
Resolução: Cálculo da pressão de amônia, a partir da utilização da equação dos gases ideais: p = =
V
17
( ) x 0,08206 x (427+273) 1 x 0,08206 x 700
17
5,0
= 5,0
= 11,49 atm

112
1600 FÍSICO-QUÍMICA APLICADA EXERCÍCIOS COMENTADOS - IME – ITA – OLIMPÍADA

Questão 234 – (IME) Tomou-se uma amostra de 130 gramas de zinco metálico para reagir com uma solução aquosa
diluída de ácido clorídrico em quantidade estequiométrica. Dessa reação, observou-se a formação de gás, que foi
aquecido a 227°C e transportado para um balão fechado de cinquenta litros. Esse balão continha, inicialmente, iodo
em fase gasosa a 227°C e 3,28 atm. Após o equilíbrio, verificou-se que a constante de equilíbrio Kc a 227°C é igual a
160. Considerando que a temperatura permaneceu constante durante o processo, determine a pressão final total no
balão.
Resolução: Equação química: Zn(s) + HCl(aq) → ZnCl2(aq) + H2(g)
mZn 130
Cálculo do número de mol de Zinco: nZn = = = 2,0 mol
<MM>Zn 65

Cálculo do número de mol de gás hidrogênio produzido, a partir da reação de deslocamento: Zn(s) + HCl(aq) → ZnCl2(aq)
+ H2(g)
1 mol de Zn --------------- 1 mol de H2
2,0 mol de Zn ------------- nH2
nH2 = 2,0 mol

Cálculo do número de mol de iodo (I2) no seu estado inicial, a partir da utilização dos gases ideais: nI2 =
3,28 x 50 164
0,08206 x (227+273)
= 41,03 ≅ 4,00 mol

Equação química para a formação do iodeto de hidrogênio: H2(g) + I2(g) ⇄ 2 HI(g)


Tabela de equilíbrio químico com base de cálculo: 2 mol de H2 e 4 mol de I2
H2(g) (mol) I2(g) (mol) ⇄ 2 HI(g) (mol)
Início 2 4 0
Reage α α 2α
Equilíbrio 2 - α 4-α 2α

Cálculo do número de mol total (nT) dos gases: nT = nH2 + nI2 + nHI = 2 – α + 4 – α + 2α = 6 mol
246,18
Cálculo da pressão final (pT) do balão, a partir da utilização da equação dos gases ideais: pT = 50
= 4,92 atm

Questão 235 – A decomposição térmica do pentacloreto de fósforo gasoso ocorre segundo a seguinte equação
química balanceada: PCl5(g) ⇄ PCl3(g) + Cl2(g). Determine a expressão do grau de dissociação (α) em função da
densidade da mistura gasosa (d).

Resolução: Para a resolução deste exercício será necessário a utilização de uma tabela de equilíbrio químico. Base
de cálculo: n(mol) inicial de pentacloreto de fósforo.
Tabela de equilíbrio químico:
PCl5(g) PCl3(g) Cl2(g)
Início n 0 0
Reage nα nα nα
Equilíbrio n - nα nα nα

113
1600 FÍSICO-QUÍMICA APLICADA EXERCÍCIOS COMENTADOS - IME – ITA – OLIMPÍADA

Cálculo do número de mol total (nT) da mistura gasosa, no equilíbrio: nTotal = nPCl5 + nPCl3 + nCl2 == n - nα + nα + nα = n
+ nα = n(1 + α)

m
Relação da densidade (d) com o grau de dissociação (α): ptotal x V = x (1 + α) x R x T
<MM>
m
ptotal x < MM > = x (1 + α) x R x T

V
𝑑

ptotal x < MM >


𝑑=
(1 + α) x R x T

Questão 236 – A massa específica do gás que resulta no aquecimento do pentacloreto de fósforo é igual a 3,43 g.L-
1e ocorre a uma temperatura igual a 227°C e pressão de uma atmosfera. Determine:
a) A massa molecular da mistura gasosa.
b) O grau de dissociação (α).
Resolução:
dxRxT
a) Determinação da massa molar, a partir da equação dos gases ideais: < MM > = p

3,43 x 0,08206 x (227 + 273) 140,73 g


< MM >= = = 140,73
1 100 mol
ptotal x <MM>
b) Cálculo do grau de dissociação (α), a partir da euação da densidade: d = (1+α) x R x T

1,0 x (31 + 5 𝑥 35,5)


3,43 =
(1 + α) x 0,08206 x (227 + 273)

α = 0,4815 (48,15%)

Questão 237 – What is the total kinectic energy of translation of 2 moles of a perfect gas at 27°C in calories?
Resolução: Sabendo que a energia cinética de translação de uma partícula gasosa é apresentada pela seguinte
3 3 cal
expressão matemática: Ecinética = x n x R x T = x 2 mol x 1,987
2 2
x (27 + 273)K = 1788,3 cal
mol x K

Questão 238 –
a) Determine a densidade do gás hidrogênio em g.L-1, a 25°C e pressão de 1,50 atm, considerando que este se
comporta como um gás ideal.
b) Uma amostra de um gás ideal desconhecido, com massa igual a 190 g desloca 35,50 dm³ de ar atmosférico,
medidos a 27°C e 1200 mmHg. A partir desta informação, calcule a massa molecular da referida substância.
c) Certa massa de gás Hélio (He) está contida num recipiente de 8,0 L, a 127°C, exercendo uma pressão de
2,0 atm. Se o volume dessa massa de gás se reduzisse a 75% do valor inicial, determine a temperatura para que sua
pressão se torne também 75% inferior ao valor inicialmente estabelecido no recipiente.

Resolução:
m
a) A relação entre a densidade e a pressão é dada pela seguinte equação: p. V = <MM> . R. T

114
1600 FÍSICO-QUÍMICA APLICADA EXERCÍCIOS COMENTADOS - IME – ITA – OLIMPÍADA

p < MM >
d=
RT
1,50 x (2,0)
Cálculo da densidade do gás hidrogênio: dH2 = (0,08206)x (25+273) = 0,123g. L−1

1200 190
b) Sabendo que 1 atm equivale a 760 mmHg, temos: ( 760 ) x 35,5 = (<MM>) x 0,08206 x (27 + 273)

g
< MM > = 83,39 ⁄mol

O gás mais próximo que apresenta esta massa molar é o gás nobre Kriptônio (Kr).

c) Dado que o gás se comporta como ideal, a razão entre o produto PV e T será constante e numericamente igual ao
produto do número de mols e da constante dos gases (R).

p.V pV p1 .V1 p2 .V2


n. R = T
. Considerando que a relação T
é constante, temos: ( T1
) =( T2
)

Sendo “1” o estado inicial e “2” o estado final, a temperatura do estado final (2), será:

Estado 1: P1 = 2,0 atm; V1 = 8,0 L e T1 = 127 + 273 = 400 K

3 3 3 3
Estado 2: P2 = x P1 = x 2,0 = 1,50 atm; V2 = x V1 = x 8,0 = 6,00 L
4 4 4 4

2,0 x 8,0 1,50 x 6,0


A partir da utilização da equação combinada dos gases ideais: 400
= T2

T2 = 225 K

Questão 239 – Dez gramas de gás metano foram colocadas em um recipiente com capacidade de 1 dm³ a 25°C.
a) Calcule a pressão exercida pelo gás, considerando que o mesmo se comporta como ideal.
b) Calcule a pressão exercida pelo gás, considerando que o mesmo apresente comportamento real de Van der
Waals, com parâmetros a e b respectivamente iguais a 2,25 L².atm.mol-1 e 0,0428 L.mol-1.

Resolução:
10
a) Cálculo da pressão do gás: p x 1 = 16 x 0,08206 x (25 + 273)

p = 15,28 atm

b) Considerando que o gás apresenta um comportamento volumétrico descrito pela equação de van der Waals,
convém calcular inicialmente o volume molar do gás nas condições termodinâmicas de interesse. Este, por sua vez,
pode ser determinado através da razão entre o volume ocupado pelo gás e o número de mols.
m 10
Número de mol: n = <MM> = 16
= 0,625 mol

1
Volume molar: Vmolar = 0,625 = 1,60 L. mol−1

115
1600 FÍSICO-QUÍMICA APLICADA EXERCÍCIOS COMENTADOS - IME – ITA – OLIMPÍADA

Substituindo o volume molar, a temperatura e as constantes a e b na equação de Van der Waals, temos: preal =
RT a 0,08206 x (25+273) 2,25
Vmolar −b
− V2 = 1,6−0,0428
− (1,6)2 = 14,82 atm
molar

Questão 240 - Uma mistura gasosa é constituída por três gases: 450 g de etano, 250 g de argônio e 350 g de dióxido
de carbono. Sabendo que a pressão parcial do gás nobre a 300 K é igual a 8000 Pa, calcule:
a) as frações molares de cada componente;
b) a pressão total da mistura;
c) o volume ocupado pela mistura.

Resolução: Primeiramente, para a resolução das alternativas (a, b e c), será necessário calcular o número de mol de
cada composto gasoso:
mC2H6 450 mAr 250 mCO2 350
nC2 H6 = = = 15 mol; nAr = = = 6,35 mol e nCO2 = = =
<MM>C2H6 30 <MM>Ar 40 <MM>CO2 44

7,95 mol

Número de mol total (nT): nT = nC2H6 + nAr + nCO2 = 15 + 6,25 + 7,95 = 29,2 mol

a) As frações molares de cada componente podem ser determinadas através da razão entre o número de mol
calculado do gás pelo número de mol total. O somatório das frações molares calculadas deve ser obrigatoriamente
igual a um.
nC2H6 15 nAr 6,25 nCO2 7,95
X C2 H6 = nT
= 29,2
= 0,514; X Ar = nT
= 29,2
= 0,214 e X CO2 = nT
= 29,2
= 0,272

b) Por definição, a pressão parcial de um componente qualquer em um gás ideal é o resultado do produto da
sua fração molar característica e a pressão total exercida pelo gás. Sabendo que 1 atm corresponde a 101325 Pa,
temos: PAr = X Ar x Ptotal
1 atm
8000 Pa x (101325 Pa) = 0,21 x Ptotal

Ptotal = 0,376 atm

c) Como a mistura se comporta como um gás ideal, o volume ocupado pode ser calculado a partir do
conhecimento da pressão, temperatura e número total de mols.
0,376 x V = 29,2 x 0,08206 x 300

V = 1911,8 L

Questão 241 - Em um laboratório de síntese de nanopartículas há um reator que é preenchido com 0,25 kg de argônio
a temperatura de 450°C. Sabendo que o volume interno do reator é de 30 L, calcule:
a) a pressão exercida pelo gás enquanto ideal.
b) a pressão exercida pelo gás considerando que o mesmo se comporta de acordo com a equação de Van der
Waals, sendo o parâmetro atrativo igual a 1,4 atm.L2.mol-2 e o repulsivo igual a 0,032 L.mol-1.
c) o fator de compressibilidade do gás nas condições dadas.

116
1600 FÍSICO-QUÍMICA APLICADA EXERCÍCIOS COMENTADOS - IME – ITA – OLIMPÍADA

Resolução:
250
x 0,08206 x (450+273)
a) Substituindo os valores fornecidos pelo problema, tem-se: p = 40
= 12,36 atm
30

m 250
b) Cálculo do número de mol: n = <MM> = 40
= 6,25 mol

V 30
Cálculo do volume molar: Vmolar = n
= 6,25 = 4,8 L. mol−1

Substituindo-se o volume molar na equação de estado de van der Waals, juntamente com o conhecimento da
temperatura e dos valores de a e b característicos para o argônio, a pressão pode ser diretamente computada.

a
{preal + 2 }x {Vmolar − b} = R x T
Vmolar

1,4
{preal + } x {4,8 − 0,032} = 0,08206 x (450 + 273)
(4,8)²

preal = 12,38 atm

c) A razão entre o fator de compressibilidade do gás real e aquele do mesmo gás na ausência de interações
interatômicas (comportamento ideal – Z = 1), em verdade é a razão entre as pressões no equilíbrio nas referidas
condições. Cabe ressaltar, que, no presente problema, os volumes molares do gás na presença e na ausência de
interações são ambos iguais ao volume molar calculado anteriormente – 4,8 L/mol.

Preal x ϑreal
Zreal RxT Preal 12,38
= = = = 1,002 → Zreal = 1,002
Zideal Pideal x ϑideal Pideal 12,36
RxT

Para este valor encontrado para o fator de compressibilidade (Z) do gás real sugere que o argônio deve se comportar
de forma muito similar ao comportamento esperado para o mesmo, caso não houvessem interações entre seus
átomos.

Questão 242 - Um freezer recém-adquirido, foi fechado e ligado quando a temperatura ambiente estava a 27°C.
Considerando que o ar se comporta como um gás ideal e a vedação é “perfeita”, determine a pressão no interior do
freezer quando for atingida a temperatura de –19°C.

Resolução: Para a resolução desta questão, como se trata do mesmo composto gasoso, utilizamos a equação
combinada dos gases ideais.

Situação I: PI = 1 atm; V = constante e TI = 273 + (27) = 300 K

Situação II: PII; V = constante; TII = 273 + (- 19) = 254 K

Cálculo da pressão II (PII), considerando o processo como sendo isovolumétrico (VI = VII), ou também conhecido como
isocórico.

117
1600 FÍSICO-QUÍMICA APLICADA EXERCÍCIOS COMENTADOS - IME – ITA – OLIMPÍADA

pI x VI pII x VII
TI
= TII
(Equação combinada dos gases ideais)

1 pII
=
300 254

pII = 0,85 atm

Questão 243 - A decomposição térmica do pentacloreto de fósforo gasoso ocorre segundo a seguinte equação
química balanceada: PCl5(g) ⇄ PCl3(g) + Cl2(g). Determine a expressão da densidade (d) em função do grau de
dissociação (α) da mistura gasosa.

Resolução: Base de cálculo: n(mol) inicial de pentacloreto de fósforo. Equação química: PCl5(g) ⇄ PCl3(g) + Cl2(g)
Tabela de equilíbrio químico:

PCl5(g) PCl3(g) Cl2(g)


Início n 0 0
Reage nα nα nα
Equilíbrio n - nα nα nα

Cálculo do número de mol total (nT) da mistura gasosa, no equilíbrio: nTotal = nPCl5 + nPCl3 + nCl2 = n - nα + nα + nα = n
+ nα = n x (1 + α)

Relação da densidade (d) com o grau de dissociação (α), a partir da equação dos gases ideais: p. < MM >=
m
V
. (1 + α). R. T

p. < MM >= d x (1 + α) x R x T

p x < MM >
d=
(1 + α) x R x T

Questão 244 - A uma temperatura de aproximadamente 250 K e pressão de 20 atm, o fator de compressibilidade de
um determinado gás específico é igual a 0,86. Calcule o volume ocupado por 7,0 milimol deste gás nessas condições.

Resolução: Cálculo do volume a partir da equação dos gases reais, em função do fator de compressibilidade (Z).

Z x n x R x T 0,86 x 7,0 x 10−3 x 0,08206 x 250


V= = = 6,175 x 10−3 L
p 20

Questão 245 - Um gás encontra-se a uma temperatura de 295 K e pressão de 40 atm, apresentando um volume
molar real 22,50% maior do que o calculado pela lei dos gases ideais. A partir desta informação, determine:
a) o fator de compressibilidade;
b) o volume molar do gás;
c) que forças são dominantes deste gás nestas condições?

118
1600 FÍSICO-QUÍMICA APLICADA EXERCÍCIOS COMENTADOS - IME – ITA – OLIMPÍADA

Resolução:
a) Considerando um aumento de 22,50% em relação ao calculado pela lei dos gases ideais, a relação entre o
volume molar real com o volume molar ideal é:
Real Ideal
Vmolar = 1,225 x Vmolar
VReal 1,2250 x VIdeal
Cálculo do fator de compressibilidade: Z = molar
= molar
= 1,2250
VIdeal
molar VIdeal
molar

b) Cálculo do volume molar do gás, a partir da equação dos gases reais (com a presença do fator de
compressibilidade):
V ZxRxT
=

n p
Vmolar

1,225 x 0,08206 x 295


Vmolar = = 0,741 L. mol−1
40

c) Para Z = 1,2250, as forças dominantes são as repulsivas, uma vez que Z > 1.

Questão 246 - Sabe-se que nas condições normais de temperatura e pressão, um mol de um determinado gás ocupa
volume de 22,4 litros. Utilizando a equação dos gases reais, de Van der Waals, determine a pressão de um mol de
gás oxigênio na mesma temperatura e compare com o valor esperado. Dados: constantes de van der Waals para o
gás oxigênio: a = 1,378 L².atm.mol-2 e b = 3,183 x 10-4 L.mol-1.

22,4 L
Resolução: Cálculo do volume molar: Vmolar = 1,0 mol = 22,4 L. mol−1

a
Analisando o gás oxigênio com comportamento de um gás real: {preal + V2molar
}x {Vmolar − b} = R x T

1,378
{preal + } x {22,4 − 3,183 x 10−4 } = 0,08206 x (0 + 273)
(22,4)²

preal = 0,997 atm

Diante do resultado obtido da pressão através da equação de van der Waals, a pressão calculada pela equação dos
gases ideais não apresenta diferenças significativas.

Questão 247 - (GRILLO) A constante a de van der Waals de um gás é igual a 0,50 m6.Pa.mol-2, apresentando um
volume molar igual a 5,00 x 10-4 m³.mol-1, a 273 K e pressão de 3,0 MPa. A partir destes dados, calcule:
a) a constante b de van der Waals.
b) o fator de compressibilidade do gás nessas condições de temperatura e pressão.

Resolução: Item a) Cálculo da constante de van der Waals (b), a partir da equação de van der Waals:
a
{preal + V2molar
} x {Vmolar − b} = R x T

119
1600 FÍSICO-QUÍMICA APLICADA EXERCÍCIOS COMENTADOS - IME – ITA – OLIMPÍADA

0,50
{3,0 x 106 + } x (5,0 x 10−4 − b) = 8,314 x 273
(5,0 x 10−4 )²

b = 4,60 x 10-5 m³.mol-1

V
Cálculo da pressão considerando comportamento ideal: p x (⏟
n
) =RxT
𝑉𝑜𝑙𝑢𝑚𝑒 𝑚𝑜𝑙𝑎𝑟
(𝑉𝑚 )

8,314 x 273
p= = 4,54 x 106 Pa
5,0 x 10−4

preal 3,0 x 106


Item b) Cálculo do fator de compressibilidade (Z): Z = pideal = 4,54 x 106 = 0,66

Para Z = 0,66, as forças dominantes são atrativas, uma vez que Z < 1.

Questão 248 - (GRILLO) Explique a Lei de Dalton das pressões parciais e faça as devidas deduções.

Resolução: Considere um determinado recipiente de volume V contendo três gases distintos (A, B e C), com
temperatura constante T. Cálculo da pressão total (PT), a partir da equação dos gases ideais:

pT x VT = nT x R x T

pT x (VA + VB + VC ) = (nA + nB + nC ) x R x T

nA nB nC
pT = ( + + ) xRxT
VA VB VC

nA nB nC
pT = ( xRxT+ xRxT + x R x T)
VA
⏟ VB
⏟ VC

𝑃𝐴 𝑃𝐵 𝑃𝐶

pT = PA + PB + PC

Lei de Dalton das pressões parciais: Quando dois ou mais gases que não reagem quimicamente são colocados num
mesmo reservatório, a pressão exercida por cada gás na mistura é a mesma que ele teria se estivesse sozinho no
reservatório. A Pressão exercida por cada gás numa mistura é chamada de Pressão Parcial. “A pressão total exercida
por uma mistura é a soma das pressões parciais”.

Questão 249 - (GRILLO) Explique a Lei de Amagat dos volumes parciais.

Resolução: Considere um determinado recipiente de volume V contendo três gases distintos (A, B e C), com
temperatura constante T. Cálculo do volume total (VT), a partir da equação dos gases ideais:

VT x (pA + pB + pC ) = (nA + nB + nC ) x R x T

120
1600 FÍSICO-QUÍMICA APLICADA EXERCÍCIOS COMENTADOS - IME – ITA – OLIMPÍADA

nA nB nC
VT = ( + + ) xRxT
pA pB pC

nA nB nC
VT = ( xRxT+ xRxT + x R x T)
p
⏟A p
⏟ p
⏟C
VA VB VC

VT = VA + VB + VC

Lei de Amagat dos volumes parciais: Quando dois ou mais gases que não reagem quimicamente são colocados num
mesmo reservatório, o volume exercido por cada gás na mistura é a mesma que ele teria se estivesse sozinho no
reservatório. O volume exercido por cada gás numa mistura é chamada de Volume Parcial. “O Volume total exercido
por uma mistura é a soma dos volumes parciais”.

Questão 250 – (GRILLO) Mostre que a constante universal dos gases (R) vale 0,08206 atm.L.mol-1.K-1.

pxV
Resolução: Através da equação dos gases ideais, temos: R = n x T e considerando que o processo esteja nas
pxV 1 x 22,4 atm.L
Condições Normais de Temperatura e Pressão (CNTP), temos: R = = = 0,08206
nxT 1 x 273 mol.K

Questão 251 – (GRILLO) Calcule os parâmetros de van der Waals para um determinado gás que apresenta a
temperatura crítica igual a 277 K e pressão crítica igual a 55,5 atmosferas.

Resolução: Sabendo que a temperatura crítica e a pressão crítica estão apresentadas pelas seguintes equações a
seguir:

8.𝑎
𝑇𝑐𝑟í𝑡𝑖𝑐𝑎 = 27.𝑏.𝑅 (Equação A)

𝑎
𝑃𝑐𝑟í𝑡𝑖𝑐𝑎 = 27.𝑏² (Equação B)

8a
A partir da temperatura crítica igual a 277 K, temos: Tcrítica = 27bR

8a
277 = 27bR

a
8a = 277 x 27 x b x (0,08206) → b = 76,71 (Equação C)

a
A partir da pressão crítica igual a 55,5 atmosferas, temos: Pcrítica = 27b²

𝑎
55,5 = 27𝑏²

𝑎 1
55,5 = 27𝑏 𝑥 𝑏

121
1600 FÍSICO-QUÍMICA APLICADA EXERCÍCIOS COMENTADOS - IME – ITA – OLIMPÍADA

𝑎 1
55,5 𝑥 27 = 𝑥
𝑏 𝑏

𝑎 1
1498,5 = 𝑏 𝑥 𝑏 (Equação D)

𝑎
Sabendo que pela equação C, a relação 𝑏 é igual a 76,71, então a constante b de van der Waals é igual a:

1
1498,5 = (76,71) 𝑥
𝑏

b = 5,12 x 10-2 L.mol-1

𝑎 𝑎
Cálculo da constante a de van der Waals, a partir da equação C: 𝑏
= 76,71 → 5,12 𝑥 10−2 = 76,71 → a = 3,93

atm.L².mol-2

Questão 252 – A constante de van der Waals b, para os gases reais, pode ser usada para determinar o tamanho de
um átomo ou de uma molécula no estado gasoso, sendo o valor de b em relação ao N2 igual a 39,4 x 10-6 m3.mol-1,
determine o raio hipotético de uma molécula de N2 desse gás.

Resolução: Considerando que o gás nitrogênio apresente como configuração matemática uma configuração esférica,
4πR³
o raio da molécula de nitrogênio gasoso, pode ser calculado através da seguinte expressão: V = 3

4πR³ b
=
3 NA

3.b 3 3.b
R³ = →R= √
4π.NA 4π.N A

Onde b é a constante de van der Waals, relacionado ao volume da molécula gasosa e N A = constante de Avogadro.
Sendo a constante de van de Waals b = 39.4 x 10-6 m3.mol-1, temos:

3 3. b 3 3 x 39,4 x 10−6 3
R= √ =√ = √1,56 x 10−29 = 2,50 x 10−9 m
4π. NA 4 x 3,14 x 6,02 x 1023

Questão 253 – (GRILLO) Considere um litro de um recipiente com um determinado gás que apresenta as seguintes
características: PINICIAL = 340 atmosferas, temperatura inicial igual a 444 K é comprimido para uma pressão de 1200
atmosferas e -23°C. O fator de compressibilidade para o estado inicial é igual a 1,077 e para o estado final é igual a
1,876. A partir destas informações, determine o volume deste gás para o estado final.

Resolução: Dados do problema:


Situação inicial: PINICIAL = 340 atm; VINICIAL = 1,0 L; TINICIAL = 444 K; ZINICIAL = 1,077

Situação final: PFINAL = 1200 atm; VFINAL = ?; TFINAL = (- 23 + 273) = 250 K; ZFINAL = 1,876

Desenvolvendo a equação dos gases reais com a presença do fator de compressibilidade, temos:

122
1600 FÍSICO-QUÍMICA APLICADA EXERCÍCIOS COMENTADOS - IME – ITA – OLIMPÍADA

pV = ZnRT (Equação A)

Para a resolução deste problema, será considerado tanto para o estado inicial quanto para o estado final uma
quantidade de 1 mol. Isolando o produto constante (nR) da equação A, temos:

pV
nR =
ZT

pV pV
nR = ( ) =( )
ZT INICIAL ZT FINAL

340 x 1,0 1200 x V


(1,077 x 444) = (1,876 x 250) → VFINAL = 0,278 L
INICIAL FINAL

Questão 254 – (ITA) Num cilindro contendo uma mistura de gás oxigênio e gás argônio, a pressão total é de 10
atmosferas. Sabendo que a pressão parcial do oxigênio é 5,0 vezes maior do que a pressão parcial do argônio, no
cilindro o valor da relação (Massa do oxigênio em gramas / Massa do argônio) em gramas é:
a) 2,0
b) 3,5
c) 4,0
d) 5,0
e) 6,3

Resolução: Alternativa C.

Sabendo que pela lei de Dalton, o somatório das pressões parciais é igual a pressão total, temos: PTOTAL = PO2 + PAR

PO2 + PAR = 10 atm

Informação do problema: A pressão parcial do O2 é cinco vezes maior que a pressão parcial do Argônio, logo: PO2 =
5 x PAR.

Cálculo da pressão parcial do gás Argônio: PO2 + PAR = 10 atm

5.PAR + PAR = 10 atm

PAR = 1,67 atm

Cálculo da pressão parcial do gás Oxigênio: PO2 = 5 x PAR

PO2 = 5 x 1,67 = 8,33 atm

Sabendo que a pressão parcial é o produto da fração molar com a pressão total, temos: PAr = X Ar x pTOTAL

X Ar x pTOTAL = 1,67

nAr
x pTOTAL = 1,67
nTOTAL

123
1600 FÍSICO-QUÍMICA APLICADA EXERCÍCIOS COMENTADOS - IME – ITA – OLIMPÍADA

m
(< MMAr> )
Ar
x pTOTAL = 1,67
nTOTAL

66,8
mAr = x pTOTAL = 1,67
nTOTAL

66,8
mAr = pTOTAL
x nTOTAL (Equação I)

Analisando para o gás oxigênio: PO2 = X O2 x pTOTAL

X O2 x pTOTAL = 1,67

nO2
x pTOTAL = 8,33
nTOTAL

mO
(< MM 2> )
O2
x pTOTAL = 8,33
nTOTAL

266,56
m O2 = pTOTAL
x nTOTAL (Equação II)

266,56 x nTOTAL
mO2 PTOTAL 266,56
Dividindo a equação II pela equação I, temos: = 66,8 x nTOTAL = = 4,01
mAr 66,8
PTOTAL

Questão 255 – (ITA) Um frasco fechado contém dois gases cujo comportamento é considerado ideal: hidrogênio
molecular e monóxido de nitrogênio. Sabendo que a pressão parcial do monóxido de nitrogênio é igual a 3/5 da
pressão parcial do hidrogênio molecular, e que a massa total da mistura é de 20 g, assinale a alternativa que fornece
a porcentagem em massa do hidrogênio molecular na mistura gasosa.
a) 4%
b) 6%
c) 8%
d) 10%
e) 12%

Resolução: Alternativa D.

3
Dados do problema: PNO = (5) x pH2 e mNO + mH2 = 20 g

3
Relação matemática entre a massa do monóxido de nitrogênio com o hidrogênio molecular: PNO = (5) x pH2

3
X NO x PTOTAL = ( ) x X H2 x PTOTAL
5

124
1600 FÍSICO-QUÍMICA APLICADA EXERCÍCIOS COMENTADOS - IME – ITA – OLIMPÍADA

nNO 3 nH2
x PTOTAL = ( ) x x PTOTAL
(nNO + nH2 ) 5 (nH2 + nNO )

m m H2
(< MMNO> ) 3
(
< MM >H2
)
NO
x PTOTAL = ( ) x x PTOTAL
(nNO + nH2 ) 5 (nH2 + nNO )

mNO 3 m H2
= ( )x
< MM >NO 5 < MM >H2

mNO 3 mH2
30
= (5) x 2
→ mNO = 9 x mH2

Cálculo da massa de hidrogênio molecular, resolvendo o seguinte sistema de equações: m NO = 9.mH2 e mNO + mH2 =
20 g

A massa de Hidrogênio calculada no sistema acima é igual a 2,0 g. Logo, o percentual de hidrogênio na mistura
2,0
gasosa é de: (%)𝐻2 = 20,0 = 0,10 (10%)

Questão 256 – (GRILLO) Usando a equação de van der Waals, calcule a pressão desenvolvida através de 100
gramas de dióxido de carbono contido em um volume de cinco litros a uma temperatura igual a 40°C. Além disso,
compare este valor com o calculado usando a lei dos gases ideais e determine a percentagem devido a idealidade,
considerando as constantes de van der Waals do dióxido de carbono igual a: a = 3,6 atm.L².mol-2 e b = 44 cm³.mol-1.

nCO 100
Resolução: Cálculo do número de mol de CO2: nCO2 = <MM>2 = 44
= 2,27 mol
CO2

V 5,0
Cálculo do volume molar (Vm): Vmolar = n = 2,27
= 2,20 L. mol−1

cm³ 10−3 dm³


Cálculo da pressão considerando o gás com comportamento real: b = 44 mol
x( 1 cm³
) =

44 x 10−3 dm³. mol−1

a
{preal + 2 }x {Vmolar − b} = R x T
Vmolar

3,6
{𝑝𝑟𝑒𝑎𝑙 + (2,20)²
}𝑥 {2,20 − 44 𝑥 10−3 } = 0,08206 𝑥 (40 + 273)

𝑝𝑟𝑒𝑎𝑙 = 11,17 𝑎𝑡𝑚

Cálculo da pressão considerando o gás com comportamento ideal: pideal x 5 = 2,27 x 0,08206 x 313

pideal = 11,66 atm

11,66−11,17
Cálculo do percentual: (%) = 11,66
= 4,20 𝑥 10−2 (4,20 %)

125
1600 FÍSICO-QUÍMICA APLICADA EXERCÍCIOS COMENTADOS - IME – ITA – OLIMPÍADA

Questão 257 – (GRILLO) Considere que o gás etileno, gás muito utilizado no setor químico industrial, apresentando
a temperatura crítica igual a 282 K e pressão crítica igual a 50 atm. A partir destes dados, determine as constantes de
van der Waals.

Resolução: Sabendo que a temperatura crítica é definida através da seguinte equação matemática, temos:
8.𝑎
𝑇𝑐𝑟í𝑡𝑖𝑐𝑎 = 27 x b x R
8. 𝑎
282 =
27 x b x R

𝑎 27 𝑥 0,08206 𝑥 282
= = 78,10
𝑏 8
𝑎
Sabendo que a pressão crítica é definida como sendo: 𝑃𝑐𝑟í𝑡𝑖𝑐𝑎 = 27.𝑏²

a
50 =
27 x b²
a
= 27 x 50

a 1
x = 1350
b b
a 1
Sabendo que a relação é igual: a / b = 78,10, temos: b x b
= 1350

1
78,10 x = 1350
b

b = 0,058 L. mol−1

a
Cálculo da constante de van der Waals (a): = 78,10
b

a
= 78,10
0,058

a = 4,52 atm. L². mol−2

Questão 258 – Uma mistura composta por dois gases nobres (Hélio e Neônio) apresenta uma massa total igual a
5,50 gramas ocupando um volume de 6,80 litros a 300 K e 1,0 atm. A partir desta informação, calcule a composição
da mistura em bases percentuais.

Resolução: Dados do problema: mtotal = mNe + mHe = 5,50g

Cálculo do número de mol total para um volume de 6,80 litros, 300 K e 1 atm, a partir da utilização da equação dos
gases ideais:

126
1600 FÍSICO-QUÍMICA APLICADA EXERCÍCIOS COMENTADOS - IME – ITA – OLIMPÍADA

pV 1 x 6,80 6,80
ntotal = = = = 0,276 mol
RT 0,08206 x 300 24,62

Equação (1): nNe x < MM >Ne + nHe x < MM >He = 5,50

20 𝑥 nNe + 4 x nHe = 5,50

Dividindo a equação acima por quatro, temos: 5 𝑥 nNe + nHe = 1,375

Equação (2): nNe + nHe = 0,276

Resolvendo o sistema de equações (1) e (2): 5 𝑥 nNe + nHe = 1,375

nNe + nHe = 0,276

Multiplicando a segunda equação por menos cinco (-5):

5 𝑥 nNe + nHe = 1,375

−5 x nNe + −5 x nHe = −1,38

−4 x nHe = −0,005

nHe = 0,00125 mol

Logo, o número de mol de gás neônio é de 0,275 mol. Cálculo das frações molares dos gases nobres:

nNe 0,275
X Ne = = = 0,9963 (99,64%)
ntotal 0,276

nHe 0,00125
X He = = = 0,00453 (0,453%)
ntotal 0,276

Questão 259 – (GRILLO) Um gás a 220 K e 11 atm tem o volume molar 12% maior do que o calculado pela lei dos
gases perfeitos. Calcule o fator de compressibilidade nestas condições e o volume molar do gás. Que forças são
dominantes no gás, as atrativas ou as repulsivas?

real ideal
Resolução: Dados do problema: Vmolar = 1,12 x Vmolar

Vreal 1,12 x Videal


Sabendo que o fator de compressibilidade é definido como: Z = molar
= molar
= 1,12
Videal
molar Videal
molar

Cálculo do volume molar a partir da equação dos gases reais em função do fator de compressibilidade:

Z x R x T 1,12 x 0,08206 x 220 L


Vmolar = = = 1,84
p 11 mol

127
1600 FÍSICO-QUÍMICA APLICADA EXERCÍCIOS COMENTADOS - IME – ITA – OLIMPÍADA

Questão 260 - (IME) A que temperatura, em °C, o etileno, a 800 mm Hg, terá a mesma densidade absoluta que o
oxigênio, a 700 mm Hg e a 20°C?

𝑝 𝑥<𝑀𝑀>
Resolução: A partir da equação dos gases ideais, a relação entre a densidade e a pressão será: 𝑑 = 𝑅𝑥𝑇
.

800 𝑥 28
Analisando o Gás Etileno: 𝑑𝐶2 𝐻4 = (Equação 1)
𝑅 𝑥 𝑇𝐶2 𝐻4

700 𝑥 32
Analisando o Gás Oxigênio: 𝑑𝑂2 = 𝑅 𝑥 (20+273) (Equação 2)

Igualando as equações das densidades (1) e (2), temos: dC2H4 = dO2

800 x 28 700 x 32
dC2 H4 = = = dO2
R x TC2 H4 R x (20 + 273)

800 x 28 700 x 32
=
R x TC2 H4 R x 293

TC2 H4 = 293 K (20°C)

Questão 261 – (IME) Para medir o volume de um recipiente A, de formato irregular, contendo oxigênio a 27°C e 24,6
atmosferas, usou-se outro recipiente, indeformável, de 6,0 litros de volume. O recipiente B, quando completamente
vazio pesou 422 gramas. Fez-se a ligação entre A e B deixando que o gás passasse de A para B até atingir o equilíbrio.
Nessas condições, B pesou 470 gramas. Calcule o volume de A.

Resolução: Dados do tanque A: pA = 24,6 atm; VA; TA = 27 + 273 = 300K

Dados do tanque B: PB; VB = 6,0 Litros; TB = 27 + 273 = 300K

Cálculo da massa de oxigênio: mO2 = 470 g – 422 g = 48 g

48
Cálculo da pressão do tanque B, utilizando a equação dos gases ideais: pB x 6,0 = (32) x 0,08206 x (27 +

273)

pB = 6,15 atm

Cálculo do volume do tanque A, sabendo que o volume do tanque B é igual a VB = VA + 6,0, temos: pA x VA =
pB x VB

24,6 x VA = 6,15 x (VA + 6,0)

VA = 2,0 L

128
1600 FÍSICO-QUÍMICA APLICADA EXERCÍCIOS COMENTADOS - IME – ITA – OLIMPÍADA

Questão 262 – (IME) Um reservatório, com capacidade de metano de 2000 m3, é submetido à temperatura máxima
de 47,0°C no verão e à temperatura mínima de 7,0°C no inverno. Calcule em quantos quilogramas a massa do gás
armazenado no inverno excede àquela do verão, sendo submetido a uma pressão de 0,1 MPa. Despreze as variações
de volume do reservatório com a temperatura e considere o metano como um gás ideal.

m
Resolução: Através da equação dos gases ideais, temos a seguinte relação: pideal x V = xR xT
<MM>

p x V x<MM>
Cálculo da massa de gás metano no verão, a uma temperatura de 47°C: mverão = RxT

0,10 x 106 x 2,0 x 103 x 16


mverão = = 1202790,47 g
8,314 x (47 + 273)

p x V x<MM>
Cálculo da massa de gás metano no inverno, a uma temperatura de 7°C: minverno = RxT

0,10 x 106 x 2,0 x 103 x 16


minverno = = 1374617,68 g
8,314 x (7 + 273)

Cálculo da variação de massa (Δm) de gás metano: Δm = minverno – mverão = 1374617,68 – 1202790,47 = 171827,21 g
(≈171,83 kg)

Questão 263 – (CONCURSO PARA DOCENTE - IFRJ) Uma mistura de monóxido de carbono (CO) e dióxido de
carbono (CO2), com comportamento ideal, apresenta massa específica igual a 1,332 kg.m-³, quando se encontra sob
pressão de 750 mmHg a temperatura de 25oC. Então, responda a estes questionamentos.

a) Qual é a massa molar da mistura?


b) Qual é a composição da mistura?
c) Qual é a pressão parcial de cada gás que constitui essa mistura?
d) Calcule o fator de compressibilidade dessa mistura, sabendo que o seu volume molar real é 23,50 L.mol -1.

𝑘𝑔 1𝑚³ 1000 𝑔
Resolução: Conversão de unidade da densidade: 𝑑 = 1,332 𝑚³
𝑥 1000 𝐿
𝑥 1 𝑘𝑔
= 1,332 𝑔. 𝐿−1

p x<MM>
a) Cálculo da massa molar (<MM>) da mistura gasosa: d = RxT
750
(760) x < MM >Mistura
d= = 1,332
R x (25 + 273)

1,322 x 760 x 0,08206 x 298


< MM >Mistura = = 33,01 g. mol−1
750

b) Cálculo da composição da mistura: X CO x < MM >CO + X CO2 x < MM >CO2 = X mistura x <
MM >mistura e X CO + X CO2 = 1
28 x X CO + 44 x X CO2 = 1 x < MM >mistura

X CO + X CO2 = 1

129
1600 FÍSICO-QUÍMICA APLICADA EXERCÍCIOS COMENTADOS - IME – ITA – OLIMPÍADA

Resolvendo o sistema acima, as frações molares das espécies gasosas são as seguintes: XCO = 0,6875 (68,75%) e
XCO2 = 0,3125 (31,25%)

c) Cálculo das pressões parciais das substâncias gasosas:


PCO = X CO x pTOTAL = 0,6875 x 750 = 515,625 mmHg

PCO2 = X CO x pTOTAL = 0,3125 x 750 = 234,375 mmHg

RxT 0,08206 x 298


d) Cálculo do volume molar para um gás ideal: Vmolar = = = 24,78 L. mol−1
p (750⁄760)

VReal 23,50 L.mol−1


Cálculo do fator de compressibilidade (Z): Z = molar
= = 0,948
VIdeal
molar 24,78 L.mol−1

Questão 264 – (CONCURSO PARA DOCENTE - IFRJ) Uma amostra de 30,0 gramas de C2H6 encontra-se na
condição I (temperatura 0,0°C e volume de 22,414 L), sendo comprimida e aquecida até a condição II (temperatura =
727°C e volume de 100 cm³). Analise a transformação no sistema gasoso e responda aos questionamentos dos itens
A e B.
a) Qual é a pressão desse gás, comportando-se como:
A.1) gás ideal na condição I?
A.2) gás de Van der Waals na condição I?
A.3) gás ideal na condição II?
A.4) gás de Van der Waals na condição II?
b) Calcule o valor aproximado da constante de compressibilidade na condição I e na condição II, considerando
que o gás de Van der Waals representa o comportamento real do gás. Dados gerais da questão: a (C2H6) = 5,49
L².atm.mol-2; b(C2H6) = 0,064 L.mol-1. Resolução:
30
A.1) Cálculo do número de mol do gás Etano (C2H6): 𝑛 = 30 = 1,0 𝑚𝑜𝑙

Cálculo da pressão, considerando o sistema gasoso como ideal: pideal x 22,41 = 1,0 x 0,08206 x 273

pideal = 1,0 atm

a
A.2) Cálculo da pressão pela equação de Van der Waals: {preal + V2molar
}x {Vmolar − b} = R x T

5,49 22,414
{preal + }x { − 0,064} = 0,08206 x 273
22,414 1
( 1 )²

preal = 0,991 atm

A.3) Cálculo da pressão do gás considerando ideal: pideal x 0,100 = 1,0 x 0,08206 x 273

pideal = 820,6 atm

130
1600 FÍSICO-QUÍMICA APLICADA EXERCÍCIOS COMENTADOS - IME – ITA – OLIMPÍADA

a
A.4) Cálculo da pressão pela equação de Van der Waals: {preal + V2molar
}x {Vmolar − b} = R x T

5,49 0,100
{preal + }x { − 0,064} = 0,08206 x 273
0,100 1
( )²
1

preal = 1730,44 atm

𝑃𝑅𝑒𝑎𝑙 0,99 𝑎𝑡𝑚


B) Cálculo do valor aproximado do fator de compressibilidade (Z) na condição I: 𝑍 = 𝑃𝐼𝑑𝑒𝑎𝑙
= 1,00 𝑎𝑡𝑚
=

0,99 𝑎𝑡𝑚.

PReal 1730,44 atm


Cálculo do valor aproximado do fator de compressibilidade (Z) na condição II: Z = PIdeal
= 820,6 atm
=

2,11 atm.

Questão 265 – (IME) Para se encher um balão de borracha, até certo tamanho, na temperatura de -3°C necessitou-
se de 0,20 gramas de hidrogênio. Para se encher o mesmo balão, até o mesmo tamanho, a uma outra temperatura,
necessitou-se de 0,18 gramas do mesmo gás. Se a elasticidade do balão não variou nos dois casos, determinar a
segunda temperatura.

Resolução: Relação matemática entre a massa do gás e a temperatura, considerando o gás como ideal:

m
pideal x V = ( )x R x T
< MM >

pideal x V x < MM >


= (m x T)constante
R

Considerações a serem feitas para a resolução deste problema:

✓ Processo isobárico, pelo fato do enchimento do balão ocorrer ao nível do mar;


✓ Processo isocórico, pois não houve variação na elasticidade do balão;
✓ R é apresentado como a constante dos gases
✓ A massa molar é a mesma para os dois casos, uma vez que se trata do mesmo gás, hidrogênio molecular
(H2).

Situação I: P = constante; V = constante; TI = 273 + (-3°C) = 270K e mI = 0,20 g

Situação II: P = constante; V = constante; TII = ? e mII = 0,18 g

Cálculo da temperatura II, através da equação combinada dos gases ideais: mI x TI = mII x TII

0,20 g x 270 K = 0,18 g x TII

TII = 300 K (27°C)

131
1600 FÍSICO-QUÍMICA APLICADA EXERCÍCIOS COMENTADOS - IME – ITA – OLIMPÍADA

Questão 266 – (IME) Uma determinada reação química gera um produto gasoso, do qual foi coletada uma amostra
para análise. Verificou-se que a amostra, pesando 0,32 gramas, ocupa 492 cm³ a 27°C e 1 atm de pressão, obedece
a lei dos gases ideais e é formada por 75% em peso de carbono e 25% em peso de hidrogênio. Determine:

a) Qual o peso molecular deste gás?


b) Qual a sua fórmula molecular mínima?

Resolução:

m
Item a) Cálculo da massa molar do gás, considerando comportamento ideal: pideal x V = <MM> x R x T

mxR xT 0,32 x 0,08206 x 300


< MM > = = = 16,02 g. mol−1
pideal x V 1 x 0,492

Item b) Cálculo do número de átomo de carbono:


16,01 g.mol-1 ---------------- 100%
<MM>Carbono ----------------- 75%
<MM>Carbono = 12 g.mol-1
12 𝑔.𝑚𝑜𝑙−1
Cálculo do número de átomos de carbono = 12 𝑔.𝑚𝑜𝑙−1 = 1

Cálculo da massa molar de hidrogênio: <MM>Hid. = (<MM>Carbono - 12 g.mol-1) = 16 g.mol-1 - 12 g.mol-1 = 4 g.mol-1

4
Cálculo do número de átomos de hidrogênio = (1) = 4. Fórmula molecular: CH4 (Metano).

Questão 267 – (IME) Um gás ideal desconhecido contendo 80% em massa de carbono e 20% em massa de
hidrogênio tem massa específica 1,22 g.L-1, quando submetido a pressão de uma atmosfera e a temperatura de 27°C.
Calcule a massa molecular e escreva a fórmula molecular desse gás. Resolução: Cálculo da massa molar do gás
p x <MM>
ideal desconhecido, através da relação entre a densidade de um gás e sua pressão: d = ( )
RxT

1 x <MM>
1,22 = (0,08206 x 300)

<MM> = 30,03 g.mol-1

Cálculo do número de átomo de carbono:

30,03 g.mol-1 --------------- 100%


<MM>Carbono ---------------- 80%
<MM>Carbono = 24 g.mol-1
𝑔
24
Número de átomos de carbono = 𝑚𝑜𝑙
𝑔 =2
12
𝑚𝑜𝑙

Cálculo do número de átomo de hidrogênio: <MM>Hid. = <MM>Carbono - 12 g.mol-1

132
1600 FÍSICO-QUÍMICA APLICADA EXERCÍCIOS COMENTADOS - IME – ITA – OLIMPÍADA

<MM>Hid. = 30 g.mol-1 - 24 g.mol-1 = 6 g.mol-1


𝑔
6
Cálculo do número de átomos de hidrogênio = 𝑚𝑜𝑙
𝑔 = 6. Logo, a fórmula molecular é o C2H6 (Etano).
1
𝑚𝑜𝑙

Questão 268 – (IME) Tendo-se uma mistura gasosa, formada de 0,60 mol de CO2, 1,50 mol de O2 e 0,90 mol de N2,
cuja massa específica é de 1,35 g.L-1 a 27°C, calcule:

a) As pressões parciais dos gases;


b) O volume da mistura.

Resolução:

Item a) Cálculo do número de mol total da mistura gasosa: nMistura = nCO2 + nO2 + nN2 = 0,60 + 1,50 + 0,90 = 3,00 mol

Cálculo da massa molecular da mistura: {(%) x <MM>}CO2 + {(%) x <MM>}O2 + {(%) x <MM>}N2 = {(%) x <MM>}Mistura

{(%) x <MM>}Mistura = {(%) x <MM>}CO2 + {(%) x <MM>}O2 + {(%) x <MM>}N2

{1,0 x <MM>}Mistura = {(0,60/3,0) x 44}CO2 + {(1,50/3,00) x 32}O2 + {(0,90/3,00) x 28}N2

<MM>Mistura = 33,20 g.mol-1

1,35 x 0,08206 x 300


Cálculo da pressão total do sistema (pT): pT = 33,20
= 1,0 𝑎𝑡𝑚

Cálculo das pressões parciais:

0,60
Pressão parcial do CO2: PCO2 = XCO2 x pT = ( 3,0 ) x 1 atm = 0,30 atm
1,50
Pressão parcial do O2: PCO2 = XO2 x pT = ( )x 1 atm = 0,50 atm
3,0
0,90
Pressão parcial do N2: PCO2 = XN2 x pT = ( 3,0 ) x 1 atm = 0,30 atm

Item b) Cálculo do volume da mistura: pT x VT = (nCO2 + nO2 + nN2 ) x R x T

1 x VT = 3,0 x 0,08206 x 300

VT = 73,85 L

Questão 269 – (IME) A equação do gás ideal só pode ser aplicada para gases reais em determinadas condições
especiais de temperatura e pressão. Na maioria dos casos práticos é necessário empregar uma outra equação, como
a de van der Waals. Considere um mol do gás hipotético A contido num recipiente hermético de 1,1 litros a 27ºC. Com
auxílio da equação de van der Waals, determine o erro cometido no cálculo da pressão total do recipiente quando se
considera o gás A como ideal. Dados: Constante universal dos gases: R = 0,08206 atm.L.mol-1.K-1; Constantes da
equação de van der Waals: a = 1,21 atm.L2.mol-2 e b = 0,10 L.mol-1.

Resolução: Dados do problema: n = 1 mol; V = 1,1 L; T = 27ºC (300 K)

133
1600 FÍSICO-QUÍMICA APLICADA EXERCÍCIOS COMENTADOS - IME – ITA – OLIMPÍADA

Considerando que o gás apresente comportamento ideal: pideal x 1,1 = 1,0 x 0,08206 x (27 + 273)

pideal = 22,38 atm

1,21
Considerando que o gás apresente comportamento real: {preal + (1,1)²
}x {1,1 − 0,1} = 0,08206 x (27 +

273)

preal = 23,62 atm

23,62−22,38
Cálculo do erro (%): %(erro) = ( ) = 0,0524 (5,24%)
22,38

Questão 270 – (IME) Determinada quantidade de nitrogênio ocupa um volume de dez litros a uma temperatura de
127°C e a uma pressão de 4,92 atmosferas. Adicionaram-se ao nitrogênio 9,03 x 1023 moléculas de oxigênio (O2).
Sabendo-se que a pressão final de equilíbrio do sistema é de 6,15 atmosferas, calcule a temperatura final de equilíbrio.

Resolução: Cálculo do número de mol de N2, a partir da equação dos gases ideais: pideal x V = nN2 x R x T

4,92 x 10 = nN2 x 0,08206 x (127 + 273)

nN2 = 1,50 mol

9,03 x 1023 moléculas


Cálculo do número de mol de O2: nO2 = = 1,50 mol
6,02 x 1023 moléculas x mol−1

Cálculo do número de mol total: nTOTAL = nO2 + nN2 = 1,50 mol + 1,50 mol = 3,00 mol

Cálculo da temperatura final de equilíbrio (T), a partir da utilização da equação dos gases ideais: 6,15 x 10 =
3,0 x 0,08206 x T

T = 250 K

Questão 271 – (IME) O gás obtido pela completa decomposição térmica de uma amostra de carbonato de cálcio com
50,0% de pureza é recolhido em um recipiente de 300 mL a 27,0°C. Sabendo-se que a pressão no recipiente é de
1,66 MPa, determine:

a) a massa de gás produzido, admitindo que seu comportamento seja ideal;


b) a massa da amostra utilizada.
10−3 L 1 dm³ 1 m³
Resolução: Conversão de unidade do volume (mL para m³): Volume = 300 mL x 1 mL
x 1L
x 1 dm³
=

3,0 x 10−4 m³

Item a) Cálculo do número de mol do gás produzido, utilizando a equação dos gases ideais:

pideal x V 1,66 x 106 x 3,0 x 10−4


nCO2 = = = 0,200 mol
RxT 8,314 x (27 + 273)

Massa de dióxido de carbono: mCO2 = nCO2 x <MM>CO2 = 0,200 x 44 = 8,8 g

134
1600 FÍSICO-QUÍMICA APLICADA EXERCÍCIOS COMENTADOS - IME – ITA – OLIMPÍADA

Item b) Cálculo da massa de carbonato de cálcio: CaCO3(s) → CaO(s) + CO2(g)

1 mol de CaCO3(s) ------------ 1 mol de CO2(g)


100 g ---------------------------- 44 g
0,50 x mCO2 --------------------- 8,78 g
mCO2 = 39,93 g

Questão 272 – (CONCURSO PARA DOCENTE - IFRJ) Uma massa de 1,37 gramas de amônia no estado gasosos,
está confinada em um recipiente de volume correspondente a 5,00 litros e a temperatura de 100°C. Então, comprove
que, nessas condições, o gás se comporta idealmente. Dados: Constante de van der Waals para o NH3: a = 4,169
atm.L².mol² e b = 3,710 x 10-2 L.mol-1.

𝑚 1,37
Resolução: Cálculo do número de mol: 𝑛 = <𝑀𝑀> = 17
= 8,06 𝑥 10−2 𝑚𝑜𝑙

Cálculo da pressão do gás NH3, considerando comportamento ideal: pideal x 5,0 =


8,06 x 10−2 x 0,08206 x (100 + 273)

pideal = 0,493 atm

V 5,0
Cálculo do volume molar: Vmolar = = = 62,04 L. mol−1
n 8,06 x 10−2

a
Cálculo da pressão, considerando o gás nitrogênio com comportamento real: {preal + }x {Vmolar − b} =
V2molar

RxT

4,169
{preal + } x {62,04 − 3,710 x 10−2 } = 0,08206 x (100 + 273)
(62,04)²

preal = 0,49 atm

O gás apresenta comportamento ideal uma vez que as pressões tanto no ideal como no real são iguais.

Questão 273 – (IME) Borbulha-se oxigênio por meio de uma coluna de água e, em seguida, coletam-se 100 cm³ do
gás úmido a 23°C e 1,06 atmosferas. Sabendo que a pressão de vapor da água a 23°C pode ser considerada igual a
0,03 atmosferas, calcule o volume coletado de oxigênio seco nas CNTP.

Resolução: Pressão de O2 = 1,06 atm – 0,03 atm = 1,03 atm

Cálculo da quantidade do número de mol de oxigênio coletado: 1,03 x 1,0 = nO2 x 0,08206 x (23 + 273)

1,03 = nO2 x 0,08206 x 300

1,03
nO2 = = 4,24 x 10−3 mol
0,08206 x 300

Cálculo do volume de oxigênio coletado medido nas Condições Normais de Temperatura e Pressão (CNTP):
pideal x VO2 = nO2 x R x T

135
1600 FÍSICO-QUÍMICA APLICADA EXERCÍCIOS COMENTADOS - IME – ITA – OLIMPÍADA

1,0 x VO2 = 4,24 x 10−3 x 0,08206 x (23 + 273)

VO2 = 0,095 L (95 cm3 )

Questão 278 – (IME) Num reator selado de 1,5 litros, sob vácuo, um certo volume de um composto orgânico, tóxico
e volátil, de peso molecular 126, foi aquecido até 600 K. Nesta temperatura, metade do composto original se
decompôs, formando monóxido de carbono e cloro. Se a pressão final no recipiente foi de 32,8 atm, determine:

a) a fórmula estrutural plana do composto orgânico original;


b) o número inicial de moléculas do composto orgânico.
Dados: Constante de Avogadro = 6,02 × 1023 e Constante dos Gases = 0,082 L x atm x mol–1 x K–1.

Resolução:

Item a) Determinação da fórmula molecular do composto orgânico: Através da questão, a reação de decomposição
do composto orgânico resulta na formação do monóxido de carbono e cloro, ambos na fase gasosa.

Equação química: COCl2(g) → CO(g) + Cl2(g)

Logo, temos: (CO)xCl2(g) → x CO(g) + Cl2(g)

Determinação de x: x + 2 x (35,5) = 126

x = 55 g.mol-1

Levando em consideração que a molécula apresenta dois átomos de oxigênio, temos:

<MM> = 55 g.mol-1 – 2 x 16 g.mol-1 = 55 – 32 = 23 g.mol-1

23
Cálculo do número de átomos de carbono: (12 ) ≅ 2

Equação química balanceada da decomposição: C2O2Cl2(g) → 2 CO(g) + Cl2(g)

Item b) Base de cálculo: n (mol) inicial de C2O2Cl2(g).

Para a resolução do problema será necessário levar em consideração a utilização da tabela de equilíbrio químico com
grau de dissociação de 50% (α).

C2O2Cl2(g) 2 CO(g) Cl2(g)


Início n 0 0
Reage nα 2nα nα
Equilíbrio n - nα 2nα nα

Cálculo do número de mol total da mistura gasosa: nTOTAL = nC2O2Cl2l + nCO + nCl2

nTOTAL = n - nα + 2nα + nα = n(1 + 2α)

136
1600 FÍSICO-QUÍMICA APLICADA EXERCÍCIOS COMENTADOS - IME – ITA – OLIMPÍADA

Para um grau de dissociação igual a 50%, temos: nTOTAL = n(1 + 2 x 0,50) = 2n

Cálculo do número de mol do C2O2Cl2(g): ptotal x VO2 = ntotal x R x T

32,8 x 1,50 = 2n x 0,08206 x 600

n = 0,50 mol

6,02 x 1023 moléculas


Cálculo do número de moléculas de C2O2Cl2: Moléculas = 0,50 mol x ( 1 mol
) =

3,10 x 1023 moléculas

Questão 279 – (IME) Uma amostra de gás monoatômico desconhecido ocupa um volume de 230 cm³, a 300 K e 1
atm. Sabendo-se que os átomos na amostra ocupam uma fração de volume de 2.10-4, calcule o raio dos átomos do
gás.

Resolução: Dados do problema: Vátomos = 2,0 x 10-4 x Vrecipiente

Sabendo que o volume do recipiente é igual a 230 cm³, o volume dos átomos será igual a:

Vátomos = 2,0 x 10-4 x 0,230 = 0,460 x 10-4 dm³

Cálculo do número de mol de átomos do gás monoatômico, considerando comportamento ideal:

1,0 x 0,230 = nO2 x 0,08206 x 300

nO2 = 9,34 x 10−3 mol

0,460 x 10−4 dm³


Cálculo da constante de van der Waals (b): b = 9,34 x 10−3 mol
= 4,92 x 10−3 dm³. mol−1

b
Cálculo do raio do gás monoatômico: V = N

4πR³ b
Considerando que o gás apresenta configuração esférica, temos: 3
=N

3 3b
R = √4πN

3 3 x 4,92 x 10−3 3 −9
R= √ = √1,95 x 10−27 = 1,25 x 10
⏟ metros = 1,25 nm
4 x 3,14 x 6,02 x 1023 nanômetro

Questão 280 – (IME) Qual a massa que um balão contendo 10,000 L a 25°C e 1 atm pode suportar mantendo-se
com o poder ascensional igual a zero, ou seja, em equilíbrio com o ar?

m
Resolução: Analisando o balão com a presença do gás hélio: pideal x V = <MM>
He
xRxT
He

pideal x V x < MM >He 1x 1000 x 4,0


mHe = = = 1635,73 g
RxT 0,08206 x (25 + 273)

137
1600 FÍSICO-QUÍMICA APLICADA EXERCÍCIOS COMENTADOS - IME – ITA – OLIMPÍADA

m
Analisando o balão com a presença de ar atmosférico: pideal x V = <MM>
ar
xRxT
ar

pideal x V x < MM >ar 1x 1000 x 28,8


mar = = = 11777,27 g
RxT 0,08206 x (25 + 273)

Para que o processo fique em equilíbrio: ∆massa = massaar – massahélio

∆massa = 11777,27 – 1635,73 = 10141,5 g (10,14 kg)

Questão 281 – (GRILLO) A partir das constantes de van der Waals do gás nitrogênio, a = 1,390 atm.L².mol-2 e b =
0,0391 L.mol-1, e considerando que esta amostra gasosa encontra-se a 0°C confinado em um recipiente de 62,30
mililitros, calcule:

a) A pressão calculada pela equação dos gases ideais, para 1 mol;


b) A pressão calculada pela equação dos gases reais (van der Waals), para 1 mol;
c) A diferença percentual entre os dois valores obtidos, a partir da equação dos gases reais;
d) O valor do raio da molécula gasosa de nitrogênio;
e) A temperatura de Boyle.

Resolução: Item a) Cálculo da pressão, considerando com comportamento ideal: pideal x (62,30 x 10−3 ) =
1 x (0,08206) x (0 + 273)

pideal = 359,59 atm

1,390 62,30 x 10−3


Item b) Cálculo da pressão, considerando com comportamento ideal: {preal + 62,30 x 10−3
}x ( )−
( )² 1
1

0,0391 = 0,08206 x (0 + 273)


1,390 0,08206 x 273
{preal + }=
−3
(62,30 x 10 )² 62,30 x 10−3 − 0,0391

0,08206 x 273 1,390


preal = − = 607,49 atm
62,30 x 10 − 0,0391 (62,30 x 10−3 )2
−3

607,49−359,59
Item c) Cálculo da diferença percentual, a partir da equação dos gases reais: Erro = 607,49
= 0,4080 (≅

41%)
4πR³ b
Item d) Cálculo do raio da molécula gasosa de nitrogênio: 3
=N

4 x 3,14 x R³ 0,0391
=
3 6,02 x 1023

R³ = 1,55 x 10−26

3
R = √1,55 x 10−26 = 2,49 x 10−9 = 2,49 nm

138
1600 FÍSICO-QUÍMICA APLICADA EXERCÍCIOS COMENTADOS - IME – ITA – OLIMPÍADA

N a 1,390
Item e) Cálculo da temperatura de Boyle: TBOYLE
2
= = = 433,22 K
bxR 0,0391 x 0,08206

Questão 282 - (GRILLO) Consultando a tabela das constantes de Van der Waals, determine a temperatura de Boyle
e o raio das seguintes moléculas gasosas:

a) Cloro
b) Nitrogênio
c) Oxigênio
d) Argônio

Resolução: Organizando as constantes de van der Waals para as seguintes moléculas na tabela apresentada a
seguir:

a (atm.L².mol-2) b (10-2.L.mol-1)
Cl2(g) 6,579 5,622
N2(g) 1,408 3,913
O2(g) 1,378 3,183
Ar(g) 1,363 3,219

4πR³ b
a) Analisando o cálculo do raio para o cloro gasoso: =
3 N

4 x 3,14 x R³ 5,622 x 10−2


=
3 6,02 x 1023

3
R = √2,23 x 10−26 = 2,81 x 10−9 m = 2,81 nm

Cl a 6,579
Temperatura de Boyle para o cloro gasoso: TBOYLE
2
= b x R = 5,622 x 10−2 x 0,08206 = 1426,06 K

4𝜋𝑅³ 𝑏
b) Analisando para o cálculo do raio do nitrogênio gasoso: 3
=𝑁

4 x 3,14 x R³ 3,913 x 10−2


=
3 6,02 x 1023

3
R = √1,55 x 10−26 = 2,49 x 10−9 m = 2,49 nm

N a 1,408
Temperatura de Boyle para o nitrogênio gasoso: TBOYLE
2
= = = 438,49 K
bxR 3,913 x 10−2 x 0,08206

4𝜋𝑅³ 𝑏
c) Analisando o cálculo do raio para o oxigênio gasoso: 3
=𝑁

4 x 3,14 x R³ 3,183 x 10−2


=
3 6,02 x 1023

3
R = √1,26 x 10−26 = 2,33 x 10−9 m (2,33 nm)

O a 1,378
Temperatura de Boyle para o oxigênio gasoso: TBOYLE
2
= b x R = 3,183 x 10−2 x 0,08206 = 527,57 K

139
1600 FÍSICO-QUÍMICA APLICADA EXERCÍCIOS COMENTADOS - IME – ITA – OLIMPÍADA

4πR³ b
d) Analisando o raio para o argônio gasoso: 3
=N

4 x 3,14 x R³ 3,219 x 10−2


=
3 6,02 x 1023

3
R = √1,277 x 10−26 = 2,34x 10−9 m = 2,34 nm

Ar a 1,363
Temperatura de Boyle para o argônio gasoso: TBOYLE = = = 515,99 K
bxR 3,183 x 10−2 x 0,08206

Questão 283 – (GRILLO) Em um laboratório de síntese de fármacos há um reator que é preenchido com 20 kg de
Argônio a uma temperatura de 3000C. Sabendo que o volume do reator é de 2 m3, determine:

a) a pressão no reator, considerando o Ar como gás perfeito;


b) a pressão no reator, considerando o Ar como gás real;
c) o fator de compressibilidade nestas condições;
d) o valor do raio da partícula gasosa;
e) a temperatura de Boyle. Dados para a constante de van der Waals: a = 1,363 atm.L2.mol-2; b = 3,219 x 10-2
L.mol-1.

20000
Resolução: Cálculo do número de mol de gás argônio (Ar): 𝑛𝐴𝑟 = 40
= 500 𝑚𝑜𝑙

2000
Cálculo do volume molar: Vmolar = = 4,0 L. mol−1
500

a) Cálculo da pressão pela equação de gás perfeito: 𝑝𝑖𝑑𝑒𝑎𝑙 𝑥 2000 = 500 𝑥 0,08206 𝑥 (300 + 273)
𝑝𝑖𝑑𝑒𝑎𝑙 = 11,75 𝑎𝑡𝑚

b) Cálculo da pressão com comportamento real através da equação de Van der Waals:
1,363
{𝑝𝑟𝑒𝑎𝑙 + } 𝑥 {4,0 − 3,219 𝑥 10−2 } = 0,08206 𝑥 (300 + 273)
(4)²

𝑝𝑟𝑒𝑎𝑙 = 11,76 𝑎𝑡𝑚

𝑃𝑅𝑒𝑎𝑙 11,76 𝑎𝑡𝑚


c) Cálculo do fator de compressibilidade (Z): 𝑍 = 𝑃𝐼𝑑𝑒𝑎𝑙 = 11,75 𝑎𝑡𝑚
≈ 1,00 𝑎𝑡𝑚
4𝜋𝑅³ 𝑏
d) Considerando que a partícula gasosa apresenta uma configuração esférica, temos: 3
=𝑁

4 x 3,14 x R³ 3,219 x 10−2


=
3 6,02 x 1023

R³ = 1,28 x 10−26

3
R = √1,28 x 10−26 = 2,33 x 10−9 m (2,33 nm)

Ar a 1,363
e) Cálculo da temperatura de Boyle: TBOYLE = = = 516 K
bxR 3,219 x 10−2 x 0,08206

140
1600 FÍSICO-QUÍMICA APLICADA EXERCÍCIOS COMENTADOS - IME – ITA – OLIMPÍADA

Questão 284 – (GRILLO) Três recipientes com volume de 0,2 litros cada um e temperatura de aproximadamente
igual a 25°C, contém em cada recipiente três gases diferentes, apresentando respectivamente três valores de
pressão, atm, 6 atm e 8 atm, são misturados por meio da abertura de duas válvulas. Calcule a pressão do sistema e
as pressões parciais dos gases na mistura.

pV 4 x 0,2 0,80
Resolução: Cálculo do número de mol (n1) para o balão de pressão 4 atm: n1 = RT = RT
= RT

pV 6 x 0,2 1,20
Cálculo do número de mol (n2) para o balão de pressão 6 atm: n2 = RT = RT
= RT

pV 8 x 0,2 1,60
Cálculo do número de mol (n3) para o balão de pressão 8 atm: n3 = RT = RT
= RT

0,80 1,20 1,60 3,60


Cálculo do número de mol total (nT): nTotal = RT
+ RT + RT = RT

3,6
nT x R x T ( )xRxT
Cálculo da pressão total (pT): pT = VT
= RxT
0,60
= 6,0 atm

0,80
Cálculo da pressão parcial do gás 1: p1 = X1 x pTOTAL = ( RXT
3,6 ) x 6,0 atm = 1,33 atm
RXT

1,20

Cálculo da pressão parcial do gás 2: p2 = X 2 x pTOTAL = ( RXT


3,6 ) x 6,0 atm = 2,00 atm
RXT

1,60
Cálculo da pressão parcial do gás 3: p3 = X 3 x pTOTAL = ( RXT
3,6 ) x 6,0 atm = 2,66 atm
RXT

Questão 285 – (GRILLO) A uma temperatura de aproximadamente igual a 503 K e sob pressão do sistema igual a
0,50 atm, o cloreto de nitrosila dissocia-se conforme a equação química não balanceada a seguir: NOCl(g)  NO(g) +
Cl2(g). Quando o equilíbrio é atingido, a pressão parcial do cloro na mistura gasosa é igual a 0,07 atmosferas. A partir
destas informações, calcule:

a) o grau de dissociação (α);


b) a composição do monóxido de nitrogênio;
c) a composição do cloreto de nitrosila;
d) a densidade da mistura gasosa.

Resolução: Base de cálculo: n (mol) inicial de cloreto de nitrosila. Utilização da tabela de equilíbrio químico:
2 NOCl(g) → 2 NO(g) Cl2(g)
Início n 0 0
Reage 2nα 2nα nα
Equilíbrio n - 2nα 2nα nα

Cálculo do número de mol total da mistura gasosa: nT = nNOCl + nNO + nCl2 = n - 2nα + 2nα + nα = n + nα = n.(1 + α)

a) Cálculo da pressão parcial do cloro gasoso: pCl2 = X Cl2 x pTOTAL

141
1600 FÍSICO-QUÍMICA APLICADA EXERCÍCIOS COMENTADOS - IME – ITA – OLIMPÍADA

pCl2 0,07
X Cl2 = = = 0,14
pTOTAL 0,50

Sabendo que a fração molar do cloro gasoso é a razão do número de mol de cloro pelo número de mol total, temos:
nCl2
X Cl2 = n
TOTAL


Pela tabela de equilíbrio, o gás cloro no equilíbrio é igual a nα, então: X Cl2 =
n x (1+ α)

𝛼
0,14 =
(1 + 𝛼)

𝛼 = 0,163

nNO 2nα 2 x 0,163


b) Cálculo da composição do monóxido de nitrogênio: X NO = n = n(1+ α) = (1+ 0,163) =
TOTAL

0,280 (28%)
n n x (1−2α) (1−2 x 0,163)
c) Cálculo da composição do cloreto de nitrosila: X NOCl = n NOCl = n x (1+ α)
= (1+0,163)
=
TOTAL

0,58 (58%)
p x<MM> 0,50 x65,5
d) Cálculo da densidade da mistura gasosa: d = (1+ α)x R x T = (1+0,163)x 0,08206 x (230+273) = 0,68 g. L−1

Questão 286 – (OLIMPÍADA DE QUÍMICA DO RIO DE JANEIRO) Um cilindro de GNV, com capacidade de 7,0 m³,
foi enchido com uma mistura de metano e etano. A mistura dentro do cilindro se encontra a 22,0 atm e a uma
temperatura de 27°C. A pressão parcial do etano na cilindro é de 16,4 atm. Considere o comportamento ideal.

a) Indique o número de moléculas de metano contidas no cilindro.


b) Qual seria o volume da mistura gasosa, em m³, se a mesma se encontrasse nas condições de 1,0 atm e
0,0°C?
c) Qual a massa de CO2, em toneladas emitida pela combustão total do etano contido no cilindro? Observação:
considerar combustão completa.
Resolução: Cálculo da pressão parcial de gás metano: PTOTAL = PC2H6 + PCH4
PCH4 = 22,0 – 16,4 = 5,60 atm

Cálculo do número de mol total da mistura gasosa: 22 x 700 = nT x 0,08206 x (27 + 273)

nTotal = 625,56 mol

Cálculo do número de mol de gás metano, a partir da pressão parcial: pCH4 = X CH4 x pTOTAL

nCH4 PCH4
=
nCH4 pTOTAL

nCH4 5,6
=
625,55 22,4

142
1600 FÍSICO-QUÍMICA APLICADA EXERCÍCIOS COMENTADOS - IME – ITA – OLIMPÍADA

nCH4 = 156,39 mol

6,02 x 1023 moléculas


Item a) Cálculo do número de moléculas de gás metano: nCH4 = 156,39 mol x =
1 mol

9,41 x 1025 moléculas

Item b) Cálculo do volume da mistura gasosa, a partir da utilização da equação combinada dos gases ideais, temos:
pI x VI pII x VII
TI
= TII

22 x 7 1 x VII
=
300 273

VII = 140,14 m³ (Volume da mistura gasosa)

Cálculo do número de mol de etano, a partir da equação da pressão parcial: pC2 H6 = X C2 H6 x pTOTAL

nC2 H6
16,4 = x 22
625,55

nC2 H6 = 466,32 mol

Equação química da combustão completa do gás etano: C2H6(g) + 7/2 O2(g) → 2 CO2(g) + 3 H2O(g)

1 mol de C2H6 ---------------- 2 mol de CO2

466, 32 mol ------------------- nCO2

nCO2 = 932,64 mol

Item c) Cálculo da massa de dióxido de carbono, a partir da equação do número de mol: mCO2 = nCO2 x <MM>CO2 =
932,64 x 44 = 41036 g (41,36 kg)

Questão 287 – (ITA) Um reator de volume constante continha, inicialmente, 361 g de uma mistura gasosa constituída
por um alcano e um éter, ambos de massa molecular 58, a 398 K e 1,47 atm. Neste reator, injetou-se uma quantidade
de oxigênio correspondente ao dobro do mínimo necessário para realizar a combustão completa. Após a reação de
combustão, a mistura final foi resfriada até a temperatura inicial, atingindo uma pressão de 20,32 atm. Supondo
combustão completa, calcule a composição molar da mistura original.

Resolução: Fórmula molecular geral do hidrocarboneto alcano: CnH2n + 2 → Massa molecular: 14n + 2 = 58

n=4

Para n = 4, o hidrocarboneto é o alcano denominado butano (C4H10).

Fórmula molecular geral do Éter: CnH2n + O 2 (supondo ligações simples e cadeia aberta)

14 x n + 2 + 16 = 58

143
1600 FÍSICO-QUÍMICA APLICADA EXERCÍCIOS COMENTADOS - IME – ITA – OLIMPÍADA

40
n = 12 não resulta em um número inteiro, logo esta suposição é falsa.

* CnH2n + O 2 (supondo ligação dupla ou cadeia fechada)

14 x n + 16 = 58

n = 3, logo suposição verdadeira. Fórmula molecular: C3H6O

Considerando nC4H10 = x e nC3H6O = y, temos as equações de combustão:

13
x C4H10 + O2 → 4x CO2 + 5x H2O
2

y C3H6O + 4y O2 → 3y CO2 + 3y H2O

Pelo balanceamento das equações de combustão, teremos:

13𝑥
*nO2 necessário = 2
+ 4𝑦

13𝑥 13𝑥
Logo nO2 colocado = ( + 4𝑦) 𝑥 2, sendo ( + 4𝑦) o número de mol de O2 em excesso:
2 2

nCO2 = 4x + 3y e nH2O= 5x + 3y

No recipiente antes da injeção de O2 temos apenas o alcano e o éter, logo:

𝑚1 𝑚2 𝑚1 + 𝑚2
nTOTAL = nC4H10 + nC3H6O = 58
+ 58
= 58
. Sendo m1 + m2 = mTOTAL = 361g

𝑚1 + 𝑚2 361
nTOTAL = 58
= 58
=𝑥+𝑦

nxRxT 361 x R x 398


Cálculo de volume do recipiente: V = =
pideal pideal

p 20,32 361 x R x 398


Cálculo do número de mol ao final do processo: ntotal = xV = x = 86,04 mol
RxT 398 x R 58 x 1,47

nTOTAL = nO2(excesso) + nCO2 + nH2O = 86

13𝑥
2
+ 4y + 4x + 3y + 5x + 3y = 86 → 15,5x + 10y = 86

361
x+y= 58

y = 6,22 – x (Equação 1)

15,5x + 10y = 86 (Equação 2)

Substituindo a equação 1 na equação 2, temos:

15,5x + 10.(6,22 – x) = 86

15,5x – 10x = 86 – 62,2

144
1600 FÍSICO-QUÍMICA APLICADA EXERCÍCIOS COMENTADOS - IME – ITA – OLIMPÍADA

23,8
x= 5,5
= 4,33 mol de C4H10.

y = 6,22 – 4,33 → y = 1,89 mols de C3H6O

Resposta: 4,33 mol de alcano e 1,89 mol de éter.

Questão 288 – (ITA) Uma chapa de ferro é colocada dentro de um reservatório contendo solução aquosa de ácido
clorídrico. Após um certo tempo observa-se a dissolução do ferro e formação de bolhas gasosas sobre a superfície
metálica. Uma bolha gasosa, de massa constante e perfeitamente esférica, é formada sobre a superfície do metal a
2,0 metros de profundidade. Calcule:

a) O volume máximo dessa bolha de gás que se expandiu até atingir a superfície do líquido, admitindo-se que
a temperatura é mantida constante e igual a 25°C e que a base do reservatório está posicionada ao nível do mar.
b) A massa de gás contida no volume de expansão da bolha. Sabe-se que no processo corrosivo a formação
de bolha de gás foram consumidos 3,0 x 1015 átomos de ferro. Dado: massa específica da solução aquosa de HCl é
igual a 1020 kg.m-3na temperatura de 25°C.

Resolução: Cálculo do número de mol de hidrogênio gasoso: Fe(s) + 2 HCl(aq) → FeCl2(aq) + H2(g)

1 mol de Fe ------------------------ 1 mol de H2

6 x 1023 átomos de Fe ------------ 1 mol de H2

3 x 1015 átomos de Fe ------------ nH2

nH2 = 5,0 x 10-9 mol

Cálculo da massa de hidrogênio gasoso: mH2 = 5,0 x 10-9 x 2 = 10-8 g

nxRxT
Cálculo do volume do gás de hidrogênio, utilizando a equação dos gases ideais: VH2 = =
pideal

5,0 x 10−9 x 0,08206 x (25+273)


1,0
= 1,22 x 107 L

Questão 289 – (ITA) Estime a massa de ar contida em uma sala de aula. Indique claramente quais as hipóteses
utilizadas e os quantitativos estimados das variáveis empregadas.

Resolução: Considerando que a sala apresente as seguintes dimensões: Comprimento da sala = 10 metros; Largura
da sala = 10 metros; Altura da sala = 3 metros e volume da sala = 10 x 10 x 3 = 300 m³ (300000 litros)

Cálculo da massa molar do ar, considerando que a composição do ar seja de 20% de O2 e 80% de N2: {Xar x <MM>ar}
= {XO2 x <MM>O2} + {XN2 x <MM>N2}

1,0 x <MM>ar = {0,20 x 32,0} + {0,80 x 28,0}

<MM>ar = 28,80 g.mol-1

145
1600 FÍSICO-QUÍMICA APLICADA EXERCÍCIOS COMENTADOS - IME – ITA – OLIMPÍADA

Considerando que a pressão esteja a uma atmosfera 1 atm e que a temperatura seja igual a 25°C, temos:
mar
1 x 3,0 x 105 = ( ) x 0,08206 x (25 + 273)
28,80

mar = 353318,2 g (353,32 kg)

Questão 290 – (IME) Um balão de material permeável às variedades alotrópicas do oxigênio é cheio com ozônio e
colocado em um ambiente de oxigênio à mesma pressão e igual temperatura do balão. Responda, justificando
sumariamente: o balão se expandirá ou se contrairá?
Resolução: Segundo a Lei de Graham, quanto maior a massa molar, menor será a velocidade de efusão, com isso,
teremos a seguinte relação: VO3 < VO2. Desta forma, o balão irá se expandir.

1 ∂V
Questão 291 – O coeficiente de expansão térmica está definido por α = ( ) . ( ) . Aplicando a equação de
V ∂T p
estado, calcule o valor de α para um gás ideal.
Resolução: Sabendo que o gás ideal é dado pela seguinte equação matemática: pV = nRT, temos:
nRT
V= p
. Derivando em função da temperatura, a partir do momento em que o processo ocorre a pressão constante.

∂V 1
α=( ) x
∂T p v
1 nR
α= x
v p
nR 1
Sabendo que a equação de estado é dado por: pV = T, logo a equação matemática da expressão térmica é dado por:
1
α = T.
1 ∂V
Questão 292 – O coeficiente de compressibilidade 𝛽 está definido por 𝛽 = − ( ) . ( ) . Calcule o valor de 𝛽 para
V ∂p T
um gás ideal.
Resolução: Sabendo que o gás ideal é dado pela seguinte equação matemática: pV = nRT, temos:
nRT
V= p
. Derivando em função da temperatura, a partir do momento em que o processo ocorre a temperatura
constante.
∂V −n x R x T 1 nRT
(∂T) = p²
, em que 𝛽 = − v x (− p²
)
T
nRT
Sabendo que a equação de estado é dado por: pV
= 1, logo a equação matemática da expressão térmica é dado
1
por: 𝛽 = p.

∂p
Questão 293 – Expressar a derivada (∂T) de um gás em função de α e β.
V

∂p nR
Resolução: Da equação de estadão se obtém: ∂T = V
, mas como temos a seguinte relação termodinâmica a partir
nR 𝑃 ∂p 1 α
da equação dos gases ideais, V
= 𝑇 , temos: (∂T) = 𝑇 . 𝑝 = 𝛽 .
V

146
1600 FÍSICO-QUÍMICA APLICADA EXERCÍCIOS COMENTADOS - IME – ITA – OLIMPÍADA

Questão 294 – Expressar a pressão parcial de uma mistura gasosa em função das concentrações das quantidades
de matéria.
Resolução: A partir da equação dos gases ideais, temos: pV = nRT e sabendo que a concentração da quantidade
concentração n
de matéria é a razão entre o número de mol pelo volume ([ ] = V), logo:
molar
Para um gás 1: p1 x V = n1 x R x T
n1
p1 = xRxT
V
concentração
p1 = [ ]x R x T
molar
Então, a relação da pressão parcial do gás 1 pela concentração da quantidade de matéria é dada pela seguinte
concentração
equação matemática: p1 = [ ]x R x T .
molar

Questão 295 – (GRILLO) Considere a reação de dissociação do tetróxido de dinitrogênio, representada pela seguinte
equação química: N2 O4(g) ⇌ 2 NO2(g) , sabendo que a constante de equilíbrio em função das pressões parciais
seja igual a 0,115 a 25°C. Sendo n o número de mol de moléculas de N2 O4(g) que estariam presentes, determine o
volume molar (V/n) a três pressões distintas, (a) 2,0 atm; (b) 1,0 atm e (c) 0,50 atm, levando em consideração que a
mistura gasosa em seu estado de equilíbrio apresentem comportamento ideal.
Resolução: Para a resolução deste problema será necessário utilizar a tabela de equilíbrio químico, sabendo que a
base de cálculo é igual a n(mol) inicial de N2 O4(g) .

Tabela de equilíbrio químico:


N2 O4(g) (mol) ⇄ 2 NO2(g) (mol)
Início n 0
Reage nα 2nα
Equilíbrio n - nα 2nα

Cálculo do número de mol total: ntotal = n − nα + 2nα = n + nα


Determinação das pressões parciais:
n x (1 − α) (1 − α)
pN2 O4 = x ptotal = xp
n x (1 + α) (1 + α) total
2nα 2α
p𝑁O2 = x ptotal = xp
n x (1 + α) (1 + α) total
p2NO2
Expressão da constante de equilíbrio em função das pressões parciais: K p = p
N2 O4

Relação da constante de equilíbrio em função das pressões parciais (Kp = 0,115) com o grau de dissociação (α):
4α²
Kp = xp = 0,115
1 − α² total

4α2 𝑥 ptotal = 0,115 − 0,115α²

Cálculo do grau de dissociação considerando que a pressão seja igual a 2,0 atm:

147
1600 FÍSICO-QUÍMICA APLICADA EXERCÍCIOS COMENTADOS - IME – ITA – OLIMPÍADA

4α2 𝑥 2,0 = 0,115 − 0,115 x α²

0,115
α=√ = 0,119
8,115

Cálculo do volume molar para uma pressão igual a 2,0 atm: ptotal x V = ntotal x R x T
ptotal x V = n x (1 + α) x R x T
V
ptotal x 𝑛 = (1 + α) x R x T
V
2,0 x 𝑛 = (1 + 0,119) x 0,08206 x 298

V L
= 13,68
n mol
Cálculo do grau de dissociação considerando que a pressão seja igual a 1,0 atm:
4α2 𝑥 1,0 = 0,115 − 0,115 x α²

0,115
α=√ = 0,167
4,115

Cálculo do volume molar para uma pressão igual a 1,0 atm: ptotal x V = ntotal x R x T
ptotal x V = n x (1 + α) x R x T
V
1,0 x 𝑛 = (1 + 0,167) x 0,08206 x 298

V L
= 28,54
n mol
Cálculo do grau de dissociação considerando que a pressão seja igual a 0,50 atm:

4α2 𝑥 0,50 = 0,115 − 0,115 x α²

0,115
α=√ = 0,233
2,115

Cálculo do volume molar para uma pressão igual a 0,50 atm: ptotal x V = ntotal x R x T
ptotal x V = n x (1 + α) x R x T
V
0,50 x 𝑛 = (1 + 0,233) x 0,08206 x 298

V L
= 60,30
n mol

148
1600 FÍSICO-QUÍMICA APLICADA EXERCÍCIOS COMENTADOS - IME – ITA – OLIMPÍADA

Questão 296 – (GRILLO) Considerando a mistura gasosa do exercício 53, mostre que a pressão se aproxima a zero
(p→0) e com isso o fator de compressibilidade se aproxima igual a dois (Z→2).
Resolução: Sabendo que a equação dos gases reais em função do fator de compressibilidade é dado pela seguinte
equação matemática:
ptotal x V = Z x ntotal x R x T
ptotal x V = Z x n x (1 + α) x R x T
ptotal x V
=Z
n x (1 + α) x R x T
ptotal x V ptotal x V
Z=n xRxT
, então: n xRxT
= (1 + α)
total total

Logo: Z = 1 + α
Analisando a pressão total tendendo a zero (p→0), a partir da expressão da constante de equilíbrio em função das
4α2 x ptotal
pressões parciais (Kp): K p =
1−α²

4α2 𝑥 0
Kp =
1 − α²
0
Kp =
1 − α²
(1 − α2 ) 𝑥 𝐾𝑝 = 0

(1 − α²) = 0

α=1
Logo, para uma pressão total tendendo a zero, o grau de dissociação tende a um (α→1). Diante desta análise, o grau
de dissociação tende a dois, pois:
Z→1+α
Z→1+1
Z→2
Questão 297 – Certo gás a uma temperatura de 0°C e pressão igual a uma atmosfera, apresenta fator de
compressibilidade igual a 1,0054. A partir desta informação, calcule o valor de b da constante de van der Waals.

Resolução: Pela equação apresentada por Gilbert Castellan (Fundamentos de Físico-Química), a relação matemática
bxp
entre Z e b é a seguinte: Z = 1 + .
RxT

b x 1,0
Substituindo os valores: 1,0054 = 1 + 0,08206 x (0+273)

L
b = 0,121
mol

Questão 298 – Considere que o fator de compressibilidade seja igual a 2,0064 a 27°C e 1,20 atm. Sabendo que a
temperatura de Boyle é igual a 177 K, calcule as constantes de van der Waals a e b.

149
1600 FÍSICO-QUÍMICA APLICADA EXERCÍCIOS COMENTADOS - IME – ITA – OLIMPÍADA

Resolução: Pela equação apresentada por Gilbert Castellan (Fundamentos de Físico-Química), a relação matemática
bxp b x 1,20
entre Z e b é a seguinte: Z = 1 + R x T. Substituindo os valores: 2,0064 = 1 + 0,08206 x (27+273)

L
b = 20,64
mol
a
Cálculo da constante de van der Waals, a partir da equação da temperatura de Boyle: TBoyle = b x R.

a
Substituindo os valores: 177 = 2,64 x 0,08206

atm x L²
a = 299,79
mol²

Questão 299 – (IME) No equipamento esquematizado na figura abaixo, as torneiras A, B e C estão inicialmente
fechadas. O compartimento 1 de volume 2,00 L contém oxigênio sob pressão de 1,80 atm. 0 compartimento 2 contém
nitrogênio. O compartimento 3 de volume 1,00 L contém nitrogênio e uma certa quantidade de sódio metálico.
Executam-se, então, isotermicamente, as três operações descritas a seguir:

1ª) Mantendo a torneira A fechada, abrem-se B e C e faz-se o vácuo nos recipientes 2 e 3, sem alterar a
quantidade de sódio existente em 3;
2ª) Fecham-se B e C e abre-se A, constatando que, após atingir o equilíbrio, o manômetro M1 indica uma pressão
de 1,20 atm;
3ª) Fecha-se A e abre-se B, verificando que, atingido o equilíbrio, o manômetro M2 indica uma pressão de 0,300
atm.
Finalmente, fecha-se a torneira B e eleva-se a temperatura do recipiente 3 até 77,0°C, quando então, a pressão
indicada por M2 é de 0,400 atm. Calcule a massa inicial de sódio, considerando que, antes da elevação da temperatura,
todo o sódio se transformara em óxido de sódio, e que os volumes das tubulações e dos sólidos (sódio e seu óxido)
são desprezíveis.

Resolução: Através da figura dada pelo problema, iremos colocar os dados na própria figura:

Situação inicial:

150
1600 FÍSICO-QUÍMICA APLICADA EXERCÍCIOS COMENTADOS - IME – ITA – OLIMPÍADA

Com a abertura da válvula A, fechando as torneiras B e C após a evacuação de gás nitrogênio, temos:

1,8 atm ---------- 1

1,2 atm ---------- 1/V2

1,2 x (2 + V2) = 1,8 x 2

1,2 x (2 + V2) = 3,6

V2 = 1 L

Após a situação 1 e 2, o diagrama ficará da seguinte maneira:

Após a situação 3, o diagrama ficará da seguinte maneira:

pI x VI pII x VII
Cálculo da temperatura antes do aquecimento: =
TI TII

0,30 x 1 0,40 x 1,0


=
TI 350

TI = 262,50 K

Observação: Como as operações 1,2 e 3 ocorrem de forma isotérmica, temos: 1,2 x 1 = n x 0,08206 x 262,5

n = 0,057 mol

151
1600 FÍSICO-QUÍMICA APLICADA EXERCÍCIOS COMENTADOS - IME – ITA – OLIMPÍADA

Cálculo de oxigênio, após a reação com sódio: 0,40 x 1 = n x 0,08206 x 350

n = 0,0139 mol

Logo, a quantidade de oxigênio que sofre reação é igual a: 𝑛𝑂2 = 0,056 − 0,0139 = 0,042 𝑚𝑜𝑙

Através da equação química, temos: 2 Na(s) + ½ O2(g) → Na2O(s)

2 mol de Na --------------- ½ mol de O2


46 gramas de Na ---------- ½ mol de O2
mNa -------------------------- 0,042 mol de O2
mNa = 3,86 g de sódio metálico

Questão 301 – (IME) Na figura abaixo, o cilindro A de volume VA contém um gás inicialmente a pressão P0 e encontra-
se conectado, através de uma tubulação dotada de uma válvula (1), a um vaso menor B de volume V B, repleto do
mesmo gás a uma pressão p tal que P0 > p > Patm, onde Patm é a pressão atmosférica local. Abre-se a válvula 1 até
que a pressão fique equalizada nos dois vasos, após o que, fecha-se esta válvula e abre-se a válvula 2 até que a
pressão do vaso menor B retorne ao seu valor inicial p, completando um ciclo de operação. Sabendo-se que o sistema
é mantido a uma temperatura constante T, pede-se uma expressão para a pressão do vaso A após N ciclos.

B
2 1

Resolução: Vamos considerar as seguintes nomenclaturas:

pAi = pressão no cilindro A após o ciclo i

ni = número total de mol em A + B depois do ciclo i

nAi = número de mol do cilindro A depois do ciclo i

pxV p0 x VA
Analisando o ciclo 1: n1 = ( R x TB ) + ( RxT
)

VA
Sabendo que nA = n1 x (V ), logo
A +VB

nA1 x R x T p x VB + p0 x VA p −p
PA1 = ( VA
) = ( VA +VB
) = p + V 0+V x (VA )
A B

pxV pAi x VA
Analisando o ciclo 2: n2 = ( R x TB ) + ( RxT
)

152
1600 FÍSICO-QUÍMICA APLICADA EXERCÍCIOS COMENTADOS - IME – ITA – OLIMPÍADA

( p0 −p ) x VA
V n xRxT p x VB + (p+ )
VA +VB
Sendo nA2 = n2 x ( A ), logo PA2 = ( A2 ) =
VA +VB VA VA +VB

(p0 − p)x VA2


PA2 = p +
(VA + VB )²

pxV pAi x VA
Analisando o ciclo 3: n3 = ( R x TB ) + ( RxT
)

( p0 −p ) x VA
V n xRxT p x VB + (p+ )
VA +VB
Sendo nA2 = n2 x ( A ), logo PA2 = ( A2 ) =
VA +VB VA VA +VB

(p0 − p)x VA2


PA2 = p +
(VA + VB )²

p x VB pA2 x VA
n2 = +
RT RT
VA
Sendo nA3 = n3 x
VA + VB

( p0 −p ) x V2
A)
p x VB + (p+
nA3 x R x T (VA +VB )2
Logo: PA3 = ( ) =
VA VA +VB

(p0 − p)x VA3


PA3 = p +
(VA + VB )3

(𝑝0 − 𝑝)𝑥 𝑉𝐴𝑁


Analisando para um ciclo N: 𝑃𝐴𝑁 = 𝑝 + (𝑉𝐴 +𝑉𝐵 )𝑁

Questão 302 – (ITA) Dois frascos, A e B, contém soluções aquosas concentradas em HCl e NH3, respectivamente.
Os frascos são mantidos aproximadamente a um metro de distância entre si, à mesma temperatura ambiente. Aberto
os frascos, observa-se a formação de um aerossol branco entre os mesmos. Descreva o fenômeno e justifique por
que o aerossol branco se forma em uma posição mais próxima a um dos frascos do que ao outro.

Resolução: O HCl vaporiza naturalmente com o recipiente aberto tal qual a amônia que deixa a solução na forma
gasosa. Como os dois gases estão no mesma temperatura, a seguinte relação é válida:

HCl(g) + NH3(g) → NH
⏟ 4 Cl(s)
Sólido branco

2 2
< MM >HCl x vHCl = < MM >NH3 x vNH 3

MMHCl > MMNH3 → VNH3 > VHCl , onde <MM> é a massa molar do composto e V é a velocidade.

Velocidade de difusão do gás clorídrico é maior que a da amônia, sendo assim, o sólido se forma mais próximo do
recipiente de HCl. A fumaça branca torna-se mais próxima do frasco que contém HCl, pois a amônia se difunde mais

153
1600 FÍSICO-QUÍMICA APLICADA EXERCÍCIOS COMENTADOS - IME – ITA – OLIMPÍADA

rapidamente já que as velocidades de difusão gasosa são inversamente proporcionais às massas molares dos gases
VNH3 M
que se difundem, sob temperatura constante, segundo a Lei de Graham. Portanto, VHCl
= √M HCl .
NH3

Questão 302 – (OLIMPÍADA DE QUÍMICA DO DISTRITO FEDERAL) Do ponto de vista histórico, o efeito das
interações intermoleculares e sua manifestação sobre o comportamento de sistemas químicos começou a mais de
dois séculos, com os experimentos pioneiros em sistemas gasosos, realizados por cientistas como Robert Boyle,
Lacques Charles, Joseph-Louis Gay Lussac e Johannes Van der Waals. A combinação das relações obtidas por
Boyle, Charles e Gay-Lussac levou a conhecida equação de estado dos gases ideias: pV = nRT, em que p é a pressão,
V é o volume, T a temperatura absoluta, n o número de mol do gás contidos no recipiente e R é a constante universal
dos gases ideais. Um gás ideal é, por definição, um sistema gasoso constituído de partículas pontuais e não
interagentes, isto é, não existe nenhuma interação entre as partículas constituintes do gás, quer sejam elas átomos
ou moléculas. A análise da equação de estado dos gases ideais nos revela algumas curiosidades. Por exemplo, a
dada pressão e temperatura, 1 mol de qualquer gás ocupará o mesmo volume. Isto é devido ao fato que a equação
dos gases ideais não contempla nenhuma informação inerente ao sistema gasoso sob estudo, o que faz com que as
propriedades termodinâmicas calculadas sejam as mesmas para qualquer gás. (Química Nova na Escola – QNEsq –
número 4 – Maio de 2001)

a) Sob quais condições de pressão e temperatura o comportamento de um gás se aproxima daquele esperado
para um gás ideal? Justifique.
b) No primeiro parágrafo o autor comentou que a lei dos gases ideais surgiu da combinação de algumas
relações propostas por Boyle, Charles e Gay-Lussac. Explique como a equação de estado do gás perfeito pode ser
obtida usando a lei de Boyle, a lei de Charles e o princípio de Avogadro.

Resolução: Item a)

Van der Waals →1873 → procurou adaptar a equação dos gases ideais para descrever o comportamento de um gás
real. Ele adicionou duas constantes, uma devido ao fato das partículas ocuparem um volume finito (b) e outro devido
as interações entre as partículas constituintes do gás (a). O modelo de gás ideal assume que não existem interações
entre as partículas de um gás. No entanto, as partículas de um gás real interagem entre si através das forças de Van
der Waals. Outra falha no modelo do gás ideal é a desconsideração total do chamado “volume próprio das partículas”.
As partículas de um gás real apresentam volume mesmo quando a temperatura tende a zero ou quando a pressão
tende ao infinito”.

154
1600 FÍSICO-QUÍMICA APLICADA EXERCÍCIOS COMENTADOS - IME – ITA – OLIMPÍADA

Item b)
Lei de Boyle → Descreve o comportamento de um gás ideal apenas quando sua temperatura é mantida constante
(para um processo em que a temperatura se mantém constante é chamada de isotérmica). Quanto mais você aumenta
a pressão no gás, o seu volume também diminuirá. Logo perceberá que as grandezas volume e pressão são
diretamente proporcionais. Então a lei de Boyle diz matematicamente que: p.V = k, onde k é uma constante que
depende da massa, temperatura e da natureza desse gás.

Lei de Charles - Gay Lussac → Descreve o comportamento de um gás ideal apenas quando a pressão é mantida
constante (para um processo em que a pressão se mantém constante é chamada de isobárica). Para uma dada massa
de gás à pressão constante, o volume ocupado pelo gás é diretamente proporcional à temperatura absoluta. Um
aumento da temperatura absoluta acarreta um aumento do volume ocupado pelo gás, de maneira que o quociente
seja constante V/T = k. A relação entre volume e temperatura foi inicialmente observada em 1787 por Jacques Charles
e quantificada em 1802 por Joseph Gay-Lussac, sendo conhecida como lei de Charles Gay-Lussac.

Lei de Avogadro → Em 1811, A. Avogadro enunciou sua famosa lei, também conhecida por hipótese de Avogadro,
segundo a qual volumes iguais de diferentes gases, a uma mesma temperatura e pressão, contém o mesmo número
de moléculas.

Questão 303 – (GRILLO) Partindo da equação de van der Waals para os gases reais, expresse a pressão deste
composto gasoso para um mol.

V V
Resolução: Sabendo que o volume molar é a razão do volume pelo número de mol, temos: Vmolar = n = 1
=V

a
Equação dos gases reais de van der Waals: {preal + V2molar
}x {Vmolar − b} = R x T

a
{preal + } x {V − b} = R x T

RxT a
preal = −
(V − b) V²

Questão 304 – Calcular, usando a equação de van der Waals, a pressão necessária para manter 10 gramas de amônia
num volume de 289,0 cm3 a 0°C. Além da pressão, calcule o raio desta partícula. Dados: Constantes de Van der Waals:
a = 4,39 atm.L².mol-2 e b = 0,037 L.mol-1.

𝑚𝑁𝐻 10
Resolução: Cálculo do número de mol de amônia: 𝑛𝑁𝐻3 = <𝑀𝑀> 3 = 17 = 0,0588 𝑚𝑜𝑙
𝑁𝐻3

𝑉 0,289
Cálculo do volume molar: 𝑉𝑚𝑜𝑙𝑎𝑟 = 𝑛 = 0,588
= 0,491 𝑚𝑜𝑙. 𝐿−1

𝑎
Cálculo da pressão do NH3: {𝑝𝑟𝑒𝑎𝑙 + 2 } 𝑥 {𝑉𝑚𝑜𝑙𝑎𝑟 − 𝑏} = 𝑅 𝑥 𝑇
𝑉𝑚𝑜𝑙𝑎𝑟

155
1600 FÍSICO-QUÍMICA APLICADA EXERCÍCIOS COMENTADOS - IME – ITA – OLIMPÍADA

𝑟𝑒𝑎𝑙
4,39 289 𝑥 10−3
{𝑝 + }𝑥 { − 0,037} = 0,08206 𝑥 273
289 𝑥 10−3 0,588
( )²
0,588

𝑝𝑟𝑒𝑎𝑙 = 31,12 𝑎𝑡𝑚

b
Cálculo do raio da molécula de NH3: Vesfera = N

4πR³ b
=
3 N

4 x 3,14 x R³ 0,037
=
3 6,02 x 1023

3
𝑅 = √1,46 𝑥 10−26 = 2,45 𝑥 10−9 𝑚𝑒𝑡𝑟𝑜𝑠 (2,45 𝑛𝑚)

Questão 305 – (IME) Ao desejar identificar o conteúdo de um cilindro contendo um gás monoatômico puro, um
estudante de química coletou uma amostra desse gás e determinou sua densidade, d = 5,38 g.L -1, nas seguintes
condições de temperatura e pressão: 15°C e 0,97 atm. Com base nestas informações, e assumindo o modelo do gás
ideal.

a) Calcular a massa molar do gás.


b) Identificar o gás.

Resolução:

a) Cálculo da massa molar (<MM>) do referido gás, a partir da relação entre a densidade e a pressão: d =
p x <MM>
RxT
dxRxT 5,38 x 0,08206 x (15+273)
Isolando a massa molar: < MM > = p
= 0,97
= 131,08 g. mol−1

b) O gás em questão refere-se ao Xenônio (grupo 18 - quinto período).

Questão 306 – Massa de 12,76 gramas de pentacloreto de fósforo é introduzida em recipiente com capacidade igual
a 3,5 litros. Sabendo que o aquecimento do sistema levará à dissociação completa do PCl5, determinar o grau de
dissociação, a partir das seguintes situações:

a) a 200°C observa-se que a pressão no recipiente, após atingir valor constante, mantém-se igual a 753,5 mm
Hg;
b) a 250°C a pressão estabiliza-se em 102l,5 mm Hg.

156
1600 FÍSICO-QUÍMICA APLICADA EXERCÍCIOS COMENTADOS - IME – ITA – OLIMPÍADA

Resolução: Base de cálculo: n(mol) inicial de pentacloreto de fósforo. Tabela de equilíbrio químico:

PCl5(g) → PCl3(g) Cl2(g)


Início n 0 0
Reage nα nα nα
Equilíbrio n - nα nα nα

Cálculo do número de mol total da mistura gasosa: nTotal = nPCl5 + nPCl3 + nCl2 = n - nα + nα + nα = n.(1 + α)

753,50 12,76
a) Cálculo do grau de dissociação (α): ( 760,0 ) x 3,50 = 208,5 x (1 + α) x (0,08206) x (200 + 273)

α = 0,461 (46,10%)

m
Item b) Cálculo do grau de dissociação (α): pideal x V = <MM> x R x T
1021,50 12,76
( ) x 3,50 = x (1 + α) x (0,08206) x (250 + 273)
760,0 208,5

α = 0,7911 (79,11%)

Questão 307 – (GRILLO) Considere uma quantidade de 5,50 mol de monóxido de carbono estejam presentes em um
reservatório de capacidade igual a 3,50 litros a uma temperatura de aproximadamente de 37°C. Calcule a pressão do
referido gás, considerando:

a) comportamento ideal;
b) comportamento real, com a utilização da equação de van der Waals;
c) calcule o fator de compressibilidade e diga se as forças são dominantes ou repulsivas.
Informações para a resolução do problema: coeficientes de van der Waals: a = 1,453 atm.L².mol -2; b = 3,95 x 10-2
L.mol-1.

Resolução: Item a) Cálculo da pressão do monóxido de carbono, considerando comportamento ideal:

pideal x 3,50 = 5,50 x (0,08206) x (37 + 273)

pideal = 39,97 atm

Item b) Cálculo da pressão do monóxido de carbono, considerando comportamento real: {preal +

a
V2molar
} x {Vmolar − b} = R x T
V 3,50
Cálculo do volume molar: Vmolar = n = 5,50
= 0,64 L. mol−1

1,453
{preal + } x {0,64 − 3,95 x 10−2 } = 0,08206 x (37 + 273)
(0,64)²
preal = 38,81 atm
𝑃𝑟𝑒𝑎𝑙 38,81 𝑎𝑡𝑚
Item c) Cálculo do fator de compressibilidade (Z): 𝑍 = 𝑃𝑖𝑑𝑒𝑎𝑙
= 39,97 𝑎𝑡𝑚
= 0,971

157
1600 FÍSICO-QUÍMICA APLICADA EXERCÍCIOS COMENTADOS - IME – ITA – OLIMPÍADA

Para Z < 1, as forças dominantes são as forças atrativas.

Questão 308 – (GRILLO)

a) Calcule a densidade do gás metano, em g.L-1, considerando que a pressão seja igual a 1,20 atmosferas com
uma temperatura igual a 55°C.
b) O diborano, com fórmula molecular B2H6, é um composto químico formado pelos átomos boro e hidrogênio.
Trata-se de um gás incolor a temperatura ambiente que apresenta um odor com característica adocicada. Este
composto pode ser queimado na presença de oxigênio molecular, produzindo óxido bórico e água no estado líquido.
Calcule o volume em litros de oxigênio necessário para a produção de 20,0 gramas de óxido, quando o local apresenta
pressão igual a 3,00 atmosferas e 27°C.
c) Utilizando a (I) equação dos gases ideais e a (II) equação dos gases reais, por intermédio da equação de
van der Waals, calcule a temperatura em graus Celsius para que 30,0 gramas de gás etano estejam confinados em
um recipiente de volume igual a 10 litros, sendo aquecido para que a pressão seja de aproximadamente igual a 50
atmosferas. Informação para a resolução do problema: constante de van der Waals: a = 4,42 L².atm.mol-2; b = 0,057
L.mol-1.

Resposta:

p x <MM> 1,20 x 16
Item a) Sabendo que a relação entre a densidade e a pressão é dada por: d = RxT
= 0,08206 x (55+273) =

0,71 g. L−1
Item b) Equação química: B2H6(g) + 3 O2(g) → B2O3(s) + 3 H2O(l).
3 mol de O2(g) --------------- 1 mol de B2O3(s)
(3 x 32) g.mol-1 ------------- 1 x (2 x 11 + 3 x 16) g.mol-1
mO2 --------------------------- 20 g
mO2 = 27,43 g de O2(g)
27,43
nxRxT ( ) x 0,08206 x (27+273)
Cálculo do volume de oxigênio (O2) consumido: VO2 = = 32
= 7,03 L
pideal 3,0

30
Item c) I) Considerando o gás com comportamento ideal: 50 x 10 = 30 x 0,08206 x T

T = 6093,10 K

4,42
II) Considerando o gás com comportamento real, utilizando a equação de van der Waals: {50 + (10)²
} x {10 −

0,057} = 0,08206 x T

T = 6063,73 K

Questão 309 – Considere a mistura gasosa apresentada pela seguinte reação reversível hipotética apresentada
abaixo: XY5(g) ⇄ XY3(g) + Y2(g). Sabendo que a pressão total dos gases neste recipiente é igual a uma atmosfera e a
densidade da mistura gasosa é igual a 7,063 g/L, a uma temperatura igual a 27°C e que a massa molar do composto

158
1600 FÍSICO-QUÍMICA APLICADA EXERCÍCIOS COMENTADOS - IME – ITA – OLIMPÍADA

que sofre a decomposição é igual a 208,5 g/mol, determine o grau de dissociação (α). Resolução: Base de cálculo:
n (mol) inicial de XY5(g).

Tabela de equilíbrio químico:

XY5(g) (mol) ⇄ XY3(g) (mol) Y2(g) (mol)


Início n 0 0
Reage nα nα nα
Equilíbrio n - nα nα nα

Cálculo do número de mol total da mistura reacional: nTOTAL = nXY5 + nXY3 + nY2 = n - nα + nα + nα = n + nα

Considerando que os gases apresentam comportamento ideal, temos: p x < MM > = d x (1 + α). R. T

p x<MM>
Cálculo do grau de dissociação (α), a partir da Equação I: d = (1+
α)x R x T

(1 + α) x R x T x d = p x < MM >

(1 + α) x 0,08206 x (27 + 273) x 7,063 = 1 x 208,5

α = 0,199 (≅ 20,0 %)

Questão 310 – (IME) Dar as fórmulas moleculares dos gases, adiante relacionados, separando-os conforme suas
densidades em relação ao ar atmosférico.

✓ Gás carbônico
✓ Anidrido sulfuroso
✓ Acetileno
✓ Neônio
✓ Argônio
✓ Amoníaco

Resolução: Cálculo da massa molecular do ar atmosférico (<MM>ar), considerando que a composição apresenta 21%
(v/v) de O2 e 79% (v/v) de N2.

<MM>ar = {(%)N2 x <MM>N2} + {(%)O2 x <MM>O2} = {0,79 x 28} g/mol + {0,21 x 32} g/mol = 28,84 g/mol

Fórmula molecular dos compostos gasosos mencionados, massa molecular e comparação com a densidade do ar
atmosférica.

✓ Gás carbônico → CO2 = 44 g/mol → mais denso que o ar atmosférico;


✓ Anidrido sulfuroso → SO2 = 64 g/mol → mais denso que o ar atmosférico;
✓ Acetileno → C2H2 = 26 g/mol → menos denso que o ar atmosférico;
✓ Neônio → Ne = 20 g/mol → menos denso que o ar atmosférico;
✓ Argônio → Ar = 40 g/mol → mais denso que o ar atmosférico;

159
1600 FÍSICO-QUÍMICA APLICADA EXERCÍCIOS COMENTADOS - IME – ITA – OLIMPÍADA

✓ Amoníaco → NH3 = 17 g/mol → menos denso que o ar atmosférico.

Questão 311 – (IME) Em um parque de diversões, um certo dia quente, um homem enchia balões com gás hélio. Se
o volume médio dos balões, depois de cheios, era de 10 L, a temperatura média do dia era de 37°C e a pressão de
gás no balão era de 2,50 atm, qual a massa de He em cada balão?

Resolução: Cálculo da massa de gás Hélio (He), a partir do comportamento ideal: 2,50 x 10,0 =
mHe
4,0
x 0,08206 x (37 + 273)

mHe = 3,93 g

Questão 312 – A temperatura crítica do gás etano (C2H6) é igual a 33°C, a pressão crítica sendo igual a 48,20 atm.
A partir das informações apresentadas, calcule:

a) o volume crítico, aplicando a equação dos gases ideais;


p xV 3
b) o volume crítico a partir da equação de van der Waals, a partir da seguinte relação matemática: c c = .
R x Tc 8

Resolução: Item a) A partir da utilização da equação dos gases ideais: pc x Vc = n x R x Tc

Vc R x Tc 0,08206 x (33 + 273) L


= = = 0,521
n pc 48,20 mol

pc x Vc 3
Item b) A partir da equação apresentada: =
R x Tc 8

3 x R x Tc 3 𝑥 0,08206 𝑥 (33 + 273) L


Vc = = = 0,195
8 x pc 8 𝑥 48,20 mol

Questão 313 –

a) A temperatura crítica (Tc) é uma temperatura bem particular em que o gás real não se liquefaz. Para esta
temperatura, os indicativos são também observados através da pressão critica (Pc) e volume crítico (Vc). Considere
que as constantes de van der Waals a e b para o benzeno são respectivamente iguais a 18 atm.L².mol-2 e 0,115 L.mol-
1. A partir desta informação, determine as constantes críticas.
b) A temperatura crítica e a pressão crítica do naftaleno são respectivamente 475 K e 41 atmosferas. A partir
desta informação determine as constantes de van der Waals e o volume crítico para o referido composto.

160
1600 FÍSICO-QUÍMICA APLICADA EXERCÍCIOS COMENTADOS - IME – ITA – OLIMPÍADA

Resolução: a) Cálculo do volume crítico: VC = 3 x b = 3 x 0,115 = 0,345 L.mol-1

a 18
Cálculo da pressão crítica: Pcrítica = 27 x b² = 27 x (0,115)²
= 50,41 atm

8xa 8 x 18
Cálculo da temperatura crítica: Tcrítica = 27 x b x R = 27 x 0,115 x 0,08206 = 565,16 K

b) Determinação das constantes de van der Waals (a e b). Sabendo que a temperatura crítica é dada por: Tcrítica =
8xa
27 x b x R

8a
475 =
27 x 0,08206 x b
a
b
= 131,55 (Equação 1)

a
A partir da pressão crítica, pode-se tirar a seguinte relação: Pcrítica = 27 x b2

a
41 =
27 x b 2
a
= 1107

a 1
b
x b
= 1107 (Equação 2)

a
A partir da equação 2, temos: b²
= 1107

a 1
x = 1107
b b
a 1
Substituindo a equação 1 na equação 2, temos: b x = 1107
b

1
131,55 x = 1107
b

b = 0,119 L.mol-1.

a
Cálculo da constante de van der Waals (a). Substituindo o valor de b na equação 1, temos: b = 131,55

a
= 1107
0,119

a = 15,65 atm. L². mol−2

Cálculo do volume crítico (Vc): Vc = 3 x b = 3 x 0,119 = 0,357 L.mol-1

161
1600 FÍSICO-QUÍMICA APLICADA EXERCÍCIOS COMENTADOS - IME – ITA – OLIMPÍADA

Questão 314 – Considere um recipiente com capacidade de 12,50 litros e que tenha em seu interior 750 gramas de
gás cloro a uma temperatura de 127°C. Diante dessas informações, determine:
a) a pressão do gás considerando comportamento ideal;
b) a pressão do gás considerando comportamento real;
c) o fator de compressibilidade;
d) a temperatura de Boyle;
e) a temperatura, pressão e volume crítico.
Informações para a resolução do problema: constante de van der Waals: a = 6,49 at.L².mol-2; b = 0,0592 L.mol-1.

750
Resolução: Item a ) Considerando o gás com comportamento ideal: pideal x 12,50 = 71
x 0,08206 x (127 +

273)

pideal = 12,27 atm

6,49
Item b) Considerando o gás com comportamento real: {preal + 12,50
} x {1,18 − 0,0592} =
(750 )²
⁄71

0,08206 x (127 + 273)


preal = 24,62 atm

Preal 24,62 atm


Item c) Cálculo do fator de compressibilidade: Z = = = 0,89
Pideal 27,74 atm

Para Z < 1 → Forças atrativas.

a 6,49
Item d) Cálculo da temperatura de Boyle (TB): TB = = = 1335,95 K
bxR 0,0592 x 0,08206

Item e) Volume crítico: Vcrítico = 3 x b = 3 x (0,0592) = 0,178 L. mol−1

a 6,49
Pressão crítica: Pcrítica = 27.b² = 27 x (0,0592)²
= 68,59 atm

8.a 8 x 6,49
Temperatura crítica: Tcrítica = 27.b.R = 27 x 0,0592 x 0,08206
= 395,84 K

162
1600 FÍSICO-QUÍMICA APLICADA EXERCÍCIOS COMENTADOS - IME – ITA – OLIMPÍADA

Questão 315 – Considere os seguintes dados referentes ao gás cloro, conforme mostrado na tabela abaixo:

Gás Cloro
Massa (g) 550
Volume em litros de Cl2 no recipiente 25
Temperatura operacional (°C) 125
Parâmetro de van der Waals (a) (atm.L².mol-2) 6,49
Parâmetro de van der Waals (b) (L.mol-1) 5,62 x 10-2

A partir da tabela com as características apresentadas do gás cloro, calcule os seguintes itens abaixo.

a) Calcule a pressão do gás cloro, considerando comportamento ideal.


b) Calcule a pressão do gás cloro, considerando comportamento real de van der Waals.
c) O fator de compressibilidade (Z).
d) Qual a força predomina para este comportamento do gás cloro, forças atrativas ou repulsivas?
e) A pressão a partir do fator de compressibilidade.
f) Calcule a pressão, temperatura e volume crítico.
g) A temperatura de Boyle.

mCl 550
Resolução: Cálculo do número de mol de gás cloro: nCl2 = <MM>2 = 71
= 7,75 mol
Cl2

25
Cálculo do volume molar: Vmolar = 7,75 = 3,22 L. mol−1

a) Considerando que o gás apresente comportamento ideal: pideal x (25) = 7,75 x (0,08206) x (125 + 273)
pideal = 10,12 atm

6,49
b) Considerando que o gás apresente comportamento real, temos: {preal + (3,22)²
}x {3,22 −

5,62 x 10−2 } = 0,08206 x (125 + 273)


preal = 9,70 atm

c) Cálculo do fator de compressibilidade (Z): Sabendo que o fator de compressibilidade é a relação entre o
PReal 9,70 atm
comportamento real e ideal, temos: Z = PIdeal
= 10,12 atm
= 0,958. A forção dominante á a dominante, uma vez

que Z < 1.

d) Cálculo da pressão de Cl2, considerando comportamento real, a partir do fator de compressibilidade:


0,958 x 0,08206 x 398
p= = 9,72 atm
3,22

e) Calculo da pressão crítica, temperatura crítica e volume crítico:


a 6,49
Pcrítica = = = 76,10 atm
27 x b² 27 x (5,62 x 10−2 )²

163
1600 FÍSICO-QUÍMICA APLICADA EXERCÍCIOS COMENTADOS - IME – ITA – OLIMPÍADA

8xa 8 x 6,49
Tcrítica = = = 416,97 K
27 x b x R 27 x 5,62 x 10−2 x 0,08206

Vcrítico = 3 x b = 3 x 0,0562 = 0,169 L. mol−1

a 6,49
f) Cálculo da temperatura de Boyle (TBOYLE): TB = b x R = 5,62 x 10−2 x 0,08206 = 1407,27 K

Questão 316 – (IME) Determine a temperatura Celsius em que o etileno, a 800 mmHg, terá a mesma densidade
absoluta que o oxigênio, a 700 mmHg e a 20°C.

Resolução: Para a resolução deste problema será necessário considerar os gases como ideais, relacionando a
densidade com a pressão.
p x < MM >
d=
RxT
p x <MM> 800 x 28 22400
Estudando em função do gás etileno: dC2 H4 = RxT
= RxT = RxT
C 2 H4 C 2 H4

p x <MM> 700 x 32 22400


Estudando em função do gás oxigênio: dO2 = RxT
= R x (20+273) = 293 x R

Igualando as densidades dos compostos gasosos: dC2 H4 = dO2

22400 22400
=
R x TC2 H4 293 x R

22400 22400
=
R x TC2 H4 293 x R

TC2 H4 = 293 K

Questão 319 – (IME) Mistura-se um fluxo de ar seco com vapor d’água para se obter ar úmido com 2,0%, em volume,
de umidade. Admitindo o comportamento ideal dos gases e a massa molecular média do ar seco como 28,96 g/mol,
calcule a massa específica do ar úmido a 14,25o C e 1,00 x 105 Pa.

Resolução: Equação química: Fluxo de ar seco + vapor d′ água → Fluxo de ar úmido

Cálculo da massa molar do fluxo de ar úmido:

(98% x 28,96) + (2% x 18) 2874,08 g


< MM >fluxo de ar úmido = = = 28,74
100% 100 mol
100000
Conversão da pressão em atmosferas: p = = 0,987 atm
101325

p x <MM>fluxo de ar úmido 0,987 x 28,74


Cálculo da massa específica do ar úmido: ρfluxo de ar úmido = RxT
= 0,08206 x (14,25+273) =
g
1,20 L

164
1600 FÍSICO-QUÍMICA APLICADA EXERCÍCIOS COMENTADOS - IME – ITA – OLIMPÍADA

Questão 320 – (IME) Uma amostra de uma substância pesando 0,08 gramas desloca 30 cm³ de ar, medidos a 27 °C
e pressão de 720 mmHg. Determine a massa molecular da substância.

760 x 0,08 x 0,08206 x (27+273) 1496,77


Resolução: A partir da equação dos gases ideais, temos: < MM > = 720 x 30 x 10−3
= 21,6
=
g
69,29
mol

Questão 321 – (ITA) Dois balões esféricos de mesmo volume são unidos por um tubo de volume desprezível, provido
de torneira. Inicialmente o balão A contém 1,00 mol de gás ideal, e em B há vácuo.

Os dois balões são mantidos às temperaturas indicadas no desenho acima. A torneira é aberta durante certo tempo.
Voltando a fechá-la, verifica-se que a pressão em B é 0,81 do valor da pressão em A. Quanto do gás deve ter sobrado
no balão A?

Resolução: Para a resolução deste problema será necessário a utilização da equação dos gases ideais.
Informação do problema: pB = 0,81 x pA

Analisando o tubo A: pA x VA = nA x R x 400

Analisando o tubo B: 0,81 𝑥 pA x VB = nB x R x 324

Considerando que o processo seja isocórico (volume constante) e dividindo a equação A pela equação B, temos:
pA x VA nA x R x 400
=
0,81 𝑥 pA x VB nB x R x 324

0,81 𝑥 nA x R x 400 = nB x R x 324

nA x 324 = nB x 324

nA = nB

Logo irão sobrar 0,50 mol no balão A.

Questão 322 – (GRILLO) Partindo de 1000 gramas de sulfeto de cobre II, determine o volume do dióxido de nitrogênio
e de dióxido de enxofre a 27°C e 2,0 atmosferas. Equação química: CuS(s) + HNO3(aq) → Cu(NO3)2(s) + H2O(l) + NO2(g)
+ SO2(g).

Resolução: Primeiramente será necessário realizar o balanceamento pelo método REDOX, em que o sulfeto de cobre
II é o agente redutor e a solução de ácido nítrico é o agente oxidante.

Equação química balanceada: CuS(s) + 8 HNO3(aq) → Cu(NO3)2(s) + 4 H2O(l) + 6 NO2(g) + SO2(g)

165
1600 FÍSICO-QUÍMICA APLICADA EXERCÍCIOS COMENTADOS - IME – ITA – OLIMPÍADA

1000
Cálculo do número de mol de sulfeto de cobre II: 𝑛𝐶𝑢𝑆 = 95,55 = 10,46 𝑚𝑜𝑙

Cálculo do número de mol de dióxido de nitrogênio e dióxido de enxofre, a partir da relação estequiométrica:

1 mol de CuS − − − − − − 6 mol de NO2 − − − 1 mol de SO2

10,46 mol de CuS − − − − nNO2 − − − − − − − nSO2

nNO2 = 62,76 mol e nSO2 = 10,46 mol

Cálculo do volume dos gases produzidos, utilizando a equação dos gases ideais:

nNO2 x R x T 62,76 x 0,08206 x (27+273)


Dióxido de nitrogênio: VNO2 = p
= 2,0
= 772,54 L

nSO2 x R x T 10,46 x 0,08206 x (27+273)


Dióxido de enxofre: VSO2 = p
= 2,0
= 128,75 L

Questão 344 – (GRILLO) Adicionando-se carbonato de amônio a 490 gramas de ácido fosfórico. A partir desta
informação, faça o que se pede.

a) Determine a massa de sal formado.


b) Nomeie os produtos formados.
c) Determine o volume de dióxido de carbono liberado nas CNATP.
d) Determine o volume de dióxido de carbono nas CNTP.

Resolução: Equação química: 3 (NH4 )2 CO3 + 2 H3 PO4 → 2 (NH4 )3 PO4 + 3 H2 O + 3 CO2

a) Cálculo da massa de sal formada, a partir da estequiometria:

2 mol de H3 PO4 − − − − 2 mol de (NH4 )3 PO4

2 x 98 g − − − − − − − 2 x 149 g

490 g − − − − − − − − msal

490 x 2 x 149 146020


msal = = = 745 g
2 x 98 196

b) Os produtos formados são: fosfato de amônio, água e dióxido de carbono.

490
c) Cálculo do número de mol de ácido fosfórico: nácido = 98
= 5 mol

Cálculo do número de mol de dióxido de carbono, a partir da relação estequiométrica: 3 (NH4 )2 CO3 + 2 H3 PO4 →
2 (NH4 )3 PO4 + 3 H2 O + 3 CO2

2 mol de H3 PO4 − − − − − − 3 mol de CO2


5 mol de H3 PO4 − − − − − − nCO2

166
1600 FÍSICO-QUÍMICA APLICADA EXERCÍCIOS COMENTADOS - IME – ITA – OLIMPÍADA

5x3
nCO2 = = 7,50 mol
2
nCO2 x R x T 7,50 x 0,08206 x (25+273)
Cálculo do volume de dióxido de carbono nas CNATP: VCO2 = p
= 1
= 183,40 L

L
d) Cálculo do volume de dióxido de carbono nas CNTP: VCO2 = 22,4 x nCO2 = 22,4 x 7,50 = 168 L
mol

Questão 345 – Sabe-se que 1,50 mol de gás metano ocupam 1700 dm³ a 0°C e pressão de 0,9 atm. Considerando
que o metano se comporta de acordo com a equação de Van der Waals (a = 2,25 L².atm.mol-1 e b = 0,0428 L.mol-1),
calcule o que se pede:

a) Fator de compressibilidade do gás.

b) Pressão, temperatura e volume crítico do CH4.

c) Temperatura de Boyle do CH4.

Resolução:

pxV 0,9 x 1700 1530


a) Cálculo do fator de compressibilidade (Z): Z = n x R x T = 1,50 x 0,08206 x (0+273) = 33,60 = 45,53

b) Cálculo da pressão, volume e temperatura crítica:

L
Volume crítico: Vc = 3 x b = 3 x 0,0428 = 0,128 mol
8xa 8 x 2,25 18
Temperatura crítica: Tc = 27 x b x R = 27 x 0,0428 x 0,08206 = 0,948 = 18,99 K

a 2,25
Pressão crítica: Tc = 27 x b2 = 27 x (0,0428)2 = 45,49 atm

a 2,25
c) Cálculo da temperatura de Boyle: T BOYLE = b x R = 0,0428 x 0,08206 = 640,63 K

Questão 346 – Hematita (Fe2O3) pode ser produzida mediante oxidação da pirita (FeS2). 4 FeS2(s) + 11 O2(g) → 2
Fe2O3(s) + 8 SO2(g) Determine o volume de ar atmosférico necessário para reagir com 100 g de sulfeto de ferro a 2,5
atm e 225°C. Considere que o ar atmosférico contém 21% em bases molares de O2, e que a alimentação do processo
é estequiométrica.

Resolução: Cálculo do número de mol de gás nitrogênio, a partir de 11 mol de gás oxigênio: 4 FeS2(s) + 11 O2(g) → 2
Fe2O3(s) + 8 SO2(g)

11 mol de O2 − − − − − 21%
nN2 − − − − − − − − − − 79%
11 x 79
nN2 = = 41,38 mol
21
Equação química com a presença do gás nitrogênio: 4 FeS2(s) + 11 O2(g) + 41,38 N2(g) → 2 Fe2O3(s) + 8 SO2(g) + 41,38
N2(g)

167
1600 FÍSICO-QUÍMICA APLICADA EXERCÍCIOS COMENTADOS - IME – ITA – OLIMPÍADA

100 100
Cálculo do número de mol de pirita: 𝑛𝑝𝑖𝑟𝑖𝑡𝑎 = (56+64) = 120 = 0,83 mol

Cálculo do número de mol de ar atmosférico: 4 FeS2(s) + 11 O2(g) + 41,38 N2(g) → 2 Fe2O3(s) + 8 SO2(g) + 41,38 N2(g)

(11 + 41,38) mol de ar − − − − − 4 mol de FeS2


nar − − − − − − − − − − − − − − 0,83 mol de FeS2
0,83 x 52,38
n𝑎𝑟 = = 10,87 mol
4
10,87 x 0,08206 x (225+273)
Cálculo do volume de ar atmosférico, aplicando a equação dos gases ideais: V = =
2,5

177,68 Litros

Questão 347 – A 10°C e a uma pressão de 75 atm, o fator de compressibilidade do gás nitrogênio é igual a 0,933.
Calcule a massa de nitrogênio necessária para encher o cilindro de 89 L de capacidade.

Massa
Resolução: Aplicando a equação dos gases reais em função do fator de compressibilidade: pV = Z x <MM> x RT

p x V x < MM > 75 x 89 x 28 186900


Massa = = = = 8629 g (8,63 kg)
ZxRxT 0,933 x 0,08206 x (10 + 273) 21,66

Questão 348 – Uma mistura gasosa é constituída por três gases distintos: 0,65 mol de dióxido de carbono, 1,75 mol
de oxigênio e 0,95 mol de nitrogênio, cuja densidade é igual a 1,80 g.L-1, a 47°C. Calcule:

a) o volume da mistura gasosa;


b) os volumes parciais dos gases;
c) as pressões parciais dos gases.

Resolução: Cálculo do número de mol total: nTotal = nCO2 + nO2 + nN2 = 0,65 mol + 1,75 mol + 0,95 mol = 3,35 mol

nCO2
Cálculo da massa molar da mistura (<MM>mistura): < MM >mistura x X mistura = < MM >CO2 x ( )+
nTotal
nO2 n𝐍2
< MM >O2 x ( ) + < MM >N2 x ( )
nTotal nTotal

0,65 1,75 0,95 g


< MM >mistura x 1,0 = 44,0 x ( ) + 32,0 x ( ) + 28,0 x ( ) = 33,20
3,35 3,35 3,35 mol

dxRxT
Cálculo da pressão total da mistura (pT), a partir da relação com a densidade da mistura: pTotal = <MM> =
mistura

1,80 x 0,08206 x (47+273)


33,20
= 1,42 atm

nTotal x R x T 3,35 x 0,08206 x (47+273)


a) Cálculo do volume total da mistura gasosa (VTotal): VTotal = pTotal
= 1,42
= 61,95 L

b) Cálculo dos volumes parciais, aplicando a Lei de Amagat:


nCO2 0,65
VCO2 = X CO2 x VTotal = ( ) x VTotal = ( ) x 61,95 L = 12,02 L
nTotal 3,35

168
1600 FÍSICO-QUÍMICA APLICADA EXERCÍCIOS COMENTADOS - IME – ITA – OLIMPÍADA

nO2 1,75
VO2 = X O2 x VTotal = ( ) x VTotal = ( ) x 61,95 L = 32,36 L
nTotal 3,35

nN2 0,95
VO2 = X N2 x VTotal = ( ) x VTotal = ( ) x 61,95 L = 17,57 L
nTotal 3,35

Questão 349 –

a) A temperatura de 280 K e pressão igual a 19 atmosferas, o fator de compressibilidade é igual a 0,80. Calcule
o volume ocupado deste gás que apresenta 6,0 mmol.
b) Considere a decomposição térmica do carbonato de cálcio, que apresenta 45% de pureza e rendimento de
75%. O gás formado é colocado em um reservatório de 500 mL, a 37°C e 1,50 Mpa. A partir desta informação,
determine a massa de gás produzido e a massa de amostra utilizada.
c) A 600 K e sob pressão de 0,50 atmosferas, a reação de dissociação do pentacloreto de fósforo, PCl 5(g) =
PCl3(g) + Cl2(g), resulta em mistura gasosa que apresenta massa específica igual a 1,42 kg/m 3. Determinar o grau de
dissociação do pentacloreto de fósforo, nesta temperatura.
Resolução:
Item a) Cálculo do volume do gás (V), a partir da equação dos gases reais em função do fator de compressibilidade:

Z x n x R x T 0,80 x 6,0 x 10−3 x 0,08206 x 280


V= = = 5,80 x 10−3 L (5,80 mL)
p 19


Item b) Equação química: CaCO3(s) →
⏞ CaO(s) + CO2(g)

1 atm
Sabendo que uma atmosfera corresponde aproximadamente 105 Pascal, temos: P = 1,50 x 106 Pa x 105 Pa =

15,0 atm

p x V x <MM>CO2 15 x 0,50 x 44 330


Cálculo da massa de dióxido de carbono: mCO2 = RxT
= 0,08206 x (37+273) = 25,44 = 12,97 g

Cálculo da massa de carbonato de cálcio, a partir do processo estequiométrico: 1 mol de CaCO3 --------- 1 mol de
CO2

100 g de CaCO3 ---------- 44 g de CO2 x (0,75)


0,45 x mCaCO3 -------------- 12,87 g
12,87 x 100 1287
mCaCO3 = = = 86,67 g
44 x 0,75 x 0,45 14,85

Item c) Base de cálculo: n mol inicial de pentacloreto de fósforo e utilizando a tabela de equilíbrio químico:

PCl5(g)  PCl3(g) Cl2(g)


Início n 0 0
Reage nα nα nα

169
1600 FÍSICO-QUÍMICA APLICADA EXERCÍCIOS COMENTADOS - IME – ITA – OLIMPÍADA

Equilíbrio n - nα nα nα

Cálculo do número de mol total (nTotal): nTotal = n - nα + nα + nα = n + nα = n(1 + α)

Considerando que os gases que estão no equilíbrio e que apresentam comportamento ideal, a relação entre a
densidade e o grau de dissociação é dada pela seguinte equação matemática: pV = 𝑛(1 + 𝛼)RT

m x (1 + α) x R x T
< MM > =
Vxp

d x (1 + α) x R x T
< MM > =
p

Sabendo que a densidade proposta pelo problema é igual a 1,42 kg.m-3 o que equivale a 1,42 g.L-1, o grau de
1,42 x (1+α) x 0,08206 x 600
dissociação (α) será: 208,5 =
0,50

104,25
1+α=
69,91

α = 1,49 − 1,0 = 0,49 (49%)

Questão 350 – Uma das etapas para a produção de ferro metálico, trata-se da decomposição da pirita e da formação
de óxido de ferro III, conforme pode ser observado pela equação química balanceada a seguir: 4 FeS2(s) + 11 O2(g) →
2 Fe2O3(s) + 8 SO2(g). A partir desta informação, determine o volume de ar atmosférico que é necessário para reagir
com 100 quilogramas de sulfeto de ferro a 2,5 atm e 225C. Considere que o ar atmosférico contém 21% em volume
de O2.

Resolução: Sabendo que o ar atmosférico apresenta composição igual a 21% de oxigênio e 79% de nitrogênio, o
número de mol de N2 será:

4 FeS2(s) + 11 O2(g) → 2 Fe2O3(s) + 8 SO2(g)


11 mol de O2 --------- 21%
nN2 --------------------- 79%
nN2 = 41,38 mol

Equação química com a presença de nitrogênio fica da seguinte maneira: 4 FeS2(s) + 11 O2(g) + 41,38 N2(g) → 2 Fe2O3(s)
+ 8 SO2(g) + 41,38 N2(g)

Cálculo do número de mol de ar: 4 FeS2(s) + 11 O2(g) + 41,38 N2(g) →

4 mol de FeS2 ------------------- (11 + 41,38) de ar


100000
120
mol ----------------------- nar

100000 52,38
𝑛𝑎𝑟 = = 10912,50 mol
4 𝑥 120

170
1600 FÍSICO-QUÍMICA APLICADA EXERCÍCIOS COMENTADOS - IME – ITA – OLIMPÍADA

nar RT
Cálculo do volume de ar, considerando que o gás apresenta comportamento ideal: Var = 𝑝
=
10912,50 𝑥 0,08206 𝑥 (225+273)
2,50
= 178379 L

Questão 351 – Três esferas de 200 mililitros cada, a 27°C, contendo cada um deles um gás distinto e também a
pressão diferente, são misturadas por meio da abertura de duas válvulas, conforme pode ser observada na figura
abaixo. A partir desta informação, determine a pressão total e as pressões parciais dos gases na mistura.

2 atm 4 atm

6 atm

pV
Resolução: Cálculo do número de mol (n1) para o recipiente que apresenta pressão igual a 2 atm: n1 = RT =
2 x 0,2 0,40 0,40
0,08206 x (27+273)
= 0,08206 x 300 = 24,618 = 1,62 x 10−2 mol

pV
Cálculo do número de mol (n2) para o recipiente que apresenta pressão igual a 4 atm: n2 = RT =
4 x 0,2 0,80 0,80
0,08206 x (27+273)
= 0,08206 x 300 = 24,618 = 3,25 x 10−2 mol

pV
Cálculo do número de mol (n3) para o recipiente que apresenta pressão igual a 6 atm: n2 = RT =
6 x 0,2 1,20 1,20
= = = 4,87 x 10−2 mol
0,08206 x (27+273) 0,08206 x 300 24,618

Cálculo do número de mol total (nTotal): nTotal = 1,62 x 10−2 mol + 3,25 x 10−2 mol + 4,87 x 10−2 mol =
9,74 x 10−2 mol
nTotal x R x T
Cálculo da pressão total do sistema, a partir da utilização da equação dos gases ideais: pTotal = V
=
9,74 x 10−2 x 0,08206 x 300
= 4,0 atm
0,60

Cálculo das pressões parciais:


1,62 x 10−2 mol
P1 = X1 x ptotal =( ) x 4,0 atm = 0,66 atm
9,74 x 10−2 mol

3,25 x 10−2 mol


P2 = X 2 x ptotal = ( ) x 4,0 atm = 1,33 atm
9,74 x 10−2 mol

4,87 x 10−2 mol


P3 = X 3 x ptotal = ( ) x 4,0 atm = 2,00 atm
9,74 x 10−2 mol

171
1600 FÍSICO-QUÍMICA APLICADA EXERCÍCIOS COMENTADOS - IME – ITA – OLIMPÍADA

Questão 352 – (IME) Um hidreto gasoso tem fórmula empírica XH3 (massa molar de X = 13 g.mol-1) e massa específica
de 6,0 g.L-1 numa dada condição de temperatura e pressão. Sabendo-se que, na mesma temperatura e pressão, 1,0
L de O2 gasoso tem massa de 3,0 g, pode-se afirmar que a fórmula molecular do hidreto é:

a) X0,5H1,5
b) XH3
c) X4H12
d) X2H6
e) X6H18

Resolução: Alternativa C.

(XH3)n: Massa molar do referido composto: <MM> = 16.n e d = 6 g.L-1

3
Analisando para o gás oxigênio: p x 1 = 32 x RT (Equação 1)

𝑚
Analisando para o hidreto: p x V = <𝑀𝑀> x R x T (Equação 2)

px1 (3⁄32) x R x T
Dividindo a equação (1) pela equação (2): =
pxV (m⁄<MM>)x R x T

1 (3⁄ ) x R x T
= m 32
V ( ⁄
16n) x R x T

m 3n
=
V 2
3𝑛
Sabendo que a razão entre a massa e o volume corresponde a densidade d = m/V, o valor de n será: 𝑑 = 2

3𝑛
6=
2

𝑛=4

Questão 353 – (IME) Num garrafão de 3,5 L de capacidade, contendo 1,5 L de solução 1,0 mol.L-1 de ácido sulfúrico,
introduzem-se 32,7 g de aparas de zinco; fecha-se rapidamente com rolha de borracha. Supondo que a temperatura
do ambiente onde essa perigosa experiência está sendo feita seja de 20°C, o incremento máximo de pressão interna
(P) do frasco será de:

a) 0,41 atm
b) 3,4 atm
c) 5,6 atm
d) 6,0 atm
e) 12,0 atm

172
1600 FÍSICO-QUÍMICA APLICADA EXERCÍCIOS COMENTADOS - IME – ITA – OLIMPÍADA

Resolução: Alternativa D.

Determinação do reagente limitante, calculando o número de mol para cada reagente químico:

m 32,7
Para o zinco: nZn = <MM>
Zn
= 65,4 = 0,50 mol (Reagente limitante)
Zn

Para o ácido sulfúrico: nH2 SO4 = [H2 SO4 ] x Vsolução = 1,0 x 1,50 = 1,50 mol

Cálculo do número de mol de gás Hidrogênio: Zn(s) + H2 SO4(aq) → ZnSO4(aq) + H2(g)

1 mol de Zn(s) − − − − − 1 mol de H2(g)


0,50 mol de Zn(s) − − − − nH2
nH2 = 0,50 𝑚𝑜𝑙
O volume de gás hidrogênio produzido é igual a 2,0 litros (3,5 L − 1,5L).

nxRxT
Cálculo da pressão de gás hidrogênio, considerando comportamento ideal: p= V
=
0,50 x 0,08206 x (20+273)
2,0
= 6,0 atm

Questão 354 – (ITA) Um recipiente fechado, mantido a volume e temperatura constantes, contém a espécie química
X no estado gasoso a pressão inicial Po. Esta espécie decompõe-se em Y e Z de acordo com de acordo com a
seguinte equação química: X(g) → 2 Y(g) + ½ Z(g). Admita que X, Y e Z tenham comportamento de gases ideais.
Assinale a opção que apresenta a expressão correta da pressão (P) no interior do recipiente em função do andamento
da reação, em termos da fração α de moléculas de X que reagiram.

a) P = [1+(1/2).α].Po
b) P = [1+(2/2).α].Po
c) P = [1+(3/2).α].Po
d) P = [1+(4/2).α].Po
e) P = [1+(5/2).α].Po

Resolução: Alternativa C. Equação química de decomposição química: X(g) ⇄ 2 Y(g) + ½ Z(g)

Base de cálculo: Pressão inicial (Po) para a espécie gasosa X e utilizando a tabela de equilíbrio químico:

X(g) ⇄ 2Y(g) ½ Z(g)


Início P0 0 0
Reage P0α 2 P0.α ½ P0.α
Equilíbrio P0 - P0α 2 P0.α ½ P0.α

Determinação da pressão total (𝑃𝑡𝑜𝑡𝑎𝑙 ), a partir do equilíbrio observado na tabela de equilíbrio.


Ptotal = PX + PY + PZ

173
1600 FÍSICO-QUÍMICA APLICADA EXERCÍCIOS COMENTADOS - IME – ITA – OLIMPÍADA

1
Ptotal = P0 − P0 α + 2P0 α + P0 α
2
1 3
Ptotal = P0 + P0 α + P0 α = P0 x [1 + ( ) α]
2 2
Questão 355 – (OLIMPÍADA DE QUÍMICA DO RIO DE JANEIRO) Um cilindro com 8,0 litros de capacidade contém
160 g de gás oxigênio a 27 °C. Abrindo-se a válvula do cilindro, deixou-se escapar o gás até que a pressão fosse
reduzida a 9 atm. Supondo a constante universal dos gases ideais igual a 0,08, volume e temperatura constante, qual
a massa, em gramas, de gás oxigênio que foi liberada do recipiente?

a) 8
b) 16
c) 32
d) 64
e) 96

Resolução: Alternativa D.

Considerando que o processo é isovolumétrico (volume constante) e isotérmico (temperatura constante), temos a
seguinte situação. Situação I: m1 = 160g; V1 = 8 L e T1 = 27 + 273 = 300 K

160
nRT ( ) x 0,08 x (27+273)
Cálculo da pressão inicial para a situação I: pI = V
= 32
8
= 15 atm

m
Relação entre a pressão e a massa do composto gasoso: pV = <𝑀𝑀> RT

V x < MM > m
=
⏟ 𝑅𝑥𝑇 𝑝
𝑐𝑜𝑛𝑠𝑡𝑎𝑛𝑡𝑒

mI mII
Cálculo da massa no estado final (mII): =
pI pII

160 mII 160 x 9


15
= 9
→ mII = 15
= 96 𝑔

Logo a massa final de oxigênio que escapa do recipiente é: mfinal = 160 g − 96 g = 64 g

Questão 356 – (OLIMPÍADA DE QUÍMICA DO RIO DE JANEIRO) Um caminhão com tanque criogênico transportou
510 kg de um gás armazenado sob pressão de 20 atm e temperatura aproximada de 23 °C negativos. O tanque deste
caminhão consiste num cilindro com volume de 30 m3. Considere a constante dos gases ideais igual a 0,08. Com
base nos dados fornecidos, o gás transportado por este caminhão possui a fórmula:

a) NH3
b) CO2
c) C2H2
d) O2

174
1600 FÍSICO-QUÍMICA APLICADA EXERCÍCIOS COMENTADOS - IME – ITA – OLIMPÍADA

e) Ar

Resolução: Alternativa A.

mxRxT
Cálculo da massa molecular, a partir da equação dos gases ideais: < MM > = pxV
=
510000 x 0,08 x (−23+273) 10200000 g
20 x 30000
= 600000
= 17 mol (𝑁𝐻3 , 𝑎𝑚ô𝑛𝑖𝑎)

Questão 357 – (OLIMPÍADA DE QUÍMICA DO RIO DE JANEIRO) Uma amostra de um gás X e outra de um gás Z
possuem mesmo volume, pressão e temperatura. A amostra do gás X tem massa igual a 1,60 g e a amostra do gás
Z tem uma massa 3,5 g. Supondo que X é o gás oxigênio, qual é massa molar, em g.mol-1, do gás Z?

a) 16
b) 32
c) 35
d) 70
e) 90

Resolução: Alternativa D.

pV
Propriedades do gás X: = nX x R
T

pV
Propriedades do gás Z: T
= nZ x R

Considerando que os gases, conforme enunciado, apresentam a mesma pressão, volume e temperatura, temos:
nX x R = nZ x R

mX mZ
xR = xR
< MM >Z < MM >Z

1,60 3,50
=
32,0 < MM >Z

32,0 x 3,50 g
< MM >Z = = 70
1,60 mol

Questão 358 – (ITA) Supondo um comportamento de gás ideal, assinale a opção que indica, aproximadamente, a
massa em gramas, de 1,0 L de C3H8 nas CNTP:

a) 2 x 10-3 g
b) 0,50 g
c) 2,0 g
d) 22,4 g
e) 44,0 g

175
1600 FÍSICO-QUÍMICA APLICADA EXERCÍCIOS COMENTADOS - IME – ITA – OLIMPÍADA

Resolução: Alternativa C.

p x V x <MM>
Cálculo da massa de gás propano, a partir da utilização da equação dos gases ideais: m = RxT
=
1,0 x 1,0 x 44 44
0,08206 x 273
= 22,40 = 1,96 g

Questão 359 – (ITA) 2,7 g de alumínio são dissolvidos em 500 ml de uma solução aquosa 1,00 mol.L -1 em ácido
clorídrico. Todo o hidrogênio produzido é recolhido. Após a secagem, o volume de hidrogênio à pressão de 1 atm e
25C é:

a) 1,2 litros
b) 1,6 litros
c) 2,4 litros
d) 3,6 litros
e) 12 litros

Resolução: Alternativa D.
Para a resolução deste problema será necessário determinar o reagente limitante. Equação química: Al(s) +
3
3 HCl(aq) → AlCl3(aq) + 2 H2(g)
2,70
nAl = = 0,10 mol (Reagente limitante)
27
0,500 x 1,0
nHCl = nHCl x [HCl] = 3
= 0,167 mol

Comparando o número de mol de cada reagente, o alumínio é o reagente limitante.


3
1 mol de Al(s) − − − − − − − mol de H2(g)
2
2,70
mol de Al(s) − − − − − nH2(g)
27
nH2(g) = 0,15 mol
nH xRxT
2(g)
Cálculo do volume de gás hidrogênio produzido, a partir da equação dos gases ideais: VH2(g) = p
=
0,15 x 0,08206 x (25+273)
1,0
= 3,66 L

Questão 360 – (OLIMPÍADA DE QUÍMICA DO RIO DE JANEIRO) Amostras de He, O2 e N2 contêm, cada uma, 2,00
g. Suponha que os gases sejam colocados conjuntamente em um recipiente de 15,0 L a 100°C. Considere o
comportamento ideal e calcule a pressão total em atmosfera.

a) 2,13 atm
b) 0,620 atm
c) 1,29 atm
d) 2,58 atm
e) 2,21 atm

176
1600 FÍSICO-QUÍMICA APLICADA EXERCÍCIOS COMENTADOS - IME – ITA – OLIMPÍADA

Resolução: Alternativa C.

2,0 2,0
Cálculo do número de mol para cada espécie gasosa: nHe = = 0,50 mol; nO2 = = 0,0625 mol e
4,0 32,0
2,0
nN2 = 28,0 = 0,0714 mol

Número de mol total: ntotal = nHe + nO2 + nN2 = 0,50 mol + 0,0625 mol + 0,0714 mol = 0,6339 mol

nxRxT 0,6339 x 0,08206 x (100+273)


Aplicando a equação dos gases ideais: p = 𝐕
= 15,0
= 1,29 atm

Questão 361 – (IME) Um recipiente de paredes rígidas, contendo apenas ar, aberto para a atmosfera, é aquecido de
27ºC a 127ºC. Calcule a percentagem mássica de ar que saiu do recipiente, quando atingido o equilíbrio final.

a) 79%
b) 75%
c) 30%
d) 25%
e) 21%

Resolução: Alternativa D.

Estudando a situação inicial: p1 ; V1 ; T1 = 27 + 273 = 300K

Estudando a situação final: p2 ; V2 ; T2 = 127 + 273 = 400K

A partir da equação dos gases ideais e considerando que tanto a pressão quanto o volume sejam constantes, temos:

p1 x V1 n1 x R x T1
=
p2 x V2 n2 x R x T2

p1 x V1 n1 x R x 300
=
p2 x V2 n2 x R x 400

n2 300
= = 0,75 (75%)
n1 400

O percentual de massa de ar que saiu é igual a 100% - 75% = 25%

Questão 362 – (ITA) Um recipiente de 240 litros de capacidade contém uma mistura dos gases ideais hidrogênio e
dióxido de carbono, a 27°C. Sabendo que a pressão parcial do dióxido de carbono é três vezes menor que a pressão
parcial do hidrogênio e que a pressão total da mistura gasosa é de 0,82 atmosferas, assinale a alternativa que
apresenta, respectivamente, as massas de hidrogênio e de dióxido de carbono contidas no recipiente:

a) 2 g e 44 g
b) 6 g e 44 g
c) 8 g e 88 g

177
1600 FÍSICO-QUÍMICA APLICADA EXERCÍCIOS COMENTADOS - IME – ITA – OLIMPÍADA

d) 12 g e 88 g
e) 16 g e 44 g

Resolução: Alternativa D.

1
Dados do problema: PCO2 = 3 x PH2 e PTOTAL = PCO2 + PH2 = 0,82 atm

1
Resolvendo as equações apresentadas acima, temos: PCO2 = 3 x PH2

1
x PH2 + PH2 = 0,82 atm
3

PH2 = 0,615 atm

Cálculo da pressão parcial de dióxido de carbono (CO2): PCO2 + PH2 = 0,82 atm

PCO2 + 0,615 atm = 0,82 atm

PCO2 = 0,205 atm

Cálculo da pressão total (pTOTAL): PH2 = 0,615 = X H2 x PTOTAL

nH2
0,615 = n x PTOTAL (Equação I)
TOTAL

PCO2 = 0,615 = X CO2 x PTOTAL

nCO2
0,205 = n x PTOTAL (Equação II)
TOTAL

Cálculo do número de mol total (nTOTAL), utilizando a equação dos gases ideais: 0,82 x 240 =
nTOTAL x 0,08206 x 300

nTOTAL = 8,00 mol

nH2
Substituindo os valores na equações I, temos: 0,615 = n x PTOTAL
TOTAL

m H2
< MM >H2
0,615 = x PTOTAL
nTOTAL

mH2
2,0
0,615 = x 0,82
8

mH2 = 12,0 g

nCO2
Substituindo os valores na equações II, temos: 0,205 = n x PTOTAL
TOTAL

178
1600 FÍSICO-QUÍMICA APLICADA EXERCÍCIOS COMENTADOS - IME – ITA – OLIMPÍADA

mCO2
44,0
0,205 = x 0,82
8,0

mCO2 = 88,0 g

Questão 363 – (ITA) Uma porção de gás pode ser aquecida sob pressão constante ou sob volume constante. A
questão é saber como irá variar a densidade do gás em cada uma dessas maneiras de aquecimento:

Pressão Constante Volume Constante


a) aumenta não varia
b) aumenta diminui
c) não varia aumenta
d) diminui diminui
e) diminui não varia

Resolução: Alternativa E.

𝑝 𝑥 <𝑀𝑀>
Sabendo que a relação entre a densidade de um determinado gás é dado por 𝑑 = 𝑅𝑥𝑇
, então analisando cada

proposta colocada pelo problema, temos:

Aumentando a temperatura com a pressão constante, a densidade da amostra gasosa diminui.

Aumentando a temperatura com o volume constante, tanto a massa quanto o volume constantes, a densidade torna-
se constante.

Questão 364 – (GRILLO) Considere que 1,0 mol de gás cloro (Cl2) esteja confinado dentro de um recipiente de volume
igual a 22,4 litros a uma temperatura igual a 0°C. Diante destes dados, calcule o valor correto do fator de
compressibilidade. Parâmetros de van der Waals: a = 6,49 L².atm.mol-2 e b = 5,62 x 10-2 L.mol-1.

V 22,4 L
Resolução: Cálculo do volume molar do gás cloro: Vmolar = = = 22.4 L. mol−1
n 1,0 mol

6,49
Cálculo da pressão utilizando a equação de van der Waals (preal): {preal + (22,4)²
} x {22,4 − 5,62 x 10−2 } =

0,08206 x 273

preal = 0,99 atm

PReal 0,99 atm


Cálculo do fator de compressibilidade (Z): Z = PIdeal = 1,00 atm = 0,99

Questão 365 – (GRILLO) A uma temperatura igual a 500 K e sob uma pressão de 1 atm, a dissociação do cloreto de
nitrosila, 2 NOCl(g) → 2 NO(g) + Cl2(g), resulta em uma mistura gasosa que apresenta densidade igual a 1,302 kg.m-3.
A partir destes dados, determine o grau de dissociação do cloreto de nitrosila, nesta mesma temperatura.

179
1600 FÍSICO-QUÍMICA APLICADA EXERCÍCIOS COMENTADOS - IME – ITA – OLIMPÍADA

Resolução: Base de cálculo: n(mol) inicial de cloreto de nitrosila e utilizando a tabela de quilíbrio químico para a
seguinte equação: 2 NOCl(g) → 2 NO(g) + Cl2(g).

2 NOCl(g) 2 NO(g) Cl2(g)


Início n 0 0
Reage 2nα 2nα nα
Equilíbrio n - 2nα 2nα nα

Cálculo do número de mol total da mistura gasosa: nTotal = nNOCl + nNO + nCl2 = n - 2nα + 2nα + nα = n + nα = n.(1 + α)

p x <MM>
Cálculo do grau de dissociação (α) do cloreto de nitrosila: d = (1+ α)x R x T

1 x 65,5
1,302 =
(1 + α) x 0,08206 x (227 + 273)

α = 0,23 (23%)

Questão 366 – (GRILLO) Um gás é usado como um fluido refrigerante em uma unidade de ar-condicionado. Uma
tabela de parâmetros de van der Waals mostra que: a = 16,2 atm.L².mol-2 e b = 8,4x10-2 L.mol-1. Calcule a pressão
quando 1,50 mol foram confinados em um volume de 5,00 litros na temperatura de 273K.

16,2 5,0
Resolução: Aplicando a equação de van der Waals: {𝑝𝑟𝑒𝑎𝑙 + 5,0 } 𝑥 {1,50 − 8,40 𝑥 10−2 } =
( )²
1,50

0,08206 𝑥 (0 + 273)

𝑝𝑟𝑒𝑎𝑙 = 5,44 𝑎𝑡𝑚

Através do valor da pressão calculada e da temperatura fornecida, observa-se uma característica predominante para
o comportamento do gás real  Alta pressão e baixa temperatura.

Questão 367 – (GRILLO) Um determinado recipiente apresenta uma mistura gasosa constituído por: 320 miligramas
de metano; 175 miligramas de argônio e 225 miligramas de neônio. A pressão parcial do neônio, a 300 K é de
aproximadamente 8,87 kPa. Calcule a pressão total da mistura e o volume da mistura.

mCH 320 x 10−3


Resolução: Cálculo do número de mol de metano (CH4): nCH4 = <MM> 4 = 16
= 0,020 mol
CH4

mAr 175 x 10−3


Cálculo do número de mol de Argônio (Ar): nAr = <MM>Ar
= 40
= 0,0044 mol

mNe 225 x 10−3


Cálculo do número de mol de Neônio (Ne): nNe = = = 0,01125 mol
<MM>Ne 20

Cálculo do número de mol total (nT) da mistura gasosa: nT = 0,020 + 0,0044 + 0,01125 = 0,0356 mol

nNe
Cálculo da pressão total (pT) do sistema, a partir da pressão parcial do Neônio: PNe = X Ne x PT = n x PT
total

180
1600 FÍSICO-QUÍMICA APLICADA EXERCÍCIOS COMENTADOS - IME – ITA – OLIMPÍADA

0,01125
8,87 = x PT
0,0356

PT = 28,07 kPa

Cálculo do volume da mistura: 280170 x VTotal = 0,0356 x 8,314 x 300

VTotal = 3,17 x 10−4 m³

Questão 368 – (GRILLO) A partir das constantes de van der Waals do gás etano (a = 5,507 dm³.atm.mol-1; b = 3,19
x 10-2 dm³.mol-1), calcule de forma aproximada o valor do raio desta molécula gasosa, em nanômetros (nm).

b
Resolução: Sabendo que para a determinação do volume supostamente esférica, temos que V = , onde V é o
N
4πR³
volume da partícula esférica (V = 3
), b é a constante de van der Waals e N a constante de Avogadro.

4πR³ 3,19 x 10−2


Cálculo do raio da partícula gasosa: 3
= 6,02 x 1023

3
R = √1,26 x 10−26 = 2,33 x 10−9 (2,33 nm)

Questão 369 – 9,10 litros de monóxido de carbono nas CNTP são injetados em um cilindro indeformável,
apresentando uma capacidade de aproximadamente 100 litros. Neste mesmo recipiente foram colocados mais 26,6
litros de gás hidrogênio nas CNATP. A mistura, assim obtida, é mantida a uma temperatura constante e igual a 300
K. A partir destas informações, assinale a alternativa que apresenta o valor aproximado da massa específica, em g.L-
1.

a) 0,12
b) 0,14
c) 0,16
d) 0,18
e) 0,20

Resolução: Alternativa B.

Para o cálculo da massa específica, será necessário primeiramente calcular a pressão do sistema.

pV 1 x 9,1
Cálculo do número de mol de monóxido de carbono (CO): nCO = RT = 0,08206 x 273 = 0,41 mol

pV 1 x 26,6
Cálculo do número de mol de H2: nH2 = RT = 0,08206 x (25+ 273) = 1,09 mol

Cálculo do número de mol total: nTOTAL = nH2 + nCO = 1,09 + 0,41 = 1,50 mol.

1,50 x 0,08206 x (25+273)


Cálculo da pressão total do recipiente: pTotal = 100
= 0,369 atm

181
1600 FÍSICO-QUÍMICA APLICADA EXERCÍCIOS COMENTADOS - IME – ITA – OLIMPÍADA

Cálculo da massa molar da mistura: X CO x < MM >CO + X H2 x < MM >H2 = X mistura x < MM >mistura

0,41 1,09
x 28 + x 2,0 = 1,0 x < MM >mistura
1,50 1,50

< MM >mistura = 9,11 g. mol−1

pT x <MM>mistura 0,369 x 9,11 0,369 x 9,11


Cálculo da massa específica (μ): μ = RxT
= 0,08206 x (27+273) = 0,08206 x 300 = 0,136 g. L−1

Questão 370 – (ITA – MODIFICADO) Calcule a densidade do hidrogênio gasoso a 25°C e pressão de 1,5 atmosferas.

a) 0,123 g.L-1
b) 1,23 g.L-1
c) 12,3 g.L-1
d) 123 g.L-1
e) 1230 g.L-1

p x<MM>
Resolução: Alternativa A. Cálculo da densidade da amostra gasosa nas condições apresentadas: d = =
RxT
1,50 x 2,0 3,0 g
0,08206 x (25+273)
= 0,08206 x 298 = 0,123 L

Questão 371 – Um container é dividido em dois compartimentos. O compartimento A apresenta um gás ideal A
apresentando uma temperatura de 400 K e pressão de 5 atmosferas. Já o compartimento B está fechado
apresentando um gás ideal a 400 K e 8 atmosferas. A partição entre os compartimentos é removida e os gases são
misturados. A fração molar do gás A na mistura é igual a XA = 0,58. O volume total dos compartimentos é igual a 29
litros. Assinale a alternativa que apresenta os volumes dos compartimentos A e B, respectivamente.

a) 20 litros e 9 litros
b) 9 litros e 20 litros
c) 19 litros e 10 litros
d) 10 litros e 19 litros
e) Nenhuma das respostas anteriores.

Resolução: Alternativa A.

nA
Estudando o compartimento A: X A = nT

nA
0,58 =
nT

nA = 0,58 x nT

pA x VA = nA x R x TA

182
1600 FÍSICO-QUÍMICA APLICADA EXERCÍCIOS COMENTADOS - IME – ITA – OLIMPÍADA

5 x VA = 0,58 x nT x R x TA (Equação A)

Estudando o compartimento B: X A + X B = 1

XB = 1 − XA

X B = 1 − 0,58 = 0,42

nB
XB =
nT

nB
0,42 =
nT

nB = 0,42 x nT

pB x VB = nB x R x TB

8 x VB = 0,42 x nT x R x TB (Equação B)

8xV 0,42 x R x T
Dividindo a equação 2 pela equação 1, temos: 5 x VB = 0,58 x R x TB
A A

8 x VB 0,42
Sendo TA = TB (processo isotérmico), temos: =
5 x VA 0,58

VA = 2,21 x VB

Sabendo que a soma dos volumes (VA + VB) é igual a 29 litros, temos: VA + VB = 29,0

VA = 2,21 x VB

Resolvendo o sistema: VA = 19,96 L e VB = 9,03 L

Questão 372 – (ITA) Por ocasião do jogo Brasil versus Bolívia disputado em La Paz, um comentarista esportivo
afirmou que: “Um dos maiores problemas que os jogadores da seleção brasileira de futebol terão que enfrentar é o
fato de o teor de oxigênio no ar, em La Paz, ser cerca de 40% menor do que aquele ao nível do mar”. Lembrando que
a concentração de oxigênio nível do mar é aproximadamente 20% (v/v) e supondo que no dia em que o comentarista
fez esta afirmação a pressão atmosférica em La Paz fosse igual, aproximadamente, a 450 mmHg, qual das opções
abaixo contém a afirmação que mais se aproxima daquilo que o comentarista poderia ter dito?

a) a concentração de oxigênio no ar é cerca de 12% (v/v)


b) a fração molar do oxigênio no ar é cerca de 0,12
c) a pressão parcial do oxigênio no ar é aproximadamente expressa por: (0,20 x 760 mmHg x 0,60)
d) a pressão parcial do oxigênio no ar é cerca de 152 mmHg
e) a pressão parcial do oxigênio no ar é aproximadamente expressa por: (0,20 x 760 mmHg x 0,40)

183
1600 FÍSICO-QUÍMICA APLICADA EXERCÍCIOS COMENTADOS - IME – ITA – OLIMPÍADA

Resolução: Alternativa C.

Sendo a pressão do oxigênio no ar, em La Paz, cerca de 40% menor do que ao nível do mar, a pressão lá será apenas
60% do que é a pressão ao nível do mar, ou seja: pressão do O2 em La Paz = (100 – 40)% da pressão ao nível do
mar.

Questão 373 – (ITA) Uma amostra de 7,5 gramas de um hidrocarboneto ocupa um volume de 5,6 litros nas CNTP,
podemos concluir que esse hidrocarboneto corresponde a um:

a) Alcano
b) Alceno
c) Alcino
d) Ciclano
e) Cicleno

Resolução: Alternativa A.

mxRxT 7,50 x 0,08206 x 273


Cálculo da massa molar do hidrocarboneto desconhecido: < MM ≥ = pxV
= 1 x 5,6
=

30,00 𝑔. 𝑚𝑜𝑙 −1. Determinação do hidrocarboneto:

✓ Alcano (CnH2n+2): CnH2n+2 = 30


12n + 2n + 2 = 30

n=2

✓ Alceno (CnH2n-2): CnH2n-2 = 30


12n + 2n - 2 = 30

n = 2,28

✓ Alcino (CnH2n): CnH2n = 30


12n + 2n = 30

n = 2,14

✓ Ciclano (CnH2n): CnH2n = 30


12n + 2n = 30

n = 2,14

✓ Cicleno (CnH2n-2): CnH2n = 30


12n + 2n -2 = 30

n = 2,28

184
1600 FÍSICO-QUÍMICA APLICADA EXERCÍCIOS COMENTADOS - IME – ITA – OLIMPÍADA

Questão 374 – (ITA) A que temperatura deve ser aquecido um frasco aberto para expulsar a metade da massa de
cloro que nele se encontra a 25°C?

a) 50°C
b) 75°C
c) 298°C
d) 323°C
e) 596°C

Resolução: Alternativa D.

𝑝 𝑥 𝑉 𝑥 < 𝑀𝑀 >
[ ] = mxT
R 𝑐𝑜𝑛𝑠𝑡𝑎𝑛𝑡𝑒

Situação I: P = constante; V = constante; R = constante; TI = 273 + 20 = 293K; mI = m

Situação II: P = constante; V = constante; R = constante; TII = ?; mII = m/2

Cálculo da temperatura II, através da equação combinada dos gases ideais: mI x TI = mII x TII

mI
mI x 298 = x TII
2

TII = 596 K (323°C)

Questão 375 – (IME) Assinale a alternativa correta.

a) Um veículo de testes para redução de poluição ambiental, projetado para operar entre – 40ºC e 50ºC,
emprega H2 e O2, os quais são estocados em tanques a 13 MPa. Pode-se afirmar que a lei dos gases ideais não é
uma aproximação adequada para o comportamento dos gases no interior dos tanques. (Dado: 1 atm = 101,3 kPa).
b) A pressão de vapor de um líquido independe da temperatura.
c) Um recipiente de 500 mL, inicialmente fechado e contendo um líquido em equilíbrio com seu vapor, é aberto.
Pode-se afirmar que a pressão de vapor do líquido aumentará.
d) Na equação pv = nRT, o valor numérico de R é constante e independe do sistema de unidades empregado.
e) De acordo com o princípio de Avogadro, pode-se afirmar que, dadas as condições de temperatura e pressão,
o volume molar gasoso depende do gás considerado.

Resolução: Alternativa A.

O modelo do gás ideal considera que o tamanho das partículas que compõem o gás é desprezível. As interações
intermoleculares entre estas são, também, desprezadas, uma vez que a distância entre as partículas é muito grande.
As interações, no gás ideal, devem-se apenas ás colisões entre as moléculas do gás. O elevado valor de pressão, de
aproximadamente 130 atm não permite que a hipótese do gás ideal seja adotada, pois, nessa situação, as interações
intermoleculares das partículas do gás não serão desprezíveis.

185
1600 FÍSICO-QUÍMICA APLICADA EXERCÍCIOS COMENTADOS - IME – ITA – OLIMPÍADA

Questão 376 – (OLIMPÍADA DE QUÍMICA DO RIO DE JANEIRO) O ácido sulfúrico é um dos agentes da chuva
ácida. Ao precipitar, a chuva ácida reage com monumentos de mármore (carbonato de cálcio), “destruindo-os”. Qual
o volume de gás produzido a 27,0°C e 1,00 atm quando 2,45 gramas de ácido sulfúrico precipitam na forma de chuva
ácida e reagem com estes monumentos?

a) 1,23 x 10³ mL
b) 615 mL
c) 560 mL
d) 111 mL
e) 55,3 mL

Resolução: Alternativa B.

mH SO4 2,45
Cálculo do número de mol de ácido sulfúrico: nH2 SO4 = <MM>2 = 98
= 0,025 mol
H2 SO4

Cálculo do número de mol de dióxido de carbono produzido: H2SO4 + CaCO3 → CaSO4 + H2O + CO2

1 mol de H2SO4 --------------- 1 mol de CO2

0,025 mol de H2SO4 ---------- nCO2

nCO2 = 0,025 mol.

nCO2 .R.T 0,025 x 0,08206 x 300


Cálculo do volume de dióxido de carbono produzido: VCO2 = p
= 1
=

0,615 litros (615 mL)

Questão 377 – (PETROBRÁS – ENGENHEIRO DE PROCESSAMENTO JÚNIOR) Os valores aproximados dos


coeficientes da Equação de Van der Waals para o metano são: a = 0,2 Pa.m 6.mol-1 e b = 4 x 10-5 m³.mol-1. Para 100
mol do gás ocupando o volume de 1 m³ a 300 K, a diferença, em kPa, entre a pressão calculada por essa relação e o
verificado para um gás ideal, nas mesmas condições R = 8,0 Pa.m3.mol-1.K-1, é:

a) -2
b) -1
c) 0
d) +2
e) +3

Resolução: Alternativa B.

V 1 m³
Cálculo do volume molar: Vmolar = = = 10−2 m³. mol−1
n 100 mol

0,2
Analisando o gás metano como real: {preal + (10−2 )²
} x {10−2 − 4,0 x 10−5 } = 8 x 300

186
1600 FÍSICO-QUÍMICA APLICADA EXERCÍCIOS COMENTADOS - IME – ITA – OLIMPÍADA

preal = 239 kPa

Analisando o gás metano como ideal: pideal x 1 = 1000 x 8 x 300

pideal = 240 kPa

Cálculo da variação de pressão (Δp): ∆p = preal − pideal = 239 kPa − 240 kPa = −1 kPa

Questão 378 – (ITA) Um reservatório de 30 litros contém gás nitrogênio diatômico, a temperatura ambiente de 20°C.
Seu medidor de pressão indica uma pressão de 3,00 atm. A válvula do reservatório é aberta momentaneamente e
uma certa quantidade de gás escapa para o meio ambiente. Fechada a válvula, o gás atinge novamente a temperatura
ambiente. O medidor de pressão do reservatório indica agora uma pressão de 2,40 atmosferas. Quantos gramas,
aproximadamente, de nitrogênio escaparam?

a) 10,50 g
b) 31 g
c) 15 g
d) 3g
e) 21 g

Resolução: Alternativa E.

Situação I: Gás confinado a uma pressão de 3,0 atmosferas.

p x V x<MM> 28 x 3 x 30
Cálculo da massa da espécie gasosa (mI): mI = RxT
= 0,08206 x 293 = 104,81 g

Situação II: Gás confinado a uma pressão de 2,40 atmosferas.

p x V x<MM> 28 x 2,40 x 30
Cálculo da massa (mII): mII = = = 84,85 g
RxT 0,08206 x 293

Situação III: Cálculo da massa de gás (N2) de escape: mN2 = mII – mI = 104,81g – 83,85g = 20,96 g

Questão 379 – (MESTRE JOÃO ROBERTO DA PACIÊNCIA NABUCO) Determine a temperatura a que se deve
aquecer um recipiente aberto para que saia metade da massa nele contida a 20ºC.

m
Resolução: Aplicando a equação dos gases ideais: pV = xRxT
<MM>

p x V x < MM >
{ } = massa x Temperatura
R constante

Situação I: P = constante; V = constante; TI = 273 + (20) = 293K; mI = m

Situação II: P = constante; V = constante; TII = ?; mII = m/2

Cálculo da temperatura II, através da equação combinada dos gases ideais: mI x TI = mII x TII

187
1600 FÍSICO-QUÍMICA APLICADA EXERCÍCIOS COMENTADOS - IME – ITA – OLIMPÍADA

mI
mI x 293 = x TII
2

TII = 586 K (313°C)

Questão 380 – (OLIMPÍADA DE QUÍMICA DO RIO DE JANEIRO) A que temperatura as moléculas de O2 teriam a
mesma velocidade média dos átomos de He a 27°C?

a) 216°C
b) 300°C
c) 1000°C
d) 2127°C
e) 2700°C

Resolução: Alternativa D. Informação do problema: c²O2 = c²He, em que c² é a velocidade média quadrática.
3 x R x TO2 3 x R x THe
cO2 2 = = 2
= cHe
< MM >O2 < MM >He
3 x R x TO2 3 x R x THe
=
< MM >O2 < MM >He
TO2 THe
=
< MM >O2 < MM >He
TO2 (27 + 273)
=
32 4
TO2 = 2400 K (2127°C)

Questão 381 – (ITA) Considere um recipiente de 320 litros, ao qual são adicionados gases ideais nas seguintes
condições:

I. Hélio: 30000 cm³ a 760 cmHg e 27°C


II. Monóxido de carbono: 250 litros a 1140 mmHg e -23°C
III. Monóxido de nitrogênio: 2 m³ a 0,273 atm e 0°C
Sabendo que a pressão total da mistura gasosa é de 4,5 atmosferas, assinale a opção que apresenta a pressão
parcial do hélio na mistura gasosa:

a) 0,1 atm
b) 0,2 atm
c) 0,5 atm
d) 1,0 atm
e) 2,0 atm

188
1600 FÍSICO-QUÍMICA APLICADA EXERCÍCIOS COMENTADOS - IME – ITA – OLIMPÍADA

Resolução: Alternativa D.

Cálculo do número de mol de cada participante gasoso, considerando comportamento ideal:

7600
Para o gás Hélio (He): 760
x 30 = nHe x 0,08206 x (27 + 273)

𝑛𝐻𝑒 = 12,19 𝑚𝑜𝑙

1140
Para o monóxido de carbono (CO): 760
x 250 = nCO x 0,08206 x (−23 + 273)

nCO = 18,28 mol

Para o gás monóxido de nitrogênio (NO): pideal x V = nNO x R x T

0,273 x 2000 = nNO x 0,08206 x (0 + 273)

nNO = 24,37 mol

Cálculo do número de mol total (nT): nT = nNO + nCO + nHe = 24,37 mol + 18,28 mol + 12,19 mol = 54,84 mol

12,19
Cálculo da pressão parcial do gás nobre Hélio: PHe = XHe x PTOTAL = 54,84 x 4,50 = 1,0 atm

Questão 382 – (ITA) Uma amostra de 4,4 gramas de um gás ocupa um volume de 3,1 litros a 10°C e 566 mmHg.
Assinale a alternativa que apresenta a razão entre as massas específicas deste gás e a do hidrogênio gasoso nas
mesmas condições de pressão e temperatura.

a) 2,2
b) 4,4
c) 10
d) 22
e) 44

Resolução: Alternativa D.

O problema coloca que os dois gases apresentam a mesma pressão e a mesma temperatura.

𝑝 𝑥 <𝑀𝑀>𝑥
𝑑𝑥 ( 𝑥 )
𝑅 𝑥 𝑇𝑥
𝑑𝐻2
= 𝑝𝐻 𝑥 <𝑀𝑀>𝐻
, onde X representa um gás hipotético. Para esta relação, a relação entre as densidades fica
( 2 2)
𝑅 𝑥 𝑇𝐻
2

dx <MM>x
reduzida da seguinte maneira: = .
dH2 <MM>H2

mxRxT
Cálculo da massa molar da espécie gasosa X (<MM>X): < MM >X = pideal x V

4,4 x 0,08206 x (10 + 273)


< MM >X = = 44,26 g. mol−1
566
( ) x 3,1
760

189
1600 FÍSICO-QUÍMICA APLICADA EXERCÍCIOS COMENTADOS - IME – ITA – OLIMPÍADA

d <MM> 44,26
Cálculo da razão entre as densidades: d x = <MM> x = 2,00
= 22,13
H2 H2

Questão 383 – (ITA) A 25ºC, uma mistura de metano e propano ocupa um volume (V), sob uma pressão total de
0,080 atm. Quando é realizada a combustão desta mistura e apenas dióxido de carbono é coletado, verifica-se que a
pressão desse gás é de 0,12 atm, quando este ocupa o mesmo volume (V) e está sob a mesma temperatura da
mistura original. Admitindo que os gases têm comportamento ideal, assinale a opção que contém o valor correto da
concentração, em fração em mol, do gás metano na mistura original.

a) 0,01
b) 0,25
c) 0,50
d) 0,75
e) 1,00
Resolução: Alternativa D. Informação do problema: metano + propano = 0,080 atm

Segundo Avogadro, quando dois gases distintos apresentam a mesma temperatura, mesmo volume e mesma
pressão, os gases irão apresentar o mesmo número de mol. Logo: nm + np = 0,080

Equação química: CH4 + 2 O2 → CO2 + H2O

1 mol de CH4 ---------- 1 mol de CO2


nCH4 --------------------- nCO2
nCH4 = nCO2

Equação química: C3H8 + 5 O2 → 3 CO2 + 4 H2O

1 mol de C3H8 ---------- 3 mol de CO2


nC3H8 ---------------------- nCO2
nCO2 = 3 x nC3H8

3 x nC3H8 + nCO2 = 0,12

Resolvendo o sistema de equações apresentadas anteriormente, temos: 𝑛𝐶𝐻4 + 𝑛𝐶3 𝐻8 = 0,08

𝑛𝐶𝐻4 + 3 𝑥 𝑛𝐶3 𝐻8 = 0,12

𝑛𝐶𝐻4 = 0,06

𝑛𝐶3 𝐻8 = 0,02

nCH4 0,06 0,06


Cálculo da fração molar do gás metano: X CH4 = n = 0,06+0,02 = 0,08 = 0,75
CH4 +nC3 H8

190
1600 FÍSICO-QUÍMICA APLICADA EXERCÍCIOS COMENTADOS - IME – ITA – OLIMPÍADA

Questão 384 – (U.S. NATIONAL CHEMISTRY OLYMPIAD) Uma mistura de 0,50 mol de H2 e 1,3 mol de Ar encontra-
se em um recipiente fechado de 4,82 litros. Se a temperatura da mistura é de 50°C, qual é a pressão parcial de H2 na
mistura?

a) 1,5 atm
b) 2,8 atm
c) 7,2 atm
d) 9,9 atm

Resolução: Alternativa B.

Mistura gasosa: 0,50 mol de H2 + 1,3 mol de Ar


Cálculo do número de mol total (nT): nT = 0,5 + 1,3 = 1,8 mol.

Cálculo da pressão total da mistura gasosa:

pTOTAL x 4,82 = 1,80 x 0,08206 x (50 + 273)

pTOTAL = 9,90 atm

Cálculo da pressão parcial do gás hidrogênio: PH2 = X H2 . PT

nH2 0,50
PH2 = x PT = x 9,90 atm = 2,75 atm
nT 1,80

Questão 385 – (ITA) Calcular a massa de gás hélio, contida num balão, sabendo-se que o gás ocupa um volume
igual a 5,0 cm3 e está a uma temperatura de - 23°C e a uma pressão de 30 cmHg.

a) 0,186 mg
b) 0,46 mg
c) 0,96 mg
d) 0,186 mg
e) 0,385 mg

Resolução: Alternativa E.

m
Cálculo da massa do gás Hélio: pideal x V = <MM> x R x T

300 −3
pideal x V x < MM > (760) x 5,0 x 10 x 4
m= = = 3,85 x 10−4 g (0,385 mg)
RxT 0,08206 x 250

191
1600 FÍSICO-QUÍMICA APLICADA EXERCÍCIOS COMENTADOS - IME – ITA – OLIMPÍADA

Questão 386 – (ITA) Considere o volume de 5,0 L de uma mistura gasosa contendo 20% (V/V) do isótopo 40 do
argônio e 80% (V/V) do isótopo 20 do neônio. Na temperatura de 273 K, a mistura exerce a pressão de 20 atm. A
quantidade (em mol) de argônio nesta mistura:

a) 2 / 22,4
b) 10 / 22,4
c) 20 / 22,4
d) 50 / 22,4
e) 100 / 22,4

Resolução: Alternativa D.

Convertendo o volume para as Condições Normais de Temperatura e Pressã e utilizando a equação combinada dos
pI x VI pII x VII
gases ideais, temos: TI
= TII

20 x 5,0 1 x VII
=
(273 + 273) 273

100 1 x VII
=
546 273

VII = 50 L

Cálculo do número de mol (n):

1 mol ---------- 22,4 Litros


n --------------- 50 Litros
50
n = (22,4) mol
20 50 10
Cálculo do número de mol de Argônio: nAr = x mol = mol
100 22,4 22,4

Questão 387 – (ITA) A pressão total do ar no interior de um pneu era de 2,30 atmosferas quando a temperatura do
pneu era de 27°C. Depois de ter rodado um certo tempo com este pneu, mediu-se novamente sua pressão e verificou-
se que está agora era de 2,53 atmosferas. Supondo variação de volume do pneu desprezível, uma nova temperatura
será:

a) 29,7°C
b) 57,0°C
c) 33°C
d) 330°C
e) Nenhuma das respostas anteriores

192
1600 FÍSICO-QUÍMICA APLICADA EXERCÍCIOS COMENTADOS - IME – ITA – OLIMPÍADA

Resolução: Alternativa B.

Utilizando a equação combinada dos Gases Ideais e considerando que o volume é desprezível, temos que:

Situação I: PI = 2,30 atm; VI = Vdesprezível; TI = 300K

Situação II: PII = 2,53 atm; VII = Vdesprezível; TII

pI .VI pII .VII


Cálculo da temperatura II através da equação combinada dos Gases Ideais: TI
= TII

pI pII
=
TI TII

2,30 2,53
= → TII = 330 K (330 − 273 = 57°C)
300 TII

Questão 388 – (ITA) A figura mostra cinco curvas de distribuição de velocidade molecular para diferentes gases (I, II,
III, IV e V) a uma dada temperatura.

Assinale a opção que relaciona corretamente a curva de distribuição de velocidade molecular a cada um dos gases.

a) I = H2; II = He; III = O2; IV = N2; V = H2O


b) I = O2; II = N2; III = H2O; IV = He; V = H2
c) I = He; II = H2; III = N2; IV = O2; V = H2O
d) I = N2; II = O2; III = H2; IV = H2O; V = He
e) I = H2O; II = N2; III = O2; IV = H2; V = He

Resolução: Alternativa B.

Segundo a Lei de Graham, a velocidade de efusão e a de difusão dos gases é proporcional às respectivas massas
𝑣1 <𝑀𝑀>
molares, ou seja: = √ 2
. A ordem crescente das velocidades moleculares será então: H2 (2 g.mol−1) > He (4
𝑣2 <𝑀𝑀> 1

g.mol−1) > H2O (18 g.mol−1) > N2 (28 g.mol−1) > O2 (32 g.mol−1). Note que as velocidades estão marcadas no eixo das
abscissas e não nas ordenadas.

193
1600 FÍSICO-QUÍMICA APLICADA EXERCÍCIOS COMENTADOS - IME – ITA – OLIMPÍADA

Questão 389 – (ITA) Um vaso de pressão com volume interno de 250 cm3 contém gás nitrogênio (N2) quimicamente
puro, submetido a temperatura constante de 250ºC e pressão total de 2,0 atm. Assumindo que o N2 se comporta como
gás ideal, assinale a opção correta que representa os respectivos valores numéricos do número de moléculas e da
massa específica, em kg.m-3, desse gás quando exposto às condições de pressão e temperatura apresentadas.

a) 3,7 x 1021 e 1,1


b) 4,2 x 1021 e 1,4
c) 5,9 x 1021 e 1,4
d) 7,2 x 1021 e 1,3
e) 8,7 x 1021 e 1,3

Resolução: Alternativa D.

ptotal x V 2,0 x 0,250


Cálculo do número de mol de gás nitrogênio (nN2): nN2 = RxT
= 0,08206 x (250+273) = 1,16 x 10−2 mol

6,02 x 1023
Cálculo do número de moléculas (X) de gás nitrogênio (N2): X = 1,16 x 10−2 mol x 1 mol
=

7,01 x 1021 moléculas

p x <MM> 2 x 28
Cálculo da densidade de gás nitrogênio (N2), em g.L-1: d = RxT
= 0,08206 x (250+273) = 1,30 g. L−1

1 kg 1L
Conversão da unidade da densidade para kg.m-3: d = 1,30 g. L−1 x ( ) x( ) = 1,30 kg. m−3
1000 g 10−3 m³

Questão 390 – (ITA) Um recipiente contendo gás hidrogênio (H2) é mantido à temperatura constante de 0°C.
Assumindo que, nessa condição, o H2 é um gás ideal e sabendo-se que a velocidade média das moléculas desse
gás, nessa temperatura é de 1,85 x 10³ m.s-1, assinale a alternativa correta que apresenta o valor calculado da energia
cinética média, em J, de uma única molécula de H2.

a) 3,1 x 10-24
b) 5,7 x 10-24
c) 3,1 x 10-21
d) 5,71 x 10-21
e) 2,8 x 10-18

Resolução: Alternativa D. Sabendo que a energia cinética é calculada a partir da seguinte fórmula matemática: 𝐸𝑐 =
𝑚 𝑥 𝑣²
2
, em que m é a massa e v é a velocidade. Cálculo da massa em quilogramas (kg), sabendo que a massa

atômica do átomo de hidrogênio é igual a 1,0 g.mol-1.

g 1 mol 1kg
m H2 = 2 x 23
x = 3,32 x 10−27 kg
mol 6,02 x 10 1000 g

194
1600 FÍSICO-QUÍMICA APLICADA EXERCÍCIOS COMENTADOS - IME – ITA – OLIMPÍADA

m x v² 3,32 x 10−27 x (1850)²


Cálculo da energia cinética: Ec = 2
= 2
= 5,68 x 10−21 J

Questão 391 – (ITA) Um recipiente de aço de volume Vl, contém ar comprimido na pressão Pl. Um segundo recipiente
de aço de volume V2, contém ar menos comprimido na pressão P2. Ambos os cilindros estão na pressão ambiente.
Caso sejam interligados por uma tubulação de volume desprezível, a pressão final em ambos os cilindros será igual
a:

(V1 P1 + V2 P2 )
a) (V1 + V2 )
(V1 P2 + V2 P1 )
b) (V1 + V2 )
(V1 P1 + V2 P2 )
c) (P1 + P2 )
(V1 P2 + V2 P1 )
d) (P1 + P2 )
P P
e) (V1 + V2 ) x (V1 x V2 )1/2
1 2

Resolução: Alternativa A.

𝑝1 𝑥 𝑉1
Analisando o recipiente 1 - Cálculo do número de mol: 𝑛1 = 𝑅𝑥𝑇

𝑝2 𝑥 𝑉2
Analisando o recipiente 2 - Cálculo do número de mol: 𝑛2 = 𝑅𝑥𝑇

p1 x V1 p2 x V2
Cálculo do número de mol total (nT): nTotal = n1 + n2 = RxT
+ RxT

1
nTotal = x (p1 x V1 + p2 x V2 )
RxT

Cálculo da pressão total (pT): ptotal x V = nTotal x R x T

1
ptotal x (V1 + V2 ) = [ x (p1 x V1 + p2 x V2 )] x R x T
RxT

ptotal x (V1 + V2 ) = p1 x V1 + p2 x V2

p1 x V1 + p2 x V2
ptotal =
V1 + V2

Questão 392 – (ITA) Nitrogênio gasoso, inicialmente na temperatura ambiente, é passado por um tubo mantido num
forno. A vazão do gás é tão baixa que a pressão na saída (quente) é praticamente a da entrada (frio). Chamemos as
vazões do gás (cm³.s-1) na entrada de v1 e na saída de v2. A densidade do gás (cm³.s-1) na entrada é designada por
d1 e na saída por d2. Nas condições acima teremos que:

a) v1 < v2; d1 < d2


b) v1 < v2; d1 > d2

195
1600 FÍSICO-QUÍMICA APLICADA EXERCÍCIOS COMENTADOS - IME – ITA – OLIMPÍADA

c) v1 > v2; d1 < d2


d) v1 > v2; d1 > d2
e) v1 = v2; d1 = d2

Resolução: Alternativa B. Analisando as densidades, como a temperatura na saída (T2) é maior que a temperatura
na entrada (T1), logo a relação entre as densidades ficarão da seguinte maneira: quanto maior a temperatura, menor
v d
a densidade. Conclui-se que d2 < d1. Analisando as vazões pela lei de Graham, temos: v1 = √d2. Como a densidade
2 1

na entrada (d1) é maior do que a densidade de saída (d2), a relação das velocidades é o inverso das densidades, logo
v2 > v1.

Questão 393 – (ITA) Um termômetro em uma sala de 8,0 x 5,0 x 4,0 m indica 22°C e um higrômetro indica que a
umidade relativa é de 40%. Qual é a massa de vapor d´água na sala se sabemos que nessa temperatura o ar saturado
contém 19,33 gramas de água por metro cúbico?

a) 1,24 kg
b) 0,351 kg
c) 7,73 kg
d) 4,8 x 10-1 kg
e) Outro valor

Resolução: Alternativa A.

A umidade relativa do ar estuda a relação entre a umidade absoluta do ar e a umidade absoluta do mesmo ar no ponto
de saturação, a mesma temperatura.

Representado pela sigla (%) UR.

40
mvapor = (8 x 5 x 4) x x 19,33 = 1237,12 g (1,24 kg)
100

Questão 394 – (ITA) Consideremos um gás formado de moléculas todas iguais e que corresponde ao que considera
um gás ideal. Este gás é mantido num recipiente de volume constante. Dentre as afirmações abaixo, todas referentes
ao efeito do aumento de temperatura, assinale a correta, em relação ao caminho livre médio das moléculas e a
frequência das colisões entre as mesmas:

Caminho livre médio Frequência de colisões

a) Inalterado Aumenta
b) Diminui Inalterada
c) Aumenta Aumenta
d) Inalterado Diminui
e) Diminui Aumenta

196
1600 FÍSICO-QUÍMICA APLICADA EXERCÍCIOS COMENTADOS - IME – ITA – OLIMPÍADA

Resolução: Alternativa A.

O caminho livre médio das moléculas é uma propriedade dos gases que é independente da temperatura, mas sim do
número de moléculas presentes em um determinado recipiente de volume V. Já em relação a frequências de colisões
de partículas gasosas é uma propriedade muito dependente da temperatura, sendo diretamente proporcional.

Questão 395 – (ITA) O gráfico abaixo representa o comportamento real da relação densidade/pressão (d/p) em função
da pressão (p) do hidrogênio, a temperatura constante.

Se o hidrogênio tivesse, de fato, comportamento de um gás ideal, a curva seria uma reta:

a) Paralela AB, mas passando pelo zero


b) Que passa por X e tem coeficiente angular negativo
c) Paralela ao eixo p e passando por X
d) Paralela ao eixo d/p e passando por A
e) Que passa pelo zero e tem coeficiente angular positivo

Resolução: Alternativa C.

p x<MM> <𝑀𝑀>
Sabendo que a relação entre a densidade e a pressão é dada por: d = RxT
e a razão 𝑅 𝑥 𝑇 é constante, então
𝑑 <𝑀𝑀>
juntando as duas equações, temos: = . Esta relação apresentada é constante para qualquer temperatura.
𝑝 𝑅𝑥𝑇

Diante disso, a melhor representação trata-se do item (c).

197
1600 FÍSICO-QUÍMICA APLICADA EXERCÍCIOS COMENTADOS - IME – ITA – OLIMPÍADA

Questão 396 – (IME) Um tambor selado contém ar seco e uma quantidade muito pequena de acetona líquida em
equilíbrio dinâmico com a fase vapor. A pressão parcial da acetona é de 180,0 mm Hg e a pressão total no tambor é
de 760,0 mmHg. Em uma queda durante seu transporte, o tambor foi danificado e seu volume interno diminuiu para
80% do volume inicial, sem que tenha havido vazamento. Considerando-se que a temperatura tenha se mantido
estável a 20ºC, conclui-se que a pressão total após a queda é de:

a) 950,0 mm Hg
b) 1175,0 mm Hg
c) 760,0 mm Hg
d) 832,0 mm Hg
e) 905,0 mm Hg

Resolução: Alternativa E.

Estado inicial:

Pressão parcial do ar seco: par = 760 mmHg – 180 mmHg = 580 mmHg

Observação: 180 mmHg é o valor da pressão máxima de vapor da acetona a 20%.

Estado final:

A redução do volume a 80% de seu valor inicial promove um aumento da pressão parcial do ar, que pode ser calculado
por:

580 x V = (par )final x 0,8 x V

580
(par )final = = 725 mmHg
0,80

O fato de parte da acetona da fase vapor ter sofrido condensação não altera a sua pressão máxima de vapor, pois
não ocorreu variação de temperatura. Logo, a pressão total final será: (Ptotal)final = 725 mmHg + 180 mmHg = 905
mmHg

Questão 397 – (ITA) Temos um frasco aberto contendo um gás a temperatura de 127°C. Querendo expulsar do frasco
1/3 do número de moléculas desse gás, devemos aquecê-lo a:

a) 42,50°C
b) 377 K
c) 447°C
d) 42,50 K
e) 600 K

Resolução: Alternativa E.

198
1600 FÍSICO-QUÍMICA APLICADA EXERCÍCIOS COMENTADOS - IME – ITA – OLIMPÍADA

Considerações a serem feitas para a resolução do problema:

✓ considerar que o processo seja isobárico (≈ pressão de 1 atm);


✓ o volume do frasco é constante - processo isocórico;
✓ R sendo a constante dos gases e a massa molar é a mesma para os dois casos, pois se trata do mesmo gás.
𝑝𝑥𝑉
Relação matemática entre o número de mol com a temperatura: { }
𝑅 𝑐𝑜𝑛𝑠𝑡𝑎𝑛𝑡𝑒
=𝑛𝑥𝑇

Situação I: P = constante; V = constante; TI = 273 + (127°C) = 400 K; nI

1 2
Situação II: P = constante; V = constante; TII = ?; nII = nI − x nI = x nI
3 3

Cálculo da temperatura II, através da equação combinada dos gases ideais: nI x TI = nII x TII

2 2
Sabendo que nII = 3 x nI , temos: nI x TI = 3
x nI x TII

2
nI x 400 = x nI x TII
3

2
x TII = 400
3

TII = 600K (600 – 273 = 327°C)

Questão 398 – (ITA – MODIFICADA) Foram misturados 10 gramas de gás carbônico e 15 gramas de gás nitrogênio
num recipiente de 9,50 litros de capacidade a temperatura de 27°C. Qual a pressão total da mistura em atmosferas?

a) 2,00 atm
b) 2,50 atm
c) 3,00 atm
d) 3,50 atm
e) 4,00 atm

Resolução: Alternativa A.

10 15
Cálculo do número de mol para cada composto gasoso: nCO2 = 44 = 0,227 mol; nN2 = 28 = 0,536 mol

Cálculo do número de mol total: nT = nCO2 + nN2 = 0,227 mol + 0,536 mol = 0,763 mol

ntotal x R x T 0,762 x 0,08206 x (27+273)


Cálculo da pressão total: ptotal = V
= 9,50
= 1,96 atm

Questão 399 – (ITA) Num recipiente está contida uma mistura de 5,6 g de N2(gás), com 6,4 g de O2(gás). A pressão total
da mistura é de 2,5 atmosferas. Nestas condições, a pressão parcial do N2 na mistura é:

a) (0,2/0,4) x 2,5 atm


b) (0,4/0,2) x 2,5 atm

199
1600 FÍSICO-QUÍMICA APLICADA EXERCÍCIOS COMENTADOS - IME – ITA – OLIMPÍADA

c) 0,2 x 2,5 atm


d) 0,4 x 2,5 atm
e) (0,2 + 0,4) x 2,5 atm

Resolução: Alternativa A.
mN2 5,6
Cálculo do número de mol de N2(gás): nN2 = <MM>N2
= 28
= 0,20 mol

mO2 6,4
Cálculo do número de mol de O2(gás): nO2 = <MM>O2
= 32
= 0,20 mol

Cálculo do número de mol total (nT): nT = nN2 + nO2 = 0,2 + 0,2 = 0,4 mol

0,2 mol
Pressão parcial de N2(gás): PN2 = X N2 x pT = (0,4 mol) x 2,50atm

Questão 400 – (ITA) Considere um gás perfeito monoatômico na temperatura de 0°C, sob uma pressão de 1 atm,
ocupando um volume de 56 litros. A velocidade quadrática média das moléculas é 1840 m.s-1. Então, a massa do gás
é:

a) 55 g
b) 100 g
c) 5g
d) 150 g
e) 20 g

Resolução: Alternativa C.
Sabendo que o produto (pV) = nRT e que a equação da velocidade quadrática média das moléculas (c) é dada por
3xRxT
c² = , onde c trata-se da velocidade média quadrática, R a constante dos gases ideais, T é a temperatura
<MM>
pxV 3xRxT
absoluta e <MM> é a massa molar. Substituindo a relação R x T = , temos: c² =
n <MM>

3xpxV 3xpxV
c2 = = m
n x < MM > (
< MM >) x < MM >

3xpxV
c² =
m

3xpxV
A massa do gás pode ser resolvida através da seguinte equação matemática: massa = . Sabendo que 1

3xpxV
atmosfera corresponde a 105 N.m-2 (105 kg.m-1.s-2) e que 56 litros corresponde a 56 x 10-3 m³, temos: m = =

3 x 105 x 56 x 10−3
(1840)²
= 4,96 x 10−3 kg (4,96 g)

200
1600 FÍSICO-QUÍMICA APLICADA EXERCÍCIOS COMENTADOS - IME – ITA – OLIMPÍADA

Questão 401 – (ITA) Certa substância gasosa a 0°C, submetida a uma pressão de 0,333 atm, apresentou uma
densidade de 0,656 g.L-1. Sua massa molecular é:

a) 132
b) 67
c) 44
d) 22
e) 15

Resolução: Alternativa C.
dxRxT 0,656 x 0,08206 x 273
Cálculo da massa molar, a plicando a equação dos gases ideais: < MM > = pideal
= 0,333
=

44,13 g. mol−1 .

Questão 402 – (ITA) Supondo um comportamento de gás ideal, assinale a opção que indica, aproximadamente, a
massa em gramas, de 1,0 litro de C3H8 nas CNTP.

a) 2 x 10-3 g
b) 0,50 g
c) 2,0 g
d) 22,4 g
e) 44,0 g

Resolução: Alternativa C.

m
Cálculo da massa de gás propano, a partir da equação dos gases ideais: 1 x 1 = 44
x 0,08206 x (0 + 273)

m
1= x 0,08206 x 273
44

44
m= = 1,96 g
0,08206 x 273

Questão 403 – (OLIMPÍADA BRASILEIRA DE QUÍMICA) Um acidente em um laboratório provocou a intoxicação de


um grupo de pessoas por inalação de um gás. Um analista coletou uma amostra desse gás e a introduziu em um
recipiente inelástico de 1 dm³, à temperatura de 27°C. A amostra de gás contida no recipiente pesou 1,14 gramas e
a pressão medida no recipiente foi de 1 atm. Assim, pode-se afirmar que este gás é:

a) CO
b) H2S
c) NO2
d) C2H2
e) NO

201
1600 FÍSICO-QUÍMICA APLICADA EXERCÍCIOS COMENTADOS - IME – ITA – OLIMPÍADA

mxRxT 1,14 x 0,08206 x 300 28,06


Resolução: Cálculo da massa molar da espécie gasosa: < MM >= pxV
= 1x1
= 1
=

28,06 g. mol−1. Determinação da massa molar de cada espécie química:

a) CO = (12 + 16) = 28 g.mol-1


b) H2S = (2x1 + 32) = 34 g.mol-1
c) NO2 = (1x14 + 2x16) = 46 g.mol-1
d) C2H2 = (2x12 + 2x1) = 26 g.mol-1
e) NO = (14 + 16) = 30 g.mol-1

Questão 404 – (OLIMPÍADA DE QUÍMICA DO RIO DE JANEIRO) Qual a pressão parcial do SO2 se 100 g de O2 são
misturados com 100 g de SO2, e a pressão total é 600 mmHg?

a) 500 mmHg
b) 400 mmHg
c) 300 mmHg
d) 200 mmHg
e) 100 mmHg

Resolução: Alternativa D. Cálculo do número de mol de cada gás e também do número de mol total:

mSO2 100
nSO2 = = = 1,5625 mol
< MM >SO2 64

m O2 100
nO2 = = = 3,125 mol
< MM >O2 32

ntotal = nSO2 + nO2 = 1,5625 mol + 3,125 mol = 4,6875 mol

nSO2
Cálculo da pressão total da mistura gasosa: PSO2 = X SO2 x PT = n x PT
SO2 + nO2

1,5625
8,87 = x PT
4,6875

PT = 200,0 kPa

Questão 405 - (OLIMPÍADA BAIANA DE QUÍMICA – MODIFICADA) Calcule o volume ocupado por 10,0 gramas de
dióxido de carbono (CO2) nas CNTP.

a) 5,09 L
b) 5,59 L
c) 6,59 L
d) 7,59 L
e) 8,59 L

202
1600 FÍSICO-QUÍMICA APLICADA EXERCÍCIOS COMENTADOS - IME – ITA – OLIMPÍADA

Resolução: Alternativa A.

10
Cálculo do volume do gás através da equação dos gases ideais: 1 x V = 44 x 0,08206 x (300 + 273)

10
V= x 0,08206 x 573 = 10,69 L
44

203
1600 FÍSICO-QUÍMICA APLICADA EXERCÍCIOS COMENTADOS - IME – ITA – OLIMPÍADA

Resolução: As equações
químicas balanceadas estão
apresentadas
abaixo:CAPÍTULO III

SOLUÇÕES E
ESTEQUIOMETRIA COM
SOLUÇÕES

PROFESSOR ALEXANDRE
VARGAS GRILLO
204
1600 FÍSICO-QUÍMICA APLICADA EXERCÍCIOS COMENTADOS - IME – ITA – OLIMPÍADA

Questão 406 – Defina soluções.


Resposta: Define-se soluções como dispersões monofásicas, misturas homogêneas de duas ou mais substâncias
presentes em sua composição, apresentando soluto e solvente.
Questão 407 – Defina soluto e solvente.
Resposta: Solvente trata-se da substância que vai dissolver o soluto. O solvente mais conhecido e utilizado e a água,
por isso, que é reconhecidamente como solvente universal. Soluto é a substância que será dissolvida com a presença
do solvente.
Questão 408 – Defina concentração comum.
Resposta: Concentração comum ou também conhecido como concentração mássica é a razão da massa do soluto
pelo volume da solução.
Questão 409 – (IME) Se temos duas soluções de acetato de sódio, sendo uma a 20ºC e outra a 50ºC, ambas com 30
g.L-1, suas molaridades são iguais? Por quê?
Resposta: Não, as concentrações em mol.L-1 também conhecida como concentração molar ou o mais correto como
concentração em quantidade de matéria apresentam valores distintos. Isso se deve ao fato desta propriedade ser
dependente da temperatura, diferentemente da molalidade. Ou seja, temperaturas diferentes terão concentrações
molares diferentes, devido a variações de volume causada pela dilatação.
Questão 410 – Defina densidade de solução.
Resposta: Densidade de solução é a razão da massa da solução pelo volume da solução.
Questão 411 – Defina fração molar.
Resposta: É a razão entre o número de mols do componente considerado e o número de mols do sistema como um
todo. Muito análogo a fração molar abordada para o sistema gasoso.
Questão 412 – Defina normalidade.
Resposta: É a razão entre a massa do soluto pelo produto entre o número de equivalente-grama (eqg) com o volume
da solução, em litros.
Questão 413 – Defina o processo de diluição.
Resposta: Diluição de uma solução é diminuir a quantidade relativa de soluto, através da adição de solvente.
Questão 414 – Defina título em massa.
Resposta: Define-se título em massa (m/m) como a razão entre a massa do soluto pela massa da solução.
Questão 415 – Defina título em volume.
Resposta: Define-se título em volume (v/v) como a razão entre o volume do soluto pelo volume da solução.
Questão 416 – (ITA) Normalidade só pode ser definida para ácidos e bases. Certo ou errado?
Resposta: Errado. Trata-se de uma frase falsa, uma vez que a normalidade ou concentração normal (N) pode ser
calculada para ácidos, bases e sais.
Questão 417 – (OLIMPÍADA NORTE/NORDESTE DE QUÍMICA) Compare um gás e um líquido, no que se refere as
seguintes propriedades:
a) densidade;
b) compressibilidade;
c) capacidade de se misturar com outras substâncias, na mesma fase, e formar misturas homogêneas.

205
1600 FÍSICO-QUÍMICA APLICADA EXERCÍCIOS COMENTADOS - IME – ITA – OLIMPÍADA

Resposta:
a) Em relação a densidade, o líquido apresenta maior densidade em comparação ao gás.
b) Com relação a compressibilidade, os gases apresentam maior compressibilidade comparado aos líquidos. É
apresentado como um coeficiente conhecido como coeficiente de compressibilidade (β), determinado pela variação
do volume da substância com o aumento da pressão a temperatura constante, conforme pode ser observado pela
1 ∂V
equação a seguir, β = − V x (∂P) .
T
c) Os gases apresentam como características a capacidade de formar misturas definidas como misturas
homogêneas, já em relação aos líquidos depende de diversos fatores, podendo ser miscíveis (mistura homogênea),
como por exemplo água e etanol ou imiscíveis (mistura heterogênea), como por exemplo água e óleo.

Questão 418 – (ITA) Definir e exemplificar coloides e citar três de suas características.
Resposta: Coloides ou também conhecido como sistemas coloidais são misturas em que o diâmetro (d) das partículas
dispersas está compreendido entre 10 e 10³ Å (angstrom). O coloide pode ser formado por agregados de moléculas,
átomos ou íons. Exemplos: argila; geleia; leite; espumas em geral.

Questão 419 – (GRILLO) Um laboratório de análises químicas realizada no IFRJ – Nilópolis, apresenta uma solução
de ácido nítrico com 58,9% em massa (m/m) e densidade igual a 1,33 g.cm-3. Calcule a concentração em quantidade
de matéria desta solução ácida. Resolução: Dados do problema:
58,9 gramas de HNO3 para cada 100 gramas de solução
1,33 gramas de solução para cada 1 cm³ de solução

Cálculo do volume da solução:


1,33 gramas de solução ---------- 1 cm³ de solução
100 gramas de solução ----------- Vsolução
Vsolução = 75,19 cm3 (75,19 x 10-3 L)
mHNO
3
nsoluto <MM>HNO
Cálculo da concentração da quantidade de matéria do ácido nítrico: [HNO3 ] = = 3
=
Vsolução Vsolução
58,9
0,935
63
= = 12,43 mol. L−1
75,19 x 10−3 75,19 x 10−3

Questão 420 – (U.S. NATIONAL CHEMISTRY OLYMPIAD) A solução aquosa de hidróxido de sódio apresenta 30%
em massa (m/m) e densidade igual a 1,33 g.mL-1. Qual é a concentração em quantidade de matéria de NaOH nesta
solução?
a) 8,25
b) 9,98
c) 16,0
d) 33,2

Resolução: Alternativa B.

Dados do problema: 30 gramas de NaOH para cada 100 gramas de solução e 1,33 gramas de solução para cada 1
mL de solução.

Cálculo do volume da solução (Vsolução):


1,33 gramas de solução ---------- 1 mL de solução
100 gramas de solução ----------- Vsolução
Vsolução = 75,19 mL (75,19 x 10-3 L)

206
1600 FÍSICO-QUÍMICA APLICADA EXERCÍCIOS COMENTADOS - IME – ITA – OLIMPÍADA

𝑚𝑁𝑎𝑂𝐻 30
𝑛 <𝑀𝑀>𝑁𝑎𝑂𝐻
Cálculo da concentração da quantidade de matéria: [𝑁𝑎𝑂𝐻] = 𝑉 𝑁𝑎𝑂𝐻 = 𝑉𝑠𝑜𝑙𝑢çã𝑜
= 40
75,19 𝑥 10−3
=
𝑠𝑜𝑙𝑢çã𝑜
0,75 𝑚𝑜𝑙
75,19 𝑥 10−3
= 9,97 𝐿

Questão 421 – (U.S. NATIONAL CHEMISTRY OLYMPIAD) Qual é a concentração em quantidade de matéria de
uma solução aquosa 0,500 mol.kg-1 de nitrato de cálcio que apresenta 1,045 g.mL-1 de densidade?
a) 0,483 mol.L-1
b) 0,500 mol.L-1
c) 0,522 mol.L-1
d) 0,567 mol.L-1

Resolução: Alternativa A.
nCa(NO3 )2 mCa(NO3 )2
Cálculo da massa de nitrato de cálcio, a partir da molalidade (W): W = =
msolvente
(kg) <MM>Ca(NO3 )2 x msolvente
(kg)

mCa(NO3 )2 = W x < MM >Ca(NO3 )2 x msolvente


(kg)

mCa(NO3 )2 = 1,0 x 164 x 0,500 = 82,0 g

Cálculo da massa da solução: msolução = msoluto + msolvente = 82 g + 1000 g = 1082 g

Cálculo do volume da solução, a partir da densidade da solução:

1 mL de solução ------------------ 1,045 gramas de solução


Vsolução ------------------------------ 1082 gramas de solução
Vsolução = 1035,41 mL (1,035 L)
msoluto 82
( ) ( )
<MM>soluto
Cálculo da concentração da quantidade de matéria do nitrato de cálcio: [Ca(NO3 )2 ] = = 164
=
Vsolução 1,035
0,48 mol. L−1
Questão 422 – (GRILLO) Calcule a normalidade, a molalidade e também a concentração em quantidade de matéria
de uma solução que apresenta 22% em massa (m/m) de cloreto de cálcio, sabendo que na temperatura do
experimento, a densidade da solução é igual a 1,45 g.cm-3.
Resolução: Dados do problema:
22 gramas de CaCl2 para cada 100 gramas de solução
1,45 gramas de solução para cada 1 cm³ de solução

Cálculo do volume da solução:


1,45 gramas de solução ---------- 1 cm³ de solução
100 gramas de solução ----------- Vsolução
Vsolução = 68,96 cm³ (68,96 x 10-3 L)
22 30
Cálculo do número de mol (n) do cloreto de cálcio: nCaCl2 = 111 = 40 = 0,198 mol

0,198
Cálculo da concentração da quantidade de matéria do cloreto de cálcio: [CaCl2 ] = = 2,87 mol. L−1
68,96 x 10−3
mCaCl2
Primeira maneira para calcular a Normalidade (N) do cloreto de cálcio: NCaCl2 = eqg x Vsolução
=
CaCl2
22
111 = 5,74 eqg. L−1
( ) x 68,96 x 10−3
2

Segunda maneira para calcular a Normalidade do cloreto de cálcio: N = [CaCl 2].x, onde x é igual a +2. Este número
corresponde à carga total do cátion cálcio.

207
1600 FÍSICO-QUÍMICA APLICADA EXERCÍCIOS COMENTADOS - IME – ITA – OLIMPÍADA

N = 2,87 x 2 = 5,74 eqg.L-1


Cálculo da massa do solvente: msolução = msoluto + msolvente
100 g = 22 g + msolvente
msolvente = 78 g

nsoluto 0,198 mol


Cálculo da molalidade (W): W = = 78 x 10−3 kg = 2,54 mol. kg −1
msolvente
(kg)

Questão 423 – (ITA) 1,7 toneladas de amônia vazaram e se espalharam uniformemente em certo volume da
atmosfera terrestre, a 27°C e 760 mmHg. Medidas mostraram que a concentração da amônia neste volume da
atmosfera era de 25 partes por milhão, em volume, de ar. O volume da atmosfera contaminada por essa quantidade
de amônia em m³, de:
a) 0,9 x 10²
b) 1,0 x 10²
c) 2,0 x 107
d) 10 x 107
e) 25 x 108

Resolução: Alternativa D.
Dados do problema: 25 ppm, ou seja, 25 partes de amônia para 106 partes de ar; Pressão = 760 mmHg (1 atm) e T =
27 + 273 = 300 K.
760 1,70 x 106
Cálculo do volume de amônia, a partir da equação dos gases ideais: 760 x VNH3 = 17
x 0,08206 x (27 +
273)

VNH3 = 2,46 x 106 L

Cálculo do volume de ar contaminado pela amônia:


25 litros de NH3 ------------------- 106 litros de ar
2,46 x 106 litros de NH3 ---------- Var
Var = 9,85 x 1010 L o que equivale a 9,85 x 1010 dm³. Este volume representa 10,0 x 107 m³ (1,0 x 108 m³)

Questão 424 – (IME) Determinar os volumes, em litros, de HCl (N/2) e de HCl (N/10), que devemos misturar para
obtermos 2 litros de solução (N/5).

Resolução: O problema leva em consideração o processo de mistura de mesmo soluto.


HCl(1) + HCl(2) → HCl(3)

Equação (1): (V1 x N1) + (V2 x N2) = VTOTAL x NTOTAL


V1 x (N/2) + V2 x (N/10) = 2 x (N/5)
Equação (2): V1 + V2 = 2

Desenvolvendo a equação (1), o sistema de equações pode ser reescrito da seguinte forma:
Equação (1): 5V1 + V2 = 4
Equação (2): V1 + V2 = 2

Resolvendo o sistema que compõem as equações (1) e (2), temos: isolando V2, através da equação (2) e substituindo
na equação (1), temos: V2 = 2 – V1
5 x V1 + (2 – V1) = 4
5 x V1 – V1 + 2 = 4
4 x V1 = 2
V1 = 0,50 L

208
1600 FÍSICO-QUÍMICA APLICADA EXERCÍCIOS COMENTADOS - IME – ITA – OLIMPÍADA

Consequentemente, V2 = 1,50 L.

Questão 425 – (GRILLO) Calcule o volume de sulfeto de hidrogênio, medido na pressão de 675 mmHg e temperatura
de 37°C, quando este gás reage com hidróxido de potássio que apresenta volume igual a 375 cm³ e concentração
igual a 0,250 mol.L-1.

Resolução: Através da reação química, temos a seguinte relação estequiométrica: H2S(g) + 2 KOH(aq) → K2S(aq) + 2
H2O(l)
1 mol de H2S ---------- 2 mol de KOH
nH2S --------------------- nKOH
2 x nH2S = nKOH

Levando em consideração que o sulfeto de hidrogênio apresenta comportamento de gás ideal, temos: nKOH =
2 x n H2 S

p x VH2 S
[KOH] x Vsolução = 2 x
RxT
675
( ) x VH2 S
0,250 x 0,375 x 10 −3
=2x 760
0,08206 x (37 + 273)

VH2 S = 1,34 x 10−3 L

Questão 426 – (GRILLO) Considere a neutralização de 15,5 mL de uma solução de ácido clorídrico por meio de uma
solução de hidróxido de cálcio 0,55 mol.L-1. Sabendo que a viragem ocorre na presença de um indicador, quando são
consumidos 22,50 mL de hidróxido, calcule a concentração, em mol.L-1, do ácido.

Resolução: Equação química: 2 HCl(aq) + Ca(OH)2(aq) → CaCl2(aq) + 2 H2O(l)


Relação estequiométrica entre o ácido clorídrico e o hidróxido de cálcio:
2 mol de HCl ---------- 1 mol de Ca(OH)2
nácido --------------------- nbase

Cálculo da concentração da quantidade de matéria do ácido clorídrico (mol.L-1): [HCl] x Vsolução =


2 x [Ca(OH)2 ] x Vsolução
[HCl] x 15,50 = 2 x 0,55 x 22,50

[HCl] = 1,60 mol x L−1

Questão 427 –
a) Considere que a solução de bicarbonato de sódio apresenta as seguintes informações no seu rótulo: 2,80
mol.kg-1 e 3,10 mol.L-1. Calcule a densidade desta solução.
b) A solução de ácido sulfúrico apresenta densidade de 1,60 g/cm³ e 88% em massa (m/m) de H2SO4. Calcule
a concentração em mol.L-1.
Resolução:

Item a) Cálculo da massa de solvente:

2,80 mol de NaHCO3 --------- 1 kg


3,10 mol de NaHCO3 --------- m
Massa de solvente (m) = 1,107 kg

209
1600 FÍSICO-QUÍMICA APLICADA EXERCÍCIOS COMENTADOS - IME – ITA – OLIMPÍADA

A massa de bicarbonato de sódio pode ser relacionada a partir da concentração da quantidade de matéria, conforme
msal
( )
<MM>sal
observado a seguir: [NaHCO3 ] = Vsolução

m
( sal )
84
1,0
= 3,10

msal = 84 x 3,10 = 260,4 g

Logo, a massa total da solução (mtotal) pode ser calculada a partir do somatório da massa do solvente e do soluto, ou
seja: mtotal = msolução = msoluto + msolvente = 1107 + 260,4 = 1367,4 g

Finalmente, sabendo que a densidade é a razão entre a massa da solução pelo volume da solução, temos: d =
msolução 1367,40 g
= = 1367,20 g. L−1
Vsolução 1,0 L

Item b) Considerando os dados do enunciado, o volume da solução pode ser determinado a partir de uma regra de
três simples.
1,60 g de solução ---------- 10-3 L
100 g de solução ----------- Vsolução
Vsolução = 0,0625 L

O número de mols de ácido presente, necessário para a determinação da concentração da quantidade de matéria
será calculado considerando como base de cálculo 100 g de solução, onde teremos 88 g de H2SO4.
mH SO4 88
Cálculo do número de mol de ácido sulfúrico: nH2 SO4 = <MM>2 = 98 = 0,898 mol
H2 SO4

A concentração da quantidade de matéria da solução ácida é então computada mediante a razão entre o número de
mols encontrado e o volume total da solução para uma base de cálculo de 100 g.
nH2 SO4 0,898 mol
[H2 SO4 ] = = = 14,37 mol. L=1
Vsolução 0,0625 L

Questão 428 – Uma amostra constituída de 250 g de cloreto de cálcio foi dissolvida em 880 g de água, formando uma
solução com um volume de 550 ml a T = 27ºC. Para esta solução, calcule:
a) Concentração mássica (g.L-1);
b) Concentração da quantidade de matéria em mol.L-1;
c) Molalidade;
d) Fração molar;
e) Densidade da solução.

Resolução:
a) O cálculo da concentração mássica (g.L-1) requer o conhecimento do volume da solução (550 mL) e da massa
𝑚𝐶𝑎 𝐶𝑙 250 𝑔 𝑔
de soluto (CaCl2). Logo: 𝐶𝐶𝑎2 𝐶𝑙2 = 𝑉 2 2 = 0,55 𝐿 = 454,54 𝐿
𝑠𝑜𝑙𝑢çã𝑜
msal 250
( ) ( )
<MM>sal
b) Cálculo da concentração da quantidade de matéria do cloreto de cálcio: [CaCl2 ] = Vsolução
= 0,55
111
=
4,09 mol. L−1
250
nsoluto ( )
c) Cálculo da molalidade (W): W = (kg) = 0,88
111
= 2,56 mol. kg −1
msolvente
d) Por definição, a fração molar associada ao CaCl2 consiste na razão entre o número de mol deste e o número
de mol total encerrado na mistura.

210
1600 FÍSICO-QUÍMICA APLICADA EXERCÍCIOS COMENTADOS - IME – ITA – OLIMPÍADA

mCaCl 250
ncaCl2 ) (< MM > 2
( )
X CaCl2 = =
caCl2
= 111 = 0,044
nCaCl2 + nH2 O ( mCaCl2 ) + ( mH2 O ) (250) + (880)
< MM >CaCl2 < MM >H2 O 111 18

e) O cálculo da densidade da solução requer o conhecimento da massa total da solução (soluto e solvente) e
do volume total da mistura.

msolução (250 + 880)g


dsolução = = = 2,05 g. mL−1
Vsolução 550 mL

Questão 429 – (U.S. NATIONAL CHEMISTRY OLYMPIAD) What is the concentration of calcium ions in 350 mL of
an aqueous solution containing 7,50 g CaCl2?

a) 0,0676 mol.L-1
b) 0,193 mol.L-1
c) 0,284 mol.L-1
d) 0,535 mol.L-1

Resolução: Alternativa B.
msal 7,50
( ) ( )
<MM>sal
Cálculo da concentração da quantidade de matéria do cloreto de cálcio: [CaCl2 ] = Vsolução
= 111
0,350
=
0,193 mol. L−1
H2 O
CaCl2(s) → Ca+2(aq) + 2 Cl-(aq)
1 mol de CaCl2 ------------ 1 mol de Ca+2
0,193 mol.L-1 -------------- [Ca+2]
[Ca+2] = 0,193 mol.L-1

Questão 430 – (U.S. NATIONAL CHEMISTRY OLYMPIAD) What is the molarity of Na+ ions in a solution made by
dissolving 4,20 g of NaHCO3 (M = 84,0) and 12,6 g of Na2CO3 (M = 126) in water and diluting to 1,00 L?

a) 0,050 mol.L-1
b) 0,100 mol.L-1
c) 0,150 mol.L-1
d) 0,250 mol.L-1

Resolução: Alternativa D.
4,20
( )
Cálculo da quantidade de matéria de bicarbonato de sódio: [NaHCO3 ] = 84
= 0,05 mol. L−1
1,0

H2 O
Cálculo da quantidade de matéia de íons sodio [Na+]: NaHCO3(s) → Na+ (aq) + HCO3-(aq)

1 mol de NaHCO3 ------------ 1 mol de Na+


0,05 mol.L-1 ------------------- [Na+]
[Na+] = 0, 05 mol.L-1

Analisando a solução de carbonato de sódio. Cálculo da quantidade de matéria do carbonato de sódio: [Na2 CO3 ] =
msal 12,60
( ) ( )
<MM>sal
Vsolução
= 126
1,0
= 0,100 mol. L−1

H2 O
Cálculo da quantidade de matéria de íons sodio [Na+]: Na2CO3(s) → 2 Na+ (aq) + CO3-2(aq)

211
1600 FÍSICO-QUÍMICA APLICADA EXERCÍCIOS COMENTADOS - IME – ITA – OLIMPÍADA

1 mol de Na2CO3 --------------- 2 mol de Na+


0,100 mol.L-1 ------------------- [Na+]
[Na+] = 0, 200 mol.L-1

Logo, a concentração da quantidade de matéria total dos íons sódio é igual a: [Na +]TOTAL = 0,050 + 0,200 = 0,250
mol.L-1

Questão 431 – (U.S. NATIONAL CHEMISTRY OLYMPIAD) What is the molarity of a hydrochloric acid solution if
20,00 mL of it neutralizes 18,46 mL of a 0,0420 M Ba(OH)2 solution?

a) 0,0194 mol.L-1
b) 0,0388 mol.L-1
c) 0,0455 mol.L-1
d) 0,0775 mol.L-1

Resolução: Alternativa D.

Cálculo da concentração da quantidade de matéria do ácido clorídrico: 2 HCl(aq) + Ba(OH)2(aq) → BaCl2(aq) + 2 H2O(l)
2 mol de HCl --------------- 1 mol de Ba(OH)2
nHCl --------------------------- nBa(OH)2
nHCl = 2 x nBa(OH)2

Cálculo da concentração da quantidade de matéria do ácido clorídrico: [HCl] x Vsolução = 2 x [Ba(OH)2] x Vsolução
[HCl] x 20 = 2 x 0,0420 x 18,46
[HCl] = 0,0775 mol.L-1

Questão 432 – Considere que uma solução de ácido sulfúrico apresenta as seguintes características, considerando
que a temperatura seja igual a 20°C. Dados analíticos do ácido sulfúrico: densidade = 1,79 g.cm-3; título em massa =
96%. Diante das informações do ácido, determine:
a) O valor da normalidade deste ácido;
b) O valor da quantidade de matéria do ácido;
c) Se 100 cm³ deste ácido forem utilizados na transformação de prata em solução de sulfato de prata, calcule
a massa de prata utilizada. Utilizando este ácido sulfúrico para realizar um procedimento de titulação com 20 cm³ de
uma solução de hidróxido de lítio, verifica-se que o volume é igual a 1,50 cm³. Calcule a concentração normal desta
base,

Resolução: Dados do problema:


1 cm³ ----------------------- 1,79 g de solução
96 g de H2SO4 ------------ 100 g de solução

Item a) Cálculo do volume da solução:


1 cm³ --------- 1,79 g de solução
V -------------- 100 g de solução
V = 55,86 cm³ (55,86 x 10-3 dm³)
msoluto 96
Cálculo da normalidade da solução: N = = 98 = 35,07 eqg. L−1
eqgsoluto x Vsolução ( ) x 55,86 x 10−3
2
mH SO
2 4 96
nH2 SO4 <MM>H SO mol
Item b) Cálculo da concentração da quantidade de matéria: [H2 SO4 ] = V = 2 4
Vsolução
98
= 55,86 x 10−3 L
=
solução
0,979 mol −1
55,86 x 10−3 L
= 17,52 mol. L

Item c) Equação química: 2 Ag(s) + H2SO4(aq) → Ag2SO4(aq) + H2(g)

212
1600 FÍSICO-QUÍMICA APLICADA EXERCÍCIOS COMENTADOS - IME – ITA – OLIMPÍADA

Cálculo da concentração comum (C), a partir da concentração da quantidade de matéria: C = [H2SO4] x <MM>H2SO4 =
17,52 x 98 = 1716,96 g.L-1

Cálculo da massa de ácido sulfúrico em 100 cm³, a partir da concentração comum: mH2SO4 = C x Vsolução = 1716,96 x
0,100 = 171,69 g

Cálculo da massa de prata, a partir da estequiometria:

2 mol de Ag(s) -------------- 1 mol de H2SO4(aq)


216 g ------------------------ 98 g
mAg --------------------------171,69 g
mAg = 378,42 g

Item d) Equação química: H2SO4(aq) + 2 LiOH → Li2SO4(aq) + 2 H2O(g)


1 mol de H2SO4(aq) ------------ 2 mol de LiOH
EqgH2SO4 ------------------------ EqgLiOH

2 x EqgH2SO4 = EqgLiOH
2 x (NH2SO4 x Vsolução) = (NLiOH x Vsolução)
2 x (35,07 x 1,50 cm³) = (NLiOH x 2,0 cm³)
NLiOH = 52,60 eqg.L-1

Questão 433 - Uma quantidade de um composto orgânico, butan-2-ol, apresenta uma massa igual a 77,60 gramas é
misturada com 278,60 gramas de água, formando uma solução que apresenta uma densidade de 0,931 g.mL-1, a
20°C. A partir desta informação, determine:
a) Concentração da quantidade de matéria;
b) Concentração comum e g x mL-1;
c) Molalidade.

Resolução: Dados do problema: massa do soluto = 77,60 g (C4H10O) e massa do solvente = 278,60 g (H2O)

Cálculo da massa da solução: msolução = msoluto + msolvente = 77,60 g + 278,60 g = 356,20 g = 356,20 g

Cálculo do volume da solução:


1 mL de solução ------------- 0,931 g de solução
Vsolução ------------------------- 356,20 g de solução
Vsolução = 382,60 mL (382,60 x 10-3 L)
77,60
n
Cálculo da concentração da quantidade de matéria do soluto: [soluto] = V soluto = 74
382,60 x 10−3
=
solução
1,05
382,60 x 10−3
= 2,74 mol. L−1

m 77,60
Cálculo da concentração comum da solução: Csoluto = V soluto = 382,60 = 0,203 g. L−1
solução

nsoluto 1,05 mol


Cálculo da molalidade (W): W = = = 3,77 mol. kg −1
msolvente
(kg) 0,2786 kg

213
1600 FÍSICO-QUÍMICA APLICADA EXERCÍCIOS COMENTADOS - IME – ITA – OLIMPÍADA

Questão 434 – Calcule os seguintes itens apresentados a seguir.


a) Um técnico de química pesou 8,80 gramas de cloreto de amônio em 200,0 gramas de água. Se a densidade
da solução é 1,92 g/mL, determine a concentração da quantidade de matéria do referido sal.
b) Determine a concentração de quantidade de matéria e também a molalidade que apresenta uma solução de
álcool etílico na presença de água, considerando que a fração molar do álcool é igual a 0,025, sabendo que a
densidade da solução seja igual a 0,98 g.mL-1.

Resolução: Item a) Cálculo do volume da solução (Vsolução):


1 mL de solução ------------ 1,92 g de solução
Vsolução ------------------------ (200,0 + 8,80) g de solução
Vsolução = 108,75 mL
mNH Cl
4
nNH4Cl <MM>NH Cl
Cálculo da concentração da quantidade de matéria do cloreto de amônio: [NH4 Cl] = V = VNH4Cl
4
=
NH4 Cl
8.80
0,164 mol
53,50
108,75 x 10−3
= 108,75 x 10−3 = 1,51 mol. L−1

nC2 H5OH
Item b) Cálculo da massa de etanol, a partir da fração molar: X C2 H5 OH = nTOTAL
= 0,025

nC2 H5 OH
X C2 H5 OH = = 0,025
nTOTAL
nC2 H5 OH
X C2 H5 OH = = 0,025
nC2 H5 OH + nH2 O

nC2 H5 OH = 0,025 x (nC2 H5 OH + nH2 O )

mC2H5 OH
<MM>C2H5 OH
= 0,025 x (nC2 H5 OH + nH2 O )

mC2H5 OH mC2H5OH mH2O


= 0,025 x ( + )
46 46 18

mC2H5 OH 9 x mC2H5OH 23 x mH2 O


46
= 0,025 x ( 414
+ 414
)

9 x mC2 H5 OH = 0,225 x mC2 H5 OH + 0,575 x mH2 O

mH2 O = 15,26 x mC2 H5 OH

Cálculo da massa total da solução para um litro de solução:


1 mL de solução -------------------------- 0,98 g de solução
1000 mL de solução (1 L) --------------- msolução
msolução = 980 g de solução

Logo, temos um sistema, com duas equações e duas incógnitas: mágua = 15,26 x metanol e mágua + metanol = 980

Resolvendo o sistema, temos: mC2H6O = 60,27 g e mH2O = 919,70 g


mC2H5OH 60,27
Cálculo do número de mol de etanol: nC2 H5 OH = = = 1,31 mol
<MM>C2H5 OH 46

Cálculo do volume da solução:


1 mL de solução -------------------------- 0,98 g de solução
Vsolução -------------------------------------- (60,27 g + 919,70 g)
Vsolução = 999,97 mL (999,97 x 10-3 L)

214
1600 FÍSICO-QUÍMICA APLICADA EXERCÍCIOS COMENTADOS - IME – ITA – OLIMPÍADA

nC2H5OH 1,31 mol


Cálculo da concentração da quantidade de matéria de etanol: [C2 H5 OH] = = ≅
Vsolução 999,97 x 10−3
1,31 mol. L−1

nsoluto 1,31 mol


Cálculo da molalidade (W): W = = = 1,42 mol. kg −1
msolvente
(kg) 0,920 kg

Questão 435 – Considere uma massa de aproximadamente 26 gramas de P2O3 que apresenta 79% de grau de pureza
que reage com 0,12239 dm³ de ácido clorídrico (0,250 mol.L-1), conforme a seguinte equação química não -
balanceada a seguir: P2O3(s) + HCl(aq) → PH3(g) + Cl2O(g). Sabendo que o rendimento desta reação apresenta um
percentual de aproximadamente igual a 77%, determine o reagente limitante e reagente em excesso, além da pressão
total dos gases que são recolhidos em um vaso com capacidade de 20 litros a 327°C.

Resolução: Equação química: P2O3(s) + 6 HCl(aq) → 2 PH3(g) + 3 Cl2O(g)

Determinação do reagente limitante e do reagente em excesso:

mP2 O3 26
nP2 O3 = = = 0,236 mol
< MM >P2 O3 110

nHCl = [HCl] x Vsolução = 0,250 x 122,39 x 10−3

0,0306
nHCl = [HCl] x Vsolução = = 5,10 x 10−3 mol (reagente limitante)
6

Cálculo do número de mol dos gases produzidos: P2O3(s) + 6 HCl(aq) → 2 PH3(g) + 3 Cl2O(g)
6 mol de HCl(aq) ------------------------- 2 mol de PH3(g)
6 mol de HCl(aq) ------------------------- (2 + 3) mol de gases

6 mol de HCl ------------------------------ 5 mol de gases x 0,77


0,79 x 5,10 x 10-3 mol de HCl ----------- ngases
ngases = 2,58 x 10-3 mol

Considerando que os gases apresentam comportamento ideal, temos: p x 20 =


2,58 x 10−3 x 0,08206 x (327 + 273)
p = 0,00635 atm

Questão 436 – 55 mililitros de 0,150 mol.L-1 de sulfato de alumínio e 65 mililitros de 0,450 mol.L-1 de cloreto de
alumínio são adicionados em uma quantidade de água para a formação de uma solução final de íons alumínio igual
a 0,550 mol.L-1, apresentando um volume final de 500 mililitros. A partir deste experimento, determine a concentração
da quantidade de matéria final de íons positivos na solução final.

Resolução: Analisando a solução (1) de sulfato de alumínio: Al2(SO4)3(aq) → 2 Al+3(aq) + 3 SO4-2(aq)


1 mol de Al2(SO4)3 ---------- 2 mol de Al+3
0,150 mol.L-1 ----------------- [Al+3]1
[Al+3]1 = 0,300 mol.L-1

Analisando a solução (2) de cloreto de alumínio: AlCl3(aq) → Al+3(aq) + 3 Cl-(aq)


1 mol de AlCl3 --------------- 1 mol de Al+3
0,450 mol.L-1 ----------------- [Al+3]2
[Al+3]2 = 0,450 mol.L-1

Cálculo da concentração da quantidade de matéria final (3) de cátions alumínio: ([Al+3]1 x V1) + ([Al+3]2 x V2) = ([Al+3]3
x V3)
0,300 x 55 + 0,450 x 65 = [Al+3]3 x 500

215
1600 FÍSICO-QUÍMICA APLICADA EXERCÍCIOS COMENTADOS - IME – ITA – OLIMPÍADA

[Al+3]3 = 0,0915 mol.L-1

Questão 437 – Responda e calcule os seguintes itens abaixo.


a) 555 m3 de solução aquosa de NaOH com concentração igual a 2,25 mol.L-1, deve ser obtido utilizando duas
outras soluções disponíveis também de NaOH, com suas concentrações iguais a 1,0 mol.L-1 e 5,0 mol.L-1. Determine
o volume para cada solução de NaOH.
b) Considere uma solução aquosa de ácido sulfúrico que apresenta 10,0% (m/m), cuja densidade a 22°C, é
1,09 g/cm3. A partir desta informação, determine:
b.1 concentração comum;
b.2 concentração da quantidade de matéria;
b.3 a molalidade.
c) Calcule a fração molar de carbonato de potássio em uma solução aquosa que apresenta 65% em massa
desta substância.

Resolução: Item a) O problema trata da mistura de mesmo soluto, logo iremos relacionar duas equações
matemáticas:
[NaOH]1 x V1 + [NaOH]2 x V2 = [NaOH]3 x V3
V1 + V2 = V3

1,0 x V1 + 5,0 x V2 = 2,25 x 555


V1 + V2 = 555

V1 + 5,0 x V2 = 1248,75
V1 + V2 = 555

Resolvendo o sistema acima, os valores encontrados são: V1 = 381,56 m³ e V2 = 173,44 m³.

Item b)

b.1 Cálculo da concentração comum (C): C = 10 x d x (%) = 10 x 1,09 x 10 = 109 g.L-1


CH SO4 109
b.2 Cálculo da concentração da quantidade de matéria: [H2 SO4 ] = <MM>2 = 98
= 1,11 mol. L−1
H2 SO4

b.3 Cálculo da massa da solução para um litro (1 L):

1,09 gramas de solução ----------- 1 cm³


msolução ------------------------------- 1 L (1000 cm³)
msolução = 1090 g
mH SO4 10
Cálculo do número de mol de H2SO4: nH2 SO4 = <MM>2 = 98
= 0,102 mol
H2 SO4

mH SO
2 4 10
nH2 SO4 <MM>H SO 0,102 mol
Cálculo da molalidade (W): W = (kg) = (kg)
2 4
= 98
10,90
= 10,90
= 0,00936 mol. kg −1
msolvente msolvente

Item c) Base de cálculo: 100 gramas de solução.

Cálculo da massa de K2CO3: mK2CO3 = 0,65 x 100 = 65 g

Cálculo da massa de H2O: msolução = mK2CO3 + mH2O


100 g = 65 g + mH2O
mH2O = 35 g

216
1600 FÍSICO-QUÍMICA APLICADA EXERCÍCIOS COMENTADOS - IME – ITA – OLIMPÍADA

mK CO
2 3 65
nK2CO3 <MM>K CO
Cálculo da fração molar de carbonato de potássio: X K2 CO3 = n = mK CO
2 3
mH O = 65
138
35 =
K2 CO3 + nH2 O 2 3 + 2 +
<MM>K CO <MM>H O 138 18
2 3 2
0,471
= 0,195
0,471 + 1,94

Questão 438 – (OLIMPÍADA NORTE/NORDESTE DE QUÍMICA) Um jovem químico decidiu medir o volume de uma
gota de água. Ele encontrou que 110 gotas eram formadas quando 3,00 cm³ de água eram escoados através de uma
bureta. De acordo com os handbooks de química o comprimento aproximado de uma molécula de água é de 1,50 A
(ângstron) e 1A = 10-10 m. A densidade da água é 1,00 g.cm-3; a constante de Avogadro é igual a 6,02 x 1023 moléculas.
Calcule, para uma gota de água:
I) O volume;
II) A massa;
III) O número de moléculas.
Use os dados determinados para uma gota de água (no item a) e calcule a concentração, em mol/L, da água. Calcule
o comprimento de uma cadeia formada por todas as moléculas de água contidas em uma gota.

Resolução: Item I) Cálculo do volume de uma única gota:


110 gotas ------ 3 cm³
1 gota ---------- Vgota
Vgota = 2,73 x 10-2 cm³

Item II) Cálculo da massa de uma única gota, a partir da densidade: Massa = d x V = 1,0 x 2,73 x 10-2 = 2,73 x 10-2 g

Item III) Cálculo do número de moléculas em uma única gota de água:

1 mol de água ------------- 18 g ----------------- 6,02 x 1023


2,73 x 10-2 g ----------------- N
N = 9,1 x 1020 moléculas

Para a determinação da concentração da quantidade de matéria da água, será necessário primeiramente calcular o
número de gotas.

110 gotas ------------- 3 cm³


X ---------------------- 1000 cm³, onde X corresponde o número de gotas em um litro de solução.
X = 3,67 x 104 gotas

Como cada gota apresenta uma massa que corresponde a 2,73 x 10-2 gramas, temos: mTOTAL =
0,0273 g
3,67 x 104 gotas x ( ) = 1001 g
1 gota

mH O
2 1001
nH2 O <MM>H O
Cálculo da concentração da quantidade de matéria: [H2 O] = Vsolução
= 2
Vsolução
= 18
1L
= 55,61 mol. L−1

Cálculo do comprimento da cadeia formada por todas as moléculas:

1 molécula --------------------------- 1,50 x 10-10 m


9,01 x 1020 moléculas -------------- X
X = 1,365 x 1011 m

217
1600 FÍSICO-QUÍMICA APLICADA EXERCÍCIOS COMENTADOS - IME – ITA – OLIMPÍADA

Questão 439 – (IME) Uma determinada amostra constituída de 204 gramas de cloreto de zinco anidro é dissolvida
em 720 gramas de água, formando uma solução com um volume de 750 ml a 20ºC. Para o cloreto de zinco nesta
solução, calcular:
a) Concentração da quantidade de matéria;
b) Normalidade;
c) Molalidade;
d) Fração molar.

Resolução: Dados do prblema: ZnCl2 = cloreto de zinco (soluto); H2O = água (solvente); Volume = 750 mL (0,750 L);
mZnCl2 = 204 g; mH2O = 720 g (0,72 kg)
204
Item a) Cálculo do número de mol do soluto: nZnCl2 = 136,4 = 1,49 mol

nZnCl2 1,49
Cálculo da quantidade de matéria do soluto: [ZnCl2 ] = V = 0,750 = 1,99 mol. L−1
solução

mZnCl2 1,49
Item b) Cálculo da concentração normal (N): N = = 136,4 = 3,99 eqg. L−1
eqgZnCl2 x Vsolução ( )x 0,750
2

nZnCl2 1,49
Item c) Cálculo da molalidade (W): W = = 0,720 = 2,07 mol. kg −1
msolvente
(kg)

Item d) Sabendo que a fração molar (X) do cloreto de zinco é a razão do número de mol do referido sal pelo número
de mol total, ou seja, a soma do número de mol do soluto e do solvente. Logo:
720
Cálculo do número de mol da água: nH2 O = = 40 mol
18

nZnCl2 1,49 1,49


Cálculo da fração molar em relação ao cloreto de zinco: X ZnCl2 = = = = 3,60 x 10−2
nZnCl2 +nH2O 1,49+40,0 41,49

Questão 440 – (IME) Uma solução 6,5 mol.L-1 de etanol em água tem massa específica 0,95 g.cm-3. Calcule a
molalidade e a fração molar de etanol dessa solução.

Resolução: Informações para a resolução do problema: etanol = C2H5OH (soluto) e água = H2O (solvente)

1 litro de solução ---------- 6,50 mol de etanol


1 cm³ de solução ---------- 0,95 g de solução

Cálculo da massa de etanol, a partir da concentração da quantidade de matéria: metanol = [etanol] x Vsolução x <MM>etanol
= 6,50 x 1,0 x 46 = 299 g

Cálculo da massa da solução:


0,95 g de solução ---------- 1 cm³ de solução (10-3 L)
msolução ----------------------- 1,0 Litro de solução
msolução = 950 g de solução

Cálculo da massa do solvente: msolução = msoluto + msolvente


950 g de solução = 299 g + msolvente
msolvente = 651 g de solvente
nC2 H5OH 6,50 mol
Cálculo da Molalidade (W): W = = = 9,98 mol. kg −1
msolvente
(kg) 0,651 kg

mH2O 651
Cálculo do número de mol da água: nH2 O = = = 36,17 mol
<MM>H2O 18

218
1600 FÍSICO-QUÍMICA APLICADA EXERCÍCIOS COMENTADOS - IME – ITA – OLIMPÍADA

nC2 H5OH 6,50 6,50


Cálculo da fração molar de etanol: X C2 H5 OH = n = 6,50+36,17 = 42,67 = 0,152
C2 H5 OH + nH2 O

Questão 441 – (IME) Em determinadas condições uma solução de carbonato de potássio em água é 3,10 mol.kg-1 e
2,79 mol.L-1. Determinar a sua massa específica.

Resolução: Cálculo da massa de solvente (água):

1 kg de solvente ---------- 3,10 mol de K2CO3


msolvente --------------------- 2,79 mol de K2CO3
msolvente = 0,90 kg
msoluto
Cálculo da massa de soluto, a partir do número de mol: nsoluto =
<MM>soluto

g
msoluto = nsoluto x < MM >soluto = 2,79 mol x 138 = 385,02 g
mol

Cálculo da massa da solução: msolução = msoluto + msolvente = 385,02 g + 900 g = 1285,02 g


msolução 1285,02 g
Cálculo da massa específica da solução: μ = Vsolução
= 1,0 L
= 1285,02 g. L−1

Questão 442 – (IME) Calcule a massa de 1 litro de uma solução aquosa de nitrato de zinco cuja concentração é
expressa por 0,643 mol.L-1 e por 0,653 mol.kg-1.

Resolução: Cálculo da massa de solvente (água):

1 kg de solvente ---------- 0,653 mol de Zn(NO3)2


msolvente --------------------- 0,643 mol de Zn(NO3)2
msolvente = 0,985 kg
m
Cálculo da massa de nitrato de zinco II, a partir do número de mol do soluto: nsoluto = <MM>
soluto
soluto
g
msoluto = nsoluto x < MM >soluto = 0,643 mol x 189,36 = 121,75 g
mol

Cálculo da massa da solução: msolução = msoluto + msolvente = 121,75 g + 985 g = 1106,76 g


Questão 443 – (IME) Uma solução aquosa de NaOH possui as seguintes características: fração molar de NaOH igual
a 0,01; massa específica da solução igual a 1,04 g.mL-1 e um litro dessa solução neutraliza 2 litros de solução aquosa
de ácido ortofosfórico. Calcule para a solução de H3PO4:
a) Molaridade;
b) Normalidade;
c) Concentração em g.L-1.

Resolução: Antes de calcularmos os diferentes tipos de concentrações para o ácido fosfórico, será necessário
calcular a concentração da quantidade de matéria do hidróxido de sódio. O problema apresenta como informação a
fração molar do hidróxido de sódio como 0,01. Sabendo que a fração molar é definida através da seguinte relação
nNaOH
matemática: X NaOH = = 0,01
nNaOH + nH2O

Para uma fração molar de hidróxido de sódio igual a 0,01, significa que o número de mol desta base corresponde
igual a um (1).
nNaOH = 1 mol.
Logo, o número de mol total será igual a 100, ou seja: nNaOH + nH2O = 100

Com isso, o número de mol de água será de: nH2O = 100 – 1 = 99 mol

219
1600 FÍSICO-QUÍMICA APLICADA EXERCÍCIOS COMENTADOS - IME – ITA – OLIMPÍADA

Cálculo da massa de soluto (NaOH): mNaOH = nNaOH x <MM>NaOH = 1 x 40 = 40 g

Cálculo da massa de solvente (H2O): mH2O = nH2O x <MM>H2O = 99 x 18 = 1782 g

Cálculo da massa da solução: msolução = msoluto + msolvente = 40 g + 1782 g = 1822 g

Cálculo do volume da solução:


1,04 g de solução ----------- 1 mL de solução
1822 g de solução ---------- Vsolução
Vsolução = 1751,92 mL
nNaOH
Cálculo da concentração da quantidade de matéria (mol x L-1) de hidróxido de sódio: [NH4 Cl] = =
Vsolução
40
mNaOH ( )
= 40
= 0,571 mol. L−1
<MM>NaOH x Vsolução 1751,92 x 10−3

Item a) Cálculo da concentração da concentração da quantidade de matéria do ácido fosfórico: H3PO4 + 3 NaOH →
Na3PO4 + 3 H2O

1 mol de H3PO4 ---------- 3 mol de NaOH


nH3PO4 ---------------------- nNaOH
3 x nH3PO4 = nNaOH

3 x [H3 PO4 ] x Vsolução = [NaOH] x Vsolução

3 x [H3 PO4 ] x 2 = 0,571 x 1

[H3 PO4 ] = 0,0952 mol. L−1

Item b) Cálculo da normalidade: N = [H3 PO4 ]. x, em que x corresponde o número de hidrogênios: N =


0,0952 x 3 = 0,286 eqg x L−1

Item c) Cálculo da concentração comum (C): CH3 PO4 = [H3 PO4 ] x < MM >H3 PO4 = 0,0952 x 98 =
9,33 g. L−1

Questão 444 - (IME) Determinar o volume máximo de ácido nítrico 0,5 mol.L-1 que pode ser obtido pela oxidação de
17 gramas de amônia.

Resolução: Cálculo do volume de ácido nítrico: NH3(g) + 2 O2(g) → HNO3(aq) + H2O(l)


1 mol de NH3 ---------- 1 mol de HNO3
nNH3 --------------------- nHNO3
mNH3
[HNO3 ] x Vsolução =
< MM >NH3

17
0,50 x Vsolução =
17

Vsolução = 2,0 L

220
1600 FÍSICO-QUÍMICA APLICADA EXERCÍCIOS COMENTADOS - IME – ITA – OLIMPÍADA

Questão 445 – (IME) A um excesso de bicarbonato de potássio adicionou-se 125 mL de solução de ácido sulfúrico.
O volume do gás liberado, medido a 20°C e 765 mmHg foi de 2,50 litros. Calcular a concentração do ácido expressa
em número de mol por litro.
Resolução: Cálculo da concentração da quantidade de matéria de ácido sulfúrico: 2 KHCO3 + H2SO4 → K2SO4 + 2
H2O + 2 CO2
1 mol de H2SO4 ---------- 2 mol de CO2
nH2SO4 ---------------------- nCO2

p x VCO2
2 x [H2 SO4 ] x Vsolução =
RxT
765
(760) x 2,50
2 x [H2 SO4 ] x 0,125 =
0,08206 x (20 + 273)

[H2 SO4 ] = 0,419 mol. L−1

Questão 446 – (IME) Determine o menor volume de solução de ácido clorídrico 0,250 mol.L-1 necessário para dissolver
completamente 13,50 gramas de alumínio metálico granulado.

Resolução: Cálculo do volume de ácido clorídrico, através da estequiometria: Al(s) + 3 HCl(aq) → AlCl3(aq) + 3/2 H2(g).
1mol de Al ----------- 3 mol de HCl
nAl --------------------- nHCl
nHCl = 3 x nAl
mAl
[HCl] x Vsolução = 3 x
< MM >Al

13,50
0,25 x Vsolução = 3 x
27,0

1
Vsolução = 4 x 3 x = 6,0 L
2

Questão 447 – (IME) Uma solução aquosa A preparada a partir de ácido bromídrico, é diluída com água até que sua
concentração seja reduzida à metade. Em titulação, 50 mL da solução diluída consomem 40 mL de uma solução
hidróxido de potássio 0,25 mol.L-1. Determine a concentração da solução A, em g.L-1.

Resolução: Cálculo da concentração da quantidade de matéria do ácido bromídrico (HBr): HBr + KOH → KBr + H2O
1 mol de HBr ---------- 1 mol de KOH
nHBr ---------------------- nKOH
nHBr = nKOH

[HBr] x Vsolução = [KOH] x Vsolução

[HBr]
x 50 = 0,25 x 40
2
20
[HBr] = = 0,40 mol. L−1
50

Cálculo da concentração comum de ácido bromídrico (g.L-1): CHBr = [HBr] x < MM >HBr = 0,40 x 81 =
32,4 g. L−1

221
1600 FÍSICO-QUÍMICA APLICADA EXERCÍCIOS COMENTADOS - IME – ITA – OLIMPÍADA

Questão 448 – (IME) O sulfato cúprico anidro é obtido a partir da reação de uma solução aquosa de ácido sulfúrico
98% (em massa), a quente, com cobre. Sabendo que a solução aquosa de ácido sulfúrico tem uma massa específica
1,84 g.cm-3 e que o ácido sulfúrico é o reagente limitante, calcule a massa de sulfato cúprico obtida a partir da reação
de 10,87 mL da solução aquosa do ácido sulfúrico.
Resolução: Equação química: Cu + 2 H2SO4 → CuSO4 + SO2 + 2 H2O.

2 mol de H2SO4 -------------- 1 mol de CuSO4


nH2SO4 -------------------------- nCuSO4

Pela relação estequiométrica, temos: nH2SO4 = 2 x nCuSO4

C 10 x d x (%) 10 x 1,84 x 98
Cálculo da quantidade de matéria de ácido sulfúrico: [H2 SO4 ] = <MM> = <MM> = 98
=
H2 SO4 H2 SO4
18,4 mol. L−1
mCuSO4
Cálculo da massa de sulfato cúprico: [H2 SO4 ] x Vsolução = 2 x <MM>CuSO4

mCuSO4
18,4 x 10,87 x 10−3 = 2 x
159,5

mCuSO4 = 15,95 g

Questão 449 – (ITA) O ácido nítrico concentrado encontrado no comércio tem 67% em massa de HNO3. Que massa
dessa solução deve ser usada para preparar 1 litro de uma solução 1,0 mol.L-1 de ácido nítrico?
Resolução: Cálculo da massa do ácido nítrico (HNO3), a partir da concentração da quantidade de matéria: [HNO3 ] =
nHNO3 mHNO3
V
= <MM> xV
solução HNO3 solução

mHNO3 = [HNO3 ] x < MM >HNO3 x Vsolução = 1,0 x 63 x 1,0 = 63,0 g

Cálculo da massa da solução:

67 gramas de ácido ---------------- 100 gramas de solução


63 gramas de ácido ---------------- X massa da solução
X = 94,03 g de solução

Questão 450 – (ITA) Calcular quantos mililitros de uma solução 0,200 mol.L-1 de ácido sulfúrico são necessários para
neutralizar 2,00 gramas de hidróxido de magnésio.
Resolução: Cálculo do volume de ácido sulfúrico: H2SO4(aq) + Mg(OH)2(aq) → MgSO4(aq) + 2 H2O(l)

1 mol de H2SO4 -------------- 1 mol de Mg(OH)2


nH2SO4 -------------------------- nMg(OH)2
nH2 SO4 = nMg(OH)2

mMg(OH)2
[H2 SO4 ] x Vsolução =
< MM >Mg(OH)2

2,00
0,200 x Vsolução =
58,0

[H2 SO4 ] x Vsolução = 0,1724 L (172,4 mL)

222
1600 FÍSICO-QUÍMICA APLICADA EXERCÍCIOS COMENTADOS - IME – ITA – OLIMPÍADA

Questão 451 – Uma solução aquosa realizada em um dos laboratórios do Instituto Federal do Rio de Janeiro – campus
Nilópolis, os íons presentes e suas concentrações mássicas estão apresentadas na tabela abaixo:
Íons presentes na solução C (g.L-1)
Li+ 14
Mg+2 22
Tl +3 55
SO4-2 70
I- X
A partir da tabela apresentada, determine o valor de X.

Resolução: Para a resolução deste exercício, será necessário calcular a concentração da quantidade de matéria de
cada íons na solução.

C 14
[Li+ ] = = = 2,0 mol. L−1
< MM >Li 7

C 22
[Mg +2 ] = = = 0,92 mol. L−1
< MM >Mg 24

C 55
[Tl+3 ] = = = 0,27 mol. L−1
< MM >Mg 204

C 70
[SO−2
4 ]= = = 0,73 mol. L−1
< MM >Mg 96

Solução resultante deve estar eletricamente neutra (∑Cargas(+) + ∑Cargas(-) = 0). Para um volume de um litro, temos:
{[Li+ ] x (+1)} + {[Mg +2 ] x (+2)} + {[Tl+3 ] x (+3)} + {[SO−2 −
4 ] x (−2)} + {[I ] x (−1)} = 0

{2,0 x (+1)} + {0,92 x (+2)} + {0,27 x (+3)} + {0,73 x (−2)} + {[I − ] x (−1)} = 0

[I − ] = 3,19 mol. L−1

Cálculo da concentração comum de iodeto: CI− = [I − ] x < MM >I− = 3,19 x 127 = 405,4 g. L−1

Questão 452 – (ITA) A 45°C, a densidade da água pura é 0,99 g.cm-3. Partindo desta informação, calcule [H2O], isto
é, o número de mol de água por litro de água pura nesta temperatura. Deixe bem claro o raciocínio utilizado nos
cálculos.

Resolução: Informação do problema: dH2O = 0,99 g.cm-3

Cálculo da massa de água, a partir de sua densidade:


1 cm³ (10-3 L) ---------- 0,99 g
1,0 L -------------------- mH2O
mH2O = 990 g
nH O
Cálculo da concentração da quantidade de matéria da água para um litro de solução: [H2 O] = V 2 =
solução
mH O
2 990
<MM>H O 55 mol
2
Vsolução
= 18
1,0 L
= 1,0 L
= 55 mol. L−1

223
1600 FÍSICO-QUÍMICA APLICADA EXERCÍCIOS COMENTADOS - IME – ITA – OLIMPÍADA

Questão 453 – (ITA) Deseja-se preparar 57 gramas de sulfato de alumínio [Al2(SO4)3] a partir de alumínio sólido (Al),
praticamente puro, e ácido sulfúrico (H2SO4). O ácido sulfúrico disponível é uma solução aquosa 96% (m\m), com
massa específica de 1,84 g.cm-3.
a) Qual a massa, em gramas, de alumínio necessária para preparar a quantidade de Al2(SO4)3 especificada?
Mostre os cálculos realizados.
b) Qual a massa, em gramas, de ácido sulfúrico necessária para preparar a quantidade de Al 2(SO4)3
especificada? Mostre os cálculos realizados.
c) Nas condições normais de temperatura e pressão (CNTP), qual é o volume, em litros, de gás formado durante
a preparação da quantidade de Al2(SO4)3 especificada? Mostre os cálculos realizados.
d) Caso a quantidade especificada de Al2(SO4)3 seja dissolvida em água acidulada, formando 1 litro de solução,
qual a concentração de íons Al+3 e de íons SO42- existentes nesta solução?

Resolução:

Item a) Cálculo da massa de alumínio (mAl): 2 Al(s) + 3 H2SO4(aq) → Al2(SO4)3(aq) + 3 H2(g)


2 mol de Al(s) ---------------------- 1 mol de Al2(SO4)3(aq)
2 mol x (27 g.mol-1) -------------- 1 mol x (342 g.mol-1)
mAl ---------------------------------- 57 g
mAl = 9,0 g

Item b) Cálculo da massa de ácido sulfúrico (mH2SO4): 2 Al(s) + H2SO4(aq) → Al2(SO4)3(aq) + H2(g)
3 mol de H2SO4(aq) ---------------------- 1 mol de Al2(SO4)3(aq)
3 mol x (98 g.mol-1) -------------------- 1 mol x (342 g.mol-1)
mH2SO4 ------------------------------------ 57 g
mH2SO4 = 49,0 g

Item c) Cálculo do volume de gás formado (VH2): 2 Al(s) + H2SO4(aq) → Al2(SO4)3(aq) + H2(g)
1 mol de Al2(SO4)3(aq) ---------------------- 3 mol de H2 (g)
1 mol x (342 g.mol-1) ---------------------- 3 mol x (22,4 L.mol-1)
57 g ------------------------------------------- VH2
VH2 = 11,2 L
nAl2 (SO4 )3
Item d) Cálculo da concentração da quantidade de matéria do sulfato de alumínio: [Al2 (SO4 )3 ] = Vsolução
=
57 𝑚𝑜𝑙
342 𝑥 1
= 0,167 𝐿
Cálculo das concentrações dos íons: Al2(SO4)3(aq) → 2 Al+3(aq) + 3 (SO4)-3(aq)
1 mol de Al2(SO4)3(aq) ---------- 2 mol de Al+3(aq) ---------- 3 mol de SO4-3
0,167 mol.L-1 -------------------- [Al+3] ---------------------- [SO4-3]

[Al+3] = 0,334 mol.L-1


[SO4-3] = 0,501 mol.L-1

Questão 454 - Determine a concentração da quantidade de matéria de uma solução de um ácido hipotético do tipo
diácido, H2X (massa molar = 160 g.mol-1), para o qual se sabe: d = 1,40 kg.L-1; % (m/m) H2X = 60%.
Resolução: Dados do problema:
1,40 kg de solução --------------------- 1,0 litro de solução
60 gramas de soluto -------------------- 100 gramas de solução

Cálculo do volume da solução:


1400 g de solução --------------------- 1,0 litro de solução
100 gramas de solução --------------- Vsolução
Vsolução = (1/14) L de solução

224
1600 FÍSICO-QUÍMICA APLICADA EXERCÍCIOS COMENTADOS - IME – ITA – OLIMPÍADA

60
nsoluto
Cálculo da concentração da quantidade de matéria do soluto: [Soluto] = = 160
1 = 5,25 mol. L−1
Vsolução ( )
14

Questão 455 - (GRILLO) Uma solução de ácido sulfúrico tem densidade igual a 1,60 g/cm³ e 88% em massa de
H2SO4. A partir destes dados, determine:
a) Concentração da quantidade de matéria;
b) Concentração comum;
c) Molalidade;
d) Normalidade.

CH2 SO4 10 x d x (%)


Resolução: a) Cálculo da concentração da quantidade de matéria: [H2 SO4 ] = = =
<MM>H2 SO4 <MM>H2 SO4
10 x 1,60 x 88 1408
98
= 98
= 14.37 mol. L−1

g
b) Cálculo da concentração comum: CH2 SO4 = 10 x d x (%) = 10 x 1,60 x 88 = 1408 L

nH2SO4 mH2SO4 88
c) Cálculo da molalidade: W = = = 12 g = 74,83 mol. kg −1
msolvente
(kg) <MM>H2SO4 x msolvente
(kg) 98 x {( ) x 1 kg}
1000 g
d) Cálculo do volume da solução:

1,60 g de solução ---------- 1 cm³


100 g de solução ----------- Vsolução
Vsolução = 62,50 cm³ (62,50 x 10-3 dm³)
mH2SO4 88
Cálculo da normalidade (concentração normal): N = eqg = 98 = 28,74 eqg. L−1
H2 SO4 x Vsolução ( ) x 62,50 x 10−3
2

Questão 456 – (ITA) Uma determinada amostra de hidróxido de cálcio impuro apresenta uma massa de
aproximadamente 2,00 gramas. Esta quantidade foi neutralizada com 50 mL de ácido sulfúrico (0,500 mol.L -1).
Determine a pureza da amostra de hidróxido.
Resolução: Cálculo da massa de hidróxido de cálcio: H2SO4(aq) + Ca(OH)2(aq) → CaSO4(aq) + 2 H2O(l)

1 mol de H2SO4 ---------- 1 mol de Ca(OH)2


na --------------------------- nb
na = nb
mCa(OH)2
[H2 SO4 ] x Vsolução =
< MM >Ca(OH)2

mCa(OH)2
0,500 x 50 x 10−3 =
74

mCa(OH)2 = 1,85 g

1,85
Cálculo da pureza de hidróxido de cálcio: (%)pureza = = 0,925 (92,5%)
2,0

225
1600 FÍSICO-QUÍMICA APLICADA EXERCÍCIOS COMENTADOS - IME – ITA – OLIMPÍADA

Questão 457 – (OArQ) Em um laboratório se preparam duas soluções, uma de sulfato de sódio e outra de nitrato de
sódio, dissolvendo-se 9,00 gramas de soluto para obter 100 cm³ de solução. Uma delas tem concentração igual a
1,059 mol/dm³. De qual solução se trata? Resolução: Cálculo da concentração da quantidade de matéria para as
duas soluções apresentadas pelo problema.
mNa SO4 9
Cálculo do número de mol: nNa2 SO4 = <MM>2 = 142
= 0,063 mol
Na2 SO4
mNa SO
2 4
nNa2 SO4 <MM>Na SO
Cálculo da concentração da quantidade de matéria do sulfato de sódio: [Na2 SO4 ] = Vsolução
= 2 4
Vsolução
=
9
0,0634
142
0,100
= 0,100
= 0,634 mol. L−1

mNaNO
3
nNaNO3 <MM>NaNO
Cálculo da concentração da quantidade de matéria do sulfato de sódio: [NaNO3 ] = = 3
=
Vsolução Vsolução
9
85
0,100
= 1,07 mol. L−1

Logo, a solução que apresenta a concentração da quantidade de matéria determinada trata-se da solução de nitrato
de sódio (NaNO3).

Questão 458 – (OArQ) Para realizar uma experiência são necessários 250 cm³ de solução aquosa de HCl 2,15 mol.L-
1 (densidade = 1,02 g.cm-3).

a) Quais são as massas de soluto e solvente necessárias?


b) Qual será a molalidade da solução?

Resolução:
Item a) Cálculo das massas de soluto e solvente:
Cálculo da massa da solução:
1,02 g de solução ---------- 1 cm³
msolução ---------------------- 250 cm³
msolução = 255 g
nHCl
Cálculo da massa de soluto (ácido clorídrico), a partir da concentração da quantidade de matéria: [HCl] = V =
solução
mH2 SO4
<MM>H2 SO4 x Vsolução

mH2 SO4
2,15 =
36,5 x 0,250

mH2 SO4 = 2,15 x 36,5 x 0,250 = 19,62 g

Cálculo da massa de solvente (H2O): msolução = msoluto + msolvente


255 = 19,62 + msolvente
msolvente = 235,38 g

Cálculo do número de mol do soluto em 250 cm³ de solução, a partir da concentração da quantidade de matéria: nHCl
= [HCl] x Vsolução
nHCl = 2,15 x 0,250 = 0,5375 mol
𝑛𝐻𝐶𝑙 0,5375 𝑚𝑜𝑙
Item b) Cálculo da molalidade: 𝑊 = 𝑠𝑜𝑙𝑣𝑒𝑛𝑡𝑒 = 0,235 𝑘𝑔
= 2,29 𝑚𝑜𝑙. 𝑘𝑔−1
𝑚(𝑘𝑔)

226
1600 FÍSICO-QUÍMICA APLICADA EXERCÍCIOS COMENTADOS - IME – ITA – OLIMPÍADA

Questão 459 – (OLIMPÍADA MINEIRA DE QUÍMICA) Conhecer soluções e suas concentrações é uma ferramenta
importante para o químico. Sobre elas, responda:
a) O bicarbonato de sódio é usado em ferramentas, extintores de incêndio, na manufatura de plásticos e
cerâmicos, como antiácido, entre outras coisas. Quando 16,8 gramas do composto foram dissolvidos em água
suficiente para preparar 200 mL de solução, qual será a concentração em quantidade de matéria desta solução?
b) Qual o volume de ácido clorídrico concentrado é necessário para se preparar 500 mL de solução de ácido
clorídrico a 0,10 mol.L-1? Dado: HCl (d = 1,18 g.mL-1; 36% em massa).
Resolução: Item a) Cálculo da concentração da quantidade de matéria do bicarbonato de sódio: [NaHCO3 ] =
mNaHCO
3 16,9
nNaHCO3 <MM>NaHCO 0,10
Vsolução
= 3
Vsolução
= 168
0,20
= 0,20 = 0,50 mol. L−1

Item b) Cálculo da concentração comum: C = 10 x d x (%) = 10 x (1,18) x 36 = 424,8 g.L-1


C 424,8
Cálculo da concentração da quantidade de matéria: [HCl] = <MM> = 36,5
= 11,64 mol. L−1

Cálculo do volume do ácido clorídrico concentrado (V1): [HCl]1 x V1 = [HCl]2 x V2


11,64 x V1 = 0,10 x 500
V1 = 4,29 mL

Questão 460 – (GRILLO - IFRJ) Em um experimento sobre eletrólise, realizado no IFRJ – Campus Nilópolis, os alunos
tiveram que preparar uma solução de sulfato de zinco. Foi pesado 2,0 gramas deste sal em um balão volumétrico de
capacidade de um litro. Sabendo que a densidade da água é igual a 0,9970 g/mL a 25°C, determine:
a) a fração molar do soluto;
b) a fração molar do solvente;
c) a concentração da quantidade de matéria;
d) a molalidade;
e) a densidade da solução.

Resolução: Dados do problema: massa de sulfato de zinco = 2,0 g e densidade da água (25°C) = 0,9970 g/mL.
mH2 O
Cálculo da massa de água (mH2O), a partir da densidade: d = V

g 1000 mL
mH2 O = 0,9970 x = 997 g
mL 1L
mZnSO4 2,0 g
Cálculo do número de mol (ZnSO4): nZnSO4 = <MM> = 161,41 g/mol = 1,24 x 10−2 mol
ZnSO4

mH O 997 g
Cálculo do número de mol (H2O): nH2 O = <MM>2 = 18 g/mol = 55,39 mol
H2 O

nZnSO4 1,24 x 10−2 mol


a) Cálculo da fração molar do soluto (sulfato de zinco): X ZnSO4 = n = 1,24 x 10−2 mol+55,39 mol =
ZnSO4 +nH2 O

2,24 x 10−4
nH2O 55,39 mol 55,39 mol
b) Cálculo da fração molar do solvente (água): X H2 O = n = 1,24 x 10−2 mol+55,39 mol = 55,40 mol =
ZnSO4 +nH2 O
0,9998
nZnSO4 1,24 x 10−2 mol
c) Cálculo da concentração da quantidade de matéria do sulfato de zinco: [ZnSO4 ] = V = 1L
=
solução

1,24 x 10−2 mol. L−1


nZnSO4 1,24 x 10−2 mol
d) Cálculo da molalidade (W): W = kg = 0,997 kg
= 1,24 x 10−2 mol. kg −1
msolvente

227
1600 FÍSICO-QUÍMICA APLICADA EXERCÍCIOS COMENTADOS - IME – ITA – OLIMPÍADA

msolução msoluto + msolvente 2,0 g + 997 g


e) Cálculo da densidade da solução: d = = = = 999 g. L−1
Vsolução Vsolução 1L

Questão 461 – Juntam-se 300 mL de ácido clorídrico com concentração molar igual a 0,40 mol.L-1 e 200 mL de
hidróxido de sódio com concentração molar igual a 0,80 mol.L-1. Determine as concentrações molares (mol.L-1) da
solução final:
a) em relação ao ácido clorídrico;
b) em relação ao hidróxido de sódio;
c) em relação ao sal formado.

Resolução: Cálculo do número de mol do ácido clorídrico: nHCl = [HCl] x Vsolução = 0,40 x 0,30 = 0,120 mol

Cálculo do número de mol do hodróxido de sódio: nNaOH = [NaOH] x Vsolução = 0,80 x 0,20 = 0,160 mol

Tabela de equilíbrio químico: HCl(aq) + NaOH(aq) → NaCl(aq) + H2O(l)

HCl(aq) (mol) NaOH(aq) (mol) → NaCl(aq) (mol) H2O(l)


Início 0,12 0,16 0 0
Reage 0,12 0,12 0,12 0,12
Equilíbrio 0 0,04 0,12 0,12
Cálculo do volume total = 200 mL + 300 mL = 500 mL
nHCl 0
a) Concentração da quantidade de matéria em relação ao ácido: [HCl] = = =0
Vsolução 0,50
n 0,04
b) Concentração da quantidade de matéria em relação ao hidróxido de sódio: [NaOH] = V NaOH = 0,50 =
solução
0,08 mol. L−1
n 0,12
c) Concentração da quantidade de matéria em relação ao sal: [NaCl] = V NaCl = 0,50 = 0,24 mol. L−1
solução

Questão 462 – O ácido sulfúrico concentrado apresenta 96,0% em massa de H2SO4. Determine as frações molares
do ácido e da água.
Resolução: Informação do problema: 96% em massa de ácido sulfúrico = significa que na solução há 96 gramas de
ácido para 100 gramas de solução.

Massa da solução = massa de soluto + massa de solvente


100 g = 96 g + massa de solvente
Massa de solvente = 4 gramas.
mH SO4 96
Cálculo do número de mol de ácido sulfúrico: nH2 SO4 = <MM>2 = 98 = 0,979 mol
H2 SO4

mH O 4
Cálculo do número de mol de água: nH2 O = <MM>2 = 18 = 0,222 mol
H2 O

Cálculo do número de mol total (nTOTAL): nTOTAL = nH2O + nH2SO4 = 0,222 + 0,979 = 1,201 mol

Cálculo das frações molares, tanto do solvente quanto para o soluto:

nH2 O 0,222
X H2 O = = = 0,185
nH2 O + nH2 SO4 1,201

nH2 SO4 0,979


X H2 SO4 = = = 0,815
nH2 O + nH2 SO4 1,201

228
1600 FÍSICO-QUÍMICA APLICADA EXERCÍCIOS COMENTADOS - IME – ITA – OLIMPÍADA

Questão 463 – (ITA) São misturados volumes iguais de soluções aquosas de duas substâncias distintas, ambas as
soluções com concentração 5,00 x 10 -3 mol.L-1. Dentre os pares abaixo, assinale aquele para o qual não irá ocorrer
reação perceptível.
a) Na2SO3 + HCl
b) KCl + MgSO4
c) HI + Pb(NO3)2
d) CaCl2 + K2CO3
e) CO2 + Ba(OH)2

Resolução: Alternativa B.

A concentração fornecida no problema foi apenas para informar que as soluções apresentam uma concentração da
quantidade de matéria igual a 5,00 x 10 -3 mol.L-1. Os efeitos que são perceptiveis quando uma reação química é
processada podem ser assim listadas: mudança de cor; odor característico; possível liberação de algum produto no
estado gasoso; mudança na coloração da solução; formação de precipitado, etc. Para a reação proposta neste
exercício, a reação entre o cloreto de potássio com o sulfato de magnésio é uma reação classificada como dupla troca,
que a equação química balanceada está apresentada da seguinte forma: 2 KCl(aq) + MgSO4(aq) → K2SO4(aq) + MgCl2(aq).
Os produtos formados, sulfato de potássio (K2SO4) e cloreto de magnésio (MgCl2) não apresentam nenhum efeito que
possa ser perceptível, uma vez que a reação ocorre entre dois sais, em que os produtos formados são sais que não
irão apresentar nenhuma mudança que seja perceptível.

Questão 464 - (OLIMPÍADA DE BRASILEIRA DE QUÍMICA) Calcule quantos mililitros de uma solução de HCl 0,250
mol.L-1, são necessários para:
a) Preparar 100 mL de uma solução de HCl 0,100 mol.L-1;
b) Neutralizar 50 mL de uma solução de Ba(OH)2 0,200 mol.L-1;
c) Dissolver 0,200 g de carbonato de cálcio, de acordo com a reação CaCO 3(s) + 2 H+(aq) → Ca+2(aq) + H2O(l) +
CO2(g).

Resolução:
Item a) Processo de diluição do ácido clorídrico: [HCl]1 x V1 = [HCl]2 x V2
0,250 x V1 = 100 x 0,100
V1 = 40 mL

Item b) Equação química: 2 HCl + Ba(OH)2 → BaCl2 + 2 H2O


2,0 mol de HCl ---------- 1 mol de Ba(OH)2
na -------------------------- nb
0,250 x Vácido = 2,0 x 0,200 x 50
Vácido = 80 mL
Item c) Equação química: CaCO3(s) + 2 HCl(aq) → CaCl2(aq) + 2 H2CO3(aq)
CaCO3(s) + 2 H+(aq) + 2Cl-(aq) → Ca+2(aq) + 2Cl-(aq) + H2O(l) + CO2(g)
CaCO3(s) + 2 H+(aq) → Ca+2(aq) + H2O(l) + CO2(g)

1 mol de CaCO3(s) ---------- 2 mol de H+(aq)


ns ----------------------------- na
nácido = 2 x nsal
msal
[H + ] x Vsolução = 2 x
< MM >sal

0,200
[H + ] x 0,250 = 2 x
100

[H + ] = 0,016 mL (16 mL)

229
1600 FÍSICO-QUÍMICA APLICADA EXERCÍCIOS COMENTADOS - IME – ITA – OLIMPÍADA

Questão 465 - (IME) Foram misturados 500 mL de uma solução 0,40 mol.L-1 de Na2SO4 com 500 mL 0,20 mol.L-1 de
CaCl2, verificando-se a formação de precipitado. Calcule as concentrações dos íons no equilíbrio, em solução, e a
massa do precipitado.

Resolução: Equação química: Na2SO4(aq) + CaCl2(aq) → CaSO4(s) + 2 NaCl(aq)

O precipitado formado trata-se do sulfato de cálcio (CaSO4).

Cálculo do número de mol de sulfato de sódio (1), a partir da concentração da quantidade de matéria: n1 = [Na2SO4]
x Vsolução = 0,40 x 0,500 = 0,200 mol

Cálculo do número de mol de cloreto de cálcio (2), a partir da concentração da quantidade de matéria: n2 = [CaCl2] x
Vsolução = 0,20 x 0,500 = 0,100 mol

Reagente limitante: cloreto de cálcio (CaCl2)

Pela estequiometria, temos: Na2SO4(aq) + CaCl2(aq) → CaSO4(s) + 2 NaCl(aq)

1 mol de CaCl2(aq) ------------1 mol de Na2SO4(aq)


nCaCl2 --------------------------- nCaSO4
nCaCl2 = nCaSO4
mCaSO4
Cálculo da massa de sulfato de cálcio: nCaSO4 = <MM>
CaSO4
mCaSO4 = 0,10 𝑥 136 = 13,60 g

Cálculo da concentração de íons sódio [Na+], a partir do sulfato de sódio, sabendo que o volume total da mistura
corresponde a 1000 mL (1 L):

Na2SO4(aq) → 2 Na+(aq) + SO4-2(aq)


1 mol de Na2SO4(aq) -------------- 2 mol de Na+(aq)
0,40 mol de Na2SO4(aq) ---------- nNa+
nNa+ = 0,80 mol
nNa+ 0,80
Cálculo da concentração da quantidade de matéria para os íons sódio, [Na +]: [Na+ ] = V = 1,0
=
solução
−1
0,80 mol. L

Cálculo da concentração da quantidade de matéria para os íons sódio, [Na+]2: [Na+ ]1 x V1 = [Na+ ]2 x V2

0,80 x 500 = [Na+ ]2 x (500 + 500)

400 = [Na+ ]2 x 1000

[Na+ ]2 = 0,40 mol. L−1

Cálculo da concentração de íons sulfato [SO4-], a partir do sulfato de sódio, sabendo que o volume total da mistura
corresponde a 1000 mL (1 L):

Na2SO4(aq) → 2 Na+(aq) + SO4-2(aq)


1 mol de Na2SO4(aq) ------------------ 1 mol de SO4-2(aq)
0,20 mol.L-1 de Na2SO4(aq) ---------- nSO4-2
nSO4-2 = 0,20 mol.L-1

Cálculo da concentração da quantidade de íons sulfato, [SO4-2]: [SO−2 −2


4 ]1 x V1 = [SO4 ]2 x V2

230
1600 FÍSICO-QUÍMICA APLICADA EXERCÍCIOS COMENTADOS - IME – ITA – OLIMPÍADA

0,20 x 500 = [Na+ ]2 x (500 + 500)

[SO−2
4 ]2 = 0,10 mol. L
−1

Cálculo da concentração de íons cloreto [Cl-], a partir do cloreto de cálcio, sabendo que o volume total da mistura
corresponde a 1000 mL (1 L):

CaCl2 (aq) → Ca+2(aq) + 2 Cl-(aq)


1 mol de CaCl2(aq) ------------------- 2 mol de Cl-(aq)
0,20 mol.L-1 de CaCl2(aq) ----------- nCl-
nCl-= 0,40 mol.L-1

Cálculo da concentração de íons cloreto, [Cl-]: [Cl− ]1 x V1 = [Cl− ]2 x (V1 + V2 )

0,40 x 500 = [Cl− ]2 x (500 + 500)


[Cl− ]2 = 0,20 mol. L−1

Questão 466 - (IME) Foram neutralizados 25,0 ml de solução aquosa saturada de hidróxido de cálcio com igual
volume de solução M/40 de ácido sulfúrico. Determinar a solubilidade do hidróxido de cálcio nestas condições,
expressando o resultado em gramas por litro de solução.

Resolução: Equação química: H2SO4(aq) + Ca(OH)2(aq) → CaSO4(aq) + 2 H2O(l)

1 mol de H2SO4 ------------- 1 mol de Ca(OH)2


nácido --------------------------- nbase
nácido = nbase

(624/760) x V
Cálculo do volume da solução de ácido clorídrico: 0,08206 x (27+273)
HCl
= 0,500 x 0,200

VHCl = 3,0 L

Cbase 1
=
< MM >base 40

Cbase 1
=
74 40

Cbase = 1,85 g. L−1

Questão 467 - (IME) Um litro de uma solução aquosa, contendo inicialmente 9,8 g de ácido sulfúrico, foi submetida a
eletrólise pela passagem de uma corrente de 17,5 ampères, durante 900 segundos. Pede-se:
a) A normalidade da solução aquosa, antes da eletrólise.
b) A normalidade da solução aquosa, após a eletrólise, considerando desprezível a variação de volume da solução
aquosa;

c) O abaixamento relativo da pressão máxima de vapor d’água P, após a eletrólise, podendo-se considerar:
constante tonométrica da água, 20ºC, Kt = 0,018; pressão máxima de vapor d’água a 20ºC, p = 17,5mmHg e densidade
da solução a 20ºC: d = 1g/cm3.

Resolução:
mH2SO4 9,80
a) Cálculo da normalidade (N) antes do processo de eletrólise: NH2 SO4 = = 98 =
eqgH2SO4 x Vsolução ( )x1
2
0,200 eqg. L−1

231
1600 FÍSICO-QUÍMICA APLICADA EXERCÍCIOS COMENTADOS - IME – ITA – OLIMPÍADA

b) Cálculo da massa após o processo de eletrólise: H2SO4(aq) → 2H+(aq) + SO4-2(aq)

Observa-se a participação de 2 mol de elétrons:


98 g -------------- {2 mol de e- x 96500 C / mol de elétrons}
mH2SO4 ----------- 17,5 x 900 C
mH2SO4 = 8,00 g
mH2SO4 9,80−8,0 1,80
Cálculo da normalidade (N) depois do processo da eletrólise: NH2 SO4 = eqgH2 SO4 x Vsolução
= 98 = 49
=
( )x1
2
0,0367 eqg. L−1

Cálculo da massa de solução, a partir da densidade: Volume = 1 dm³ (1000 cm³).


m=dxV
m = 1 g/cm³ x (1000 cm³) = 1000 g

Massa da solução = massa do soluto + massa do solvente


1000 g = 8,00 g + massa do solvente
Massa do solvente = 992,0 g
∆P
Cálculo do abaixamento relativo da pressão de vapor, considerando um grau de dissociação igual a 100%: p°
=
8,0⁄
0,018 x ( 98) x [1 + (3 − 1) x 1] = 4,44 x 10−3
0,992

Questão 468 - Determine a molalidade e a fração molar do cloreto de amônio, que apresenta 0,850% em massa de
cloreto de amônio.

Resolução: A solução de cloreto de amônio (NH4Cl) apresenta 0,85 gramas para cada 100 gramas de solução.

Cálculo da massa de solvente (H2O): msolução = msoluto + msolvente


100 g = 0,85 g + msolvente
msolvente = 99,15 g
mNH Cl
4 0,850
nNH4Cl <MM>NH Cl 0,0159 mol
Cálculo da molalidade (W): W = = 4
= 53,5
= 99,15 x 10−3 kg = 0,160 mol. kg −1
msolvente
(kg) msolvente
(kg) msolvente
(kg)

mNH Cl
4 0,850
nNH4ClH <MM>NH Cl
Cálculo da fração molar (XNH4Cl): X NH4 Cl = n = mNH Cl
4
mH O = 53,5
0,850 99,15 =
NH4 Cl + nH2 O 4 + 2 +
<MM>NH Cl <MM>H O 53,5 18
4 2
0,0159 mol
= 2,88 x 10−3
0,0159 mol + 5,51 mol

Questão 469 – Responda e calcule os seguintes itens abaixo.


a) Uma quantidade de cloreto de magnésio apresentando uma massa igual a 77,0 g é adicionada a 112,0 g de
água para a formação de uma solução que apresenta uma densidade igual a 1,76 g.mL-1. A partir desta informação,
determine:
i) A percentagem em massa;
ii) A fração molar do sal;
iii) A concentração da quantidade de matéria;
iv) A molalidade.
b) Suponha que uma solução apresente as seguintes soluções para um volume total de 350 mL: 30 mL de
cloreto de sódio (0,050 mol.L-1), 55 mL de cloreto de potássio (0,200 mol.L-1) e 45 mL de cloreto de alumínio (0,450
mol.L-1). Considerando que todos os eletrólitos presentes são fortes, determine a concentração final de íons cloreto.
c) Determine o volume de cloreto de hidrogênio medido a 10 atm e 37°C, necessários para neutralizar 2,00
litros de hidróxido de bário (0,25 mol.L-1).

232
1600 FÍSICO-QUÍMICA APLICADA EXERCÍCIOS COMENTADOS - IME – ITA – OLIMPÍADA

Resolução:
𝑚 77,0
a) % = 𝑚 𝑠𝑜𝑙𝑢𝑡𝑜 = 77,0+112,0 = 0,41 (41%)
𝑠𝑜𝑙𝑢çã𝑜
mMgCl
2
nMgCl2 <MM>MgCl
i) Fração molar do cloreto de magnésio: X MgCl2 = = mMgCl
2
mH O =
nMgCl2 + nH2O 2 + 2
<MM>MgCl <MM>H O
2 2
0,810
0,810 + 6,22
= 0,115
ii) Cálculo do volume da solução:
1 mL de solução ---------- 1,76 g de solução
Vsolução ---------------------- 189 g de solução
Vsolução = 107,39 mL

𝑀𝑔𝐶𝑙2 𝑛
iii) Cálculo da concentração da quantidade de matéria do cloreto de magnésio: [𝑀𝑔𝐶𝑙2 ] = 𝑉 =
𝑠𝑜𝑙𝑢çã𝑜
77,95 −1
= 7,55 𝑚𝑜𝑙. 𝐿
107,39 𝑥 10−3
𝑛𝑀𝑔𝐶𝑙2 0,810
iv) Cálculo da molalidade (W): 𝑊 = 𝑠𝑜𝑙𝑣𝑒𝑛𝑡𝑒 = 0,112
= 7,23 𝑚𝑜𝑙. 𝑘𝑔−1
𝑚(𝑘𝑔)

H2 O
b) Analisando a solução de cloreto de sódio: NaCl(aq) → Na+ −
(aq) + Cl(aq)
1 mol de NaCl -------------------- 1 mol de Cl-
0,050 mol.L-1 de NaCl ----------- [Cl-]1
[Cl-]1 = 0,050 mol.L-1
H2 O
Analisando a solução de cloreto de potássio: KCl(aq) → K+ −
(aq) + Cl(aq)
1 mol de KCl -------------------- 1 mol de Cl-
0,200 mol.L-1 de KCl ---------- [Cl-]2
[Cl-]2 = 0,200 mol.L-1
H2 O
Analisando a solução de cloreto de alumínio: AlCl3 (aq) → Al+3 −
(aq) + 3 Cl(aq)
1 mol de KCl -------------------- 3 mol de Cl-
0,450 mol.L-1 de KCl ---------- [Cl-]3
[Cl-]3 = 1,35 mol.L-1

Cálculo da concentração da quantidade de matéria final de cloreto, [Cl-]FINAL: [Cl-]1 x V1 + [Cl-]2 x V2 + [Cl-]3 x V3 = [Cl-
]FINAL x VFINAL
(0,030 x 50) + (0,200 x 55) + (1,35 x 45) = [Cl-]FINAL x (30 + 55 + 45)
[Cl-]FINAL = (73,25/130) = 0,56 mol.L-1

c) Equação química: 2 HCl(g) + Ba(OH)2(aq) → BaCl2(aq) + H2O(l)

2 mol de HCl(g) ---------- 1 mol de Ba(OH)2(aq)


ngás ------------------------- nbase
ngás = 2 x nbase
p x VHCl
Considerando que o gás (cloreto de hidrogênio) apresenta comportamento ideal, temos: RxT
=
2 x [Ba(OH)2 ] x Vsolução

10 x VHCl
= 2 x 0,25 x 2,0
0,08206 x (37 + 273)

VHCl = 2,54 L

233
1600 FÍSICO-QUÍMICA APLICADA EXERCÍCIOS COMENTADOS - IME – ITA – OLIMPÍADA

Questão 470 – (GRILLO) Uma solução de soda cáustica apresenta densidade igual a 1,55 g/L e 37,50% em massa.
a) Calcule a molalidade dessa solução e dos dados apresentados, qual deles não é necessário para o cálculo.
b) Calcule a fração molar da soda cáustica nessa solução.
c) Calcule a concentração da quantidade de matéria.

Resolução:
a) A partir do título em massa, sabemos que: 37,50 g de NaOH/100 g de solução

Cálculo da massa de solução: msolução = msoluto + msolvente


msolvente = msolução - msoluto = 100 g – 37,50 g = 62,50 g
mNaOH 37,50
nNaOH <MM>NaOH 0,9375
Cálculo da molalidade (W): W = = = 40
= = 15,0 mol. kg −1
msolvente
(kg) msolvente
(kg) 62,50 x 10−3 62,50 x 10−3

Observação: A densidade da solução (d) não foi necessário para o cálculo da molalidade.

b) Sabendo que o número de mol da soda cáustica já calculada no item b é igual a 0,9375 mol, a fração molar
n nNaOH 0,9375
será: X NaOH = n NaOH+n
= mH O
2
= 62,50 = 0,213
NaOH H2 O nNaOH + 0,9375+
<MM>H O 18
2

c) Cálculo da concentração comum: CNaOH = 10 x d x (%) = 10 x 1,55 x 37,50 = 581,25 g.L-1


C 581,25
Cálculo da concentração da quantidade de matéria: [NaOH] = V = 40
= 14,53 mol. L−1
solução

Questão 471 – (OLIMPÍADA BRASILEIRA DE QUÍMICA) Uma solução saturada de nitrato de potássio constituída
além do sal, 100 g de água, está a temperatura de 70°C. Essa solução é resfriada a 40°C, ocorrendo precipitação de
parte dosal dissolvido. Com base nesses dados e no gráfico apresentado abaixo:

Pode-se afirmar que a massa de sal que precipitou foi de aproximadamente:


a) 20 g
b) 40 g
c) 60 g
d) 80 g
e) 100 g

Resolução: Alternativa D.

Dados obtidos através do gráfico que relaciona a solubilidade do nitrato de potássio com a temperatura.

234
1600 FÍSICO-QUÍMICA APLICADA EXERCÍCIOS COMENTADOS - IME – ITA – OLIMPÍADA

No gráfico os pontos destacados são os seguintes: a 70°C há 140 g de KNO 3 para cada 100 g de H2O e a 40°C há
60 g de KNO3 para cada 100 g de H2O. Então a massa de nitrato de potássio que precipitou é igual a: 140 g – 60 g =
80 g.

Questão 472 - (OLIMPÍADA BRASILEIRA DE QUÍMICA) Uma amostra de um líquido diprótico pesando 12,25 g foi
dissolvida em água e o volume da solução completado para 500 mL. Se 25,0 mL desta solução são neutralizados com
12,5 mL de uma solução de KOH 1,00 mol.L-1, a massa molar desse ácido, em g.mol-1, considerando que os dois
prótons foram neutralizados, é igual a:
a) 22,5
b) 24,5
c) 49,0
d) 98,0
e) 122,5

Resolução: Alternativa D. Como 2 prótons do ácido (2 H+) são dissociáveis, logo há uma relação estequiométrica
de dois mols de ácido para 1 mol de base.

2 x [ácido] x Vsolução = [base] x Vsolução

mácido
2x ( ) x Vsolução = [base] x Vsolução
< MM >ácido x Vsolução

12,25
2x ( ) x 25,0 = 1,0 x 12,5
< MM >ácido x 0,500
g
< MM >ácido = 98
mol
Questão 473 – (IME) A figura a seguir representa as curvas de solubilidade de duas substâncias A e B.

Com base nela, pode-se afirmar que:


a) No ponto 1, as soluções apresentam a mesma temperatura mas as solubilidades de A e B são diferentes.
b) A solução da substância A está supersaturada no ponto 2.
c) As soluções são instáveis no ponto 3.
d) As curvas de solubilidade não indicam mudanças na estrutura dos solutos.
e) A solubilidade da substância B segue o perfil esperado para a solubilidade de gases em água.

235
1600 FÍSICO-QUÍMICA APLICADA EXERCÍCIOS COMENTADOS - IME – ITA – OLIMPÍADA

Resolução: Alternativa E.
a) Falso. Neste ponto observa-se a mesma temperatura e também solubilidades iguais.
b) Falso. Não se trata de supersaturação, e sim um ponto de inflexão, normalmente criado pela coexistência de
duas “formas” de hidratação, como CuSO4·5H2O e CuSO4.
c) Falso. O ponto A seria insaturada, logo estável. B seria supersaturada, logo instável.
d) Falso. O ponto (2) é ponto de inflexão da substância A, logo mudança de estrutura do soluto.
e) Verdadeiro. A solubilidade dos gases é decrescente com a temperatura.

Questão 474 – (IME) Um erlenmeyer contém 10,0 mL de uma solução de ácido clorídrico, juntamente com algumas
gotas de uma solução de fenolftaleína. De uma bureta, foi-se gotejando uma solução 0,100 mol.L-1 de hidróxido de
sódio até o aparecimento de leve coloração rósea. Nesse momento, observou-se um consumo de 20,0 mL da solução
alcalina. Pode-se afirmar que a concentração de HCl na solução ácida original era de:
a) 3,65 x 10–3 g.cm-3
b) 7,30 x 10–3 g.cm-3
c) 4,00 x 10–3 g.cm-3
d) 3,20 x 10–3 g.cm-3
e) 2,00 x 10–3 g.cm-3

Resolução: Alternativa B.

Cálculo do número de mol de ácido clorídrico (HCl): HCl(aq) + NaOH(aq) → NaCl(aq) + H2O(l)

HCl(aq) + NaOH(aq) → NaCl(aq) + H2O(l)


1 mol de HCl(aq) ---------- 1 mol de NaOH(aq)
nHCl ------------------------- nNaOH
nHCl = nNaOH

[HCl] x Vsolução = [NaOH] x Vsolução


[HCl] x 10 = 0,100 x 20
[HCl] = 0,200 mol.L-1

Cálculo da concentração comum do HCl: C = [HCl] x < MM >HCl = 0,200 x 36,5 = 7,30 g. L−1

g 1L 1 dm3
Convertendo para g.cm-3, temos: C = 7,30 L x x 1000 cm³ = 7,30 x 10−3 g. cm−3
1 dm³

Questão 475 – (ITA) O rótulo de um frasco diz que contém solução 1,50 molal de LiNO3 em etanol. Isto quer dizer
que a solução contém:
a) 1,50 mol de LiNO3 /quilograma de solução
b) 1,50 mol de LiNO3 /litro de solução
c) 1,50 mol de LiNO3 /quilograma de etanol
d) 1,50 mol de LiNO3 /litro de etanol
e) 1,50 mol de LiNO3 /mol de etanol

Resolução: Alternativa C.
A molalidade indica o número de mol do soluto por quilograma de solvente, portanto, uma solução 1,5 mol.kg -1 de
nitrato de lítio apresenta 1,50 mol de LiNO3 por quilograma de etanol.

Questão 476 – (ITA) Para preparar 80 litros de uma solução aquosa 12% (massa/massa) de KOH (massa específica
da solução = 1,10 g/cm3) foram adicionados x litros de uma solução aquosa 44% (massa/massa) de KOH (massa
específica da solução = 1,50 g/cm3) e y litros de água deionizada (massa específica = 1,00 g/cm3). Os valores de x e
de y são respectivamente:
a) 12 L e 68 L
b) 16 L e 64 L
c) 30 L e 50 L

236
1600 FÍSICO-QUÍMICA APLICADA EXERCÍCIOS COMENTADOS - IME – ITA – OLIMPÍADA

d) 36 L e 44 L
e) 44 L e 36 L.
Resolução: Alternativa B.

KOH(1) + H2O → KOH(2)


44% (m/m) 12% (m/m)
d = 1,50 g.cm-³ d = 1,10 g.cm-³
x(L) y(L) 80 L

Cálculo da concentração da quantidade de matéria do hidróxido de potássio (1): C1 = 10 x (%) x d = 10 x 44 x 1,50 =


660 g.L-1.

Cálculo da concentração comum do hidróxido de potássio (2): C2 = 10 x (%) x d = 10 x 12 x 1,10 = 132 g.L-1

Cálculo do volume de KOH (1): C1 x V1 = C2 x V2


660. V1 = 132 x 80
x = 16 L

Sendo o volume de uma mistura uma propriedade aditiva, tem: x + y = 80 L


16 L + y = 80 L
y = 64 L

Questão 477 - (ITA) 1.000 cm3 de solução aquosa de HCl 5,0 N devem ser obtidos utilizando-se apenas duas soluções
de HCl, respectivamente, 3,0 N e 6,0 N. Admitindo-se que os volumes se somem, devem ser misturados:
a) 444 cm3 de HCl 3,0 N com 556 cm3 de HCl 6,0 N
b) 429 cm3 de HCl 3,0 N com 571 cm3 de HCl 6,0 N
c) 375 cm3 de HCl 3,0 N com 625 cm3 de HCl 6,0 N
d) 333 cm3 de HCl 3,0 N com 667 cm3 de HCl 6,0 N
e) 222 cm3 de HCl 3,0 N com 778 cm3 de HCl 6,0 N

Resolução: Alternativa D.

HCl(1) + HCl(2) → HCl(3)


V1 x N1 + V2 x N2 = V3 x N3

Equação (1): V1 x 3,0 N + V2 x 6,0 N = 1000 x 5,0 N


Equação (2): volumes são aditivos: V1 + V2 = 1000

Através da equação (2): Isolando V2: V2 = 1000 – V1

Substituindo na equação (1): V1 x (3,0 N) + V2 x (6,0 N) = 5000


3 x V1 + (1000 – V1) x 6,0 = 5000
3 x V1 = 1000

V1 = 333,33 cm³
V2 = 1000 – V1 = 666,67 cm³

Questão 478 – (ITA) O volume, em litros, de uma solução 0,30 mol.L-1 de sulfato de alumínio que contém 3,0 mol de
cátion de alumínio, é:
a) 2,50
b) 3,30
c) 5,0
d) 9,0
e) 10,0

237
1600 FÍSICO-QUÍMICA APLICADA EXERCÍCIOS COMENTADOS - IME – ITA – OLIMPÍADA

Resolução: Alternativa C.

Equação química de ionização do sulfato de alumínio: Al2(SO4)3(s) → 2 Al+3(aq) + 3(SO4)-3(aq)

1 mol de Al2(SO4)3 -------------- 2 mol de Al+3


nsal --------------------------------- 3 mol
nsal = 1,50 mol de Al2(SO4)3

Cálculo do volume de sulfato de alumínio, a partir da concentração da quantidade de matéria: Vsolução =


nAl2(SO4 )3 1,50
[Al2 (SO4 )3 ]
= 0,30 = 5,0 L

Questão 479 – (ITA) Um litro de uma solução aquosa contém 0,30 mol de íons Na+, 0,28 mol de íons Cl–, 0,10 mol
de íons SO42– e x mol de íons Fe3+. A concentração de íons Fe3+ (em mol.L-1) presentes nesta solução é:
a) 0,03
b) 0,06
c) 0,08
d) 0,18
e) 0,26

Resolução: Alternativa B.
A solução resultante deve estar eletricamente neutra (∑Carga(+) + ∑Carga(-) = 0), logo em um litro de solução temos:

0,30 mol de íons Na+ corresponde a 0,30 mol de cargas positivas+


0,28 mol de íons Cl– corresponde a 0,28 mol de cargas negativas–
0,10 mol de íons SO2–4 corresponde a 0,20 mol de cargas negativas–
x mol de íons Fe3+ corresponde a (3x) mol de cargas positivas+
Total de cargas positivas = (0,30 + 3.[Fe+3]) mol
Total de cargas negativas = (0,28 + 0,20) mol

0,30 mol + 3 x [Fe+3] = 0,48 mol


3 x [Fe+3] = 0,48 mol – 0,30 mol
3 x [Fe+3] = 0,18 mol
[Fe+3] = 0,06 mol, para um volume de solução de 1,0 litro, a concentração da quantidade de matéria do Fe3+ será igual
0,06 mol mol
a [Fe+3 ] = 1 L = 0,06 L

Questão 480 – (ITA) A análise quantitativa de certo líquido não aquoso, de dissociação iônica desprezível, revelou
conter somente íons Na+, Mg+2, Cl- e SO4-2. Por método analíticos quantitativos estabeleceu-se que o líquido contém:
0,170 mol.L-1 de Mg+2
0,120 mol.L-1 de SO42–
0,300 mol.L-1 de Cl–.
Com estas informações se pode concluir que a concentração de Na+ na amostra:
a) Deve ser de 0,250 mol.L-1
b) Deve ser de 0,550 mol.L-1
c) Deve ser de 0,200 mol.L-1
d) Tem um valor diferente das três opções acima
e) Pode ter mais de um valor

Resolução: Alternativa C. Em 1,0 litro de solução, temos:

Total de cargas positivas = [(0,170 x 2) + W x (+1)]


Total de cargas negativas = [0,120 x (-2) + 0,300 x (-1)] mol.L-1

∑Carga(+) + ∑Carga(-) = 0
[0,170 x 2 + W x (+1)] + [0,120 x (-2)] + 0,300 x (-1)] = 0

238
1600 FÍSICO-QUÍMICA APLICADA EXERCÍCIOS COMENTADOS - IME – ITA – OLIMPÍADA

0,340 + W – 0,240 – 0,300 = 0


0,340 + W – 0,540 = 0
W = 0,200 mol.L-1

Questão 481 – (ITA) Sabe-se que uma solução só contém os seguintes íons:
0,10 mol.L-1 de K+
0,16 mol.L-1 de Mg+2
0,16 mol.L-1 de Cl– e
x mol.L-1 de SO42–. Este x deve ser igual a:
a) 0,10
b) 0,13
c) 0,26
d) 0,42
e) 0,52

Resolução: Alternativa B. Em 1,0 litro de solução, temos:


Total de cargas positivas = (0,10 x 1) + (0,16 x 2) mol.L-1
Total de cargas negativas = [(0,16 x (-1)] + [(-2).x] mol.L-1

∑Carga(+) + ∑Carga(-) = 0
(0,10 x 1) + (0,16 x 2) + [0,16 x (-1)] + (-2.x) = 0
0,10 + 0,32 – 0,16 – 2x = 0
0,42 – 0,16 – 2x = 0
0,26 – 2x = 0
x = 0,13 mol.L-1

Questão 482 – (ITA) Um recipiente aberto continha inicialmente 1000 mL de uma solução aquosa contendo 31,80 g
de carbonato de sódio. Parte da água foi evaporada, obtendo-se um volume final de 800 mL da solução. Pode-se
afirmar que, em relação ao carbonato de sódio:
a) A solução inicial era 0,600 mol.L-1
b) A solução final é 0,750 normal
c) A solução final é 0,750 mol.L-1
d) A solução final é 0,480 normal
e) Nenhuma das afirmações acima

Resolução: Alternativa B.

Cálculo da concentração da quantidade de matéria inicial do carbonato de sódio (Na2CO3): [Na2 CO3 ] =
mNa2CO3 31,80
<MM> xV
= 106 x 1,0 = 0,300 mol. L−1
Na2 CO3 solução

mNa2 CO3
Cálculo da concentração normal (normalidade) da solução final: N = eqg =
Na2 CO3 x Vsolução
mNa2CO3 31,80
<MM>Na CO = 106 = 0,750 eqg. L−1
( 2 3) x V
solução
( )x 0,80
x 2

Cálculo da concentração da quantidade de matéria final de carbonato de sódio: [Na2 CO3 ] =


mNa2CO3 31,80
<MM> xV
= 106 x 0,80 = 0,375 mol. L−1
Na2 CO3 solução

239
1600 FÍSICO-QUÍMICA APLICADA EXERCÍCIOS COMENTADOS - IME – ITA – OLIMPÍADA

Questão 483 – (ITA) Juntamos 500 mL de solução 0,40 mol.L-1 de NaOH. A solução obtida adicionamos água até
completar 1,0 litro. Essa solução final terá concentração molar:
a) 0,20 mol.L-1
b) 0,50 mol.L-1
c) 0,40 mol.L-1
d) 1,00 mol.L-1
e) 0,30 mol.L-1

Resolução: Alternativa A. Dados da solução no estado 1: {NaOH]1 = 0,40 mol.L-1 e V1 = 500 mL (0,50 L)

Dados da solução no estado 2: [NaOH]2 (mol.L-1) e V2 = 1000 mL (1,00 L)


Cálculo da concentração da quantidade de matéria solução no estado 2: [NaOH]1 x V1 = [NaOH]2 x V2
0,40 x 0,50 = [NaOH]2 x 1,0
[NaOH]2 = 0,20 mol.L-1

Questão 484 – (ITA) Num recipiente, mantido a 25°C, misturam-se 50 mL de uma solução 5,0 milimol.L-1 de HCl, 50
mL de água destilada e 50 mL de uma solução 5,0 milimol.L-1 de NaOH. A concentração de íons H+, em mol.L-1, na
solução resultante é:
a) 1,3 × 10-11
b) 1,0 × 10-7
c) 0,8 × 10-3
d) 1,0 × 10-3
e) 3,3 × 10-3

Resolução: Alternativa B.

Equação química: HCl(aq) + NaOH(aq) → NaCl(aq) + 2 H2O(l)


1 mol de HCl ----------------- 1 mol de NaOH
nHCl ---------------------------- nNaOH
nHCl = nNaOH

Cálculo do número de mol do HCl, a partir da quantidade de matéria: nHCl = [HCl] x Vsolução
nHCl = (5,0 x 10-3) x 50 x 10-3 = 2,50 x 10-4 mol

Cálculo do número de mol do NaOH (nNaOH), a partir da quantidade de matéria: nNaOH = [NaOH] x Vsolução
nHCl = 5,0 x 10-3 x 50 x 10-3 = 2,50 x 10-4 mol

Conclusão: nHCl = nNaOH

Como o número de mol tanto do ácido quanto da base são iguais, então a solução resultante será neutra, ou seja,
[H+] = [OH-] = 10-7 mol.L-1, ou seja, pH = pOH = 7.

Questão 485 – (ITA) 11,9 gramas de KBr foram dissolvidas em água suficiente para formar 0,500 litros de solução. A
concentração molar desta solução é:
a) 5,00 x 10-2 mol.L-1
b) 2,00 x 10-2 mol.L-1
c) 1,00 x 10-1 mol.L-1
d) 2,00 x 10-1 mol.L-1
e) 5,00 x 10-1 mol.L-1

Resolução: Alternativa D.
n
Cálculo da concentração da quantidade de matéria do sal inorgânico - brometo de potássio: [KBr] = V KBr =
solução
mKBr 11,9
<MM>KBr 119 0,100 mol
Vsolução
= 0,500
= 0,500 = 0,200 L

240
1600 FÍSICO-QUÍMICA APLICADA EXERCÍCIOS COMENTADOS - IME – ITA – OLIMPÍADA

Questão 486 – (ITA) Qual é a concentração molar de CaCl2 em uma solução obtida por dissolução de 11,1 gramas
de CaCl2 anidro em tanta água quanto necessária para que o volume da solução seja de 500 mL?
a) 0,100 mol.L-1
b) 11,1 mol.L-1
c) 0,400 mol.L-1
d) 0,00200 mol.L-1
e) 0,200 mol.L-1

Resolução: Alternativa E.
mCaCl
2 11,1
nCaCl2 <MM>CaCl
Cálculo da quantidade de matéria do soluto iônico: [CaCl2 ] = V = 2
Vsolução
= 111
0,500
= 0,200 mol. L−1
solução

Questão 487 – (OLIMPÍADA BRASILEIRA DE QUÍMICA) Um estudante, a pedido de seu professor, precisa preparar
400 mL de uma solução de amônia 5 mol.L-1. Ao olhar frasco lacrado de amônia que utilizará para produzir sua
solução, o estudante observou as seguintes informações:
✓ Concentração (m/m): 29,0%
✓ Densidade: 0,9 g.cm-3
✓ Massa molar: 17,02 g.mol-1
A partir das informações, o volume de solução concentrada medida pelo estudante para preparar a solução solicitada
pelo professor foi de:
a) 86,00 mL
b) 94,15 mL
c) 112,03 mL
d) 130,46 mL
e) 145,31 mL
Resolução: Alternativa D. Cálculo da concentração comum (C): C = 10 x d x (%) = 10 x 0,90 x 29 = 261 g.L-1
C 262
Cálculo da concentração da quantidade de matéria a patir da concentração comum: Minicial = <MM> = 17,02 =
15,33 mol. L−1

Cálculo do volume inicial: [M]inicial x Vinicial = [M]final x Vfinal

15,35 x Vinicial = 400 x 5,0


Vinicial = 130,27 mL

Questão 488 – (ITA) O volume de SO2 gasoso, medido nas CNTP, necessário para transformar completamente 250
cm³ de solução aquosa 0,100 mol.L-1 de NaOH em solução de Na2SO3, é:
a) 0,14 L
b) 0,28 L
c) 0,56 L
d) 1,12 L
e) 2,24 L

Resolução: Alternativa B. Cálculo do volume de dióxido de enxofre nas CNTP (0°C e 1 atm), a partir da seguinte
equação química: SO2(g) + 2 NaOH(aq) → Na2SO3(aq) + H2O(l)

1 mol de SO2 ---------- 2 mol de NaOH


nSO2 ---------------------- nNaOH
nNaOH = 2 x nSO2

p x VSO2
[NaOH] x Vsolução = 2 x
RxT

241
1600 FÍSICO-QUÍMICA APLICADA EXERCÍCIOS COMENTADOS - IME – ITA – OLIMPÍADA

1 x VSO2
0,100 x 0,250 = 2 x
0,08206 x 273

VSO2 = 0,28 L

As questões 489, 490 e 491 referem-se ao seguinte problema prático:

(ITA) Precisamos preparar 500 mL de uma solução 0,30 mol.L-1 em Fe2(SO4)3. O sal disponível é o Fe2(SO4)3.9H2O.
Esta solução é preparada colocando a quantidade correta do sal sólido num balão volumétrico de 500 mL e
acrescentando água, aos poucos, até que todo sal esteja dissolvido. Após isso, continua-se a colocar água até atingir
a marca existente no balão.

Questão 489 - A quantidade, em mol, de Fe2(SO4)3.9H2O utilizado é:


a) 0,10
b) 0,15
c) 0,30
d) 0,60
e) 0,90

Questão 490 - A massa em gramas, do Fe2(SO4)3.9H2O utilizado é:


a) 60
b) 63
c) 84
d) 120
e) 169

Questão 491 - A concentração, em mol.L-1, de íons sulfato em solução será:


a) 0,10
b) 0,15
c) 0,30
d) 0,60
e) 0,90

Resolução das três questões (489; 490 e 491):

Questão 489 – (ITA) Alternativa B.

Cálculo do número de mol do sal, a partir da concentração da quantidade de matéria: nsal = [Sal] x Vsolução
nsal = 0,30 x 0,50 = 0,15 mol

Questão 490 – (ITA) Alternativa C.

Cálculo da massa do sal, a partir do número de mol: msal = nsal x <MM>sal


msal = 0,15 x (112 + 96 + 192 + 162)
msal = 0,15 x 562 = 84,30 g

Questão 491 – (ITA) Alternativa E.

Equação química: Al2(SO4)3 → 2 Al+3(aq) + 3 SO4-2(aq)


1 mol de Al2(SO4)3 ---------- 3 mol de SO4-2
0,30 mol.L-1 ------------------ [SO4-2]
[SO4-2] = 0,90 mol.L-1

242
1600 FÍSICO-QUÍMICA APLICADA EXERCÍCIOS COMENTADOS - IME – ITA – OLIMPÍADA

Questão 492 – (ITA) Dispomos no laboratório de álcool a 95% em massa (densidade de 0,809 g.mL-1), e queremos
preparar 250 mL de álcool a 30% em massa (densidade de 0,957 g.mL-1). Para isso, qual deve ser o volume que
devemos tomar do álcool disponível:
a) 75,6 mL
b) 93,5 mL
c) 101 mL
d) 117 mL
e) Nenhuma das respostas anteriores

Resolução: Alternativa B.

Cálculo da concentração comum no estado inicial (C1): C1 = 10 x d x (%) = 10 x 0,809 x 95 = 768,55 g,mL-1

Cálculo da concentração comum no estado inicial (C2): C2 = 10 x d x (%) = 10 x 0,957 x 30 = 287,1 g.mL-1

Cálculo do volume no estado inicial (V1): C1 x V1 = C2 x C2


768,55 x V1 = 287,1 x 250
V1 = 93,39 mL

Questão 493 – (ITA) O volume, em litros, de NH3 gasoso medido nas condições normais de temperatura e pressão
necessário para transformar completamente, em solução de (NH4)2SO4, 250 cm3 de uma solução aquosa 0,100 mol.L-
1 de H SO é:
2 4
a) 0,56
b) 1,12
c) 2,24
d) 3,36
e) 4,48

Resolução: Alternativa B.

Cálculo do volume de amônia gasosa, nas CNTP (0°C e 1 atm), a partir da seguinte equação química: 2 NH3(aq) +
H2SO4(aq) → (NH4)2SO4(aq):
2 mol de NH3 ---------- 1 mol de H2SO4
nNH3 ---------------------- nH2SO4
nNH3 = 2 x nH2 SO4

p x VNH3
= 2 x [H2 SO4 ] x Vsolução
RxT
1 x VNH3
= 2 x 0,100 x 0,250
0,08206 x 273

VNH3 = 1,12 L

Questão 494 – (OLIMPÍADA DE QUÍMICA DO RIO DE JANEIRO) O ácido nítrico concentrado é uma solução aquosa
de concentração 65,0 (m/m). Se a densidade da solução é de 1,40 g.cm-3, a concentração desta solução, em mol.L-1,
é:
a) 8,75
b) 10,3
c) 14,4
d) 22,2
e) 24,2

Resolução: Alternativa C.

243
1600 FÍSICO-QUÍMICA APLICADA EXERCÍCIOS COMENTADOS - IME – ITA – OLIMPÍADA

Dados do problema: 65,0 gramas de HNO3 para cada 100 gramas de solução e 1,40 gramas de solução para cada 1
cm³ de solução

Cálculo do volume da solução:


1,40 gramas de solução ---------- 1 cm³ de solução
100 gramas de solução ----------- Vsolução
Vsolução = 71,43 cm3 (71,43 x 10-3 dm³)
mHNO3 65,0
Cálculo do número de mol (n) de ácido nítrico: nHNO3 = <MM> = 63,0
= 1,03 mol
HNO3

nHNO3 1,03
Cálculo da concentração da quantidade de matéria: [HNO3 ] = V = 71,43 x 10−3 = 14,42 mol. L−1
solução

Questão 495 – (ITA) Em um copo de 500 mL, são misturados 100 mL de ácido clorídrico 1,00 mol.L-1 em 100 mL de
hidróxido de sódio 0,50 mol.L-1. A solução resultante no copo é:
a) 1x10-7 mol.L-1 em OH-
b) 1x10-7 mol.L-1 em H+
c) 0,05 mol.L-1 em H+
d) 0,25 mol.L-1 em H+
e) 0,50 mol.L-1 em H+

Resolução: Alternativa D.
Equação química: HCl + NaOH → NaCl + H2O

Cálculo do número de mol de ácido clorídrico, a partir da quantidade de matéria: nHCl = [HCl] x Vsolução = 1,00 x 0,10 =
0,10 mol

Cálculo do número de mol de hidróxido de sódio, a partir da quantidade de matéria: nNaOH = [NaOH] x Vsolução =
0,50 x 0,10 = 0,05 mol

Pelo cálculo do número de mol de cada reagente, observa-se que a quantidade de ácido é maior que a da base. Logo,
o ácido é o reagente em excesso.

Cálculo do número de mol de excesso de HCl: nHCl = 0,10 mol − 0,05 mol = 0,05 mol
nHCl 0,05
Cálculo da concentração da quantidade de matéria do ácido (HCl): [HCl] = V = 0,20 = 0,25 mol. L−1
solução

Questão 496 – (ITA) Qual o valor da massa de sulfato de ferro (III) anidro que deve ser colocada em um balão
volumétrico de 500 mL de capacidade para obter uma solução aquosa 20 milimol.L-1 em íons férricos após completar
o volume do balão com água destilada?
a) 1,50 g
b) 2,0 g
c) 3,0 g
d) 4,0 g
e) 8,0 g

Resolução: Alternativa B.

Cálculo da concentração da quantidade de matéria de sulfato férrico: Fe2(SO4)2(s) → 2 Fe+3(aq) + 3 (SO4-2)(aq)

Fe2(SO4)2 ------------------------ 2 mol de Fe+3


1 mol de Fe2(SO4)2 ------------- 2 Fe+3(aq)
MFe2(SO4)2 ------------------------- 20.10-3 mol.L-1
MFe2(SO4)2 = 10-2 mol.L-1

244
1600 FÍSICO-QUÍMICA APLICADA EXERCÍCIOS COMENTADOS - IME – ITA – OLIMPÍADA

Cálculo da massa de sulfato de ferro III, a partir da concentração da quantidade de matéria: mFe2(SO4)3 = [Fe2(SO4)3] x
<MM>Fe2(SO4)3 x Vsolução
mFe2(SO4)3 = 10-2 x 400 x 0,500 = 2,0 gramas

Questão 497 – (ITA) Num equipamento adequado para permitir adição de solução, assim como coleta e medida de
volume de gases, fez-se seguinte experiência: após colocar neste equipamento 100 cm3 de uma solução aquosa
contendo 1,06 g de carbonato de sódio por litro de solução, adiciona-se um excesso de solução de ácido clorídrico.
Admitindo que, nesta experiência, todo o gás que pudesse ser produzido pela reação entre as duas soluções foi de
fato coletado, qual o volume medido, em cm3, sabendo-se que a experiência foi realizada na temperatura de 27°C e
pressão de 750 mmHg?
a) 0,10 x 22,4
b) 1,10 x 24,9
c) 100 x 0,0827
d) 0,20 x 62,3
e) 0,40 x 62,3

Resolução: Alternativa E.

Cálculo da massa de carbonato de cálcio utilizada: Na2CO3 + 2 HCl → 2 NaCl + H2O + CO2

1 litro de solução ------------------ 1,06 g


0,100 litros de solução ----------- mNa2CO3
mNa2CO3 = 0,106 g

Cálculo do volume de CO2, a partir da equação dos gases ideais: Na2CO3 + 2 HCl → 2 NaCl + H2O + CO2

1 mol de Na2CO3 ---------- 1 mol de CO2


nNa2CO3 ---------------------- nCO2
nNa2CO3 = nCO2

750
(760) x VCO2 0,106
=
0,08206 x 300 106

VCO2 = 0,025 L (25 cm3 )

Questão 498 – (ITA) O volume, em litros, de uma solução 1,0 x 10-4 mol.L-1 de cloreto de cálcio que contém o mesmo
número de Avogadro de cátions é:
a) 5,0 x 10³
b) 1,0 x 104
c) 2,0 x 104
d) 6,0 x 1019
e) 6,0 x 1023

Resolução: Alternativa D.
H2 O
Equação química: CaCl2(s) → Ca+2(aq) + 2 Cl-(aq)

Cálculo do volume:
1,0 litro de solução ----------- 1,0 x 10-4 mol x (6,0 x 1023 cátions/mol)
Vsolução -------------------------- 6,0 x 1023 cátions
Vsolução = 6,0 x 1019 L

245
1600 FÍSICO-QUÍMICA APLICADA EXERCÍCIOS COMENTADOS - IME – ITA – OLIMPÍADA

Questão 499 – (ITA) Adicionou-se um excesso de solução de sulfato de sódio a 500 mL de uma solução de nitrato
de chumbo, tendo se tornado um precipitado de sulfato de chumbo de massa igual a 3,0 g. A concentração molar
inicial do sal de chumbo na solução era:
a) 2,0 x 10-3 mol.L-1
b) 1,0 x 10-2 mol.L-1
c) 2,0 x 10-2 mol.L-1
d) 5,0 x 10-2 mol.L-1
e) 2,0 x 10-1 mol.L-1

Resolução: Alternativa C.

Fórmula molecular do sulfato de sódio: Na2SO4 (Soluto 1)

Fórmula molecular do nitrato de chumbo: Pb(NO3)2 (Soluto 2)

Equação química: Na2SO4 + Pb(NO3)2 → 2 NaNO3 + PbSO4

Cálculo da concentração da quantidade de matéria inicial (M1) de nitrato de chumbo:


1 mol de Pb(NO3)2 ---------- 1 mol de PbSO4
n1 ------------------------------ n2
n1 = n2
msoluto2 3,0
[soluto1 ] x Vsolução = → [soluto1 ] x 0,50 = → [soluto1 ] = 0,0198 mol. L−1
<MM>soluto2 303

Questão 500 – (ITA) 0,200 mol de hidróxido de cálcio sólido são adicionados a 2,00 litros de uma solução 0,400 mol.L-
1 de ácido clorídrico. Depois de a reação ter sido completada, a concentração de cátions hidrogênio, será:

a) 0,050 mol.L-1
b) 0,100 mol.L-1
c) 0,200 mol.L-1
d) 0,300 mol.L-1
e) 0,400 mol.L-1

Resolução: Alternativa D. Equação química: 2 HCl(aq) + Ca(OH)2(s) → CaCl2(aq) + 2 H2O(l)

Determinação do reagente limitante e do reagente em excesso, calculando o número de mol de cada reagente, a
partir da concentração da quantidade de matéria:
Para o ácido clorídrico: nHCl = [HCl] x Vsolução = 0,400 x 2 = (0,800 mol / 2) = 0,400 mol

Para o hidróxido de sódio: nNaOH = [NaOH] x Vsolução = 0,200 x 2 = 0,400 mol

Através da reação química, para cada dois mol de ácido terá 0,400 mol de HCl, logo o ácido é o reagente em excesso
e a base é o reagente limitante.
Cálculo do número de mol do reagente em excesso: nHCl = 0,800 mol – 0,200 mol = 0,600 mol
nHCl 0,600
Cálculo da concentração da quantidade de matéria de ácido em excesso: [HCl]excesso = V = 2,0
=
solução
mol
0,300 L

Equação química de ionização do ácido clorídrico: HCl(aq) → H+(aq) + Cl-(aq)


1 mol de HCl(aq) ---------- 1 mol de H+(aq)
0,300 mol.L-1 ------------ [H+]
[H+] = 0,300 mol.L-1

Questão 501 – (ITA) Mistura-se 20 mL de uma solução 0,50 mol x L-1 de NaOH com 40 mL de uma solução 0,30 mol
x L-1 de NaOH. Admitindo-se a aditividade de volume, a concentração da solução final, em mol/L, será:

246
1600 FÍSICO-QUÍMICA APLICADA EXERCÍCIOS COMENTADOS - IME – ITA – OLIMPÍADA

a)
(0,50 + 0,30) / (20 + 40)
b)
(20 x 0,50 + 40 x 0,30) / (20 + 40)
c)
(40 x 0,50 + 20 x 0,30) / (20 + 40)
d)
(20 + 0,50) / (40 x 0,30)
e)
(11,50 + 0,30) / (40 + 20)

Resolução: Alternativa B. Mistura de mesmo soluto: NaOH(1) + NaOH(2) → NaOH(T)


V1 x M1 + V2 x M2 = VT x MT

Mistura de mesmo soluto: (20 mL x 0,50) + (40 mL x 0,30) = (20 mL + 40 mL) x [NaOH]

[NaOH] = {(20 mL x 0,50) + (40 mL x 0,30)} / (20 mL + 40 mL)

Resolvendo a equação matemática acima, temos: [NaOH] = {(20 mL x 0,50) + (40 mL x 0,30)} / (20 mL + 40 mL)

[NaOH] = {10 + 12} / 60 = 22/60 = 0,367 mol.L-1


Questão 502 – (ITA) Fazendo-se borbulhar gás cloro através de 1,0 litro de uma solução de hidróxido de sódio,
verificou-se ao final do experimento que todo hidróxido de sódio foi consumido, e que na solução resultante foram
formados 2,5 mol de cloreto de sódio. Considerando que o volume da solução não foi alterado durante todo o
processo, e que na temperatura em questão tenha ocorrido apenas a reação correspondente a seguinte equação
química, não balanceada: OH-(aq) + Cl2(g) → Cl-(aq) + ClO3-(aq) + H2O(l). Qual deve ser a concentração inicial do hidróxido
de sódio?
a) 6,0 mol.L-1
b) 5,0 mol.L-1
c) 3,0 mol.L-1
d) 2,5 mol.L-1
e) 2,0 mol.L-1

Resposta: Alternativa C.

Realizando o balanceamento pelo método da oxirredução, a equação química fica da seguinte forma: 6 OH -(aq) + 3
Cl2(g) → 5 Cl-(aq) + ClO3-(aq) + 3 H2O(l)

Cálculo do número de mol de hidroxila: 6 OH-(aq) + 3 Cl2(g) → 5 Cl-(aq) + ClO3-(aq) + 3 H2O(l)


6 mol de OH-(aq) ---------- 5 mol de Cl-(aq)
nOH-------------------------- nCl-
5 x nOH- = 6 x nCl

5 x nOH− = 6 x 2,50
5 x nOH− = 15,0
nOH− = 3,0 mol

nOH− 3,0 mol


Para o volume de um litro, a concentração da quantidade de matéria será: [OH − ] = = = 3,0
Vsolução 1,0 L

Questão 503 – (OLIMPÍADA DE QUÍMICA DO RIO DE JANEIRO / ITA) 2,70 gramas de alumínio são dissolvidos em
500 mL de uma solução aquosa 1,00 mol.L-1 em ácido clorídrico. Todo hidrogênio produzido é recolhido. Após a
secagem, o volume de hidrogênio a pressão de 1 atmosferas e 25°C, é:
a) 1,20 litro
b) 1,60 litro
c) 2,40 litros
d) 3,60 litros
e) 12 litros

247
1600 FÍSICO-QUÍMICA APLICADA EXERCÍCIOS COMENTADOS - IME – ITA – OLIMPÍADA

Resolução: Alternativa D.

Determinação do reagente limitante e do reagente em excesso, a partir do cálculo do número de mol: Al(s) + 3 HCl(aq)
→ AlCl3(aq) + 3/2 H2(g)
m 0,27
Analisando o alumínio metálico: nAl = <MM>
Al
= 27
= 0,10 mol (Reagente limitante)
Al

Analisando o ácido clorídrico, a partir da concentração de quantidade de matéria: nHCl = [HCl] x Vsolução = 1,0 x 0,500 =
0,500 mol
mHCl 0,500
Como a reação química apresenta 3 mol de ácido clorídrico, temos: nHCl = = = 0,167 mol
<MM>HCl 3
Cálculo do número de mol de gás hidrogênio produzido:

1 mol de Al(s) ---------- 3/2 mol de H2(g)


nAl ----------------------- nH2
nH2 = (3/2) x nAl
p x VH2 3
Considerando que o gás hidrogênio apresenta comportamento ideal, temos: RxT
= 2 x nAl

1 x VH2 3 2,70
= x
0,08206 x ( 25 + 273) 2 27

3
VH2 = 0,08206 x 298 x x 0,10 = 3,67 L
2

Questão 504 - (OLIMPÍADA DE QUÍMICA DO RIO DE JANEIRO) 20 mL de uma solução de 0,100 mol.L-1 de nitrato
de bário foram misturados com 30 mL de uma solução 0,400 mol.L-1 de nitrato de amônio. A concentração de nitrato
na solução resultante é:
a) 0,100 mol.L-1
b) 0,250 mol.L-1
c) 0,280 mol.L-1
d) 0,320 mol.L-1
e) 0,400 mol.L-1

Resolução: Alternativa D.

Analisando a primeira solução de nitrato de bário: Ba(NO3)2(aq) → Ba+2(aq) + 2 NO3-(aq). Cálculo da concentração da
quantidade de matéria de nitrato, [NO3-]1:
1,0 mol de Ba(NO3)2(aq) ------------ 2 mol de [NO3-(aq)]1
0,100 mol.L-1 ----------------------- [NO3-(aq)]1
[NO3-(aq)]1 = 0,200 mol.L-1

Analisando a segunda solução de nitrato de amônio: NH4NO3(aq) → NH4+(aq) + NO3-(aq). Cálculo da concentração da
quantidade de matéria de nitrato, [NO3-]2:
1,0 mol de NH4NO3(aq) ------------- 1 mol de [NO3-]2
0,400 mol.L-1 ----------------------- [NO3-]1
[NO3-]1 = 0,400 mol.L-1
Cálculo do volume total (VT): VT = V1 + V2 = 20 mL + 30 mL = 50 mL

Cálculo da concentração da quantidade de matéria da solução final de nitrato, [NO 3-(aq)]T: V1.[NO3-]1 + V2[NO3-]1 =
VT[NO3-]T
20 mL x 0,200 mol.L-1 + 30 mL x 0,400 mol.L-1 = (20 + 30) mL x [NO3-]T
[NO3-]T = 0,320 mol.L-1

248
1600 FÍSICO-QUÍMICA APLICADA EXERCÍCIOS COMENTADOS - IME – ITA – OLIMPÍADA

Questão 505 - (OLIMPÍADA BRASILEIRA DE QUÍMICA) Quando se mistura 200 mL de uma solução a 5,85% (m/v)
de cloreto de sódio com 200 mL de uma solução de cloreto de cálcio que contém 22,20 gramas do soluto e adiciona-
se 200 ml de água, obtém-se uma nova solução cuja concentração de íons cloreto é de:
a) 0,10 mol.L-1
b) 0,20 mol.L-1
c) 1,0 mol.L-1
d) 2,0 mol.L-1
e) 3,0 mol.L-1

Resolução: Alternativa C.

Analisando a primeira solução de cloreto de sódio: NaCl(aq) → Na+ (aq) + Cl-(aq). Cálculo da massa de NaCl:

5,85 g de NaCl ---------- 100 mL de solução


mNaCl ---------------------- 200 mL de solução
mNaCl = 11.70 g
mNaCl 11,70
nNaCl <MM>NaCl
Cálculo da concentração da quantidade de matéria de cloreto de sódio: [NaCl] = = = 58,5
=
Vsolução Vsolução 0,200
−1
1 mol. L

Cálculo da concentração da quantidade de matéria de cloreto do cloreto de sódio: NaCl(aq) → Na+(aq) + Cl-(aq)

1 mol de NaCl(aq) ---------- 1 mol de [Cl-]1


1 mol.L-1 -------------------- [Cl-]1
[Cl-]1 = 1 mol.L-1

Analisando a segunda solução de cloreto de cálcio: CaCl2(aq) → Ca+2 (aq) + 2 Cl-(aq)


mCaCl
2
nCaCl2 <MM>CaCl
Cálculo da concentração da quantidade de matéria de cloreto de cálcio: [𝐶𝑎𝐶𝑙2 ] = V = 2
Vsolução
=
solução
22,20
111
0,200
= 1 mol. L−1

Cálculo da concentração da quantidade de matéria de cloreto do cloreto de cálcio: CaCl2(aq) → Ca+2(aq) + 2 Cl-(aq)

1 mol de CaCl2 (aq) ---------- 2 mol de [Cl-]2


1 mol.L-1 --------------------- [Cl-]2
[Cl-]2 = 2 mol.L-1

Cálculo do volume total: VT = V1 + V2 +VH2O = 200 mL + 200 mL + 200 mL = 600 mL

Cálculo da concentração da quantidade de matéria do cloreto na mistura, [Cl-]T: V1 x [Cl-]1 + V2 x [Cl-]1 = VT x [Cl-]T
200 mL x 1,0 mol.L-1 + 200 mL x 2,0 mol.L-1 = 600 mL x [Cl-]T
[Cl-]T = 1,0 mol.L-1

Questão 506 - (OLIMPÍADA DE BRASILEIRA DE QUÍMICA) O volume de ácido nítrico 0,1 mol.L-1 necessário para
neutralizar uma mistura 0,40 g de hidróxido de sódio e 1,71 g de hidróxido de bário é:
a) 20 mL
b) 30 mL
c) 50 mL
d) 200 mL
e) 300 mL

Resolução: Alternativa E.

249
1600 FÍSICO-QUÍMICA APLICADA EXERCÍCIOS COMENTADOS - IME – ITA – OLIMPÍADA

Analisando a primeira reação de neutralização (ácido nítrico + hidróxido de sódio →): HNO3(aq) + NaOH(aq) →
NaNO3(aq) + H2O(l)

Cálculo do volume de ácido nítrico:

1,0 mol de HNO3 ---------- 1 mol de NaOH


nHNO3 ------------------------- nNaOH
nHNO3 = nNaOH
mNaOH
[HNO3 ] x Vsolução =
< MM >NaOH

0,40
1,0 x Vsolução =
40
Vsolução = 0,10 L (100 mL)

Analisando a segunda reação de neutralização (ácido nítrico + hidróxido de Bário →): 2 HNO3 + Ba(OH)2 → Ba(NO3)2
+ 2 H2O

Cálculo do volume de ácido nítrico:

2,0 mol de HNO3 ---------- 1 mol de Ba(OH)2


nHNO3 ------------------------- nNaOH
nHNO3 = 2 x nBa(OH)2
mBa(OH)2
[HNO3 ] x Vsolução = 2 x
< MM >Ba(OH)2

1,71
0,10 x Vsolução = 2 x
171

Vsolução = 0,20 L (200 mL)

Cálculo do volume total (VTOTAL): VTOTAL = V1 + V2 = 100 mL + 200 mL = 300 mL

Questão 507 - (OLIMPÍADA MARANHENSE DE QUÍMICA) Uma das formas de monitoramento da água de caldeiras
de indústrias, responsável pelo fornecimento de energia, é a medição do teor de íons cloreto. Um método de análise
de cloreto é a adição a água de caldeira de uma solução de nitrato de prata (AgNO3), segundo a equação: Cl-(aq) +
AgNO3(aq) → AgCl(s) + NO3-(aq). A análise de 10,0 mL de água de caldeira, contendo Cl-, com solução aquosa de nitrato
de prata, formou um precipitado de 0,287 g de cloreto de prata. A concentração molar de cloreto presente na amostra
de água de caldeira é igual a:
a) 2
b) 0,2
c) 0,02
d) 0,002
e) 0,0002

Resolução: Alternativa B.

Cálculo do volume de ácido nítrico: Cl-(aq) + AgNO3(aq) → AgCl(s) + NO3-(aq)


1,0 mol de Cl-(aq) ----------- 1 mol de AgCl(aq)
nCl- --------------------------- nAgCl
nCl- = nAgNO3

250
1600 FÍSICO-QUÍMICA APLICADA EXERCÍCIOS COMENTADOS - IME – ITA – OLIMPÍADA

[Cl-] x Vsolução = (mAgCl / <MM>AgCl)


[Cl-] x 10 x 10-3 = (0,287 / 143,5)
0,287
[Cl-] = 143,5 𝑥 0,01 = 0,20 mol.L-1

Questão 508 - (OLIMPÍADA DE BRASILEIRA DE QUÍMICA) Uma amostra de um ácido diprótico pesando 12,25
gramas foi dissolvida em água e o volume da solução completado para 500 mL. Se 25,0 mL desta solução são
neutralizados com 12,5 mL de uma solução KOH 1,00 mol.L-1, a massa molar desse ácido, considerando que os dois
prótons foram neutralizados, é igual a:
a) 2,25
b) 24,5
c) 49,0
d) 98,0
e) 122,5

Resolução: Alternativa D.
12,25
Expressão da concentração da quantidade de matéria inicial do ácido diprótico (H2X): [H2 X] = 0,50 x <MM>
H2 X
Cálculo da massa molar do ácido diprótico (H2X): H2X + 2 KOH → K2X + 2 H2O

1,0 mol de H2X ---------- 2 mol de KOH


na --------------------------- nKOH
nKOH = 2 x nH2X

[KOH] x Vsolução = 2 x [H2X] x Vsolução


12,25
1,0 x 12,5 = 2 x x 25,0
0,50 x <MM>H2X

<MM>H2X = 98 g.mol-1

Questão 509 – (OLIMPÍADA BRASILEIRA DE QUÍMICA) A soma das concentrações dos íons presentes em uma
solução preparada a partir da dissolução completa de 1,25 gramas de sulfato cúprico penta-hidratado em volume de
água suficiente para completar 500 mL de solução, expressa em mmol.L-1, é de aproximadamente:
a) 10
b) 20
c) 30
d) 40
e) 50
Resolução: Alternativa B.

Cálculo da concentração da quantidade de matéria do sulfato cúprico penta-hidratado: [CuSO4 . 5 H2 O] =


1,25 mol
249,50 x 0,50
= 10−2 L

Cálculo da concentração da quantidade de matéria dos íons: CuSO4(aq) → Cu+2(aq) + SO4-2(aq)

1 mol de CuSO4(aq) ---------- 1 mol de Cu+2(aq) --------- 1 mol de SO4-2(aq)


10-2 mol.L-1 ------------------ [Cu+2] ---------------------- [SO4-2]

[Cu+2] = 10-2 mol.L-1


[SO4-2] = 10-2 mol.L-1

Soma das concentrações da quantidade de matéria dos íons ([Cu+2] + [SO4-2]): [Cu+2] + [SO4-2] = 10-2 mol.L-1 + 10-2
mol.L-1 = 2 x 10-2 mol.L-1 (20 mmol.L-1)

251
1600 FÍSICO-QUÍMICA APLICADA EXERCÍCIOS COMENTADOS - IME – ITA – OLIMPÍADA

Questão 510 – (OLIMPÍADA BRASILEIRA DE QUÍMICA) Hipoclorito de sódio pode ser obtido através da seguinte
reação: Cl2(g) + 2 NaOH(aq) → NaCl(aq) + NaOCl(aq) + H2O(l). Considerando a existência de cloro gasoso em excesso,
qual o volume de uma solução de NaOH de concentração 2 mol.L-1 necessário para produzir hipoclorito em quantidade
suficiente para preparar 2,0 L de uma solução 0,50 mol.L-1 de NaOCl?
a) 1,0 L
b) 2,0 L
c) 3,0 L
d) 4,0 L
e) 5,0 L

Resolução: Alternativa A. Cálculo do volume da solução de hidróxido de sódio (NaOH): Cl2(g) + 2 NaOH(aq) → NaCl(aq)
+ NaOCl(aq) + H2O(l)

2 mol de NaOH(aq) ----------- 1 mol de NaOCl(aq)


nNaOH -------------------------- nNaOCl
nNaOH = 2.nNaOCl
[NaOH] x Vsolução = 2 x [NaOCl] x Vsolução
2 x Vsolução = 2 x (0,50) x 2,0
Vsolução = 1,0 L

Questão 511 – (ENADE) Segundo um estudo norte-americano publicado na revista Proceedings of the National
Academy of Sciences, as temperaturas na superfície da Terra não subiram tanto entre 1998 e 2009, graças ao efeito
resfriador dos gases contendo enxofre, emitidos pelas termelétricas a carvão (as partículas de enxofre refletem a luz
e o calor do Sol). O enxofre é um dos componentes do ácido sulfúrico (H2SO4), cujo uso é comum em indústrias na
fabricação de fertilizantes, tintas e detergentes. Sabendo-se que o ácido sulfúrico concentrado é 98,0% em massa de
H2SO4 e densidade 1,84 g.mL-1, conclui-se que a sua concentração, em mol.L-1, é igual a:
a) 18,0
b) 18,2
c) 18,4
d) 18,6
e) 18,8

Resolução: Alternativa D.

Dados do problema:
98 g de H2SO4 -------------- 100 g de solução
1,84 g de solução ---------- 1 mL de solução

Cálculo do volume da solução:


1,84 g de solução -------------- 1 mL de solução
100 g de solução --------------- Vsolução
Vsolução = 54,35 x 10-3 L
mH SO
2 4
nH2 SO4 <MM>H SO
Cálculo da concentração da quantidade de matéria do ácido sulfúrico: [H2 SO4 ] = V = 2 4
Vsolução
=
solução
1,0
1,0
54,35 x 10−3
= 18,40 mol x L−1

Questão 512 – (OLIMPÍADA PORTUGUESA DE QUÍMICA / GRÉCIA) Quantos mL de solução de NaOH 1,00 mol/L
são necessários para neutralizar 100 mL de solução 0,100 mol/L de H3PO4?
a) 10,0
b) 3,30
c) 30,0
d) 300,0

Resolução: Alternativa C.

252
1600 FÍSICO-QUÍMICA APLICADA EXERCÍCIOS COMENTADOS - IME – ITA – OLIMPÍADA

Equação química: H3PO4 + 3 NaOH → Na3PO4 + 3 H2O


1 mol de H3PO4 ---------- 3 mol de NaOH
nH3PO4 ---------------------- nNaOH
nNaOH = 3 x nH3PO4

[NaOH] x Vsolução = 3 x [H3PO4] x Vsolução


1,0 x Vsolução = 3 x 0,100 x 100
Vsolução = 30 mL

Questão 513 - (OLIMPÍADA PARAENSE DE QUÍMICA) As baterias novas possuem soluções aquosas de ácido
sulfúrico a 38% em massa e densidade de 1,30 g.mL-1. A concentração molar dessa solução será:
a) 0,05 mol.L-1
b) 1,50 mol.L-1
c) 2,50 mol.L-1
d) 5,00 mol.L-1
e) 7,00 mol.L-1

Resolução: Alternativa D.

Cálculo do volume da solução:


1,3 g de solução --------- 100 mL
100 g de solução -------- Vsolução
Vsolução = 76,92 mL (76,92 x 10-3 L)
mH SO
2 4 38
nH2SO4 <MM>H SO
Cálculo da concentração da quantidade de matéria (mol.L-1): [H2 SO4 ] = = 2 4
= 98
=
Vsolução Vsolução 76,92 x 10−3
5,04 mol x L−1

Questão 514 – (OLIMPÍADA CEARENSE DO ENSINO SUPERIOR DE QUÍMICA) Os íons tiossulfatos (S2O3)-2
desproporcionam-se em uma solução ácida produzindo o enxofre sólido (S) e o íon hidrogênio sulfito (HSO3-): 2 S2O3-
(aq) + 2 H3O (aq) → 2 HSO3 (aq) + 2 H2O(l) + 2 S(s). Assumindo que a reação se completa e que uma solução 10,1 mL
2 + -

de íons (HSO3) , 55,0% em massa cuja densidade é 1,45 g.cm-3, é obtida na reação, determine a massa de (S2O3)-2,
-

presente inicialmente na reação.


a) 11,13 g
b) 14,64 g
c) 8,05 g
d) 0,750 g
e) 23,30 g

Resolução: Alternativa A. Equação química: 2 S2O3-2(aq) + 2 H3O+(aq) → 2 HSO3-(aq) + 2 H2O(l) + 2 S(s)


2 mol de S2O3-2(aq) ---------- 2 mol de HSO3-(aq)
nS2O3-2 ------------------------ nHSO3-
𝑛S2 𝑂3−2 = 𝑛HS𝑂3−
nS2 O−2 3
= [HSO− 3 ] x Vsolução
< MM >S2 O−2 3

C 10 x d x (%)
Cálculo da concentração da quantidade de matéria do íon sulfito: [HSO−
3] = = =
<MM>S O−2 <MM>HSO−
2 3 3
10 x 1,45 x 55 797,50
= = 9,84 mol. L−1
81 81
Cálculo da massa de íons tiossulfato: 𝑛S2 𝑂3−2 = 𝑛HS𝑂3−
mS2 O−2
3
= [HSO− 3 ] x Vsolução
< MM >S2 O−23

mS2 O−2
3
= 102 x 9,84 x 10,1 x 10−3 = 10,14 g

253
1600 FÍSICO-QUÍMICA APLICADA EXERCÍCIOS COMENTADOS - IME – ITA – OLIMPÍADA

Questão 515 – (U. S. NATIONAL CHEMISTRY OLYMPIAD) Qual é a concentração de íons Na+ formado por uma
mistura de 20 mL de solução de Na2SO4 (0,10 mol.L-1) com 50 mL de solução 0,30 mol.L-1 de Na3PO4?
a) 0,15 mol.L-1
b) 0,24 mol.L-1
c) 0,48 mol.L-1
d) 0,70 mol.L-1

Resolução: Alternativa D. Analisando a solução de sulfato de sódio: Na2SO4(aq) → 2 Na+(aq) + SO4-2(aq)


1 mol de Na2SO4 ------------------- 2 mol de Na+
0,10 mol.L-1 de Na2SO4 ----------- [Na+]
[Na+] = 0,20 mol.L-1

Analisando a solução de fosfato de sódio: Na3PO4(aq) → 3 Na+(aq) + PO4-3(aq)


1 mol de Na3PO4 ------------------- 3 mol de Na+
0,30 mol.L-1 de Na3PO4 ----------- [Na+]
[Na+] = 0,90 mol.L-1

Analisando a solução de sódio final, para o cálculo da concentração da quantidade de matéria dos íons Na+: [Na+]1.V1
+ [Na+]2.V2 = [Na+]T.VT
[0,20 x 20] + [0,90 x 50] = [Na+]T x 70
[Na+]T = 0,70 mol.L-1

Questão 516 – (OLIMPÍADA DE QUÍMICA DO RIO DE JANEIRO) Um técnico em química misturou acidentalmente
três soluções no laboratório: 200 mL de cloreto de potássio 0,10 mol.L-1, 300 mL de cloreto de magnésio 0,20 mol.L-1
e 200 mL de cloreto de sódio 0,50 mol.L-1. Qual é a concentração de íons cloreto na solução final obtida pelo técnico?
a) 0,012 mol.L-1
b) 0,67 mol.L-1
c) 1,2 mol.L-1
d) 0,26 mol.L-1
e) 0,34 mol.L-1

Resolução: Alternativa E.

Cálculo da concentração da quantidade de matéria de íons cloreto do KCl: KCl → K+ + Cl-


1 mol de KCl ---------- 1 mol de Cl-
0,10 mol.L-1 ----------- [Cl-]
[Cl-]1 = 0,10 mol.L-1

Cálculo da concentração da quantidade de matéria de íons cloreto [Cl-]: MgCl2 → Mg+2 + 2 Cl-
1 mol de MgCl2 ---------- 2 mol de Cl-
0,20 mol.L-1 -------------- [Cl-]
[Cl-]2 = 0,40 mol.L-1

Cálculo da concentração da quantidade de matéria de íons cloreto do NaCl: NaCl(s) → Na+(aq) + Cl-(aq)
1 mol de NaCl ---------- 1 mol de Cl-
0,50 mol.L-1 ------------- [Cl-]
[Cl-]3 = 0,50 mol.L-1

Cálculo da concentração da quantidade de matéria de íons cloreto da solução final: [Cl-]1 x V1 + [Cl-]2 x V2 + [Cl-]3.V3 =
[Cl-]FINAL x (V1 + V2)
0,10 mol.L-1 x 200 + 0,40 mol.L-1 x 300 + 0,50 mol.L-1 x 200 = [Cl-]FINAL x (200 + 300 + 200)
240
[Cl− ]FINAL = = 0,343 mol. L−1
700

254
1600 FÍSICO-QUÍMICA APLICADA EXERCÍCIOS COMENTADOS - IME – ITA – OLIMPÍADA

Questão 517 - (GRILLO) Considere que uma solução seja constituída pelas seguintes substâncias: 52,6% em água,
33,5% em etanol e o restante composto de ácido clorídrico, sendo todos estes valores fornecidos em percentuais
(m/m). A partir da informação apresentada, determine a fração molar para cada constituinte da mistura, a variação da
energia livre molar de Gibbs e a variação de entropia molar da mistura a uma temperatura de 25°C.
Resolução: Base de cálculo: massa total da solução igual a 100 gramas.
Cálculo da massa de cada constituinte:
mágua = 0,526 x 100 g = 52,60 g

málcool = 0,335 x 100 g = 33,50 g


mácido = 0,139 x 100 g = 13,90 g
52,60 33,50
Cálculo do número de mol de cada constituinte: nágua = 18
= 2,92 mol; nálcool = 46
= 0,728 mol e
13,90
nácido = 36,50
= 0,381 mol

Cálculo do número de mol total: ntotal = nágua + nálcool + nácido = 4,03 𝑚𝑜𝑙
nágua 2,92 mol nálcool 0,728 mol
Cálculo da fração molar de cada constituinte: X água = = = 0,724; X álcool = = =
ntotal 4,03 mol ntotal 4,03 mol
nácido 0,381 mol
0,181 e X ácido = ntotal
= 4,03 mol
= 0,094

Cálculo da energia livre de Gibbs molar da solução:

∆G = ntotal x R x T x [X água x ln X água + X álcool x ln X álcool + X ácido x ln X ácido ]

∆G = 4,03 x 8,314 x (25 + 273) x [0,724 x ln(0,724) + 0,181 x ln(0,181) + 0,094 x ln(0,094)]
∆G = 4,03 x 8,314 x 298 x [−0,234 − 0,309 − 0,222]
J
∆G = 4,03 x 8,314 x 298 x (−0,765) = −7638,23 (processo espontâneo)
mol
Cálculo da entropia molar da solução: ∆G = ∆H − T x ∆S. Como o processo ocorre a temperatura constante, ∆H =
0, logo:

ntotal x R x T x [X água x ln X água + X álcool x ln X álcool + X ácido x ln X ácido ] = 0 − T x ∆S

ntotal x R x T x [X água x ln X água + X álcool x ln X álcool + X ácido x ln X ácido ] = 0 − T x ∆S

∆S = − ntotal x R x [X água x ln X água + X álcool x ln X álcool + X ácido x ln X ácido ]

∆S = − 4,03 x 8,314 x [0,724 x ln(0,724) + 0,181 x ln(0,181) + 0,094 x ln(0,094)]


J
∆S = − 4,03 x 8,314 x (−0,765) = +25,63 (processo espontâneo)
K

255
1600 FÍSICO-QUÍMICA APLICADA EXERCÍCIOS COMENTADOS - IME – ITA – OLIMPÍADA

Questão 518 – (ITA) O equivalente – grama do ferro (massa atômica = 55,85) na reação 2 FeCl2 + Cl2 → FeCl3, é:
f) 55,85 / 2
g) 55,85 x 2
h) 55,85
i) 55,85 / 3
j) Nenhuma das respostas anteriores

Resolução: Alternativa C.
O número de oxidação do ferro varia de +2 → +3.
A variação do número de oxidação é ∆ = 3 – 2 = 1
<MM>Fe 55,85
Cálculo do equivalente-grama: Eqg Fe = ∆
= 1,0
= 55,85

Questão 519 – (ITA) Faz-se reagir 5 mL de ácido sulfúrico 0,1 mol x L-1 com NaOH. O ácido estará completamente
neutralizado se forem empregados:
a) 6,0 mL de NaOH 0,05 mol x L-1
b) 5,0 mL de NaOH 0,05 N
c) 10 mL de NaOH 0,1 N
d) 10 mL de NaOH 0,2 N
e) Nenhuma das respostas anteriores

Resolução: Alternativa C.

Equação química da reação de neutralização: H2 SO4(aq) + 2 NaOH(aq) → Na2 SO4(aq) + 2 H2 O(l)

Relação estequiométrica entre o ácido sulfúrico com o hidróxido de sódio:

1 mol de H2 SO4(aq) − − − − 2 mol de NaOH(aq)


n(ácido) − − − − − − − − − n(base)
2 x n(ácido) = n(base)

Cálculo do número de mol da base: n(base) = 2 x 0,10 x 5 x 10−3 = 10−3 mol

Analisando a alternativa C: n(base) = [NaOH] x Vsolução

Para a solução de hidróxido de sódio a concentração da quantidade de matéria desta solução é igual a concentração
normal.
n(base) = 2 x [NaOH] x Vsolução
n(base) = 0,10 x 10 x 10−3 = 10−3 mol

Questão 520 – (ITA) Dada a reação 2 KMnO4 + 5 H2C2O4 + 3 H2SO4 → K2SO4 + 2 MnSO4 + 8 H2O + 10 CO2, quantos
mL de H2C2O4 0,1 N reagem completamente com 50 mL de KMnO4 0,2 M?

Resolução: Cálculo da concentração normal da solução de permanganato de potássio: N = [KMnO4 ]. x, onde x


corresponde ao número de oxidação em módulo do ânion (𝑀𝑛𝑂4− ). Pela estequiometria apresenta pela equação
química, temos: 2 mol de KMnO4 − − − − − − 5 mol de H2 C2 O4

2 x N1 x V1 = 5 x N2 x V2

2 x N1 x V1 2 x 0,2 x 50
V2 = = = 40 mL
5 x N2 5 x 0,1

256
1600 FÍSICO-QUÍMICA APLICADA EXERCÍCIOS COMENTADOS - IME – ITA – OLIMPÍADA

Questão 521 – (ITA) Definir:


a) título;
b) percentagem em massa;
c) normalidade;
d) molaridade;
e) molalidade, de uma solução.

Resposta:
a) Título = é qualquer forma de expressar a concentração de uma solução. Trata-se de um tipo de concentração em
percentagem que pode ser (m/m) e (v/v).
b) percentagem em massa = relaciona a massa do soluto em gramas pela massa da solução. Exemplo: 36% (m/m)
de NaOH. Significa que há 36 gramas de soda caústica por 100 gramas de solução.
c) normalidade: é a razão entre o número de equivalente-grama de um determinado soluto (ácido, base ou sal) pelo
volume da solução em litros.
d) Molaridade = hoje em dia conhecido como concentração da quantidade de matéria. É a razão entre o número de
mol do soluto pelo volume da solução em litros.
e) Molalidade = é a razão entre o número de mol do soluto pela massa do solvente em quilogramas.

Questão 522 – (ITA) Escrever as equações representativas da dissociação iônica do cloreto de sódio, hidróxido de
amônio, ácido sulfúrico e ácido acético em soluções aquosas, e da água.
H2 O
⏞ Na+
Resolução: Analisando a solução de cloreto de sódio: NaCl(aq) → −
(aq) + Cl(aq)

H2 O
⏞ NH4 +
Analisando a solução de hidróxido de amônio: NH4 OH(aq) → −
(aq) + OH(aq)

H2 O
+
Analisando a solução de ácido sulfúrico: H2 SO4 (aq) →
⏞ H(aq) + HSO−
4(aq)
H2 O
HSO− +
⏞ H(aq)
4(aq) → + SO−2
4(aq)
H2 O
⏞ CH3 COO−
Analisando a solução de ácido acético: CH3 COOH(aq) → +
(aq) + H(aq)

H2 O
+ −
Analisando a água: H2 O(l) →
⏞ H(aq) + OH(aq)

257
1600 FÍSICO-QUÍMICA APLICADA EXERCÍCIOS COMENTADOS - IME – ITA – OLIMPÍADA

EXERCÍCIOS DE FIXAÇÃO

Questão 523 – (AUSTRALIAN SCIENCE OLYMPIADS) In an analysis of a solution containing barium ions (Ba2+),
50,0 mL of the solution gave 0,244 g of barium sulfate (BaSO4) (molar mass = 233 g.mol–1) upon addition of sufficient
sulfuric acid to precipitate all the Ba2+ ions present. What is the concentration (in mol.L–1) of the Ba2+ ions in the
solution? Resposta: [Ba+2] = 2,09 x 10-3 mol x L-1

Questão 524 – A solution of KNO3 contains 192,6 g of salt per liter of solution. The density of the solution is 1,1432.
Calcule the concentration in terms of (a) molarity; (b) weight per cent. Resposta: (a) [KNO3] = 1,906 mol x L-1; (b)
16,84% (m/m)
Questão 525 – (AUSTRALIAN SCIENCE OLYMPIADS) What is the concentration of a NaOH solution, 25,0 mL of
which completely reacts with 35,7 mL of a 0,126 mol.L–1 H2SO4 solution?
a) 0,088 mol.L–1
b) 0,126 mol.L–1
c) 0,176 mol.L–1
d) 0,180 mol.L–1
e) 0,360 mol.L–1

Questão 526 – (AUSTRALIAN SCIENCE OLYMPIADS) One way to measure the concentration of a substance is
through molality. A 1 molal solution is defined as 1 mol of substance per 1 kg of the solvent used to make up the
solution. What is the molality of a solution made by dissolving 100 g of bromothymol blue (C27H28Br2O5S) in 1,00 L of
ethanol on a winter’s day at 10ºC? The density of ethanol at this temperature is 0,7979 kg.L-1.
a) 0,100 mol.kg–1
b) 0,128 mol.kg–1
c) 0,160 mol.kg–1
d) 0,201 mol.kg–1
e) 0,252 mol.kg–1

Questão 527 – (OLIMPÍADA DE QUÍMICA DO RIO DE JANEIRO) O vinagre comercial no Brasil apresenta uma
concentração de 3 a 5% de ácido etanoico. Para determinar se certa marca de vinagre está dentro das especificações
brasileiras foi feito o seguinte teste: titulou-se 10 mL do vinagre com uma solução 40 g/L de hidróxido de sódio.
Admitindo a densidade do vinagre comercial como 1,12 g/cm3, o volume, em litros, da solução de hidróxido de sódio
gasto na titulação foi de:
a) 9,3 × 100
b) 9,3 × 10–1
c) 9,3 × 10–2
d) 9,3 × 10–3
e) 9,3 x 10-4

Questão 528 – (OLIMPÍADA DE QUÍMICA DO RIO DE JANEIRO) Uma solução aquosa de ácido clorídrico, HCl, cuja
densidade 1,13 g mL-1 contém 25,7% de HCl em massa. Considerando que um volume de 1,25 mL da solução aquosa
de HCl reagiu com 1,00 g de ferro, formando o cátion ferroso, calcule o rendimento percentual sabendo que 0,0070 g
de gás hidrogênio, H2, foram obtidos. Resposta: Rendimento = 0,7040 (70,40%).

Questão 529 – (OLIMPÍADA DE QUÍMICA DO RIO DE JANEIRO) Para se determinar a concentração de uma
solução de hidróxido de sódio, um estudante extraiu uma alíquota de 50,00 mL e, utilizando um balão volumétrico,
avolumou para 250,0 mL. Desta diluição, ele extraiu 20,00 mL e titulou com uma solução de ácido sulfúrico com a
concentração de 0,2 mol⋅L-1. O volume de ácido gasto foi de 18,50 mL. A concentração da solução inicial de hidróxido
de sódio é de: a) 0,18 mol⋅L-1 (b) 0,37 mol⋅L-1 c) 0,92 mol⋅L-1 d) 1,85 mol⋅L-1 e) 2,15 mol⋅L-1.

Questão 530 – (OLIMPÍADA DE QUÍMICA DO RIO DE JANEIRO) Um laboratório possui 200 gramas de uma solução
10% m/m e deseja aumentar sua concentração para 20% m/m. Que massa de soluto, em gramas, deve ser adicionada
sem alterar a quantidade de solvente? a) 10; b) 20; c) 22,5; d) 25; e) 45

258
1600 FÍSICO-QUÍMICA APLICADA EXERCÍCIOS COMENTADOS - IME – ITA – OLIMPÍADA

Questão 531 – (OLIMPÍADA DE QUÍMICA DO RIO DE JANEIRO) 30 g de uma amostra impura de hidróxido de sódio
foram dissolvidos em água suficiente para a obtenção de 200 mL de solução. Uma alíquota de 25,0 mL dessa solução
foi neutralizada totalmente quando titulada com 46,88 mL de solução de ácido sulfúrico 0,80 mol/L. Determine o teor
de pureza da amostra de hidróxido de sódio, admitindo que as impurezas não reagem com o ácido. a) 40% b) 20% c)
10% d) 90% e) 80%
Questão 532 – (OLIMPÍADA DE QUÍMICA DO RIO DE JANEIRO) Um técnico de laboratório realizou a mistura de
200 mL de solução aquosa 0,3 mol/L de CaCl2 com 300 mL de solução aquosa 0,2 mol/L de NaCl. Em seguida, ele
adicionou 100 mL de água destilada à mistura. Determine a concentração, em mol/L, de íons cloreto na solução
resultante. a) 0,10 b) 0,12 c) 0,30 d) 0,20 e) 0,36
Questão 533 – (OLIMPÍADA DE QUÍMICA DO RIO DE JANEIRO) O ácido acetilsalicílico (C9H8O4) é um fármaco da
família dos salicilatos sendo utilizado como medicamento para combater a dor, a febre e a inflamação devido ao seu
efeito inibidor não seletivo da ciclo-oxigenase. Um teste comum para determinar a quantidade de ácido acetilsalicílico
(C9H8O4) num comprimido analgésico foi feito segundo o procedimento: pesar 2,0 g do comprimido e dissolver numa
mistura de etanol e água. Em uma titulação, essa solução consumiu 40 mL de solução aquosa de Hidróxido de sódio,
de concentração 0,20 mol/L segundo a equação química abaixo: C9H8O4(aq) + NaOH(aq)→ NaC9H7O4(aq) + H2O(ℓ). A
porcentagem em massa de ácido acetilsalicílico no comprimido é de, aproximadamente: a) 2% b) 18% c) 55% d) 60%
e) 72%
Questão 534 – (GRILLO) Considere que uma solução definida como homogênea seja constituída pelas seguintes
substâncias, em massa: 50% em água, 35% em etanol e o restante composto de ácido acético, sendo todos estes
percentuais em massa. A partir da informação apresentada, determine a fração molar para cada constituinte da
mistura. Resposta: Xágua = 0,735; Xálcool = 0,201 e Xácido = 0,0640
Questão 535 – (GRILLO) Uma solução de carbonato de sódio (106 g x mol-1) é obtida por um processo de dissolução
de 22,50 g de carbonato de sódio decahidratado (286 g x mol-1) em água até completar 200 cm³. Sabendo que a
densidade da solução é igual a 1,040 g x cm-³, determine o valor da concentração da quantidade de matéria, a
molalidade e a normalidade. Resposta: concentração da quantidade de matéria = 0,393 mol x L-1; molalidade =
0,394 mol x kg-1 e N = 0,786 eqg x L-1
Questão 536 – (GRILLO) Calcule a energia livre de Gibbs e a entropia da mistura para duas situações: (a) uma
mistura composta por 1,0 mol de nitrogênio gasoso e 1,0 mol de oxigênio gasoso; (b) uma mistura composta por 2
mol de argônio, 1 mol de hélio e 3 mol de gás hidrogênio. A partir das informações apresentadas, levar em
consideração que a pressão (1 bar) e a temperatura esteja constante (25°C). Resposta: (a) ∆Gmistura = -3433,91 J x
mol-1 e ∆Smistura = + 11,52 J x K-1; (b) ∆Gmistura = -15034,96 J x mol-1 e ∆Smistura = + 50,45 J x K-1
Questão 537 – How many of water must be added to 20,0 g of urea to prepare a 5,00% aqueous urea solution by
weight? Resposta: 380 g de água
Questão 538 – (OLIMPÍADA BRASILEIRA DE QUÍMICA 2005) Determinou-se, em uma solução aquosa, a presença
dos seguintes íons: Na+, Cl– e SO42–. Se, nesta solução, as concentrações dos íons Na+ e SO42– são, respectivamente,
0,05 mol/L e 0,01 mol/L, a concentração, em mol/L, de íons Cl– será:
a) 0,01
b) 0,02
c) 0,03
d) 0,04
e) 0,05

259
1600 FÍSICO-QUÍMICA APLICADA EXERCÍCIOS COMENTADOS - IME – ITA – OLIMPÍADA

Questão 539 – (OLIMPÍADA PORTUGUESA DE QUÍMICA) O Oceano Atlântico tem, em média, 28 g de cloreto de
sódio por 1 kg de água. A concentração de cloreto de sódio em ppm é:
a) 28000
b) 28
c) 28 × 106
d) Nenhuma das anteriores

Questão 540 – What is (a) the molarity and (b) the molality of a 13% solution (by weight) of sulphuric acid with a
density of 1,02 g/mL? (c) To what volume should 100 mL of this acid be diluted in order to prepare a 1,5 N solution?
Resposta: a) [H2SO4] = 1,35 mol x L-1; b) W = 1,52 mol x kg-1; c) V = 180 mL.
Questão 541 – (ITA) São preparadas duas misturas: uma de água e sabão e a outra de etanol e sabão. Um feixe de
luz visível incidindo sobre essas duas misturas é visualizado somente através da mistura de água e sabão. Com base
nestas informações, qual das duas misturas pode ser considerada uma solução? Por quê? Resposta: Etanol e
sabão, que é opticamente vazia em relação ao efeito Tyndall.

Questão 542 - A glicose, com fórmula estrutural C6H12O6, é um açúcar simples e é também a principal fonte de energia
para os seres humanos e outros vertebrados. Açúcares mais complexos podem ser convertidos em glicose. Numa
série de reações a glicose combina-se com o oxigênio que respiramos e produz, após muitos compostos
intermediários, dióxido de carbono e água, com liberação de energia. A alimentação intravenosa hospitalar consiste
usualmente em uma solução de glicose em água com adição de sais minerais. Considere que 1,50 g de glicose sejam
dissolvidos em 64,0g de água.
a) Calcule a molalidade da solução resultante. Resposta: W = 0,130 mol x kg-1
b) Calcule as frações molares da glicose e da água nesta solução. Resposta: Xágua = 0,998 e Xglicose = 0,002

Questão 543 – (OLIMPÍADA BRASILEIRA DE QUÍMICA) Um técnico dispões de um frasco de ácido nítrico em cujo
rótulo está escrito: Concentração = 60% em massa e Densidade = 1,48 g x mL-1.
a) Escreva a fórmula do ácido nítrico.
b) Escreva a equação química correspondente à neutralização do ácido nítrico pelo hidróxido de cálcio.
c) Determine a concentração em mol x L-1 do ácido nítrico contido no frasco.
d) Que volume desse ácido nítrico seria necessário para preparar 500mL de uma nova solução de ácido nítrico de
concentração 2,0 mol x L-1.
e) Que volume de uma solução de hidróxido de sódio de concentração 20 g x L-1 seria necessário para neutralizar 20
mL da solução de ácido nítrico preparada no item anterior (item d)? Resposta: a) HNO3; b) 2 HNO3(aq) + Ca(OH)2(aq)
→ Ca(NO3)2(aq) + 2 H2O(l); c) [HNO3] = 14,09 mol x L-1; d) Vsolução = 70,97 mL; e) V = 40 mL

Questão 544 – (MESTRE JOÃO ROBERTO DA PACIÊNCIA NABUCO) Calcule o volume de solução 2,0 N de
ácido neutraliza 40 mL de uma solução de hidróxido de cálcio 5,0 g x L-1. Resposta: 2,70 mL.

Questão 545 – (MESTRE JOÃO ROBERTOO DA PACIÊNCIA NABUCO) Misturamos 100 mL de ácd fosfórico 2 mol
x L-1 com 400 mL do mesmo ácido 2,0 N. Da solução resultante retiramos 100 mL aos quais adicionamos 400 mL de
ácido 1,0 N e diluímos com água suficiente para formar um litro de solução. Calcule a normalidade final desta última
solução. Resposta: 0,20 eqg x L-1

260
1600 FÍSICO-QUÍMICA APLICADA EXERCÍCIOS COMENTADOS - IME – ITA – OLIMPÍADA

Resolução: As equações
químicas balanceadas estão
apresentadas
abaixo:CAPÍTULO IV

PROPRIEDADES
COLIGATIVAS

PROFESSOR ALEXANDRE
VARGAS GRILLO

261
1600 FÍSICO-QUÍMICA APLICADA EXERCÍCIOS COMENTADOS - IME – ITA – OLIMPÍADA

262
1600 FÍSICO-QUÍMICA APLICADA EXERCÍCIOS COMENTADOS - IME – ITA – OLIMPÍADA

Questão 546 – Defina propriedades coligativas.

Resposta: Propriedades coligativas são propriedades provocadas pela presença de um soluto não volátil que
dependem do número de partículas dissolvidas e não da natureza do soluto, ou seja, não importa se é um sal orgânico
ou inorgânico, açúcar, ácido, mas sim da quantidade que foi dissolvida.

Questão 547 – Quais são os efeitos coligativos que são estudadas?

Resposta: As propriedades coligativas estudadas são as seguintes:

➢ Abaixamento da pressão de vapor ou também conhecido como tonoscopia (tonometria);


➢ Elevação da temperatura de ebulição ou também conhecido como elulioscopia (ebuliometria);
➢ Abaixamento da temperatura de solidificação ou também conhecido como crioscopia (criometria);
➢ Pressão osmótica (Osmometria).

Questão 548 – Quais são os aspectos comuns das propriedades coligativas?

Resposta: Todo estudo de um processo coligativo provém da diminuição do potencial químico (μ) do solvente líquido
provocado pela presença de um soluto não volátil. Para as soluções diluídas ideais, a diminuição faz o potencial de
𝜇𝐴∗ que indica que este potencial o solvente está puro, passando para 𝜇𝐴∗ + 𝑅 𝑥 𝑇 𝑥 𝑙𝑛𝑋𝐴 , onde esta equação indica
a presença do soluto não volátil. O termo XA é a fração molar, que para este caso, torna-se menor que 1, XA < 1, o
que faz com que lnXA seja negativo e automaticamente o potencial fica menor.

Questão 549 – Enuncie a lei de Raoult.

Resposta: A lei de Raoult, estudada e enunciada por François Marie Raoult um notável químico francês, introduziu o
conceito de solução ideal. “O abaixamento relativo de pressão de vapor de um solvente puro, devido à adição de um
soluto não volátil, é igual a fração molar do soluto na solução.”

Questão 550 – Defina a propriedade coligativa tonoscópica.

Resposta: Abaixamento da pressão de vapor ou tonoscopia é uma das propriedades coligativas que estuda o
abaixamento da pressão máxima de vapor de um líquido, quando se dissolve um soluto não volátil.

Questão 551 – A água do mar apresenta maior ou menor pressão de vapor em comparação a água pura e cristalina.

Resposta: A água do mar vai apresentar menor pressão de vapor, segundo a lei de Raoult. Isso é explicado pelo fato
da água do mar apresentar sais dissolvidos, tais como o cloreto de sódio (NaCl).

Questão 552 – Defina a propriedade coligativa ebulioscópica.

Resposta: A propriedade coligativa ebulioscópica é o aumento da temperatura de ebulição de uma solução, a partir
do momento que é adicionado um soluto não-volátil em um solvente puro a pressão constante.

Questão 553 – Defina a propriedade coligativa crioscópica.

263
1600 FÍSICO-QUÍMICA APLICADA EXERCÍCIOS COMENTADOS - IME – ITA – OLIMPÍADA

Resposta: A propriedade coligativa crioscópica refere-se a medida do abaixamento da temperatura de solidificação


provocado pela adição de um soluto não-volátil.

Questão 554 – Conceitue o processo de Osmose.

Resposta: O processo de osmose é uma difusão seletiva, em que após um determinado tempo a concentração entre
as duas fases separadas por uma membrana semipermeável irá apresentar uma concentração igual em qualquer
ponto da solução final, o que significa que o solvente se difundiu através da solução.

Questão 555 – Defina osmometria.

Resposta: É o estudo da osmose, ou seja, a passagem espontânea do solvente para a solução mais concentrada
por uma membrana semipermeável.

Questão 556 – Enuncie as duas leis da osmometria.

Resposta: As leis da osmometria são as seguintes:

1° lei da osmometria: A temperatura constante, a pressão osmótica é diretamente proporcional a concentração da


quantidade de matéria da solução.

παM

2° lei da osmometria: à concentração da quantidade de matéria sendo constante, a pressão osmótica é diretamente
proporcional a temperatura.

παT

Questão 557 – Classifique as soluções quanto a osmometria.

Resposta:

Solução A hipotônica em relação a solução B, então a relação entre as pressões osmóticas fica da seguinte maneira:
πA < πB;
Solução A hipertônica em relação a solução B, então a relação entre as pressões osmóticas fica da seguinte maneira:
πA > πB;
Solução A isotônica em relação a solução B, então a relação entre as pressões osmóticas fica da seguinte maneira:
πA = πB.

Questão 558 – (ITA) Explique por que a água pura exposta à atmosfera e sob pressão de 1,0 atm entra em ebulição
em uma temperatura de 100°C, enquanto água pura exposta à pressão atmosférica de 0,7 atm entra em ebulição
em uma temperatura de 90°C.

264
1600 FÍSICO-QUÍMICA APLICADA EXERCÍCIOS COMENTADOS - IME – ITA – OLIMPÍADA

Resposta: A pressão de vapor de uma substância aumenta com o aumento da temperatura. Quando a pressão de
vapor se iguala à pressão atmosférica (pressão local), o líquido entra em ebulição, logo, em um determinado local
onde a pressão atmosférica é 0,7 atmosferas, a água entra em ebulição em uma temperatura menor que 100°C.

Questão 559 – (ITA) Esboce graficamente o diagrama de fases (pressão versus temperatura) da água pura (linhas
cheias). Neste mesmo gráfico, esboce o diagrama de fases de uma solução aquosa 1 mol.kg-1 em etilenoglicol (linhas
tracejadas).

Resposta: Através do diagrama de fases da água pura, podemos concluir as seguintes transformações:

𝑠𝑜𝑙𝑢çã𝑜
𝑠𝑜𝑙𝑣𝑒𝑛𝑡𝑒
Transformação Líquido → Gasoso na solução etilenoglicol: 𝑝𝑣𝑎𝑝𝑜𝑟 < 𝑝𝑣𝑎𝑝𝑜𝑟 . Assim é mais difícil fazer a
transição L → G para a solução, então para pressão constante, a temperatura de ebulição da solução é maior que a
da água pura.

Transformação Líquido → Sólido na solução etilenoglicol: Ocorre com maior dificuldade para uma pressão constante,
assim a temperatura de congelamento da solução é menor que a da água pura

Transformação Sólido → Gasoso: Com o abaixamento da pressão de vapor é necessária também uma temperatura
mais alta para sublimação considerando pressão constante.

Questão 560 – (ITA) Sólidos têm pressão de vapor. Certo ou errado?

Resposta: A frase está correta. Todas as três fases (sólido, líquido e gasoso) apresentam pressão de vapor.

Questão 561 – (ITA) Em igualdade de concentração, duas substâncias não-voláteis quaisquer fornecem soluções
com as mesmas propriedades. Certo ou errado?

Resposta: A frase encontra-se errada. Mesmo apresentando a mesma concentração molar, é necessário saber se
as substâncias não-voláteis são moleculares ou iônicas. Este fator diferencia e bastante as propriedades coligativas.

265
1600 FÍSICO-QUÍMICA APLICADA EXERCÍCIOS COMENTADOS - IME – ITA – OLIMPÍADA

Questão 562 – (ITA) Descreva como se poderia determinar, experimentalmente, a pressão de vapor do etanol na
temperatura de 30C. Sua descrição deve incluir um esquema da aparelhagem que poderia ser utilizada.

Resposta: Numa câmara barométrica do tipo Torricelli, mantida a temperatura de 30ºC, coloca-se, por meio de um
conta-gotas, o etanol líquido a a mesma temperatura, ou seja, T = 30ºC. Este é menos denso que o mercúrio e, por
isso, sobe para a câmara barométrica vazia, onde se vaporiza. O vapor de etanol abaixa a coluna de mercúrio pelo
fato de exercer pressão sobre ela. O processo vai ocorrendo em gota a gota o etanol até a câmara ficar saturada de
vapor, isto é, uma nova gota de etanol introduzida não se vaporizará, permanecendo líquida na superfície do mercúrio
da câmara barométrica. Por isso, que ddizemos que os vapores do etanol são saturantes a 30ºC e o abaixamento da
coluna de mercúrio será igual à pressão (máxima) de vapor do etanol naquela temperatura. A figura abaixo mostra
exatamente o processo descrito.

Questão 563 – (ITA) Escrever as expressões matemáticas das leis de Raoult à criometria e à tonometria de soluções
diluídas de não eletrólitos, dando-o significado dos símbolos empregados.

Resposta:Analisando o efeito coligativo crioscópico - abaixamento da temperatura de congelamento: ∆Tc = K c x W


∆Tc = variação da diminuição da temperatura de congelamento
K c = constante crioscópico molal

R x (T 0 )2
Kc =
1000 x LF

K c = constante crioscópico molal; R = constante dos gases ideais; T° = temperatura absoluta do solvente puro;
LF = calor latente de fusão do solvente.

∆p
Analisando o efeito tonoscópico (abaixamento da pressão de vapor: p°
= kt x W

p − p°
= kt x W

p = pressão de vapor da solução; p° = pressão de vapor do solvente; k t


= constante molal tonoscópica; W = molalidade.

p − p° < MM >solvente nsoluto


= x (kg) xi
p° ⏟ 1000 m
𝑘𝑡 solvente

266
1600 FÍSICO-QUÍMICA APLICADA EXERCÍCIOS COMENTADOS - IME – ITA – OLIMPÍADA

Questão 564 – Definir pressão osmótica de uma solução.


Resposta: Pressão osmótica de uma solução é a pressão desenvolvida na osmose e corresponde à pressão externa
que é preciso aplicar sobre a solução para impedir o processo osmótico. A determinação da pressão osmótica (π) é
objeto do efeito coligativo da osmometria. A equação da pressão osmótica é dada por: π = [soluto] x R x T x i
Questão 565 – Sabe-se que 1 fórmula – grama de glicose dissolvida em 1 kg de água provoca o abaixamento de
1,86°C na temperatura de congelamento da mesma. Qual o abaixamento máximo dessa temperatura que será
causado por uma mistura da mesma. Qual o abaixamento máximo dessa temperatura que será causado por uma
mistura de 5,85 g de cloreto de sódio e 26,14 g de nitrato de bário, dissolvidos em 1 kg de água?
H2 O
⏞ Na+
Resolução: Analisando a solução de cloreto de sódio: NaCl: NaCl(aq) → −
(aq) + Cl(aq) . Levando em
consideração que o grau de dissociação seja de 100%.
i = 1 + (2 – 1) x 1 = 2
5,85

Cálculo da temperatura crioscópica (1): ∆TC1 = −1,86 x 58,5


1,0
x 2,0 = −0,372°C
H2 O
⏞ Ba+2
Analisando a solução de nitrato de bário: NaCl: Ba(NO3 )2(aq) → −
(aq) + 2 NO3(aq) . Levando em consideração
que o grau de dissociação seja de 100%.

i = 1 + (3 – 1) x 1 = 3
2,61
Cálculo da temperatura crioscópica (1): ∆TC2 = −1,86 x 261
1,0
x 3,0 = −0,558°C

Cálculo da temperatura máxima de congelamento: ∆TC1 + ∆TC2 = −0,372°C − 0,558°C = −0,930°C

Questão 566 – Duas soluções A e B têm a seguinte composição: solução A: 18,9 g de antraceno (massa molecular
= 178) em 100 mL de benzeno; solução B: 6,4 g de naftaleno (massa molecular = 128) em 100 mL de benzeno.
Determine a relação entre a pressão de vapor pA e pB dessas soluções, à mesma temperatura.
a) p A = pB
b) p A > pB
c) p A < pB
d) faltam dados para responder
e) Nenhuma das respostas anteriores

Resolução: Alternativa B.
p−p°
Para a resolução deste problema será levado em consideração que o processo coligativo seja tonoscópico: p°
=
<MM>solvente nsoluto
⏟ 1000 x (kg) xi
msolvente
𝑘𝑡
18,9
∆𝑝𝐴 <MM>benzeno
Analisando a solução A: p°
= ⏟ 1000 x 178
(kg) x 1,0
mbenzeno
𝑘𝑡
6,8
∆𝑝𝐵 <MM>benzeno
Analisando a solução B: p°
=⏟ 1000
128
x (kg) x 1,0
mbenzeno
𝑘𝑡
18,9
<MM>benzeno 178
∆𝑝𝐴 ⏟ 1000 x (kg) x 1,0
m
p° 𝑘𝑡 benzeno
Cálculo da razão entre as pressões pA e p :B ∆𝑝𝐵 = 6,8
<MM>benzeno 128

⏟ 1000 x (kg) x 1,0
m
𝑘𝑡 benzeno

267
1600 FÍSICO-QUÍMICA APLICADA EXERCÍCIOS COMENTADOS - IME – ITA – OLIMPÍADA

18,9
∆pA
= 178 ≅ 2,0
∆pB 6,8
128

Questão 567 – A 20°C a pressão de vapor da água apresenta uma pressão igual a 17,54 mmHg. Foram dissolvidos
114 gramas de sacarose em 1000 gramas de água, o que ocasionou em uma diminuição da pressão de vapor em
0,11 mmHg. A partir desta informação, determine a massa molecular deste soluto.

Resolução: Dados do problema: Pressão de vapor da água pura (P°) = 17,54 mmHg; Massa do soluto (sacarose) =
114 g; Massa do solvente (água) = 1000 g e Variação da pressão absoluta (∆P) = 0,11 mmHg.

n 114
Número de mol do soluto: nsoluto = <MM>
soluto
= <MM>
soluto

n 1000
Número de mol do solvente: nsolvente = <MM>
solvente
= 18
= 55,56 mol
solvente

114
∆P nsoluto <MM>soluto
Através do efeito coligativo tonoscópico, temos: = = 114
P° nsoluto + nsolvente +55,56
<MM>soluto
114
0,11 < MM >soluto
=
17,54 114
< MM >soluto + 55,56

114 114
6,27 x 10−3 x { + 55,56} =
< MM >soluto < MM >soluto
g
< MM >soluto = 325,52
mol

Questão 568 – Calcule o abaixamento relativo e o abaixamento absoluto da pressão máxima de vapor quando 5,0
gramas de ureia, CO(NH2)2, são dissolvidos em 500 gramas de água, sabendo que a pressão máxima de vapor da
água pura é igual a 32 torr a 28°C.

Resolução: Para este exercício será apresentado duas maneiras de resolução. Primeira maneira de resolução:
Dados do problema: Pressão de vapor da água pura (P°) = 32 torr; Massa do soluto (ureia) = 5,0 g e Massa do
solvente (água) = 500 g.
m 5
Cálculo do número de mol do soluto: nureia = <MM>
ureia
= 60
= 0,083 mol
ureia

m 500
Cálculo do número de mol do solvente: nsolvente = <MM>
solvente
= 18
= 27,78 mol
solvente

∆P ∆P
Cálculo do abaixamento relativo ( P° ) da pressão máxima de vapor: P° = X soluto

∆P nsoluto 0,083
= = = 2,99 x 10−3
P° nsoluto + nsolvente 0,083 + 27,78
∆P
Cálculo do abaixamento absoluto (∆P) da pressão máxima de vapor: P°
= 2,99 x 10−3

∆P
= 2,99 x 10−3
32 torr

268
1600 FÍSICO-QUÍMICA APLICADA EXERCÍCIOS COMENTADOS - IME – ITA – OLIMPÍADA

∆P = 2,99 x 10−3 x 32 torr = 0,0957 torr

Segunda maneira de resolução:


<MM>solvente 18
Cálculo da constante tonoscópica molal: K t = 1000
= 1000

nsoluto 0,083
Cálculo da molalidade: W = = 0,500 = 0,167 mol. kg −1
msolvente
(kg)

∆P <MM>solvente
Cálculo do abaixamento relativo (∆P/P°) da pressão máxima de vapor: P° = K t x ⏟ 1000
Molalidade
∆P 18
= 0,167 x = 2,99 x 10−3
P° 1000
∆P
Cálculo do abaixamento absoluto (∆P) da pressão máxima de vapor: P°
= 2,99 x 10−3

∆P
= 2,99 x 10−3 = 0,0957 torr
32 torr

Questão 569 – (OLIMPÍADA DE QUÍMICA DO RIO DE JANEIRO) A pressão de vapor do benzeno (C6H6) e do
tolueno (C7H8) a 25°C são 95,1 mmHg e 28,4 mmHg, respectivamente. Uma solução das duas substâncias é
preparada com uma fração molar de tolueno de 0,60. Considerando a solução ideal, a pressão total de vapor sobre a
solução, em mmHg, seria:
a) 38,4 mmHg
b) 123,5 mmHg
c) 55,1 mmHg
d) 68,4 mmHg
e) 75,2 mmHg

Resolução: Alternativa C.
Sabendo que a soma das frações molares é igual a 1, e que Xtolueno = 0,60, temos: Xtotal = Xtolueno + Xbenzeno
Xbenzeno = 1 - Xtolueno

Xbenzeno = 1 – 0,60 = 0,40


0 0
Cálculo da pressão total de vapor (PTOTAL): Ptotal = X tolueno x Ptolueno + X benzeno x Pbenzeno = 0,60 x 28,4 +
0,40 x 95,1 = 55,08 mmHg
Questão 570 – Calcule a temperatura de ebulição de uma solução que apresenta 15,60 gramas de benzeno
dissolvidos na presença de 500 gramas de acetona, sabendo que a pressão normal, a acetona entra em ebulição a
60°C. Informação para a resolução do problema: constante ebulioscópica molal da acetona (Keb) = 1,71°C.kg.mol-1.

n 15,60
Resolução: Número de mol do soluto (benzeno): nsoluto = <MM>
soluto
= 78
= 0,20 mol
soluto

nsoluto 0,200
Cálculo da molalidade (W): W = = 0,500 = 0,400 mol. kg −1
msolvente
(kg)

Cálculo da temperatura da solução, a partir da variação da temperatura ebulioscópica: ∆Teb = k acetona


eb xWxi

0,200
T − 60 = 1,71 x [ ] x1
0,500
T = 60 + 0,684 = 60,68°C

269
1600 FÍSICO-QUÍMICA APLICADA EXERCÍCIOS COMENTADOS - IME – ITA – OLIMPÍADA

Questão 571 – A pressão osmótica de uma dispersão de moléculas de poliestireno em tolueno é 0,78 mmHg a 25°C.
Considerando que a dispersão continha aproximadamente 3,0 x 10-3 gramas de poliestireno por mililitro, determine a
massa molecular do poliestireno.
Resolução: Cálculo da massa molecular do poliestireno, tendo que levar em consideração que esta molécula
apresenta coeficiente da van’ t Hoff igual a um (i = 1) e a pressão seja igual a 1 atm (760 mmHg).
π =MxRxTxi
0,78 3,0 x 10−3
= x 0,08206 x (25 + 273) x 1
760 < MM > x 10−3
0,78 3,0
= x 0,08206 x 298
760 < MM >
< MM > = 73361 g. mol−1
Questão 572 – (GRILLO) O fator de Van´t Hoff para uma solução de nitrato de bário com concentração igual a 0,256
mol.L-1 é igual a 2,44. Calcule o grau de dissociação deste sal.

Resolução: Cálculo do grau de dissociação (α), a partir do fator de Van´t Hoff (i): Ba(NO3)2(aq) → Ba+2(aq) + 2 NO3-(aq)
i = 1 + α.[n – 1]
2,44 = 1 + α.[(2 + 1) – 1]
2,44 = 1 + 2α
1,44 = 2α

1,44
𝛼= = 0,72 (72%)
2

Questão 573 – Responda os itens que se seguem.


a) Calcule a pressão osmótica de uma solução aquosa contendo 5,5 gramas de sacarose por 1,23 dm³ à 25°C.
b) Calcule a pressão osmótica de uma solução aquosa contendo 5,5 gramas de hidróxido de cálcio por 1,23
dm³ à 25°C.

Resolução:
a) Cálculo da pressão osmótica da solução molecular – sacarose - C12H22O11: Sabendo que a sacarose é uma
solução molecular (i = 1), temos:
msacarose 5,50
< MM >sacarose
πsacarose = [sacarose] x R x T = { } x R x T = { 342 } x 0,08206 x (25 + 273)
Vsolução 1,23
= 1,31 x 10−2 x 0,08206 x 298 = 0,320 atm

b) Cálculo da pressão osmótica de uma solução iônica - hidróxido de sódio. Cálculo do fator de van´t Hoff para
o hidróxido de cálcio, considerando α = 100%: Ca(OH)2(aq) → Ca+2(aq) + 2 OH-(aq)
mbase 5,50
< MM >base
πbase ={ } x R x T = { 74 } x 0,08206 x (25 + 273) x {1 + (3 − 1) x 1}
Vsolução 1,23
5,50
={ } x 0,08206 x 298 x 3 = 4,43 atm
74 x 1,23

Observação: O efeito coligativo para uma solução iônica é maior em comparação à uma solução molecular.

270
1600 FÍSICO-QUÍMICA APLICADA EXERCÍCIOS COMENTADOS - IME – ITA – OLIMPÍADA

Questão 574 – Considere que 3,0 gramas de uma substância desconhecida seja colocada em 100 gramas de
tetracloreto de carbono, e, com isso, o ponto de ebulição seja igual a 0,60°C. Sabendo que a constante ebulioscópica
é igual a 5,03°C.kg.mol-1 e a constante crioscópica vale 31,8°C.kg.mol-1 para o mesmo solvente, determine:
a) o abaixamento do ponto de congelamento do CCl4;
b) a massa molecular do referido soluto.

Resolução:
Item a) Cálculo da massa molar do soluto desconhecido (<MM>soluto), a partir da variação de temperatura
ebulioscópica: ∆Teb = K eb x W x i

3,0
(< MM > )
soluto
0,60 = 5,03 x x1
0,100

0,60 x 0,100 x < MM >soluto = 5,03 x 3,0 x 1

15,09
< MM >soluto = = 251,5 g x mol−1
0,060
3,0
( )
Item b) Cálculo da variação da temperatura crioscópica (∆Tc): ∆Tc = K c x W x i = 31,08 x 251,5
0,100
x 1 = 3,71°C

Questão 575 – Responda os seguintes itens que seguem abaixo.


a) Uma solução é preparada pela dissolução de 1,0 grama de etilenoglicol (C2H6O2) na presença de 200 gramas
de solvente, sendo este solvente a água. Sabendo que a constante crioscópica da água é igual a 1,86°C.kg.mol -1 e
que a temperatura da água corresponde a 0°C, determine a temperatura de congelamento, em graus Celsius. Dado:
Constante crioscópica = 1,86°C.kg.mol-1.
b) 350 mL de uma solução 0,50 mol.L-1 de NaOH com 400 mL de uma solução 0,30 mol.L-1 de NaOH. Determine
a concentração da solução final, em mol/L.
c) 0,820 gramas de gelatina foi dissolvido em água suficiente para preparar 150 mL de solução. A pressão
osmótica da solução encontrada foi 67,20 mmHg, a 27°C. Determine a massa molecular aparente desta gelatina.

Resolução: Item a) Sabendo que o efeito coligativo crioscópico é dado por ∆Tc = Kc x W x i e o fator de van´t Hoff
sendo igual a um (i = 1), o cálculo da temperatura da solução será:
msoluto
T 0 − T solução = K c x xi
< MM >soluto x Vsolução

1,0
0 − T solução = 1,86 x x1
62,0 x 0,20

− 1,86
T solução = = −0,15°C
12,4

Item b) Cálculo da concentração da quantidade de matéria da solução final:

Solução (1): [NaOH]1 = 0,50 mol.L-1; V1 = 350 mL

Solução (2): [NaOH]2 = 0,30 mol.L-1; V1 = 400 mL

Solução Final: [NaOH]final = X; Vfinal = (350 + 400) mL

[NaOH]1 x V1 + [NaOH]2 x V2 = [NaOH]final x Vfinal

271
1600 FÍSICO-QUÍMICA APLICADA EXERCÍCIOS COMENTADOS - IME – ITA – OLIMPÍADA

[NaOH]1 x V1 + [NaOH]2 x V2 0,50 x 350 + 0,30 x 400


[NaOH]final = = = 0,393 𝑚𝑜𝑙. 𝐿−1
Vfinal 750

Item c) Sabendo que a pressão osmótica é definida como sendo: π = M x R x T x i e sendo a gelatina uma solução
molecular, o fator de van´t Hoff é igual a 1. Cálculo da massa molecular da gelatina:

mgelatina 0,820
< MM >gelatina < MM >gelatina
πgelatina ={ }x R x T = { } x 0,08206 x (27 + 273)
Vsolução 0,150

0,820
8,84 x 10−2 = { } x 0,08206 x 300
0,150 x < MM >gelatina

< MM >gelatina = 1522,38 g. mol−1

Questão 576 – Responda os seguintes itens a seguir que se apresentam a seguir.


a) Em um centro de pesquisa foi isolada uma certa proteína, a partir de uma amostra de soro sanguíneo. Uma
dispersão coloidal de massa igual a 885 mg desta proteína na presença de água formou uma solução com volume
igual a 100 mL, apresentado pressão osmótica igual a 0,28 atm a 17°C. A partir desta informação, determine a massa
molecular da referida proteína.
b) Em um determinado meio aquoso, uma solução de glicose apresenta concentração molar igual a 0,13 mol.L-
1 a uma temperatura correspondente a -23°C. Esta solução molecular é uma solução isotônica de uma outra solução,

sendo esta iônica de cloreto de cálcio, a 27°C. Sabe-se que o grau de dissociação iônica deste sal é igual a 80%.
Diante destas informações, calcule a concentração da quantidade de matéria do referido sal.

Resolução:

Item a) Sabendo que a pressão osmótica é dada por: π = M x R x T x i e a proteína é uma substância molecular, o
fator de Van´t Hoff (i) é igual a 1.

0,885
< MM >proteína
0,28 = { } x 0,08206 x (17 + 273)
0,100

0,885
0,28 = { } x 0,08206 x 290
0,100 x < MM >proteína

0,028 x < MM >proteína = 0,885 x 0,08206 x 290

< MM >proteína = 752,17 g. mol−1

Item b) Sendo a glicose uma solução molecular, o seu fator de Van´t Hoff é igual a 1, logo: πglicose =
[glicose] x R x T

πglicose = 0,13 x R x (−23 + 273)


πglicose = 0,13 x R x 250 = 32,50 x R

Analisando a solução iônica de cloreto de cálcio: πCaCl2 = [CaCl2 ] x R x T = [CaCl2 ] x R x (27 + 273)x [1 +
(3 − 1) x 0,80] = 780 x R x [CaCl2 ]

Como as soluções são isotônicas, as pressões osmóticas são iguais. πglicose = πCaCl2

32 x R = 780 x R x [CaCl2 ]

272
1600 FÍSICO-QUÍMICA APLICADA EXERCÍCIOS COMENTADOS - IME – ITA – OLIMPÍADA

[CaCl2 ] = 0,0410 mol. L−1

Questão 577 – Em um laboratório de análises químicas, forma preparadas quatro soluções, apresentando a mesma
concentração molar (W) e o mesmo grau de dissociação (α) foi preparada quatro soluções aquosas indicadas abaixo:
(a) Glicose
(b) Fosfato de cálcio
(c) Cloreto de magnésio
(d) Cloreto de sódio
O objetivo desta análise química foi determinar a pressão osmótica de cada solução. Diante disso, coloque em ordem
decrescente de pressão osmótica (π).

Resolução:
a) Para a solução de glicose, o fator de Van´t Hoff é igual 1 (i = 1), por ser uma solução molecular.

b) Solução de fosfato de cálcio: Ca3(PO4)2(s) → 3 Ca+2(aq) + 2 PO4-(aq)


n = 3 cargas para o cátion + 2 cargas para o ânion = 5
i = 1 + (n – 1).α
i = 1 + (5 – 1).α
i = 1 + 4.α

c) Solução de cloreto de magnésio: MgCl2(s) → Mg+2(aq) + 2 Cl-(aq)


n = 1 cargas para o cátion + 2 cargas para o ânion = 3
i = 1 + (n – 1) x α
i = 1 + (3 – 1) x α
i = 1 + 2.α

d) Solução de cloreto de sódio: NaCl(s) → Na+(aq) + Cl-(aq)


n = 1 cargas para o cátion + 1 cargas para o ânion = 2
i = 1 + (n – 1).α
i = 1 + (2 – 1).α
i=1+α

Logo, a ordem decrescente da pressão osmótica será a seguinte: πb > πc > πd > πa.

Questão 578 – Responda os seguintes itens a seguir.


a) Calcule a pressão de vapor de 0,50 litros de solução aquosa que contém como solutos 60 gramas de cloreto
de sódio e 23,8 gramas de brometo de potássio a T = 20°C, considerando que os respectivos sais são solutos não
voláteis. Dado: P°H2O = 17,60 mmHg (20°C) e a densidade da solução igual a 1,05 g.mL-1.
b) Foram dissolvidos 34,2 gramas de sacarose e 6,00 gramas de glicose em 250 gramas de etanol. Determine
a pressão de vapor da solução resultante desta mistura, a 25°C? Dado: Pressão de vapor do etanol puro = 60,00
mmHg a 25°C.
n 60
Resolução: Item a) Cálculo do número de mol de cloreto de sódio: nNaCl = <MM>
NaCl
= 58,5 = 1,025 mol
NaCl

n 23,8
Cálculo do número de mol de brometo de potássio: nKBr = <MM>
KBr
= 119
= 0,20 mol
KBr

Cálculo da massa de solução, a partir da densidade da solução:

1 mL ------------- 1,05 gramas de solução


500 mL ---------- msolução
msolução = 525 gramas

273
1600 FÍSICO-QUÍMICA APLICADA EXERCÍCIOS COMENTADOS - IME – ITA – OLIMPÍADA

Cálculo da massa de solvente (H2O): msolução = msoluto + msolvente


msolução = (mNaCl + mKBr) + mH2O
mH2O = msolução - (mNaCl + mKBr) = 525 g - (60 + 23,8) g = 441,2 gramas
nH O 441,5
Cálculo do número de mol de H2O: nH2 O = <MM>2 = 18
= 24,63 mol
H2 O

Cálculo do número de mol total: nTOTAL = nH2O + nNaCl + nKBr = 24,53 + 1,025 + 0,20 = 25,75 mol
nH2O 24,63
Cálculo da fração molar da água (XH2O): X H2 O = n = 25,75
= 0,956 mol
TOTAL

Cálculo da pressão de vapor da solução: P = XH2O x P°H2O = 0,956 x 17,60 = 16,82 mmHg
mC H22 O11 34,2
Item b) Cálculo do número de mol de sacarose (C12H22O11): nC12 H22 O11 = <MM>12 = 342
= 0,10 mol
C12 H22 O11

mC H12 O6 6,0
Cálculo do número de mol de glicose (C6H12O6): nC6 H12 O6 = <MM>6 = 192 = 0,03125 mol
C6 H12 O6

nC H O 250,0
Cálculo do número de mol de solvente - Etanol (C2H5OH): nC2 H6 O = <MM>
2 6
= 46
= 5,43 mol
C 2 H6 O

Cálculo do número de mol total (nTOTAL): nTOTAL = nC12H22O11 + nC6H12O6 + nC2H6O = 0,10 + 0,03125 + 5,43 = 5,56 mol
nC2H6O 5,43
Cálculo da fração molar do etanol (XC2H5OH): X C2 H6 O = = = 0,977 mol
nC2 H6O + nC6H12O6 5.56

Cálculo da pressão de vapor da solução: P = XC2H6O x P°C2H6O = (0,976) x 60,0 = 58,56 mmHg

Questão 579 – (GRILLO) Considere uma solução aquosa composta por 480 gramas de glicose e 700 gramas de
água, com esta solução processada a temperatura ambiente. Sabendo que a pressão máxima de vapor da água nesta
temperatura ambiente é igual a 22 mmHg, calcule:
a) o valor do abaixamento relativo da pressão máxima de vapor.
b) o valor do abaixamento absoluto da pressão máxima de vapor.
c) o valor da pressão máxima de vapor da solução.
<𝑀𝑀>𝑠𝑜𝑙𝑣𝑒𝑛𝑡𝑒 18
Resolução: Item a) Cálculo da constante tonoscópica (Kt): 𝐾𝑡 = =
1000 1000
480
𝑛𝑠𝑜𝑙𝑢𝑡𝑜 ( )
Cálculo da molalidade (W): 𝑊 = 𝑠𝑜𝑙𝑣𝑒𝑛𝑡𝑒
180
= 0,700 = 3,81 𝑚𝑜𝑙. 𝑘𝑔−1
𝑚(𝑘𝑔)

∆P <MM>solvente 18
Cálculo do abaixamento relativo da pressão máximo de vapor: P°
= 1000
x W x i = 1000 x 3,81 x 1 =
0,0686
∆𝑃 18
Item b) Cálculo do abaixamento absoluto da pressão de vapor (Δp): 22 = 1000 𝑥 3,81
18
∆𝑃 = 22 𝑥 𝑥 3,81 = 1,51 𝑚𝑚𝐻𝑔
1000

Item c) Cálculo da pressão máxima de vapor da solução (P): ∆P = 1,51 mmHg


P° - P = 1,51 mmHg
22 – P = 1,51 mmHg
P = 20,49 mmHg

274
1600 FÍSICO-QUÍMICA APLICADA EXERCÍCIOS COMENTADOS - IME – ITA – OLIMPÍADA

Questão 580 – (GRILLO) Responda os seguintes itens a seguir que se apresentam a seguir.
a) A adição de 425 gramas de um determinado composto não volátil na presença de 1250 gramas de benzeno
provocou um abaixamento coligativo crioscópico igual a 23,5 K. A partir desta informação, calcule a massa molecular
do composto, sabendo que a constante crioscópica do benzeno é igual a 5,12 K.kg.mol-1.
b) A pressão de vapor do benzeno puro a 55°C é igual a 450 mmHg, porém com a adição de 40 gramas de um
composto orgânico não-volátil, esta pressão cai para 399 mmHg quando estão presentes 450 gramas de benzeno. A
partir destas informações, determine a massa molar do composto orgânico.
c) Uma solução foi preparada usando os seguintes solutos: 46,0 gramas de glicose, 150 gramas de cloreto de
cálcio com 75% de grau de dissociação e 250 gramas de cloreto de sódio com 90% de grau de dissociação. Todos
estão dissolvidos em água, formando uma solução com um litro de capacidade. Determine a pressão osmótica do
sistema a 27°C.
d) A pressão osmótica de uma solução de nitrato de magnésio 0,125 eqg.L-1 a 37°C é igual a quatro atmosferas.
Determine o grau de dissociação do referido sal.

Resolução: Item a) O problema deixa evidente o efeito coligativo crioscópico. Sendo a solução molecular o fator de
van´t Hoff é igual a 1 (i = 1). ∆Tc = K c x W x i

425
23,5 = 5,12 x ( )x 1
< MM > x 1,25

23,5 425
=( )
5,12 < MM > x 1,25

425
< MM > = = 74,04 g. mol−1
5,74

Item b) O efeito coligativo mencionado é o tonoscópico. Sendo a solução molecular o fator de van´t Hoff é igual a 1 (i
P°− P <MM>solvente n
= 1). = x soluto
etanol x i
P° 1000 mkg
40
450 − 399 78 < MM >soluto
= x x1
450 1000 0,45

< MM >soluto = 61,18 g. mol−1

Item c) Cálculo da pressão osmótica (π) da glicose (C6H12O6), sabendo que o fator de van´t Hoff para este caso é igual
a 1:
mglicose 46
< MM >glicose 180
πglicose ={ }x R x T = { } x 0,08206 x (27 + 273) = 6,29 atm
Vsolução 1,0

Cálculo da pressão osmótica (π) do cloreto de cálcio (CaCl2), considerando que o grau de dissociação seja igual a
75%: CaCl2(s) → Ca+2(aq) + 2 Cl-(aq)

mCaCl2
< MM >CaCl2
πCaCl2 ={ } x R x T x [1
Vsolução

150
+ (n − 1) x α] { 111 } x 0,08206 x (27 + 273) x [1 + (3 − 1) x 0,75] = 83,17 atm
1,0

275
1600 FÍSICO-QUÍMICA APLICADA EXERCÍCIOS COMENTADOS - IME – ITA – OLIMPÍADA

Cálculo da pressão osmótica (π) do cloreto de sódio (NaCl), considerando que o grau de dissociação colocado pelo
problema é igual a 90%: NaCl(s) → Na+(aq) + Cl-(aq)

mNaCl 250
< MM >NaCl 58,5
πNaCl ={ } x R x T x [1 + (n − 1) x α] = { } x 0,08206 x 300 x [1 + 1 x 0,90]
Vsolução 1,0

= 199,89 atm

Cálculo da pressão osmótica total (π): π = 6,29 atm + 83,17 atm + 199,89 atm = 289,35 atm

Item d) Cálculo da concentração em quantidade de matéria, a partir da concentração normal: N = M.x, sendo x se
refere a carga do cátion total da magnésio.

Mg(NO3)2(s) → Mg+2(aq) + 2 NO3-(aq)

N 0,125 mol
M= = = 0,0625
2 2 L

Cálculo do grau de dissociação, a partir da equação da pressão osmótica (π): π = [M] x R x T x [1 + (n − 1) x α]

4 = 0,0625 x 0,08206 x (37 + 273)x [1 + (3 − 1). α]

4 = 0,0625 x 0,08206 x 310 x [1 + 2α]

2,51
= [1 + α]
1,59

α = 1,58 − 1,0 = 0,58 (58%)

Questão 581 – (IME) Uma solução contendo 0,994 gramas de um polímero, de fórmula geral (C2H4)n, em 5,00 g de
benzeno, tem ponto de congelamento 0,51ºC mais baixo que o do solvente puro. Determine o valor de n. Dado:
constante crioscópica do benzeno = 5,10ºC/molal.

Resolução: Dados do problema:


Soluto = polímero
Solvente = benzeno
Fator de Van’t Hoff: i = 1 (solução molecular)

Cálculo do massa molar (<MM>) do polímero: ∆Tc = K c x W x i

0,994
(< MM >)
0,51 = 5,10 x x1
5
(1000)

0,51 994
=
5,10 5 x < MM >

994
0,10 =
5 x < MM >

< MM > = 1988 g. mol−1

Sabendo que a fórmula geral do polímero é dado por (C2H4)n, logo: (C2H4) x n = 1988
(24 + 4) x n = 1988

276
1600 FÍSICO-QUÍMICA APLICADA EXERCÍCIOS COMENTADOS - IME – ITA – OLIMPÍADA

n = 71

Questão 582 – (IME) Uma solução aquosa 0,28 mol.L-1 de glicose é isotônica em relação a uma solução aquosa 0,10
mol.L-1 de um cloreto de metal alcalino-terroso, na mesma temperatura. Calcule o grau de dissociação aparente do
sal.

Resolução: Expressão da pressão osmótica para a glicose: [glicose] = 0,28 mol.L-1; i = 1 (solução molecular)

πglicose = [glicose] x R x T x i = 0,28 x R x T x 1

Reação: XCl2(aq) → X+2(aq) + 2 Cl-(aq)

[XCl2] = 0,10 mol.L-1

i = 1 + (n – 1).α (solução iônica)


Expressão da pressão osmótica para o cloreto de metal alcalino-terroso (XCl2): πXCl2 = [XCl2 ] x R x T x i
πXCl2 = 0,10 x R x T x {1 + (3 – 1).α}
πXCl2 = 0,10 x R x T x {1 + 2α}

Como a solução é isotônica, as pressões osmóticas são iguais, logo: πglicose = πXCl2

0,28 x R x T = 0,10 x R x T x (1 + 2α)

2,8 x R x T = R x T x (1 + 2α)

α = 0,90 (90%)

Questão 583 – (IME) Uma solução foi preparada dissolvendo-se 2,76 g de um álcool puro em 100,00 g de acetona.
O ponto de ebulição da acetona pura é 56,13°C e o da solução é 57,16°C. Determine:
a) a massa molecular do álcool;
b) a fórmula molecular do álcool.
Dado: Keb = 1,72°C.Kg.mol-1 (constante molal de elevação do ponto de ebulição da acetona).

Resolução: Dados do problema: Soluto = álcool puro; Solvente = acetona e Fator de Van’t Hoff: i = 1 (solução
molecular)

a) Cálculo da massa molar do álcool puro, a partir do efeito coligativo ebulioscópico: ∆Teb = K eb x W x i
2,76
(57,16 − 56,13) = 1,72 x MM > x 1
<
0,100

2,76
1,03 = 1,72 x
0,100 x < MM >
4,75
< MM >= = 46,0 g. mol−1
0,103

b) Para uma massa molecular igual a 46 g.mol-1, o solvente é o álcool etílico ou também conhecido como etanol
(C2H6O).

Questão 584 – (IME) Um produto anticongelante foi adicionado a 10,0 litros de água de um radiador para que a
temperatura de congelamento da mistura fosse –18,6ºC. A análise elementar do anticongelante forneceu o seguinte
resultado em peso: C = 37,5%, O = 50,0% e H = 12,5%. Sabe-se que a constante crioscópica molal da água é
1,86°C.kg.mol-1 e sua massa específica é 1,00 kg.dm-3. Determine:
a) a fórmula estrutural plana e o nome do produto utilizado;
b) a massa de produto necessária para alcançar este efeito.

277
1600 FÍSICO-QUÍMICA APLICADA EXERCÍCIOS COMENTADOS - IME – ITA – OLIMPÍADA

Resolução: Dados do problema: Soluto = produto anticongelante; Solvente = água e Fator de Van’t Hoff: i = 1 (solução
molecular)
mH2 O
Cálculo da massa de solvente (água), a partir da sua densidade: d =
Vsolução
mH2 O = d x Vsolução = 1 x 10 = 10 kg

Item a) Fórmula molecular do produto anticongelante com base de cálculo igual a 100 gramas de C, H e O. Cálculo
da massa de cada átomo:

37,5
C: ( ) x 100 g = 37,5 g
100

50
O: ( ) x 100 g = 50 g
100

12,5
H: ( ) x 100 g = 12,5 g
100

Cálculo do número de mol de cada átomo:

37,5
nC = ( ) = 3,125 mol
12

50
nO = ( ) = 3,125 mol
16

12,5
nH = ( ) = 12,5 mol
1

Cálculo do número de átomos, dividindo cada um pelo menor número de mol (3,125 mol):

3,125 mol
C: ( )=1
3,125 mol

3,125 mol
O: ( )=1
3,125 mol

12,5 mol
H: ( )=4
3,125 mol
Fórmula molecular: CH4O (metanol)

Item b) Cálculo da massa do produto anticongelante, a partir do efeito crioscópico: ∆Tc = K c x W x i

m
( )
18,6 = 1,86 x 32 x1
10
18,6 m
=
1,86 320

m = 3200 g

278
1600 FÍSICO-QUÍMICA APLICADA EXERCÍCIOS COMENTADOS - IME – ITA – OLIMPÍADA

Questão 585 – (IME) Determine o abaixamento relativo da pressão de vapor do solvente quando 3,04 g de cânfora
(C10H16O) são dissolvidos em 117,2 mL de etanol a 25°C. Dado: densidade do álcool: 785 kg.m-³.

Resolução: Dados do problema: Soluto = Cânfora; Solvente = Etanol e Fator de Van’t Hoff: i = 1 (solução molecular)

10−3 L 1 dm³ 10−3 m³


Conversão do volume para m³: Vetanol = 117,20 mL x 1 mL
x 1 L
x 1 dm³ = 117,20 x 10−6 m³

metanol
Cálculo da massa de etanol, a partir de sua densidade: detanol = V

kg
metanol = 785 x 117,20 x 10−6 m3 = 92002 x 10−6 kg
m3
∆𝑃 <𝑀𝑀>𝑠𝑜𝑙𝑣𝑒𝑛𝑡𝑒 𝑛𝑐â𝑛𝑓𝑜𝑟𝑎
Cálculo do abaixamento relativo da pressão de vapor (∆P/P°): 𝑃°
= 1000
𝑥 𝑒𝑡𝑎𝑛𝑜𝑙 𝑥𝑖
𝑚𝑘𝑔
3,04
∆𝑃 46 152
= 𝑥 𝑥 1 = 1,0 𝑥 10−2
𝑃° 1000 92002 𝑥 10−6

Questão 586 – (IME) A pressão osmótica de uma solução de poli-isobutileno sintético em benzeno foi determinada a
25ºC. Uma amostra contendo 0,20 gramas de soluto por 100 cm3 de solução subiu até uma altura de 2,4 mm quando
foi atingido o equilíbrio osmótico. A massa específica da solução no equilíbrio é 0,88 g.cm -3. Determine a massa
molecular do poli-isobutileno? Dados: Aceleração da gravidade = 9,81 m.s-2; 1,0 N.m-2 = 9,869 x 10-6 atm; constante
universal dos gases: R = 0,082 atm.L.mol–1.K–1.

Resolução: Dados do problema: Soluto = Poli-isobutileno; Solvente = Benzeno e Fator de Van’t Hoff: i = 1 (solução
molecular)
0,20
Expressão da pressão osmótica (π) sobre a solução: π = 0,10 x<MM> x 0,08206 x (25 + 273) x 1 =
0,20
0,10 x<MM>
x 0,08206 x 298

Expressão e cálculo da pressão (P) sobre o pistão: P= ρxgxh=


g 1 kg 1 cm³ m −3 kg
0,88 3 x x x 9,81 x 2,40 x 10 m= 20,72 (Pa)
cm 1000 g 10−6 g s² m.s²

Cálculo da massa molecular (<MM>) do poli-isobutileno, a partir da igualdade das pressões (P = π): 𝑃 =
[𝑠𝑜𝑙𝑢𝑡𝑜] 𝑥 𝑅 𝑥 𝑇 𝑥 𝑖

0,20
1 atm
20,72 Pa x ( ) = [ MM > ] x 0,08206 x 298 x 1
<
101325 Pa 0,10

279
1600 FÍSICO-QUÍMICA APLICADA EXERCÍCIOS COMENTADOS - IME – ITA – OLIMPÍADA

2,0
0,204 = [ ] x 0,08206 x 298
< MM >

< MM > = 2,39 x 105 g. mol−1

Questão 587 – (IME) Uma solução com 102,6 gramas de sacarose (C12H22O11) em água apresenta concentração de
1,20 mol.L-1 e densidade de 1,0104 g.cm-3. Os diagramas de fase dessa solução e da água pura estão representados
abaixo:

Com base nos efeitos coligativos observados nesses diagramas, calcule as constantes molal ebuliométrica (K e) e
criométrica (Kc) da água.

Resolução: Dados do problema: Soluto = sacarose (C12H22O11); Solvente = água e Fator de Van’t Hoff: i = 1 (solução
molecular).

Cálculo do volume da solução a partir da concentração da quantidade de matéria: [C12 H22 O11 ] =
mC12H22 O11
<MM>C12 H22O11 x Vsolução
102,6
1,20 =
342 x Vsolução

Vsolução = 0,25 L (250 mL)

Cálculo da massa da solução:


1 cm³---------- 10-3 dm³ ------------ 1,0104 gramas de solução
0,25 dm³ ----------- msolução
msolução = 252,6 g

Cálculo da massa do solvente (H2O): msolução = msolvente + msoluto


252,6 gramas = msolvente + 102,6 gramas
msolvente = 150 g

Cálculo da constante ebulioscópica molal (Keb):

∆Tc = K c x W x i

280
1600 FÍSICO-QUÍMICA APLICADA EXERCÍCIOS COMENTADOS - IME – ITA – OLIMPÍADA

102,6
( )
|374 − 373| = K c x 342 x 1
0,150

K eb = 0,50 K. kg. mol−1

Cálculo da constante crioscópica molal (Kc):

∆Tc = K c x W x i

102,6
( 342 )
|269 − 273| = K c x x1
0,150

K c = 2,0 K. kg. mol−1

Questão 588 – (IME) Qual é a temperatura de congelamento de uma solução aquosa de glicerina (C3H8O3) a 20%
em peso, sabendo-se que a constante criométrica molal da água é de Kc = 1,86°C.kg.mol-1.

Resolução: Dados do problema: 20 gramas de C3H8O3 para cada 100 gramas de solução; Fator de Van´t Hoff (i) = 1
(solução molecular)

Cálculo da massa da solvente: Massa da solução = massa do soluto + massa do solvente


100 g = 20 g + massa do solvente
Massa do solvente = 80g
20
Cálculo do número de mol do soluto: nsoluto = = 0,22 mol
92

0,217
Cálculo da temperatura de congelamento: ∆Tc = K c x W x i = 1,86 x 80 x 10−3 x 1 = 5,05°C

Questão 589 – (IME) 7,10 gramas de uma substância A2B, de peso molecular 174, são dissolvidos em água até
completar um litro de solução. Sabendo-se que nestas condições o A2B fica 80% dissociado, pede-se calcular a
pressão osmótica, em atmosferas, sendo a temperatura de 17°C. Resposta: 𝛑 = 𝟐, 𝟓𝟐 𝐚𝐭𝐦

Resolução: Equação química: A2B: A2B(aq) → 2 A+(aq) + B-2(aq)

Cálculo do fator de Van’t Hoff (i):


i = 1 + [3 – 1) x 0,80]
i = 1 + (2 x 0,80)
i = 2,60
7,10

Cálculo da pressão osmótica (π): π = M x R x T x i = { 174


1,0
} x 0,08206 x (17 + 273) x 2,60 =
7,10
{ 174 } x 0,08206 x 290 x 2,60 = 2,52 atm

281
1600 FÍSICO-QUÍMICA APLICADA EXERCÍCIOS COMENTADOS - IME – ITA – OLIMPÍADA

Questão 590 – (IME) Uma solução aquosa de glicerol (C3H8O3) começa a congelar a –20°C. Considerando a
constante molal do abaixamento crioscópico do ponto de fusão da água como sendo 1,84, determinar a porcentagem
em peso de glicerol na solução. Resolução:
mglicerol
<MM>glicerol
Cálculo da relação entre a massa de glicerol com a massa da água (mglicerol/mágua): ∆Tc = K c x água xi
mkg
mglicerol
|0 − (−20,0)| = 1,86 x 92 x 10−3 x 1
água
mkg
mglicerol
água
=1
mkg

mglicerol
Sabendo que o somatório das massas corresponde a 100%, temos: água =1
mkg
água
mglicerol = mkg

água
mglicerol = mkg = 0,50 (50%)

Questão 591 – (IME) Qual o volume de metanol, de massa específica 0,80 g.mL-1, que deve ser adicionado ao
radiador de um veículo, contendo 9,0 litros de água, para que o congelamento não ocorra antes da temperatura cair
abaixo de -10,30°C. Dados: Constante crioscópica molal da água = 1,86°C.mol.kg-1; densidade da água = 1,0 g.cm-3.

Resolução: Dados do problema: Soluto = metanol (CH3OH); Solvente = água e Fator de Van’t Hoff: i = 1 (solução
molecular)

Cálculo do massa de solvente (água), a partir da densidade: mH2 O = d x V = 1,0 x 9000 = 9000 g
mmetanol
<MM>metanol
Cálculo do massa de metanol: ∆Tc = K c x W x i = K c x água xi
mkg

mmetanol
|− 10,30 − 0| = 1,86 x 32 x1
9,0
mmetanol
10,30 = 1,86 x
288

mmetanol = 1594,84 g
mH2 O 1594,84 g
Cálculo do volume de metanol: Vmetanol = d
= 0,80 g.mL−1 = 1993,55 mL

282
1600 FÍSICO-QUÍMICA APLICADA EXERCÍCIOS COMENTADOS - IME – ITA – OLIMPÍADA

Questão 592 – (IME) Uma solução contendo 16,90 gramas de uma substância não dissociativa em 250 gramas de
água, tem um ponto de solidificação de 0,744°C. A substância é composta de 57,20% de carbono, 4,70% de
hidrogênio e 38,10% de oxigênio. Qual é a fórmula molecular desse composto? Dado: constante crioscópica da água
= 1,86°C.kg.mol-1.

Resolução: Dados do problema: Soluto = substância não dissociativa (molecular); Solvente = água e Fator de Van’t
Hoff: i = 1 (solução molecular)
16,90
<MM>X
Cálculo da massa molecular do soluto, a partir do efeito coligativo crioscópico: 0,744 = 1,86 x 0,250
x1

< MM >X = 169 g. mol−1

Determinação da fórmula molecular: Base de cálculo: 100 gramas de C, H e O.

Cálculo da massa de cada átomo:

57,20
C: ( ) x 100 g = 57,20 g
100
38,10
O: ( ) x 100 g = 38,10 g
100
4,70
H: ( ) x 100 g = 4,70 g
100

Cálculo do número de mol de cada átomo:

57,20
nC = ( ) = 4,77 mol
12
38,10
nO = ( ) = 2,38 mol
16
4,70
nH = ( ) = 4,70 mol
1,0

Cálculo do número de átomos, dividindo cada um pelo menor número de mol, ou seja, por 2,38 mol:

4,77 mol
C: ( ) ≅ 2,0
2,38 mol

2,38 mol
O: ( ) = 1,0
2,38 mol

4,70
H: ( ) ≅ 2,0
2,38

Fórmula molecular mínima: (C2H2O)n. Para uma molécula com massa molecular igual a 169 g.mol-1, a fórmula
molecular será: (C2 H2 O)n = 169
(24 + 2 + 16) x n = 169
42 x n = 169
n ≅ 4 (C2H2O)4

283
1600 FÍSICO-QUÍMICA APLICADA EXERCÍCIOS COMENTADOS - IME – ITA – OLIMPÍADA

Questão 593 – (IME) Calcular o número de partículas dispersas numa solução de 1 mol de sulfato de alumínio,
suposto 70% dissociado.

Resolução: Cálculo do fator de Van´t Hoff: Al2(SO4)3 → 2 Al+3(aq) + 3 SO4-2(aq)

i = 1 + [(5 – 1) x 0,70]
i = 1 + [0,70 x 4]
i = 1 + 2,80
i = 3,80

Cálculo do número de partículas dispersas (X): X = 3,80 x 6,02 x 1023 = 2,29x 1024

Questão 594 – (IME) Na produção de uma solução de cloreto de sódio em água a 0,90% (m/m), as quantidades de
solvente e solução são pesadas separadamente, e, posteriormente, promove-se a solubilização. Certo dia suspeitou-
se que a balança de soluto estivesse descalibrada. Por este motivo, a temperatura de ebulição de uma amostra de
solução foi medida, obtendo-se 100,14°C. Considerando o sal totalmente dissociado determine a massa de soluto a
ser acrescentada de modo a produzir um lote de 1000 kg com a concentração correta. Dados: Keb = 0,52°C.kg.mol-1.
Resolução: Equação química: NaCl(aq) → Na+(aq) + Cl-(aq)

Considerando um grau de dissociação igual a 100% o fator de Van’t Hoff será a seguinte:
i = 1 + {(n – 1) x α}
i = 1 + [(2 – 1)] x 1}
i=2

soluçãosolvente
Cálculo da concentração molal (W), a partir da elevação da temperatura de ebulição: Tebulição − Tebulição =
K eb x W x i

100,14 − 100,0 = 0,52 x W x 2

0,14
W= = 0,135 mol. kg −1
1,04

Cálculo da massa em quilogramas de solvente (água): 0,9% (m/m)

0,90 gramas de NaCl 9g 9 kg


= =
100 gramas de solução 100 g 1000 kg

Massa do solvente = massa da solução – massa do soluto = 1000 kg – 9 kg = 991 kg


nNaCl
Cálculo do número de mol de NaCl, a partir da molalidade (W): W = solvente
mkg

nNaCl = W x msolvente
kg = W x msolvente
kg = 0,135 x 991 = 133,78 g

m
Cálculo da massa de NaCl, a partir do número de mol: nNaCl = <MM>
NaCl
NaCl

mNaCl = nNaCl x < MM >NaCl = 133,78 x 58,5 = 7826,42 g (≅ 7,83 kg)

Cálculo da massa de NaCl adicionado: mNaCl = 9,0 kg − 7,83 kg = 1,17 kg

Questão 595 – (IME) Um gás possui uma taxa de efusão que corresponde a 25,0% da taxa do gás hidrogênio. Uma
massa mx desse gás, que ocupa um volume de 1,00 L a 1,00 atm e a 39,5°C é a mesma de sulfanilamida, um soluto
não volátil, dissolvida em 100 gramas de acetona. Se a pressão de vapor da acetona pura a 39,5oC é 400 mmHg,
calcule:

284
1600 FÍSICO-QUÍMICA APLICADA EXERCÍCIOS COMENTADOS - IME – ITA – OLIMPÍADA

a) a massa mx;
b) a pressão de vapor da solução de sulfanilamida (C6H8O2N2S) em acetona à mesma temperatura.

Resolução: Item a) Dados do problema: A velocidade de efusão do gás X é dada pela seguinte relação:
VX = 0,25 x VH2 .

VH2 <MM>
Através da Lei de Graham temos: VX
= √<MM> X .
H2

Massa do solvente (acetona: H3C – CO – CH3) = 100 gramas (0,100 kg).

VH2 <MM>
Cálculo da massa molecular do gás X: = √ X
VX <MM> H2

VH2 <MM>X
0,25 x VH2
= √ 2

1 <MM>X
= √
0,25 2

<MM>X
4= √ 2
<MM>X
(4)² = 2

< MM >X = 16 x 2 = 32,0 g. mol−1


m
Cálculo da massa da espécie gasosa X, considerando comportamento ideal: p. V = <MM>
X
. R. T
X

mX
1x1 = x 0,08206 x (39,50 + 273)
32
32
mX = = 1,25 g
25,64
∆P
Item b) Cálculo da pressão de vapor da solução de sulfanilamida (i=1/solução molecular): =

<MM>solvente nsoluto
x xi
1000 msolvente
kg
1,25
400 − p 58 1,72
= x x1
400 1000 0,100

p = 398,5 mmHg

285
1600 FÍSICO-QUÍMICA APLICADA EXERCÍCIOS COMENTADOS - IME – ITA – OLIMPÍADA

Questão 596 – (ITA) Motores de automóveis refrigerados a água normalmente apresentam problemas de
funcionamento em razões muito frias. Um desses problemas está relacionado ao congelamento da água de
refrigeração do motor. Admitindo que não ocorra corrosão, qual das ações abaixo garantiria o maior abaixamento de
temperatura de início do congelamento da água utilizada num sistema de refrigeração com capacidade de quatro litros
de água? Justifique.
a) Adição de 1 mol de glicerina na água.
b) Adição de 1 mol de sulfato de sódio na água.
c) Adição de 1 mol de nitrato de sódio na água.

Resolução: A ação que irá garantir maior abaixamento de temperatura (crioscopia) será aquela que apresentar maior
número de partículas dispersas (n), o que irá ocasionar maior efeito coligativo.

a) Glicerina: solução molecular → n = 1

b) Sulfato de sódio: Na2SO4

Cálculo do número (n) de íons presenets no sulfato de sódio: Na2SO4(aq) → 2 Na+(aq) + SO4-2(aq)
1 mol de Na2SO4(aq) ---------- 2 mol de Na+(aq) ---------- 1 mol de SO4-2(aq)
n=1+2=3

c) Nitrato de sódio: NaNO3.


NaNO3(aq) → Na+(aq) + NO3-(aq)
1 mol de Na2SO4(aq) ---------- 1 mol de Na+(aq) ----------------------- 1 mol de NO3-(aq)
n=1+1=2

Conclusão: O efeito crioscópico que irá apresentar maior abaixamento de temperatura de congelamento da água será
o sulfato de sódio (n = 3).

Questão 597 – (ITA) Deseja-se desdobrar dois litros de uma solução aquosa 0,15 mol.L-1 de cloreto de sódio em: um
litro de água pura e um litro de solução 0,30 mol.L-1 de cloreto de sódio, isto sem haver afastamento de temperatura
ambiente e sem usar destilação, mas utilizando apenas os princípios envolvidos no fenômeno da osmose. Explique
como isto poderia ser feito e que tipo de trabalho estaria em jogo. Ilustre sua resposta com uma figura que deixe claro
a aparelhagem a ser utilizada.

Resolução:

Exercendo sobre a solução de 0,15 mol.L-1 de cloreto de sódio uma pressão maior que a pressão osmótica vai haver
passagem da água da solução para a água pura (processo de osmose reversa). Quando o volume da solução reduzir
para um litro o processo vai parar, ficando um litro de solução 0,30 mol.L-1 de cloreto de sódio no compartimento (I).
Já no compartimento (II) vai haver três litros de água pura. Com qualquer aparelho de medição de volume, separa-se
um litro de água pura dos dois litros de água iniciais.

286
1600 FÍSICO-QUÍMICA APLICADA EXERCÍCIOS COMENTADOS - IME – ITA – OLIMPÍADA

Questão 598 - (ITA) Prepara-se, a 25°C, uma solução por meio da mistura de 25 mL de n-pentano e 45 mL de n-
hexano. Dados: massa específica do n-pentano = 0,63 g.mL-1; massa específica do n-hexano = 0,66 g.mL-1; pressão
de vapor do n-pentano = 511 torr; pressão de vapor do n-hexano = 150 torr. Determine os seguintes valores,
mostrando os cálculos efetuados:
a) Fração molar do n-pentano na solução.
b) Pressão de vapor da solução.
c) Fração molar do n-pentano no vapor em equilíbrio com a solução.

Resolução: Cálculo da massa de n-pentano (C5H12), a partir da massa específica (μ): Massa = μ x Volume = 0,63 x
25 = 15,75 g
mC5H12 15,75
Cálculo do número de mol de n-pentano (C5H12): nC5 H12 = = = 0,22 mol
<MM>C5H12 72

Cálculo da massa de n-hexano (C6H14), a partir da massa específica: Massa = μ x Volume = 0,66 x 45 = 29,70 g
𝑚𝐶 𝐻14 29,7
Cálculo do número de mol de n-hexano (C6H14): 𝑛𝐶6 𝐻14 = <𝑀𝑀>6 = 86
= 0,34 𝑚𝑜𝑙
𝐶6 𝐻14

𝑛𝐶5𝐻12 0,22 0,22


a) Cálculo da fração molar do n-pentano (XC5H12): 𝑋𝐶5 𝐻12 = 𝑛 = = 0,56 = 0,39
𝐶5 𝐻12 + 𝑛𝐶6 𝐻14 0,22+0,34

b) Cálculo da fração molar do n-hexano: X C5 H12 + X C6 H14 = 1


X C6 H14 = 1 − 0,39 = 0,61

Cálculo da pressão de vapor da solução: Psolução = PC5H12 + PC6H14 = (XC5H12 x P°pentano) + (XC6H14 x P°hexano) = (0,39 x
51) + (0,61 x 150) = 290,79 torr
vapor 0
c) Cálculo da fração molar do pentano na solução: Psolução = X pentano x Ppentano

0
X ′ . Psolução = X pentano x Ppentano

X ′ . 290,79 = 0,39 x 511

X ′ = 0,68

Questão 599 – (IME) Certo composto β é produzido através da reação:

Dois bécheres são colocados em um sistema fechado, mantido a 40°C. O bécher da esquerda contém 200 mL de
etanol, enquanto o da direita contém uma solução de 500 mg do composto β em 200 mL de etanol, conforme a
representação a seguir:

Assinale a alternativa que melhor representa os níveis de líquido nos bécheres três horas após o início do
confinamento.

287
1600 FÍSICO-QUÍMICA APLICADA EXERCÍCIOS COMENTADOS - IME – ITA – OLIMPÍADA

Resposta: Alternativa E. O bécher da direita contém uma solução de soluto não-volátil. Por isso sua pressão de
vapor é menor que a do frasco da esquerda, e a vaporização do frasco à esquerda é a mais fácil. Em consequência,
haverá transferência do etanol para o frasco que contém a solução. O nível do líquido do frasco à esquerda será
menor.

Questão 600 – A adição de 100 g de um composto X a uma quantidade de 750 g de tetracloreto de carbono provocou
um abaixamento crioscópico de aproximadamente 10,50 K. Determine a massa molar do composto X. Dado: constante
crioscópica do CCl4 = 30°C.kg.mol-1. Resposta: < 𝐌𝐌 >𝐱 = 𝟑𝟖𝟎, 𝟗𝟓 𝐠. 𝐦𝐨𝐥−𝟏

Resolução: Dados do problema: Soluto: X; Solvente: tetracloreto de carbono (CCl4); Fator de Van’t Hoff: i = 1 (solução
molecular)

Cálculo da massa molar do soluto desconhecido, a partir da propriedade coligativa crioscópica: ∆𝑇𝑐 = 𝐾𝑐 𝑥 𝑊 𝑥 𝑖

100
(< MM > )
x
10,50 = 30 x x 1,0
0,750

< MM >x = 380,95 g. mol−1

Questão 601 – O benzeno puro congela a 5,45ºC. Verificou-se que uma solução contendo 7,24 g de C2H2Cl4 em 115
g de benzeno congela a 3,55ºC. Qual o valor da constante molal de abaixamento do ponto de congelamento do
benzeno?

Resolução: Cálculo da constante crioscópica molal (Kc): ∆Tc = K c x W x i

Tsolução − Tsolvente = K c x W x i

7,24
(3,55 − 5,45)°C = K c x ( 168 ) x 1
0,115

K c = −5,08 kg. °C. mol−1

Questão 602 – Foram dissolvidos uma quantidade de 34,2 g de sacarose em uma quantidade de 180 g de água.
Sabendo que a temperatura experimental a pressão de vapor da água pura é de aproximadamente igual a 17,5 mmHg,
determine:
a) O abaixamento relativo da pressão de vapor;
b) O abaixamento absoluto da pressão de vapor;
c) A pressão de vapor da solução aquosa de sacarose.

288
1600 FÍSICO-QUÍMICA APLICADA EXERCÍCIOS COMENTADOS - IME – ITA – OLIMPÍADA

∆P <MM>água nsacarose
Resolução: Item a) Cálculo do abaixamento relativo da pressão de vapor: P°
= 1000
x água xi
mkg

34,2
∆P 18
= x 342 x 1 = 10−2
P° 1000 0,180

Cálculo do abaixamento absoluto da pressão de vapor (∆p): Através do item a, o abaixamento relativo da pressão de
∆p
vapor é igual a 10-2, então o abaixamento absoluto da pressão de vapor será igual a: 17,5 mmHg = 10−2

∆p = 0,175 mmHg

c) Cálculo da pressão de vapor da solução de sacarose (p): Através do item b, o abaixamento absoluto da pressão de
vapor é igual a 0,175 mmHg, então a pressão de vapor da solução de sacarose será igual a: ∆𝑝 = 0,175 𝑚𝑚𝐻𝑔

p − p0 = 0,175 mmHg

p − 17,5 mmHg = 0,175 mmHg

p = 17,67 mmHg

Questão 603 – (OLIMPÍADA NORTE – NORDESTE DE QUÍMICA) A adição de 3 gramas de determinada substância
a 100 gramas de CCl4 eleva o ponto de ebulição deste último de 0,60°C.
a) Calcule o abaixamento de sua temperatura de congelação.
b) Calcule a pressão osmótica da solução a 25°C.
c) Determine a massa molar da substância. Dados para a resolução da questão: Keb (CCl4) = 5,03°C/mol.kg-1;
Kc (CCl4) = 31,8°C/mol.kg-1; densidade do CCl4: d = 1,59 g/cm3, a 25°C; constante dos gases: R = 8,31 J/K.mol; (1
atm = 1,013 x 105 Pa; 1 Pa = 1 N/m2; 1 J = 1 N.m).

Resolução: Item c) Cálculo da massa molar, a partir da propriedade coligativa ebulioscópica com i = 1 (solução
molecular): ∆Teb = K eb x W x i

3,0
0,60 = 5,03 x { < MM >} x 1
0,100

3,0
0,60 = 5,03 x { }
0,100 x < MM >

< MM > = 251,5 g. mol−1


3,0 𝑔

Item a) Cálculo da temperatura do abaixamento crioscópico: ∆𝑇𝑐 = 𝐾𝑐 𝑥 𝑊 𝑥 𝑖 = 31,8 𝑥 { 0,100


251,5
} 𝑥 1 = 3,79°𝐶

Como a temperatura é crioscópica, levaremos em consideração o sinal de negativo, ∆𝑇𝑐 = −3,79°𝐶


𝑚 100
Item b) Cálculo do volume da solução, a partir da densidade do tetracloreto de carbono: 𝑉 = = =
𝑑 1,59
−3
62,89 𝑐𝑚³ (62,89 𝑥 10 𝐿)
𝑚

Cálculo da pressão osmótica: 𝜋 = { 𝑉<𝑀𝑀> } 𝑥 𝑅 𝑥 𝑇 𝑥 [1 + (𝑛 − 1) 𝑥 𝛼]


𝑠𝑜𝑙𝑢çã𝑜

289
1600 FÍSICO-QUÍMICA APLICADA EXERCÍCIOS COMENTADOS - IME – ITA – OLIMPÍADA

3,0 3,0
251,5 251,5
𝜋={ } 𝑥 0,08206 𝑥 (25 + 273) = { } 𝑥 0,08206 𝑥 298 = 4,65 𝑎𝑡𝑚
62,89 𝑥 10−3 62,89 𝑥 10−3

Questão 604 – A dissolução de 3,00 g de uma substância A em 100,0 g de tetracloreto de carbono eleva o ponto de
ebulição do tetracloreto de carbono eleva o ponto de ebulição do tetracloreto de carbono de 0,600°C. Calcular:
a) O abaixamento do ponto de congelamento do solvente;
b) O abaixamento relativo da pressão de vapor do solvente;
c) A massa molar da substância A. Dados eventualmente necessários ao tetracloreto de carbono: K c =
31,8°C.mol.kg-1; Keb = 5,03°C.mol.kg-1; Densidade = 1,5 g.mL-1.

Resolução: Dados do problema: Soluto = Substância A; Solvente = tetracloreto de carbono (CCl4); Fator de Van’t
Hoff: i = 1 (solução molecular). Para a resolução deste problema, será necessário inicialmente calcular o item c, ou
seja, a massa molecular da substância A.

Item c) Cálculo da massa molar da substância A: ∆Teb = K eb x W x i


3,0
(< MM > )
A
0,600 = 5,03 x x1
0,100

< MM >A = 251,5 g. mol−1

Item a) Cálculo do abaixamento do ponto de congelamento do solvente: ∆Tc = K c x W x i

3,0
( )
251,5
∆𝑇𝑐 = 31,8 𝑥 𝑥1
0,100

∆𝑇𝑐 = 3,79°𝐶

∆P <MM>CCl4 nsubstância A
Item b) Cálculo do abaixamento relativo da pressão de vapor do solvente: = x CCl xi
P° 1000 mkg 4
3,0
∆P 154 251,5 1,19 x 10−2
= x x 1 = 0,154 x = 0,154 x 0,119 = 1,84 x 10−2
P° 1000 0,100 0,100

Questão 605 – A uma temperatura de 25°C, as pressões de vapor do benzeno e do tolueno são 12,5 kPa e 3,59 kPa,
respectivamente. Qual a pressão que deverá ser aplicada a uma solução preparada com 60,0 gramas de benzeno e
40,0 gramas de tolueno para que ferva a uma temperatura de 25°C?
m C H6 60,0
Resolução: Cálculo do número de mol de benzeno (C6H6): nC6 H6 = <MM>6 = 78
= 0,769 mol
C 6 H6

mC H8 40,0
Cálculo do número de mol de tolueno (nC7H8): nC7 H8 = <MM>7 = 92
= 0,435 mol
C 7 H8

Cálculo do número de mol total (nTOTAL): nTOTAL = nbenzeno + ntolueno = 0,769 + 0,435 = 1,204 mol

mistura C H C H
Cálculo da pressão de vapor da solução: Psolução 6 6
= Psolução 7 8
+ Psolução = X C6 H6 x PC06 H6 + X C7 H8 x PC07 H8 =
0,768 0,435
(1,204) x 12,50 kPa + (1,204) x 3,59 kPa = 9,28 kPa

290
1600 FÍSICO-QUÍMICA APLICADA EXERCÍCIOS COMENTADOS - IME – ITA – OLIMPÍADA

Questão 606 – O heptano apresenta uma pressão de vapor igual a 105 kPa a uma temperatura igual a 100°C. A
mesma temperatura, o hidrocarboneto octano apresenta uma pressão de vapor igual a 46 kPa. Qual será a pressão
de vapor a uma mistura que compreende 25,0 g de heptano e 35,0 g de octano? Observação: Considere ideal o
comportamento da solução.

Resolução: Cálculo do número de mol de heptano, número de mol do octano e consequentemente do número de mol
total:

mC7 H16 mC7 H16 25,0 35,0


nTotal = nC7 H16 + nC8 H18 = + = + = 0,56 mol
< MM >C7 H16 < MM >C7 H16 100 114

C H 0,25 0,25
Cálculo da pressão de vapor de heptano na solução: Psolução
7 16
= (0,25+ 0,31) x 105 kPa = (0,56) x 105 kPa =
46,87 kPa

C H 0,31
Cálculo da pressão de vapor do octano na solução: Psolução
8 18
= (0,31+0,25) x 47 kPa = 26,02 kPa

mistura C H C H
Cálculo da pressão de vapor da mistura: Psolução 7 16
= Psolução 8 18
+ Psolução = 46,87 kPa + 26,02 kPa =
72,89 kPa

Questão 607 – O ponto de ebulição do tolueno puro é 110,60°C. Uma solução contendo 5,00 g de difenilo, C12H10,
em 100 g de tolueno ferve a 111,68°C. Uma solução contendo 6,00 g de uma substância desconhecida não-volátil,
em 200 g de tolueno, ferve a 112,00°C. Calcular a massa molecular da amostra desconhecida.

Resolução: Dados do problema: Solvente = Tolueno (C7H8); Soluto = difenilo (C12H10) e Fator de Van’t Hoff: i = 1
(solução molecular)

solução solvente
Cálculo da constante ebulioscópica molal: Tebulição − Tebulição = K eb x W x i
5,0
( )
111,68 − 110,60 = K eb x | 154 | x1
0,100

K eb = 3,33°C. kg. mol−1


Cálculo da massa molecular (<MM>) da amostra desconhecida: ∆Teb = K eb x W x i

ebulição ebulição
Tsolução − Tsolvente = K eb x W x i

6,0
(< MM >)
112,0 − 110,60 = 3,33 x x1
0,200

19,98
< MM > = = 71,36 g. mol−1
0,280

Questão 608 – Determine a temperatura de ebulição normal de uma solução que contém 4,7 g de fenol em 500 g de
álcool etílico. Dados: Temperatura de ebulição normal: Teb = 78,4°C; Constante ebulioscópica molal: Keb =
1,2°C.kg.mol-1. Resolução: Dados do problema: Soluto = Fenol (C6H6O); Solvente = álcool etílico (C2H6O) e Fator de
Van’t Hoff: i = 1 (solução molecular).

Cálculo da temperatura de ebulição da solução: ∆Teb = K eb x W x i

ebulição ebulição
Tsolução − Tnormal = K eb x W x i

291
1600 FÍSICO-QUÍMICA APLICADA EXERCÍCIOS COMENTADOS - IME – ITA – OLIMPÍADA

4,70
( )
− 78,40 = 1,20 x 94 x 1
ebulição
Tsolução
0,500
ebulição
Tsolução = 78,40 + 0,12 = 78,52°C

Questão 609 – Determine a massa de naftaleno que dissolvida em éter etílico (42,8 gramas), forma uma solução que
apresenta temperatura de ebulição normal superior em 0,2°C a temperatura de ebulição normal de éter etílico. Dado:
Keb = 2,14°C.kg.mol-1. Resolução: Dados do problema: Soluto = Naftaleno (C10H8); Solvente = éter etílico (C4H10O) e
Fator de Van’t Hoff: i = 1 (solução molecular)

Cálculo da molalidade (W), a partir do efeito ebulioscópico: ∆Teb = K eb x W x i

0,20 = 2,14 x W x 1

0,20
W= = 9,34 x 10−2 mol. kg −1
2,14
nsoluto
Cálculo da massa do soluto: W = (kg)
msolvente

m
(128)
−2
9,34 x 10 =
42,8 x 10−3

m = 0,512 g

Questão 610 - A temperatura de congelamento de uma solução aquosa (Kc da água = 1,86) 0,40 mol.kg -1 de um
sulfato metálico, onde o sal está totalmente dissociado é -3,72°C. Determine o número de oxidação do metal neste
sal? Resolução: Dados do problema: Soluto = sulfato metálico [M2(SO4)a]; Solvente = água.

Equação de dissociação do sulfato metálico: M2(SO4)a(aq) → 2 M+a(aq) + a SO4-2(aq)

Cálculo do número de oxidação do sulfato metálico, considerando 100% dissociado, a partir do efeito coligativo
crioscópico: ∆Tc = K c x W x i

⌊0 − (−3,72)⌋ = 1,86 x 0,40 x {1 + (2 + a − 1) x 1}

3,72
= {1 + (2 + a − 1) x 1}
0,744

a = +3

Questão 611 – (GRILLO) A pressão de vapor do benzeno, a uma temperatura de 60,50°C é de aproximadamente
400 torr. Quando são dissolvidos 19,0 g de um determinado composto orgânico não-volátil em 500 g de benzeno, a
pressão de vapor do benzeno passa a ser de 386 torr. Diante dessa informação, determine a massa molar do
composto desconhecido. Resolução: Dados do problema: Soluto = desconhecido (19,0 g); Solvente = benzeno (0,50
kg) e Fator de Van’t Hoff: i = 1 (solução molecular)
<MM>solvente 78
Cálculo da constante tonoscópica molal (Kt): K t = 1000
= 1000 = 78 x 10−3

∆P <MM>solvente nsoluto
Cálculo da massa molecular (<MM>) do soluto desconhecido: = x xi
P° 1000 mbenzeno
kg
19
386 − 400 78 < MM >soluto
| |= x x1
400 1000 0,500

292
1600 FÍSICO-QUÍMICA APLICADA EXERCÍCIOS COMENTADOS - IME – ITA – OLIMPÍADA

< MM >solvente = 84,68 g. mol−1

Questão 612 – Sabendo que a pressão de vapor do propan-2-ol é 50,0 kPa a 340°C, mas cai a 49,6 kPa quando se
dissolve, em 250g de propan-2-ol, 8,69 g de um composto orgânico desconhecido não volátil. A partir destas
informações, calcule a massa molar do composto. Resolução: Dados do problema: Soluto: desconhecido; Solvente:
propano-2-ol (C3H8O) e Fator de Van’t Hoff: i = 1 (solução molecular)
∆P <MM>solvente nsoluto
Cálculo da massa molar do composto desconhecido: P°
= 1000
x propan−2−ol xi
mkg
8,69
50,0 − 49,6 60 < MM >X
| |= x x1
50,0 1000 0,250

< MM >X = 260,7 g. mol−1

Questão 613 – (GRILLO) Uma pequena amostra de proteína de hemoglobina com uma massa de aproximadamente
0,500 g foi dissolvida em água suficiente para preparar 100,0 mL de solução. A pressão osmótica da solução medida
a uma temperatura ambiente foi de 1,35 mmHg. Calcule a massa molecular da hemoglobina.

Resolução: Dados do problema: Soluto: hemoglobina; Solvente: água e Fator de Van’t Hoff: i = 1 (solução molecular).
0,50
1,35 mmHg <MM>hemoglobina
Cálculo da massa molecular da hemoglobina (<MM>): ={ } x 0,08206 x (25 +
760 mmHg 0,10

273) x 1

1,35 mmHg 5
={ } x 0,08206 x 298
760 mmHg < MM >hemoglobina

12,23
< MM >hemoglobina = = 68833,14 g. mol−1
0,000178

Questão 614 – (GRILLO) Quimicamente, a gelatina é uma proteína de origem animal, um polímero formado de
aminoácidos. Ela é derivada de colágeno, que é uma proteína na grande maioria dos mamíferos tem basicamente a
mesma composição; o colágeno dos peixes e répteis apresentam composição ligeiramente diferente. Considere uma
quantidade de aproximadamente 0,614 g de gelatina que foi dissolvido em água suficiente para preparar 125 mL de
solução. A 25°C, a pressão osmótica da solução encontrada foi 67,20 mmHg. Calcule a massa molecular aparente da
gelatina. Resolução: Dados do problema: Soluto: gelatina; Solvente: água e Fator de Van’t Hoff: i = 1 (solução
molecular)
m

Cálculo da massa molecular (<MM>) da gelatina: π = { V<MM> } x R x T x i


solução

0,614
67,20 mmHg
= { MM > } x 0,08206 x (25 + 273) x 1,0
<
760 mmHg 0,125

< MM > = 1358,47 g. mol−1

Questão 615 – A pressão osmótica de uma solução (1) contendo 32 g.L-1 de um soluto é 8,20 atm a 27°C. Qual a
pressão osmótica a 47°C de uma solução (2) contendo 10 g.L-1 do mesmo soluto no mesmo solvente?
m
C
Resolução: Analisando a solução (1): π1 = { V <MM>
} x R x T1 x i = {<MM>} x R x T1 x i
solução

293
1600 FÍSICO-QUÍMICA APLICADA EXERCÍCIOS COMENTADOS - IME – ITA – OLIMPÍADA

32
Substituindo os dados do problema na equação acima temos: 8,20 = {<MM>} x R x (27 + 273) x i

8,20 x < MM > = 32 x R x (27 + 273) x i

8,20 x < MM > = 9600 x R x i (Equação A)


m
C
Analisando a solução (2): π2 = { V<MM> } x R x T2 x i = {<MM>} x R x T2 x i
solução

C
Substituindo os dados do problema na equação acima temos: π2 = {<MM>} x R x T2 x i

π2 x < MM > = 10 x R x (47 + 273)x i (Equação B)

8,20 x<MM> 9600 x R x i


Dividindo a Equação (A) pela Equação (B): π2 x <MM>
= 10 x R x (47+273)x i

8,20 x < MM >


=3
π2 x < MM >

8,20
π2 = = 2,73 atm
3

Questão 616 – Quando 8,05 g de um composto desconhecido foram dissolvidos em 100 g de benzeno, a pressão de
vapor do benzeno diminui de 100,0 mmHg para 94,8 mmHg a uma temperatura de 30°C. Determine:
a) A fração molar do benzeno;
b) A massa molar do composto desconhecido.

Resolução: Dados do problema: Soluto: composto desconhecido; Solvente: benzeno e Fator de Van’t Hoff: 1 (solução
molecular)

Item b) Determinação da massa molar do composto desconhecido (<MM>X):


<MM>solvente 78
Cálculo da constante tonoscópica (Kt): K t = 1000
= 1000 = 78 x 10−3

∆P <MM>solvente nsoluto
Cálculo da massa molecular do composto A (<MM>X): = x xi
P° 1000 mbenzeno
kg
8,05
94,8 − 100,0 78 < MM >X
| |= x x1
100,0 1000 0,100

627,90
0,052 =
100 x < MM >X

< MM >X = 120,75 g. mol−1


𝑚 100
Item b) Cálculo do número de mol do benzeno: 𝑛𝑏𝑒𝑛𝑧𝑒𝑛𝑜 = <𝑀𝑀>
𝑏𝑒𝑛𝑧𝑒𝑛𝑜
= 78
= 1,28 𝑚𝑜𝑙
𝑏𝑒𝑛𝑧𝑒𝑛𝑜

𝑚 8,05
Cálculo do número de mol do composto desconhecido (X): 𝑛𝑋 = <𝑀𝑀>
𝑋
= 120,75 = 6,7 𝑥 10−2 𝑚𝑜𝑙
𝑋

Cálculo do número de mol total (nbenzeno + nX): nTOTAL = nbenzeno + nX = 1,28 + 0,067 = 1,35 mol

294
1600 FÍSICO-QUÍMICA APLICADA EXERCÍCIOS COMENTADOS - IME – ITA – OLIMPÍADA

nBenzeno 1,28
Cálculo da fração molar (Xbenzeno): X Benzeno = n = ≅ 0,95
Benzeno + nX 1,28+0,067

Questão 617 – A adição de 10,0 g de um composto X a 250 g de naftaleno provocou um abaixamento crioscópico de
10,5 K. Calcule a massa molar do composto X. Dado: Kc = 6,94°C.kg.mol-1.

Resolução: Dados do problema: Soluto: composto X; Solvente: naftaleno (C10H8) e Fator de Van’t Hoff: i = 1 (solução
molecular).

Cálculo da massa molar do composto X: ∆Tc = K c x W x i

10,0
< MM >x
10,5 = 6,94 x { }x 1
0,250

10,0
10,5 = 6,94 x { }x 1
0,250 x < MM >x
69,4
10,5 =
0,250 x < MM >x

< MM >x = 26,44 g. mol−1

Questão 618 – O benzeno puro congela a uma temperatura de aproximadamente igual a 5,45°C. Verificou-se que
uma solução contendo 7,24 g de C2H2Cl2 em 115 g de benzeno congela a uma temperatura de aproximadamente de
3,55°C. Determine o valor da constante molal de abaixamento do ponto de congelamento do benzeno.

Resolução: Dados do problema: Soluto: C2H2Cl2; Solvente: benzeno (C6H6) e Fator de Van’t Hoff: i = 1 (solução
molecular)

Cálculo da massa molar do soluto desconhecido, a partir do efeito crioscópico: ∆Tc = K c x W x i

7,24
3,55 = K c x { 97 } x 1
0,500

K c = 5,47°C. kg. 𝑚𝑜𝑙 −1

Questão 619 – Uma solução igual a 30 g.L-1, de um soluto não-iônico de massa molecular 90, é isotônica de uma
solução de sulfato férrico 20 gramas de soluto em 600 mL de solução. Qual o grau de dissociação do sulfato férrico
na segunda solução?

C 30
Resolução: Analisando a solução molecular (i = 1): πmolecular = {<MM>} x R x T x i = {90} x R x T x 1,0 =
RxT
3

Analisando o sulfato de ferro III: Fe2(SO4)3(aq) → 2 Fe+3(aq) + 3 SO4-2(aq)

Fator de van´t Hoff:


i = 1 + [(3+2) -1].α
i = 1 + 4.α

20
πiônica = { } x R x T x (1 + 4α)
400 x 0,60

295
1600 FÍSICO-QUÍMICA APLICADA EXERCÍCIOS COMENTADOS - IME – ITA – OLIMPÍADA

1
πiônica = { } x R x T x (1 + 4α)
12
RxT 1
Como as pressões osmóticas são iguais (solução isotônica): πmolecular = 3
= {12} x R x T x (1 + 4α) =
πiônica
1 1
= { } x (1 + 4α)
3 12

α = 0,75 (75%)

Questão 620 – 10 gramas de uma substância de massa molecular igual a 266 foram dissolvidos em 500 gramas de
tetracloreto de carbono. Qual é a temperatura de ebulição da solução, sob pressão normal? Dados relacionados ao
tetracloreto de carbono: Temperatura de ebulição = 77°C e Calor latente de ebulição = 46 cal.g-1.

Resolução: Dados do problema: Solvente = tetracloreto de carbono (CCl4); Soluto = substância de massa igual a 10
gramas e massa molecular igual a 266.
R x T2eb
Fator de Van’t Hoff (i) = 1 (solução molecular). Cálculo da constante ebulioscópica (Keb): K eb = 1000 x Lv
, onde:
Keb = constante ebulioscópica molal;
R = constante dos gases;
Te = temperatura absoluta de ebulição do solvente puro;
Lv = calor latente de vaporização do solvente puro.
2
R x Teb
K eb =
1000 x Lv

2 x (77 + 273)²
K eb =
1000 x 46

K eb = 5,33°C. kg. mol−1

Cálculo da variação da temperatura ebulioscópica: ∆Teb = K eb x W x i

10,0
( 266 )
∆Teb = 5,33 x [ ] x 1 = 0,400°C
0,500

Cálculo da temperatura de ebulição da solução: ∆Teb = 0,40°C

T − 77°C = 0,40°C

T = 0,40°C + 77℃ = 77,40°C

Questão 621 - (OArQ) A que temperatura uma solução aquosa 1,00 mol.L-1 de sacarose teria uma pressão osmótica
de 1,00 atm. É razoável a resposta?

Resolução: Cálculo da temperatura da solução através da osmometria. Sendo i é igual a 1, por se tratar de uma
solução molecular, temos:

π = [sacarose] x R x T x i, onde M é a concentração da quantidade de matéria.

1,0 = 1,0 x 0,8206 x T x 1,0

T = 12,19 K

296
1600 FÍSICO-QUÍMICA APLICADA EXERCÍCIOS COMENTADOS - IME – ITA – OLIMPÍADA

Para 12,19 K (-260,81°C) NÃO se trata de temperatura razoável para uma solução aquosa de sacarose.

Questão 622 – Supondo os solutos completamente dissociados, determine a temperatura de congelamento de uma
solução que contém 2 g de hidróxido de sódio e 14,2 g de sulfato de sódio, dissolvidos em 500 g de água. Dado:
constante crioscópica molal da água = 1,86°C.kg.mol-1.

Resolução: Informações colocadas pelo problema: Solvente: água; Soluto (1): hidróxido de sódio (NaOH) e Soluto
(2): sulfato de sódio (Na2SO4)

Analisando a solução de hidróxido de sódio: NaOH(aq) → Na+(aq) + OH-(aq)

2,0 2,0
( 40 ) ( 40 )
∆Tc = K c x W x i = 1,86 x [ ] x {1 + (2 − 1) x 1} = 1,86 x [ ] x {1 + (2 − 1) x 1}
0,500 0,500
= 0,372°C

Analisando a solução de sulfato de sódio: Na2SO4(aq) → 2 Na+(aq) + SO4-2(aq)

14,2
( 142 )
∆Tc = K c x W x i = 1,86 x [ ] x {1 + (3 − 1) x 1} = 1,12°C
0,500
A temperatura de congelamento total será: ∆Ttotal = 0,372°𝐶 + 1,12°𝐶 = 1,492°𝐶. Como se trata de um
processo crioscópica, a temperatura ficará negativa, ou seja, ∆Tc = - 1,49°C.

Questão 623 - (OLIMPÍADA NORTE – NORDESTE DE QUÍMICA)


I.Uma solução preparada a partir de 20,0 g de um soluto não volátil e 154 g de solvente, tetracloreto de carbono, tem
uma pressão de vapor de 504 mmHg a 65°C. Considerando que a pressão de vapor do tetracloreto de carbono é de
531 mmHg, a 65°C, qual será a massa molar aproximado do soluto?
II.As propriedades coligativas dependem da natureza do __________ e da concentração __________. Preencha cada
lacuna com a letra correspondente a opção correta.
a) do soluto
b) do solvente
c) de soluto e do solvente
III.O abaixamento do ponto de concentração de um solvente, provocado pela adição de um soluto não volátil (ΔTc) é
igual ao produto da constante crioscopica do solvente (Kc) pela concentração da solução, expressa em:
a) Molalidade
b) molaridade
c) normalidade
d) osmolaridade
∆P <MM>solvente nsoluto
Resolução: Item I) Cálculo da massa molar do soluto: = x CCl xi
P° 1000 mkg 4

20
531 − 504 154 < MM >soluto
= x( ) x1
531 1000 0,154

3080
< MM >soluto = = 392,16 g. mol−1
7,854

Item II) As propriedades coligativas dependem da natureza do solvente e da concentração da quantidade de matéria
do soluto.

297
1600 FÍSICO-QUÍMICA APLICADA EXERCÍCIOS COMENTADOS - IME – ITA – OLIMPÍADA

nsoluto
Item III) Molalidade (W), representado pela seguinte equação matemática: W =
msolvente
(kg)

Questão 624 - (OArQ) Quando 0,154 gramas de enxofre finamente dividido se fundem com 4,38 gramas de cânfora
(Kc = 40 K.kg.mol-1), o ponto de congelamento deste último baixa 5,47°C. Qual é a fórmula molecular do enxofre, com
base nesta medida crioscópica?

Resolução: Dados do problema: Soluto = enxofre (Sx); Solvente = cânfora e Fator de Van’t Hoff: i = 1 (solução
molecular)

Cálculo da massa molecular do soluto, a partir do efeito crioscópico: ∆Tc = K c x W x i

T solução − T solvente = K c x W x i

0,154
(< MM > )
soluto
5,47 = 40 x [ ]x 1
4,38 x 10−3

5,47 x 4,38 x 10−3 x < MM >soluto = 0,154 x 40

< MM >soluto = 257,11 g. mol−1


257
Cálculo do número de átomos de enxofre: x = 32
≅ 8. Fórmula molecular: S8.

Questão 625 – Quando 2,25 g de um composto desconhecido se dissolvem em uma quantidade de 1,50 x 10² g de
cicloexano, o ponto de ebulição do cicloexano aumenta 0,481°C. Através destas informações, determine a massa
molar do composto desconhecido. Dado: constante ebulioscópica molal do cicloexano (Keb): 2,79°C.kg.mol-1.

Resolução: Dados do problema: Soluto: composto desconhecido; Solvente: cicloexano e Fator de Van’t Hoff: 1
(solução molecular). Cálculo da massa molar (<MM>) do composto desconhecido, a partir do efeito ebulioscópico:
∆Teb = K eb x W x i

T solução − T solvente = K eb x W x i

2,25
( )
< MM >soluto
0,481 = 2,79 x [ ] x1
0,150

0,481 2,25
=[ ]
2,79 0,150 x < MM >soluto

< MM >soluto = 87,01 g. mol−1

298
1600 FÍSICO-QUÍMICA APLICADA EXERCÍCIOS COMENTADOS - IME – ITA – OLIMPÍADA

Questão 626 – Qual será o ponto de ebulição normal de uma solução que contém 111 g de cloreto de cálcio em 1000
g de água, admitindo a dissociação completa do sal? Dado: constante ebulioscópica molal da água = +
0,512°C.kg.mol-1.

Resolução: Dados do problema: Soluto: Cloreto de cálcio (CaCl2); Solvente: Água (H2O).

Cálculo do fator de Van´t Hoff (i) com grau de dissociação igual a 100%: CaCl2(aq) → Ca+2(aq) + 2 Cl-1(aq)

i = 1 + (n – 1) x α
i = 1 + [(2 + 1) – 1] x 1
i=2+1=3

Cálculo da Temperatura de ebulição da solução: ∆Teb = K eb x W x i

T solução − T solvente = K eb x W x i

111
(111)
solução
T − 100 = 0,512 x [ ] x3
1,0

T solução = 100 + (0,512 x 1 x 3) = 100 + 1,536 = 101,54°C

Questão 627 – Qual será o ponto de congelamento normal de uma solução que contém 58,5 g de cloreto sódio em
1000 g de água, admitindo a dissociação completa do sal? Dado: constante crioscópica molal da água = + 1,86
°C.kg.mol-1.

Resolução: Dados do problema: Soluto: Cloreto de sódio (NaCl) e Solvente: Água.

Considerando que a dissociação (α) seja igual a 100%, temos: NaCl(aq) → Na+(aq) + Cl-1(aq)

i = 1 + (n – 1).α
i = 1 + [(2 - 1) x 1]
i=1+1
i=2

Cálculo da temperatura de congelamento da solução: T solução − T solvente = K c x W x i


58,5
( )
solução 58,5
T − 0 = 1,86 x x2
1,0

58,5
T solução = 1,86 x x 2 = 1,86 x 1 x 2 = 3,76°C
58,5

Questão 628 – 12,0 gramas de uma determinada substância X, dissolvida em 500 gramas de água, sob pressão
normal, entram em ebulição a uma temperatura de aproximadamente igual a 100,12°C (constante ebulioscópica da
água = 0,52 °C.kg.mol-1). A partir destas informações, determine a massa molecular de X.

Resolução: Dados do problema: Soluto = substância X; Solvente = água e levando em consideração que a solução
seja molecular (Fator de Van’t Hoff: i = 1).

Cálculo da massa molar do soluto X, a partir do efeito ebulioscópico: T solução − T solvente = K eb x W x i

12
(< MM > )
x
100,12 − 100,0 = 0,52 x x1
0,500

299
1600 FÍSICO-QUÍMICA APLICADA EXERCÍCIOS COMENTADOS - IME – ITA – OLIMPÍADA

12
0,12 = 0,52 x
0,500 x < MM >x

0,06 x < MM >x = 0,52 x 12

< MM >x = 104 g. mol−1

Questão 629 – Determine a massa molar e também a fórmula molecular de um composto não dissociável, cuja fórmula
empírica é a seguinte: C4H2N, se 3,84 gramas deste determinado composto foi dissolvido em 500 gramas de benzeno,
provocando um abaixamento do ponto de congelamento igual a 0,307°C.

Resolução: Dados do problema: Soluto = composto não-dissociável (C4H2N); Solvente = benzeno (C6H6); Fator de
Van’t Hoff: i = 1 (solução molecular) e Constante crioscópica do benzeno: Kc = 5,12 °C.kg.mol-1

Cálculo da massa molar (<MM>) do soluto, a partir do efeito crioscópico: ∆Tc = K c x W x i

T solução − T solvente = K c x W x i
3,84
(< MM > )
x
(0,307 − 0) = 5,12 x x1
0,500

0,1535 x < MM >x = 19,66


19,66
< MM >x = = 128,08 g. mol−1
0,1535

Cálculo da fórmula molecular: (C4H2N)n = 128,08


(48 + 2 + 14) x n = 128,08
n = 2 [Fórmula molecular: (C4H2N)n → (C4H2N)2 → C8H4N2].

Questão 630 – Determine o ponto de congelamento e o ponto de ebulição de uma solução que contém 6,50 g de
etilenoglicol, composto este muito utilizado como um anticongelante em automóveis, em 200 g de água. Informação
para a resolução do problema: constante ebulioscópica molal da água: Keb = 0,51°C.kg.mol-1 e Constante crioscópica
molal da água: Kc = 1,86°C.kg.mol-1.

Resolução: Dados do problema: Soluto = etilenoglicol (C2H6O2); Solvente = água (H2O) e Fator de Van’t Hoff = 1
(solução molecular)

∆Tc = K c x W x i

6,50
( ) 6,50
∆Tc = 1,86 x [ 50 ] x 1 = 1,86 x = 1,21°C
0,200 10

∆Teb = K c x W x i

6,50
( ) 6,50
∆Teb = 0,51 x [ 50 ] = 0,51 x = 0,33°C
0,200 10
Questão 631 – Calcular a pressão osmótica de uma solução de sacarose, C12H22O11, a 30°C, a 5% (m/m), com
densidade de 1,017 g.mL-1.

Resolução: Cálculo da concentração comum (g.L-1): C = 10 x d x (%) = 10 x (1,017) x 5 = 50,85 g.L-1.

300
1600 FÍSICO-QUÍMICA APLICADA EXERCÍCIOS COMENTADOS - IME – ITA – OLIMPÍADA

Csacarose 50,85
Cálculo da concentração da quantidade de matéria (mol.L-1): [Sacarose] = <MM>sacarose
= 342
=
−1
0,149 mol. L

Cálculo da pressão osmótica (π), considerando a solução molecular (i=1): π = [Sacarose] x R x T x i

π = 0,148 x 0,08206 x (30 + 273) x 1,0 = 3,70 atm

Questão 632 – A solução aquosa da ureia de concentração 30 g.L-1 é isotônica da solução aquosa 0,125 F de um
eletrólito A2B3. Qual o grau de dissociação aparente do eletrólito?

Resolução: Analisando a solução de ureia: Fórmula molecular da ureia: (NH2)2CO (<MM>ureia = 60 g.mol-1); Fator de
Van’t Hoff: i = 1 (solução molecular)

30 1
πureia = xRxTx1= xRxT
60 2

Analisando a solução do eletrólito A2B3: A2B3(aq) → 2 A+3(aq) + 3 B-2(aq)

Cálculo da concentração da quantidade de matéria do A2B3, considerando que o volume da solução seja igual a 1,0
litro:

1 mol de A2B3 ---------- (1mol de e- x 96500 C.mol-1) ---------- 1 F


nA2B3 -------------------------------------------------------------------- 0,125 F
nA2B3 = 0,125 mol

nA2 B3 0,125 mol


[A2 B3 ] = = = 0,125 mol. L−1
Vsolução 1L

Expressão da pressão osmótica do A2B3: πA2 B3 = 0,125 x R x T x {1 + (5 − 1). α}


πA2 B3 = 0,125 x R x T x {1 + 4. α}

1
Como o processo é isotônico (πureia = πA2 B3 ), temos: 2 x R x T = 0,125 x R x T x {1 + 4α}

1
= 0,125 x {1 + 4α}
2
3
α= = 0,75 (75%)
4

Questão 633 – (GRILLO) A pressão de vapor do benzeno, a uma temperatura de 62°C, é de aproximadamente 54
kPa. São adicionados 19,0 g de um composto orgânico não volátil e dissolvido em 500 g de benzeno, ocasionando
uma pressão de 50 kPa. Determine a massa molar deste composto desconhecido.

Resolução: Dados do problema: Soluto = desconhecido (19,0 g); Solvente = benzeno (500 gramas = 0,50 kg) e Fator
de Van’t Hoff: i = 1 (solução molecular)
∆P
Cálculo da massa molecular (<MM>) do soluto desconhecido: P°
= Kt x W x i

∆P < MM >solvente nsoluto


= x benzeno x i
P° 1000 mkg

301
1600 FÍSICO-QUÍMICA APLICADA EXERCÍCIOS COMENTADOS - IME – ITA – OLIMPÍADA

19
50 − 54 78 ( )
< MM >soluto
| |= x x1
54 1000 0,500

19
4000 (< MM > )
soluto
=
4212 0,500

< MM >soluto = 40,0 g. mol−1

Questão 634 - A uma temperatura de 30ºC, a pressão de vapor do benzeno puro é 0,160 atm. Dissolvendo-se 15,0
g de um soluto não volátil em 250 g de benzeno, obtém-se uma solução, cuja pressão de vapor é 0,158 atm. Determine
a massa molar aproximada do soluto.

Resolução: Dados do problema: Soluto = desconhecido (15 gramas); Solvente = benzeno (C6H6) = 250 gramas (0,25
kg) e Fator de Van’t Hoff: i = 1 (solução molecular)
<𝑀𝑀>𝑠𝑜𝑙𝑣𝑒𝑛𝑡𝑒 78
Cálculo da constante tonoscópica molal (Kt): 𝐾𝑡 = = = 78 𝑥 10−3
1000 1000

∆𝑃
Cálculo da massa molecular do soluto desconhecido, a partir do efeito coligativo tonoscópico: =
𝑃°
<𝑀𝑀>𝑠𝑜𝑙𝑣𝑒𝑛𝑡𝑒 𝑛𝑠𝑜𝑙𝑢𝑡𝑜
1000
𝑥 𝑠𝑜𝑙𝑣𝑒𝑛𝑡𝑒 𝑥𝑖
𝑚𝑘𝑔
15
0,158 − 0,160 < 𝑀𝑀 >𝑠𝑜𝑙𝑢𝑡𝑜
| | = 78 𝑥 10−3 𝑥 𝑥1
0,160 0,250

− 0,002 15
| | = 78 𝑥 10−3 𝑥
0,160 0,250 𝑥 < 𝑀𝑀 >𝑠𝑜𝑙𝑢𝑡𝑜

187,2
< 𝑀𝑀 >𝑠𝑜𝑙𝑢𝑡𝑜 = = 374,4 𝑔. 𝑚𝑜𝑙 −1
0,50

Questão 635 – Uma solução foi preparada dissolvendo 0,750 g de uma determinada proteína em 125 mL de água. A
uma temperatura de 4°C, a pressão osmótica corresponde uma elevação de aproximadamente 2,6 mm da solução.
Determine a massa molar da proteína.

Resolução: Cálculo da pressão (P) da coluna: P = ρ x g x h

g kg 1 mL 1L 1 dm3 m
P = 1,00 x1 x −3 x 3
x −3 3
x 9,81 2 x 2,60 x 10−3 m
mL 1000 g 10 L 1 dm 10 m s
Kg
= 25,51 (25,51 Pa)
m x s²
1 atm
Convertendo para atm, temos: P = 25,51Pa x 101325 Pa = 2,52 x 10−3 atm

m 0,750

Equação da pressão osmótica (π): π = { <MM>


} x R x T x i = { <MM> } x 0,08206 x (4 + 273) x 1 =
Vsolução 0,125
0,750
{0,125 x <MM>} x 0,08206 x 277

Igualando as pressões, temos: P = π

0,750
2,52 x 10−3 atm = { } x 0,08206 x 277
0,125 x < MM >

302
1600 FÍSICO-QUÍMICA APLICADA EXERCÍCIOS COMENTADOS - IME – ITA – OLIMPÍADA

< MM > = 54120 g. mol−1

Questão 636 – (ITA) Considere duas soluções aquosas apresentadas a seguir, ambas na mesma temperatura.
Solução I: contém 1,0 milimol de glicose e 2,0 milimol de CaCl2, por quilograma de água.
Solução II: contém apenas Fe2(SO4)3 dissolvido em água.
Supondo soluções ideais e eletrólitos totalmente dissociados, considerando que as duas soluções terão os mesmos
valores para suas propriedades coligativas, determine a quantidade em mmol de Fe 2(SO4)3 da solução II, por
quilograma de água.

água
Resolução: Analisando a solução I – Glicose + Cloreto de Cálcio: ∆TXI = (K X x W x i) +
glicose
água
(K X x W x i)
CaCl2

água nglicose água nCaCl2


∆TXI = K X x x 1 + KX x x {1 + (3 − 1) x 1}
1 1

água nglicose água nCaCl2


∆TXI = K X x + 3 x KX x
1 1

água 10−3 água 2 x 10


−3
∆TXI = KX x + 3 x KX x
1 1
água água água
∆TXI = K X x 10−3 + 6 x 10−3 x K X = 7 x 10−3 x K X

água
Analisando a solução II – Sulfato de ferro III: ∆TXII = K X xWxi

água nFe2 (SO4 )3


∆TXII = K X x xi
msolvente
(kg)
água nFe2 (SO4 )3
∆TXII = K X x x {1 + (5 − 1). α}
1
água nFe2 (SO4 )3 água nFe2 (SO4 )3
∆TXII = K X x x {1 + 4 x 1} = K X x x5
1 1

Conforme o propósito do problema as temperaturas são iguais, logo: ∆TXI = ∆TXII

água água
(7 x 10−3 x K X ) = (5 x K X x nFe2 (SO4 )3 )

água
7 x 10−3 x K X
nFe2 (SO4 )3 = água
5 x KX

nFe2 (SO4 )3 = 1,40 x 10−3 mol (1,4 mmol)

Questão 637 – (ITA)


a) Considerando que a pressão osmótica da sacarose (C12H22O11) a 25°C é igual a 15 atm, calcule a massa da
sacarose para preparar 1,0 litro de sua solução aquosa a temperatura ambiente.
b) Calcule a temperatura do ponto de congelamento de uma solução contendo 5,0 g de glicose (C6H12O6) em
25 g de água. Sabe-se que a constante do ponto de congelamento da água é igual a 1,86°C.kg.mol-1.
c) Determine a fração molar de hidróxido de sódio em uma solução aquosa contendo 50% em massa desta
espécie.

303
1600 FÍSICO-QUÍMICA APLICADA EXERCÍCIOS COMENTADOS - IME – ITA – OLIMPÍADA

Resolução: Item a) Cálculo da massa de sacarose a partir do processo de osmometria: 𝜋 =


𝑚𝑠𝑎𝑐𝑎𝑟𝑜𝑠𝑒
<𝑀𝑀>𝑠𝑎𝑐𝑎𝑟𝑜𝑠𝑒
{ 𝑉𝑠𝑜𝑙𝑢çã𝑜
}𝑥 𝑅 𝑥 𝑇 𝑥 𝑖

Sacarose = solução molecular → i = 1.

𝑚𝑠𝑎𝑐𝑎𝑟𝑜𝑠𝑒
15 = { } 𝑥 0,08206 𝑥 (25 + 273)𝑥 1,0
1,0

𝑚𝑠𝑎𝑐𝑎𝑟𝑜𝑠𝑒 = 209,78 𝑔.
𝑛𝑠𝑜𝑙𝑢𝑡𝑜
Item b) Cálculo da temperatura do ponto de congelamento (Tsolução): ∆𝑇𝑐 = 𝐾𝑡 𝑥 𝑠𝑜𝑙𝑣𝑒𝑛𝑡𝑒 𝑥𝑖
𝑚(𝑘𝑔)

5,0
(180)
∆𝑇𝑐 = 1,86 𝑥 𝑥 1 = 2,06°𝐶
25,0 𝑥 10−3

Sabendo que a variação da temperatura crioscópica é definida como a diferença entre a temperatura do solvente puro
(água a 0°C) e a temperatura da solução, então a temperatura da solução será igual a: ∆𝑇 = 2,06°𝐶

solvente
Tpuro − T solução = 2,06°C

0°C − T solução = 2,06°C

T solução = − 2,06°C

Item c) Dado do problema: 50 g de NaOH para cada 100 g de solução. Cálculo da massa do solvente:

Massa da solução = massa do soluto + massa do solvente


100 g = 50 g + massa do solvente
Massa do solvente = 50 g
m 50
Cálculo do número de mol do soluto e do solvente: nsoluto = <MM>
soluto
= 40 = 1,25 mol e nsolvente =
soluto
msolvente 50
<MM>solvente
= 18 = 2,78 mol

nNaOH nNaOH 1,25


Cálculo da fração molar (X) do hidróxido de sódio: X NaOH = = = = 0,310
nTOTAL nNaOH + nH2O 1,25+2,78

Questão 638 – (OArQ) Suponha que colocamos alguns microorganismos unicelulares em várias soluções aquosas
de cloreto de sódio. Observamos que as células se mantém sem perturbação em NaCl 0,7% em massa, enquanto
que se encolhem em soluções mais concentradas e se expandem em soluções mais diluídas. Supondo que o NaCl
se comporta como um eletrólito ideal 1:1, calcule a pressão osmótica do fluido aquoso dentro das células a 25°C.

Resolução: Para esta situação, a densidade plausível a ser considerada será igual a 1 g.mL-1.

Informação do problema: 0,70 gramas de NaCl por 100 gramas de solução.


1 gramas de solução -------------------- 1 mL de solução
100 gramas de solução ----------------- Vsolução
Vsolução = 100 mL de solução
m m

Cálculo da pressão osmótica (π): π = { V<MM> } x R x T x i = { V<MM> } x R x T x {1 + (n − 1) x α}


solução solução

304
1600 FÍSICO-QUÍMICA APLICADA EXERCÍCIOS COMENTADOS - IME – ITA – OLIMPÍADA

Considerando que o cloreto de sódio apresenta 100% de dissociação (α = 1), temos: NaCl(aq) → Na+(aq) + Cl-(aq)

0,70
58,5
π={ } x 0,08206 x (25 + 273)x {1 + (2 − 1) x 1} = 5,87 atm
0,10

Questão 639 – Considere uma solução aquosa composta por 480 g de glicose, sendo a glicose com fórmula molecular
igual a C6H12O6 e 700 g de água, com esta solução sendo processada a uma temperatura ambiente. Sabendo que
máxima de vapor da água nesta temperatura ambiente é igual a 22 mmHg, calcule:
a) O valor do abaixamento relativo da pressão máxima de vapor;
b) O valor do abaixamento absoluto da pressão máxima de vapor;
c) O valor da pressão máxima de vapor da solução.
<MM>solvente 18
Resolução: Item a) Cálculo da constante tonoscópica molal (Kt): K t = 1000
= 1000

480
nsoluto ( )
Cálculo da molalidade (W): W = 180
= 0,700 = 3,81 mol. kg −1
msolvente
(kg)

Cálculo do abaixamento relativo da pressão máxima de vapor (∆p/p°), considerando que a solução seja molecular (i
= 1).

∆P 18
= Kt x W x i = x 3,81 x 1 = 6,86 x 10−2
P° 1000
Item b) Cálculo do abaixamento absoluto da pressão máxima de vapor (∆p), a partir da pressão de vapor do solvente
∆P
puro: P° = 6,86 x 10−2

∆P
= 6,86 x 10−2
22 mmHg

∆P = 22 mmHg x 6,86 x 10−2 = 1,51 mmHg

Item c) Cálculo da pressão máxima de vapor da solução: ∆P = 1,51 mmHg

P° − Psolução = 1,51 mmHg

22 mmHg − Psolução = 1,51 mmHg

Psolução = 22 mmHg − 1,51 mmHg = 20,49 mmHg

Questão 640 – (ITA) Cinco recipientes contêm cada um 500 gramas de água e, respectivamente, 1,0 grama de um
dos sais, conforme as alternativas abaixo. Os recipientes são resfriados. Pergunta-se: em qual deles o início de
solidificação irá ocorrer em temperatura mais baixa: Dado: constante crioscópica molal (Kc) da água vale
1,86°C.kg.mol-1.
a) LiCl
b) KCl
c) NaBr
d) MgCl2
e) Na2SO4

Resolução: Alternativa A.
Item a) Analisando o cloreto de lítio: LiCl(aq) → Li+(aq) + Cl-(aq)

305
1600 FÍSICO-QUÍMICA APLICADA EXERCÍCIOS COMENTADOS - IME – ITA – OLIMPÍADA

1,0⁄
42,5 1,0
∆Tc = K c x W x i = 1,86 x ( ) x [1 + (2 − 1) x 1] = 1,86 x ( ) x [1 + 1]
0,500 42,5 x 0,500

1,0
∆Tc = 1,86 x ( ) x 2 = 0,175°C
21,25
Item b) Analisando o cloreto de potássio: KCl(aq) → K+(aq) + Cl-(aq)

1,0⁄
(39,0 + 35,5)
∆Tc = K c x W x i = 1,86 x ( ) x [1 + (2 − 1) x 1]
0,500
1,0⁄
74,5
= 1,86 x ( ) x [1 + (2 − 1) x 1] = 9,98 x 10−2 °C
0,500

Item c) Analisando o brometo de sódio: NaBr(aq) → Na+(aq) + Br-(aq)

1,0⁄ 1,0⁄
(23 + 80) 103) x 2
∆Tc = K c x W x i = 1,86 x ( ) x [1 + (2 − 1) x 1] = 1,86 x (
0,500 0,500
1,0
= 1,86 x ( ) x 2 = 0,0722°C
51,50
Item d) Analisando o cloreto de magnésio: MgCl2(aq) → Mg+2(aq) + 2 Cl-(aq)

1,0⁄ 1,0⁄
(24 + 71) 95) x 3
∆Tc = K c x W x i = 1,86 x ( ) x [1 + (3 − 1) x 1] = 1,86 x (
0,500 0,500
1,0
= 1,86 x ( ) x 3 = 0,117°C
47,50
Item e) Analisando o sulfato de sódio: Na2SO4(aq) → 2 Na+(aq) + 2 SO4-2(aq)

1,0⁄
∆Tc = K c x W x i = 1,86 x ( 142) x [1 + (3 − 1)x 1] = 1,86 x (1,0) x 3 = 0,0786°C
0,500 71

Observação: Considerando que cada temperatura calculada seja negativa, a menor temperatura é o cloreto de lítio
(A).

Questão 641 – (ITA) Considere as afirmações abaixo, todas relativas à pressão de 1 atm:
I. A temperatura de fusão do ácido benzoico puro é 122°C, enquanto que a da água pura é 0°C
II. A temperatura de ebulição de uma solução aquosa 1,00 mol.L-1 de sulfato de cobre é maior do que a de uma
solução aquosa 0,10 mol.L-1 deste mesmo sal
III. A temperatura de ebulição de uma solução aquosa saturada em cloreto de sódio é maior do que a da água
pura
IV. A temperatura de ebulição do etanol puro é 78,4°C, enquanto que a de uma solução alcoólica 10% (m/m)
em água é 78,2°C. Das diferenças apresentadas em cada das afirmações acima, está(ão) relacionada(s) com
propriedades coligativas:
a) Apenas I e III
b) Apenas I
c) Apenas II e III
d) Apenas II e IV

306
1600 FÍSICO-QUÍMICA APLICADA EXERCÍCIOS COMENTADOS - IME – ITA – OLIMPÍADA

e) Apenas III e IV

Resolução: Alternativa C.

Os efeitos coligativos são propriedades físico-químicas das soluções que depende da quantidade das partículas de
soluto não volátil dissolvida em um determinado solvente, não dependendo da natureza do soluto. Logo, as afirmações
II e III estão diretamente relacionadas as propriedades coligativas.

Questão 642 – (IME) Admita que uma solução aquosa 0,0400 mol x L-1 de ácido tricloroacético congele a – 0,1395°C.
Considere ainda que a constante de abaixamento do ponto de congelamento (Kc) da água seja 1,860°C.Kg.mol-1 e
que 1,00 L de solução contenha 1,00 Kg de solvente. O valor da constante de dissociação (Ka) do ácido tricloroacético
será:
a) 4,90 x 10-7
b) 3,28 x 10-5
c) 7,66 x 10-3
d) 1,36 x 10-2
e) 2,45 x 10-1

Resolução: Alternativa E. A questão afirma que a molalidade é igual a concentração da quantidade de matéria e
que o efeito coligativo é o da crioscopia. Cálculo do fato de Van’t Hoff (i): ∆Tc = K c x W x i
∆Tc |− 0,1395|
i= = = 1,875
K c x W 1,86 x 0,0400
O tricloroacético é um ácido do tipo monoácido em que o número de íons gerados é igual a dois por molécula que se
ioniza.
i = 1 + (n − 1). α
1,875 = 1 + (2 − 1). α
α = 0,875 (87,50 %)
[ácido].α² 0,0400 x (0,8750)² 0,030625
Cálculo da constante do ácido tricloroacético (Ka): K a = 1− α
= 1− 0,8750
= 0,125
= 0,245

Questão 643 – 6,0 gramas de glicose foi adicionada em uma mistura contendo dois líquidos distintos, formando uma
mistura homogênea. O primeiro líquido refere-se ao etanol, apresentando uma massa igual a 252 gramas e o segundo
líquido é a água, com uma massa igual a 180 gramas. A partir desses dados importantes, assinale a pressão de vapor
da solução total a 25°C. Informação para a resolução do problema: pressão de vapor da água pura = 24,0 mmHg a
25°C; pressão de vapor do etanol = 59,0 mmHg a 25°C.
a) 30 mmHg
b) 36,3 mmHg
c) 39 mmHg
d) 40,7 mmHg
e) 45,9 mmHg

Resolução: Alternativa B.
180 g
Cálculo do número de mol de água: nH2 O = 18 g.mol−1 = 10 mol

252 g
Cálculo do número de mol de etanol: nC2 H6 O = 46 g.mol−1 = 5,48 mol

Cálculo do número de mol total: ntotal = nágua + nálcool + nglicose = 10 mol + 5,58 mol + 3,33 x 10-2 mol = 15,51 mol

307
1600 FÍSICO-QUÍMICA APLICADA EXERCÍCIOS COMENTADOS - IME – ITA – OLIMPÍADA

5,48 mol 10 mol


Cálculo da fração molar do etanol e da fração molar da água: X C2 H6 O = 15,51 mol = 0,353 e X H2 O = 15,51 mol =
0,645

Cálculo da pressão total (PTOTAL): PTOTAL = P° x XH2O + P° x XC2H6O = (24,00 x 0,645) + (59,00 x 0,353) = 36,31 mmHg

Questão 644 – (ITA) Que concentração molar do soluto deve ter uma solução aquosa de BaCl 2 para que o
abaixamento crioscópico seja praticamente igual o mesmo que o observado na solução aquosa 0,030 mol.L-1 de NaCl?
a) 0,015
b) 0,020
c) 0,030
d) 0,045
e) 0,060

BaCl2
Resolução: Alternativa B. Informação importante do problema: ∆Tc = ∆TcNaCl

Analisando o cloreto de bário: BaCl2(aq) → Ba+2(aq) + 2 Cl-(aq)


nBaCl2 nBaCl2
∆Tc = K c x W x i = K c x ( H O
) x [1 + (3 − 1) x α] = K c x ( H O
) x [1 + 2α]
mkg2 mkg2

Analisando o cloreto de sódio: NaCl (aq) → Na+ (aq) + Cl-(aq)


nNaCl nNaCl
∆Tc = K c x W x i = K c x ( H O
) x [1 + (2 − 1) x α] = K c x ( H O
) x [1 + α]
mkg2 mkg2

Levando em consideração que ambos os sais (BaCl2 e NaCl) apresentam dissociação igual a 100%, temos:
BaCl
∆Tc 2 = ∆TcNaCl

nBaCl2 nNaCl
Kc x ( H O
) x [1 + 2α] = K c x ( H O
) x [1 + α]
mkg2 mkg2

Considerando que o grau de dissociação de ambos os sais sejam iguais a 100%, temos: 3 x nBaCl2 = 2 x nNaCl

3 x [BaCl2 ] x Vsolução = 2 x [NaCl] x Vsolução

Considerando que os volumes são iguais, temos: 3 x [BaCl2 ] = 2 x [NaCl]


3 x [BaCl2 ] = 2 x (0,030)

[BaCl2 ] = 0,020 mol. L−1

Questão 645 – (ITA) Qual das soluções abaixo deve ter maior pressão osmótica?
a) 0,010 mol.L-1 de ácido acético
b) 0,010 mol.L-1 de cloreto de etila
c) 0,010 mol.L-1 de ácido clorídrico
d) 0,010 mol.L-1 de cloreto de rubídio
e) 0,010 mol.L-1 de cloreto de magnésio

Resolução: Alternativa E.

Analisando a solução de ácido acético (CH3-COOH), com fator de Van’t Hoff igual a 1: π1 = 0,010 x R x T x 1

Analisando o cloreto de etila (CH3 – CH2 - Cl), com fator de Van’t Hoff igual a 1: π2 = 0,010 x R x T x 1

308
1600 FÍSICO-QUÍMICA APLICADA EXERCÍCIOS COMENTADOS - IME – ITA – OLIMPÍADA

Analisando o ácido acético (H3C - COOH), com fator de Van’t Hoff igual a 1: π3 = 0,010 x R x T x 1

Analisando o ácido clorídrico (HCl): HCl(aq) → H+(aq) + Cl-(aq)

π4 = M x R x T x i = M x R x T x {1 + (n − 1). α}

π4 = 0,010 x R x T x {1 + (2 − 1). α} = 0,010 x R x T x {1 + α}

Analisando o cloreto de rubídio (RbCl): RbCl(aq) → Rb+(aq) + Cl-(aq)

π5 = M x R x T x i = M x R x T x {1 + (n − 1). α}

π5 = 0,010 x R x T x {1 + (2 − 1). α} = 0,010 x R x T x {1 + α}

Analisando o cloreto de magnésio (MgCl2): MgCl2(aq) → Mg+2(aq) + 2 Cl-(aq)


π6 = M x R x T x i = M x R x T x {1 + (n − 1). α}

π6 = 0,010 x R x T x {1 + (3 − 1). α} = 0,010 x R x T x {1 + 2α}

Através de cada composto analisado, o cloreto de magnésio apresenta maior pressão osmótica.

Questão 646 – (ITA) Uma solução aquosa 0,15 mol.L-1 de um ácido fraco HX é isotônica com uma solução aquosa
0,20 mol.L-1 de glicose. Qual o grau de dissociação, α = {[X-] / [X-] + [HX]}, de HX na solução 0,15 mol.L-1?
a) ¼
b) 1/3
c) ½
d) 2/3
e) 1

Resolução: Alternativa B.

Analisando o ácido HX: HX(aq) → H+(aq) + X-(aq)

πácido = [ácido] x R x T x {1 + (n − 1 ). α}

πácido = 0,15 x R x T x {1 + (2 − 1 ). α} = 0,15 x R x T x (1 + α)

Analisando a solução de glicose: πglicose = [glicose] x R x T x i = 0,20 x R x T

Processo isotônico: πácido = 0,15 x R x T x (1 + α) = 0,20 x R x T = πglicose

0,15 x (1 + α) = 0,20

α = 0,33 (33%)

309
1600 FÍSICO-QUÍMICA APLICADA EXERCÍCIOS COMENTADOS - IME – ITA – OLIMPÍADA

Questão 647 – (ITA) Considere os valores da temperatura de congelação de soluções 1 milimol.L-1 das seguintes
substâncias:
I. Aℓ2(SO4)3
II. Na2B4O7
III. K2Cr2O7
IV. Na2CrO4
V. Aℓ(NO3)3.9H2O
Assinale a alternativa CORRETA relativa à comparação dos valores dessas temperaturas.
a) I < II < V < III < IV
b) I < V < II ≈ III ≈ IV
c) II < III < IV < I < V
d) V < II < III < IV < I
e) V ≈ II < III < IV < I

Resolução: Alternativa B.
As soluções (I, II, III, IV e V) apresentam a mesma concentração da quantidade de matéria e também o mesmo grau
de dissociação (α). Logo, a solução que irá apresentar a maior temperatura de congelamento será aquela que
apresenta a maior incógnita n. Esta incógnita encontra-se presente na equação de van’t Hoff , dada pela seguinte
equação matemática: I = 1 + (n – 1).α

Analisando cada alternativa:


I) Al2(SO4)3(aq) → 2 Al+3(aq) + 3SO4-2(aq); n = 2 + 3 = 5
II) Na2B4O7(aq) → 2 Na+(aq) + (B4O7)-2 (aq); n = 2 + 1 = 3
III) K2Cr2O7(aq) → 2 K+(aq) + (Cr2O7)-2 (aq); n = 2 + 1 = 3
IV) Na2CrO7(aq) → 2 Na+(aq) + (Cr2O7)-2 (aq); n = 2 + 1 = 3
V) Al(NO3)3(aq) → Al (aq) + 3 NO3 (aq); n = 3 + 1 = 4
+3 -

Ordem crescente de temperatura de congelamento: I < V < II ≈ III ≈ IV.

Questão 648 - (ITA) Dois béqueres, X e Y, contêm, respectivamente, volumes iguais de soluções aquosas,
concentrada e diluída de cloreto de sódio, na mesma temperatura. Dois recipientes hermeticamente fechados,
mantidos á mesma temperatura constante, são interconectados por uma válvula, inicialmente fechada, cada qual
contendo um dos béqueres. Aberta a válvula, após o restabelecimento do equilíbrio químico, verifica-se que a pressão
de vapor nos dois béqueres é Pf. Assinale a opção que indica respectivamente as comparações corretas entre os
volumes inicial (VXi) e final (VXf) da solução no béquer X e entre as pressões de vapor inicial (PYi) e final (Pf), no
recipiente que contém o béquer Y.
a) VXi < VXf e PYi = Pf
b) VXi < VXf e PYi > Pf
c) VXi < VXf e PYi < Pf
d) VXi > VXf e PYi > Pf
e) VXi > VXf e PYi < Pf

Resolução: Alternativa D.
X é a solução mais concentrada e menos volátil. Logo, a pressão de vapor é menor que a de Y. Ao abrir a válvula,
haverá transferência de água da solução Y para a solução X, devido à diferença na pressão de vapor. Com isso, a
pressão de vapor de Y diminui até atingir o equilíbrio. Em consequência deste processo em relação a pressão, o
volume de X aumenta (VXi > VXf) e a pressão de Y diminui (PYi > Pf).

310
1600 FÍSICO-QUÍMICA APLICADA EXERCÍCIOS COMENTADOS - IME – ITA – OLIMPÍADA

Questão 649 - (ITA) Uma solução líquida é constituída de 1,2 - dibromoeltileno (C2H2Br2) e 2,3 – dibromopropreno
(C3H4Br2). A 85°C, a concentração do 1,2 – dibromoeltileno nesta solução é igual a 0,40 (mol/mol). Nessa temperatura
as pressões de vapor saturantes do 1,2 – dibromoeltileno e do 2,3 – dibromopropeno puros são, respectivamente,
iguais a 173 mmHg e 127 mmHg. Admitindo que a solução tem comportamento ideal, é correto afirmar que a
concentração em (mol/mol) de 2,3 – dibromopropeno na fase gasosa é igual a:
a) 0,40
b) 0,42
c) 0,48
d) 0,52
e) 0,60

Resolução: Alternativa D.

Analisando a solução líquida de 1,2 - dibromoeltileno (C2H2Br2): p1 = X x p° = 0,60 x 173 mmHg = 69,20 mmHg

Analisando a solução líquida de 2,3 - dibromopropeno (C2H2Br2): X1 + X2 = 1


0,60 + X2 = 1
X2 = 0,40

Cálculo da pressão de vapor (p2) do 1,2 - dibromoeltileno (C2H2Br2) na solução: p2 = X x p° =


0,40 x 127 mmHg = 76,20 mmHg

Cálculo da pressão total de vapor (pT) da solução: ptotal = p1 + p2 = 69,20 mmHg + 76,20 mmHg = 145,40 mmHg
p 76,20
Cálculo da fração molar (X1) do 2,3 - dibromopropeno (C3H4Br2): X = = = 0,52
pT 145,20

Questão 650 - (ITA) Em relação a água pura, é de se esperar que uma solução de 10 gramas de sacarose em 150
gramas de água tenha, respectivamente:
Ponto de ebulição Ponto de solidificação Pressão de Vapor
a) menor maior menor
b) menor menor menor
c) maior menor menor
d) maior menor maior
e) Nenhuma das respostas

Resolução: Alternativa C.
Analisando o ponto de ebulição: Quando se adiciona um determinado soluto não-volátil a um solvente líquido vai
ocorrer um aumento da temperatura de ebulição.
Analisando o ponto de solidificação: O estudo do abaixamento do ponto de congelamento de um solvente diminui
quando se adiciona a ele um soluto não-volátil.
Analisando a pressão de vapor: O estudo do abaixamento de pressão máxima de vapor de um líquido é ocasionado
pela dissolução de um soluto não-volátil. Isso ocorre pelo fato das partículas dispersas constituem uma barreira que
dificulta a movimentação das moléculas do solvente do líquido para a fase gasosa.

Questão 651 – (U.S. NATIONAL CHEMISTRY OLYMPIAD) O interferão é uma proteína solúvel em água. Uma
solução preparada por dissolução de 15,0 mg de interferão em 2,50 mL de H2O apresenta uma pressão osmótica de
5,80 mmHg a 25°C. Qual é a massa molar do interferão?
a) 1.92 × 104 g.mol–1
b) 1.92 × 107 g.mol–1
c) 1.95 × 106 g.mol–1
d) 1.61 × 103 g.mol–1

Resolução: Alternativa A.

311
1600 FÍSICO-QUÍMICA APLICADA EXERCÍCIOS COMENTADOS - IME – ITA – OLIMPÍADA

m
Cálculo da massa molar, a partir do efeito coligativo osmótico: π = <MM>x V xRxTxi
solução
−3
5,80 15 x 10
= x 0,08206 x (25 + 273) x 1
760 < MM > x 2,50 x 10−3

15 x 10−3
7,63 x 10−3 = x 0,08206 x 298
< MM > x 2,50 x 10−3

< MM > = 19230 g. mol−1

Questão 652 – (ITA) A pressão de vapor da água pura é de 23,8 torr a 25°C. São dissolvidos 10,0 gramas de cloreto
de sódio em 100,0 gramas de água pura a 25°C. Assinale a opção que indica o valor do abaixamento da pressão de
vapor da solução, em torr:
a) 22,4
b) 11,2
c) 5,6
d) 2,8
e) 1,4

Resolução: Alternativa E.
Dados do problema: Soluto = cloreto de sódio (NaCl); Solvente = água (H2O)
Cálculo do fator de Van’t Hoff (i), considerando que o grau de dissociação seja igual a 100%: NaCl(aq) → Na+(aq) + Cl-
(aq)

∆P <MM>solvente nsoluto
Cálculo do abaixamento da pressão de vapor: = x xi
P° 1000 metanol
kg
msoluto
∆P < MM >solvente < MM >soluto
= x água
x [1 + (n − 1) x α]
P° 1000 mkg
10
∆P 18 58,5
= x( ) x [1 + (2 − 1) x 1]
23,8 1000 0,10

∆P 360
=
23,8 5850

360 x 23,8 8568


∆P = = = 1,46 torr
5850 5850

Questão 653 – Qual das soluções abaixo apresenta maior grau de dissociação iônica?
a) CaCl2 com fator vant’Hoff igual a 2,5
b) FeCl3 com fator vant’Hoff igual a 3
c) NaCl com fator vant’Hoff igual a 2
d) Na2CO3 com fator vant’Hoff igual a 2,6
e) Al2(SO4)3 com fator vant’Hoff igual a 4,6

Resolução: Alternativa C.
a) Equação química: CaCl2(aq) → Ca+2(aq) + 2 Cl-(aq)

i = 1 + (n – 1).α

2,5 = 1 + (3 – 1).α

α = 0,75 (75%)

312
1600 FÍSICO-QUÍMICA APLICADA EXERCÍCIOS COMENTADOS - IME – ITA – OLIMPÍADA

b) Equação química: FeCl3(aq) → Fe+3(aq) + 3 Cl-(aq)

i = 1 + (n – 1).α

3,0 = 1 + (4 – 1).α

α = 0,67 (67%)
c) Equação química: NaCl (aq) → Na+ (aq) + Cl-(aq)

i = 1 + (n – 1).α

2,0 = 1 + (2 – 1).α

α = 1,0 (100%) – Maior grau de dissociação

d) Equação química: Na2CO3 (aq) → 2 Na+ (aq) + CO3-2(aq)

i = 1 + (n – 1).α

2,6 = 1 + (3 – 1).α

α = 0,80 (80%)

e) Equação química: Al2(SO4)3 (aq) → 2 Al+3 (aq) + 3 SO4-2(aq)

i = 1 + (n – 1).α

4,6 = 1 + (5 – 1).α

α = 0,90 (90%)

Questão 654 – (OLIMPÍADA BRASILEIRA DE QUÍMICA) Considere as seguintes soluções aquosas:


Solução A: contém 0,10 mol de NaCl por 1000 gramas de solvente.
Solução B: contém 0,10 mol de sacarose por 1000 gramas de solvente.
Solução C: contém 0,10 mol de CaCl2 por 1000 gramas de solvente.
Assinale a opção na qual estas soluções estão citadas em ordem crescente de ponto de ebulição.
a) A, B, C
b) A, C, B
c) B, A, C
d) B, C, A
e) C, A, B

Resolução: Alternativa C.
Vamos considerar que o solvente seja a água, apresentando constante ebulioscópica igual a 0,51°C.kg.mol -1 e grau
de dissociação igual a 100%.

Cálculo da temperatura ebulioscópica para a solução A (cloreto de sódio): NaCl(aq) → Na+(aq) + Cl-(aq).

água 0,10
∆Tebulição = K ebul. x W x i = 0,51 x x [1 + (2 − 1) x 1] = 0,102°C
1
água
Cálculo da temperatura ebulioscópica para a solução B (sacarose): ∆Tebulição = K ebul. x W x i =
0,10
0,51 x 1
x 1 = 0,051°C

313
1600 FÍSICO-QUÍMICA APLICADA EXERCÍCIOS COMENTADOS - IME – ITA – OLIMPÍADA

Cálculo da temperatura ebulioscópica para a solução C (cloreto de cálcio): CaCl2(aq) → Ca+2(aq) + 2 Cl-(aq)

água 0,080 0,080


∆Tebulição = K ebul. x W x i = 0,51 x ( ) x [1 + (3 − 1) x 1] = 0,51 x ( ) x [1 + 2]
1 1
= 1,53 x 0,080 = 0,122°C

Logo, a ordem crescente de temperatura de ebulição é dada por: B < A < C.

Questão 655 – (ITA) Temos as seguintes soluções aquosas 1 mol.L-1:


I.ureia, CO(NH2)2;
II.glicerina, C3H8O3;
III.glicose, C6H12O6.
Podemos afirmar que a temperatura de início de ebulição de:
a) I é praticamente três vezes menor que o de III
b) II é praticamente 1,5 vezes maior que o de I
c) III é praticamente duas vezes maior que o de II
d) I, II e III será a mesma
e) Faltam dados para responder

Resolução: Alternativa E. Por mais que três soluções aquosas sejam moleculares, ou seja, apresentam fator de
van’t Hoff igual a um (1), não é possível responder a esta questão, pelo fato de faltar dados para os devidos cálculos.
𝑛𝑠𝑜𝑙𝑢𝑡𝑜
Para o cálculo da temperatura de ebulição é necessário o cálculo da molalidade, 𝑊 = 𝑠𝑜𝑙𝑣𝑒𝑛𝑡𝑒 , e para este
𝑚(𝑘𝑔)
parâmetro não há dados suficientes.
Questão 656 - (ITA) Considere as seguintes soluções diluídas:
I. x mol de sacarose / quilograma de água
II. y mol de cloreto de sódio / quilograma de água
III. z mol de sulfato de magnésio / quilograma de água
IV. w mol de cloreto de magnésio / quilograma de água
Para que nestas quatro soluções, durante o resfriamento, possa começar a aparecer gelo na mesma temperatura,
digamos a -1,3OC, é necessário que, em primeira aproximação, tenhamos:
a) x=y=z=w
b) 1x = 2y = 4z = 4w
c) 1x = 2y = 4z = 3w
d) x/1 = y/2 = z/2 = w/3
e) x/1 = y/2 = z/4 = w/4

Resolução: Alternativa C.
Observação: Para todas as soluções (I, II, III e IV) levaremos em consideração que o grau de dissociação seja igual
a 100%.

Analisando a solução de sacarose: Fator de Van’t Hoff = 1 (solução molecular)

∆Tc = K c x W x i

− 1,3 = K c . X . 1
− 1,3 = K c . X
(−1,3)
X= K (Equação A)
c

Analisando a solução de cloreto de sódio: NaCl(aq) → Na+ (aq) + Cl-(aq)

∆Tc = K c x W x i

− 1,3 = K c . Y . {1 + (2 − 1). 1}

314
1600 FÍSICO-QUÍMICA APLICADA EXERCÍCIOS COMENTADOS - IME – ITA – OLIMPÍADA

− 1,3 = K c . 2Y
(−1,3)
Y = 2.K (Equação B)
c
(−1,3)
Pela equação A, X = Kc
, logo X = 2Y

Analisando a solução de sulfato de magnésio: MgSO4(aq) → Mg+2 (aq) + SO4-2(aq)


∆Tc = K c x W x i

− 1,3 = K c . Z . {1 + (4 − 1). 1}
− 1,3 = K c . 4Z
(−1,3)
Z= (Equação C)
4 . Kc
(−1,3)
Pela equação A, X = , logo X = 4Z
Kc

Analisando a solução de cloreto de magnésio: MgCl2(aq) → Mg+2 (aq) + Cl-(aq)


∆Tc = K c x W x i

− 1,3 = K c . S . {1 + (3 − 1). 1}
− 1,3 = K c . 3S
(−1,3)
S= (Equação D)
3.Kc

(−1,3)
Pela equação A, X = Kc
, logo X = 3S

Questão 657 – (ITA) Qual das opções a seguir contém a substância no estado sólido que, adicionada a 100 mL de
água pura na temperatura de 25°C e em quantidade igual a 0,10 mol, produzirá uma solução aquosa com maior
pressão osmótica?
a) Ag2O
b) Na2O2
c) MgO
d) Ba(OH)2
e) Aℓ(OH)3

Resolução: Alternativa B.

A solução que irá apresentar maior pressão osmótica, será aquela que apresentar maior fator de Van’t Hoff (i).
As alternativas a) óxido de prata, c) óxido de magnésio e e) hidróxido de alumínio, são pouco solúveis, apresentando
um grau de dissociação baixo (α).

Analisando o item b):


Na2O2 + 2 H2O → 2 Na+(aq) + 2 OH-(aq) + H2O2(aq)
1 mol de Na2O2 ---------- 4 mol de (Na+ + OH-)
0,10 mol ------------------ n(Na+ + OH-)
n(Na+ + OH-) = 0,40 mol

Analisando o item d):


Ba(OH)2 → Ba+2(aq) + 2 OH-(aq)
1 mol de Ba(OH)2 ---------- 3 mol de (Ba+2 + OH-)
0,10 mol ---------------------- n(Ba+2 + OH-)
n(Ba+2 + OH-) = 0,30 mol

A partir dos resultados, o peróxido de sódio apresenta maior pressão osmótica.

315
1600 FÍSICO-QUÍMICA APLICADA EXERCÍCIOS COMENTADOS - IME – ITA – OLIMPÍADA

Questão 658 – (ITA) O abaixamento da temperatura de congelamento da água numa solução aquosa com
concentração molal de soluto igual a 0,100 mol/kg é 0,55°C. Sabe-se que a constante crioscópica da água vale
1,86°C.kg.mol-1. Qual das opções abaixo contém a fórmula molecular correta do soluto?
a) (AgNH3)Cl
b) (Pt(NH3)4Cl2)Cl2
c) Na[Al(OH)4]
d) K3[Fe(CN)6]
e) K4[Fe(CN)6]

Resolução: Alternativa B.

Sabendo que o fenômeno crioscópico é dado por: ΔTc = Kc x W x i, para a resolução desta questão será necessário
calcular o fator de Van’t Hoff. Substituindo os dados do problema, o fator de van’t Hoff será: ∆Tc = K c x W x i

0,55 = 1,86 x 0,100 x i

0,55 = 0,186 x i

0,55
i= = 2,96 ≅ 3
0,186

Logo será necessário determinar qual das opções apresenta fator de Van’t Hoff igual a três. Analisando item por item,
temos: i = 1 + (n – 1) x α

Para i = 3 e considerando α = 1 (100%), n será igual a: i = 1 + (n – 1) x α

3 = 1 + (n – 1) x 1
n=3

a) (AgNH3)Cl, Íons: (AgNH3)+1 e Cl-1; n = 1 + 1 = 2


b) (Pt(NH3)4Cl2)Cl2, Íons: [Pt(NH3)4Cl2]+2 e Cl-1; n = 2 + 1 = 3
c) Na[Al(OH)4], Íons: Na+ e [Al(OH)4]-1; n = 1 + 1 = 2
d) K3[Fe(CN)6], Íons: K3+ e [Fe(CN)6]-3; n = 1 + 3 = 4
e) K4[Fe(CN)6], Íons: K4+ e [Fe(CN)6]-4; n = 1 + 4 = 5

Questão 659 – (OLIMPÍADA PARAENSE DE QUÍMICA) 64 gramas de um composto A dissolvido em 546 gramas
de benzeno, dão uma solução cuja pressão de vapor é igual a 70 mmHg, a 20°C. A pressão de vapor do benzeno, a
20°C, é igual a 75 mmHg. Calcule a massa do composto A é:
a) 42 g.mol-1
b) 64 g.mol-1
c) 96 g.mol-1
d) 128 g.mol-1
e) 192 g.mol-1

Resolução: Alternativa D.

Dados do problema: Soluto = Composto A; Solvente = Benzeno; Fator de Van’t Hoff: i = 1 (solução molecular).
<𝑀𝑀>
Cálculo da constante tonoscópica molal (Kt): 𝐾𝑡 = 𝑠𝑜𝑙𝑣𝑒𝑛𝑡𝑒
1000
78
𝐾𝑡 = = 78 𝑥 10−3
1000
∆𝑃
Cálculo da massa molecular (<MM>) do composto A, a partir do efeito tonoscópico: 𝑃°
=
<𝑀𝑀>𝑠𝑜𝑙𝑣𝑒𝑛𝑡𝑒 𝑛𝑠𝑜𝑙𝑢𝑡𝑜
𝑥 𝑏𝑒𝑛𝑧𝑒𝑛𝑜 𝑥𝑖
1000 𝑚𝑘𝑔

316
1600 FÍSICO-QUÍMICA APLICADA EXERCÍCIOS COMENTADOS - IME – ITA – OLIMPÍADA

64
70 − 75 78 ( )
< 𝑀𝑀 >𝑠𝑜𝑙𝑢𝑡𝑜
| |= 𝑥 𝑥1
70 1000 0,546

< 𝑀𝑀 >𝑠𝑜𝑙𝑢𝑡𝑜 = 128 𝑔. 𝑚𝑜𝑙 −1

Questão 660 – (OLIMPÍADA PERUANA DE QUÍMICA) Um químico dissolve 5,00 gramas de um polímero em 100
mL de agua e a pressão osmótica medida é igual a 0,021 atm a 25°C. Qual é a massa molar aproximado do polímero?
a) 7 000 g/mol
b) 16 000 g/mol
c) 23 000 g/mol
d) 58 000 g/mol

Resolução: Alternativa: D. Dados do problema: Soluto = Polímero; Solvente = Água (H2O) e Fator de Van’t Hoff: i =
1 (solução molecular). Cálculo da massa molar do polímero (i = 1), a partir da pressão osmótica:
mpolímero
< MM >polímero
π={ }x R x T x i
Vsolução

5,0
< MM >polímero
0,021 = { } x 0,08206 x (25 + 273) x 1
0,100

< MM >polímero = 58223,52 g, mol−1

Questão 661 – (OLIMPÍADA PERUANA DE QUÍMICA) Determine a temperatura de ebulição de uma solução aquosa
dissolvendo 11,4 gramas de amoníaco (NH3) em 200 g de solvente. A constante ebulioscópica da água é igual a Kb =
0,52°C/m.
a) 98,4°C
b) 99,3°C
c) 101,7°C
d) 102,6°C

Resolução: Alternativa C.

Dados do problema: Soluto = Polímero; Solvente = Água (H2O) e Fator de Van’t Hoff: i = 1 (solução molecular)
mpolímero
<MM>polímero
Cálculo da massa molar do polímero (i = 1), a partir da pressão osmótica: π = { Vsolução
}x R x T x i

5,0
< MM >polímero
0,021 = { } x 0,08206 x (25 + 273) x 1
0,100

< MM >polímero = 58223,52 g, mol−1

317
1600 FÍSICO-QUÍMICA APLICADA EXERCÍCIOS COMENTADOS - IME – ITA – OLIMPÍADA

Questão 662 – Utilizando-se em um radiador de automóvel uma solução aquosa de etilenoglicol (10% em massa), a
temperatura mais baixa que o radiador poderá operar será, aproximadamente:
a) 0,0°C
b) – 2,0°C
c) + 2,0°C
d) – 3,3°C
e) + 3,3°C

Resolução: Alternativa D.

Dados do problema: 10% em massa de etilenoglicol. Esta relação significa que há: 10 g de etilenoglicol / 100 gramas
de solução, ou seja, 90 gramas em solvente (H2O).

Fórmula molecular e massa molar do etilenoglicol: C2H4(OH)2 (62 g.mol-1) e constante crioscópica molal da água (Kc):
Kc = 1,86°C.kg.mol-1

Cálculo da temperatura da solução de etilenoglicol: ∆Tc = K c x W x i

10
(62)
∆Tc = (−1,86) x [ ]x 1
90 x 10−3

10000
Tsolução − Tágua = (−1,86) x [ ]
90 x 62
10000
Tsolução − 0 = (−1,86) x [ ]
5580
10000
Tsolução = −1,86 x [ ] = −3,33°C
5580

Questão 663 – Uma solução de X gramas de glicerina em 250 gramas de água apresenta o mesmo ponto de
solidificação de uma solução que contém 2,5 gramas de sacarose dissolvidas em 125 gramas de água. Sabendo que
a constante crioscópica da água é igual a 1,86°C.mol-1.kg, é possível afirmar que a massa X de glicerina é:
a) 2,6 g
b) 1,3 g
c) 0,50 g
d) 5,4 g
e) 13,0 g

Resolução: Alternativa B.

Analisando a solução de glicerina: C3H8O (<MM> = 92 g.mol-1)

𝑋
𝑔𝑙𝑖𝑐𝑒𝑟𝑖𝑛𝑎
(92)
∆𝑇𝑐 = −1,86 𝑥 [ ]𝑥 1
250 𝑥 10−3

Analisando a solução de sacarose: C12H22O11 ( <MM> = 342 g.mol-1)

2,5
(342)
∆𝑇𝑐𝑠𝑎𝑐𝑎𝑟𝑜𝑠𝑒 = −1,86 𝑥 [ ]𝑥 1
0,125

318
1600 FÍSICO-QUÍMICA APLICADA EXERCÍCIOS COMENTADOS - IME – ITA – OLIMPÍADA

glicerina
Apresentando a mesma temperatura de solidificação, temos: ∆Tc = ∆Tcsacarose

X 2,5
(92) (342)
−1,86 x [ ] x 1 = −1,86 x [ ]x 1
250 x 10−3 0,125

92 x 0,25 x 2,50 57,5


X= = = 1,34 g
0,125 x 342 42,75

Questão 664 – O fator de van’t Hoff para uma solução 0,10 mol.L-1 de nitrato de bário é igual a 2,74. O grau de
dissociação deste sal é igual a:
a) 91,3%
b) 87%
c) 100%
d) 74%
e) 69%

Resolução: Alternativa B. Equação química: Ba(NO3)2(aq) → Ba+2(aq) + 2 NO3-(aq)

Cálculo do grau de dissociação (α): i = [1 + (n – 1) x α]

2,74 = [1 + (3 – 1) x α]
α = 0,87 (87%)

Questão 665 – (GRILLO) Uma solução de cloreto de sódio apresenta um grau de dissociação igual a 85%, entra em
ebulição a uma temperatura sob uma pressão normal igual a 102,5°C. A partir destes dados, assinale a alternativa
que apresenta o valor correto da molalidade, sabendo que a constante ebulioscópica molal da água é igual a
0,50°C.mol-1.kg.
a) 2,70
b) 2,80
c) 2,60
d) 3,00
e) 2,00

Resolução: Alternativa A.

Cálculo da molalidade (W): NaCl(aq) → Na+(aq) + Cl-(aq)

∆Teb = K eb x W x i
solução solução
Teb − Teb = K eb x W x {1 + (n − 1). α}

102,5 − 100 = 0,50 x W x {1 + (2 − 1) x 0,85}


2,50 = 0,50 x W x (1,85)
2,50
W= = 2,70 mol. kg −1
1,85 x 0,50
Questão 666 – (OLIMPÍADA PARAENSE DE QUÍMICA) O fator de Van´t Hoff “i” para o cloreto de cálcio com grau
de dissociação igual a 50% é:
a) 0,80
b) 0,20
c) 2,6
d) 3,0
e) 2,0

319
1600 FÍSICO-QUÍMICA APLICADA EXERCÍCIOS COMENTADOS - IME – ITA – OLIMPÍADA

Resolução: Alternativa D. Equação química: CaCl2(aq) → Ca+2(aq) + 2 Cl-(aq)


i = 1 + (n – 1).α
i = 1 + (3 – 1) x 0,50 = 2,0

Questão 667 – (ITA) Uma amostra de 2x10-2 g de um determinado composto orgânico é dissolvida em 300 mL de
água a 25°C, resultando numa solução de pressão osmótica 0,027 atm. Pode-se afirmar, então, que o composto
orgânico é o (a):
a) ácido etanoico (ácido acético)
b) 1,2-etanodiol (etileno glicol)
c) etanol (álcool etílico)
d) metanodiamida (ureia)
e) tri-fluor-carbono

Resolução: Alternativa D.
O objetivo desta questão do ITA é o cálculo da massa molecular deste composto X a partir do efeito coligativo que se
trata da osmometria. Sabendo que o fator de van´t Hoff para o composto orgânico é igual a 1 e sendo a pressão
osmótica representada pela seguinte equação matemática: π = [X] x R x T
mX
π= xRxT
< MM >X x Vsolução

2,0 x 10−2
0,027 = x 0,08206 x (25 + 273)
< MM >X x 0,300

0,027 x < MM >X x 0,300 = 2,0 x 10−2 x 0,08206 x 298

0,489 𝑔
< MM >X = = 60,37
0,0081 𝑚𝑜𝑙

Pela massa molecular calculada, a molécula orgânica é a ureia (CH4N2O).

320
1600 FÍSICO-QUÍMICA APLICADA EXERCÍCIOS COMENTADOS - IME – ITA – OLIMPÍADA

EXERCÍCIOS DE FIXAÇÃO

Questão 668 – (MESTRE JOÃO ROBERTO DA PACIÊNCIA NABUCO) 1,0 g de cloreto de magnésio é dissolvido
em 500 g de água e a solução é resfriada até a solidificação. Determinar em que temperatura se verifica a solidificação,
sabendo-se que o grau de dissociação do sal é 80%. Resposta: T = -0,10°C

Questão 669 – (MESTRE JOÃO ROBERTO DA PACIÊNCIA NABUCO) Determinar a pressão osmótica desenvolvida
por 500 mL de uma solução 1 normal de ácido sulfúrico a 25°C, sabendo-se que nessa temperatura o grau de
ionização do ácido é 72%. Resposta: π = 29,83 atm

Questão 670 – (MESTRE JOÃO ROBERTO DA PACIÊNCIA NABUCO) Um polímero de fórmula geral (C2H4)n
abaixou o ponto de congelação do benzeno de 0,36°C, quando 1,0 g do mesmo foi dissolvido em 5,0 g de benzeno.
°C
K c = 5,04 mol x kg. Determine o valor de n. Resposta n = 100

Questão 671 – (MESTRE JOÃO ROBERTO DA PACIÊNCIA NABUCO) Determinar o abaixamento relativo e o
abaixamento absoluto da pressão máxima de vapor que 50 gramas de ureia [CO(NH2)2] dissolvidos em 5,0 kg de água
a 30°C, provocando na água pura a essa temperatura, sabendo-se que então a tensão máxima de vapor da água seja
∆𝐩
31,82 torr. Resposta: = 𝟎, 𝟎𝟎𝟑 e ∆𝐩 = 𝟎, 𝟎𝟗𝟓𝟓 𝐭𝐨𝐫𝐫
𝐩𝟎

Questão 672 – (MESTRE JOÃO ROBERTO DA PACIÊNCIA NABUCO) Determine a temperatura de ebulição de
uma solução que contém 156 gramas de benzeno que estão dissolvidos em 555 gramas de acetona, sabendo-se que
sob pressão normal, a acetona entra em ebulição a uma temperatura igual a 57°C. Resposta: ∆𝐓𝐞𝐛 = 𝟒, 𝟑𝟗°𝐂

Questão 673 – (MESTRE JOÃO ROBERTO DA PACIÊNCIA NABUCO) Calcule o ponto de ebulição de uma solução
que contém 24 gramas de soluto de massa molecular igual a 58 em 600 gramas de água, quando a pressão
barométrica é tal que a água pura entra em ebulição a 99,725°C. Resposta: 0,352°C
Questão 674 – (MESTRE JOÃO ROBERTO DA PACIÊNCIA NABUCO) Uma solução contendo 4,50 gramas de um
não – eletrólito dissolvidos em 250 gramas de água congela a uma temperatura igual a -0,372°C. Qual a massa
molecular aproximada do soluto? Resposta: 90 g x mol-1

Questão 675 – (MESTRE JOÃO ROBERTO DA PACIÊNCIA NABUCO) A que temperatura congela uma solução de
metanol a 25% em massa? Resposta: ∆𝐓𝐞𝐛 = −𝟏𝟗, 𝟑𝟕°𝐂

Questão 676 – (MESTRE JOÃO ROBERTO DA PACIÊNCIA NABUCO) Uma solução foi obtida dissolvendo-se 3,75
gramas de um composto orgânico puro em 95 gramas de acetona, numa pressão barométrica tal que o ponto de
ebulição da acetona pura é igual a 55,95°C e o da solução, 56,50°C. Sabendo-se que a constante ebuliométrica da
𝐠
acetona é 1,71 °C x kg x mol-1, determinar a massa molecular do soluto. Resposta: < 𝐌𝐌 > = 𝟏𝟐𝟐, 𝟕𝟑 𝐦𝐨𝐥

Questão 677 – (MESTRE JOÃO ROBERTO DA PACIÊNCIA NABUCO) Qual a pressão osmótica em atm de uma
solução 0,20 N de um diácido 60% ionizado, sendo a temperatura igual a 27°C. Resposta: 5,41 atm.
Questão 678 – (MESTRE JOÃO ROBERTO DA PACIÊNCIA NABUCO) Calcule o número de partículas dispersas
por cada mol de sulfato férrico, 90% dissociado. Resposta: 2,17 x 1024 partículas dispersas.
Questão 679 – (MESTRE JOÃO ROBERTO DA PACIÊNCIA NABUCO) Calcule o número de partículas dispersas
em 200 cm³ de solução 0,60 normal (0,60 N) de fosfato de potássio 80% dissociado. Resposta: 8,16 x 1022 partículas
dispersas.
Questão 680 – (GRILLO) Calcule a energia livre de Gibbs e a entropia da mistura para duas situações: (a) uma
mistura composta por 1,0 mol de nitrogênio gasoso e 1,0 mol de oxigênio gasoso; (b) uma mistura composta por 2
mol de argônio, 1 mol de hélio e 3 mol de gás hidrogênio. A partir das informações apresentadas, levar em
consideração que a pressão (1 bar) e a temperatura esteja constante (25°C). Resposta: (a) ∆Gmistura = -3433,91 J x
mol-1 e ∆Smistura = + 11,52 J x K-1; (b) ∆Gmistura = -15034,96 J x mol-1 e ∆Smistura = + 50,45 J x K-1

321
1600 FÍSICO-QUÍMICA APLICADA EXERCÍCIOS COMENTADOS - IME – ITA – OLIMPÍADA

Questão 681 – (GRILLO) A partir de uma mistura de 20 gramas de etano e 350 gramas de gás oxigênio dentro de
um recipiente com capacidade igual a 5,50 litros e temperatura de 27°C, calcule a pressão total do sistema, a energia
livre de Gibbs da mistura e e a variação da entropia da mistura. A partir das informações apresentadas, levar em
consideração que a pressão (1 bar) e a temperatura esteja constante (25°C). Resposta: ∆Gmistura = -6359,38 J x mol-
1; ∆S
mistura = + 21,20 J x K e pressão total igual a 51,92 atm.
-1

Questão 682 – A solution contains 5,0 g of urea (M2 = 60,05) per 100 g of water. What will be the vapor pressure of
this solution at 25°C? The vapour pressure of purê H2O at this temperature is 23,756 mmHg. Resposta: P = 23,40
mmHg

Questão 683 – A solution composed of 10 g of a nonvolatile organic solute in 100 g of diethyl ether has a vapor
pressure of 426,0 mmHg at 20°C. If the vapor pressure of the pure ether is 442,20 mmHg at the same temperature.
What is the molecular weight of the solute? Resposta: <MM> = 202 g x mol-1

Questão 684 – If 30 g of diphenyl are dissolved in 250 g of benzene, what will be the boiling point of the resulting
solution under atmosphere pressure? Resposta: 82,09°C

Questão 685 – A 0,20 molal aqueous solution of KCl freezes at - 0,680°C. Calculate i and osmotic pressure at )°C.
Assume volume to be that of pure H2O. Resposta: i = 1,83 e π = 8,20 atm

Questão 686 – A 0,40 molal aqueous solution of K2SO4 freezes at – 1,52°C. Assuming that i is constant with
temperature, calculate the vapour pressure at 25°C and the normal boiling point of the solution. Resposta: Psolução =
24,31 mmHg e Tsolução = 100,41°C

Questão 687 – A 0,10 molal solution of a week electrolyte ionizing into two ions freezes at – 0,208°C. Calculate the
degree of dissociation. Resposta: α = 0,1183 (11,83%)
Questão 688 – An aqueous solution contains 20 g of glucose per liters. Assuming the solution to be ideal, calculate
its osmotic pressure at 25°C. Resposta: π = 2,71 atm

Questão 689 – What weight of glycerol would have to be added to 1000 g of water in order to lowerits freezing point
10°C? Resposta: Massa = 494,62 g

Questão 690 – A 2,00 molal HCl solution freezes at -8,86°C. Calculate the apparent percentage of dissociation and
explain your answer. Resposta: α = 0,19 (19%)

Questão 691 – The vapor pressure of a solution containing 13 g of solute in 100 g of H2O at 28°C is 27,371 mmHg.
Calculate the molecular weight of the solute. The vapor pressure of water at this temperature is 28,065 mmHg.
Resposta: 94,63 g x mol-1

Questão 692 – (GRILLO) A 25°C, uma solução contém 25,0 g de uma substância na presença de uma solução com
capacidade de 225 cm³ e, com uma pressão osmótica de 650 Pa. A partir desta informação, determine a massa molar
desta substância. Resposta: <MM> = 4,23 x 105 g x mol-1

Questão 693 – A compound weighing is dissolved in 30,0 g of acetic acid. The freezing point of the solution is found
K x mol
to be 1,50 K below that of the purê solvente. Calculate the molar mass of the compound. K ácido
cr = −3,90 .
kg
𝐠
Resposta: < 𝐌𝐌 >𝐬𝐨𝐥𝐮𝐭𝐨 = 𝟑𝟗, 𝟔𝟗 𝐦𝐨𝐥

Questão 694 – Two aqueous urea solution have osmotic pressures of 2,60 atm and 5,40 atm, respectively, at a certain
temperature. What is the osmotic pressure of a solution prepared by mixing equal volumes of these two solutions at
the same temperature? Resposta: 𝛑𝐦𝐢𝐬𝐭𝐮𝐫𝐚 = 𝟒, 𝟎 𝐚𝐭𝐦

Questão 695 – A adição de 27,0 gramas de um composto desconhecido a 760 gramas de tetracloreto de carbono
baixou em 5,40 K o ponto de congelamento do solvente. Calcule a massa molar do composto. Resposta: <MM> =
197,39 g x mol-1

322
1600 FÍSICO-QUÍMICA APLICADA EXERCÍCIOS COMENTADOS - IME – ITA – OLIMPÍADA

Questão 696 – Determine freezing point of a 0,50 mol x L-1 solution of dichloroscetic acid (Ka = 0,05). Assume density
of the solution to be 1 g x cm-3 and Kcr for water is 1,86. Resposta: -1,26°C.

Questão 697 – (MESTRE JOÃO ROBERTO DA PACIÊNCIA NABUCO) Calcule a pressão máxima de vapor de uma
solução de cloreto de magnésio contendo 19 gramas do mesmo em 360 gramas de água a uma temperatura de 28°C.
A pressão máxima de vapor da água a esta temperatura é igual a 32,0 mmHg e a solução de cloreto de magnésio se
encontra 80% dissociado. Resposta: 𝐏 = 𝟑𝟏, 𝟏𝟕 𝐦𝐦𝐇𝐠

323
1600 FÍSICO-QUÍMICA APLICADA EXERCÍCIOS COMENTADOS - IME – ITA – OLIMPÍADA

Resolução: As equações
químicas balanceadas estão
apresentadas
abaixo:CAPÍTULO V

TERMODINÂMICA &
TERMOQUÍMICA

PROFESSOR ALEXANDRE
VARGAS GRILLO

324
1600 FÍSICO-QUÍMICA APLICADA EXERCÍCIOS COMENTADOS - IME – ITA – OLIMPÍADA

Questão 698 - (IME) O valor experimental para o calor liberado na queima de benzeno líquido a 25ºC, com formação
de dióxido de carbono e água liquida, é 780 kcal/mol. A combustão é feita em uma bomba calorimétrica a volume
constante. Considerando comportamento ideal para os gases formados e R = 2,0 cal/mol.K, determine:
a) O calor padrão de combustão do benzeno a 25ºC;
b) Se o calor calculado no item anterior é maior ou menor quando a água é formada no estado gasoso. Justifique
sua resposta.

Resolução: Através da primeira lei da termodinâmica, temos: ∆U = Q + W


Sabendo que o processo ocorre a volume constante (processo isocórico), logo o trabalho é igual a zero (W = 0).
kcal
∆U = Q + 0 = Q V = − 780
mol
Item a) Equação química da combustão do benzeno líquido, formando água no estado líquido: C6H6(l) + 15/2 O2(g) →
6 CO2(g) + 3 H2O(l)

Cálculo da variação de entalpia: ∆H = ∆U + ∆nRT


∆H = −780000 + (6 − 7,50) x 2,0 x (25 + 273) = −780894 cal (≅ −780,9 kcal)
Item b) Equação química da combustão do benzeno líquido, formando água no estado gasoso: C6H6(l) + 15/2 O2(g) →
6 CO2(g) + 3 H2O(g)

Considerando o comportamento como ideal: ∆H = ∆U + ∆n x R x T


∆H = −780000 + {(9 − 7,50) x 2,0 x (25 + 273)} = −779106,0 cal (≅ − 779,11 kcal)
Questão 699 – (IME) Calcule a mudança de energia interna, em kJ, para a reação de formação de dois mol de SOCl2(g)
a partir de S(g), O2(g) e Cl2(g) a 298 K. Dados: ΔH°f S(g) = 277 kJ.mol-1; ΔH°f SOCl2(g) = -210 kJ.mol-1 e R = 8,314 J.mol-
1.K-1.

Resolução: Equação química: S(g) + ½ O2(g) + Cl2(g) → SOCl2(g).


0 0 0
Cálculo da variação da entalpia padrão (∆H°): ∆Hreação = ∑ Hprodutos − ∑ Hreagentes

0 1 0 1
0
∆Hreação = HSOCl − {1 x HS0 + x HO0 2 + 1 x HCl } = −210,0 − {1 x (277) + x 0 + 1 x 0}
2 2 2 2
kJ
= − 487,0
mol

Cálculo da variação da energia interna (∆U): ∆H = ∆U + ∆n x R x T

− 487000 = ∆U + {(1 − 2,50) x 8,314 x (25 + 273)}


− 487000 = ∆U − {1,50 x 8,314 x 298}
∆U = − 483,28 kJ. 𝑚𝑜𝑙 −1
O problema pede para dois mol de cloreto de sulfurila:

1 mol de SO2Cl2 ---------- (- 483,28) kJ


2 mol de SO2Cl2 ---------- ΔU (kJ.mol-1)
ΔU = - 966,57 kJ

Questão 700 – Sabendo que a reação de combustão completa do etanol formando água no estado vapor, apresenta
energia interna igual a ΔU = - 1373 kJ.mol-1. A partir desta informação, determine a variação da entalpia padrão para
este processo de queima, a T = 25°C.

325
1600 FÍSICO-QUÍMICA APLICADA EXERCÍCIOS COMENTADOS - IME – ITA – OLIMPÍADA

Resolução: Equação química (C2H6O): C2H6O(l) + 3 O2(g) → 2 CO2(g) + 3 H2O(g)

Cálculo da entalpia (∆H): ∆H = ∆U + ∆n x R x T


∆H = ∆U + (nprodutos – nreagentes) x R x T
∆H = -1373000 + {(5 – 3) x (8,314) x (25 + 273)}
∆H = -1373000 + {2 x (8,314) x 298} = -1373000 + 4955,14 = -1368,04 kJ.mol-1
Questão 701 - (GRILLO) Considere a reação de combustão completa do acetileno, representada pela equação
química a seguir: C2H2(g) + O2(g) – 1255 kJ → CO2(g) + H2O(g). Determine a quantidade de energia na forma de calor,
que é liberada a partir de 130 g de acetileno, sabendo que esse processo apresenta rendimento de aproximadamente
igual a 80%.

Resolução: Equação química: C2H2(g) + 5/2 O2(g) – 1255 kJ → 2 CO2(g) + H2O(g)

1 mol de C2H2(g) --------------------- (– 1255 kJ)


26 g de C2H2(g) ----------------------- (– 1255 kJ) x 0,80
130 g de C2H2(g) --------------------- ∆H
∆H = - 5020 kJ
Observação: O sinal negativo é apenas um indicativo para informar que o processo é exotérmico.
Questão 702 – (GRILLO) Determine a entalpia da reação de oxidação do álcool etílico, C2H5OH(l) + ½ O2(g) → C2H4O(l)
+ H2O(l), a partir das reações químicas apresentadas abaixo.

Equação química (1): C2H5OH(l) + 3 O2(g) → 2 CO2(g) + 3 H2O(l) ΔH = -327,6 kcal

Equação química (2): C2H4O(l) + 5/2 O2(g) → 2 CO2(g) + 2 H2O(l) ΔH = -279,0 kcal

Resolução: Para a resolução deste problema será necessário aplicar a Lei de Hess.
C2H5OH(l) + 3 O2(g) → 2 CO2(g) + 3 H2O(l) ΔH = -327,6 kcal

2 CO2(g) + 2 H2O(l) → C2H4O(l) + 5/2 O2(g) ΔH = +279,0 kcal

Somando as equações químicas acima:

C2H5OH(l) + 3 O2(g) → 2 CO2(g) + 3 H2O(l) ΔH = - 327,6 kcal

2 CO2(g) + 2 H2O(l) → C2H4O(l) + 5/2 O2(g) ΔH = + 279,0 kcal +

C2H5OH(l) + (3 – 5/2) O2(g) → C2H4O(l) + H2O(l) ΔH = - 327,6 kcal + 279,0 kcal

C2H5OH(l) + 1/2 O2(g) → C2H4O(l) + H2O(l) ΔH = - 48,6 kcal (processo exotérmico).

Questão 703 – (OLIMPÍADA BRASILEIRA DE QUÍMICA) A partir das entalpias das reações dadas abaixo:

2 C(grafite) + 2 H2(g) → C2H4(g) ∆H° = + 52,0 kJ

C2H4Cl2(g) → Cl2(g) + C2H4(g) ∆H° = + 116,0 kJ

Podemos concluir que a entalpia molar de formação (em kJ.mol-1) do C2H4Cl2(g), será igual a:

a) – 64 kJ.mol-1
b) + 64 kJ.mol-1
c) - 168 kJ.mol-1

326
1600 FÍSICO-QUÍMICA APLICADA EXERCÍCIOS COMENTADOS - IME – ITA – OLIMPÍADA

d) + 168 kJ.mol-1
e) + 220 kJ.mol-1

Resolução: Alternativa A. Invertendo a segunda equação química, temos:

2 C(grafite) + 2 H2(g) → C2H4(g) ∆H° = + 52,0 kJ


Cl2(g) + C2H4(g) → C2H4Cl2(g) ∆H° = - 116,0 kJ +
2 C(grafite) + 2 H2(g) + Cl2(g) → C2H4Cl2(g) ∆H° = + 52,0 + (- 116) = - 64 kJ

Questão 704 – (ITA) Assinale a opção que indica a variação correta de entalpia, em kJ.mol-1, da reação química a
298,15 K e 1 bar, representada pela seguinte equação: C4H10(g) → C4H8(g) + H2(g). Dados eventualmente necessários:
ΔH°f (C4H8(g)) = - 11,4; ΔH°f (CO2(g)) = - 393,5; ΔH°f (H2O (l)) = - 285,8 e ΔH°c (C4H10(g)) = - 2877,6, em que ΔH°f e ΔH°c,
em kJ.mol-1, representam as variações de entalpia de formação e de combustão a 298,15 K e 1 bar, respectivamente:
a) – 3568,3
b) – 2186,9
c) + 2186,9
d) + 125,4
e) + 114,0

Resolução: Alternativa E.
Organizando as informações fornecidas pelo problema, temos:

Reação de formação do buteno: 4 C(grafite) + 4 H2(g) → C4H8(g) ΔH°f = - 11,4 kJ.mol-1

Reação de formação da água: H2(g) + ½ O2(g) → H2O(l) ΔH°f = - 285,8 kJ.mol-1

Reação de formação do dióxido de carbono: C(grafite) + O2(g) → CO2(g) ΔH°f = - 393,5 kJ.mol-1

Reação de combustão completa do gás butano: C4H10(g) + 13/2 O2(g) → 4 CO2(g) + 5 H2O(l) ΔH°c = - 2877,6
kJ.mol-1

Para a determinação do valor da entalpia de desidrogenação do gás butano, temos:

C4H10(g) + 13/2 O2(g) → 4 CO2(g) + 5 H2O(l) ΔH°c = - 2877,6 kJ.mol-1


4 C(grafite) + 4 H2(g) → C4H8(g) ΔH°f = - 11,4 kJ.mol-1
H2O(l) → H2(g) + ½ O2(g) ΔH°f = 5 x (+ 285,8) kJ.mol-1
4 CO2(g) → 4 C(grafite) + 4 O2(g) ΔH°f = 4 x (+ 393,5) kJ.mol-1

Juntando as informações e somando as reações químicas, temos:

C4H10(g) + 13/2 O2(g) → 4 CO2(g) + 5 H2O(l) ΔH° = - 2877,6 kJ.mol-1


4 C(grafite) + 4 H2(g) → C4H8(g) ΔH° = - 11,4 kJ.mol-1
5 H2O(l) → 5 H2(g) + 5/2 O2(g) ΔH° = 5 x (+ 285,8) kJ.mol-1
4 CO2(g) → 4 C(grafite) + 4 O2(g) ΔH° = 4 x (+ 393,5) kJ.mol-1 +
C4H10(g) → C4H8(g) + H2(g) ΔH° = + 114,0 kJ.mol-1

Questão 705 – A hidrazina (H2N – NH2) é muito utilizada como combustível em motores de foguete. Sua reação de
combustão pode ser representada pela equação química balanceada: H2N – NH2(g) + O2(g) → N2(g) + 2 H2O(g). Na
tabela abaixo, estão apresentados os valores das entalpias por mol de ligações químicas. A partir desta informação,
determine:

327
1600 FÍSICO-QUÍMICA APLICADA EXERCÍCIOS COMENTADOS - IME – ITA – OLIMPÍADA

Ligação química Entalpia (kJ.mol-1)


H-H 436
H-O 463
N-N 163
N=N 514
N≡N 946
C-H 415
N-H 391
O=O 498
O-O 134
C=O 724

a) a entalpia de combustão da hidrazina.


b) a variação da energia interna, em kJ.mol-1, a 25°C.

Resolução: Item a ) Cálculo da entalpia de combustão da hidrazina:


Cálculo da variação de entalpia de ligação para os reagentes (∆Hreagentes): H2N – NH2(g) + O2(g) →

Analisando as ligações químicas dos reagentes observa-se as seguintes configurações eletrônicas: quatro ligações
N-H, uma ligação N – N e uma ligação dupla entre os átomos de oxigênio, O = O. Para os reagentes, o processo é
endotérmico, ou seja, o sistema absorve calor para a quebra das ligações químicas.

∆Hreagentes = + [4 x HN−H + 1 x HN−N + 1 x HO=O ] = + [4 x (389) + 1 x (163) + 1 x (498)]

kJ
∆Hreagentes = + [1556 + (163) + (498)] = + 2217
mol

Cálculo da variação de entalpia de ligação para os produtos (∆Hprodutos): → N2(g) + 2 H2O(g)

Analisando as ligações químicas dos produtos, observa-se as seguintes configurações eletrônicas: uma ligação tripla
para o nitrogênio, N ≡ N, duas ligações O-H na molécula de água. Para os produtos, o processo é exotérmico, ou seja,
o sistema perde calor para a formação das novas ligações químicas.

kJ
∆Hprodutos = − [1 x HN≡N + 2 x (2 x HO−H )] = − [1 x 946 + 2 x 2 x 463)] = − 2798
mol

Cálculo da entalpia da reação da combustão da hidrazina: ∆Hreação = ∆Hreagentes + ∆Hprodutos = + 2217 + (- 2798) = - 581
kJ.mol-1

Item b) Equação química: H2N – NH2(g) + O2(g) → N2(g) + 2 H2O(g)

Cálculo da variação da energia interna, à 25C: ∆H = ∆U + ∆n x R x T = ∆H – {(nprodutos – nreagentes) x R x T}

∆H = - 581000 – {(3 – 2) x 8,314 x 298} = -583,48 kJ.mol-1

328
1600 FÍSICO-QUÍMICA APLICADA EXERCÍCIOS COMENTADOS - IME – ITA – OLIMPÍADA

Questão 706 – Calcule a temperatura adiabática de chama da queima de gás metano a 1 atm com: (a) oxigênio puro
e (b) ar atmosférico.

Espécie H°298 (cal.mol-1) Cpmédio (cal/mol.K)


CH4(g) - 17895 9,85
O2(g) 0 9,17
N2(g) 0 5,95
H2O(g) - 57798 10,04
CO2(g) - 94050 14,78

Resolução: Item a) Considerando o oxigênio puro: CH4(g) + 2 O2(g) → CO2(g) + 2 H2O(g)


0 0
Cálculo da variação de entalpia padrão da reação de combustão do gás metano (CH4): ∆Hreação = ∑ Hprodutos −
0
∑ Hreagentes
0 0 0
0
∆Hreação = [1 x HCO 2
+ 2 x HH 2O
] − [1 x HCH 4
+ 2 x HO0 2 ]

0
∆Hreação = [1 x (−94050) + 2 x (−57798)] − [1 x (−17895) + 2 x 0]

0
cal
∆Hreação = [−94050 − 115596] − [−17895] = −191751
mol

Análise do balanço térmico:

Cálculo da temperatura adiabática de chama com a presença de O2: Q p = ∑ ∆H


298 298 T T
Qp = ∫ (1 x 9,85)dT + ∫ (2 x 9,17)dT + ∫ (1 x 14,78)dT + ∫ (2 x 10,04)dT
298 298 298 298
+ (−191751)

Q p = 0 + 0 + 14,78. (T − 298) + 20,08. (T − 298) − 191751

Q p = (T − 298) x [14,78 + 20,08] − 191751

Sabendo que o processo é adiabático (Q = 0), temos: 0 = (T − 298) x [14,78 + 20,08] − 191751

T = 5798,60 K

Item b) Cálculo do número de mol de gás nitrogênio (N2), considerando que a composição do ar atmosférico seja
igual a 20% de O2(g) e 80% de N2(g):

2 mol de O2 ---------- 20%


nN2 --------------------- 80%
nN2 = 8,00 mol de N2(g)

329
1600 FÍSICO-QUÍMICA APLICADA EXERCÍCIOS COMENTADOS - IME – ITA – OLIMPÍADA

Equação química balanceada de combustão completa com a presença de N2(g):


CH4(g) + 2 O2(g) + 8 N2(g) → CO2(g) + 2 H2O(g) + 8 N2(g)

0 0 0
∆Hreação = ∑ Hprodutos − ∑ Hreagentes

0 0
∆𝐻𝑟𝑒𝑎çã𝑜 = [1 𝑥 𝐻𝐶𝑂2
+ 2 𝑥 𝐻𝐻02 𝑂 + 8 𝑥 𝐻𝑁02 ] − [1 𝑥 𝐻𝐶𝐻
0
4
+ 2 𝑥 𝐻𝑂02 + 8 𝑥 𝐻𝑁02 ]

0
∆𝐻𝑟𝑒𝑎çã𝑜 = [1 𝑥 (−94050) + 2 𝑥 (−57798) + 8 𝑥 0)] − [1 𝑥 (−17895) + 2 𝑥 0 + 8 𝑥 0]

0
𝑐𝑎𝑙
∆𝐻𝑟𝑒𝑎çã𝑜 = [−94050 − 115596] − [−17895] = −94050 − 115596 + 17895 = −191751
𝑚𝑜𝑙

Estudo do balanço térmico, com a presença do nitrogênio gasoso:

Cálculo da temperatura adiabática de chama (T) com a presença do ar atmosférico: 𝑄𝑝 = ∑ ∆𝐻


298 298 298 T
Qp = ∫ (1 x 9,8)dT + ∫ (2 x 9,17)dT + ∫ (8 x 5,95)dT + ∫ (1 x 14,78)dT
298 298 298 298
T T
+ ∫ (8 x 5,95)dT + ∫ (2 x 10,04)dT + (−191751)
298 298
Q p = 0 + 0 + 0 + 14,78. (T − 298) + 20,08. (T − 298) + 47,6. (T − 298) − 191751

Sabendo que o processo é adiabático (Q = 0), temos: 0 = (T − 298) x [14,78 + 20,08 + 47,6] − 191751

191751 = (T − 298) x [82,46]

T = 2623,38 K

Questão 707 – (GRLLO) Que quantidade de calor é liberada por uma reação química que é capaz de elevar 10°C até
78°C a temperatura de 1,50 kg de água? Dado: calor específico da água = 1 cal/g.oC.
Resolução: Considerando que este processo ocorre a pressão constante (Qp = ΔH), logo:
𝒄𝒂𝒍
𝑸 = ∆𝑯𝒓𝒆𝒂çã𝒐 = 𝒎𝒂𝒔𝒔𝒂 𝒙 𝒄𝒑 𝒙 ∆𝑻 = 𝟏𝟓𝟎𝟎 𝒈 𝒙 𝟏, 𝟎 𝒙 (𝟕𝟖 − 𝟏𝟎)°𝑪
𝒈 𝒙 °𝑪
= 𝟏𝟎𝟐𝟎𝟎𝟎 𝒄𝒂𝒍 (𝟏𝟎𝟐 𝒌𝒄𝒂𝒍)

330
1600 FÍSICO-QUÍMICA APLICADA EXERCÍCIOS COMENTADOS - IME – ITA – OLIMPÍADA

Questão 708 – (ITA) Sabe-se que a 25°C, as entalpias de combustão (em kJ.mol-1) de grafita, gás hidrogênio e gás
metano são, respectivamente: -393,5; -285,9 e –890,5. Assinale a alternativa que apresenta o valor correto da entalpia
da seguinte reação: C(grafita) + 2 H2(g) → CH4(g).
a) -211,1 kJ.mol-1
b) – 74,8 kJ.mol-1
c) 74,8 kJ.mol-1
d) 136,3 kJ.mol-1
e) 211,1 kJ.mol-1

Resolução: Alternativa B.

Equação química de combustão da grafita: C(grafita) + O2(g) → CO2(g) ΔH = - 393,5 kJ.mol-1


Equação química de combustão do gás hidrogênio: H2(g) + ½O2(g) → H2O(g) ΔH = - 285,9 kJ.mol-1
Equação química de combustão do gás metano: CH4(g) + 2 O2(g) → CO2 + 2H2O(g) ΔH = - 890,5 kJ.mol-1

Aplicando a Lei de Hess, temos:

C(grafita) + O2(g) → CO2(g) ΔH = - 393,5 kJ.mol-1


2 H2(g) + O2(g) → 2 H2O(g) ΔH = 2 x (- 285,9 kJ.mol-1)
CO2 + 2H2O(g) → CH4(g) + 2 O2(g) ΔH = + 890,5 kJ.mol-1 +
C(grafita) + 2 H2(g) → CH4(g) ΔH = – 74,8 kJ.mol-1

Questão 709 – Explique o que representa a entalpia e como pode ser medida experimentalmente.

Resolução: Se a pressão é mantida constante durante um processo que envolve somente trabalho mecânico, temos
que: δW = −pdV, e a Primeira Lei fornece: dU = δQ p + δW = δQ p − pdV, onde Qp é o calor trocado entre o
sistema e as suas vizinhanças a pressão constante (processo isobárico). Daí, dU = δQ p − pdV

dQ p = dU + pdV = d(U + pV).

Como a expressão U + PV contém somente funções de estado, ela também é uma propriedade de estado extensiva,
ou seja, uma propriedade dependente da massa, definida como Entalpia, H, isto é: dH = d(U + pV)
H = U + pV
Portanto, para um processo a pressão constante, δQ p = dH. Assim, a mudança de entalpia de um sistema
representa o calor transferido entre o sistema e as suas vizinhanças durante um processo a pressão constante. A
δQp
razão δT
entre o calor trocado pelo sistema com as vizinhanças e a variação da temperatura do sistema a pressão
constante é definida como a capacidade calorífica a pressão constante, Cp (expressa, por exemplo, em cal/mol x K):
δQ
Cp = (dT)
p

dH
Cp = ( )
dT p
𝑇
Portanto, a variação de entalpia H pode ser calculada como: ∆𝐻 = ∫𝑇 2 𝐶𝑝 𝑥 𝑑𝑇.
1

Experimentalmente, a variação de entalpia de uma substância pode ser medida em um calorímetro a pressão
constante, cujo Cp é conhecido (tabelado). O tipo clássico de calorímetro é o calorímetro de queda, onde uma pequena
massa da substância a Ts >> Tamb é deixada cair sob gravidade no interior do calorímetro a Tc = Tamb, conforme
esquematizado na figura abaixo:

331
1600 FÍSICO-QUÍMICA APLICADA EXERCÍCIOS COMENTADOS - IME – ITA – OLIMPÍADA

O interior do calorímetro é preenchido com um fluido bom condutor de calor e as variações de temperatura no
calorímetro são medidas através de termopares. A variação total de entalpia do sistema combinado "substância +
calorímetro" é nula, ou seja:

Htotal = (HTf,c + H298,s) – (H298,c + HTs,s) = 0 => Hs + Hc = 0


Não havendo mudança de fase da substância até o equilíbrio ser atingido,

Cp,s. Ts + Cp,c Tc = 0 => mscs (298 – Ts) = Cp,c (Tf – 298),
onde ms é a massa (g) e cs o calor específico a pressão constante (cal/g.K) da substância. Como Cp,c e ms são
previamente conhecidos, o calorímetro efetua as medições de temperatura para determinar Ts e Tc e fornece como
resultado o valor Hs = mscsTs
Questão 710 – Explique o que representa a energia interna e como pode ser medida.

Resolução: Nos experimentos realizados por Joule, energia é adicionada à água sob a forma de trabalho, mas é
extraída sob a forma de calor. Esta energia entre o instante da sua adição à água sob a forma de trabalho e o instante
da sua retirada sob a forma de calor é definida como energia interna (U). Essa energia se refere à energia das
moléculas que a compõem a substância que se encontram em movimento e possuem energia cinética de translação,
rotação e vibração interna. Não deve ser deixado de lado também, a energia potencial resultante das forças
intermoleculares. Então, a energia interna é uma energia que é o somatório de todas as energias presentes em um
determinado corpo (sistema) a ser estudado. Varia com a pressão, temperatura, volume, número de moléculas, etc.
T

A denominação desta forma de energia de interna a distingue das energias cinéticas e potencial que o sistema pode
possuir como consequência de sua posição ou seu movimento, que podem ser encaradas como formas externas de
energia. Pela Primeira Lei da Termodinâmica postula a existência da Energia Interna, U como uma função de estado
(depende do início e do final do processo) de um sistema e afirma que a energia interna de qualquer sistema é
conservada, independente dos processos que ocorram no sistema. A quantidade de calor e trabalho existentes no
sistema está relacionado ao sistema com a vizinhança. Assim sendo, quando um sistema muda de um estado inicial
I para um estado final II, a conservação de energia requer para frações finitas: UII = UI + Q + W ou ∆U = UII −

332
1600 FÍSICO-QUÍMICA APLICADA EXERCÍCIOS COMENTADOS - IME – ITA – OLIMPÍADA

UI = Q + W. Geralmente a medição é feita em sistemas fechados, onde a massa é constante não havendo
transferência de energia interna pelas fronteiras do sistema. Toda a troca de energia entre um sistema fechado e suas
vizinhanças é na forma de calor e trabalho. Sendo assim no sistema fechado, supondo que não sofra variações de
energia cinética e potencial, podemos escrever: ∆𝑈 = 𝑄 + 𝑊. Para processos que causam variações infinitesimais
do estado do sistema, a expressão passa a ser dU = δQ + δW.
Questão 711 – Considere a reação de decomposição do gás amoníaco, apresentado pela seguinte equação química
balanceada: NH3(g) →1/2 N2(g) + 3/2 H2(g). Sabendo que a variação da entalpia padrão é igual a 46150,0 J.mol-1,
caracterizando um processo endotérmico, calcule a variação da energia interna (∆U).

Resolução: NH3(g) →1/2 N2(g) + 3/2 H2(g)

∆H = ∆U + pV = ∆U + ∆n x R x T

∆H = ∆U + (nprodutos − nreagentes ) x R x T

1 3
+ 46150 = ∆U + {( + ) − 1} x 8,314 x (25 + 273)
2 2

+ 46150 = ∆U + {2 − 1} x 8,314 x 298

J kJ
∆U = + 43672,43 (+ 43,67 )
mol mol

Questão 712 – Calcule o valor da entalpia padrão a uma temperatura igual a 125℃, a partir da seguinte reação
química balanceada: C(grafite) + H2O(g) → CO(g) + H2(g). Considere que a entalpia padrão deste processo a 25℃ seja
igual a 131,30 kJ.mol-1, caracterizando um processo endotérmico. Dados para a resolução do problema:

Substância ̅𝒑
𝑪 𝒎é𝒅𝒊𝒐
[J/mol.K]
C(grafite) 8,64
H2O(g) 33,58
CO(g) 29,14
H2(g) 28,84
Resolução: Para a resolução do problema será necessário
utilizar a lei de kirchoff.

Variação da entalpia padrão (∆H°): C(grafite) + H2O(g) → CO(g) + H2(g)

0 0 0
∆𝐻𝑟𝑒𝑎çã𝑜 = ∑ 𝐻𝑝𝑟𝑜𝑑𝑢𝑡𝑜𝑠 0
− ∑ 𝐻𝑟𝑒𝑎𝑔𝑒𝑛𝑡𝑒𝑠 = [1 𝑥 𝐻𝐶𝑂 + 1 𝑥 𝐻𝐻02 ] − [1 𝑥 𝐻𝐶0 + 1 𝑥 𝐻𝐻02 𝑂 ]
𝑘𝐽
= 131,30 .
𝑚𝑜𝑙

Cálculo da variação da capacidade calortífica médio a pressão constante: ∆C̅pmédio = ∑ C̅p médio

dos produtos

∑ C̅p médio
dos reagentes

A partir da tabela fornecida, temos: ∆C̅pmédio = [1 x 29,14 + 1 x 28,8] − [1 x 8,64 + 1 x 33,58]

J
∆C̅pmédio = [29,14 + 1 x 28,8] − [8,64 + 33,58] = 57,94 − 42,22 = +15,72
mol x K

Cálculo da variação de entalpia para T = 125℃:

333
1600 FÍSICO-QUÍMICA APLICADA EXERCÍCIOS COMENTADOS - IME – ITA – OLIMPÍADA

(125+273) 398
0
∆H = ∆Hreação +∫ C̅pmédio dT = 131300 + ∫ (15,72) x dT
(25+273) 298
J kJ
∆H = 131300 + 15,72 x (398 − 298) = 131300 + (15,72 x 100) = +132872 (+132,87 )
mol mol

Questão 713 – Considere as seguintes equações termoquímicas apresentadas a seguir:


a) Ag2O(s) + 2 HCl(g) → 2 AgCl(s) + H2O(l) ∆H° = - 77,610 kcal.mol-1
b) 2 Ag (s) + 1/2 O2(g) → Ag2O(s) ∆H° = - 7,30 kcal.mol-1
c) ½ H2(g) + 1/2 Cl2(g) → HCl(g) ∆H° = - 22,10 kcal.mol-1
d) H2(g) + 1/2 O2(g) → H2O(l) ∆H° = - 68,30 kcal.mol-1
Calcule a variação da energia de entalpia para a reação de formação de cloreto de prata a 25℃. Calcule também a
variação da energia interna à mesma temperatura.

Resolução: Ag (s) + 1/2 Cl2(g) → AgCl(s) ∆H° = ?

Para a resolução deste exercício, será necessário utilizar a lei de Hess.

Multiplicando a terceira equação química por dois: H2(g) + Cl2(g) → 2HCl(g) ∆H° = 2 x (- 22,10 kcal.mol-1)

Invertendo a quarta equação química: H2O(l) → H2(g) + 1/2 O2(g) ∆H° = + 68,30 kcal.mol-1

Permanecendo as mesmas características para as reações (a) e (b), temos:

Ag2O(s) + 2 HCl(g) → 2 AgCl(s) + H2O(l) ∆H° = - 77,61 kcal.mol-1

2 Ag (s) + 1/2 O2(g) → Ag2O(s) ∆H° = - 7,30 kcal.mol-1

H2(g) + Cl2(g) → 2 HCl(g) ∆H° = 2 x (- 22,10 kcal.mol-1)

H2O(l) → H2(g) + 1/2 O2(g) ∆H° = + 68,30 kcal.mol-1 +

2 Ag (s) + Cl2(g) → 2 AgCl(s) ∆H° = - 77,61 kcal.mol-1 + (- 7,30 kcal.mol-1) + (- 44,20 kcal.mol-1) + 68,30
kcal.mol-1 = - 60,81 kcal.mol-1
60,81 𝑘𝑐𝑎𝑙
Dividindo a equação química obtida por dois: 2 Ag (s) + Cl2(g) → 2 AgCl(s) ∆H° = − 2
=
𝑘𝑐𝑎𝑙
−30,405 𝑚𝑜𝑙

Cálculo da variação da energia interna a 25℃: ∆H = ∆U + ∆n x R x T

∆H = ∆U + (nprodutos − nreagentes ) x R x T

1
− 30405 = ∆U + (0 − ) x 1,987 x (25 + 273)
2
cal
∆U = − 23624,36
𝑚𝑜𝑙

334
1600 FÍSICO-QUÍMICA APLICADA EXERCÍCIOS COMENTADOS - IME – ITA – OLIMPÍADA

Questão 714 – Considere a reação de produção de zinco gasoso e dióxido de carbono conforme a equação química
balanceada a seguir: ZnO(s) + CO(g) → Zn(g) + CO2(g). A expressão da enatlpia é dada pela seguinte equação:

∆𝐇(𝐉. 𝐦𝐨𝐥−𝟏 ) = 𝟒𝟕, 𝟑𝟗 − 𝟎, 𝟕𝟎𝐓 − 𝟑, 𝟐𝟗 𝐱 𝟏𝟎−𝟑 𝐓 𝟐 + 𝟏, 𝟐𝟓 𝐱 𝟏𝟎−𝟔 𝐓 𝟑

A partir das informações apresentadas pelo exercício, calcule a variação da energia interna em função da temperatura
e calcule os valores de ∆H e ∆U à 227℃. Informação para a resolução do problema: expressão geral da capacidade
calorífica a pressão constante: 𝐶𝑝 = A + BT + CT 2 + DT 3 .

Resolução: Sabendo que a equação termodinâmica que relaciona ∆H com ∆U é dadao por: ∆H = ∆U +
∆n x R x T = ∆U + (nprodutos − nreagentes ) x R x T

Considerando que os gases apresentam comportamento idel, temos:

47,39 − 0,70. T − 3,29 x 10−3 . T 2 + 1,25 x 10−6 . T 3 = ∆U + (nprodutos − nreagentes ) x R x T

47,39 − 0,70. T − 3,29 x 10−3 . T 2 + 1,25 x 10−6 . T 3 = ∆U + (2 − 1) x 8,314 x T

47,39 − 0,70. T − 3,29 x 10−3 . T 2 + 1,25 x 10−6 . T 3 = ∆U + 8,314 . T

∆U = 47,39 − 0,70. T − 3,29 x 10−3 . T 2 + 1,25 x 10−6 . T 3 − 8,314. T

∆𝐔 = 𝟒𝟕, 𝟑𝟗 − 𝟗, 𝟎𝟏𝟒. 𝐓 − 𝟑, 𝟐𝟗 𝐱 𝟏𝟎−𝟑 . 𝐓 𝟐 + 𝟏, 𝟐𝟓 𝐱 𝟏𝟎−𝟔 . 𝐓 𝟑

Cálculo da variação de entalpia a T = 227°C: ∆H(J. mol−1 ) = 47,39 − 0,70. T − 3,29 x 10−3 . T 2 +
1,25 x 10−6 . T 3

∆H(J. mol−1 ) = 47,39 − 0,70x (227 + 273) − 3,29 x 10−3 𝑥 (227 + 273)2 + 1,25 x 10−6 𝑥 (227
+ 273)³

∆H(J. mol−1 ) = 47,39 − 0,70 x 500 − 3,29 x 10−3 x (500)2 + 1,25 x 10−6 𝑥 (500)³

∆H(J. mol−1 ) = 47,39 − 350 − 822,5 + 156,25

∆H(J. mol−1 ) = −968,86

Cálculo da variação da energia interna a T = 227°C: ∆U(J. mol−1 ) = 47,39 − 9,014. T − 3,29 x 10−3 . T 2 +
1,25 x 10−6 . T 3

∆U(J. mol−1 ) = 47,39 − 9,014 x (227 + 273) − 3,29 x 10−3 𝑥 (227 + 273)2 + 1,25 x 10−6 𝑥 (227
+ 273)³

∆U(J. mol−1 ) = 47,39 − 9,014 x 500 − 3,29 x 10−3 𝑥 (500)2 + 1,25 x 10−6 𝑥 (500)³

∆U(J. mol−1 ) = 47,39 − 4507 − 822,5 + 156,25 = −5125,95

335
1600 FÍSICO-QUÍMICA APLICADA EXERCÍCIOS COMENTADOS - IME – ITA – OLIMPÍADA

Questão 715 – Usando as constantes de capacidade calorífica molar a pressão constante, apresentada na tabela a
seguir para o dióxido de carbono, calcule a quantidade de calor a pressão constante e também a volume constante,
quando são utilizados 350 g desta mesma amostra gasosa de 300 à 500 K.
Substância 𝐂̅𝐩 [J/mol.K]
CO2(g) A B x 10³ C x 107 D x 109
5,166 15,177 - 95,78 2,260
350 g
Resolução: Cálculo do número de mol de dióxido de carbono: n = g = 7,95 mol
44
mol

Expressão da equação matemática da capacidade calorífica a pressão constante: Cp = 5,166 −


15,177 x 103 x T − 95,78 x 107 x T 2 + 2,260 x 109 x T 3

Analisando primeiramente o processo a pressão constante. Cálculo da variação da entalpia, considerando a variação
de temperatura igual a 300 – 500 K:

∆H = ∫ n x Cp x dT

500 K
∆H = ∫ 7,95 x (5,166 + 15,177 x 103 x T − 95,78 x 107 x T 2 + 2,260 x 109 x T 3 ) x dT
300 K

500 K 500 K 500 K


∆H = 7,95 x {∫ 5,166 x dT + ∫ 15,177 x 103 x T x dT − ∫ 95,78 x 107 x T 2 x dT
300 K 300 K 300 K
500 K
+∫ 2,260 x 109 x T 3 x dT}
300 K

500 500
T² T3
∆H = 7,95 x {5,166 x T|500
300
3
+ 15,177 x 10 x ( )| − 95,78 x 107 x ( )|
2 300 3 300
500
T4
9
+ 2,260 x 10 x ( )| }
4 300

(500)2 (300)2
∆H = 7,95 x {5,166 x (500 − 300) + 15,177 x 103 x [ − ]
2 2
(500)3 (300)3 (500)4 (300)4
− 95,78 x 107 x [ − ] + 2,260 x 109 x [ − ]}
3 3 4 4

∆H = 7,95 x {5,166 x 200 + 15,177 x 103 x 80000 − 95,78 x 107 x 3,26 x 107
+ 2,260 x 109 x 1,36 x 1010 }

J
∆H = 7,95 x {1033,2 + 1,21 x 109 − 3,12 x 1016 + 3,07 x 1019 } = 2,44 x 1020
mol

Analisando o processo a volume constante. Cálculo da capacidade calorífica a volume constante (CV): Cp − CV = R

5,166 + 15,177 x 103 x T − 95,78 x 107 x T 2 + 2,260 x 109 x T 3 − CV = R

CV = 5,166 + 15,177 x 103 x T − 95,78 x 107 x T 2 + 2,260 x 109 x T 3 − 8,314

CV = −3,148 + 15,177 x 103 x T − 95,78 x 107 x T 2 + 2,260 x 109 x T 3

336
1600 FÍSICO-QUÍMICA APLICADA EXERCÍCIOS COMENTADOS - IME – ITA – OLIMPÍADA

Cálculo da variação da energia interna, considerando a variação de temperatura igual a 300 – 500 K: ∆U =
∫ n x CV x dT
500 K
∆U = ∫ 7,95 x (−3,148 + 15,177 x 103 x T − 95,78 x 107 x T 2 + 2,260 x 109 x T 3 ) x dT
300 K

500 K 500 K 500 K


∆U = 7,95 x {∫ (−3,148) x dT + ∫ 15,177 x 103 x T x dT − ∫ 95,78 x 107 x T 2 x dT
300 K 300 K 300 K
500 K
+∫ 2,260 x 109 x T 3 x dT}
300 K

500 500
T² T3
∆U = 7,95 x {(−3,148) x T|500
300 + 15177 x ( )| − 95,78 x 107 x ( )|
2 300 3 300
500
T4
+ 2,260 x 109 x ( )| }
4 300

(500)2 (300)2
∆U = 7,95 x {−3,148 x (500 − 300) + 15,177 x 103 x [ − ]
2 2
(500)3 (300)3 (500)4 (300)4
− 95,78 x 107 x [ − ] + 2,260 x 109 x [ − ]}
3 3 4 4

∆U = 7,95 x {−629,6 + 15,177 x 103 x 80000 − 95,78 x 107 x 3,26 x 107


J
+ 2,260 x 109 x 1,36 x 1010 } = 2,44 x 1020
mol

Questão 716 – (IME) Considerando que 100% do calor liberado na combustão de CH4 sejam utilizados para converter
100 kg de água a 100C em vapor a 1000C, calcule o volume de metano consumido, medido nas CNTP, supondo que
ele se comporte como um gás ideal. Dados: Constante universal dos gases (R) = 0,08206 atm.L.mol-1.K-1; Calor latente
de vaporização da água = 2260 J.g-1; Calor específico da água = 4,2 J.g-1.0C-1; Calor de combustão do metano = 890
kJ.mol-1.

Resolução: Equação química da combustão completa do gás metano: CH4(g) + 2O2(g) → CO2(g) + 2H2O(l)
H = - 890 kJ.mol-1.

Cálculo da quantidade de calor total (QTOTAL) necessário para a conversão de 100 kg de H2O(l) em H2O(v):

Cálculo do calor Latente: QL = m x LV


QL = 100000 g x 2260 J/g = 226000000 J = 2,26 x 108 J (QL = 2,26 x 105 kJ)

Cálculo do calor sensível: QS = m x c x ΔT


QS = 100000 g x 4,2 J/gºC x (100 - 10)ºC = 100000 g x 4,2 J/gºC x 90ºC = 37800000 J = 3,78 x 107J (QS = 3,78 x 104
kJ)

QTOTAL = QL (calor latente) + QS (calor sensível)


QTOTAL = (2,26 x 105 + 3,78 x 104) kJ
QTOTAL = 2,638 x 105 kJ

Cálculo do número de mol de gás metano (nCH4):


1 mol de CH4 ------------ 890 kJ
nCH4 ----------------------- 2,638 x 105 kJ
nCH4 = 296,40 mol

337
1600 FÍSICO-QUÍMICA APLICADA EXERCÍCIOS COMENTADOS - IME – ITA – OLIMPÍADA

Cálculo do volume de gás metano medido nas condições normais de temperatura e pressão (CNTP): VCH4 =
nCH4 x R x T 296,4 x 0,08206 x 273
= = 6640,06 L
p 1

Questão 717 – (IME) Determine a massa de água que, com uma variação de temperatura de 30ºC, fornece energia
equivalente ao calor de formação de um mol de sulfeto de carbono sólido. Dados: calor de combustão do sulfeto de
carbono = - 265 kcal.mol-1; calor de formação do gás sulfuroso = - 71 kcal.mol-1; calor de formação do dióxido de
carbono = - 96 kcal.mol-1; capacidade calorífica da água líquida = 1,0 cal.g-1.

Resolução:

Equação química da combustão do sulfeto de carbono: CS2 + 3 O2 → CO2 + 2 SO2 ΔH = – 265 kcal.mol –1

Equação química de formação do dióxido de enxofre: S + O2 → SO2 ΔH = - 71 kcal.mol-1

Equação química da combustão do dióxido de carbono: C + O2 → CO2 ΔH = - 96 kcal.mol-1

Aplicando o princípio da Lei de Hess, temos:


CO2 + 2 SO2 → CS2 + 3 O2 ΔH = – 265 kcal.mol –1
2 S + 2 O2 → 2 SO2 ΔH = (- 71 kcal.mol-1) x 2
C + O2 → CO2 ΔH = - 96 kcal.mol-1 +
C + 2 S → CS2 ΔH = + 27 kcal.mol -1

Sabendo que o processo a pressão constante, a variação da entalpia é igual a quantidade de calor, ΔH = Q p. Então:
ΔHf (CS2) = Qp = + 27,0 kcal/mol = 27000 cal/mol de CS

Cálculo da capacidade calorífica a pressão constante (Cp): Qp = Cp x ΔT


27000 cal = Cp x 30
Cp = 900 cal/oC

Cálculo da massa de água capaz de fornecer 27000 calorias quando resfriada a 30°C: Q = m x Cp
900 = m x Cp
m x 1 cal.g –1°C –1 = 900 cal/oC
Massa de água = 900 g

Questão 718 - (IME) Que quantidade de calor é liberada por uma reação química que é capaz de elevar 20°C para
28°C a temperatura de 2 kg de água? Dado: calor específico da água = 1 cal/g.oC.

Resolução: Considerando que este processo ocorre a pressão constante: ∆Hreação = Q p

cal
∆Hreação = m x c x ∆T = 2000 g x 1 x (28 − 20)°C = 16000 cal
g x °C

338
1600 FÍSICO-QUÍMICA APLICADA EXERCÍCIOS COMENTADOS - IME – ITA – OLIMPÍADA

Questão 719 – (IME) O consumo de água quente de uma casa é de 0,489 m3 por dia. A água está disponível a 10,0°C
e deve ser aquecida até 60,0°C pela queima de gás propano. Admitindo que não haja perda de calor para o ambiente
e que a combustão seja completa, calcule o volume (em m3) necessário deste gás, medido a 25,0°C e 1,00 atm, para
atender a demanda diária. Dados: Constante universal dos gases (R) = 82,0 x 10-6 m³.atm.K-1.mol-1; Massa específica
da água = 1,00 x 10³ kg.m-3; Calor especifico da água = 1,00 kcal.kg-1.°C-1. Dados de formação a 298 K a partir de
seus elementos: C3H8(g) = - 25,0 kcal.mol-1; H2O(g) = - 58,0 kcal.mol-1; CO2(g) = - 94,0 kcal.mol-1.
𝑚
Resolução: Cálculo da massa de água consumido diariamente, a partir da sua densidade: 𝑑 = á𝑔𝑢𝑎
𝑉
𝑘𝑔
𝑚á𝑔𝑢𝑎 = 𝑑 𝑥 𝑉 = 1000 𝑥 0,489 𝑚3 = 489 𝑘𝑔 (489000 𝑔)
𝑚³
Cálculo da variação da temperatura, sabendo que a temperatura inicial é igual a 10°C e a temperatura final é de 60°C:
∆T = TF – Ti = 50°C

Cálculo da quantidade de calor para o aquecimento da água: Q = m x c x ΔT


Q = 489000 g x 1 cal.g-1.°C-1 x (60 - 10)°C = 24450000 cal

Cálculo da variação de entalpia de combustão do gás propano: C3H8 + 5 O2 → 3 CO2 + 4 H2O.

∆H = {3 x (-94,0) + 4 x (-58,0)} – {(-25,0) + 5 x 0} = - 489 kcal/mol de propano queimado

Cálculo do número de mol de gás propano queimado:


1 mol de gás propano ----------- 489000 cal
npropano ----------------------------- 24450000 cal
npropano = 50 mol de gás propano

Cálculo do volume do gás propano, medido nas Condições Normais Ambientais de Temperatura e Pressão (CNATP),
utilizando a equação dos gases ideais, temos:

pxV=nxRxT
1 x V = 50 x 82 x 10−6 x 298
V = 1,22 m³

Questão 720 – (IME) Calcule o valor da reação H2(g) + Cl2(g) → 2 HCl(g) a 25°C, sabendo-se que as energias de ligação
de H2, Cl2 e HCl a mesma temperatura a 1 atm são iguais a + 436 kJ.mol -1, + 243 kJ.mol-1 e + 431 kJ.mol-1,
respectivamente.

Resoluçao: Organizando os dados em uma tabela para a energia de ligação:

Ligação Energia (kJ.mol-1)


H-H + 436
Cl - Cl + 243
H - Cl + 431

Cálculo da entalpia (H) kJ.mol-1 dos reagentes (processo endotérmico): Hreagentes = +[ 1 x (H − H) +


𝑘𝐽
1 x (Cl − Cl)] = + [ 1 x (+ 436) + 1 x (+243)] = + 679 𝑚𝑜𝑙

Cálculo da entalpia (H) em kJ.mol-1 dos produtos (processo exotérmico): Hprodutos = −2 x (H − Cl) =
𝑘𝐽
−2 x (+431) = − 862
𝑚𝑜𝑙

𝑘𝐽 𝑘𝐽
Cálculo da entalpia da reação (ΔHreação): ∆H𝑟𝑒𝑎çã𝑜 = Hprodutos + Hreagentes = −862 𝑚𝑜𝑙 + 679 𝑚𝑜𝑙 =
𝑘𝐽
−183 𝑚𝑜𝑙

339
1600 FÍSICO-QUÍMICA APLICADA EXERCÍCIOS COMENTADOS - IME – ITA – OLIMPÍADA

Questão 721 – (IME) Calcule a quantidade de calor liberado na queima de 130 kg de acetileno a 25°C e 1 atm. Dados
a 298K: (ΔHof) CO2 = - 94,1 kcal.mol-1; (ΔHof) H2O = - 57,8 kcal.mol-1;(ΔHof) C2H2 = + 54,2 kcal.mol-1.

Resolução: Equação química: C2H2(g) + 5/2 O2(g) → 2 CO2(g) + H2O(g).


0 0 0
Cálculo da variação de entalpia da combustão do gás acetileno: ∆Hreação = ∑ Hprodutos − ∑ Hreagentes
0 0 0 5
0
∆Hreação = [2 x HCO + 1 x H H O ] − [1 x HC H + x HO0 2 ]
2 2 2 2 2
5 kcal
= [2 x (−94,10) + 1 x (−57,80)] − [1 x (+54,20) + x 0] = −300,20
2 mol
mC2H2 130000 g
Cálculo do número de mol do gás acetileno: nC2 H2 = = = 5000 mol
<MM>C2H2 26 g.mol−1
Cálculo da entalpia para 130 kg de acetileno:
1 mol de C2H2 ----------------- (- 300,20 kcal)
5000 mol de C2H2 ------------ Qp = ΔH°
Qp = ΔH° = 1,50 x 106 kcal

Questão 722 – (IME) Calcule o calor de formação do meta-dimetilbenzeno, sabendo-se que:


a) o calor de combustão do meta-dimetilbenzeno é 1087,90 kcal.mol-1;
b) o calor de formação do gás carbônico é 94,00 kcal.mol-1;
c) o calor de formação da água é 68,30 kcal.mol-1.

Resolução:

Cálculo do valor da variação de entalpia da reação de formação do meta-dimetilbenzeno: 8 C + 5 H2 → C8H10


ΔH = ?

Calor de combustão do meta-dimetilbenzeno: C8H10 + 21/2 O2 → 8 CO2 + 5 H2O ΔH = - 1087,90 kcal.mol-1

Calor de formação do dióxido de carbono: C + O2 → CO2 ΔH = - 94,00 kcal.mol-1

Calor de formação da água: H2 + ½ O2 → H2O ΔH = - 68,30 kcal.mol-1

Para a resolução do problema será necessário aplicar o princípio da lei de Hess.

Invertendo a primeira equação química: 8 CO2 + 5 H2O → C8H10 + (21/2) O2 ΔH = - 1087,90 kcal.mol-1

Multiplicando a segunda equação por oito: 8 C + 8 O2 → 8 CO2 ΔH = 8 x (- 94,00) kcal.mol-1

Multiplicando a terceira equação por cinco: 5 H2 + 5/2 O2 → 5 H2O ΔH = 5 x (- 68,30) kcal.mol-1

Somando as equações químicas, temos:


8 CO2 + 5 H2O → C8H10 + (21/2) O2 ΔH = + 1087,90 kcal.mol-1
8 C + 8 O2 → 8 CO2 ΔH = 8 x (- 94,00) kcal.mol-1
5 H2 + 5/2 O2 → 5 H2O ΔH = 5 x (- 68,30) kcal.mol-1 +
8 C + 5 H2 → C8H10 ΔHreação = + 1087,90 kcal.mol-1+ [5 x (-68,30 kcal.mol-1) + [8 x (- 94,00)
kcal.mol ] = - 5,60 kcal.mol
-1 -1

340
1600 FÍSICO-QUÍMICA APLICADA EXERCÍCIOS COMENTADOS - IME – ITA – OLIMPÍADA

Questão 723 – (IME) Calcular a variação da energia interna, a 25°C, para a combustão de 1 (um) mol de benzeno
líquido à pressão constante. Dados a 25°C: (∆H°Formação)H2O = - 68320 cal.mol-1; (∆H°Formação)CO2 = - 94050 cal.mol-1;
(∆H°Formação)C6H6 = + 11720 cal.mol-1.

Resolução: Equação química da combustão completa do benzeno líquido: C6H6(l) + 15/2 O2(g) → 6 CO2(g) + 3 H2O(g)
0 0 0
Cálculo da variação de entalpia para a combustão do benzeno líquido: ∆Hreação = ∑ Hprodutos − ∑ Hreagentes

0 0 15
0
∆Hreação = [6 x HCO + 3 x HH 2O
] − [1 x HC06 H6 + x HO0 2 ]
2 2
15 cal
= [6 x (−94050) + 3 x (−68320)] − [1 x (+11720) + x 0] = −780980
2 mol

Considerando a constante dos gases igual a R = 2,0 cal.mol.K-1, temos: ∆H = ∆U + ∆nRT


cal cal
−780980 = ∆U + (9 − 7,50)mol x 2 x (25 + 273)K
mol mol x K

−780980 cal = ∆U + 1,50 x 2 cal x 298

∆U = −780980 cal − 894 cal = −781874 cal

Questão 724 – (IME) Calcule a mudança da energia interna, em kJ, para a reação de formação de dois mol de
SO2Cl2(g), a partir de S(g), O2(g) e Cl2(g), a 298 K. Dados: (∆H°FORMAÇÃO)S(g) = + 277 kJ.mol-1; (∆H°FORMAÇÃO)SOCl2 = - 210
kJ.mol-1.

Resolução: Equação química: 2 S(g) + O2(g) + 2 Cl2(g) → 2 SO2Cl2(g)


0 0 0
Cálculo da variação de entalpia: ∆Hreação = ∑ Hprodutos − ∑ Hreagentes

0 0
0
∆Hreação = [2 x HSO 2 Cl2
] − [2 x HS0 + 1 x HO0 2 + 2 x HCl2
]
kJ
= [2 x (−210)] − [2 x (+277) + 1 x 0 + 2 x 0] = −420 − 554 = −974
mol

Considerando que os gases que participam do processo químico apresentam apresentam comportamento ideal,
temos: ∆H = ∆U + (nprodutos − nreagentes ) x R x T

−974000 = ∆U + (2 − 5) x 8,314 x 298

−974000 = ∆U − 7432,72

−974000 + 7432,72 = ∆U

J kJ
∆U = −966567,28 (−966,57 )
mol mol

341
1600 FÍSICO-QUÍMICA APLICADA EXERCÍCIOS COMENTADOS - IME – ITA – OLIMPÍADA

Questão 725 – (IME) A combustão de 1 mol de naftaleno (C10H8) sólido a 25°C, realizada em um calorímetro
adiabático a volume constante, libera 1227 kcal com formação apenas de um produto gasoso, o gás carbônico. Calcule
o calor liberado por essa reação a pressão constante.

Resolução: Equação química da combustão do naftaleno: C10H8(s) + 12 O2(g) → 10 CO2(g) + 4 H2O(l).

Considerando que o processo ocorre a volume constante, o trabalho será igual a zero (W = 0). Através da primeira lei
da termodinâmica, temos:
∆U = Q + W
∆U = Q + 0
∆U = QV = - 1227 kcal

Através da definição termodinâmica da entalpia, temos: ∆H = ∆U + (nprodutos − nreagentes ) x R x T

∆H = −1227000 + {(10 − 12) x 1,987 x (25 + 273)} = −1228184,25 cal (−1228,82 kcal)

Questão 726 – (IME) A partir dos dados fornecidos abaixo, calcule o calor de hidrogenação do propeno, indicando se
a reação é exotérmica ou endotérmica. Dados:
a) Calor de formação da água no estado líquido, 68,3 kcal.mol-1;
b) Calor de combustão do propano, 530,6 kcal.mol-1;
c) Calor de combustão do propeno, 491,9 kcal.mol-1.

Resolução:

a) H2(g) + ½ O2(g) → H2O(l) ΔH = - 68,3 kcal.mol-1


b) C3H8(g) + 5 O2(g) → 3 CO2(g) + 4 H2O(l) ΔH = - 530,6 kcal.mol-1
c) C3H6(g) + 9/2 O2(g) → 3 CO2(g) + 3 H2O(l) ΔH = - 491,9 kcal.mol-1

Reação de hidrogenação do gás propeno: C3H6(g) + H2(g) → C3H8(g) ΔH = ? Aplicando o princípio da Lei de Hess:

H2(g) + ½ O2(g) → H2O(l) ΔH = - 68,3 kcal.mol-1


3 CO2(g) + 4 H2O(l) → C3H8(g) + 5 O2(g) ΔH = + 530,6 kcal.mol-1
C3H6(g) + 9/2 O2(g) → 3 CO2(g) + 3 H2O(l) ΔH = - 491,9 kcal.mol-1 +
C3H6(g) + H2(g) → C3H8(g) ΔH = - 29,60 kcal.mol-1

Questão 727 – (IME) O valor experimental para o calor liberado na queima de benzeno líquido a 25ºC, com formação
de dióxido de carbono e água liquida, é 780 Kcal/mol. A combustão é feita em uma bomba calorimétrica a volume
constante. Considerando comportamento ideal para os gases formados e R = 2,0 cal/mol.K, determine:
a) calor padrão de combustão do benzeno a 25ºC;
b) se o calor calculado no item anterior é maior ou menor quando a água é formada no estado gasoso.
Justifique sua resposta.

Resolução:

Item a) Para um processo à volume constante, o trabalho realizado é igual a zero (W = 0). A partir da primeira lei da
termodinâmica, temos a seguinte relação:
∆U = Q + W
∆U = Q + 0
∆U = QV = - 780000 kcal.mol-1

Item b) Equação da combustão do benzeno líquido: C6H6(l) + 7,5 O2(g) → 6 CO2(g) + 3 H2O(l)
H = U + pV
∆H = ∆U + (nprodutos − nreagentes ) x R x T
∆H = −780000 + {(6 − 7,5) x 2,0 x (25 + 273) }
∆H = −780000 − (1,50 x 2,0 x 298) = −780000 − 894 = −780894 cal (−780,89 kcal)

342
1600 FÍSICO-QUÍMICA APLICADA EXERCÍCIOS COMENTADOS - IME – ITA – OLIMPÍADA

Equação de combustão do benzeno líquido, havendo a produção de vapor d´água: C6H6(l) + 7,5 O2(g) → 6 CO2(g) + 3
H2O(v)

∆H = ∆U + (nprodutos − nreagentes ) x R x T
∆H = −780000 + {(9 − 7,5) x 2,0 x (25 + 273)} = −780000 + 894 = −779106 cal (−779,11 kcal)

Questão 728 – (IME) Uma amostra de 0,640 gramas de naftaleno sólido (C10H8) foi queimada num calorímetro de
volume constante, produzindo somente dióxido de carbono e água. Após a reação, verificou-se um acréscimo de
2,4°C na temperatura do calorímetro. Sabendo-se que a capacidade calorífica do calorímetro era de 2.570 cal/oC e
considerando-se que a variação de pressão foi muito pequena, calcule a entalpia de formação do naftaleno. Dados:
Entalpia de formação do CO2 (g): – 94,1 kcal/mol; Entalpia de formação da água (l): – 68,3 kcal/mol.

Resolução: Equação química da combustão do naftaleno: C10H8(s) + 12 O2(g) → 10 CO2(g) + 4 H2O(l)

Sendo o processo a volume constante, o trabalho realizado é igual a zero (W = 0). Logo: ∆U = Q + 0 = QV

Cálculo da quantidade de calor a volume constante: ∆U = Q v = n x Q v x ∆T


cal
∆U = Q v = 2570 x 2,40°C
°C

∆U = Q v = 6168 cal
massa 0,640
Cálculo do número de mol de naftaleno: nC10 H8 = <MM> = 128
= 0,0050 mol

Cálculo da quantidade de calor a volume constante (kcal.mol-1): ∆U = Q v = 6168 cal

6168 cal cal kcal


Qv = = 1233600 (1233,60 )
0,0050 mol mol mol

Como o processo ocorre através da queima de um hidrocarboneto, o que caracteriza que o processo é exotérmico,
temos: ∆U = - 1233,60 kcal.mol-1.

Considerando que a pressão seja muito baixa, p → 0, logo: ∆U = ∆H = - 1233,60 kcal.mol-1


0 0 0
Cálculo da entalpia de formação do naftaleno: ∆Hreação = ∑ Hprodutos − ∑ Hreagentes

0 0
0
∆Hreação = [10 x HCO 2
+ 4 x HH 2O
] − [1 x HC010 H8 + 12 x HO0 2 ]

− 1233,60 = [10 x (−94,10) + 4 x (−68,30)] − [1 x HC010 H8 + 12 x 0]

kcal
HC010 H8 = +19,40
mol

Questão 729 – (ITA) A 25ºC e pressão de 1atm, a queima completa de um mol de n-hexano produz dióxido de carbono
e água no estado gasoso e libera 3883 kJ, enquanto que a queima completa da mesma quantidade de n-heptano
produz as mesmas substâncias no estado gasoso e libera 4498 kJ.
b) Escreva as equações químicas, balanceadas, para as reações de combustão em questão.
c) Utilizando as informações fornecidas no enunciado desta questão, faça uma estimativa do valor do calor de
combustão do n-decano. Deixe claro o seu raciocínio.
d) Caso a água formada na reação de combustão do estado líquido, a quantidade de calor liberado seria maior,
menor ou igual a 3383 kJ? Por quê?

Resolução:

343
1600 FÍSICO-QUÍMICA APLICADA EXERCÍCIOS COMENTADOS - IME – ITA – OLIMPÍADA

Item a) Equações químicas balanceadas:

C6H14(l) + 19/2 O2(g) → 6 CO2(g) + 7 H2O(g) ΔH = -3883 kJ


C7H16(l) + 11 O2(g) → 7 CO2(g) + 8 H2O(g) ΔH = - 4498 Kj

Item b) C10H22() + 31/2 O2(g) → 10 CO2(g) + 11 H2O(g) ΔH = ?

Na combustão do heptano foram produzidos uma molécula a mais de dióxido de carbono (CO2) e também uma
molécula a mais de água (H2O) em comparação a combustão do hexano, e com isso, haverá maior liberação da
quantidade de calor, na ordem de 615 kJ a mais.

Item c) Considerando que a variação de entalpia do C6H14, ΔH do C7H16 e também do C10H22 não apresentam
diferenças tão significativas, pode-se estimar que o valor da variação de entalpia da combustão do C10H22 sendo igual
a (4 x 615) a mais do que o valor da variação de entalpia de combustão do C6H14 em função da produção de quatro
moléculas de dióxido de carbono e também de quatro moléculas de água (4 CO2 + 4 H2O).

Cálculo da entalpia de combustão do n-decano: ΔH = 4 x (- 615) + ΔHcombustão do n-hexano


ΔH = 4 x (-615 kJ) + (-3883 kJ) = - 2460 kJ – 3883 kJ = - 6343 kJ
Item d) Para esta alternativa será necessário aplicar a Lei de Hess.
Sabendo que a entalpia de formação da água líquida é menor que a entalpia de formação da água no estado vapor,
temos com isso a seguinte relação: ΔHf H2O(l) < ΔHf H2O(g)

C6H14 + 19/2 O2 → 6 CO2 + 7 H2O(g) ΔH = - 3883 kJ


7 H2O(g) → 7 H2O(l) ΔH = - X +
C6H14 + 19/2 O2 →6 CO2 + 7 H2O(l) ΔH = - (3883 + X)

Questão 730 – (OLIMPÍADA BRASILEIRA DE QUÍMICA) A queima de metano na presença de oxigênio pode
produzir três produtos distintos, contendo carbono: fuligem (partículas muito pequenas de grafite), monóxido de
carbono gasoso e dióxido de carbono gasoso.
a) Escreva três equações químicas equilibradas, correspondendo às reações de metano gasoso com oxigênio
que levam a cada um dos produtos acima citados. Em todos os casos admita que o outro produto é a água, H2O(l);
b) Determina as entalpias padrões de cada uma das reações do item (a);
c) Por que, havendo oxigênio em quantidade suficiente, o CO2(g) é o produto "carbônico" predominante na
combustão do metano? Dados: ΔHf0 (kJ.mol-1): CO(g) = -110,5; CO2(g) = -393,5; H2O(l) = -285,83; CH4(g) = -74,81.

Resolução:

Item a) Equação química (a) da combustão completa do gás metano: CH4(g) + O2(g) → CO2(g) + 2 H2O(l)

Equação química (b) de combustão incompleta do gás metano: CH4(g) + 3/2 O2(g) → CO(g) + 2 H2O(l)

Equação química (c) de combustão incompleta do gás metano: CH4(g) + O2(g) → C(s) + 2 H2O(l).
0
Item b) Cálculo da variação de entalpia da combustão completa do gás metano da equação química (a): ∆Hreação =
0 0
∑ Hprodutos − ∑ Hreagentes

0 0 0
0
∆Hreação = [1 x HCO 2
+ 2 x HH 2O
] − [1 x HCH 4
+ 1 x HO0 2 ]
kJ
= [1 x (−393,5) + 2 x (−285,83)] − [1 x (−74,81) + 1 x 0 ] = −890,35
mol
0
Cálculo da variação de entalpia da combustão incompleta do gás metano da equação química (b): ∆Hreação =
0 0
∑ Hprodutos − ∑ Hreagentes

344
1600 FÍSICO-QUÍMICA APLICADA EXERCÍCIOS COMENTADOS - IME – ITA – OLIMPÍADA

0 0 0 3
0
∆Hreação = [1 x HCO + 2 x HH 2O
] − [1 x HCH + x HO0 2 ]
4 2
3 kJ
= [1 x (−110,5) + 2 x (−285,83)] − [1 x (−74,81) + x 0 ] = −607,35
2 mol
0
Cálculo da variação de entalpia de combustão incompleta do gás metano da equação química (c): ∆𝐻𝑟𝑒𝑎çã𝑜 =
0 0
∑ 𝐻𝑝𝑟𝑜𝑑𝑢𝑡𝑜𝑠 − ∑ 𝐻𝑟𝑒𝑎𝑔𝑒𝑛𝑡𝑒𝑠

0
∆Hreação = [1 x HC0 + 2 x HH
0
2O
0
] − [1 x HCH 4
+ 1 x HO0 2 ]
kJ
= [1 x (0) + 2 x (−285,83)] − [1 x (−74,81) + 1 x 0 ] = −496,85
mol

Item c) O dióxido de carbono (CO2) é o produto mais predominante pelo fato de ser o mais exotérmico, o que pode
ser observado no valor obtido na equação química do item b.

Questão 731 - (OLIMPÍADA BRASILEIRA DE QUÍMICA) O trióxido de boro é um composto utilizado como aditivo
de fibra óptica, na produção de vidro de borossilicato, entre outros. Esse composto é obtido pela desidratação do
ácido bórico, porém também é possível consegui-lo a partir das seguintes etapas de reação:
Etapa 1: B2O3(s) + 3 H2O(g) → 3 O2(g) + B2H6(g) 2035 ΔH/kJ
Etapa 2: H2O(l) → H2O(g) 44 ΔH/kJ
Etapa 3: H2(g) + 1/2 O2(g) → H2O(l) - 286 ΔH/kJ
Etapa 4: 2 B(s) + 3 H2(g) → B2H6(g) 36 ΔH/kJ
Tendo como base as reações acima e suas respectivas entalpias, calcule a entalpia geral de formação do trióxido de
boro e informe sua equação global.

Resolução: Aplicando a Lei de Hess.

Invertendo a etapa 1: 3 O2(g) + B2H6(g) → B2O3(s) + 3 H2O(g) - 2035 ΔH/kJ

Multiplicando por três e invertendo a etapa 2: 3 H2O(g) → 3 H2O(l) 3x (- 44) ΔH/kJ

Multiplicando por três e invertendo a etapa 3: 3 H2O(l) → 3 H2(g) + 3/2 O2(g) 3x (+ 286) ΔH/kJ

Etapa 4: 2 B(s) + 3 H2(g) → B2H6(g) 36 ΔH/kJ

Somando as equações químicas e as entalpias:

3 O2(g) + B2H6(g) → B2O3(s) + 3 H2O(g) 2035 ΔH/kJ


3 H2O(g) → 3 H2O(l) 3x (- 44) ΔH/kJ
3 H2O(l) → 3 H2(g) + 3/2 O2(g) 3x (+ 286) ΔH/kJ
2 B(s) + 3 H2(g) → B2H6(g) 36 ΔH/kJ +
2 B(s) + 3/2 O2(g) → B2O3(s) ΔHTOTAL = - 2035 ΔH/kJ + 3x (- 44) ΔH/kJ + 3x (+ 286) ΔH/kJ + 36 ΔH/kJ
= - 1273 ΔH/kJ

Questão 732 – Determine os calores de formação da amônia e a hidrogenação do propeno. Informações para a
resolução do problema: Energias de ligação (kcal.mol-1): (C – C) = 82,6; (N ≡ N) = 225,8; (N – H) = 93,4; (C = C) =
154,8; (C – H) = 98,8; (H – H) = 104,2.

Resolução: Equação química: N2(g) + 3 H2(g) → 2 NH3(g)

Cálculo das entalpias (Reagentes e Produtos):

kcal
Hreagentes = + [1 x HN≡N + 3 x HH−H ] = + [1 x 225,8 + 3 x 104,2] = + 538,40
mol

345
1600 FÍSICO-QUÍMICA APLICADA EXERCÍCIOS COMENTADOS - IME – ITA – OLIMPÍADA

kcal
Hprodutos = −[2 x 3 x HN−H ] = −[6 x 93,40] = −560,40
mol

Cálculo da variação de entalpia da reação (ΔHreação): ∆Hreação = Hprodutos + Hreagentes

kcal
∆Hreação = −560,40 + 538,40 = −22,00
mol

Analisando a segunda reação – reação de hidrogenação do propeno: C3H6(g) + 3 H2(g) → 2 C3H8(g)

Hreagentes = +[1 x HC=C + 1 x HC−C + 6 x HC−H ] + [1 x HH−H ]

kcal
Hreagentes = +[1 x 145,8 + 1 x 82,6 + 6 x 98,8] + [1 x 104,2] = + 925,40
mol
kcal
Hprodutos = −[2 x HC−C + 8 x HC−H ] = − [2 x 82,60 + 8 x 98,80] = − 955,60
mol

Cálculo da variação de entalpia da reação (ΔHreação): ∆Hreação = Hprodutos + Hreagentes

kcal
∆Hreação = −955,60 + 925,40 = − 30,20
mol

Questão 733 – (IME) Qual a quantidade de calor produzida na combustão de 1 mol de sacarose, sabendo que m
gramas de sacarose libertam q kcal na sua combustão?

Resolução: Equação química: C12H22O11 + 35/2 O2 → 12 CO2 + 11 H2O.

Considerando que a reação ocorra a pressão constante, logo: Qp = ΔH.


1 mol de sacarose ------ 342 g ---------------- q’
m g ------------------- q
𝒒

Logo: 𝒒 = 𝟑𝟒𝟐 𝒙 ( )
𝒎

Questão 734 – Óxido de zinco e carbono, ambos a 25°C, são alimentados em quantidades estequiométricas em um
reator operado a 1,0 atm, onde ocorre a seguinte reação química: ZnO(s) + C(s) → Zn(g) + CO(g). A reação é completa
e os produtos saem do reator a 1,0 atm e a uma temperatura de 1000°C. Calcule o calor que é adicionado ou removido
do reator, por quilograma de zinco.

Espécie Cp [J/(mol.K)] ΔHo298 [kJ.mol-1]


ZnO(s) 52,5 -350,5
C(s) 19,8 0
Zn(g) 20,9 130,4
CO(g) 31,6 -110,5

346
1600 FÍSICO-QUÍMICA APLICADA EXERCÍCIOS COMENTADOS - IME – ITA – OLIMPÍADA

Resolução: Equação química: ZnO(s) + C(s) → Zn(g) + CO(g)

Cálculo da variação de entalpia a 298 K:

0 0 0
0
∆Hreação = ∑ Hprodutos 0
− ∑ Hreagentes 0
= [1 x HZn + 1 x HCO ] − [1 x HZnO + 1 x HC0 ]

0
kJ
∆Hreação = [1 x (130,40) + 1 x (−110,50)] − [1 x (−350,5) + 1 x 0 ] = + 370,40
mol

Estudo do balanço térmico:

𝑇 298
Cálculo da entalpia em relação ao ZnO(s): ∆𝐻1 = ∫𝑇 2 1 𝑥 𝐶𝑝 𝑥 𝑑𝑇 = ∫298 1 𝑥 𝐶𝑝 𝑥 𝑑𝑇 = 0
1

𝑇 298
Cálculo da entalpia em relação ao C(s): ∆𝐻2 = ∫𝑇 2 1 𝑥 𝐶𝑝 𝑥 𝑑𝑇 = ∫298 1 𝑥 𝐶𝑝 𝑥 𝑑𝑇 = 0
1

𝑇 1273
Cálculo da entalpia em relação ao Zn(g): ∆𝐻3 = ∫𝑇 2 1 𝑥 𝐶𝑝 𝑥 𝑑𝑇 = ∫298 1 𝑥 20,90 𝑥 𝑑𝑇
1

1273
∆𝐻3 = ∫298 20,90 𝑥 𝑑𝑇

∆𝐻3 = 20,90 𝑥 𝑇|1273


298

𝐽
∆𝐻3 = 20,90 𝑥 (1273 − 298) = 20377,50
𝑚𝑜𝑙

𝑇 1273
Cálculo da entalpia em relação ao CO(g): ∆𝐻4 = ∫𝑇 2 1 𝑥 𝐶𝑝 𝑥 𝑑𝑇 = ∫298 1 𝑥 31,60 𝑥 𝑑𝑇
1

1273
∆𝐻4 = ∫298 31,60 𝑥 𝑑𝑇
∆𝐻3 = 31,60 𝑥 𝑇|1273
298

J
∆H3 = 31,60 x (1273 − 298) = 30810 mol

Sabendo que o processo ocorre a pressão constante, então: Q p = ∑ ∆H

0
Q p = ∆Hreação + ∆H1 + ∆H2 + ∆H3 + ∆H4

kJ kJ kJ kJ
Q p = 370,40 + 0 + 0 + 20,38 + 30,81 = 421,59
mol mol mol mol

Cálculo da quantidade de calor para 1 quilograma de zinco gasoso:


1 mol de Zn ---------- 65,4 g de Zn ---------------------- 421,59 kJ
1000 g de Zn ----------------------- Qp
Qp = ΔH = 6446,33 kJ

347
1600 FÍSICO-QUÍMICA APLICADA EXERCÍCIOS COMENTADOS - IME – ITA – OLIMPÍADA

Questão 735 – Calcular a variação de entalpia de 2 mol de RbF quando aquecido de 300 K até 1200 K.

Cp/R ΔHfusão [J.mol-1]


Tf = 1048K
Faixa de T(K) 298 – 1048 1048 – 1500

RbF 7,3 8,8 26300

T Fusão T
Resolução: ∆HRbF = ∫T 2 Cp x dT + HRbF + ∫T 3 Cp x dT
1 3

1048 1200
Fusão
∆HRbF = ∫ 7,3 x R x dT + HRbF +∫ 8,8 x R x dT
300 1048

∆HRbF = 7,3 x R x T|1048 1200


300 + 26300 + 8,8 x R x T|1048

∆HRbF = 7,3 x R x (1048 − 300) + 26300 + 8,8 x R x (1200 − 1048)

∆HRbF = 7,3 x 8,314 x (1048 − 300) + 26300 + 8,8 x 8,314 x (1200 − 1048)

J kJ
∆HRbF = 82818,57 (82,82 )
mol mol

Resposta: Logo, para dois mol de RbF a entalpia será 165,64 kJ.

Questão 736 – Usando os dados abaixo, calcule o calor de reação a 70°C e 1,0 atm para a seguinte reação química
descrita a seguir: H2S(g) + 3/2 O2(g) → H2O(l) + SO2(g).

ΔHo298 (kJ.mol-1) CP (J.mol-1.K-1)


H2S(g) -20,6 26,6 + (24,1 x 10-3).T
O2(g) 0 25,8
H2O(l) -285,8 29,9
SO2(g) -296,8 25,8 + (58,2 x10-3).T

0 0 0
Resolução: Cálculo da variação da entalpia a 298K: ∆Hreação = ∑ Hprodutos − ∑ Hreagentes
0 0 0 3
0
∆Hreação = [1 x HH 2O
+ 1 x HSO ] − [1 x HH 2S
+ x HO0 2 ]
2 2

0
3
∆Hreação = [1 x (−285,80) + 1 x (−296,80)] − [1 x (−20,6) + x 0 ] = −582,60 + 20,60
2
kJ
= −562,00
mol

Cálculo da entalpia a uma temperatura igual a 70°C:


298 298 343 343
70°C
3
∆Hreação = ∆H ° + ∫ CpH2S x dT + x ∫ CpO2 x dT + ∫ 1 x CH2 O x dT + ∫ 1 x CSo2 x dT
298 2 298 298 298

298 298 343 343


70°C °
3
∆Hreação = ∆H + ∫ CpH2S x dT + x ∫ CpO2 x dT + ∫ CH2 O x dT + ∫ CSo2 x dT
298 2 298 298 298

343 343
70°C
∆Hreação = ∆H ° + 0 + 0 + ∫ CH2 O x dT + ∫ CSo2 x dT
298 298

348
1600 FÍSICO-QUÍMICA APLICADA EXERCÍCIOS COMENTADOS - IME – ITA – OLIMPÍADA

343 343
70°C
∆Hreação = ∆H ° + ∫ CH2 O x dT + ∫ CSo2 x dT
298 298

343 343
70°C
∆Hreação = ∆H ° + ∫ 29,9 x dT + ∫ (25,8 + 58,2 x 10−3 T) x dT
298 298

343
70°C

∆Hreação °
= ∆H + 29,9 x T|343
298 + {25,8 x T|343
298 + 58,2 x 10 −3
x ( )| }
2 298

70°C
343² 298²
∆Hreação = ∆H ° + 29,90 x (343 − 298) + {25,8 x (343 − 298) + 58,2 x 10−3 x ( )− ( )}
2 2

70°C
∆Hreação = ∆H ° + (29,90 x 45) + {25,8 x 45 + 58,2 x 10−3 x (58824,50 − 44402)}

70°C
J kJ
∆Hreação = −562000 + 1345,50 + {1161 + 839,39} = −558654,11 (−558,65 )
mol mol

Questão 737 – Calcule a temperatura adiabática de chama da queima completa de um gás natural contendo metano
puro a 1 atm com:
a) oxigênio puro;
b) ar atmosférico.

Espécie H°298 (kJ.mol-1) Cpmédio [J/(mol.K)]


CH4(g) -74,8 35,3
O2(g) 0 29,4
N2(g) 0 29,1
H2O(g) -241,8 34,4
CO2(g) -393,5 37,1

Resolução:

Item a) Considerando o oxigênio puro. Equação química da combustão completa do gás metano: CH4(g) + 2 O2(g) →
CO2(g) + 2 H2O(g)
0 0 0
Cálculo da variação de entalpia padrão da reação de combustão: ∆𝐻𝑟𝑒𝑎çã𝑜 = ∑ 𝐻𝑝𝑟𝑜𝑑𝑢𝑡𝑜𝑠 − ∑ 𝐻𝑟𝑒𝑎𝑔𝑒𝑛𝑡𝑒𝑠

0 0 0
0
∆Hreação = [1 x HCO 2
+ 2 x HH 2O
] − [1 x HCH 4
+ 2 x HO0 2 ]
= [1 x (−393,5) + 2 x (−241,8)] − [1 x (−74,8) + 2 x 0]

0
kJ
∆Hreação = −393,5 − 483,6 + 74,8 = −802,30
mol

Estudo do balanço térmico:

349
1600 FÍSICO-QUÍMICA APLICADA EXERCÍCIOS COMENTADOS - IME – ITA – OLIMPÍADA

0
Cálculo da temperatura adiabática de chama com a presença de O2: Q p = ∆Hreação + ∆H1 + ∆H2 + ∆H3 +
∆H4

Q p = −802300
298 298 T
+∫ 1 x 35,30 x dT + ∫ 2 x 29,40 x dT + ∫ 1 x 37,10 x dT
298 298 298
T
+ ∫ 2 x 34,40 x dT
298

298 298 T T
Q p = −802300 + ∫ 35,30 x dT + ∫ 58,80 x dT + ∫ 37,10 x dT + ∫ 68.80 x dT
298 298 298 298

T T
Q p = −802300 + 0 + 0 + ∫ 37,10 x dT + ∫ 68.80 x dT
298 298

Q p = −802300 + 37,10 x (T − 298) + 68,80 x (T − 298)

Processo adiabático – Q p = 0, temos: 0 = −802300 + 37,10 x (T − 298) + 68,80 x (T − 298)

802300 = (37,10 + 68,80) x (T − 298)

802300
(T − 298) =
(37,10 + 68,80)

802300
T= + 298
(37,10 + 68,80)

T = 7874,01 K

Item b) Levando em consideração que o ar atmosférico apresenta 20% de O2(g) e 80% de N2(g). Cálculo do número de
mol de gás nitrogênio:

2 mol de O2 ---------- 20%


nN2 --------------------- 80%
nN2 = 8 mol de N2(g)

Equação química balanceada com a presença de N2(g): CH4(g) + 2 O2(g) + 8 N2(g) → CO2(g) + 2 H2O(g) + 8 N2(g)

0 0 0
∆Hreação = ∑ Hprodutos − ∑ Hreagentes

0 0 0
0
∆Hreação = [1 x HCO 2
+ 2 x HH 2O
0
+ 8 x HN 2
] − [1 x HCH 4
+ 2 x HO0 2 + 8 x HN
0
2
]

350
1600 FÍSICO-QUÍMICA APLICADA EXERCÍCIOS COMENTADOS - IME – ITA – OLIMPÍADA

0
∆Hreação = [1 x (−393,5) + 2 x (−241,8) + 8 x 0] − [1 x (−74,8) + 2 x 0 + 8 x 0]
kJ
= −393,5 − 483,6 + 74,8 = −802,30
mol

Estudo do balanço térmico com a presença de nitrogênuio gasoso:

0
Cálculo da temperatura adiabática de chama com a presença de ar atmosférico: 𝑄𝑝 = ∆𝐻𝑟𝑒𝑎çã𝑜 + ∆𝐻1 + ∆𝐻2 +
∆𝐻3 + ∆𝐻4

Q p = −802300
298 298 298
+∫ 1 x 35,30 x dT + ∫ 2 x 29,40 x dT + ∫ 8 x 29,10 x dT
298 298 298
T T T
+∫ 1 x 37,10 x dT + ∫ 2 x 34,40 x dT + ∫ 8 x 29,10 x dT
298 298 298

Q p = −802300 + 0 + 0 + 0 + 37,10 x (T − 298) + 68,80 x (T − 298) + 232,8 x (T − 298)

Q p = −802300 + 37,10 x (T − 298) + 68,80 x (T − 298) + 232,8 x (T − 298)

Processo adiabático: Q p = 0:

0 = − 802300 + (37,10 + 68,80 + 232,80) x (T − 298)

802300
T − 298 =
(37,10 + 68,80 + 232,80)

802300 802300
T= + 298 = + 298 = 2666,76 K
(37,10 + 68,80 + 232,80) 338,70

Questão 738 – Calcule a temperatura adiabática de chama da queima completa de metano a 1 atm e 50% de excesso
de ar.

Espécie H°298 (cal.mol-1) Cpmédio [cal/(mol.K)]


CH4(g) -17895 9,85
O2(g) 0 9,17
N2(g) 0 5,95
H2O(g) -57798 10,04
CO2(g) -94050 14,78

Resolução: Equação química da combustão completa do gás metano: CH4(g) + 2 O2(g) → CO2(g) + 2 H2O(g)

Cálculo do número de mol de O2(excesso):


2 mol de O2 ----------- 100%
nO2 ---------------------- 50%

351
1600 FÍSICO-QUÍMICA APLICADA EXERCÍCIOS COMENTADOS - IME – ITA – OLIMPÍADA

nO2 = 1 mol

reagido
nexcesso
O2 = ninicial
O2 + n O2 = 2 mol + 1 mol = 3 mol

Cálculo do número de mol de N2(g), considerando que o ar atmosférico apresenta em sua composição 21% de O2(g) e
79% de N2(g):

3 mol de O2 ---------- 21%


nN2 -------------------- 79%
nN2 = 11,28 mol de N2(g)

Equação química da combustão completa do gás metano com a presença de N2(g):


CH4(g) + 3 O2(g) + 11,28 N2(g) → CO2(g) + 2 H2O(g) + 11,28 N2(g)
0 0
Cálculo da variação de entalpia padrão da reação de combustão do gás metano: ∆Hreação = ∑ Hprodutos −
0
∑ Hreagentes

0 0 0
0
∆Hreação = [1 x HCO 2
+ 2 x HH 2O
0
+ 11,28 x HN 2
] − [1 x HCH 4
+ 3 x HO0 2 + 11,28 x HN
0
2
]

0
∆Hreação = [1 x (−94050) + 2 x (−57798) + 11,28 x 0] − [1 x (−17895) + 3 x 0 + 11,28 x 0]

0
cal
∆Hreação = −191751
mol

Estudo do balanço térmico:

0
Cálculo da temperatura adiabática de chama com a presença de ar atmosférico: Q p = ∆Hreação + ∆H1 + ∆H2 +
∆H3 + ∆H4

Q p = −191751
298 298 298
+∫ 1 x 9,85 x dT + ∫ 3 x 9,17 x dT + ∫ 11,28 x 5,95 x dT
298 298 298
T T T
+∫ 1 x 14,78 x dT + ∫ 2 x 10,04 x dT + ∫ 11,28 x 5,95 x dT
298 298 298

Q p = −191751 + 0 + 0 + 0 + 14,78 x (T − 298) + 20,08 x (T − 298) + 67,12 x (T − 298)

Q p = −191751 + 14,78 x (T − 298) + 20,08 x (T − 298) + 67,12 x (T − 298)

Processo adiabático – Q p = 0:

0 = − 191751 + (14,78 + 20,08 + 67,12) x (T − 298)

191751 = (14,78 + 20,08 + 67,12) x (T − 298)

352
1600 FÍSICO-QUÍMICA APLICADA EXERCÍCIOS COMENTADOS - IME – ITA – OLIMPÍADA

191751
T − 298 =
(14,78 + 20,08 + 67,12)

191751
T= + 298 = 2178,28 K
(14,78 + 20,08 + 67,12)

Questão 739 – (IME) Nas combustões completas de x gramas de acetileno e de y gramas de benzeno são liberados,
respectivamente, Q1 kcal e Q2 kcal. Determine o calor liberado, em kcal, na formação de z gramas de benzeno a partir
do acetileno.

Resolução: Equação de combustão completa do acetileno: C2H2(g) + 5/2 O2 → 2 CO2 + H2O (Equação 1)

Sendo a massa molar do acetileno igual a 26 g.mol-1, temos:

∆𝐻 = 𝑄 𝑥 𝑛

− 26 𝑥 𝑄1
∆𝐻1 =
𝑥

Equação de combustão completa do benzeno: C6H6 + 15/2 O2 → 6 CO2 + 3 H2O (Equação 2)

Sendo a massa molar do benzeno igual a 76 g.mol-1, temos:

∆𝐻 = 𝑄 𝑥 𝑛

− 78 𝑥 𝑄1
∆𝐻2 =
𝑦

Equação química de formação do benzeno a partir do acetileno, utilizando a Lei de Hess: 3 C2H2(g) → C6H6

Multiplicando a primeira reação por três, temos: 3 C2H2(g) + 15/2 O2 → 6 CO2 + 3 H2O

− 26 𝑥 𝑄1 − 78 𝑥 𝑄1
∆𝐻1 = ( )𝑥 3 = ( )
𝑥 𝑥

Invertendo a segunda equação química, temos: 6 CO2 + 3 H2O → C6H6 + 15/2 O2

+ 78 𝑥 𝑄2
∆𝐻2 =
𝑦

Somando as duas equações químicas, temos:

3 C2H2(g) + 15/2 O2 → 6 CO2 + 3 H2O


6 CO2 + 3 H2O → C6H6 + 15/2 O2 +
3 C2H2(g) → C6H6

78 𝑥 𝑄1 78 𝑥 𝑄2 𝑄 𝑄2 𝑘𝑐𝑎𝑙
∆𝐻𝑡𝑜𝑡𝑎𝑙 = ∆𝐻1 + ∆𝐻2 = − 𝑥
+ 𝑦
= − 78 𝑥 { 𝑥1 − }
𝑦 𝑚𝑜𝑙
(Calor liberado por mol)

𝑄 𝑄2 𝑘𝑐𝑎𝑙 1 𝑚𝑜𝑙 𝑑𝑒 𝑍 𝑄 𝑄2 𝑘𝑐𝑎𝑙


Calor liberado por z gramas de benzeno: ∆𝐻𝑡𝑜𝑡𝑎𝑙 = − 78 𝑥 { 𝑥1 − }
𝑦 𝑚𝑜𝑙
𝑥 78
= − 𝑧 𝑥 { 𝑥1 − }
𝑦 𝑚𝑜𝑙

353
1600 FÍSICO-QUÍMICA APLICADA EXERCÍCIOS COMENTADOS - IME – ITA – OLIMPÍADA

Questão 740 – (ITA) A toda reação química corresponde uma variação de energia interna, U, e uma variação de
entalpia, H. Explique em que condições U tem valor igual ao de H.
Resolução: Considerando que o sistema gasoso para a formação do cloreto de hidrogênio apresenta comportamento
ideal, temos a seguinte relação termodinâmica: ∆H = ∆U + ∆n x R x T.

Equação química: H2(g) + Cl2(g) → 2 HCl(g)

∆H = ∆U + (nprodutos − nreagentes ) x R x T = ∆U + {(2 − 2) x R x T }


∆H = ∆U

Questão 741 – (IME) A combustão completa de uma amostra de certo hidrocarboneto gasoso, acíclico, realizada em
recipiente a pressão constante, consumiu 62,5 cm³ de oxigênio e liberou 0,359 kcal. A mistura gasosa continha 50,0
cm³ de um gás que foi totalmente absorvido em hidróxido de potássio. Considerando que todas as medidas foram
realizadas nas CNTP, e que a hidrogenação de um mol desse hidrocarboneto consome dois mols de hidrogênio,
determine o seu calor de formação. Dados: ∆Hf do CO2 = - 94,3 kcal/mol e ∆Hf do H2O = - 68,8 kcal/mol.

Resolução: Como o processo de hidrogenação de 1 mol do hidrocarboneto consume 2 mol de H2, o possível
hidrocarboneto apresenta cadeia acíclica, podendo ser um alcino ou alcadieno, cuja fórmula global é CnH2n-2.
3n−1
Equação química geral da combustão de um alcino ou de um alcadieno: Cn H2n−2 + O2 → nCO2 +
2
(n − 1)H2 O

Dados do problema: A reação consumiu 62,5 cm³ de O e KOH absorve CO 2, assim foi gerado na combustão 50 cm³
de CO2.

Em reações que ocorrem sob pressão constante e temperatura constante, os volumes dos participantes gasosos são
proporcionais ao número de mol.
3n−1
Assim: Cn H2n−2 + 2
O2 → nCO2 + (n − 1)H2 O

VO2 VCO2
=
3n − 1 n
( 2 )

62,50 50
=
3n − 1 n
( 2 )

n=2

Trata-se então do gás acetileno, com fórmula molecular igual a C2H2. Assim a reação de combustão balanceada será:
C2H2 + 5/2 O2 → 2 CO2 + H2O. Nas Condições Normais de Temperatura e Pressão (CNTP), temos a seguinte
proporção em quantidade de matéria - energia:

2 mol de CO2 ---------- ΔH(combustão do acetileno)

50 cm³
mol de CO2 = −0,359 kcal
22,4 x 103 cm³

Informação do problema: ΔH(combustão do acetileno) = - 326,1 kcal

A partir da Lei de Hess, temos: C2H2 + 5/2 O2 → 2 CO2 + H2O

354
1600 FÍSICO-QUÍMICA APLICADA EXERCÍCIOS COMENTADOS - IME – ITA – OLIMPÍADA

0 0 0
∆Hreação = ∑ Hprodutos − ∑ Hreagentes

2 2 C H
−321,60 = [2 x (−94,30) + (−68,80)] − ∆Hformação

C H
2 2
kcal
∆Hformação = + 64,20
mol

Questão 742 – (OLIMPÍADA DE QUÍMICA DO DISTRITO FEDERAL) Qual o ∆H da reação C(g) + 2 H2(g) → CH4(g),
tendo como dados as reações abaixo?
C(g) + O2(g) → CO2(g) ∆H = - 94,1 kcal
H2(g) + ½ O2(g) → H2O(l) ∆H = - 68,3 kcal
CH4(g) + 2O2(g) → CO2(g) + 2 H2O(l) ∆H = - 212,8 kcal

Resolução:

C(g) + O2(g) → CO2(g) ∆H = - 94,1 kcal


Multiplicando a equação química por dois: 2 H2(g) + O2(g) → 2 H2O(l) ∆H = 2 x (- 68,3) kcal
Invertendo a equação química: CO2(g) + 2 H2O(l) → CH4(g) + 2 O2(g) ∆H = + 212,8 kcal

Aplicação da lei de Hess e somando as três equações químicas, temos:

C(g) + O2(g) → CO2(g) ∆H = - 94,1 kcal


2 H2(g) + O2(g) → 2 H2O(l) ∆H = 2 x (- 68,3) kcal
CO2(g) + 2 H2O(l) → CH4(g) + 2 O2(g) ∆H = + 212,8 kcal +
C(g) + 2 H2(g) → CH4(g) ∆H = - 94,1 kcal + [2 x (- 68,3) kcal] + 212,8 kcal = -17,90 kcal

Questão 743 – (OLIMPÍADA NORTE – NORDESTE DE QUÍMICA) Alguns anos atrás, Texas City (Texas – USA), foi
abalada por uma explosão de um depósito de nitrato de amônio, composto muito usado como fertilizante. Este
composto, quando aquecido, pôde decompor exotermicamente em N2O e água, conforme a equação:
NH4NO3(s) → N2O(g) + 2 H2O(g) (Equação 1).
Se o calor liberado nesta reação ficar aprisionado, altas temperaturas sendo atingidas, e assim, o NH 4NO3 pode
decompor explosivamente em N2, H2O e O2.
2 NH4NO3(s) → 2 N2(g) + 4 H2O(g) + O2(g) (Equação 2).
Usando as informações fornecidas abaixo, responda:
a) Qual o calor liberado (à pressão constante de 1 atm e à temperatura de 25°C) na primeira reação?
b) Se 8,00 kg de nitrato de amônio são misturados com alumínio em pó, em excesso estequiométrico, qual a
quantidade de calor produzida (à pressão constante)?
c) Nitrato de amônio reage com alumínio em pó, produzindo Al2O3, segundo a equação:
2 Al(s) + 3 NH4NO3(s) → 3 N2(g) + 6 H2O(g) + Al2O3(s)
Se 8,00 kg de nitrato de amônio são misturados com alumínio em pó, em excesso estequiométrico, qual a quantidade
de calor produzida (à pressão constante)?
Dados a 25°C (kJ.mol-1): Al2O3(g) = - 1675,7; H2O(g) = - 241,8; NH4NO3(s) = - 365,6; N2O(g) = 82,0.

Resolução: Item a) Cálculo da quantidade de calor para a primeira reação química: NH4NO3(s) → N2O(g) + 2 H2O(g)

0 0 0
∆Hreação = ∑ Hprodutos − ∑ Hreagentes

0 0 0 0
∆Hreação = [1 x HNO 2
+ 2 x HH 2O
] − [1 x HNH 4 NO3
]

0
kJ
∆Hreação = [1 x 82,0 + 2 x (−241,8)] − [1 x (−365,6)] = −36,0
mol

355
1600 FÍSICO-QUÍMICA APLICADA EXERCÍCIOS COMENTADOS - IME – ITA – OLIMPÍADA

Item b) Cálculo da quantidade de calor considerando 8,00 kg de nitrato de amônio: 2 NH4NO3(s) → 2 N2(g) + 4 H2O(g) +
O2(g)

0 0
0
∆Hreação = ∑ Hprodutos 0
− ∑ Hreagentes 0
= [2 x HN 2
+ 4 x HH 2O
+ 1 x HO0 2 ] − [2 x HNH
0
4 NO3
]

0
kJ
∆Hreação = [1 x 0 + 4 x (−241,8) + 1 x 0] − [2 x (−365,6)] = −263,0
mol

Pela estequiometria da reação, temos:

160 g de NH4NO3 -------------- (- 263,00) kJ


8000 g de NH4NO3 ------------ Qp
Qp = - 13150,0 kJ

Item c) Cálculo da quantidade de calor considerando 8,00 kg de nitrato de amônio: 2 Al(s) + 3 NH4NO3(s) → 3 N2(g) + 6
H2O(g) + Al2O3(s)

0 0 0
∆Hreação = ∑ Hprodutos − ∑ Hreagentes
0 0 0 0 0
= [3 x HN 2
+ 6 x HH 2O
+ 1 x HAl2 O3
] − [2 x HAl + 3 x HNH 4 NO3
]

0
kJ
∆Hreação = [3 x 0 + 6 x (−241,8) + 1 x (−1675,7)] − [2 x 0 + 3 x (−365,6)] = −2030,0
mol

Pela estequiometria da reação, temos:

240 g de NH4NO3 -------------- (- 2030,00) kJ


8000 g de NH4NO3 ------------ Qp
Qp = - 67666,67 kJ

Questão 744 – (ITA) 300 gramas de gelo a 0°C foram adicionados a 400 gramas de água a 55°C. Determine a
temperatura final do sistema em condição adiabática. Dados: calor de fusão do gelo = 80 cal.g -1; calor específico do
gelo = 0,50 cal.g-1.°C-1; calor específico da água = 1,0 cal.g-1.°C-1.

Resolução: Cálculo da quantidade de calor (Q1) para o processo de fusão do gelo:


Q1 = m x L
Q1 = 300 x 80 = 24000 cal

Cálculo da temperatura final (equilíbrio) do sistema água/gelo:

Q 1 + Q2 = 0
24000 + 400 x 1,0 x (T – 55) = 0
T = - 5°C

A esta temperatura calculada, não há processo de fusão.

Questão 745 – (GRILLO) Considere a equação da capacidade calorífica a pressão constante do brometo de
hidrogênio: Cp = 6,5776.T + 0,9549 x10-3.T + 1,581x10-7.T². A partir desta informação, determine a quantidade de calor
para um mol deste gás, de 0°C a 500°C.

Resolução: Informação do problema: Cp = 6,5776 + 0,9549x10-3.T + 1,581x10-7.T² para 1 mol de brometo de


hidrogênio (HBr). Cálculo da quantidade de calor para o brometo de hidrogênio: ∆H = Q p = n x Cp x dT

∆H = Q p = 1 x (6,5776 + 0,9549 x 10−3 x T + 1,581 x 10−7 x T −2 ) dT

356
1600 FÍSICO-QUÍMICA APLICADA EXERCÍCIOS COMENTADOS - IME – ITA – OLIMPÍADA

773 773 773


∆H = Q p = ∫ 6,5776 x dT + ∫ (0,9549 x 10−3 x T) x dT + ∫ (1,581 x 10−7 x T 2 ) x dT
273 273 273

773 773
T2 T³
∆H = Q p = 6,5776 x T|773
273 + 0,9549 x 10 −3
x ( )| + (1,581 x 10−7 x ( )|
2 273 3 273

(773)² (273)²
∆H = Q p = 6,5776 x (773 − 273) + 0,9549 x 10−3 x { − }
2 2
(773)³ (273)³
+ 1,581 x 10−7 x { − }
3 3

cal
∆H = Q p = 3288,8 + 249,71 + 23,27 = + 3561,78
mol

Questão 746 – (IME) Em função do calor de formação do dióxido de carbono (ΔH°f,CO2); do calor de formação do
vapor d’água (ΔH°f,H2O(g)); e do calor da combustão completa de uma mistura de metano e oxigênio, em proporção
estequiométrica (ΔHr), deduza a expressão do calor de formação do metano (ΔH°f,CH4).

Resolução: Dados do problema:

Calor de formação do gás metano: C(s) + 2 H2(g) → CH4(g) ΔH°f,CH4 = ?

Calor de formação do dióxido de carbono: C(s) + O2(g) → CO2(g) (ΔH°f,CO2)

Calor de formação de vapor d´água: H2(g) + ½ O2(g) → H2O(g) (ΔH°f,H2O)

Calor de combustão do gás metano: CH4(g) + 2 O2(g) → CO2(g) + 2 H2O(g) (ΔHr)

Aplicando a lei de Hess para a determinação da entalpia de formação do gás metano (CH4):

Reação de formação do dióxido de carbono: C(s) + O2(g) → CO2(g) (ΔH°f,CO2)

Multiplicando por dois a reação de formação do vapor d´água: 2 H2(g) + O2(g) → 2 H2O(g) (2 x ΔH°f,H2O)

Invertendo a reação de combustão do gás metano: CO2(g) + 2 H2O(g) → CH4(g) + 2 O2(g) (- ΔHr)

Somando as equações químicas, temos:

C(s) + O2(g) → CO2(g) (ΔH°f,CO2)


2 H2(g) + O2(g) → 2 H2O(g) (2 x ΔH°f,H2O)
CO2(g) + 2 H2O(g) → CH4(g) + 2 O2(g) (- ΔHr) +
C(s) + 2 H2(g) → CH4(g) ΔH°f,CH4 = (ΔH°f,CO2) + 2 x ΔH°f,H2O - ΔHr

Logo, a entalpia de formação do gás metano fica representado através da seguinte expressão matemática: ΔH°f,CH4 =
ΔH°f,CO2 + 2 x ΔH°f,H2O - ΔHr.

357
1600 FÍSICO-QUÍMICA APLICADA EXERCÍCIOS COMENTADOS - IME – ITA – OLIMPÍADA

Questão 747 – O sulfeto de hidrogênio é aquecido de 77°C para 227°C dentro de um reator fluidixado. Determine a
variação de entalpia por ocasião deste aquecimento, sabendo que a capacidade calorífica a pressão constante média
é dada pela seguinte equação: Cp = 6,037 + 5,886 x 10-3.T + 0,387 x 10-5.T-2 – 1,175 x 10-6.T2, onde Cp = cal/mol.K.

Resolução: Base de cálculo: 1 mol de sulfeto de hidrogênio [H2S(g)]

Cálculo da variação de entalpia (∆H) do H2S em que dH = Q p = n x Cp x dT, onde Qp é a quantidade de calor a
pressão constante.

∆H = (6,037 + 5,886 x 10−3 x T + 0,387 x 10−5 x T −2 − 1,175 x 10−6 x T 2 )dT


500 500 500
∆H = ∫ 6,037 x dT + ∫ (5,886 x 10−3 x T) x dT − ∫ (1,175 x 10−6 x T 2 ) x dT
350 350 350

500 500
T² T³
∆H = 6,037 x T|500
350 + 5,886 x 10 −3
x ( )| − 1,175 x 10−6 x ( )|
2 350 3 350

(500)² (350)²
∆H = 6,037 x (500 − 350) + 5,886 x 10−3 x { − }
2 2
(500)³ (350)³
− 1,175 x 10−6 x { − }
3 3

∆H = 6,037 x 150 + 5,886 x 10−3 x (125000 − 61250)


− 1,175 x 10−6 x (4166666,67 − 14291666,67)

∆H = 6,037 x 150 + 5,886 x 10−3 x (63750) − 1,175 x 10−6 x (27375000)


cal kcal
= 905,55 + 375,23 − 32,16 = 1248,62 (≅ 1,25 )
mol mol

Questão 748 – (IME) A reforma com vapor d´água, a temperatura altas, é um método industrial para produção de
hidrogênio a partir de metano. Calcule a entalpia de reação desse processo. Dados:
I. Entalpias de combustão:
C(grafite) ... ∆H° = - 394 kJ.mol-1
H2(g) ... ∆H° = - 286 kJ.mol-1 (forma água líquida)
CH4(g) ... ∆H° = - 890 kJ.mol-1 (forma água líquida)
II. CO(g) + H2(g) → C(grafite) + H2O(g) ∆H° = - 131 kJ.mol-1

Resolução:

Equação química CH4(g) + H2O(g) → 3 H2(g) + CO(g) ΔH° = ?

Calor de combustão do carbono grafite: C(grafite) + O2(g) → CO2(g) ΔH° = - 394 kJ.mol-1

Calor de combustão do hidrogênio gasoso: H2(g) + ½ O2(g) → H2O(l) ΔH° = - 286 kJ.mol-1

Calor de combustão do gás metano: CH4(g) + 2 O2(g) → CO2 (g) + 2 H2O(l) ΔH° = - 890 kJ.mol-1

Equação química fornecida: CO(g) + H2(g) → C(grafite) + H2O(g) ΔH° = - 131 kJ.mol-1

Aplicando a lei de Hess para a determinação da equação representativa do gás metano com o vapor d´água, temos:

CO2(g) → C(grafite) + O2(g) ΔH° = + 394 kJ.mol-1


CH4(g) + 2 O2(g) → CO2 (g) + 2 H2O(l) ΔH° = - 890 kJ.mol-1

358
1600 FÍSICO-QUÍMICA APLICADA EXERCÍCIOS COMENTADOS - IME – ITA – OLIMPÍADA

C(grafite) + H2O(g) → CO(g) + H2(g) ΔH° = + 131 kJ.mol-1


2 H2O(l) → 2 H2(g) + O2(g) ΔH° = 2 x (+ 286) kJ.mol-1 +
CH4(g) + H2O(g) → CO(g) + 3 H2(g) ΔH° = + 394 kJ.mol-1 - 890 kJ.mol-1 + 131 kJ.mol-1 + 2 x (+ 286) kJ.mol-1 =
+ 207 kJ.mol-1

Questão 749 – (U. S. NATIONAL CHEMISTRY OLYMPIAD) A combustão de etano produz dióxido de carbono e
água líquida a 25°C.
a) Escreva a equação para esta reação.
b) Sabendo que ∆H°combustão para o etano sob estas condições é -1560 kJ.mol-1 de etano, calcule:
i.∆H°f para o etano;
Dado:

ΔH° (kJ.mol-1)
CO2(g) - 393,5
H2O(l) - 285,8
ii.A energia de ligação para a ligação C = O.
Energia de ligação (kJ.mol-1)
C - C 347
H - C 413
H - O 164
O = O 195
c) Sabendo que ∆G° = - 1467,5 kJ.mol-1. Calcule ∆S° para esta reação em J.mol-1.K-1.

Resolução:

Item a) Equação química da combustão completa do etano: C2H6(l) + 7/2 O2(g) → 2 CO2(g) + 3 H2O(l).

Item b) i

0 0 0 7
0
∆Hreação = ∑ Hprodutos 0
− ∑ Hreagentes = [(2 x HCO + 3 x HH 2O
) − (1 x HC02 H6 + x HO0 2 )]
2 2
7
− 1560 = [2 x (−393,5) + 3 x (−285,8)] − [(1 x HC02 H6 + x 0)]
2

− 1560 = −787 − 857,40 − HC02 H6

− 1560 = −1644,40 − HC02 H6

HC02 H6 = −84,40 kJ

ii

Cálculo das entalpias tanto para os reagentes quanto para os produtos.

REAGENTES:
0
7
Hreagentes = + [1 x 1 x HC−C + 6 x HC−H + x HO=O ]
2

0
7
Hreagentes = + [347 + 6 x (413) + x 495]
2

359
1600 FÍSICO-QUÍMICA APLICADA EXERCÍCIOS COMENTADOS - IME – ITA – OLIMPÍADA

0
kJ
Hreagentes = + 4557,50
mol

PRODUTOS:
0
Hprodutos = −[2 x 2 x HC=O + 3 x 2 x HO−H ]

0
Hprodutos = −[4 x HC=O + 6 x HO−H ]

0
Hprodutos = −[4 x HC=O + 6 x (464)]

0
Hprodutos = −[4 x HC=O + 2784]

Finalizando:
0
∆Hreação = − 4 x HC=O − 2784 + 4557,50

− 1560 = − 4 x HC=O − 2784 + 4557,50

− 1560 = − 4 x HC=O + 1773,50

− 1560 − 1773,50 = − 4 x HC=O

− 1560 − 1773,50 kJ
HC=O = = + 833,37
−4 mol

Item c) Cálculo da entropia padrão (∆S°):∆G° = ∆H° - T x ∆S°

- 1467500 = - 1560500 – (25 + 273) x ∆S°


∆S° = - 312,50 J.mol-1.K-1

Questão 750 – (U. S. NATIONAL CHEMISTRY OLYMPIAD) A entalpia de combustão do octano líquido, C8H18(l) para
a formação de produtos gasosos, é – 5090 kJ.mol-1. Use este valor para responder as questões abaixo, assumindo
uma temperatura de 100°C.
a) Escreva a equação balanceada para a combustão completa do octano líquido.
b) Determine a entalpia molar de formação, ∆H°f, para o estado líquido.
Dados: H°CO2 = - 393,5 kJ.mol-1; H°H2O = - 241,8 kJ.mol-1.
c) Calcule o valor da energia interna, para a reação de combustão.
d) Se ∆G° para a combustão do octano é – 5230 kJ.mol-1, calcule o valor da ∆S°.

Resolução:

Item a) Equação química da combustão do octano líquido: C8H18(l) + 25/2 O2(g) → 8 CO2(g) + 9 H2O(g).
0 0 0
Item b) Cálculo da entalpia molar de formação do octano líquido: ∆Hreação = ∑ Hprodutos − ∑ Hreagentes =
0 0 25
[8 x HCO 2
+ 9 x HH 2O
] − [1 x HC08 H18 + 2
x HO0 2 ]
25
− 5090 = [8 x (−393,5) + 9 x (−241,8)] − [1 x HC08 H18 + x 0]
2

− 5090 = [8 x (−393,5) + 9 x (−241,8)] − 1 x HC08 H18 ]

− 5090 = −5324,20 − HC08 H18

360
1600 FÍSICO-QUÍMICA APLICADA EXERCÍCIOS COMENTADOS - IME – ITA – OLIMPÍADA

kJ
HC08 H18 = −234,20
mol

Item c) Cálculo da energia interna, a partir da seguinte relação estequiométrica: ∆H = ∆U + pV, levando em
consideração que os gases apresentam comportamento ideal (pV = nRT)

∆H = ∆U + (nprodutos – nreagentes) x R x T
- 5090000 = ∆U + (17 – 12,5) x (8,314) x (100 + 273)
∆U = - 5090000 – 13955,089 J.mol-1 = - 5103,95 kJ.mol-1

Item d) Cálculo da entropia (∆S°): ∆G° = ∆H° - T x ∆S°

- 5230 + 5090 = - (100 + 273) x ∆S°

∆S° = + 0,375 kJ.mol-1.K-1

Questão 751 – (GRILLO) Dadas as reações químicas (1) e (2), determine a variação de entalpia da reação (3):
(1) H2(g) + I2(g) → 2HI(g) H(1) = + 52,96 kJ/mol
(2) 2H2(g) + O2(g) → 2H2O(g) H(2) = - 483,64 kJ/mol
(3) 4HI(g) + O2(g) → 2 I2(g) + 2H2O(g) H(3) = ?

Resolução: Para a resolução deste exercício será necessário aplicar a lei de Hess. Invertendo a reação (1) e
multiplicando por dois, temos:

4 HI(g) → 2 H2(g) + 2 I2(g) H(1) = 2 x (- 52,96) kJ.mol-1


2 H2(g) + O2(g) → 2 H2O(g) H(2) = - 483,64 kJ.mol -1

4 HI(g) + O2(g) → 2 I2(g) + 2 H2O(g) H(3) = -105,92 – 483,64 = - 589,56 kJ.mol-1

Questão 752 – (OLIMPÍADA PORTUGUESA DE QUÍMICA) Dados um copo com água, outro com HCl(aq) e outro com
NaOH(aq) concentrados, todos a mesma temperatura, como fazer para retirar, o mais rapidamente possível, um
diamante retido num cubo de gelo, usando estes líquidos. Justifique.

Resolução: Para retirar o diamante rapidamente que está preso no cubo de gelo, será necessário misturar ácido
clorídrico com hidróxido de sódio, o que vai gerar uma reação de neutralização. Para este processo, trata-se de uma
reação exotérmica, logo colocando na presença do cubo de gelo, haverá o derretimento de forma rápida e com isso
o diamante ficará livre.

Questão 753 – A 1000 K, o valor da entalpia para a síntese da amônia N2(g) + 3 H2(g) → 2 NH3(g), é igual a – 123,77
kJ.mol-1. Informação do problema: N2(g) + 3 H2(g) → 2 NH3(g), ∆H°1000K = -123,77 kJ.mol-1.

̅̅̅
Substância 𝐶𝑝⁄
𝑅
N2(g) 3,502
H2(g) 3,466
NH3(g) 4,217

Determine o valor da entalpia de síntese da amônia à 300 K, para cada mol de NH3(g).

361
1600 FÍSICO-QUÍMICA APLICADA EXERCÍCIOS COMENTADOS - IME – ITA – OLIMPÍADA

̅
𝐶𝑝
Resolução: Cálculo da variação da capacidade calorífica molar: ( 𝑅 ) = 𝐶𝑝̅ 𝑝𝑟𝑜𝑑𝑢𝑡𝑜𝑠 − 𝐶𝑝̅ 𝑟𝑒𝑎𝑔𝑒𝑛𝑡𝑒𝑠
𝑅𝑒𝑎çã𝑜
C̅p
( ) = 2 x (4,217) − [1 x (3,502) + 3 x (3,466)]
R Reação

C̅p
( ) = 8,434 − 13,90
R Reação

C̅p
( ) = −5,47
R Reação

0 300
Utilizando a lei de Kirchoff: ∆H300K = −123770 − 5,47 x ∫1000 dT
0
J
∆H300K = −123770 − 5,47 x (300 − 1000) = −123770 + 3829 = −119941
mol

A partir da relação estequiométrica da síntese da amônia, há a formação de dois mol. Logo, para 1 mol temos:
2 mol de NH3(g) ---------- (- 119941)
1 mol de NH3(g) ---------- ∆H° (J.mol-1)
∆H° = - 59971,9 J.mol-1 (- 59,97 kJ.mol-1)

Questão 754 – A capacidade calorífica do óxido de chumbo sólido, PbO, é dada pela seguinte equação da capacidade
calorífica: Cp = 44,35 + 1,67 x 10-3T. A partir desta informação, determine a variação de entalpia do óxido, à pressão
constante, de 500 K para 300 K.

Resolução: Cálculo do valor da entalpia (∆H) do óxido de chumbo (PbO): dH = Q p = n x Cp x dT

300 500
∆H = ∫ 44,35 x dT + ∫ (1,67 x 10−3 x T) x dT
500 350

300

∆H = 44,35 x T|300
500 + 1,67 x 10
−3
x( )|
2 500

(300)² (500)²
∆H = 44,35 x (300 − 500) + 1,67 x 10−3 x { − }
2 2

J kJ
∆H = − 8870 + 1,67 x 10−3 x {45000 − 125000} = − 9003,6 (−9,0 )
mol mol

Questão 755 – (ITA) Considere a reação de combustão do composto X, de massa molar igual a 27,7 g.mol –1,
representada pela seguinte equação química balanceada: X(g) + 3 O2(g) → Y(s) + 3 H2O(g); ∆H° = - 2035 kJ.mol-1.
Calcule o valor numérico, em kJ, da quantidade de calor liberado na combustão de:
a) 1,0 X 10³ g de X;
b) 1,0 x 10² mol de X;
c) 2,6 x 1022 moléculas de X;
d) uma mistura de 10,0 g de X e 10,0 g de O2.

362
1600 FÍSICO-QUÍMICA APLICADA EXERCÍCIOS COMENTADOS - IME – ITA – OLIMPÍADA

Resolução: Considere a reação de combustão do composto X, de massa molar igual a 27,7 g.mol –1, representada
pela seguinte equação química:

X(g) + 3 O2(g) → Y(s) + 3 H2O(g); ∆H° = - 2035 kJ.mol-1.

Calcule o valor numérico, em kJ, da quantidade de calor liberado na combustão de:

Item a) 1,0 x 10³ g de X

X(g) + 3 O2(g) → Y(s) + 3 H2O(g); ∆H° = - 2035 kJ.mol-1.

1 mol de X(g) ---------- 27,7 g ---------- (- 2035 kJ)


1000 g ---------- ∆H°
∆H° = (- 7,35 x 104 kJ)

Item b) 1,0 x 10² mol de X.

X(g) + 3 O2(g) → Y(s) + 3 H2O(g); ∆H° = - 2035 kJ.mol-1.

1 mol de X(g) --------------- (- 2035 kJ)


100 mol de X(g) ------------ ∆H°
∆H° = - 2,035 x 105 kJ

Item c) 2,6 x 1022 moléculas de X

X(g) + 3 O2(g) → Y(s) + 3 H2O(g); ∆H° = - 2035 kJ.mol-1

1 mol de X(g) --------------- 6,02 x 1023 moléculas de X --------- (- 2035 kJ)


2,6 x 1022 moléculas de X ----------- ∆H°
∆H° = - 87,89 kJ

Item d) uma mistura de 10,0 g de X e 10,0 g de O2


m 10,0 m
Cálculo do número de mol para cada reagente químico: nX = <MM>
X
= 27,7
= 0,361 mol e nO2 = <MM>
X
=
O2 X
10,0 0,3125
32,0
= 3
= 0,104 mol

Através do número de mol de cada reagente químico, o oxigênio molecular é o reagente limitante, logo:
X(g) + 3 O2(g) → Y(s) + 3 H2O(g); ∆H° = - 2035 kJ.mol-1.

3 mol de O2(g) --------------- (3 x 32) g de O2(g) -------------------- (- 2035 kJ)


10 g de O2(g) --------------------------- ∆H°
∆H° = (- 211,98 kJ)

Questão 756 – (IME) Uma pequena célula eletroquímica blindada por eletrodos de alumínio e de níquel, deve operar
a temperatura constante de 298 K. Para tanto, recebe uma camisa de refrigeração, isolada do meio externo, contendo
100 g de água. Supondo que a célule transfere ao exterior, de maneira reversível, uma carga de 1 Faraday, calcule a
elevação da temperatura que ocorrerá na água dentro da camisa de refrigeração. Ademais, sabe-se que essa célula
apresenta uma variação de potencial na razão de 1,5 x 10-4 V/K. Considere que o calor específico da água de
refrigeração é de 4,20 J/g.K.

Resolução: Cálculo da energia (E): E = 1,50 x 10-4 (V/K) x 96500 C = 4313,55 J

Cálculo da variação da temperatura (∆T):

363
1600 FÍSICO-QUÍMICA APLICADA EXERCÍCIOS COMENTADOS - IME – ITA – OLIMPÍADA

E = Qs = m x c x ∆T, em que Qs é a quantidade de calor sensível.

E = Qs = m x c x ∆T

4313,55 = 100 x 4,2 x ∆T

4313,55 = 420 x ∆T

∆T = + 10,27 K

Questão 757 – Defina a ciência Termodinâmica.

Resposta: A Termodinâmica Clássica ou também conhecida como Termodinâmica de Equilíbrio é uma das áreas
consolidadas principalmente pela Física, por uma estrutura de conhecimento bem definida e autoconsistente. A
essência da estrutura teórica da Termodinâmica trata-se de um conjunto de leis naturais que governam o
comportamento de sistemas físicos macroscópicos. As leis termodinâmicas foram estudadas através de observações
puramente experimentais e são, em grande parte, independentes de quaisquer hipóteses relativas à natureza
microscópica da matéria.
Questão 758 – Enuncie: sistema, sistema aberto, sistema fechado e sistema isolado.

Resposta: Sistema → Conjunto de corpos materiais (sólido, líquido ou gasosos) de objeto de interesse podendo
estar junto ou isolado, reagindo quimicamenteuns uns com os outros, mas que possa ser medido e controlado através
de certas grandezas termodinâmicas, tais como a pressão, volume e temperatura que são importantes para a
termodinâmica.
Sistema aberto → troca quantidade de massa e energia com o meio externo (vizinhança).
Sistema fechado → troca quantidade de energia com o meio externo, mas não troca quantidade de massa.
Sistema isolado → Sistema idealizado, que não troca energia e nem massa.
Questão 759 – Defina o estudo da Termoquímica.

Resposta: Um dos ramos da Físico-química, mas precisamente da primeira lei da termodinâmica que esuda a
quantidade de calor de processos químicos (reações químicas) a pressão constante.
.Questão 760 – Enuncie a lei zero da termodinâmica.

Resposta: Considere três corpos, designados pelas letras A, B e C. Se A está em equilíbrio térmico com B e em
equilíbrio térmco com C, então B e C também estão em equilíbrio térmico.

Questão 761 – Enuncie a primeira lei da termodinâmica.

Resposta: Quando o estado de um sistema termodinâmico se altera de um estado A para um estado B, a soma
algébrica das diversas transformações de energia que se efetuam sobre o sistema só depende da energia interna do
estado inicial A (UA) e da energia interna do estado final B (UB). Se depois de várias transformações de energia,o
sistema termodinâmico volta ao estado inicial A, logo a variação de energia interna será igual a zero (ΔU = 0).
Questão 762 – Enuncie a segunda lei da termodinâmica.

Resposta: Se o calor flui de um corpo A para um corpo B, ambos estando em contato térmico, o fluxo de calor inverso
não é possível de acontecer.

Questão 763 – Apresente o enunciado da segundo lei da termodinâmica segundo o postulado de Kelvin-Planck.

364
1600 FÍSICO-QUÍMICA APLICADA EXERCÍCIOS COMENTADOS - IME – ITA – OLIMPÍADA

Resposta: É impossível construir uma máquina térmica que, operando num ciclo, extraia uma quantidade de calor de
uma fonte e a transforme totalmente em trabalho. Em outras palavras, não é possível construir uma máquina de
movimento perpétuo de segunda espécie.

Questão 764 - Por que a capacidade calorífica a pressão constante é sempre maior que a capacidade calorífica a
volume constante para uma mesma substância?

Resposta: Por definição a entalpia é definida por dH = dQ = CpdT e a energia interna a volume constante por dU =
CvdT. A entalpia também pode ser definida por H = U + pV, sendo assim temos que dH = dU + pdV.
Substituindo a entalpia e a energia interna na expressão nos fornece a seguinte relação termodinâmica:
Cp dT = Cv dT + pdV
pdV
Cp = Cv + dT
. Sendo assim Cp de qualquer substância sempre será maior que o Cv, devido a diferença existente
a temperatura constante proveniente do trabalho realizado. Quando se tem um mol (1 mol) de gás ideal a expressão
passa a ser: Cp − Cv = R, onde R é a constante dos gases ideais.

Questão 765 - Enuncie a terceira lei da termodinâmica ou lei de Nernst.

Resposta: O zero absoluto é inatingível, ou também pode ser explicado que o estado de ordem absoluta só é
alcançado quando a temperatura for igual a zero, ou seja, para T = 0K → dS = 0.

Questão 766 – O que representa a entalpia?

Resposta: Para um processo a pressão constante durante um processo que envolve somente trabalho mecânico é
dado pela seguinte equação termodinâmica: 𝛿𝑊 = −𝑝𝑑𝑉
Pela primeira lei da termodinâmica, temos: 𝑑𝑈 = 𝛿𝑄𝑝 + 𝛿𝑊

𝑑𝑈 = 𝛿𝑄𝑝 − 𝑝𝑑𝑉, onde: Qp é o calor trocado entre o sistema e as suas vizinhanças a pressão constante.

Daí:
dU = δQ p − pdV

dQ p = dU + pdV

dQ p = dU + pdV + Vdp

dQ p = d(U + pV)

Como a expressão U + pV apresenta somente funções de estado, ela também é uma propriedade de estado
extensiva (dependente da massa), definida como Entalpia (H), isto é:
dQ p = d(U + pV)

H = U + pV
Portanto, para um processo a pressão constante, dQ p = dH.

Questão 767 – Defina calor específico.

Resposta: É a quantidade de calor necessário para fornecer à unidade de uma determinada substância, de modo
que a temperatura aumente em um grau (1°C).

Questão 768 – Defina caloria.

365
1600 FÍSICO-QUÍMICA APLICADA EXERCÍCIOS COMENTADOS - IME – ITA – OLIMPÍADA

Resposta: Uma caloria é a quantidade de calor que é necessária para aumentar de 14,50°C para 15,50°C a
temperatura de 1 g de água.

Questão 769 – (IME) Em um gráfico de pressão versus volume, represente o diagrama do ciclo idealizado por Carnot
(máquina térmica) para uma transformação cíclica, ininterrupta, e sem perdas de calor e trabalho, e vice-versa.
Identifique e denomine as quatro etapas dessa transformação cíclica.

Resposta: O ciclo de Carnot é constituído de quatro processos reversíveis sucessivos. São eles:

1 – Processo Isotérmico

2 – Processo Adiabático

3 – Processo Isotérmico

4 – Processo Adiabático

Ou seja, no ciclo de Carnot há duas isotermas e duas adiabáticas.

Questão 770 - (IME) Considere o diagrama de fase hipotético representado esquematicamente na figura ao lado. O
que representa os pontos A, B, C, D e E?

Resposta:

A – Equilíbrio sólido-vapor
B – Ponto Triplo
C – Equilíbrio sólido-líquido
D – Equilíbrio líquido-vapor
E – Ponto Crítico

Observação:
Pc – Pressão crítica
Tc – Temperatura crítica

Questão 771 – Em poucas palavras, faça um paralelo entre a primeira e a segunda lei da termodinâmica.

366
1600 FÍSICO-QUÍMICA APLICADA EXERCÍCIOS COMENTADOS - IME – ITA – OLIMPÍADA

Resposta: O segundo princípio da termodinâmica apareceu pelo fato da primeira lei não ser suficiente (na época) em
resolver problemas termodinâmicos. Enquanto o primeiro é importante para estabelecer o sentido das transformações
naturais e também para fornecer as condições de equilíbrio termodinâmico de uma transformação, o segundo já não
é. Enquanto a primeira lei expressa uma igualdade, a segunda lei é apresentada por uma desigualdade. A primeira
lei interessa apenas as relações energéticas, o que para a segunda lei dá o sentido em que elas ocorrem.

Questão 772 – (ITA) Retardantes de chama são substâncias que atenuam e/ou inibem o processo de combustão de
um material. Considere os seguintes fenômenos:
I. Criação de um dissipador de calor usando um composto que se decompõe em um processo altamente exotérmico,
gerando produtos voláteis não combustíveis.
II. Aumento da transferência de calor na superfície em combustão por eliminação do material fundido.
III. Envenenamento da chama pela evolução de espécies químicas que capturam os radicais H ou OH que são
atrativos na propagação da termoxidação da chama.
IV. Limitação da transferência de calor e massa pela criação de uma camada de carbonização isolante na superfície
do material sólido em combustão.
Assinale a opção que apresenta corretamente o(s) fenômeno(s) que pode(m) ser atribuído(s) a ações de retardantes
de chama.
a) Apenas I e II
b) Apenas I e IV
c) Apenas II, III e IV
d) Apenas III
e) Todos

Resolução: Alternativa C.
Retardante de chama é uma substância química que apresenta como característica principal na diminuição da chama,
acrescentados em uma grande variedade de produtos.
I. A afirmativa está incorreta, pois o processo tem que ser endotérmico e não exotérmico.
II. Afirmativa correta.
III. Afirmativa Correta.
IV. Afirmativa Correta.

Questão 773 – Descreva processo reversível x processo irreversível.

Resposta: Processo reversível = Em bases termodinâmicas caracteriza-se pelo fato da entropia ser igual a zero
(nula).

Processo irreversível = Em bases termodinâmicas, um processo é caracterizado como irreversível se a variação da


entropia for maior que zero (positiva).

Questão 774 – Descreva processo de condução de calor.

Resposta: Primeira definição: Quando a transferência de energia ocorre em um meio estacionário, que pode ser um
sólido ou também um fluido, em virtude de um gradiente de temperatura (diferença de temperatura). A este processo
de transmissão de calor é definido como transferência de calor por condução.

Segunda definição: Troca de energia entre as partes de um meio contínuo que, estando em diferentes temperaturas,
transferem energia térmica pela transferência de energia cinética entre as partículas individuais ou grupo de partículas,
no nível atômico.

Gases: choque entre as partículas.

367
1600 FÍSICO-QUÍMICA APLICADA EXERCÍCIOS COMENTADOS - IME – ITA – OLIMPÍADA

Metais: movimento de elétrons livres.


Líquidos e outros sólidos: vibrações de estrutura reticular.

Questão 775 – Descreva processo de convecção de calor.

Resposta: A transferência de calor por convecção se dá quando a transferência de energia ocorre entre uma
superfície e um fluido em movimento em virtude da diferença de temperatura.

Questão 776 – Descreva processo de transmissão de calor por radiação.

Resposta: Processo de transmissão de calor através de ondas eletromagnéticas (ondas de calor). A energia emitida
por um corpo (energia radiante) se propaga até o outro, através do espaço que os separa.

Questão 777 – (GRILLO) Um recipiente apresenta 2,50 mol de argônio a uma temperatura igual a 327°C. Este gás
é comprimido por um pistão com uma pressão de aproximadamente igual a 40 atmosferas. Depois de um certo
momento, a pressão do sistema fica reduzida a ¼ do estado inicial e com isso o gás se expande adiabaticamente e
reversível. A partir destas informações, calcule os seguintes itens apresentados a seguir.
a) o volume no estado inicial;
b) o volume no estado final;
c) a temperatura no estado final.

Resposta:

Item a) Considerando que o argônio (Ar) apresenta comportamento ideal, temos a seguinte situação termodinâmica
no estado inicial e final.
Situação inicial:
Pinicial = 40 atm
Vinicial = ?
Tinicial = 327 + 273 = 600 K

Situação Final:
Pfinal = ¼ . (40 atm) = 10 atm
Vfinal = ?
Tfinal = ?

Cálculo do volume inicial a partir da equação dos gases ideais: 40 x Vinicial = 2,50 x (0,08206) x (327 + 273)
40 x Vinicial = 123,09

123,09
Vinicial = = 3,08 L
40
Item b) Cálculo do volume final (Vfinal), sendo primeiramente necessário calcular a capacidade calorífica a pressão
constante e posteriormente o coeficiente termodinâmico (γ), sendo o gás argônio um gás monoatômico. Então, a
capacidade calorífica a volume constante é igual CV = (3R/2). Logo, a capacidade calorífica a pressão constante será:
Cp − CV = R

3R
Cp − =R
2
3R 5R
Cp = R + =
2 2

368
1600 FÍSICO-QUÍMICA APLICADA EXERCÍCIOS COMENTADOS - IME – ITA – OLIMPÍADA

Cálculo da constante termodinâmica (γ) que relaciona a capacidade calorífica a pressão constante (C p) e a volume
C
constante (Cv): γ = Cp
V

5R
(2) 5
γ= = = 1,67
3R
(2) 3
γ γ
Cálculo do volume final (Vfinal), sabendo que o processo é adiabático: Pinicial x Vinicial = Pfinal x Vfinal
1,67
40 x (3,08)1,67 = 10 x Vfinal

1,67 40 x (3,08)1,67
Vfinal =
10

1,67
40 x (3,08)1,67
Vfinal = √ = 7,06 L
10
γ−1 γ−1
Item c) Cálculo da temperatura final (Tfinal), sendo o processo adiabático: Tinicial x Vinicial = Tfinal x Vfinal

γ−1
Tinicial x Vinicial Vinicial γ−1
Tfinal = γ−1 = Tinicial x ( )
Vfinal Vfinal

3,08 L 1,67−1
Tfinal = (327 + 273) x ( )
7,06 L

Tfinal = 600 x (0,44)0,67 = 346,15 K

Questão 778 – Dez gramas de gás nitrogênio se encontra inicialmente a 17°C. Esta massa gasosa é comprimida
adiabaticamente e reversívelmente a partir de um volume inicial de 8,0 litros para um volume final igual a 5,0 litros. A
partir desta informação, calcule a temperatura final e o trabalho realizado pelo gás. Além disso, calcule a variação da
energia interna (∆U) e a variação da entalpia (∆H). Informação para a resolução do problema: Cp = (7/2)R.
Resolução:
Situação Inicial:
Vinicial = 8,0 L
Tinicial = 17 + 273 = 290 K

Situação Final:
VFinal = 5,0 L
TFinal = ?

Cálculo da capacidade calorífica a volume constante (CV): Cp − CV = R

7R
− CV = R
2
7R 5R
CV = −R=
2 2
7R
Cp ( ) 7
Cálculo da constante termodinâmica (γ): γ = C = 2
5R = 5 = 1,40
V ( )
2

γ−1 γ−1
Cálculo da temperatura final (TF), considerando o processo adiabático: Tinicial x Vinicial = Tfinal x Vfinal

369
1600 FÍSICO-QUÍMICA APLICADA EXERCÍCIOS COMENTADOS - IME – ITA – OLIMPÍADA

γ−1
Tinicial x Vinicial Vinicial γ−1
Tfinal = γ−1 = Tinicial x ( )
Vfinal Vfinal

8,0 1,40−1
Tfinal = 290 x ( )
5,0

Tfinal = 290 x (1,60)0,40

Tfinal = 290 x 1,21 = 351 K


mN 10
Cálculo do número de mol de gás nitrogênio: nN2 = <MM>2 = 28 = 0,36 mol
N2

5
Cálculo da variação da energia interna (ΔU): ∆U = n x CV x ∆T = 0,36 x 2 x 8,314 x (350 − 290) = 448,96 J
7
Cálculo da variação da entalpia (ΔH): ∆H = n x Cp x ∆T = 0,36 x x 8,314 x (350 − 290) = 628,54 J
2

Como o processo ocorre de forma adiabática (Q = 0), o trabalho realizado será igual a variação da energia interna,
através da primeira lei da termodinâmica: ∆U = Q + W

∆U = 0 + W

∆U = W = + 448,96 J

Questão 779 – Considere a seguinte situação termodinâmica: em um recipiente são colocados 10 mol de gás
nitrogênio molecular a uma temperatura igual a 27°C por um pistão a 40 atmosferas de pressão. Depois de um certo
momento do processo, a pressão do sistema é reduzida para 10 atm e o gás se expande de forma adiabaticamente.
Sabendo que a capacidade calorífica a volume constante é igual a 20,8 J.mol-1.K-1, calcule cada item abaixo:
a) O volume inicial;
b) A temperatura final e o volume final;
c) A quantidade de calor;
d) a variação da energia interna;
e) a variação da entalpia;
f) o trabalho realizado.

Resolução: Item a) Considerando que o gás nitrogênio (N2) apresenta comportamento ideal, temos:

Situação Inicial:
Pinicial = 40 atm
Vinicial = ?
Tinicial = 27 + 273 = 300 K

Situação Final:
PFinal = 10 atm
VFinal = ?
TFinal = ?

Cálculo do volume inicial: 40 x Vinicial = 10 x (0,08206) x (27 + 273)


40 x Vinicial = 10 x 0,08206 x 300
Vinicial = 6,15 L

Item b) Para o cálculo da temperatura final, sendo primeiramente necessário calcular a capacidade calorífica a pressão
constante (Cp), coeficiente termodinâmico (γ) e o volume final (V2):

370
1600 FÍSICO-QUÍMICA APLICADA EXERCÍCIOS COMENTADOS - IME – ITA – OLIMPÍADA

Cálculo da capacidade calorífica a volume constante (CV): Cp − CV = R

Cp − 20,8 = 8,314

J
Cp = 20,8 + 8,314 = 29,11
mol x K
C 29,11
Cálculo da constante termodinâmica (γ): γ = Cp = 20,8
= 1,40
V

γ γ
Cálculo do volume final (VFinal): Pinicial x Vinicial = Pfinal x Vfinal
1,40
40 x (6,15)1,40 = 10 x Vfinal

1,40 40 x (6,15)1,40
Vfinal =
10

1,40
40 x (6,15)1,40
Vfinal = √
10
1,40
Vfinal = √50,87 = 16,55 L
γ−1 γ−1
Cálculo da temperatura final (TFINAL): Tinicial x Vinicial = Tfinal x Vfinal
γ−1
Tinicial x Vinicial Vinicial γ−1
Tfinal = γ−1 = Tinicial x ( )
Vfinal Vfinal

Vinicial γ−1
Tfinal = Tinicial x ( )
Vfinal

6,15 1,40−1
Tfinal = 300 x ( )
16,55

Tfinal = 300 x (0,372)0,40 = 202 K

Item c) Sabendo que o processo é adiabático, a quantidade de calor é igual a zero (Q = 0).
J
Item d) Cálculo da variação da energia interna: ∆U = n x CV x ∆T = 10 mol x 20,8 mol x K x (202 − 300) =
−20384 J
J
Item e) Cálculo da variação da entalpia (ΔH): ∆H = n x Cp x ∆T = 10 mol x 29,11 mol x K x (202 − 300) =
−28528 J

Item f) Através da primeira lei da termodinâmica, temos: ∆U = Q + W

∆U = 0 + W

∆U = W = - 20384 J

Questão 780 – Um mol de um gás apresentando comportamento ideal, expande a partir de uma pressão inicial igual
a 6,50 atmosferas e temperatura inicial de 550 K para uma situação final igual a 3 atmosferas de pressão.
Considerando esta trajetória termodinâmica (a) adiabática; (b) isotérmica e (c) isocórica, admitindo que o processo

371
1600 FÍSICO-QUÍMICA APLICADA EXERCÍCIOS COMENTADOS - IME – ITA – OLIMPÍADA

apresenta reversibilidade mecânica, calcule Q, W, ΔU e ΔH para cada processo. Informações para a resolução do
problema: Cp = 7R/2 e CV = 5R/2.
Resposta:

Situação Inicial:
Pinicial = 6,50 atm
Vinicial = ?
Tinicial = 550 K

Situação Final:
Pfinal = 3,0 atm
Vfinal = ?
Tfinal = ?

Item a) Sendo o processo adiabático, a quantidade de calor é igual a zero. Cálculo do volume no estado inicial, a partir
da equação dos gases ideais:

6,50 x Vinicial = 1,0 x 0,08206 x 550


Vinicial = 6,94 L
7R
Cp 7
Cálculo da constante termodinâmica (γ): γ = = 2
5R = = 1,40
CV 5
2

γ γ
Cálculo do volume final (VFinal), sendo o processo adiabático: Pinicial x Vinicial = Pfinal x Vfinal
1,40
6,50 x (6,94)1,40 = 3,0 x Vfinal

1,40 6,40 x (6,94)1,40


Vfinal =
3,0

1,40 6,40 x (6,94)1,40


Vfinal = √
3,0
1,40
Vfinal = √32,13

Vfinal = 11,92 L
γ−1 γ−1
Cálculo da temperatura final (TFinal): Tinicial x Vinicial = Tfinal x Vfinal
γ−1
Tinicial x Vinicial Vinicial γ−1
Tfinal = γ−1 = Tinicial x ( )
Vfinal Vfinal

Vinicial γ−1
Tfinal = Tinicial x ( )
Vfinal

6,94 1,40−1
Tfinal = 550 x ( ) = 550 x (0,582)0,40 = 550 x 0,805 = 443 K
11,92
5
Cálculo da variação da energia interna (ΔU): ∆U = n x CV x ∆T = 1,0 mol x 2
x 8,314 x (443 − 550) =
−2223,99 J
Cálculo do trabalho realizado, a partir da primeira lei da termodinâmica: ∆U = Q + W

372
1600 FÍSICO-QUÍMICA APLICADA EXERCÍCIOS COMENTADOS - IME – ITA – OLIMPÍADA

∆U = 0 + W (Q = 0, por ser um processo adiabático)

∆U = W = - 2223,99 J
7
Cálculo da variação da entalpia (ΔH): ∆H = n x Cp x ∆T = 1,0 mol x 2
x 8,314 x (443 − 550) =
−3113,59 J
Item b) Sendo o processo isotérmico (temperatura constante), a energia interna e a entalpia são iguais a zero, ∆U =
P
∆H = 0. Cálculo do trabalho realizado (W) considerando o processo isotérmico: W = −n x R x T x ln (P final )
inicial

J 6,50 atm
W = −1,0 mol x 8,314 x 550 K x ln ( ) = −3535,56 J
mol x K 3,00 atm

Cálculo da quantidade de calor (Q), a partir da primeira lei da termodinâmica: ∆U = Q + W


0=Q+W

Q = -W

Q = -W = - (- 3535,56) = + 3535,56 J

Item c) Sendo o processo isocórico (volume constante), o trabalho realizado será igual a zero (W = 0). Cálculo da
Pinicial x Vinicial Pfinal x Vfinal
temperatura final, a partir da utilização da equação combinada dos gases ideais: =
Tinicial Tfinal

6,50 x Vinicial 3,0 x Vinicial


=
550 Tfinal

1650
Tfinal = = 254 K
6,50
5
Cálculo da variação da energia (ΔU): ∆U = n x CV x ∆T = 1,0 mol x 2
x 8,314 x (254 − 550) =
−6152,36 J
7
Cálculo da variação da entalpia (ΔH): ∆H = n x Cp x ∆T = 1,0 mol x 2
x 8,314 x (254 − 550) =
−8613,30 J
Cálculo da quantidade de calor (Q), a partir da primeira lei da termodinâmica: ∆U = Q + W
∆U = Q + 0 = - 6152,36 J

∆U = Q = - 6152,36 J

Questão 781 – (IME) O tetracloreto de carbono é um composto orgânico apolar, líquido à temperatura ambiente.
Dentre outras aplicações, foi amplamente utilizado no século passado como solvente, como pesticida e na síntese de
agentes refrigerantes. Seu emprego comercial, entretanto, foi progressivamente reduzido quando se tornaram
evidentes os seus efeitos a saúde humana e ao meio ambiente. Estudos constataram que a inalação é a principal via
de exposição ao tetracloreto de carbono para trabalhadores e para a população em geral em razão de sua pressão
de vapor relativamente elevada e de sua lenta degradação no ambiente. Supondo que as energias livres padrão de
formação ( G 0f ) do tetracloreto de carbono, nos estados líquido e vapor a 25°C, sejam – 68,6 kJ.mol-1 e – 64,0 kJ.mol-
1respectivamente, determine a sua pressão de vapor, à mesma temperatura, em função da constante e (número de
Neper).

373
1600 FÍSICO-QUÍMICA APLICADA EXERCÍCIOS COMENTADOS - IME – ITA – OLIMPÍADA

Resolução:

Analisando a formação de tetracloreto de carbono na fase líquida: C(s) + 2 Cl2(g) ⇄ CCl4(l)


Analisando a formação de tetracloreto de carbono na fase gasosa: C(s) + 2 Cl2(g) ⇄ CCl4(g)
Determinação da variação da energia livre de Gibbs para a vaporização do tetracloreto de carbono:
Invertendo a reação de formação de tetracloreto de carbono na fase líquida, temos:
CCl4(l) ⇄ C(s) + 2 Cl2(g) ΔG° = + 68,6 kJ.mol-1
C(s) + 2 Cl2(g) ⇄ CCl4(g) ΔG° = - 64,0 kJ.mol-1 +
CCl4(l) ⇄ CCl4(g) ΔG° = + 4,6 kJ.mol-1
Cálculo da pressão de vapor do tetracloreto de carbono: CCl4(l) ⇄ CCl4(g)
∆G° = −R x T x lnK p
J J
4600 mol = −8,314 mol x K x (25 + 273) x lnK p

J J
4600 = −8,314 x 298 x lnK p
mol mol x K
4600
lnK p = − = −1,86
2477,572
K p = e−1,86

Questão 782 - (IME) Dado: C(grafite) + CO2(g) → 2 CO(g).


ΔH° = + 172,5 kJ (25°C, 1 atm)
ΔS° = + 176,3 J (25°C, 1 atm)
a) Determine ΔG° (25°C, 1 atm) dessa reação.
b) A que temperatura essa reação é espontânea?
0 0 0
Resolução: Item a) Utilizando a equação de Gibbs, temos: ∆Greação = ∆Hreação − T x ∆Sreação

0
∆Greação = 172500 − T x 176,3

0 0 0
Para uma temperatura igual a 25°C, temos: ∆Greação = ∆Hreação − T x ∆Sreação

0
J
∆Greação = 172500 − (25 + 273)x 176,3 = 119962,50
mol
0
Item b) Condição de espontaneidade: ∆Greação <0

0 0 0
∆Greação = ∆Hreação − T x ∆Sreação <0

172500 − T x 176,3 < 0

172500
T> = 978,44 K
176,3

374
1600 FÍSICO-QUÍMICA APLICADA EXERCÍCIOS COMENTADOS - IME – ITA – OLIMPÍADA

Questão 783 – Considere a reação apresentada a seguir: C(grafite) + H2O(g) → CO(g) + H2(g), apresentando como
variação da entalpia padrão igual a ∆H°298 = 31,39 kcal. Sabendo que os valores de Cp (cal / mol x K) são: grafite =
2,066; H2O(g) = 8,025; CO(g) = 6,965 e H2(g) = 6,892. A partir das informações apresentadas, calcule o valor da variação
de entalpia à 125°C.

Resolução: Cálculo da variação do calor específico para a reação química de simples troca citada pela questão:
H H O
∆Cpmédio = [1 x CpCO + 1 x Cp 2 ] − [1 x CpC + 1 x Cp 2 ]

cal
∆Cpmédio = [1 x 6,965 + 1 x 6,892] − [1 x 2,066 + 1 x 8,025] = +3,766
mol x K
125°C 25°C
Cálculo da da variação da entalpia à 125°C: ∆Hreação = ∆Hreação + ∫ Cpmédio x dT

398
125°C
∆Hreação = 31390 + ∫ 3,766 x dT
298

125°C
J
∆Hreação = 31390 + 3,766 x (398 − 298) = 31766,6
mol

Questão 784 – (IME) Abaixo são apresentadas três equações químicas com as respectivas entalpia e entropia de
reação a 27°C e 1 atm. Analise-as, separadamente e indique, em cada caso, se a 27°C e 1 atm a transformação é
espontânea ou não.

I.TiO2 + 2 Cl2 → TiCl4 + O2; ΔH = + 38,7 kcal; ΔS = - 9,18 cal.K-1


II.Fe2O3 + 2 Al → 2 Fe + Al2O3; ΔH = -202,6 kcal; ΔS = - 11,86 cal.K-1
III.2 HNO3 → H2 + N2 + 3 O2; ΔH = + 82,8 kcal; ΔS = + 149,6 cal.K-1

Resolução:

Item I) Analisando a primeira reação química: TiO2 + 2 Cl2 → TiCl4 + O2

ΔH = + 38,7 kcal; ΔS = - 9,18 cal.K-1

Substituindo os valores termodinâmicos na equação, temos:

∆G = ∆H − T x ∆S
𝐽
∆G = 38700 − (27 + 273)x (−9,18) = + 41454 𝑚𝑜𝑙
(Processo não espontâneo)

Item II) Analisando a segunda reação química: Fe2O3 + 2 Al → 2 Fe + Al2O3

ΔH = -202,6 kcal; ΔS = - 11,86 cal.K-1

Substituindo os valores termodinâmicos na equação, temos: ∆G = ∆H − T x ∆S


∆G = −202600 − (27 + 273) x (−11,86)
𝐽
∆G = −202600 − 300 x (−11,86) = −199042 𝑚𝑜𝑙 (Processo espontâneo)

Item III) Analisando a terceira reação química: 2 HNO3 → H2 + N2 + 3 O2

ΔH = + 82,8 kcal; ΔS = + 149,6 cal.K-1

Substituindo os valores termodinâmicos na equação, temos:

∆G = ∆H − T x ∆S

375
1600 FÍSICO-QUÍMICA APLICADA EXERCÍCIOS COMENTADOS - IME – ITA – OLIMPÍADA

∆G = +82800 − (27 + 273) x (+149,6)


𝐽
∆G = +82800 − 300 x (+149,6) = +37920 𝑚𝑜𝑙
(Processo não espontâneo)

Conclusão: Item I) Processo não espontâneo; Item II) Processo espontâneo e Item III) Processo não espontâneo.

Questão 785 - O calor específico do monóxido de carbono é dado pela seguinte equação:
Cp (J.mol-1.K-1) = 6,395 + 6,77 x 10-4T + 1,30 x 10-7T2. Qual é a variação de entalpia associada com o aquecimento do
monóxido de carbono de 500°C até 1000°C?

Resolução: Cálculo da entalpia do monóxido de carbono (HCO) no intervalo de 500 a 1000°C.


1273
HCO = ∫ (6,395 + 6,77 x 10−4 T + 1,30 x 10−7 T²)dT
773
1273 1273 1273
HCO = ∫ 6,395 dT + ∫ 6,77 x 10−4 . T dT + ∫ 1,30 x 10−7 . T² dT
773 773 773
1273 1273 1273
HCO = ∫ 6,395 dT + ∫ 6,77 x 10−4 . T dT + ∫ 1,30 x 10−7 . T² dT
773 773 773
1273 1273
T² T³
HCO = 6,395 x T|1273
773 + 6,77 x 10 −4
x | + 1,30 x 10−7 x |
2 773 3 773

1273² 773²
HCO = 6,395 x (1273 − 773) + 6,77 x 10−4 x {( )− ( )}
2 2
1273³ 773³
+ 1,30 x 10−7 x {( )− ( )}
3 3

HCO = 6,395 x 500 + 6,77 x 10−4 x 511500 + 1,30 x 10−7 x 5,34 x 108
J
HCO = 3197,5 + 346,28 + 69,42 = + 3613,20
mol
Questão 786 – A entalpia de fusão do alumínio é igual a 10,7 kJ.mol-1, e a entropia de fusão é igual a 11,4 J.mol-1.K-
1. Calcule a temperatura de fusão do alumínio.

Resolução: Através da equação de Gibbs, temos: ∆G = +10700 − T x 11,4

Condição de equilíbrio, ΔG = 0.

∆G = +10700 − T x 11,4

0 = +10700 − T x 11,4

11,4 x T = +10700

10700
T= = 938,60 K
11,4

376
1600 FÍSICO-QUÍMICA APLICADA EXERCÍCIOS COMENTADOS - IME – ITA – OLIMPÍADA

Questão 787 – Calcule a temperatura mínima para que ocorra a redução do óxido de manganês por carbono sólido
em um vácuo a 10-3 mmHg.
MnO(s) → Mn(s) + ½ O2(g) ΔG° (cal.mol-1) = 91950 - 17,40 x T
C(s) + ½ O2(g) → CO (g) ΔG° (cal.mol-1) = - 26700 – 20,95 x T
Resolução: Somando as duas reações químicas, temos:
MnO(s) → Mn(s) + ½ O2(g) ΔG° (cal.mol-1) = 91950 - 17,40 x T
C(s) + ½ O2(g) → CO(g) ΔG° (cal.mol-1) = - 26700 – 20,95 x T +

MnO(s) + C(s) → Mn(s) + CO(g) ΔG° (cal.mol-1) = 65250 - 38,35 x T


Cálculo da temperatura mínima (Tmin): ∆G° = 65250 − 38,35 x T
− R x T x ln K p = 65250 − 38,35 x T
1
− 1,987 x T x ln {10−3 x } = 65250 − 38,35 x T
760
1
− 1,987 x T x ln {10−3 x } = 65250 − 38,35 x T
760
− 1,987 x T x (−13,54) = 65250 − 38,35 x T
26,91 x T = 65250 − 38,35 x T
26,91 x T + 38,35 x T = 65250
65250
T = = 999,85 K
65,26
Questão 788 – (OLIMPÍADA NORTE/NORDESTE DE QUÍMICA) Hidrogênio gasoso pode ser produzido a partir da
reação entre carvão e vapor d´água, como mostra a reação: C(s) + H2O(g) → CO(g) + H2(g).
a) Calcule a ΔG° para esta reação a 25°C, considerando que C(s) é grafite.
b) Calcule Kp para esta reação a 25°C.
c) Esta reação ocorre espontaneamente sob estas mudanças? Se não, a que temperatura ela se tornará
espontânea? Dado: R = 8,314 J.mol-1.K-1.

Espécie ΔHo298 [kJ.mol-1] ΔSo298 [J.mol-1.K-1] ΔGo298 [kJ.mol-1]


H2O(g) - 241,8 + 188,8 - 228,6
CO(g) - 110,5 + 197,7 - 137,2
C(s) 0 + 5,740 0
H2(g) 0 + 130,68 0

Resolução: Item a) Cálculo da variação de entalpia da reação a 298 K, a partir da equação química:

C(s) + H2O(g) → CO(g) + H2(g)


0 0 0
Cálculo da variação de entalpia da reação a 298 K: ∆𝐻𝑟𝑒𝑎çã𝑜 = ∑ 𝐻𝑝𝑟𝑜𝑑𝑢𝑡𝑜𝑠 − ∑ 𝐻𝑟𝑒𝑎𝑔𝑒𝑛𝑡𝑒𝑠

0 0
∆𝐻𝑟𝑒𝑎çã𝑜 = [1 𝑥 𝐻𝐶𝑂 + 1 𝑥 𝐻𝐻02 ] − [1 𝑥 𝐻𝐶0 + 5 𝑥 𝐻𝐻02 𝑂 ]

0
𝐽
∆𝐻𝑟𝑒𝑎çã𝑜 = [1 𝑥 (−110500) + 1 𝑥 (0)] − [0 + 1 𝑥 (−241800)] = +131300
𝑚𝑜𝑙
0 0 0
Cálculo da variação de entropia a 298 K: ∆𝑆𝑟𝑒𝑎çã𝑜 = ∑ 𝑆𝑝𝑟𝑜𝑑𝑢𝑡𝑜𝑠 − ∑ 𝑆𝑟𝑒𝑎𝑔𝑒𝑛𝑡𝑒𝑠

377
1600 FÍSICO-QUÍMICA APLICADA EXERCÍCIOS COMENTADOS - IME – ITA – OLIMPÍADA

0 0
∆𝑆𝑟𝑒𝑎çã𝑜 = [1 𝑥 𝑆𝐶𝑂 + 1 𝑥 𝑆𝐻02 ] − [1 𝑥 𝑆𝐶0 + 5 𝑥 𝑆𝐻02 𝑂 ]

0
𝐽
∆𝑆𝑟𝑒𝑎çã𝑜 = [1 𝑥 (+197,70) + 1 𝑥 (130,68)] − [5,740 + 1 𝑥 (188,8)] = +133,84
𝑚𝑜𝑙 𝑥 𝐾
0 0 0
Cálculo da variação da energia livre de Gibbs a 298 K: ∆Greação = ∆Hreação − T x ∆Sreação

0
∆Greação = 131300 − (25 + 273) x 133,84

0
𝐽
∆Greação = 131300 − 298 x 133,84 = + 91415,7
𝑚𝑜𝑙
0 𝐽
Item b) Cálculo da constante de equilíbrio: ∆Greação = + 91415,7 = −𝑅 𝑥 𝑇 𝑥 𝑙𝑛𝐾𝑝
𝑚𝑜𝑙
𝐽
+ 91415,7 = −8,314 𝑥 298 𝑥 𝑙𝑛𝐾𝑝
𝑚𝑜𝑙
+ 91415,7
𝑙𝑛𝐾𝑝 = = −36,90
−2477,572

𝐾𝑝 = 𝑒 −36,90

0 0 0
Item c) Cálculo da temperatura para que o processo passe a ser espontâneo: ∆Greação = ∆Hreação − T x ∆Sreação

131300 − T x 133,84 < 0

T x 133,84 > 131300

131300
T>
133,84

T > 981,02 K (Passa a ser espontânea, a partir de 981,02 K).

Questão 789 - Óxido de zinco e carbono, ambos a 25°C, são alimentados, em quantidades estequiométricas, em um
reator operado a 1,0 atm, onde ocorre a seguinte reação química: ZnO(s) + C(s) → Zn(g) + CO(g). A reação é completa
e os produtos saem do reator a 1,0 atm e a uma temperatura de 1000°C. Calcule o calor que é adicionado ou removido
do reator, por quilograma de zinco.
Espécie Cp [J/(mol.K)] ΔHo298 [kJ.mol-1]
ZnO(s) 52,5 -350,5
C(s) 19,8 0
Zn(g) 20,9 130,4
CO(g) 31,6 -110,5

Resolução: Equação química balanceada: ZnO(s) + C(s) → Zn(g) + CO(g)


0 0 0
Cálculo da variação de entalpia a 298 K: ∆Hreação = ∑ Hprodutos − ∑ Hreagentes

0 0
0
∆Hreação 0
= [1 x HZn + 1 x HCO ] − [1 x HZnO + 5 x HC0 ]

0
∆Hreação = [1 x (130,4) + 1 x (−110,5)] − [1 x (−350,5) + 1 x (0)]

378
1600 FÍSICO-QUÍMICA APLICADA EXERCÍCIOS COMENTADOS - IME – ITA – OLIMPÍADA

0
kJ
∆Hreação = [+ 19,9] − [−350,5] = + 370,4
mol

Estudo do balanço térmico:

Cálculo da entalpia em relação ao monóxido de zinco (ZnO): dH = n x Cp x dT


∫ dH = ∫ n x Cp x dT

298
∆HZnO = ∫ n x Cp x dT
298
∆HZnO = 0

Cálculo da entalpia em relação ao C(s): dH = n x Cp x dT


∫ dH = ∫ n x Cp x dT
298
∆HC = ∫ n x Cp x dT = 0
298

Cálculo da entalpia em relação ao Zn(g): dH = n x Cp x dT


∫ dH = ∫ n x Cp x dT
1273
J
∆HZn = ∫ 1 x 20,9 x dT = 20,90 x (1273 − 298) = 20377,50
298 mol

Cálculo da entalpia em relação ao CO(g): dH = n x Cp x dT


∫ dH = ∫ n x Cp x dT
1273
∆HCO = ∫ 1 x 31,6 x dT
298
1273
J
∆HCO = ∫ 31,6 x dT = 31,6 x (1273 − 298) = 30810
298 mol

Sabendo que o processo ocorre a pressão constante, então: Q p = ∑ ∆H


0
Q p = ∆Hreação + ∆HZnO + ∆HC + ∆HZn + ∆HCO
J kJ
Q p = 370400 + 0 + 0 + 20377,50 + 30810 = 421587,50 (421,59 )
mol mol

Cálculo da quantidade de calor para 1 quilograma de zinco gasoso:

1 mol de Zn ---------- 65,4 g de Zn ------------------- 421,59 kJ


1000 g de Zn ------------------- Qp

379
1600 FÍSICO-QUÍMICA APLICADA EXERCÍCIOS COMENTADOS - IME – ITA – OLIMPÍADA

Qp = ΔH = 6446,33 kJ

Questão 790 – (U.S. NATIONAL CHEMISTRY OLYMPIAD) A entalpia de combustão do octano líquido, C8H18(l) para
a formação de produtos gasosos, é – 5090 kJ.mol-1. Use este valor para responder as questões abaixo, assumindo
uma temperatura de 100°C.
a) Escreva a equação balanceada para a combustão completa do octano líquido.
0
b) Determine a entalpia molar de formação, ∆𝐻𝑓𝑜𝑟𝑚𝑎çã𝑜 , para o estado líquido. Dados: Hco 0
2
=
kJ 0 kJ
−393,5 mol ; HH 2O
= −241,8 mol
c) Calcule o valor da energia interna, para a reação de combustão.
d) Se G 0 para a combustão do octano é – 5230 kJ.mol-1, calcule o valor da S 0 .
e) State whether the heat associated with the combustion of liquid octane in a bomb calorimeter representes
H or E 0 . Explain your reasoning.
0

Resolução: Item a) Equação química da combustão do octano líquido: C8H18(l) + 25/2 O2(g) → 8 CO2(g) + 9 H2O(g)
0 0 0
Item b) Cálculo da entalpia molar de formação do octano líquido: ∆Hreação = ∑ Hprodutos − ∑ Hreagentes

0 0 25
0
∆Hreação = (8 x HCO + 9 x HH 2O
) − (1 x HC08 H18 + x HO0 2 )
2 2

−5090 = [8 x (−393,5) + 9 x (−241,8) − 1 x HC08 H18 ]

−5090 = 8 x (−393,5) + 9 x (−241,8) − HC08 H18

−5090 + 3148 + 2176,2 = −HC08 H18

kJ
HC08 H18 = −234,02
mol

Item c) Cálculo da energia interna, considerando que os gases apresentam comportamento ideal.

∆H = ∆U + ∆n x R x T

− 5090000 = ∆U + (17 − 12,50) x 8,314 x (100 + 273)

𝐽 𝑘𝐽
∆U = −5103955,05 (−5103,95 )
𝑚𝑜𝑙 𝑚𝑜𝑙
0 0 0
Item d) Cálculo da entropia (∆𝑆 0 ): ∆𝐺𝑟𝑒𝑎çã𝑜 = ∆𝐻𝑟𝑒𝑎çã𝑜 − 𝑇 𝑥 ∆𝑆𝑟𝑒𝑎çã𝑜

0
− 5230 = −5090 − (100 + 273) x ∆Sreação

0
− 5230 = −5090 − 373 x ∆Sreação

0
−140 kJ
∆Sreação = = + 0,375
−373 mol x K

O sinal da variação da entropia do processo de combustão do octano líquido está correto com a quantidade de gases
produzidos na reação química. São 17 mol de gases no produto “contra” 12,5 mol de gás no reagente.
Item e) Representa a variação de energia interna padrão, uma vez que o processo ocorre a volume constante, na
queima de octano líquido.

380
1600 FÍSICO-QUÍMICA APLICADA EXERCÍCIOS COMENTADOS - IME – ITA – OLIMPÍADA

Questão 791 – (U.S. NATIONAL CHEMISTRY OLYMPIAD) 2 NO2(g) + O3(g) → N2O5(g) + O2(g), ΔH° = - 198 kJ e ΔS°
= - 168 J.K-1. Ozônio reage com dióxido de nitrogênio de acordo com a equação química acima.
a) Calcule a entalpia padrão de formação do dióxido de nitrogênio em kJ.mol-1. Dado: [ΔH°(kJ.mol-1):
O3(g) = 143 e N2O5(g) = 11].
b) Explique o significado do sinal de ΔS°.
c) Calcule o valor de ΔG° para 25°C.
d) Calcule o valor da constante de equilíbrio.
e) Utilize a tabela a seguir, determine a lei de taxa para a reação entre NO2(g) e O3(g).

NO2 (mol.L-1) O3 (mol.L-1)


Taxa (mol.L-1.s-1)
0,0015 0,0025 1,8 x 10-8
0,0022 0,0025 7,2 x 10-8
0,0022 0,0050 1,4 x 10-8
f) Calcule a velocidade específica para a seguinte reação.
0 0 0
Resolução: Item a) Cálculo da entalpia do dióxido de nitrogênio: ∆Hreação = ∑ Hprodutos − ∑ Hreagentes

0
0
∆Hreação = [1 x HN 2 O5
+ 1 x HO0 2 ] − [2 x HNO
0
2
+ 1 x HO0 3 ]
0 0
−198 = [HN 2 O5
+ 1 x 0] − [2 x HNO 2
+ 1 x HO0 3 ]
0 0
−198 = HN 2 O5
− (2 x HNO 2
+ HO0 3 )
0
−198 = 11 − 2 x HNO 2
− 143

0 − 66 kJ
HNO = = + 33
2 −2 mol

Item b) O sinal da entropia padrão é negativo, pois observa-se na reação que há um decréscimo do número de mol,
ou seja: 3 mol de gases no reagente → 2 mol de gases no produto.
0 0 0
Item c) Cálculo da energia livre de Gibbs (ΔG°): ∆Greação = ∆Hreação − T x ∆Sreação
0
∆Greação = −198000 − (25 + 273) x 168

0
J kJ
∆Greação = −248064 (−248,1 )
mol mol

Item d) Cálculo da constante de equilíbrio (K): − 248000 = −8,314 x 298 x lnK


− 248000
lnK = = 100,12
−2477,572
K = e100,12

0,0022 a
Item e) Cálculo da ordem de reação em relação ao dióxido de nitrogênio, através do experimento 1 e 2: (0,0015) =
7,2 x 10−8
4,8 x 10−8

(1,47)a = 1,50

381
1600 FÍSICO-QUÍMICA APLICADA EXERCÍCIOS COMENTADOS - IME – ITA – OLIMPÍADA

Aplicando o logaritmo natural, temos: ln(1,47)a = ln(1,50)


a = 1,05
0,0050 b 1,4 x 10−7
Cálculo da ordem de reação em relação ao ozônio, através do experimento 2 e 3: (0,0025) = 7,2 x 10−8

(2)b = (1,94)

Aplicando o logaritmo natural, temos: ln(2)b = ln(1,94)


b = 0,96

Equação da velocidade de reação: vreação = k x [NO2 ]1,05 x [O3 ]0,96

Item f) Cálculo da velocidade específica (k), através dos dados do primeiro experimento:
vreação = k x [NO2 ]1,05 x [O3 ]0,96
mol mol 1,05 mol 0,96
4,80 x 10−8 = k x (0,0015 ) x (0,0025 )
Lxs L L
mol mol1,05 mol0,96
4,80 x 10−8 = k x (0,0015)1,05 x 1,05 x (0,0025)0,96 x 0,96
Lxs L L
mol mol
4,80 x 10−8
k= L x s = 1,39 x 10 −2 L x s
−6 mol2,01 mol2,01
3,45 x 10 x 2,01
L L2,01
mol L2,01 L1,01
k = 1,39 x 10−2 x = 1,39 x 10 −2
L x s mol2,01 mol1,01 x s

Questão 792 – A entalpia de fusão do alumínio é igual a 10,7 kJ.mol-1, e a entropia de fusão é igual a 11,4 J.mol-1.K-
1. Calcule a temperatura de fusão do alumínio.

Resolução: Através da equação de Gibbs, temos: ∆G = 10700 − T x 11,4

Considerando que o processo está em equilíbrio termodinâmico (∆G = 0): ∆G = 10700 − T x 11,4

0 = 10700 − T x 11,4

10700
T= = 938,60 K
11,4

382
1600 FÍSICO-QUÍMICA APLICADA EXERCÍCIOS COMENTADOS - IME – ITA – OLIMPÍADA

Questão 793 – (U.S. NATIONAL CHEMISTRY OLYMPIAD) Glucose, C6H12O6, is readily metabolized in the body.
a) Write a balanced equation for the metabolism of C6H12O6 to CO2 and H2O.
b) Calculate ΔGo metabolism for glucose. Given: The free energy of formation, ΔGf°, is – 917 kJ.mol–1 for
C6H12O6(s); –394,4 kJ.mol–1 for CO2(g); – 237,2 kJ.mol–1 for H2O(l).
c) If ΔHo for this process is –2801,3 kJ, calculate ΔSo at 25°C.

Resolução:
Item a) Equação química: C6H12O6 + 6 O2(g) → 6 CO2(g) + 6 H2O(g).
0 0 0
Item b) Cálculo da energia livre de Gibbs: ∆𝐺𝑟𝑒𝑎çã𝑜 = ∑ 𝐺𝑝𝑟𝑜𝑑𝑢𝑡𝑜𝑠 − ∑ 𝐺𝑟𝑒𝑎𝑔𝑒𝑛𝑡𝑒𝑠

0 0
0
∆Greação = [(6 x GCO 2
+ 6 x GH 2O
) − 1 x GC06 H12 O6 ]

0
∆Greação = [6 x (−394,4) + 6 x (−237,2) − 1x (−917)]

0
kJ
∆Greação = −2366,4 − 1423,2 + 917 = −2872,60
mol
0 0 0
Item c) Aplicando a equação de Gibbs, temos: ∆Greação = ∆Hreação − T x ∆Sreação

0
− 2872,60 = −2801,30 − (25 + 273) x ∆Sreação

0
− 71,30 = −298 x ∆Sreação

0
− 71,3 kJ
∆Sreação = = + 0,239
− 298 mol x K

Questão 794 - O calor específico do acetileno é representado através da seguinte equação empírica: 7,33 + 12,62 x
10-3T – 3,89 x 10-6T-², em cal.mol-1.K-1, com a temperatura em Kelvin. Calcule o calor específico médio em um intervalo
de temperatura que compreende entre 298 K e 500 K?

Resolução: Sabendo que o cálculo da entalpia para uma temperatura diferente de 25°C pode ser determinado através
de duas equações termodinâmicas:
T
∆H = ∫T 2 Cp dT (Equação A)
1

∆H = Cpmédio x (T2 − T1 ) (Equação B)


T
Igualando as equações: Cpmédio x (T2 − T1 ) = ∫T 2 Cp dT
1

T
∫T 2 Cp dT
1
Cpmédio =
(T2 − T1 )
T
∫T 2 Cp dT
Cálculo do Cpmédio : Cpmédio = 1
(T2 −T1 )

500
∫ (7,33 + 12,62 x 10−3 x T − 3,89 x 10−6 x T −2 ) dT
Cpmédio = 298
(500 − 298)
500 500
−3 T² T −2+1
7,33 x T|500
298 + 12,62 x 10 x |
2 298 − 3,89 x 10 −6
x |
(−2 + 1) 298
Cpmédio =
202

383
1600 FÍSICO-QUÍMICA APLICADA EXERCÍCIOS COMENTADOS - IME – ITA – OLIMPÍADA

500
−3 T² (−1) 500
7,33 x T|500
298 + 12,62 x 10 x |
2 298 − 3,89 x 10 −6
x T |298
Cpmédio =
202
500
−3 T² 1 500
7,33 x T|500
298 + 12,62 x 10 x |
2 298 + 3,89 x 10 −6
x T|298
Cpmédio =
202
(500)2 (298)2 1 1
7,33 x (500 − 298) + 12,62 x 10−3 x [ −6
2 − 2 ] + 3,89 x 10 x (500 − 298)
Cpmédio =
202
7,33 x 202 + 12,62 x 10−3 x (80598) + 3,89 x 10−6 x (− 1,35 x 10−3 )
Cpmédio =
202
1480,66 + 1017,15 − 5,25 x 10−9 cal
Cpmédio = = 12,36
202 mol x K
Observação: A capacidade calorífica é uma propriedade termodinâmica diretamente relacionada com a temperatura.
É representada pelas expressões: A + B.T + CT-2 ou A + BT + CT², com A, B e C valores constantes (tabelados). Os
cálculos são bastante trabalhosos, como foi observado na primeira questão acima e a maioria dos exercícios é mais
comum a utilização do calor específico médio (<Cp>médio), tomando como base a temperatura de 25°C.
Questão 795 – (OLIMPÍADA BRASILEIRA DE QUÍMICA) Óxidos metálicos podem ser reduzidos pelo hidrogênio
gasoso, produzindo os respectivos metais: ZnO(s) + H2(g) → Zn(s) + H2O(g). Prediga os efeitos de cada um dos seguintes
procedimentos sobre a posição do equilíbrio, ou seja, para que lado o equilíbrio será deslocado em cada caso:
a) Adição de ZnO(s)
b) Adição de H2(g)
c) Retirada do Zn(s)
d) Retirada de vapor d´água
Usando os dados termodinâmicos fornecidos abaixo, responda os quesitos que se seguem:
e) Em que direção o equilíbrio será deslocado por um aumento de temperatura?
f) Calcule o valor de ΔG° para esta reação?
g) Esta reação deverá ocorrer espontaneamente ou não?
h) A constante de equilíbrio para esta reação é maior ou menor que 1?
Espécie ΔH°f (298,15 K) kJ.mol-1 S°f (298,15 K) J.mol-1.K-1 ΔG°f (298,15 K) J.mol-1.K-1
H2O(g) - 241,818 188,825 - 228,572
ZnO(s) - 348,28 43,64 - 318,30
Fonte: The NBS Tables Chemical Thermodynamics Properties, 1982.

Resolução:

Item a) Adicionando monóxido de zinco, o equilíbrio não será deslocado/afetado, uma vez que trata-se de um sólido
e a atividade deste composto é igual a 1 (aZnO = 1).

Item b) Adicionando hidrogênio gasoso, o equilíbrio será deslocado para a direita, favorecendo zinco metálico e vapor
d´água.

Item c) Retirando zinco metálico, o equilíbrio não será deslocado, uma vez trata-se de um sólido e a atividade é igual
a 1 (aZn = 1).

Item d) Retirando vapor d´água, o equilíbrio será deslocado para a direita, favorecendo maior produção de zinco
metálico.
0
Item e) Cálculo da variação de entalpia (∆Hreação ): ZnO(s) + H2(g) → Zn(s) + H2O(g)

384
1600 FÍSICO-QUÍMICA APLICADA EXERCÍCIOS COMENTADOS - IME – ITA – OLIMPÍADA

0 0 0
∆Hreação = ∑ Hprodutos − ∑ Hreagentes

0 0 0 0 0
∆Hreação = [1 x HZn + 1 x HH 2O
] − [1 x HZnO + 1 x HH 2
]

0
∆Hreação = [1 x 0 + 1 x (−241,818)] − [1 x (−348,28) + 1 x 0]

0
kJ
∆Hreação = −241,818 + 348,28 = +106,46
mol

Aumentando a temperatura do sistema, o equilíbrio será deslocado para a direita, pelo fato do processo ser
0
endotérmico (∆Hreação > 0).

0
Item f) Determinação da variação da energia livre de Gibbs (∆Greação ):

0 0 0 0 0 0 0
∆Greação = ∑ Gprodutos − ∑ Greagentes = [1 x GZn + 1 x GH 2O
] − [1 x GZnO + 1 x GH 2
]

0
∆Greação = [1 x 0 + 1 x (−228,572)] − [1 x (−318,30) + 1 x 0]

0
kJ
∆Greação = −228,572 + 318,30 = + 89,73
mol

Item g) Observando o valor da variação da energia livre de Gibbs (Greação


0
 0) , a reação é não espontânea.

Item h) Para uma variação da energia livre de Gibbs maior que zero, a constante de equilíbrio será menor que um.

0
kJ
∆Greação = +89,73
mol

− 8,314 x 298 x lnK = +89730

89730
lnK = −
2477,572

K = e−36,22

Questão 796 – (OLIMPÍADA BRASILEIRA DE QUÍMICA / CANADIAN CHEMISTRY OLYMPIAD FINAL


SELECTION EXAMINATION) Uréia, CO(NH2)2, reage com água produzindo dióxido de carbono e amônia. Os dados
termodinâmicos para os possíveis reagentes e produtos são dados abaixo (negligencie a solubilidade do dióxido de
carbono e da amônia em água líquida).

Composto ΔH°f (kJ.mol-1) S°f (J.mol-1.K-1)


CO(NH2)2(s) - 333,51 104,60
H2O(l) - 285,83 69,91
H2O(g) - 241,82 188,83
CO2 (g) - 393,51 213,74
NH3(g) - 46,11 192,45

a) Considere a hidrólise de ureia com H2O(l) (reação A) e com H2O(g) (reação B), respectivamente. Calcule ∆H 0 ,
∆S e ∆G0 a 25°C, para cada reação e especifique se a reação é espontânea ou não.
0

b) Considerando que ambos, ∆H 0 e ∆S 0, são independentes da temperatura, encontre a temperatura na qual


a reação A ocorrerá espontaneamente.

385
1600 FÍSICO-QUÍMICA APLICADA EXERCÍCIOS COMENTADOS - IME – ITA – OLIMPÍADA

c) Calcule Kp a 25°C para cada reação, expressando esse valor em unidades apropriadas.

Resolução:

Item a) Analisando a reação A:

Equação química: CO(NH2)2 + H2O(l) → CO2(g) + 2 NH3(g)


0 0 0 0
Cálculo da variação de entalpia (∆Hreação ) para a reação A: ∆Hreação = ∑ Hprodutos − ∑ Hreagentes

0 0 0 0 0
∆Hreação = [1 x HCO 2
+ 2 x HNH 3
] − [1 x HCO(NH2 )2
+ 1 x HH 2O
]

0
∆Hreação = [1 x (−393,51) + 2 x (−46,11)] − [1 x (−333,51) + 1 x (−285,83)]

0 kJ
∆Hreação = −485,73 + 619,34 = + 133,61 (Processo endotérmico)
mol

0 0 0 0
Cálculo da variação de entropia (∆Sreação ) para a reação A: ∆Sreação = ∑ Sprodutos − ∑ Sreagentes

0 0 0 0 0
∆Sreação = [1 x SCO 2
+ 2 x SNH3
] − [1 x SCO(NH 2 )2
+ 1 x SH 2O
]

0
∆Sreação = [1 x (213,74) + 2 x (192,45)] − [1 x (104,60) + 1 x (69,91)]

0 J
∆Sreação = 598,64 − 174,51 = + 424,13
mol x K

0 0 0 0
Cálculo da variação de energia livre de Gibbs (∆Greação ) para a reação A: ∆Greação = ∆Hreação − T x ∆Sreação

0
∆Greação = + 133610 − (25 + 273) x 424,13

0 J
∆Greação = + 133610 − 126390,74 = + 7219,26 mol (Processo não espontâneo)

Analisando a reação B:

Equação química: CO(NH2)2 + H2O(g) → CO2(g) + 2 NH3(g)


0 0 0 0
Cálculo da variação de entalpia (∆Hreação ) para a reação B: ∆Hreação = ∑ Hprodutos − ∑ Hreagentes

0 0 0 0 0
∆Hreação = [1 x HCO 2
+ 2 x HNH 3
] − [1 x HCO(NH2 )2
+ 1 x HH 2O
]

0
∆Hreação = [1 x (−393,51) + 2 x (−46,11)] − [1 x (−333,51) + 1 x (−241,82)]

0 kJ
∆Hreação = −485,73 + 575,33 = + 89,60 (Processo endotérmico)
mol

0 0 0 0
Cálculo da variação de entropia (∆Sreação ) para a reação B: ∆Sreação = ∑ Sprodutos − ∑ Sreagentes

0 0 0 0 0
∆Sreação = [1 x SCO 2
+ 2 x SNH3
] − [1 x SCO(NH 2 )2
+ 1 x SH 2O
]

0
∆Sreação = [1 x (213,74) + 2 x (192,45)] − [1 x (104,60) + 1 x (188,83)]

0 J
∆Sreação = 598,64 − 293,43 = + 305,21
mol x K

386
1600 FÍSICO-QUÍMICA APLICADA EXERCÍCIOS COMENTADOS - IME – ITA – OLIMPÍADA

0 0 0 0
Cálculo da variação de energia livre de Gibbs (∆Greação ) para a reação B: ∆Greação = ∆Hreação − T x ∆Sreação

0
∆Greação = + 89600 − (25 + 273) x 305,21
0 J
∆Greação = + 89600 − 90952,58 = − 1352,58 mol (Processo espontâneo)

0 0 0
Item b) Equação da energia livre de Gibbs para a reação A: ∆Greação = ∆Hreação − T x ∆Sreação

0
∆Greação = + 133610 − T x 424,13

0
Critério de espontaneidade: ∆Greação <0
+ 133610 − T x 424,13 < 0

424,13 x T > 133610

133610
T>
424,13

T > 315,02 K
0
Item c) Analisando a reação A: ∆Greação = −R x T x lnK

7219,26 = −8,314 x 298 x lnK

− 7219,26
lnK =
2477,572

K = e−2,91
0
Analisando a reação B: ∆Greação = −R x T x lnK

− 1352,58 = −8,314 x 298 x lnK

− 1352,58
lnK =
− 2477,572

K = e+0,546

Questão 797 – (U.S. NATIONAL CHEMISTRY OLYMPIAD) O primeiro passo para a produção de silício com alta
pureza para semicondutores está representado pela seguinte equação química: SiO2(s) + 2 C(s) → Si(s) + 2 CO(g)
ΔH° = + 689,9 kJ
0 0
a) Calcule ∆Hformação para SiO2(s). Dado: ∆Hformação para CO(g) = - 110,5 kJ.mol-1.
0
b) Calcule ∆Sreação para a produção de silício puro.
J J J J
Dado: SC0 = 5,7 mol x K ; SCO
0 0
= +197,6 mol x K ; SSi = 18,8 mol x K
0
e SSiO = 41,8 mol x K.
0
c) Determine ∆Greação para uma temperatura igual a 25°C.
d) Determine a constante de equilíbrio.
e) Calcule a temperatura em graus de Celsius, para que o processo seja espontâneo. Assuma que
0 0
∆Hreação e ∆Sreação e não variam com a temperatura.

0
Resolução: Item a) Cálculo da variação da entalpia do dióxido de silício: SiO2(s) + 2 C(s) → Si(s) + 2 CO(g), ∆Hreação =
0 0
∑ Hprodutos − ∑ Hreagentes

387
1600 FÍSICO-QUÍMICA APLICADA EXERCÍCIOS COMENTADOS - IME – ITA – OLIMPÍADA

0 0 0
0
∆Hreação = [1 x HSi + 2 x HCO ] − [1 x HSiO 2
+ 2 x HC0 ]

0 0
∆Hreação = [1 x 0 + 2 x (−110,5)] − [1 x HSiO 2
+ 2 x 0]

0
+689,9 = −221,0 − HSiO 2

0 kJ
HSiO 2
= −910,9 mol

0 0
Item b) Cálculo da variação da entropia padrão da reação: SiO2(s) + 2 C(s) → Si(s) + 2 CO(g), ∆Sreação = ∑ Sprodutos −
0
∑ Sreagentes

0 0 0
0
∆Sreação = [1 x SSi + 2 x SCO ] − [1 x SSiO 2
+ 2 x SC0 ]

0
∆Sreação = [1 x (18,8) + 2 x (+197,6)] − [1 x (41,8) + 2 x (5,7)]

0
J
∆Sreação = 414,0 − 53,20 = + 360,8
mol x K
0 0 0
Item c) Cálculo da variação da entropia padrão da reação: ∆Greação = ∆Hreação − T x ∆Sreação

0
∆Greação = + 689900 − (25 + 273) x 360,80

0
∆Greação = + 89600 − 90952,58
0 J kJ
∆Greação = − 582381,60 mol (− 582,4 mol
) (Processo espontâneo)
0
Item d) Cálculo da constante de equilíbrio: ∆Greação = −R x T x lnK

− 582381,6 = −8,314 x 298 x lnK


− 582381,6
lnK = = 235,06
− 2477,572

K = e+235,06
0
Item e) Critério de espontaneidade: ∆Greação <0
0 0 0
∆Greação = ∆Hreação − T x ∆Sreação

0
∆Greação = + 689900 − T x 360,80

0
Critério de espontaneidade: ∆Greação <0

+ 689900 − T x 360,80 < 0

360,80 x T > 689900

689900
T>
360,80

T > 1911,66 K

388
1600 FÍSICO-QUÍMICA APLICADA EXERCÍCIOS COMENTADOS - IME – ITA – OLIMPÍADA

Questão 798 – (U.S. NATIONAL CHEMISTRY OLYMPIAD) Consider the thermodynamic data given below:
Espécies ΔHºf (kJ x mol-1) Sºf (J x mol-1 x K-1)
H+(aq) 0 0
OH-(aq) - 229,9 -
H2O(l) - 285,83 69,95
The autoionization of water can be described according to the equation below. Its equilibrium constant, Kw, is 1,0  10-
14 at 25ºC.

H2O(l) ⇌ H+(aq) + OH-(aq)


a) Calculate ∆Hº for the autoionization of water.
b) Calculate ∆Gº (at 298 K) for the autoionization of water.
c) Calculate ∆Sº for the autoionization of water and rationalize its sign.
d) Calculate Sº for OH–(aq).
e) Calculate Kw at 50ºC.

Resolução:
0 0 0
Item a) Cálculo da variação de entalpia padrão (∆Hº): ∆Hreação = ∑ Hprodutos − ∑ Hreagentes

0 0 0 0
∆Hreação = [1 x HH + + 1 x HOH− ] − [1 x HH
2O
]

0
kJ
∆Hreação = [1 x 0 + 1 x (−229,9)] − [1 x (−285,83)] = +55,93
mol
0
Item b) Cálculo da variação da energia livre de Gibbs padrão (∆Gº): ∆Greação = −R x T x lnK

0
∆Greação = −8,314 x (25 + 273) x ln (10−14 )

0
J kJ
∆Greação = +79867,48 (79,87 )
mol mol

0 0 0
Item c) Cálculo da variação de entropia padrão: ∆Greação = ∆Hreação − T x ∆Sreação

0
79,87 = + 55,93 − (25 + 273) x ∆Sreação

0
79,87 − 55,93 J
∆Sreação =− = − 8,03 x 10−2
298 mol x K
0 0 0 0 0
Item d) Cálculo da entropia de OH- à 25°C: ∆Sreação = ∑ Sprodutos − ∑ Sreagentes = [1 x SH + + 1 x SOH− ] −

0
[1 x SH 2O
]

0
−80 = [1 x 0 + 1 x SOH − ] − [1 x (69,95)]

0 J J
SOH − = −80 + 69,95 = −10,05
mol x K mol x K

dlnK ∆H0
Item e) Aplicando a equação de Van´t Hoff: =
dT R x T²

+ ∆H° 1 1
lnK W1 − lnK W2 = x{ − }
R T2 T1

389
1600 FÍSICO-QUÍMICA APLICADA EXERCÍCIOS COMENTADOS - IME – ITA – OLIMPÍADA

+ 55930 1 1
ln (10−14 ) − lnK W2 = x{ − }
8,314 (50 + 273) (25 + 273)

+ 55930 1 1
ln (10−14 ) − lnK W2 = x{ − }
8,314 323 298

+ 55930
ln (10−14 ) − lnK W2 = x (− 2,60 x 10−4 )
8,314
−32,24 − lnK W2 = − 1,75

lnK W1 = − 30,49

K W1 = e−30,49 = 5,74 x 10−14

0 0 0
Segunda maneira de resolução, a partir da equação livre de Gibbs: ∆Greação = ∆Hreação − T x ∆Sreação

0
∆Greação = + 55930 − (50 + 273) x (−80)

0
J
∆Greação = + 55930 + 25840 = + 81770
mol
0
Cálculo da constante de ionização a 50°C: ∆Greação = −R x T x lnK W

81770 = −8,314 x (50 + 273) x ln K W

81770
ln K W = − = −30,44
2685,42
K W = e(−30,44) = 6,03 x 10−14

Questão 799 – (OLIMPÍADA DE QUÍMICA DO RIO DE JANEIRO – MODIFICADA) Um reator foi projetado para
promover a hidrogenação do benzeno à cicloexano.
C6H6(l) + 3 H2(g) → C6H12(l) Reação I
Para a execução do projeto fez-se necessário o levantamento dos dados termodinâmicos da reação.

Entalpias-padrão de combustão
(considerando combustão completa das espécies)

Compostos ΔH°c (kJ.mol-1)


C6H6(l) - 3267
H2(g) - 285,8
C6H12(l) - 3930
Entropias-padrão
Compostos S° (J.K-1.mol-1)
C6H6(l) 174
H2(g) 130,7
C6H12(l) 204

a) Determine a variação de entalpia para a reação 1.

390
1600 FÍSICO-QUÍMICA APLICADA EXERCÍCIOS COMENTADOS - IME – ITA – OLIMPÍADA

b) Determine a variação de entropia para a reação 1.


c) Na temperatura ambiente, 25°C, a reação 1 é espontânea? Justifique.
d) Calcule a constante de equilíbrio.

Resolução: Item a) Cálculo da entalpia para a reação 1:

Equação química I: C6H6(l) + 15/2 O2(g) → 6 CO2(g) + 3 H2O(l) ΔH° = - 3267 kJ.mol-1
Equação química II: H2(g) + ½ O2(g) → H2O(l) ΔH° = - 285,8 kJ.mol-1
Equação química III: C6H12(l) + 9 O2(g) → 6 CO2(g) + 6 H2O(l) ΔH° = - 3267 kJ.mol-1
Utilizando o princípio da lei de Hess, multiplicando a segunda equação por três e invertendo a terceira reação química:
C6H6(l) + 15/2 O2(g) → 6 CO2(g) + 3 H2O(l) ΔH° = - 3267 kJ.mol-1
3 H2(g) + 3/2 O2(g) → 3 H2O(l) ΔH° = 3 x (- 285,8) kJ.mol-1
6 CO2(g) + 6 H2O(l) → C6H12(l) + 9 O2(g) ΔH° = + 3267 kJ.mol-1 +

C6H6(l) + 3 H2(g) → C6H12(l) ΔH° = - 1194,4 kJ.mol-1


0 0 0
Item b) Cálculo da variação de entropia padrão: ∆𝑆𝑟𝑒𝑎çã𝑜 = ∑ 𝑆𝑝𝑟𝑜𝑑𝑢𝑡𝑜𝑠 − ∑ 𝑆𝑟𝑒𝑎𝑔𝑒𝑛𝑡𝑒𝑠

0
∆𝑆𝑟𝑒𝑎çã𝑜 = [1 𝑥 𝑆𝐶06 𝐻12 ] − [1 𝑥 𝑆𝐶06 𝐻6 + 3 𝑥 𝑆𝐻02 𝑂 ] = [1 𝑥 204] − [1 𝑥 174 + 3 𝑥 130,7]
𝐽
= + 204 − 566,1 = −362,1
𝑚𝑜𝑙 𝑥 𝐾
0 0 0
Item c) Cálculo da energia livre de Gibbs padrão: ∆𝐺𝑟𝑒𝑎çã𝑜 = ∆𝐻𝑟𝑒𝑎çã𝑜 − 𝑇 𝑥 ∆𝑆𝑟𝑒𝑎çã𝑜

0
∆𝐺𝑟𝑒𝑎çã𝑜 = −1194400 − 𝑇 𝑥 362,1

0 𝐽 𝑘𝐽
∆𝐺𝑟𝑒𝑎çã𝑜 = −1194400 − 298 𝑥 362,1 = − 1302305,8 (−1302,30 ), processo espontâneo.
𝑚𝑜𝑙 𝑚𝑜𝑙

0
Item d) Cálculo da constante de equilíbrio (K): ∆Greação = −R x T x lnK W
− 1302305,8 = −8,314 x 298 x ln K

− 1302305,8
ln K = = + 525,64
−2477,572

K = e(+525,64)

Questão 800 - Propano é queimado completamente em um reator com uma quantidade estequiométrica de oxigênio.
Calcule a temperatura adiabática de chama, assumindo que os gases são alimentados a 25 °C. Calcule também
quando a alimentação é feita com ar ao invés de oxigênio.

Espécie ΔHof (298K) (kJ x mol-1) Cp (J/mol x K)


C3H8(g) -105,0 73,5
CO2(g) -393,5 45,4
H2O(g) -241,8 34,4
N2(g) 0 29,1
O2(g) 0 29,4

Resolução: Equação química da combustão completa do gás propano: C3H8(g) + 5 O2(g) → 3 CO2(g) + 4 H2O(g)

391
1600 FÍSICO-QUÍMICA APLICADA EXERCÍCIOS COMENTADOS - IME – ITA – OLIMPÍADA

0 0
Cálculo da variação de entalpia padrão da reação de combustão do gás propano: ∆Hreação = ∑ Hprodutos −
0
∑ Hreagentes

0 0
0
∆Hreação = [3 x HCO 2
+ 4 x HH 2O
] − [1 x HC03 H8 + 5 x HO0 2 ]

0
∆Hreação = [3 x (−393,5) + 4 x (−241,8)] − [1 x (−105,0) + 1 x (0)]

0
kJ
∆Hreação = −1180,5 − 967,2 + 105,0 = −2042,7
mol

Estudo do balanço térmico:

Cálculo da temperatura (T) adiabática de chama: Q p = ∑ ∆H


3
Q p = ∆HC13 H8 + ∆HO2 2 + ∆HCO 2
4
+ ∆HH 2O
0
+ ∆Hreação

298 298 T T
Qp = ∫ 1 x 73,5 dT + ∫ 5 x 29,4 dT + ∫ 3 x 45,4 dT + ∫ 4 x 34,4 dT − 2042700
298 298 298 298

Q p = 0 + 0 + 136,2 x (T − 298) + 137,6 x (T − 298) − 2042700

Q p = 136,2 x (T − 298) + 137,6 x (T − 298) − 2042700

Sabendo que o processo é adiabático, Q p = 0.

0 = 136,2 x (T − 298) + 137,6 x (T − 298) − 2042700

136,2 x (T − 298) + 137,6 x (T − 298) = 2042700

2042700
T − 298 =
(136,2 + 137,6)

2042700
T= + 298 = 7758,55 K
(136,2 + 137,6)

Analisando e determinando a temperatura adiabática de chama com a presença do ar atmosférico. Cálculo do número
de mol de N2: C3H8(g) + 5 O2(g) → 3 CO2(g) + 4 H2O(g)
5 mol de O2 ----------- 21%
nN2 --------------------- 79%
nN2 = 18,81 mol
Equação química da combustão completa do gás propano com a presença de gás nitrogênio: C3H8(g) + 5 O2(g) + 18,81
N2(g)→ 3 CO2(g) + 4 H2O(g) + 18,81 N2(g)

392
1600 FÍSICO-QUÍMICA APLICADA EXERCÍCIOS COMENTADOS - IME – ITA – OLIMPÍADA

0 0
Cálculo da variação de entalpia padrão da reação de combustão do gás propano: ∆Hreação = ∑ Hprodutos −
0
∑ Hreagentes
0 0
0
∆Hreação = [3 x HCO 2
+ 4 x HH 2O
0
+ 18,81 x HN 2
] − [1 x HC03 H8 + 5 x HO0 2 + 18,81 x HN
0
2
]

0
∆Hreação = [3 x (−393,5) + 4 x (−241,8) + 18,81 x 0] − [1 x (−105,0) + 1 x (0) + 18,81 x 0]

0
kJ
∆Hreação = −1180,5 − 967,2 + 105,0 = −2042,7
mol

Estudo do balanço térmico:

Cálculo da temperatura adiabática de chama (T) com a presença de ar atmosférico: Q p = ∑ ∆H


5
Q p = ∆HC13 H8 + ∆HO2 2 + ∆HN
3
2
4
+ ∆HCO 2
+ ∆HH 2O
6
+ ∆HN 2
0
+ ∆Hreação

298 298 298 T


Qp = ∫ 1 x 73,5 dT + ∫ 5 x 29,4 dT + ∫ 18,81 x 29,1 dT + ∫ 3 x 45,4 dT
298 298 298 298
T T
+ ∫ 4 x 34,4 dT + ∫ 18,81 x 29,1 dT − 2042700
298 298

T T T
Q p = 0 + 0 + 0 + ∫ 3 x 45,4 dT + ∫ 4 x 34,4 dT + ∫ 18,81 x 29,1 dT − 2042700
298 298 298

0 = 0 + 0 + 136,2 x (T − 298) + 137,6 x (T − 298) + 547,4 x (T − 298) − 2042700

2042700 = (136,2 + 137,6 + 547,4) x (T − 298)

2042700
T − 298 =
821,2

2042700
T= + 298 = 2785,46 K
821,2

Questão 801 – (OLIMPÍADA BRASILEIRA DE QUÍMICA) Durante a titulação de uma solução de triiodeto (I3-) com
uma solução de tiossulfato de sódio (Na2S2O3) ocorrem as seguintes reações, com seus potenciais padrão de redução:
I3- + 2e- → 3 I- E° = + 0,54 V
S4O6-2 + 2e- → 2 S2O3-2 E° = + 0,08 V
a) Escreva a reação global da redução do triiodeto pelo íon tiossulfato e calcule o potencial de redução padrão
desta reação.

393
1600 FÍSICO-QUÍMICA APLICADA EXERCÍCIOS COMENTADOS - IME – ITA – OLIMPÍADA

b) O íon triiodeto (I3-), em solução aquosa, permanece em equilíbrio com o iodo molecular (I2) e íon iodeto (I-),
de acordo com a equação I. Equação química: I- + I2 → I3-

Os dados termodinâmicos, a temperatura de 25°C, para essas três espécies encontram-se na tabela abaixo.

Espécie ∆H°f (kJ.mol-1) ∆G°f (kJ.mol-1) ∆S°f (J.mol-1)


I- 55,2 - 51,6 111,3
I2 22,5 16,4 137,2
I- 3 51,5 - 51,4 239,3

Calcule o valor da constante de equilíbrio, K, para a reação apresentada na equação I, utilizando os dados da tabela.
Resolução:

Item a) Invertendo a segunda reação química, temos:

I3- + 2e- → 3 I- E° = + 0,54 V ΔG° = -2 x F x (+ 0,54)


2 S2O3-2 → S4O6-2 + 2e- E° = + 0,08 V ΔG° = -2 x F x (- 0,08) +

I3- + 2 S2O3-2 → 3 I- + S4O6-2 ΔG°TOTAL = - 2 x F x (+ 0,54) + [-2 x F x (- 0,08)]


ΔG°TOTAL = - 2 x F x (+ 0,54) - 2 x F x (- 0,08)
-2 x F x E° = -2 x F x (+ 0,54) –2 x F x (- 0,08)
E° = + 0,54 – 0,08 = + 0,46V
0
Item b) Cálculo da constante de equilíbrio (K): ∆Greação = −R x T x lnK

Equação química balanceada: I- + I2 → I3-


0 0
∑ Gprodutos − ∑ Greagentes = −R x T x lnK

{(1 x − 51,4)} − {(1 x − 51,6) + (1 x 16,4)} = − 8,314 x 298 x ln K

−51,4 + 35,2 = − 8,314 x 298 x ln K

16,2
ln K = = 6,54 x 10−3
2477,572
−3 )
K = e(−6,54 x 10

394
1600 FÍSICO-QUÍMICA APLICADA EXERCÍCIOS COMENTADOS - IME – ITA – OLIMPÍADA

Questão 802 – (U.S. NATIONAL CHEMISTRY OLYMPIAD) A combustão de etano produz dióxido de carbono e água
líquida a 25°C.
a) Escreva a equação para esta reação.
0
b) Sabendo que ∆Hcombustão para o etano sob estas condições é - 1560 kJ.mol-1 de etano, calcule:
0
i. ∆Hformação para o etano. Dado:
ΔH° (kJ.mol-1)
CO2(g) - 393,5
H2O(l) - 285,8
ii. A energia de ligação para a ligação C = O.

Energia de ligação
(kJ.mol-1)
C-C 347
H-C 413
H-O 164
O=O 195
c) Sabendo que ∆G0 = - 1467,5 kJ.mol-1. Calcule ∆𝑆 0 para esta reação em J.mol-1.K-1.
Resolução:
Item a) Equação química: C2H6(l) + 7/2 O2(g) → 2 CO2(g) + 3 H2O(l)
Item b)
0 0 0
i. Cálculo da entalpia padrão do etano: ∆Hreação = ∑ Hprodutos − ∑ Hreagentes

0 0 7
0
∆Hreação = [2 x HCO + 3 x HH 2O
] − [1 x HC02 H6 + x HO0 2 ]
2 2
7
− 1560 = [{2 x (−393,5) + 3 x (−285,8)} − 1 x HC02 H6 − x 0]
2

− 1560 = −1644,40 − HC02 H6

HC02 H6 = −1644,40 + 1560 = −84,40 kJ

ii. Através da tabela apresentada pelo problema

Energia de ligação
(kJ.mol-1)
C - C 347
H - C 413
H - O 164
O = O 195

0 7
Cálculo da entalpia dos reagentes: Hreagentes = + [1 x 1 x HC−C + 6 x HC−H + x HO=O ]
2

0
7 kJ
Hreagentes = + [1 x 1 x 347 + 6 x 413 + x 195] = + 3507,5
2 mol
0
Cálculo da entalpia dos produtos: Hprodutos = − [2 x 2 x HC=O + 3 x 2 x HO−H ]

0
Hprodutos = − [4 x HC=O + 6 x 164]

395
1600 FÍSICO-QUÍMICA APLICADA EXERCÍCIOS COMENTADOS - IME – ITA – OLIMPÍADA

0
Hprodutos = − 4 x HC=O − 984

0 0 0
∆Hreação = Hreagentes + Hprodutos

− 1560 = + 3507,5 − 4 x HC=O − 984

− 1560 − 3507,5 + 984 = − 4 x HC=O

− 4083,5 kJ
HC=O = = +1020,87
−4 mol
0
Item c) Cálculo da entropia padrão (∆Sreação ). Sabendo que ∆G0 = - 1467,5 kJ.mol-1 (processo espontâneo) e
0 0 0 0
∆Hreação = 1560 kJ.mol-1 (processo exotérmico), temos: ∆Greação = ∆Hreação − T x ∆Sreação

0
− 1467,5 + 1560 = −298 x ∆Sreação

0
− 1467,5 + 1560 kJ
∆Sreação = = − 0,310
−298 mol x K

Questão 803 – Considere um tubo cilíndrico com condutividade térmica igual a 47 W/m.K, com diâmetro interno igual
a 8 centímetros e 5,5 milímetros de espessura apresentando uma temperatura interna igual a 325 K. O mesmo tubo
cilíndrico é coberto por uma camada de 9,0 centímetros de isolante com condutividade térmica igual a 50 W/m.K e
com uma outra camada, agora com 4,0 centímetros de um segundo isolante e condutividade térmica igual a 0,35
W/m.K. Considerando que a temperatura da superfície externa do isolante seja igual a 55 K, determine o valor da
quantidade de calor por comprimento.

Resolução: Desenvolvendo a equação de Fourier para uma configuração cilíndrica e sabendo que a área é A = 2.π.r.L
dT
e aproximando dx ≅ dr, temos a seguinte relação: q = −k. A. ( dx )

dT
q = −k. (2. π. r. L). ( )
dr
1
( ) . dr. q = −k. (2. π. L). dT
r
r2 T2
q 1
. [∫ ( ) . dr] = −2. π. k. [∫ dT]
L r1 r T1

q
. [lnr2 − lnr1 ] = −2. π. k. (T2 − T1 )
L
r2
q ln (r1 )
.[ ] = (T1 − T2 )
L 2. π. k

r2
q ln (r1 )
.[ ] = ∆T
L 2. π. k

396
1600 FÍSICO-QUÍMICA APLICADA EXERCÍCIOS COMENTADOS - IME – ITA – OLIMPÍADA

r r r
ln( 2) ln( 3) ln( 4)
r1 r2 r3
Cálculo da resistência térmica do conjunto: R t = [ 2.π.k + 2.π.k
+ 2.π.k
]

4 + 0,55 4 + 0,55 + 9 4 + 0,55 + 9 + 4


ln ( ) ln ( 4 + 0,55 ) ln ( 4 + 0,55 + 9 )
Rt = [ 4 + + ]
2. π. (47) 2. π. (50) 2. π. 0,35

0,129 1,091 0,259 K


Rt = [ + + ] = 0,122
295,16 314 2,198 W

K
R t = 0,122
W
q
Cálculo da quantidade de calor por comprimento: L x R t = ∆T

q
x (0,122) = (325 − 55)
L
q W
= 2217,73
L m

Questão 804 – Um material civil com espessura igual a 10 centímetros e condutividade térmica igual a 0,7 W/m.K,
está exposta a um vento frio de três graus negativos apresentando coeficiente convectivo igual a 40 W/m².K. O outro
lado do material está em contato com o ar quente a temperatura igual a 57°C, apresentando coeficiente convectivo
igual a 10 W/m2.K. Diante destas informações, determine a quantidade de calor em função da área.
1 x
Resolução: Cálculo da resistência total do sistema de transmissão de calor: q . {( ) + ( ) +
h.A quente k.A material
1
( ) } = ∆T
h.A frio

q 1 x 1
. {( ) + ( ) + ( ) } = ∆T
A h. quente k material h frio

q 1 0,10 1
. {( ) + ( ) + ( ) } = (273 + 57) − (273 − 3)
A 10. quente 0,7 material 40 frio

q
x {0,268} = 330 − 270
A
q 60 W
= = 223,80 2
A 0,268 m

397
1600 FÍSICO-QUÍMICA APLICADA EXERCÍCIOS COMENTADOS - IME – ITA – OLIMPÍADA

Questão 805 – Um tubo de carbono apresenta condutividade térmica igual a 60,3 W/m.°C. Em sua estrutura apresenta
duas medidas, uma com 10 cm de diâmetro externo e 2 cm de espessura. Sabendo que a temperatura da parede
interna do tubo é igual a 200°C e a externa é igual a 20°C. A partir desta informação, determine a quantidade de calor
por metro de comprimento de tubo (W/m).

Resolução: Desenvolvendo a equação de Fourier para uma configuração cilíndrica e sabendo que a área é A = 2.π.r.L
e aproximando dx ≅ dr, temos a seguinte relação:
𝑑𝑇
𝑞 = −𝑘. 𝐴. ( )
𝑑𝑥
𝑑𝑇
𝑞 = −𝑘. (2. 𝜋. 𝑟. 𝐿). ( )
𝑑𝑟
1
( ) . 𝑑𝑟. 𝑞 = −𝑘. (2. 𝜋. 𝐿). 𝑑𝑇
𝑟
𝑟2 𝑇2
𝑞 1
. [∫ ( ) . 𝑑𝑟] = −2. 𝜋. 𝑘. [∫ 𝑑𝑇]
𝐿 𝑟1 𝑟 𝑇1

𝑞
. [𝑙𝑛𝑟2 − 𝑙𝑛𝑟1 ] = −2. 𝜋. 𝑘. (𝑇2 − 𝑇1 )
𝐿
𝑞 𝑟2
. [𝑙𝑛 ( )] = 2. 𝜋. 𝑘. (𝑇1 − 𝑇2 )
𝐿 𝑟1

𝑞 5
Inserindo os valores na equação matemática acima, temos: 𝐿 𝑥 [𝑙𝑛 (3)] = 2 𝑥 (3,14) 𝑥 (60,5) 𝑥 (200 − 20)

𝑞 68389,2 𝑊
= = 133,88
𝐿 5 𝑚
𝑙𝑛 (3)

Questão 806 – Uma parede é formada por três materiais diferentes apresentadas logo abaixo.

Cobre, com espessura igual a 2,50 cm e condutividade térmica igual a 390 W/m°C;

Amianto, com espessura igual a 3,20 mm e condutividade térmica igual a 0,160 W/m°C;

Fibra de vidro, com espessura igual a 5,00 cm e condutividade térmica igual a 0,048 W/m°C.

Sabendo que a diferença de temperatura entre as duas fases que compreende estaa parede é igual a 560°C e diante
de todas estas informações, determine o fluxo de calor por unidade de área (W/m²).

Resolução: Através da equação de Fourier, temos a seguinte relação:

x x
q . {k.A} = ∆T, em que a relação {k.A} é definida como resistência térmica (Rt).

Como as paredes estão em sequência, ou seja, em série, teremos uma situação igual a resistência em série para
eletricidade.

𝑥 𝑥 𝑥
𝑞 . {( ) + ( ) + ( ) } = ∆𝑇
𝑘. 𝐴 𝑐𝑜𝑏𝑟𝑒 𝑘. 𝐴 𝑎𝑚𝑖𝑎𝑛𝑡𝑜 𝑘. 𝐴 𝑓𝑖𝑏𝑟𝑎 𝑑𝑒 𝑣𝑖𝑑𝑟𝑜

398
1600 FÍSICO-QUÍMICA APLICADA EXERCÍCIOS COMENTADOS - IME – ITA – OLIMPÍADA

2,50 𝑥 10−2 3,20 𝑥 10−4 5,00 𝑥 10−2


𝑞 . {( ) + ( ) + ( ) } = 560
390. 𝐴 𝑐𝑜𝑏𝑟𝑒
0,166. 𝐴 𝑎𝑚𝑖𝑎𝑛𝑡𝑜 0,048. 𝐴 𝑓𝑖𝑏𝑟𝑎 𝑑𝑒 𝑣𝑖𝑑𝑟𝑜

𝑞 2,50 𝑥 10−2 3,20 𝑥 10−4 5,00 𝑥 10−2


. {( ) + ( ) + ( ) } = 560
𝐴 390 𝑐𝑜𝑏𝑟𝑒
0,166 𝑎𝑚𝑖𝑎𝑛𝑡𝑜
0,048 𝑓𝑖𝑏𝑟𝑎 𝑑𝑒 𝑣𝑖𝑑𝑟𝑜

𝑞
. {1,061} = 560
𝐴
𝑞 𝑊
= 527,89 2
𝐴 𝑚

Questão 807 – (ITA - Adaptado) Duas salas idênticas estão separadas por uma divisória de espessura L = 5,0 cm,
área A = 100m2 e condutividade térmica k = 2,0 W/m.K. O ar contido em cada sala encontra-se, inicialmente, à
temperatura T1 = 47°C e T2 = 27°C, respectivamente. Considerando o ar como um gás ideal e o conjunto das duas
salas um sistema isolado, calcule o fluxo de calor através da divisória relativo às temperaturas iniciais T 1 e T2.

Resolução: Cálculo da quantidade de calor através da equação de transmissão de calor por condução,
𝑑𝑇
𝑞 = −𝑘. 𝐴. (𝑑𝑥).

x T
Aplicando a integral na equação acima, temos: q ∫0 dx = −k. A. ∫T II dT
I

0,05 300
q∫ dx = −k. A. ∫ dT
0 320

W
q. (0,05 − 0) = −2,0 . 100 m2 . (300 − 320)K
m. K
4000
q= = 80000 W (80 kW)
0,05

Questão 808 – A parede de um reator CSTR (continuous flow stirred tank reactors) é feita por uma camada com
espessura igual a quinze centímetros de um refratário que apresenta condutividade térmica expressa por uma
equação em função da temperatura apresentada a seguir, k = 0,15 + 10-4 x T.

Resolução: Cálculo da quantidade de calor através da equação de transmissão de calor por condução,
𝑑𝑇
𝑞 = −𝑘 𝑥 𝐴 𝑥 (𝑑𝑥).

𝑞
( ) 𝑥 𝑑𝑥 = −(0,15 + 10−4 𝑇) 𝑥 𝑑𝑇
𝐴
𝑞 0,15 250
Aplicando a integral na equação acima, temos: (𝐴) . ∫0 𝑑𝑥 = − ∫1050(0,15 + 10−4 𝑇). 𝑑𝑇

250
′ 250
T2
q′ x (0,15 − 0) = −[0,15 x T|1050 ] + 10−4 x | ]
2 1050

(250)² (1050)²
q′′ x 0,15 = −[0,15 x (250 − 1050)] + 10−4 x { − }
2 2

q′′ x 0,15 = −{[[0,15 x (250 − 1050)] + 10−4 x (31250 − 551250)]}

399
1600 FÍSICO-QUÍMICA APLICADA EXERCÍCIOS COMENTADOS - IME – ITA – OLIMPÍADA

q′′ x 0,15 = − (−120 − 52)

172 W
q′′ = = 1147,67 2
0,15 m

Questão 809 – Um fluido a 300C com coeficiente convectivo igual a 35 W/m2.K toca a superfície de uma placa metálica
que apresenta espessura igual a 0,15 metros, área superficial igual a 0,080 m2 e condutividade térmica (k) igual a 237
W/m.oC. Sabendo que a temperatura da face direita da placa é igual a 135°C, determine as seguintes alternativas.
a) A quantidade de calor trocada pelo sistema.
b) A temperatura da face esquerda da placa metálica.

Resolução:
Item a) Cálculo da quantidade de calor trocada a partir da quantidade de calor convectivo:
q = h. A. (Ts − T∞ ), onde Ts é a temperatura da superfície e T∞ é a temperatura do fluido.

q = h. A. (Ts − T∞ )

q = 35 x 0,080 x (135 − 30)

q = 294 W
0,15
Item b) Cálculo da temperatura da face esquerda, utilizando o processo de transmissão condutivo: q. ∫0 dx =
T
−k. A. ∫T d=135°C dT
e

294 x (0,15 − 0) = −(237) x 0,080 x (135 − Td )

− 44,1
= 135 − Td
18,96

Td = 137,32°C

Questão 810 – Uma determinada parede plana com condutividade térmica (k) igual a 40 W/m.K apresenta uma
espessura designada pela letra W com área transversal igual a 0,20 m2. Este material apresenta sua temperatura
esquerda fixada em aproximadamente igual a 1800C. A partir destas informações, calcule a espessura da placa se no
lado direito da parede plana tivermos fluido (ar) a 200C com coeficiente convectivo igual a 20 W/m2K, se 2000 W
tiverem que ser dissipados através do conjunto.

Resolução: Cálculo da temperatura da face direita aplicando a equação convectiva de Newton:

q = h. A. (Ts − T∞ )

2000 = 20 x 0,20 x (Td − 20)

500 = Td − 20

A temperatura da superfície que se trata da temperatura da face direita da parede é igual a 540°C.

Cálculo da espessura da parede a partir do processo de transmissão condutivo.


W 180
q. ∫ dx = −k. A. ∫ dT
0 540

2000 x (W − 0) = − (40) x 0,20 x (180 − 540)

400
1600 FÍSICO-QUÍMICA APLICADA EXERCÍCIOS COMENTADOS - IME – ITA – OLIMPÍADA

2000 x W = −8 x (−340)

W = 1,36 m

Questão 811 – Considere a parede de um reator CSTR construída com tijolo refratário apresentando 45 centímetros
de espessura e com condutividade térmica igual a 1,70 W/m.K. Um engenheiro de operações industriais realizou
medidas ao longo do processo de produção em estado estacionário, apresentando um decréscimo de temperaturas
de 1400 K e 1250 K proveniente das paredes interna e externa. Determine a taxa de calor, sabendo que a parede do
reator apresenta uma altura de 1,30 metros e 0,50 metros de largura.

Resolução: Desenvolvendo a equação de Fourier, temos: q. dx = −k. A. dT

0,45 1250
q. ∫ dx = −k. A. ∫ dT
0 1400

q. (0,45 − 0) = −(1,70). (0,50 x 1,30). (1250 − 1400)

q. (0,45) = −(1,70). (0,65). (−150)

q = 368,33 W

Questão 812 – Determine a espessura de uma parede de alvenaria que apresenta condutividade térmica de 0,75
W/m.K, considerando que a taxa de transferência de calor através dessa parede seja 75% da taxa de T.C. de uma
parede com condutividade térmica de 0,25 W/m.K e espessura de 100 mm. Observação: as superfícies das paredes
apresentam a mesma diferença de temperatura.

Resolução: Dados do problema: kalvenaria = 0,75 W/m.°C; kparede = 0,25 W/m.°C e Espessura da parede = 100 mm.

Sabendo que a transferência de calor por condução é dada pela equação matemática de Fourier, temos: 𝑞 =
𝑑𝑇
−𝑘. 𝐴. (𝑑𝑥). Desenvolvendo a equação de Fourier, temos: 𝑞. 𝑑𝑥 = −𝑘. 𝐴. 𝑑𝑇

𝑥 𝑇2
𝑞. ∫ 𝑑𝑥 = −𝑘. 𝐴. ∫ 𝑑𝑇
0 𝑇1

𝑞. (𝑥 − 0) = −𝑘. 𝐴. (𝑇2 − 𝑇1 )
𝑥
𝑞. { } = (𝑇1 − 𝑇2 )
𝑘. 𝐴

Informação importante do problema: 𝑞𝑎𝑙𝑣𝑒𝑛𝑎𝑟𝑖𝑎 = 0,75. 𝑞𝑝𝑎𝑟𝑒𝑑𝑒 .

Cálculo da espessura da parede de alvenaria:

𝑞𝑎𝑙𝑣𝑒𝑛𝑎𝑟𝑖𝑎 = 0,75. 𝑞𝑝𝑎𝑟𝑒𝑑𝑒

(𝑇1 − 𝑇2 ) (𝑇1 − 𝑇2 )
[ 𝑥 ] = 0,75. [ 𝑥 ]
{𝑘. 𝐴} {𝑘. 𝐴}
𝑎𝑙𝑣𝑒𝑛𝑎𝑟𝑖𝑎 𝑝𝑎𝑟𝑒𝑑𝑒

𝑘. 𝐴 𝑘. 𝐴
[ ] = 0,75. [ ]
𝑥 𝑎𝑙𝑣𝑒𝑛𝑎𝑟𝑖𝑎 𝑥 𝑝𝑎𝑟𝑒𝑑𝑒

401
1600 FÍSICO-QUÍMICA APLICADA EXERCÍCIOS COMENTADOS - IME – ITA – OLIMPÍADA

0,75 0,25
[ ] = 0,75. [ ]
𝑥 𝑎𝑙𝑣𝑒𝑛𝑎𝑟𝑖𝑎 100 𝑚𝑚 𝑝𝑎𝑟𝑒𝑑𝑒

1 0,25
[ ] = [ ]
𝑥 𝑎𝑙𝑣𝑒𝑛𝑎𝑟𝑖𝑎 100 𝑚𝑚 𝑝𝑎𝑟𝑒𝑑𝑒

x = 400 mm

Questão 813 – Considere que uma parede de uma residência seja composta por uma camada de 20 cm de espessura
de tijolo com condutividade térmica igual a k = 0,60 W/m.K e também junto a este tijolo uma outra camada de gesso
com espessura igual a 5,0 cm e condutividade térmica igual a k = 0,46 W/m.K. Determine a taxa de transferência de
calor por área, se a face externa composta pelo conjunto parede + gesso se encontra à 35°C e a face interna à 20°C.

Resolução: Sabendo que a transferência de calor por condução é dada pela equação matemática de Fourier, temos:
𝑑𝑇
𝑞 = −𝑘. 𝐴. ( )
𝑑𝑥

Desenvolvendo a equação de Fourier, temos:

𝑞. 𝑑𝑥 = −𝑘. 𝐴. 𝑑𝑇
𝑥 𝑇2
𝑞. ∫ 𝑑𝑥 = −𝑘. 𝐴. ∫ 𝑑𝑇
0 𝑇1

𝑞. (𝑥 − 0) = −𝑘. 𝐴. (𝑇2 − 𝑇1 )
𝑥
𝑞. { } = (𝑇1 − 𝑇2 )
𝑘. 𝐴
𝑥
Definindo a relação como a resistência térmica por condução e sabendo que o conjunto parede e gesso
𝑘.𝐴
encontram-se em série, temos:

𝑥 𝑥
𝑞. [{ } + { } ] = (𝑇1 − 𝑇2 )
𝑘. 𝐴 𝑝𝑎𝑟𝑒𝑑𝑒 𝑘. 𝐴 𝑔𝑒𝑠𝑠𝑜

0,20 0,05
𝑞. [{ } + { } ] = (35 − 20)
0,60. 𝐴 𝑝𝑎𝑟𝑒𝑑𝑒 0,46. 𝐴 𝑔𝑒𝑠𝑠𝑜

𝑞 0,20 0,05
.[ + ] = 15
𝐴 0,60 0,46
𝑞
. [0,442 ] = 15
𝐴
𝑞 15 𝑊
= = 33,94 2
𝐴 0,442 𝑚

Questão 814 – (IME) Mistura-se a água contida em dois recipientes, designados por A e B, de forma adiabática. Cada
um contém a mesma massa m de água no estado líquido. Inicialmente, as temperaturas são T no recipiente A e T +
ΔT no recipiente B. Após a mistura, a água atinge a temperatura final de equilíbrio térmico. Mostre que a variação de
entropia do processo de mistura é positiva. Dado: onde ∆S = m x Cp x ln(T2/T1), onde T2 e T1 são duas temperaturas
em dois estados diferentes do processo e cp é o calor específico da água, considerado constante.

402
1600 FÍSICO-QUÍMICA APLICADA EXERCÍCIOS COMENTADOS - IME – ITA – OLIMPÍADA

Resolução: Considere dois sistemas caracterizados pelas letras A e B, inseridos dentro de um reservatório.
água
Sistema A: calor específico de A = c e mA
água
Sistema B: calor específico de B = c e mB

Como o processo ocorre a pressão constante, Qp = H

Como o processo é adiabático, Qp = 0, logo: H = 0

Como são dois sistemas HA +HB = 0. (Equação A)


Desenvolvendo a equação A, temos: mA x CA x (Teq – TA) + mB x CB x (Taq – TB) = 0
Como as massas são iguais (mA = mB) e o fluido é o mesmo nos dois reservatórios (água), então a capacidade
calorífica é a mesma (cA = cB).
m x c x (Teq – TA) + m x c x (Teq – TB) = 0
m x c x (Teq – TA) = – m x c x (Teq –TB)
Teq – TA = –Teq + TB
2 x Teq = TB + TA (Equação B)
TB + TA
Definiremos a equação B da seguinte forma Teq = 2
e sabendo que a entropia de todo o sistema é dado por:
S = SA +SB, temos:
Ts Ts
∆S = m x cp x ln ( ) + m x cp x ln ( )
TA TB
Ts Ts
∆S = m x cp x {n ( ) x ( )}
TA TB
T2s
∆S = m x cp x {n (T )} (Equação C)
A x TB

Substituindo a equação B na equação C, temos:

T +T 2
( A 2 B)
∆S = m x cp x ln {[ ]}
TA x TB

(TA + TB )2
∆S = m x cp x ln { }
4 x TA x TB

TA2 + 2 x TA x TB + TB2
∆S = m x cp x ln
4 x TA x TB
TA2 + 2 x TA x TB + TB2
∆S = m x cp x ln
4 x TA x TB
(TA +TB )2
∆S = m x cp x ln (Equação D)
2 x √TA x TB

Sabendo que pela desigualdade das médias, a média aritmética (M.A.) é maior ou igual à média geométrica (M.G.),
ou seja, M. A. ≥ M. G.
Colocando em relação a temperatura, a função fica da seguinte maneira:

403
1600 FÍSICO-QUÍMICA APLICADA EXERCÍCIOS COMENTADOS - IME – ITA – OLIMPÍADA

TA + TB
≥ √TA x TB
2
(TA +TB )2
Manipulando a equação D apresentada acima, temos: ∆S = m x cp x ln
2 x √TA x TB

(TA + TB )
∆S = m x cp x ln x (TA + TB )
2 x √TA x TB

≥1

(TA + TB )
≥1
2 x √TA x TB
(TA + TB )
≥√
⏟TA x TB
⏟ 2
M.A M.G.

Logo ∆S > 0, pois as temperaturas são distintas.


Questão 815 – (IME) Num calorímetro a 17ºC colocamos 100 cm3 de água a 30ºC e 100 cm3 de água a 15ºC. A
temperatura de equilíbrio é 22ºC. Qual o equivalente em água do calorímetro?

Resolução:
Calor absorvido pela água fria: Q1 = m1x C x θ1 = 100 x 1 x (22 - 15) = 100.(7) = + 700 cal
Calor perdido pela água quente: Q2 = m2 x C x θ2 = 100 x 1 x (22 - 30) = 100.(-8) = - 800 cal
Calor absorvido pelo calorímetro: Qc = mc x C x θc = mc x 1 x (22 - 17) = 5 x mc
Q 1 + Q2 + Qc = 0
700 cal + (- 800 cal) + 5 x mc = 0
- 100 cal = - 5 x mc
mc = 20 g
Questão 816 - Uma amostra de dióxido de carbono sofre expansão isotermicamente a uma temperatura de 0°C de
0,224 cm3 para 0,448 cm3, realizando trabalho reversível de 300 kJ. A partir destas informações, determine a massa
deste gás no sistema, a variação da energia interna e o calor trocado.

Resolução: Informações do problema:

Massa de dióxido de carbono = ?


Massa molecular do dióxido de carbono = 44 g/mol
Vinicial = 0,224 cm³
Vfinal = 0,448 cm³
T = 0°C (273 K)

Cálculo da massa de dióxido de carbono, a partir do trabalho (W) realizado, considerando que o processo ocorre
isotermicamente.

mCO2 Volumefinal
W = −( ) x R x T x ln ( )
< MM >CO2 Volumeinicial

mCO2 0,448
− 300000 = − ( ) x 8,314 x 273 x ln ( )
44 0,224

404
1600 FÍSICO-QUÍMICA APLICADA EXERCÍCIOS COMENTADOS - IME – ITA – OLIMPÍADA

mCO2
300000 = ( ) x 8,314 x 273 x ln(2)
44
300000 x 44
mCO2 = = 8390,27 g (8,39 kg)
8,314 x 273 x ln(2)

Cálculo da variação da energia interna: ∆U = n x CV x ∆T = n x CV x 0 = 0

Cálculo da quantidade de calor trocada a partir da primeira lei da termodinâmica: ∆U = Q + W

0=Q+W

Q = − W = − (−300 kJ) = + 300 kJ

Questão 817 - (OLIMPÍADA DE QUÍMICA DA ARGENTINA) Em um compartimento de uma bomba calorimétrica


(volume constante) rodeado de 945 g de água, a combustão de 1,048 g de benzeno, C6H6(l), elevou a temperatura da
água desde 23,640°C a 32,692°C. A capacidade do calorímetro é de 891 kJ/°C e a da água é 4,184 J/g.°C.
a) Escreva a equação química balanceada para a reação de combustão do benzeno.
b) Calcule ΔUcombustão (C6H6) expresso em kJ/mol.

Resolução:

Item a) Equação química da combustão do benzeno: C6H6(l) + 15/2 O2(g) → 6 CO2(g) + 3 H2O(l).

Item b) Como o problema pede a variação da energia interna e o processo ocorre a volume constante, pela primeira
lei da termodinâmica, o trabalho realizado será igual a zero. Então: ∆U = Q + W = Q + 0

∆U = Q

O calor calculado será o sistema formado pelo calorímetro + água.

∆U = Q + W

∆U = Q + 0 = Q V

∆U = (m x c x ∆T)calorímetro + (m x c x ∆T)água

J J
∆U = [891 x (32,692 − 23,640)°C] + [945 g x 4,18 (32,692 − 23,640)°C]
°C g x °C

J J
∆U = [891 x (32,692 − 23,640)°C] + [945 g x 4,18 (32,692 − 23,640)°C]
°C g x °C

∆U = 8065,33 J + 35756,30 J = 43821,64 J

Se para:
1,048 g ---------- 43821,64 J
78 g -------------- QV
QV = 3,26 x 106 J

Como o processo é exotérmico, o valor da quantidade de calor a volume constante é igual a QV = - 3,26 x 106 J.

405
1600 FÍSICO-QUÍMICA APLICADA EXERCÍCIOS COMENTADOS - IME – ITA – OLIMPÍADA

Questão 818 – A capacidade calorífica de 2,50 mol de um gás ideal apresenta a seguinte expressão: Cp = 6 + 2,50 x
10-2T. A partir desta informação, determine a variação de entalpia que este gás sofre, a partir de um aquecimento em
que a temperatura varia de 125°C até 625°C.
Resolução: Sabendo que a entalpia é definida como dH = n x Cp x dT.
Cálculo da variação da entalpia: 𝑑𝐻 = 𝑛 𝑥 𝐶𝑝 𝑥 𝑑𝑇
𝐻 𝑇
Aplicando a integral, temos: ∫𝐻 2 𝑑𝐻 = ∫𝑇 2 𝑛 𝑥 𝐶𝑝 𝑥 𝑑𝑇
1 1

𝑇2
𝐻2 − 𝐻1 = ∫ 𝑛 𝑥 𝐶𝑝 𝑥 𝑑𝑇
𝑇1

𝑇2
∆𝐻 = ∫ 𝑛 𝑥 𝐶𝑝 𝑥 𝑑𝑇
𝑇1

(625+273)
∆𝐻 = ∫ 2,50 𝑥 (6 + 2,50 𝑥 10−2 )𝑑𝑇
(125+273)

898
∆𝐻 = 2,50 𝑥 ∫ (6 + 2,50 𝑥 10−2 )𝑑𝑇
398
898 898
∆𝐻 = 2,50 𝑥 (∫ 6 𝑥 𝑑𝑇 + ∫ 2,50 𝑥 10−2 𝑥 𝑇 𝑥 𝑑𝑇)
398 398
898
𝑇²
∆𝐻 = 2,50 𝑥 (6 𝑥 𝑇|898
398 + 2,50 𝑥 10 −2
𝑥 | )
2 398

(898)² (398)²
∆𝐻 = 2,50 𝑥 {6 𝑥 (898 − 398) + 2,50 𝑥 10−2 𝑥 [ − ]}
2 2

∆𝐻 = 2,50 𝑥 {3000 + 2,50 𝑥 10−2 𝑥 324000} = + 27750 𝑐𝑎𝑙 (27,75 𝑘𝑐𝑎𝑙)


Questão 819 – Calcule o calor de combustão do resfriamento de 800 g de oxigênio molecular, em que a temperatura
varia de 200°C para 10°C. Informação para a resolução do problema: Cp = 29,96 + 4,18 x 10-3 T – 1,67 x 10-5 T³,
𝑐𝑎𝑙
sabendo que a unidade da capacidade calorífica a pressão constante é dada por 𝑚𝑜𝑙 𝑥 °𝐶 .
mO 800
Resolução: Cálculo do número de mol de oxigênio molecular: nO2 = <MM>2 = 32
= 25 mol
O2
Cálculo da variação da entalpia: dH = n x Cp x dT
H T
Aplicando a integral, temos: ∫H 2 dH = ∫T 2 n x Cp x dT
1 1

T2
H2 − H1 = ∫ n x Cp x dT
T1

T2
∆H = ∫ n x Cp x dT
T1

(10+273)
∆H = ∫ 25 x (29,96 + 4,18 x 10−3 T − 1,67 x 10−5 T³ )dT
(200+273)

283
∆H = 25 x ∫ (29,96 + 4,18 x 10−3 T − 1,67 x 10−5 T³ )dT
473

406
1600 FÍSICO-QUÍMICA APLICADA EXERCÍCIOS COMENTADOS - IME – ITA – OLIMPÍADA

283 283 283


∆H = 25 x ∫ 29,96 dT + ∫ 4,18 x 10−3 T. dT − ∫ 1,67 x 10−5 T³ . dT
473 473 473
283 283
T² T4
∆H = 25 x (29,96 x T|283
473 + 4,18 x 10
−3
x | − 1,67 x 10−5 x | )
2 473 4 473

∆H = 25 x {29,96 x (−190) + 4,18 x 10−3 x (−71820) − 1,67 x 10−5 x (−10910104380)}

∆H = 25 x {−5692,4 + 300,21 + 182198,74 } = 25 x 176806,55 = 4,42 x 106 cal

Questão 820 – 1000 dm³ de um determinado gás ideal que se encontra inicialmente a 600 K e 1000 kPa sofre
expansão para uma situação cinco vezes maior o volume inicial. A partir desta informação, determine a temperatura
final, a pressão final, a energia interna e o trabalho realizado pelo gás para cada item proposto, sabendo que a
capacidade calorífica a pressão constante é igual a 21 J/mol.K.
a) Através de um processo isotérmico e reversível.
b) Através de um processo adiabático e reversível.

Resolução: Item a) Através de um processo isotérmico e reversível:

Estado (I): PI = 1000 kPa; VI = V e TI = 600 K

Estado (II): PII = ?; VII = 5 x VI e TII = ?

Como o processo é isotérmico, a temperatura final será igual a temperatura inicial, ou seja, TI = TII = 600K
pI x VI pII x VII
Cálculo da pressão da situação final (pII), a partir da equação combinada dos gases ideais: TI
= TII

1000 kPa x VI pII x 5 x VI


=
600 TII

1000 kPa pII x 5


=
600 600

pII = 200 kPa

A variação da energia interna será igual à zero (ΔU = 0), pois o processo ocorre a temperatura constante.

Cálculo do trabalho, considerando o processo sendo isotérmico e reversível. Primeiramente será necessário calcular
o número de mol a partir da situação inicial, utilizando a equação dos gases ideais.

106
x 1000 = nI x 0,08206 x 600
105
10000
nI = = 203,10 mol
49,24

Volumefinal
Cálculo do trabalho (W): W = −n x R x T x ln ( )
Volumeinicial

5V
W = −203,10 x 8,314 x 600 x ln ( )
V

W = −203,10 x 8,314 x 600 x ln(5) = − 1630,59 kJ

407
1600 FÍSICO-QUÍMICA APLICADA EXERCÍCIOS COMENTADOS - IME – ITA – OLIMPÍADA

Item b) Analisando para um processo adiabático e reversível. Cálculo da capacidade calorífica a volume constante:
Cp − CV = R

CV = Cp − R

J
CV = 21 − 8,314 = 12,69
mol x K
C
Cálculo da relação entre a capacidade calorífica a pressão constante e a volume constante (γ): γ = Cp =
V
J
21,0
mol x K
J = 1,65
12,69
mol x K

γ γ
Cálculo da pressão final (PII): PI x VI = PII x VII
V 1,65
1000 x ( ) = PII
5V

1 1,65
PII = 1000 x ( ) = 70,26 kPa
5
γ−1 γ−1
Cálculo da temperatura final (TII): TI x VI = TII x VII

VI1,65−1
600 x = TII
VII1,65−1

V 1,65−1
TII = 600 x ( )
5xV

1 0,65
TII = 600 x ( ) = 210,77 K
5

Sendo o processo adiabático, Q = 0. Pela primeira lei da termodinâmica, temos: ∆U = Q + W


∆U = 0 + W

∆U = W

Cálculo da energia interna (ΔU): ∆U = W = n x CV x ∆T


∆U = W = 203,10 x 12,69 x (210,77 − 600)

∆U = W = −203,10 x 12,69 x 389,23 = −1003177,66 J ( −1003,18 kJ)

Questão 821 – Considere as duas situações que ocorrem para um gás ideal poliatômico que sofre transformação
adiabática no qual a relação P1 x V1γ é válida.
Estado (I): PI = ?; VI = 110 L e TI = 320 K

Estado (II): PII = 1,0 atm; VII = 250 L e TII = ?

Determine o coeficiente termodinâmico γ, a pressão inicial no estado (I) e a temperatura final no estado (II).

Resolução: Processo adiabático consiste em um processo idealizado em que não há troca de calor (Q = 0).

408
1600 FÍSICO-QUÍMICA APLICADA EXERCÍCIOS COMENTADOS - IME – ITA – OLIMPÍADA

Cálculo da capacidade a pressão constante, a partir da relação das capacidades caloríficas e considerando o gás
como poliatômico (CV = 3R): Cp − CV = R
Cp − 3R = R
Cp = 4R

Cálculo da relação adiabática (γ) entre a capacidade calorífica a pressão constante e a capacidade calorífica a volume
C 4R
constante (γ): γ = Cp = 3R = 1,33
V

γ γ
Cálculo da pressão do estado I (pI): PI x VI = PII x VII

PI x (110)1,33 = 1,0 x (250)1,33

250 1,33
PI = 1,0 x ( ) = 2,98 atm
110
γ−1 γ−1
Cálculo da temperatura no estado II (TII): TI x VI = TII x VII

γ−1
TI VII
=
TII V γ−1
I

320 250 1,33−1


=( )
TII 110

320
= (2,27)0,33
TII

320 320
TII = = = 244,27 K
(2,27)0,33 1,31

Questão 822 – Considere uma amostra gasosa monoatômico de 1,25 mol, apresentando as seguintes características
quanto a pressão e temperatura no estado inicial: pI = 1,00 atm e TI = 350 K. Este gás é aquecido de forma reversível
até 450 K, considerando que o processo ocorre a volume constante, determine:
a) A pressão final
b) O trabalho realizado
c) A energia interna
d) A quantidade de calor
e) A entalpia

Resolução:

Item a) Cálculo da pressão final (PII), a partir da equação combinada dos gases ideais:

Estado (I): PI = 1,00 atm; VI = V e TI = 350 K

Estado (II): PII = ?; VII = V e TII = 450 K

pI x VI pII x VII
=
TI TII

1,0 x VI pII x VI
=
350 450
pII = 1,28 atm

409
1600 FÍSICO-QUÍMICA APLICADA EXERCÍCIOS COMENTADOS - IME – ITA – OLIMPÍADA

Item b) Como o processo é isovolumétrico, ou seja, o sistema ocorre a volume constante, o trabalho será igual a zero,
W = 0.

Item c) Sendo a molécula monoatômica, a capacidade calorífica a volume constante será calculada a partir da seguinte
relação:

3 3 J
𝐶𝑉 = 𝑥 𝑅 = 𝑥 8,314 = 12,47
2 2 mol x K

Cálculo da energia interna (ΔU): ∆𝑈 = 𝑛 𝑥 𝐶𝑉 𝑥 ∆𝑇

𝐽
∆𝑈 = 1,25 𝑚𝑜𝑙 𝑥 12,47 𝑥 (450 − 350)𝐾 = +1558,75 𝐽
𝑚𝑜𝑙 𝑥 𝐾

Item d) Através da primeira lei da termodinâmica, temos: ∆𝑈 = 𝑄 + 𝑊

∆𝑈 = 𝑄 + 0

∆𝑈 = 𝑄 = +1558,75 𝐽

Item e) Cálculo da capacidade calorífica a pressão constante e da variação de entalpia (ΔH):

Cp − CV = R

3R
Cp − =R
2
5R
Cp =
2

Cálculo da variação de entalpia (ΔH): ∆𝐻 = 𝑛 𝑥 𝐶𝑝 𝑥 ∆𝑇

5
∆𝑈 = 1,25 𝑥 𝑥 8,314 𝑥 (450 − 350) = + 2598,12 𝐽
2

Questão 823 – Um determinado gás ideal apresenta capacidade calorífica a volume constante igual a 6,76 cal.mol -
1.K-1. Se 10 mols deste gás são aquecidos em um intervalo entre 373 K a 473 K, determine a variação da energia

interna e a entalpia do referido processo.


Resolução: Dados do problema: Capacidade calorífica a volume constante: CV = 6,76 cal.mol-1.K-1; Número de mol:
10 mol.
Cálculo da energia interna (ΔU): ∆U = n x CV x ∆T

cal
∆U = 10 mol x 6,76 x (473 − 373)K
mol x K
cal
∆U = 10 mol x 6,76 x 100 K
mol x K

∆U = +6760 cal

Cálculo da capacidade calorífica a pressão constante: Cp − CV = R

Cp − 6,76 = 1,987

410
1600 FÍSICO-QUÍMICA APLICADA EXERCÍCIOS COMENTADOS - IME – ITA – OLIMPÍADA

cal
Cp = 8,75
mol x K

Cálculo da entalpia (ΔH): ∆H = n x Cp x ∆T

cal
∆H = 10 mol x 8,75 x (473 − 373)K
mol x K
cal
∆H = 10 mol x 8,75 x 100K = + 8750 cal
mol x K

Questão 824 – Um gás monoatômico (n = 1,50 mol) e considerando comportamento ideal tem o seu volume
comprimido mediante um pistão, de 3 m³ para 1,2 m³, isso por causa de uma compressão isotérmica a uma
temperatura igual a 300 K. Calcule:
a) O trabalho realizado pelo processo.
b) A variação da energia interna.
c) O calor envolvido pelo processo. O sistema recebe ou cede calor?

Resolução: Dados do problema: n = 1,50 mol; Volume inicial = 3 m³; Volume final = 1,20 m³ e T = 300 K

Item a) Para um processo isotérmico e compressível, temos: Cálculo do trabalho realizado (W): W =
Volume
−n x R x T x ln (Volume final )
inicial

cal 1,20 m3
W = −1,50 mol x 1,987 x 300 K x ln ( ) = + 819,04 cal
mol x K 3,0 m3

Item b) A variação da energia interna será igual a zero, uma vez que o processo é isotérmico (temperatura constante).

Item c) Através da primeira lei da termodinâmica, temos: ∆U = Q + W

0=Q+W

Q = −W = − 819,04 cal

Questão 825 – Dois litros de nitrogênio gasoso a 0°C e pressão igual a 5 atm expande reversivelmente e
isotermicamente, ficando confinado a uma pressão igual a 1 atm. Assumindo que o gás apresenta comportamento
ideal, calcule W, q, ΔU e ΔH para este processo de expansão.
Resolução: Informação do problema: Processo isotérmico (temperatura constante) e reversível: ∆U = ∆H = 0.
Cálculo do volume final (2) e do número de mol:
Estado I: PI = 5 atm; VI = 2,0 L e TI = 0 + 273 = 273 K
Estado II: PII = 1 atm; VII = ? e TII = 273 K
5x2 1 x VII
Utilizando a equação combinada dos gases ideais, temos: 273
= 273

VII = 10 L
pxV 5x2 10
Cálculo do número de mol de nitrogênio gasoso, a partir da situação inicial (1): n = R x T = 0,08206 x 273 = 22,40 =
0,446 mol

411
1600 FÍSICO-QUÍMICA APLICADA EXERCÍCIOS COMENTADOS - IME – ITA – OLIMPÍADA

Volume
Cálculo do trabalho (W) realizado reversivelmente e isotermicamente: W = −n x R x T x ln (Volume final )
inicial

cal 10
W = −0,446 mol x 1,987 x 273 x ln ( )
mol x K 2,0
cal
W = −0,446 mol x 1,987 x 273 x ln(5) = −389,38 cal
mol x K

Cálculo da quantidade de calor, a partir da primeira lei termodinâmica: ∆U = Q + W

0=Q+W

Q = −W = −(−389,38 cal) = + 389,38 cal

Questão 826 – Um quilograma de ar atmosférico é aquecido de forma reversível a pressão constante de 27°C e 1
atm, até que o seu volume triplique. Calcule W, Q, ΔU e ΔH para o processo sabendo que o ar apresenta capacidade
calorifica a pressão constante igual a 29 J/mol.K.
Resolução: Cálculo da massa molar do ar atmosférico (<MM>ar), considerando a seguinte composição: 20% de O2 e
80% de N2.
<MM>ar = 0,20 x <MM>O2 + 0,80 x <MM>N2
<MM>ar = (0,20 x 32) g.mol-1 + (0,80 x 28) g.mol-1 = 28,80 g.mol-1
m 1000
Cálculo do número de mol do ar atmosférico: nar = <MM>
ar
= 28,80
= 34,72 mol
ar

Estado I: PI = 1,0 atm; VI = V e TI = 27 + 273 = 300 K

O problema considera que o processo é isobárico (pressão constante).


Estado II: PII = P; VII = 3V e TII = ?
pI x VI pII x VII
Cálculo da temperatura II, a partir da equação combinada dos gases ideais: TI
= TII

p x V p x 3V
=
300 TII
1 3
=
300 TII
TII = 900 K
Cálculo da capacidade a volume constante (CV): Cp − CV = R
29 − CV = 8,314
J
CV = 20,69
mol x K

Cálculo da variação da energia interna (ΔU): ∆U = n x CV x ∆T


J
∆U = 34,72 mol x 20,69 x (900 − 300)K = +431014,08 J (431,01 kJ)
mol x K

Cálculo da variação da entalpia: ∆H = n x Cp x ∆T


J
∆H = 34,72 mol x 29 x (900 − 300)K = +604128 J (604,13 kJ)
mol x K

412
1600 FÍSICO-QUÍMICA APLICADA EXERCÍCIOS COMENTADOS - IME – ITA – OLIMPÍADA

Cálculo do trabalho realizado, a partir da definição da entalpia: ∆H = ∆U + pV

∆H = ∆U − W

604,13 kJ = 431,01 kJ − W

W = −173,12 kJ
Cálculo da quantidade de calor (Q): ∆H = Q = + 604,13 kJ
Questão 827 – Um mol de gás ideal apresenta uma capacidade calorífica a volume constante igual a CV = (5R/2),
apresentando as seguintes situações destacadas abaixo, ocorrendo transformação reversível.
Estado (I): PI = 8,00 atm; V1 = ? e T1 = 600 K

Estado (2): P2 = 1 atm; V2 = ? e T2 = ?

Estes estados estão de acordo com os seguintes processos descritos em


a) processo isocórico;
b) processo isotérmico;
c) processo adiabático.

Resolução: Item a) Considerando o processo isocórico (volume constante), o trabalho realizado é igual a zero (W =
0).

1 x 0,08206 x 600
Cálculo do volume para o processo (I), considerando comportamento ideal: VI = 8
= 6,15 L

pI x VI pII x VII
Cálculo da temperatura para o processo (II) utilizando a equação combinada dos gases ideais: TI
= TII

8,0 x V 1,0 x V
=
600 TII

TII = 75 K

Cálculo da energia interna (ΔU): ∆U = Q + W

∆U = Q V + 0

∆U = Q V , onde QV é a quantidade de calor a volume constante.

∆U = Q V = n x CV x ∆T

5
∆U = Q V = 1 mol x x R x ∆T
2
5
∆U = Q V = 1 mol x x 8,314 x (75 − 600) = −10912,125 J (−10,91 kJ)
2

Cálculo da entalpia (ΔH): ∆H = n x Cp x ∆T

∆H = n x (R + CV ) x ∆T

413
1600 FÍSICO-QUÍMICA APLICADA EXERCÍCIOS COMENTADOS - IME – ITA – OLIMPÍADA

5R 7R
∆H = n x (R + ) x ∆T = n x ( ) x ∆T
2 2
7 J
∆H = 1 mol x 2 x 8,314 mol x K x (75 − 600) = −15276,8975 J ( −15,28 kJ)

Item b) Para um processo isotérmico (temperatura é constante), logo ∆U = ∆H = 0.

Cálculo da quantidade de calor (Q) e do trabalho (W) a partir da primeira lei da termodinâmica: ∆U = Q + W

0=Q+W

Q=−W

Pressãoinicial
Cálculo do trabalho (W) realizado: W = −n x R x T x ln ( )
Pressãofinal
J 8 atm
W = −1 mol x 8,314 x 600 K x ln ( )
mol x K 1 atm

W = −10373,09 J (−10,4 kJ)

Logo, o calor realizado é igual a Q = - W = - (-10,4 kJ) = + 10,4 kJ.

Item c) Considerando o processo adiabático, a quantidade de calor será igual a zero (Q = 0). Pela primeira lei da
termodinâmica, sabemos da equação da conservação de energia.

∆U = Q + W
∆U = 0 + W
Q=+W

Estado (I): P1 = 8,00 atm; V1 = 6,15 L e T1 = 600 K

Estado (II): P2 = 8,00 atm; V2 = ? e T2 = ?


7R
Cp 7
Cálculo da relação entre a capacidade calorífica a pressão constante e a volume constante (γ): γ = CV
= 2
5R =5=
2
1,40
γ γ
Cálculo do volume (II), considerando o processo adiabático: PI x VI = PII x VII

8 atm x (6,15)1,40 = 1,0 atm x VII1,40


8 atm x (6,15)1,40
VII1,40 =
1,0 atm
1,40
VII = √101,74 = 27,16 L
γ−1 γ−1
Cálculo da temperatura (2), considerando processo adiabático: TI x VI = TII x VII
γ−1
TI VII
=
TII V γ−1
I

600 27,16 1,40−1


=( )
TII 6,15

414
1600 FÍSICO-QUÍMICA APLICADA EXERCÍCIOS COMENTADOS - IME – ITA – OLIMPÍADA

600
= (4,42)0,40
TII

600
TII = = 331,49 K
1,81
5
Cálculo da variação da energia interna (ΔU): ∆U = n x CV x ∆T = 1 mol x 2 x 8,314 x (331,49 − 600) =
−5580,98 J (−5,58 kJ)

Questão 828 – Um fluido gasoso está em uma tubulação cilíndrica de diâmetro D equipado por um pistão se
deslocando sem nenhum atrito e adiabático.
Pergunta 1: A energia na forma de trabalho pode ser transferida para o fluido?
Pergunta 2: Determine a variação de energia interna quando há aumento da pressão de PA para PB.
Resolução: Resposta da pergunta 1: Não pode haver transferência na forma de trabalho, pois como o pistão move-
se livremente, a expansão é caracterizada como livre, ou seja, W = 0.
Resposta da pergunta 2: Como o processo é adiabático, a troca de calor (Q) é igual a zero. Logo, pela primeira lei da
termodinâmica, temos:
ΔU = Q + W
ΔU = 0 + 0 = 0
ΔU = 0
Além disso, considerando o fluido como incompressível, não irá apresentar variação de pressão e também nem
variação em relação a temperatura, o que caracteriza que a energia interna igual a zero (ΔU = 0).
Questão 829 – O gás nitrogênio está confinado em um recipiente esférico com um pistão. A pressão inicial de gás
nitrogênio é igual a 7 atm e o volume inicial corresponde a 100 dm³. O pistão está fixo através de travas na parede
esférica do recipiente. Todo o conjunto (gás + recipiente esférico + pistão) é colocado no vácuo. A partir desta
informação, determine a variação da energia interna se as travas forem removidas de forma que o gás aumente o seu
volume inicial até dobrar. Observação: o pistão fica novamente fixo pelas travas no final do processo.
Resolução: O sistema a ser analisado e calculado refere-se ao gás nitrogênio (N2) junto com o recipiente esférico e
o pistão. Não há trabalho realizado, pois há forças externas sobre o sistema e além disso também não há nenhum
indício de transmissão de calor, uma vez que o sistema encontra-se no vácuo.
Logo, pela primeira lei da termodinâmica, temos:
ΔU = Q + W
ΔU = 0 + 0
ΔU = 0
Questão 830 – Um mol de nitrogênio molecular apresenta comportamento ideal considerando que a temperatura
medida foi igual a 25°C. Este gás se expande reversivelmente e adiabaticamente de 20 atm a 1 atm. Determine a
temperatura final, levando em consideração que a capacidade calorífica a volume constante é igual a 3/2R.
Resolução: Cálculo da capacidade a pressão constante, a partir da capacidade calorífica a volume constante:
𝐶𝑝 − 𝐶𝑉 = 𝑅

3𝑅
𝐶𝑝 − =𝑅
2

415
1600 FÍSICO-QUÍMICA APLICADA EXERCÍCIOS COMENTADOS - IME – ITA – OLIMPÍADA

3𝑅 5𝑅
𝐶𝑝 = +𝑅 =
2 2

Cálculo da relação entre a capacidade calorífica a pressão constante e a volume constante (γ):
5𝑅
𝐶𝑝 5
𝛾= = 2 = = 1,67
𝐶𝑉 3𝑅 3
2
1 x 0,08206 x (25+273)
Cálculo do volume inicial (I), a partir da utilização da equação dos gases ideais: VI = = 1,22 𝐿
20

Estado I: PI = 20 atm; VI = 1,22 L e TI = 298 K


Estado II: PII = 1 atm; VII = ? e TII = ?
𝛾 𝛾
Cálculo do volume (II), considerando processo adiabático: 𝑃𝐼 𝑥 𝑉𝐼 = 𝑃𝐼𝐼 𝑥 𝑉𝐼𝐼

20 𝑎𝑡𝑚 𝑥 (1,22)1,67 = 1,0 𝑎𝑡𝑚 𝑥 𝑉𝐼𝐼 1,67


20 𝑎𝑡𝑚 𝑥 (1,22)1,67
𝑉𝐼𝐼 1,67 =
1,0 𝑎𝑡𝑚
𝑉𝐼𝐼 1,67 = 27,88
1,67
𝑉𝐼𝐼 = √27,88 = 7,33 𝐿

Questão 831 – Cinco mol de um gás perfeito expande de forma isotérmica a uma temperatura constante igual a
127°C, para quatro vezes o seu volume. A partir desta informação, determine o trabalho realizado pelo gás e o fluxo
de calor em relação ao sistema.

Resolução: Cálculo do trabalho realizado, sabendo que o processo é isotérmico (temperatura constante): W =
V
−n x R x T x ln (V final )
inicial

J 5V
W = − 5 mol x 8,314 x (127 + 273) K x ln ( )
mol x K V

W = − 5 x 8,314 J x 400 x ln(5)

W = − 5 x 8,314 J x 400 x 1,61 = −26771 J (−26,77 kJ)

Pela primeira lei da termodinâmica e sabendo que o processo é isotérmico, a variação da energia interna é igual a
zero (ΔU = 0):
ΔU = Q + W

0=Q+W
Q=-W
Q = - (- 26,77 kJ) = + 26,77 kJ

416
1600 FÍSICO-QUÍMICA APLICADA EXERCÍCIOS COMENTADOS - IME – ITA – OLIMPÍADA

Questão 832 – Considere que um gás ideal sofre um processo de compressão adiabática e reversível, do volume

passando do estado I (VI) para o estado II (VII). A partir desta informação diga quais das grandezas apresentadas a
seguir serão nulas, positivas e negativas.

a) Temperatura no estado II (TII)


b) Variação da temperatura (ΔT)
c) Variação da pressão (ΔP)
d) Trabalho (W)
e) Quantidade de calor (Q)
f) Variação da entropia (ΔS)
g) Variação da energia interna (ΔU)
h) Variação da entalpia (ΔH)

Resolução:

a) Temperatura no estado II (TII) = Positivo. Como o processo ocorre com uma compressão adiabática, a
tendência da temperatura no estado II é ser maior em comparação a temperatura I, TII > TI.
b) Variação da temperatura (ΔT) = Positivo. Como a temperatura no estado II é maior que do estado I, TII > TI,
logo a variação será positiva.
c) Variação da pressão (ΔP) = Positivo. Como o processo se caracateriza como uma compressão, logo a
pressão do estado II é maior que do estado I, PII > PI. Ou, pode-se explicar da seguinte maneira: sendo uma
compressão, o volume diminui e a pressão aumenta.
d) Trabalho (W) = Positivo. Há trabalho realizado no gás.
e) Quantidade de calor (Q) = Nulo. Como o processo é adiabático, a quantidade de calor é igual a zero.
f) Variação da entropia (ΔS) = Nulo. Sendo o processo de compressão adiabática reversível, a variação da
entropia é isoentrópico.
g) Variação da energia interna (ΔU) = Positivo. Através da primeira lei da termodinâmica, temos a seguinte
equação da conservação de energia.
∆𝑈 = 𝑄 + 𝑊
∆𝑈 = 0 + 𝑊
∆𝑈 = 𝑊. Como o trabalho é positivo, a variação da energia interna será também positiva.
h) Variação da entalpia (ΔH) = Positivo. Pela definição de entalpia:
∆𝐻 = ∆𝑈 + 𝑝𝑉
∆𝐻 = ∆𝑈 + ∆𝑛 𝑥 𝑅 𝑥 𝑇
Como ∆𝑈 e ∆𝑇 são funções de estado positivas, com isso, ∆𝐻 também será positiva.

Questão 833 – Em um botijão de gás, a pressão interna é igual a dez atmosferas à uma temperatura igual a 20°C. A
partir desta informação, calcule a temperatura do gás quando sai do botijão e fica exposto a pressão constante.
Considere que o coeficiente termodinâmico que relaciona as capacidades caloríficas seja igual a 7/5 (γ).

Resolução: Cálculo do volume do estado I considerando como base de cálculo igual a 1 mol da espécie gasosa.

1 x 0,08206 x (20 + 273)


VI = = 2,40 L
10
γ γ
Cálculo do volume do estado II: PI x VI = PII x VII

10 atm x (2,40)1,40 = 1,0 atm x VII1,40

1,40 10 atm x (2,40)1,40


VII =
1,0 atm
VII1,40 = 34,10

417
1600 FÍSICO-QUÍMICA APLICADA EXERCÍCIOS COMENTADOS - IME – ITA – OLIMPÍADA

1,40
VII = √34,10 = 12,44 L
γ−1 γ−1
Cálculo da temperatura II: TI x VI = TII x VII

293 K x (2,40)1,40−1 = TII x (12,44)1,40−1


293 K x (2,40)1,40−1
TII =
(12,44)1,40−1
293 K x (2,40)0,40 293 K x 1,42
TII = = = 151,85 K (−121, 15°C)
(12,44)0,40 2,74
Questão 834 – (ITA – ADAPTADO) Certa quantidade de oxigênio (considerado aqui como gás ideal) ocupa um
volume V1 a uma temperatura T1 e pressão p1. A seguir, toda essa quantidade é comprimida, por meio de um processo
𝑣
adiabático e quase estático, tendo reduzido o seu volume para 𝑣𝑓𝑖𝑛𝑎𝑙 = 𝑖𝑛𝑖𝑐𝑖𝑎𝑙
2
. Dê o valor da expressão do trabalho
realizado sobre esse gás.

Resolução: Sendo o oxigênio ser uma molécula diatômica, então a capacidade calorífica a volume constante será
5𝑅
expressa pela seguinte expressão: 𝐶𝑉 = 2
.

Cálculo da capacidade calorífica a pressão constante: 𝐶𝑝 − 𝐶𝑉 = 𝑅

5R
Cp − =R
2
5R 7R
Cp = +R =
2 2
7R
Cp 7
Cálculo da relação entre a capacidade calorífica a pressão constante e a volume constante (γ): γ = = 2
5R = =
CV 5
2
1,40
Sabendo que a relação entre a pressão e o volume para um processo adiabático é dado por: p x V γ = constante
γ γ
PInicial x VInicial = Pfinal x Vfinal
1,40 1,40
PInicial x VInicial = Pfinal x Vfinal
(Pfinal x Vfinal − Pinicial x Vinicial )
Aplicando a equação do trabalho adiabático, temos: Wad = 1− γ

21,40
( 2 − 1) x (Pinicial x Vinicial )
Wad =
1 − 1,40
21,40 − 2
( 2 ) x (Pinicial x Vinicial )
Wad =
− 0,40
5
Wad = − x (20,40 − 1) x (Pinicial x Vinicial )
2

418
1600 FÍSICO-QUÍMICA APLICADA EXERCÍCIOS COMENTADOS - IME – ITA – OLIMPÍADA

Questão 835 – Considere que 1 mol de um gás ideal que expande de forma reversível e isotermicamente à 300 K
quando o volume triplica em relação a situação inicial. A partir desta informação, determine a variação de entropia do
universo (∆𝑆𝑢𝑛𝑖𝑣𝑟𝑠𝑜 ).
𝑉
Resolução: Cálculo da variação de entropia do sistema (∆𝑆𝑠𝑖𝑠𝑡𝑒𝑚𝑎 ): ∆𝑆𝑠𝑖𝑠𝑡𝑒𝑚𝑎 = 𝑛 𝑥 𝑅 𝑥 𝑇 𝑥 𝑙𝑛 (𝑉 𝑓𝑖𝑛𝑎𝑙 )
𝑖𝑛𝑖𝑐𝑖𝑎𝑙

𝑛 𝑥 𝑅 𝑥 𝑇 𝑥 𝑙𝑛(3)
∆𝑆𝑠𝑖𝑠𝑡𝑒𝑚𝑎 =
𝑇

∆𝑆𝑠𝑖𝑠𝑡𝑒𝑚𝑎 = 𝑛 𝑥 𝑅 𝑥 𝑇 𝑥 𝑙𝑛(3)
𝑄
Cálculo da entropia do meio externo (∆𝑆𝑒𝑥𝑡𝑒𝑟𝑛𝑜 ): ∆𝑆𝑠𝑖𝑠𝑡𝑒𝑚𝑎 = 𝑇

Como o processo é isotérmico, a variação da energia interna é igual a zero, logo: ΔU = Q + W

0=Q+W
Q=-W
𝑉𝑓𝑖𝑛𝑎𝑙
𝑄 = − 𝑛 𝑥 𝑅 𝑥 𝑇 𝑥 𝑙𝑛 ( )
𝑉𝑖𝑛𝑖𝑐𝑖𝑎𝑙
3𝑉
𝑄 = −1𝑚𝑜𝑙 𝑥 8,314 𝑥 𝑇 𝑥 𝑙𝑛 ( )
𝑉
𝑄 = −1𝑚𝑜𝑙 𝑥 8,314 𝑥 𝑇 𝑥 𝑙𝑛(3)
1 𝑚𝑜𝑙 𝑥 8,314 𝑥 𝑇 𝑥 𝑙𝑛(3)
∆𝑆𝑒𝑥𝑡𝑒𝑟𝑛𝑜 = −
𝑇

∆𝑆𝑒𝑥𝑡𝑒𝑟𝑛𝑜 = −1 𝑚𝑜𝑙 𝑥 8,314 𝑥 𝑙𝑛(3)

Cálculo da entropia do universo (∆𝑆𝑢𝑛𝑖𝑣𝑒𝑟𝑠𝑜 ): ∆𝑆𝑢𝑛𝑖𝑣𝑒𝑟𝑠𝑜 = ∆𝑆𝑠𝑖𝑠𝑡𝑒𝑚𝑎 + ∆𝑆𝑒𝑥𝑡𝑒𝑟𝑛𝑜

∆𝑆𝑢𝑛𝑖𝑣𝑒𝑟𝑠𝑜 = [1 𝑚𝑜𝑙 𝑥 8,314 𝑥 𝑙𝑛(3)] + [−1 𝑚𝑜𝑙 𝑥 8,314 𝑥 𝑙𝑛(3)]


∆𝑆𝑢𝑛𝑖𝑣𝑒𝑟𝑠𝑜 = 0
Questão 836 – (IME) Uma fábrica de cal (CaO) necessita reduzir o custo da produção para se manter no mercado
com preço competitivo para seu produto. A direção da fábrica solicitou ao departamento técnico o estudo da viabilidade
de reduzir a temperatura do forno de calcinação de carbonato de cálcio, dos atuais 1500 K para 800 K. Considerando
apenas o aspecto termodinâmico, pergunta-se: o departamento técnico pode aceitar a nova temperatura de
calcinação? Em caso afirmativo, o departamento técnico pode oferecer uma outra temperatura de operação que
proporcione maior economia? Em caso negativo, qual é a temperatura mais econômica para se operar o forno de
calcinação? Dados:

So (J/mol.K) Ho (kJ/mol)
CaCO3(s) 92,9 -1206,9
CaO(s) 39,8 -635,1
CO2(g) 213,6 -393,5
Observação: Desconsidere a variação das propriedades como a temperatura.
Resolução: Equação química: CaCO3(s) → CaO(s) + CO2(g)

419
1600 FÍSICO-QUÍMICA APLICADA EXERCÍCIOS COMENTADOS - IME – ITA – OLIMPÍADA

𝑜 0 0 0 0
Cálculo da variação da entalpia padrão (∆𝐻𝑟𝑒𝑎çã𝑜 ): ∆𝐻𝑟𝑒𝑎çã𝑜 = ∑ 𝐻𝑝𝑟𝑜𝑑𝑢𝑡𝑜𝑠 − ∑ 𝐻𝑟𝑒𝑎𝑔𝑒𝑛𝑡𝑒𝑠 = [1 𝑥 𝐻𝐶𝑎𝑂 +
0 0
1 𝑥 𝐻𝐶𝑂2
] − [1 𝑥 𝐻𝐶𝑎𝐶𝑂3
]
0
∆𝐻𝑟𝑒𝑎çã𝑜 = [1 𝑥 (−635,1) + 1 𝑥 (−393,5) − 1 𝑥 (−1206,9)]

0
𝑘𝐽
∆𝐻𝑟𝑒𝑎çã𝑜 = −635,1 − 393,5 + 1206,9 = +178,30
𝑚𝑜𝑙
𝑜 0 0 0 0
Cálculo da variação da entropia padrão (∆𝑆𝑟𝑒𝑎çã𝑜 ): ∆𝑆𝑟𝑒𝑎çã𝑜 = ∑ 𝑆𝑝𝑟𝑜𝑑𝑢𝑡𝑜𝑠 − ∑ 𝑆𝑟𝑒𝑎𝑔𝑒𝑛𝑡𝑒𝑠 = [1 𝑥 𝑆𝐶𝑎𝑂 +
0 0
1 𝑥 𝑆𝐶𝑂2 ] − [1 𝑥 𝑆𝐶𝑎𝐶𝑂3 ]
0
∆𝑆𝑟𝑒𝑎çã𝑜 = [1 𝑥 (+ 39,8) + 1 𝑥 (+ 213,6) − 1 𝑥 (+92,9)]

0
𝐽
∆𝑆𝑟𝑒𝑎çã𝑜 = 253,4 − 92,9 = + 160,50
𝑚𝑜𝑙 𝑥 𝐾
0 0 0
Equação termodinâmica de Gibbs: ∆𝐺𝑟𝑒𝑎çã𝑜 = ∆𝐻𝑟𝑒𝑎çã𝑜 − 𝑇 𝑥 ∆𝑆𝑟𝑒𝑎çã𝑜 . Substituindo os valores, temos:

0 0 0
∆𝐺𝑟𝑒𝑎çã𝑜 = ∆𝐻𝑟𝑒𝑎çã𝑜 − 𝑇 𝑥 ∆𝑆𝑟𝑒𝑎çã𝑜

∆𝑮𝟎𝒓𝒆𝒂çã𝒐 = 𝟏𝟕𝟖𝟑𝟎𝟎 − 𝑻 𝒙 𝟏𝟔𝟎, 𝟓𝟎

0
Para T = 1500 K, temos: ∆𝐺𝑟𝑒𝑎çã𝑜 = 178300 − 1500 𝑥 160,50

0
𝐽
∆𝐺𝑟𝑒𝑎çã𝑜 = −62450
𝑚𝑜𝑙

Para T = 1500 K, o processo é espontâneo.

Analisando o processo para uma temperatura igual a 800 K.


0
∆𝐺𝑟𝑒𝑎çã𝑜 = 178300 − 800 𝑥 160,50

0
∆𝐺𝑟𝑒𝑎çã𝑜 = 178300 − 128400

0
𝐽
∆𝐺𝑟𝑒𝑎çã𝑜 = +49900
𝑚𝑜𝑙

Para T = 800 K, o processo é não espontâneo.

Por isso, pelo aspecto termodinâmico, o departamento técnico não pode aceitar a nova temperatura de calcinação.

Cálculo da temperatura para que o processo seja mais econômico para a operação do forno de calcinação:
0
∆𝐺𝑟𝑒𝑎çã𝑜 = 178300 − 160,50 𝑥 𝑇

0
Condição de espontaneidade: ∆𝐺𝑟𝑒𝑎çã𝑜 ≤ 0.

0
∆𝐺𝑟𝑒𝑎çã𝑜 = 178300 − 160,50 𝑥 𝑇 ≤ 0

178300 − 160,50 𝑥 𝑇 ≤ 0

178300 ≤ 160,50 𝑥 𝑇

420
1600 FÍSICO-QUÍMICA APLICADA EXERCÍCIOS COMENTADOS - IME – ITA – OLIMPÍADA

178300
𝑇≥
160,50

𝑇 ≥ 1110,90 𝐾

Questão 837 – (IME) Calcule o valor da variação da energia livre, a 25°C, para a reação a seguir: 2 Na2O2(s) + 2 H2O(l)
→ 4 NaOH(s) + O2(g). Dados:
Entalpia de formação a 25°C S° a 25°C
Substância
(kJ.mol-1) (J.mol-1.K-1)
H2O(l) -286,0 69,69
Na2O2(s) -510,9 94,60
NaOH(s) -426,8 64,18
O2(g) 0 205,00

Resolução: Equação química: 2 Na2O2(s) + 2 H2O(l) → 4 NaOH(s) + O2(g)

o 0 0 0
Cálculo da variação da entalpia padrão (∆Hreação ): ∆Hreação = ∑ Hprodutos − ∑ Hreagentes
0
0
∆Hreação = [4 x HNaOH + 1 x HO0 2 ] − [2 x HNa
0
2 O2
0
+ 2 x HH 2O
]

0
∆Hreação = [4 x (−426,8) + 1 x 0] − [2 x (−510,9) + 2 x (−286,0)]

0
∆Hreação = −1707,2 − (−1021,8 − 572,0)

0
∆Hreação = −1707,2 − (−1593,8)

0 kJ
∆Hreação = −1707,2 + 1593,8 = −113,40 mol (Processo Exotérmico)

o 0 0 0
Cálculo da variação da entropia padrão (∆Sreação ): ∆Sreação = ∑ Sprodutos − ∑ Sreagentes
0
0
∆Sreação = [4 x SNaOH + 1 x SO0 2 ] − [2 x SNa
0
2 O2
0
+ 2 x SH 2O
]

0
∆Sreação = [4 x (64,18) + 1 x (205)] − [2 x (94.60) + 2 x (69,69)]

0
∆Sreação = [256,72 + 205] − [189,20 + 139,38]

0
J
∆Sreação = 461,72 − 328,58 = +133,14
mol x K
0 0 0
Equação termodinâmica de Gibbs: ∆Greação = ∆Hreação − T x ∆Sreação . Substituindo os valores, temos:
0 0 0
∆Greação = ∆Hreação − T x ∆Sreação

𝟎
∆𝐆𝐫𝐞𝐚çã𝐨 = −𝟏𝟏𝟑𝟒𝟎𝟎 − 𝐓 𝐱 𝟏𝟑𝟑, 𝟏𝟒

Para T = 25°C, temos:


0
∆Greação = −113400 − (25 + 273) x 133,14

0 J kJ
∆Greação = −113400 − 298 x 133,14 = −153075,7 (−153,07 ) (Processo espontâneo)
mol mol

Questão 838 – (IME) Calcule a que temperatura a reação abaixo é espontânea: CO(g) + ½ O2(g) → CO2(g).

421
1600 FÍSICO-QUÍMICA APLICADA EXERCÍCIOS COMENTADOS - IME – ITA – OLIMPÍADA

Entalpia de formação a 25°C S° a 25°C


Substância
(kJ.mol-1) (J.mol-1.K-1)
CO(g) -110,5 198
CO2(s) -393,5 214
O2(g) 0 205

Resolução:

Equação química: CO(g) + ½ O2(g) → CO2(g)


𝑜 0 0 0
Cálculo da variação da entalpia padrão (∆𝐻𝑟𝑒𝑎çã𝑜 ): ∆𝐻𝑟𝑒𝑎çã𝑜 = ∑ 𝐻𝑝𝑟𝑜𝑑𝑢𝑡𝑜𝑠 − ∑ 𝐻𝑟𝑒𝑎𝑔𝑒𝑛𝑡𝑒𝑠

0 0 0
1
∆𝐻𝑟𝑒𝑎çã𝑜 = [1 𝑥 𝐻𝐶𝑂 − (1 𝑥 𝐻𝐶𝑂 + 𝑥 𝐻𝑂02 )]
2 2
0 1 𝑘𝐽
∆𝐻𝑟𝑒𝑎çã𝑜 = [1 𝑥 (−393,5)] − [1 𝑥 (−110,5) + 2 𝑥 (0)] = − 283,0 𝑚𝑜𝑙 (Processo Exotérmico)

𝑜 0 0 0
Cálculo da variação da entropia padrão (∆𝑆𝑟𝑒𝑎çã𝑜 ): ∆𝑆𝑟𝑒𝑎çã𝑜 = ∑ 𝑆𝑝𝑟𝑜𝑑𝑢𝑡𝑜𝑠 − ∑ 𝑆𝑟𝑒𝑎𝑔𝑒𝑛𝑡𝑒𝑠

0 0 0
1
∆𝑆𝑟𝑒𝑎çã𝑜 = [1 𝑥 𝑆𝐶𝑂 − (1 𝑥 𝑆𝐶𝑂 + 𝑥 𝑆𝑂02 )]
2
2

0
1
∆𝑆𝑟𝑒𝑎çã𝑜 = [1 𝑥 214 − 1 𝑥 198 − 𝑥 205]
2

0
𝐽
∆𝑆𝑟𝑒𝑎çã𝑜 = [214 − 198 − 102,5] = −171,5
𝑚𝑜𝑙 𝑥 𝐾
0 0 0
Equação termodinâmica de Gibbs: ∆𝐺𝑟𝑒𝑎çã𝑜 = ∆𝐻𝑟𝑒𝑎çã𝑜 − 𝑇 𝑥 ∆𝑆𝑟𝑒𝑎çã𝑜 .

Substituindo os valores, temos: ∆𝑮𝟎𝒓𝒆𝒂çã𝒐 = −𝟐𝟖𝟑𝟎𝟎𝟎 + 𝑻 𝒙 𝟏𝟕𝟏, 𝟓

0
Condição para que o processo seja espontâneo: ∆𝐺𝑟𝑒𝑎çã𝑜 ≤ 0.

0
∆Greação = −283000 + T x 171,5 ≤ 0

T x 171,5 ≤ 283000

283000
T≤ ≤ 1650,14 K
171,5

Questão 839 – (IME) Considere a proposta de um processo para a obtenção da cementita, esquematizada abaixo.

422
1600 FÍSICO-QUÍMICA APLICADA EXERCÍCIOS COMENTADOS - IME – ITA – OLIMPÍADA

Sabe-se que a energia livre de Gibbs molar está relacionada diretamente com a constante de equilíbrio de uma reação
química, conforme a seguinte equação termodinâmica: ∆Greação = - R x T x lnKp. Determine as frações molares na fase
gasosa, na situação de equilíbrio, e avalie se o processo é viável.

Fe(s) Fe3C(s) CH4(g) H2(g)


ΔHof (kJ.mol-1) 0 25,10 - 74,80 0
ΔSof (J.mol-1.K-1) 27,30 104,6 186,2 130,6

Resolução: Equação química: 3 Fe(s) + CH4(g) → Fe3C(s) + 2 H2(g)


o 0 0 0
Cálculo da variação da entalpia padrão (∆Hreação ): ∆Hreação = ∑ Hprodutos − ∑ Hreagentes
0 0 0 0 0
∆Hreação = [1 x HFe 3C
+ 2 x HH 2
− 3 x HFe − 1 x HCH 4
] = [1 x 25,10 + 2 x 0 − 3 x 0 − 1 x (−74,80)]

0 kJ
∆Hreação = [25,10 + 74,80] = + 99,90 mol (Processo Endotérmico)

o 0 0 0
Cálculo da variação de entropia padrão (∆Sreação ): ∆Sreação = ∑ Sprodutos − ∑ Sreagentes
0 0 0 0 0
∆Sreação = [1 x SFe 3C
+ 2 x SH 2
− 3 x SFe − 1 x SCH 4
]
= [1 x 104,60 + 2 x (130,6) − 3 x 27,30 − 1 x 186,2]

0
J
∆Sreação = [365,80 − 268,1] = +97,7
mol x K
0 0 0
Cálculo da variação da energia livre de Gibbs: ∆Greação = ∆Hreação − T x ∆Sreação

0
∆Greação = 99900 − T x 97,7

0
Para T = 25°C: ∆Greação = 99900 − (25 + 273) x 97,7

0 J
∆Greação = +70785,4 mol (Processõ não espontâneo)

0 J
Cálculo da constante de equilíbrio em fução das pressões parcias (Kp): ∆Greação = +70785,4
mol
J
− R x T x lnK p = +70785,4
mol
J
− 8,314 x (25 + 273) x lnK p = +70785,4
mol
− 70785,4
lnK p = = −28,57
2477,572

K p = e−28,57 = 3,91 x 10−13

Para uma constante de equilíbrio menor que 1, significa que a o equilíbrio está deslocado para a esquerda, ou seja,
o processo não é viável para a produção de cementita.
0
Critério de espontaneidade: ∆Greação ≤0

0
∆Greação = 99900 − T x 97,7 ≤ 0

97,7 x T ≥ 99900

423
1600 FÍSICO-QUÍMICA APLICADA EXERCÍCIOS COMENTADOS - IME – ITA – OLIMPÍADA

99900
T≥ ≥ 1022,52 K
97,7

Cálculo das frações molares (X) das espécies gasosas presentes na reação química:

Base de cálculo: 1 atm de gás metano.

3 Fe(s) + CH4(g) ⇌ Fe3C(s) + 2 H2(g)


Início - 1 atm - 0
Reage - α - 2α
Equilíbrio - 1-α - 2α
P2H2
Cálculo do grau de dissociação (α): K p =
PCH4

(2α)²
Kp =
1−α

4α²
Kp = = 3,91 x 10−13
1−α

Sabendo que o valor do x calculado será bem baixo, logo (1 – x) → 1.

4α² = 3,91 x 10−13

3,91 x 10−13
α=√
4

α = √9,77 x 10−14

α = 3,13 x 10−7

Cálculo das frações molares a partir das pressões parcias, levando em consideração que a pressão total seja igual a
1 atm. Em relação ao gás hidrogênio: PH2 = X H2 x PTOTAL

PH2
X H2 =
PTOTAL

PH2
X H2 =
PTOTAL

2. α
X H2 =
1

X H2 = 2 x (3,13 x 10−7 ) = 6,26 x 10−7

Em relação ao gás metano: X CH4 + X H2 = 1

X CH4 = 1 − X H2

X CH4 = 1 − 6,26 x 10−7 ≅ 1

424
1600 FÍSICO-QUÍMICA APLICADA EXERCÍCIOS COMENTADOS - IME – ITA – OLIMPÍADA

Questão 840 – (IME) O oxigênio e o hidrogênio combinam-se, em células de combustível, produzindo água líquida e
gerando corrente elétrica. O máximo trabalho elétrico útil que essas células produzem é dado por ΔG° = - 237 x 10³
J.mol-1. Com base nos dados fornecidos, calcule o ponto de ebulição da água. Aproxime ΔH por ΔH° e ΔS por ΔS°.
Dados termodinâmicos:

O2(g) H2(g) H2O(l) H2O(g)


S° = 206 J.mol-1.K-1 S° = 131 J.mol-1.K-1 S° = 70,0 J.mol-1.K-1 S° = 189,0 J.mol-1.K-1
H° = -242x10³ J.mol-1
Resolução: Tabela fornecida pelo problema:

O2(g) H2(g) H2O(l) H2O(g)


S° = 206 J.mol-1.K-1 S° = 131 J.mol-1.K-1 S° = 70,0 J.mol-1.K-1 S° = 189,0 J.mol-1.K-1
H° = -242x10³ J.mol-1
Equação química: H2(g) + ½ O2(g) → H2O(l)
o 0 0 0
Cálculo da variação de entropia padrão (∆Sreação ): ∆Sreação = ∑ Sprodutos − ∑ Sreagentes

0 1
0
∆Sreação = [1 x SH 2O
0
− (1 x SH + x SO0 2 )]
2 2

0
1 J
∆Sreação = [1 x 70 − 1 x 131 − x 206] = [70 − 131 − 103] = −164
2 mol x K
o 0 0 0
Cálculo da entalpia da água no estado líquido (HH 2 O(l)
), a 25°C: ∆Greação = ∆Hreação − T x ∆Sreação

o 1
0
∆Greação = {HH 2 O(l)
0
− (1 x HH + x HO0 2 )} − T x ∆Sreação
0
2 2

o 1
− 237000 = {HH 2 O(l)
− (1 x 0 + x 0)} − (25 + 273) x (−164)
2
o
− 237000 = HH 2 O(l)
+ 298 x 164

o J kJ
HH 2 O(l)
= − 237000 − 48872 = −285872 (−285,87 )
mol mol

Cálculo da variação de entropia padrão para o processo de vaporização (ΔS°), a partir da equação química
balanceada: H2O(l) → H2O(g):

0 0 0 0 0
∆Svaporização = ∑ Sprodutos − ∑ Sreagentes = [SH 2 O(g)
− SH 2 O(l)
] = [1 x 189 − 1 x 70]
J
= +119
mol x K

Cálculo da variação de entalpia padrão para o processo de vaporização (ΔH°), a partir da equação química
balanceada: H2O(l) → H2O(g):

0 0 0 0 0
∆Hvaporização = ∑ Hprodutos − ∑ Hreagentes = [HH 2 O(g)
− HH 2 O(l)
] = −242000 − (−285872)
J
= + 43872
mol

Como o processo ocorre no equilíbrio líquido → gasoso, a variação da energia livre de Gibbs é igual a zero, logo:
∆G = ∆H − Tebulição x ∆S

425
1600 FÍSICO-QUÍMICA APLICADA EXERCÍCIOS COMENTADOS - IME – ITA – OLIMPÍADA

0 = 43872 − Tebulição x 119

43872
Tebulição = = 368,67 K
119

Questão 841 – A partir da reação de combustão do carbono, calcule o trabalho realizado a uma temperatura de
aproximadamente igual a 27°C.

Resolução: Equação química da combustão completa do carbono: C(s) + O2(g) → CO2(g)

Relação termodinâmica entre a entalpia e a energia interna: dH = dU + pV (diferencial infinitesimal)

Considerando que os participantes gasosos apresentam comportamento ideal, temos:

∆H = ∆U + ∆n x R x T

∆H − ∆U = ∆n x R x T

𝑊

W = (1 − 1) x R x T

W=0

Questão 842 – (IME) Um mol de um gás ideal sofre uma expansão adiabática reversível de um estado inicial cuja
𝐶𝑝
pressão é Pi e o volume é Vi para um estado final em que a pressão é Pf e o volume é Vf. Sabe-se que 𝛾 = 𝐶 é o
𝑉
expoente de Poisson, em que Cp e CV são os respectivos calores molares a pressão e a volume constantes. Obtenha
a expressão do trabalho realizado pelo gás em função de Pi, Vi, Pf, Vf e γ.

Resolução: Segundo o problema, o gás sofre um processo de expansão adiabática, logo, a quantidade de calor é
igual a zero (Q = 0).

Pela primeira lei da termodinâmica, temos: ΔU = Q + W

ΔU = 0 + W

ΔU = W

Sabendo que a variação da energia interna ocorre a volume constante, sua expressão é dada pela seguinte relação:
∆𝑈 = 𝑛 𝑥 𝐶𝑉 𝑥 ∆𝑇.

Para n = 1 mol, ∆𝑈 = 𝐶𝑉 𝑥 ∆𝑇 e sabendo que a relação entre as capacidades caloríficas é dada pela seguinte
equação termodinâmica: Cp − CV = R.

W = ∆U

W = CV x ∆T

W = CV x (Tf − Ti )

Pf x Vf Pi x Vi
W = CV x [( )−( )]
R R

426
1600 FÍSICO-QUÍMICA APLICADA EXERCÍCIOS COMENTADOS - IME – ITA – OLIMPÍADA

CV
W= x [Pf x Vf − Pi x Vi ]
R
CV
W= x [Pf x Vf − Pi x Vi ]
CP − CV

1
W= x [Pf x Vf − Pi x Vi ]
CP
CV − 1

1 1
W= x [Pf x Vf − Pi x Vi ] = x [Pf x Vf − Pi x Vi ]
CP γ − 1
−1
C
⏟V
γ

Questão 843 – (SELETIVA PARA A 37TH INTERNATIONAL CHEMISTRY OLIMPIAD) Uma mistura de 11,02 mmol
de H2S e 5,48 mmol de CH4 foi colocada em um reator com um catalisador de platina e, o equilíbrio H2S(g) + CH4(g) ⇄
H2(g) + CS2(g), foi estabelecido em 700°C e 762 torr. Retirou-se o catalisador da mistura reacional e em seguida a
mistura foi resfriada. Através de análises no equilíbrio encontrou-se 0,711 mmol de CS2. Determine:
a) As frações molares de cada substância envolvida;
b) As porcentagens molares;
c) As pressões parciais;
d) Kp e o ΔG° para a reação em 700°C.

Resolução:

Equação química: 2 H2S(g) + CH4(g) ⇄ 4 H2(g) + CS2(g)

Base de cálculo: 11,02 mmol de H2S(g) e 5,48 mmol de CH4(g).

Tabela de equilíbrio químico:

2 H2S(g) (mmol) CH4(g) (mmol) ⇄ 4 H2(g) (mmol) CS2(g) (mmol)


Início 11,02 5,48 0 0
Reage 2α α 4α α
Equilíbrio 11,02 - 2α 5,48 - α 4α α = 0,7

Cálculo do número de mol para cada espécie gasosa no equilíbrio:

nH2 S = 11,02 − 2α = 11,02 − 2 x (0,711) = 11,02 − 1,422 = 9,60 mmol

nCH4 = 5,48 − α = 5,48 − 0,711 = 4,77 mmol

nH2 = 4. α = 4 x (0,711) = 2,84 mmol

nCS2 = 0 + α = 0,711 mmol

Cálculo do número de mol total (nTOTAL): nTOTAL = nH2 S + nCH4 + nH2 + nCS2

nTOTAL = 9,60 mmol + 4,77 mmol + 2,84 mmol + 0,711 mmol = 17,92 mmol

Item a) Frações molares para cada espécie gasosa:

427
1600 FÍSICO-QUÍMICA APLICADA EXERCÍCIOS COMENTADOS - IME – ITA – OLIMPÍADA

n H2 S 9,60 mmol
X H2 S = = = 0,536
nTOTAL 17,92 mmol

nCH4 4,77 mmol


X CH4 = = = 0,266
nTOTAL 17,92 mmol

nH2 2,84 mmol


X H2 = = = 0,158
nTOTAL 17,92 mmol

nCS2 0,711 mmol


X CS2 = = = 3,97 x 10−2
nTOTAL 17,92 mmol

Item b) Cálculo do percentual (%) para cada espécie gasosa:

n H2 S 9,60 mmol
X H2 S = = = 0,536 (53,6%)
nTOTAL 17,92 mmol

nCH4 4,77 mmol


X CH4 = = = 0,266 (26,6%)
nTOTAL 17,92 mmol

nH2 2,84 mmol


X H2 = = = 0,158 (15,8%)
nTOTAL 17,92 mmol
nCS2 0,711 mmol
X CS2 = n = 17,92 mmol = 3,97 x 10−2 (3,97 %)
TOTAL

Item c e d) Cálculo das pressões parciais:

PH2 S = X H2 S x PTOTAL = 0,536 x 762 torr = 408,43 torr

PCH4 = X CH4 x PTOTAL = 0,266 x 762 torr = 202,69 torr

PH2 = X H2 x PTOTAL = 0,158 x 762 torr = 120,40 torr

PCS2 = X CS2 x PTOTAL = 3,97 x 10−2 x 762 torr = 30,25 torr

P4H2 x PCS2
Equação da constante de equilíbrio em função das pressões parciais: K p =
P2H S x PCH4
2

P4H x PCS2 (120,40)4 x 30,25


Cálculo da constante de equilíbrio em função das pressões parciais: K p = P2 2 = (408,43)2 x 202,69 = 188,0
H S x PCH4
2

Cálculo da variação da energia livre de Gibbs (ΔG°): ∆G° = −R x T x lnK p

J J kJ
∆G° = −8,314 x (700 + 273)x ln(188) = −42389,1 (−42,39 ) (Processso
mol x K mol mol
espontâneo)

428
1600 FÍSICO-QUÍMICA APLICADA EXERCÍCIOS COMENTADOS - IME – ITA – OLIMPÍADA

Questão 844 – (IFRJ) Num recipiente de 20 litros, inseriu-se 1,0 mol de bicarbonato de sódio. Após algum tempo,
estabeleceu-se o equilíbrio apresentado a seguir. Ao longo de todo o processo, a temperatura permaneceu constante,
25°C. Equação química balanceada: 2 NaHCO3(s) ⇄ Na2CO3(s) + H2O(g) + CO2(g).

ΔHo298 So298
Substância
(kJ.mol-1) (J.mol-1.K-1)
NaHCO3(s) - 947 102
Na2CO3(s) - 1133 136
H2O(g) - 242 189
CO2(g) - 393 213

Admitindo que todos os gases envolvidos apresentam comportamento de gás ideal, determine a variação da energia
livre de Gibbs, a variação da energia interna do sistema reacional e também a constante de equilíbrio para o sistema
na temperatura em questão.

Resolução:

Equação química: 2 NaHCO3(s) ⇄ Na2CO3(s) + H2O(g) + CO2(g)


𝑜 0 0 0
Cálculo da variação da entalpia padrão (∆𝐻𝑟𝑒𝑎çã𝑜 ): ∆Hreação = ∑ Hprodutos − ∑ Hreagentes

0 0 0 0 0
∆Hreação = [1 x HNa 2 CO3
+ 1 x HH 2O
+ 1 x HCO 2
− 2 x HNaHCO 3
]

0
∆Hreação = [1 x (−1133) + 1 x (−242) + 1 x (−393) − 2 x (−947)]

0 kJ
∆Hreação = −1768 + 1894 = +126 mol (Processo Endotérmico)

𝑜 0 0 0
Cálculo da variação de entropia padrão (∆𝑆𝑟𝑒𝑎çã𝑜 ): ∆Sreação = ∑ Sprodutos − ∑ Sreagentes

0 0 0 0 0
∆Sreação = [1 x SNa 2 CO3
+ 1 x SH 2O
+ 1 x SCO 2
− 2 x SNaHCO 3
]

0
∆Sreação = [1 x 136 + 1 x 189 + 1 x 213 − 2 x 102]

0
J
∆Sreação = [538 − 204] = +334
mol x K
o 0 0 0
Cálculo da variação da energia livre de Gibbs (∆Greação ): ∆Greação = ∆Hreação − T x ∆Sreação

0
∆Greação = 12600 − T x 334

0
∆Greação = 12600 − (25 + 273) x 334

0 J
∆Greação = 12600 − 99532 = + 26468 (Processo não espontâneo)
mol

0 J
Cálculo da constante de equilíbrio (K): ∆Greação = + 26468 mol = −R x T x ln K

J J
+ 26468 = −8,314 x 298 K x ln K
mol mol x K

429
1600 FÍSICO-QUÍMICA APLICADA EXERCÍCIOS COMENTADOS - IME – ITA – OLIMPÍADA

− 26468 = 2477,572 x ln K

− 26468
ln K =
2477,572

ln K = −10,68

K = e(−10,68)

Cálculo da variação da energia interna do sistema (∆𝑈): ∆H = ∆U + pV

∆H = ∆U + ∆n x R x T

∆H = ∆U + (𝑛𝑝𝑟𝑜𝑑𝑢𝑡𝑜𝑠 − 𝑛𝑟𝑒𝑎𝑔𝑒𝑛𝑡𝑒𝑠 ) x R x T

𝐽
126000 = ∆U + (2 − 0) x 8,314 x 298 K
𝑚𝑜𝑙 𝑥 𝐾

126000 − 4955,14 = ∆U

∆U = 121044,86 J (+121,04 kJ)

Questão 845 – Considere que três mols de um gás ideal apresentando capacidade calorífica a pressão constante
igual a (5/2R) esteja sujeita à transformação irreversível segundo a seguinte sequência:
600K; 10 atm → 300 K; 5 atm
Diante deste informação, calcule as seguintes propriedades termodinâmicas:
a) a variação da entalpia;
b) variação da energia interna;
c) a quantidade de calor;
d) o trabalho realizado.

Resolução: Inicialmente iremos calcular o volume inicial e o volume final para cada situação.
Cálculo do volume para o estado inicial, considerando comportamento ideal: p x Vinicial = n x R x T
10 x Vinicial = 3,0 x 0,08206 x 600
Vinicial = 14,77 L

Cálculo do volume para o estado final, considerando comportamento ideal: p x Vfinal = n x R x T


5 x Vfinal = 3,0 x 0,08206 x 300
Vfinal = 14,77 L

Diante dos valores calculados dos volumes, inicial e final, observa-se que o processo é isocórico (volume constante).
Logo, como o processo é isocórico, o trabalho realizado pelo sistema é igual a zero (W = 0).

Cálculo da capacidade calorífca a volume constante: Cp − CV = R

5R
− CV = R
2
5R 3R
CV = −R=
2 2
430
1600 FÍSICO-QUÍMICA APLICADA EXERCÍCIOS COMENTADOS - IME – ITA – OLIMPÍADA

Cálculo da energia interna (ΔU): ∆U = n x CV x ∆T


3
∆U = 3 mol x x 8,314 x (300 − 600)
2
3
∆U = −3 mol x x 8,314 x 300 = −11223,90 J (−11,22 kJ)
2
Cálculo da entalpia (ΔH): ∆H = n x CP x ∆T
5
∆H = 3 mol x x 8,314 x (300 − 600)
2
5
∆H = −3 mol x x 8,314 x 300 = −18706,50 J (−18,71 kJ)
2
Cálculo da quantidade de calor (Q), a partir da primeira lei da termodinâmica: ∆U = Q + W

∆U = Q + 0 = Q = −11223,90 J (−11,22 kJ)

Questão 846 – Um mol de um gás a uma temperatura igual a 27°C é comprimido isotermicamente e reversivelmente
a partir de um volume inicial de 10 litros para um volume final de 0,20 L. Calcule o trabalho realizado no sistema a
partir da equação do gás real (Vm – b)P = RT, sabendo que a constante de Van der Waals de b é igual a 0,03 L/mol.

Resolução: Sabendo que o trabalho é definido como: 𝒅𝑾 = − 𝒑𝒆𝒙𝒕𝒆𝒓𝒏𝒂 𝒙 𝒅𝑽

RxT
A partir da equação dos gases mencionado pelo problema, temos: P =
Vmolar −b

RxT
Substituindo na equação acima, temos: dW = − (V ) x dV
molar −b

V −b
Desenvolvendo a equação, temos: W = −R x T x ln (V2 −b)
1

V2 −b
Substituindo os valores proposto pelo problema: W = −R x T x ln ( )
V1 −b

0,20−0,03
W = − 8,314 x (27 + 273) x ln ( )
10−0,03

W = − 8,314 x 300 x ln(1,70 x 10−2 ) = +10155 J


Questão 847 – Determine o valor da variação da energia interna e da variação de entalpia, nas condições normais
de temperatura e pressão, quando 100 L de gás hélio são aquecidos de 0°C até 100°C em um recipiente fechado,
considerando que a capacidade calorífica a volume contente é igual a CV = 3R/2.
V
Resolução: Primeiramente será necessário calcular o valor do número de mol de gás hélio: nHe = V =
molar
100 L
L = 4,46 mol
22,4
mol

Cálculo da variação de energia interna (U): ∆U = n x CV x ∆T

3 3 J kJ
∆U = n x x R x ∆T = 4,46 mol x ( x 8,314) x (100 + 273) = 20746,51 (20,75 )
2 2 mol mol

Cálculo da capacidade calorífica a pressão constante (Cp): Cp − CV = R

431
1600 FÍSICO-QUÍMICA APLICADA EXERCÍCIOS COMENTADOS - IME – ITA – OLIMPÍADA

3R
Cp − =R
2
3R 5R
Cp = +R =
2 2

Cálculo da quantidade de calor a pressão constante (ΔH): ∆H = n x Cp x ∆T

5 5 J kJ
∆H = n x x R x ∆T = 4,46 mol x ( x 8,314) x (100 + 273) = 34577,51 (34,58 )
2 2 mol mol

Questão 848 – (ITA) Um frasco aberto contém ar a 7°C. Calcular a que temperatura deve ser aquecido o frasco a fim
de que escape ¼ do ar nele contido.

Resolução:

Estado (1): P1 = P; V1 = V; T1 = 7 + 273 = 280 K; n1 = n

Estado (2): P2 = P; V2 = V; T2 = 7 + 273 = 280 K; n2 = (n – ¼.n) = n.(1 – ¼) = 3n/4

Equação (1): p1 x V1 = n1 x R x T1

p x V = n x R x (7 + 273)

p x V = n x R x 280 (Equação A)

Equação (2): p2 x V2 = n2 x R x T2

1
p x V = (1 − ) x n x R x T2
4
3
pxV= x n x R x T2 (Equação B)
4

pxV n x R x 280
Dividindo a equação (A) pela equação (B), temos: p x V = 3
x n x R x T2
4
280
1=
3
x T2
4
3
x T2 = 280
4
280 x 4
T2 = = 373,33 K
3

Questão 849 – (IME) Um corpo recebe 40 Joules de calor de um outro corpo e rejeita 10 Joules para um ambiente.
Simultaneamente, o corpo realiza um trabalho de 200 Joules. Estabeleça baseado na primeira lei da termodinâmica,
o que acontece com a temperatura do corpo em estudo.

Resolução: Baseado na descrição colocada pelo problema, o sistema recebe 40 J de calor, o que caracteriza um
processo endotérmico (+ 40 J) e libera para a vizinhança 10 J, caracterizando um processo exotérmico (- 10 J).

Quantidade de calor nesta primeira etapa será igual a: Q = + 40 J + (-10 J) = + 30 J

432
1600 FÍSICO-QUÍMICA APLICADA EXERCÍCIOS COMENTADOS - IME – ITA – OLIMPÍADA

O sistema realiza trabalho para a vizinhança no valor de 200 J, ou seja, - 200J.

Através da primeira lei da termodinâmica, como a energia interna é a soma da quantidade de calor pelo trabalho, ∆U
= Q + W, a energia interna diminui, com isso, a temperatura do sistema também irá diminuir.

Questão 850 – Considere um mol de ar atmosférico a uma temperatura de aproximadamente 423 K e pressão de
7,90 atm sofre um processo reversível. Esta quantidade de ar expande de forma isotérmica até que a pressão fique a
três atmosferas, quando a temperatura seja 323 K, com todo este processo ocorrendo a volume constante.
Considerando que a capacidade calorífica a volume constante seja igual a (5R/2) e a capacidade calorífica a pressão
constante igual a (7R /2), calcule Q, W, ΔU e ΔH.

Resolução: O processo descrito no problema ocorre primeiramente com uma expansão isotérmica seguida de um
processo a volume constante.

Processo (I): PI = 7,90 atm; VI = V; TI = 423 K e n = 1 mol de ar atmosférico

Processo (II): PII = ?; VII = ?; TII = 423 K

Processo (III): PIII = 3 atm; VIII = ? e TIII = 323 K

Entre o processo a/b o processo se caracteriza como isotérmico, logo a variação da energia interna e a variação de
entalpia é igual a zero, ∆H = ∆U = 0.

Cálculo do volume para o processo I, considerando que o ar atmosférico apresenta comportamento ideal:

1 x 0,08206 x 423
VI = = 4,39 L
7,90

PII x VII PIII x VIII


Cálculo da pressão para o processo II, a partir da equação combinada dos gases ideais: TII
= TIII
PII 3
=
423 323

PII = 3,93 atm


1 x 0,08206 x 423
Cálculo do volume para o processo II: VII = = 8,83 L
3,93

Como o processo é isocórico entre o processo II e o processo III, logo Vb = Vc = 8,83 litros.

V
Cálculo do trabalho realizado entre o processo I/II: WI/II = − n x R x T x ln (V final )
inicial
J 8,83
WI/II = − 1 mol x 8,314 x 423 K x ln ( ) = −2437,67 J
mol x K 4,39
A partir da primeira lei da termodinâmica, temos: ∆UI/II = Q I/II + WI/II

Como o processo entre o processo a e o b é isotérmico, a energia interna é igual a zero.

∆UI/II = Q I/II + WI/II

0 = Q I/II + WI/II

Q I/II = − WI = −(−2437,67 J) = +2437,67 J


II

433
1600 FÍSICO-QUÍMICA APLICADA EXERCÍCIOS COMENTADOS - IME – ITA – OLIMPÍADA

Entre o processo II/III o processo é isocórico, logo o trabalho (WII/III ) é igual a zero.

Cálculo da variação da energia interna para o processo II/III: ∆U = n x CV x ∆T

5 5
∆U = n x x R x ∆T = 1,0 mol x ( x 8,314) x (323 − 423) = −2078,5 J
2 2

Cálculo da quantidade de calor para o processo II/III: ∆U = Q + W


∆U = Q + 0
∆U = Q = −2078,5 J
7
Cálculo da entalpia para o processo II/III: ∆H = n x CP x ∆T = n x 2 x R x ∆T =
7
1,0 mol x ( x 8,314) x (323 − 423) = −2909,9 J
2

Questão 851 – Para qual temperatura a velocidade média quadrática das moléculas de nitrogênio seja igual a
velocidade média quadrática das moléculas de hidrogênio a 20,0°C?

Resolução: Sabendo que a velocidade média quadrática (c²) é definida pela seguinte equação matemática: c² =
3xRxT
.
<MM>

2 2
cN 2
= cH 2

3 x R x TN2 3 x R x TH2
=
< MM >N2 < MM >H2

TN2 TH2
=
< MM >N2 < MM >H2

TN2 (20 + 273)


=
28,0 2,0

TN2 = 293 x 14 = 4102 K

Questão 852 – (GRILLO) Calcular a energia liberada na forma de quantidade de calor, para a combustão completa
de gás acetileno, do qual se encontra em um reator industrial, do tipo CSTR, de volume igual a 100 litros a 147°C e
pressão de 25 atm. Informações para a resolução do problema:

Substâncias ΔHfo (kJ/mol)


C2H2(g) - 227
O2(g) 0
H2O(g) - 242
CO2(g) - 394

Resolução: Equação química: C2H2(g) + 3/2 O2(g) → 2 CO2(g) + H2O(g)


0
Cálculo da variação de entalpia da reação de combustão do gás acetileno (∆Hreação ):

0 0 0
0
∆Hreação = ∑ Hprodutos 0
− ∑ Hreagentes = [2 x HCO 2
+ 1 x HH 2O
] − [1 x HC02 H2 + 1 x HO0 2 ]

434
1600 FÍSICO-QUÍMICA APLICADA EXERCÍCIOS COMENTADOS - IME – ITA – OLIMPÍADA

0
∆Hreação = [2 x (−394) + 1 x (−242)] − [1 x (−227) + 1 x 0] = [−788 − 242] − [−227 + 0] =
kJ
−1030 + 227 = − 803 mol (Processo exotérmico)

Cálculo do número de mol do gás acetileno, a partir da equação dos gases ideais: 25 x 100 =
nC2 H2 x 0,08206 x (147 + 273)

2500
nC2 H2 = = 72,54 mol
34,46

0 kJ
Cálculo da quantidade de calor a partir de 72,54 mol de gás acetileno: ∆Hreação = −803 mol x 72,54 mol =
− 58249,62 kJ

Questão 853 – (GRILLO) O óxido de cobre é reduzido a cobre metálico por hidrogênio a uma temperatura de
aproximadamente igual a 1200°C.
a) Apresente a reação química de redução do óxido de cobre na presença de gás hidrogênio.
b) Calcule a variação da entalpia padrão e diga se o processo é endotérmico ou exotérmico.
c) Calcule a energia interna deste processo. Informações para a resolução do problema a 25°C:

ΔH°fomação
Substância
(kJ.mol-1)
CuO(s) -157
H2(g) 0
Cu(s) 0
H2O(g) -242

Resolução: Item a) Equação química: CuO(s) + H2(g) → Cu(s) + H2O(g)


0
Item b) Cálculo da variação de entalpia da reação (∆Hreação ), respeitando os valores tabelados a seguir:

0 0 0 0 0 0 0
∆Hreação = ∑ Hprodutos − ∑ Hreagentes = [1 x HCu + 1 x HH 2O
] − [1 x HCuO + 1 x HH 2
]

0 kJ
∆Hreação = [1 x 0 + 1 x (−242)] − [1 x (−157) + 1 x 0] = −242 + 157 = −85 (processo
mol
exotérmico)

Item c) Cálculo da variação da energia interna (ΔU): ∆H = ∆U + (nprodutos − nreagentes ) x R x T

∆H = ∆U + (1 − 1) x R x T

kJ
∆H = ∆U = − 85
mol

Questão 854 – (ITA) Um cilindro provido de um pistão móvel, que se desloca sem atrito, contém 3,2 g de gás hélio
que ocupa um volume, de 19,0 L sob pressão 1,2 x 105 N.m-2. Mantendo a pressão constante, a temperatura do gás
é diminuída de 15 K e o volume ocupado pelo gás diminui para 18,2 L. Sabendo que a capacidade calorífica molar do
gás hélio à pressão constante é igual a 20,8 J.K-1.mol-1, a variação da energia interna neste sistema é
aproximadamente igual a:
a) – 0,35 kJ
b) – 0,25 kJ
c) – 0,20 kJ
d) – 0,15 kJ

435
1600 FÍSICO-QUÍMICA APLICADA EXERCÍCIOS COMENTADOS - IME – ITA – OLIMPÍADA

e) – 0,10 kJ

Resolução: Alternativa A.

O problema coloca um processo de compressibilidade do gás. Cálculo do trabalho realizado:


10−3 m3
W = − pexterna x ∆V = − 1,20 x 105 Pa x (18,20 − 19,0)L x ( ) = + 96 J
1L

3,20 g J
Cálculo da quantidade de calor: Q = n x Cp x ∆T = g x 20,8 mol x K x 15 K = 249,6 J
4,0
mol

A partir da primeira lei da termodinâmica, temos: ∆U = Q + W = + 96 J + 249,6 J = + 345,6 J

Como há uma diminuição da temperatura, a variação da energia interna será negativa, ou seja, (- 350 J), o que é
equivalente a – 0,35 kJ.

Questão 855 – (ITA) Num cilindro, provido de um pistão móvel sem atrito, é realizada a combustão completa de
carbono (grafita). A temperatura no interior do cilindro é mantida constante desde a introdução dos reagentes até o
final da reação. Considere as seguintes afirmações:
I. A variação da energia interna do sistema é igual a zero.
II. O trabalho realizado pelo sistema é igual a zero.
III. A quantidade de calor trocada entre o sistema e a vizinhança é igual a zero.
IV. A variação da entalpia do sistema é igual à variação da energia interna.
Destas afirmações, está(ão) correta(s):
a) apenas I
b) apenas I e IV
c) apenas I, II e III
d) apenas II e IV
e) apenas III e IV

Resolução: Alternativa D.

I) Para um processo de combustão do carbono grafita, C(s) + O2(g) → CO2(g), a variação da entalpia é igual a
energia interna, pelo fato da variação do número de mol, Δn = nprodutos – nreagentes ser igual a zero (1 – 1 = 0). Como a
equação é ΔH = ΔU + Δn.R.T, com Δn = 0 , temos ΔH = ΔU.
II) Se o pistão é móvel e não apresenta atrito, o trabalho será igual a zero.
III) A quantidade de calor trocada entre o sistema e a vizinhança não é igual a zero, pois se trata de uma reação
de combustão (ΔH = Qp < 0).
IV) A variação da entalpia do sistema é igual a variação da energia interna, conforme mostrado no item I.

Questão 856 – (ITA) Para transformar completamente 1cm3 de água a 100°C e 1 atm em vapor (que ocupará
1671cm3) a 100°C e 1 atm, é necessário fornecer 539 calorias. Nestas condições, o trabalho realizado pelo gás em
expansão e o aumento da energia interna serão, respectivamente (valores aproximados):
a) 0,17 kJ e 2,09 kJ
b) 2,09 kJ e 0,17 kJ
c) 0,17 kJ e 2,26 kJ
d) 1,13 kJ e 1,13 kJ
e) Nenhum dos resultados acima

436
1600 FÍSICO-QUÍMICA APLICADA EXERCÍCIOS COMENTADOS - IME – ITA – OLIMPÍADA

Resolução: Alternativa A.
Cálculo do trabalho (W), considerando que a pressão externa seja constante: W = − pest x ∆V

105 Pa 10−6 m3
W = − 1 atm x ( ) x (1671 − 1)cm3 x ( ) = −167 J (≅ −0,17 kJ)
1 atm 1 cm3

O problema afirma que o sistema absorve calor, logo o processo é endotérmico.


Q = + 539 cal
4,18 J
Convertendo esta quantidade de calor para Joules, temos: Q = +539 cal x ( ) = +2253,02 J (+2,25 kJ)
1 cal

Cálculo da variação da energia interna, a partir da primeira lei da termodinâmica: ∆U = Q + W = + 2,25 kJ +


(−0,17 kJ) = + 2,08 kJ
Questão 857 – (IME) Deseja-se minimizar a taxa de transferência de calor em uma parede feita de um determinado
material, de espessura conhecida, submetendo-a a um diferencial de temperatura. Isso é feito adicionando-se uma
camada isolante refratária de 15% da espessura da parede, de forma que cuidadosas medidas experimentais indicam
que a taxa de transferência de calor passa a ser 40% em relação à situação original. Supondo que o diferencial de
temperatura entre as extremidades livres da parede original e da parede composta seja o mesmo, pode-se afirmar
que a condutividade térmica do material refratário é numericamente igual a:
a) 10 % da condutividade térmica do material da parede.
b) 15 % da condutividade térmica do material da parede.
c) 4,5 % da condutividade térmica do material da parede.
d) 22,22 % da condutividade térmica do material da parede.
e) 33,33 % da condutividade térmica do material da parede.

Resolução: Alternativa A.
k x A x ∆T
Fórmula de Fourier para o fluxo térmico: q =
∆x
x
Fórmula da resistência térmica (RT): q =
kxA

Condição inicial:

Condição final:

2 = 0,41

Como os materiais formam um circuito térmico em série, temos que: R equivalente = R1 + R 2

437
1600 FÍSICO-QUÍMICA APLICADA EXERCÍCIOS COMENTADOS - IME – ITA – OLIMPÍADA

x 0,15 . x
R equivalente = +
k1 . A k2 . A
∆T ∆T A.∆T
q2 = = 1 x 0,15.x = x 0,15.x
Requivalente .( + ) ( + )
A k1 k2 k1 k2

Sabendo que k 2 = 0,4. k 2 + 0,06. k1 , temos: k 2 − 0,4. k 2 = 0,06. k1

0,6. k 2 = 0,06. k1
0,06
k2 = . k = 0,1. k1
0,6 1
k2
= 0,1 (10%)
k1

Questão 858 - (ITA) Uma pessoa dorme sob um cobertor de 2,5 cm de espessura e de condutibilidade térmica 3,3 x
10–4 J.cm–1.s–1.(°C)–1. Sua pele está a 33°C e o ambiente a 0°C. O calor transmitido pelo cobertor durante uma hora,
por m2 de superfície, é:
a) 4,4 x 10–3 J
b) 4,3 x 102 J
c) 1,6 x 102 J
d) 2,8 x 102 J
e) 1,6 x 105 J

Resolução: Alternativa E.
x T
Utilizando o processo de condução de calor por condução: q. ∫0 dx = −k. A. ∫T 2 dT
1
0,0250 0
J 1 cm
q. ∫ dx = − (3,3 x 10−4 ). ( −2 ) . ∫ dT
0 cm x s x °C 10 m 33

J 1 cm
q. (0,0250 − 0) m = − (3,3 x 10−4 ) . ( −2 ) . (0 − 33)°C
cm x s x °C 10 m
J 1
q. (0,0250 − 0) m = − (3,3 x 10−4 ) . ( −2 ) . (−33)
s 10 m
108,9 x 10−2 J J J 3600 s J
q=+ 0,0250 m2 x s
. Convertendo para m² x h, temos: q = 43,52 m² x s
x 1h
= 1,57 x 105 m² x h

Questão 859 – (IME) A figura composta por dois materiais sólidos diferentes A e B, apresenta um processo de
condução de calor, cujas temperaturas não variam com o tempo.

É correto afirmar que a temperatura T2 da interface desses materiais, em kelvins, é:


a) 400
b) 500

438
1600 FÍSICO-QUÍMICA APLICADA EXERCÍCIOS COMENTADOS - IME – ITA – OLIMPÍADA

c) 600
d) 700
e) 800
Observações:
• T1: Temperatura da interface do material A com o meio externo
• T3: Temperatura da interface do material B com o meio externo
• KA: Coeficiente de condutividade térmica do material A
• KB: Coeficiente de condutividade térmica do material B

Resolução: Alternativa B.

dT
Analisando o material A, através da equação de Fourier: q = −k. A. dx

Desenvolvendo a equação de Fourier, temos: qA . dx = −k. A. dT


0,10 300
qA
x ∫ dx = −k. ∫ dT
A 0 T

q′′A x 0,10 = T − 300

T−300
q′′A = 0,10
(Equação A)

dT
Analisando o material B: q = −k. A. dx

Desenvolvendo a equação de Fourier, temos: qB . dx = −k. A. dT

0,10 T
qB
x ∫ dx = −0,2. ∫ dT
A 0 1500

q′′B x 0,10 = −0,20 x (T − 1500)


0,10
q′′B x = (1500 − T)
0,20

q′′B = 2 x (1500 − T) (Equação B)

T−300
Igualando ambas as equações A e B, temos: q′′A = 0,10
= 2 x (1500 − T) = q′′B

T − 300
= 3000 − 2T
0,10

T − 300 = 300 − 0,20. T

T + 0,20. T = 300 + 300

600
T= = 500 K
1,20

Questão 860 – (IME) Considere as reações abaixo:


H2(g) + ½ O2(g) → H2O(l) (I)

439
1600 FÍSICO-QUÍMICA APLICADA EXERCÍCIOS COMENTADOS - IME – ITA – OLIMPÍADA

H2(g) + ½ O2(g) → H2O(g) (II)


Assinale a alternativa correta.
a) o decréscimo de entropia é menor na reação (I) do que na reação (II)
b) o acréscimo de entropia na reação (I) é maior do que na reação (II)
c) o decréscimo de entropia é menor na reação (II) do que na reação (I)
d) o acréscimo de entropia na reação (II) é maior do que na reação (I)
e) a variação de entropia é igual em ambas as reações.

Resolução: Alternativa C.
Observando as duas reações de síntese apresentados, I e II, para a reação I há a formação da água na fase líquida
enquanto para a reação II a água está sendo formada na fase gasosa. Com isso, a entropia na reação I é maior do
que a reação II.

Questão 861 – (IME) 100 gramas de água líquida foram aquecidos utilizando o calor liberado na combustão de 0,25
gramas de etanol. Sabendo que a variação da temperatura da água foi de 12,5oC, assinale a alternativa que apresenta
o valor correto para a entalpia molar de combustão do etanol. Considere que a capacidade calorífica da água é igual
a 4,18 kJ,kg-1.oC-1 e que a energia liberada na combustão do etanol foi utilizada exclusivamente no aquecimento da
água.
a) – 961 kJ
b) - 5,2 kJ
c) + 4,2 kJ
d) + 5,2 kJ
e) + 961 kJ

Resolução: Alternativa A.
Equação química: C2H6O + 7/2 O2 → 2 CO2 + 3 H2O.

Cálculo da quantidade de calor recebida pela água (∆HH2 O ): ∆HH2 O = m x c x ∆T =


−3 kJ
100 x 10 kg x 4,18 kg x °C x 12,50°C = + 5,225 kJ

A energia responsável pelo aquecimento da água é proveniente do processo de combustão do etanol [C2H6O]. O
problema pede o valor da entalpia molar do etanol. Como 1 mol de etanol há 46 gramas, logo:

0,25 g ------------ + 5,225 kJ


46 g -------------- ∆H
∆H = + 961,4 kJ

Como a reação de queima do etanol é um processo exotérmico, logo, ∆H = − 961,4 kJ.

Questão 862 – (ITA) Diferentemente da dinâmica newtoniana, que não distingue passado e futuro, a direção temporal
tem papel marcante no nosso dia. Assim, por exemplo, ao aquecer uma parte de um corpo macroscópico e o isolarmos
termicamente, a temperatura deste se torna gradualmente uniforme, jamais se observando o contrário, o que indica a
direcionalidade do tempo. Diz-se então que os processos macroscópicos são irreversíveis, evoluem do passado para
o futuro e exibem o que o famoso cosmólogo Sir Arthur Eddington denominou de seta do tempo. A lei física que melhor
traduz o tema do texto é:
a) a segunda lei de Newton
b) a lei de conservação da energia
c) a segunda lei da termodinâmica
d) a lei zero da termodinâmica
e) a lei de conservação da quantidade de movimento

440
1600 FÍSICO-QUÍMICA APLICADA EXERCÍCIOS COMENTADOS - IME – ITA – OLIMPÍADA

Resolução: Alternativa C.
O texto refere-se à entropia de um determinado sistema, ou melhor, do aumento da entropia dos sistemas
termodinâmicos, o que é apresentado pela segunda lei da termodinâmica, que afirma o seguinte: “nunca será
observado, com o passar do tempo, um acúmulo de energia térmica em apenas um ponto do corpo.”

Questão 863 – (ITA) Considere os valores das seguintes variações de entalpia (ΔH) para as reações químicas
representadas pelas equações I e II, onde (graf) significa grafite. Com base nestas informações e considerando que
todos ΔH se referem à temperatura e pressão citadas acima, assinale a opção correta:

a) C(graf) + ½ O2(g) → CO(g) ΔH = + 110 kJ


b) 2C(graf) + O2(g) → 2CO(g) ΔH = – 110 kJ
c) 2C(graf) + ½ O2(g) → C(graf) + CO(g) ΔH = + 110 kJ
d) 2C(graf) + 2O2(g) → 2CO(g) + O2(g) ΔH = + 220 kJ
e) C(graf) + O2(g) → CO(g) + ½ O2(g) ΔH = – 110 kJ

Resolução: Alternativa E.
Aplicando a Lei de Hess, através das reações apresentadas, será necessário inverter a reação II:

C(graf) + O2(g) → CO2(g); ΔH(298 K; 1atm) = – 393 kJ


CO2(g) → CO(g) + ½ O2(g) ΔH(298 K; 1atm) = + 283 kJ +
C(graf) + O2(g) → CO(g) + ½ O2(g) ΔH(298 K; 1atm) = - 393 + 283 kJ = -110 kJ

Observação: As quantidades de calor pode ser determinado diretamente, o que nem sempre é possível, ou então,
podem ser estimados com a utilização do princípio de Hess (1840), partindo de valores conhecidos para os calores
de formação dos diversas compostos.

Questão 864 – (ITA) Assinale a opção errada que apresenta (em kJ.mol-1) a entalpia padrão de formação (∆Hf) da
substância a 25°C:
a) ∆Hf (H2(g)) = 0
b) ∆Hf (F2(g)) = 0
c) ∆Hf (N2(g)) = 0
d) ∆Hf (Br2(g)) = 0
e) ∆Hf (Cl2(g)) = 0

Resolução: Alternativa D.
Todas as substâncias químicas com exceção do bromo estão na sua forma física mais estável. Convencionou-se que
a entalpia de qualquer substância elementar em seu estado físico mais estável a uma temperatura igual a 25°C (298
K) e sob uma pressão igual a uma atmosfera (1 atm) seja igual a zero. Então, na forma estável à estas condições, o
bromo encontra-se na fase líquida e não na fase gasosa, conforme é apresentado.

Questão 865 – (ITA) Qual das opções a seguir apresenta a equação química balanceada para a reação de formação
de óxido de ferro (II) sólido nas condições-padrão?
a) Fe(s) + Fe2O3(s) → 3 FeO(s)
b) Fe(s) + ½ O2(g) → FeO(s)
c) Fe2O3(s) → 2 FeO(s) + ½ O2(g)
d) Fe(s) + CO (g) → FeO(s) + C(grafite)
e) Fe(s) + CO2(g) → FeO(s) + C(grafite) + ½ O2(g)

Resolução: Alternativa B.
O calor de formação é a variação da entalpia que acompanha a formação de um mol de um determinado composto a
partir de seus constituintes elementares na sua forma estável, à temperatura de 25°C e 1 atmosfera. Logo, para a
formação de óxido de ferro II, temos: Fe(s) e O2(g) encontram-se na sua forma mais estável.

441
1600 FÍSICO-QUÍMICA APLICADA EXERCÍCIOS COMENTADOS - IME – ITA – OLIMPÍADA

Questão 866 – (ITA) Num cilindro, provido de um pistão móvel sem atrito, é realizada a combustão completa de
carbono (grafita). A temperatura no interior do cilindro é mantida constante desde a introdução dos reagentes até o
final da reação. Considere as seguintes afirmações:
I. A variação da energia interna do sistema é igual a zero.
II. O trabalho realizado pelo sistema é igual a zero.
III. A quantidade de calor trocada entre o sistema e a vizinhança é igual a zero.
IV. A variação da entalpia do sistema é igual à variação da energia interna.
Destas afirmações, está(ão) correta(s):
a) apenas I
b) apenas I e IV
c) apenas I, II e III
d) apenas II e IV
e) apenas III e IV

Resolução: Alternativa D.
Considerando que o processo ocorra com pistão móvel sem atrito, W = 0.
Equação química de combustão do grafite: C(grafite) + O2(g) → CO2(g)

Através da definição de entalpia, temos: ΔH = ΔU + Δn x R x T


ΔH = ΔU + (nprodutos – nreagentes) x R x T
ΔH = ΔU + (1 – 1) x R x T = ΔU + 0 x R x T
ΔH = ΔU

Conclusão: Para a reação de combustão do grafite, a variação da entalpia será igual a variação de energia interna,
ΔH = ΔU.
I. Falso. A variação da energia interna do sistema é igual variação da entalpia.
II. Verdadeiro. Como o processo ocorre através de uma expansão livre, o trabalho realizado pelo sistema é igual
a zero.
III. Falso. A quantidade de calor trocada entre o sistema e a vizinhança é a variação da energia interna.
IV. Verdadeira. A variação da entalpia do sistema é igual à variação da energia interna.

Questão 867 – (ITA) Sabe-se que a 25°C, as entalpias de combustão (em kJ.mol-1) de grafita, gás hidrogênio e gás
metano são, respectivamente: -393,5; -285,9 e –890,5. Assinale a alternativa que apresenta o valor correto da entalpia
da seguinte reação: C(grafita) + 2 H2(g) → CH4(g).
a) -211,1 kJ.mol-1
b) – 74,8 kJ.mol-1
c) 74,8 kJ.mol-1
d) 136,3 kJ.mol-1
e) 211,1 kJ.mol-1

Resolução: Alternativa B.

Equação química de combustão da grafita: C(grafita) + O2(g) → CO2(g) ΔH = - 393,5 kJ.mol-1


Equação química de combustão do gás hidrogênio: H2(g) + ½O2(g) → H2O(g) ΔH = - 285,9 kJ.mol-1
Equação química de combustão do gás metano: CH4(g) + 2 O2(g) → CO2 + 2H2O(g) ΔH = - 890,5 kJ.mol-1

Aplicando a Lei de Hess, temos:


C(grafita) + O2(g) → CO2(g) ΔH = - 393,5 kJ.mol-1
2 H2(g) + O2(g) → 2 H2O(g) ΔH = 2 x (- 285,9 kJ.mol-1)
CO2(g) + 2 H2O(g) → CH4(g) + 2 O2(g) ΔH = + 890,5 kJ.mol-1 +
C(grafita) + 2 H2(g) → CH4(g) ΔH = – 74,8 kJ.mol -1

442
1600 FÍSICO-QUÍMICA APLICADA EXERCÍCIOS COMENTADOS - IME – ITA – OLIMPÍADA

Questão 868 – (ITA) Assinale a opção que indica a variação correta de entalpia, em kJ.mol -1, da reação química a
298,15 K e 1 bar, representada pela seguinte equação: C4H10(g) → C4H8(g) + H2(g). Dados eventualmente necessários:
ΔH°f (C4H8(g)) = - 11,4; ΔH°f (CO2(g)) = - 393,5; ΔH°f (H2O (l)) = - 285,8 e ΔH°c (C4H10(g)) = - 2877,6, em que ΔH°f e ΔH°c,
em kJ.mol-1, representam as variações de entalpia de formação e de combustão a 298,15 K e 1 bar, respectivamente:
a) – 3568,3
b) – 2186,9
c) + 2186,9
d) + 125,4
e) + 114,0

Resolução: Alternativa E.

Entalpia de formação do buteno: 4 C(S) + 4 H2(g) → C4H8(g) ΔH° = - 11,4 kJ.mol-1


(Equação A)

Entalpia de formação do dióxido de carbono: C(S) + O2(g) → CO2(g) ΔH° = - 393,5 kJ.mol-1
(Equação B)

Entalpia de formação da água no estado líquido: H2 (g) + ½ O2(g) → H2O(g) ΔH° = - 285,8 kJ.mol-1
(Equação C)

Entalpia de combustão completa do butano: C4H10(g) + 13/2 O2(g) → 4 CO2(g) + 5 H2O(g) ΔH° = - 2877,6 kJ.mol-1
(Equação D)

Aplicando a Lei de Hess pela análise , a partir das equações descritas acima, temos:

4 C(S) + 4 H2(g) → C4H8(g) ΔH° = - 11,4 kJ.mol-1

Invertendo e multiplicando por quatro a equação B:

4 CO2(g) → 4 C(S) + 4 O2(g) ΔH° = 4 x (+ 393,5) kJ.mol-1

Invertendo e multiplicando por cinco a equação C:

5 H2O(g) → 5 H2 (g) + 5/2 O2(g) ΔH° = 5 x (+ 285,8) kJ.mol-1

C4H10(g) + 13/2 O2(g) → 4 CO2(g) + 5 H2O(g) ΔH° = - 2877,6 kJ.mol-1

Somando as quatro reações químicas, temos:

4 C(S) + 4 H2(g) → C4H8(g) ΔH° = - 11,4 kJ.mol-1


4 CO2(g) → 4 C(S) + 4 O2(g) ΔH° = 4 x (+ 393,5) kJ.mol-1
5 H2O(g) → 5 H2(g) + 5/2 O2(g) ΔH° = 5 x (+ 285,8) kJ.mol-1
C4H10(g) + 13/2 O2(g) → 4 CO2(g) + 5 H2O(g) ΔH° = - 2877,6 kJ.mol-1 +
C4H10(g) → H2(g) + C4H8(g) ΔH°TOTAL = - 11,4 + [4 x (+ 393,5)] + [5 x (+ 285,8)] + (- 2877,6) = + 114,0
kJ.mol-1

443
1600 FÍSICO-QUÍMICA APLICADA EXERCÍCIOS COMENTADOS - IME – ITA – OLIMPÍADA

Questão 869 – (ITA) A figura abaixo mostra como a entalpia dos reagentes e dos produtos de uma reação química
do tipo A(g) + B(g) → C(g) varia com a temperatura.

Levando em consideração as informações fornecidas nesta figura, e sabendo que a variação de entalpia (ΔH) é igual
ao calor trocado pelo sistema à pressão constante, é errado afirmar que:
a) Na temperatura T1 a reação ocorre com liberação de calor
b) Na temperatura T1, a capacidade calorífica dos reagentes é maior que a dos produtos
c) No intervalo de temperatura compreendido entre T1 e T2 , a reação ocorre com absorção de calor (ΔH > zero)
d) O ΔH, em módulo, da reação aumenta com o aumento da temperatura
e) Tanto a capacidade calorífica dos reagentes como a dos produtos aumentam com o aumento da temperatura

Resolução: Alternativa C.
No intervalo de temperatura entre T1 e T2, a reação ocorre com liberação de calor, uma vez que o patamar dos
reagentes (Hreagentes) é maior que a dos produtos (Hprodutos), o que caracteriza como exotérmico (ΔH < 0).

Item a) Alternativa verdadeira, pois como o sentido da reação é dos reagentes para os produtos, observamos que o
patamar da entalpia dos reagentes é maior que a dos produtos, conforme podemos observar no gráfico a seguir:

Item b) Alternativa verdadeira, pois como a entalpia é definida como o produto da capacidade calorífica com a
temperatura, ΔH = n x Cp x dT, logo podemos concluir que: quanto maior a entalpia, consequentemente maior a
capacidade calorífica, uma vez que Cp = f(T).
Item d) Alternativa verdadeira, considerando que seja em módulo, uma vez que a reação é exotérmica.
Item e) Alternativa verdadeira, o que se trata de uma característica importante da capacidade calorífica, ou seja, trata-
se de uma propriedade diretamente relacionada com a temperatura, Cp = f(T).

Questão 870 - (OLIMPÍADA BRASILEIRA DE QUÍMICA) A partir das entalpias das reações dadas abaixo:
2 C(grafite) + 2 H2(g) → C2H4(g); ΔH0 = + 52,0 kJ
C2H4Cl2(g) → Cl2(g) + C2H4(g) . ΔH0 = + 116,0 kJ

444
1600 FÍSICO-QUÍMICA APLICADA EXERCÍCIOS COMENTADOS - IME – ITA – OLIMPÍADA

Podemos concluir que a entalpia molar de formação (em kJ.mol-1) do C2H4Cl2(g), será igual a:
a) - 64 kJ.mol-1
b) + 64 kJ.mol-1
c) - 168 kJ.mol-1
d) + 168 kJ.mol-1
e) + 220 kJ.mol-1

Resolução: Alternativa A.

Aplicando a lei de Hess, temos:


Equação química (1): 2 C(grafite) + 2 H2(g) → C2H4(g) ΔH0 = + 52,0 kJ
Invertendo a segunda equação (2): C2H4(g) + Cl2(g) → C2H4Cl2(g). ΔH0 = - 116,0 kJ

2 C(grafite) + 2 H2(g) → C2H4(g) ΔH0 = + 52,0 kJ


C2H4(g) .+ Cl2(g) → C2H4Cl2(g). ΔH0 = - 116,0 kJ +

2 C(grafite) + 2 H2(g) .+ Cl2(g) → C2H4Cl2(g).


ΔH°REAÇÃO = - 64,00 kJ para cada 1 mol de formação de C2H4Cl2(g)

Questão 871 – (OLIMPÍADA BRASILEIRA DE QUÍMICA) O acetileno ou etino (C2H2) é um gás de grande uso
comercial, sobretudo em maçaricos de oficinas de lanternagem. Assinale a opção que corresponde à quantidade de
calor liberada pela combustão completa de 1 mol de acetileno, a 25°C, de acordo co a reação abaixo: 2 C 2H2(g) + 5
O2(g) → 4 CO2(g) + 2 H2O(g). Dados: ΔH°f [C2H2(g)] = + 227 kJ.mol-1; ΔH°f [CO2(g)] = - 394 kJ.mol-1; ΔH°f [H2O(g)] = - 242
kJ.mol-1.
a) 204 kJ
b) 409 kJ
c) 863 kJ
d) 1257 kJ
e) 2514 kJ

Resolução: Alternativa D.

Cálculo da entalpia padrão(∆H°reação), a partir da reação de combustão do acetileno: 2 C2H2(g) + 5 O2(g) → 4 CO2(g) +
2 H2O(g)

0 0 0
0
∆Hreação = ∑ Hprodutos 0
− ∑ Hreagentes = [4 x HCO 2
+ 2 x HH 2O
] − [2 x HC02 H2 + 5 x HO0 2 ]

0
kJ
∆Hreação = [4 x (−394) + 2 x (−242)] − [2 x (+ 227) + 5 x 0] = − 2514
mol

Cálculo da quantidade de calor para a queima de 1 mol de acetileno:


2 mol de C2H2 ---------- (- 2514 kJ)
1 mol de C2H2 ---------- ΔH°
ΔH° = - 1257,00 kJ

Questão 872 - (OLIMPÍADA BRASILEIRA DE QUÍMICA) Os produtos da combustão do H2S(g) são H2O(g) e SO2(g).
Usando as informações dadas nas equações termoquímicas abaixo:

H2(g) + S(s) → H2S(g) ΔH = - 21 kJ


S(s) + O2(g) → SO2(g) ΔH = - 297 kJ
H2(g) + ½ O2(g) → H2O (g) ΔH = - 242 kJ

445
1600 FÍSICO-QUÍMICA APLICADA EXERCÍCIOS COMENTADOS - IME – ITA – OLIMPÍADA

Conclui-se que a energia desprendida na combustão de 1 mol de H2S(g) é:

a) – 67 kJ
b) 34 kJ
c) – 560 kJ
d) – 34 kJ
e) – 518 kJ

Resolução: Alternativa E.

Equação química: H2S(g) + 3/2 O2(g) → SO2(g) + H2O(g)

Invertendo a equação química: H2S(g) → H2(g) + S(s) ΔH = + 21 kJ


S(s) + O2(g) → SO2(g) ΔH = - 297 kJ
H2(g) + ½ O2(g) → H2O (g) ΔH = - 242 kJ

Somando as equações químicas, temos: H2S(g) + 3/2 O2(g) → SO2(g) + H2O(g)

ΔHTOTAL = + 21 kJ + (- 297 kJ) + (- 242 kJ) = - 518 kJ

Questão 873 - (OLIMPÍADA BRASILEIRA DE QUÍMICA) Assinale a opção que apresenta a equação química da
reação, cuja entalpia é a entalpia padrão de formação do gás etano (C2H6):

a) 2 C(g) + 6 H(g) → C2H6(g)


b) 2 C(s) + 6 H(g) → C2H6(g)
c) 2 C(g) + 3 H2(g) → C2H6(g)
d) 2 C(s) + 3 H2(g) → C2H6(g)
e) CH4(g) + H2(g) → C2H6(g)

Resolução: Alternativa D.
A entalpia de formação do gás etano (ΔHf) ocorre segundo as substâncias na sua forma mais estável. Logo, a reação
de formação refere-se a seguinte equação química balanceada: 2 C(s) + 3 H2(g) → C2H6(g).

Questão 874 - (ITA) Considere que os quatro processos químicos, descritos a seguir nos itens I a IV, são realizados
isobárica e isotermicamente:

I. KNO3(s) → K+(aq) + NO3-(aq)


II. H2O(l) → H2O(g)
III. C(grafite) → C(diamante)
IV. 2 Na(s) + ½ O2(g) → Na2O(s)

Qual das opções abaixo contém os processos químicos cuja variação de energia interna é nula?

a) Apenas I e II
b) Apenas I, II e III
c) Apenas II e III
d) Apenas III e IV
e) Nenhum processo

Resolução: Alternativa C.

I) Cálculo da quantidade de calor da água, aquecimento e mudança de fase para vaporizar 1 L de água pura a partir
de 25°C:

Q TOTAL = Q SENSÍVEL + Q VAPORIZAÇÃO

446
1600 FÍSICO-QUÍMICA APLICADA EXERCÍCIOS COMENTADOS - IME – ITA – OLIMPÍADA

1000 1000
Q TOTAL = [( ) x 75,9 x (100 − 25)] + [( ) x 40660] = 2,57 x 106 J
18 18
1000 1000
Q TOTAL = [( ) x 75,9 x 75] + [( ) x 40660]
18 18

Q TOTAL = 2,57 x 106 J

II) Calor da quantidade de calor de combustão do gás metano:CH4(g) + 2O2(g) → CO2(g) + 2H2O(g)

0 0
∆𝐻𝑟𝑒𝑎çã𝑜 = [1 𝑥 𝐻𝐶𝑂2
+ 2 𝑥 𝐻𝐻02 𝑂 ] − [1 𝑥 𝐻𝐶𝐻
0
4
+ 2 𝑥 𝐻𝑂02 ]

0
𝑘𝐽
∆𝐻𝑟𝑒𝑎çã𝑜 = [1 𝑥 (−393,5) + 2 𝑥 (−285,83)] − [1 𝑥 (− 74,81) + 2 𝑥 0] = − 890,35
𝑚𝑜𝑙

III) Cálculo do número de mol de gás metano:

890, 35 kJ -------------- 1 mol de CH4


2575, 14 kJ ------------ X
X  2,90 mol de CH4

Questão 875 – (ITA) Nitrato de amônio pode explodir porque a sua decomposição é exotérmica. Qual das opções a
seguir contém a equação química, envolvendo este composto que representa a reação mais exotérmica?
a) NH4NO3(c) → 2N(g) + 4H(g) + 3O(g)
b) NH4NO3(c) → NH3(g) + HNO3(g)
c) NH4NO3(c) → N2(g) + H2O(g) + H2O2(g)
d) NH4NO3(c) → N2(g) + 2H2O(g) + ½ O2(g)
e) NH4NO3(c) → N2(g) + 2H2(g) + ½ O2(g)

Resolução: Alternativa E.
A reação química mais exotérmica refere-se à alternativa E, pelo fato da formação de água no estado gasoso. Para a
alternativa D, há a formação de água oxigenada, o que não torna a reação mais exotérmica em comparação a
alternativa E.

Questão 876 – (IME) Considere as seguintes afirmativas:

I – Uma reação química a temperatura e pressão constantes será espontânea se a variação da energia livre de Gibbs
(∆G) for menor que zero.
II – Em um sistema reacional onde a única forma de trabalho observável é o trabalho de expansão, a variação da
entalpia (∆H) é igual à quantidade de calor liberada ou absorvida pela reação, a pressão constante.
III – Para uma substância simples que admite mais de uma forma alotrópica, não há variação de entalpia na conversão
de uma forma em outra.
São corretas:
a) Somente I
b) Somente II
c) Somente III
d) I e II
e) I e III
Resolução: Alternativa D.
I – Verdadeiro. ∆G é uma propriedade termodinâmica extensiva (dependente da massa) definida como a variação da
energia livre de Gibbs e que mede o trabalho útil, ou seja, trabalho total menos o trabalho executado contra uma
pressão externa, o que se traduz pela relação termodinâmica.
∆G = ∆A + p x ∆V
∆G = Wmáximo + Wexpansão

447
1600 FÍSICO-QUÍMICA APLICADA EXERCÍCIOS COMENTADOS - IME – ITA – OLIMPÍADA

∆G < 0

Para ∆G < 0, o processo se caracteriza como um espontâneo.

II – Verdadeiro. A partir da primeira lei da termodinâmica, temos:

∆U = Q + W
∆U = Q − p∆V
UB − UA = Q − p x (VB − VA )
U (U )
⏟B + pVB − ⏟ A + pVA = Q

HA
HB

∆H = Q p

A variação da entalpia é igual ao calor trocado à pressão constante.

III – Falso. Por convenção, a entalpia para substâncias simples na sua forma mais estável é igual a zero (formas
alotrópicas). Para formas não alotrópicas, a entalpia será diferente de zero.

Questão 877 – (ITA) Sob 1 atm e a 25C, qual das reações abaixo equacionadas deve ser a mais exotérmica?

a) H2(g) + F2(g) → 2HF(g)


b) H2(g) + Cl2(g) → 2HCl(g)
c) H2(g) + I2(g) → 2HI(g)
d) Br2(g) + I2(g) → 2BrI(g)
e) Cl2(g) + Br2(g) → 2ClBr(g)

Resolução: Alternativa A.
A maior diferença de eletronegatividade entre os elementos químicos apresentados ocorre entre o flúor (F) e o
hidrogênio (H). Por isso, a ligação H – F, é a reação mais exotérmica.

Questão 878 – (ITA) Considere as informações contidas nas seguintes equações termoquímicas, todas referentes à
temperatura de 25°C e pressão de uma atmosfera:

1. H2O(l) → H2O(g) ΔH1 = 44,0 kJ.mol-1


2. CH3CH2OH(l) → CH3CH2OH (g) ΔH2 = 42,6 kJ.mol-1
3. CH3CH2OH(l) + 7/2 O2(g) → 2 CO2(g) + 3 H2O(l) ΔH3 = - 1366,8 kJ.mol-1
4. CH3CH2OH(l) + 7/2 O2(g) → 2 CO2(g) + 3 H2O(g) ΔH4 = ?
5. CH3CH2OH(g) + 7/2 O2(g) → 2 CO2(g) + 3 H2O(l) ΔH5 = ?
6. CH3CH2OH(g) + 7/2 O2(g) → 2 CO2(g) + 3 H2O(g) ΔH6 = ?

Em relação ao exposto acima, é errado afirmar que:

a) As reações representadas pelas equações 1 e 2 são endotérmicas;


b) As reações representadas pelas equações 3, 4, 5 e 6 são exotérmicas;
c) ΔH4 = - 1234,8 kJ.mol-1
d) ΔH5 = - 1324,2 kJ.mol-1
e) ΔH6 = - 1277,4 kJ.mol-1

448
1600 FÍSICO-QUÍMICA APLICADA EXERCÍCIOS COMENTADOS - IME – ITA – OLIMPÍADA

Resolução: Alternativa D.

a) As equações químicas 2 e 3 são processos endotérmicos, pois ΔH > 0.


b) Aplicando a lei de Hess para a obtenção da reação 3, temos:

Multiplicando a primeira reação por três, temos:


3 H2O(l) → 3 H2O(g) ΔH1 = 3 x (44,0) kJ.mol-1
CH3CH2OH(l) + 7/2 O2(g) → 2 CO2(g) + 3 H2O(l) ΔH3 = - 1366,8 kJ.mol-1 +
CH3CH2OH(l) + 7/2 O2(g) → 2 CO2(g) + 3 H2O(g) ΔH3 = - 1234,8 kJ.mol-1

Aplicando a lei de Hess para a obtenção da reação 5, temos:

Multiplicando a primeira reação por três, temos:


CH3CH2OH(g) → CH3CH2OH (l) ΔH2 = - 42,6 kJ.mol-1
CH3CH2OH(l) + 7/2 O2(g) → 2 CO2(g) + 3 H2O(l) ΔH3 = - 1366,8 kJ.mol-1 +
CH3CH2OH(g) + 7/2 O2(g) → 2 CO2(g) + 3 H2O(l) ΔH3 = - 1409,4 kJ.mol-1

Aplicando a lei de Hess para a obtenção da reação química 6, multiplicando a primeira reação por três, temos:
CH3CH2OH(g) → CH3CH2OH (l) ΔH2 = - 42,6 kJ.mol-1
CH3CH2OH(l) + 7/2 O2(g) → 2 CO2(g) + 3 H2O(l) ΔH3 = - 1366,8 kJ.mol-1
3 H2O(l) → 3 H2O(g) ΔH1 = 3 x (44,0) kJ.mol-1 +
CH3CH2OH(g) + 7/2 O2(g) → 2 CO2(g) + 3 H2O(g) ΔH3 = - 1277,4 kJ.mol-1

Questão 879 – (IME) Considere os seguintes processos conduzidos a 25°C e 1 atm:


(1) 4 Fe(s) + 3 O2(g) → 2 Fe2O3(s)
(2) H2O(s) → H2O(l)
(3) CH4(g) + 2 O2(g) → CO2(g) + 2 H2O(g)
(4) Cu2S(s) → 2 Cu(s) + S(s), com ΔG = + 82,2 kJ
(5) S(s) + O2(g) → SO2(g), com ΔG = - 300,4 kJ
(6) Cu2S(s) + O2(g) → 2 Cu(s) + SO2(g)
(7) 2NO(g) + O2(g) → 2 NO2(g)

Assinale a afirmativa correta.


a) Os processos (1), (4) e (5) não são espontâneos
b) Os processos (2) é exotérmico e apresenta variação de entropia positiva
c) Os processos (3) é endotérmico e apresenta variação de entropia negativa
d) Os processos (2) e (7) apresentam variação de entropia positiva
e) Os processos (1), (2) e (6) são espontâneos

Resolução: Alternativa E.

a) O processo é espontâneo, uma vez que se trata do processo de corrosão do ferro. Logo ΔG < 0.
b) O processo é endotérmico, uma vez que se trata da fusão do gelo.
c) A reação de combustão do gás metano é um processo exotérmico.
d) A reação (7) apresenta variação de entropia negativa, pelo fato do menor número de mol de gases no
produto.
e) Alternativa correta.

Questão 880 - (IME) A entalpia de fusão de uma determinada substância é 200 kJ.kg-1, e seu ponto de fusão normal
é 27°C. Após a solidificação de 3 kg do material, pode-se afirmar que a entalpia desse sistema:
a) Diminui 2 kJ.K-1
b) Diminui 600 kJ.K-1
c) Não variou

449
1600 FÍSICO-QUÍMICA APLICADA EXERCÍCIOS COMENTADOS - IME – ITA – OLIMPÍADA

d) Aumentou 2 kJ.K-1
e) Aumentou 600 kJ.K-1

Resolução: Alternativa A.

Cálculo da quantidade de calor para uma massa igual a 3 kg:

1 kg --------------- 200 kJ
3 kg --------------- Q
Q = 600 kJ
Qreversível
Sabendo que o processo de solidificação reduz a entropia do sistema, temos: dS = −
dT

600 kJ 600 kJ kJ
dS = − =− = −2
(27 + 273)K 300 K kg

Questão 881 – (ITA) Considere a energia liberada em:


I. combustão completa (estequiométrica) do octano e em
II. célula de combustível de hidrogênio e o oxigênio.
Assinale a opção que apresenta a razão correta entre a quantidade de energia liberada por átomo de hidrogênio na
combustão do octano e na célula de combustível. Dados: Energias de ligação, em kJ mol -1:
C–C 347 H–H 436
C–H 413 H–O 464
C=O 803 O=O 498
a) 0,280
b) 1,18
c) 2,35
d) 10,5
e) 21,0

Resolução: Alternativa C.

Cálculo da entalpia de ligação para os reagentes: C8H18 + 25/2 O2 → 8 CO2 + 9 H2O


HReagentes = + {(7 x 347) + (18 x 416) + (12,5 x 498)} = + 16142 kJ.mol-1

Cálculo da entalpia de ligação para os produtos: HProdutos = - {(16 x 803) + (18 x 464)} = - 21200 kJ.mol-1

Cálculo da entalpia de reação:


HReação = HProdutos + HReagentes
HReação = - 21200 kJ.mol-1 + 16142 kJ.mol-1
HReação = - 5058,00 kJ.mol-1 (para cada mol de octano)
− 5058,00 𝑘𝐽
Para cada mol de átomo de hidrogênio (H), teremos: ∆𝐻𝐻 = = −281
18 𝑚𝑜𝑙

Analisando a reação para a célula combustível – Cálculo da entaplia de ligação: H2(g) + 1/2 O2(g) → H2O(g)

HReagentes = + {(1 x 436) + (0,50 x 498)} = + 436 + 249 = + 685 kJ.mol-1

HProdutos = - (2 x 464) = - 928 kJ.mol-1

450
1600 FÍSICO-QUÍMICA APLICADA EXERCÍCIOS COMENTADOS - IME – ITA – OLIMPÍADA

Cálculo da entalpia de reação: HReação = HProdutos + HReagentes = + 685 kJ.mol-1 + (- 928 kJ.mol-1) = - 243 kJ.mol-1 (para cada mol
de gás hidrogênio)
− 281,00
Para cada mol de átomo de hidrogênio (H), teremos: ∆𝐻𝐻 = −121,50
= +2,31

Questão 882 – (ITA) Considere as reações representadas pelas seguintes equações químicas balanceadas:
a) C2H5OH(l) + O2(g) → 2C(s) + 3H2O(g); ∆HI(T); ∆UI(T).
b) C2H5OH(l) + 2O2(g) → 2CO(g) + 3H2O(l); ∆HII(T); ∆UII(T).
Sendo ∆H(T) e ∆U(T), respectivamente, a variação da entalpia e a energia interna do sistema na temperatura T. Admita
que as reações acima são realizadas sob pressão constante, na temperatura T, e que a temperatura dos reagentes é
igual à dos produtos. Considere que, para as reações representadas pelas equações acima, sejam feitas as seguintes
comparações:
I. |∆EI | = |∆UII|.
II. |∆HI| = |∆HII|.
III. |∆HII | > |∆UII|.
IV. |∆HI| < |∆UI |.
Das comparações acima, está(ão) correta(s):
a) apenas I
b) apenas I e II
c) apenas II.
d) apenas III
e) apenas IV

Resolução: Alternativa E.

I. Falso. Analisando a primeira reação química, a partir da definição termodinâmica da entalpia.

∆HI = ∆UI + ∆n x R x T
∆HI = ∆UI + (3 − 1) x R x T
∆HI = ∆UI + 2 x R x T

Analisando a segunda reação química, a partir da definição termodinâmica da entalpia.

∆HII = ∆UII + ∆n x R x T
∆HII = ∆UII + (2 − 2) x R x T
∆HII = ∆UII

II. Falso. Como as reações químicas são diferentes, a variação química da entalpia são distintas, ou seja,
∆HI ≠ ∆HII

III. Falso. Através da reação química II, temos: C2H5OH(l) + 2O2(g) → 2CO(g) + 3H2O(l); ∆HII(T); ∆UII(T).

∆HII = ∆UII + ∆n x R x T
∆HII = ∆UII + (2 − 2) x R x T
∆HII = ∆UII
IV. Verdadeiro. Através da reação química I, temos: C2H5OH(l) + O2(g) → 2C(s) + 3H2O(g); ∆HI(T); ∆EI(T)

∆HI = ∆UI + ∆n x R x T
∆HI = ∆UI + (3 − 1) x R x T
∆HI = ∆UI + 2 x R x T

Como se trata de uma reação exotérmica, a equação termodinâmica ficará da seguinte maneira:

451
1600 FÍSICO-QUÍMICA APLICADA EXERCÍCIOS COMENTADOS - IME – ITA – OLIMPÍADA

|∆UI | = |− ∆HI − 2 x R x T|
|∆UI | > | ∆HI |

Questão 883 – (ITA) Para determinar a entalpia de vaporização do composto hipotético MX4(l), o mesmo foi colocado
num recipiente equipado com uma serpentina de aquecimento resistivo, a 80ºC e sob pressão de 1,0 bar. Para a
manutenção da temperatura, foi utilizada uma fonte de 30 V com passagem de corrente de 900 mA durante 30
segundos, tendo sido vaporizados 2,0 g de MX4(l). Sabendo que a massa molar desse composto é 200 g.mol–1, assinale
a opção que apresenta a entalpia molar de vaporização em kJ.mol–1, a 80ºC.
a) 4,1
b) 8,1
c) 81
d) 405
e) 810

Resolução: Alternativa C.
2,0
Cálculo do número de mol de MX4(l): n = 200 = 10−2 mol

Cálculo da potência: P = V x i = 30 x (900 x 10-3) = 27 W (27 J.s-1)

J
Q = 27,0 x 30 s = 810,0 J
s
810 J J kJ
Cálculo da entalpia molar de vaporização: ∆Hvaporização = 10−2 mol = 81000 mol (81 mol)

Questão 884 – (ITA) Considere as seguintes afirmações a respeito da variação, em módulo, da entalpia e da energia
interna das reações químicas, respectivamente representadas pelas equações química abaixo, cada uma mantida a
temperatura e pressão constantes.
I. H2O(g) + ½ O2(g) → H2O2(g) │∆HI│>│∆UI│
II. 4 NH3(g) + N2(g) → 3 N2H4(g) │∆HII│<│∆UII│
III. H2(g) + F2(g) → 2 HF(g) │∆HIII│<│∆UIII│
IV. HCl(g) + 2 O2(g) → HClO4(l) │∆HIV│<│∆UIV│
V. CaO(s) + 3 C(s) → CO(g) + CaC2(s) │∆HV│>│∆UV│
Das afirmações acima, estão corretas:
a) Apenas I, II e V
b) Apenas I, III e IV
c) Apenas II, IV e V
d) Apenas III e V
e) Todas

Resolução: Alternativa C.

Analisando a primeira reação química - I: ∆HI = ∆UI + ∆n x R x T


∆HI = ∆UI + (1 − 1,50) x R x T
∆HI = ∆UI − 0,50 x R x T
∆UI = ∆HI + 0,50 x R x T
Conclusão: |∆HI | < |∆UI |

Analisando a segunda reação química - II: ∆HII = ∆UII + ∆n x R x T


∆HII = ∆UII + (3 − 5) x R x T
∆HII = ∆UII − 2 x R x T
∆UII = ∆HII + 2 x R x T
Conclusão: |∆HII | < |∆UII |

452
1600 FÍSICO-QUÍMICA APLICADA EXERCÍCIOS COMENTADOS - IME – ITA – OLIMPÍADA

Analisando a terceira reação química - III: ∆HIII = ∆UIII + ∆n x R x T


∆HIII = ∆UIII + (2 − 2) x R x T
∆HIII = ∆UIII + 0 x R x T
Conclusão: |∆HIII | = |∆UIII |

Analisando a quarta reação química - IV: ∆HIV = ∆UIV + ∆n x R x T


∆HIV = ∆UIV + (0 − 3) x R x T
∆HIV = ∆UIV − 3 x R x T
∆UIV = ∆HIV + 3 x R x T
Conclusão: |∆HIV | < |∆UIV |

Analisando a quinta reação química - V:


∆HV = ∆UV + ∆n x R x T
∆HV = ∆UV + (1 − 0) x R x T
∆HV = ∆UV + 1 x R x T
∆HV = ∆UV + R x T
Conclusão: |∆HV | > |∆UV |

453
1600 FÍSICO-QUÍMICA APLICADA EXERCÍCIOS COMENTADOS - IME – ITA – OLIMPÍADA

Resolução: As equações
químicas balanceadas estão
apresentadas
abaixo:CAPÍTULO VI

CINÉTICA QUÍMICA –
ESTUDO DAS VELOCIDADES
DAS REAÇÕES QUÍMICAS

PROFESSOR ALEXANDRE
VARGAS GRILLO
454
1600 FÍSICO-QUÍMICA APLICADA EXERCÍCIOS COMENTADOS - IME – ITA – OLIMPÍADA

455
1600 FÍSICO-QUÍMICA APLICADA EXERCÍCIOS COMENTADOS - IME – ITA – OLIMPÍADA

Questão 885 - (ITA - MODIFICADA) A reação entre íons brometo e bromato, em meio aquoso e ácido, pode ser
representada pela seguinte equação química balanceada: 5 Br-(aq) + BrO3-(aq) + 6 H+(aq) → 3 Br2(aq) + 3 H2O(l). Sabendo
que a velocidade de desaparecimento do íon bromato é igual a 5,63 x 10-6 mol.L-1.s-1, determine:
a) a velocidade da reação, expressa em mol.L-1.s-1;
b) a velocidade de aparecimento do bromo, Br2, expressa em mol.L-1.s-1;
c) a velocidade de desaparecimento de H+, expressa em mol.L-1.s-1.

Resolução:
1 d[Br − ] 1 d[BrO− 3] 1 d[H + ] 1 d[Br2 ] 1 d[H2 O]
Vreação = − x =− x =− x =+ x =+ x
5 dt 1 dt 6 dt 3 dt 3 dt
1 1 1 1 1
Vreação = − x VBr− = − x VBrO−3 = − x VH+ = + x VBr2 = + x VH2 O
5 1 6 3 3
1 1 1 1 1
Vreação = − x VBr− = − x(5,63 x 10−6 ) = − x VH+ = + x VBr2 = + x VH2 O
5 1 6 3 3

a) Vreação = 5,63 x 10-6 mol.L-1.s-1

b) VBr2 = 1,69 x 10-5 mol.L-1.s-1

c) VH+ = 3,38 x 10-5 mol.L-1.s-1

Questão 886 – (ENADE) Em um laboratório, foram efetuadas diversas experiências para a reação: 2 H2(g) + 2 NO(g)
→ N2(g) + 2 H2O(g). Os resultados das velocidades iniciais obtidos estão representados na tabela a seguir.

Concentração de H2 Concentração de NO Velocidade Inicial da reação


Experiência
(mol.L-1) (mol.L-1) (mol.L-1.s-1)
1 0,10 0,10 0,10
2 0,20 0,10 0,20
3 0,10 0,20 0,40
4 0,30 0,10 0,30
5 0,10 0,30 0,90
Com base nesses dados, a equação da velocidade para a reação é:

a) v = k.[H2]
b) v = k.[NO]
c) v = k.[H2].[NO]
d) v = k.[H2]².[NO]
e) v = k.[H2].[NO]²

Resolução: Alternativa E. Primeira maneira de resolução: Aplicando o método das velocidades iniciais.
Equação da velocidade: v = k.[H2]α.[NO]β, onde α e β correspondem as ordens de reação dos reagentes A e B,
respectivamente. Aplicando a função logarítmica na equação acima, temos: Log v = log k + α.log[H2] + β.log[NO].
Através do experimento 1, temos: Log (0,10) = log k + α.log(0,10) + β.log(0,10)

-1 = log k – α – β

log k = α + β -1 (Equação 1)

Através do experimento 2, temos: Log (0,20) = log k + α.log(0,20) + β.log(0,10)

-0,70 = log k – 0,70α – β

456
1600 FÍSICO-QUÍMICA APLICADA EXERCÍCIOS COMENTADOS - IME – ITA – OLIMPÍADA

log k = 0,70α + β - 0,70 (Equação 2)

Através do experimento 3, temos: Log (0,40) = log k + α.log(0,10) + β.log(0,20)


-0,40 = log k – α – 0,70β

-0,40 = log k – 1 – 0,70β

log k = 0,70β + 0,60 (Equação 3)

Igualando as equações (1) e (2): α + β -1 = - 0,70 + 0,70α + β


α - 0,70α = - 0,70 + 1
0,30.α = 0,30
α = 1,0
Igualando as equações (1) e (3), temos: α + β -1 = 0,60 + 0,70α
1 + β -1 = 0,60 + 0,70α
β = 0,60 + 0,70α
0,30 x β = 0,60
β = 2,0
A ordem de reação para o gás hidrogênio é igual a 1 e a ordem de reação em relação ao monóxido de nitrogênio é
igual a 2. Equação da velocidade: v = k.[H2].[NO]².
É possível resolver a questão de uma maneira mais rápida e direta. Vamos a esta resolução...

Segunda maneira de resolução: Determinação da ordem de reação em relação ao gás hidrogênio (α): Observa-se
que entre o experimento 1 e o 2, a concentração da quantidade de matéria do monóxido de nitrogênio encontra-se
0,20 𝛼 0,20
constante, enquanto a concentração de gás hidrogênio dobra e a velocidade também dobra. Logo: (0,20) = 0,10

2𝛼 = 2

𝛼=1

Determinação da ordem de reação em relação ao monóxido de nitrogênio (β): Observa-se que entre o experimento 1
e o 3, a concentração da quantidade de matéria do gás hidrogênio encontra-se constante, enquanto a concentração
0,20 𝛽 0,40
de monóxido de nitrogênio dobra, e a velocidade quadruplica. Logo: (0,10) = 0,10

2𝛽 = 4

𝛽=2

Equação da velocidade: v = k.[H2].[NO]².

Questão 887 – (OLIMPÍADA MARANHENSE DE QUÍMICA) Num laboratório, foram efetuadas diversas experiências
para a reação: 2 H2(g) + 2 NO(g) → N2(g) + 2 H2O(g). Com os resultados das velocidades iniciais obtidos, montou-se a
seguinte tabela:
Experiência [H2] (mol.L-1) [NO] (mol.L-1) V (mol.L-1.s-1 )
1 0,10 0,10 0,10

457
1600 FÍSICO-QUÍMICA APLICADA EXERCÍCIOS COMENTADOS - IME – ITA – OLIMPÍADA

2 0,20 0,10 0,20


3 0,10 0,20 0,40
4 0,30 0,10 0,30
5 0,10 0,30 0,90

Baseando-se na tabela acima, podemos afirmar que a lei da velocidade para a reação é:
a) v = k.[H2]
b) v = k.[NO]
c) v = k.[H2].[NO]
d) v = k.[H2]2.[NO]
e) v = k.[H2].[NO]2

Resolução: Alternativa E.
Cálculo da ordem de reação em relação ao H2, entre o experimento 1 e 2: a concentração da quantidade de matéria
do monóxido de nitrogênio encontra-se constante, enquanto a concentração de gás hidrogênio dobra e a velocidade
0,20 𝑎 0,20
também dobra, logo: ( ) =
0,20 0,10

2𝑎 = 2
𝑎=1
Cálculo da ordem de reação em relação ao NO, entre o experimento 1 e 3: a concentração da quantidade de matéria
do hidrogênio gasoso encontra-se constante, enquanto a concentração de NO dobra e a velocidade quadruplica, logo:
0,20 𝑏 0,40
(0,10) = 0,10

2𝑏 = 4
𝑏=2
Equação da velocidade da reação: v = k.[H2] x [NO2]²
Ordem de reação em relação ao [NO] = 2, ordem de reação em relação ao [H2] = 1 e ordem de reação global = 2 + 1
= 3.
Questão 888 - (GRILLO) O mecanismo a seguir foi proposto para explicar a destruição da camada de ozônio:
O3 + Cl ⇄ ClO + O2
𝑘𝑎
ClO + O → Cl + O2
Através de experimentos laboratoriais verificou-se que, a primeira reação atingiu o equilíbrio de forma bastante rápida.
Sendo a constante de equilíbrio igual a K, deduza uma expressão para a velocidade de formação de oxigênio
molecular, em termos de K e ka além das concentrações de [O] e [O3].

Resolução: Para a reação reversível, a constante de equilíbrio fica representada pela seguinte equação: 𝐾𝑐 =
[𝐶𝑙𝑂] 𝑥 [𝑂2 ]
[𝑂3 ] 𝑥 [𝐶𝑙]
(Equação I). Através do par de reações químicas, tanto na etapa rápida (primeira reação) quanto na etapa
lenta (segunda reação), [ClO] é a espécie intermediária. Isolando a [ClO] na Equação I, temos: [𝐶𝑙𝑂] =
𝐾𝑐 𝑥 [𝑂3 ] 𝑥 [𝐶𝑙] 𝑘𝑎
[𝑂2 ]
(Equação II). A etapa lenta representada pela equação química balanceada ClO + O → Cl + O2,
apresenta como taxa temporal do Oxigênio está representado pela equação III.
d[O2 ]
dt
= k a x [ClO] x [O] (Equação III)

d[O2 ]
Substituindo a expressão da concentração de [ClO] (equação II) na equação III, temos: = k a x [ClO] x [O]
dt

458
1600 FÍSICO-QUÍMICA APLICADA EXERCÍCIOS COMENTADOS - IME – ITA – OLIMPÍADA

d[O2 ] K c x [O3 ] x [Cl]


= ka x x [O]
dt [O2 ]

d[O2 ] k a x K c x [O3 ] x [Cl] x [O]


=
dt [O2 ]

Questão 889 – (OLÍMPÍADA NORTE – NORDESTE DE QUÍMICA) Cloreto de sulfurila, SO2Cl2(g) se decompõe em
fase gasosa, produzindo SO2(g) e Cl2(g). A concentração do SO2Cl2(g) foi acompanhada em uma experiência e verificou-
se que o gráfico ln[SO2Cl2] contra o tempo é linear e que, em 240 segundos, a concentração caiu de 0,400 mol.L -1
para 0,280 mol.L-1.
a) Qual a constante de velocidade da reação SO2Cl2(g) → SO2(g) + Cl2(g)?
b) Qual o tempo de meia-vida desta reação?

[SO Cl ]
Resolução: Item a) Cálculo da constante cinética (k): ln ([SO 2Cl 2] Final ) = −k x t
2 2 Inicial
0,280
ln ( ) = −k x 240
0,400
k = 0,0015 s −1
ln2 0,693
Item b) Cálculo do tempo de meia-vida, para uma cinética de primeira ordem: t 1⁄ = k
= 0,0015 s−1 = 462 s
2

Questão 890 – A reação gasosa com cinética de primeira ordem está representado pela seguinte equação química
balanceada: SO2Cl2(g) → SO2(g) + Cl2(g), apresentando uma constante cinética igual a k1 = 2,20 x 10-5.min-1 a uma
temperatura de aproximadamente 600 K. Qual o valor da percentagem de uma amostra de SO2Cl2 que se decompõe
no aquecimento a mesma temperatura durante 1 hora, e também determine o tempo de meia-vida, em dias.
Resolução: Dados do problema: K = 2,20 x 10-5 min-1 (600 K) e tempo = 1 hora (60 minutos)
[SO Cl ]
Cálculo da percentagem de cloreto de sulfurila durante 1 hora (60 minutos): ln ([SO 2Cl 2] Final ) = −k. t
2 2 Inicial

[SO2 Cl2 ]Final −5


= e−(2,20 x 10 x 60) = e−0,00132 = 0,9986
[SO2 Cl2 ]Inicial
ln2 0,693
O valor percentual de decomposição é igual a 99,86%. Cálculo do tempo de meia-vida: t 1⁄ = = =
2 k 2,20 𝑥 10−5
31500 𝑠
Questão 891 – A decomposição em fase gasosa do aldeído acético, também conhecido como acetaldeído, ocorre
através de uma cinética de segunda ordem com velocidade específica de aproximadamente 0,30 L.mol -1.s-1. A partir
destas informações, determine:
a) O tempo de meia-vida, sabendo que a concentração inicial do aldeído é igual a 0,05 mol.L-1.
b) O tempo necessário para que a concentração deste composto diminua de 0,50 mol.L-1 para 0,10 mol.L-1.

Resolução: Equação química de decomposição: C2H4 O(g) → CH4(g) + CO(g).


1 1
Item a) Cálculo do tempo de meia-vida:t 1⁄ = k x [A] = 0,30 x 0,050 = 66,67 s
2 Inicial

1 1
Item b) Cálculo do tempo (t): [A] = [A] +kxt
Final Inicial

1 1
= + 0,30 x t
0,10 0,50

459
1600 FÍSICO-QUÍMICA APLICADA EXERCÍCIOS COMENTADOS - IME – ITA – OLIMPÍADA

1 1
− = 0,30 x t
0,10 0,50
t = 26,67 s
Questão 892 – (OLIMPÍADA BRASILEIRA DE QUÍMICA - MODIFICADA) Moléculas de butadieno podem acoplar
para formar C8H12. A expressão da velocidade para esta reação é v = k.[C4H6]² e a constante de velocidade estimada
é 0,014 L.mol-1.s-1. Se a concentração inicial de butadieno é 0,016 mol.L-1, determine o tempo necessário para que a
concentração decaia para 0,0016 mol.L-1.
Resolução: Cálculo do tempo sabendo que o processo é de segunda ordem.
1 1
= +kxt
[C4 H6 ]Final [C4 H6 ]Inicial

1 1
= + 0,014 x t
0,0016 0,016

t = 40178,57 s

Questão 893 – A reação química do tipo 3A → P, apresenta cinética de terceira ordem, com constante cinética na
ordem de 3,50 x 10-4 L².mol-2.s-1. Sabendo que a espécie gasosa apresenta um decréscimo de concentração igual a
7,70 x 10-2 mol.L-1 para 2,1 x 10-2 mol.L-1, calcule o tempo necessário para esta diminuição.
1 1
Resolução: Cálculo do tempo (t), considerando a cinética de terceira ordem: − [A]2 =2xkxt
[A]2Final Inicial

1 1
− = 2 x (3,50 x 10−4 ) x t
(0,021)² (0,077)²
t = 2998444,56 segundos (≅ 34,70 dias)
Questão 894 – (GRILLO) As constantes de velocidade para a reação de segunda ordem de produção de iodo gasoso
e de cloreto de hidrogênio, 2 ICl(g) + H2(g) → I2(g) + 2 HCl(g), são 0,163 e 0,348 L.mol-1.s-1, a 503 K e 513 K,
respectivamente. Determine a energia de ativação (Eat) e o fator pré-exponencial (A) para este processo cinético.
Resolução: Dados do problema: k1 (503 K) = 0,163 L.mol-1.s-1 e k2 (513 K) = 0,348 L.mol-1.s-1.
k Eat 1 1
Cálculo da energia de ativação (Eat), a partir da equação de Arrhenius: ln ( 1 ) = x[ − ]
k2 R T T
2 1

0,163 Eat 1 1
ln ( )= x[ − ]
0,348 8,314 513 503
Eat = 1,63 x 105 kJ. mol−1
Eat
(− )
Determinação do fator pré-exponencial (A): k1 = A. e R.T

1,63 x 105
(− )
0,163 = A x e 8,314 x 503

0,163 = A x e(−38,91) → A = 1,29 x 1016 L. mol−1 . s −1


Questão 895 – (ITA) Computadores químicos são sistemas desenvolvidos para resolver diversos problemas de
ciência e engenharia, por meio de reações químicas. Considere dois exemplos de aplicação desses computadores:
I. Desenvolvimento de circuitos de controle molecular e procedimentos terapêuticos inteligentes utilizando um conjunto
de velocidades de reações químicas como linguagem de programação para controlar a síntese de DNA.

460
1600 FÍSICO-QUÍMICA APLICADA EXERCÍCIOS COMENTADOS - IME – ITA – OLIMPÍADA

II. Definição das melhores rotas de deslocamento entre dois pontos de um mapa, de forma mais rápida do que
qualquer sistema de navegação convencional. O computador utiliza um mapa preenchido com um líquido alcalino. O
ponto de partida contém partículas de corante e o ponto de destino contém um gel misturado com ácido.
Assinale a opção que apresenta os conceitos/processos que melhor descrevem o princípio de funcionamento dos
computadores químicos exemplificados em I e II, respectivamente.
a) Primeira lei da termodinâmica e solubilidade
b) Mecanismos reacionais e variação da tensão superficial
c) Entalpia de formação dos produtos e entalpia de neutralização
d) Princípio de Le Chatelier e Lei de Hess
e) Energia de Ativação e reação de Cannizzaro

Resolução: Alternativa B.
A afirmativa I apresenta o seguinte termo: “conjunto de velocidades de reações químicas”. Este fator reflete no
processo de mecanismo cinético reacional. Com relação a afirmativa II está presente o seguinte termo: “contém
partículas de corante e o ponto de destino contém gel misturado com ácido”. Essa afirmativa caracteriza o que
definimos de Tensão Superficial. Trata-se de um processo físico que ocorre diretamente na interface entre duas fases
químicas.
Questão 896 – (OLIMPÍADA BAIANA DE QUÍMICA) As figuras abaixo representam à reação A + B ⇄ Produtos.
Após analisá-las, use a teoria das colisões para responder:
a) Em qual dos três recipientes (a), (b) ou (c), a reação é mais rápida?
b) Em qual recipiente, (d) ou (e), a reação é mais rápida?

Resolução:

Item a) Segundo a Teoria das Colisões, para que uma reação ocorra, é necessário que partículas dos reagentes
entrem obrigatoriamente em contato umas com as outras, o que irá ocorrer no estudo de uma forma de energia em
consequência dos choques elásticos para a quebra das ligações nessas partículas. Outro requisito para que a colisão
seja efetiva, é que as partículas entre em colisão com uma determinada orientação adequada, de modo que os átomos
possam formar novas ligações. Logo, quanto maior a frequência das colisões, maior será o número de colisões por
unidade de tempo, e com isso, maior será a probabilidade de ocorrerem colisões efetivas, isto é, as partículas se
chocam com uma energia mínima necessária para que a reação ocorra. Quanto maior a quantidade de partículas em
um dado recipiente, maior será a frequência das colisões. Ao analisar os recipientes (a), (b) e (c), os recipientes (a) e
(b) apresentam o mesmo volume, menor que o recipiente (c), e o número de partículas de (b) é igual ao de (c) é maior
que o de (a), o que leva a concluir que em (b), o número de partículas por unidade de volume é maior. Desse modo,
em (b) as partículas terão maior frequência de colisão em comparação com os outros recipientes. Conclusão: a
velocidade do recipiente (b) será maior que os demais.

Item b) Sabendo que a quantidade de partículas por unidade de volume é igual em (d) e (e) e que a temperatura do
sistema é maior em (d) do que em (e), podemos afirmar que o processo ocorre mais rápido em (e). Este fato pode ser
explicado a partir da Teoria das Colisões.

Questão 897 – (OLIMPÍADA BAIANA DE QUÍMICA) Um modelo usado em cinética química é o da teoria das
colisões. Segundo esse modelo, para uma reação ocorrer, partículas dos reagentes ao colidirem umas com as outras,

461
1600 FÍSICO-QUÍMICA APLICADA EXERCÍCIOS COMENTADOS - IME – ITA – OLIMPÍADA

algumas ligações devem ser quebradas e outras devem ser formadas. Mas nem todas as colisões levam à formação
de produtos. A partir do entendimento dessa teoria, responda:
a) Quais são os dois requisitos para que os choques entre as partículas resultem em reações?
b) Por que ao reduzir o volume do recipiente no qual está ocorrendo uma reação na fase gasosa, a velocidade
da reação aumenta?

Resolução:
Item a) Os requisitos para que os choques sejam efetivos são os seguintes: Energia das colisões e orientação das
partículas.
Item b) Pois diminuindo o volume do recipiente, aumentam o número de colisões por unidade de área e também
aquelas entre as próprias partículas, isto é, aumenta a pressão, pois aumenta a frequência das colisões. Maior
frequência das colisões significa maior número de choques por unidade de tempo. Assim, com maior número de
choques por unidade de tempo, aumenta a probabilidade de acontecerem choques que resultem em reações, ou seja,
em quebra de ligações.

Questão 898 – (ITA) A equação química hipotética A → D ocorre por um mecanismo que envolve as três reações
unimoleculares abaixo (I, II e III). Nestas reações, ∆Hi representa as variações de entalpia, e Eai, as energias de
ativação.
I. A → B; Rápida, ∆HI , EaI
II. B → C; Lenta, ∆HII , EaII
III. C → D; Rápida, ∆HIII , EaIII
Trace a curva referente à energia potencial em função do caminho da reação A → D, admitindo que a reação global
A → D seja exotérmica e considerando que: ∆HII > ∆HI > 0; EaI < EaIII .

Resolução: Com relação às energias potenciais das substâncias, deve-se prestar atenção à variação de entalpia das
reações. Temos que a reação I e II são endotérmicas e portando caminho A→B→C apresenta energia potencial
crescente. Como a reação global A → D é exotérmica (libera energia), temos que D possui energia potencial menor
que A. Ainda vale identificar que HI é menor que HII (enunciado). Para identificar as magnitudes relativas das
energias de ativação, utiliza-se basicamente as informações do enunciado, com exceção da EaII. Pela etapa II ser a
etapa lenta, chega-se à conclusão que EaII será a maior energia de ativação. Logo, chega-se ao gráfico:

EP

EaII
EaIII

HII
EaI B

A HI
HIII

Caminho da reação

Questão 899 – (OLIMPÍADA PARAENSE DE QUÍMICA – MODIFICADA) Em geral, reação química não ocorre toda
vez que acontece uma colisão entre espécies potencialmente reativas. A reação ocorre quando as espécies reativas
possuem um mínimo de energia no momento da colisão. É uma barreira que as espécies que colidem devem suplantar
para produzir os produtos. A partir do texto apresentado, qual é esta energia e defina em bases cinéticas.

Resolução: Energia de Ativação. A energia de ativação é a mínima energia necessária para que as espécies químicas
dos reagentes devem possuir para que a reação ocorra. A energia de ativação é um parâmetro que foi estudado por
𝐸𝑎𝑡
(− )
Arrhenius, representado pela seguinte equação matemática apresentada a seguir: 𝑘 = 𝐴 𝑥 𝑒 𝑅.𝑇 , onde: k =

462
1600 FÍSICO-QUÍMICA APLICADA EXERCÍCIOS COMENTADOS - IME – ITA – OLIMPÍADA

constante cinética ou também conhecida como velocidade específica; A = fator pré-exponencial; Eat = energia de
ativação; R = constante dos gases ideais e T = temperatura absoluta.

Questão 900 – (ITA) Considere que na figura abaixo, o frasco A contém peróxido de hidrogênio, os frascos B e C
contêm água e que se observa borbulhamento de gás no frasco C. O frasco A é aberto para a adição de 1 grama de
dióxido de manganês e imediatamente fechado. Observa-se então, um aumento do fluxo de gás no frasco C. Após
um período de tempo, cessa o borbulhamento de gás no frasco C, observando-se que ainda resta sólido no frasco A.
Separando-se este sólido e secando-o, verifica-se que sua massa é igual a 1 grama.
a) Escreva a equação química que descreve a reação que ocorre com o peróxido de hidrogênio, na ausência
de dióxido de manganês.
b) Explique por que o fluxo de gás no frasco C aumenta quando da adição de dióxido de manganês ao peróxido
de hidrogênio.

Resolução:
Item a) Equação química da decomposição do peróxido de hidrogênio: H2O2(l) → H2O(l) + ½ O2(g)

Item b) Observa-se que no próprio enunciado, a quantidade de dióxido de manganês depositada inicialmente no
experimento é a mesma quantidade recuperada na fase final do processo. Além disso, a adição deste mesmo óxido
ocasiona um aumento na velocidade da reação química, o que leva a concluir que o referido óxido é um catalisador.

Questão 901 – (ITA) Um recipiente aberto, mantido à temperatura ambiente, contém uma substância A(s) que se
transforma em B(g) sem a presença de catalisador. Sabendo-se que a reação acontece segundo uma equação de
velocidade de ordem zero, responda com justificativas às seguintes perguntas:
a) Qual a expressão algébrica que pode ser utilizada para representar a velocidade da reação?
b) Quais os fatores que influenciam na velocidade da reação?
c) É possível determinar o tempo de meia-vida da reação sem conhecer a pressão de B(g)?

Resolução:

Item a) A expressão algébrica da velocidade é a seguinte: v = k.[A]0, ou seja, v = k. Para uma reação química com
cinética de ordem zero, a velocidade é diretamente dependente, diretamente proporcional a constante cinética e
independente da concentração do reagente.
Item b) Para este caso específico os fatores que influenciam na velocidade da reação são os seguintes: Energia de
ativação; Temperatura; Superfície de contato.
Item c) Sim. A equação do tempo de meia-vida para uma reação de ordem zero é dada pela seguinte equação
[A]
matemática: t 1⁄ = 2 x 0k.
2

Questão 902 – (GRILLO) Através da equação química hipotética apresentada a seguir: 4A(g) + 3B(g) → 2W(g), e
sabendo que a velocidade da reação seja igual a 1,5 mol.L-1.h-1, determine as velocidades de formação e de consumo
dos participantes gasosos da referida reação.

Resolução: Equação química hipotética: 4 A(g) + 3 B(g) → 2 W(g)

463
1600 FÍSICO-QUÍMICA APLICADA EXERCÍCIOS COMENTADOS - IME – ITA – OLIMPÍADA

Velocidade da reação = 1,5 mol.L-1.h-1

1 d[A] 1 d[B] 1 d[W]


Vreação = − x = − x = + x
4 dt 3 dt 2 dt
mol 1 d[A] 1 d[B] 1 d[W]
1,50 = − x = − x = + x
L. h 4 dt 3 dt 2 dt
mol 1 1 1
1,50 = − x VA = − x VB = + x VW
L. h 4 3 2
mol
VA = − 6,0
Lxh
mol
VB = − 4,50
Lxh
mol
VW = 3,00
Lxh
Questão 903 – (GRILLO) Sabendo que a velocidade de consumo de A é igual a 1,75 mol.L-1.s-1 na reação gasosa do
tipo 3A(g) + 2B(g) → W(g) + 5Z(g). A partir das informações apresentadas, determine as velocidades de formação, a
velocidade de consumo dos participantes gasosos e também da velocidade de reação.

Resolução: Equação química hipotética na fase gasosa: 3A(g) + 2B(g) → W(g) + 5Z(g)
1 d[A] 1 1 1
Vreação = − x = − x VB = + x VW = + x VZ
3 dt 2 1 5
1 mol 1 1 1
Vreação = − x (1,75 ) = − x VB = + x VW = + x VZ
3 L. s 2 1 5
mol
VB = − 1,17
Lxs
mol
Vreação = VW = − 0,583
Lxh
mol
VZ = + 2,92
Lxh
Questão 904 - (IME) Estudos cinéticos demonstram que a equação 4 A + B + C → 2 D + 2E ocorre em três etapas,
segundo o mecanismo a seguir:
Etapa 1: A + B + C → 2 F (lenta)
Etapa 2: 2 F + A → 2 G (rápida)
Etapa 3: G + A → D + E (rápida)
Os dados cinéticos de quatro experimentos conduzidos a mesma temperatura são apresentados na Tabela 1.
Tabela 1 - Dados cinéticos da reação em estudo.
Concentração inicial das espécies químicas (mol.L-1)
Experimento Velocidade inicial (mol.L-1.s-1)
A B C F G
1 90 9 3 3 2 2
2 60 9 2 3 2 1
3 120 9 3 4 1 1
4 3 3 3 3 0,5 0,5

464
1600 FÍSICO-QUÍMICA APLICADA EXERCÍCIOS COMENTADOS - IME – ITA – OLIMPÍADA

Determine:
a) A equação da velocidade da reação.
b) A ordem global da reação.
c) O valor da constante de velocidade.

Resolução:
3 𝑎 3
Item a) Determinação da ordem de reação (a) em relação ao reagente A entre os experimentos 1 e 4: (9) = 90
1 𝑎 1
( ) =
3 30

1 𝑎 1
𝑙𝑜𝑔 ( ) = 𝑙𝑜𝑔 ( )
3 30

𝑎 𝑥 (log 3)−1 = (log 30)−1

𝑎 ≅ 3,0

2 𝑏 60
Determinação da ordem de reação (b) em relação ao reagente B, entre os experimentos 1 e 2: (3) = 90

2 𝑏 2
( ) =
3 3

𝑏=1

4 𝑐 120
Determinação da ordem de reação (c) em relação ao reagente C, entre os experimentos 1 e 3: (3) = 90

4 𝑐 4
( ) =
3 3

𝑐=1

Equação da velocidade: v = k.[A]³.[B].[C]

Item b) Determinação da ordem global: β = a + b + c = 3 + 1 + 1 = 5

Item c) Determinação da constante cinética (k), a partir do experimento 3: v = k x [A]3 x [B]x [C]

mol mol 3 mol mol


120 = k x (9 ) x3 x4
L. s L L L

L4
k = 1,3
mol4 x s

Observação: Há uma contradição entre os dados fornecidos na Tabela 1 e as etapas da reação química (lenta e
rápida). A etapa lenta é a etapa dominante em um processo cinético. E, para esta etapa a reação seria de terceira
ordem (𝑣 = 𝑘. [𝐴]3 𝑥 [𝐵]𝑥 [𝐶]).
Já pela tabela a reação global é de quinta ordem (v = k.[A]³.[B].[C]). Para que o problema estivesse de acordo com a
teoria, tanto a etapa lenta como os dados fornecidos de forma experimental deveriam obter a mesma ordem de reação
global.

Questão 905 – (IME) Para a reação foram realizados três experimentos, conforme a tabela abaixo:

465
1600 FÍSICO-QUÍMICA APLICADA EXERCÍCIOS COMENTADOS - IME – ITA – OLIMPÍADA

Experimento [A] (mol.L-1) [B] (mol.L-1) Velocidade de reação (mol.L-1.s-1)


I 0,10 0,10 2,0 x 10-3
II 0,20 0,20 8,0 x 10-3
III 0,10 0,20 4,0 x 10-3

Determine:
a) a lei da velocidade da reação acima;
b) a constante de velocidade;
c) a velocidade de formação de C quando as concentrações de A e B forem ambas 0,50 mol.L-1.

0,10 𝑎 4,0 𝑥 10−3


Resolução: Cálculo da ordem de reação em relação ao reagente A, entre o experimento II e III: (0,20) = 8,0 𝑥 10−3

𝑎=1

0,20 𝑏 4,0 𝑥 10−3


Cálculo da ordem de reação em relação ao reagente B, entre o experimento I e III: (0,10) = 2,0 𝑥 10−3

𝑏=1

Item a) Equação da velocidade da reação química: v = k.[A].[B]

Item b) Cálculo da constante cinética (k), através do experimento I: 𝒗 = 𝒌 𝒙 [𝑨] 𝒙 [𝑩]


𝟐, 𝟎 𝒙 𝟏𝟎−𝟑 = 𝒌 𝒙 (𝟎, 𝟏𝟎) 𝒙 (𝟎, 𝟏𝟎)
𝑳
𝒌 = 𝟎, 𝟐𝟎
𝒎𝒐𝒍. 𝒎𝒊𝒏
d[C]
Item c) Cálculo da velocidade de C: Vreação = + dt
= k x [A] x [B]
d[C]
+ = 0,20 x 0,50 x 0,50
dt
d[C] mol
+ = 0,05
dt L. min

Questão 906 - (IME) A decomposição do aldeído acético ocorre segundo a reação: CH3CHO(g) → CH4(g) + CO(g). A
velocidade inicial da reação foi medida na mesma temperatura para duas concentrações do aldeído, fornecendo os
resultados abaixo:
[CH3CHO] Velocidade da reação
(mol.L-1) (mol.L-1.s-1)
0,10 0,020
0,20 0,081
Determine a constante de velocidade e a ordem dessa reação.

Resolução: Cálculo da ordem de reação em relação ao acetaldeído gasoso, entre o primeiro e segundo experimento:
𝟎,𝟐𝟎 𝒂 𝟎,𝟎𝟖𝟏
(𝟎,𝟏𝟎) = 𝟎,𝟎𝟐𝟎
2a = 4
a=2

Lei da velocidade da reação química: Vreação = k x [A]²

Cálculo da constante cinética (k), através do primeiro experimento: 𝑽𝒓𝒆𝒂çã𝒐 = 𝒌 𝒙 [𝑨]²


𝟎, 𝟎𝟐𝟎 = 𝒌 𝒙 (𝟎, 𝟏𝟎)²
𝑳
𝒌 = 𝟐, 𝟎
𝒎𝒐𝒍 𝒙 𝒔

466
1600 FÍSICO-QUÍMICA APLICADA EXERCÍCIOS COMENTADOS - IME – ITA – OLIMPÍADA

Questão 907 – (IME) A reação em fase gasosa: aA + bB → cC + dD foi estudada em diferentes condições, tendo
sido obtidos os seguintes resultados experimentais:
Concentração inicial (mol.L-1)
Velocidade inicial (mol.L-1.h-1)
[A] [B]
1 x 10-3 1 x 10-3 3 x 10-5
2 x 10 -3 1 x 10 -3 12 x 10-5
2 x 10 -3 2 x 10 -3 48 x 10-5
A partir dos dados acima, determine a constante de velocidade da reação.

Resolução: Cálculo da ordem de reação em relação ao reagente A, entre o primeiro e segundo experimento: 2ª = 4
a=2

Cálculo da ordem de reação em relação ao reagente B, entre o segundo e terceiro experimento: 2b = 4


b=2

Ordem de reação global da reação (a + b): a + b = 2 + 2 = 4

Lei da velocidade da reação química: 𝑽𝒓𝒆𝒂çã𝒐 = 𝒌 𝒙 [𝑨]² 𝒙 [𝑩]²

Cálculo da constante cinética (k), através do terceiro experimento: 𝑽𝒓𝒆𝒂çã𝒐 = 𝒌 𝒙 [𝑨]² 𝒙 [𝑩]²
𝟐
𝟒𝟖, 𝟎 𝒙 𝟏𝟎−𝟓 = 𝒌 𝒙 (𝟐, 𝟎 𝒙 𝟏𝟎−𝟑 ) 𝒙 (𝟐, 𝟎 𝒙 𝟏𝟎−𝟑 )²
𝟒𝟖, 𝟎 𝒙 𝟏𝟎−𝟓 𝑳³
𝒌= −𝟗
= 𝟑, 𝟎 𝒙 𝟏𝟎+𝟕
𝟏𝟔, 𝟎 𝒙 𝟏𝟎 𝒎𝒐𝒍³ . 𝒉

Questão 908 – (IME) Para a reação hipotética A + B → Produtos, tem-se os seguintes dados:
A (mol.L-1) B (mol.L-1) v (mol.L-1.h-1)
10,00 10,00 100,0
Considerando a mesma reação, verificou-se também a seguinte correlação:
A (mol.L-1) B (mol.L-1) v (mol. L-1.h-1)

10  

onde  e  são, respectivamente, as ordens da reação em relação a A e B. Sabendo que (α/β) = 10,0, determine:
a) a constante de velocidade k;
b) os valores numéricos das ordens parciais e global da reação.

Resolução: A equação da velocidade pode ser expressa da seguinte maneira: Vreação = k x [A]α x [B]β

Equação da velocidade para a primeira correlação: Vreação = k x [A]α x [B]β


𝟏𝟎𝟎 = 𝒌 𝒙 (𝟏𝟎)𝜶 𝒙 (𝟏𝟎)𝜷 (Equação A)
α
Sabendo que o problema apresenta a seguinte relação entre α e β é dado por β
= 10, então α = 10 x β.
Substituindo na equação A, temos: 100 = k x (10) 10β β
x (10)

100 = k x (10)10β + β

10² = k x (10)11β

10²
k=
(10)11β

467
1600 FÍSICO-QUÍMICA APLICADA EXERCÍCIOS COMENTADOS - IME – ITA – OLIMPÍADA

k = 102 − 11β (Equação I)

Através da equação da velocidade, temos: αβ x αα = k x (10α)α x (β)β

Sabendo que 𝜶 = 𝟏𝟎𝜷, temos: (𝟏𝟎𝜷)𝜷 𝒙 (𝟏𝟎𝜷)𝟏𝟎𝜷 = 𝒌 𝒙 (𝟏𝟎 𝒙 𝟏𝟎𝜷)𝟏𝟎𝜷 𝒙 (𝜷)𝜷

(𝟏𝟎𝜷)𝟏𝟏𝜷 = 𝒌 𝒙 (𝟏𝟎)𝟏𝟎𝜷 𝒙 (𝟏𝟎𝜷)𝟏𝟎𝜷 𝒙 (𝜷)𝜷

(𝟏𝟎𝜷)𝟏𝟏𝜷
(𝟏𝟎)𝟏𝟎𝜷 𝒙 (𝜷)𝜷 𝒙 𝒌 =
(𝟏𝟎𝜷)𝟏𝟎𝜷

(𝟏𝟎)𝟏𝟎𝜷 𝒙 (𝜷)𝜷 𝒙 𝒌 = (𝟏𝟎𝜷)𝜷

(𝟏𝟎)𝟏𝟎𝜷 𝒙 (𝜷)𝜷 𝒙 𝒌 = (𝟏𝟎)𝜷 𝒙 (𝜷)𝜷

(𝟏𝟎)𝟏𝟎𝜷 𝒙 (𝜷)𝜷 𝒙 𝒌 = (𝟏𝟎)𝜷 𝒙 (𝜷)𝜷

(𝟏𝟎)𝜷
𝒌 = (𝟏𝟎)𝟏𝟎𝜷 = (𝟏𝟎)−𝟗𝜷 (Equação B)

Igualando as equações A e B, temos: 𝟏𝟎𝟐 − 𝟏𝟏𝜷 = (𝟏𝟎)−𝟗𝜷

𝟐 − 𝟏𝟏𝜷 = −𝟗𝜷

𝟐
𝜷= =𝟏
𝟐
𝜶
Sendo 𝜷 = 𝟏𝟎, 𝒍𝒐𝒈𝒐 𝜶 = 𝟏𝟎.

A ordem global é o somatório das ordens parciais, α + β = 11.

L10
Item a) Cálculo da velocidade específica, a partir da equação B: k = (10)−9β = (10)−9 x 1 = 10−9 mol10 x h

Item b) As ordens parciais são as seguintes: α = 10 e β = 1.

Questão 909 – (IME) No estudo da cinética da reação 2 NO(g) + H2(g) → N2O(g) + H2O(g) ocorrendo à temperatura de
700°C, foram obtidos os dados constantes da tabela abaixo:

C (concentração inicial) (mol.L-1)


Vo (velocidade inicial)
(mol.L-1.s-1)
NO H2
0,025 0,01 2,4 x 10-6
0,025 0,005 1,2 x 10-6
0,0125 0,01 0,6 x 10-6
Pede-se:
a) a ordem global da reação;
b) a constante de velocidade a esta temperatura.

0,0125 a
Resolução: Item a) Cálculo da ordem de reação em relação ao reagente NO, entre o experimento I e III: (0,0250) =
0,6 x 10−6
2,4 x 10−6

468
1600 FÍSICO-QUÍMICA APLICADA EXERCÍCIOS COMENTADOS - IME – ITA – OLIMPÍADA

1 a 1
( ) =
2 4

a=2

0,005 a 1,2 x 10−6


Cálculo da ordem de reação em relação ao reagente H2, entre o experimento I e II: (0,010) = 2,4 x 10−6
b=1

Ordem global: a + b = 2 + 1 = 3.

Item b) Cálculo da constante cinética (k): Vreação = k x [NO]2 x [H2 ]1


𝟐, 𝟒 𝒙 𝟏𝟎−𝟔 = 𝒌 𝒙 (𝟎, 𝟎𝟐𝟓)𝟐 𝒙 (𝟎, 𝟎𝟏𝟎)𝟏
𝒌 = 𝟎, 𝟑𝟖𝟒 𝑳². 𝒎𝒐𝒍−𝟐 . 𝒔−𝟏

Questão 910 - (OLIMPÍADA BRASILEIRA DE QUÍMICA) Observa-se que uma reação de decomposição de primeira
ordem tem as seguintes constantes de velocidade nas temperaturas mencionadas:
T (°C) k (10-3 s-1)
0 2,46
20,0 25,1
40,0 576
Estime a energia de ativação.

Resolução: Refazendo a tabela fornecida pelo problema:

T (°C) T (K) k (s-1) 1/T Ln k


0 273 2,46 x 10-3 0,003663 - 6,01
20,0 293 25,1 x 10-3 0,003413 - 3,68
40,0 313 576 x 10-3 0,003195 - 0,55

Plotando um gráfico ln k versus 1/T, temos:

-1

-2

-3
Ln k

-4

-5

-6

0.0032 0.0033 0.0034 0.0035 0.0036 0.0037


1/T

− 𝐸𝑎𝑡 ∆𝑦
Cálculo da energia de ativação (Eat): Através do coeficiente angular da reta, temos: 𝑅
= ∆𝑥

− Eat − 0,55 − (−6,01)


=
8,314 0,003195 − 0,003663

469
1600 FÍSICO-QUÍMICA APLICADA EXERCÍCIOS COMENTADOS - IME – ITA – OLIMPÍADA

− Eat 5,46
=
8,314 −0,00468

J
Eat = 96996,97
mol

Questão 911 – (IME) Um primeiro estudo da cinética da reação SO2(g) + O3(g) → SO3(g) + O2(g) foi feito a 250 K,
fornecendo os dados da tabela abaixo:
[SO2] [O3] Taxa
(mol.L-1) (mol.L-1) (mol.L-1.s-1)
0,25 0,40 0,118
0,25 0,20 0,118
0,75 0,20 1,062
Um segundo estudo foi então realizado a 400 K, fornecendo:
[SO2] (mol.L-1) [O3] (mol.L-1) Taxa, (mol.L-1.s-1)
0,50 0,30 1,425
Com base nesses dados, estime a energia de ativação da referida reação.

Resolução: Cálculo da ordem de reação (a) em relação ao dióxido de enxofre entre o segundo e o terceiro
0,75 a 1,062
experimento: (0,25) = 0,118

𝑎=2

0,20 b 0,118
Cálculo da ordem de reação (b) em relação ao ozônio entre o primeiro e o segundo experimento: (0,40) = 0,118

b=0

Cálculo da constante cinética (k1) em relação ao primeiro estudo, através do primeiro experimento: Vreação =
k1 x [SO2 ]2 x [O3 ]0

𝟎, 𝟏𝟏𝟖 𝑳
𝒌𝟏 = = 𝟏, 𝟖𝟗
𝟎, 𝟐𝟓 𝒎𝒐𝒍. 𝒔

Cálculo da constante cinética (k2) em relação ao segundo estudo: Vreação = k 2 x [SO2 ]2 x [O3 ]0
𝟏, 𝟒𝟐𝟓 𝑳
𝒌𝟐 = = 𝟓, 𝟕𝟎
𝟎, 𝟓𝟎 𝒎𝒐𝒍. 𝒔
k Eat 1 1
Cálculo da energia de ativação, a partir da equação de Arrhenius: ln (k1 ) = R
x {T − T }
2 2 1

1,89 Eat 1 1
ln ( )= x{ − }
5,70 8,314 400 250

J
Eat = 6118,49
mol

Questão 912 – (IME) A decomposição térmica do SO2Cl2, gasoso a 320°C, segue uma cinética idêntica à
desintegração radioativa, formando SO2 e Cl2 gasosos, com uma constante de velocidade k = 2,20 x 10-5.s-1. Calcule
a percentagem de SO2Cl2 que se decompõe por aquecimento a 320°C, durante 4h 25min.

Resolução: Convertendo o tempo para segundos: Tempo = (4 x 3600)s + (25 x 60)s = 14400 s + 1500 s = 15900
[SO Cl ]
segundos. Cálculo da percentagem de cloreto de sulfurila decomposto:ln ([SO 2Cl 2] Final ) = −k x tempo
2 2 Inicial

470
1600 FÍSICO-QUÍMICA APLICADA EXERCÍCIOS COMENTADOS - IME – ITA – OLIMPÍADA

[SO2 Cl2 ]Final


ln ( ) = −(2,20 x 10−5 )x 15900
[SO2 Cl2 ]Inicial

[SO2 Cl2 ]Final


= e−0,350
[SO2 Cl2 ]Inicial

[SO2 Cl2 ]Final


= 0,7047 (70,47%)
[SO2 Cl2 ]Inicial

Questão 913 – (IME) Considere o decaimento radioativo do 24Na como um processo cinético de primeira ordem,
conforme mostrado no gráfico abaixo.

Para este radioisótopo, determine:


a) a constante de decaimento, k;
b) o tempo de meia-vida, em horas. Dados: ln 2 = 0,693; ln 3 = 1,099; ln 5 = 1,609.

[Na]
Resolução: Item a) Cálculo da constante de decaimento (k): ln ([Na] Final ) = −k x tempo
Inicial

12
ln ( ) = −k x 46
100

k = 4,61 x 10−2 s−1

Item b) Cálculo do tempo de meia-vida, considerando a cinética do processo como de primeira ordem.

ln2 0,693
t 1⁄ = = = 15,05 h
2 k 4,61 x 10−2

471
1600 FÍSICO-QUÍMICA APLICADA EXERCÍCIOS COMENTADOS - IME – ITA – OLIMPÍADA

Questão 914 – (IME – MODIFICADA) Em 1889, o químico sueco August Svante Arrhenius demonstrou que, para
uma reação com energia de ativação constante Eat, a variação da velocidade específica, k, com a temperatura é
expressa pela equação: k = A.e-Eat/R.T, onde: A é o fator de frequência, R é a constante universal dos gases, e é a base
dos logaritmos neperianos e T é a temperatura Kelvin. Certa reação obedece a uma lei velocidade onde os valores
de k são 0,00001 e 0,00010 L.mol–1.s–1, a 312,50 K e 357,14 K, respectivamente. Usando essas informações, calcule:
a) a ordem da reação;
b) a energia de ativação;
c) o fator de frequência;
d) a temperatura na qual a reação é 10 vezes mais lenta que a 312,50 K. Dado: R = 2,00 cal.K.–1.mol–1.

Resolução: Item a) Para uma unidade de constante cinética (k) igual a L.mol-1.s-1, a reação apresenta cinética de
segunda ordem.

Item b) Cálculo da energia de ativação, considerando os seguintes dados:


k1 (312,50 K) = 10-5 L.mol-1.s-1 e k1 (357,14 K) = 10-4 L.mol-1.s-1.

k Eat 1 1
Aplicando a equação de August Svante Arrhenius: ln ( 1 ) = x{ − }
k2 R T T2 1

10−5 Eat 1 1
ln ( −4 )= x{ − }
10 2,00 357,14 312,50

cal
Eat = 11513,57
mol
Eat
(− )
Item c) Cálculo do fator pré-exponencial (A), através de k1. k1 = A. e R.T

11513,57
(− )
10−5 = A. e 625,00

10−5 = A. e(−18,42)

10−5 10−5 L
A= = −8
= 999,32
e(−18,42) 1,00 x 10 mol. s

Item d) Cálculo da temperatura (T) em que a reação é dez vezes mais lenta em relação a 312,50 K:

k1 Eat 1 1
ln ( )= x{ − }
k2 R T T1
10−5 11513,57 1 1
ln ( −6 ) = x{ − }
10 2,00 T 312,50

T = 277,78 K

Questão 915 – (IME) A reação abaixo segue a mesma cinética do decaimento radioativo. A → 2B + ½C. Ao se
acompanhar analiticamente o desenvolvimento desta reação na temperatura T1, obtêm-se o Gráfico 1, o qual
estabelece uma relação entre a concentração molar da substância A no meio reacional e o tempo de reação.

472
1600 FÍSICO-QUÍMICA APLICADA EXERCÍCIOS COMENTADOS - IME – ITA – OLIMPÍADA

Ao se conduzir esta mesma reação em diversas temperaturas, obtêm-se diferentes valores para a constante de
velocidade de reação k, conforme os dados da Tabela 1.

Tabela 1 - Efeito da temperatura na constante de velocidade (k).


Temperatura (°C) 25 45 55 65
Constante de velocidade, k (s-1) 3,2 x 10-5 5,1 x 10-4 1,7 x 10-3 5,2 x 10-3
Finalmente, com um tratamento matemático dos dados da Tabela 1, pode-se construir o Gráfico 2, o qual fornece uma
relação entre a constante de velocidade e a temperatura. Com base nas informações fornecidas, considerando ainda
que ln 2 = 0,69 e que a constante universal dos gases é igual a 8,3 J.mol-1.K-1, determine:
a) a temperatura T1;
b) a energia de ativação, em kJ/mol, da reação.

Resolução: Item a) Cálculo da temperatura T1: Para a determinação da temperatura T1, será calculado a constante
cinética a partir do gráfico 1 e comparado com a tabela 1 fornecida pelo problema. A reação de decomposição obedece
uma cinética química de primeira ordem, uma vez que a constante cinética encontra-se na unidade (s-1), conforme
pode ser observado também na própria tabela.

[A]Final
ln ( ) = −k x tempo
[A]Inicial

0,010
No intervalo entre 0 e 400 segundos, temos: ln (0,020) = − 400 x k
1
ln ( ) = − 400 x k
2

k = 1,73 x 10−3 s−1

Observando a tabela 1, a temperatura correspondente é de 55°C.

Item b) Através de dois pontos do gráfico 2, temos:

473
1600 FÍSICO-QUÍMICA APLICADA EXERCÍCIOS COMENTADOS - IME – ITA – OLIMPÍADA

Eat 1 1
Utilizando a equação de Arrhenius: ln k1 − ln k 2 = x{ − }
R T T 1

Eat
− 5,25 − (−7,5) = x {0,0031 − 0,0029}
8,3

J
Eat = 9,33 x 104
mol

Questão 916 – (IME) A reação 3 ClO- → ClO3- + 2 Cl-, pode ser representada pelo seguinte diagrama de energia
potencial (EP) pela coordenada da reação:

Pede-se:
a) Propor um mecanismo para a reação, composto por reações elementares.
b) A expressão da velocidade de reação global. Justifique a resposta.

Resolução: Item a)
Mecanismo da reação:
Etapa lenta (por apresentar maior energia de ativação, Eat1): 3 ClO-(aq) → ClO2-(aq) + Cl-(aq) + ClO-(aq)
Etapa rápida (por apresentar menor energia de ativação, Eat2): ClO2-(aq) + Cl-(aq) + ClO-(aq) → ClO3-(aq) + 2 Cl-(aq)

Organizando as reações, temos:


2 ClO-(aq) → ClO2-(aq) + Cl-(aq)
ClO2-(aq) + ClO-(aq) → ClO3-(aq) + Cl-(aq)
Equação química global: 3 ClO-(aq) → ClO3-(aq) + 2 Cl-(aq)

Item b) Lei da velocidade: v = k x [ClO-]³

Questão 917 - (IME) À temperatura de 147°C, a decomposição do peróxido de diterbutila, em fase gasosa, obedece
à equação: (CH3)3COOC(CH3)3 → 2 CH3COCH3 + C2H6. O estudo cinético dessa reação pela medida da pressão total
da mistura em função do tempo, a volume constante, mostrou que a mesma é de 1ª ordem. Com base na tabela

474
1600 FÍSICO-QUÍMICA APLICADA EXERCÍCIOS COMENTADOS - IME – ITA – OLIMPÍADA

abaixo, calcular a velocidade média da reação em relação de di-t-butila, no intervalo de 0 a a10 minutos, em mol.L –
1.min –1 .

T(min) 0 6 10 14 22
P(mmHg) 180 200 210 220 240

Dado: Constante dos gases ideais (R) = 62,50 mm Hg.L.mol-1.K-1.

Resolução: Neste problema a tabela está em função da pressão total. Como a cinética estuda a velocidade de
decomposição em relação ao reagente, que neste caso é o peróxido de diterbutila. Com isso será necessário calcular
a pressão em relação a este reagente, ou seja, PA (pressão parcial do peróxido de diterbutila). Representando
(CH3)3COOC(CH3)3 por A, CH3COCH3 por B e C2H6 por C, a tabela de equilíbrio químico fica da seguinte maneira:

Equação química A → 2B C
Início P0 0 0
Reage P 2P P
Equilíbrio P0 - P 2P P

Cálculo da pressão total do sistema do sistema: Ptotal = Po – P + 2P + P, sendo Po = 180 mmHg:


Ptotal = 180 – P + 2P + P
Ptotal = 180 + 2P

Em t = 6 min, a pressão total é de 200 mmHg, logo a pressão do peróxido de diterbutila neste tempo será igual a: p o
+ 2p = 200  180 + 2p = 200 e p = 10  pA = po – p = 170 mmHg. Repetindo o raciocínio para os demais intervalos
de tempo, tem-se:

T(min) 0 6 10 14 22
po + 2p 180 200 210 220 240
2p 0 20 30 40 60
p(mmHg) 0 10 15 20 30

pA(mmHg) 180 170 165 160 150

(165 − 180)
n10 − n0
PV (p10 − p0 ) | | | − 15 |
RT RT 62,32 x (147 + 273) 26174,40
Vmédia = = = = =
t10 − t 0 t10 − t 0 t10 − t 0 10 − 0 10
−5
mol
= 5,73 x 10
L. min

Questão 918 – (IME) A reação do óxido nítrico com hidrogênio, a 827ºC, fornece nitrogênio e vapor d’água. Nestas
condições, foram obtidos os seguintes dados cinéticos:
Experiência PH2 (torr) PNO (torr) Velocidade inicial (torr.s –1)
1 400 152 0,28
2 400 300 1,08
3 400 359 1,55
4 300 400 1,44
5 289 400 1,39
6 205 400 0,98
7 147 400 0,70
Considerando a reação ocorrendo em um recipiente de 1,00 L, na temperatura dada e com uma pressão inicial de NO
de 100 torr, calcule o número de mol de H2 necessário para que a velocidade inicial seja de 3,75 x 10 –2 torr.s –1.

475
1600 FÍSICO-QUÍMICA APLICADA EXERCÍCIOS COMENTADOS - IME – ITA – OLIMPÍADA

Resolução: 2 NO(g) + 2 H2(g) → N2(g) + 2 H2O(v)

Equação da velocidade de reação: v = k.[NO]x.[H2]y

𝑉2 𝑘 𝑥 (300)𝑦
Cálculo da ordem de reação em relação ao óxido nítrico, entre o experimento 1 e 2: 𝑉1
= 𝑘 𝑥 (153)𝑦

1,08 (300)𝑦
=
0,28 (153)𝑦
3,86 = 1,96𝑦

ln(3,86) = 𝑦 . ln (1,96)

1,351
𝑦= = 2,0
0,673

Cálculo da ordem de reação em relação ao hidrogênio, entre o experimento 4 e 5:

𝑉4 𝑘 𝑥 (300)𝑥
=
𝑉5 𝑘 𝑥 (289)𝑥

(300)𝑥
1,038 =
(153)𝑥

1,038 = 1,035𝑥

ln(1,038) = 𝑥 . ln (1,035)

𝑥 = 1,0
1 2
Equação da velocidade: v = k x 𝑝𝐻 2
𝑥 𝑝𝑁𝑂

1 2
Cálculo da constante cinética, a partir do primeiro experimento: v = k x 𝑝𝐻 2
𝑥 𝑝𝑁𝑂

𝑣 0,28 𝑡𝑜𝑟𝑟
k= 1 2 = = 3,03 𝑥 10−8
𝑝𝐻 2
𝑥 𝑝𝑁𝑂 400 𝑥 (152)² 𝑠𝑒𝑔𝑢𝑛𝑑𝑜𝑠

1 2
Cálculo da pressão de hidrogênio ao valor desejado para a velocidade inicial: v = k x 𝑝𝐻 2
𝑥 𝑝𝑁𝑂

1
3,75 x 10−2 = 3,0 x 10−8 x 𝑝𝐻 2
𝑥 (100)²

3,75 x 10−2
𝑝𝐻2 = = 123,77 torr (0,163 atm)
3,0 x 10−8 𝑥 (100)²

Cálculo do número de mol de H2(g), considerando comportamento ideal:

𝑝𝑉 0,163 𝑥 1
𝑛= = = 6,61 𝑥 10−3 𝑚𝑜𝑙
𝑅𝑇 0,08206 𝑥 300

Questão 919 – (IME) A reação 2A(l) + 2B(l) → 3C(l), onde A, B e C representam substâncias puras foi realizada,
isotermicamente, em um béquer, repetidas vezes. As concentrações iniciais dos reagentes usados e as velocidades
iniciais de cada uma das reações realizadas são mostradas no quadro abaixo. Calcule a ordem parcial da referida
reação em relação a cada um dos reagentes.

476
1600 FÍSICO-QUÍMICA APLICADA EXERCÍCIOS COMENTADOS - IME – ITA – OLIMPÍADA

REAÇÃO CONCENTRAÇÃO INICIAL (MOL.L-1) VELOCIDADE INICIAL


N° (MOL.L-1.MIN-1)
A B C
1 4,000 0,5000 0 12,13
2 4,000 0,8000 0 17,67
3 4,000 2,0000 0 36,76
4 2,000 4,0000 0 27,86
5 0,800 4,0000 0 9,86
6 0,500 4,0000 0 5,65
DADOS
Valor Logaritmo neperiano Valor Logaritmo neperiano
12,13 2,50 17,67 2,87
36,76 3,60 27,86 3,33
9,86 2,29 5,65 1,73
4,0 1,39 2,0 0,69
0,5 - 0,69 0,8 - 0,22

0,500 a
Resolução: Cálculo da ordem de reação em relação ao reagente A, entre a reação de número 4 e 6: (2,000) =
5,65
27,86

0,500 a 5,65
Aplicando a função logarítmica natural, temos:𝑙𝑛 ( ) = ln ( )
2,000 27,86

0,500
𝑎 𝑥 𝑙𝑛 ( ) = ln(5,65) − ln(27,56)
2,000

0,500
𝑎 𝑥 𝑙𝑛 ( ) = ln(5,65) − ln(27,56)
2,000

𝑎 = 1,16

2,000 b 36,76
Cálculo da ordem de reação em relação ao reagente B, entre a reação de número 1 e 3: (0,500) = 12,13

2,000 b 36,76
Aplicando o logaritmo neperiano na equação acima, temos: 𝑙𝑛 (0,500) = ln (12,13)

2,000
𝑏 𝑥 𝑙𝑛 ( ) = ln(36,76) − ln(12,13)
0,500

1,10
𝑏= = 0,79
1,39

Ordem de reação em relação ao reagente C: c = 0. Logo, a lei da velocidade fica da seguinte forma: 𝑣𝑟𝑒𝑎çã𝑜 =
k x [𝐴]1,16 𝑥 [𝐵]0,79 𝑥 [𝐶]0

Questão 920 – (ITA) Velocidades iniciais (vi) de decomposição do peróxido de hidrogênio foram determinadas em
três experimentos (A, B e C), conduzidos na presença de I-(aq) sob as mesmas condições, mas com diferentes
concentrações iniciais de peróxido [H2O2], de acordo com os dados abaixo:
Experimento [H2O2] (mol.L-1) Vi (10-3 mol.L-1.s-1)
A 0,750 2,745
B 0,500 1,83

477
1600 FÍSICO-QUÍMICA APLICADA EXERCÍCIOS COMENTADOS - IME – ITA – OLIMPÍADA

C 0,250 0,915
Com base nestes dados, para a reação de decomposição do peróxido de hidrogênio:
a) Escreva a equação estequiométrica que representa a reação;
b) Indique a ordem desta reação;
c) Escreva a lei de velocidade da reação;
d) Determine o valor numérico da constante de velocidade;
e) Indique a função do I-(aq) na reação.

Resolução: Item a) Equação química: H2O2(l) → H2O(l) +1/2 O2(g)

Item b) Cálculo da ordem de reação do peróxido de hidrogênio: entre o experimento A e C, temos:

0,250 a 0,915
( ) =
0,750 2,745

1 a 1
( ) ≅
3 3
a ≅ 1 (reação de decomposição com cinética de primeira ordem)

Item c) Equação da velocidade: v = k x [H2 O2 ]1


Item d) Através do experimento A, temos: v = k x [H2 O2 ]1
mol mol
2,745 x 10−3 = k x 0,750
L. s L

k = 0,0036 s −1

Item e) Para este processo, o I- (iodeto) funciona como um catalisador para a referida reação de decomposição do
peróxido de hidrogênio.

Questão 921 – (ITA) A equação química que representa a reação de decomposição do gás N2O5 é: 2 N2O5(g) → 4
NO2(g) + O2(g). A variação da velocidade de decomposição do gás N2O5 é dada pela equação algébrica: v = k.[N2O5],
em que k é a constante de velocidade desta reação, e [N2O5] é a concentração, em mol/L do N2O5, em cada tempo.
A tabela a seguir fornece os valores de ln[N2O5] em função do tempo, sendo a temperatura mantida constante.
Tempo (segundos) ln [N2O5]
0 - 2,303
50 - 2,649
100 - 2,996
200 - 3,689
300 - 4,382
400 - 5,075
a) Determine o valor da constante de velocidade (k) desta reação de decomposição. Mostre os cálculos
realizados.
b) Determine o tempo de meia-vida do N2O5 no sistema reagente. Mostre os cálculos realizados.

[𝑵 𝑶 ]
Resolução: Item a) Cálculo da constante cinética ordem (k): 𝒍𝒏 ([𝑵 𝟐𝑶 𝟓] 𝑭𝒊𝒏𝒂𝒍 ) = −𝒌 𝒙 𝒕𝒆𝒎𝒑𝒐
𝟐 𝟓 𝑰𝒏𝒊𝒄𝒊𝒂𝒍

[N2 O5 ]Final − [N2 O5 ]Inicial = −k x tempo

− 5,075 − (−2,303) = −k x 400

k = 0,00693 s−1

𝒍𝒏 𝟐 𝒍𝒏 𝟐
Item b) Cálculo do tempo de meia-vida: 𝒕𝟏⁄ = 𝒌
= 𝟎,𝟎𝟎𝟔𝟗𝟑 = 𝟏𝟎𝟎 𝒔
𝟐

478
1600 FÍSICO-QUÍMICA APLICADA EXERCÍCIOS COMENTADOS - IME – ITA – OLIMPÍADA

Questão 922 – (IFRJ) A decomposição química de um composto A apresenta cinética de primeira ordem. Um estudo
sobre o decréscimo de concentração do composto versus o tempo é mostrado na seguinte tabela:
[A] (mol.L-1) Tempo (horas)
25,00 0
18,95 1
10,88 3
6,25 5
De acordo com os dados obtidos, calcule:
a) o tempo de meia-vida para o decaimento do composto;
b) o valor da constante de velocidade;
c) a concentração da substância A, decorridos 4 horas de reação;
d) o tempo necessário para a decomposição de 99% do composto.
𝒍𝒏 𝟐 𝒍𝒏 𝟐
Resolução: Item a) Cálculo do tempo de meia-vida para o composto A: 𝒕𝟏⁄ = 𝒌
= 𝟎,𝟐𝟐𝟕 = 𝟐, 𝟓𝟎 𝒉𝒐𝒓𝒂𝒔
𝟐
Item b) Cálculo da constante de velocidade (k):

6,25
ln ( ) = −k x 5
25,0
k = 0,277 h−1

[𝑨]𝑭𝒊𝒏𝒂𝒍
Item c) Cálculo da concentração de A, decorridos 4 horas: 𝒍𝒏 ( ) = −𝟎, 𝟐𝟕𝟕 𝒙 𝟒
𝟐𝟓,𝟎

𝒎𝒐𝒍
[𝑨]𝑭𝒊𝒏𝒂𝒍 = 𝟐𝟓 𝒙 𝒆− (𝟒 𝒙 𝟎,𝟐𝟕𝟕) = 𝟖, 𝟐𝟓
𝑳

Item d) Cálculo do tempo para uma decomposição de 99%: Decomposição de 99%, restando 1%, logo a relação entre
a concentração final e inicial ficará da seguinte maneira: [A]Final = 0,01 x [A]Inicial.

0,01 x [A]Inicial
ln ( ) = −0,277 x tempo
[A]Inicial

ln(0,01) = −0,277 x tempo

tempo = 16,62 h

Questão 923 – (IFRJ) A reação de decomposição do pentóxido de dinitrogênio (N2O5) possui uma constante de
velocidade a 25°C igual a 3,38 x 10-4.s-1. Considerando que a equação que representa a reação é: 2 N2O5(g) → 4
NO2(g) + O2(g).
a) Calcule o tempo de meia vida do N2O5.
b) Construa um gráfico qualitativo, do qual possa extrair o valor da constante.

ln 2 ln 2
Resolução: Item a) Cálculo do tempo de meia-vida para o composto A: 𝑡1⁄ = 𝑘
= 3,38 𝑥 10−4 = 2050,73 𝑠
2

Item b) Desenvolvendo a equação para a cinética de primeira ordem, temos:


ln[N2 O5 ]Final − ln[N2 O5 ]Inicial = −k x tempo

Chamando: ln[N2O5]Final de eixo y; ln[N2O5]Inicial como o coeficiente linear; t como o eixo x e k (constante cinética) será
o coeficiente angular. O gráfico qualitativo ficará da seguinte maneira:

479
1600 FÍSICO-QUÍMICA APLICADA EXERCÍCIOS COMENTADOS - IME – ITA – OLIMPÍADA

Questão 924 – (IFRJ) Esta reação, expressa na equação a seguir, foi realizada na temperatura de 20°C, com
concentração inicial de NO3(g), igual a 0,05 mol.L-1. Decorridos 60 minutos, foi verificado que a concentração de NO3
passou a ser de 0,0358 mol.L-1. NO3(g) → NO2(g) + ½ O2(g). Sabendo que a decomposição de NO3(g) segue uma cinética
de segunda ordem, determine o seguinte:
a) O valor da constante cinética;
b) A concentração de NO3(g) decorridos 145 minutos;
c) O tempo necessário para que reste 1% da concentração inicial do reagente.
1 1
Resolução: Item a) Cálculo da constante cinética para uma reação com cinética de segunda ordem: 0,0358 − 0,05 =
k x 60

L
k = 0,132
mol. min
1 1
Item b) Cálculo da concentração da quantidade de matéria após 145 minutos: [NO − 0,05 = 0,132 x 143
3 ]Final

mol
[NO3 ]Final = 2,55 x 10−2
L

Item c) Informação do problema: [NO3 ]Final = 0,01 x [NO3 ]Inicial

1 1
− = 0,132 x tempo
0,01 x [NO3 ]Final [NO3 ]Inicial

99
= 0,132 x tempo
[NO3 ]Final

99
= 0,132 x tempo
0,05

tempo = 15000 min

Questão 925 – (IFRJ) Dados cinéticos foram levantados a uma determinada temperatura para esta reação: C2H5I(g)
→ C2H4(g) + HI(g). Com base nesses dados, construíram-se os seguintes gráficos:

480
1600 FÍSICO-QUÍMICA APLICADA EXERCÍCIOS COMENTADOS - IME – ITA – OLIMPÍADA

O efeito da temperatura sobre essa mesma reação foi também estudado. A tabela a seguir mostra os resultados
obtidos. Para esta reação:

Temperatura (oC) 300 325 375 400


Tempo de meia vida (h) 86,69 13,85 0,54 0,13

a) Descreva a lei de velocidade e justifique sua resposta;


b) Calcule os parâmetros de Arrhenius;
c) Determine a constante de velocidade da reação e a temperatura em que os dados empregados na construção
dos gráficos foram obtidos.

Resolução: Item a) Através do gráfico que relaciona ln[C2H5I] versus o tempo (t) está linearizado, o que comprova
que a reação de decomposição apresenta um comportamento cinético de primeira ordem.

Item b) Cálculo dos parâmetros de Arrhenius.

ln 2
Cálculo da constante cinética (k) para a 300°C: k 300°C = = 0,0080 h−1
86,69

ln 2
Cálculo da constante cinética (k) para a temperatura igual a 400°C: k 400°C = 0,13
= 5,33 h−1

k300°C Eat 1 1
Cálculo da energia de ativação: ln (k400°C ) = R
. [(400+273) − (300+273)]

J
Eat = 298479,82
mol

Cálculo do fator do fator de frequência (A), a partir da equação de Arrhenius, a uma temperatura de 400oC: k 400°C =
Eat
(− )
A. e R.T

− 208479,82
{( )}
5,33 = A. e 8,314 x (400+273)
A = 8,06 x 1016 h−1
E
(− at )
Caso, o fator de frequência fosse calculado a uma temperatura de 300oC, temos: k 300°C = A x e RxT

− 208479,82
{( )}
0,0080 = A x e 8,314 x (300+273)

A = 8,10 x 1016 h−1

Conclusão: tanto faz o cálculo do fator de frequência pode ser realizado para qualquer temperatura dado pelo
problema.

481
1600 FÍSICO-QUÍMICA APLICADA EXERCÍCIOS COMENTADOS - IME – ITA – OLIMPÍADA

[C H I]
Item c) A partir do gráfico ln[C2H5I] versus o tempo, temos: ln [C 2H 5I] Final = − k x tempo
2 5 Inicial

ln[C2 H5 I]Final − ln[C2 H5 I]Inicial = −k x tempo

Para a equação y = - 0,9163 – 0,2704.x, a constante cinética é igual a k = 0,2704 h-1.

k E 1 1
Cálculo da temperatura para a constante cinética igual a k = 0,2704 h-1: ln (k300°C ) = 8,314
at
x [(300+273) − (T)]

0,2704 298479,82 1 1
ln ( )= x[ − ]
0,0080 8,314 573 T

T = 623,13 K

k Eat 1 1
Caso a temperatura fosse calculado em função de k400°C, temos: ln ( ) = x [( ) − ( )]
k400°C 8,314 400+273 T

0,2704 298479,82 1 1
ln ( )= x[ − ]
5,33 8,314 673 T

T = 623,13 K

Questão 926 – Considere uma reação hipotética em fase aquosa que apresenta cinética de segunda ordem, do tipo
A(aq) + B(aq) → P(aq). Este processo foi conduzido inicialmente em 0,050 mol.L-1 de Q e 0,080 mol.L-1 em B. Após o
decorrer da reação, a concentração da quantidade de matéria de A caiu para 0,020 mol.L -1 em um período de uma
hora. Determine a constante cinética em L.mol-1.s-1.

Resolução: Tabela de equilíbrio químico:

A(aq) (mol.L-1) B(aq) (mol.L-1) P(aq) (mol.L-1)


Início 0,050 0,080 0

Reage 0,030 = α 0,030 0,030
Equilíbrio 0,020 0,050 0,030

Sabendo que a equação matemática que expressa a cinética de segunda ordem para uma reação do tipo A(aq) + B(aq)
→ P(aq) em que as concentrações de ambos os reagentes são distintas, temos:

1 [A]0 x ([B]0 − α)
k x tempo = x ln [ ]
[B]0 − [A]0 ([A]0 − α) x [B]0

1 0,050 x (0,080 − 0,030)


kx1= x ln [ ]
0,080 − 0,050 (0,050 − 0,030) x 0,080

1 0,050 x (0,080 − 0,030)


kx1= x ln [ ]
0,030 (0,050 − 0,030) x 0,080

1 (0,050)² 0,00250
k= x ln [ ] = 33,33 x ln [ ] = 33,33 x ln(1,5625)
0,030 (0,020) x 0,080 0,0016
L 1h L
= 14,38 x( ) = 0,00413
mol x h 3600 s mol x s

Questão 927 – Considere a reação de segunda ordem do tipo A(aq) + 2B(aq) → P(aq). Este processo apresentou
inicialmente 0,075 mol.L-1 em A e 0,080 mol.L-1 em B. Após um período de aproximadamente uma hora, a

482
1600 FÍSICO-QUÍMICA APLICADA EXERCÍCIOS COMENTADOS - IME – ITA – OLIMPÍADA

concentração da quantidade de matéria de A foi igual a 0,045 mol.L-1. Através destas informações, determine a
constante de velocidade em L.mol-1.s-1.

Resolução: Tabela de equilíbrio químico:

A(aq) (mol.L-1) 2B(aq) (mol.L-1) P(aq) (mol.L-1)


Início 0,075 0,080 0

Reage 0,030 = α 2 x 0,030 = 0,060 0,030
Equilíbrio 0,045 0,020 0,030

Sabendo que a cinética de segunda ordem para uma reação do tipo A(aq) + B(aq) → P(aq) em que as concentrações dos
reagentes são distintas é dada pela equação matemática abaixo:

2α 1 [A]0 x ([B]0 − 2α)


k x tempo = + 2
x ln [ ]
(2 x [A]0 − [B]0 ) x ([B]0 − 2α) x [B]0 (2 x [A]0 − [B]0 ) ([A]0 − α) x [B]0

Cálculo da constante cinética:

2α 1 [A]0 x ([B]0 − 2α)


k x tempo = + x ln [ ]
(2 x [A]0 − [B]0 ) x ([B]0 − 2α) x [B]0 (2 x [A]0 − [B]0 )2 ([A]0 − α) x [B]0
2 x (0,030)
kx1=
(2 x 0,075 − 0,080) x (0,080 − 2 x 0,030) x 0,080
1 0,075 x (0,080 − 2 x 0,030)
+ 2
x ln [ ]
(2 x 0,075 − 0,080) (0,075 − 0,030) x 0,080

0,060 1 0,075 x (0,080 − 0,060)


k = + 2
x ln [ ]
(0,070) x (0,020) x 0,080 (0,070) (0,045) x 0,080

0,060 1 0,075 x 0,020


k= + 2
x ln [ ]
(0,070) x (0,020) x 0,080 (0,070) (0,045) x 0,080

3 1 0,075
k= + x ln [ ]
0,0056 0,0049 (0,045) x 4

L 1h L
k = 357,04 x = 0,099
mol x h 3600 s mol x s

Questão 928 – (ITA – MODIFICADA) A figura apresenta cinco curvas (I, II, III, IV e V) da concentração de uma espécie
X em função do tempo. Considerando uma reação química hipotética representada pela equação X(g) → Y(g),
determine qual a curva correspondente a uma reação química que obedece a uma lei de velocidade de segunda
ordem em relação à espécie X.

Resolução: Se a reação química obedece a uma lei cinética de segunda ordem em relação à espécie química X, a
equação da velocidade será expressa da seguinte maneira: v = k.[X]2. Desenvolvendo a equação cinética, temos: v =
d[X]
− dt

483
1600 FÍSICO-QUÍMICA APLICADA EXERCÍCIOS COMENTADOS - IME – ITA – OLIMPÍADA

d[X]
− = k x [X]²
dt

d[X]
+ k x [X]2 = 0
dt

Então, temos que a solução desta equação diferencial respeita uma equação da seguinte maneira:
d[X]
[X] = at-1. Verificando na equação: dt + k x [X]2 = 0

− at −2 + k x a²t −2 x [A]2 = 0

ak − 1 = 0

1
a=
k

Logo, para uma função do tipo [X] = (kt)-1, trata-se de uma hipérbole equilátera, que equivalente a curva V.

Questão 929 – Uma reação elementar do tipo W + Z → X foi realizada a uma temperatura ambiente com
concentrações iniciais iguais a dos dois reagentes, [W]0 e [Z]0, de 10-3 mol.L-1 e obteve-se uma constante cinética, k =
8 x 10-3 (unidade). Responda:
a) Qual a ordem da reação?
b) Qual a unidade da velocidade específica?
c) Calcule o tempo de meia-vida da reação.

Resolução:
Item a) Como o problema afirma que se trata de uma reação elementar, logo a equação da velocidade química é dada
pela seguinte equação:
vreação = k x [W]x [Z]

Ordem de reação em relação ao reagente W = 1


Ordem de reação em relação ao reagente Z = 1
Ordem de reação global: 1 + 1 = 2 (cinética de segunda ordem)

Item b) Considere que a velocidade da reação apresenta a seguinte unidade: mol.L-1.s-1 e a concentração da
quantidade de matéria dos reagentes como mol.L-1. Com isso, a velocidade específica apresenta a seguinte unidade:

vreação = k x [W]x [Z]

mol mol mol


=kx x
Lxs L L
L
k=
mol x s
1 1
Item c) Cálculo do tempo de meia-vida com cinética de segunda ordem: t 1⁄ = k x [W] = 0,008 x 0,001 =
2 0
125000 s (34,72 horas)

Questão 930 – (GRILLO) A energia de ativação para a reação de decomposição do iodeto de hidrogênio é de
aproximadamente 182 kJ.mol-1. A constante de velocidade a 700°C é 1,57x10-3 L.mol-1.s-1. Calcule:
a) a velocidade específica a 600°C;
b) o fator pré-exponencial (A) a 600°C.

Resolução: 2 HI(g) → H2(g) + I2(g); k1 (700°C/973 K) = 1,53 x 10-3 L.mol-1.s-1 e k2 (600°C/873 K) = ?.

484
1600 FÍSICO-QUÍMICA APLICADA EXERCÍCIOS COMENTADOS - IME – ITA – OLIMPÍADA

k Eat 1 1
Item a) Determinação da constante de velocidade, k2: ln (k1 ) = R
x [T − T ]
2 2 1

1,53 x 10−3 182000 1 1


ln ( )= x[ − ]
k2 8,314 873 973

1,53 x 10−3
ln ( ) = 21890,79 x (1,18 x 10−4 )
k2

1,53 x 10−3
( ) = e2,58
k2

L
k 2 = 1,16 x 10−4
mol x s
Eat
(− )
Item b) Determinação do fator pré-exponencial (A): k 2 = A x e R x T2

182000
(− )
1,16 x 10−4 = A x e 8,314 x 873

L
A = 8,99 x 106
mol x s

Questão 931 – (IME) O propeno pode ser obtido através da reação de isomerização do ciclopropano, conforme
apresentado na reação abaixo:

(g) (g)

O estudo teórico da cinética, considerando diferentes ordens para esta reação, fornece as seguintes equações:
[∆] = 0,100 – k x t, se a reação for de ordem zero;


ln ( ) = −k x t, se a reação for de primeira ordem; e
0,100

1 1

− 0,100 = k x t, se a reação for de segunda ordem, onde k é a constante de velocidade.

Seguindo este estudo, foram obtidos dados experimentais da concentração de ciclopropano [Δ] ao longo do tempo t,
apresentados nos gráficos abaixo em três formas diferentes. Considerando as informações mencionadas, determine
a expressão da velocidade de reação para a isomerização do ciclopropano.

0,1 -2,1
0,09 -2,3
0,08 -2,5
In []

0,07
[]

-2,7
0,06
0,05 -2,9
0,04 -3,1
0,03 -3,3
0 5 10 15 20 25 0 5 10 15 20 25
Tempo (mín) Tempo (mín)

485
1600 FÍSICO-QUÍMICA APLICADA EXERCÍCIOS COMENTADOS - IME – ITA – OLIMPÍADA

25
20
15
1/[]

10
5
0
0 5 10 15 20 25
Tempo (mín)

Resolução: Para que uma reação química seja de uma determinada ordem cinética, o gráfico deverá apresentar uma
característica linear (retilínea). Por isso, o único gráfico que apresenta esta característica trata-se do gráfico ln[Δ]
versus o tempo (t). Logo, a reação de isomerização é de primeira ordem.
𝑘
Questão 932 - (ITA) A reação química hipotética representada pela seguinte equação: 2 AB2C → ⏞ 2 AB2 + C2 foi
acompanhada experimentalmente, medindo-se as concentrações das espécies [AB2C], [AB2] e [C2] em função do
tempo. A partir destas informações experimentais, foram determinada a constante de velocidade (k) e a lei de
velocidade da reação. Com base nessa lei de velocidade, o mecanismo abaixo foi proposto e aceito:
Mecanismo:
k1
AB2C → AB2 + C (Etapa Lenta)
k2
AB2 + C → AB2 + C2 (Etapa Rápida)
Explique como foi possível determinar a constante de velocidade (k).

Resolução: Sabendo que a etapa lenta é a etapa determinante do processo cinético, temos: v = k1.[AB2C] (cinética
de primeira ordem).

d[AB2 C]
− = k x [AB2 C]
dt
[AB C] 1 t
Realização a integral da função acima, temos: ∫[AB 2C] Final [AB
d[AB2 C] = ∫0 − k x dT
2 Inicial 2 C]

[AB C]
ln[AB2 C]|[AB22 C]Final
Inicial
= −k x (t − 0)

ln[AB2 C]Final − ln[AB2 C]Inicial = −k x t

Conhecendo-se as concentrações de AB2C ao longo do tempo, pode-se determinar a constante k. O gráfico ln [AB2C]
x t ficaria:

ln [AB 2C]

ln [AB 2C] 0

k seria o coeficiente angular.

486
1600 FÍSICO-QUÍMICA APLICADA EXERCÍCIOS COMENTADOS - IME – ITA – OLIMPÍADA

Questão 933 - (ITA) A reação de combustão 2 SO2(g) + O2(g) → 2 SO3(g) é lenta e pode ser representada pela figura:
Energia (Kcal/mol)

Caminho da reação

Esta mesma reação pode ser catalisada pelo NO2(g) em duas etapas, sendo que a primeira é bem mais lenta que a
segunda. Numa mesma figura, esboce o perfil da curva da reação não catalisada e da reação catalisada pelo NO2(g).

Resolução: O perfil da curva da reação não-catalisada e da reação catalisada pelo dióxido de nitrogênio será dado
pela seguinte figura abaixo:

reação sem
energia

catalizador

caminho da reação

reação com
catalizador

Questão 934 - (IME) Para a reação A + B → C, foram constatados experimentalmente os seguintes perfis de
concentração molar (mol.L-1) versus tempo (h):

Observa-se ainda, experimentalmente, que a concentração do reagente B cai à metade do valor inicial em 195 h.
Determine:
a) A lei de velocidade;
b) O valor da constante de velocidade.

Resolução:

Item a) A lei de velocidade para este processo é de primeira ordem em relação ao B e ordem zero em relação ao
reagente A.
[B]Inicial

Item b) Cálculo da velocidade específica: ln ( [B] 2


) = − k x tempo
Inicial

1
ln ( ) = − k x 195
2

k = 3,55 x 10−3 h−1

487
1600 FÍSICO-QUÍMICA APLICADA EXERCÍCIOS COMENTADOS - IME – ITA – OLIMPÍADA

Questão 935 – (IME) Com base nos dados de variação da concentração com o tempo, obtidos em laboratório para a
reação 2 A → B + C foi levantado o gráfico a seguir. Valendo-se exclusivamente de métodos gráficos, pôde-se
determinar a velocidade específica como sendo k = 2.s-1.

a) Diga a ordem da reação;


b) Descreva o método utilizado.

Resolução: Item a) Observando a unidade da constante cinética ou também conhecida como constante específica
(k), o processo apresenta cinética de primeira ordem.

Item b)

O método utilizado para a determinação da constante cinética de primeira ordem (k), foi a relação entre a concentração
da quantidade de matéria do reagente em função do tempo, através da seguinte equação matemática:

[A]Final
ln = − k x tempo
[A]Inicial

A partir da quantidade inicial ([A]inicial) e da quantidade final ([A]Final), conforme pode ser visualizado na figura acima,
em um determinado intervalo de tempo (t), a constante cinética (k) poderá ser calculada.

Questão 936 – (EXAME NACIONAL DE CURSOS – ENGENHARIA QUÍMICA) Dados experimentais obtidos a partir
de uma reação A → B são apresentados na tabela abaixo:
Tempo (segundos) Concentração de A (mol.L-1)
100 0,50
200 0,25
300 0,20
Determine:
a) A ordem de reação.
b) O valor da constante de velocidade.
c) O tempo de meia-vida para uma concentração inicial de 1 mol.L-1.

Resolução: Cálculo da constante cinética entre 100 e 300 segundos: [A]Final − [A]Inicial = −k x t

0,25 − 0,50 = −k x 200

mol
k = 1,25 x 10−3
Lxs

488
1600 FÍSICO-QUÍMICA APLICADA EXERCÍCIOS COMENTADOS - IME – ITA – OLIMPÍADA

Cálculo da constante cinética entre 100 e 400 segundos: [A]Final − [A]Inicial = −k x t

0,20 − 0,50 = −k x 300

mol
k = 10−3
Lxs

Observando as constantes cinéticas, em diferentes intervalos, concluímos que não se trata de uma reação com
cinética de ordem zero.

Analisando a reação para uma cinética de primeira ordem. Ajustando a tabela para uma reação de primeira ordem,
temos:

Tempo (s) 100 200 300


Concentração de A (mol.L-1) 0,50 0,25 0,20
ln[A] - 0,693 - 1,386 - 1,609

Cálculo da constante cinética entre 100 e 300 segundos: 𝑙𝑛[𝐴]𝐹𝑖𝑛𝑎𝑙 − 𝑙𝑛[𝐴]𝐼𝑛𝑖𝑐𝑖𝑎𝑙 = − 𝑘 𝑥 𝑡𝑒𝑚𝑝𝑜

− 1,386 − (−0,693) = − k x 200

k = 3,46 x 10−3 s−1

Cálculo da constante cinética entre 300 e 400 segundos: 𝑙𝑛[𝐴]𝐹𝑖𝑛𝑎𝑙 − 𝑙𝑛[𝐴]𝐼𝑛𝑖𝑐𝑖𝑎𝑙 = − 𝑘 𝑥 𝑡𝑒𝑚𝑝𝑜

− 1,609 − (−1,386) = − k x 100

k = 2,23 x 10−3 s−1

Observando as constantes cinéticas, em diferentes intervalos, concluímos que não se trata de uma reação com
cinética de primeira ordem.

Analisando a reação para uma cinética de segunda ordem. Fazendo uma tabela para uma reação de primeira ordem,
temos:

Tempo (s) 100 200 300


Concentração de A (mol.L-1) 0,50 0,25 0,20
1/[A] 2 4 5

1 1
Cálculo da constante cinética entre 100 e 300 segundos: [A] − [A] =kxt
Final Inicial

4 − 2 = k x 200

L
k = 10−2
mol x s
1 1
Cálculo da constante cinética entre 300 e 400 segundos: [A] − [A] =kxt
Final Inicial

5 − 4 = k x 100

489
1600 FÍSICO-QUÍMICA APLICADA EXERCÍCIOS COMENTADOS - IME – ITA – OLIMPÍADA

L
k = 10−2
mol x s

Item a) Observando as constantes cinéticas, em diferentes intervalos, concluímos que a reação apresenta cinética de
segunda ordem.

Item b) Constante cinética (k) = 10-2 L x mol-1 x s-1


1 1
Item c) Cálculo do tempo de meia-vida para uma reação com cinética de segunda ordem: t 1⁄ = k x [A] = 10−2 x 1 =
2 0
100 s

Questão 937 – (OLIMPÍADA DE QUÍMICA DO RIO GRANDE DO SUL) 2 HgCl2 + C2O42- → 2 Cl- + 2 CO2(g) + Hg2Cl2(s)
foi estudada em solução aquosa, segundo o número de mol de cloreto mercuroso que precipita por litro de solução
por minuto. Os dados obtidos estão na tabela.

[HgCl2] (mol.L-1) [C2O42-] (mol.L-1) Velocidade (mol.L-1.min-1)


0,100 0,15 1,8 x 10-5
0,100 0,30 7,2 x 10-5
0,050 0,30 3,6 x 10-5

a) Determine a equação de velocidade de reação.


b) Calcule o valor da constante de velocidade da reação.
c) Qual será a velocidade da reação quando [HgCl2] = 0,010 mol.L-1 e [C2O42-] = 0,010 mol.L-1?

Resolução:
Item a) Cálculo da ordem de reação em relação ao reagente HgCl2, entre o segundo e o terceiro experimento:
𝟎,𝟎𝟓𝟎 𝒂 𝟑,𝟔𝟎 𝒙 𝟏𝟎−𝟓
( ) =
𝟎,𝟏𝟎𝟎 𝟕,𝟐𝟎 𝒙 𝟏𝟎−𝟓

1 a 1
( ) =
2 2

a=1

𝟎,𝟏𝟓 𝒃
Cálculo da ordem de reação em relação ao reagente C2O4-2, entre o primeiro e segundo experimento: (𝟎,𝟑𝟎) =
𝟏,𝟖𝟎 𝒙 𝟏𝟎−𝟓
𝟕,𝟐𝟎 𝒙 𝟏𝟎−𝟓

1 b 1
( ) =
2 4

b=2

𝟐
Lei da velocidade: 𝒗𝒓𝒆𝒂çã𝒐 = 𝒌 𝒙 [𝑯𝒈𝑪𝒍𝟐 ] 𝒙 [𝑪𝟐 𝑶−𝟐
𝟒 ]

Item b) Cálculo da constante cinética (k): Através do experimento I, temos: 𝟏, 𝟖𝟎 𝒙 𝟏𝟎−𝟓 =


𝒌 𝒙 (𝟎, 𝟏𝟎𝟎) 𝒙 (𝟎, 𝟏𝟓)𝟐
𝑳²
𝒌 = 𝟖, 𝟎 𝒙 𝟏𝟎−𝟑
𝒎𝒐𝒍² 𝒙 𝒎𝒊𝒏

Item c) Cálculo da velocidade da reação química: 𝒗𝒓𝒆𝒂çã𝒐 = 𝟖, 𝟎 𝒙 𝟏𝟎−𝟑 𝒙 𝟎, 𝟎𝟏𝟎 𝒙 (𝟎, 𝟎𝟏𝟎)𝟐 =
𝒎𝒐𝒍
𝟖, 𝟎 𝒙 𝟏𝟎−𝟗 𝑳 𝒙 𝒎𝒊𝒏

490
1600 FÍSICO-QUÍMICA APLICADA EXERCÍCIOS COMENTADOS - IME – ITA – OLIMPÍADA

Questão 938 – (OLIMPÍADA DE QUÍMICA DO RIO DE JANEIRO) Para a reação genérica abaixo, a 298 K: 3 X2Y +
WZ3 → Produtos. Foram obtidos os seguintes dados cinéticos:

Concentração (mol.L-1) Velocidade inicial


Experimento
[X2Y]0 [WZ3]0 (mol.L-1.s-1)
I 1,72 2,44 0,68
II 3,44 2,44 5,44
III 1,72 0,10 2,8 x 10-2
IV 2,91 1,33 ?

a) Em relação a cada reagente, determine a ordem da reação. Determine também, a ordem global da reação.
b) A partir das informações da tabela, determine a lei de velocidade para a reação.
c) Utilizando os dados, determine o valor da constante de velocidade para a reação genérica acima.
d) Utilizando os dados fornecidos, calcule a velocidade de reação para o experimento IV.
e) A velocidade de reação aumenta por um fator de 100 da presença de um catalisador, a 298 K. A energia de
ativação aumentará, diminuirá ou permanecerá a mesma? Justifique.

𝟑,𝟒𝟒 𝒂 𝟓,𝟒𝟒
Resolução: Item a) Cálculo da ordem de reação (a) em relação ao [X2Y]0, entre o experimento I e II: (𝟏,𝟕𝟐) = 𝟎,𝟔𝟖
(2)a = 8
a=3

𝟐,𝟒𝟒 𝒃 𝟎,𝟔𝟖
Cálculo da ordem de reação (b) em relação ao [WZ3]0, entre o experimento I e III: (𝟎,𝟏𝟎) = 𝟐,𝟖 𝒙 𝟏𝟎−𝟐
(24,4)b ≅ 24,4
b=1

Cálculo da ordem global (somatório das ordens parciais): a + b = 3 + 1 = 4

Item b) Equação da lei de velocidade: 𝒗𝒓𝒆𝒂çã𝒐 = 𝒌 𝒙 [𝑿𝟐 𝒀]𝟑 𝒙 [𝑾𝒁𝟑 ]

Item c) Cálculo da constante cinética (k) a partir do experimento IV: 𝒗𝑰𝑽 = 𝒌 𝒙 [𝑿𝟐 𝒀]𝟑 𝒙 [𝑾𝒁𝟑 ]

𝟎, 𝟔𝟖 = 𝒌 𝒙 (𝟏, 𝟕𝟐)³ 𝒙 𝟐, 𝟒𝟒

𝑳³
𝒌 = 𝟓, 𝟒𝟖 𝒙 𝟏𝟎−𝟐
𝒎𝒐𝒍³ 𝒙 𝒔

Item d) Determinação da velocidade no experimento IV: vIV = k x [X 2 Y]3 x [WZ3 ]

𝒗𝑰𝑽 = 𝟓, 𝟒𝟖 𝒙 𝟏𝟎−𝟐 𝒙 (𝟐, 𝟗𝟏)³ 𝒙 𝟏, 𝟑𝟑

𝒎𝒐𝒍
𝒗𝑰𝑽 = 𝟏, 𝟕𝟗
𝑳𝒙𝒔

Item e) Com a presença de um catalisador específico, a energia de ativação diminui.

491
1600 FÍSICO-QUÍMICA APLICADA EXERCÍCIOS COMENTADOS - IME – ITA – OLIMPÍADA

Questão 939 – (ITA) Considere a curva de variação de energia potencial das espécies A, B, C, D e E, envolvidas em
uma reação química genérica, em função do caminho da reação, apresentada na figura ao lado.

Suponha que a reação tenha sido acompanhada experimentalmente, medindo-se as concentrações de A, B e C em


função do tempo.
a) Proponha um mecanismo de reação para o processo descrito na figura, indicando a reação global.
b) Indique a etapa lenta do processo e escreva a lei de velocidade da reação.
c) Baseado na sua resposta no item b e conhecendo as concentrações de A, B e C em função do tempo,
explique como determinar a constante de velocidade desta reação.

Resolução: Item a)
A+B→C (Primeira etapa)
C→D+E (Segunda etapa) +
A + B → D + E (Reação global)

Item b) A etapa lenta é a etapa que apresenta a maior energia de ativação, conforme pode ser observado no seguinte
gráfico abaixo:

Etapa Lenta: C → D + E
Equação da velocidade: 𝑣𝑟𝑒𝑎çã𝑜 = k x [C]

Item c) Se for conhecida a [C] em função do tempo, temos que a velocidade da reação pode ser dada por: 𝑣𝑟𝑒𝑎çã𝑜 =
𝑑[𝐶] 𝑑[𝐶]
𝑑𝑡
, onde 𝑑𝑡
é a derivada da concentração em função do tempo. mas também sabemos que 𝑣𝑟𝑒𝑎çã𝑜 = k x [C],
d[C]
logo: k x [C] =
dt

492
1600 FÍSICO-QUÍMICA APLICADA EXERCÍCIOS COMENTADOS - IME – ITA – OLIMPÍADA

1 d[C]
k= x
[C] dt

Questão 940 – (OLIMPÍADA BRASILEIRA DE QUÍMICA) Uma das reações que ocorrem nos motores de carro e
sistemas de exaustão é: NO2(g) + CO(g) → NO(g) + CO2(g). Os dados experimentais para esta reação são os seguintes:
[NO2] inicial [CO] inicial Velocidade inicial
Experimento
(mol.dm-3) (mol.dm-3) (mol.dm-3.s-1)
1 0,10 0,10 0,0050
2 0,40 0,10 0,0800
3 0,10 0,20 0,0050
a) Escreva a equação da lei de velocidade desta reação, considerando o seguinte mecanismo para esta reação:
Etapa 1: NO2 + NO2 → NO3 + NO
Etapa 2: NO3 + CO → NO2 + CO2
b) Qual a etapa determinante da reação? Justifique.
c) Desenhe um diagrama de energia (energia versus caminho da reação) para esta reação.

Resolução:

Item a) Cálculo da ordem de reação em relação ao dióxido de nitrogênio, entre o experimento 1 e 2:

0,40 a 0,0800
( ) =
0,10 0,0050

(4)a = 16

a=2

Cálculo da ordem de reação em relação ao monóxido de carbono, entre o experimento 1 e 3:

0,20 b 0,0050
( ) =
0,10 0,0050

(2)b = 1

b=0

Equação da velocidade: 𝑣𝑟𝑒𝑎çã𝑜 = k x [𝑁𝑂2 ]2

Item b) A etapa 1 é a etapa determinante, uma vez que nesta etapa o catalisador encontra-se como reagente (NO2) e
o intermediário (NO3) encontra-se como produto.

Item c) Gráfico que representa a Energia versus caminho da reação (C.R.).

493
1600 FÍSICO-QUÍMICA APLICADA EXERCÍCIOS COMENTADOS - IME – ITA – OLIMPÍADA

Questão 941 – (OLIMPÍADA PORTUGUESA DE QUÍMICA) Considere o seguinte mecanismo proposto para a
decomposição do peróxido de hidrogênio:
H2O2(aq) + I-(aq) → H2O(l) + IO-(aq)
H2O2(aq) + IO-(aq) → H2O(l) + I-(aq) + O2(g)
a) Escreva a reação global.
b) Diga qual é o catalisador desta decomposição. Justifique.
c) Quais são os intermediários desta reação? Justifique.

Resolução:

Item a) Determinação da reação global:


H2O2(aq) + I-(aq) → H2O(l) + IO-(aq)
H2O2(aq) + IO-(aq) → H2O(l) + I-(aq) + O2(g) +
2H2O2(aq) → 2 H2O(l) + O2(g) (Reação global)

Item b e c) O catalisador para este determinado mecanismo trata-se do I-(aq). A identificação do catalisador consiste
em ser um reagente na primeira reação química e como produto na segunda reação química. Significa dizer que toda
quantidade de iodeto inserido na primeira reação, será consumido na segunda reação química. O único intermediário
presente neste mecanismo trata-se do IO-(aq). O intermediário “liga” uma etapa a outra, não aparecendo na reação
global. O intermediário consiste em ser um produto da primeira reação química e reagente na segunda reação química.

494
1600 FÍSICO-QUÍMICA APLICADA EXERCÍCIOS COMENTADOS - IME – ITA – OLIMPÍADA

Questão 942 – (EXAME NACIONAL DE CURSOS – ENGENHARIA QUÍMICA) Para a reação de decomposição do
ozônio 2 O3(g) → 3 O2(g) foi proposto um mecanismo em duas etapas, a primeira envolvendo a reação de equilíbrio O3
⇄ O2 + O e a segunda envolvendo a reação irreversível O + O3 → 2 O2.
a) Desenvolva a equação da velocidade de reação em termos de O2 para o mecanismo proposto.
b) Mostre que a cinética da reação é de segunda ordem em relação ao O3 e de ordem -1 em relação ao O2.

Resolução:

Item a) Analisando a primeira reação – reação reversível:

Expressão da constante de equilíbrio:


[𝑂2 ] 𝑥 [𝑂] 𝑘1
𝐾= = ′
[𝑂3 ] 𝑘1

𝐾 𝑥 [𝑂3 ]
Isolando o intermediário ([O]): [𝑂] = [𝑂2 ]

𝑘1 [𝑂3 ]
[𝑂] = 𝑥
𝑘1′ [𝑂2 ]
(Equação A)

Analisando a segunda reação – reação irreversível:

Expressão da velocidade da reação: 𝑣𝑟𝑒𝑎çã𝑜 = k x [O] 𝑥 [𝑂3 ] (Equação B)

Substituindo a equação A na equação B, temos: 𝑣𝑟𝑒𝑎çã𝑜 = k 2 x [O] 𝑥 [𝑂3 ]

k1 [O3 ]
vreação = k 2 x x x [O3 ]
k1′ [O2 ]

k1 [O3 ]2
vreação = k 2 x x
k1′ [O2 ]

1 𝑑[𝑂2 ]
Sabendo que a velocidade da reação em relação ao O2 é dada por 𝑣𝑟𝑒𝑎çã𝑜 = + 2 𝑥 𝑑𝑡
. Sabendo que 𝑣𝑟𝑒𝑎çã𝑜 =
𝑘1 [𝑂3 ]2
k2 x 𝑥 [𝑂 ] , temos:
𝑘1′ 2

1 d[O2 ] k1 [O3 ]2
+ x = k2 x ′ x
2 dt k1 [O2 ]

d[O2 ] 2 x k1 x k 2 x [O3 ]2
=
dt k1′ x [O2 ]

Item b) Observando a equação em relação a velocidade de formação do Oxigênio, observamos que


𝑑[𝑂2 ] 2 𝑥 𝑘1 𝑥 k2 𝑥 [𝑂3 ]2
𝑑𝑡
= 𝑘1′ 𝑥 [𝑂2 ]
.

A ordem de reação em relação ao [O3] é igual a dois e em relação ao [O2] é igual a -1.

Questão 943 – (GRILLO) A reação gasosa X(g) + Y(g) → Z(g) ocorre em duas etapas, de acordo com o seguinte
mecanismo:
2 X(g) ⇄ W(g)
k1
Y(g) + W(g) → X(g) + Z(g)

495
1600 FÍSICO-QUÍMICA APLICADA EXERCÍCIOS COMENTADOS - IME – ITA – OLIMPÍADA

A primeira etapa deste processo ocorre de forma rápida, atingindo o equilíbrio e a constante de equilíbrio é designada
pela letra K. Deduza uma expressão para a velocidade de formação em relação a Z, em termos de K, k 1 e das
concentrações de X e Y.

Resolução: Através da reação reversível, a constante de equilíbrio para esta reação gasosa é definido como: 𝐾 =
[𝑊]
[𝑋]2
(Equação 1)

Equação da velocidade, através da segunda reação química, gasosa e irreversível: 𝑣𝑟𝑒𝑎çã𝑜 = 𝑘1 𝑥 [𝑌] 𝑥 [𝑍]
(Equação 2)

𝑑[Z]
Relação da velocidade da reação com o produto gasoso Z(g): + 𝑑𝑡
= k1 x [𝑌] 𝑥 [𝑊]
(Equação 3)

[𝑊]
Isolando a concentração molar W, através da equação (1): 𝐾 = [𝑋]2

[𝑊] = 𝐾 𝑥 [𝑋]2 (Equação 4)

d[Z]
Substituindo a concentração da quantidade de matéria de W (equação 4) na equação 3: + dt
= k1 x [Y] x [W]

d[Z]
+ = k1 x [Y] x K x [X]2
dt

d[Z]
+ = K x k1 x [Y] x [X]2
dt

Questão 944 – (OLIMPÍADA MUNDIAL DE QUÍMICA) Uma determinada substância sofre decomposição segundo
uma cinética de primeira ordem, e sua dependência em relação à temperatura segue uma lei empírica chamada de
equação de Arrhenius. Os tempos de meia-vida determinados a 95ºC e 85ºC foram 15,4 minutos e 57,8 minutos,
respectivamente. A partir destes dados:
a) Calcule a energia de ativação e, supondo que esta permaneça constante, independente da temperatura,
estime o tempo de meia-vida a 25ºC.
b) Estime também a energia de ativação por meio de um gráfico do logaritmo natural da constante de
velocidade versus o inverso da temperatura (em Kelvin).

ln 2 ln 2
Resolução: Item a) Cálculo da constante cinética a T = 85°C: 𝑘85°𝐶 = 85°𝐶 = = 0,012 𝑚𝑖𝑛−1
𝑡1/2 57,8

ln 2 ln 2
Cálculo da constante cinética a T = 95°C: 𝑘95°𝐶 = 95°𝐶 = 15,4 = 0,045 𝑚𝑖𝑛−1
𝑡1/2

𝑘 𝐸𝑎𝑡 1 1
Cálculo da energia de ativação (Eat): 𝑙𝑛 (𝑘95°𝐶 ) = 𝑅
𝑥 [𝑇 −𝑇]
85°𝐶 2 1

0,045 𝑚𝑖𝑛−1 𝐸𝑎𝑡 1 1


𝑙𝑛 ( −1 )= 𝑥[ − ]
0,012 𝑚𝑖𝑛 8,314 (85 + 273) (95 + 273)

𝐸𝑎𝑡 1 1
𝑙𝑛(3,75) = 𝑥[ − ]
8,314 358 368

𝑙𝑛(3,75) 𝑥 8,314 1,32 𝑥 8,314 10,32 𝐽


𝐸𝑎𝑡 = = = = 144774,19
1 1 1 1 1 1 𝑚𝑜𝑙
[358 − 368] [358 − 368] [358 − 368]

496
1600 FÍSICO-QUÍMICA APLICADA EXERCÍCIOS COMENTADOS - IME – ITA – OLIMPÍADA

𝑘 𝐸𝑎𝑡 1 1
Cálculo da constante cinética a 25°C, a partir dos dados referentes a 95°C: 𝑙𝑛 (𝑘25°𝐶 ) = 𝑅
𝑥 [𝑇 −𝑇]
95°𝐶 2 1

𝑘25°𝐶 144774,19 1 1
𝑙𝑛 ( −1
)= 𝑥[ − ]
0,045 𝑚𝑖𝑛 8,314 (95 + 273) (25 + 273)

𝑘25°𝐶 144774,19 1 1
𝑙𝑛 ( −1
)= 𝑥[ − ]
0,045 𝑚𝑖𝑛 8,314 368 298

𝑘25°𝐶
( ) = 𝑒 (−11,11)
0,045 𝑚𝑖𝑛−1

𝑘25°𝐶 = 6,73 𝑥 10−7 𝑚𝑖𝑛−1

𝑘 𝐸𝑎𝑡 1 1
Caso a constante cinética a 25°C fosse calculada a partir de 85°C, temos: 𝑙𝑛 (𝑘25°𝐶 ) = 𝑅
𝑥 [𝑇 −𝑇]
85°𝐶 2 1

𝑘25°𝐶 144774,19 1 1
𝑙𝑛 ( −1
)= 𝑥[ − ]
0,012 𝑚𝑖𝑛 8,314 (85 + 273) (25 + 273)

𝑘25°𝐶 144774,19 1 1
𝑙𝑛 ( −1
)= 𝑥[ − ]
0,012 𝑚𝑖𝑛 8,314 358 298

𝑘25°𝐶
( ) = 𝑒 (−9,79)
0,012 𝑚𝑖𝑛−1

𝑘25°𝐶 = 6,72 𝑥 10−7 𝑚𝑖𝑛−1


25°𝐶 ln 2 ln 2
Cálculo do tempo de meia-vida a 25°C: 𝑡1/2 = = = 1,03 𝑥 106 𝑚𝑖𝑛𝑢𝑡𝑜𝑠
𝑘 6,72 𝑥 10−7

𝐸𝑎𝑡
(− )
Item b) Sabendo que a equação de Arrhenius é dada por: 𝑘 = 𝐴 𝑥 𝑒 𝑅.𝑇

𝐸𝑎𝑡
(− )
Aplicando a função logarítmica natural (ln), temos: 𝑙𝑛𝑘 = 𝑙𝑛𝐴 + 𝑙𝑛 [𝑒 𝑅.𝑇 ]

𝐸𝑎𝑡
𝑙𝑛𝑘 = 𝑙𝑛𝐴 −
𝑅. 𝑇
𝐸𝑎𝑡
Chamando ln k = y, ln A = a (coeficiente linear), 1/T = x e a razão (− 𝑇
) será o coeficiente angular (m). Logo, a
∆𝑦 𝐸𝑎𝑡
energia de ativação pode ser calculado a partir da seguinte relação matemática: 𝑚 = ∆𝑥 = − 𝑇
, conforme pode
ser visualizado a partir do gráfico abaixo.

497
1600 FÍSICO-QUÍMICA APLICADA EXERCÍCIOS COMENTADOS - IME – ITA – OLIMPÍADA

Questão 945 – (GRILLO) Escreva a expressão para a velocidade de aparecimento do composto D, considerando os
dois mecanismos apresentados a seguir, considerando que cada etapa descrita ocorre de acordo com uma cinética
de primeira ordem:
a) A⇄B
B+C→D
b) A + B ⇄ AB
AB + C → D

Resolução: Item a) Analisando a primeira equação química, através da expressão da constante de equilíbrio:
[𝐵]
𝐾𝑐 = (Equação I)
[𝐴]

𝑑[𝐷]
Analisando a equação de velocidade, através da segunda equação química: 𝑣𝑟𝑒𝑎çã𝑜 = 𝑑𝑡
= k x [B] 𝑥 [C]
(Equação II)

Isolando a concentração da quantidade de matéria do reagente B a partir da equação I, pois se refere do intermediário,
temos:

[𝐵] = 𝐾𝑐 𝑥 [𝐴] (Equação III)

𝑑[𝐷]
Substituindo a concentração da quantidade de matéria de B da equação III na equação II: 𝑣𝑟𝑒𝑎çã𝑜 = 𝑑𝑡
=
k x [B] 𝑥 [C]

d[D]
dt
= k x K c x [A] x [C]

[𝐴𝐵]
Item b) Analisando a primeira equação química através da expressão da constante de equilíbrio: 𝐾𝑐 = [𝐴] 𝑥 [𝐵]
(Equação I)

𝑑[𝐷]
Analisando a equação de velocidade, através da segunda equação química: 𝑣𝑟𝑒𝑎çã𝑜 = 𝑑𝑡
= k x [AB] 𝑥 [C]
(Equação II)

Isolando a concentração da quantidade de matéria [AB] a partir da equação I, uma vez que se trata do intermediário,
[𝐴𝐵]
temos: 𝐾𝑐 = [𝐴]𝑥 [𝐵]

[𝐴𝐵] = 𝐾𝑐 𝑥 [𝐴] 𝑥 [𝐵] (Equação III)

𝑑[𝐷]
Substituindo a concentração molar de AB da equação III na equação II: 𝑣𝑟𝑒𝑎çã𝑜 = 𝑑𝑡
= k x [AB] 𝑥 [C]

498
1600 FÍSICO-QUÍMICA APLICADA EXERCÍCIOS COMENTADOS - IME – ITA – OLIMPÍADA

d[D]
vreação = dt
= k x K c x [A] x [B] x [C]

Questão 946 – (GRILLO) Considere o seguinte mecanismo proposto para a reação a seguir:
AH + B ⇄ BH+ + A-
A- + AH → P
Encontre a lei de velocidade, expressando o valor da constante de velocidade em função das constantes de velocidade
das etapas elementares.

[BH+ ] x [A− ]
Resolução: Analisando a primeira equação química reversível: K c = [AH] x [B]
(Equação I)

Analisando a segunda equação química da velocidade:


vreação = k x [A− ] x [AH] (Equação II)

Isolando a concentração da quantidade de matéria do reagente [A-] a partir da equação I de equilíbrio , pois se refere
do intermediário:

[BH + ] x [A− ] = K c x [AH] x [B]

Kc x [AH] x [B]
[A− ] = (Equação III)
[BH+ ]

Substituindo a concentração da quantidade de matéria de [A-] da equação III na equação II: vreação =
k x [A− ] x [AH]

K c x [AH] x [B]
vreação = k x x [AH]
[BH + ]

k x K c x [AH]2 x [B]
vreação =
[BH + ]

Questão 947 – (ITA) Considere uma reação endotérmica entre reagentes, todos no estado gasoso.
a) Esboce graficamente como deve ser a variação da constante de velocidade em função da temperatura.
b) Conhecendo-se a função matemática que descreve a variação da constante de velocidade com a
temperatura é possível determinar a energia de ativação da reação. Explique como e justifique.
c) Descreva um método que pode ser utilizado para determinar a ordem da reação.
𝐸𝑎𝑡
(− )
Resolução: Item a) Desenvolvendo a equação de Arrhenius, temos: 𝑘 = 𝐴 𝑥 𝑒 𝑅.𝑇

Eat
(− )
Aplicando a função logarítmica natural (ln) na equação acima, temos: lnk = lnA + ln [e R.T ]

Eat
lnk = lnA −
R. T
𝐸𝑎𝑡 1
Chamando ln k = y, ln A = a, (− ) = b e = 𝑥, onde a é o coeficiente linear e b o coeficiente angular, o gráfico
𝑅 𝑇
de ln k versus 1/T está apresentado através da seguinte figura abaixo:

499
1600 FÍSICO-QUÍMICA APLICADA EXERCÍCIOS COMENTADOS - IME – ITA – OLIMPÍADA

lnR

1
T

Item b) Sim, é possível determinar a energia de ativação (Eat). Através do cálculo do coeficiente angular da reta (b) no
gráfico ln k x 1/T, temos a seguinte relação matemática:

𝐸𝑎𝑡 ∆𝑦
(− 𝑅
) = 𝑏 = tg 𝜃 = ∆𝑥, R é a constante dos gases e Eat é a energia de ativação.

Item c) Basta verificar como varia a concentração de um reagente em função do tempo. Com isso é possível construir
o gráfico [Reagente] versus o tempo (t), e através do comportamento da curva obtida determina-se a ordem da reação.

Questão 948 – (ITA) Considere duas reações químicas (I e II) envolvendo um reagente X. A primeira (I) é de primeira
ordem em relação a X, e o tempo de meia-vida é igual a 50 segundos. A segunda (II) é de segunda ordem em relação
a X e tem tempo de meia vida igual à metade da primeira reação. Considere que a concentração inicial de X nas duas
reações é igual a 1,00 mol.L-1. Em um gráfico de concentração de X (mol.L-1) versus tempo (de 0 a 200 s), em escala,
trace as curvas de consumo de X para as duas reações. Indique com I a curva que representa a reação de primeira
ordem e, com II, a que representa a reação de segunda ordem.
𝑙𝑛2
Resolução: Analisando a reação I, com cinética de primeira ordem: 𝑡1⁄ = 𝑘1
2

𝑙𝑛2 ln 2
𝑘1 = = 1,38 𝑥 10−2 𝑠 −1
𝑡1⁄ 50
2

1
Analisando a reação II, com cinética de segunda ordem: 𝑡1⁄ = 𝑘 𝑥 [𝑋]𝐼𝑛𝑖𝑐𝑖𝑎𝑙
2 2

1 1 𝐿
𝑘2 = = = 0,04
𝑡1⁄ 𝑥 [𝑋]𝐼𝑛𝑖𝑐𝑖𝑎𝑙 1,0 𝑥 25 𝑚𝑜𝑙 𝑥 𝑠
2

Traçando os gráficos da evolução temporal da [X], teremos o seguinte esboço:

0,9

0,8

0,7

0,6
[X] mol/l

0,5

0,4

0,3

0,2

0,1

0
0 20 40 60 80 100 120 140 160 180 200
Tempo (s)

Reação I - Ordem 1 Reação II - Ordem 2

Questão 949 – (ITA) A figura apresenta a variação de velocidade em função do tempo para a reação química
hipotética não catalisada representada pela equação A2 + B2 → 2 AB. Reproduza esta figura no caderno de soluções,

500
1600 FÍSICO-QUÍMICA APLICADA EXERCÍCIOS COMENTADOS - IME – ITA – OLIMPÍADA

incluindo no mesmo gráfico, além das curvas da reação catalisada, as da reação não catalisada, explicitando ambas
as condições.

Resolução: Como o catalisador atua tanto na reação direta quanto na reação inversa, o valor absoluto da velocidade
da reação no equilíbrio não se altera. O efeito do catalisador é o de antecipar o equilíbrio. Logo, o gráfico que
representa a reação catalisada e a não catalisada está apresentada abaixo.

Questão 950 – (ITA) Considere uma reação genérica reversível A + B ⇄ 2 C e os dados cinéticos para a reação
direta (D) e inversa (I):

Sentido da reação Constante de velocidade Energia de ativação


A + B → 2C kD Ea,D
2C → A + B K1 = (3/2).kD Ea,I = (1/2).Ea,D

a) Desenhe o gráfico de energia potencial versus coordenada da reação direta.


b) Determine o valor numérico da constante de equilíbrio da reação.
c) Qual o sentido da reação?

Resolução: Item a) A seguir está apresentado o gráfico da energia potencial versus o caminho da reação (C.R.).

501
1600 FÍSICO-QUÍMICA APLICADA EXERCÍCIOS COMENTADOS - IME – ITA – OLIMPÍADA

[C]2 k kdireta 2
Item b) Expressão da constante de equilíbrio da reação: K c = [A]x[B] = k direta = 3 =3
inversa x kdireta
2

Item c) Observando o diagrama de energia, a energia de ativação da reação direta é maior em comparação ao
processo inverso. A reação química caracteriza-se como
um processo endotérmico, uma vez que o patamar da energia dos produtos (Hprodutos) ser maior em comparação ao
reagentes (Hreagentes), Hprodutos > Hreagentes.

Questão 951 – (EXAME NACIONAL DE CURSOS – ENGENHARIA QUÍMICA) Calcule a constante de equilíbrio a
350°C para a reação H2(g) + I2(g) ⇄ 2 HI(g), a partir dos dados abaixo.
Reação Eat (kJ.mol-1) A (L.mol-1.s-1)
Sentido 1 165,1 1,6 x 1011
Sentido 2 186,0 1,2 x 1010
Eat é a energia de ativação e A é o fator de frequência ou pré-exponencial.

k1
Resolução: Cálculo da constante cinética para a reação direta, ou seja, H2(g) + I2(g) → 2HI(g)

Eat
(− )
k1 = A1 x e R.T

165100 L
(− )
k1 = (1,6 x 1011 ) x e 8,314 x (350+273) = (1,6 x 1011 ) x e(−31,87) = 2,30 x 10−3
mol x s
𝑘2
Cálculo da constante cinética para a reação inversa, ou seja, 2𝐻𝐼(𝑔) → 𝐻2(𝑔) + 𝐼2(𝑔)

186000 L
(− )
k 2 = (1,20 x 1010 ) x e 8,314 x (350+273) = (1,20 x 1010 ) x e(−35,91) = 3,04 x 10−6
mol x s
𝐿
𝑘 2,30 𝑥 10−3
Cálculo da constante de equilíbrio (K): 𝐾 = 𝑘1 = 𝑚𝑜𝑙 𝑥 𝑠
𝐿 = 756,58
2 3,04 𝑥 10−6
𝑚𝑜𝑙 𝑥 𝑠

Questão 952 – (ITA) Dado o seguinte mecanismo reacional, constituído de duas etapas elementares (I e II).

502
1600 FÍSICO-QUÍMICA APLICADA EXERCÍCIOS COMENTADOS - IME – ITA – OLIMPÍADA

Escreva a expressão para a taxa de variação temporal da concentração do:


a) reagente A.
b) intermediário M.
c) produto C.

Resolução:

𝑑[𝐴]
Item a) Determinação da taxa temporal em relação a A: 𝑑𝑡
= − 𝑘1 𝑥 [𝐴] + 𝑘−1 𝑥 [𝑀] − 𝑘2 𝑥 [𝑀] 𝑥 [𝐴]

𝑑[𝑀]
Item b) Determinação da taxa temporal em relação ao intermediário M: 𝑑𝑡
= + 𝑘1 𝑥 [𝐴] − 𝑘−1 𝑥 [𝑀] −
𝑘2 𝑥 [𝑀] 𝑥 [𝐴]

𝑑[𝐶]
Item c) Determinação da taxa temporal em relação a C: 𝑑𝑡
= + 𝑘2 𝑥 [𝑀] 𝑥 [𝐴]

Questão 953 – (GRILLO) Considere a seguinte reação de formação do dióxido de nitrogênio, representado pela
seguinte equação química, 2 NO + O2 → 2 NO2. Sabendo que o mecanismo proposto está representado da seguinte
maneira:
k1
NO + NO → N2O2
k2
N2O2 → 2 NO
k3
N2O2 + O2 → 2 NO2
Deduza uma equação da taxa de velocidade em relação a formação de NO2, utilizando a aproximação do estado
estacionário em relação à espécie estacionária.
d[N2 O2 ]
Resolução: Determinação da taxa temporal em relação ao composto intermediário (N2O2): dt
=
+ k1 x [NO]x [NO] − k 2 x [N2 O2 ] − k 3 x [N2 O2 ] x [O2 ]

d[N2 O2 ]
dt
= + k1 x [NO]2 − k 2 x [N2 O2 ] − k 3 x [N2 O2 ] x [O2 ]

d[N2 O2 ]
Como a concentração do intermediário, [N2O2], é praticamente constante ao longo do processo, temos: dt
= 0
, logo:
d[N2 O2 ]
dt
= + k1 x [NO]2 − k 2 x [N2 O2 ] − k 3 x [N2 O2 ] x [O2 ]

0 = + k1 x [NO]2 − k 2 x [N2 O2 ] − k 3 x [N2 O2 ] x [O2 ]

k1 x [NO]2 = k 2 x [N2 O2 ] + k 3 x [N2 O2 ] x [O2 ]

k1 x [NO]2 = [N2 O2 ] x {k 2 + k 3 x [O2 ]}

k1 x [NO]2
[N2 O2 ] =
k 2 + k 3 x [O2 ]

Através da equação química balanceada N2O2 + O2 → 2 NO2, a taxa temporal em relação ao dióxido de nitrogênio
1 d[NO ]
será igual a: 2 x dt 2 = + k 3 x [N2 O2 ] x [O2 ]

d[NO2 ]
dt
= 2 x k 3 x [N2 O2 ] x [O2 ]

d[NO2 ] k1 x [NO]2
dt
= 2 x k3 x k2 + k3 x [O2 ]
x [O2 ]

503
1600 FÍSICO-QUÍMICA APLICADA EXERCÍCIOS COMENTADOS - IME – ITA – OLIMPÍADA

d[NO2 ] 2 x k3 x k1 x [NO]2 x [O2 ]


dt
= k2 + k3 x [O2 ]

Questão 954 – (ITA) Os diagramas seguintes, traçados numa mesma escala, referem-se, respectivamente, aos
equilíbrios, em fase gasosa e numa mesma temperatura, representados pelas seguintes equações químicas:
I. AB + CD ⇄ AD + CB K1
II. AX + CY ⇄ AY + CX K2

Comparando as informações apresentadas nos dois diagramas, pedem-se:


a) Qual das constantes de equilíbrio, K1 ou K2 terá maior valor? Justifique sua resposta. Dado eventualmente
necessário: A relação entre a energia da Energia Livre de Gibbs padrão (ΔG°) e a constante de equilíbrio (K) de uma
reação é dada por ΔG° = - R x T x lnK.
b) Para as seguintes misturas numa mesma temperatura:

Mistura I Mistura II
[AB]Inicial = 0,10 mol.L-1 [AD]Inicial = ZERO [AX]Inicial = 0,10 mol.L-1 [CY]Inicial = 0,20 mol.L-1
[CD]Inicial = 0,20 mol.L-1 [CD]Inicial = ZERO [AY]Inicial = ZERO [CX]Inicial = ZERO

Qual das reações químicas, expressa pela equação I e II, atinge o equilíbrio mais rapidamente? Justifique sua
resposta.

Resolução:

Item a) Segundo os diagramas, a reação I é mais exotérmica do que a reação II. Da equação de van’t Hoff k =
∆H
(− )
Axe R.T , sabe-se que:

✓ K: constante de equilíbrio;
✓ A: fator pré-exponencial ou fator de frequência;
✓ ΔH: variação de entalpia;
✓ R: constante dos gases;
✓ T: temperatura na escala kelvin.

A partir disto, deduz-se que quanto mais exotérmica for a reação, tanto maior será a constante de equilíbrio. Logo, KI
> KII.

Item b) Nas duas reações, as concentrações iniciais de reagente são iguais. Segundo a equação de Arrhenius, k =
E
(− at )
A x e R.T , o sistema que possuir maior energia de ativação terá k com o menor valor. A mistura 2 é a que tem
menor Ea, e portanto maior k e maior velocidade.

504
1600 FÍSICO-QUÍMICA APLICADA EXERCÍCIOS COMENTADOS - IME – ITA – OLIMPÍADA

Questão 955 – (U.S. NATIONAL CHEMISTRY OLYMPIAD) O seguinte mecanismo foi proposto para a reação:

Usando a aproximação de estado estacionário, derivar a lei de taxa prevista por este mecanismo.

𝑑[𝑂]
Resolução: Analisando o intermediário, através das duas reações: 𝑑𝑡
= + 𝑘1 𝑥 [𝑂3 ] − 𝑘−1 𝑥 [𝑂2 ] 𝑥 [𝑂] −
𝑘2 𝑥 [𝑂] 𝑥 [𝑂3 ]

𝑑[𝑂]
Como a concentração da espécie intermediária é praticamente constante, temos: 𝑑𝑡
=0

d[O]
. dt
= + k1 x [O3 ] − k −1 x [O2 ] x [O] − k 2 x [O] x [O3 ]

0 = + k1 x [O3 ] − k −1 x [O2 ] x [O] − k 2 x [O] x [O3 ]

k −1 x [O2 ] x [O] + k 2 x [O] x [O3 ] = + k1 x [O3 ]

[O] x {k −1 x [O2 ] + k 2 x [O3 ]} = + k1 x [O3 ]

k1 x [O3 ]
[O] =
k −1 x [O2 ] + k 2 x [O3 ]

Equação da taxa de reação: 𝑣𝑟𝑒𝑎çã𝑜 = k 2 x [𝑂] 𝑥 [𝑂3 ]

Substituindo a concentração do intermediário, [O], na equação de taxa, temos: 𝑣𝑟𝑒𝑎çã𝑜 = k 2 x [𝑂] 𝑥 [𝑂3 ]

k1 x [O3 ]
vreação = k 2 x x [O3 ]
k −1 x [O2 ] + k 2 x [O3 ]

k 2 x k1 x [O3 ]2
vreação =
k −1 x [O2 ] + k 2 x [O3 ]

Questão 956 – (ITA) Qual foi a contribuição de Arrhenius para o entendimento da cinética das reações químicas?

Resolução: Svante August Arrhenius, químico, físico e matemático sueco, além de ter estudado a teoria dos ácidos
e das bases, como até hoje é muito estudado em todas as universidades e centros de pesquisas, também desenvolveu
outros trabalhos na área da Físico-Química, mas especificamente na área da velocidade das reações químicas
(cinética química). Estudou a dependência da constante cinética ou também conhecido como velocidade específica
(k) com a temperatura. A equação matemática que relaciona a constante cinética com a temperatura é a seguinte:

dln k Eat
=
dt R x T²

Onde: Eat = Energia de ativação; R = constante dos gases ideais; T = temperatura absoluta; k = constante cinética ou
também conhecida como velocidade específica.

505
1600 FÍSICO-QUÍMICA APLICADA EXERCÍCIOS COMENTADOS - IME – ITA – OLIMPÍADA

A equação de Arrhenius é útil pelo fato de expressar a relação quantitativa entre a temperatura, a energia de ativação
e a constante de velocidade. Uma de suas principais aplicações é a determinação da energia de ativação de uma
reação química, a partir de dados cinéticos experimentais a diferentes temperaturas.
Questão 957 – (GRILLO) Considere o seguinte mecanismo proposto para a reação a seguir:
𝑘𝑎
2A+2B→ C+D
2A⇄B+C
Encontre a lei de velocidade, expressando o valor da constante de velocidade em função das constantes de
velocidades das etapas elementares.
[B] x [C]
Resolução: Expressão da constante de equilíbrio: K = [A]2
(Equação 1)

Analisando a reação irreversível (equação da taxa):

Expressão da velocidade: vreação = k a x [A]2 x [B]2 (Equação 2)

[B] x [C]
Isolando a concentração do intermediário, [B], na equação 1: K = [A]2

K x [A]2
[B] =
[C]
(Equação 3)

Substituindo a equação o intermediário da equação 3 na equação 2, temos: vreação = k a x [A]2 x [B]2


2
K x [A]2
vreação = k a x [A]2 x{ }
[C]

K 2 x [A]4
vreação = k a x [A]2 x
[C]2

k a x K 2 x [A]6
vreação =
[C]2

Questão 958 – (GRILLO) Considere o seguinte mecanismo proposto para a reação a seguir:
A+B⇄C+I
k1
I+A→ ⏞ Produtos
Encontre a lei de velocidade, expressando o valor da constante de velocidade em função das constantes de
velocidades das etapas elementares.

[C]x [I]
Resolução: Expressão da constante de equilíbrio: K = [A]x [B] (Equação 1)

Expressão da velocidade: vreação = k1 x [I] x [A] (Equação 2)

[C]x [I]
Isolando a concentração da quantidade de matéria do intermediário na equação 1: K = [A]x [B]

K x [A]x [B] = [C] x [I]

K x [A]x [B]
[I] =
[C]
(Equação 3)

Substituindo a equação 3 na equação 2, temos: vreação = k1 x [I] x [A]


K x [A]x [B]
vreação = k1 x x [A]
[C]

506
1600 FÍSICO-QUÍMICA APLICADA EXERCÍCIOS COMENTADOS - IME – ITA – OLIMPÍADA

k1 x K x [A]2 x [B]
vreação =
[C]

Questão 959 – (GRILLO) Deduza a equação da velocidade, mostrando que se trata de cinética de terceira ordem.
2A ⇄ I
𝑘𝑎
I+B→
⏞ Produtos

[𝐼]
Resolução: Expressão da constante de equilíbrio para a reação reversível: 𝐾 = [𝐴]² (Equação 1)

Analisando a reação irreversível, expressando a taxa de velocidade: vreação = k a x [I] x [B] (Equação 2)

[𝐼]
Isolando a concentração do intermediário, [I], na equação 1: 𝐾 = [𝐴]2

[𝐼] = 𝐾 𝑥 [𝐴]² (Equação 3)

Substituindo a equação 3 na equação 2, temos: vreação = k a x [I] x [B]

vreação = k a x K x [A]² x [B]

vreação = K x k a x [A]² x [B]

Ordem de reação em relação a [A] = 2


Ordem de reação em relação a [B] = 1
Ordem global: 2 + 1 = 3

Questão 960 – (IME) Considere a seguinte reação: 2 A + B → C. A partir dos dados fornecidos na tabela abaixo,
calcule a constante de velocidade da reação e o valor da concentração X. Considere que as ordens de reação em
relação aos reagentes são iguais aos respectivos coeficientes estequiométricos.

Velocidade da reação
Teste [A] (mol.L-1) [B] (mol.L-1)
(mol.L-1.s-1)
1 10 X v
2 X 20 2v
3 15 30 13.500

Resolução: Como a reação é elementar, a equação da velocidade ficará da seguinte forma: vreação =
k x [A]2 x [B]

Cálculo da constante cinética (k): Substituindo os dados do terceiro teste, temos: vreação = k x [A]2 x [B]

13500 = k x (15)2 x 30

13500 L²
k= =2
6750 mol² x s

Substituindo os dados dos testes 1 e 2, temos:

Teste (1): v = k x [A]² x [B]¹


v = k.(10)².X
v = k.100.X
Teste (2): v = k x [A]² x [B]¹

507
1600 FÍSICO-QUÍMICA APLICADA EXERCÍCIOS COMENTADOS - IME – ITA – OLIMPÍADA

2v = k. X² . 20

v k.100.X
Dividindo o teste (1) pelo teste (2), temos: 2v = k.X².20

mol
X = 10
L

Igualando os testes (2) e (3), temos: 200. X = 20. X²

mol
X = 10
L

Questão 961 – (IME) Uma mistura gasosa ideal de propano e ar é queimada a pressão constante, gerando 720 litros
de CO2 por hora, medidos a 20°C. Sabe-se que o propano e o ar encontram-se em proporção estequiométrica.
Determine a velocidade média de reação da mistura em relação ao ar, considerando a composição do ar 21% de
oxigênio e 79% de nitrogênio, em volume.

Resolução: C3H8(g) + 5O2(g) → 3CO2(g) + 4H2O(g)

Cálculo do número de mol de gás nitrogênio (N2):


5 mol de O2 --------------- 21%
nN2 mol de N2 ------------ 79%
nN2 = 18,81 mol de N2

Equação de combustão completa do propano com a presença do gás nitrogênio:


C3H8(g) + 5 O2(g) + 18,81 N2(g) → 18,81 N2(g) + 3 CO2(g) + 4 H2O(g)

Cálculo da velocidade média do ar a partir da velocidade do gás carbônico:


1 d[ar] 1 d[CO2 ]
− x = − x
23,81 dt 3 dt

1 1
− x Var = − x VCO2
23,81 3

1 1 L
− x Var = − x 720
23,81 3 h

L
Var = −5714,40
h

Observação: o sinal negativo apenas indica que o ar está sendo consumido.

Questão 962 – (GRILLO) Após alguns estudos experimentais de uma certa reação, foi identificado que a Equação
de Arrhenius era satisfeita para um valor de fator de frequência igual a 4x1013.s-1 e de energia de ativação igual a 98,6
kJ.mol-1. Sabendo-se ainda que a reação é de primeira ordem, calcule a temperatura na qual o tempo de meia vida
da reação será igual a 10 minutos?
Eat
(− )
Resolução: Aplicando o logaritmo neperiano na equação de Arrhenius, temos: k = A. e R.T

Eat
(− )
lnk = lnA + ln e R.T

Eat
lnk = lnA −
RxT

508
1600 FÍSICO-QUÍMICA APLICADA EXERCÍCIOS COMENTADOS - IME – ITA – OLIMPÍADA

Cálculo da velocidade específica para t = 10 minutos (600 segundos), a partir da equação do tempo de meia-vida com
ln2 ln 2
cinética de primeira ordem: k = t = 600 s = 1,15 x 10−3 s −1
1⁄
2

Eat
Cálculo da temperatura: lnk = lnA − R.T

98600
ln(1,15 x 10−3 ) = ln(4,0 x 1013 ) −
8,314 x T

98600
− 6,77 = 31,32 −
8,314 x T

− 38,09 x 8,314 x T = − 98600

98600
T= = 311,35 K
316,68

Questão 963 – (ITA) Existem reações que, apesar de termodinamicamente possíveis, ocorrem com velocidade tão
pequena que pode levar dias para que sua ocorrência seja percebida, ao passo que outras ocorrem com velocidade
tão grande que chegam a ser explosivas. Como, num laboratório de química, você procederia para:
a) Acelerar uma reação muito lenta? (cite alguns procedimentos, justificando-os).
b) Retardar uma reação muito rápida? (cite alguns procedimentos, justificando-os).

Resolução:
Item a) Para acelerar uma reação muito lenta, há vários procedimentos:
• O aumento da temperatura provoca aumento da velocidade de reação;
• Adição de um catalisador provoca um aumento na reação química;
• Para um processo heterogêneo, regido por uma reação heterogênea, pode ter uma a velocidade aumentada,
por causa do aumento da superfície de contato entre os reagentes;
• Para uma reação homogênea gasosa, aumentando a pressão, mantendo o volume constante, há um
aumento da velocidade, pois aumento o número de colisões efetivas.
Item b) Para retardar uma reação rápida, há vários procedimentos. Vamos a eles.
• Reduzir a temperatura;
• Adicionar um inibidor, substância esta que aumenta a energia de ativação e consequentemente reduz a
velocidade da reação;
• Em uma reação heterogênea, usar aglomerados dos reagentes, o que faz reduzir a superfície de contato;
• Em um meio gasoso, a redução de pressão a volume constante.

Questão 964 – (GRILLO) A decomposição térmica do N2O5 forma NO2 e O2 apresentando cinética de primeira ordem,
com velocidade específica igual a 5,1 x 10-4 s-1 a 313 K.
a) Calcule o tempo de meia vida do processo.
b) A 70°C a velocidade específica é igual a 6,82 x 10-3 s-1. Suponha que no início o número de N2O5 seja de 0,3
mol, quantos mol de N2O5 restarão após 1,5 min de reação?
c) Qual o tempo de meia vida do N2O5 a 70°C?

Resolução:

ln 2 ln 2
Item a) Cálculo do tempo de meia-vida para T = 313 K: t 1⁄ = k
= 5,10 x 10−4 = 1359,11 s
2

n
Item b) Sendo o processo com cinética de primeira ordem, temos: ln (n Final ) = −k x tempo
Inicial

nFinal 60 segundos
ln ( ) = −(6,82 x 10−3 x 1,50 minutos x )
0,30 1 minuto

509
1600 FÍSICO-QUÍMICA APLICADA EXERCÍCIOS COMENTADOS - IME – ITA – OLIMPÍADA

nFinal
= e(−0,6138)
0,30

nFinal = 0,162 mol

ln 2 ln 2
Item c) Cálculo do tempo de meia-vida à 70°C: t 1⁄ = k
= 6,82 x 10−3 = 101,63 s
2

Questão 965 – (GRILLO) Considere o seguinte mecanismo abaixo:


Z2(g) ⇄ 2 Z(g) (Etapa rápida)

ka
Z(g) + W(g) → Produtos (Etapa lenta)

Deduza a lei de cinética para este processo.

Resolução: Analisando a reação reversível, aplicando a expressão da constante de equilíbrio: Z2(g) ⇄ 2 Z(g)
[Z]2
K=
[Z2 ]

ka
Analisando a reação irreversível, apresentando a taxa de reação: Z(g) + W(g) → Produtos

vreação = k a x [Z] x [W]

[Z]2
Isolando o intermediário na expressão da constante de equilíbrio, temos: K = [Z
2]

[Z]2 = K x [Z2 ]

[Z] = √K x [Z2 ]

Substituindo o intermediário na equação de taxa, temos: vreação = k a x [Z] x [W]

vreação = k a x √K x [Z2 ] x [W]

vreação = K1/2 x k a x √[Z2 ] x [W]

Questão 966 – (GRILLO) A reação química dada por A + B → C ocorre em dois estágios atarvés do seguinte
mecanismo:
2 A(g) ⇄ D(g)
𝑘𝑎
B(g) + D(g) → A(g) + C(g)
O primeiro estágio atinge rapidamente o equilíbrio. Deduza a lei de cinética para o produto C, em termos de ka, K, [A]
e [B].

[D]
Resolução: Analisando a reação reversível, a partir da constante de equilíbrio: K =
[A]2

Equação da velocidade da reação, a partir da equação irreversível: vreação = k a x [B] x [D]

Isolando o intermediário, [D], na equação do equilíbrio químico, temos: [D] = K x [A]2

Substituindo na equação da velocidade, temos: vreação = k a x [B] x [D]

510
1600 FÍSICO-QUÍMICA APLICADA EXERCÍCIOS COMENTADOS - IME – ITA – OLIMPÍADA

vreação = k a x [B] x K x [A]2

vreação = K x k a x [B] x [A]2

d[C]
Equação da velocidade em função do produto C: vreação = + = K x k a x [A]2 x [B]
dt

Questão 967 – (GRILLO) Considere a seguinte reação de síntese da sacarose com a presença da água no estado
líquido, conforme pode ser observado pela equação química não balaneada, C12H22O11(aq) + H2O(l) → C6H12O6(aq). A
cinética química desta reação foi estudada e os dados obtidos foram colocados em uma tabela pelos meus alunos da
Qim241 do curso técnico de Química - IFRJ – Campus Nilópolis.

Tempo (minutos) [SACAROSE] (mol.L-1)


0 0,316
39 0,274
80 0,238
140 0,190
210 0,146

a) Prove que estes dados referem-se a uma cinética de primeira ordem.


b) Calcule a constante cinética.

Resolução: Item a) Calculando o logaritmo neperiano para cada concentração da sacarose, a tabela fica da seguinte
maneira:

Tempo (minutos) [C12H22O11] (mol.L-1) ln [C12H22O11]


0 0,316 1,152
39 0,274 1,295
80 0,238 1,435
140 0,190 1,661
210 0,146 1,924

Gráfco plotado que relaciona ln(sacarose) versus o tempo.

-1.1

-1.2

-1.3

-1.4
Ln [sacarose]

-1.5

-1.6

-1.7

-1.8

-1.9

-2.0
0 50 100 150 200 250

Tempo (minutos)

A partir dos dados fornecidos, como a relação ln [sacarose] versus o tempo é uma reta, logo a reação química
apresenta cinética de primeira ordem.

Item b) Cálculo da velocidade específica: ln[sacarose]final − ln[sacarose]inicial = − k x tempo

− 1,924 − (−1,152) = − k x 210

511
1600 FÍSICO-QUÍMICA APLICADA EXERCÍCIOS COMENTADOS - IME – ITA – OLIMPÍADA

k = 0,00368 min−1

Questão 968 – (GRILLO) A velocidade de uma determinada reação química gasosa a 100°C é três vezes mais rápida
que a de 35°C. Estime a energia de ativação para esta reação.

Resolução: Informação do problema: v1100°C = 3 x v235°C; k1 x [A]n = 3 x k2 x [A]n e k1 = 3.k2

k Eat 1 1
Cálculo da energia de ativação (Eat): ln (k1 ) = R
x {T − T }
2 2 1

3 x k2 Eat 1 1
ln ( )= x {( )−( )}
k2 8,314 308 373

Eat 1 1
ln(3) = x {( )−( )}
8,314 308 373

J
Eat = 16143,61
mol

Questão 969 – (U.S. NATIONAL CHEMISTRY OLYMPIAD – SEGUNDA FASE) Pentóxido de dinitrogênio gasoso,
N2O5, decompões para formar oxigênio gasoso e dióxido de carbono com uma taxa inicial a 25°C, conforme a tabela
abaixo:
[N2O5] (mol.L-1) Taxa (mol.L-1min-1)
0,150 3,42 x 10-4
0,350 7,98 x 10-4
0,650 1,48 x 10-3
a) Escreva a equação balanceada para esta reação.
b) Use os dados fornecidos para escrever a lei de taxa e calcule o valor de k para esta reação. Mostre os
cálculos.
c) Calcule o tempo necessário para que a concentração de N2O5 diminua de 0,150 mol.L-1 para 0,050 mol.L-1.
d) A taxa inicial para a reação de uma amostra 0,150 mol.L-1 a 40°C é 2,37 x 10–3 mol.L–1.min–1. Determine a
energia de ativação para esta reação.

Resolução:

Item a) Equação química: N2O5(g) → 2 NO2(g) + ½ O2(g)

0,650 a 1,48 x 10−3


Item b) Cálculo da ordem de reação (a) entre o experimento I e III: ( ) =
0,150 3,42 x 10−4

(4,33)a = 4,33

a = 1 (Reação de decomposição com cinética de primeira ordem)

Cálculo da constante cinética (k), através do experimento I: vreação = k x [N2 O5 ]

3,42 x 10−4 = k x (0,150)

k = 0,00228 min−1

[𝑵 𝑶 ]
Item c) Cálculo do tempo (t): 𝒍𝒏 ([𝑵 𝟐𝑶 𝟓] 𝑭𝒊𝒏𝒂𝒍 ) = −𝒌 𝒙 𝒕𝒆𝒎𝒑𝒐
𝟐 𝟓 𝑰𝒏𝒊𝒄𝒊𝒂𝒍

𝟎, 𝟎𝟓𝟎
𝒍𝒏 ( ) = −𝟎, 𝟎𝟎𝟐𝟐𝟖 𝒙 𝒕𝒆𝒎𝒑𝒐
𝟎, 𝟏𝟓𝟎

512
1600 FÍSICO-QUÍMICA APLICADA EXERCÍCIOS COMENTADOS - IME – ITA – OLIMPÍADA

𝒕𝒆𝒎𝒑𝒐 = 𝟒𝟖𝟐 𝒎𝒊𝒏

Item d) Cálculo da velocidade específica (k2) a 40°C: vreação = k x [𝑁2 𝑂5 ]

2,37 x 10−3 = k 2 x (0,150)

k 2 = 0,0158 𝑚𝑖𝑛−1

Utilizando a equação de Arrhenius e considerando a constante dos gases (R) igual a 8,314 J.mol-1K-1, temos: k1 (25°C)
= 0,00228 min-1 e k2 (40°C) = 0,0158 min-1

k1 𝐸𝑎𝑡 1 1
𝑙𝑛 ( )= 𝑥 {( ) − ( )}
k2 𝑅 T2 T1

0,00228 𝐸𝑎𝑡 1 1
𝑙𝑛 ( )= 𝑥 {( )−( )}
0,0158 8,314 313 298

𝐸𝑎𝑡 1 1
𝑙𝑛(0,144) = 𝑥 {( )−( )}
8,314 313 298

𝐸𝑎𝑡
− 1,94 = 𝑥 (− 1,61 𝑥 10−4 )
8,314

8,314 𝑥 1,94 𝐽
𝐸𝑎𝑡 = = 100181,12
1,61 𝑥 10−4 𝑚𝑜𝑙

Questão 970 – (U.S. NATIONAL CHEMISTRY OLYMPIAD - SEGUNDA FASE) A decomposição térmica do HO2NO2,
tem sido estudado e foi observado que se trata de uma cinética de primeira ordem. A constante de taxa foi determinada
para 2,1 x 10–1 s–1 a 331K e 1,1 s–1 a 342K.
a) Calcule o tempo de meia-vida para o HO2NO2 a 331K, em segundos.
b) Para uma amostra de HO2NO2 com uma concentração inicial igual a 7,1 x 10–8 mol.L–1, calcule a concentração
de HO2NO2 depois de 0,40 minutes a 331K.
c) Calcule a energia de ativação para esta reação.

Resolução:
ln2 0,693
Item a) Cálculo do tempo de meia-vida (t1/2) a 331 K: t 1⁄ = k = 2,10 x 10−1 = 3,30 s
2

[HO NO ]
Item b) Cálculo da concentração final da quantidade de matéria: ln ([HO 2NO 2] Final ) = −k x tempo
2 2 Inicial

[HO2 NO2 ]Final 60 s


ln ( −8
) = −(2,10 x 10−1 ) x {0,40 minutos x }
7,10 x 10 1 minuto

[HO2 NO2 ]Final


ln ( ) = − 5,04
7,10 x 10−8

[HO2 NO2 ]Final


= e−5,04
7,10 x 10−8

[HO2 NO2 ]Final = 7,10 x 10−8 x e−5,04

mol
[HO2 NO2 ]Final = 4,60 x 10−10
L

513
1600 FÍSICO-QUÍMICA APLICADA EXERCÍCIOS COMENTADOS - IME – ITA – OLIMPÍADA

2,1.10−1
Item c) Cálculo da energia de ativação (Eat), a partir da utilização da equação de Arrhenius: ln ( 1,10
) =
Eat 1 1
8,314
x [342 − 331]
J kJ
Eat = 141683,97 (141,7 )
mol mol

Questão 971 – (IFRJ – MODIFICADA) O seguinte mecanismo foi proposto para a obtenção do ácido hipobromoso:

Etapa 1: HBr + O2 ⇌ HOBr


k2

Etapa 1: HOOBr + HBr →
⏞ 2 HOBr

Onde k1 e k´1 são, respectivamente, na etapa 1, as constantes de velocidade das reações direta e inversa e k 2 é a
constante de velocidade da reação na etapa 2. Baseando-se no mecanismo proposto, determine a lei de velocidade
da reação de formação de HOBr.

Resolução: Analisando a cinética do intermediário, HOOBr:

d[HOOBr]
= + k1 x [HBr] x [O2 ] − k1′ x [HOOBr] − k 2 x [HOOBr] x [HBr]
dt
𝑑[𝐻𝑂𝑂𝐵𝑟]
Admitindo que a espécie intermediária, 𝑑𝑡
não apresente variação ao longo do processo cinético, temos:
𝑑[𝐻𝑂𝑂𝐵𝑟]
𝑑𝑡
= 0.

d[HOOBr]
= + k1 x [HBr] x [O2 ] − k1′ x [HOOBr] − k 2 x [HOOBr] x [HBr]
dt

0 = + k1 x [HBr] x [O2 ] − k1′ x [HOOBr] − k 2 x [HOOBr] x [HBr]

+ k1 x [HBr] x [O2 ] = k1′ x [HOOBr] + k 2 x [HOOBr] x [HBr]

+ k1 x [HBr] x [O2 ] = [HOOBr] x {k1′ + k 2 x [HBr]}

k1 x [HBr] x [O2 ]
[HOOBr] =
{k1′ + k 2 x [HBr]}

Cálculo da taxa cinética d[HOOBr]/dT, substituindo expressão do intermediário na equação da taxa em função do
HOOBr:

1 d[HOOBr]
+ x = +k 2 x [HOOBr]x [HBr]
2 dt
+ k1 x [HBr] x [O2 ]
Sabendo que [HOOBr] = {k′1 + k2 x [HBr]}
, temos:

1 d[HOOBr] k 2 x k1 x [HBr]² x [O2 ]


+ x =
2 dt {k1′ + k 2 x [HBr]}

d[HOOBr] 2 x k 2 x k1 x [HBr]² x [O2 ]


=
dt {k1′ + k 2 x [HBr]}

514
1600 FÍSICO-QUÍMICA APLICADA EXERCÍCIOS COMENTADOS - IME – ITA – OLIMPÍADA

Questão 972 – (GRILLO) O mecanismo a seguir foi proposto para explicar a destruição da camada de ozônio, que
pode ser observado através das seguintes reações químicas:
O3 + Cl → ClO + O2
ClO + O → Cl + O2
Diante deste processo, determine:
a) Qual o intermediário da reação?
b) Qual o catalisador da reação?
c) Qual a reação global?

Resolução:

Item a) O intermediário deste processo é o ClO.

Item b) O catalisador deste processo é o Cl.

Item c) Determinação da reação global, somando as reações químicas apresentadas:

O3 + Cl → ClO + O2
ClO + O → Cl + O2 +

O3 + O → 2 O2
Questão 973 – (GRILLO) O íon tricloroacetato em determinados solventes ionizantes que contém H+, decompõe-se
em dióxido de carbono e clorofórmio. Foram levantados alguns dados da constante cinética em função da temperatura,
conforme pode ser observado no quadro abaixo:
k (s-1) T (°C)
3,11 x 10-4 90
7,62 x 10-5 80
1,71 x 10-5 70
Sabendo que o processo apresenta uma cinética de primeira ordem, determine o valor da constante cinética a uma
temperatura de 60°C.

Resolução: v Informações do problema: 90°C (363 K) → k = 3,11 x 10-4 s-1 e 70°C (343 K) → k = 1,71 x 10-5 s-1
k90°C Eat 1 1
Cálculo da energia de ativação (Eat): ln (k70°C ) = R
x [(70+273) − (90+273)]

3,11 x 10−4 Eat 1 1


ln ( −5 )= x[ − ]
1,71 x 10 8,314 343 363
J
Eat = 149800
mol
Cálculo da constante cinética a 60°C: 90°C (363 K) → k = 3,11 x 10-4 s-1 e 60°C (333 K) → k = ?

k 60°C Eat 1 1
ln ( 90°C )= x [( )−( )]
k R 90 + 273 60 + 273

k 60°C 149800 1 1
ln ( ) = x [ − ]
3,11 x 10−4 8,314 363 333

k 60°C
= e(−4,47)
3,11 x 10−4
k 60°C = 3,56 x 10−6 s −1

515
1600 FÍSICO-QUÍMICA APLICADA EXERCÍCIOS COMENTADOS - IME – ITA – OLIMPÍADA

Questão 974 – (GRILLO) A reação de decomposição do monóxido de dinitrogênio fornece nitrogênio gasoso e
oxigênio também na fase gasosa, a 750°C. Este processo ocorre com cinética de primeira ordem, com constante
cinética igual a 2,25 x 10-2 h-1. Considerando que a pressão inicial seja igual a 375 torr, calcule a pressão de equilíbrio
aproximado ao final de 2,50 horas.

Resolução: Equação química: N2O(g) → N2(g) + ½ O2(g)


[N O]
Equação química com cinética de primeira ordem: ln [N 2O] Final = −k x tempo
2 Inicial

pFinal
NO2
Em função da pressão parcial do dióxido de nitrogênio, a equação acima fica da seguinte maneira: ln =
pInicial
NO2

−k x tempo

pFinal
NO
Substituindo na equação cinética de primeira ordem, temos: ln 375 torr
2
= −2,25 x 10−2 x 2,50

pFinal
NO2
= e(−0,05625)
375 torr

pFinal
NO2 = 375 torr x e
(−0,05625)
= 354,49 torr

Questão 975 – The rate constante for na isomerization reaction, A → B is 4,50 x 10-3 min-1. If the initial concentration
of A is 1,0 mol x L-1, calculate the rate of the reaction after 1 h.

Resolução: Para a resolução deste problema será necessário utilizar a tabele de equilíbriio químico.

Equação
A
química B
Início 1,0 0
Reage x x
Equilíbrio 1,0 - x x

Sabendo que o processo apresenta cinética de primeira ordem, uma vez que o indicativo disso seja a unidade da
[𝐴]
constante cinética. Com isso, utilizando a equação para a cinética de primeira ordem: ln [A] Final = −k x tempo
Inicial

1−x
ln = −4,50 x 10−3 x 60
1,0

1−x
ln = −0,27
1,0

1−x
= e−0,27
1,0

mol
x = 0,237
L

Cálculo da taxa após 1 hora (60 minutos): k = (1 − x) x k = (1 − 0,237) x 4,50 x 10−3 = 3,43 x 10−3 min−1

Questão 976 – In the Arrhenius equation for a certain reaction, the value of A and Eat (activation energy) are 4,0 x
1013 sec-1 and 98,6 kJ x mol-1 respectively. If the reaction is of first order, at what temperature will its half-life period be
tem minutes.

516
1600 FÍSICO-QUÍMICA APLICADA EXERCÍCIOS COMENTADOS - IME – ITA – OLIMPÍADA

ln2
Resolução: Cálculo da constante cinética, a partir de um processo cinético de primeira ordem: t 1⁄ = k
= 10 𝑥 60
2

ln 2
k=
600
Eat

k =Axe 8,314 x T

ln 2 98600

= 4,0 x 1013 x e 8,314 x T
600
T = 311,14 K

Questão 977 – At 380°C, the half-life period for the first order decomposition of H2O2 is 360 min. The energy of
activation of the reactioin is 200 kJ x mol-1. Calculate the time required for 75% decomposition at 450°C.

Resolução:
ln 2
Cálculo da constante cinética a 380°C: k = 360 = 0,001925 min−1

0,001925 200000 1 1
Cálculo da constante cinética a 450°C, aplicando a equação de Arrhenius: ln ( k
) = 8,314
x [723 − 653]

0,001925
ln ( ) = −3,57
k

k = 0,0692 min−1
[A]
Aplicando a equação para um prcesso cinético de primeira ordem: ln [A] Final = −k x tempo
Inicial

0,25 x [A]inicial
ln = −0,0692 x tempo
[A]Inicial

tempo = 20,03 min

Questão 978 – The rate constant of a reaction is 1,50 x 107 s-1 at 50°C and 1,50 x 107 s-1 at 100°C. Evaluate the
Arrhenius parameters A and Eat.

1,50 x 107 E 1 1
Resolução: Cálculo da energia de ativação: ln (4,50 x 107 ) = 8,314
at
x [373 − 323]

J
Eat = 22008,77
mol
22008,77

Cálculo do fator de frequência (A): 1,50 x 107 = A x e 8,314 x 323

A = 5,44 x 10+10 s−1

517
1600 FÍSICO-QUÍMICA APLICADA EXERCÍCIOS COMENTADOS - IME – ITA – OLIMPÍADA

Questão 979 – (IME) O gráfico abaixo ilustra as variações de energia devido a uma reação química conduzida nas
mesmas condições iniciais de temperatura, pressão, volume de reator e quantidades de reagentes em dois sistemas
diferentes. Estes sistemas diferem apenas pela presença de catalisador. Com base noráfico, é possível afirmar que:

a) A curva 1 representa a reação catalisada, que ocorre com absorção de calor.


b) A curva 2 representa a reação catalisada, que ocorre com absorção de calor.
c) A curva 1 representa a reação catalisada com energia de ativação dada por E1 + E3.
d) A curva 2 representa a reação não catalisada, que ocorre com liberação de calor e a sua energia de ativação é
dada por E2 + E3.
e) A curva 1 representa a reação catalisada, que ocorre com liberação de calor e a sua energia de ativação é dada
por E1.

Resolução: Alternativa E.
a) Falsa. A reação é exotérmica, pois o patamar da entalpia dos produtos é menor em comparação a entalpia
dos reagentes.
b) Falsa. A reação catalisada é a representada pela curva 1, que tem energia de ativação (Eat) menor.
c) Falsa. A energia de ativação é E1.
d) Falsa. A reação é não catalisada, ocorrendo a liberação de calor (ΔH < 0), mas sua energia de ativação é
dada por E2.
e) Correta.

Questão 980 – (IME) Considere a reação catalisada descrita pelo mecanismo a seguir.
Primeira etapa: A + BC → AC + B
Segunda etapa: AC + D → A + CD
O perfil energético dessa reação segue a representação do gráfico abaixo.

Diante das informações apresentadas é correto afirmar que:


a) Os intermediários de reação são representados por (2) e (3) e equivalem, respectivamente, aos compostos
BC e AC;
b) Os reagentes, representados por (1), são os compostos A e D;
c) O complexo ativado representado por (4) tem estrutura A ----- C ----- D;
d) O produto, representado por (5), é único e equivale ao composto CD;
e) A presença do catalisador A torna-se a reação exotérmica.

518
1600 FÍSICO-QUÍMICA APLICADA EXERCÍCIOS COMENTADOS - IME – ITA – OLIMPÍADA

Resolução: Alternativa C.
Primeira etapa: A + BC → AC + B
Segunda etapa: AC + D → A + CD +

Reação Global: BC + D → CD + B
a) Falso. Os (3) = AC, B, D
b) Falso. Os reagentes são os seguintes: A, D, BC.
c) Correta.
d) Falso. CD, B, A.
e) Falso. A presença do catalisador não interfere quanto a quantidade de calor (ΔH). A presença do catalisador
diminui a energia de ativação.

Questão 981 – (IME) “A olimpíada deve ser disputada sem o fantasma da fraude química dentro do princípio de que,
tanto quanto é importante competir, vencer é a prova de competência”. (Jornal “O Globo”, 28/05/2016). Considere que
um atleta tenha consumido 64 mg de um anabolizante e que, após 4 dias, o exame antidoping tenha detectado apenas
0,25 mg deste composto. Assumindo que a degradação do anabolizante no organismo segue uma cinética de primeira
ordem, assinale a alternativa que apresenta o tempo de meia-vida da substância no organismo do atleta.
a) 4 horas
b) 6 horas
c) 8 horas
d) 12 horas
e) 48 horas

Resolução: Alternativa D.
𝑚𝐴𝐹𝑖𝑛𝑎𝑙
Cálculo da constante cinética (k), considerando o processo com cinética de primeira ordem: 𝑙𝑛 ( )=
𝑚𝐴𝐼𝑛𝑖𝑐𝑖𝑎𝑙
−𝑘 𝑥 𝑡𝑒𝑚𝑝𝑜
0,25 𝑚𝑔
𝑙𝑛 ( ) = −𝑘 𝑥 4
64 𝑚𝑔
− 8 𝑥 𝑙𝑛(2) = −𝑘 𝑥 4
2 𝑥 𝑙𝑛(2) = 𝑘
ln2
Sabendo que o tempo de meia-vida para uma cinética de primeira ordem é definida como sendo t 1⁄ = k
, temos:
2

2 x ln(2) = k
ln2
2x =1
k
2 x t1/2 = 1

𝑡1/2 = 0,50 dias, o que corresponde a 12 horas.

519
1600 FÍSICO-QUÍMICA APLICADA EXERCÍCIOS COMENTADOS - IME – ITA – OLIMPÍADA

Questão 982 – (ITA) A equação 2A + B → PRODUTOS representa uma determinada reação química que ocorre no
estado gasoso. A lei de velocidade para esta reação depende da concentração de cada um dos reagentes, e a ordem
parcial desta reação em relação a cada um dos reagentes é igual aos respectivos coeficientes estequiométricos. Seja
v1 a velocidade da reação quando a pressão parcial de A e B é igual a pA e pB, respectivamente, e v2 a velocidade da
reação quando essas pressões parciais são triplicadas. A opção que fornece o valor correto da razão v2 /v1 é:
a) 1
b) 3
c) 9
d) 27
e) 81

Resolução: Alternativa D.
pA 2 pB
vreação = k x ( ) x ( )
RT RT
p 2 p
1
Analisando a situação I: vreação A
= k x (RT ) x (RTB ) (Equação A) .

𝟑 𝒙 𝒑𝑨 𝟐 𝟑 𝒙 𝒑𝑩
Analisando a situação II: Triplicando ambas as pressões parciais, temos: 𝒗𝟐𝒓𝒆𝒂çã𝒐 = 𝒌 𝒙 ( 𝑹𝑻
) 𝒙( 𝑹𝑻
)

𝒑𝑨 𝟐 𝒑𝑩
𝒗𝟐𝒓𝒆𝒂çã𝒐 = 𝟐𝟕 𝒙 𝒌 𝒙 ( ) 𝒙( ) (Equação B)
𝑹𝑻 𝑹𝑻

𝒑 𝟐 𝒑
𝒗𝟐𝒓𝒆𝒂çã𝒐 𝟐𝟕 𝒙 𝒌 𝒙 ( 𝑨 ) 𝒙 ( 𝑩 )
Dividindo a Equação B pela Equação A, temos: = 𝑹𝑻 𝑹𝑻
𝒗𝟏𝒓𝒆𝒂çã𝒐 𝒑 𝟐 𝒑
𝒌 𝒙 ( 𝑨) 𝒙 ( 𝑩 )
𝑹𝑻 𝑹𝑻
𝒗𝟐𝒓𝒆𝒂çã𝒐
= 𝟐𝟕
𝒗𝟏𝒓𝒆𝒂çã𝒐

Questão 983 – (ITA) A reação entre íons brometo e bromato, em meio aquoso e ácido, pode ser representada pela
seguinte equação química balanceada: 5 Br-(aq) + BrO3-(aq) + 6 H+(aq) → 3 Br2(aq) + 3 H2O(l). Sabendo que a velocidade
de desaparecimento do íon bromato é igual a 5,63 x 10-6 mol.L-1.s-1, assinale a alternativa que apresenta o valor correto
para a velocidade de aparecimento do bromo, Br2, expressa em mol.L-1.s-1.
a) 1,69 x 10-5
b) 5,63 x 10-6
c) 1,90 x 10-6
d) 1,13 x 10-6
e) 1,80 x 10-16

Resolução: Alternativa A.
Relação entre a velocidade da reação em função do consumo do bromato e de formação do bromo: vreação =
d[BrO−3] 1 d[Br2 ]
− dt
=+ 3 dt

1 d[Br2 ]
5,63 x 10−6 = + x
3 dt
d[Br2 ] mol
= 3 x 5,63 x 10−6 = 1,69 x 10−5
dt Lxs
Questão 984 – (ITA) A equação de Arrhenius k = A.e(-Eat/RT) mostra a relação de dependência da constante de
velocidade (k) de uma reação química com a temperatura (T) em kelvin (K), a constante universal dos gases (R), o
fator pré-exponencial A e a energia de ativação (Eat). A curva abaixo mostra a variação da constante de velocidade
com o inverso da temperatura absoluta, para uma dada reação química que obedece à equação acima. A partir da
análise desse gráfico, assinale a opção que apresenta o valor da razão Eat/R para essa reação.

520
1600 FÍSICO-QUÍMICA APLICADA EXERCÍCIOS COMENTADOS - IME – ITA – OLIMPÍADA

a) 0,42
b) 0,50
c) 2,0
d) 2,4
e) 5,5

Resolução: Alternativa C.
Para a resolução deste exercício será necessário aplicar a equação de Arrhenius. Equação de Arrhenius: 𝑘 =
−𝐸𝑎𝑡
( ) 𝐸𝑎𝑡
𝐴𝑥𝑒 𝑅𝑥𝑇 . Aplicando o logaritmo neperiano na equação de Arrhenius, temos: ln 𝑘 = ln 𝐴 −
𝑅𝑥𝑇
− Eat
Chamando ln k = y e 1/T = x, o coeficiente angular será obtido através da seguinte relação matemática: R
=
− (2−4)
1,75−0,75

− Eat − 2
=
R 1
Eat
=2
R

Questão 985 - (ITA) Dentre as afirmações a seguir, todas relativas a ação de catalisadores, assinale a errada:
a) um bom catalisador para certa polimerização também é um bom catalisador para a respectiva despolimerização
b) enzimas são catalisadores via de regra muito específicos
c) as vezes, as próprias paredes de um recipiente podem catalisar uma reação numa solução contida no mesmo
d) a velocidade da reação catalisada depende da natureza do catalisador, mas não da sua concentração na fase
reagente
e) fixadas as quantidades iniciais dos reagentes postos em contato, as concentrações no equilíbrio final independem
da concentração do catalisador adicionado

Resolução: Alternativa D. A concentração do catalisador influencia na velocidade da reação catalisada, logo a


velocidade da reação depende da natureza do catalisador além da sua concentração na fase reagente.
Questão 986 - (ITA) Considere a reação química genérica A → B + C. A concentração do reagente [A] foi
acompanhada ao longo do tempo, conforme apresentada na tabela que também registra os logaritmos neperianos (n)
desses valores e os respectivos recíprocos (1/[A]).

Tempo (s) [A] (mol.L-1) Ln[A] 1/[A] (L.mol -1)


0 0,150 -0,11 1,11
100 0,350 -0,46 1,59
200 0,650 -0,84 2,33
300 0,3 -1,20 3,33

521
1600 FÍSICO-QUÍMICA APLICADA EXERCÍCIOS COMENTADOS - IME – ITA – OLIMPÍADA

400 0,21 -1,56 4,76


500 0,14 -1,97 7,14
600 0,10 -2,3 10,00

Assinale a opção que contém a constante de velocidade correta desta reação:


a) 4 x 10-3. s-¹
b) 4 x 10-3 mol.L-1.s-1
c) 4 x 10-3 L.mol-1.s-1
d) 4 x 103. s-¹
e) 4 x 103. mol.L-1.s-1

Resolução: Alternativa A.
1 1
Analisando o processo para uma cinética de segunda ordem, entre 0 a 600 segundos: = +kxt
[A]Final [A]Inicial

1 1
= + k x 600
10 1,11

L
k = 1,51 x 10−5
mol x s

Analisando o processo para uma cinética de ordem zero, entre 0 a 600 segundos: [𝐴]𝐹𝑖𝑛𝑎𝑙 − [𝐴]𝐼𝑛𝑖𝑐𝑖𝑎𝑙 = −𝑘 𝑥 𝑡

− (0,100 − 0,150) mol


k= = 8,33 x 10−5
600 Lxs

Analisando o processo para uma cinética de primeira ordem, entre 0 a 600 segundos: ln[𝐴]𝐹𝑖𝑛𝑎𝑙 − ln[𝐴]𝐼𝑛𝑖𝑐𝑖𝑎𝑙 =
−𝑘. 𝑡

− 2,30 − (−0,11) = −k x 600

− 2,30 − (−0,11) − 2,19


−k = =
600 600

k = 3,65 x 10−3 s−1

522
1600 FÍSICO-QUÍMICA APLICADA EXERCÍCIOS COMENTADOS - IME – ITA – OLIMPÍADA

Questão 987 – (ITA) O cloreto de sulfurila, SO2Cl2, no estado gasoso, decompõe-se nos gases cloro e dióxido de
enxofre em uma reação química de primeira ordem (análogo ao decaimento radioativo). Quantas horas irá demorar
para que ocorra a decomposição de 87,5% de SO2Cl2 a 320oC? Dados: constante de velocidade da reação de
decomposição (320°C) = 2,20 x 10-5 s-1; ln (0,50) = - 0,693.
a) 1,58
b) 8,75
c) 11,1
d) 26,3
e) 52,5

Resolução: Alternativa D.
Cálculo do tempo para uma decomposição de 87,5%, sobrando 12,5%, ou seja, [SO2Cl2]Final = 0,125 x [SO2Cl2]Inicial:
[𝑺𝑶 𝑪𝒍 ]
𝒍𝒏 ([𝑺𝑶 𝟐𝑪𝒍 𝟐] 𝑭𝒊𝒏𝒂𝒍 ) = −𝒌 𝒙 𝒕
𝟐 𝟐 𝑰𝒏𝒊𝒄𝒊𝒂𝒍

0,1250 x [SO2 Cl2 ]Inicial


ln ( ) = −2,20 x 10−5 x t
[SO2 Cl2 ]Inicial

ln(0,1250 ) = −2,20 x 10−5 x t

125
ln ( ) = −2,20 x 10−5 x t
1000

5 3
ln ( ) = −2,20 x 10−5 x t
10
−3 x 0,693
t= = 94500 segundos (26,25 h)
−2,20 x 10−5

Questão 988 – (ITA) Considere que a decomposição do N2O5, representada pela equação química global 2 N2O5 →
4 NO2 + O2, apresente lei de velocidade de primeira ordem. No instante inicial da reação, a concentração de N2O5 é
de 0,10 mol.L-1 e a velocidade de consumo desta espécie é de 0,022 mol.L-1.min-1. Assinale a opção que apresenta o
valor da constante de velocidade da reação global, em min-1:
a) 0,0022
b) 0,011
c) 0,022
d) 0,11
e) 0,22

Resolução: Alternativa D.
Equação química: 2 N2O5(g) → 4 NO2(g) + O2(g)
1 d[N2 O5 ] 1 d[NO2 ]
Relação da velocidade da reação (vreação) com os componentes gasosos: vreação = − 2 x dt
= + 4x dt
=
d[O ]
+ 2
dt

1 d[N2 O5 ]
vreação = − x = k x [N2 O5 ]
2 dt
1
vreação = − x vN2 O5 = k x [N2 O5 ]
2
mol
Informação do problema: vN2 O5 = 0,022 L

523
1600 FÍSICO-QUÍMICA APLICADA EXERCÍCIOS COMENTADOS - IME – ITA – OLIMPÍADA

1
x 0,022 = k x 0,10
2
k = 0,11 min−1
Questão 989 – (ITA) Considere as seguintes afirmações relativas a reações químicas em que não haja variação de
temperatura e pressão:
I - Uma reação química realizada com a adição de um catalisador é denominada heterogênea se existir uma superfície
de contato visível entre os reagentes e o catalisador
II - A ordem de qualquer reação química em relação à concentração do catalisador é igual a zero.
III - A constante de equilíbrio de uma reação química realizada com a adição de um catalisador tem valor numérico
maior do que o da reação não catalisada.
IV - A lei de velocidade de uma reação química realizada com a adição de um catalisador, mantidas constantes as
concentrações dos demais reagentes, é igual àquela da mesma reação não catalisada.
V - um dos produtos de uma reação química pode ser o catalisador da mesma reação.
Das afirmações feitas, estão corretas:
a) apenas I e III
b) apenas I e V
c) apenas I, II e IV
d) apenas II, IV e V
e) apenas III, IV e V

Resolução: Alternativa B.
I – Verdadeiro. Quando o catalisador e os reagentes estão em fases diferentes, a catálise é denominada de
heterogênea.
II – Falso. Quando o catalisador afetar a velocidade de uma ração química a ordem da reação será diferente de zero
em relação ao catalisador.
III – Falso. Somente a temperatura consegue alterar a constante de equilíbrio de uma reação química.
IV – Falso. A adição de um catalisador modifica o caminho da reação, o número de etapas e a natureza do complexo
ativado. Assim, a expressão da lei para uma reação não catalisada necessariamente não é a mesma de uma reação
catalisada.
V – Verdadeiro. Trata-se de uma auto-catálise. Por exemplo, pode-se citar a reação de Belousov-Zhabothiskii.
BrO3- + HBrO2 + H3O+ → 2 BrO2 + 2H2O
2 BrO2 + 2 Ce3+ + 2H3O+ → 2 HBrO2 + Ce4+ + 2H2O
Pelas etapas das reações pode-se perceber que o catalisador é o HBrO2, assim, se a sua concentração for aumentada
a velocidade da primeira etapa será aumentada.
Questão 990 - (ITA) Uma reação hipotética de decomposição de uma substância gasosa catalisada em superfície
metálica tem lei de velocidade de ordem zero, com uma constante de velocidade (k) igual a 10-3 atm.s-1. Sabendo que
a pressão inicial do reagente é igual a 0,6 atm, assinale a opção que apresenta o tempo necessário, em segundos,
para que um terço do reagente se decomponha.
a) 0,00001
b) 200
c) 400
d) 600
e) 693

Resolução: Alternativa C.
1
Dados do problema: 𝑃𝐴𝐼𝑛𝑖𝑐𝑖𝑎𝑙 = 0,60 𝑎𝑡𝑚; 𝑃𝐴𝐹𝑖𝑛𝑎𝑙 = 3 𝑥 𝑃𝐴𝐼𝑛𝑖𝑐𝑖𝑎𝑙

524
1600 FÍSICO-QUÍMICA APLICADA EXERCÍCIOS COMENTADOS - IME – ITA – OLIMPÍADA

1
𝑃𝐴𝐹𝑖𝑛𝑎𝑙 = 𝑥 0,60 = 0,20 𝑎𝑡𝑚
3
Cálculo do tempo (t): PAFinal − PAInicial = −k x t

0,20 − 0,60 = −(10−3 ) x t


t = 400 s
Questão 991 – (OLIMPÍADA DE QUÍMICA DO RIO DE JANEIRO) A reação (CH3)3COH + Br- → (CH3)3CBr + OH-
ocorre segundo as etapas:
(CH3)3COH + Br- → (CH3)3C+ + OH- (Etapa Lenta)
(CH3)3C+ + Br- → (CH3)3CBr (Etapa Rápida)
A lei da velocidade da reação pode ser dada por:
a) v = k. [(CH3)3COH].[Br-]
b) v = k. [(CH3)3COH]
c) v = k. [(CH3)3CBr].[OH-]
d) v = k. [(CH3)3C+].[OH-]
e) v = k. [(CH3)3C+].[Br-]

Resolução: Alternativa A. A etapa determinante é a etapa lenta, logo a equação da velocidade é dada pela seguinte
equação de taxa: vreação = k x [(CH3 )3 COH] x [Br − ].

Questão 992 – (OLIMPÍADA BRASILEIRA DE QUÍMICA) Obtém-se uma reta quando, para uma reação A → B, de
primeira ordem, constrói-se um gráfico de:
a) ln[A] versus t
b) ln[A] versus 1/t
c) [A] versus t
d) [A] versus 1/t
e) 1/[A] versus t

Resolução: Alternativa A. Equação química: A(g) → B(g).


𝒅[𝑨]
Fazendo a integração da concentração do reagente A em função do tempo, temos: − 𝒅𝒕
= 𝒌 𝒙 [𝑨]

[𝑨]𝑭𝒊𝒏𝒂𝒍 𝒕
𝟏
∫ 𝒅[𝑨] = − 𝒌 𝒙 ∫ 𝒅𝒕
[𝑨]𝑰𝒏𝒊𝒄𝒊𝒂𝒍 [𝑨] 𝟎

𝒍𝒏[𝑨]𝑭𝒊𝒏𝒂𝒍 − ⏟
⏟ 𝒍𝒏[𝑨]𝑰𝒏𝒊𝒄𝒊𝒂𝒍 = − 𝒌
⏟𝒙⏟
𝒕
𝒚 𝒂 𝒃 𝒙
Esboço do gráfico que relaciona o logaritmo natural da concentração da quantidade de matéria do reagente A(gasoso)
versus o tempo.

525
1600 FÍSICO-QUÍMICA APLICADA EXERCÍCIOS COMENTADOS - IME – ITA – OLIMPÍADA

Questão 993 – (OLIMPÍADA DE QUÍMICA DO RIO GRANDE DO Sul) Em março de 2011, um reator nuclear da
cidade de Fukoshima, apresentou danos devido a um grande terremoto no Japão, e houve vazamento de material
radioativo. Esse material continha Iodo-131 e contaminou toda região. Sabendo-se que este radioisótopo 53I131 emite
radiação beta e perde 75% de sua atividade em 16 dias, qual é a meia-vida deste radioisótopo?
a) 2 dias
b) 4 dias
c) 6 dias
d) 8 dias
e) 10 dias

Resolução: Alternativa D.
Sabendo que 75% representa perda de sua atividade, então há 25% de sobra, logo: [I]Final = 0,25 x [I]inicial.
[I]
Cálculo da constante cinética radioativa (λ) para uma reação com cinética de primeira ordem: ln ([I] Final ) = −λ x t
Inicial

ln(0,25) = −λ x 16 dias

1,386
λ= dias −1 = 0,087 dias−1
16
𝑙𝑛2 0,693
Cálculo do tempo de meia-vida: 𝑡1⁄ = 𝑘
= 0,087 = 8 𝑑𝑖𝑎𝑠
2

Questão 994 – (OLIMPÍADA BRASILEIRA DE QUÍMICA) O cobre – 64 é usado na forma de acetato de cobre (II),
no tratamento de tumores cerebrais. Se a meia-vida desse radioisótopo é de 12,8 horas, a quantidade que restará,
após 2 dias e 16 horas, de uma amostra com 15,0 mg de acetato de cobre II estará entre:
a) 0,1 e 0,5 mg
b) 0,5 e 1,0 mg
c) 1,0 e 2,0 mg
d) 2,0 e 3,0 mg
e) 3,0 e 5,0 mg

ln2
Resolução: Alternativa A. Cálculo da constante cinética radioativa: t 1⁄ = λ
2

ln2 0,693
λ= = = 0,0541 h−1
t 1⁄ 12,8
2

m
Sabendo que o tempo de dois dias e dezesseis horas corresponde a 64 horas, temos: ln (m Final ) = −λ x t
Inicial

526
1600 FÍSICO-QUÍMICA APLICADA EXERCÍCIOS COMENTADOS - IME – ITA – OLIMPÍADA

mFinal
ln ( ) = −0,0541 x 64
15 mg
mFinal
= e−3,46
15 mg

mFinal = 0,471 mg

Questão 995 – (U.S. NATIONAL CHEMISTRY OLYMPIAD) Qual é a constante de velocidade de primeira ordem para
uma reação que apresenta 36,5% completo em 0,0200 segundos?
a) 50,4 s-1
b) 27,7 s-1
c) 22,7 s-1
d) 9,86 s-1

Resolução: Alternativa C.
[A]
Cálculo da constante cinética: ln ([A] Final ) = −k x t
Inicial

Dado do problema: [A]Final = 0,625 x [A]inicial e tempo igual a 0,0200 s.


0,635 𝑥 [A]Inicial
ln ( ) = − k x 0,0200
[A]Inicial

ln(0,635)
k=− = 22,71 𝑠 −1
0,0200

Questão 996 – (OLIMPÍADA DE QUÍMICA DO RIO DE JANEIRO) No estudo da cinética da reação a uma
temperatura de 700°C entre o óxido nítrico e o gás hidrogênio formando óxido nitroso e água foram obtidos os dados
constantes na tabela abaixo:
Concentração (mol.L-1) Velocidade inicial
Óxido nítrico Gás hidrogênio (mol.L-1.s-1)
0,0250 0,0100 2,40 x 10-6
0,0250 0,0050 1,20 x 10-6
0,0125 0,0100 0,60 x 10-6
A ordem global para esta reação é:
a) 1
b) 2
c) 3
d) 4
e) 5

Resolução: Alternativa C.
𝟎,𝟎𝟏𝟐𝟓 𝒂 𝟎,𝟔𝟎 𝒙 𝟏𝟎−𝟔
Cálculo da ordem de reação em relação ao óxido nítrico, entre o experimento I e III, temos: (𝟎,𝟎𝟐𝟓𝟎) = 𝟐,𝟒𝟎 𝒙 𝟏𝟎−𝟔
a=2

𝟎,𝟎𝟎𝟓𝟎 𝒃 𝟏,𝟐𝟎 𝒙 𝟏𝟎−𝟔


Cálculo da ordem de reação em relação ao gás hidrogênio, entre o experimento I e II, temos: (𝟎,𝟎𝟏𝟎𝟎) = 𝟐,𝟒𝟎 𝒙 𝟏𝟎−𝟔
b=1

Equação da velocidade: 𝒗𝒓𝒆𝒂çã𝒐 = 𝒌 𝒙 [𝑵𝑶]² 𝒙 [𝑯𝟐 ]

Cálculo da ordem global: 2 + 1 = 3 (terceira ordem)

527
1600 FÍSICO-QUÍMICA APLICADA EXERCÍCIOS COMENTADOS - IME – ITA – OLIMPÍADA

Questão 997 – (U.S. NATIONAL CHEMISTRY OLYMPIAD) Esta reação é de primeira ordem em relação ao N2O5. 2
N2O5(g) → 4 NO2(g) + O2(g). Se esta reação é de primeira ordem e o tempo de meia – vida é igual a 19,0 minutos, qual
é a constante cinética?
a) 0,0158 min–1
b) 0,0263 min–1
c) 0,0365 min–1
d) 0,0526 min–1

Resolução: Alternativa C.
ln2
Cálculo da constante cinética, a partir do tempo de meia-vida de uma cinética de primeira ordem: t 1⁄ = k
2

ln2 0,693
k= = = 0,0365 min−1
t 1⁄ 19,0
2

Questão 998 – A decomposição térmica do N2O4 em NO2 é uma reação química que apresenta uma cinética de
primeira ordem, apresentando uma constante cinética k igual a 4,5 x 103 s-1 a uma temperatura de 1°C com uma
Ea
energia de ativação 58 kJ.mol-1. Em que temperatura o valor de k seria 1,0 x 104 s-1? Dados: k = A. e-RT ; ln (4,5) =
1,5; ln (10) = 2,3.
a) 300 K
b) 287 K
c) 273 K
d) 213 K
e) 200 K

Resolução: Alternativa B.
Equação química: N2O4(g) → 2 NO2(g).
Dados para o cálculo da Equação de Arrhenius: k1(274 K) = 4,50 x 103 s-1; k2(T2) = 1,0 x 104 s-1; Eat = 58000 J/mol
k Eat 1 1
Cálculo da T2, aplicando a equação de Arrhenius: ln ( 1 ) = x[ − ]
k2 R T2 T1

4,50 x 103 58000 1 1


ln ( ) = x [ − ]
1,0 x 104 8,314 T2 274

T2 = 287 K

Questão 999 – (ITA) Um equilíbrio químico genérico representado por A + B ⇆ C + D, pode ser discutido em termos
de um diagrama do tipo apresentado abaixo.

Qual das opções abaixo explica o comportamento observado quando da adição de um catalisador?
a) só aumenta x

528
1600 FÍSICO-QUÍMICA APLICADA EXERCÍCIOS COMENTADOS - IME – ITA – OLIMPÍADA

b) só diminui z
c) só diminui y
d) só diminuem y e z
e) diminuem igualmente z, y e z.

Resolução: Alternativa C.
A presença do catalisador diminui a energia de ativação, ou seja, haverá a diminuição do parâmetro y.

Questão 1000 - (ITA) Uma certa reação química é representada pela equação: 2A(g) + 2B(g) → C(g), onde “A”,”B” e “C”
significam as espécies químicas que são colocadas para reagir. Verificou-se experimentalmente, numa certa
temperatura, que a velocidade desta reação quadruplica com duplicação da concentração da espécie “A”, mas não
depende das concentrações das espécies “B” e ”C”. Assinale a opção que contém, respectivamente, a expressão
correta da velocidade e o valor correto da ordem da reação.
a) v = k[A]2.[B]2 e 4;
b) v = k[A]2.[B]2 e 3;
c) v = k[A]2.[B]2 e 2;
d) v = k[A]2 e 4;
e) v = k[A]2 e 2.

Resolução: Alternativa E.
Como a velocidade da reação quadruplica com a duplicação da concentração da quantidade de matéria o reagente
A, então, a ordem de reação em relação a este reagente será de segunda ordem. Diante da mudança da velocidade
com a mudança da concentração do reagente A, e não havendo nenhuma mudança em relação ao reagente B,
conclui-se que o reagente B é de ordem zero. Logo, a equação da velocidade será: vreação = k x [A]² x [B]0 , ou
seja, vreação = k x [A]², com ordem global igual a dois.

Questão 1001 – (ITA) O processo físico de transformação do milho em pipoca pode ser um exemplo de reação
química. Se for assim entendido, qual é a ordem dessa reação, considerando um rendimento de processo de 100%?
a) Zero
b) Um
c) Dois
d) Três
e) pseudozero

Resolução: Alternativa B.
Considerando a seguinte reação química de transformação do milho em pipoca: Milho → pipoca.
A velocidade de cozimento do milho vai depender da quantidade de milho, o que deve ser representado através da
seguinte equação matemática:
vreação = k x N, onde: vreação = velocidade da reação química; k = constante cinética; N = quantidade de milho.
Considerando que na formação da pipoca a partir do milho ocorre uma transformação na qual a quantidade de pipoca
gerada é tanto maior quanto maior for o número de grãos do milho, podemos fazer uma analogia com uma reação
química cuja velocidade é maior quanto maior a concentração dos reagentes. Desta maneira, em t = 0, temos uma
velocidade máxima e esta velocidade diminui ao longo do tempo com a diminuição do número de grãos de milho.
Além disso, a transformação que ocorre não depende dos choques entre uma partícula de milho e outra (como seria
um típico mecanismo de uma reação de uma ordem superior). Portanto, pode-se fazer uma analogia entre a
transformação de milho em pipoca com uma reação cuja cinética seja de primeira ordem.

Questão 1002 - (ITA) Considere a reação representada pela equação química 3A(g) + 2B(g) → 4E(g). Essa reação
ocorre em várias etapas, sendo que a etapa mais lenta corresponde a reação representada pela seguinte equação
∆[𝐴]
química: A(g) + C(g) → D(g). A velocidade inicial desta última reação pode ser expressa por - ∆𝑡 = 5,0 𝑚𝑜𝑙. 𝑠 −1 .
Qual é a velocidade inicial da reação (mol.s-1) em relação a espécie E?

529
1600 FÍSICO-QUÍMICA APLICADA EXERCÍCIOS COMENTADOS - IME – ITA – OLIMPÍADA

a) 3,8
b) 5,0
c) 6,7
d) 20
e) 60

Resolução: Alternativa C.
Equação química: A(g) + C(g) → D(g)
1 𝑑[𝐴] 1 𝑑[𝐶]
Relação da velocidade da reação (vreação) com os componentes gasosos: 𝑣𝑟𝑒𝑎çã𝑜 = − 1 𝑥 𝑑𝑡
= − 1𝑥 𝑑𝑡
=
𝑑[𝐷]
+ 𝑑𝑡

𝑣𝑟𝑒𝑎çã𝑜 = − 𝑣𝐴 = − 𝑣𝐶 = + 𝑣𝐷
𝑚𝑜𝑙
𝑣𝑟𝑒𝑎çã𝑜 = −5 = − 𝑣𝐶 = + 𝑣𝐷
𝑠
Através da primeira equação química, temos: 3 A(g) + 2 B(g) → 4 E(g)
1 𝑑[𝐴] 1 𝑑[𝐶] 1 𝑑[𝐷]
𝑣𝑟𝑒𝑎çã𝑜 = − 𝑥 = − 𝑥 =+ 𝑥
3 𝑑𝑡 2 𝑑𝑡 4 𝑑𝑡
1 1 1
𝑣𝑟𝑒𝑎çã𝑜 = − 𝑥 𝑣𝐴 = − 𝑥 𝑣𝐶 = + 𝑥 𝑣𝐷
3 2 4
1 𝑑[𝐴] 1 𝑑[𝐶] 1 𝑑[𝐷]
Cálculo da velocidade em relação ao produto D: 𝑣𝑟𝑒𝑎çã𝑜 = − 3 𝑥 𝑑𝑡
= − 2𝑥 𝑑𝑡
= + 4𝑥 𝑑𝑡

1 𝑚𝑜𝑙 1 1
𝑣𝑟𝑒𝑎çã𝑜 = − 𝑥 (5 ) = − 𝑥 𝑣𝐶 = + 𝑥 𝑣𝐷
3 𝑠 2 4
1 𝑚𝑜𝑙 1
− 𝑥 (5 ) = + 𝑥 𝑣𝐷
3 𝑠 4
20 𝑚𝑜𝑙
𝑣𝐷 = = 6,67
3 𝑠

Questão 1003 – (OLIMPÍADA MARANHENSE DE QUÍMICA) A fabricação industrial do ácido sulfúrico, H2SO4, é
realizada a partir de enxofre, oxigênio e água, em três etapas, representadas pelo diagrama energético abaixo:

S + O2

A indústria usa um catalisador para aumentar a velocidade da fabricação do ácido sulfúrico. É correto afirmar que
o catalisador aumenta a velocidade:
a) da primeira etapa.
b) da segunda etapa.
c) da terceira etapa.
d) das três etapas.
e) das duas primeiras etapas

Resolução: Alternativa A.

530
1600 FÍSICO-QUÍMICA APLICADA EXERCÍCIOS COMENTADOS - IME – ITA – OLIMPÍADA

O catalisador apresenta como principal objetivo a diminuição da energia de ativação, que consequentemente aumenta
a velocidade de uma reação química. Logo, a primeira etapa apresenta a presença do catalisador. Observando a
Figura abaixo, podemos constatar que a menor energia de ativação (menor seta) trata-se da etapa 1.

Questão 1004 – (U.S. NATIONAL CHEMISTRY OLYMPIAD) A reação CHCl3(g) + Cl2(g) → CCl4(g) + HCl(g) é
processada através do seguinte mecanismo:
Cl2(g) → 2Cl(g) (Etapa Rápida)
Cl(g) + CHCl3(g) → HCl(g) + CCl3(g) (Etapa Lenta)
CCl3(g) + Cl(g) → CCl4(g) (Etapa Rápida)
Qual é a equação da taxa de velocidade consistente com este mecanismo?
a) Taxa = k.[Cl2]
b) Taxa = k.[Cl].[CHCl3]
c) Taxa = k.[Cl2].[CHCl3]
d) Taxa = k.[Cl2]1/2.[CHCl3]

Resolução: Alternativa B.
A etapa determinante é a etapa lenta. Logo a equação da velocidade é dada pela seguinte relação, vreação =
k x [Cl] x [CHCl3 ].
Questão 1005 – (U.S. NATIONAL CHEMISTRY OLYMPIAD) Uma substância decompõe-se através de uma reação
de primeira ordem, com uma constante de taxa igual a 6,70 x 10-4.s-1. Se a concentração inicial da substância é 1,50
mol.L-1, qual é a concentração após 500 s?
a) 1,07 mol.L-1
b) 0,503 mol.L-1
c) 0,335 mol.L-1
d) 0,128 mol.L-1

Resolução: Alternativa A.
[A]
Cálculo da concentração da quantidade de matéria final, a partir da cinética de primeira ordem: ln ([A] Final ) =
Inicial
−k x t
[A]Final
ln ( ) = −6,70 x 10−4 x 500
1,50

[A]Final
= 0,715
1,50

531
1600 FÍSICO-QUÍMICA APLICADA EXERCÍCIOS COMENTADOS - IME – ITA – OLIMPÍADA

𝑚𝑜𝑙
[A]Final = 1,07
𝐿
Questão 1006 – (U.S. NATIONAL CHEMISTRY OLYMPIAD) Uma reação apresenta uma constante cinética igual a
k = 8,54 x 10-4 L.mol-1.s-1 a uma temperatura de 45°C e energia de ativação, Eat = 90,8 kJ.mol-1. Qual é o valor da
constante cinética k a 25°C?
a) 4,46 x 10-5 L.mol-1.s-1
b) 8,54 x 10-5 L.mol-1.s-1
c) 8,52 x 10-4 L.mol-1.s-1
d) 8,54 x 10-3 L.mol-1.s-1

Resolução: Alternativa B.
Cálculo da constante cinética a 25°C: 45°C (318 K) → k1 = 8,54 x 10-4 L.mol-1.s-1; 25°C (298 K) → k2 = ?
k1 Eat 1 1
ln ( )= x[ − ]
k2 R T2 T1
8,54 x 10−4 90800 1 1
ln ( )= x[ − ]
k2 8,314 298 318
L
k 2 = 8,60 x 10−5
mol x s
Questão 1007 - (ITA) A figura mostra o perfil reacional da decomposição de um composto X por dois caminhos
reacionais diferentes, I e II.

Baseado nas informações apresentadas nessa figura, assinale a opção errada.


a) O caminho reacional II envolve duas etapas.
b) A quantidade de energia liberada pelo caminho reacional I é igual à do caminho reacional II.
c) O composto K é um intermediário no processo reacional pelo caminho II.
d) O caminho reacional I mostra que a decomposição de X é de primeira ordem.
e) O caminho reacional II refere-se à reação catalisada.

Resolução: Alternativa D.

a) Verdadeiro. O caminho reacional II envolve realmente duas etapas, com transformação de 2X + 2Y + 2W → X +2T
+ K na primeira etapa e X + 2T + K → 2T + Z + 2Y + 2W na segunda etapa.
b) Verdadeiro. A energia liberada nos dois caminhos é igual, pois é a diferença entre os patamares dos reagentes e
dos produtos, e nos dois casos temos os mesmos patamares.
c) Verdadeiro. Durante o processo dois pode-se identificar a presença de K durante o processo, mas não ao final,
caracterizando assim K como um intermediário.

532
1600 FÍSICO-QUÍMICA APLICADA EXERCÍCIOS COMENTADOS - IME – ITA – OLIMPÍADA

d) Falso. Não é possível caracterizar a ordem do processo no gráfico fornecido. É necessária a avaliação das
variações de concentrações no tempo.
e) Verdadeiro. A reação I, em presença de 2Y + 2W (catalisador), é de fato a reação II, que ocorre com menor energia
de ativação que a reação I, evidenciando a catálise.
Questão 1008 - (U.S. NATIONAL CHEMISTRY OLYMPIAD) The rate constant k for the reaction shown os 1,63 x 10-
6 L.mol-1.s-1. What is the overall order of the reaction? 2 ICl + H
(g) 2(g) → HCl(g) + I2(g)
a) 0
b) 1
c) 2
d) 3

Resolução: Alternativa C.
Para este exercício a determinação da ordem de reação tem que ser obtido a partir da unidade da constante cinética.
Para este caso a unidade é L.mol-1.s-1, o processo apresenta cinética de segunda ordem.

Questão 32 - (ITA) A reação hipotética A(s) + B(aq) → C(g) + D(aq) + E(l) é autocatalisada por C(g). Considerando que
essa reação ocorre em sistema fechado, volume constante e sob atmosfera inerte, assinale a opção que apresenta a
curva que melhor representa a variação da massa de A(s), mA, em função do tempo, desde o início da reação até
imediatamente antes do equilíbrio químico ser estabelecido dentro do sistema.

Resolução: Alternativa E. Conforme dados do enunciado, o sistema é fechado com volume constante. A equação
química em questão é a seguinte: A(s) + B(aq) → C(g) + D(aq) + E(liq). Como o enunciado informa que a reação é
autocatalisada pela espécie C(g), concluímos que a velocidade inicial da decomposição de A(s) é relativamente
pequena. Conforme a reação se processa, temos a formação do catalisador (e produto) C(g). A velocidade de
decomposição de A(S) aumenta rapidamente pela presença do catalisador. Porém, como a reação é reversível,
próximo ao equilíbrio temos que a velocidade de decomposição da espécie A(s) começa a se igualar com a velocidade
da formação de A(S) (produtos formando novamente os reagentes), consequentemente, a massa da espécie A(s) tende
a ficar constante.

533
1600 FÍSICO-QUÍMICA APLICADA EXERCÍCIOS COMENTADOS - IME – ITA – OLIMPÍADA

Questão 1009 – (ITA) A decomposição química de um determinado gás A(g) é representada pela equação: A(g) → B(g)
+ C(g). A reação pode ocorrer numa mesma temperatura por dois caminhos diferentes (I e II), ambos com lei de
velocidade de primeira ordem. Sendo v a velocidade da reação, k a constante de velocidade, ΔH a variação de entalpia
da reação e t1/2 o tempo de meia-vida da espécie A, é correto afirmar que:
a) ∆HI < ∆HII
k (t1/2 )
b) k I = II
II (t1/2 )
I
[B] x [C]
c) kI = [A]
[B] x [C]
d) vII = kII x [A]
v k
e) v I = kII
II I

Resolução: Alternativa B.
A partir do tempo de meia-vida para uma reação química com cinética de primeira ordem,
𝑙𝑛2
𝑡1⁄ = 𝑘
, temos:
2

Para o caminho I: ln2 = (t 1⁄ ) x k I


2 I

Para o caminho II: ln2 = (t 1⁄ ) x k II


2 II

Igualando as equações (caminhos): (t 1⁄ ) x k I = (t 1⁄ ) x k II


2 I 2 II

k I (t 1⁄2 )II
=
k II (t 1 )
⁄ 2 I

Questão 1010 – (U.S. NATIONAL CHEMISTRY OLYMPIAD) Uma reação de primeira ordem apresenta uma
constante de velocidade igual a 0,320 min-1. Para uma concentração de reagente inicial igual a 1,22 mol.L-1, determine
o tempo necessário para que a concentração caia para 0,150 mol.L-1?
a) 0,671 min
b) 2,60 min
c) 6,55 min
d) 25,4 min

Resolução: Alternativa C

[A]
Cálculo do tempo, a partir da equação cinética de primeira ordem: ln ([A] Final ) = −k x t
Inicial

0,150
ln ( ) = −(0,320)x t
1,22
2,095
t= = 6,55 min
0,320

534
1600 FÍSICO-QUÍMICA APLICADA EXERCÍCIOS COMENTADOS - IME – ITA – OLIMPÍADA

Questão 1011 - (U.S. NATIONAL CHEMISTRY OLYMPIAD) Qual gráfico é linear para uma reação de segunda ordem
em relação a [A]?
a) [A] vs time
b) ln[A] vs time
c) [A]² vs time
d) 1/[A] vs time

Resolução: Alternativa D.

Expressão matemática que relaciona a concentração com o tempo, para uma reação que apresenta cinética de
segunda ordem: A(g → P1 + P2
𝑑[𝐴]
− 𝑑𝑡
= 𝑘 𝑥 [𝐴]2 (Equação diferencial para uma cinética de segunda ordem)
[𝐴] 𝑑[𝐴]
Separando as variáveis, temos: ∫[𝐴] 𝐹𝑖𝑛𝑎𝑙 [𝐴]2
= − 𝑘 𝑥 𝑑𝑡
𝑖𝑛𝑖𝑐𝑖𝑎𝑙

[𝐴]𝐹𝑖𝑛𝑎𝑙 [𝐴]−2+1 𝑡
∫ 𝑑[𝐴] = − 𝑘 𝑥 ∫ 𝑑𝑡
[𝐴]𝑖𝑛𝑖𝑐𝑖𝑎𝑙 −1 0

[𝐴]𝐹𝑖𝑛𝑎𝑙 𝑡
1
∫ 𝑑[𝐴] = − 𝑘 𝑥 ∫ 𝑑𝑡
[𝐴]𝑖𝑛𝑖𝑐𝑖𝑎𝑙 [𝐴] 0

1 1
− = −𝑘 𝑥 𝑡
[𝐴]𝐹𝑖𝑛𝑎𝑙 [𝐴]𝐼𝑛𝑖𝑐𝑖𝑎𝑙
1 1
= + −𝑘𝑥𝑡
[𝐴]𝐹𝑖𝑛𝑎𝑙 [𝐴]𝐼𝑛𝑖𝑐𝑖𝑎𝑙
1
Chamando [𝐴] = 𝑦.
𝐹𝑖𝑛𝑎𝑙

Questão 1012 – (ITA) Considere as seguintes equações que representam reações químicas genéricas e suas
respectivas equações de velocidade:
I. A → produtos; VI = kI [A]
II. 2B → produtos; VII = kII [B]2
Considerando que, nos gráficos, [X] representa a concentração de A e de B para as reações I e II, respectivamente,
assinale a opção que contém o gráfico que melhor representa a lei de velocidade das reações I e II,

a) 1
II
[x]
I

Tempo

b) [x]

II

Tempo

535
1600 FÍSICO-QUÍMICA APLICADA EXERCÍCIOS COMENTADOS - IME – ITA – OLIMPÍADA

c) [x]

II
Tempo

d) 1
[x]

I
II

Tempo

d) In [x]

II

Tempo

Resolução: Alternativa A.
Durante a reação a concentração diminui com o tempo, logo o inverso da concentração da quantidade de matéria
aumenta com o tempo. Para ambos os casos apresentados, a única alternativa é a letra A.
Questão 1013 – (ITA) O 214Pb82 desintegra-se por emissão de partículas Beta, transformando-se em 214Bi83 que, por
sua vez, se desintegra também por emissão de partículas Beta, transformando-se em 214Po84. A figura ao lado mostra
como varia, com o tempo, o número de átomos, em porcentagem de partículas, envolvidos nestes processos de
desintegração. Admita ln 2 = 0,69. Considere que, para estes processos, sejam feitas as seguintes afirmações:

100
Números de átomos (%)

80

60

0 20 40 60 80 100
Tempo (min)

I. O tempo de meia-vida do chumbo é de aproximadamente 27 min.


II. A constante de velocidade da desintegração do chumbo é de aproximadamente 3 x 10-2 min-1.
III. A velocidade de formação de polônio é igual à velocidade de desintegração do bismuto.
IV. O tempo de meia-vida do bismuto é maior que o do chumbo.
V. A constante de velocidade de decaimento do bismuto é de aproximadamente 1 x 10-2 min-1.
Das afirmativas acima, estão corretas:
a) apenas I, II e III
b) apenas I e IV
c) apenas II, III e V.
d) apenas III e IV
e) apenas IV e V

Resolução: Alternativa A.
I. Verdadeiro. Observando o gráfico, o chumbo varia de 100% a 50% em um tempo igual a 27 min.

536
1600 FÍSICO-QUÍMICA APLICADA EXERCÍCIOS COMENTADOS - IME – ITA – OLIMPÍADA

𝑙𝑛2
II. Verdadeiro. 𝑘 = 27 = 0,03 𝑚𝑖𝑛−1
III. Verdadeiro. Pois para cada átomo de bismuto desintegrado gera um átomo de polônio, assim: v formação
(Po) = vdesintegração (Bi).
IV. Falso. De acordo com o gráfico no instante t  30 min o percentual de bismuto atinge um máximo. Nesse
ponto, a velocidade líquida total de formação do bismuto é igual a zero. Equacionando o problema, temos:
vBi = vformação(Bi) – vdesintegração(Bi) = 0. Mas sabe-se que:

vformação(Bi) = vdesitnegração(Pb) = KPb x NPb

vdesintegração(Bi) = KBi x NBi

v(Bi) = KPb x NPb – KBi x NBi = 0

Em t = 30 min: NPb  50% e NBi  30%

Assim: KPb x 50 – KBi x 30 = 0

KBi = (50/30) x KPb


KBi = (5/3) x 3 x 10-2 = 5 x 10-2 min-1 > KPb
Como a constante de velocidade do bismuto é maior, seu tempo de meia-vida é menor.
V. Falso. KBi = 5 x 10-2.
Observação: A visualização do gráfico não dá a certeza do tempo de meia-vida do chumbo em 27 minutos podendo
o mesmo variar no intervalo de 25 – 30 min.

Questão 1014 – (ITA) A figura a seguir representa o resultado de dois experimentos diferentes (I) e (II) realizados
para uma mesma reação química genérica (reagentes → produtos). As áreas hachuradas sob as curvas representam
o número de partículas reagentes com energia cinética igual ou maior que a energia de ativação da reação (Eat).

Baseado nas informações apresentadas nesta figura é correto afirmar que


a) a constante de equilíbrio da reação nas condições do experimento I é igual à da reação nas condições do
experimento II
b) a velocidade medida para a reação nas condições do experimento I é maior que a medida nas condições do
experimento II
c) a temperatura do experimento I é menor que a temperatura do experimento II.
d) a constante de velocidade medida nas condições do experimento I é igual à medida nas condições do experimento
II
e) a energia cinética média das partículas, medida nas condições do experimento I, é maior que a medida nas
condições do experimento II

537
1600 FÍSICO-QUÍMICA APLICADA EXERCÍCIOS COMENTADOS - IME – ITA – OLIMPÍADA

Resolução: Alternativa C.

Alternativa a) Incorreta, pois a constante de equilíbrio depende do conhecimento da concentração dos produtos e dos
reagentes da reação, o que não pode ser deduzido do gráfico acima, que apresenta apenas a relação entre a energia
cinética das partículas e a quantidade de partículas com determinados níveis de energia em ambos os experimentos.
Alternativa b) Incorreta, pois o experimento II apresenta maior quantidade de partículas com energia maior ou igual à
energia de ativação da reação, o que implica em um maior número de colisões efetivas e portanto em uma maior
velocidade de reação.
Alternativa c) Correta, pois maior quantidade de partículas com nível de energia cinética alto no experimento II significa
maior temperatura a que aquelas partículas estão submetidas.
Alternativa d) Incorreta, conforme a explicação da alternativa b.
Alternativa e) Incorreta, pois o experimento II apresenta uma quantidade de partículas com alto nível de energia
cinética bem maior do que o experimento I, o qual apresenta a maior parte de suas partículas em condições de baixo
nível de energia.

Questão 1015 – (ITA) A figura a seguir mostra como o valor do logaritmo da constante de velocidade (k) da reação
𝑘
representada pela equação química A →
⏞ R varia com o recíproco da temperatura.

Considere que, em relação às informações mostradas na figura, sejam feitas as afirmações seguintes:
I. O trecho a – b da curva mostra a variação de ln k da reação direta (A → R) com o recíproco da temperatura,
enquanto o trecho b – c mostra como varia ln k da reação inversa (R → A) com o recíproco da temperatura.
II. Para temperaturas menores que Tb, o mecanismo controlador da reação em questão é diferente daquele para
temperaturas maiores que Tb.
III. A energia de ativação da reação no trecho a – b é menor que a no trecho b – c.
IV - A energia de ativação da reação direta (A → R) é menor que a da reação inversa (R → A).
Das afirmações acima, está(ão) correta(s):
a) apenas I e IV
b) apenas I, II e IV
c) apenas II
d) apenas II e III
e) apenas III

Resolução: Alternativa D.
I. Incorreta. Tanto o trecho a – b quanto no trecho b – c, o gráfico mostra a variação de ln k com a reação direta (A →
R).
II. Correta. A partir da utilização da equação de Arrhenius o gráfico ln k versus (1/T) é uma reta, a menos que haja
alteração da energia de ativação, pois ki e R são constantes. Nota-se que a energia de ativação para temperaturas
menores que Tb é diferente daquela para temperaturas maiores que Tb (segmentos de reta com diferentes inclinações),
logo o mecanismo controlador da reação é diferente para temperaturas menores que Tb do que aquele para
temperaturas maiores que Tb.

538
1600 FÍSICO-QUÍMICA APLICADA EXERCÍCIOS COMENTADOS - IME – ITA – OLIMPÍADA

III. Correta. A inclinação do segmento de reta no trecho a – b é menor do que no trecho b – c, indicando um menor
coeficiente angular, isto é, um menor valor para Ea (vide equação do item II).
IV. Incorreta. Nada podemos afirmar sobre a energia de ativação da reação inversa.
Questão 1016 – (ITA) Considere quatro séries de experimentos em que quatro espécies químicas (X, Y, Z e W)
reagem entre si, à pressão e temperatura constantes. Em cada série, fixam-se as concentrações de três espécies e
varia-se a concentração (C0) da quarta. Para cada série, determina-se a velocidade inicial da reação (v0) em cada
experimento. Os resultados de cada série são apresentados na figura, indicados pelas curvas X, Y, Z e W,
respectivamente. Com base nas informações fornecidas, assinale a opção que apresenta o valor CORRETO da ordem
global da reação química.

a) 3
b) 4
c) 5
d) 6
e) 7

Resolução: Alternativa C.
É sabido que v0 = k.[X]x.[Y]y.[Z]z.[W]w, com x, y, z e w determinados experimentalmente, assim temos:
log v0 = log k + x.log[X] + y.log[Y] + z.log[Z] + w.log[W]
Mantendo a temperatura constante e a concentração da quantidade de matéria de todas as substâncias com exceção
de uma, temos, que, quando log [ESPÉCIE] passa de -1 a -0,7 (varia 0,3 unidades):
I) para variação de [X]: log v0 se mantém constante. Assim, x = 0;
II) para variação de [Y]: log v0 passa de -1 para -0,7 (varia 0,3 unidades). Assim, y = 1;
III) para variação de [Z]: log v0 passa de -1 para -0,4 (varia 0,6 unidades). Assim, z = 2;
IV) para variação de [W]: log v0 passa de -0,8 para -0,2 (varia 0,6 unidades). Assim, w = 2.
Portanto, a lei da velocidade da reação é: Vreação = k[X]0.[Y]1.[Z]2.[W]2
Como a soma x + y + z + w corresponde a ordem de reação global, temos:
Ordem global = 0 + 1 + 2 + 2 = 5
Questão 1017 – (ITA) A figura apresenta cinco curvas (I, II, III, IV e V) da concentração de uma espécie X em função
do tempo. Considerando uma reação química hipotética representada pela equação X(g) → Y(g), assinale a opção
correta que indica a curva correspondente a uma reação química que obedece a uma lei de velocidade de segunda
ordem em relação à espécie X.

539
1600 FÍSICO-QUÍMICA APLICADA EXERCÍCIOS COMENTADOS - IME – ITA – OLIMPÍADA

a) Curva I
b) Curva II
c) Curva III
d) Curva IV
e) Curva V

Resolução: Alternativa E.

Se a reação química obedece a uma lei de velocidade de segunda ordem em relação à espécie química X, temos:
vreação = k.[X]2.

d[X]
vreação = −
dt
d[X]
− = k x [X]2
dt
d[X]
+ k x [X]2 = 0
dt
d[X]
Assim, temos que a solução desta equação diferencial deve ser da forma: [X] = at -1. Verificando na equação: +
dt
k x [X]2 = 0
−at 2 + ka2 t −2 = 0

a. k − 1 = 0

1
a=
k

Assim, [X] = (kt)-1, ou seja, esta expressão representa uma hipérbole equilátera. Portanto a curva correspondente é a
curva V.

Questão 1018 – (ITA) Um recipiente fechado contendo a espécie química A é mantido a volume (V) e temperatura
(T) constantes. Considere que essa espécie se decomponha de acordo com a equação: A(g) → B(g) + C(g). A tabela
abaixo mostra a variação da pressão total (Pt) do sistema em função do tempo (t):
Tempo (s) 0 55 220 380 495 640 820
Pt (mm Hg) 55 60 70 80 85 90 95
Considere sejam feitas as seguintes afirmações:
I. A reação química obedece à lei de velocidade de ordem zero;
II. O tempo de meia-vida da espécie A independe da sua pressão parcial;
III. Em um instante qualquer, a pressão parcial de A, PA, pode ser calculada pela equação: PA = 2P0 - Pt, em que P0 é
a pressão do sistema no instante inicial.
IV. No tempo de 640 s, a pressão Pi é igual a 45mm Hg, em que Pi é a soma das pressões parciais de B e C.
Então, das afirmações acima, está(ão) correta(s):
a) apenas I e II.
b) apenas I e IV.
c) apenas II e III.
d) apenas II e IV.
e) apenas IV.

540
1600 FÍSICO-QUÍMICA APLICADA EXERCÍCIOS COMENTADOS - IME – ITA – OLIMPÍADA

Resolução: Alternativa E.

Tabela de equilíbrio químico, levando em consideração que a pressão inicial do composto gasoso A(g) é igual a P0.

A(g) B(g) C(g)


Início p0 0 0
Rege -p +p +p
Equilíbrio p0 - p +p +p

A pressão total (Ptotal) é dada por: Ptotal = p0 − p + p + p = p0 + p. Logo, a diferença entre a pressão num
instante qualquer e a pressão inicial fornece o valor da pressão parcial da espécie C ou da espécie B. Desta maneira,
a variação da pressão no tempo fornece a velocidade de formação de B e C, as quais são iguais à velocidade de
consumo de A.
A afirmação I está incorreta. Observando os dados da tabela, tem-se que a taxa de variação da pressão não é
constante nos intervalos de tempo. Com isso, a velocidade de consumo de A não é constante.
A afirmação II está correta. A velocidade de consumo de A indica ocorrência de fenômeno de primeira ordem.

Variação da pressão parcial de A(g)

60

50
Pressão (mmHg)

40

30

20

10

0
0 100 200 300 400 500 600 700 800 900
Tempo (s)

Pressão parcial A (mmHg)

Variação da pressão parcial de A(g)

4,50
4,00
3,50
Ln (Pressão A)

3,00
2,50
2,00
1,50
1,00
0,50
0,00
0 100 200 300 400 500 600 700 800 900
Tempo (s)

(Pressão A)

Os gráficos anteriores indicam velocidade de primeira ordem, pois a variação do ln(PA) é linear com o tempo. Nos
ln2
fenômenos de primeira ordem, o tempo de meia-vida é dado por: t 1⁄ = k
, em que o k é a constante de velocidade.
2

A afirmação III está correta. A pressão de A pode ser obtida por: PA = p0 − p = p + p = p0 + p


PA = P0 − (Ptotal − P0 )
PA = 2. P0 − Ptotal
A afirmação IV está incorreta. No tempo de 640 segundos, a pressão parcial de A é: PA = 2. P0 − Ptotal
PA = (2 x 55) − 90 = 110 mmHg − 90 mmHg = 20 mmHg
Assim, as pressões parciais das espécies gasosas B e C são as seguintes: PA = 20 mmHg

541
1600 FÍSICO-QUÍMICA APLICADA EXERCÍCIOS COMENTADOS - IME – ITA – OLIMPÍADA

PB = PC = 35 mmHg
Já a pressão inicial (p0) será igual a: Ptotal = p0 + p

90 mmHg = p0 + 20 mmHg

p0 = 70 mmHg

Questão 1019 – (ITA) Qual o gráfico que apresenta a curva que melhor representa o decaimento de uma amostra
contendo 10,0 g de um material radioativo ao longo dos anos?

a) b)

c) d)

e)

Resolução: Alternativa B.

Uma amostra de material radioativo sofre decaimento que ocorre proporcionalmente ao número de átomos radioativos
presentes. Desta forma, a curva que apresenta comportamento compatível é a curva do item B.

Questão 1020 – (ITA) Considere a reação química representada pela seguinte equação: 4 NO2(g) + O2(g) → 2 N2O5(g).
Num determinado instante de tempo t da reação, verifica-se que o oxigênio está sendo consumido a uma velocidade
de 2,4 x 10-2 mol L-1 s-1. Nesse tempo t, a velocidade de consumo de NO2 será de
a) 6,0 x 10-3 mol L-1s-1
b) 1,2x10-2 mol L-1s-1
c) 2, 4 x 10-2 mol L-1s-1
d) 4,8 x 10-2 mol L-1s-1
e) 9, 6 x 10-2 mol L-1s-1

Resolução: Alternativa E. Equação química: 4 NO2(g) + O2(g) → 2 N2O5(g)

1 d[NO2 ] 1 d[O2 ] 1
Vreação = − x =− x = + x [N2 O5 ]
4 dt 1 dt 2

542
1600 FÍSICO-QUÍMICA APLICADA EXERCÍCIOS COMENTADOS - IME – ITA – OLIMPÍADA

1 d[NO2 ] 1 1
Vreação = − x = − x (2,40 x 10−2 ) = + x [N2 O5 ]
4 dt 1 2
mol
VNO2 = −4 x (2,40 x 10−2 ) = 9,60 x 10−2
Lxs

Questão 1021 – (ITA) O acidente nuclear ocorrido em Chernobyl (Ucrânia), em abril de 1986, provocou a emissão
radioativa predominantemente de Iodo-131 e Césio - 137. Assinale a opção CORRETA que melhor apresenta os
respectivos períodos de tempo para que a radioatividade provocada por esses dois elementos radioativos decaia para
1% de seus valores iniciais. Considere o tempo de meia-vida do Iodo - 131 igual a 8,1 dias, e do Césio - 137 igual a
30 anos. Dados ln(100) = 4,6; ln(2) = 0,69.
a) 45 dias e 89 anos
b) 54 dias e 201 anos
c) 61 dias e 235 anos
d) 68 dias e 274 anos
e) 74 dias e 296 anos

Resolução: Alternativa B.
ln 2
Iodo-131: Cálculo da constante radioativa do Iodo-131: 𝑡1/2 = 𝜆

ln 2 0,693
λ= = = 8,56 x 10−2 dia−1
t1/2 8,1

Cálculo do tempo: [I]Final = 0,01 x [I]Inicial

0,01 x [I]Inicial
ln ( ) = −8,56 x 10−2 x tempo
[I]Inicial

ln(0,01) = −8,56 x 10−2 x tempo

tempo = 53,80 dias


ln 2
Césio-137: Cálculo da constante radioativa do Césio-137: 𝑡1/2 = 𝜆

ln 2 0,693
λ= = = 0,0231 anos −1
t1/2 30

Cálculo do tempo: [Cs]Final = 0,01 x [Cs]Inicial

0,01 x [Cs]Inicial
ln ( ) = −0,0231 x tempo
[Cs]Inicial

ln(0,01) = −0,0231 x tempo

tempo = 199,36 anos

Questão 1022 – (ITA) Considere a seguinte mecanismo de relação genérica:


A+4 + B+2 → A+3 + B+3 (Etapa lenta)
A+4 + B+3 → A+3 + B+4 (Etapa rápida)
C+ + B+4 → C+3 + B+2 (Etapa rápida)
Com relação a este mecanismo, assinale a opção errada.
a) A relação global é representada pela equação C+ + 2 A+4 → C+3 + 2A+3
b) B+2 é catalisador

543
1600 FÍSICO-QUÍMICA APLICADA EXERCÍCIOS COMENTADOS - IME – ITA – OLIMPÍADA

c) B+3 e B+4 são intermediários da reação.


d) A lei de velocidade é descrita pela equação v = k.[C+].[A+4]
e) A reação é de segunda ordem

Resolução: Alternativa D. A questão apresenta o seguinte mecanismo:


A+4 + B +2 → A+3 + B +3 (Etapa Lenta)
A+4 + B +3 → A+3 + B +4 (Etapa Rápida)

C + + B +4 → C +3 + B +2 (Etapa Rápida)
Somando as três equações químicas:
A+4 + B +2 → A+3 + B +3
A+4 + B +3 → A+3 + B +4

C + + B +4 → C +3 + B +2 +

2A+4 + C +4 → 2A+3 + C +3 (Equação química global)


Com isso a alternativa A está correta.
A espécie B +2 é consumida no início da reação e, ao termino desta, é consumida, sendo por isso caracterizado como
catalisador, tornando correta a alternativa B.
B +3 e B +4 são produzidos e consumidos ao longo do processo, logo são intermediários da reação. Alternativa C está
correta.
A expressão da lei da velocidade de uma reação, quando esta ocorre em mais de uma etapa, é determinada pela
etapa lenta. Daí, temos que a expressão para lei de velocidade é vreação = k x [B +2 ]x [A+4 ] e não vreação =
k x [C + ]x [A+4 ]. Logo a alternativa D está errada, sendo então a resposta a ser assinalada.
A ordem de uma reação é determinada pela soma das atividades de cada participante na lei de velocidade. Então a
reação é de segunda ordem.

Questão 1023 – (ITA) Assinale a opção que apresenta a afirmação correta sobre uma reação genérica de ordem zero
em relação ao reagente X.
a) A velocidade inicial de X é maior que sua velocidade média.
b) A velocidade inicial de X varia com a concentração inicial de X.
c) A velocidade de consumo de X permanece constante durante a reação.
d) O gráfico do logaritmo natural de X versus o inverso do tempo é representado por uma reta.
e) O gráfico da concentração de X versus tempo é representado por uma curva exponencial decrescente.

Resolução: Alternativa C. Numa reação de ordem zero, a velocidade independe da concentração do reagente,
podendo ser representada pela seguinte equação: vreação = k.

544
1600 FÍSICO-QUÍMICA APLICADA EXERCÍCIOS COMENTADOS - IME – ITA – OLIMPÍADA

Questão 1024 – (ITA) Considere uma reação química hipotética representada pela equação X → Produtos. São
feitas as seguintes proposições relativas a essa reação:
I. Se o gráfico de [X] em função do tempo for uma curva linear, a lei de velocidade da reação dependerá somente da
constante de velocidade.
II. Se o gráfico e 1/[X] em função do tempo for uma curva linear, a ordem de reação será 2.
III. Se o gráfico da velocidade da reação em função de [X] for uma curva linear, a ordem de reação será 1.
IV. Se o gráfico da velocidade de reação em função de [X]2 for uma curva linear, a ordem de reação será 2.
Das proposições acima, está(ão) CORRETA(S)
a) apenas I
b) apenas I e Il
c) apenas I, lll e IV
d) apenas III
e) todas

Resolução: Alternativa E.
Δ[X]
A velocidade média de uma reação é dada por: vmédia = Δt

Δ[X] d[X]
Fazendo ∆t → 0, temos a velocidade instantânea: vinstantânea = lim =
∆t→0 Δt dt

Integrando a expressão acima, temos a chamada lei de velocidade integrada, onde V é expresso em função de [X], e
α é a ordem da reação em relação à espécie química X.
d[X]
vreação = = −k x [X]α
dt

I. Verdadeiro. Se [X] varia linearmente com o tempo, então V independe da concentração da quantidade de matéria
de X, [X]. Logo α = 0 e vreação = k.
d[X]
II. Verdadeiro. Se α = 2, temos dt
= −k x [X]2 . Integrando a expressão, chegamos a:

1 1
= +kxt
[X]Final [X]Inicial
1
Logo, o gráfico de [X] em função do tempo será uma reta.

III. Verdadeiro. Se α = 1, temos vreação = −k x [X]1 . Logo, V em função de [X] é uma reta.

IV. Verdadeiro. Se α = 2, temos vreação = −k x [X]2 . Logo, V em função de [X]² é uma reta.

Questão 1025 – (OLIMPÍADA CEARENSE DO ENSINO SUPERIOR DE QUÍMICA) A decomposição térmica da


fosfina (PH3) em fósforo e hidrogênio molecular trata-se de uma reação de primeira ordem. A meia vida da reação é
35,0 s à 680°C. Determine o tempo necessário para a decomposição de 95% da fosfina.
a) t = 151,3 s
b) t = 250 s
c) t = 50 s
d) t = 15,01 s
e) t = 500 s

Resolução: Alternativa A. Equação química: 4 PH3(g) → P4(g) + 6 H2(g)


ln2
Cálculo da constante cinética (k), a partir do tempo de meia-vida (t1/2): 35,0 = k

ln2
k= = 1,98 x 10−2 s −1
35,0

545
1600 FÍSICO-QUÍMICA APLICADA EXERCÍCIOS COMENTADOS - IME – ITA – OLIMPÍADA

Informação fornecida pelo problema: [Fosfina]FINAL = 0,05 x [Óxido]INICIAL

[Fosfina]Final
ln ( ) = −k x tempo
[Fosfina]Inicial

0,05 x [Fosfina]Inicial
ln ( ) = −0,0198 x tempo
[Fosfina]Inicial

ln(0,05 ) = −0,0198 x tempo

ln(0,05 )
tempo =
−0,0198

tempo = 151,3 s

Questão 1026 – (ITA) A respeito de reações químicas descritas pela equação de Arrhenius, são feitas as seguintes
proposições:
I. Para reações bimoleculares, o fator pré-exponencial na equação de Arrhenius é proporcional à frequência de
colisões, efetivas ou não, entre as moléculas dos reagentes.
II. O fator exponencial na equação de Arrhenius é proporcional ao número de moléculas cuja energia cinética
relativa é maior ou igual à energia de ativação de reação.
III. Multiplicando-se o negativo da constante dos gases (–R) pelo coeficiente angular da reta ln k versus 1/T obtém-
se o valor da energia de ativação da reação.
IV. O fator pré-exponencial da equação de Arrhenius é determinado pela intersecção da reta ln k versus 1/T com o
eixo das abcissas.
Das proposições acima, está(ão) errada(s):
a) apenas I
b) apenas I e II
c) apenas I e IV
d) apenas II e III
e) apenas IV

Resolução: Alternativa E.
−Eat
( )
Apresentação da Equação de Arrhenius: k = A x e RxT , onde Eat é a energia de ativação.
I. Correto. O fator pré-exponencial A é proporcional ao total de colisões entre os reagentes.
−Eat
( )
II. Correto. O fator e R x T está ligado à fração de moléculas com energia maior ou igual à energia de ativação,
segundo a distribuição de Boltzmann.
III. Correto. Aplicando o logaritmo natural na equação de Arrhenius, temos:
−Eat
lnk = lnA + ln
RxT
Eat
lnk = lnA −
RxT
E
Coeficiente angular = Rat, multiplicando por (–R) irá resultar na Energia de Ativação.

IV. Incorreto. Pela equação logarítmica acima, o fator pré-exponencial A é obtido a partir do coeficiente linear, isto
1
é, a intersecção da reta lnk x , com o eixo das ordenadas.
T

Questão 1027 – (U.S. NATIONAL CHEMISTRY OLYMPIAD) A 651,5 K, o tempo de meia-vida para a decomposição
de primeira ordem um determinado óxido é de aproximadamente 6,03 minutos, e uma energia de ativação igual a 52
kcal.mol-1. A partir desta informação, assinale a alternativa do tempo aproximado em segundos, necessário para que
o referido óxido se decomponha em 75%, a 723 K.
a) 10,0 segundos

546
1600 FÍSICO-QUÍMICA APLICADA EXERCÍCIOS COMENTADOS - IME – ITA – OLIMPÍADA

b) 10,5 segundos
c) 12,4 segundos
d) 12,8 segundos
e) 13,8 segundos

Resolução: Alternativa E. Cálculo da constante cinética para um tempo de meia-vida igual a 6,03 minutos a 651,50
ln2 ln 2
K: k 651,5 K = t = 6,03 min = 0,115 min−1
1⁄
2

Cálculo da velocidade específica a 723 K, utilizando a equação de Arrhenius: k1 (651,50 K) = 0,115 min-1 e k2 (723 K)
=?

k 651,5 K Eat 1 1
ln ( 723 K ) = x [( )−( )]
k R 723 651,5

0,115 52000 1 1
ln ( 723 K
)= x [( )−( )]
k 1,987 723 651,5

0,115
ln ( ) = −3,97
k 723 K
0,115
= e−3,97
k 723 K
0,115
k 723 K = = 6,12 min−1
1,88 x 10−2

Informação fornecida pelo problema: [Óxido]Final = 0,25 x [Óxido]Inicial


0,25 x [óxido]Inicial
Cálculo do tempo (t) em minutos, a partir da cinética de primeira ordem: ln ( [óxido]Inicial
) = −k x tempo

ln(0,25) = − 6,12 x tempo

ln(0,25) 60 s
tempo = = 0,226 min x ( ) = 13,56 s
−6,12 1 min

Questão 1027 – (IME) Um grupo de alunos desenvolveu um estudo sobre três reações irreversíveis de ordens zero,
um e dois. Contudo, ao se reunirem para confeccionar o relatório, não identificaram a correspondência entre as
colunas da tabela abaixo e as respectivas ordens de reação.
t (s) C1 (mol/L) C2 (mol/L) C3 (mol/L)
200 0,8000 0,8333 0,8186
210 0,7900 0,8264 0,8105
220 0,7800 0,8196 0,8024
230 0,7700 0,8130 0,7945
240 0,7600 0,8064 0,7866
Considere que o modelo ∆C/∆t = -k.Cn descreva adequadamente as velocidades das reações estudadas. Considere
ainda que as magnitudes das constantes de velocidade específica de todas as reações são idênticas à da reação de
segunda ordem, que é 1,0 x10–3L/mol.s. Assim, pode-se afirmar que C1, C2 e C3 referem-se, respectivamente, a reações
de ordem:
a) 1, 2 e 0
b) 0, 1 e 2
c) 0, 2 e 1
d) 2, 0 e 1
e) 2, 1 e 0

547
1600 FÍSICO-QUÍMICA APLICADA EXERCÍCIOS COMENTADOS - IME – ITA – OLIMPÍADA

Resolução: Alternativa C.
Analisando os dados para uma cinética de primeira reação na primeira coluna, é possível perceber que para intervalos
de tempos iguais, a variação de concentração é a igual, o que indica que a primeira reação é de ordem zero. Desta
forma a alternativa correta será a letra B ou C.
1 1
Cinética de segunda ordem: substituindo valores de C(200), C(210) e o t entre 200 e 210, temos: C −C =
Final Inicial
+ k x tempo

1 1
− = + k x 10
0,8264 0,833

0,00956 = k x 10

k = 9,56 x 10−4

Este valor para a constante cinética confirma o valor de k apresentado pelo problema. Com isso, concluímos que a
terceira reação química apresenta cinética de primeira ordem.

Questão 1028 – (OLIMPÍADA BRASILEIRA DE QUÍMICA) Assinale a opção que corresponde à fração de substrato
que reagiu, em uma reação de primeira ordem, após um período de quatro vezes a meia-vida:
a) 15/16
b) 1/16
c) 7/8
d) ¾
e) ¼

Resolução: Alternativa B. Informação fornecida pelo problema: tempo = 4 x (t1/2). Sabendo que o processo cinético
[substrato]
para uma reação de primeira ordem é dada por: ln [substrato] Final = −k x tempo e o tempo de meia-vida para
Inicial
ln2
uma cinética de primeira ordem é definida a partir da seguinte expressão matemática: t 1⁄ = k
. Cálculo da fração
2
[substrato]
de substrato que sofreu reação: ln [substrato] Final = −k x tempo
Inicial

[substrato]Final
ln = −k x 4 x t 1⁄
[substrato]Inicial 2

[substrato]Final ln 2
ln = −k x 4 x ( )
[substrato]Inicial k

[substrato]Final
= e−2,772
[substrato]Inicial

[substrato]Final
= 0,0625
[substrato]Inicial

[substrato]Final 1
=
[substrato]Inicial 16

548
1600 FÍSICO-QUÍMICA APLICADA EXERCÍCIOS COMENTADOS - IME – ITA – OLIMPÍADA

Questão 1029 – (U.S. NATIONAL CHEMISTRY OLYMPIAD) A taxa de formação de O3(g) é igual a 2,0 x 10-7 mol.L-
1.s-1 para a seguinte reação 3 O
2(g) → 2 O3(g). Qual é a taxa de desaparecimento de O2(g) em mol.L s ?
-1 -1

a) 1,3 x 10-7
b) 2,0 x 10-7
c) 3,0 x 10-7
d) 4,5 x 10-7

Resolução: Alternativa C. Equação química: 3 O2(g) → 2 O3(g)

1 d[O2 ] 1 d[O3 ]
vreação = − x = + x
3 dt 2 dt

1 d[O2 ] 1
− x = + x (2,0 x 10−7 )
3 dt 2
1 1
− x vO2 = + x (2,0 x 10−7 )
3 2
mol
vO2 = 3,0 x 10−7
Lxs

Questão 1030 – (U.S. NATIONAL CHEMISTRY OLYMPIAD) Quando a temperatura de uma reação é aumentada de
300 K para 310 K, a taxa de reação dobra. Determine a energia de ativação, Eat, associada com esta reação.
a) 6,45 kJ.mol-1
b) 23,3 kJ.mol-1
c) 53,6 kJ.mol-1
d) 178 kJ.mol-1

Resolução: Alternativa C.

Informações apresentadas pelo problema: k1 (300 K) e k2 (310) = 2 x k1


k Eat 1 1
Cálculo da energia de ativação, a partir da equação de Arrhenius: ln (k1 ) = R
x [T −T ]
2 2 1

k1 Eat 1 1
ln ( )= x[ − ]
2 x k1 8,314 310 300

1 Eat 1 1
ln ( ) = x[ − ]
2 8,314 310 300

J kJ
Eat = 53594,28 (53,59 )
mol mol

Questão 1031 – (OLIMPÍADA CEARENSE DO ENSINO SUPERIOR DE QUÍMICA) Um recipiente contém uma
mistura dos compostos A e B que se decompõem segundo uma cinética de primeira ordem. As meias-vidas são de
50,0 minutos para A e 18,0 minutos para B. Se as concentrações de A e B forem iguais no início, que tempo será
necessário para que a concentração de A seja quatro vezes a concentração de B?
a) 62 min
b) 56 min
c) 68 min
d) 32 min
e) 45 min

Resolução: Alternativa B.

549
1600 FÍSICO-QUÍMICA APLICADA EXERCÍCIOS COMENTADOS - IME – ITA – OLIMPÍADA

𝑙𝑛2
Cálculo da velocidade específica (k) para o gás A: 𝑘 𝐴 = 𝑡
1⁄
2

ln 2
𝑘𝐴 = = 1,39 𝑥 10−2 𝑚𝑖𝑛−1
50 𝑚𝑖𝑛
𝑙𝑛2
Cálculo da velocidade específica para o gás B: 𝑘 𝐵 = 𝑡
1⁄
2

𝑙𝑛2
𝑘𝐵 = = 3,85 𝑥 10−2 𝑚𝑖𝑛−1
18 𝑚𝑖𝑛

Dividindo a equação do gás A, [𝐴]𝐹𝑖𝑛𝑎𝑙 = [𝐴]𝐼𝑛𝑖𝑐𝑖𝑎𝑙 𝑥 𝑒 −𝑘 𝑥 𝑡𝑒𝑚𝑝𝑜 , pela equação do gás B, [𝐵]𝐹𝑖𝑛𝑎𝑙 =
[𝐵]𝐼𝑛𝑖𝑐𝑖𝑎𝑙 𝑥 𝑒 −𝑘 𝑥 𝑡𝑒𝑚𝑝𝑜 , temos:

[𝐴]𝐹𝑖𝑛𝑎𝑙 [𝐴]𝐼𝑛𝑖𝑐𝑖𝑎𝑙 𝑥 𝑒 −𝑘 𝑥 𝑡𝑒𝑚𝑝𝑜


=
[𝐵]𝐹𝑖𝑛𝑎𝑙 [𝐵]𝐼𝑛𝑖𝑐𝑖𝑎𝑙 𝑥 𝑒 −𝑘 𝑥 𝑡𝑒𝑚𝑝𝑜

Informação do problema: [𝐴]𝐼𝑛𝑖𝑐𝑖𝑎𝑙 = [𝐵]𝐼𝑛𝑖𝑐𝑖𝑎𝑙 e [𝐴]𝐹𝑖𝑛𝑎𝑙 = 4 𝑥 [𝐵]𝐹𝑖𝑛𝑎𝑙

[𝐴]𝐹𝑖𝑛𝑎𝑙 [𝐵]𝐼𝑛𝑖𝑐𝑖𝑎𝑙 𝑥 𝑒 −𝑘 𝑥 𝑡𝑒𝑚𝑝𝑜


=
[𝐵]𝐹𝑖𝑛𝑎𝑙 [𝐵]𝐼𝑛𝑖𝑐𝑖𝑎𝑙 𝑥 𝑒 −𝑘 𝑥 𝑡𝑒𝑚𝑝𝑜
−2
4 𝑥 [𝐵]𝐹𝑖𝑛𝑎𝑙 [𝐵]𝐼𝑛𝑖𝑐𝑖𝑎𝑙 𝑥 𝑒 −1,39 𝑥 10 𝑥 𝑡𝑒𝑚𝑝𝑜
=
[𝐵]𝐼𝑛𝑖𝑐𝑖𝑎𝑙 [𝐵]𝐼𝑛𝑖𝑐𝑖𝑎𝑙 𝑥 𝑒 −3,85 𝑥 10−2 𝑥 𝑡𝑒𝑚𝑝𝑜
−2
𝑒 −1,39 𝑥 10 𝑥 𝑡𝑒𝑚𝑝𝑜
4= −2
𝑒 −3,85 𝑥 10 𝑥 𝑡𝑒𝑚𝑝𝑜

−2 + 3,85 𝑥 10−2 )
4 = 𝑒 (−1,39 𝑥 10 𝑥 𝑡𝑒𝑚𝑝𝑜

−2 + 3,85 x 10−2 )
Aplicando o logaritmo neperiano, temos: ln4 = ln{e(−1,39 x 10 x tempo
}

1,386= 2,47 x 10−2 x tempo

1,386
tempo = = 56,11 min
2,47 x 10−2

Questão 1032 – (OLIMPÍADA BRASILEIRA DE QUÍMICA) Considere as assertivas abaixo, que se referem à ação
dos catalisadores:
I. Alteram a velocidade da reação;
II. Diminuem a energia de ativação;
III. Transformam as reações em reações espontâneas;
IV. Deslocam o equilíbrio da reação para o lado dos produtos.
Estão corretas, somente as assertivas:
a) I e II
b) I e III
c) I e IV
d) II e III
e) III e IV

Resolução: Alternativa A.

I. Correto. Com a utilização do catalisador, a velocidade da reação aumenta.

550
1600 FÍSICO-QUÍMICA APLICADA EXERCÍCIOS COMENTADOS - IME – ITA – OLIMPÍADA

II. Correto. Com a utilização do catalisador, a energia de ativação diminui.


III. Incorreto. A utilização do catalisador não influencia em reações a serem ou não espontâneas.
IV. Incorreto. A utilização do catalisador não desloca o equilíbrio da reação para nenhum dos lados, apenas
deixa a reação em equilíbrio mais rapidamente.

As alternativas corretas são os itens I e II.

Questão 1033 – (U.S. NATIONAL CHEMISTRY OLYMPIAD) O ozônio na atmosfera da Terra se decompõe de acordo
com a equação: 2O3(g) → 3O2(g). Esta reação ocorre através de dois mecanismos:
Passo 1: O3(g) = O2(g) + O(g) (Rápido, reversível)
Passo 2: O3(g) + O(g) → 2 O2(g). (Lento)
Qual a lei da taxa é consistente com este mecanismo?
a) –∆[O3]/∆t = k[O3]
b) –∆[O3]/∆t = k[O3]2
c) –∆[O3]/∆t = k[O3]2 /[O2]
d) –∆[O3]/∆t = k[O3]2 /[O2]3

Resolução: Alternativa C.
[O2 ] x [O]
Equação da constante de equilíbrio através da reação reversível: K = [O3 ]

[O2 ] x [O]
Através da equação da constante de equilíbrio, será necessário isolar o intermediário, [O]: K = [O3 ]

K x [O3 ]
[O] =
[O2 ]
Equação de taxa de velocidade: vreação = k a x [O3 ] x [O]

Substituindo a equação do intermediário na equação da taxa: vreação = k a x [O3 ] x [O]

K x [O3 ]
vreação = k a x [O3 ] x
[O2 ]

K x [O3 ]²
vreação = k a x
[O2 ]

k a x K x [O3 ]²
vreação =
[O2 ]

Analisando a velocidade da reação em função da decomposição de ozônio e definindo o produto ka x K = k, temos:


ka x K x [O3 ]²
vreação = [O2 ]

k x [O3 ]²
vreação =
[O2 ]

d[O3 ] k x [O3 ]²
− =
dt [O2 ]

Questão 1034 – (U.S. NATIONAL CHEMISTRY OLYMPIAD – Primeira Fase) A energia de ativação de uma certa
reação é 87 kJ.mol-1. Qual é a proporção das constantes de taxa para esta reação quando a temperatura diminui de
37°C para 15°C?
a) 5/1
b) 8,3/1

551
1600 FÍSICO-QUÍMICA APLICADA EXERCÍCIOS COMENTADOS - IME – ITA – OLIMPÍADA

c) 13/1
d) 24/1

Resolução: Alternativa C.
𝑘 𝐸𝑎𝑡 1 1
Através da aplicação da equação de Arrhenius, temos: 𝑙𝑛 (𝑘37°𝐶 ) = 𝑅
𝑥 [𝑇 −𝑇]
15°𝐶 2 1

k 37°C 87000 1 1
ln ( )= x[ − ]
k15°C 8,314 (15 + 273) (37 + 273)

k 37°C
= e(+2,58) = 13,18
k15°C

552
1600 FÍSICO-QUÍMICA APLICADA EXERCÍCIOS COMENTADOS - IME – ITA – OLIMPÍADA

Questão 944 – (OLIMPÍADA


MUNDIAL DE QUÍMICA) Uma
determinada substância sofre
decomposição segundo uma
cinética de primeira ordem, e
sua dependência em relação
à temperatura segue uma lei
empírica chamada de
equação de Arrhenius. Os
tempos de meia-vida
detCAPÍTULO VII
553
1600 FÍSICO-QUÍMICA APLICADA EXERCÍCIOS COMENTADOS - IME – ITA – OLIMPÍADA

ESTUDO DO EQUILÍBRIO
QUÍMICO – KC e KP

PROFESSOR ALEXANDRE
VARGAS GRILLO

554
1600 FÍSICO-QUÍMICA APLICADA EXERCÍCIOS COMENTADOS - IME – ITA – OLIMPÍADA

Questão 1035 – Defina equilíbrio químico e processo reversível.

Resposta: Equilíbrio químico é o estado que apresenta como característica fundamental a constância das
propriedades macroscópicas. Define-se reação reversível o tipo de reação que se processo de forma simultânea, nos
dois sentidos, tanto pelo sentido direto quanto no sentido inverso. Algumas reações são bem características.
a) Reação de síntese da amônia, a partir dos gases nitrogênio e hidrogênio;
b) Reação de síntese do anidrido sulfúrico, a partir do anidrido sulfuroso e oxigênio molecular;
c) Reação de esterificação entre o ácido acético e o etanol.

Questão 1036 – Deduzir a constante de equilíbrio através de bases cinéticas.

Resposta: Através da equação química hipotética, do tipo aA(g) + bB(g) ⇄ mM(g)+ nN(g). De acordo com a lei da ação
das massas, a velocidade da reação direta é dada através da seguinte equação de velocidade:

vdireta = k direta x [A]a x [B]b , onde kd é a velocidade específica da reação direta.


A velocidade da reação inversa é dada através da seguinte equação de velocidade:
vinversa = k inversa x [M]m x [N]n , onde ki é a velocidade específica da reação inversa.
Quando a reação reversível atinge o equilíbrio químico, a velocidade da reação direta é igual a velocidade da reação
inversa, logo:
vdireta = vinversa

k direta x [A]a x [B]b = k inversa x [M]m x [N]n


k direta [M]m x [N]n
=
k inversa [A]a x [B]b
A razão entre as duas velocidades específicas é definida pela constante K, conhecida como constante de equilíbrio.
Questão 1037 - Explique a relação que existe entre a constante de equilíbrio e as velocidades específicas.

Resposta: Quando um determinado sistema entra em equilíbrio químico, as velocidades da reação direta torna-se
igual a velocidade da reação inversa, logo a relação entre a constante de equilíbrio e as velocidades específicas pode
𝑘
ser representada da seguinte maneira, 𝐾 = 𝑘 𝑑𝑖𝑟𝑒𝑡𝑎 .
𝑖𝑛𝑣𝑒𝑟𝑠𝑎

Questão 1038 – Explique o significado de uma constante de equilíbrio apresentar um valor igual a 10 comparada com
outra, de valor igual a 0,10.

Resposta: Considere a seguinte equação química hipotética representada a seguir:


rReagentes(gasoso) ⇄ pProdutos(gasoso), onde r e p são coeficientes estequiométricos da reação química. Sabendo que
a constante de equilíbrio é definida como a razão da concentração da quantidade de matéria dos produtos pelo
reagente, temos a seguinte equação:
[𝑃𝑟𝑜𝑑𝑢𝑡𝑜𝑠]𝑝
𝐾= [𝑅𝑒𝑎𝑔𝑒𝑛𝑡𝑒𝑠]𝑟
.

Então, para uma constante de equilíbrio igual a 10, significa que a concentração dos produtos será dez vezes maior
[𝑃𝑟𝑜𝑑𝑢𝑡𝑜𝑠]𝑝
em comparação aos reagentes, ou seja, [𝑅𝑒𝑎𝑔𝑒𝑛𝑡𝑒𝑠]𝑟
= 10, ou seja, [Produtos]p = 10 x [Reagentes]r . Para
1
uma constante de equilíbrio igual a 10, significa que a concentração da quantidade de matéria dos reagentes é dez
vezes maior em comparação aos produtos, ou seja, [Reagentes]r = 10 x [Produtos]p . A partir desta análise,

555
1600 FÍSICO-QUÍMICA APLICADA EXERCÍCIOS COMENTADOS - IME – ITA – OLIMPÍADA

vemos que no primeiro caso o equilíbrio químico é deslocado para a direita, favorecendo a reação direta, enquanto
que no segundo caso, o equilíbrio químico é deslocado para a esquerda da reação direta.

Questão 1039 – Deduza a equação matemática que relaciona a constante de equilíbrio Kc com Kp, para uma reação
em fase gasosa. Em que circunstâncias o valor de uma será idêntico ao da outra?

Resposta: Considere a equação química gasosa do tipo, aA(g) ⇄ bB(g), onde a e b são os coeficientes
estequiométricos.
[𝑃𝐵 ]𝑏
Equação da constante de equilíbrio em função das pressões parciais: 𝐾𝑝 = [𝑃𝐴 ]𝑎
.

Considerando que os gases que compõem o sistema reacional apresentam comportamento ideal, temos: pV =
nxRxT
n
p=( ) xRxT
V
p=MxRxT
p
M = RT, onde M é definido como a concentração da quantidade de matéria (mol x L-1). Substituindo na equação da
constante de equilíbrio, temos:
[PB ]b {[B]x R x T}b ([B])b x (R x T)b [B]b (R x T)b
Kp = = = = x = K c x (R x T)b−a
[PA ]a {[A]x R x T}a ([A])a x (R x T)a [A]
⏟a (R x T)a
Kc

Definindo a diferença entre b - a = ∆n, a equação entre a constante de equilíbrio em função das pressões parciais
(Kp) e a constante de equilíbrio em função das concentrações das quantidades de matéria (Kc) é dada pela seguinte
equação matemática: 𝐾𝑝 = 𝐾𝑐 𝑥 (𝑅𝑇)∆𝑛 . As constantes de equilíbrio serão iguais (K p = K c ), quando a variação
do número de mol (∆n) entre os produtos e os reagentes for igual à zero, ∆n = 0.
Questão 1040 – Explique por que as pressões parciais podem ser utilizadas em lugar das concentrações na constante
de equilíbrio de reação envolvendo gases.

Resposta: Considerando uma reação na fase gasosa, é mais conveniente o uso da constante de equilíbrio em função
das pressões parciais, designado pela sigla Kp. Logo, para uma equação hipotética do tipo, aA(g) + bB(g) ⇄ wW(g), a
(𝑃𝑊 )𝑊
constante de equilíbrio fica da seguinte maneira: 𝐾𝑝 = (𝑃 𝑎 𝑏 .
𝐴 ) .(𝑃𝐵 )

Questão 1041 – Qual é a condição termodinâmica em que Kp seja igual a Kc?

Resposta: Sabendo que a relação matemática que relaciona a constante de equilíbrio em função das pressões
parciais (Kp) com a constante de equilíbrio em função das concentrações das molares (K c), é dada pela seguinte
equação: K p = K c x (R. T)∆n, onde:

Kp = constante de equilíbrio em função das pressões parciais;


Kc = constante de equilíbrio em função das concentrações molares;
R = constante dos gases;
T = temperatura absoluta;
∆n = variação do número de mol (número de mol dos produtos – número de mol dos reagentes).
Kp será igual a Kc, quando a variação do número de mol for igual a ZERO, ∆n = 0. Podemos dar como exemplo a
reação de formação do cloreto de hidrogênio, a partir dos componentes gasosos, H2(g) e Cl2(g).

556
1600 FÍSICO-QUÍMICA APLICADA EXERCÍCIOS COMENTADOS - IME – ITA – OLIMPÍADA

Equação química balanceada: H2(g) + Cl2(g) ⇄ 2 HCl(g)

K p = K c x (R x T)[(2)−(1+1)]

K p = K c x (R x T)0

Kp = Kc x 1

Kp = Kc

Questão 1042 – Enuncie o princípio de Le Chatelier.

Resposta: Enunciado: Se um sistema entre reagentes e produtos é perturbado através de uma pequena variação de
uma das variáveis, podendo ser a temperatura, pressão ou concentração, o sistema tende a retornar a um estado de
equilíbrio do qual se encontrava, sempre no sentido de reduzir a perturbação introduzida.

Questão 1043 – As seguintes equações químicas abaixo são exotérmicas.


a) 2 SO2(g) + O2(g) ⇄ 2 SO3(g)
b) N2(g) + 3 H2(g) ⇄ 2 NH3(g)
Explique o efeito de aumentar a temperatura e a pressão sobre a mistura em equilíbrio para cada um dos dois sistemas
reacionais.
Resposta:

Item a) Equação química balanceada: 2SO2(g) + O2(g) ⇄ 2 SO3(g). Como a equação química é exotérmica, logo, com o
aumento da temperatura do sistema, o equilíbrio será deslocado para a esquerda, favorecendo o dióxido de enxofre
e o oxigênio molecular. Aumentando a pressão do sistema, o equilíbrio será deslocado para a direita, favorecendo o
anidrido sulfúrico.

Item b) Equação química balanceada: N2(g) + 3 H2(g) ⇄ 2 NH3(g). Como a reação química é exotérmica, logo, com o
aumento da temperatura do sistema, o equilíbrio será deslocado para a esquerda, favorecendo os gases nitrogênio e
oxigênio. Aumentando a pressão do sistema, o equilíbrio será deslocado para a direita, favorecendo a amônia.

Questão 1044 – Discuta o aumento da temperatura sobre a reação de formação de sulfeto de hidrogênio, a partir dos
seus elementos constituintes mais estáveis, sabendo que a equação da variação da energia de Gibbs é representada
a partir da seguinte equação termodinâmica, ∆G° = - 180688 + 98,83T (J.mol-1). Além disso, explique porque a reação
é endotérmica ou exotérmica?

Resposta: Equação química da formação do sulfeto de hidrogênio: H2(g) + ½ S2(g) ⇄ H2S(g). Através da equação de
Gibbs, ∆G° = -180688 + 98,83 x T (∆G° = ∆H° - ∆S°.T), observa-se que a variação da entalpia é negativa, logo trata-
se de uma reação exotérmica. Com o aumento da temperatura, o equilíbrio é deslocado para a esquerda, favorecendo
os gases hidrogênio e enxofre.

Questão 1045 – Explique a importância da equação de Van´t Hoff, para a dependência de K em função da
temperatura.

Resposta: Van´t Hoff estudou a dependência da constante de equilíbrio em função da temperatura, do qual é
representada pela seguinte equação matemática:
𝐝𝐥𝐧𝐊 ∆𝐇𝟎
𝐝𝐓
= 𝐑𝐓²
(Equação A)

Para determinarmos o valor da constante de equilíbrio na temperatura designado por T2 em termos do seu valor K1
na temperatura T1, será necessário integrar a equação A.

557
1600 FÍSICO-QUÍMICA APLICADA EXERCÍCIOS COMENTADOS - IME – ITA – OLIMPÍADA

dlnK ∆H 0
=
dT RT²
dlnK ∆H0
Desenvolvendo a equação de van´t Hoff, temos: dT
= RT²

∆H 0
dlnK = ( ) . dT
RT²
lnK2
∆H 0 T2 1
∫ dlnK = . ∫ ( ) . dT
lnK1 R T1 T²

∆H 0 1 1
lnK 2 − lnK1 = x( − )
R T1 T2
Observação: Para este estudo, leva-se em consideração que a variação da entalpia padrão da reação varia pouco
com a temperatura em relação ao intervalo de integração.
Questão 1046 – A constante de equilíbrio é dimensional?

Resposta: Não. Como a constante de equilíbrio é definida em função da atividade e da fugacidade e, estes parâmetros
são adimensionais, logo, a constante de equilíbrio é também adimensional.

Questão 1047 – (IME) Que pode influir na constante de equilíbrio de uma reação química?

Resposta: O fator que influencia a constante de equilíbrio trata-se da temperatura, K = f(T).


Variação da constante de equilíbrio com a temperatura, representado pela equação de Van´t Hoff:
Em dada temperatura, afinidade-padrão e constante de equilíbrio estão relacionadas pela seguinte equação: ΔGo = -
dln K ∆G°
R x T x lnK. Derivando em relação a temperatura, temos: dT
= R x T² .

𝑑𝑙𝑛 𝐾 1 𝑑(𝐺⁄𝑇)
𝑑𝑇
= −𝑅 𝑥 𝑑𝑇
, onde a variação de pressão constante é omitido um vez que ΔGo é definida para a pressão de
1 atm e, assim, a constante de equilíbrio, K, não depende da pressão.
d(G⁄T) ∆H° dln K ∆H°
Combinando com a equação de Gibbs-Helmholtz: dT
=− T²
, obtém-se dT
= R x T² .

Esta equação, conhecida com equação de Van´t Hoff, dá a taxa de variação do logaritmo da constante de equilíbrio
com a temperatura, em torno de uma certa temperatura T, sendo ΔHo a entalpia-padrão de reação nesta mesma
temperatura T. Vê-se que, quando cresce a temperatura, K aumenta nas reações endotérmicas (ΔHo ˃ 0) e diminui
nas reações exotérmicas (ΔHo < 0).
dln K ∆H°
Lembrando que: 1 = R
.
d( )
T

“... mostrando que a variação de lnK com 1/T é linear se ΔHo for considerada independente da temperatura, o que é
admissível para um intervalo de temperatura não muito grande”.
Questão 1048 – (IME) O equilíbrio químico é estático? Por quê?

Resposta: Não, porque as reações continuam se processando, nos sentidos direto e inverso com a mesma
velocidade.

Questão 1049 – (ITA) Sabendo que a dissociação de N2O4(g) em NO2(g) é endotérmica, preveja como a constante de
equilíbrio (Kc) deve variar com a temperatura, explicando o tipo de raciocínio utilizado para esta previsão.

558
1600 FÍSICO-QUÍMICA APLICADA EXERCÍCIOS COMENTADOS - IME – ITA – OLIMPÍADA

Resposta: Equação química balanceada: N2O4(g) ⇄ 2 NO2(g) ΔH° > 0.


[NO2 ]2
Equação da constante de equilíbrio químico: K C = [N .
2 O4 ]

O processo sendo endotérmico, com o aumento da temperatura, o equilíbrio será deslocado para a direita,
favorecendo os produtos da reação direta. com isso, a constante de equilíbrio vai aumentar. Este fator também pode
ser explicado pela equação de Van´t Hoff.
dlnK ∆°H
Equação de Van´t Hoff: dT
= R x T²

Observação: A equação de Van’t Hoff trata-se da taxa de variação do logaritmo da constante de equilíbrio com a
temperatura, em torno de uma certa temperatura T, sendo ΔH° a entalpia padrão da reação nesta temperatura.
✓ Se a reação é endotérmica (ΔH° > 0) → K aumenta com a temperatura.
✓ Se a reação é exotérmica (ΔH° < 0) → K diminui com a temperatura.

Questão 1050 - (IME) Na reação N2(g) + O2(g) ⇄ 2 NO(g) - 43000 cal. Dizer o efeito sobre o seu equilíbrio, por:
a) aumento de temperatura à pressão constante;
b) diminuição de pressão à temperatura constante;
c) aumento de concentração de O2;
d) diminuição de concentração de N2;
e) aumento de concentração de NO;
f) presença de um catalisador.

Resolução: Analisando a questão pelo princípio de Le Chatelier.


a) Como a reação é endotérmica (N2(g) + O2(g) + 43000 cal ⇄ 2 NO(g)), com o aumento da temperatura o equilíbrio
será deslocado para a direita.
b) Com a diminuição da pressão do sistema, não haverá nenhuma influência sobre o deslocamento de equilíbrio.
c) Aumentando a concentração da quantidade de matéria de oxigênio molecular, [O2], o equilíbrio será deslocado
para a direita.
d) Diminuindo a concentração da quantidade de matéria de nitrogênio gasoso, [N2], o equilíbrio será deslocado para
a esquerda.
e) Aumentando a concentração da quantidade de matéria de monóxido de nitrogênio, [NO], o equilíbrio será
deslocado para a esquerda.
f) O catalisador não influencia em nada no deslocamento do equilíbrio, apenas faz com que o processo ocorra com
maior velocidade.

Questão 1051 – (IME) Uma amostra de IBr de massa 8,28 gramas é aquecida a 227°C em um recipiente de 0,25 litros
decompondo-se parcialmente em iodo e bromo. Sabendo-se que ao atingir o equilíbrio, em fase gasosa, a pressão
parcial do bromo é de 3,08 atmosferas, calcule o valor da constante de equilíbrio.

Resolução: Equação balanceada de decomposição do brometo de iodo: 2 IBr(g) ⇄ I2(g) + Br2(g).


Base de cálculo: Massa de brometo de iodo igual a 8,28 gramas.
Cálculo da pressão inicial do brometo de iodo, a partir da equação dos gases ideais:
8,28
( ) x 0,08206 x 500
pIBr = 128 = 6,56 atm
0,25
Tabela de equilíbrio químico:

2 IBr(g) (atm) I2(g) (atm) Br2(g) (atm)



Início 6,56 0 0

559
1600 FÍSICO-QUÍMICA APLICADA EXERCÍCIOS COMENTADOS - IME – ITA – OLIMPÍADA

Reage 2xα 1xα 1xα


Equilíbrio 6,56 - 2α α α

O enunciado do problema coloca a seguinte situação: “a pressão parcial do bromo é de 3,08 atm” (equilíbrio final),
logo a tabela de equilíbrio acima ficará da seguinte maneira:

PCl5(g) ⇄ PCl3(g) Cl2(g)


Início 6,56 0 0
Reage 2α = 2 x (3,08) = 6,16 α = 3,08 α = 3,08
Equilíbrio 6,56 – 6,16 = 0,40 atm 3,08 atm 3,08 atm

PPCl3 x PCl2 (3,08)²


Expressão e cálculo da constante de equilíbrio em função das pressões parciais (Kp): K p = PPCl5
= 0,40
=
59,29
Questão 1052 – (IME) A constante de equilíbrio do processo A(sólido) + 2B(gás) ⇄ 2C(gás). Tem um valor numérico igual
a 0,64. Que concentração de "C" estará em equilíbrio com 0,1 mol.L-1 de "A" e 0,5 mol.L-1 de "B"?
Resolução: Expressão da constante de equilíbrio em função das concentrações das quantidades de matéria:
[C]²
Kc = = 0,64
[B]²

[C]²
Tirando a raiz quadrada da expressão matemática acima, temos: √[B]² = √0,64

[C]
= 0,8
[B]
Cálculo da concentração da quantidade de matéria de C, sabendo que a concentração de B é igual a 0,50 mol.L -1,
[C]
temos: = 0,8
0,50

mol
[C] = 0,40
L
Questão 1053 – (ITA - MODIFICADA) Descreva a equação química balanceada para o preparo de propionato de
metila em um laboratório de química. Também mencione como a reação pode ser acelerada e como o seu rendimento
pode ser aumentado.

Resolução: Equação química a ser analisada pelo princípio de Le Chatelier:


CH3OH(aq) + C2H5COOH(aq) ⇄ C2H5COOCH3(aq) + H2O(l).
O estabelecimento do equilíbrio pode ser acelerado pela adição de ácidos minerais. Os íons H 3O+(aq) atuam como
catalisador da reação. O catalisador acelera o estabelecimento do equilíbrio, mas não aumenta o rendimento da
reação, pois o catalisador não desloca equilíbrio químico. Para aumentar o rendimento em éster (propinato de metila),
precisamos deslocar o equilíbrio para a direita, o que pode ser feito adicionando um desidratante. A retirada de água
também desloca o equilíbrio para a direita.
Questão 1054 – (ITA) A reação N2O4(g) ⇄ 2 NO2(g) é endotérmico, com ∆H° = + 56,9 kJ.
a) Aponte o que acontece com a quantidade de NO2 para as seguintes variações:
I. Adição de N2O4;
II. Abaixamento da pressão por aumento de volume do recipiente;
III. Aumento da temperatura;
IV. Adição de um catalisador ao sistema.

560
1600 FÍSICO-QUÍMICA APLICADA EXERCÍCIOS COMENTADOS - IME – ITA – OLIMPÍADA

b) Indique, dentre as variações apresentadas no item anterior, o que altera o valor de Kc. Justifique sua resposta.

Resolução:

Item a) Equação química balanceada: N2O4(g) ⇄ 2 NO2(g)


I. Adicionando N2O4(g), o equilíbrio será deslocado para a direita, aumentando NO2(g).
II. Diminuindo a pressão do sistema reacional, o equilíbrio será deslocado para a direita, aumentando NO2(g).
III. Sendo o processo endotérmico e aumentando a temperatura, o equilíbrio é deslocado para a direita,
aumentando NO2(g).
IV. A adição do catalisador específico para este processo, não influencia no equilíbrio químico.

Item b) O fator que altera a constante de equilíbrio é a temperatura. Este fator é explicado pela equação de Van´t Hoff.
Somente uma alteração de temperatura pode provocar modificação no valor da constante de equilíbrio (Kc ou Kp). Já
alterações de concentração ou de pressão não modificam os valores de Kc ou Kp. No quadro a seguir, encontra-se a
equação de Van´t Hoff, que estuda a relação da constante de equilíbrio com a temperatura.

∆H 0 1 1
lnK1 − lnK 2 = x( − )
R T2 T1
Questão 1055 – (OLIMPÍADA MINEIRA DE QUÍMICA) O processo de produção de amônia (NH3), a partir dos gases
hidrogênio (H2) e nitrogênio (N2), foi de difícil desenvolvimento e demandou vários anos de estudo. Este método
recebeu o nome de processo Haber-Bosch, em homenagem aos seus idealizadores. A equação química abaixo
descreve a síntese da amônia: 3/2 H2(g) + ½ N2(g) ⇄ NH3(g). Supondo que os gases nitrogênio e hidrogênio sejam
colocados em um reator fechado, onde não ocorra nenhuma troca de matéria com a vizinhança e na presença de um
catalisador apropriado, responda as questões que se seguem:
a) Considerando o princípio de Le Chatelier, descreva qual será a nova situação do equilíbrio químico no interior
do reator, se sua pressão interna for duplicada.
b) Calcule a pressão que 17 gramas do gás amônia exercem em um reator de volume igual a 5 L e operando a
427°C.

Resolução:

Item a) Avaliação do número de mol dos reagentes e produtos pela reação direta:
Equação química balanceada: 3/2 H2(g) + ½ N2(g) ⇄ NH3(g)
3 1
( + )V ⇄ 1V
2 2
2V ⇄ 1V
Aumentando a pressão interna do reator, consequentemente vai haver uma diminuição do volume do sistema, com
isso o equilíbrio químico será deslocado para a direita, ou seja, favorecendo a formação de amônia.
Item b) Cálculo da pressão de amônia (NH3), a partir da utilização da equação dos gases ideais:

17
nNH3 x R x T (17) x 0,08206 x (427 + 273) 57,44
pNH3 = = = = 11,49 atm
V 5 5

561
1600 FÍSICO-QUÍMICA APLICADA EXERCÍCIOS COMENTADOS - IME – ITA – OLIMPÍADA

Questão 1056 – (IME) Em um recipiente fechado, mantido a temperatura constante de 700 K, foram introduzidos
0,450 mol de dióxido de carbono e 0,450 mol de hidrogênio. Após certo tempo, estabeleceu-se o equilíbrio,
aparecendo como espécies novas monóxido de carbono e vapor d’água. Foram então adicionados 0,500 mol de uma
mistura equimolecular de dos reagentes. Calcule a nova composição de equilíbrio, sabendo-se que nessas condições
Kc = 0,160.

Resolução: Para este exercício apresentado pelo IME, faremos de três maneiras distintas.

Primeira maneira de resolução:


Base de cálculo: 0,450 mol inicial de CO2(g) e 0,450 mol inicial de H2(g).
Tabela de equilíbrio químico:

CO2(g) (mol) H2(g) (mol) CO(g) (mol) H2O(g) (mol)


Início 0,450 + 0,250 0,450 + 0,250 ⎯
⎯⎯

⎯ 0 0
Reage α α α α
Equilíbrio 0,700 - α 0,700 - α α α

Expressão das concentrações das quantidades de matéria (mol.L-1) considerando que o volume da solução seja igual
a V:
0,700 − α
[CO2 ] =
V
0,700 − α
[H2 ] =
V
α
[CO] =
V
α
[H2 O] =
V
Cálculo da constante α, a partir da constante de equilíbrio (Kc): CO2(g) + H2(g) ⇄ CO(g) + H2O(g)
α α
[CO] x [H2 O] (V) x (V)
Kc = =
[CO2 ] x [H2 ] 0,700 − α 0,700 − α
( V )x( V )

(α) x (α)
Kc =
(0,700 − α) x (0,700 − α)
(α)2
Kc =
(0,700 − α)2
(α)2
0,160 =
(0,700 − α)2

(α)2
Tirando a raiz quadrada da equação matemática acima, temos: √(0,700−α)2 = √0,160

α
= 0,40
0,700 − α
α = 0,40 x (0,700 − α)
α = 0,280 − 0,40α
α + 0,40α = 0,280

562
1600 FÍSICO-QUÍMICA APLICADA EXERCÍCIOS COMENTADOS - IME – ITA – OLIMPÍADA

0,280
α= = 0,20
1,40
Concentração das espécies químicas, levando em consideração que o volume seja de um litro (1L):
0,700 − α 0,700 − 0,200 0,500
[CO2 ] = = =
V V V
0,700 − α 0,700 − 0,200 0,500
[H2 ] = = =
V V V
α 0,200
[CO] = =
V V
α 0,200
[H2 O] = =
V V
Segunda maneira de resolução:
Base de cálculo: 0,450 mol de CO2(g) e 0,450 mol de H2(g) inicial. Tabela de equilíbrio químico:

CO2(g) (mol) H2(g) (mol) CO(g) (mol) H2O(g) (mol)


Início 0,450 0,450 0 0

Reage α α α α
Equilíbrio 0,450 - α 0,450 - α α α

O problema afirma que foram adicionados 0,500 equimolecular para os reagentes, logo a tabela de equilíbrio químico
fica da seguinte maneira:

CO2(g) (mol) H2(g) (mol) CO(g) (mol) H2O(g) (mol)


Início 0,450 – α + 0,250 0,450 – α + 0,250 α α

Reage β β β β
Equilíbrio 0,700 – α - β 0,700 – α - β α+β α+β

Expressão da concentração das quantidades de matéria (mol.L-1) considerando que o volume da solução seja igual a
V. Chamando z = α + β, temos:
0,700 − α − β
[CO2 ] =
V
0,700 − α − β
[H2 ] =
V
α+ β
[CO] =
V
α+ β
[H2 O] =
V
Chamando 𝛼 + 𝛽 = 𝑧, temos:
0,700 − z
[CO2 ] =
V
0,700 − z
[H2 ] =
V
z
[CO] =
V

563
1600 FÍSICO-QUÍMICA APLICADA EXERCÍCIOS COMENTADOS - IME – ITA – OLIMPÍADA

z
[H2 O] =
V
Cálculo da constante z, a partir da constante de equilíbrio (Kc): CO2(g) + H2(g) ⇄ CO(g) + H2O(g)
[CO] x [H2 O] (z) x (z) z2
Kc = = =
[CO2 ] x [H2 ] (0,700 − z) x (0,700 − z) (0,700 − z)2

z2
Kc =
(0,700 − z)2
z2
0,160 =
(0,700 − z)2

𝑧2
Tirando a raiz quadrada da equação matemática acima, temos: √(0,700−𝑧)2 = √0,160

z
= 0,40
0,700 − z
z = 0,40 x (0,700 − z)
z = 0,280 − 0,40z
z + 0,40z = 0,280
0,280
z= = 0,200
1,40
Logo, a concentração das espécies químicas são as seguintes, considerando o volume igual a 1 litro:
0,700 − z 0,700 − 0,200 mol
[CO2 ] = = = 0,500
V 1 L
0,700 − z 0,700 − 0,200 mol
[H2 ] = = = 0,500
V 1 L
z 0,200 mol
[CO] = = = 0,200
V 1 L
z 0,200 mol
[H2 O] = = = 0,200
V 1 L
Terceira maneira de resolução:
Base de cálculo: 0,450 mol de CO2(g) e 0,450 mol de H2(g) inicial.
Primeira tabela de equilíbrio químico:

CO2(g) (mol) H2(g) (mol) CO(g) (mol) H2O(g) (mol)


Início 0,450 0,450 0 0

Reage α α α α
Equilíbrio 0,450 - α 0,450 - α α α

Expressão das concentrações das quantidsades de matéria (mol.L-1) para um volume de solução V:
0,450 − α
[CO2 ] =
V
0,450 − α
[H2 ] =
V

564
1600 FÍSICO-QUÍMICA APLICADA EXERCÍCIOS COMENTADOS - IME – ITA – OLIMPÍADA

α
[CO] =
V
α
[H2 O] =
V
[CO] x [H2 O] (α) x (α) z2
Cálculo de α, a partir da constante de equilíbrio: K c = [CO2 ] x [H2 ]
= (0,450−α) = (0,450−z)2
x (0,700−α)
α2
0,160 =
(0,450 − α)2

𝛼2
Tirando a raiz quadrada da equação matemática acima, temos: √(0,450 − 𝛼)2 = √0,160
α
= 0,40
0,450 − α

α = 0,40 x (0,450 − α)

0,18
α= = 0,130
1,40

Foram adicionados 0,500 equimolecular para os reagentes, logo:


Segunda tabela de equilíbrio químico:

CO2(g) (mol) H2(g) (mol) CO(g) (mol) H2O(g) (mol)


Início 0,450 + 0,250 – 0,130 0,450 + 0,250 – 0,130 0,130 0,130

Reage β β β β
Equilíbrio 0,570 - β 0,570 – β 0,130 + β 0,130 + β

Expressão das concentrações das quantidades de matéria (mol.L-1) para um volume de solução V:
0,570 − β
[CO2 ] =
V
0,570 − β
[H2 ] =
V
0,130 + β
[CO] =
V
0,130 + β
[H2 O] =
V
0,130+β 0,130+β
[CO] x [H2 O] ( )x( )
Cálculo de 𝛽, a partir da constante de equilíbrio: K c = [CO2 ] x [H2 ]
= V
0,570−β
V
0,570−β
( )x( )
V V

0,130 + β 2
( V )
Kc =
0,570 − β 2
( V )

(0,130 + β)2
Kc =
(0,570 − β)2
(0,130 + β)2
0,160 =
(0,570 − β)2

565
1600 FÍSICO-QUÍMICA APLICADA EXERCÍCIOS COMENTADOS - IME – ITA – OLIMPÍADA

(0,130+𝛽)2
Tirando a raiz quadrada da equação matemática acima, temos: √(0,570−𝛽)2 = √0,160
0,130 + β
= 0,40
0,570 − β

0,130 + β = 0,40 x (0,570 − β)

0,130 + β = 0,228 − 0,40β


1,40 x β = 0,228 − 0,130

0,098
β= = 0,07
1,40
Logo, a concentração das espécies químicas são as seguintes, considerando o volume igual a 1 litro:
0,570 − β 0,570 − 0,07 mol
[CO2 ] = = = 0,500
V 1 L
0,570 − β 0,570 − 0,07 mol
[H2 ] = = = 0,500
V 1 L
0,130 + β 0,130 + 0,07 mol
[CO] = = = 0,200
V 1 L
0,130 + β 0,130 + 0,07 mol
[H2 O] = = = 0,200
V 1 L

Questão 1057 – (SELETIVA PARA A 39TH INTERNATIONAL CHEMISTRY OLYMPIAD – MOSCOU – RÚSSIA –
OBQ) Para a reação H2(g) + I2(g) ⇄ 2 HI(g), tem-se K 448°C
p = 50,0 e K 350°C
p = 66,9. Calcule ΔH° para essa reação.

Resolução: Equação de Van´t Hoff:


∂lnK ∆H°
=− 2
∂T RT
∂lnK ∆H° 1
= − 2d( )
∂T RT T

∂lnK ∆H° 1
= − 2d( )
∂T RT T

∂lnK ∆H°
=− 2
1
∂T( ⁄T) RT

A equação anterior mostra que, como Ho é aproximadamente constante (pelo menos em intervalos de temperatura
não muito grandes), um gráfico de ln K contra 1/T é linear, tendo um coeficiente angular igual a −Ho/R. Ela mostra
ainda como o efeito da temperatura sobre K é influenciado pelo sinal e pela magnitude de Ho para a reação Se
Ho0 (reação endotérmica), K cresce com o aumento da temperatura; se Ho0 (reação exotérmica), K decresce
com o aumento da temperatura. E quanto maior é o módulo de H, maior a influência de uma variação de temperatura
sobre a constante de equilíbrio. A equação também pode ser escrita da seguinte maneira:
−∆H° 1 1
lnK p2 − lnK p2 = x{ − }
R T2 T1
−∆H° 1 1
Resolvendo o problema: ln (50) − ln (66,9) = x{ − }
8,314 623 721

566
1600 FÍSICO-QUÍMICA APLICADA EXERCÍCIOS COMENTADOS - IME – ITA – OLIMPÍADA

50 −∆H° 1 1
ln ( )= x{ − }
66,9 8,314 623 721
−∆H° 1 1
ln (0,747) = x{ − }
8,314 623 721
− 0,292 x 8,314 = −∆H° x {1,60 x 10−3 − 1,39 x 10−3 }
− 0,292 x 8,314 = −∆H° x {1,60 x 10−3 − 1,39 x 10−3 }
− 2,43 = −∆H° x (0,00021)
J
∆H° = + 11571,43
mol
Questão 1058 – (ITA) Em um balão fechado e sob temperatura de 27°C, N2O4(g) está em equilíbrio com NO2(g). A
pressão total exercida pelos gases dentro do balão é igual a 1,0 atm, nestas condições, N2O4(g) encontra-se 20%
dissociado.
a) Determine o valor da constante de equilíbrio para a reação de dissociação do N 2O4(g). Mostre os cálculos
realizados.
b) Para a temperatura de 27°C e pressão total dos gases dentro do balão igual a 0,10 atm, determine o grau de
dissociação do N2O4(g). Mostre os cálculos realizados.

Resolução: Equação química: N2O4(g) ⇄ 2 NO2(g).


Base de cálculo: n (mol) inicial de N2O4(g)
Tabela de equilíbrio químico:

N2O4(g) ⇄ 2 NO2(g)
Início n 0
Reage nα 2nα
Equilíbrio n - nα 2nα

Cálculo do número de mol total (nT) das espécies gasosas: nT = n – nα + 2nα = n + nα = n(1 + α)
Cálculo das pressões parciais dos componentes gasosos:
n(1−α) (1−α)
Pressão parcial do N2O4: PN2 O4 = X N2 O4 x pt = [n(1+α)] x pt = [(1+α)] x pt

2nα 2α
Pressão parcial do NO2: PNO2 = X NO2 x pt = [n(1+α)] x pt = [1+α] x pt

2α 2
p2NO2 {[ ]x pt } 4α²pt
Expressão da constante de equilíbrio: K p = P = 1+α
2α = 1−α²
N2 O4 [ ]x pt
1+α

Item a) Cálculo da constante de equilíbrio a pressão constante, considerando como grau de dissociação igual a 0,20
(α = 0,20):

4α²pt 4α²pt 4 x (0,20)2 x 1 0,16


Kp = = = = = 0,167
1 − α² 1 − α² 1 − (0,20)2 1,96

Item b) Como a temperatura (27°C) é a mesma para o item (a), logo Kp = 0,167. Cálculo do grau de dissociação (𝛼):

567
1600 FÍSICO-QUÍMICA APLICADA EXERCÍCIOS COMENTADOS - IME – ITA – OLIMPÍADA

4𝛼²𝑝𝑡
𝐾𝑝 =
1 − 𝛼²

4α² x 0,10
0,167 =
1 − α²

0,40 x α²
0,167 =
1 − α²

0,167 − 0,167α² = 0,40 x α²

0,567α² = 0,167

0,167
α=√ = 0,5427 (54,27%)
0,567

Questão 1059 – (ITA) O método de Haber para a produção de amônia é baseado no estabelecimento do seguinte
equilíbrio N2(g) + 3H2(g) ⇄ 2 NH3(g); K; a partir de misturas de nitrogênio e hidrogênio comprimido. Pensando em alguns
aspectos do problema, poderia se chegar à conclusão de que é mais interessante trabalhar em temperaturas mais
baixas. Pensando em outros aspectos, poderia se chegar à conclusão contrária. Discuta o problema envolvido em um
e em outro caso.
Resolução: A reação de síntese do NH3 é um processo exotérmico, conforme pode ser observado a seguir.

N2(g) + 3H2(g) ⇄ 2 NH3(g), H < 0.


Pelo princípio de Le Chatelier, ao acrescentar calor ao sistema, ele será deslocado para a esquerda, com isso, irá
ocorrer a diminuição do rendimento da reação (Kc diminui).
Com a diminuição da temperatura, o equilíbrio será deslocado para a direita, (Kc aumenta), com isso, o rendimento
aumenta. Porém, em baixas temperaturas, sendo o rendimento maior, a velocidade do processo é muito pequena
tanto para o processo direto quanto para o processo inverso, o que acarreta que o equilíbrio vai demorar a se
estabilizar. Então, o que deve ser feito é a obtenção de valores intermediários para a temperatura, isto é, nem muito
baixa para comprometer o rendimento do processo. Esta temperatura ocorre em torno de 400°C.
Questão 1060 – (OLIMPÍADA DE QUÍMICA DO RIO DE JANEIRO) Em 1914, trabalhando com ligas de ferro como
catalisadores, Fritz Haber e colaboradores conseguiram sintetizar a amônia a partir de seus elementos, H2 e N2. Este
processo, conhecido como hoje como processo Haber-Bosch, tornou a Alemanha pré – Primeira Guerra independente
na fabricação de explosivos a base de nitrato. Este catalisador mudou o caminho da história, fazendo da Alemanha
uma grande potência militar no começo do século XX. Nos dias de hoje, decorridos quase 100 anos, o processo
Haber-Bosch ainda é fundamental na fabricação de amônia, e o catalisador utilizado possui a mesma composição
básica. Num reator com volume de 200 litros, foram colocados 75 mol de gás nitrogênio, 180 mol de gás hidrogênio e
10 mol de gás amônia. Após certo tempo, verificou-se que o sistema atingiu o equilíbrio. A temperatura registrada
neste momento foi de 25°C e observou-se que 50 mol de gás nitrogênio foram consumidos na reação.
a) Calcule o valor da constante de equilíbrio Kc na referida temperatura.
b) Explique que efeito teria a adição de 2,5 kg de catalisador nas concentrações das espécies em equilíbrio.
c) Que efeito teria o aumento da pressão nas concentrações das espécies em equilíbrio?

Resolução: Equação química: N2(g) + 3 H2(g) ⇄ 2 NH3(g)


[NH3 ]2
Equação da constante de equilíbrio: K c =
[N2 ] x [H2 ]3

Base de cálculo: 75 mol de N2(g), 180 mol de H2(g) e 10 mol de NH3(g) no estado inicial.

568
1600 FÍSICO-QUÍMICA APLICADA EXERCÍCIOS COMENTADOS - IME – ITA – OLIMPÍADA

Tabela de equilíbrio químico:

N2(g) (mol) 3 H2(g) (mol) ⇄ 2 NH3(g) (mol)


Início 75 180 10
Reage 50 150 100
Equilíbrio 75 – 50 = 25 mol 180 – 150 = 30 mol 10 + 100 = 110 mol

Cálculo das concentrações das quantidades de matéria (mol.L-1) molares das espécies químicas:
25 mol mol
[N2 ] = = 0,125
200 L L
30 mol mol
[H2 ] = = 0,150
200 L L
110 mol mol
[NH3 ] = = 0,550
200 L L
[NH3 ]2 [0,550]2
Item a) Cálculo da constante de equilíbrio: K c = [N 3 = [0,125] x [0,150]3 = 717,04
2 ] x [H2 ]

Item b) A adição de catalisador não influencia em nada nas concentrações das espécies em equilíbrio. O objetivo da
aplicação de um catalisador é o aumento da velocidade da reação química.
Item c) Pelo Princípio de Le Chatelier, o aumento da pressão total do sistema desloca o equilíbrio para a direita,
aumentando a concentração da quantidade de matéria da amônia e diminuindo as concentrações do nitrogênio e do
hidrogênio.

Questão 1061 – (GRILLO) Considere a síntese de uma reação hipotética A2(g) + B2(g) ⇄ 2 AB(g), em que as constantes
de equilíbrio apresentam os seguintes valores em diferentes temperaturas, K 650°C
p = 99,0 e K 225°C
p = 44,9. A partir
dessas informações, preveja se esta reação de síntese é endotérmica ou exotérmica.

Resolução: Informações do problema: 𝐾𝑝650°𝐶 = 99,0 e 𝐾𝑝225°𝐶 = 44,9


−∆H° 1 1
lnK p2 − lnK p2 = x{ − }
R T2 T1
−∆H° 1 1
Resolvendo o problema: ln (99) − ln (44,9) = 8,314
x {650+273 − 225+273}

99 −∆H° 1 1
ln ( )= x{ − }
44,9 8,314 923 498
−∆H°
ln (2,20) = x {1,08 x 10−3 − 2,0 x 10−3 }
8,314
0,788 x 8,314 = −∆H° x {−0,00092}
0,788 𝑥 8,314 J
∆H° = = + 7121,12 (𝑝𝑟𝑜𝑐𝑒𝑠𝑠𝑜 𝑒𝑛𝑑𝑜𝑡é𝑟𝑚𝑖𝑐𝑜)
0,00092 mol
Questão 1062 – (ITA) As espécies químicas A e B reagem segundo a reação representada pela seguinte equação
química: 2A + B ⇄ 4C. Numa temperatura fixa, as espécies são colocadas para reagir em um recipiente com volume
constante. A figura abaixo mostra como a concentração das espécies químicas A, B e C varia com o tempo.

569
1600 FÍSICO-QUÍMICA APLICADA EXERCÍCIOS COMENTADOS - IME – ITA – OLIMPÍADA

A partir da análise desta figura, determine a constante de equilíbrio, Kp, para esta reação.
Resolução: Equação química: 2A(g) + B(g) ⇄ 4C(g)

P4
Expressão da constante de equilíbrio em função das pressões parciais: K p = P2 xCP
A B
Observando o gráfico, observa-se que no equilíbrio as concentrações das concentrações em relação as pressões
parciais de cada participante são as seguintes: pA = 0,4 atm; pB = 0,2 atm e pC = 0,8 atm.

P4 (0,8)4
Cálculo da constante de equilíbrio: K p = P2 xCP = (0,4)4 x 0,2 = 80
A B

Questão 1063 – (OLIMPÍADA NORTE/NORDESTE DE QUÍMICA) Abaixo, tem-se a equação não-balanceada da


reação de dióxido de enxofre com cloro para a produção de óxido de dicloro: SO2(g) + Cl2(g) → OSCl2(g) + Cl2O(g).
a) Escreva a equação balanceada desta reação.
b) Considerando a equação balanceada, uma redução do volume do recipiente, mantendo as quantidades de
reagentes, favorecia ou desfavorecia a produção de Cl2O(g)? Justifique.
c) Nesta reação, que elemento é oxidado e que elemento é reduzido?
d) Enumere todos os compostos envolvidos nesta reação, em ordem crescente de velocidade de efusão.

Resolução:

Item a) Realizando o balanceamento da reação pelo método de tentativas:


SO2(g) + 2 Cl2(g) → OSCl2(g) + Cl2O(g).

Item b) Diminuindo o volume do sistema, automaticamente a pressão aumenta. Pelo princípio de Le Chatelier,
diminuindo o volume, o equilíbrio será deslocado para a direita, favorecendo o Cl2O(g).

Item c) O cloro tanto se reduz como se oxida.

Item d) Como a velocidade de efusão é inversamente proporcional à massa molar, então, quanto maior a massa
molar, menor será a velocidade de efusão. Logo: SO2(g) = 64 g.mol-1; Cl2(g) = 71 g.mol-1; OSCl2(g) = 119 g.mol-1 e Cl2O(g)
= 87 g.mol-1.

Resposta: VOSCl2 < VCl2O < VCl2 < VSO2.


Questão 1064 – (IME) Considere a reação: AB2(g) + A(s) ⇄ 2 AB(g). Atingido o equilíbrio nas CNTP, a fase gasosa
apresenta fração molar de AB2 igual a 0,1. Em que pressão, à mesma temperatura, a fração molar de AB na fase
gasosa, no equilíbrio, seria igual a 0,8?

Resolução: Analisando a situação I:


Equação química: AB2(g) + A(s) → 2 AB(g)

570
1600 FÍSICO-QUÍMICA APLICADA EXERCÍCIOS COMENTADOS - IME – ITA – OLIMPÍADA

Informação do problema: XAB2 = 0,1


XAB2 + XAB = 1,0
XAB = 1,0 – 0,1 = 0,9
Logo, para a situação I, as frações molares são as seguintes: XAB2 = 0,1 e XAB = 0,9
P2 [XAB .pT ]2
Equação da constante de equilíbrio em função das pressões parciais para a situação I: K I = P AB = =
AB2 [XAB2 .pT ]
X2AB .pT (0,90)2 .pT
= 0,10
[XAB2 ]

K I = 8,1 x pT
Nas condições normais de temperatura e pressão (0°C e 1 atm), a constante de equilíbrio I será igual a KI = 8,1
Analisando a situação II:
Equação química: AB2(g) + A(s) → 2 AB(g)
Informação do problema: XAB = 0,8
XAB2 + XAB = 1,0
XAB2 = 1,0 – 0,8 = 0,2
Como o processo ocorre na mesma temperatura, temos: KI = KII.
Logo, para a situação II, as frações molares são as seguintes: XAB2 = 0,2 e XAB = 0,8
P2 [XAB .pT ]2
Equação da constante de equilíbrio em função das pressões parciais para a situação II: K II = P AB = =
AB2 [XAB2 .pT ]
X2AB .pT (0,80)2 x pT
= (0,20)
[XAB2 ]

Cálculo da pressão total, sabendo que a constante KI é igual a 8,1: K I = K II


(0,80)2 x pT
8,1 =
0,20
8,1 x 0,20
pT = = 2,53 atm
(0,80)2
Questão 1065 – (IME) O tetracloreto de carbono é um composto orgânico apolar, líquido à temperatura ambiente.
Dentre outras aplicações, foi amplamente utilizado no século passado como solvente, como pesticida e na síntese de
agentes refrigerantes. Seu emprego comercial, entretanto, foi progressivamente reduzido quando se tornaram
evidentes os seus efeitos a saúde humana e ao meio ambiente. Estudos constataram que a inalação é a principal via
de exposição ao tetracloreto de carbono para trabalhadores e para a população em geral em razão de sua pressão
de vapor relativamente elevada e de sua lenta degradação no ambiente. Supondo que as energias livres padrão de
formação (∆G°f) do tetracloreto de carbono, nos estados líquido e vapor a 25°C, sejam – 68,6 kJ.mol-1 e – 64,0 kJ.mol-
1 respectivamente, determine a sua pressão de vapor, à mesma temperatura, em função da constante e (número de

Neper).
Resolução:

Formação de tetracloreto de carbono na fase líquida: C(s) + 2 Cl2(g) ⇄ CCl4(l)


Formação de tetracloreto de carbono na fase gasosa: C(s) + 2 Cl2(g) ⇄ CCl4(g)
Cálculo da variação da energia livre de Gibbs para a vaporização do tetracloreto de carbono:
Invertendo a reação de formação de tetracloreto de carbono na fase líquida, temos:

571
1600 FÍSICO-QUÍMICA APLICADA EXERCÍCIOS COMENTADOS - IME – ITA – OLIMPÍADA

0 kJ
CCl4(l) ⇄ C(s) + 2 Cl2(g) ∆Greação = +68,6 mol

0 kJ
C(s) + 2 Cl2(g) ⇄ CCl4(g) ∆Greação = −64,0 mol +

0 kJ
CCl4(l) ⇄ CCl4(g) ∆Greação = +4,6 mol

Cálculo da pressão de vapor do tetracloreto de carbono na fase gasosa: CCl4(l) ⇄ CCl4(g)

0
J
∆Greação = + 4600 = −R x T x ln K
mol
vapor
− 8,314 x (25 + 273) x ln PCCl4 = + 4600
vapor
8,314 x 298 x ln PCCl4 = − 4600

vapor 4600
ln PCCl4 =−
2477,572
4600
vapor (− )
PCCl4 =e 2477,572 = e(−1,86)

Questão 1066 – (IME) A reação de desidrogenação do etano a eteno, conduzida a 1060 K, tem constante de equilíbrio
Kp igual a 1,0. Sabendo-se que a pressão da mistura racional no equilíbrio é igual a 1,0 atm, determine:
a) a pressão parcial, em atmosferas, do eteno no equilíbrio;
b) a fração de etano convertido a eteno.

Resolução: Item a) Equação química: C2H6(g) ⇄ C2H4(g) + H2(g)


Base de cálculo: n (mol) inicial de gás etano (C2H6(g))
Tabela de equilíbrio químico:
C2H6(g) (mol) C2H4(g) (mol) H2(g) (mol)
Início n 0 0

Reage nα nα nα
Equilíbrio n - nα nα nα

Cálculo do número de mol total: nTOTAL = n - nα + nα + nα = n + nα = n(1 + α)


Expressão das pressões parciais:
n−nα 1−α
Pressão Parcial do C2H6: PC2H6 = XC2H6 x pT = (n+nα) x pT = (1+ α) x pT

nα α
Pressão Parcial do C2H4: PC2H4 = XC2H4 x pT = (n+nα) x pT = (1+ α) x pT

nα α
Pressão Parcial do C2H4: PC2H4 = XH2 x pT = (n+nα) x pT = (1+ α) x pT

𝑃𝐶2 𝐻4 𝑥 𝑃𝐻2
Expressão da constante de equilíbrio em função das pressões parciais (Kp): 𝐾𝑝 = 𝑃𝐶2 𝐻6

Cálculo do grau de dissociação (α), a partir da equação do equilíbrio químico em função das pressões parciais: K p =
PC2H4 x PH2
P C 2 H6

572
1600 FÍSICO-QUÍMICA APLICADA EXERCÍCIOS COMENTADOS - IME – ITA – OLIMPÍADA

α α α
(1 + α) x pT x (1 + α) x pT (1 + α) x pT x (α) α2 x pT α2 x pT
Kp = = = =
1−α (1 − α) (1 + α) x (1 − α) 1 − α2
(1 + α) x pT

Sabendo que a constante de equilíbrio é igual a 1 e a pressão total do sistema é também igual a 1 atm, temos: K p =
α2 x pT
1−α2

α2 x 1
1=
1 − α2
1 − α2 = α2

2α2 = 1

√2
α=
2
√2
α
Cálculo da pressão parcial do gás eteno: PC2 H4 = 1+ α x pT = 2
√2
x 1 atm = (√2 − 1) atm
(1+ )
2

√2 1,41
Item b) Cálculo do grau de dissociação (α): α = 2
= 2
= 0,7071 (70,71%)

Questão 1067 – (IME) A reação dada pela equação abaixo: CH3COOH + C2H5OH ⇄ CH3COOC2H5 + H2O tem
constante de equilíbrio (Kc) igual a 4,00 a temperatura de 100°C. Calcule as concentrações de equilíbrio em mol por
litro de cada componente, partindo da condição inicial de 120,0 gramas de ácido acético e de 92,0 gramas de etanol,
a um volume V do sistema.

Resolução: Equação química: CH3COOH + C2H5OH ⇄ CH3COOC2H5 + H2O


Base de cálculo: 120 g de ácido e 92 g de álcool.
m 120
Cálculo do número de mol de ácido acético: nácido = <MM>
ácido
= 60
= 2,0 mol
ácido

m 92
Cálculo do número de mol de álcool: nálcool = <MM>
álcool
= 46 = 2,0 mol
álcool

Tabela de equilíbrio:

CH3COOH C2H5OH ⇄ CH3COOC2H5 H2O


Início 2 2 0 0
Reage α α α α
Equilíbrio 2-α 2-α α α

[CH3 COOC2 H5 ] x [H2 O]


Expressão da constante de equilíbrio (Kc): K c = [CH3 COOH]x [C2 H5 OH]

Cálculo do grau de dissociação (α), a partir da constante de equilíbrio (Kc):


[CH3 COOC2 H5 ] x [H2 O] 𝛼²
Kc = = =4
[CH3 COOH] x [C2 H5 OH] (2 − 𝛼)²

𝛼²
Tirando a raiz quadrada da equação matemática acima, temos: √(2 − 𝛼)² = √4

𝛼
=2
2 − 𝛼

573
1600 FÍSICO-QUÍMICA APLICADA EXERCÍCIOS COMENTADOS - IME – ITA – OLIMPÍADA

4
𝛼=
3
Cálculo das concentrações das espécies químicas:
4 2
[ácido] = [álcool] = 2 − 3
= 3𝑉
4
[éster] = [água] = α = 3𝑉

Questão 1068 – (IME) Tomou-se uma amostra de 130 gramas de zinco metálico para reagir com uma solução aquosa
diluída de ácido clorídrico em quantidade estequiométrica. Dessa reação, observou-se a formação de gás, que foi
aquecido a 227°C e transportado para um balão fechado de cinquenta litros. Esse balão continha, inicialmente, iodo
em fase gasosa a 227°C e 3,28 atm. Após o equilíbrio, verificou-se que a constante de equilíbrio Kc a 227°C é igual a
160. Considerando que a temperatura permaneceu constante durante o processo, determine a pressão final total no
balão.

Resolução: Equação química: Zn(s) + HCl(aq) → ZnCl2(aq) + H2(g)


𝑚𝑍𝑛 130
Cálculo do número de mol de Zinco: 𝑛𝑍𝑛 = = = 2,0 𝑚𝑜𝑙
<𝑀𝑀>𝑍𝑛 65

Cálculo do número de mol de gás hidrogênio produzido:


Zn(s) + HCl(aq) → ZnCl2(aq) + H2(g)
1 mol de Zn --------------- 1 mol de H2
2,0 mol de Zn ------------- nH2
nH2 = 2 mol

O gás hidrogênio é transportado para um balão de 50 litros, que já continha gás iodo.

Cálculo do número de mol de iodo (I2) no estado inicial, a partir da equação dos gases ideais:
3,28 𝑥 50 = 𝑛𝐼2 𝑥 0,08206 𝑥 (227 + 273)
164
𝑛𝐼2 = ≅ 4,00 𝑚𝑜𝑙
41,03

574
1600 FÍSICO-QUÍMICA APLICADA EXERCÍCIOS COMENTADOS - IME – ITA – OLIMPÍADA

Equação química: H2(g) + I2(g) ⇄ 2 HI(g)


Base de cálculo: 2 mol de H2 e 4 mol de I2
Tabela de equilíbrio químico:
H2(g) (mol) I2(g) (mol) 2 HI(g) (mol)
Início 2 4 0
Reage α α 2α
Equilíbrio 2-α 4-α 2α

Cálculo do número de mol total (nT) dos gases: nT = nH2 + nI2 + nHI = 2 – α + 4 – α + 2α = 6 mol
Cálculo da pressão final (pT) do balão: pT x 50 = 6 x 0,08206 x 500
246,18
pT = = 4,92 𝑎𝑡𝑚
50
Questão 1069 – (ITA) Um cilindro de volume V contém as espécies A e B em equilíbrio químico representado pela
seguinte equação: A(g) ⇄ 2B(g). Inicialmente, os números de mols de A e de B são, respectivamente, iguais a n A1 e
nB1. Realiza-se, então, uma expansão isotérmica do sistema até que o seu volume duplique (2V) de forma que os
números de mols de A e de B passem a ser, respectivamente, n A2 e nB2. Demonstrando o seu raciocínio, apresente
a expressão algébrica que relaciona o número final de mols de B (n B2) unicamente com nA1, nA2 e nB1.

Resolução: Analisando a situação I: Equação química: A(g) ⇄ 2 B(g)


nA1
Concentração das quantidades de matéria das espécies químicas presentes no sistema reacional: [A] = V
;
nB1
[B] =
V

[B]
Expressão da constante de equilíbrio (K1): K1 = [A]

nB 2
( 1)
[B]2
Relação entre a constante de equilíbrio com o número de mol de cada espécie química: K1 = [A]
= V
nA
1
=
V
n2B1
nA1 x V

Analisando a situação II:


nA2 nB2
Concentrações das quantidades de matéria das espécies químicas: [A] = 2V
; [B] = 2V

[B]2
Expressão da constante de equilíbrio (K2): K 2 = [A]

nB 2
( 2)
[B]2
Relação entre a constante de equilíbrio com o número de mol de cada espécie química: K 2 = [A]
= 2V
nA
1
=
2V
n2B2
nA2 x 2V

Como a temperatura se mantém constante para as duas situações, logo as constantes de equilíbrio (K 1 e K2) são
n2B2 n2B1
iguais. Igualando as constantes, temos: n x 2V
=n xV
A2 A1

575
1600 FÍSICO-QUÍMICA APLICADA EXERCÍCIOS COMENTADOS - IME – ITA – OLIMPÍADA

n2B2 n2B1
=
nA2 x 2 n A1

2 x nA2 x n2B1
n2B2 =
nA1

2 x nA2 x n2B1 2 x nA2


nB2 =√ = n B1 x √
nA1 nA1

Questão 1070 – O pentacloreto de fósforo é obtido pela reação de tricloreto de fósforo e cloro segundo a seguinte
equação química, PCl3(g) + Cl2(g) ⇄ PCl5(g). Sabe-se, por outro lado, que o pentacloreto de fósforo tende a dissociar-
se nas respectivas matérias-primas, sendo a extensão dessa reação medida pelo grau de dissociação (α), definido
como:

𝐐𝐮𝐚𝐧𝐭𝐢𝐝𝐚𝐝𝐞 𝐝𝐞 𝐦𝐨𝐥 𝐝𝐢𝐬𝐬𝐨𝐜𝐢𝐚𝐝𝐚


𝛂=
𝐐𝐮𝐚𝐧𝐭𝐢𝐝𝐚𝐝𝐞 𝐢𝐧𝐢𝐜𝐢𝐚𝐥 𝐝𝐞 𝐦𝐨𝐥

Pede-se:
a) O grau de dissociação do PCl5, a temperatura de 250°C e sob pressão de 1,0 atm, se a densidade da mistura
gasosa no equilíbrio é de 2,695 g.L-1.
b) A constante de equilíbrio (Kp) da reação de dissociação nessa temperatura.
c) A pressão total do sistema gasoso a 250°C para um grau de dissociação de 30%.

Resolução:
Item a) Cálculo do grau de dissociação, PCl5(g) ⇄ PCl3(g) + Cl2(g):
Base de cálculo: n (mol) inicial de PCl5(g).
Tabela de equilíbrio:

PCl5(g) ⇄ PCl3(g) Cl2(g)


Início n 0 0
Reage nα nα nα
Equilíbrio n - nα nα nα

Cálculo do número de mol total: nT = nPCl5 + nPCl3 + nCl2 = n – nα + nα + nα = n + nα = n(1+ α)


Relação entre a densidade do gás com a pressão, a partir da equação dos gases ideais: p x V = n x (1 +
α) x R x T
m
pxV= x (1 + α) x R x T
< MM >
p x < MM >= d x (1 + α) x R x T
p x < MM >
d=
(1 + α) x R x T
1 x 208,5
Substituindo os valores na equação acima, temos: 2,696 = (1+ α) x 0,08206 x (250+273)

208,5
1+ α=
115,70
α = 0,80 (80%)

576
1600 FÍSICO-QUÍMICA APLICADA EXERCÍCIOS COMENTADOS - IME – ITA – OLIMPÍADA

Item b) Expressão das pressões parciais dos participantes químicos:


n−nα 1−α
Pressão Parcial do PCl5: PPCl5 = XPCl5 x pT = (n+nα) x pT = (1+ α) x pT

nα α
Pressão Parcial do PCl3: PPCl3 = XPCl3 x pT = (n+nα) x pT = (1+ α) x pT

nα α
Pressão Parcial do C2H4: PCl2 = XCl2 x pT = ( )x pT = ( ) x pT
n+nα 1+ α

α2
PPCl3 x PCl2 [(1+α)2 ]x p2T
Expressão da constante de equilíbrio em função das pressões parciais (K p): K p = = (1−α) =
PPCl5 [(1+α)]x pT
α2 x pT (0,80)2 x 1
(1−α2 )
= 1−(0,80)2 = 1,78

Item c) Como a temperatura do processo é constante (250°C), o valor da constante de equilíbrio será também
α² x pT
constante, ou seja, Kp = 1,78, mesmo apresentando um grau de dissociação distinto (0,30). Logo:K p = (1−α²)
=
(0,30)2 x p T
1−(0,30)²
= 1,78

(0,30)2 x pT
= 1,78
1 − (0,30)²
0,09 x pT
= 1,78
0,91
1,78 x 0,91
pT = = 17,98 atm
0,09
Questão 1071 – (OLIMPÍADA DE QUÍMICA - RGS) Um recipiente fechado contém o sistema gasoso, representado
pela equação: 2 SO2(g) + O2(g) ⇄ 2 SO3(g).
a) Calcular o valor da constante de equilíbrio, em termos de concentrações (Kc), considerando que 4 mol.L-1 de
SO2 reagem com 2 mol.L-1 de O2, obtendo-se, no equilíbrio, 2 mol.L-1 de SO3.
b) Calcular o valor da constante de equilíbrio, em termos de pressões parciais (Kp), considerando o recipiente
sob pressão de 3 atm, cuja composição no equilíbrio é de 0,8 mol de SO2, 0,4 mol de O2 e 0,8 mol de SO3.

Resolução: Base de cálculo: 4 mol.L-1 de SO2 e 2 mol.L-1 de O2 no estado inicial.


Equação química: 2 SO2(g) + O2(g) ⇄ 2 SO3(g)
Primeira tabela de equilíbrio químico:

2 SO2(g) (mol.L-1) O2(g) (mol.L-1) ⇄ 2 SO3(g) (mol.L-1)


Início 4 2 0
Reage 2α α 2α
Equilíbrio 4 - 2α 2-α 2 mol.L-1

Cálculo do α, sabendo que há 2 mol.L-1 de SO3(g) no equilíbrio: 0 + 2α = 2


α=1
Segunda tabela de equilíbrio químico:

2 SO2(g) (mol.L-1) O2(g) (mol.L-1) ⇄ 2 SO3(g) (mol.L-1)


Início 4 2 0
Reage 2x1 1 2x1
Equilíbrio 2 1 2

577
1600 FÍSICO-QUÍMICA APLICADA EXERCÍCIOS COMENTADOS - IME – ITA – OLIMPÍADA

[SO3 ]2 (2)2
Item a) Cálculo da constante de equilíbrio (Kc): K c = [SO 2 = (2)2 x 1 = 1
2 ] x [O2 ]

Item b) Cálculo do número de mol total (nT): nT = nSO2 + nO2 + nSO3 = 0,8 mol + 0,4 mol + 0,8 mol = 2,0 mol
Cálculo das pressões parciais:
0,80
Pressão Parcial do SO2: PSO2 = XSO2 x pT = 2
x 3 atm = 1,20 atm

0,40
Pressão Parcial do O2: PO2 = XO2 x pT = 2
x 3 atm = 0,60 atm

0,80
Pressão Parcial do HI: PSO3 = XSO3 x pT = 2
x 3 atm = 1,20 atm

P2SO3 (1,20)2
Cálculo da constante de equilíbrio em função das pressões parciais (Kp): K p = P2 = (1,20)2 x 0,60
= 1,67
SO2 x PO2

Questão 1072 – (SELETIVA PARA A 37TH INTERNATIONAL CHEMISTRY OLIMPIAD) Uma mistura de 11,02 mmol
de H2S e 5,48 mmol de CH4 foi colocada em um reator com um catalisador de platina e, o equilíbrio H2S(g) + CH4(g) ⇄
H2(g) + CS2(g), foi estabelecido em 700°C e 762 torr. Retirou-se o catalisador da mistura reacional e em seguida a
mistura foi resfriada. Através de análises no equilíbrio encontrou-se 0,711 mmol de CS2. Determine:
a) as frações molares de cada substância envolvida;
b) as porcentagens molares;
c) as pressões parciais;
d) Kp e o ΔG° para a reação em 700°C.

Resolução: Equação química balanceada: 2 H2S(g) + CH4(g) ⇄ 4 H2(g) + CS2(g)


Base de cálculo: 11,02 mmol de H2S(g) e 5,48 mmol de CH4(g) no estado inicial.
Tabela de equilíbrio químico:
2 H2S(g) (mmol) CH4(g) (mmol) 4 H2(g) (mmol) CS2(g) (mmol)
Início 11,02 5,48 0 0

Reage 2α α 4α α
Equilíbrio 11,02 - 2α 5,48 - α 4α α = 0,711 mmol

Cálculo do número de mol para cada espécie gasosa:


nH2S = 11,02 - 2α = 11,02 – 2 x (0,711) = 9,60 mmol
nCH4 = 5,48 - α = 5,48 - 0,711 = 4,77 mmol
nH2 = 4α = 4 x (0,711) = 2,84 mmol
nCS2 = 0 + α = 0,711 mmol
Cálculo do número de mol total: nT = nH2S + nCH4 + nH2 + nCS2 = 9,60 mmol + 4,77 mmol + 2,84 mmol + 0,711 mmol =
17,92 mmol
Item a) Frações molares de cada espécie gasosa:

nH2 S 9,60 mmol


X H2 S = = = 0,536
nT 17,92 mmol
nCH4 4,77 mmol
X CH4 = = = 0,266
nT 17,92 mmol

578
1600 FÍSICO-QUÍMICA APLICADA EXERCÍCIOS COMENTADOS - IME – ITA – OLIMPÍADA

nH2 2,84 mmol


X H2 = = = 0,158
nT 17,92 mmol
nCS2 0,711 mmol
X CS2 = = = 0,0397
nT 17,92 mmol
Item b) Determinação das percentagens molares: X H2 S = 0,536 (53,6%); X CH4 = 0,266 (26,6%); X H2 =
0,158 (15,8%) e X CS2 = 0,0397 (3,97%)

Item c) Cálculo das pressões parciais:

PH2 S = X H2 S x pTotal = 0,536 x 762 torr = 408,4 torr

PCH4 = X CH4 x pTotal = 0,266 x 762 torr = 202,7 torr

PH2 = X H2 x pTotal = 0,158 x 762 torr = 120,4 torr

PCS2 = X CS2 x pTotal = 0,0397 x 762 torr = 30,25 torr

Item d) Cálculo da constante de equilíbrio em função das pressões parciais:

Equação química: 2 H2S(g) + CH4(g) ⇄ 4 H2(g) + CS2(g)


P4H x PCS2 (120,4)4 x 30,25
Cálculo da constante de equilíbrio (Kp): K p = P2 2 x P = (408,4)2 x 202,7
= 188,20
H2 S CH4

Cálculo da variação da energia livre de Gibbs (ΔG°): ∆G° = −R x T x lnK p


J
∆G° = −8,314 x (700 + 273)K x ln (188,20)
mol x K
J
∆G° = −8,314 x 973 x 5,24
mol
J kJ
∆G° = −42389,09 (−42,4 ) , processo espontâneo
mol mol
Questão 1973 – Demonstre que para o seguinte equilíbrio químico, representado pela equação química N2O4(g) ⇄ 2
4α²
NO2(g), que a constante Kp é dada pela seguinte relação K p = x pTOTAL .
(1− α2 )

Resolução: Equação química: N2O4(g) ⇄ 2 NO2(g)


Base de cálculo: n (mol) inicial de N2O4(g) no estado inicial.
Tabela de equilíbrio:

N2O4(g) ⇄ 2 NO2(g)
Início n 0
Reage nα 2nα
Equilíbrio n - nα 2nα

Cálculo do número de mol total (nT) das espécies gasosas: nT = n – nα + 2nα = n + nα = n(1 + α)
Expressão das pressões parciais:
n−nα 1−α
Pressão Parcial do N2O4: PN2O4 = XN2O4 x pT = (n+nα) x pT = (1+ α) x pT

579
1600 FÍSICO-QUÍMICA APLICADA EXERCÍCIOS COMENTADOS - IME – ITA – OLIMPÍADA

2nα 2α
Pressão Parcial do NO2: PNO2 = XNO2 x pT = (n+nα) x pT = (1+ α) x pT

Expressão da constante de equilíbrio em função do grau de dissociação (𝛼):


4α2 2 4α2
2
PNO [ x p TOTAL ] [ x p2TOTAL ] 4α²
2 (1 + α)2 (1 + α)2
Kp = = = = x PTOTAL
PN2 O4 (1 − α) (1 − α) 1 − α²
[ x PTOTAL ] [ x PTOTAL ]
(1 + α) (1 + α)
Questão 1074 – (U. S. NATIONAL CHEMISTRY OLYMPIAD) Uma amostra de XCl3(g) de 0,0343 mol é adicionado
em um recipiente com capacidade de 1,50 litros. O recipiente é selado e aquecido a 100°C, e com isso, o equilíbrio
foi estabelecido através da seguinte equação química: 2 XCl3(g) ⇄ 2 X(g) + 3Cl2(g). No equilíbrio, a pressão no recipiente
é igual a 0,800 atm.
a) Calcule a pressão de XCl3(g) no recipiente a 100°C se não ocorre reação.
b) Calcule a pressão parcial no equilíbrio de PXCl3, PX e PCl2.
c) Determine o valor a 100°C do Kp e ∆G° em kJ.mol-1.
d) Calcule o valor de Kc a 100°C.

Resolução: Item a) Cálculo da pressão de XCl3(g), a partir da equação dos gases ideais:

0,0343 x 0,08206 x (100 + 273)


pXCl3 = = 0,700 atm
1,50

Item b) Equação química balanceada: 2 XCl3(g) ⇄ 2 X(g) + 3Cl2(g)

Base de cálculo: 0,700 atm de XCl3(g) no estado inicial


Tabela de equilíbrio químico:
2 XCl3(g) (atm) 2 X(g) (atm) 3 Cl2(g) (atm)
Início 0,700 0 0
Reage 2X 2X 3X
Equilíbrio 0,700 – 2X 2X 3X

Cálculo do X, considerando que a pressão total é igual a 0,800 atm: 0,700 – 2X + 2X + 3X = 0,800
X = 0,033
Determinação das pressões parciais:
pXCl3 = 0,700 − 2X = 0,700 − 2 x (0,033) = 0,634 atm

pX = 2X = 2 x (0,033) = 0,066 atm

pCl2 = 3X = 3 x (0,033) = 0,099 atm

P2X x P3Cl2 (0,066)2 x (0,099)3


Cálculo da constante de equilíbrio (Kp): K p = = = 1,05 x 10−5
P2XCl (0,634)²
3

Cálculo da variação da energia livre de Gibbs (∆G°): ∆G° = −R x T x lnK p


𝐽 𝑘𝐽
∆G° = −8,314 x (100 + 273) x ln(1,05 x 10−5 ) = 35551,68 (35,55 )
𝑚𝑜𝑙 𝑚𝑜𝑙
Sendo ∆G° > 0, o processo se caracteriza como não espontâneo.

580
1600 FÍSICO-QUÍMICA APLICADA EXERCÍCIOS COMENTADOS - IME – ITA – OLIMPÍADA

Para uma temperatura de 373 K e com uma constante dos gases igual a 0,08206 atm.L.mol -1.K-1, o valor de Kc será
igual a:

K p = K c x (RT)∆n

1,05 x 10−5 = K c x (0,08206 x 373)3

1,05 x 10−5
Kc = = 3,66 𝑥 10−10
(0,08206 x 373)3

Questão 1075 – (IME – MODIFICADA) Em um recipiente de dois litros, que se encontrava completamente vazio, sob
vácuo, foi colocada uma amostra de 10,0 gramas de PCl5. Em seguida, a amostra foi aquecida a 500 K, ocorrendo a
decomposição do PCl5, conforme a equação química: PCl5(g) ⇄ PCl3(g) + Cl2(g). Sabendo que, no equilíbrio, a pressão
medida no recipiente foi de 1,551 atmosferas e que todos os gases envolvidos são de comportamento ideal, calcule
a constante de equilíbrio na reação de decomposição do PCl5.

Resolução: Equação química: PCl5(g) ⇄ PCl3(g) + Cl2(g)


Base de cálculo: n (mol) inicial de pentacloreto de fósforo no estado inicial.
Tabela de equilíbrio químico:

PCl5(g) ⇄ PCl3(g) Cl2(g)


Início n 0 0
Reage nα nα nα
Equilíbrio n - nα nα nα

Cálculo do número de mol total (nT): nT = n – nα + nα + nα = n + nα = n(1 + α)

Cálculo do grau de dissociação (α) do pentacloreto de fósforo: p x V = n x (1 + α) x R x T


m
pxV= x (1 + α) x R x T
< MM >
10
1,551 x 2 = x (1 + α) x 0,08206 x 500
208,5
1,551 x 2 x 208,5
1+α=
10 x 0,08206 x 500
α = 0,58 (58%)
Expressão das pressões parciais dos compostos gasosos:
n−nα 1−α
Pressão Parcial do PCl5: PPCl5 = XPCl5 x pT = (n+nα) x pT = (1+ α) x pT

nα α
Pressão Parcial do PCl3: PPCl3 = XPCl3 x pT = (n+nα) x pT = (1+ α) x pT

nα α
Pressão Parcial do C2H4: PCl2 = XCl2 x pT = (n+nα) x pT = (1+ α) x pT

581
1600 FÍSICO-QUÍMICA APLICADA EXERCÍCIOS COMENTADOS - IME – ITA – OLIMPÍADA

α2
PPCl3 x PCl2 [(1+α)2 ]x p2T
Expressão da constante de equilíbrio em função das pressões parciais (K p): K p = PPCl5
= (1−α) =
[(1+α)]x pT
α² x pT
(1−α²)

α2
PPCl3 x PCl2 [(1+α)2]x p2T α2 x p
Cálculo da constante de equilíbrio (Kp): K p = = (1−α) = (1−α2T)
PPCl5 [(1+α)]x pT

(0,58)2 x 1,551 0,522


Kp = = = 0,786
1 − (0,58)2 0,664
Questão 1076 – (ITA) Dentro de um forno, mantido numa temperatura constante, temos um recipiente contendo 0,50
moles de Ag(s), 0,20 moles de Ag2O(s) e oxigênio gasoso exercendo uma pressão de 0,20 atm. As três substâncias
estão em equilíbrio químico. Caso a quantidade de Ag2O(s) dentro do recipiente, na mesma temperatura, fosse 0,40
moles, a pressão, em atmosferas, do oxigênio no equilíbrio seria:
a) 0,10
b) 0,20
c) 0,40
d) (0,20)1/2
e) 0,80

Resolução: Alternativa B.
A quantidade de óxido de prata [Ag2O(s)] não influencia na pressão parcial em relação ao oxigênio, pois sendo o óxido
de prata um sólido, sua atividade é igual a um (𝒂𝑨𝒈𝟐𝑶 = 𝟏), logo não faz parte da equação da constante de equilíbrio
em função da pressão parcial (Kp). Com isso, a pressão parcial de oxigênio ficará a mesma, ou seja, 0,20 mol.

Questão 1077 – (IME) Dada a reação química abaixo que ocorre na ausência de catalisadores, H2O(g) + C(s) +
31,40kcal ⇄ CO(g) + H2(g) pode-se afirmar:
a) O denominador da expressão da constante de equilíbrio é [H2O] x [C];
b) Se for adicionado mais monóxido de carbono ao meio reacional, o equilíbrio se desloca para a direita;
c) O aumento da temperatura da reação favorece a formação dos produtos;
d) Se fossem adicionados catalisadores, o equilíbrio iria se deslocar tendo em vista uma maior formação de
produtos;
e) O valor da constante de equilíbrio é independente da temperatura.

Resolução: Alternativa C.
[CO] x [H2 ]
a) Falsa. A expressão da constante de equilíbrio é a seguinte: K c = [H2 O]
, ou seja, no denominador da
expressão da constante de equilíbrio há a concentração da quantidade de matéria da água;
b) Falsa. A adição de monóxido de carbono o equilíbrio será deslocado para a esquerda;
c) Correta. Sendo a reação endotérmica e com o aumento da temperatura, o equilíbrio será deslocado para a
direita, favorecendo os produtos da reação direta;
d) Falsa. A função do catalisador é aumentar a velocidade da reação e por consequência acarreta a diminuição
da energia de ativação, não havendo influência na constante de equilíbrio;
e) Falsa. A constante de equilíbrio é dependente diretamente da temperatura, o que pode ser observado pela
dlnK ∆H0
equação de Van´t Hoff, dT
= R x T².

Questão 1078 – (IME) Entre as reações abaixo, seguidas das respectivas constantes de equilíbrio, assinale aquela
que uma vez atingido o equilíbrio, a partir das quantidades estequiométricas dos reagentes, pode ser considerada a
mais completa no sentido da esquerda para a direita:
a) CH3COOH(aq) ⇄ H+(aq) + CH3COOH-(aq) Kc = 1,8 x 10-5
b) CdS(s) ⇄ Cd (aq) + S (aq)
+2 -2 Kc = 7,1 x 10-28
c) H+(aq) + HS-(aq) ⇄ H2S(aq) Kc = 1,0 x 107
d) 2 HI(g) ⇄ H2(g) + I2(g) Kc = 9,0

582
1600 FÍSICO-QUÍMICA APLICADA EXERCÍCIOS COMENTADOS - IME – ITA – OLIMPÍADA

e) CO2(g) + H2(g) ⇄ CO(g) + H2O(g) Kc = 67

Resolução: Alternativa C.
Aa constante de equilíbrio é definida como a razão da concentração da quantidade de matéria produtos pela
concentração da quantidade dos reagentes, então pelas alternativas apresentadas a que será mais completa em
atingir o equilíbrio química, no sentido da esquerda para direita trata-se da alternativa C. Isto se deve ao fato desta
reação apresentar o maior valor de Kc.
[H2 S]
Kc = = 1,0 x 10+7
[H ] x [HS − ]
+

[H2 S] = 1,0 x 10+7 x [H + ] x [HS − ]


A relação acima comprova que o equilíbrio será deslocado para a direita.
Questão 1079 – (IME) Considere a reação, em equilíbrio, de produção do alvejante gasosos dióxido de cloro, que
ocorre em um sistema reacional: Cl2(g) + 2 NaClO4(s) ⇄ 2 ClO2(g) + 2 NaCl(s). Nessa situação, assinale a alternativa
correta:
a) A adição de mais clorito de sódio ao sistema desloca o equilíbrio da reação, de forma a produzir mais
alvejante gasoso.
𝐾
b) A razão entre as constantes de equilíbrio 𝑝 é igual a 0,0820568 x T, em que T é a temperatura do sistema
𝐾𝑐
reacional, medida em Kelvin.
c) A retirada parcial de cloreto de sódio do sistema desloca o equilíbrio da reação, de forma a produzir menos
alvejante.
d) A constante de equilíbrio Kp é igual a constante de equilíbrio Kc.
e) Para duas diferentes temperaturas do sistema reacional, desde que elevadas e compatíveis com a
manutenção do equilíbrio, o valor numérico da constante de equilíbrio Kp é o mesmo, mantendo inalterada a produção
de alvejante gasoso.

Resolução: Alternativa B.
a) Incorreta. A adição de clorito de sódio (NaClO2) não afeta o equilíbrio químico, uma vez que se trata de um
composto sólido, ou seja, apresenta atividade unitária (aNaClO2 = 1).

b) Correta. Sabendo que a relação entre a constante de equilíbrio em função da pressão parcial com a constante de
equilíbrio em função da concentração da quantidade de matéria é definida pela seguinte equação: K p =
K c x (R x T)∆n , onde Δn = nprodutos – nreagentes = 2 – 1 = 1. Logo:

Kp
= (R x T)∆n = (R x T)1 = 0,08206 x T
Kc
c) Incorreta. A diminuição de cloreto de sódio não influencia no equilíbrio químico, uma vez que, por convenção a
atividade de um sólido é unitário, ou seja, (aNaCl = 1).

Incorreta. A relação entre a constante de equilíbrio em função da pressão parcial (Kp) com a constante de equilíbrio
Kp
em função da concentração da quantidade de matéria é dada pela seguinte relação Kc
= R x T, conforme já
mencionado no item b.
dlnK ∆H0
d) Incorreta. Através da equação de Van’t Hoff, dT
= R x T² a constante de equilíbrio (K) apresenta variação com
a mudança de temperatura.

583
1600 FÍSICO-QUÍMICA APLICADA EXERCÍCIOS COMENTADOS - IME – ITA – OLIMPÍADA

Questão 1080 – (ITA) As opções a seguir se referem a equilíbrios químicos que foram estabelecidos dentro de
cilindros providos de êmbolo. Se o volume interno em cada cilindro for reduzido à metade, a temperatura
permanecendo constante, em qual das opções a seguir o ponto de equilíbrio será alterado?
a) H2(g) + l2(g) ⇄ 2HI(g)
b) CaCO3(s) ⇄ CaO(s) + CO2(g)
c) PbS(s) + O2(g) ⇄ Pb(s) + SO2(g)
d) CH4(g) + 2O2(g) ⇄ CO2(g) + 2H2O(g)
e) Fe2O3(s) + 3CO(g) ⇄ 2Fe(s) + 3CO2(g)

Resolução: Alternativa B.
Observa-se que a única reação química que não será alterada é a alternativa b. Isto se deve ao fato de que esta
reação é a única em que o número de mol de gás não se encontra equilibrada, o que nas demais reações químicas
apresentadas.
a) 2 mol de gás nos reagentes e também 2 mol de gás nos produtos;
b) 0 mol de gás nos reagentes e 1 mol de gás nos produtos;
c) 1 mol de gás nos reagentes e no produto;
d) 3 mol de gás nos reagentes e nos produtos;
e) 3 mol de gás nos reagentes e nos produtos.

Questão 1081 – (ITA) Dentro de um recipiente fechado, de volume V, se estabelece o seguinte equilíbrio: N2(g) + 3H2(g)
⇄ 2NH3(g); KC. As quantidades (mol) de N2, H2 e NH3 no equilíbrio são, respectivamente, nN2, nH2 e nNH3. Assinale a
opção que contém a expressão que representa corretamente a constante KC para o equilíbrio equacionado acima:
n2NH3
a) 3
nH x nN2
2
nNH3 𝑥 𝑉
b) nH2 x nN2
n2NH3 𝑥 𝑉²
c)
n3H x nN2
2
n3H2 x nN2
d)
n2NH3 𝑥 𝑉²
4 𝑥 n2NH3 x 𝑉²
e) 27 𝑥 n3H x nN2
2

Resolução: Alternativa C.
N2(g) + 3H2(g) ⇄ 2NH3(g)
[NH3 ]2
Expressão da constante de equilíbrio: K c = [N 3
2 ] x [H2 ]

[NH3 ]2
Relação do número de mol de cada substância com o volume: K c = [N 3
2 ] x [H2 ]

nNH 2
[ V 3]
Kc =
nN nH 3
[ V2 ] x [ V2 ]
2
[nNH3 ] 𝑥 V 2
Kc = 3
nN2 x [nH2 ]

584
1600 FÍSICO-QUÍMICA APLICADA EXERCÍCIOS COMENTADOS - IME – ITA – OLIMPÍADA

Questão 1082 - (ITA) Carbamato de amônio sólido (NH2COONH4) decompõe-se em amônia e dióxido de carbono,
ambos gasosos. Considere que uma amostra de carbamato de amônio sólido esteja em equilíbrio químico com CO 2(g)
e NH3(g) na temperatura de 50°C, em recipiente fechado e volume constante. Assinale a opção correta que apresenta
a constante de equilíbrio em função da pressão total P, no interior do sistema.
a) 3P
b) 2P²
c) P³
2
d) 9
𝑥 𝑃2
4
e) 27
𝑥 𝑃3

Resolução: Alternativa E.
Equação de decomposição do carbamato de amônio sólido: NH2COONH4(s) ⇄ 2 NH3(g) + CO2(g)
Base de cálculo: n (mol) inicial de carbamato de amônio sólido no início da reação.
Tabela de equilíbrio químico:

NH2COONH4(s) (mol) 2 NH3(g) (mol) CO2(g) (mol)


Início n 0 0
Reage nα 2nα nα
Equilíbrio n - nα 2nα nα

Cálculo do número de mol total somente das espécies gasosas: nT = nNH3 + nCO2 = 2nα + nα = 3nα

Determinação das das pressões parciais dos compostos gasosos:


2nα 2
Amônia: pNH3 = x ptotal = x ptotal
3nα 3
nα 1
Gás carbônico: pCO2 = x ptotal = x ptotal
3nα 3

Cálculo da constante de equilíbrio em função das pressões parciais (Kp):


2
2 1 4
Kp = p2NH3 x pCO2 = [ x ptotal ] x x ptotal = x p3total
3 3 27

Questão 1083 – (ITA) Calcular a constante de equilíbrio Kp, à temperatura T, do sistema H2O(g) ⇌ 2 H2(g) + O2(g),
sabendo que nessa temperatura existem em equilíbrio 7 moles de vapor d’ água, 1 mol de oxigênio e 2 moles de
hidrogênio. Qual é a unidade de medida de Kp, neste caso?

Resolução: Equação química: 2 H2O(g) ⇌ 2 H2(g) + O2(g)

Cálculo do número de mol total: ntotal = nH2 O + nH2 + nO2 = 7 mol + 2 mol + 1 mol = 10 mol

Cálculo das pressões parciais dos compostos gasosos:

nH2 O 7 mol
PH2 O = x Ptotal = x 10 atm = 7 atm
ntotal 10 mol

nH2 2 mol
PH2 = x Ptotal = x 10 atm = 2 atm
ntotal 10 mol

nO2 1 mol
PO2 = x Ptotal = x 10 atm = 1 atm
ntotal 10 mol

(PH2 )² x PO2 (2)² x 1 4


Cálculo da constante de equilíbrio em função das pressões parciais: K p = = (7)²
= 49
(PH2O )²

585
1600 FÍSICO-QUÍMICA APLICADA EXERCÍCIOS COMENTADOS - IME – ITA – OLIMPÍADA

A constante de equilíbrio não apresenta unidade.


Questão 1084 – (ITA) Um recipiente fechado, mantido a volume e temperatura constantes, contém a espécie química
X no estado gasoso a pressão inicial Po. Esta espécie decompõe-se em Y e Z de acordo com de acordo com a seguinte
equação química: X(g) → 2Y(g) + ½Z(g). Admita que X, Y e Z tenham comportamento de gases ideais. Assinale a opção
que apresenta a expressão correta da pressão (P) no interior do recipiente em função do andamento da reação, em
termos da fração α de moléculas de X que reagiram.
1
a) P = [1 + (2) . α] . P0
2
b) P = [1 + (2) . α] . P0
3
c) P = [1 + (2) . α] . P0
4
d) P = [1 + ( ) . α] . P0
2
5
e) P = [1 + (2) . α] . P0

Resolução: Alternativa C.
Equação química de decomposição química a ser estudada: X(g) ⇄ 2 Y(g) + ½ Z(g)
Base de cálculo: pressão inicial (Po) para a espécie gasosa X.
Tabela de equilíbrio químico:

X(g) ⇄ 2Y(g) ½ Z(g)


Início P0 0 0
Reage P0α 2P0α ½P0α
Equilíbrio P0 - P0α 2P0α ½P0α

Determinação da pressão total (P): P = Px + PY + PZ


1
P = P0 − P0 α + 2P0 α + P0 α
2
(2P0 + 2P0 α + P0 α)
P=
2
2 𝑥 P0 (3P0 α) 3 3
P= + = P0 + P0 α = P0 𝑥 [1 + α]
2 2 2 2

Questão 1085 – (ITA) Num recipiente de volume constante igual a 1,00 litro, inicialmente evacuado, foi introduzido
1,00 mol de pentacloreto de fósforo gasoso e puro. O recipiente foi mantido a 250°C e no equilíbrio final foi verificada
a existência de 0,47 mol de gás cloro. Qual das opções a seguir contém o valor aproximado da constante (K c) do
equilíbrio estabelecido dentro do cilindro e representado pela seguinte equação química: PCl5(g) ⇄ PCl3(g) + Cl2(g).
a) 0,179
b) 0,22
c) 0,42
d) 2,38
e) 4,52

Resolução: Alternativa C.
Equação química de decomposição do pentacloreto de fósforo: PCl5(g) ⇄ PCl3(g) + Cl2(g).
Base de cálculo: 1 mol inicial de PCl5(g).
Tabela de equilíbrio químico:

586
1600 FÍSICO-QUÍMICA APLICADA EXERCÍCIOS COMENTADOS - IME – ITA – OLIMPÍADA

PCl5(g) ⇄ PCl3(g) Cl2(g)


Início 1mol 0 0
Reage α α α
1−α α α
Equilíbrio
V V V

O enunciado do problema coloca a seguinte situação: “equilíbrio final foi verificada a existência de 0,47 mol de gás
cloro“, logo a tabela de equilíbrio acima ficará da seguinte maneira:

PCl5(g) ⇄ PCl3(g) Cl2(g)


Início 1mol 0 0
Reage 1α = 0,47 mol 1.α = 0,47 mol 0,47 mol
Equilíbrio 1 - 0,47 = 0,53mol / 1L 0,47 mol / 1 L 0,47 mol / 1 L

[PCl3 ] x [Cl2 ] (0,47).(0,47)


Expressão e cálculo da constante de equilíbrio (Kc): K c = [PCl5 ]
= 0,53
= 0,42

Questão 1086 – (ITA) A reação de síntese do metanol a partir do gás d’água é representada por: CO(g) + 2 H2(g) ⇄
CH3OH(g) + calor. Com base no princípio de Le Chatelier é possível prever como, partindo de uma certa quantidade
de CO, pode-se aumentar a quantidade de metanol gasoso resultante do equilíbrio. Abaixo são dadas algumas
alterações que poderiam contribuir para esse aumento. Uma delas não está relacionada com o princípio de Le
Chatelier. Qual é?
a) Aumento da quantidade de hidrogênio a volume constante.
b) Acréscimo de catalisador para a reação.
c) Diminuição da temperatura do sistema.
d) Aumento da pressão sobre o sistema.
e) Condensação do metanol à medida que ele se forma.

Resolução: Alternativa B.
O objetivo da questão é de que o equilíbrio seja deslocado para a direita, ou seja, favorecendo maior produção de
metanol. Possíveis alternativas:
Aumento da concentração da quantidade de matéria de monóxido de carbono gasoso;
Aumento da concentração da quantidade de matéria de hidrogênio gasoso;
Diminuição da temperatura do sistema reacional;
Aumento da pressão do sistema reacional;
Condensação do metanol.
Observação: O uso do catalisador não modifica o equilíbrio químico de uma reação química.
Questão 1087 – (ITA) Considere a reação de dissociação do N2O4(g) representada pela segunda equação: N2O4(g) ⇄
2 NO2(g). Assinale a opção correta que relaciona a fração percentual (α) de N2O4(g) dissociado com a pressão total (P)
e com a constante de equilíbrio em termos de pressão (Kp).
𝐾𝑝
a) 𝛼 = √4𝑝+ 𝐾
𝑝
4𝑝+ 𝐾𝑝
b) 𝛼=√ 𝐾𝑝
𝐾𝑝
c) 𝛼 = 2𝑝+ 𝐾
𝑝
2𝑝+ 𝐾𝑝
d) 𝛼= 𝐾𝑝
𝐾𝑝
e) 𝛼 = 2+𝑝

Resolução: Alternativa A.

587
1600 FÍSICO-QUÍMICA APLICADA EXERCÍCIOS COMENTADOS - IME – ITA – OLIMPÍADA

Equação química: N2O4(g) ⇄ 2 NO2(g)

Base de cálculo: n (mol) inicial de tetróxido de dinitrogênio no início da reação.

Tabela de equilíbrio químico:

N2O4(g) (mol) ⇄ 2 NO2(g) (mol)


Início n 0
Reage nα 2nα
Equilíbrio n - nα 2nα

Cálculo do número de mol total das espécies gasosas: nT = nN2O4 + nNO2


nT = n - nα + 2nα = n.(1 + α)

Expressões das pressões parciais:

2nα 2α
PNO2 = x pT = x pT
n. (1 + α) (1 + α)

n. (1 − α) (1 − α)
PN2 O4 = x pT = x pT
n. (1 + α) (1 + α)

Determinação do grau de dissociação (α) em função da pressão total (P) e da constante de equilíbrio (K p): K p =
2α 2
P2NO2 [
(1+α)
x pT ]
PN2O4
= (1−α)
x pT
(1+α)

4α²
[ ] . p2
(1 + α)² T 4α². pT
Kp = =
(1 − α) (1 − α²)
[ ].p
(1 + α) T

4α².p
Isolando o grau de dissociação, temos: K p = (1−α²)T

K p . (1 − α²) = 4α². pT

Chamando a pressão total de P, temos: K p . (1 − α²) = 4α². P


α2 . (4P + K p ) = K p
Kp
α2 =
(4P + K p )

Kp
α=√
4P + K p

588
1600 FÍSICO-QUÍMICA APLICADA EXERCÍCIOS COMENTADOS - IME – ITA – OLIMPÍADA

Questão 1088 – (ITA) Quantidades iguais de H2(g) e I2(g) foram colocadas em um frasco, com todo o sistema à
temperatura T, resultando na pressão total de 1 bar. Verificou-se que houve a produção de HI(g), cuja pressão parcial
foi de 22,8 kPa. Assinale a alternativa que apresenta o valor que mais se aproxima do valor correto da constante de
equilíbrio desta reação.
a) 0,295
b) 0,350
c) 0,490
d) 0,590
e) 0,700

Resolução: Alternativa B.

Equação química balanceada: H2(g) + I2(g) ⇄ 2 HI(g)


Base de cálculo: n mol de H2 e n mol de I2 no estado inicial.
Tabela de equilíbrio químico:
H2(g) (mol) I2(g) (mol) ⇄ 2 HI(g) (mol)
Início n n 0
Reage nα nα 2nα
Equilíbrio n - nα n - nα 2nα

Cálculo do número de mol total (nT) dos gases: nT = nH2 + nI2 + nHI
nT = n – nα + n – nα + 2nα = 2n
Expressões das pressões parciais:
n(1−α) (1−α)
Pressão Parcial do H2: PH2 = X H2 x ptotal = 2n
x ptotal = 2
x ptotal

n(1−α) (1−α)
Pressão Parcial do I2: PI2 = X I2 x ptotal = 2n
x ptotal = 2
x ptotal

2nα
Pressão Parcial do HI: P𝐻𝐼 = X HI x ptotal = 2n
x ptotal = α. ptotal

Cálculo do grau de dissociação a partir da pressão parcial do iodeto de hidrogênio: PHI = α.pT
22,8 kPa = α.100 kPa
22,8 kPa
α= = 0,228
100 kPa
Cálculo das pressões parciais de H2 e I2:
(1−0,228)
Pressão Parcial do I2: PI2 = X I2 x ptotal = x 100 kPa = 38,6 kPa
2

(1−0,228)
Pressão Parcial do H2: PH2 = X H2 x ptotal = 2
x 100 kPa = 38,6 kPa

P2HI (22,8)2
Cálculo da constante de equilíbrio em função das pressões parciais: K p = P = (38,6)2 = 0,349
H2 x PI2

589
1600 FÍSICO-QUÍMICA APLICADA EXERCÍCIOS COMENTADOS - IME – ITA – OLIMPÍADA

Questão 1089 – (ITA) O princípio de Le Chatelier pode ser enunciado assim: “se um sistema em equilíbrio é sujeito
a uma perturbação, ocorre um deslocamento do equilíbrio, que tende a minimizar a perturbação imposta”. Assim,
podemos esperar um deslocamento para a direita, na(s) seguinte(s) reação(ões), se a mistura de gases é comprimida:
I. H2 + I2 ⇄ 2 HI
II. N2 + 3H2 ⇄ 2 NH3
III. CO + NO2 ⇄ CO2 + NO
IV. N2O4 ⇄ 2NO2
a) II
b) II e IV
c) I e III
d) IV
e) todas

Resolução: Alternativa A.
O objetivo da questão é que o equilíbrio seja deslocado para a direita. A partir do momento em que os gases sejam
comprimidos, ou seja, diminuição do volume consequentemente aumento da pressão do sistema, a única opção que
retrata este propósito é a síntese da amônia.
Questão 1090 – (ITA) Num recipiente mantido a pressão e temperatura ambiente, foram introduzidas 1,00 mol de
etanol, x mol de ácido acético, um pouco de um catalisador adequado e um solvente inerte para que o volume final
da mistura homogênea líquida fosse 5,0 litros. Nestas condições se estabelece o equilíbrio correspondente à equação
química: C2H5OH(SOLV) + CH3COOH(SOLV) ⇄ CH3COOC2H5(SOLV) + H2O(SOLV). A constante de equilíbrio é 4,0 na
temperatura ambiente. Uma vez atingido o equilíbrio, verifica-se que o sistema contém 0,50 mol de acetato de etila.
Destas informações podemos concluir que a quantidade x inicial posta de ácido acético é:
a) 0,25
b) 0,38
c) 0,50
d) 0,63
e) 0,75

Resolução: Alternativa D.
Base de cálculo: 1 mol de etanol no início da reação, x mol de ácido acético no início da reação e 0,5 mol de acetado
de etila no estado de equilíbrio.
Tabela de equilíbrio químico:
CH3COOH(aq) C2H5OH(aq) ⇄ CH3COOC2H5(aq) H2O(l)
Início x 1 0 0
Reage 1α 1α 1α 1α
Equilíbrio x - α 1-α α α

[CH COOC H ].[H2 O]


Expressão da constante de equilíbrio (Kc): K c = [CH 3COOH]2x [C
5
3 2 H5 OH]

Cálculo de incógnita α: α + 0 = 0,50


α = 0,50
[CH COOC2 H5 ].[H2 O]
Cálculo do valor de α, a partir da constante de equilíbrio: K c = [CH 3
3 COOH] x [C2 H5 OH]

[CH3 COOC2 H5 ] x [H2 O] (0,50)²


Kc = =
[CH3 COOH] x [C2 H5 OH] 0,50 x (α − 0,50)
(0,50)²
Para Kc = 4,0, temos: 0,50 x (α−0,50) = 4

(0,50)2 = 4 x 0,50 x (α − 0,50)

590
1600 FÍSICO-QUÍMICA APLICADA EXERCÍCIOS COMENTADOS - IME – ITA – OLIMPÍADA

(0,50)2 = 2 x (α − 0,50)

α = 0,625 mol
Questão 1091 - (ITA) Num cilindro com pistão móvel provido de torneira conforme figura ao lado, se estabeleceu o
equilíbrio:

N2O4(g) ⇄ 2NO2(g); Kc

Mantendo a temperatura constante, pode-se realizar as seguintes modificações:


(I) Reduzir o volume, por deslocamento do pistão.
(II) Introduzir mais NO2(g) pela torneira, o pistão permanecendo fixo.
(III) Introduzir mais N2O4(g) pela torneira, o pistão permanecendo fixo.
(IV) Introduzir Argônio pela torneira, o pistão permanecendo fixo.
Qual ou quais das alternativas acima irá ou irão provocar deslocamento do equilíbrio para a esquerda, isto é, irá
acarretar a produção de mais N2O4(g) dentro do cilindro?
a) Apenas (I)
b) Apenas (III)
c) Apenas (I) e (II)
d) Apenas (II) e (IV)
e) Apenas (I), (II) e (III)

Resolução: Alternativa C.
Para maior produção de tetróxido de dinitrogênio, o equilíbrio deverá ser deslocado para a esquerda, através dos
seguintes procedimentos:

• Aumentar a pressão do sistema reacional;


• Aumentar a pressão parcial do dióxido de nitrogênio, NO2(g);
• Diminuir o volume do sistema reacional.

Questão 1092 - (ITA) Considere o equilíbrio 2 HI(g) ⇆ H2(g) + I2(g), Kc sabe-se que a reação no sentido da esquerda
para a direita é endotérmica na faixa de temperatura considerada. Assinale a afirmação incorreta:
a) o valor da constante de equilíbrio a 500ºC será maior que o valor a 250ºC
b) se [H2] for aumentada por acréscimo de iodo, [HI] também aumentará
c) o equilíbrio será deslocado para a direita se a pressão for aumentada
d) o equilíbrio não será afetado pela adição de um catalisador
e) das afirmações no enunciado também segue que: ½ H2(g) + ½ I2(g) ⇆ HI(g), K’ c = (1/Kc)1/2

Resolução: Alternativa C.
a) Correto. Como a reação é endotérmica, quanto maior a temperatura, maior a constante de equilíbrio, o que pode
ser observado na equação de Van´t Hoff;
b) Correto. Aumentando a concentração de iodo, o equilíbrio será deslocado para a esquerda, aumentando a
concentração de iodeto de hidrogênio;
c) Incorreta. Na reação a variação de número de mol gasoso é igual a zero, logo o número de mol de gás no produto
é o mesmo do reagente. Neste caso, a pressão não desloca o equilíbrio químico.
d) Correto. A adição de catalisador não influencia a constante de equilíbrio.
e) Correto. A constante de equilíbrio para a reação direta 2 HI(g) ⇆ H2(g) + I2(g) é dada pela seguinte equação: 𝐾𝑐 =
[𝐻2 ].[𝐼2 ]
[𝐻𝐼]2
. A constante de equilíbrio para a reação inversa e multiplicada por dois é representada pela seguinte equação:
[𝐻𝐼] 1 1/2
𝐾𝑐´ = [𝐼 1/2 .[𝐻 ]1/2 . Logo, a relação entre a primeira equação com a segunda equação: 𝐾𝑐´ = (𝐾 ) .
2] 2 𝑐

591
1600 FÍSICO-QUÍMICA APLICADA EXERCÍCIOS COMENTADOS - IME – ITA – OLIMPÍADA

Questão 1093 - (OLIMPÍADA DE QUÍMICA DO RIO DE JANEIRO) Recomenda-se aos fumantes que abandonem o
vício, já que, dentre os vários produtos formados pela queima do fumo está o monóxido de carbono. Esse composto
não reage com a água, pois se trata de um óxido neutro; porém, reage com a hemoglobina que existe no sangue,
impedindo-a de transportar o oxigênio para as várias partes do organismo. De acordo com a OMS, em ambientes
fechados, o monóxido de carbono à concentração de 10% é fatal em dois minutos. (Adaptado de "Época", 09/06/2003).
O equilíbrio se estabelece com base na reação HmO2(aq) + CO(g) ⇄ HmCO(aq) + O2(g), sendo o valor de K = 210. Estima-
se que os pulmões de um fumante estejam expostos a uma concentração de monóxido de carbono igual a 2,2 x 10–6
mol.L-1 e de gás oxigênio igual a 8,8 x 10–3 mol.L-1. Nesse caso, a razão entre a concentração de hemoglobina ligada
ao monóxido de carbono [HmCO] e a concentração de hemoglobina ligada ao oxigênio [HmO2] é de:
a) 5,75 x 10–2
b) 5,25 x 10–2
c) 2,50 x 10–2
d) 4,50 x 10–3
e) 4,00 x 10–3

Resolução: Alternativa B.
Equação química balanceada: HmO2(aq) + CO(g) ⇄ HmCO(aq) + O2(g)
[HmCO] x [O2 ]
Expressão da constante de equilíbrio: K c = [HmO .
2 ] x [CO]

[HmCO]
Cálculo da relação [HmO ] :
2

[HmCO] x [O2 ]
Kc =
[HmO2 ] x [CO]
[HmCO] x 8,80 x 10−3
210 =
[HmO2 ] x 2,20 x 10−6
[HmCO] 210 x 2,20 x 10−6
= = 5,25 x 10−2
[HmO2 ] 8,80 x 10−3
Questão 1094 - (OLIMPÍADA BRASILEIRA DE QUÍMICA) Em um recipiente fechado tem-se o seguinte equilíbrio:
N2O4(g) ⇄ 2 NO2(g), com um ΔH = + 58,2 kJ. A concentração de NO2(g), no equilíbrio, aumentará se:
a) A temperatura do recipiente for aumentada
b) O volume do recipiente for diminuído
c) Se a pressão do sistema for aumentada
d) Se um gás inerte for adicionado
e) A temperatura do recipiente for diminuído

Resolução: Alternativa A.
A equação química N2O4(g) ⇄ 2 NO2(g) apresenta ΔH > 0 (processo endotérmico).
A concentração de NO2(g) irá aumentar se:
✓ a temperatura do recipiente for aumentada, uma vez que o processo é endotérmico.
✓ diminuindo a pressão do sistema reacional, o equilíbrio será deslocado para a direita.
✓ aumentando a pressão parcial N2O4, o equilíbrio será deslocado para a direita.
✓ diminuindo a pressão parcial NO2, o equilíbrio será deslocado para a direita.

592
1600 FÍSICO-QUÍMICA APLICADA EXERCÍCIOS COMENTADOS - IME – ITA – OLIMPÍADA

Questão 1095 – (OLIMPÍADA DE QUÍMICA DO RIO GRANDE DO NORTE) Apesar do nitrogênio (N2) ser o gás mais
abundante na composição da atmosfera da Terra, esse elemento na sua forma diatômica é muito pouco reativo. Para
reagir com o oxigênio gasoso ele precisa de grande quantidade de energia sob a forma de altas temperaturas e
pressões ou uma via catalítica adequada. Para além da conversão bioquímica que ocorrem em organismos
especialmente adaptados à fixação do nitrogênio, na natureza a oxidação do nitrogênio ocorre apenas nas descargas
elétricas das trovoadas. Fazendo os óxidos de nitrogênio compostos em geral pouco comuns. Esta situação alterou-
se profundamente nas regiões industrializadas com a introdução dos motores a explosão. Nesses motores, as
pressões e temperaturas criadas no interior dos cilindros levam à oxidação do nitrogênio do ar injetado, formando uma
complexa mistura de óxidos de nitrogênio, em geral designados por NxOy, que é libertada para a atmosfera com os
gases de escape. São estes gases que, reagindo com os componentes da atmosfera, em particular com a água,
formam ácido nitroso (HNO2) e ácido nítrico (HNO3), ácidos fortes que contribuem poderosamente para a acidificação
da chuva. Pela queima de combustíveis fósseis a altas pressões e temperaturas na presença de nitrogênio do ar,
temos que na câmara de combustão dos motores, ocorre a reação:
N2(g) + O2(g) → 2 NO(g) Kc1 = 2,3 x 10-19
O óxido de nitrogênio formado é instável nas condições atmosféricas normais, e na presença de oxigênio do ar, produz
o dióxido de nitrogênio:
NO(g) + ½ O2(g) → NO2(g) Kc2 = 3,0 x 106
A baixas temperaturas, o dióxido de nitrogênio dimeriza e se converte em tetróxido de dinitrogênio. Ambos, em contato
com a umidade ou água chuva se transforma em ácido nítrico e nitroso, responsáveis pela chuva ácida.
2 NO2(g) → N2O4(g) Kc3 = 2,2 x 102
O valor da constante de equilíbrio para a reação de formação do tetróxido de dinitrogênio, N2(g) + 2 O2(g) → N2O4(g) é:
a) 7,5 x 10-11
b) 1,5 x 10-10
c) 3,0 x 10-4
d) 4,5 x 10-4
e) 2,2 x 10²

Resolução: Alternativa D.
Primeira equação química: N2(g) + O2(g) → 2 NO(g) Kc1 = 2,3 x 10-19
Multiplicando por dois a segunda equação química: 2 NO(g) + O2(g) → 2 NO2(g) (Kc2)² = (3,0 x 106)²
Terceira equação química: 2 NO2(g) → N2O4(g) Kc3 = 2,2 x 102
Somando todas as equações químicas, temos:
N2(g) + O2(g) → 2 NO(g) Kc1 = 2,3 x 10-19
2 NO(g) + O2(g) → 2 NO2(g) (Kc2)² = (3,0 x 106)²
2 NO2(g) → N2O4(g) Kc3 = 2,2 x 102 +

N2(g) + 2 O2(g) → N2O4(g) KTOTAL = 2,3 x 10-19 x (3,0 x 106)² x 2,2 x 102 = 4,55 x 10-4
Questão 1096 – (OLIMPÍADA DE QUÍMICA DO RIO DE JANEIRO) Para a reação SO2(g) + Cl2(g) ⇄ SO2Cl2(g) a uma
temperatura particular, Kc = 55,5. Se 1 mol de SO2(g) e 1 mol de Cl2(g) são colocados em um recipiente de 10,0 L, qual
será a concentração de SO2Cl2(g) ao se atingir o equilíbrio?
a) 0,055 mol.L-1
b) 0,034 mol.L-1
c) 0,13 mol.L-1
d) 0,84 mol.L-1
e) 0,066 mol.L-1

Resolução: Alternativa E.
Equação química balanceada de síntese do cloreto de sulfurila: SO2(g) + Cl2(g) ⇄ SO2Cl2(g)

593
1600 FÍSICO-QUÍMICA APLICADA EXERCÍCIOS COMENTADOS - IME – ITA – OLIMPÍADA

[𝑆𝑂2 𝐶𝑙2 ]
Equação da constante de equilíbrio: 𝐾𝑐 =
[𝑆𝑂2 ] 𝑥 [𝐶𝑙2 ]

Base de cálculo: 1 mol inicial de SO2(g) e 1 mol inicial de Cl2(g).


Tabela de equilíbrio químico:

SO2(g) (mol) Cl2(g) (mol) ⇄ SO2Cl2(g) (mol)


Início 1 1 0
Reage α α α
Equilíbrio 1–α 1–α α
α
[SO2 Cl2 ] ( )
Cálculo do grau de dissociação (α): K c = [SO2 ] x [Cl2 ]
= 10
1−α 2
( )
10

𝛼
(10)
= 55,5
1−𝛼 2
( 10 )

100𝛼
= 55,5
10 𝑥 (1 − 𝛼)2

Resolvendo a equação do segundo grau, 55,5𝛼² − 121𝛼 + 55,5 = 0 , suas raízes serão as seguintes: α1 =
1,52 e α2 = 0,66.
Logo, o grau de dissociação válido para este processo é o α2. Cálculo da concentração da quantidade de matéria do
cloreto de sulfurila:
0,66 α mol
[SO2 Cl2 ] = = = 0,066
10 V L
Questão 1097 – (OLIMPÍADA BRASILEIRA DE QUÍMICA) Supondo que a seguinte reação ocorra por meio de um
mecanismo de uma única etapa elementar nas duas direções: A + 2 B ⇄ AB2. Sendo k1 e k2 as constantes, a uma
determinada temperatura, para as reações direta e inversa, respectivamente, assinale a alternativa correta:
k1 [AB2 ]
a) k
= K = [A] x [B]²
2
k1 [A] x [B]²
b) k2
=K= [AB2 ]
k2 [AB2 ]²
c) =K= [A] x [B]
k1
k2 [A] x [B]²
d) =K= [AB2 ]²
k1
[AB2 ]
e) k 2 + k1 = K −1
= [A]
x [B]²

Resolução: Alternativa A.

Equação química: A + 2B ⇄ AB2.


Equação da velocidade na reação direta: v1 = k1 x [A] x [B]2
Equação da velocidade na reação inversa: v2 = k 2 x [AB2 ]
Igualando as velocidades da reação direta e da reação inversa, temos: v1 = v2
k1 x [A] x [B]2 = k 2 x [AB2 ]
k1 x [A] x [B]2
=1
k 2 x [AB2 ]

594
1600 FÍSICO-QUÍMICA APLICADA EXERCÍCIOS COMENTADOS - IME – ITA – OLIMPÍADA

k1 [AB2 ]
=
k 2 [A] x [B]2

Questão 1098 – (ITA) As opções abaixo se referem a equilíbrios químicos que foram estabelecidos dentro de cilindros
providos de êmbolos. Se o volume interno em cada cilindro for reduzido à metade, a temperatura permanecendo
constante, em qual das opções abaixo o ponto de equilíbrio será alterado?
a) H2(g) + I2 (g) → 2HI(g)
b) CaCO3(s) → CaO(s) + CO2(g)
c) PbS(s) + O2(g) → Pb(s) + SO2(g)
d) CH4(g) + 2O2(g) → CO2(g) + 2H2O(g)
e) Fe2O3(s) + 3CO(g) → 2Fe(s) + 3CO2(g)

Resolução: Alternativa B.
O "ponto de equilíbrio" será alterado apenas naquele caso em que a soma dos coeficientes estequiométricos dos
gases, em ambos os membros da equação química, forem diferentes. Isso só ocorre na alternativa B.
Diminuindo o volume do cilindro, será necessário determinar entre as alternativas mencionadas, aquela em que a
pressão do sistema será maior, ou seja, aumento de pressão do sistema reacional, deslocando o equilíbrio para a
direita. Analisando item por item.

a) H2(g) + I2 (g) → 2HI(g) = não há variação de pressão neste sistema.


b) CaCO3(s) → CaO(s) + CO2(g) = aumentando a pressão do sistema, o equilíbrio será deslocado para a esquerda.
c) PbS(s) + O2(g) → Pb(s) + SO2(g) = não há variação de pressão neste sistema.
d) CH4(g) + 2O2(g) → CO2(g) + 2H2O(g) = não há variação de pressão neste sistema.
e) Fe2O3(s) + 3CO(g) → 2Fe(s) + 3CO2(g) = não há variação de pressão neste sistema.

Questão 1099 – (OLIMPÍADA BRASILEIRA DE QUÍMICA) Para a seguinte reação NO(g) + CO(g) ⇄ ½ N2(g) + CO2(g),
com ΔH = - 374 kJ. As condições que favorecem a máxima conversão de reagentes em produtos são:
a) Baixa temperatura e alta pressão
b) Baixa temperatura e baixa pressão
c) Alta temperatura e baixa pressão
d) Alta temperatura e alta pressão
e) Apenas alta temperatura

Resolução: Alternativa A.
Pelo princípio de Le Chatelier para que o equilíbrio seja deslocado para a direita é necessário que a temperatura
diminua ou que haja o aumento da pressão reacional do sistema.
Questão 1100 - (OLIMPÍADA DE QUÍMICA DO RIO DE JANEIRO) Em um recipiente fechado de 5,00 litros foram
introduzidos 1,00 mol de gás nitrogênio e mesma quantidade de gás hidrogênio. A reação para formação de amônia,
de acordo com o processo Haber, entra em equilíbrio na temperatura de 17°C. No equilíbrio é encontrado 0,400 mol
de gás hidrogênio. Qual o valor de Kp para esse processo?
a) 1,28 x 10-2
b) 1,38 x 10-1
c) 3,91
d) 7,24
e) 78,1

Resolução: Alternativa B.
Base de cálculo: 1 mol de gás nitrogênio no estado inicial e 0,400 mol de gás hidrogênio no equilíbrio.
Equação química: N2(g) + 3H2(g) ⇄ 2 NH3(g)
Tabela de equilíbrio químico:

595
1600 FÍSICO-QUÍMICA APLICADA EXERCÍCIOS COMENTADOS - IME – ITA – OLIMPÍADA

N2(g) (mol) 3 H2(g) (mol) ⇄ 2 NH3(g) (mol)


Início 1 1 0
Reage 1α 3α 2α
Equilíbrio 1-α 1 - 3α 2α

Cálculo da incógnita o α, sabendo que no equilíbrio encontram-se 0,400 mol de H2(g):


1 − 3𝛼 = 0,400
0,600
𝛼= = 0,200 𝑚𝑜𝑙
3
Cálculo das concentrações das quantidades de matéria das espécies gasosas:
nN2 1 − α 1 − 0,200 mol
[N2 ] = = = = 0,160
V V 5 L
nH2 1 − 3α 1 − 3 x 0,200 mol
[H2 ] = = = = 0,080
V V 5 L
nNH3 2α 2 x 0,200 mol
[NH3 ] = = = = 0,080
V V 5 L
[NH3 ]² (0,080)²
Cálculo da constante de equilíbrio (Kc): K c = [N = = 78,125
2 ] x [H2 ]³ 0,160 x (0,080)³

Cálculo da constante de equilíbrio (Kp), a partir da relação matemática entre Kp e Kc: Kp = Kc x (RT)Δn
Kp = 78,125 x (0,08206 x 190)-2 = 0,138
Questão 1101 – (ENADE) A 700°C e 1 atm, a constante de equilíbrio para a reação C(s, grafite) + H2O(g) ⇄ CO(g) + H2(g)
é igual a 1,6. Nessas condições, ao atingir o equilíbrio, qual a fração molar aproximada de hidrogênio na fase gasosa?
a) 0,12
b) 0,26
c) 0,33
d) 0,44
e) 0,55

Resolução: Alternativa D.
Equação química: C(s, grafite) + H2O(g) → CO(g) + H2(g).
Base de cálculo: 1 mol de carbono na fase sólida e 1 mol de água na fase gasosa, no início da reação.
Tabela de equilíbrio químico:

C(s,grafite) (mol) H2O(g) (mol) CO(g) (mol) H2(g) (mol)


Início 1 1 0 0
Reage α α α α
Equilíbrio 1 - α 1-α α α

Cálculo do número de mol total (nTotal), levando em consideração apenas as espécies gasosas:
nTotal = nH2O + nCO + nH2
nTotal = 1 - α + α + α = 1 + α

Considerando que a reação esteja sendo processando em um volume de um litro, temos:


𝛼 𝛼
[CO] 𝑥 [H2 ] = = =𝛼
𝑉 1

596
1600 FÍSICO-QUÍMICA APLICADA EXERCÍCIOS COMENTADOS - IME – ITA – OLIMPÍADA

1−𝛼 1−𝛼
[H2 𝑂] = = = 1−𝛼
𝑉 1

Cálculo da incógnita α, a partir da utilização da constante de equilíbrio:

[CO] x [H2 ] α²
Kc = = = 1,6
[H2 O] 1−α
α²
= 1,6
1−α
α2 − 1,6α − 1,6 = 0
Desenvolvendo a equação do segundo grau, as raízes são as seguintes: α1 = - 2,30 e α2 = + 0,70.
Considerando que o termo só pode ser levado em consideração valores positivos, ou seja, α > 0, logo, α = + 0,70.

Cálculo do número de mol total: nTotal = 1 + α = 1 + 0,70 = 1,70


nH2 0,70 mol
Cálculo da fração molar de hidrogênio gasoso: X H2 = n = 1,70 mol = 0,412
Total

Questão 1102 – (OLIMPÍADA CEARENSE DO ENSINO SUPERIOR DE QUÍMICA) Uma mistura reacional que
consiste de 0,4 mol de H2 e 1,6 mol de I2 foi colocado num frasco de 3,0 L e aquecidos a 300°C. No equilíbrio, 60%
do gás hidrogênio tinha reagido. Qual é a constante de equilíbrio K para a reação nessa temperatura?
a) 1,1
b) 0,20
c) 13,0
d) 0,51
e) 11,0

Resolução: Alternativa A.
Equação química: H2(g) + I2(g) ⇄ 2 HI(g)
Base de cálculo: 0,40 mol de H2 e 1,6 mol de I2
Tabela de equilíbrio químico:
H2(g) (mol) I2(g) (mol) 2 HI(g) (mol)
Início 0,40 1,6 0
Reage 0,40 x 0,60 1,6 - (0,40 x 0,60) 2 x 0,40 x 0,60
0,16 mol 0,24 mol 0,48 mol
Equilíbrio
3,0 L 3,0 L 3,0 L

[HI]² (0,16)²
Cálculo da constante de equilíbrio (K): K c = [H = 5,33 x 10−2 x 0,453 = 1,06 ≅ 1,1
2 ] x [I2 ]

Questão 1103 – (OLIMPÍADA BRASILEIRA DE QUÍMICA) Dada a reação: 2 SO2(g) + O2(g) ⇄ 2 SO3(g). A constante
de equilíbrio desta reação pode ser expressa em Kc e Kp. Qual a relação entre Kp e Kc para esta reação?
a) Kp = Kc
b) Kp = Kc x (RT)-1
c) Kp = Kc x (RT)1/2
d) Kp = Kc x (RT)2
e) Kp = Kc x (RT)

Resolução: Alternativa B.
Equação química balanceada: 2 SO2(g) + O2(g) ⇄ 2 SO3(g)

597
1600 FÍSICO-QUÍMICA APLICADA EXERCÍCIOS COMENTADOS - IME – ITA – OLIMPÍADA

Para a resolução do problema será necessário relacionar a constante de equilíbrio em função das pressões parciais
(Kp) com a constante de equilíbrio em função das concentrações molares (Kc).

K p = K c x (RT)∆n

K p = K c x (RT)2−3
Kp
= (RT)−1
Kc
Questão 1104 – (OLIMPÍADA CEARENSE DO ENSINO SUPERIOR DE QUÍMICA) Dada as seguintes reações e
suas constantes de equilíbrio, a 25°C:
N2(g) + O2(g) ⇄ 2 NO(g) KC1 = 4,1 x 10-31
N2O(g) + ½ O2(g) ⇄ 2 NO(g) KC1 = 1,7 x 10-13
Calcule Kc, para a reação abaixo: N2(g) + ½ O2(g) ⇄ N2O(g) (T = 25°C)
a) Kc = 5,8 x 10 -18

b) Kc = 2,4 x 10-18
c) Kc = 5,9 x 1012
d) Kc = 2,4 x 10-12
e) Kc = 3,4 x 1018

Resolução: Alternativa B.
Invertendo a segunda equação química, temos:
N2(g) + O2(g) ⇄ 2 NO(g) KC1 = 4,1 x 10-31
1
2 NO(g) ⇄ N2O(g) + ½ O2(g) KC1 = +
1,7 x 10−13
1
N2(g) + ½ O2(g) = N2O(g) K c = 4,1 x 10−31 x (1,7 x 10−13 ) = 2,41 x 10−18

Questão 1105 – Analise os sistemas em equilíbrio a seguir:


Sistema I: N2(g) + O2(g) ⇄ 2 NO(g) Kc = 1,0 x 10-30, a 25°C.
Sistema II: N2(g) + 3 H2(g) ⇄ 2 NH3(g) Kc = 5,0 x 108, a 25°C.
A alternativa correta em função das constantes de equilíbrio e dos fatores que podem provocar deslocamentos dos
equilíbrios é:
a) Em I, aumentando-se a pressão externa sobre o sistema, desloca-se o equilíbrio no sentido da produção de
NO.
b) Em I, as [N2] e [O2] no equilíbrio são muito pequenas se comparadas com a [NO];
c) Em II, aumentando-se a pressão parcial do N2, desloca-se o equilíbrio no sentido da produção de NH3.
d) Em II, a adição de um catalisador metálico, desloca o equilíbrio no sentido do consumo de H2;
e) Em II, a concentração de NH3 no equilíbrio, é muito pequena se comparada às concentrações de N2 e H2.

Resolução: Alternativa C.
Item a) Falso. Aumentando a pressão externa do sistema, ou seja, a pressão total, não vai haver deslocamento do
equilíbrio. Ou seja, aumentando ou até diminuindo a pressão do sistema reacional, não vai haver deslocamento.
[NO]2
Item b) Falso. Expressão da constante de equilíbrio para o sistema I: K c = [N
2 ] x [O2 ]
[NO]2
1,10 x 10−30 =
[N2 ] x [O2 ]
[NO]2
[N2 ] x [O2 ] =
1,10 x 10−30
[N2 ] x [O2 ] = 9,09 x 10+29 𝑥 [NO]2

598
1600 FÍSICO-QUÍMICA APLICADA EXERCÍCIOS COMENTADOS - IME – ITA – OLIMPÍADA

Item c) Verdadeiro. Aumentando a pressão parcial do nitrogênio gasoso, o equilíbrio será deslocado para a direita,
favorecendo amônia.
Item d) Falso. A adição de catalisador não influencia na constante de equilíbrio, mas faz com que o equilíbrio se
estabilize mais rapidamente.
Item e) Falso. Observando a relação da constante de equilíbrio:

[NH3 ]2
Kc =
[N2 ] x [H2 ]³

8
[NH3 ]2
5,0 𝑥 10 =
[N2 ] x [H2 ]³
[NH3 ]2 = 5,0 x 108 x [N2 ] x [H2 ]³

Questão 1106 – (OLIMPÍADA DE QUÍMICA DO RIO DE JANEIRO) Considerando a reação abaixo, em equilíbrio,
num recipiente de 2,0 litros: 2 H2S(g) ⇄ 2 H2(g) + S2(g). Observa-se o equilíbrio e verificam-se que estão presentes 1,0
mol de gás sulfídrico, 0,20 mol de gás hidrogênio e 0,80 mol de enxofre gasoso. Com base nos dados acima, qual o
valor da constante de equilíbrio, Kc, da reação?
a) 1,60 x 10-2
b) 1,25 x 101
c) 3,20 x 10-2
d) 6,25 x 101
e) 8,00 x 10-2

Resolução: Alternativa A.
Cálculo das concentrações das quantidades de matéria das espécies químicas:
0,20 mol mol
[H2 ] = = 0,10
2L L
0,80 mol mol
[S2 ] = = 0,40
2L L
1,00 mol mol
[H2 S] = = 0,50
2L L
[H2 ]2 x [S2 ] (0,10)2 x 0,40
Expressão e cálculo da constante de equilíbrio: K c = [H2 S]²
= = 0,016
(0,50)²

Questão 1107 – (OLIMPÍADA MINEIRA DE QUÍMICA) Considere a equação termoquímica que representa o
equilíbrio químico em um sistema fechado: 4 HCl(g) + O2(g) ⇄ 2 H2O(g) + 2 Cl2(g); ΔH = -114 kJ. É correto afirmar que:
a) A quantidade de Cl2 aumenta com o aumento da temperatura do sistema
b) A quantidade de Cl2 aumenta com a diminuição da pressão total do sistema
c) A quantidade de Cl2 aumenta com a adição de um agente higroscópico do sistema
d) A quantidade de Cl2 aumenta com a adição de NaOH ao sistema

Resolução: Alternativa C.
Item a) Falsa. Com a diminuição da temperatura, o equilíbrio é deslocado para a direita, aumentando
consequentemente a quantidade de cloro gasoso.
Item b) Falsa. Com a diminuição da pressão total do sistema, o equilíbrio é deslocado para a esquerda, diminuindo a
quantidade de cloro gasoso.
Item c) Verdadeira. O processo de higroscopia trata-se de um agente que retira água, o que consequentemente com
a retirada de água, diminuição, irá deslocar o equilíbrio para a direita, aumentando a quantidade de cloro gasoso.
Item d) Com a adição de uma solução de hidróxido de sódio, não vai influenciar em nada no equilíbrio.

599
1600 FÍSICO-QUÍMICA APLICADA EXERCÍCIOS COMENTADOS - IME – ITA – OLIMPÍADA

Questão 1108 – (OLIMPÍADA DE QUÍMICA DO RIO GRANDE DO SUL) Quando o cloreto de amônio é aquecido em
um recipiente fechado, ele atinge o equilíbrio em amônia e gás clorídrico, conforme a equação: NH4Cl(s) ⇄ NH3(g) +
HCl(g). Determinou-se que, no equilíbrio, em um recipiente de 5,0 litros, há 2 mol de amônia, X mol de HCl e 1 mol de
NH4Cl, a 500°C. Sabendo-se que Kc = 0,16, o valor de X vale:
a) 0,08
b) 0,4
c) 1,0
d) 2,0
e) 2,5

Resolução: Alternativa D.

Equação química balanceada: NH4Cl(s) ⇄ NH3(g) + HCl(g)

Cálculo das concentrações das quantidades de matéria das espécies químicas gasosas:

2 mol mol
[NH3 ] = = 0,40
5L L
X mol
[HCl] =
5 L

Expressão da constante de equilíbrio (Kc) e cálculo do número de mol (X) do cloreto de hidrogênio: K c =
[NH3 ] x [HCl]

X
0,16 = 0,40 x
5,0

X = 2,0 mol
Questão 1109 – (ITA) Amoníaco gasoso pode ser obtido por síntese a partir de reagentes gasosos. Qual das
afirmações relativas à síntese é incorreta, considerando que é uma reação exotérmica e irreversível?
a) A equação química representativa da síntese é: NH3(g) + H2O(g) ⇄ NH4OH(g);
b) Quanto maior a pressão da mistura gasosa, maior a quantidade de amoníaco obtida a partir de determinada
massa da mistura reagente, se o equilíbrio for atingido.
c) Quanto menor a temperatura da mistura gasosa, maior a quantidade de amoníaco obtida a partir de
determinada massa da mistura reagente, se o equilíbrio for atingido.
d) Quanto maior a concentração de um reagente, maior a quantidade de amoníaco obtida a partir de
determinada massa do outro reagente, se o equilíbrio for atingido.
e) É possível converter integralmente em amoníaco uma mistura estequiométrica dos reagentes, se o produto
for eliminado do sistema à medida que ele se forma.

Resolução: Alternativa A.
Item a) A reação de síntese para a formação de amoníaco (gás amônia) é representada pela seguinte equação
química balanceada: N2(g) + 3 H2(g) ⇄ 2 NH3(g).
Item b) Aumentando a pressão do sistema do sistema reacional (pressão total), o equilíbrio é deslocado para a direita,
para o sentido de formação de amônia;
Item c) Sendo o processo de síntese da amônia como um processo exotérmica, diminuindo a temperatura, o equilíbrio
é deslocado para a direita, para maior formação de amônia;
Item d) Aumentando a concentração de nitrogênio ou até de mesmo de hidrogênio, o equilíbrio será deslocado para
a direita, para maior formação de amônia;
Item e) Característica principal do estudo da constante de equilíbrio, o que caracteriza uma reação reversível.

600
1600 FÍSICO-QUÍMICA APLICADA EXERCÍCIOS COMENTADOS - IME – ITA – OLIMPÍADA

Questão 1110 – (OLIMPÍADA DE QUÍMICA DO RIO DE JANEIRO) A reação 2 A + 3 B ⇄ C + 2 D, onde todas as


substâncias se encontram no estado gasoso, ocorre com perda de calor. Após certo tempo, o sistema entra em
equilíbrio. Indique a alternativa que apresenta três maneiras de aumentar a produção de C e D.
a) Utilizar um catalisador, aumentar a pressão e aumentar a concentração de A
b) Reduzir a concentração de B, aumentar a temperatura e reduzir a pressão
c) Utilizar um catalisador, aumentar o volume e aumentar a temperatura
d) Aumentar a concentração de C, reduzir o volume e aumentar a temperatura
e) Aumentar a concentração de A, aumentar a pressão e reduzir a temperatura

Resolução: Alternativa E.
2A + 3B ⇄ C + 2D
Aplicando o princípio de Le Chatelier, iremos apresentar algumas alternativas que faça com que o equilíbrio esteja se
deslocando para a direita, ou seja, favorecendo os produtos C e D.
✓ Aumento da concentração de A;
✓ Aumento da concentração de B;
✓ Aumento da pressão total do sistema reacional;
✓ Diminuição da temperatura, uma vez que se trata de um processo exotérmico;
Observação: a utilização de um determinado catalisador não influencia na constante de equilíbrio.

Questão 1111 – (OLIMPÍADA DE QUÍMICA DO RIO DE JANEIRO) Um método de produção de cianeto de hidrogênio
é a nitrogenação do acetileno em fase gasosa, de acordo com a equação não-balanceada: N2 + C2H2 ⇄ HCN. À
temperature de 300 K, nas concentrações de 1 mol.L-1, 2 mol.L-1 e 0,1 mol.L-1, para N2, C2H2 e HCN, respectivamente,
o sitema está em equilíbrio. Calcule o valor da constante de equilíbrio para esse processo a 300 K.
a) 0,005
b) 0,05
c) 0,2
d) 20
e) 200

Resolução: Alternativa A.
Equação química balanceada: N2(g) + C2H2(g) ⇄ 2 HCN(g)
[HCN]2 (0,1)2
Expressão e cálculo da constante de equilíbrio (Kc): K c = [N = 1,0 x 2,0 = 0,005
2 ] x [C2 H2 ]

Questão 1112 – (OLIMPÍADA DE QUÍMICA DO RIO DE JANEIRO) Dada a expressão da Lei de Equilíbrio, retirado
de dados experimentais, K = [O2]-3, assinale a opção de qual reação ela pertence:
a) 4 Al(s) + 3 O2(g) ⇄ 2 Al2O3(s)
b) 3 N2(g) + 3 O2(g) ⇄ 6 NO(g)
c) C3H6O3(l) + 3 O2(g) ⇄ 3 CO2(g) + 3 H2O(l)
d) 6 NO(g) + 3 O2(g) ⇄ 6 NO2(g)
e) 2 Al2O3(s) ⇄ 4 Al(s) + 3 O2(g)

601
1600 FÍSICO-QUÍMICA APLICADA EXERCÍCIOS COMENTADOS - IME – ITA – OLIMPÍADA

Resolução: Alternativa A.
Expressão da constante de equilíbrio (Kc) para cada alternativa apresentada:
1
a) K c = [O 3 = [O2 ]−3
2]

[NO]6
b) K c = [N 3 3
2 ] 𝑥 [O2 ]

[CO2 ]3
c) K c = [O2 ]3

[NO ]6
d) K c = [NO]6 𝑥2[O 3
2]

e) K c = [O2 ]3
Questão 1113 – (OLIMPÍADA BRASILEIRA DE QUÍMICA) A redução de magnetita por hidrogênio, em alto-forno, é
um dos principais processos de obtenção de ferro. Esta reação ocorre segundo a equação (não balanceada) abaixo:
Fe3O4(s) + H2(g) ⇄ Fe(s) + H2O(g). Se esta reação é efetivada a 200°C, sob pressão total de 1,50 atmosferas e com Kp =
5,30 x 10-6, a pressão parcial de hidrogênio é de:
a) 0,80 atm
b) 1,00 atm
c) 1,26 atm
d) 1,43 atm
e) 1,62 atm

Resolução: Alternativa D.
Equação química: Fe3O4(s) + 4 H2(g) ⇄ 3 Fe(s) + 4 H2O(g)
Base de cálculo: n (mol) inicial de Fe(s) e H2(g).
Tabela de equilíbrio químico:

Fe3O4(s) (mol) 4 H2(g) (mol) → 3 Fe(s) (mol) 4 H2O(g)

Início n n 0 0
Reage - 4nα - 4nα
Equilíbrio - n – 4nα - 4nα

Cálculo do número de mol total: nTotal = nH2 + nH2O = n – 4nα + 4nα = n


Expressão das pressões parciais:
n− 4nα
Pressão parcial do H2: PH2 = X H2 x pTotal = n
= (1 − 4α) x pTotal

4nα
Pressão parcial do H2O: PH2 𝑂 = X H2 𝑂 x pTotal = n
= (4α) x pTotal

p4H2 o
Expressão da constante de equilíbrio (Kp) em função das pressões parciais: K P =
p4H2

[(4α) x pTotal ]4
5,30 𝑥 10−6 =
[(1 − 4α) x pTotal ]4

4 4 [(4α) x p ]4
Tirando a raiz quarta na equação acima: √5,30 𝑥 10−6 = √[(1− 4α) x Total 4
p Total ]

602
1600 FÍSICO-QUÍMICA APLICADA EXERCÍCIOS COMENTADOS - IME – ITA – OLIMPÍADA


4,80 𝑥 10−2 =
(1 − 4α)
α = 1,14 x 10−2
Cálculo da pressão parcial de gás hidrogênio: PH2 = (1 – 4α) x pTotal
PH2 = [1 – 4 x (1,14 x 10-2)] x 1,50 = 1,43 atm
Questão 1114 – (ITA) Em um copo se estabelece o seguinte equilíbrio heterogêneo: Ag+(aq) + Cl-(aq) → AgCl(c). Com
relação à possibilidade de se deslocar este equilíbrio para a direita, mantendo a temperatura constante, são feitas as
seguintes sugestões:
I. Acrescentar AgCl(c);
II. Retirar uma parte do AgCl(c);
III. Acrescentar um pouco de NaCl(c);
IV. Acrescentar água;
V. Evaporar água.
Das sugestões acima irá(ão) deslocar, efetivamente, o equilíbrio no sentido desejado apenas:
a) III
b) I e IV
c) II e III
d) III e V
e) II, III e V

Resolução: Alternativa D.
I. Acrescentando AgCl(c) não vai deslocar o equilíbrio, pois se trata de um composto sólido.
II. Retirar uma parte de AgCl(c) não vai deslocar o equilíbrio, pois se trata de um composto sólido.
III. Acrescentando um pouco de NaCl(c), vai haver um íon comum que se trata do cloreto (Cl-), e com isso, o
equilíbrio será deslocado para a direta.
IV. Acrescentando água o equilíbrio será deslocado para a esquerda, onde vai haver maior produção de íons,
Ag+ e Cl-.
V. Evaporando a água, o equilíbrio será deslocado para a direita, diminuindo a fase aquosa.

Questão 1115 – (IME) Foram colocados n1 mol de N2O4 num recipiente de volume constante a uma dada temperatura
T. Ao se estabelecer o equilíbrio, segundo a equação: N2O4(g) = 2 NO2(g), 20% do N2O4 estavam dissociados. A adição
de mais n2 mol de N2O4, a mesma temperatura, provocou uma variação na pressão total de equilíbrio de 2,14 atm,
ficando o N2O4 apenas 10% dissociado Determine o valor da constante de equilíbrio para esta reação, na temperatura
T.
Resolução: Equação química: N2O4(g) ⇄ 2 NO2(g)
Base de cálculo: n (mol) inicial de N2O4(g) no estado inicial.
Tabela de equilíbrio químico:

N2O4(g) ⇄ 2 NO2(g)
Início n 0
Reage nα 2nα
Equilíbrio n - nα 2nα

Cálculo do número de mol total (nT) das espécies gasosas: nT = n – nα + 2nα = n + nα = n(1 + α)
Expressão das pressões parciais dos compostos gasosos:

603
1600 FÍSICO-QUÍMICA APLICADA EXERCÍCIOS COMENTADOS - IME – ITA – OLIMPÍADA

n−nα 1−α
Pressão Parcial do N2O4: PN2O4 = XN2O4 x pT = (n+nα) x pT = (1+α) x pT

2nα 2α
Pressão Parcial do NO2: PNO2 = XNO2 x pT = (n+nα) x pT = (1+α) x pT

4α2
P2NO2 [(1+α)2 x p2TOTAL ]
Expressão da constante de equilíbrio em função do grau de dissociação (𝛼): K p = P = (1−α) =
N2 O4 [(1+α) x PTOTAL ]
4α²
1−α²
x PTOTAL

4α²
Para a primeira situação em que o grau de dissociação seja igual a 20%, ou seja, α = 0,20: K p1 = 1−α² x PTOTAL
4 x (0,20)² 0,16
K p1 = x PTOTAL = x PTOTAL
1 − (0,20)² 0,96
Para a segunda situação, a pressão será a seguinte: ∆P = P2 − P1 em que iremos considerar que a pressã tta para
a situação 1 será P1 = PTOTAL. Logo:
∆P = P2 − PTOTAL
2,14 = P2 − PTOTAL
P2 = 2,14 + PTOTAL
4α2
Cálculo da constante de equilíbrio em função das pressões parciais, para α = 0,10: K p2 = x (2,14 + PTOTAL )
1−α2
4 x (0,10)2 0,04
K p2 = 2
x (2,14 + PTOTAL ) = x (2,14 + PTOTAL )
1 − (0,10) 0,99
Como o processo ocorre a temperatura constante, tanto para a situação 1 quanto para a situação 2, a constante de
equlíbrio será igual, entã K p1 = K p2 . Cálculo da pressão total do sistema reacional.
0,16 0,04
x PTOTAL = x (2,14 + PTOTAL )
0,96 0,99
0,16 𝑥 0,99 x PTOTAL = 0,96 x 0,04 x (2,14 + PTOTAL )
0,1584 x PTOTAL = 0,0384 x (2,14 + PTOTAL )
0,1584 𝑥 PTOTAL
= 2,14 + PTOTAL
0,0384
4,125 𝑥 PTOTAL = 2,14 + PTOTAL
3,125 𝑥 PTOTAL = 2,14
2,14
PTOTAL = = 0,685 𝑎𝑡𝑚
3,125
0,16
Cálculo da constante de equilíbrio em função das pressões parciais: K 𝑝 = 0,96 x 0,685 = 0,114

604
1600 FÍSICO-QUÍMICA APLICADA EXERCÍCIOS COMENTADOS - IME – ITA – OLIMPÍADA

Questão 1116 – (OLIMPÍADA DE QUÍMICA DO RIO GRANDE DO SUL) N2O4 e NO2, gases poluentes do ar,
encontram-se em equilíbrio, como indicado: N2O4(g) ⇄ 2 NO2(g). Em uma experiência, nas condições ambientes,
introduziu-se 1,5 mol de N2O4(g) em um reator de 2,0 litros. Estabelecido o equilíbrio, a concentração de NO2(g) foi de
0,06 mol.L-1. Qual o valor da constante, em termos da concentração, desse equilíbrio?
a) 2,4 x 10-3
b) 4,8 x 10-3
c) 5,2 x 10-3
d) 8,3 x 10-3
e) 5,0 x 10-3

Resolução: Alternativa E.

Equação química: N2O4(g) ⇄ 2 NO2(g)

Base de cálculo: 1,5 mol de tetróxido de dinitrogênio no início da reação.

Tabela de equilíbrio químico:

N2O4(g) (mol) ⇄ 2 NO2(g) (mol)


1,50 mol mol
Início = 0,75 0
2,0 L L
Reage 0,75α 2 x 0,75 x α = 1,50 α
Equilíbrio 0,75 − 0,75α = 0,75 x (1 − α) 0,06 mol.L-1

Cálculo do grau de dissociação (α), partindo do equilíbrio do NO2(g): 0 + 1,5α = 0,06


1,5α = 0,06
α = 0,04 (4%)

Cálculo da concentração da quantidade de matéria das espécies químicas:

[N2O4] = 0,75.(1 – α) = 0,75.(1 - 0,04) = 0,72 mol.L-1


[NO2] = 0,06 mol x L-1

[NO2 ]2 (0,06)2
Expressão e cálculo da constante de equilíbrio (Kc): K C = [N = = 0,005
2 O4 ] 0,72

Questão 1117 – (ITA) Considere as seguintes reações químicas e respectivas constantes de equilíbrio:
N2(g) + O2(g) ⇄ 2 NO(g) K1
2 NO(g) + O2(g) ⇄ 2 NO2(g) K2
NO2(g) ⇄ ½ N2(g) + O2(g) K3
Então, K3 é igual a:
1
a) K xK
1 2
1
b) 2 x K1 x K2
1
c) 4 x K1 x K2
1 1/2
d) [(K x K )]
1 2
1 2
e) [(K x K )]
1 2

Resolução: Alternativa D.
1
Invertendo a primeira equação química e dividindo por dois, temos: NO(g) ⇄ ½ N2(g) + ½ O2(g) 1/2
K1

605
1600 FÍSICO-QUÍMICA APLICADA EXERCÍCIOS COMENTADOS - IME – ITA – OLIMPÍADA

1
Invertendo a primeira equação química e dividindo por dois, temos: NO2(g) ⇄ NO(g) + ½ O2(g) 1/2
K2

Somando as reações químicas, temos:


1
NO(g) ⇄ ½ N2(g) + ½ O2(g) 1/2
K1
1
NO2(g) ⇄ NO(g) + ½ O2(g) 1/2 +
K2

1 1 1 1 1/2
NO2(g) ⇄ N2(g) + ½ O2(g) K= 1/2 x 1/2 = [K x K ]
K1 K2 1 2

Questão 1118 – (ITA) A respeito da reação reversível N2(g) + 3 H2(g) ⇄ 2 NH3(g) e levando em conta os princípios que
regem a síntese de amoníaco, são feitas as seguintes afirmações:
I. O princípio de Le Chatelier prevê que um aumento na pressão total da mistura gasosa deve deslocar o
equilíbrio acima para o lado direito;
[NH3 ]
II. É constante, para cada valor de temperatura, a relação K = [N ]1/2 .[H ]3/2
, onde [ ] representa a
2 2
concentração de equilíbrio da espécie considerada
III. No processo industrial de preparação do amoníaco, empregam-se catalisadores cuja função é reduzir o
tempo para o estabelecimento do equilíbrio.
IV. Os gases que entram em contato com o catalisador não devem conter certas impurezas como H 2S que
poderiam desativar o catalisador.
V. A síntese sínteses industrial do amoníaco constitui um processo contínuo em que a mistura dos gases
reagentes, em proporção estequiométrica, entra no reator, que já contém o catalisador, e sai dele parcialmente
convertida em amoníaco.
Das afirmações feitas são verdadeiras:
a) Apenas I, II e III
b) Apenas I, III e V
c) Apenas II e IV
d) Apenas IV e V
e) Todas

Resolução: Alternativa E.
I. Verdadeiro. Pelo princípio de Le Chatelier, aumentando a pressão do sistema, o equilíbrio será
deslocado para a direita, favorecendo maior formação de gás amônia;
II. Verdadeiro. Pelo princípio de Le Chatelier, o valor da constante de equilíbrio é constante para cada
temperatura. Isso pode ser observado pela equação de Van´t hoff.
III. Verdadeiro. Para a obtenção de gás amônia é utilizado como catalisador ferro (Fe) com a presença de
pequenas quantidades de óxidos metálicos para que o tempo consumido pelo sistema possa atingir o
equilíbrio de forma mais rápido;
IV. Verdadeiro. O ferro, que é utilizado, é atacado pelo sulfeto de hidrogênio e, portanto, a mistura reagente
não contém H2S.
V. A entrada de hidrogênio e nitrogênio gasosos no reator é realizada em proporções estequiométricas, o
que evita desperdício de reagente e converte amoníaco na presença do catalisador.

Questão 1119 – (OLIMPÍADA DE QUÍMICA DO RIO DE JANEIRO) A equação a seguir representa um processo de
obtenção do antranilato de metila, largamente utilizado como flavorizante de uva em balas e chicletes.

Quando realizado em condições adequadas, o processo atinge o equilíbrio após um determinado período de tempo.
Com o objetivo de aumentar o rendimento na produção, foram propostas as seguintes ações:

606
1600 FÍSICO-QUÍMICA APLICADA EXERCÍCIOS COMENTADOS - IME – ITA – OLIMPÍADA

I) aumento da temperatura
II) aumento da pressão
III) adição de água
IV) retirada de água
A opção mais adequada para este objetivo seria a conjunção das ações:
a) I e II
b) I e IV
c) II e III
d) III e IV
e) I, II e IV

Resolução: Alternativa B.
Para que haja um aumento do rendimento da produção, o equilíbrio deverá ser deslocado para a direita. Para isso,
para que o equilíbrio deja deslocado para a direita, será necessário aumentar a temperatura do sistema ou retirando
água líquida e com isso o equilíbrio será deslocado para a direita.
Questão 1120 - (ITA) Considere Y a quantidade (em mol) de iodo dissolvido em 100 mL de água, X um solvente
praticamente imiscível em água e K (120) a constante de partição do iodo entre o solvente e a água a 25ºC. Assinale
a alternativa correta para o volume do solvente X necessário para extrarir 90% do iodo contido inicialmente em 100
mL de água.
7,5 mL
9,0 mL
12 mL
100 mL
120 mL

Resolução: Alternativa A.

Para o processo de partição do iodo (I2) entre a água (H2O) e um solvente X, será necessário utilizar a seguinte
NIodo(x) x Vágua
expressão matemática: K =
NIodo(aq) x VX

0,90y x 100
120 =
0,10y x VX

0,90y x 100
120 =
0,10y x VX

VX = 7,50 mL

Questão 1121 - (U. S. NATIONAL CHEMISTRY OLYMPIAD)


A) Calculate the [H+] in a 0,25 mol x L-1 solution of methylamine, CH3NH2 (Kb = 4,4 × 10-4).
(A) 1,1 × 10-4 (B) 1,0 × 10-2 (C) 9,1 × 10-11 (D) 9,5 × 10-13
B) CO(g) + Cl2(g) ⇄ COCl(g) + Cl(g) Keq = 1,5 × 10-39. If the rate constant, k, for the forward reaction above is 1,4 x 10-
28 what is k for the backward reaction?

(A) 2,1 × 10-67 (B) 1,0 × 10-11 (C) 9,3 × 1010 (D) 7,1 × 1027
Resolução:
A) Alternativa E.
No equilíbrio químico a constante de equilíbrio em relação a base é dado por: K b = M. α²

607
1600 FÍSICO-QUÍMICA APLICADA EXERCÍCIOS COMENTADOS - IME – ITA – OLIMPÍADA

Cálculo do grau de dissociação: 4,40 𝑥 10−4 = 0,25. α²


4,40 𝑥 10−4
α2 =
0,25
α2 = 17,6 𝑥 10−4

α = √17,6 𝑥 10−4 = 1,19 𝑥 10−2

Cálculo da concentração da quantidade de matéria do hidróxido: [OH − ] = M. α = 0,25 x 1,19 x 10−2


𝑚𝑜𝑙
[OH − ] = M. α = 0,1045
𝐿
Cálculo da concentração da quantidade de matéria de H+: [H + ] 𝑥 [OH − ] = 10−14
10−14 𝑚𝑜𝑙
[H + ] = = 9,55 x 10−13
0,1045 𝐿
B) Alternativa C.
Equação química balanceada: CO(g) + Cl2(g) ⇄ COCl(g) + Cl(g)
k [COCl] x [Cl]
Expressão da constante de equilíbrio: K eq = k direto = [CO] x [Cl2 ]
inverso

k direto
K eq =
k inverso
1,40 x 10−28
1,50 x 10−39 =
k inverso
1,40 x 10−28
k inverso = = 9,33 x 10+10
1,50 x 10−39
Questão 1122 – (OLIMPÍADA BRASILEIRA DE QUÍMICA) A constante de equilíbrio em termos das concentrações
(KC) de NO2 e N2O4 a 298 K é 1,70 mol–1L, enquanto a constante de equilíbrio em termos de pressões parciais (Kp) é
6,7 × 10–5 para unidade em Pascal (Pa) ou 6,8 em atmosfera (atm). Escreva as expressões para as constantes de
equilíbrio KC e Kp para esta reação e, considerando a equação de Clapeyron, deduza a relação entre KC e Kp.

Resolução: Equação química: 2 NO2(g) ⇄ N2 O4(g)

[N O4 ]
Expressão da constante de equilíbrio em função da concentração da quantidade de matéria: K c = [NO
2
2
2]

PN2O4
Expressão da constante de equilíbrio em função das pressões parciais: K p = P2NO
2

Sabendo que a razão entre o número de mol pelo volume é a concentração da quantidade de matéria, temos:

PN2 O4 = [N2 O4 ] x R x T (Equação A)

PNO2 = [NO2 ] x R x T (Equação B)

Substituindo as equações A e B na constante de equilíbrio em função das pressões parciais, temos:

PN2 O4 [N2 O4 ] x R x T [N2 O4 ] R x T [N2 O4 ]


Kp = 2 = 2
= 2
x 2
= x (R x T)1−2 = K c x (R x T)−1
PNO2 {[NO2 ] x R x T} [NO2 ] {R x T} ⏟ 2 ]2
[NO
Kc

608
1600 FÍSICO-QUÍMICA APLICADA EXERCÍCIOS COMENTADOS - IME – ITA – OLIMPÍADA

Questão 1123 – (OLIMPÍADA BRASILEIRA DE QUÍMICA) O grau de dissociação do cloro a 1600 K e 1,01325 x 105
Pa é 0,071. Quais os valores de Kp e Kc para a reação Cl2(g) ⇄ 2 Cl(g), supondo comportamento ideal.
Resolução: Equação química: Cl2(g) ⇄ 2 Cl(g)

Base de cálculo: n (mol) inicial de cloro no estado gasoso.


Tabela de equilíbrio químico:

Cl2(g) ⇄ 2Cl(g)
Início n 0
Reage nα 2nα
Equilíbrio n - nα 2nα

Cálculo do número de mol total (nT): nT = n – nα + 2nα = n + nα = n(1 + α)

Expressão das pressões parciais dos compostos gasosos:


n−nα 1−α
Pressão Parcial do Cl2: PCl2 = XCl2 x pT = (n+nα) x pT = (1+ α) x pT

2nα 2α
Pressão Parcial do Cl: PCl = XCl x pT = ( )x pT = ( ) x pT
n+nα 1+ α

Cálculo das pressões parciais:


1−α 1 − 0,071 0,929
PCl2 = ( ) x pT = ( ) x 1 atm = ( ) x 1 atm = 0,867 atm
1+ α 1 + 0,071 1,071
2α 2 x 0,071 0,142
PCl = ( )x pT = ( ) x 1 atm = x 1 atm = 0,132 atm
1+α 1+ 0,071 1,071

P2Cl (0,132)2
Cálculo da constante de equilíbrio em função das pressões parciais (Kp): K p = = = 2,0 x 10−2
PCl2 0,867

Cálculo da constante de equilíbrio em função da concentração da quantidade de matéria (Kc): K p =


K c x (R x T)(2−1)
K p = K c x (R x T)1

2,0 x 10−2 = K c x 0,08206 x 1600


2,0 x 10−2
Kc = = 1,52 x 10−4
131,30

609
1600 FÍSICO-QUÍMICA APLICADA EXERCÍCIOS COMENTADOS - IME – ITA – OLIMPÍADA

Questão 1124 – (IME) Dois experimentos foram realizados a volume constante e à temperatura T. No primeiro,
destinado a estudar a formação do gás fosgênio, as pressões parciais encontradas no equilíbrio foram 0,130 atm para
o cloro, 0,120 atm para o monóxido de carbono e 0,312 atm para o fosgênio. No segundo, estudou-se a dissociação
de n mols de fosgênio de acordo com a reação: COCl2(g) ⇄ CO(g) + Cl2(g), sendo a pressão total P, no equilíbrio a 1
atm. Calcule o grau de dissociação α do fosgênio após o equilíbrio ser alcançado.

Resolução: Análise do primeiro experimento: CO(g) + Cl2(g) ⇄ COCl2(g) . Primeira tabela de equilíbrio químico:
Pela informação do problema, temos no primeiro experimento os compostos gasosos já no equilíbrio. Com isso é
possível calcular a constante de equilíbrio (Kp).

CO(g) Cl2(g) ⇄ COCl2(g)


Início - - -
Reage - - -
Equilíbrio 0,120 atm 0,130 atm 0,312 atm

PCOCl2 0,312 0,312


Cálculo da constante de equilíbrio em função das pressões parciais (Kp): K p = P = 0,120 x 0,130 = 0,0156 =
CO x PCl2
20
Análise do segundo experimento:

Base de cálculo: n mol de COCl2 no início da reação. Segunda tabela de equilíbrio químico:

COCl2(g) ⇄ CO(g) + Cl2(g)


Início n 0 0
Reage nα nα nα
Equilíbrio n - nα nα nα

Cálculo do número de mol total: nT = n - nα + nα + nα = n + nα = n(1 + α)


Expressão das pressões parciais:
n(1−α) (1−α)
Pressão Parcial do COCl2: PCOCl2 = X COCl2 x pT = n(1+α) x pT = (1+α) x pT

nα α
Pressão Parcial do CO: PCO = X CO x pT = x pT = x pT
n(1+α) (1+α)

nα α
Pressão Parcial do Cl2: PCl2 = X Cl2 x pT = n(1+α) x pT = (1+α) x pT

α α
PCO x PCl2 ( x pT ) x ( x pT ) α² x pT 1
Cálculo do grau de dissociação (α) do fosgênio: K p = PCOCl2
= 1+α
(1−α)
1+α
= 1−α²
= 20
(1+α)
x pT

α² x pT 1
Sabendo que a pressão total do sistema é igual a 1 atm, temos: 1−α²
= 20

α² x 1 1
=
1 − α² 20
1
α² =
21
1 √21 √21 4,58
α= x = = = 0,218 (21,8%)
√21 √21 21 21

610
1600 FÍSICO-QUÍMICA APLICADA EXERCÍCIOS COMENTADOS - IME – ITA – OLIMPÍADA

Questão 1125 – (GRILLO) Considere a reação reversível que representa o equilíbrio do carbonato de cálcio, segundo
a equação química representada a seguir: Sn(s) + H2(g) + calor ⇄ H2O(vapor) + ½ Sn(fundido). Analise a quantidade de
estanho na fase sólida quando acontece os seguintes casos:
a) For adicionado estanho fundido, Sn(fundido).
b) Adição de um catalisador específico para este processo.
c) Aumentando a quantidade de vapor d´água.
d) Aumentando a temperatura do sistema.

Resolução:
Item a) Como a atividade termodinâmica de um sólido por convenção é unitário (a = 1), a adição de estanho fundido
não influencia no equilíbrio químico.
Item b) A adição de um catalisador em qualquer processo não influencia no equilíbrio químico. A função do catalisador
é aumentar a velocidade da reação.
Item c) Aumentando a concentração da quantidade de matéria de vapor de água, o equilíbrio será deslocado para a
esquerda, aumentando a concentração de estanho sólido.
Item d) Aumentando a temperatura do sistema sendo o processo endotérmico, o equilíbrio é deslocado para a direita,
aumentando tanto a concentração de vapor de água quanto a de estanho fundido e com isso diminui a quantidade de
estanho sólido.
Questão 1126 – (GRILLO) Considere a reação reversível para a seguinte equação química apresentada a seguir:
NO2(g) + ½ O2(g) ⇄ NO3(g) + calor. Analise através do princípio de Le Chatelier, os seguintes itens apresentados a
seguir:
a) Diminuir a pressão parcial do dióxido de nitrogênio.
b) Aumentar a temperatura do sistema.
c) Aumentar a pressão total do sistema.
d) Aumentar a pressão parcial do trióxido de nitrogênio.

Resolução:
Item a) Diminuindo a pressão parcial do dióxido de nitrogênio, o equilíbrio é deslocado para a esquerda.
Item b) Como a reação é exotérmica, NO2(g) + ½ O2(g) – calor ⇄ NO3(g), aumentando a temperatura do sistema
reacional, o equilíbrio é deslocado para a esquerda favorecendo dióxido de nitrogênio (NO2) e o oxigênio molecular
(O2).
Item c) Aumentando a pressão total do sistema, o equilíbrio é deslocado para a direita da reação direta, favorecendo
a formação de trióxido de nitrogênio (NO3).
Item d) Aumentando a pressão parcial do trióxido de nitrogênio, o equilíbrio é deslocado para a esquerda.

Questão 1127 – (GRILLO) Considere a reação reversível que representa o equilíbrio do carbonato de cálcio, segundo
a equação química representada a seguir: CaCO3(s) + calor ⇄ CaO(s) + CO2(g). Analise a quantidade de carbonato de
cálcio caso:
a) For adicionado monóxido de cálcio.
b) Adição de um catalisador específico para este processo.
c) Aumentando a quantidade de dióxido de carbono.
d) Aumentando o volume do recipiente.
e) Aumentando a temperatura do sistema.

Resolução:
Item a) Como a atividade termodinâmica de um sólido por convenção é unitário (a = 1), a adição ou até mesmo a
redução de monóxido de cálcio não influencia no equilíbrio químico.
Item b) A adição de um catalisador em qualquer processo não influencia no equilíbrio químico.
Item c) Aumentando a concentração da quantidade de matéria de dióxido de carbono, o equilíbrio será deslocado para
a esquerda.
Item d) Aumentando o volume do recipiente, consequentemente a pressão do sistema irá diminuir. Logo, o equilíbrio
será deslocado para a direita, aumentando a pressão parcial de dióxido de carbono.

611
1600 FÍSICO-QUÍMICA APLICADA EXERCÍCIOS COMENTADOS - IME – ITA – OLIMPÍADA

Item e) Aumentando a temperatura do sistema sendo a reação química endotérmica, o equilíbrio é deslocado para a
direita, aumentando tanto a concentração de monóxido de cálcio quanto a pressão parcial de dióxido de carbono.

Questão 1128 – (ITA) Explicar sucintamente de que maneira se estabelece o equilíbrio nas reações químicas.
Resposta: O equilíbrio se estabelece quando a velocidade da reação direta for igual a velocidade da reação inversa.
Considere a equação química representa da seguinte forma: aA(g) + bB(g) ⇌ cC(g) + dD(g) .

Aplicando a equação da velocidade direta (1): V1 = k1 x [A]a x [B]b


Aplicando a equação da velocidade inversa (1): V2 = k 2 x [C]c x [D]d

O estabelecimento do equilíbrio para as reações químicas ocorre quando a velocidade da reação direta se igualar
com a velocidade da reação, V1 = V2 .

k1 x [A]a x [B]b = k 2 x [C]c x [D]d

k1 [C]c x [D]d
= k2 x a
k
⏟2 [A] x [B]b
Keq

k
A razão entre as constantes cinéticas (k1 ) é definida como sendo a constante de equilíbrio (K eq ).
⏟2

Questão 1129 – A um recipiente fechado contendo CO2 a 0,40 atm e 1000 K, foi adicionado grafite. Calcular o valor
da constante de equilíbrio, para a pressão total no sistema de 0,75 atm, sabendo que ocorreu formação de monóxido
de carbono.
Resolução: Equação química: CO2(g) + C(s) ⇄ 2 CO(g)
Base de cálculo: 0,40 atm de pressão inicial de CO2(g).
Tabela de equilíbrio químico:
CO2(g) (atm) C(s) ⇄ 2 CO(g) (atm)
Início 0,40 - 0
Reage α - 2α
Equilíbrio 0,40 – α - 2α
Cálculo do valor do grau de dissocação (α), a partir da pressão total: PT = PCO2 + PCO
0,75 = 0,40 - α + 2α
α = 0,35
Cálculo das pressões parciais:
Pressão parcial do CO2: PCO2 = 0,40 - α = 0,40 – 0,35 = 0,05 atm

Pressão parcial do CO: PCO = 2α = 2 x (0,35) = 0,70 atm

P2CO (0,70)²
Expressão e cálculo da constante de equilíbrio (Kp): K p = = = 9,8
P2CO 0,05
2

Questão 1130 - Aqueceram-se dois mols de PCl5, num recipiente fechado, com capacidade de dois litros. Atingido o
equilíbrio, o PCl5 estava 40% dissociado em PCl3 e Cl2. Calcular a constante de equilíbrio.
Resolução: Equação química balanceada de decomposição do pentacloreto de fósforo: PCl5(g) ⇄ PCl3(g) + Cl2(g)
Base de cálculo: 2 mol (inicial) de PCl5(g)

612
1600 FÍSICO-QUÍMICA APLICADA EXERCÍCIOS COMENTADOS - IME – ITA – OLIMPÍADA

Tabela de equilíbrio químico:

PCl5(g) (mol) ⇄ PCl3(g) (mol) Cl2(g) (mol)


Início 2 0 0
Reage 2α 2α 2α
Equilíbrio 2 - 2α 2α 2α

2−(2 x 0,40) mol


Concentração da quantidade de matéria do PCl5: [PCl5 ] = = 0,60
2 L

2α mol
Concentração da quantidade de matéria do PCl3: [PCl3 ] = = α = 0,40
2 L

2α mol
Concentração da quantidade de matéria do Cl2: [Cl2 ] = 2
= α = 0,40 L

Expressão e cálculo da constante de equilíbrio em função da concentração da quantidade de matéria (Kc):

[PCl3 ] x [Cl2 ] α2 (0,40)²


KC = = = = 0,27
[PCl5 ] 1 − α 1 − 0,40

Questão 1131 – A um recipiente fechado, mantido a temperatura constante, foi introduzido CO e água em quantidades
tais que suas pressões parciais eram iguais e valiam 0,856 atm cada uma. Após certo tempo, há formação de dióxido
de carbono e hidrogênio gasoso, estabelecendo o equilíbrio com pressão parcial de monóxido de carbono igual a
0,580 atm. Determine a constante de equilíbrio em função das pressões parciais.
Resolução: Equação química: CO(g) + H2O(g) ⇄ CO2(g) + H2(g).
Base de cálculo: PCO = 0,856 atm, PH2O = 0,856 atm no estado inicial e PCO = 0,580 atm no estado de equilíbrio.
Tabela de equilíbrio químico:
CO(g) (atm) H2(g) (atm) ⇄ CO2(g) (atm) H2(g) (atm)
Início 0,856 0,856 0 0
Reage 0,276 0,276 0,276 0,276
Equilíbrio 0,856 – 0,276 = 0,580 0,856 – 0,276 = 0,580 0,276 0,276

PCO2 x PH2 (0,276)²


Cálculo da constante de equilíbrio em função das pressões parciais (Kp): K P = P = (0,580)²= 0,226
CO 𝑥 PH2 O

Questão 1132 – Considere que um mol de H2 e um mol de I2 eestejam confinados em um reator de um litro a 457°C.
Diante desta informação, determine a concentração de cada substância no equilíbrio, sabendo que a sua constante
de equilíbrio é igual a 49.

Resolução: Equação química: H2(g) + I2(g) ⇄ 2 HI(g)


Base de cálculo: 1 mol de H2 e I2 no estado inicial e volume de 1 Litro.
Tabela de equilíbrio químico:

H2(g) (mol) I2(g) (mol) ⇄ 2 HI(g) (mol)


Início 1 1 0
Reage α α 2α
Equilíbrio 1 - α 1-α 2α

Determinação das expressões das concentrações das quantidades de matéria (mol.L-1):

613
1600 FÍSICO-QUÍMICA APLICADA EXERCÍCIOS COMENTADOS - IME – ITA – OLIMPÍADA

1− 𝛼
[H2] =
𝑉

1− 𝛼
[I2] = 𝑉

2𝛼
[HI] = 𝑉

[HI]2 2𝛼 2
Cálculo do α, a partir da constante de equilíbrio: K c = [H = (1− 𝛼)2 = 49
2 ] x [I2 ]

2α2
Tirando a raiz quadrada da equação anterior, temos: √(1− α)2 = √49


= 7
(1 − α)
α = 7/9
Cálculo da concentração da quantidade de matéria (mol.L-1):
7
1− ( )
[H2] = 1
9
= 0,022 mol. L−1

7
1− ( )
[I2] = 9
= 0,022 mol. L−1
1

7
2x( ) 14
[HI] = 9
= = 1,56 mol. L−1
1 9

Questão 1133 – Na reação de esterificação, um mol de ácido acético reage com um mol de álcool etílico, a
temperatura igual a 25°C, e depois de um determinado tempo t, o equilíbrio é estabelecido quando estão presentes
2/3 mol de éster e 2/3 mol de água, ambos no equilíbrio. A partir destes dados, determine a constante de equilíbrio,
Kc.

Resolução: Base de cálculo: 1 mol de ácido acético e 1 mol de álcool etílico.


Tabela de equilíbrio químico:

CH3COOH(aq) C2H5OH(aq) ⇄ CH3COOC2H5(aq) H2O(l)


Início 1 1 0 0
Reage α α α α
Equilíbrio 1 - α 1-α α α

[éster] x [água]
Expressão da constante de equilíbrio (Kc): K c = [ácido] x [álcool]

No equilíbrio estão presentes 2/3 mol de éster e 2/3 mol de água.


Logo, a tabela de equilíbrio químico ficará da seguinte maneira:

CH3COOH(aq) C2H5OH(aq) ⇄ CH3COOC2H5(aq) H2O(l)


Início 1 1 0 0
Reage 2/3 2/3 2/3 2/3
Equilíbrio 1 – (2/3) = 1/3 1 – (2/3) = 1/3 2/3 2/3

Concentração da quantidade de matéria:


1
[CH3COOH] = 3V

614
1600 FÍSICO-QUÍMICA APLICADA EXERCÍCIOS COMENTADOS - IME – ITA – OLIMPÍADA

1
[C2H5OH] =
3V
2
[CH3COOC2H5] = 3V
2
[H2O] = 3V

[éster] x [água]
Cálculo do valor da constante α, a partir da constante de equilíbrio: K c = [ácido]
x [álcool]

2 2
(3V) x (3V)
Kc =
1 1
(3V) x (3V)

4
K c = 9V²
1
9V²
4
Kc = =4
1
Questão 1134 – Em um recipiente de 1 litro, foram colocados 4 mols de H2 e 4 mols de Cl2, ambos gasosos. Em
seguida, o recipiente foi aquecido a uma temperatura T, e logo em seguida se estabeleceu o seguinte equilíbrio
químico:
H2(g) + Cl2(g) ⇄ 2 HCl(g), com uma constante de equilíbrio de 0,25.
A partir destas informações, determine:
a) Determine as concentrações de todas as espécies químicas no equilíbrio;
b) Calcule o valor de Kc da equação inversa;
c) Calcule o valor de Kp da equação direta;
d) Mantendo a temperatura constante, descreva o que se observa no valor de Kc, com o aumento da
concentração de Cl2(g).

Resolução: Equação química: H2(g) + Cl2(g) ⇄ 2 HCl(g)


Base de cálculo: 4 mol para cada gás nos reagentes (H2 e Cl2).
Tabela de equilíbrio químico:
H2(g) (mol) Cl2(g) (mol) ⇄ 2 HCl(g) (mol)
Início 4 4 0
Reage α α 2α
Equilíbrio 4 – α 4–α 2α

Determinação das concentrações das quantidades de matéria (mol.L-1):


4− 𝛼
[H2] =
𝑉

4− 𝛼
[Cl2] = 𝑉

2𝛼
[HCl] =
𝑉

[HCl]²
Cálculo do grau de dissociação (α): K c = [H = 0,25
2 ] x [Cl2 ]

615
1600 FÍSICO-QUÍMICA APLICADA EXERCÍCIOS COMENTADOS - IME – ITA – OLIMPÍADA

2α 2
[V]
= 0,25
4−α 2
[ V ]

2𝛼 2
[ ]
Tirando a raiz quadrada da equação acima, temos: √ 𝑉
4−𝛼 2
= √0,25
[ ]
𝑉

2𝛼
= 0,5
(4 − 𝛼)
α = 0,80 (80%)
a) Cálculo das expressões das concentrações das quantidades de matéria (mol.L-1):
(4−0,8)
[H2] = 1
= 3,20 mol. L−1

(4−0,8)
[Cl2] = 1
= 3,20 mol. L−1

2 x (0,8)
[HCl] = 1
= 1,60 mol. L−1

b) Cálculo da constante de equilíbrio inversa (Kc’) para a reação direta: H2(g) + Cl2(g) ⇄ 2 HCl(g)

[HCl]2
Kc = = 0,25
[H2 ] x [Cl2 ]

Para a reação inversa: 2 HCl(g) ⇄ H2(g) + Cl2(g)


[H2 ] x [Cl2 ] 1
K ′c = = =4
[HCl]2 0,25
c) Primeira maneira de resolução:

Cálculo do número de mol total: nT = nH2 + nCl2 + nHCl = 4 – α + 4 + α + 2α = 8 mol


Cálculo das pressões parciais:
(4−α) (4−0,8) 3,2
Pressão Parcial do PH2: PH2 = 8
x pT = 8
x pT = 8
x pT = 0,4 x pT

(4−α) (4−0,8) 3,2


Pressão Parcial do PCl2: PCl2 = 8
x pT = 8
x pT = 8
x pT = 0,4 x pT

(2α) (2 x 0,8) 1,6


Pressão Parcial do PHCl: PHCl = 8
x pT = 8
x pT = 8
x pT = 0,2 x pT

P2HCl (0,2 x pT )2
Cálculo da constante de equilíbrio em função das pressões parciais: K p = P = (0,4 x p =
H2 x PCl2 T ) x (0,4 x pT )
0,04
0,16
= 0,25

Segunda maneira de resolução:

Sabendo que a relação entre Kp e Kc é dado pela seguinte equação química, temos: K p = K c x (RT)∆n

Cálculo da variação do número de mol: Δn = nprodutos – nreagentes = 2 – (1 + 1) = 0


Kp
Kc
= (R. T)0 = 1.

616
1600 FÍSICO-QUÍMICA APLICADA EXERCÍCIOS COMENTADOS - IME – ITA – OLIMPÍADA

Então, Kc = Kp = 0,25
d) Como a temperatura é constante, a constante de equilíbrio químico (Kc) permanece constante.
Questão 1135 – Num recipiente de 10 litros, estão confinados e em equilíbrio a 1000 K e pressão do sistema reacional
igual a 224 atm, 3,74 litros de N2, 5,89 litros de H2 e 0,37 litros de NH3. Calcule o valor da constante de equilíbrio (Kc)
para a reação de síntese de amônia, nas condições experimentais.
Resolução: Equação química: N2(g) + 3 H2(g) ⇄ 2 NH3(g)
RT 0,08206 x 1000
Cálculo do volume molar (L.mol-1) nas condições experimentais: Vmolar = p
= 224
= 0,37 L. mol−1 .

Cálculo do número de mol de cada espécie gasosa:


3,74 L
nN2 = 0,37 L.mol−1 = 10,11 mol

5,89 L
nH2 = = 15,92 mol
0,37 L. mol−1
0,37 L
nNH3 = = 1,00 mol
0,37 L. mol−1
Cálculo da concentração da quantidade de matéria para cada espécie gasosa:
nN2 10,11 mol
[N2 ] = = = 1,01 mol. L−1
Vsolução 10 L
nH2 15,92 mol
[H2 ] = = = 1,59 mol. L−1
Vsolução 10 L
nNH3 1,00 mol
[NH3 ] = = = 0,10 mol. L−1
Vsolução 10 L
[NH3 ]² (0,10)²
Cálculo da constante de equilíbrio (Kc): K c = [N = 1,01 x 1,59 = 6,23 x 10−2
2 ] x [H2 ]

Questão 1136 – Em um processo de esterificação a partir de um mol de ácido acético reagindo com um mol de álcool
etílico, a 25°C, o equilíbrio é estabelecido com a constante de equilíbrio (Kc) igual a 4,0. A partir desta informação,
determine o número de mol da cada espécie química no equilíbrio químico.
Resolução: Base de cálculo: 1 mol de ácido acético, 1 mol de álcool etílico e 1,0 litro de solução (volume total)
Tabela de equilíbrio químico (mol):
CH3COOH(aq) C2H5OH(aq) ⇄ CH3COOC2H5(aq) H2O(l)
Início 1 1 0 0
Reage α α α α
Equilíbrio 1 - α 1-α α α
[éster] x [água]
Expressão da constante de equilíbrio (Kc): K c = [ácido] x [álcool]

Determinação das concentrações das quantidades de matéria (mol.L-1):


1− α
[CH3COOH(aq)] = V
1− α
[C2H5OH(aq)] = V
α
[CH3COOC2H5(aq)] = V
α
[H2O(l)] = V

617
1600 FÍSICO-QUÍMICA APLICADA EXERCÍCIOS COMENTADOS - IME – ITA – OLIMPÍADA

α α α 2
x ( )
Cálculo do valor de α, a partir da constante de equilíbrio: K c = V V
1− α 1− α = V
1−α 2
x ( )
V V V

α2
Kc = =4
(1 − α)2

(α)2
Tirando a raiz quadrada, temos: √(1−α)2 = √4

α
=2
1−α
2
α=
3
Para α = 2/3, o número de mol de cada participante no equilíbrio ficará da seguinte maneira:

CH3COOH(aq) (mol) C2H5OH(aq) (mol) ⇄ CH3COOC2H5(aq) mol) H2O(l) (mol)


Início 1 1 0 0
Reage α = 2/3 α = 2/3 α = 2/3 α = 2/3
Equilíbrio 1 – 2/3 = 1/3 1 – 1/3 = 1/3 2/3 2/3

Questão 1137 – Em um processo pirometalúrgico, foram aquecidos quatro mols de XCl3, num recipiente fechado,
com capacidade de um litro. Atingido o equilíbrio, o PCl5 estava 90% dissociado em XCl e Cl2. Calcular a constante
de equilíbrio em função da concentração da quantidade de matéria (Kc) e também em função das pressões parciais
(Kp) a 727°C.
Resolução: Equação química balanceada de decomposição do cloreto de X: XCl3(g) ⇄ XCl(g) + Cl2(g)
Base de cálculo: 4 mol (inicial) de PCl5(g).
Tabela de equilíbrio químico:
XCl3(g) XCl (g) Cl2(g)

(mol) (mol) (mol)
Início 4 0 0
Reage 4α = 4 x 0,90 4 x 0,90 4 x 0,90
Equilíbrio 4 – 3,60 = 0,40 3,60 3,60

Expressão e cálculo das concentrações das quantidades de matéria das espécies gasosas (mol.L-1):
0,40 mol
[XCl3 ] =
1L
3,60 mol
[XCl] =
1L
3,60 mol
[Cl2 ] =
1L

Expressão e cálculo da constante de equilíbrio em função das concentrações das quantidades de matéria (Kc): K C =
[XCl] x [Cl2 ] (3,60)2 12,96
[XCl3 ]
= 0,40
= 0,40
= 32,40

Cálculo da constante de equilíbrio em função das pressões parciais (Kp): K p = K c x (RT)∆n


K p = 32,40 x [0,8206 𝑥 (273 + 727)]2−1

618
1600 FÍSICO-QUÍMICA APLICADA EXERCÍCIOS COMENTADOS - IME – ITA – OLIMPÍADA

K p = 32,40 x 0,08206 𝑥 1000 = 2658,74

Questão 1138 – A constante cinética para a decomposição do (CH3)2O é igual a 2,30 x 10-3 s-1 a uma temperatura de
aproximadamente igual a 500°C. A reação é realizada em um reator a volume constante. Inicialmente a pressão
parcial do (CH3)2O é igual a 0,365 atm. Determine a pressão do sistema após 6,50 minutos. Considere o sistema
reacional com comportamento ideal.

(CH3)2O(g) ⇄ CH4(g) + H2(g) + CO(g)


PFinal
(CH3 )2 O
Resolução: Cálculo da pressão de (CH3)2O após 6,50 minutos: ln ( ) = −k x tempo
PInicial
(CH ) O
3 2

Final
P(CH 3 )2 O
60 s
ln ( ) = −2,30 x 10−3 x {6,50 minutos x }
0,365 1 minuto

Final
P(CH 3 )2 O
ln ( ) = −2,30 x 10−3 x 390
0,365

Final
P(CH 3 )2 O
ln ( ) = −0,897
0,365

Final
P(CH 3 )2 O
( ) = e(−0,897)
0,365

Final
P(CH 3 )2 O
= 0,365 atm x e⏟(−0,897) = 0,149 atm
0,408

Para a determinação das pressões do sistema, será necessário utilizar a tabela de equilíbrio químico.

CH4(g) H2(g) CO(g)


(CH3)2O(g) (atm) ⇄
(atm) (atm) (atm)
Início 0,365 0 0 0
Reage 0,365 - 0,149 0,365 - 0,149 0,365 - 0,149 0,365 - 0,149
Equilíbrio 0,149 0,216 0,216 0,216

Cálculo da pressão total do sistem (Ptotal):

Ptotal = P(CH3 )2 O + PCH4 + PH2 + PCO

Ptotal = 0,149 atm + 0,216 atm + 0,216 atm + 0,216 atm = 0,797 atm

Questão 1139 – (GRILLO) Considere a reação de síntese da amônia, representada pela seguinte equação química
balanceada: ½ N2(g) + 3/2 H2(g) ⇄ NH3(g), apresentando constante de equilíbrio igual a 1,45 x 10-2 a 350℃ e 3,95 x 10-
1 a 450℃. A partir desta informação, determine o valor da variação da entalpia padrão (∆H°) e também analise pelo

princípio de Le Chatelier, o que ocorre com o sistema caso haja uma diminuição da temperatura deste sistema
reacional.

(623) (723)
Resolução: Dados do problema: k1 = 1,45 x 10−2 e k 2 = 3,95.
k ∆H0 1 1
Aplicando a equação de Van´t Hoff: ln (k1 ) = R
x {T − T }
2 2 1

619
1600 FÍSICO-QUÍMICA APLICADA EXERCÍCIOS COMENTADOS - IME – ITA – OLIMPÍADA

1,45 x 10−2 ∆H 0 1 1
ln ( )= x{ − }
3,95 8,314 723 623

∆H 0
ln(3,67 x 10−3 ) = x {− 2,22 x 10−4 }
8,314

−5,61 x 8,314 46,64 J kJ


∆H 0 = −4
= −4
= + 210090,1 (+ 210,01 )
− 2,22 x 10 2,22 x 10 mol mol

Como o sistema apresenta comportamento endotérmico (∆H° > 0), aumentando a temperatura do sistema, o equilíbrio
será deslocado para a direita.

Questão 1140 – (MESTRE JOÃO ROBERTO DA PACIÊNCIA NABUCO) Considere as seguintes reações químicas
reversíveis apresentadas a seguir:
a) 2 SO3(g) + Q1 ⇄ 2 SO2(g) + O2(g)
b) 2 NO2(g) ⇄ N2O4(g) + Q2
c) 2 HI(g) ⇄ H2(g) + I2(g) + Q3
d) CO(g) + H2O(g) ⇄ CO2(g) + H2(g) + Q4
Onde Q1, Q2, Q3 e Q4 são as quantidades de calor em kcal.mol-1. Através do princípio de Le chatelier, em que sentido
é deslocado o equilíbrio:
I. Quando aumenta a pressão do sistema, mantendo a temperatura constante.
II. Quando diminui a temperatura do sistema, mantendo a pressão constante.

Resolução:
I) Analisando a reação química (a): aumentando a pressão do sistema reacional, o equilíbrio é deslocado para a
esquerda, favorecendo dióxido de enxofre e gás oxigênio.
Analisando a reação química (b): Aumentando a pressão do sistema reacional, o equilíbrio é deslocado para a direita,
favorecendo o tetróxido de dinitrogênio.
Analisando a reação química (c): Aumentando ou até mesmo se fosse o caso de diminuição da pressão do sistema
reacional, o equilíbrio não sofreria alteração.
Analisando a reação química (d): Aumentando ou até mesmo se fosse o caso de diminuição da pressão do sistema
reacional, o equilíbrio não sofreria alteração.

II) Analisando a reação química (a): Diminuindo a temperatura do sistema reacional, o equilíbrio é deslocado para a
esquerda, favorecendo trióxido de enxofre.
Analisando a reação química (b): Diminuindo a temperatura do sistema reacional, o equilíbrio é deslocado para a
direita, favorecendo o tetróxido de dinitrogênio.
Analisando a reação química (c): Diminuindo a temperatura do sistema reacional, o equilíbrio é deslocado para a
direita, favorecendo os gases H2 e I2.
Analisando a reação química (d): Diminuindo a temperatura do sistema reacional, o equilíbrio é deslocado para a
direita, favorecendo os gases CO2 e H2.

Questão 1141 – (U. S. NATIONAL CHEMISTRY OLYMPIAD) A closed container at 450 K initially contains only PCl5(g)
and PCl3(g), each with a partial pressure of 2,7 bar. After the system reaches equilibrium, what is the partial pressure
of Cl2(g)?
PCl5(g)  PCl3(g) + Cl2(g), Kp = 0,015.
a) 0,015 bar
b) 0,12 bar
c) 0,20 bar
d) 2,7 bar

Resolução: Alternativa A.
Equação química: PCl5(g)  PCl3(g) + Cl2(g)

620
1600 FÍSICO-QUÍMICA APLICADA EXERCÍCIOS COMENTADOS - IME – ITA – OLIMPÍADA

Base de cálculo: pressão parcial inicial de PCl5(g) e Cl2(g) igual a 2,7 bar.

PCl5(g)  PCl3(g) Cl2(g)


Início 2,7 bar 2,7 bar 0
Reage X X X
Equilíbrio 2,7 - X 2,7 + X X

[PCl3 ] x [Cl2 ]
Expressão da constante de equilíbrio de equilíbrio em função das pressões parciais: K ps = [PCl5 ]

(2,7+X) .X
Cálculo do valor de X: K ps = [PCl5 ]

X² + 2,715X – 0,0405 = 0
Resolvendo a equação do segundo grau, as raízes são as seguintes: X1 = 0,0145 bar e X2 < 0.
A pressão parcial do cloro gasoso no equilíbrio é igual a 0,0145 bar.
Questão 1142 – (IME) Considerando o sistema em equilíbrio de água líquida, gelo e vapor d’água, pede-se o número
de componentes e o número de graus de liberdade do sistema. Justifique as respostas.
Resolução: Segundo a regra de fases, V + F = C + 2, onde F é o número de fases; C é o número de componentes;
V é o número de graus de liberdade, sendo grau de liberdade cada fator independente (composição, pressão,
temperatura) que se pode variar livremente num sistema em equilíbrio, sem que desapareça qualquer das fases
presentes. Nosso sistema em estudo é o seguinte: 𝑯𝟐 𝑶(𝒔) ⇄ 𝑯𝟐 𝑶(𝒍) ⇄ 𝑯𝟐 𝑶(𝒈)
F = 3 (sólida, líquida e gasosa), C = 1 (o único componente é H2O). Substituindo os valores na regra das fases: V = 1
+ 2 – 3 = 0. O número de graus de liberdade é zero e o sistema é invariante, ou seja, isso significa que não se pode
alterar a pressão nem a temperatura sem que desapareça uma das fases.

621
1600 FÍSICO-QUÍMICA APLICADA EXERCÍCIOS COMENTADOS - IME – ITA – OLIMPÍADA

Questão 944 – (OLIMPÍADA


MUNDIAL DE QUÍMICA) Uma
determinada substância sofre
decomposição segundo uma
cinética de primeira ordem, e
sua dependência em relação
à temperatura segue uma lei
empírica chamada de
equação de Arrhenius. Os
622
1600 FÍSICO-QUÍMICA APLICADA EXERCÍCIOS COMENTADOS - IME – ITA – OLIMPÍADA

tempos de meia-vida
detCAPÍTULO VIII

ESTUDO DO EQUILÍBRIO
IÔNICO – KW, KA, KB, KPS E KH

PROFESSOR ALEXANDRE
VARGAS GRILLO

Questão 1143 –
A - (U. S. NATIONAL CHEMISTRY OLYMPIAD) A rule of thumb is that a reaction’s rate roughly doubles for every
10°C increase in temperature. What is the activation energy of a reaction whose rate exactly doubles between 25,0°C
and 35,0°C?
a) 52,9 kJ.mol–1
b) 153 kJ.mol–1
c) 504 kJ.mol–1
d) 523 kJ.mol–1

Resolução: Alternativa A.

623
1600 FÍSICO-QUÍMICA APLICADA EXERCÍCIOS COMENTADOS - IME – ITA – OLIMPÍADA

k Eat 1 1
Para a resolução do problema será necessário a utilização da equação de Arrhenius: ln (k1 ) = R
x {T − T }
2 2 1

Dados do problema: k 2 = 2 x k1 ; T1 = 298 K e T2 = 308 K.


k1 Eat 1 1
ln ( )= x{ − }
2 x k1 8,314 308 298
1
ln (2) x 8,314 −0,693 x 8,314 J kJ
Eat = = = 52882,29 (52,88 )
{− 1,089 x 10−4 } −4
{− 2,64 x 10 } mol mol
B - (U. S. NATIONAL CHEMISTRY OLYMPIAD) The molar solubility of PbF2 is 2,1 × 10–3 mol.L–1. What is its Ksp?
a) 4,4 × 10–6
b) 8,8 × 10–6
c) 3,7 × 10–8
d) 9,3 × 10–9

Resolução: Alternativa C.
Reação de dissociaçõ do fluoreto de chumbo: PbF2(s) ⇄ Pb+2(aq) + 2 F-(aq)
Cálculo da concentração da quantidade de matéria dos íons: PbF2(s) ⇄ Pb+2(aq) + 2 F-(aq)
1 mol de PbF2(s) ------- 1 mol de Pb+2(aq) ---- 2 mol de F-(aq)
2,1 × 10–3 mol.L–1 ---- [Pb+2]------------------- [F-]
[Pb+2]= 2,1 × 10–3 mol.L–1
[Pb+2]= 4,2 × 10–3 mol.L–1

Cálculo da constante de equilíbrio do produto de solubilidade: K ps = [Pb+2 ] x [F − ]2 =


2,1 x 10−3 x (4,2 x 10−3 )2 = 3,70 x 10−8

Questão 1144 - (U. S. NATIONAL CHEMISTRY OLYMPIAD)


A) What is the solubility of calcium hydroxide in mol·L–1? Calcium hydroxide: 4,0 × 10–6.
(A) 1,6 × 10–2 (B) 1,0 × 10–2 (C) 2,0 × 10–3 (D) 1,0 × 10–3
B) A liquid has a vapor pressure of 40 mmHg at 19,0˚C and a normal boiling point of 78,3˚C. What is its enthalpy of
vaporization in kJ.mol-1?
(A) 42,4 (B) 18,4 (C) 5,10 (D) 1,45
C) For the rate equation, Rate = k[A].[B]2, what are the units for the rate constant, k, if the rate is given in mol.L -1.sec-
1?

(A) L.mol.sec (B) L.mol-1.sec-1 (C) L2.mol-2.sec-1 (D) L3.mol-3.sec-2


Resolução:

A) Alternativa B.
Relação entre o produto de solubilidade e a solubilidade: K ps = 4 x S 3

Cálculo da solubilidade: 4,0 x 10−6 = 4 x S 3


4,0 x 10−6
S3 = = 10−6
4

624
1600 FÍSICO-QUÍMICA APLICADA EXERCÍCIOS COMENTADOS - IME – ITA – OLIMPÍADA

3 𝑚𝑜𝑙
S = √10−6 = 10−2
𝐿
B) Alternativa A.
Dados do problema: P1 (19°C) = 40 mmHg e P1 (78,3°C) = 760 mmHg
P ∆H0 1 1
Aplicando a equação de Clausius-clapeyron: ln (P1 ) = R
x {T − T }
2 2 1

40 ∆H 0 1 1
ln ( )= x{ − }
760 8,314 (78,3 + 273) (19 + 273)

∆H 0
ln(5,26 x 10−2 ) = x {2,85 x 10−3 − 3,42 x 10−3 }
8,314

ln(5,26 x 10−2 ) x 8,314 = ∆H 0 x {−0,57 x 10−3 }

ln(5,26 x 10−2 ) x 8,314 J kJ


∆H 0 = {−0,57 x 10−3 }
= +42877,19 mol (+42,88 mol)

C) Alternativa C.
Equação da taxa de velocidade: r = k x [A] x [B]2

mol mol mol 2


=kx x( )
Lxs L L

k=
mol2 x s
Questão 1145 – (U. S. NATIONAL CHEMISTRY OLYMPIAD) How many moles of HCOONa must be added to 1,0 L
of 0.10 mol.L-1 HCOOH to prepare a buffer solution with a pH of 3,4? (HCOOH Ka = 2 × 10–4)
a) 0,01
b) 0,05
c) 0,1
d) 0,2

Resolução: Alternativa B.
O problema retrata uma situação de uma solução tampão, observando que a solução é composta por um ácido fraco
e seu sal conjugado.
[sal]
pH = pka + log
[ácido]
[sal]
pH = − log K a + log
[ácido]
nsal
Vsolução
3,4 = − log(2 x 10−4 ) + log
0,10
nsal
1,0
3,40 = −(−3,70) + log
0,10

nsal
log = − 0,30
0,10

625
1600 FÍSICO-QUÍMICA APLICADA EXERCÍCIOS COMENTADOS - IME – ITA – OLIMPÍADA

nsal
= 10(− 0,30)
0,10

nsal = 0,050 mol

Questão 1146 – (ITA) Determine a massa de hidróxido de potássio que deve ser dissolvida em 0,500 mL de água
para que a solução resultante tenha um pH ≈ 13 a 25°C.

Resolução: Equação química: KOH(s) → K+(aq) + OH-(aq)


Sabendo que na temperatura de 25°C pH + pOH = 14, temos: pH + pOH = 14
13 + pOH = 14
pOH = 1
Cálculo da concentração da quantidade de matéria da hidroxila [OH-]: pOH = - log[OH-]
1 = - log[OH-]
[OH-] = 0,10 mol.L-1
Cálculo da concentração da quantidade de matéria de KOH: KOH(s) → K+(aq) + OH-(aq)
1 mol de KOH---------------- 1 mol de OH-
[KOH] ------------------------- 0,10 mol.L-1 de OH-
[KOH] = 0,10 mol.L-1
Cálculo da massa de KOH, a partir da concentração da quantidade de matéria: mKOH = [KOH] x <
MM >KOH x Vsolução = 0,10 x 56 x 0,50 x 10−3 = 0,0028 g (2,80 mg)

Questão 1147 – (GRILLO) A 27°C, o produto de solubilidade do sulfato de chumbo II é igual a 1,0 x 10-8 e as
constantes de equilíbrio para as três equações químicas estão apresentadas a seguir:

a) PbSO4(s) + 2 I-(aq)  PbI2(s) + SO4-2(aq) K1 = 4,0 x 10-4


b) PbI2(s) + CrO4-(aq)  PbCrO4(s) + 2 I-(aq) K2 = 4,0 x 1012
c) PbS(s) + CrO4-(aq)  PbCrO4(s) + S-2(aq) K3 = 1,60 x 10-8

Através dos dados fornecidos acima, assinale a alternativa que apresenta o valor da energia livre Gibbs padrão,
considerando que a constante dos gases seja igual a 8,0 J.mol-1.K-1.

a) 4500 J/mol
b) 5000 J/mol
c) 5500 J/mol
d) 6000 J/mol
e) Nenhuma das respostas anteriores

Resolução: Alternativa C.

PbS(s)  Pb+2(aq) + S-2(aq) K5 = ?


PbS(s) + CrO4-(aq)  PbCrO4(s) + S-2(aq) K3 = 1,60 x 10-8
PbSO4(s)  Pb+2(aq) + SO4-2(aq) K4 = 1,0 x 10-8
1
PbCrO4(s) + 2 I-(aq)  PbI2(s) + CrO4-(aq) K2 = 4,0 𝑥 1012
1
PbI2(s) + SO4-2(aq)  PbSO4(s) + 2 I-(aq) K1 = 4,0 𝑥 10−4 +
1 1
PbS(s)  Pb+2(aq) + S-2(aq) K5 = 1,60 𝑥 10−8 𝑥 1,0 𝑥 10−8 𝑥 4,0 𝑥 1012 𝑥 4,0 𝑥 10−4

626
1600 FÍSICO-QUÍMICA APLICADA EXERCÍCIOS COMENTADOS - IME – ITA – OLIMPÍADA

O K5 que corresponde ao produto de solubilidade do sulfeto de chumbo II é igual a: K5 = Kps = 0,10

Cálculo da energia livre de Gibbs padrão (∆G°) pata T = 27°C: ∆𝐺° = −𝑅 𝑥 𝑇 𝑥 𝑙𝑛𝐾

Sabendo que lnK = 2,303 x logK, temos: ∆G° = −R x T x 2,303 x logK

J
∆G° = −8 x (27 + 273)x 2,303 x log(0,10) = +5527,20
mol

Questão 1148 – (OLIMPÍADA CEARENSE DO ENSINO SUPERIOR DE QUÍMICA) Qual resposta apresentam juntas
o pH da solução e a pressão do gás H2 no eletrodo normal de hidrogênio.
a) pH = 2,35 e pressão = 1,16 atm
b) pH = 2,28 e pressão = 1,00 atm
c) pH = 0,00 e pressão = 2,00 atm
d) pH = 0,00 e pressão = 1,00 atm
e) pH = 7,00 e pressão = 1,00 atm

Resolução: Alternativa D.
As condições padrões efetuadas para uma pilha galvânica são as seguintes:
p = 1 atm, [ ] = 1 mol.L-1 e T = 25°C.
Diante do fato de que a concentração da quantidade de matéria seja igual a 1, o seu pH será igual a zero (0).
Questão 1149 – (IIT-JEE) A solubilidade do hidróxido de magnésio na presença de água é igual a 9,57 x 10 -3 g.L-1.
Calcule a solubilidade em g.L-1 na presença de 0,02 mol.L-1 de nitrato de magnésio.

Resolução: Hidróxido de magnésio na presença de água: Mg(OH)2(s)  Mg+2(aq) + 2 OH-(aq)


C 9,57 x 10−3
Cálculo da solubilidade do hidróxido de magnésio (mol.L-1): S = [Mg(OH)2 ] = <MM> = 58
=
Mg(OH)2
mol
1,65 x 10−4 L

Cálculo do produto de solubilidade do hidróxido de magnésio: Mg(OH)2(s)  Mg+2(aq) + 2 OH-(aq)

1 mol de Mg(OH)2(s) ----- 1 mol de Mg+2(aq) ----- 2 mol de OH-(aq)


1 mol de Mg(OH)2(s) ----- S ------------------------ 2 S

Equação e cálculo do produto de solubilidade (Kps): K ps = S x (2S)2 = 4S³


K ps = 4S³ = 4 x (1,65 x 10−4 )3 = 1,80 x 10−11

Cálculo da solubilidade do hidróxido de magnésio com a presença de nitrato de sódio (0,02 mol.L -1): Mg(NO3)2(s) 
Mg+2(aq) + 2 NO3-(aq)
1 mol de Mg(NO3)2(s) ----- 1 mol de Mg+2(aq) ------ 2 mol de NO3-(aq)
1 mol de Mg(OH)2(s) ------ S ------------------------- 2 S
K ps = [Mg +2 ] x [OH − ]

1,80 x 10−11 = (0,02 + S) x (2S)2

1,80 x 10−11 = ⏟
(0,02 + S) x 4S²
≅0,02

1,80 x 10−11 = 0,02 x 4S²

627
1600 FÍSICO-QUÍMICA APLICADA EXERCÍCIOS COMENTADOS - IME – ITA – OLIMPÍADA

mol
S = 1,50 x 10−5
L
C
Cálculo da concentração mássica do hidróxido de magnésio (mol.L-1): S = [Mg(OH)2 ] = S = <MM>
Mg(OH)2
g
C = 1,50 x 10−5 x 58 = 1,50 x 10−4
L

Questão 1150 – (OLIMPÍADA CEARENSE DO ENSINO SUPERIOR DE QUÍMICA) Uma amostra específica de
vinagre tem pH de 2,90. Supondo que o ácido acético (CH3COOH) seja o único ácido que o vinagre contém (Ka =1,8
× 10-5), informe qual a concentração inicial do ácido acético em mol.L-1.
a) 8,8 × 10-2
b) 7,7 × 10-2
c) 7,4 × 10-2
d) 8,0 × 10-2
e) 9,6 × 10-2

Resolução: Alternativa A.
CH3COOH(aq) + H2O(l) ⇄ CH3COOH-(aq) + H3O+(aq)
Base de cálculo: M (mol.L-1) inicial de CH3COOH(aq)
Tabela de equilíbrio químico:

CH3COOH H2O(l) ⇄ CH3COOH- (aq) H3O+(aq)


Início M - 0 0
Reage Mα - Mα Mα
Equilíbrio M - Mα - Mα Mα

Espécies no estado de equilíbrio: [CH3 COOH] = M − Mα


[CH3 COO− ] = Mα
[H3 O+ ] = Mα

[CH3 CO𝑂 − ] x [H3 O+ ] Mα x Mα


Expressão da constante de equilíbrio do ácido (Ka): K a = [CH3 COOH]
=
M−Mα

Cálculo da concentração de hidrônio, a partir do pH: 2,90 = −log[H3 O+ ]

mol
[H3 O+ ] = 10−2,90 = 1,26 x 10−3
L
Como no equilíbrio a concentração de [H3O+] é igual a Mα, logo a concentração da quantidade de mat´ria designada
Mα x Mα
pelo M, temos: K a = M−Mα

[H3 O+ ] [H3 O+ ]
⏞ x Mα
Mα ⏞
1,80 𝑥 10−5 =
M − Mα

[H3 O+ ]

1,26 x 10−3 x 1,26 x 10−3


1,80 𝑥 10−5 =
M − 1,26 x 10−3
1,80 𝑥 10−5 x (M − 1,26 x 10−3 ) = 1,26 x 10−3 x 1,26 x 10−3
(1,26 x 10−3 )2 mol
M = 1,26 x 10−3 + −5
= 1,26 x 10−3 + 0,0882 = 8,95 x 10−2
1,80 𝑥 10 L

628
1600 FÍSICO-QUÍMICA APLICADA EXERCÍCIOS COMENTADOS - IME – ITA – OLIMPÍADA

Questão 1151 – (OLIMPÍADA CEARENSE DO ENSINO SUPERIOR DE QUÍMICA) Uma solução aquosa contém
0,675 g de NaOH em um volume de 500 mL. Qual o pH desta solução?
a) 2,97
b) 13,00
c) 12,52
d) 3,36
e) 6,75

Resolução: Alternativa C.
0,675 mol
Cálculo da concentração da quantidade de matéria do hidróxido de sódio: [NaOH] = 40 x 0,500 = 0,03375 L

H2 O
Reação de dissociação do hidróxido de sódio: NaOH(aq) → Na+ −
(aq) + OH(aq)

Pela estequiometria, temos:


1 mol de NaOH -------- 1 mol de OH-
0,03375 mol.L-1 -------- [OH-]
[OH-] = 0,03375 mol.L-1

Cálculo do pOH: pOH = −log[OH − ]

pOH = − log(0,03375)
pOH = −(−1,47) = +1,47

Cálculo do pH: pH = 14 − pOH = 14 − 1,47 = 12,53

Questão 1152 – (IFRJ – NILÓPOLIS - TURMA QIM) Um estudante do curso técnico de química do Instituto Federal
do Rio de Janeiro – Campus nilópolis estudaram o comportamento de uma solução de ácido acético com concentração
igual a 0,010 mol.L-1 apresentando um pH igual a 2,68. a partir desta informação, calcule a concentração da quantidade
de matéria do ácido e também o seu grau de ionização.

Resolução: Equação química: C2H4O2(aq) + H2O(l) ⇄ C2H3O2-(aq) + H3O+(aq)


Base de cálculo: M (mol.L-1) inicial de C2H4O2 (aq)
Tabela de equilíbrio químico:

C2H4O2 H2O(l) ⇄ C2H3O2-(aq) H3O+(aq)


Início M - 0 0
Reage Mα - Mα Mα
Equilíbrio M - Mα - Mα Mα
Espécies no estado de equilíbrio:
[C2 H4 O2 ] = M − Mα
[C2 H3 O−2 ] = Mα
[H3 O+ ] = Mα

[C2 H3 O− +
2 ] x [H3 O ] Mα x Mα Mα x Mα Mα²
Expressão da constante de equilíbrio do ácido (Ka): K a = [C2 H4 O2 ]
= = =
M−Mα M x (1−α) 1−α

Cálculo do grau de ionização, a partir do pH: 2,68 = −log[H + ]


mol
[H + ] = 10−2,68 = 2,09 x 10−3
L
mol
Como no equilíbrio a concentração de H+ é igual a M.α, o grau de dissociação será: M x α = 2,09 x 10−3 L

629
1600 FÍSICO-QUÍMICA APLICADA EXERCÍCIOS COMENTADOS - IME – ITA – OLIMPÍADA

mol mol
0,010 x α = 2,09 x 10−3
L L
mol mol
0,010 x α = 2,09 x 10−3
L L

2,09 x 10−3
α= = 0,209 (20,90%)
0,010

[C2 H3 O− +
2 ] x [H3 O ] Mα² 0,010 x (0,209)²
Cálculo da constante de equilíbrio do ácido: K a = [C2 H4 O2 ]
= = = 5,52 x 10−4
1−α 1−0,209

Questão 1153 – Calcule a atividade dos íons presentes em uma solução 2,0 x 10-4 mol.L-1 de cloreto de magnésio,
levando em consideração que este sal seja 100% solúvel.

Resolução: Antes de calcularmos a atividade dos íons presentes em solução, cabe aqui uma pequena definição sobre
esses conceitos físico-químicos.
A atividade no ramo da físico-química representa a concentração efetiva, levando em consideração o soluto na
presença de uma quantidade de solvente. Um íon em uma solução fica rodeado de outros íons de carga oposta a sua,
o que a separação desses íons torna-se uma tarefa impossível de ser realizada.
A atividade de uma espécie X qualquer (aX) é dada pela seguinte equação matemática, apresenta a seguir: ax =
γ±
x x [X], onde:

ax = atividade do soluto X,

γ±
x = coeficiente de atividade;

[X] = concentração da quantidade de matéria do soluto X.

Em soluções muito diluídas, a atividade se iguala a concentração da quantidade de matéria. Já com relação ao
coeficiente de atividade, é uma função da força iônica (I) presente na solução, sendo representado pela seguinte
equação matemática:

𝟏
𝐈= 𝐱 ∑ 𝐳𝐢𝟐 𝐱 [𝐢]
𝟐

Onde: I = função iônica, zi = carga do íon e [i] = concentração da quantidade de matéria do íon i.
Realizando os cálculos referente ao problema:
MgCl2(aq) → Mg +2 −
(aq) + 2Cl(aq)

Cálculo das concentrações das quantidades de matéria dos íons presentes no cloreto de magnésio:

1 mol de MgCl2 ------------------------ 1 mol de Mg+2 ---------- 2 mol de Cl-


2 x 10-4 mol.L-1 de MgCl2 ------------ [Mg+2] -------------------- [Cl-]

[Mg+2] = 2 x 10-4 mol.L-1

[Cl-] = 4 x 10-4 mol.L-1


1
Cálculo da força iônica (I): I = 2 x {zi2 x [Mg +2 ] + zi2 x [Cl− ]}

630
1600 FÍSICO-QUÍMICA APLICADA EXERCÍCIOS COMENTADOS - IME – ITA – OLIMPÍADA

1 1
I= x {(2)2 x 2,0 x 10−4 + (−1)² x 4,0 x 10−4 } = x {8,0 x 10−4 + (−1)² x 4,0 x 10−4 }
2 2
1 −4 −4
= x {12,0 x 10 } = 6,0 x 10
2

Cálculo do coeficiente de atividade, a partir da lei de Debye-Huckel: logγ±


x = −|z1 x z2 | x A x √I

logγ±
x = −|2 x (−1)| x 0,509 x √6,0 x 10
−4

logγ±
x = −2 x 0,509 x 2,45 x 10
−2

logγ±
x = − 0,0249

γ±
x = 10
(−0,0249)
= 0,944

Cálculo das atividades dos íons:

Para o cátion magnésio: aMg+2 = γ± +2


x x [Mg ]

aMg+2 = 0,944 x 2,0 x 10−4 = 1,89 x 10−4

Para o ânion cloreto: aCl− = γ± −


x x [Cl ]

aCl− = 0,944 x 4,0 x 10−4 = 3,78 x 10−4

Questão 1154 – Calcule as atividades dos íons em uma solução 10-4 mol.L-1 de CuCl2.

Resolução: Dissociação do cloreto de magnésio: CuCl2(aq) → Cu+2 −


(aq) + 2Cl(aq)

Cálculo das concentrações das quantidades de matéria dos íons presentes no cloreto de magnésio:

1 mol de CuCl2 ------------------------ 1 mol de Cu+2 ---------- 2 mol de Cl-


10-4 mol.L-1 de CuCl2 ----------------- [Cu+2] ------------------- [Cl-]

[Cu+2] = 10-4 mol.L-1

[Cl-] = 2 x 10-4 mol.L-1


1
Cálculo da força iônica (I): I = 2 x {zi2 x [Cu+2 ] + zi2 x [Cl− ]}

1 1
I= x {(2)2 x 10−4 + (−1)² x 2,0 x 10−4 } = x {4,0 x 10−4 + (−1)² x 2,0 x 10−4 }
2 2
1 −4 −4
= x {6,0 x 10 } = 3,0 x 10
2

Cálculo do coeficiente de atividade, a partir da lei de Debye-Huckel: logγ±


x = −|z1 x z2 | x A x √I

logγ±
x = −|2 x (−1)| x 0,509 x √3,0 x 10
−4

logγ±
x = − 0,0176

γ±
x = 10
(−0,0176)
= 0,960

Cálculo das atividades dos íons: aCu+2 = γ± +2


x x [Mg ]

631
1600 FÍSICO-QUÍMICA APLICADA EXERCÍCIOS COMENTADOS - IME – ITA – OLIMPÍADA

mol
aCu+2 = 0,960 x 10−4 = 9,60 x 10−5
L

aCl− = γ± −
x x [Cl ]

mol
aCl− = 0,960 x 2,0 x 10−4 = 1,92 x 10−4
L

Questão 1155 – Calcule as atividades dos íons através de uma solução de sulfato de magnésio, que apresenta
concentração igual a 3 x 10-4 mol.kg-1, obedecendo a lei de Debye-Huckel.

Resolução: Dissociação do sulfato de magnésio: MgSO4(aq) → Mg +2 −2


(aq) + SO4(aq)

Cálculo das concentrações das quantidades de matéria dos íons presentes na solução de sulfato de magnésio:
−2
1 mol de MgSO4(aq) ------------------------ 1 mol de Mg+2 ---------- 1 mol de 𝑆𝑂4(𝑎𝑞)
3,0 x 10-4 mol.L-1 de CuCl2 --------------- [Mg+2] ------------------- [𝑆𝑂4−2 ]

[Mg+2] = 3,0 x 10-4 mol.kg-1

[𝑆𝑂4−2 ] = 3,0 x 10-4 mol.kg-1


1
Cálculo da força iônica (I): I = 2 x {zi2 x [Cu+2 ] + zi2 x [Cl− ]}

1
I= x {(2)2 x 3,0 x 10−4 + (−2)² x 3,0 x 10−4 }
2
1 1
I= x {12,0 x 10−4 + 12,0 x 10−4 } = x {24,0 x 10−4 } = 12,0 x 10−4
2 2

Cálculo do coeficiente de atividade, a partir da lei de Debye-Huckel: logƔ± = −|z1 x z2 | x A x √I

logƔ± = −|2 x (−2)| x 0,509 x √12,0 x 10−4

logƔ± = −4 x 0,509 x 3,46 x 10−2

logƔ± = − 0,0704

Ɣ± = 10(−0,0704) = 0,850

Cálculo das atividades dos íons:

Para o cátion magnésio: aMg+2 = Ɣ± x WMg+2

aMg+2 = 0,850 x 3,0 x 10−4

aMg+2 = 2,55 x 10−4

Para o ânion sulfato: aSO−2


4
= Ɣ± x WSO−2
4

aSO−2
4
= 0,850 x 3,0 x 10−4

aSO−2
4
= 2,55 x 10−4

632
1600 FÍSICO-QUÍMICA APLICADA EXERCÍCIOS COMENTADOS - IME – ITA – OLIMPÍADA

Questão 1156 – Uma solução de cloreto de magnésio tem coeficiente médio de atividade calculada pela lei de Debye
– Huckel igual a 0,85. Calcule a massa de cloreto de magnésio presente em 100 gramas de água para formar essa
solução.

Resolução: Dissociação do cloreto de magnésio: MgCl2(aq) → Mg +2 −


(aq) + 2 Cl(aq)

Cálculo da força iônica (I), a partir da equação de Debye-Huckel: logƔ± = −|z1 x z2 | x A x √I

log(0,85) = −|2 x (−1)| x 0,509 x √I

−7,06 x 10−2 = −2 x 0,509 x √I

−7,06 x 10−2
√I =
− 1,018

√I = 6,93 x 10−2

I = (6,93 x 10−2 )2 = 0,0048

Relação estequiométrica com a molalidade, a partir da reação de dissociação do cloreto de magnésio: MgCl2(aq) →
Mg +2 −
(aq) + 2 Cl(aq)


1 mol de MgCl2(aq) --------------- 1 mol de Mg+2 ---------- 2 mol de 𝐶𝑙(𝑎𝑞)
W ----------------------------------- W ------------------------ 2W
1
Cálculo da massa, a partir da equação da força iônica: I = x {zi2 x [Mg +2 ] + 2 x zi2 x [Cl− ]}
2
1
I = x {(2)2 x W + 2 x (−1)² x W}
2
1
I= x {4 x W + 2 x W}
2

I=3xW
nsoluto
0,00480 = 3 x (kg)
msolvente

msoluto
0,00480 = 3 x 95
0,100

0,00480 x 9,5
msoluto = = 0,0152 g
3
Questão 1157 - (IME) O ácido clorídrico puro, no estado líquido, pode ser eletrolizado? Por que?
Resolução: O ácido clorídrico no estado líquido não pode ser eletrolisado, pelo fato deste composto não está ionizado.
Questão 1158 – (IME) Quanto à precipitação do hidróxido férrico (Kps = 1,0 x 10-36) emu ma solução 0,001 mol.L-1 de
Fe+3, é correto afirmar que:

a) Independe do pH
b) Ocorre somente na faixa da pH alcalino
c) Ocorre somente na faixa de pH ácido
d) Não ocorre para pH < 3

633
1600 FÍSICO-QUÍMICA APLICADA EXERCÍCIOS COMENTADOS - IME – ITA – OLIMPÍADA

e) Ocorre somente para pH ≥ 12

Resolução: Alternativa D.

Dissociação do hidróxido de ferro III (Hidróxido férrico): Fe(OH)3(s) → Fe+3(aq) + 3OH-(aq).

Equação da constante do produto de solubilidade: Kps = [Fe+3] x [OH-]³

Cálculo da concentração da quantidade de matéria de íons hidroxila (OH-): 1,0 x 10-36 = 10-3 x [OH-]³

[OH-]³ = 10-33
3
[OH-] = √10−33 = 10−11mol.L-1

Cálculo do pOH e do pH: pOH = -log[OH-] = - log (10-11) = 11


pH = 14 – 11 = 3
A partir deste pH, o hidróxido de ferro III começa a precipitar a partir de um pH igual a 3.

Questão 1159 – Qual a consequência da adição de um íon comum em uma solução que apresenta um ácido fraco?

Resolução: Quando houver uma solução de um ácido fraco, adicionando a esta solução um íon comum de um sal
derivado, porém comum a este ácido, o equilíbrio será deslocado para a esquerda, ocasionando com isso uma
diminuição da ionização do ácido.

Questão 1160 – Defina solução tampão.

Resolução: Soluções que apresentam como característica fundamental a não sofrerem variações acentuadas em
seu pH, mesmo quando for adicionado uma quantidade considerável de um ácido forte pou de uma base forte. A
solução tampão ocorre entre um ácido fraco e o sal derivado deste ácido e entre uma base e um sal derivado dessa
base.

Questão 1161 – Dê a equação de Handerson-Hasselbach para uma solução tampão entre um ácido fraco e seu sal
conjugado.

[sal]
Resolução: Equação matemática: pH = pK a + log [ácido]

634
1600 FÍSICO-QUÍMICA APLICADA EXERCÍCIOS COMENTADOS - IME – ITA – OLIMPÍADA

Questão 1162 – Defina hidrólise de um sal.

Resolução: Trata-se de um processo químico que consiste em uma reação química entre os íons de um sal com a
água.

Questão 1163 – Analise a hidrólise salina do cloreto de amônio.

Resolução: Fórmula molecular: NH4Cl


+
NH4 Cl(aq) ⇌ NH4(aq) + Cl−
(aq)

+
NH4(aq) + Cl−
(aq) + H2 O(l) ⇌ NH4 OH(aq) + ⏟
HCl(aq)
ácido forte

+
NH4(aq) + Cl− + −
(aq) + H2 O(l) ⇌ NH4 OH(aq) + H(aq) + Cl(aq)

+ +
NH4(aq) + H2 O(l) ⇌ NH4 OH(aq) + H
⏟(aq)
solução ácida

Questão 1164 – Analise a hidrólise salina do cianeto de sódio.

Resolução: Fórmula molecular: NaCN

NaCN(aq) ⇌ Na+ −
(aq) + CN(aq)

Na+ −
(aq) + CN(aq) + H2 O(l) ⇌ NaOH
⏟ (aq) + HCN(aq)
Base forte

Na+ − + −
(aq) + CN(aq) + H2 O(l) ⇌ Na (aq) + OH(aq) + HCN(aq)

− −
CN(aq) + H2 O(l) ⇌ HCN(aq) + OH
⏟ (aq)
solução básica

Questão 1165 – Analise a hidrólise salina do cianeto de amônio.

Resolução: Fórmula molecular: NH4CN


+ −
NH4 CN(aq) ⇌ NH4(aq) + CN(aq)

+ −
NH4(aq) + CN(aq) + H2 O(l) ⇌ ⏟
NH4 OH(aq) + ⏟HCN(aq)
⏟base fraca ácido fraco
SOLUÇÃO NEUTRA

Questão 1166 – Analise a hidrólise salina do cloreto de sódio.

Resolução: Fórmula molecular: NaCl

NaCl(aq) ⇌ Na+ −
(aq) + Cl(aq)

Na+ −
(aq) + Cl(aq) + H2 O(l) ⇌ NaOH
⏟ (aq) + HCl
⏟ (aq)
base forte ácido forte

Na+ − + − + −
(aq) + Cl(aq) + H2 O(l) ⇌ Na (aq) + OH(aq) + H(aq) + Cl(aq)

635
1600 FÍSICO-QUÍMICA APLICADA EXERCÍCIOS COMENTADOS - IME – ITA – OLIMPÍADA

− +
H2 O(l) ⇌ OH(aq) + H(aq)

Questão 1167 – Obtenha a equação matemática da constante de hidrólise (Kh) referente a reação química da questão
1163.
+ +
Resolução: Equação química: NH4(aq) + H2 O(l) ⇌ NH4 OH(aq) + H
⏟(aq)
solução ácida

[NH4 OH(aq) ] x [H+


(aq) ]
Equação química da constante de hidrólise: K h =
[NH+
4(aq) ] x ⏟
aH 2 O
=1

+ −
Sabendo que [H(aq) ] 𝑥 [OH(aq) ] = KW

KW
[NH4 OH(aq) ] x ( − )
[OH(aq) ]
Kh =
+
[NH4(aq) ]

[NH4 OH(aq) ] x K W
Kh =
+ −
[NH4(aq) ] x [OH(aq) ]

[NH+ −
4(aq) ] x [OH(aq) ] KW
Sabendo que K b = , logo: K h = .
[NH4 OH(aq) ] Kb

Questão 1168 – Obtenha a equação matemática da constante de hidrólise (Kh) referente a reação química da questão
1164.
− −
Resolução: Equação química: CN(aq) + H2 O(l) ⇌ HCN(aq) + OH
⏟ (aq)
solução básica

[HCN(aq) ] x [OH−
(aq) ]
Equação química da constante de hidrólise: K h =
[CN−
(aq) ] x ⏟
aH 2 O
=1

+ −
Sabendo que [H(aq) ] 𝑥 [OH(aq) ] = KW

KW
[HCN(aq) ] x ( )
+
[H(aq) ]
Kh = −
[CN(aq) ]

[HCN(aq) ] x K W
Kh =
− +
[CN(aq) ] x [H(aq) ]

[CN− +
(aq) ] x [H(aq) ] KW
Sabendo que K a = , logo: K h = .
[HCN(aq) ] Ka

Questão 1169 – Obtenha a equação matemática da constante de hidrólise (Kh) referente a reação química da questão
1165.

636
1600 FÍSICO-QUÍMICA APLICADA EXERCÍCIOS COMENTADOS - IME – ITA – OLIMPÍADA

+ −
Resolução: Equação química: NH4(aq) + CN(aq) + H2 O(l) ⇌ NH
⏟ 4 OH(aq) + HCN
⏟ (aq)
⏟base fraca ácido fraco
SOLUÇÃO NEUTRA

[NH4 OH(aq) ] x [HCN(aq) ]


Equação química da constante de hidrólise: K h =
[NH+ −
4(aq) ] x [CN(aq) ] x a
⏟H2O
=1

[H+ −
(aq) ] x [CN(aq) ] [NH+ −
4(aq) ] x [OH(aq) ]
Sabendo que: [HCN(aq) ] = Ka
e que [NH4 OH(aq) ] = Kb
. Substitutindo na equação
do equilíbrio de hidrólise, temos:
KW
+ ⏞ − + −
[NH4(aq) ] x [OH(aq) ] x [H(aq) ] x [CN(aq) ] KW
Kh = =
+
[NH4(aq) −
] x [CN(aq) ] x Kb x Ka Kb x Ka

KW
Kh =
Kb x Ka

Questão 1170 - (ITA) Qual será o pH de uma solução em que a concentração de íons H+ é igual a 2,0 x 10-4 mol.L-1?
a) 2,4
b) 3,0
c) 3,7
d) 4,0
e) 4,3

Resolução: Alternativa C.

Cálculo do valor da concentração de H+: pH = −log[2,0 x 10−4 ] = −[log2,0 + log10−4 ] = −[0,30 − 4,0] =
3,70
Questão 1171 – (ITA) Calcule a solubilidade do cloreto de prata, sabendo que seu produto de solubilidade é 1,80 x
10-10.

Resolução: Equação iônica: AgCl(s) ⇌ Ag + −


(aq) + Cl(aq)

1 mol de AgCl(s) − − − 1 mol de Ag + −


(aq) − − − 1 mol de Cl(aq)

Equação do produto de solubilidade: K ps = [Ag + −


(aq) ] x [Cl(aq) ]

Relação da constante do produto de solubilidade com a solubilidade: K ps = 𝑆²

1,80 x 10−10 = S²

mol
𝑆 = √1,80 x 10−10 = 1,34 x 10−5
L
Questão 1172 - (ITA) Em relação aos equilíbrios:
H2X(aq) ⇄ H+(aq) + HX-(aq) K1
HX-(aq) ⇄ H+(aq) + X-2(aq) K2
Podemos afirmar dizer, em geral, que:
a) K1 > K2
b) K1 < K2

637
1600 FÍSICO-QUÍMICA APLICADA EXERCÍCIOS COMENTADOS - IME – ITA – OLIMPÍADA

c) K1 ≈ K2
d) K1 > 0; K2 < 0
e) K1 < 0; K2 > 0

Resolução: Alternativa A.
Para cada equilíbrio apresentado na questão, pode-se escrever uma equação para a constante de equilíbrio, baseada
na Lei da Ação das Massas. Como os equilíbrios estão acoplados entre si por causa da presença de espécies
concomitantes, a cada constante é atribuído um índice, representado por K1 e K2. A cada processo de ionização para
um sistema poliprótico, a constante de equilíbrio decresce nos estágios sucessivos da ionização, isto se deve ao
decréscimo da tendência próton doadora dos sucessivos íons residuais cuja carga negativa crescente se opõe, cada
vez mais, a transferência de um próton ao solvente.

Questão 1173 – (ITA) O pH de uma solução 0,010 mol.L-1 de NaOH é, aproximadamente, igual a:
a) 1,0
b) 2,0
c) 10,0
d) 12,0
e) 14,0

Resolução: Alternativa D.
1 mol de NaOH ----------- 1 mol de [OH-]
[NaOH] = [OH-] = 10-2 mol x L-1

Cálculo do pOH: pOH = −log[OH − ]


pOH = −log(0,010) = −(−2) = 2

pH = 14 – pOH = 14 – 2 = 12

Questão 1174 – (ITA) O pH de uma solução 0,020 mol.L-1 de HCl é aproximadamente igual a:
a) 1,3
b) 1,7
c) 2,0
d) 2,3
e) 4,0

Resolução: Alternativa B.

Sabendo que a concentração da quantidade de matéria do ácido clorídrico é igual a 0,020 mol.L-1, a concentração de
íons H+ também será igual a 0,020 mol.L-1, uma vez que a proporção estequiométrica é 1:1.

Cálculo do pH: pH = −log[H + ] = −log(0,020) = −(−1,70) = 1,70


Questão 1175 – Calcule o pH e o pOH de uma solução de ácido clorídrico que apresenta 22% em massa e densidade
igual a 1,078 g.mL-1 a 25°C.

Resolução: Cálculo da concentração da quantidade de matéria do ácido clorídrico, [HCl]:

C 10 x d x (%) 10 x 1,078 x 22 mol


[HCl] = = = = 6,50
< MM >HCl < MM >HCl 36,5 L
H2 O
+
Cálculo da concentração da quantidade de matéria de íons H +, [H+]: ⏞ H(aq)
HCl(aq) → + Cl−
(aq)

+
1 mol de HCl(aq) − − − −1 mol de H(aq)

638
1600 FÍSICO-QUÍMICA APLICADA EXERCÍCIOS COMENTADOS - IME – ITA – OLIMPÍADA

+
6,50 mol. L−1 − − − − − [H(aq) ]
+ mol
[H(aq) ] = 6,50
L

Cálculo do pH: pH = −log[H + ] = − log(6,50) = −0,813

Questão 1176 – Calcule o pH de uma solução que contém 2,70 g de HCN e 0,65 g de KCN por litro de solução.
Informações para a resolução do problema: Ka = 7,0 x 10-10.

Resolução: Equação química: HCN(aq) + KOH(aq) ⇄ KCN(aq) + H2O(l)

[KCN]
Sabendo que o pH de uma solução tampão é representado pela seguinte fórmula: pH = pK a + log [HCN]
0,65
−10 ) 65 𝑥 1,0
pH = −log(7,0 𝑥 10 + log { }
2,70
27 𝑥 1
0,010
pH = −(−9,15) + log
0,10

pH = 9,15 + log(0,10)

pH = 9,15 − 1 = 8,15

Questão 1177 – Uma solução 0,10 mol.L-1 de ácido bromídrico apresenta pressão osmótica igual a 4,70 atm a 22°C.
Calcule o pH da solução ácida e também a constante do ácido.
H2 O
+ −
Resolução: Cálculo do fator de van´t Hoff: HBr(aq) →
⏞ H(aq) + Br(aq)
i = 1 + (n − 1). α
i = 1 + (2 − 1). α = 1 + α
A partir da aplicação da pressão osmótica, temos: π = [HBr] x R x T x i
4,70 = 0,10 x 0,08206 x (22 + 273) x (1 + α)
4,70 = 0,008206 x 295 x (1 + α)
4,70
1+α=
0,008206 x 295
4,70
1+α=
2,42
1 + α = 1,94
α = 0,94
Cálculo do pH: pH = −log[H + ]
pH = −log([HBr] x α) = −log(0,10 x 0,94) = −log(0,094) = 1,03
Questão 1178 – (GRILLO) Dê as expressões matemáticas das constantes do produto de solubilidade para as
seguintes soluções saturadas apresentadas a seguir:
a) Cloreto de prata
b) Cromato de prata
c) Sulfato de estrôncio

639
1600 FÍSICO-QUÍMICA APLICADA EXERCÍCIOS COMENTADOS - IME – ITA – OLIMPÍADA

d) Iodato de cálcio
e) Sulfato de bário
f) Hidróxido de alumínio
g) Azida de chumbo
h) Carbonato de sódio

Resolução:

a) AgCl(s) ⇄ Ag+(aq) + Cl-(aq); K ps = [Ag + ] x [Cl− ]


b) Ag2CrO4(s) ⇄ 2 Ag+(aq) + CrO4-2(aq); K ps = [Ag + ]2 x [Cr2 O−24 ]
−2
c) SrSO4(s) ⇄ Sr (aq) + SO4 (aq); K ps = [Sr ] x [SO4 ]
+2 -2 +2

d) Ca(IO3)2(s) ⇄ Ca+2(aq) + 2 IO3- (aq); K ps = [Ca+2 ] x [IO−3]


2

e) BaSO4(s) ⇄ Ba+2(aq) + SO4-2(aq); K ps = [Ba+2 ] x [SO−2 4 ]


f) Al(OH)3(s) ⇄ Al (aq) + 3 OH (aq); K ps = [Al ] x [OH − ]3
+3 - +3

g) Pb(N3)2(s) ⇄ Pb+2(aq) + 2 N3- (aq); K ps = [Pb+2 ] x [N3− ]2


h) Na2CO3(s) ⇄ 2 Na+(aq) + CO3-(aq); K ps = [Na+ ]2 x [CO−2 3 ]
Questão 1179 - (GRILLO) Sabendo que o produto de solubilidade do sulfato de bário é igual a 1,1 x 10-11, determine
a massa deste sal se for lavado com:
a) 250 mL de água;
b) 350 mL de uma solução aquosa de sulfato de amônio, com concentração da quantidade de matéria igual a
0,75 mol.L-1.

Resolução:
a) Para este problema, será necessário o uso da tabela de equilíbrio químico. Base de cálculo: n (mol) inicial
de sulfato de bário, BaSO4.

BaSO4(s)  Ba+2(aq) SO-24 (aq)


Início n 0 0
Reage // S S
Equilíbrio // S S

Equação do produto de solubilidade (Kps): K ps = [Ba+2 ] x [SO4 −2 ] = S x S = S 2

Cálculo da solubilidade (S): K ps = S² = 1,1 x 10−11

S = √1,1 x 10−11 = 3,32 x 10−6 mol. L−1

Cálculo da massa de sulfato de bário, na presença de 0,250 litros de água: 3,32 x 10−6 mol. L−1 =
mBaSO4
<MM>BaSO4 x Vsolução

mBaSO4 = 3,32 x 10−6 mol. L−1 x 233 x 0,250 = 193,39 x 10


⏟ −6
g = 193,39 μg
μ

Cálculo da massa de sulfato de bário, na presença de sulfato de amônio, (NH4)2SO4: com a presença desta solução
de sulfato de amônio, há a presença do íon comum, que se trata do sulfato.

(NH4)2SO4(s) → 2 NH4+(aq) + SO4-2(aq)

1 mol de (NH4)2SO4(s) ----------- 1 mol de SO4-2(aq)

640
1600 FÍSICO-QUÍMICA APLICADA EXERCÍCIOS COMENTADOS - IME – ITA – OLIMPÍADA

0,75 mol.L-1 ----------------------- [SO4-2]


[SO4-2] = 0,75 mol.L-1

BaSO4(s)  Ba+2(aq) SO-24 (aq)


Início n 0 0
Reage // S S
Equilíbrio // S S + 0,75

Equação do produto de solubilidade (Kps), na presença do íon comum: K ps = [Ba+2 ] x [SO4 −2 ]

K ps = S x ⏟
(S + 0,75)
≅ 0,75

0,75 x S = 1,1 x 10−11

1,1 x 10−11
S= 0,75
= 1,47 x 10−11 mol. L−1.

Cálculo da massa de sulfato de bário, na presença de 0,350 litros de água: 1,47 x 10−11 mol. L−1 =
mBaSO4
<MM>BaSO4 x Vsolução

mBaSO4 = 1,47 x 10−11 mol. L−1 x 233 x 0,350 = 119,88 x 10−11

Questão 1180 – A uma determinada temperatura T, o produto de solubilidade do sulfato de chumbo II é igual a 1,1 x
10-8 e as constantes de equilíbrio para as três equações químicas podem ser observadas a seguir:

a) PbSO4(s) + 2 I-(aq) ⇄ PbI2(s) + SO4-2(aq) K1 = 4,6 x 10-4


b) PbI2(s) + CrO4-(aq) ⇄ PbCrO4(s) + 2 I-(aq) K2 = 4,3 x 1012
c) PbS(s) + CrO4-(aq) ⇄ PbCrO4(s) + S-2(aq) K3 = 7,5 x 10-8

Através dos dados fornecidos acima, determine o valor da constante do produto de solubilidade do sulfeto de chumbo,
além da energia livre Gibbs padrão.

Resolução: PbS(s) ⇄ Pb+2(aq) + S-2(aq) K5 = ?

PbS(s) + CrO4-(aq) ⇄ PbCrO4(s) + S-2(aq) K3 = 7,5 x 10-8

PbSO4(s) ⇄ Pb+2(aq) + SO4-2(aq) K4 = 1,1 x 10-8

PbCrO4(s) + 2 I-(aq) ⇄ PbI2(s) + CrO4-(aq) K2 = 1 / 4,3 x 1012

PbI2(s) + SO4-2(aq) ⇄ PbSO4(s) + 2 I-(aq) K1 = 1/ 4,6 x 10-4 +

PbS(s) ⇄ Pb+2(aq) + S-2(aq) K5 = 7,5 x 10-8 x 1,1 x 10-8 x (1 / 4,3 x 1012) x (1/ 4,6 x 10-4) = 4,17 x 10-25

Cálculo da energia livre de Gibbs padrão (∆G°): ∆G° = −R x T x ln K ps =


J
−8,314 x 298 x ln(4,17 x 10−25 ) = +139082 mol

641
1600 FÍSICO-QUÍMICA APLICADA EXERCÍCIOS COMENTADOS - IME – ITA – OLIMPÍADA

Questão 1181 – Foram dissolvidos 0,002 mol em um litro de solução de nitrato de chumbo II sólido em um litro de
uma solução de ácido sulfúrico, com concentração molar igual a 0,001 mol/L. A partir desta informação, verifique se
vai haver ou não precipitação de sulfato de chumbo. Informação para a resolução do problema: produto de solubilidade
do sulfato de chumbo = 1,3 x 10–8, a 25°C.
Resolução: Para este problema, iremos fazer uma comparação de valores, entre o coeficiente reacional (Qps) com o
valor do produto de solubilidade (Kps).

Cálculo da concentração da quantidade de matéria do sulfato: H2SO4(aq) → 2 H+(aq) + SO4-2(aq)

1 mol de H2SO4(aq) ---------- 1 mol de SO4-2(aq)


0,001 mol/L ------------------- [SO4-2]
[SO4-2] = 0,001 mol.L-1
Cálculo da concentração da quantidade de matéria de chumbo [Pb+2]: Pb(NO3)2(aq) → Pb+2(aq) + 2 NO3- (aq)

1 mol de Pb(NO3)2(aq) ---------- 1 mol de Pb+2(aq)


0,002 mol/L ----------------------- [Pb+2]
[Pb+2] = 0,002 mol.L-1
Cálculo do valor da constante reacional (Q) para o sulfato de chumbo: PbSO4(aq) → Pb+2(aq) + SO4-2 (aq)

Q ps = [Pb+2 ] x [SO4 −2 ] = 0,002 x 0,001 = 2 x 10−6

Comparando os valores do produto reacional com o produto de solubilidade, observa-se que Qps é maior que o Kps
(Qps > Kps). Com isso, há sim precipitação de sulfato de chumbo.

Questão 1182 - Preparando uma solução tampão com um pH = 8,5 foram adicionados em uma solução de ácido
clorídrico cianeto de sódio, com concentração molar igual a 0,01 mol. A partir desta informação, determine o valor do
número de mol do referido sal. Informação para a resolução do problema: Ka(HCN) = 4,1 x 10-10.

Resolução: Adicionando-se x mols de ácido clorídrico à solução de cianeto de sódio, NaCN, teremos o seguinte
equilíbrio iônico colocado a seguir: Na+(aq) + H+(aq) + CN-(aq) + Cl-(aq) → Na+(aq) + Cl-(aq) + HCN.
Base de cálculo: 0,01 mol de cianeto. Tabela de equilíbrio iônico:

H+(aq) CN-(aq)  HCN


Início 0 0,01 0
Reage x x x
Equilíbrio x 0,01 - x x

Considerando que o processo se caracteriza como um tampão, temos: pH = pKa + log ([ânion] / [cátion])

8,5 = - log (4,1 x 10-10) + log (0,01 - x / x)


8,5 = 10 - 0,413 + log (0,01 - x / x)
- 0,887 = log (0,01 - x / x)
x = 8,85 x 10-3 mol

642
1600 FÍSICO-QUÍMICA APLICADA EXERCÍCIOS COMENTADOS - IME – ITA – OLIMPÍADA

Questão 1183 – Considere duas soluções: uma solução aquosa de ácido clorídrico e outra solução de hidróxido de
sódio. Para esta mistura foram utilizados 3,65 gramas o referido ácido e 6,0 gramas de hidróxido em uma quantidade
de água suficiente para um total de um litro de solução. Diante disso, calcule o pH da solução resultante.
Resolução: Equação química: HCl(aq) + NaOH(aq) → NaCl(aq) + H2O(l).
Determinação do reagente limitante e do reagente em excesso:
m 3,65 g
Analisando o HCl: nHCl = <MM>
HCl
= 36,5 g.mol−1 = 0,10 mol
HCl

m 6,0 g
Analisando o NaOH: nNaOH = <MM>
NaOH
= 40,0 g.mol−1 = 0,15 mol
NaOH

Observando o número de mol de cada reagente químico, observa-se que há um excesso de hidróxido de sódio, uma
vez que o número de mol de hidróxido é maior do que o do ácido (nNaOH > nHCl), conforme pode ser observado em
cada número de mol calculado acima.

Cálculo do número de mol em excesso de hidróxido de sódio: nexcesso = nHCl − nNaOH = 0,15 − 0,10 =
0,05 mol

Cálculo da concentração da quantidade de matéria do hidróxido de sódio em excesso, [NaOH]excesso:

NaOH
nNaOH
excesso 0,05 mol
Mexcesso = = = 0,05 mol. L−1
Vsolução 1L

Observando a estequiometria, temos o seguinte valor da concentração molar de íons hidroxila: NaOH(aq) → Na+(aq) +
OH-(aq)

1 mol de NaOH(aq) ------------------- 1mol de OH-(aq)


0,05 mol.L-1 de NaOH(aq) ----------- [OH-]
[OH-] = 0,05 mol.L-1

Cálculo do pOH: pOH = − log[OH − ] = − log(0,05) = 1,30

Cálculo do valor do pH, a partir do pOH a 25°C: pOH = 14 – 1,30 = 12,70


Questão 1184 – Um ácido hipotético do tipo HX(aq) apresenta 0,05 mol dissolvido em água, formando um litro de
solução. A partir da obtenção desta solução específica o pH foi medido e foi igual a 2,00. A partir de todas estas
observações, determine:
a) a soma das concentrações de todos os íons;
b) o valor da constante de ionização (Ka) para o ácido HX.

Resolução: Equação química: HX(aq) + H2O(l)  X-(aq) + H3O+(aq). Base de cálculo: M (mol.L-1) inicial de ácido fórmico,
HCOOH.

HX(aq) H2O(l)  X-(aq) H3O+(aq)


Início M M 0 0
Reage Mα // Mα Mα
Equilíbrio M - Mα // Mα Mα

(Mα) x (Mα) Mα²


Cálculo do grau de dissociação (α), a partir da constante de equilíbrio: K a = (M−Mα)
= 1−α

Considerando que o pH é igual a 2,0, o valor da concentração de íons H3O+(aq) será igual a: 2,0 = −log[H3 O+ ]

643
1600 FÍSICO-QUÍMICA APLICADA EXERCÍCIOS COMENTADOS - IME – ITA – OLIMPÍADA

[H3 O+ ] = 10−2 mol. L−1

Item a) Cálculo da soma dos íons: [H3 O+ ] + [X − ] = 10−2 mol. L−1 + 10−2 mol. L−1 = 2,0 x 10−2 mol. L−1

Item b) Cálculo da constante de equilíbrio (Ka) do ácido HX: Observa-se que no equilíbrio a concentração de [H3 O+ ]
é igual a de [X − ], logo será necessário calcular o grau de dissociação (α). A partir da concentração de H3 O+ , tempo
que: [H3 O+ ] = [X − ] = M. α
10−2 = Mα

0,05
x α = 10−2
1

α = 0,2 (20%)

[X − ]. [H3 O+ ] (Mα) x (Mα) 10−2 x 10−2 10−4


Ka = = = = = 2,50 x 10−3
[HX]. aH2 O (M − Mα) (0,05 − 10−2 ) 0,04

Questão 1185 – A constante do produto de solubilidade do sulfato de chumbo é igual a 2,50 x 10 -8. A partir desta
informação, calcule a solubilidade deste mesmo soluto na presença de:
a) água pura;
b) solução de nitrato de chumbo;
c) na presença de sulfato de sódio.

Resolução:
−2
Item a) Analisando a partir da presença de água pura: PbSO4(s) ⇄ Pb+2
(aq) + 𝑆𝑂4 (aq)

−2
Equação da constante de solubilidade: K ps = [Pb+2
(aq) ] x [𝑆𝑂4 (aq) ]

−2
1 mol PbSO4(s) − − − 1 mol Pb+2
(aq) (S) − − − 1 mol SO4 (aq) (S)

K ps = S x S = S²

mol
S = √2,50 x 10−8 = 1,58 x 10−4
L

Item b) Analisando a partir da presença de nitrato de chumbo:


−2
Sulfato de chumbo: PbSO4(s) ⇄ 𝐏𝐛+𝟐
(𝐚𝐪) + SO4 (aq)

−1
Nitrato de chumbo: Pb(NO3 )2 ⇄ 𝐏𝐛+𝟐
(𝐚𝐪) + 2 NO3 (aq)

Presença do íon comum: 𝐏𝐛+𝟐


(𝐚𝐪)

−2
Equação da constante de solubilidade: K ps = [Pb+2
(aq) ] x [SO4 (aq) ]

−2
1 mol PbSO4(s) − − − 1 mol Pb+2
(aq) (S) − − − 1 mol SO4 (aq) (S)

K ps = ⏟
(S + 0,010) x S
0,010

0,010 x S = 2,50 x 10−8

2,50 x 10−8 mol


S= = 2,50 x 10−6
0,010 L

644
1600 FÍSICO-QUÍMICA APLICADA EXERCÍCIOS COMENTADOS - IME – ITA – OLIMPÍADA

Item c) Analisando a partir da presença de sulfato de sódio:


−𝟐
Sulfato de chumbo: PbSO4(s) ⇄ Pb+2
(aq) + 𝐒𝐎𝟒 (𝐚𝐪)

−𝟐
Nitrato de chumbo: Na2 SO4(aq) ⇄ 2Na+2
(aq) + 𝐒𝐎𝟒 (𝐚𝐪)

Íon comum: SO4+2


−2
Equação da constante de solubilidade: K ps = [Pb+2
(aq) ] x [SO4 (aq) ]

−2
1 mol PbSO4(s) − − − 1 mol Pb+2
(aq) (S) − − − 1 mol SO4 (aq) (S)

K ps = S x ⏟
(S + 0,10)
0,10

0,10 x S = 2,50 𝑥 10−8

2,50 𝑥 10−8 mol


S= = 2,50 𝑥 10−7
0,10 L

Questão 1186 – (GRILLO) Considere que uma solução saturada de iodeto de mercúrio II apresenta uma
concentração da quantidade de matéria de íons mercúrio na ordem de 3,0 x 10-6 mol.L-1 a temperatura experimental
T. A partir desta informação, calcule o valor da constante do produto de solubilidade além também do valor do p Kps.

Resolução: Equação química: HgI2(s) ⇄ Hg +2 −


(aq) + 2I(aq)

Cálculo da concentração da quantidade de matéria de íons iodeto:

1 mol de Hg =2 −
(aq) − − − − − 2 mol de I(aq)
mol
3,0 x 10−6 − − − − − − [I − ]
L
mol
[I − ] = 6,0 x 10−6
L

Expressão e cálculo da constante do produto de solubilidade: K ps = [Hg +2 ] x [I − ]2 =


[3,0 x 10−6 ] x [6,0 x 10−6 ]2 = 1,08 x 10−16

Cálculo do pKps: pKps = −logK ps = −log(1,08 x 10−16 ) = −(−15,97) = + 15,97

Questão 1187 – Sabendo que o sulfato de prata a uma temperatura T apresenta em água uma solubilidade igual a
2,0 x 10-2 mol.L-1, determine o produto de solubilidade desse sal, nesta mesma temperatura T.

Resolução: Ag2SO4(s) ⇄ 2 Ag+(aq) + SO4-2(aq); K ps = [Ag +2 ]² x [SO−2


4 ]

1 mol de Ag2SO4(s) --- 2 mol de Ag+(aq) --- 1 mol de SO4-2(aq)

Relação entre o produto de solubilidade e a solubilidade: K ps = [2𝑆]2 x [S] = 4 x S³

Sabendo que a solubilidade é igual a 2,0 x 10-3 mol.L-1, temos: K ps = 4 x (2,0 x 10−3 )3 = 3,2 x 10−5

645
1600 FÍSICO-QUÍMICA APLICADA EXERCÍCIOS COMENTADOS - IME – ITA – OLIMPÍADA

Questão 1188 – (ITA) Uma solução aquosa saturada em fosfato de estrôncio [Sr3(PO4)2] está em equilíbrio químico
à temperatura de 25°C, e a concentração de equilíbrio do íon estrôncio, nesse sistema, é de 7,50 x 10 –7mol.L–1.
Considerando-se que ambos os reagentes (água e sal inorgânico) são quimicamente puros, assinale a alternativa
correta com o valor do pKps (25°C) do Sr3(PO4)2.
a) 7,0
b) 13,0
c) 25,0
d) 31,0
e) 35,0

Resolução: Alternativa D.
+2
Reação: Sr3 (PO4 )2(s) ⇄ 3Sr(aq) + 2PO−3
4(aq)

Equação da constante do produto de solubilidade: K ps = [Sr +2 ]³ x [PO−2


4 ]²

+2
Determinação da quantidade de matéria de íons fosfato: Sr3 (PO4 )2(s) ⇄ 3Sr(aq) + 2PO−3
4(aq)
+2
3 mol de Sr(aq) − − − − 2 mol de PO−3 4(aq)
7,50 x 10−7 − − − − − [PO−3 4(aq) ]
−3 −7
mol
[PO4(aq) ] = 5,0 x 10
L

Cálculo da constante do produto de solubilidade: K ps = (7,50 x 10−7 )3 x (5,0 x 10−7 )2 = 1,05 x 10−31

Cálculo do pKps: pKps = −logK ps = − log(1,05 x 10−31 ) = 31

Questão 1189 – (OLIMPÍADA DE QUÍMICA DO RIO DE JANEIRO) Kps para o iodato de chumbo II, Pb(IO3)2 é 3,2 x
10-14 a uma dada temperatura. Qual a solubilidade de Pb(IO3)2 em mol.L-1?
a) 1,8 x 10-7
b) 3,6 x 10-7
c) 2,0 x 10-5
d) 4,0 x 10-5
e) 8,0 x 10-5

Resolução: Alternativa C.

Fórmula molecular do iodato de chumbo II: Pb(IO3 )2(aq)

Equação química: Pb(IO3 )2(s) ⇄ Pb+2 −


(aq) + 2 IO3(aq)

Expressão da constante do produto de solubilidade: K ps = [Pb+2 ] x [IO−


3]
2

Relação da constante de produto de solubilidade com a solubilidade: K ps = S x (2S)2 = 4S³

3,2 𝑥 10−14
= S³
4

3 mol
S = √8,0 x 10−15 = 2 x 10−5
L

646
1600 FÍSICO-QUÍMICA APLICADA EXERCÍCIOS COMENTADOS - IME – ITA – OLIMPÍADA

Questão 1190 - (OLIMPÍADA DE QUÍMICA DO RIO DE JANEIRO) Nas estações de tratamento de água comumente
provoca-se a formação de flocos de hidróxido de alumínio para arrastar partículas em suspensão. Suponha que o
hidróxido de alumínio seja substituído pelo hidróxido férrico. Qual a menor concentração de íons Fe +3, em mol.L-1,
necessária para provocar a precipitação da base, numa solução que contém 1,0 x 10 -8 mol.L-1 de íons OH-? Dado:
produto de solubilidade do Fe(OH)3 = 4,0 x 10-38.
a) 2,0 x 10-16
b) 2,0 x 10-14
c) 4,0 x 10-14
d) 4,0 x 10-16
e) 2,0 x 10-18

Resolução: Alternativa C.
Cálculo da concentração da quantidade de matéria de íons [Fe+3], a partir da constante do produto de solubilidade do
hidróxido férrico ou também conhecido como hidróxido de ferro III:

Equação química: Fe(OH)3(s) ⇄ Fe+3 −


(aq) + 3 OH(aq)

Expressão da constante do produto de solubilidade: K ps = [Fe+3 ] x [OH − ]3

Cálculo da concentração da quantidade de matéria de íons Fe+3: K ps = [Fe+3 ] x [OH − ]3

K ps 4,0 x 10−38 4,0 x 10−38 mol


[Fe+3 ] = − 3
= −8 3
= −24
= 4,0 x 10−14
[OH ] (1,0 x 10 ) 1,0 x 10 L
Questão 1191 – (IME) O grau de hidrólise do NaCN, em solução 0,2 N, é 0,85% a 25°C. Calcule a constante de
ionização do HCN a 25°C.

Resolução:

Equação química (1): NaCN(aq) → Na+ −


(aq) + CN(aq)

Equação química (2): Na+ − +


(aq) + CN(aq) + H2 O(l) ⇌ HCN(aq) + Na (aq) + OH −
⏟ (aq)
solução básica
KW
Equação química da constante de hidrólise: K h = Ka

KW
[NaCN] x α² =
Ka
KW 10−14 10−14
Ka = = = = 6,925 x 10−10
[NaCN] x α² 0,20 x (0,0085)2 1,445 x 10−5

Questão 1192 – (IME) Calcule o pH de uma solução 0,100 mol.L-1 de um ácido monoprótico, sabendo que sua
constante de acidez é 1,00 x 10-11.
+ −
Resolução: Ácido monoprótico: HX(aq). Equação química: HX (aq) → H(aq) + X (aq)
Mα x Mα Mα²
Cálculo do grau de dissociação, a partir da equação da constante de equilíbrio do ácido: K a = M−Mα
= 1−α

Levando em consideração que o ácido hipotético seja fraco, alfa tende a zero (α → 0), logo 1 − α → 1

Com isso, a equação da constante de equilíbrio do ácido fica da seguinte forma: K a = Mα2

Substituindo os valores na constante de equilíbrio, temos: 10−11 = 0,10 x α2

647
1600 FÍSICO-QUÍMICA APLICADA EXERCÍCIOS COMENTADOS - IME – ITA – OLIMPÍADA

α = 10−5

Sabendo que no equilíbrio a concentração de íons hidrogeniônicos é igual a 𝑀α, então: [H + ] = M x α =


mol
0,10 x 10−5 = 10−6 L

Cálculo do pH: pH = − log[H + ] = 6

Questão 1193 – (IME) Juntam-se 90 mL de solução 0,10 mol x L-1 de hidróxido de amônio com 10 mL de solução 010
mol x L-1 de ácido clorídrico. Determinar o pH da solução obtida.

Resolução: Equação química: HCl(aq)+ NH4(OH) → NH4Cl(aq) + H2O(l)

Cálculo do número de mol para cada reagente químico:

nácido = 0,10 x 10 x 10−3 = 1,0 x 10−3 mol

nbase = 0,10 x 90 x 10−3 = 9,0 x 10−3 mol

Observando o número de mol de cada reagente químico, como o número de mol da base é maior em comparação ao
número de mol do ácido, a solução final apresentará característica alcalina.

nexcesso
base = 9,0 x 10−3 mol − 1,0 x 10−3 mol = 8,0 x 10−3 mol

Vtotal = 10 x 10−3 L + 90 x 10−3 L = 100 x 10−3 L

8,0 x 10−3 mol mol


Cálculo da concentração da quantidade de matéria dos íons hidroxilas: [OH − ] = 100 x 10−3 L
= 8,0 x 10−2 L

pOH = − log(8,0 x 10−3 ) = 1,10

pH = 14 − pOH = 14 − 1,10 = 12,90

Questão 1194 – Em que pH começa a precipitar o hidróxido de magnésio, se a concentração de Mg +2 na solução é


0,010 mol X L-1. Dado: Mg(OH)2(s) ⇄ Mg+2(aq) + 2 OH-(aq), Kps = 8,90 x 10-12.

Resolução: Cálculo da concentração hidroxiliônica, a partir do produto de solubilidade: K ps = [Mg +2 ] x [OH − ]2

8,90 𝑥 10−12 = 10−2 x [OH − ]2

8,90 𝑥 10−12 mol


[OH − ] = √ = 2,98 𝑥 10−5
10−2 L

Cálculo do potencial hidroxiliônico: pOH = − log[OH − ] = − log[2,98 x 10−5 ] = 4,52

pH = 14 − pOH = 14 − 4,52 = 9,48

Questão 1195 – Quantos miligramas de íons Mn+2 permanecem em 100 mL de uma solução de pH igual a 8,60 sem
que haja a precipitação do hidróxido de manganês? Dado: Mn(OH)2(s) ⇄ Mn+2(aq) + 2 OH-(aq), Kps = 1,90 x 10-13.
Resolução: Equação da constante de equilíbrio do produto de solubilidade: K ps = [Mn+2 ] x [OH − ]2

Cálculo do pOH e da concentração da quantidade de matéria de hidroxila à 25°C: pOH = 14 − 8,60 = 5,40

5,40 = − log[OH − ]

648
1600 FÍSICO-QUÍMICA APLICADA EXERCÍCIOS COMENTADOS - IME – ITA – OLIMPÍADA

mol
[OH − ] = 10−5,40 = 1,99 x 10−6
L

Pela relação estequiométrica: 1 mol de Mn+2 − − − − − 2 mol de OH −

1 mol de Mn+2 − − − − − 2 mol de OH −


mol
[Mn+2 ] − − − − − − − − 3,98 x 10−6 de OH −
L
3,98 x 10−6 mol
[Mn+2 ] = = 1,99 x 10−6 de Mn+2
2 L
mMn+2
Cálculo da massa de cátions manganês: <MM> x Vsolução
= 1,99 x 10−6
Mn+2

mMn+2 = 1,99 x 10−6 𝑥 54,90 𝑥 0,10 ≅ 0,011 mg

Questão 1196 – (IME) O equilíbrio da solubilidade para o composto hipotético RQ2 em água é dado pela seguinte
equação: RQ 2 → R++ + 2Q− . O produto de solubilidade à temperatura ambiente é 3,2 x 10-11. Calcule o número de
mols de RQ2 que se pode dissolver em 50 mL de água pura.
Resolução: Equação química: RQ 2 → R++ + 2Q−
1 mol de RQ 2 − − − 1 mol de R++ − − − 2 mol de Q−
Equação do produto de solubilidade: K ps = [R++ ] x [Q− ]2

Realação do produto de solubilidade com a solubilidade: K ps = [S] x [2S]2 = 4S³

3,2 𝑥 10−11 = 4S³

3 3,2 𝑥 10−11 mol


S= √ = 2,0 𝑥 10−4
4 L
nRQ2
Cálculo do número de mol de RQ2, a partir da solubilidade: 2,0 𝑥 10−4 =
50 x 10−3

nRQ2 = 2,0 𝑥 10−4 𝑥 50 x 10−3 = 1,0 x 10−5 mol

Questão 1197 – (MESTRE JOÃO ROBERTO DA PACIÊNCIA NABUCO) O grau de hidrólise do cianeto de potássio
em solução 0,1 mol.L-1 é 1,12%. Calcular o pH da solução. Dado: log (1,12) = 0,05.

Resolução: Analisando A solução de cianeto de potássio: KCN(aq) ⇌ K + −


(aq) + CN(aq)

K+ −
(aq) + CN(aq) + H2 O(l) ⇌ KOH
⏟ (aq) + HCN(aq)
Base forte

K+ − + −
(aq) + CN(aq) + H2 O(l) ⇌ K (aq) + OH(aq) + HCN(aq)

− −
CN(aq) + H2 O(l) ⇌ HCN(aq) + OH
⏟ (aq)
solução básica

A solução de cianteo de potássio é alcalina.

− mol
[OH(aq) ] = 0,10 x 0,0112 = 1,12 x 10−3
L
Cálculo do pOH: pOH = −log [1,12 x 10−3 ] = 2,95

649
1600 FÍSICO-QUÍMICA APLICADA EXERCÍCIOS COMENTADOS - IME – ITA – OLIMPÍADA

pH = 14 − pOH = 14 − 2,95 = 11,05


Questão 1198 – (MESTRE JOÃO ROBERTO DA PACIÊNCIA NABUCO) A constante de hidrólise do cloreto de
amônio 0,2 mol.L-1 é 5,0 x 10-8. Calcule Kb do hidróxido de amônio.
+
Resolução: Analisando o cloreto de amônio: NH4 Cl(aq) ⇌ NH4(aq) + Cl−
(aq)
+
NH4(aq) + Cl−
(aq) + H O
2 (l) ⇌ NH 4 OH(aq) + HCl
⏟ (aq)
ácido forte

+
NH4(aq) + Cl− + −
(aq) + H2 O(l) ⇌ NH4 OH(aq) + H(aq) + Cl(aq)

+ +
NH4(aq) + H2 O(l) ⇌ NH4 OH(aq) + H
⏟(aq)
solução ácida
KW
Kh =
Kb

10−14
5,0 𝑥 10−8 =
Kb

10−14
Kb = = 2,0 𝑥 10−7
5,0 𝑥 10−8

Questão 1199 – (OLIMPÍADA DE QUÍMICA DO RIO DE JANEIRO) Qual das seguintes substâncias, a ser adicionada
em água, não irá alterar o pH?
a) NaHCO3
b) NH4Cl
c) KCN
d) KCl
e) CO2
Resolução: Alternativa D.

a) Analisando a solução de bicarbonato de sódio: NaHCO3 (aq) ⇌ Na+ −


(aq) + HCO3(aq)

Na+ −
(aq) + HCO3(aq) + H2 O(l) ⇌ NaOH(aq) + ⏟
H2 CO3(aq)
ácido fraco

Na+ − + −
(aq) + HCO3(aq) + H2 O(l) ⇌ Na (aq) + OH(aq) + H
⏟2 CO3(aq)
ácido fraco

HCO−
3(aq) + H2 O(l) ⇌ OH −
⏟ (aq) +H
⏟2 CO3(aq)
solução alcalina ácido fraco

+
b) Analisando a solução de cloreto de amônio: NH4 Cl(aq) ⇌ NH4(aq) + Cl−
(aq)

+
NH4(aq) + Cl−
(aq) + H2 O(l) ⇌ NH4 OH(aq) + HCl
⏟ (aq)
ácido forte

+
NH4(aq) + Cl− + −
(aq) + H2 O(l) ⇌ NH4 OH(aq) + H(aq) + Cl(aq)

+ +
NH4(aq) + H2 O(l) ⇌ NH4 OH(aq) + H
⏟(aq)
solução ácida

650
1600 FÍSICO-QUÍMICA APLICADA EXERCÍCIOS COMENTADOS - IME – ITA – OLIMPÍADA

c) Analisando a solução de cianeto de potássio: KCN(aq) ⇌ K + −


(aq) + CN(aq)

K+ −
(aq) + CN(aq) + H2 O(l) ⇌ KOH
⏟ (aq) + HCN(aq)
Base forte

K+ − + −
(aq) + CN(aq) + H2 O(l) ⇌ K (aq) + OH(aq) + HCN(aq)

− −
CN(aq) + H2 O(l) ⇌ HCN(aq) + OH
⏟ (aq)
solução básica

d) Analisando a solução de cloreto de potássio: KCl(aq) ⇌ K + −


(aq) + Cl(aq)

K+ −
(aq) + Cl(aq) + H2 O(l) ⇌ KOH
⏟ (aq) + HCl
⏟ (aq)
base forte ácido forte

K+ − + − + −
(aq) + Cl(aq) + H2 O(l) ⇌ K (aq) + OH(aq) + H(aq) + Cl(aq)

− +
H2 O(l) ⇌ OH(aq) + H(aq)

e) Analisando o dióxido de carbono: Reagindo este composto com a água, resulta na formação de um ácido (H2CO3).
CO2 (g) + H2 O(l) → H
⏟2 CO3(aq)
ácido carbônico
(ácido fraco)

Questão 1200 – (ITA) Numa solução aquosa 0,100 mol.L-1 de um ácido monocarboxílico, a 25°C, o ácido está 3,7%
dissociado após o equilíbrio ter sido atingido. Assinale a opção que contém o valor correto da constante de dissociação
desse ácido nesta temperatura.
a) 1,4
b) 1,4 x 10–3
c) 1,4 x 10–4
d) 3,7 x 10–2
e) 3,7 x 10–4

Resolução: Alternativa C.
M x α²
Sabendo que a constante do ácido é dado por: K a = 1−α
. Substituindo os valores na equação da constante do
ácido, temos:
0,10 x (0,037)²
Ka = = 1,42 x 10−4
1 − 0,037
Como o ácido é fraco, a equação da constante dos ácidos se reduz a K a = M x α2 . Realizando o cálculo mediante
a esta última equação:
K a = 0,10 x (0,037)2 = 1,37 x 10−4 (≅ 1,40 x 10−4 )
Questão 1201 – (ITA) Sabendo que a solubilidade em água a 0°C do sulfato de prata (Ag2SO4) é 1,8 x 10-2 mol x L-1,
podemos concluir que o produto de solubilidade desse sal, nesta temperatura vale:
a) 1,6 x 10-4
b) 6,5 x 10-4
c) 6,0 x 10-4
d) 1,8 x 10-2
e) 5,4 x 10-2

651
1600 FÍSICO-QUÍMICA APLICADA EXERCÍCIOS COMENTADOS - IME – ITA – OLIMPÍADA

Resolução: Questão anulada por não apresentar a alternativa correta.

Equação química: Ag 2 SO4(s) ⇌ 2 Ag + −2


(aq) + SO4(aq)

1 mol de Ag 2 SO4(s) − − − 2 mol de Ag + −2


(aq) − − − 1 mol de SO4(aq)

Equação do produto de solubilidade: K ps = [Ag + ]² x [SO−2


4(aq) ]

Relação do produto de solubilidade com a solubilidade: K ps = [2S]2 x [S] = 4S 3

K ps = [2S]2 x [S] = 4 x (1,80 𝑥 10−2 )3 = 2,33 𝑥 10−5

Questão 1202 – (ITA) Sabendo que o produto de solubilidade do cloreto de prata vale 1,80 x 10-10 pode-se dizer que
a solubilidade desse sal em água é:
a) 3,24 x 10-20 mol x L-1
b) 0,90 x 10-10 mol x L-1
c) 1,80 x 10-10 mol x L-1
d) 3,60 x 10-10 mol x L-1
e) 1,34 x 10-5 mol x L-1

Resolução: Alternativa E.

Equação química: AgCl(s) ⇌ Ag + −


(aq) + Cl(aq)

1 mol de Ag 2 SO4(s) − − − 1 mol de Ag + −


(aq) − − − 1 mol de Cl(aq)

Equação do produto de solubilidade: K ps = [Ag + ] x [Cl−


(aq) ]

Realação do produto de solubilidade com a solubilidade: 1,80 𝑥 10−10 = S x S


1,80 𝑥 10−10 = S²

mol
S = √1,80 x 10−10 = 1,34 x 10−5
L

Questão 1203 – (ITA) Dentre as substâncias abaixo assinale aquela que, quando dissolvida em água, produz solução
alcalina.
a) C2H5OH
b) NaCl
c) CH3COONa
d) NH4Cl
e) KNO3

Resolução: Alternativa C.

Analisando o cloreto de sódio: NaCl(aq) ⇌ Na+ −


(aq) + Cl(aq)

Na+ −
(aq) + Cl(aq) + H2 O(l) ⇌ NaOH
⏟ (aq) + HCl
⏟ (aq)
base forte ácido forte

Na+ − + − + −
(aq) + Cl(aq) + H2 O(l) ⇌ Na (aq) + OH(aq) + H(aq) + Cl(aq)

− +
H2 O(l) ⇌ OH(aq) + H(aq)

Analisando o acetato de sódio: CH3 COONa(aq) ⇌ CH3 COO− +


(aq) + Na (aq)

652
1600 FÍSICO-QUÍMICA APLICADA EXERCÍCIOS COMENTADOS - IME – ITA – OLIMPÍADA

CH3 COO− +
(aq) + Na(aq) + H2 O(l) ⇌ CH
⏟ 3 COOH(aq) + NaOH
⏟ (aq)
ácido fraco ácido forte

CH3 COO− + +
(aq) + Na(aq) + H2 O(l) ⇌ CH3 COOH(aq) + Na (aq) + OH −
⏟ (aq)
solução básica

CH3 COO−
(aq) + H2 O(l) ⇌ CH3 COOH(aq) + OH −
⏟ (aq)
solução básica

+
Analisando o cloreto de amônio: NH4 Cl(aq) ⇌ NH4(aq) + Cl−
(aq)

+
NH4(aq) + Cl−
(aq) + H2 O(l) ⇌ NH4 OH(aq) + HCl
⏟ (aq)
ácido forte

+
NH4(aq) + Cl− + −
(aq) + H2 O(l) ⇌ NH4 OH(aq) + H(aq) + Cl(aq)

+ +
NH4(aq) + H2 O(l) ⇌ NH4 OH(aq) + H(aq)

solução ácida

Analisando o nitrato de potássio: KNO3 (aq) ⇌ K + −


(aq) + NO3(aq)

K+ −
(aq) + NO3(aq) + H2 O(l) ⇌ KOH
⏟ (aq) + HNO
⏟ 3 (aq)
base forte ácido forte

K+ − + − + −
(aq) + NO3(aq) + H2 O(l) ⇌ K (aq) + NO3(aq) + H(aq) + OH(aq)

+ −
H
⏟2 O(l) ⇌ H(aq) + OH(aq)
solução neutra

Questão 1204 – (ITA) A 25ºC, adiciona-se 1,0 mL de uma solução aquosa 0,10 mol/L em HCl a 100 mL de uma
solução aquosa 1,0 mol/L em HCl. O pH da mistura final é:
a) 0
b) 1
c) 2
d) 3
e) 4

Resolução: Alternativa A.
Cálculo da concentração da quantidade de matéria final da solução de ácido clorídrico: [HCl]1 x V1 + [HCl]2 x V2 =
[HCl]final x Vfinal

0,10 x 1,0 + 1,0 x 100 = [HCl]final x (1 + 100)


100,1
[HCl]final = = 0,991 ≅ 1,0
101
+
HCl(aq) → H(aq) + Cl−
(aq)

+ mol
[HCl] = [H(aq) ] = 1,0
L
Cálculo do pH: pH = − log[H + ] = − log(1) = 0

653
1600 FÍSICO-QUÍMICA APLICADA EXERCÍCIOS COMENTADOS - IME – ITA – OLIMPÍADA

Questão 1205 – (ITA - 1987) Uma solução 0,005 mol.L-1 de hidróxido de bário em água à temperatura ambiente, terá
pH aproximadamente igual a:
a) 0,01
b) 2
c) 5
d) 9
e) 12

Resolução: Alternativa E.
Equação química: Ba(OH)2(aq) → Ba+2(aq) + 2 OH-(aq)
1 mol de Ba(OH)2 ---------------- 2 mol de OH-(aq)
0,005 mol.L-1 ---------------------- [OH-]
[OH-] = 0,01 mol.L-1
Cálculo do pOH e do pH: pOH = -log[OH-] = - log (10-2) = 2. À 25°C: pH + pOH = 14
pH = 14 – 2 = 12
Questão 1206 – (ITA) A respeito das substâncias CO2, (NH4)2SO4 e CH3(CH2)10COONa são feitas as afirmações:
I. O pH de uma amostra de água não é alterado pela dissolução de Na2SO4.
II. A dissolução de CO2 e de (NH4)2SO4 em água produz soluções ácidas.
III. A dissolução de CH3(CH2)10COONa em água produz solução alcalina.
Das afirmações feitas está(ão) certas:
a) apenas I
b) apenas II
c) apenas III
d) apenas I e II
e) Todas

Resolução: Alternativa E.

Analisando a afirmativa I) Na2 SO4(aq) → 2Na+ −2


(aq) + SO4(aq)

2Na+ −2
(aq) + SO4(aq) + 2H2 O(l) → 2NaOH(aq) + H2 SO4(aq)

2Na+ −2 + − + −2
(aq) + SO4(aq) + 2H2 O(l) → 2Na (aq) + 2OH(aq) + 2H(aq) + SO4(aq)

− +
2H
⏟ 2 O(l) → 2OH(aq) + 2H(aq)
solução neutra

Analisando a afirmativa II) CO


⏟ 2(g) + H2 O(l) → H
⏟2 CO3(aq)
óxido ácido ácido carbônico

+
(NH4 )2 SO4(aq) ⇌ 2NH4(aq) + SO−2
4(aq)

+
2NH4(aq) + SO−2
4(aq) + H2 O(l) → 2 NH4 OH(𝑎𝑞) + H2 SO4(aq)

+
2NH4(aq) + SO−2 + −2
4(aq) + H2 O(l) → 2 NH4 OH(𝑎𝑞) + 2H(aq) + SO4(aq)

+ +
2NH4(aq) + H2 O(l) → 2 NH4 OH(𝑎𝑞) + 2H
⏟ (aq)
solução ácida

654
1600 FÍSICO-QUÍMICA APLICADA EXERCÍCIOS COMENTADOS - IME – ITA – OLIMPÍADA

Analisando a afirmativa III) CH3 (CH2 )10 COONa(aq) ⇌ Na+ −


(aq) + CH3 (CH2 )10 COO(aq)

Na+ −
(aq) + CH3 (CH2 )10 COO(aq) + H2 O(l) → CH3 (CH2 )10 COOH(aq) + NaOH(aq)

Na+ − + −
(aq) + CH3 (CH2 )10 COO(aq) + H2 O(l) → CH3 (CH2 )10 COOH(aq) + Na (aq) + OH(aq)

CH3 (CH2 )10 COO−


(aq) + H2 O(l) → CH3 (CH2 )10 COOH(aq) + OH −
⏟ (aq)
solução básica

Questão 1207 – (ITA) Considere as substâncias:


I. CH3COOK
II. NH3
III. KNO3
IV. KHCO3
V. KHSO4
VI. K2CO3
Quais são as que produzem soluções alcalinas (pH > 7), quando dissolvidos em água?
a) Apenas II e VI
b) Apenas II, IV, V e VI
c) Apenas I, II, III e IV
d) Apenas I, II, IV e VI
e) Apenas I, III, IV e V

Resolução: Alternativa D.

Analisando o composto I) Acetato de potássio: CH3 COOK (aq) ⇌ K + −


(aq) + CH3 COO(aq)

K+ −
(aq) + CH3 COO(aq) + H2 O(l) → CH3 COOH(aq) + KOH(aq)

K+ − + −
(aq) + CH3 COO(aq) + H2 O(l) → CH3 COOH(aq) + K (aq) + OH(aq)

CH3 COO−
(aq) + H2 O(l) → CH3 COOH(aq) + OH −
⏟ (aq)
solução básica

Analisando o composto II) Amônia: NH3 (g) + H2 O(l) → NH4 OH(aq)


Analisando o composto III) Nitrato de potássio: KNO3(aq) → K +
(aq) + NO3 (aq)


K+
(aq) + NO3 (aq) + H2 O(l) → KOH(𝑎𝑞) + HNO3(aq)

− −
K+ + − +
(aq) + NO3 (aq) + H2 O(l) → K (aq) + OH(aq) + H(aq) + NO3 (aq)

− +
2H
⏟ 2 O(l) → 2OH(aq) + 2H(aq)
solução neutra


Analisando o composto IV) Bicarbonato de potássio: KHCO3(aq) → K +
(aq) + HCO3 (aq)


K+
(aq) + HCO3 (aq) + H2 O(l) → KOH(𝑎𝑞) + H2 CO3(aq)


K+ + −
(aq) + HCO3 (aq) + H2 O(l) → K (aq) + OH(aq) + H2 CO3(aq)

655
1600 FÍSICO-QUÍMICA APLICADA EXERCÍCIOS COMENTADOS - IME – ITA – OLIMPÍADA

HCO3 −
(aq) + H2 O(l) → OH −
⏟ (aq) + H2 CO3(aq)
solução básica


Analisando o composto V) Bissulfato de potássio: KHSO4(aq) → K +
(aq) + HSO4 (aq)


K+
(aq) + HSO4 (aq) + H2 O(l) → KOH(𝑎𝑞) + H2 SO4(aq)


K+ + − + −
(aq) + HSO4 (aq) + H2 O(l) → K (aq) + OH(aq) + H(aq) + HSO4(aq)

− +
H2 O(l) → OH(aq)
⏟ + H(aq)
solução neutra

Analisando o composto VI) Carbonato de potássio: K 2 CO3(aq) → 2K + −2


(aq) + CO4(aq)

2K + −2
(aq) + CO3(aq) + 2H2 O(l) → 2KOH(aq) + H2 CO3(aq)

2K + −2 + −
(aq) + CO3(aq) + 2H2 O(l) → 2K (aq) + 2OH(aq) + H2 CO3(aq)

CO−2
3(aq) + 2H2 O(l) →

2OH(aq)
⏟ + H2 CO3(aq)
solução básica

Questão 1208 – (ITA) Numa solução de acetato de sódio há:


a) mais íons H+ do que OH-
b) mais íons OH- do que íons H+
c) o mesmo número de íons H+ e íons OH-
d) não há íons H+ nem íons OH-
e) o mesmo número de íons Na+ e íons CH3COO-

Resolução: Alternativa B.

Analisando a hidrólise do acetato de sódio: CH3 COONa(aq) ⇌ CH3 COO− +


(aq) + Na (aq)

CH3 COO− +
(aq) + Na(aq) + H2 O(l) ⇌ CH
⏟ 3 COOH(aq) + NaOH
⏟ (aq)
ácido fraco ácido forte

CH3 COO− + +
(aq) + Na(aq) + H2 O(l) ⇌ CH3 COOH(aq) + Na (aq) + OH −
⏟ (aq)
solução básica

CH3 COO−
(aq) + H2 O(l) ⇌ CH3 COOH(aq) + OH −
⏟ (aq)
solução básica

Questão 1209 – (ITA) Numa solução de cloreto de amônio há:


a) mais íons H+ do que OH-
b) mais íons OH- do que íons H+
c) o mesmo número de íons H+ e íons OH-
d) não há íons H+ nem íons OH-
e) o mesmo número de íons NH4+ e íons Cl-

Resolução: Alternativa A.
+
Analisando a hidrólise do cloreto de amônio: NH4 Cl(aq) ⇌ NH4(aq) + Cl−
(aq)

656
1600 FÍSICO-QUÍMICA APLICADA EXERCÍCIOS COMENTADOS - IME – ITA – OLIMPÍADA

+
NH4(aq) + Cl−
(aq) + H2 O(l) ⇌ NH4 OH(aq) + HCl
⏟ (aq)
ácido forte

+
NH4(aq) + Cl− + −
(aq) + H2 O(l) ⇌ NH4 OH(aq) + H(aq) + Cl(aq)

+ +
NH4(aq) + H2 O(l) ⇌ NH4 OH(aq) + H
⏟(aq)
solução ácida

Questão 1210 – (ITA) Em relação às soluções aquosas de cada um dos seguintes sais: NH4Cl, KNO3, CuSO4, fez-se
a seguinte afirmação constituída de três partes. “As três soluções:

I. apresentam pH menor do que 7


II. porque esses sais derivam de ácidos fortes e
III. porque esses sais derivam de bases fracas.”

a) as três partes da afirmação estão certas


b) somente a parte II está certa
c) as três partes estão erradas
d) somente a parte I está certa
e) somente a parte I está errada

Resolução: Alternativa B.

Questão 1211 – (IME) A ciência procura reunir semelhantes em classes ou grupos, com objetivo de facilitar
metodologicamente o estudo de tais entes. Na química, uma classificação inicial ocorreu em meados do século XVIII
e dividiu as substâncias em orgânicas e inorgânicas ou minerais. Abaixo, são apresentadas correlações de nomes,
fórmulas e classificações de algumas substâncias inorgânicas.
Correlação Nome da substância Fórmula Classificação
I Carbonato ácido de potássio KHCO3 Sal de hidrólise ácida
II Óxido de alumínio Al2O3 Óxido anfótero
III Cianeto de sódio NaCN Sal de hidrólise básica
IV Óxido de cálcio CaO Óxido básico
V Hidróxido estanoso Sn(OH)4 Base de Arrhenius
Assinale a alternativa na qual ambas as correlações são falsas.
a) IeV
b) II e III
c) III e V
d) I e III
e) II e IV

Resolução: Alternativa A.

Analisando a correlação I) KHCO3 (aq) ⇌ K + −


(aq) + HCO3(aq)

K+ −
(aq) + HCO3(aq) + H2 O(l) ⇌ KOH(aq) + H
⏟2 CO3(aq)
ácido fraco

K+ − + −
(aq) + HCO3(aq) + H2 O(l) ⇌ K (aq) + OH(aq) + H
⏟2 CO3(aq)
ácido fraco

HCO−
3(aq) + H2 O(l) ⇌ OH −
⏟ (aq) +⏟
H2 CO3(aq)
solução básica ácido fraco

657
1600 FÍSICO-QUÍMICA APLICADA EXERCÍCIOS COMENTADOS - IME – ITA – OLIMPÍADA

Analisando a correlação II) Verdadeiro



Analisando a correlação III) NaCN(aq) ⇌ Na(aq) + CN(aq)

Na+ − +
(aq) + CN(aq) + H2 O(l) ⇌ HCN(aq) + Na (aq) + OH −
⏟ (aq)
solução básica

− −
CN(aq) + H2 O(l) ⇌ HCN(aq) + OH
⏟ (aq)
solução básica

Analisando a correlação IV) Verdadeiro

Analisando a correlação V) Falso. A definição desta base segundo Arrhenius está correta, mas a nomenclatura está
incorreta. Nomenclatura correta: Hidróxido estânico.

Questão 1212 – (IME) Calcule a concentração de uma solução aquosa de ácido acético cujo pH é 3,00 sabendo que
a constante de dissociação do ácido é 1,75 x 10-5.

Resolução: Fórmula molecular do ácido acético: C2H4O2.



Equação química: HAc(aq) + H2 O(l) → Ac(aq) + H3 O+
(aq)

Base de cálculo: n(mol) inicial de ácido acético.

Tabela de equilíbrio químico:

HAc(aq) H2 O(l) ⇄ A−(aq) H3 O+


(aq)
Início n - 0 0
Reage nα - nα nα
n n n n
Equilíbrio − α - α α
𝑉 𝑉 𝑉 𝑉

[Ac− +
(aq) ] x [H3 O ]
Expressão da constante de equilíbrio do ácido: K a =
[HAc(aq) ] 𝑥 𝑎H2O(l)

1
Mα x Mα
Ka = = 1,75 𝑥 10−5
M − Mα

Cálculo da concentração de [H3 O+ +


(aq) ]: pH = −log[H3 O(aq) ]

3 = −log[H3 O+
(aq) ]

𝑚𝑜𝑙
[H3 O+
(aq) ] = 10
−3
𝐿

No equilíbrio a concentração de hidrônio é igual a Mα. Substituindo os valores na equação da constante de equilíbrio
do ácido acético, temos:

10−3 x 10−3
M−10−3
= 1,75 𝑥 10−5 .

Com isso, a concentração da quantidade de matéria (M) do ácido acético será igual a 5,82 x 10-3 mol x L-1.

658
1600 FÍSICO-QUÍMICA APLICADA EXERCÍCIOS COMENTADOS - IME – ITA – OLIMPÍADA

Questão 1213 – (U. S. NATIONAL CHEMISTRY OLYMPIAD) Calcium hydroxide is slightly soluble in water with a
Ksp of 1.3×10–6 . What is the pH of a saturated solution of calcium hydroxide at 25°C? (A) 12,34 (B) 12,14 (C) 12,04
(D) 11,84

Resolução: Alternativa B.

Hidróxido de cálcio: Ca(OH)2(aq) → Ca+2 −


(aq) + 2 OH(aq)

1 𝑚𝑜𝑙 𝑑𝑒 Ca(OH)2(aq) − − − 1 mol de Ca+2 −


(aq) − − − 2 mol de OH(aq)
1 𝑚𝑜𝑙 𝑑𝑒 Ca(OH)2(aq) − − − S − − − − − − − − − 2S

Relação matemática entre o produto de solubilidade e a solubilidade: K ps = [Ca+2 ] x [OH − ]2 = S x (2S)2 = 4S³

Cálculo da solubilidade do hidróxido de cálcio: 1,30 x 10−6 = 4S³

𝑚𝑜𝑙
S = 6,87 x 10−3
𝐿

Cálculo da concentração da quantidade de matéria da hidroxila, [OH − ]:

1 𝑚𝑜𝑙 𝑑𝑒 Ca(OH)2(aq) − − − 1 mol de Ca+2 −


(aq) − − − − 2 mol de OH(aq)
mol
6,87 x 10−3 − − − − − [OH − ]
L
mol
[OH − ] = 1,37 x 10−2
L

Cálculo do pOH: pOH = −log[OH − ] = − log(1,37 x 10−2 ) = 1,86

Cálculo do pH: pH = 14 − pOH = 14 − 1,86 = 12,14

Questão 1214 – (U. S. NATIONAL CHEMISTRY OLYMPIAD) What is the percent ionization of a 0.010 M HCN
solution? (Ka = 6,2 × 10–10) (A) 0,0025% (B) 0,025% (C) 0,25% (D) 2,5%

Resolução: Alternativa B.
+ −
Ácido cianídrico (ácido fraco): HCN(aq) → H(aq) + CN(aq)

[HCN] x α²
Cálculo do grau de dissociação, a partir da constante de equilíbrio do ácido: K a = . Como o ácido cianídrico
1−α
é um ácido fraco, a equação da constante de equilíbrio em função do ácido se reduz para:

K a = [HCN] x α²

Ka 6,2 x 10−10
α2 = = = 6,2 x 10−8
[HCN] 0,010

α = √6,2 x 10−8 = 2,50 𝑥 10−4 (0,025%)

Questão 1215 – (U. S. NATIONAL CHEMISTRY OLYMPIAD) The pH of a saturated solution of Fe(OH)2 is 8,67. What
is the Ksp for Fe(OH)2? (A) 5×10–6 (B) 2×10–11 (C) 1×10–16 (D) 5×10–17

Resolução: Alternativa E.

Hidróxido de ferro II: Fe(OH)2(aq) → Fe+2 −


(aq) + 2 OH(aq)

659
1600 FÍSICO-QUÍMICA APLICADA EXERCÍCIOS COMENTADOS - IME – ITA – OLIMPÍADA

Cálculo do pOH: pOH = 14 − pH = 14 − 8,67 = 5,33

Cálculo da concentração da quantidade de matéria de hidroxila: pOH = − log[OH − ]

5,33 = − log[OH − ]

𝑚𝑜𝑙
[OH − ] = 10−(5,33) = 4,68 𝑥 10−6
𝐿
Cálculo da concentração da quantidade de matéria de íons Fe+2(aq):

1 mol de Fe(OH)2(aq) − − − 1 mol de Fe+2 −


(aq) − − − − − − 2 mol de OH(aq)
mol
1 mol de Fe(OH)2(aq) − − − [Fe+2
(aq) ] − − − − − − − − − 4,68 x 10
−6
L
𝑚𝑜𝑙
[Fe+2
(aq) ] = 2,34 𝑥 10
−6
𝐿

Cálculo da constante do produto de solubilidade: K ps = [Fe+2 ] x [OH − ]2 = 2,34 𝑥 10−6 x (4,68 𝑥 10−6 )2 =
5,12 𝑥 10−17

Questão 1216 – (U. S. NATIONAL CHEMISTRY OLYMPIAD) What is the pH of a solution formed by mixing 45,0 mL
of 0,10 M HNO3, 50,0 mL of 0,20 M HCl, and 55,0 mL of 0,10 M CH3COOH? (A) 0,40 (B) 0,88 (C) 1,01 (D) 1,52

Resolução: Alternativa B.

mol
Analisando a solução de ácido nítrico: [HNO3 ] = [H + ] = 0,10 L
e V1 = 45,0 mL

mol
Analisando a solução de ácido clorídrico: [HCl] = [H + ] = 0,20 L
e V2 = 50,0 mL

mol
Analisando a solução de ácido acético: [CH3 COOH] = [H + ] = 0,10 L
e V3 = 55,0 mL

Cálculo da concentração da quantidade de matéria de íons H+ no estado final: [H + ] x V1 + [H + ] x V2 +


[H + ] x V3 = [H + ]𝑓𝑖𝑛𝑎𝑙 x Vfinal

0,10 x 45 + 0,20 x 50 + 0,10 x 55 mol


[H + ]final = = 0,133
45 + 50 + 55 L

pH = − log[H + ] = − log(0,133) = 0,875

Questão 1217 – (U. S. NATIONAL CHEMISTRY OLYMPIAD) Calculate the aqueous solubility of Ca(OH)2 in grams
per liter. [Ksp = 8,0 × 10–6] (A) 5,9 × 10–4 (B) 2,0 × 10–2 (C) 0,93 (D) 1,5

Resolução: Alternativa C.

Hidróxido de cálcio: Ca(OH)2(aq) → Ca+2 −


(aq) + 2 OH(aq)

1 𝑚𝑜𝑙 𝑑𝑒 Ca(OH)2(aq) − − − 1 mol de Ca+2 −


(aq) − − − 2 mol de OH(aq)
1 𝑚𝑜𝑙 𝑑𝑒 Ca(OH)2(aq) − − − S − − − − − − − − − 2S

Relação matemática entre o produto de solubilidade e a solubilidade: K ps = [Ca+2 ] x [OH − ]2 = S x (2S)2 = 4S³

660
1600 FÍSICO-QUÍMICA APLICADA EXERCÍCIOS COMENTADOS - IME – ITA – OLIMPÍADA

3 K 3 8,0 𝑥 10−6 mol 74 g g


ps
S= √ =√ = 1,26 x 10−2 x = 0,932
4 4 L 1 mol L

Questão 1218 – (U. S. NATIONAL CHEMISTRY OLYMPIAD) A 0,050M solution of an unknown acid is 1.0% ionized.
What is the value of its Ka? (A) 2,5×10–7 (B) 5,0×10–6 (C) 5,0×10–4 (D) 5,0×10–2

Resolução: Alternativa B.

Cálculo da constante de equilíbrio do ácido:

[HCN] x α² 0,050 x (0,01)²


Ka = = = 0,050 𝑥 (0,01)2 = 5,0 x 10−6
1−𝛼 1 − 0,01

⏟=0,99
"o que praticamente corresponde igual a 1".

2
Ka 6,2 x 10−10
α = = = 6,2 x 10−8
[HCN] 0,010

α = √6,2 x 10−8 = 2,50 𝑥 10−4 (0,025%)

Questão 1219 – (U. S. NATIONAL CHEMISTRY OLYMPIAD) Calculate the pH of a 0,15 M solution of HOCl. (Ka =
HOCl 2,9×10–8) (A) 3,77 (B) 4,18 (C) 6,71 (D) 8,36

Resolução: Alternativa B.

Nome do ácido: Ácido hipocloroso: HClO(aq) + H2 O(l) → ClO− +


(aq) + H3 O(aq)

Cálculo do grau de dissociação (α): k a = M x α2

2,90 𝑥 10−8 = 0,15 x α2

2,90 𝑥 10−8
α=√ = 1,40 𝑥 10−4
0,15

Cálculo da concentração da quantidade de matéria de H3 O+ +


(aq) : [H3 O(aq) ] = 𝑀 𝑥 𝛼 = 0,15 𝑥 1,40 𝑥 10
−4
=
−5
6,60 𝑥 10

Cálculo do pH: pH = − log[H3 O+ −5


(aq) ] = − log(6,60 𝑥 10 ) = 4,18

Questão 1220 – (U. S. NATIONAL CHEMISTRY OLYMPIAD) The Ksp of AgCl is 9,5  10–11 at 14°C and is 7,8  10–
10 at 42°C. What is ∆Hº for the dissolution of AgCl ? AgCl ⇌ Ag+
(aq) + Cl (aq)

(s) (s)
(A) – 0,49 kJ.mol–1 (B) + 0,37 kJ.mol–1 (C) + 57 kJ.mol–1 (D) +220 kJ.mol–1

Resolução: Alternativa C.

9,5 x 10−11 ∆H0 1 1


Aplicando a equação matemática de Van’t Hoff: ln ( ) = x( − )
7,8 x 10−10 8,314 42+273 14+273

ln(0,122) x 8,314 = ∆H 0 x (−3,10 x 10−4 )

ln(0,122) x 8,314 kJ kJ
∆H 0 = = 56420,79 (56,42 )
−3,10 x 10−4 mol mol

661
1600 FÍSICO-QUÍMICA APLICADA EXERCÍCIOS COMENTADOS - IME – ITA – OLIMPÍADA

Questão 1221 – (U. S. NATIONAL CHEMISTRY OLYMPIAD) Calculate the hydrogen ion in mol/L of a 0,010 M
solution of NH4Cl. Base Ionization Constant, Kb concentration of NH3: 1.8×10-5.
(A) 4.2×10-4 (B) 2,4×10-6 (C) 1,8×10-7 (D) 5,6×10-12

Resolução: Alternativa B.
+
Analisando a hidrólise do cloreto de amônio: NH4 Cl(aq) ⇌ NH4(aq) + Cl−
(aq)

+
NH4(aq) + Cl−
(aq) + H2 O(l) ⇌ NH4 OH(aq) + HCl
⏟ (aq)
ácido forte

+
NH4(aq) + Cl− + −
(aq) + H2 O(l) ⇌ NH4 OH(aq) + H(aq) + Cl(aq)

+ +
NH4(aq) + H2 O(l) ⇌ NH4 OH(aq) + H
⏟(aq)
solução ácida

KW
Cálculo do grau de dissociação, a partir da equação da hidrólise: K h =
Kb
10−14
= 5,56 x 10−10 = [NH
⏟ 4 Cl] x α2
1,80 x 10−5
Kh

α = 2,36 x 10−4

mol
Cálculo da concentração da quantidade de matéria de H+: [H + ] = 0,010 x 2,36 x 10−4 = 2,36 x 10−6
L

Questão 1222 – (OLIMPÍADA BRASILEIRA DE QUÍMICA) Uma solução pode ser denominada solução tampão,
quando contém concentrações aproximadamente iguais de um:
a) Ácido forte e um sal
b) Ácido e uma base forte
c) Ácido e base fracos
d) Ácido forte e sua base conjugada
e) Ácido fraco e sua base conjugada

Resolução: Alternativa E.
Uma solução definida como tampão se caracteriza por ser uma solução composta por um ácido fraco com o seu sal
conjugado ou uma base fraca com o seu sal conjugado.

Questão 1223 – (MESTRE JOÃO ROBERTO DA PACIÊNCIA NABUCO) O grau de hidrólise do cianeto de potássio
em solução 0,2 mol.L-1 é 0,0085 a 20°C. Calcular Kh da hidrólise do sal e Ka do ácido cianídrico a mesma temperatura.

Resolução: Estudando a hidrólise do cianeto de potássio (KCN): KCN(aq) ⇌ K + −


(aq) + CN(aq)

K+ −
(aq) + CN(aq) + H2 O(l) ⇌ KOH
⏟ (aq) + HCN(aq)
Base forte

K+ − + −
(aq) + CN(aq) + H2 O(l) ⇌ K (aq) + OH(aq) + HCN(aq)

− −
CN(aq) + H2 O(l) ⇌ HCN(aq) + OH
⏟ (aq)
solução básica

A solução de cianteo de potássio é alcalina.

Cálculo da constante de hidrólise (Kh): K h = [KCN] x α2 = 0,20 x (0,0085)2 = 1,445 x 10−5

662
1600 FÍSICO-QUÍMICA APLICADA EXERCÍCIOS COMENTADOS - IME – ITA – OLIMPÍADA

KW
Cálculo da constante de ácido (Ka): K h = Ka

10−14
1,445 x 10−5 =
Ka

10−14
Ka = = 6,92 x 10−10
1,445 x 10−5

Questão 1224 – (MONBUKAGAKUSHO, COLLEGE OF TECHNOLOGY STUDENTS E UNDERGRADUATED


STUDENTS) What is the pH of 50 mL of the 0,14 mol.L-1 HCl solution mixed with 50 mL of 0,10 mol.L-1 NaOH solution?
Write the number of the correct answer in the answer box. Log 2 = 0,30.
a) 1,5
b) 1,7
c) 1,9
d) 2,1
e) 2,3

Resolução: Alternativa B.

Equação química: NaOH(aq) + HCl(aq) → NaCl(aq) + H2 O(l)

Cálculo do número de mol para cada espécie química:

nácido = 0,14 x 50 x 10−3 = 7,0 x 10−3 mol

nbase = 0,10 x 50 x 10−3 = 5,0 x 10−3 mol -

nexcesso
ácido = 7,0 x 10−3 mol − 5,0 x 10−3 mol = 2,0 x 10−3 mol

Cálculo da concentração da quantidade de matéria do ácido em excesso, sabenso que o volume final é de 100 mL
(50 mL + 50 mL).

nexcesso
ácido 2,0 x 10−3 mol mol
[HCl] = = = 2,0 x 10−2
Vtotal 0,1 L L

mol
[HCl] = [H + ] = 2,0 x 10−2
L

Cálculo do pH: pH = − log[H + ] = −(2,0 x 10−2 ) = 1,70

Questão 1225 – Uma solução de um ácido monoprótico apresenta concentração da quantidade de matéria igual a
0,030 mol x L-1 com grau de dissociação igual a 13,90%. A partir das informações apresentadas, calcule a constante
de equilíbrio do referido ácido.

M x α² 0,030 x (0,139)2
Resolução: Cálculo da constante do ácido: K a = 1−α
= 1−0,139
= 6,73 x 10−3

Questão 1226 – Sabendo que o pH do ácido benzoico é igual a 2,75 a uma temperatura de 25°C e concentração
igual a 0,069 mol.L-1, calcule a constante de equilíbrio do referido ácido.

Resolução: Equação química: C7H6O2(aq) + H2O(l) ⇄ C7H5O2-(aq) + H3O+(aq)

663
1600 FÍSICO-QUÍMICA APLICADA EXERCÍCIOS COMENTADOS - IME – ITA – OLIMPÍADA

Base de cálculo: M (mol.L-1) inicial de C7H6O2(aq)


Tabela de equilíbrio químico:

C7H6O2 H2O(l) ⇄ C7H5O2-(aq) H3O+(aq)


Início M - 0 0
Reage Mα - Mα Mα
Equilíbrio M - Mα - Mα Mα

Espécies no estado de equilíbrio:


[C7 H6 O2 ] = M − Mα
[C7 H5 O−
2 ] = Mα
[H3 O+ ] = Mα

[C7 H5 O− +
2 ] x [H3 O ]
Expressão da constante de equilíbrio do ácido (Ka): K a = [C7 H6 O2 ]

Mα x Mα Mα x Mα Mα²
Ka = = =
M − Mα M x (1 − α) 1 − α
Cálculo do grau de ionização, a partir do pH: 2,75 = −log[H3 O+ ]

mol
[H + ] = 10−2,75 = 1,78 x 10−3
L
mol
Como no equilíbrio a concentração de H3O+ é igual a Mα, o grau de dissociação será: M x α = 1,78 x 10−3 L
mol mol
0,069 x α = 1,78 x 10−3
L L

1,78 𝑥 x 10−3
α= = 2,57 𝑥 10−2 (2,57%)
0,069
[C7 H5 O− +
2 ] x [H3 O ] Mα² 0,069 x (0,0257)²
Cálculo da constante de equilíbrio do ácido (Ka): K a = [C7 H6 O2 ]
= 1−α = 1−0,0257
= 4,68 x 10−5

Questão 1227 – Quantidade equivalente a 0,05 mol de certo ácido hipotético apresentado na forma HX, foi dissolvida
em água produzindo 1 L de solução apresentando pH igual a 2,0.

a) Determine a concentração total de todos os íons presentes após o equilíbrio ter sido alcançado.
b) Calcule o valor da constante de dissociação do ácido (Ka).
c) Calcule o valor da constante de equilíbrio (K).

Resolução: Equação química: HX(aq)  X-(aq) + H3O+(aq)


Base de cálculo: 0,05 mol de HX no início.

HX(aq)  X-(aq) H3O+(aq)


Início 0,05 0 0
Reage 0,05 α 0,05α 0,05α
Equilíbrio 0,05 x (1 - α) 0,05α 0,05α

[X− ] x [H3 O+ ]
Expressão da constante de equilíbrio do ácido: K a = [HX]

664
1600 FÍSICO-QUÍMICA APLICADA EXERCÍCIOS COMENTADOS - IME – ITA – OLIMPÍADA

[X − ] x [H3 O+ ]
Ka =
[HX]
0,05α 0,05α
( 1 )x( 1 )
Ka =
0,05 x (1 − α)
[ 1 ]

[X − ] x [H3 O+ ] 0,05α x 0,05α 0,05. α²


Ka = = =
[HX] 0,05 x (1 − α) 1−α

Como o valor do pH foi dado, a concentração de cátions: pH = −log[H3 O+ ]

2,0 = −log[H3 O+ ]

[H3 O+ ] = 10−2 mol. L−1

a) Condição de eletroneutralidade: A carga positiva total deve ser igual à carga negativa total.

(+1) x [H3 O+ ] + (−1) x [X − ] = 0

[H3 O+ ] = [X − ] = 0,01 mol. L−1

Logo, a concentração total de espécies iônicas pode ser facilmente calculada:

[H3 O+ ] + [X − ] = 10−2 mol. L−1 + 10−2 mol. L−1 = 2,0 x 10−2 mol. L−1

b) Como a concentração da quantidade de matéria de cátions hidroxiliônicos e pela tabela de equilíbrio químico

0,05. α
[H3 O+ ] = [X − ] =
1
[H3 O+ ] = [X − ] = 0,05. α = 1,0 . 10−2

1,0 x 10−2
α=
5 x 10−2
1
α= = 0,2 (20%)
5
0,05. α
[H3 O+ ] = [X − ] =
1

[H3 O+ ] = [X − ] = 0,05. α = 1,0 x 10−2

1,0 x 10−2 1
α= = = 0,2 (20%)
5 x 10−2 5

1,0 x 10−2 1
[H3 O+ ] = [X − ] = 0,05. α = 1,0 . 10−2 → α = = = 0,2 (20%)
5 x 10−2 5

0,05. α 1,0 . 10−2 1


[H3 O+ ] = [X − ] = = 0,05. α = 1,0 . 10−2 → α = = = 0,2 (20%)
1 5. 10−2 5

Inserindo-se este último resultado na equação para o cálculo de K a , tem-se:

665
1600 FÍSICO-QUÍMICA APLICADA EXERCÍCIOS COMENTADOS - IME – ITA – OLIMPÍADA

(0,05. α²) 0,05 x (0,2)2


Ka = = = 2,50 x 10−3 mol. L−1
(1 − α) (1 − 0,2)
c) A constante de equilíbrio pode ser relacionada com o valor de 𝐾𝑎 através do valor para a concentração total de
espécies presentes (Ct ).

𝐾. 𝐶t𝛿 = 𝐾𝑎
Ct = [H3 O+ ] + [X − ] + [HX]
A concentração de HX no equilíbrio pode ser determinada com base no valor do grau de dissociação encontrado no
item anterior.
[HX] = 0,05. (1 − α) = 0,05. (1 − 0,2) = 0,05.0,8 = 4 x 10−2 mol. L−1

Para o processo de ionização em questão, o valor de delta é igual a um, dado que para cada uma molécula de HX
que se dissocia, uma molécula de H3O+ e outra de X- são geradas.
𝐶𝑡𝑜𝑡𝑎𝑙 = 0,04 + 0,01 + 0,01 = 0,06 𝑚𝑜𝑙. 𝐿−1

Substituindo-se na relação entre K e K a , tem-se: K x Ct1 = K a


K a 2,5 x 10−3
K= = = 4,17 x 10−2
Ct 6 x 10−2
Questão 1228 – Calcule o pH de um tampão composto por 3,50 g de ácido cianídrico e 1,10 g de cianeto de sódio
para cada um litro. Informação para a resolução do problema: constante do ácido = 6,96 x 10-10.
3,50
Resolução: Cálculo da concentração da quantidade de matéria do ácido cianíbrico: [HCN] = (1+12+14) x 1 =
3,50 mol
27
= 0,130 L
1,10
Cálculo da concentração da quantidade de matéria de cianeto de sódio: [NaCN] = (23+12+14) =
x1
mol
2,24 x 10−2
L

Equação química: HCN(aq) + H2 O(l) ⇄ CN(aq) + H3 O+
(aq)

[CN− ] x [H3 O+ ]
Expressão da constante de equilíbrio: K a = [HCN] x aH2O

[CN− ] x [H3 O+ ]
Sendo a atividade da água igual a 1 e aplicando a função logarítmica: K a = [HCN] x aH2O

[CN − ] x [H3 O+ ]
logK a = log { }
[HCN]

logK a = log{[CN− ] x [H3 O+ ]} − log [HCN]


logK a = log[CN − ] + log[H3 O+ ] − log [HCN]
[CN − ]
− log[H3 O+ ] = ⏟
⏟ − logK a + log
[HCN]
pH pK a

[𝐂𝐍 − ]
𝐩𝐇 = 𝐩𝐊 𝐚 + 𝐥𝐨𝐠
[𝐇𝐂𝐍]

Substituindo os valores na equação em negrito: pH = −log (6,96 x 10−10 ) + log (0,172)

666
1600 FÍSICO-QUÍMICA APLICADA EXERCÍCIOS COMENTADOS - IME – ITA – OLIMPÍADA

pH = 9,16 − 0,76 = 8,40


Questão 1229 – Calcule o pH de uma solução que apresenta 0,10 mol.L-1 de ácido cianídrico e 0,01 mol.L-1 de cianeto
de potássio. Informação para a resolução do problema: Ka = 7 x 10-10.

Resolução: Observa-se que a solução é composta por um ácido fraco e seu sal conjugado, o que caracteriza uma
solução tampão.
[sal]
pH = pka + log
[ácido]
[sal]
pH = − log K a + log
[ácido]
0,10
pH = − log(7 x 10−10 ) + log
0,010
pH = −(−9,15) + log
⏟ 10 = 9,15 + 1 = 10,15
1

Questão 1230 - Calcule a massa de hidróxido de sódio que deve ser dissolvida em 0,250 mililitros de água pura para
que a solução resultante tenha um pH igual a 12,5 a uma temperatura de 25°C.
Resolução: A reação de dissociação de uma molécula de NaOH pode ser representada pela equação química abaixo.
NaOH(aq)  Na+(aq) + OH-(aq)
A 25oC, a constante de dissociação da água (Kw) apresenta um valor de 10-14 mol.L-1, a partir do conhecimento do
valor do pH, o valor de pOH pode ser prontamente determinado: pH + pOH = 14
12,5 + pOH = 14
pOH = 14 − 12,5 = 1,5
A partir da definição de pOH, tem-se: pOH = −log[OH − ]
[OH − ] = 10−pOH = 10−1,5 = 3,16 x 10−2 mol. L−1

Assumindo-se que o grau de dissociação do NaOH é de 100% nas condições dadas, como o volume da mistura não
se altera durante o processo, a concentração de OH- final deve ser idêntica à concentração de NaOH originalmente
presente.
[NaOH] = 3,16 x 10-2 mol.L-1
Com base no conhecimento do volume da solução (0,3 L) e através do valor da massa molecular do hidróxido de
sódio (40 g.mol-1), a massa requerida pode ser finalmente calculada a partir da concentração da quantidade de
matéria.
mNaOH = [NaOH] x < MM >NaOH x Vsolução = 3,16 x 10−2 x 40 x 0,30 = 0,379 g

Questão 1231 – Determine o pH de uma solução tampão que foi preparada mediante uma mistura de 0,10 mol de
ácido láctico (HLa) e 0,10 mol de lactato de potássio (LaK), para 1 L de solução a 25oC. Da literatura, sabe-se que, na
temperatura de interesse, a constante de dissociação do ácido lático apresenta um valor de 1,38 x 10-4 mol.L-1.
Resolução: Sabe-se que para uma solução tampão genérica, pH e pKa estão relacionados com as concentrações
iniciais do ácido e do sal empregados em sua formulação. Desta forma, no presente caso, tem-se:
[LaK]inicial
pH = pK a + log
[HLa]inicial

667
1600 FÍSICO-QUÍMICA APLICADA EXERCÍCIOS COMENTADOS - IME – ITA – OLIMPÍADA

[0,1]
Logo, inserindo-se os valores do enunciado, tem-se: pH = − log(1,38 x 10−4 ) + log [0,1] = 3,86 + log10 (1) =
3,86
pH = 3,86 + log10 (1) = 3,86
Questão 1232 – Determine o valor do produto de solubilidade do brometo de sódio (NaBr), sabendo que a partir da
dissolução de uma massa de 0,55 mg em 1 L de água a 25oC, observa-se o equilíbrio entre a solução e cristais de
NaBr. Sabe-se que após a filtração da solução, 0,1 mg de NaBr sólido foi recuperado.
Resolução: Como do total dissolvido, 0,1 mg permanecem no estado sólido, conclui-se que 0,4 mg foram
transformados em ânions Br- e cátions Na+, de acordo com a seguinte equação química:
NaBr(aq) ⇋ Na+ −
(aq) + Br (aq)

Portanto, a concentração de cátions Na+ pode ser determinada, dado que para cada mol de NaBr que se dissolve, um
mol de cátions é produzido em solução:
0,55 mg 10−3 g 1 mol
[Na+ ] = x x = 5,34 x 10−6 mol. L−1
L 1 mg 103 g
Pelo princípio de eletroneutralidade, a concentração de Br- deve ser igual à concentração de Na+. Logo, o produto de
solubilidade do NaBr sólido pode ser prontamente calculado:
[Na+ ] = [Br − ] = 5,34 x 10−6 mol. L−1

K ps = [Na+ ] x [Br − ] = (5,34 x 10−6 )2

K ps = 28,51 x 10−12

Questão 1233 – Determine a massa do ácido propanoico (CH3CH2COOH), que deve ser adicionada a 80,0 g de
propionato de sódio (CH3CH2COONa) para a produção de uma solução tampão com volume igual a 2,0 L, pH igual a
5,0 a 25oC. Considere que a ionização do ácido propanoico é representada pela equação abaixo, e que na temperatura
de interesse a constante de dissociação do ácido propanoico apresenta um valor de 1,34 x 10-5 mol.L-1.
CH3CH2COOH(aq)  CH3CH2COO-(aq) + H3O+(aq)
Resolução: Denominando-se o ácido propanoico por HP e o sal correspondente por NaP, o pH da solução tampão
pode ser calculado com base no conhecimento da constante de dissociação do ácido e nas concentrações iniciais do
ácido e do sal associado.
[NaP]inicial
pH = pK a + log10
[HP]inicial
A concentração inicial de NaP pode ser determinada a partir do volume da solução e da massa de sal utilizada na
80 𝑚𝑜𝑙
confecção do tampão. Sabendo-se que a massa molecular do NaP é igual a 96.g.mol-1, tem-se: [𝑁𝑎𝑃] = =
96,2 𝐿
0,4167 𝑚𝑜𝑙. 𝐿−1
Empregando-se a equação que relacionada pH e pKa é possível determinar a concentração inicial de HP.

pKa = −logK a = −log(1,34 x 10−5 ) = 4,873


[NaP]inicial
pH = pK a + log
[HP]inicial
[NaP]inicial
log = pH − pKa
[HP]inicial

668
1600 FÍSICO-QUÍMICA APLICADA EXERCÍCIOS COMENTADOS - IME – ITA – OLIMPÍADA

[NaP]inicial
log = 5 − 4,873 = 0,127
[HP]inicial
[NaP]inicial
= 10−0,127
[HP]inicial
[NaP]inicial
[HP]inicial =
10(−0,127)
0,4167
[HP]inicial = = 0,558 mol. L−1
0,7464
Com base no volume de solução (2 L) e na massa molecular do ácido propanoico (74,08 g.mol -1), a massa de ácido
pode ser diretamente calculada:
mHP
[HP]inicial =
V x < MM >HP
mHP = [HP]inicial x V x < MM >HP = 0,558 x 2 x 74,08 = 82,67 g
Questão 1234 – Considere que a constante de dissociação do ácido a 25°C seja igual a 1,47 x 10 -3. A partir das
informações apresentadas, calcule o grau de dissociação considerando que a solução apresente comportamento
ideal. Informação para a solução do problema: concentração da quantidade de matéria = 0,015 mol x L-1.

Resolução: Para a resolução deste problema será necessário utilizar a tabela de equilíbrio químico.

Equação química: HA(aq) + H2 O(l) → A−(aq) + H3 O+


(aq)

Base de cálculo: 0,015 mol x L-1 de concentração da quantidade inicial de ácido.

Tabela de equilíbrio químico:


HA(aq) H2 O(l) ⇄ A−(aq) H3 O+
(aq)
Início 0,015 - 0 0
Reage 0,015α - 0,015α 0,015α
Equilíbrio 0,015 - 0,015α - 0,015α 0,015α

[A− +
(aq) ] x [H3 O ]
Expressão da constante de equilíbrio do ácido: K a =
[HA(aq) ] 𝑥 𝑎H2O(l)

1
0,015α x 0,015α 0,015α²
Ka = = = 1,47 𝑥 10−3
0,015 − 0,015α 1−α

0,015α2 + 1,47 x 10−3 α − 1,47 x 10−3 = 0


Resolvendo a equação do segundo grau, uma das raízes corresponde a α = 0,2676, o que corresponde a 26,76%.

Questão 1235 – Calcule as concentrações das quantidades de matéria dos íons HSO4 − −
(aq) , SO4 (aq) e
H3 O+ (aq) , na solução 0,10 mol x L de bissulfato de potássio. Informação para a resolução do problema: Ka = 1,30 x
-1

10-2.

Resolução: Para a resolução deste problema será necessário utilizar a tabela de equilíbrio químico.

Equação química: KHS𝑂4 (aq) → K +
(aq) + HSO4 (aq)

Base de cálculo: 0,10 mol x L-1 de concentração da quantidade inicial de HSO4 −


(aq) .

669
1600 FÍSICO-QUÍMICA APLICADA EXERCÍCIOS COMENTADOS - IME – ITA – OLIMPÍADA

Tabela de equilíbrio químico:

HSO4 −
(aq) H2 O(l) ⇄ SO4 − +
(aq) H3 O(aq)
Início 0,10 - 0 0
Reage 0,10α - 0,10α 0,10α
Equilíbrio 0,10 - 0,10α - 0,10α 0,10α

[SO4 − +
(aq) ] x [H3 O ]
Expressão da constante de equilíbrio do ácido: K a =
[HSO4 −
(aq) ] 𝑥 𝑎
⏟H2O(l)
1
0,10α x 0,10α
Ka = = 1,30 𝑥 10−2
0,10 − 0,10α

0,10α2 + 1,30 x 10−2 α − 1,30 x 10−2 = 0


Resolvendo a equação do segundo grau, uma das raízes corresponde a α = 0,30, o que corresponde a 30%. As
concentrações das quantidades de matéria dos íons no equilíbrio químico são os seguintes:

mol
[HSO4 −
(aq) ] = 0,10 − 0,10α = 0,10 − 0,10 x 0,30 = 0,07 L
mol
[SO4 −2
(aq) ] = 0,10α = 0,10 x 0,30 = 0,03 L
mol
[H3 O+ ] = 0,10α = 0,10 x 0,30 = 0,03
L
Questão 1236 – Calcule a hidrólise através de uma solução 0,100 mol x L-1 de cloreto de amônio. Informação para a
resolução do problema: Kb(NH3) = 1,80 x 10-3.
+
Resolução: Analisando a hidrólise do cloreto de amônio: NH4 Cl(aq) ⇌ NH4(aq) + Cl−
(aq)
+
NH4(aq) + Cl−
(aq) + H O
2 (l) ⇌ NH 4 OH (aq) + HCl
⏟ (aq)
ácido forte

+
NH4(aq) + Cl− + −
(aq) + H2 O(l) ⇌ NH4 OH(aq) + H(aq) + Cl(aq)

+ +
NH4(aq) + H2 O(l) ⇌ NH4 OH(aq) + H
⏟(aq)
solução ácida
KW
Kh =
Kb

10−14
Kh = = 5,56 𝑥 10−10
1,80 𝑥 10−5

Questão 1237 – Calcule o grau de ionização, dado em percentual, de uma solução 1,0 mol.L -1 de ácido cianídrico,
sabendo que a constante de ionizaçãodo referido ácido igual a 4,80 x 10-10.

Resolução: Base de cálculo: n(mol) inicial de HCN.

Analisando a segunda tabela de equilíbrio químico:



HCN(aq) H2 O(l) ⇄ CN(aq) H3 O+
(aq)

670
1600 FÍSICO-QUÍMICA APLICADA EXERCÍCIOS COMENTADOS - IME – ITA – OLIMPÍADA

Início n - 0 0
Reage nα - nα nα
n n 𝑛 𝑛
Equilíbrio - α - α α
𝑉 𝑉 𝑉 𝑉

[CN− +
(aq) ] x [H3 O ]
Expressão da constante de equilíbrio do ácido: K a =
[HCN(aq) ] 𝑥 𝑎H2O(l)

1
Mα x Mα Mα x Mα Mα²
Ka = = = = 4,80 𝑥 10−10
M − Mα M (1 − α) 1 − α

Como o ácido é fraco, o grau de dissociação é pequeno, logo 1 − 𝛼 → 1,oi seja, o grau de dissociação tende a zero
(𝛼 → 0).

1,0 x α² = 4,80 𝑥 10−10

𝛼 = √4,80 𝑥 10−10 = 2,20 𝑥 10−5 (0,0022%)

pH = − log[H3 O+ −5 −5
(aq) ] = − log(1,0 x 2,20 x 10 ) = − log(2,20 x 10 ) = 4,66

Questão 1238 – A partir de ma solução de ácido carbônico com concentração da quantidade de matéria igual a 0,025
mol x L-1, determine os valores aproximados das concentrações dos íons H3O+, HCO3- e CO3-2, sabendo que a
constante do ácido é igual a 4,0 x 10-7 e a segunda constante do ácido é igual a Ka = 4,80 x 10-11 (HCO3-).

Resolução: Para a resolução deste problema será necessário utilizar a tabela de equilíbrio químico.
Base de cálculo: 0,025 mol x L-1 de H2CO3.
Tabela de equilíbrio químico:

H2 CO3 (aq) H2 O(l) ⇄ HCO3 − +


(aq) H3 O(aq)
Início 0,025 - 0 0
Reage 0,025α - 0,025α 0,025α
Equilíbrio 0,025 - 0,025α - 0,025α 0,025α

[HCO3− +
(aq) ] x [H3 O ]
Expressão da constante de equilíbrio do ácido: K a =
[H2 CO3 (aq) ] 𝑥 𝑎H2O(l)

1
(0,025α)2 = 4,0 x 10−7 𝑥 0,025 𝑥 (1 − α)

0,025α2 = 4,0 x 10−7 − 4,0 x 10−7 α

0,025α2 + 4,0 x 10−7 α − 4,0 x 10−7 = 0

Resolvendo a equação dos segundo grau, uma das raízes corresponde a 0,00399 (0,40%).

Cálculo das concentraçãoes das quantidades de matéria das espécies químicas no equilíbrio químico:

mol
[HCO3 −
(aq) ] =
[H3 O+ ] = 0,025 x 0,0040 = 10−4
L
mol
[H2 CO3 (aq) ] = 0,025 − 0,025 x 0,0040 = 0,0249
L

671
1600 FÍSICO-QUÍMICA APLICADA EXERCÍCIOS COMENTADOS - IME – ITA – OLIMPÍADA

Analisando a segunda tabela de equilíbrio químico:

HCO3 −
(aq) H2 O(l) ⇄ CO3 −2
(aq) H3 O+
(aq)
Início 10−4 - 0 10−4
Reage −4
10 α - 10−4α 10−4α
Equilíbrio 10 - 10 α -
−4 −4
10−4α 10−4 + 10−4α

[CO3 −2 +
(aq) ] x [H3 O ]
Expressão da constante de equilíbrio do ácido: K a =
[HCO3 −
(aq) ] 𝑥 𝑎
⏟H2O(l)
1
10−4 α x (10−4 + 10−4 α)
Ka = = 4,80 𝑥 10−11
10−4 − 10−4 α

10−4 α2 + 10−4 α − 4,80 x 10−11 = 0

Resolvendo a equação do segundo grau, uma das raízes é aproximadamente igual a 0. Logo, as concentrações das
quantidades de matéria dos participantes químicos são os seguintes:

[CO3 −2
(aq) ] = 0

mol
[HCO3 −
(aq) ] =
[H3 O+ ] = 10−4
L

Questão 1239 – A partir da concentração da quantidade de matéria inicial 0,35 mol x L-1 de acetato de sódio, determine
o grau de dissociação, o pH e o pOH. Informação para a resolução do problema: Ka = 1,75 x 10-5.

Resolução: Fórmula molecular do acetato de sódio: CH3COOH

Analisando a hidrólise do acetato de sódio: CH3 COONa(aq) ⇌ CH3 COO− +


(aq) + Na (aq)

CH3 COO− +
(aq) + Na(aq) + H2 O(l) ⇌ CH
⏟ 3 COOH(aq) + NaOH
⏟ (aq)
ácido fraco ácido forte

CH3 COO− + +
(aq) + Na(aq) + H2 O(l) ⇌ CH3 COOH(aq) + Na (aq) + OH −
⏟ (aq)
solução básica

CH3 COO−
(aq) + H2 O(l) ⇌ CH3 COOH(aq) + OH −
⏟ (aq)
solução básica

KW
Cálculo do grau de dissociação, a partir da constante de hidrólise: K h =
Ka

10−14
0,35 𝑥 α2 =
1,75 x 10−5

α = 4,04 𝑥 10−5

Cálculo da concentração da quantidade de matéria hidrogeniônica, [H + ]: [H + ] = 0,35 𝑥 4,04 𝑥 10−5 =


mol
1,41 𝑥 10−5 L

Cálculo do pH: pH = − log[H + ] = − log(1,41 𝑥 10−5 ) = 4,85

672
1600 FÍSICO-QUÍMICA APLICADA EXERCÍCIOS COMENTADOS - IME – ITA – OLIMPÍADA

Cálculo do pOH: pOH = 14 − pH = 14 − 4,85 = 9,15

Questão 1240 – Determine o valor do pH de uma solução com concentração da quantidade de matéria igual a 0,10
mol x L-1 de cloreto de amônio, que apresenta constante da base (Kb) igual a 1,80 x 10-5.

Resolução: Primeiramente será necessário analisar a hidrólise do cloreto de amônio (NH4Cl).


+
NH4 Cl(aq) ⇌ NH4(aq) + Cl−
(aq)

+
NH4(aq) + Cl−
(aq) + H2 O(l) ⇌ NH4 OH(aq) + HCl
⏟ (aq)
ácido forte

+
NH4(aq) + Cl− + −
(aq) + H2 O(l) ⇌ NH4 OH(aq) + H(aq) + Cl(aq)

+ +
NH4(aq) + H2 O(l) ⇌ NH4 OH(aq) + H
⏟(aq)
solução ácida

KW
Cálculo do grau de dissoação (α), a partir da constante de hidrólise (Kh): K h = Kb

10−14
0,10 𝑥 α2 =
1,80 x 10−5

α = 7,46 𝑥 10−5

Cálculo da concentração da quantidade de matéria hidrogeniônica, [H + ]: [H + ] = 0,10 𝑥 7,46 𝑥 10−5 =


mol
7,46 𝑥 10−6 L

Cálculo do pH: pH = − log[H + ] = − log(7,46 x 10−6 ) = 5,13

673
1600 FÍSICO-QUÍMICA APLICADA EXERCÍCIOS COMENTADOS - IME – ITA – OLIMPÍADA

Questão 944 – (OLIMPÍADA


MUNDIAL DE QUÍMICA) Uma
determinada substância sofre
decomposição segundo uma
cinética de primeira ordem, e
sua dependência em relação
à temperatura segue uma lei
empírica chamada de
equação de Arrhenius. Os

674
1600 FÍSICO-QUÍMICA APLICADA EXERCÍCIOS COMENTADOS - IME – ITA – OLIMPÍADA

tempos de meia-vida
detCAPÍTULO IX
ELETROQUÍMICA – PILHA E
ELETRÓLISE: ÍGNEA E
AQUOSA

PROFESSOR ALEXANDRE
VARGAS GRILLO

675
1600 FÍSICO-QUÍMICA APLICADA EXERCÍCIOS COMENTADOS - IME – ITA – OLIMPÍADA

Questão 1241 - (IME)


a) Determinar o potencial padrão de uma célula formada por eletrodos de Cu e Cd, onde os eletrólitos são sais de
Cu2+ e Cd2+.
b) Indicar o eletrodo positivo, o anodo é o metal que se reduz. Dados:
Cu2+(aq) + 2e- → Cu(s) ε° = + 0,337 V
Cd2+(aq) + 2e- → Cd(s) ε° = - 0,403 V

Resolução: Item a) Cálculo do potencial padrão (E°), Invertendo a semirreação de redução do Cádmio:

Cd°(s) → Cd2+(aq) + 2e- ΔG°Cd = - 2.F.(+ 0,403)


Cu2+(aq) + 2e- → Cu(s) ΔG°Cu = - 2.F.(+ 0,337)

Determinação da equação global:


Cd°(s) → Cd2+(aq) + 2e-
Cu2+(aq) + 2e- → Cu(s) +

Cd°(s) + Cu+2(aq) → Cd+2(aq) + Cu(s) (Equação global da pilha) ΔG°T = - 2.F.ET°

ΔG°T = ΔG°Cd + ΔG°Cu


- 2.F.ET° = - 2.F.(+ 0,403) + {- 2.F.(+ 0,337)}
ET° = + 0,403 + 0,337
ET° = + 0,740 V

Item b) Eletrodo positivo: eletrodo de Cádmio; Ânodo: Cádmio e Metal que se reduz: Cu+2.

Questão 1242 – (IME) Em uma pilha Ni0 / Ni2+ // Ag+1 / Ag0, os metais estão mergulhados em soluções aquosas 1,0
mol.L-1 de seus respectivos sulfatos, a 25°C. Determine:
a) a equação global da pilha;
b) o sentido do fluxo de elétrons;
c) o valor da força eletromotriz (fem) da pilha.
Dados:
Ni2+(aq) + 2e- → Ni0(s) (E0redução = - 0,25 V)
Ag+(aq) + 1e- → Ag0(s) (E0redução = + 0,80 V)

Resolução:

Item a) Invertendo a semirreação do níquel: Ni°(s) → Ni2+(aq) + 2e- ΔG1° = - 2.F.(+ 0,25)

Multiplicando a semirreação da prata por 2: 2 Ag+(aq) + 2e- → 2 Ag°(s) ΔG2° = - 2.F.(+ 0,80)

Equação global:
Ni°(s) → Ni2+(aq) + 2e-
2 Ag+(aq) + 2e- → 2 Ag°(s) +

Ni°(s) + 2 Ag+(aq) → Ag°(s) + Ni2+(aq) (Equação global da pilha) ΔG°Total = - 2.F.E°

Item b) O sentido do fluxo de elétrons sai do eletrodo de Níquel (ânodo) para o eletrodo de Prata (cátodo).

Item c)
ΔG°Total = ΔG1°Ni + ΔG2°Ag
- 2.F.E° = - 2.F.(+ 0,25) + {- 2.F.(+ 0,80)}
E° = + 0,25 + 0,80 = + 1,05 V

676
1600 FÍSICO-QUÍMICA APLICADA EXERCÍCIOS COMENTADOS - IME – ITA – OLIMPÍADA

Questão 1243 - (IME) Um certo fabricante produz pilhas comuns, nas quais o invólucro de zinco funciona como anodo,
enquanto que o catodo é inerte. Em cada uma, utilizam-se 5,87 g de dióxido de manganês, 9,2 g de cloreto de amônio
e um invólucro de zinco de 80 g. As semirreações dos eletrodos são:
Zn → Zn+2 + 2e-
NH 4+ + MnO2 + e- → 1/2 Mn2O3 + NH3 + ½ H2O
Determine o tempo que uma destas pilhas leva para perder 50% de sua carga, fornecendo uma corrente constante
de 0,08 A.

Resolução: Determinação da equação global da pilha, multiplicando a segunda semirreação por dois, temos:
Zn → Zn+2 + 2e-
2 NH 4+ + 2 MnO2 + 2e- → 1 Mn2O3 + 2 NH3 + 1 H2O +

Zn + 2 NH4+ + 2 MnO2 → Zn2+ + Mn2O3 + 2 NH3 + H2O


Logo, a proporção reacional é 1 mol de Zn, 2 mol de NH4Cl e 2 mol de MnO2. Com isso, para estes dados dos
reagentes, será necessário determinar o reagente limitante.
80
Cálculo do número de mol de zinco: nZn = = 1,22 mol
65,4

9,2 0,172
Cálculo do número de mol de cloreto de amônio: nNH4 Cl = 53,5 = 2
= 0,086 mol

0,0675
Cálculo do número de mol de dióxido de manganês: nMnO2 = 2
= 3,37 x 10−2 mol

Com base do número de mol de cada reagente, o dióxido de manganês (MnO2) é o reagente limitante.
Relacionando com a carga, e considerando o consumo de metade da quantidade de MnO2:
2 mol de MnO2 ------------------------------------- (2 mol de elétrons x 96500 C/mol de elétrons)
2 x 86,9 g de MnO2 -------------------------------- 2 x 96500 C
0,5 x 5,87 g de MnO2 ------------------------------ 0,08 x t
0,5 x 5,87 x 96500
t= = 40740,43 s (11,30 horas)
0,08 x 86,9
Questão 1244 – (IME) Para uma experiência em laboratório montou-se uma pilha com 100 ml de solução eletrolítica
e eletrodos de prata e níquel, a qual descarregou-se totalmente em 9650 segundos, apresentando uma perda de
massa de 1,475 g em um dos eletrodos.
a) A concentração inicial do eletrólito;
b) A corrente que percorreu o circuito;
c) O eletrodo positivo da pilha. Dados a 250C:
Ni2+(aq) + 2 e- → Ni(s)  0 Ni2+ ,Ni = - 0,25 V
Ag+(aq) + l e- → Ag(s)  0 Ag +
, Ag = + 0,80 V

Resolução:

Invertendo a primeira semirreação: Ni(s) → Ni2+(aq) + 2e- E°= + 0,25 V

Multiplicando a segunda semirreação: 2 Ag+(aq) + 2 e- → 2 Ag(s) E°= + 0,80 V

Somando as duas semirreações apresentadas acima:


Ni(s) → Ni2+(aq) + 2e- E°= + 0,25 V
2 Ag+(aq) + 2 e- → 2 Ag(s) E°= + 0,80 V +

677
1600 FÍSICO-QUÍMICA APLICADA EXERCÍCIOS COMENTADOS - IME – ITA – OLIMPÍADA

Ni(s) + 2 Ag+(aq) → Ni2+(aq) + 2 Ag(s) E°= + 1,05 V

Cálculo da corrente que percorre o circuito:


1 mol de Ni(s) ---------------------------------- 2 mol de elétrons x 96500 C/mol de elétrons
58,71 g de Ni(s) ------------------------------- 2 mol de elétrons x 96500 C/mol de elétrons
1,475 g g de MnO2 --------------------------- 9650 x i
i = 0,502 A
1,475
Item a) Cálculo da concentração da quantidade de matéria, M: [M] = 0,100 x 58,71

1,475 mol
[M] = = 0,251
5,871 L

Item b) i = 0,502 A
Item c) O eletrodo positivo (cátodo) é a prata.

Questão 1245 - (IME) Calcule a intensidade da corrente elétrica que deve ser utilizada para depositar 2,54 x 10 -4 kg
de cobre, de uma solução de sulfato de cobre, no tempo de 3 minutos e 20 segundos.

Resolução: CuSO4(s) → Cu+2(aq) + SO4-2(aq)


No catodo, a competição é entre o íon cobre e a água. De acordo com a escala de redução, o cobre tem uma maior
tendência a se reduzir.
Reação catódica: Cu+2(aq) + 2 e- → Cu(s)
No anodo, a competição é entre o íon sulfato e a água. De acordo com a escala de oxidação, a água predomina sobre
ânions oxigenados, logo:
Reação anódica: H2O → 2H+ (aq) + ½ O2 + 2 e-
Unindo as duas reações com a reação de decomposição do sulfato de cobre, temos:
CuSO4(s) → Cu+2(aq) + SO4-2(aq)
Cu+2(aq) + 2 e- → Cu(s)
H2O(l) → 2H+ (aq) + ½ O2(g) + 2 e- +
CuSO4(s) + H2O(l) → 2H+ + ½ O2(g) + Cu + SO4-2(aq)

Cálculo da carga (Q):


63,50 g de cobre --------------------- 2 mol de elétrons x 96500 C/mol de elétrons
2,54 x 10-4 x 1000 g ----------------- Q
Q = 772,0 C
60 s
Convertendo o tempo de horas para segundos: t = 3 min x 1 min
+ 20 s = 200 s
𝑄 772
Cálculo da corrente elétrica (i): i = 𝑡
= 200
= 3,86 A

678
1600 FÍSICO-QUÍMICA APLICADA EXERCÍCIOS COMENTADOS - IME – ITA – OLIMPÍADA

Questão 1246 – (IME) Em duas cubas eletrolíticas, ligadas em série, ocorrem as reações, cujas equações são
mostradas a seguir, pela passagem de uma corrente elétrica de 1 Ampére:
Cuba A: Ag+(aq) + 1e- → Ag(s)
Cuba B: 2 H+(aq) + 2e- → H2(g)
Pede-se:
a) o tipo de reação que está ocorrendo;
b) a denominação do eletrodo onde ocorrem essas reações;
c) o tempo necessário para que ocorra a deposição de 1,08 g de prata;
d) O volume, em litros nas CNTP, do hidrogênio produzido durante o tempo determinado na letra C.

Resolução:
Item a) O tipo de reação que ocorre tanto na cuba A quanto na cuba B apresentadas no exercício são reações de
redução.
Item b) Reações de redução = cátodo

Item c) Cálculo do tempo para a deposição de 1,08 g de prata:

108 g de Ag --------------------- 1 mol de elétrons x 96500 C/mol de elétrons


1,08 g de Ag -------------------- Q = i x tempo

108 x 1 x tempo = 1,08 x 1 x 96500


tempo = 965 s

Item d) Cálculo do volume em litros de H2, nas CNTP:

1 mol de H2 ---------- 22,4 L --------------------- 2 mol de elétrons x (96500 C/mol de elétrons)


VH2 ---------------------- 965 C

965 𝑥 22,4
𝑉𝐻2 = = 0,112 L
2 𝑥 96500

Questão 1247 – (IME) Uma bateria de automóvel apresenta as seguintes reações eletrodos durante a descarga:
no ânodo: Pb(s) + SO42-(aq) → PbSO4(s) + 2e-
no cátodo: PbO2(s) + 4 H+(aq) + SO42-(aq) + 2e- → PbSO4(s) + 2 H2O(l)
A solução inicial de ácido sulfúrico contido na bateria tem uma concentração de 40%, em massa, de ácido sulfúrico e
massa específica de 1,3 g/cm3. Após a bateria ter sido utilizada, a solução foi analisada e apresentou uma
concentração de 28%, em massa, de ácido sulfúrico com uma massa específica de 1,2 g/cm 3. Considerando fixo o
volume da solução ácida na bateria em 2,0 litros, determine o valor da carga fornecida pela bateria em ampère-hora.
Resolução: Cálculo da massa de ácido sulfúrico na etapa inicial:
40 g g 1 dm³ 1000 cm³
mInicial
H2 SO4 = x 1,30 x 2,0 L x x = 1040 g
100 g cm³ 1L 1 dm³
Cálculo da massa de ácido sulfúrico na etapa final:
28 g g 1 dm³ 1000 cm³
mFinal
H2 SO4 = x 1,20 x 2,0 L x x = 672 g
100 g cm³ 1L 1 dm³

Balanço de massa para o ácido sulfúrico: m = mInicial Final


H2 SO4 − mH2 SO4 = 1040 − 672 = 368 g

679
1600 FÍSICO-QUÍMICA APLICADA EXERCÍCIOS COMENTADOS - IME – ITA – OLIMPÍADA

Determinação da reação global:

no ânodo: Pb(s) + SO42-(aq) → PbSO4(s) + 2e-


no cátodo: PbO2(s) + 4 H+(aq) + SO42-(aq) + 2e- → PbSO4(s) + 2 H2O(l) +

Pb(s) + SO42-(aq) + PbO2(s) + 4 H+(aq) + SO42-(aq) + 2e- → 2 PbSO4(s) + 2 H2O(l) + 2e-

Na reação global, há no reagente 2 mol de ácido sulfúrico (4 H+(aq) + SO42-(aq)= 2H2SO4), logo:

Cálculo da carga (Q):

2 mol x 98 g/mol de H2SO4 -------------- 2 mol de elétrons x (96500 C/mol de elétrons)


368 g ---------------------------------------- Q

368 x 2 x 96500 71024000


Q= = = 362367,35 C
2 x 98 196
1h
Realizando a conversão: Q = 362367,35 A. s x 3600 s = 100,66 A. s

Questão 1248 – (IME) Para se recuperar o níquel, em sua forma metálica, de uma solução contendo íons Ni +2,
introduziu-se na mesma uma barra de estanho metálico. Responda:
a) O processo descrito pode ocorrer sem a participação de um agente externo ao meio reacional? Justifique.
b) Qual a ordem de grandeza da constante de equilíbrio para a reação descrita no problema (a 27°C)?
Dados: Potenciais de redução padrão a 27ºC:
Sn+2(aq) + 2e- = Sn(s) 0 = - 0,14 V
Ni (aq) + 2e = Ni(s)
+2 - 0 = - 0,25 V

Resolução:
Item a) Não. Pelo fato que sem a participação de um agente externo o processo irá apresentar uma variação da
energia livre de Gibbs maior que zero (ΔG > 0), ou seja, um processo não espontâneo.

Item b) Cálculo do potencial-padrão (E°):

Invertendo a semirreação do níquel: Ni(s) → Ni2+(aq) + 2e- ΔG° = - 2.F.(+ 0,25)


Semirreação de redução do estanho: Sn+2(aq) + 2e- → Sn(s) ΔG° = - 2.F.(- 0,14)

Determinação da reação global da pilha:


Ni(s) → Ni2+(aq) + 2e-
Sn+2(aq) + 2e- → Sn(s) +
Ni(s) + Sn+2(aq) → Ni+2(aq) + Sn(s) (Processo global da pilha) ΔG°T = - 2 x F x E°

ΔG°T = ΔG°Ni + ΔG°Sn


- 2.F.E° = - 2.F.(+ 0,25) + {- 2.F.(- 0,14)}
E° = + 0,25 - 0,14 = + 0,11 V
n x F x E°
Cálculo da constante de equilíbrio (Keq): lnK eq = RxT

2 x 96500 x 0,11 21230


lnK eq = = = 8,51
8,314 x 300 2494,2
K eq = e8,51 = 4964,16 J

Questão 1249 - (IME) Dadas as reações de meia célula:


Cu2+ + e- → Cu+ E0 = + 0,153 V

680
1600 FÍSICO-QUÍMICA APLICADA EXERCÍCIOS COMENTADOS - IME – ITA – OLIMPÍADA

I2 + 2e- → 2I- E0 = + 0,536 V


pede-se:
a) Escrever a equação que representa a reação global da célula;
b) Calcular o potencial de eletrodo global (E0);
c) Calcular a energia livre para a reação (G0), considerando que 1 mol de elétrons percorreu a célula eletroquímica.

Resolução: Item a) Reação global do processo eletroquímico:

Invertendo a semirreação do cobre e multiplicando por 2: 2 Cu+(aq) → 2 Cu+2(aq) + 2e- ΔG° = - 2.F.(- 0,153)
Semirreação de redução do Iodo: I2(g) + 2e- → 2 I-(aq) ΔG° = - 2.F.(+ 0,536)

Somando as duas semirreações:


2 Cu+(aq) → 2 Cu+2(aq) + 2e-
I2(g) + 2e- → 2 I-(aq) +
2 Cu+(aq) + I2(g) → 2 I-(aq) + 2 Cu+2(aq) (Equação global da pilha) ΔG°T = - 2.F.E°

Item b) Cálculo do potencial padrão da pilha:


2 Cu+(aq) → 2 Cu+2(aq) + 2e- ΔG° = - 2.F.(- 0,153)
I2(g) + 2e- → 2 I-(aq) ΔG° = - 2.F.(+ 0,536)

ΔG°T = ΔG°Cu + ΔG°I


- 2.F.E° = - 2.F.(- 0,153) + {- 2.F.(+ 0,536)}
E° = - 0,153 + 0,536
E° = + 0,383 V

Item c) Calcular a energia livre para a reação (G0):


96500 C
∆G0 = − n x F x E° = − 1 mol de elétrons x x (+ 0,383) = − 36959,5 J (−36,96 kJ)
mol de elétrons

Questão 1250 – (ITA) Uma célula eletroquímica fechada, usada para a produção da mistura gasosa de H2 e O2,
possui dois eletrodos inertes mergulhados numa solução diluída de hidróxido de sódio. No volume livre de 4,50 litros
acima da solução, há uma válvula e um manômetro. A temperatura da célula é mantida constante em 27°C. No início
da eletrólise, a válvula é fechada, sendo fornecida a célula uma corrente de 30 A. Calcule o tempo para a leitura do
manômetro atingir 1,64 atmosferas.
Resolução: Para a produção de H2 e O2 será necessário analisar as reações no ânodo e no cátodo.

Reação no ânodo: 2 OH-(aq) → H2O + ½ O2(g) + 2e-


Reação no cátodo: 2 H2O + 2e- → H2(g) + 2 OH-(aq)

Somando as duas semirreações eletroquímicas, temos:

2 OH-(aq) → H2O + ½ O2(g) + 2e-


2 H2O + 2e- → H2(g) + 2 OH-(aq) +
2 H2O → H2O + H2(g) + ½ O2(g)

Observa-se que há a produção de 1,50 mol de gases, a partir de dois mol de elétrons, provenientes da eletrólise da
água.

Cálculo da pressão de gás no cilindro: Pcilindro = Pmanômetro - Pbarômtero = 1,64 atm – 1,00 atm = 0,64 atm

Cálculo do número de mol dos gases no cilindro: 0,64 x 4,50 = ngases x 0,08206 x 300
0,64 x 4,50 2,88
𝑛𝑔𝑎𝑠𝑒𝑠 = = = 0,117 mol
0,08206 x 300 24,62

681
1600 FÍSICO-QUÍMICA APLICADA EXERCÍCIOS COMENTADOS - IME – ITA – OLIMPÍADA

Cálculo da carga (Q) de gases produzido:

1 mol de gases ----------------- 2 mol de e- x (96500/mol de elétrons)


0,117 mol de gases ------------ Q
Q = 15054 C
15054
Cálculo do tempo, para uma corrente igual de 30 A: 30 = t
tempo = 501,8 s

Questão 1251 – (IME) Com base nos potenciais de redução (E°red) disponíveis abaixo, determine a constante de
equilíbrio para a oxidação do íon Fe+2 por oxigênio, a 25°C, em meio ácido, de acordo com a reação: O2(g) + 4 H+(aq) +
4 Fe+2(aq) → 4 Fe+3(aq) + 2 H2O(l). Dados:
O2(g) + 4 H+(aq) + 4e- → 2 H2O(l) E°red = + 1,23 V
Fe+2(aq) + 2e- → Fe(s) E°red = - 0,450 V
Fe+3(aq) + 3e- → Fe(s) E°red = - 0,0430 V
Resolução: Cálculo do potencial entre as semirreações do Fe+2 e Fe+3:

Multiplicando a semirreação de redução do Fe+2 por quatro, temos: 4 Fe+2(aq) + 8e- → 4 Fe(s) ΔG° = -
8.F.(- 0,450)
Invertendo a semirreação de redução do Fe+3 e multiplicando por quatro, temos: 4 Fe(s) → 4 Fe+3(aq) + 12e-
ΔG° = - 12.F.(+ 0,0430)

Somando as duas semirreação:

4 Fe+2(aq) + 8e- → 4 Fe(s) ΔG° = - 8.F.(- 0,450)


4 Fe(s) → 4 Fe+3(aq) + 12e- ΔG° = - 12.F.(+ 0,0430) +

4 Fe+2(aq) + 8e- → 4 Fe+3(aq) + 12e-

4 Fe+2(aq) → 4 Fe+3(aq) + 4e- (Equação global 1) ΔG° = - 8.F.(- 0,450) + [- 12.F.(+ 0,0430)]

Somando a equação global 1 com a primeira semirreação proposta pelo problema, temos:
4 Fe+2(aq) → 4 Fe+3(aq) + 4e- ΔG° = - 8.F.(- 0,450) + [- 12.F.(+ 0,0430)]
O2(g) + 4 H+(aq) + 4e- → 2 H2O(l) ΔG° = - 4.F.(+ 1,23) +

O2(g) + 4 H+(aq) + 4e- + 4 Fe+2(aq) → 4 Fe+3(aq) + 2 H2O(l) + 4e-


ΔGT° = - 8.F.(- 0,450) + [- 12.F.(+ 0,0430)] + {- 4.F.(+ 1,23)}
- 4.F.E° = - 8.F.(- 0,450) + [- 12.F.(+ 0,0430)] + {- 4.F.(+ 1,23)}
- 4 x E° = - 8 x (- 0,450) – 12 x (+ 0,0430) – 4 x (+ 1,23)
- 4 x E° = + 3,6 – 0,516 – 4,92

− 1,836
E° = = +0,459 V
−4

Cálculo da energia livre de Gibbs padrão (G0): ∆G0 = − n x F x E°

96500 C
∆G0 = − 4 mol de elétrons x x (+0,459) = −177174 J (−177,17 kJ)
mol de elétrons
Cálculo da constante de equilíbrio (Keq): ∆G0 = − R x T x lnK eq

682
1600 FÍSICO-QUÍMICA APLICADA EXERCÍCIOS COMENTADOS - IME – ITA – OLIMPÍADA

(−177174) + 177174
lnK eq = − = = 71,51
8,314 x (25 + 273) 2477,572
K eq = 𝑒 +71,51

As questões 1252 e 1253 estão relacionadas a seguinte reação química: 2 Cr(s) + 3 Cu2+(aq) → 2 Cr3+(aq) + 3 Cu(s)
E˚ = + 0,43 V

Questão 1252 – (U. S. NATIONAL CHEMISTRY OLYMPIAD) Which expression gives the value for ∆G˚ in kJ.mol–1
for this reaction at 25°C?
a) −6 x 8,31 x 0,43 x 1000
−6 x 96500 x 0,43 x 1000
b)
8,31
−6 x 96500 x 0,43
c) 1000
−6 x 8,31 x 0,43
d) 1000

Resolução: Alternativa C.

A expressão que representa a variação da energia livre de Gibbs padrão em relação ao processo eletroquímico é dado pela
seguinte equação matemática:

J
∆G° = − n x F x E° = −6 x 96500 x 0,43 ( )
mol
−6 x 96500 x 0,43 kJ
Dividindo a expressão por 1000, temos: ∆G° = 1000
(mol)

Questão 1253 - (U. S. NATIONAL CHEMISTRY OLYMPIAD) What is the voltage for this cell when [Cu2+] = 1,0 mol.L-
1 and [Cr3+] = 0,010 mol.L-1?

a) 1,2
b) 0,87
c) 0,47
d) 0,39

Resolução: Alternativa C.
2 3
RxT RxT [Cr+3 ] x 𝑎𝐶𝑢
Aplicando a equação de Nernst, temos: E = E° − n x F x lnQ = E° − n x F x ln {𝑎2 x [Cu+2 ]3
}
𝐶𝑟
Onde as atividades dos sólidos são unitários, aCr = aCu = 1.

RxT [Cr +3 ]2 x a3Cu


E = E° − x ln { 2 }
nxF aCr x [Cu+2 ]3

8,314 x (25 + 273) (0,010)2 x (1)3 8,314 x 298


E = + 0,43 − x ln { } = + 0,43 − x ln(10−4 )
6 x 96500 (1)2 x (1)3 6 x 96500
= + 0,43 + 0,039 = +0,469 V
Questão 1254 – (IME) Dada a reação Cu + 2HCl → CuCl2 + H2 , assinale a afirmativa correta sabendo-se que os
potenciais-padrão de redução do cobre e do hidrogênio são respectivamente 0,34 V e 0,00 V.
a) A reação produz corrente elétrica.
b) A reação não ocorre espontaneamente.
c) A reação ocorre nas pilhas de Daniell.
d) O cobre é o agente oxidante.
e) O hidrogênio sofre oxidação.

683
1600 FÍSICO-QUÍMICA APLICADA EXERCÍCIOS COMENTADOS - IME – ITA – OLIMPÍADA

Resolução: Alternativa B.

O cobre metálico não reage com o ácido clorídrico, ou seja, não há formação de produtos. Diante disso, o processo
continua sendo não espontâneo, apresentando uma ddp igual a -0,34 V.

Questão 1255 – Calcule a constante de equilíbrio (25°C), para a seguinte semirreação: Sn(s) + Sn+4(aq) → 2 Sn+2(aq).
Dados dos potencias a 25°C:
Sn+4(aq) + 2e- → Sn+2(aq) E1° = + 0,15 V
Sn+2(aq) + 2e- → Sn(s) E2° = - 0,14 V
Resolução: Analisando a primeira semirreação:
Sn+4(aq) + 2e- → Sn+2(aq) E1° = + 0,15 V ΔG°1 = -2.F.(+ 0,15)
Analisando a segunda semirreação, invertendo a segunda semirreação, temos:
Sn(s) → Sn+2(aq) + 2e- E2° = + 0,14 V ΔG°2 = -2.F.(+ 0,14)
Determinação da reação global:
Sn+4(aq) + 2e- → Sn+2(aq) E1° = + 0,15 V ΔG°1 = -2.F.(+ 0,15)
Sn(s) → Sn+2(aq) + 2e- E2° = + 0,14 V ΔG°2 = -2.F.(+ 0,14) +
Sn+4(aq) + 2e- + Sn(s) → Sn+2(aq) + Sn+2(aq) + 2e-
Sn+4(aq) + Sn(s) → 2 Sn+2(aq) ΔG°T = -2.F.(E°)
Cálculo do potencial padrão da reação global (E°): ΔG°T = ΔG°1 + ΔG°2
-2.F.(E°) = -2.F.(+ 0,15) + [-2.F.(+ 0,14)]

E° = + 0,29 V

Cálculo da constante de equilíbrio (Keq): ΔG°Termodinâmico = ΔG°Eletroquímico


− n x F x E° = − R x T x lnK eq
n x F x E° 2 x 96500 x 0,29 55970
lnK eq = = = = 22,59
RxT 8,314 x 298 2477,572
K eq = e22,59

Questão 1256– Represente a eletrólise aquosa do cloreto de sódio.


Resolução:

⏞ Na+
Reação de ionização do cloreto de sódio: NaCl(aq) → −
(aq) + Cl(aq)

+ −
Reação de ionização da água: H2 O(l) → H(aq) + OH(aq)

Quem descarrega com melhor facilidade no cátodo, Na+ + + −


(aq) ou H(aq) ? 2H(aq) + 2e → H2(g)

Quem descarrega com melhor facilidade no ânodo, Cl− − −


(aq) ou OH(aq) ? 2Cl(aq) → Cl2 (𝑔) + 2𝑒

Multiplicando a primeira e a segunda semirreação por dois e somando todas as reações, temos:

2 NaCl(aq) → 2 Na+ −
(aq) + 2 Cl(aq)

684
1600 FÍSICO-QUÍMICA APLICADA EXERCÍCIOS COMENTADOS - IME – ITA – OLIMPÍADA

+ −
2H2 O(l) → 2H(aq) + 2OH(aq)
+
2H(aq) + 2e− → H2(g)

2Cl−
(aq) → Cl2 (𝑔) + 2𝑒

+

+ −
2 NaCl(aq) + 2H2 O(l) → 2
⏟Na(aq) + 2OH(aq) + H2(g) + Cl2 (g)
2 NaOH(aq)

Observação: Esta é a forma eletroquímica para a produção de hidróxido de sódio (soda cáustica) a partir do cloreto
de sódio, havendo a liberação dos gases H2(g) e Cl2(g).
Questão 1257 – Represente a eletrólise aquosa do cloreto de cálcio.
Resolução:

⏞ Ca+2
Reação de ionização do cloreto de cálcio: CaCl2 (aq) → −
(aq) + 2Cl(aq)
+ −
Reação de ionização da água: H2 O(l) → H(aq) + OH(aq)

Quem descarrega com melhor facilidade no cátodo, Ca+2 + + −


(aq) ou H(aq) ? 2H(aq) + 2e → H2(g)

Quem descarrega com melhor facilidade no ânodo, Cl− − −


(aq) ou OH(aq) ? 2Cl(aq) → Cl2 (𝑔) + 2𝑒

Multiplicando a segunda semirreação por dois e somando todas as reações, temos:

CaCl2 (aq) → Ca+2 −


(aq) + 2Cl(aq)

+ −
2H2 O(l) → 2H(aq) + 2OH(aq)
+
2H(aq) + 2e− → H2(g)

2Cl−
(aq) → Cl2 (𝑔) + 2𝑒

+

Ca+2
CaCl2 (aq) + 2H2 O(l) → ⏟ −
(aq) + 2OH(aq) + H2(g) + Cl2 (g)
2 NaOH(aq)

Observação: Na eletrólise do cloreto de cálcio há a formação dos gases hidrogênio e cloro além da produção de
hidróxido de cálcio.
Questão 1258 – Represente a eletrólise aquosa do brometo de cobre.
Resolução:

⏞ Cu+2
Reação de ionização do brometo de cobre: CuBr2 (aq) → −
(aq) + 2Br(aq)
+ −
Reação de ionização da água: H2 O(l) → H(aq) + OH(aq)

Quem descarrega com melhor facilidade no cátodo, Cu+2 + +2 −


(aq) ou H(aq) ? Cu(aq) + 2e → Cu(s)

− − −
Quem descarrega com melhor facilidade no ânodo, Br(aq) ou OH(aq) ? 2Br(aq) → Br2 (𝑙) + 2𝑒 −

Multiplicando a segunda semirreação por dois e somando todas as reações, temos:

CuBr2 (aq) → Cu+2 −


(aq) + 2Br(aq)

+ −
H2 O(l) → H(aq) + OH(aq)

Cu+2 −
(aq) + 2e → Cu(s)

685
1600 FÍSICO-QUÍMICA APLICADA EXERCÍCIOS COMENTADOS - IME – ITA – OLIMPÍADA


2Br(aq) → Br2 (𝑙) + 2𝑒 − +

+ −
CuBr2 (aq) + H2 O(l) → H
⏟(aq) + OH(aq) + Cu(s) + Br2 (l)
2 NaOH(aq)

Observação: A eletrólise do brometo de cobre produz cobre metálico [Cu(s)].


Questão 1259 – Represente a eletrólise aquosa do sulfato de cálcio.
Resolução:

−2
⏞ Ca+2
Reação de ionização do cloreto de cálcio: CaSO4 (aq) → (aq) + SO4 (𝑎𝑞)
+ −
Reação de ionização da água: H2 O(l) → H(aq) + OH(aq)

Quem descarrega com melhor facilidade no cátodo, Ca+2 + + −


(aq) ou H(aq) ? 2H(aq) + 2e → H2(g)

Quem descarrega com melhor facilidade no ânodo, SO4 −2 −


(𝑎𝑞) ou OH(aq) ?

− 1
2OH(aq) → H2 O(l) + O2(g) + 2e−
2
Multiplicando a segunda semirreação por dois e somando todas as reações, temos:
−2
CaSO4 (aq) → Ca+2
(aq) + SO4 (𝑎𝑞)
+ −
2H2 O(l) → 2H(aq) + 2OH(aq)
+ −
2H(aq) + 2e → H2(g)
− 1
2OH(aq) → H2 O(l) + 2 O2(g) + 2e− +
+2 −2 1
CaSO4 (aq) + 2 H2 O(l) → Ca
⏟ (aq) + SO4 (aq) + H2 O(l) + H2(g) + 2 O2(g)
CaSO4 (aq)

+2 −2 1
CaSO4 (aq) + H2 O(l) → Ca
⏟ (aq) + SO4 (aq) + H2(g) + 2 O2(g)
CaSO4 (aq)

1
H2 O(l) → H2(g) + O2(g)
2
Questão 1260 – As reações envolvidas no processo de oxirredução e seus respectivos potenciais padrões de redução
são os seguintes:
PbSO4(s) + 2e- → Pb(s) + SO42-(aq) Eo = - 0,36 V
PbO2(s) + 4 H (aq) + SO4 (aq) + 2e → PbSO4(s) + 2 H2O(l)
+ 2- - Eo = + 1,64 V
a) Escreva a reação de oxirredução global da pilha.
b) Calcule o valor da variação da energia livre de Gibbs padrão, Go. Esse resultado é esperado para uma
pilha? Por quê?
c) Calcule G, a 25oC quando o pH é 1 e a concentração de sulfato, [SO42-] é de 5,0 x 10-3 mol.L-1.

Resolução:

Item a) Invertendo a primeira semirreação, temos:


Pb(s) + SO42-(aq) → Pb SO4(s) + 2e- E° = + 0,36 V
PbO2(s) + 4H+(aq) + SO42-(aq) + 2e- → PbSO4(s)+ 2 H2O(l) E° = + 1,64 V
Pb(s) + PbO2(s) + 2 SO42-(aq) + 4H+(aq) → 2 PbSO4(s)+ 2 H2O(l) E° = + 0,36 + 1,64 = + 2,00 V

Item b) Cálculo da variação de energia livre de Gibbs padrão, ΔG°:

686
1600 FÍSICO-QUÍMICA APLICADA EXERCÍCIOS COMENTADOS - IME – ITA – OLIMPÍADA

∆G° = −n x F x E°
C
∆G° = −2 mol x 96500 x (+2,00 V)
mol
∆G° = −386000 J

Sim, o resultado é esperado, uma vez que caracteriza a espontaneidade da pilha apresentando um valor negativo
(ΔG° < 0 e E° > 0), o que se trata de uma das principais características da pilha.

Item c) Cálculo da concentração da quantidade de matéria de [H+], a partir do pH: pH = - log[H+]


1 = - log[H+]
[H+] = 10-1 mol.L-1

Utilizando a equação termodinâmica que relaciona a energia livres de Gibbs, temos: ∆G = ∆G° + RTlnQ

a2PbSO4 x a2H2 O
∆G = ∆G° + RTln { }
aPb x aPbO2 x [SO4 −2 ]² x [H + ]4

(1)² x (1)²
∆G = −386000 + 8,314 x (25 + 273) x ln { }
1 x 1 x (5,0 x 10−3 )2 x (10−1 )4

1
∆G = −386000 + 8,314 x 298 x ln { } = −336926,79 J (−336,93 kJ)
25,0 x 10−10

Questão 1261 – Represente a eletrólise aquosa do nitrato de prata.

Resolução:

Reação de ionização do nitrato de prata: AgNO3(aq) → Ag + −


(aq) + NO3(aq)

+ −
Reação de ionização da água: H2 O(l) → H(aq) + OH(aq)
Quem descarrega com maior facilidade no cátodo, H(aq) ou Ag +
+ + −
(aq) ? Resposta: Ag (aq) + 1e → Ag (s)
1
Quem descarrega com maior facilidade no ânodo, NO− − −
3(aq) ou OH(aq) ? Resposta: 2OH(aq) → H2 O(l) + 2 O2(g)

Multiplicando as duas primeiras semirreações por dois e somando todas as semirreações, temos:

2 AgNO3(aq) → 2Ag + −
(aq) + 2 NO3(aq)
+ −
2H2 O(l) → 2H(aq) + 2OH(aq)
2Ag + −
(aq) + 2e → 2Ag (s)
− 1
2OH(aq) → H2 O(l) + 2 O2(g) + 2𝑒 − +

+ − 1
2AgNO3(aq) + H2 O(l) → 2H
⏟ (aq) + 2 NO3(aq) + 2Ag (s) + 2 O2(g)
𝑝𝑟𝑜𝑑𝑢çã𝑜 𝑑𝑒
á𝑐𝑖𝑑𝑜 𝑛í𝑡𝑟𝑖𝑐𝑜

Questão 1262 – Represente a eletrólise ígnea do óxido de ferro III.

Resolução: Fe2 O3(s) → 2 Fe+3 −2


(l) + 3 O(l)

Semirreação que ocorre no cátodo: Fe+3 −


(l) + 3e → Fe(s)
1
Semirreação que ocorre no ânodo: O−2
(l) → 2 O2(l) + 2e

Multiplicando a semirreação do ânodo por dois: 2 O−2


(l) → O2(l) + 4e

687
1600 FÍSICO-QUÍMICA APLICADA EXERCÍCIOS COMENTADOS - IME – ITA – OLIMPÍADA

Para a determinação da reação global, será necessário multiplicar a semirreação do cátodo por quatro e a semirreação
do ânodo por três.
Multiplicando a semirreação que ocorre no cátodo por quatro: 4 Fe+3 −
(l) + 12e → 4 Fe(s)
−2
Multiplicando a semirreação do ânodo por três: 6 O(l) → 3 O2(g) + 12e −

4 Fe+3 −
(l) + 12e → 4 Fe(s)
−2
6 O(l) → 3 O2(g) + 12e− +
4 Fe+3
(l) + 6 O−2
(l) → 3 O2(g) + 4 Fe(s)

Questão 1263 – Represente a eletrólise ígnea do cloreto de magnésio.

Resolução: MgCl2(s) → Mg +2 −
(l) + 2 Cl(l)

Semirreação que ocorre no cátodo: Mg +2 −


(l) + 2e → Mg (s)

Semirreação que ocorre no ânodo: 2 Cl(l) → Cl2(g) + 2e−
Somando as duas semirreações:
Mg +2 −
(l) + 2e → Mg (s)

2 Cl(l) → Cl2(g) + 2e− +
Mg +2 −
(l) + 2 Cl(l) → Mg (s) + Cl2(g)

Questão 1264 – Foi necessário uma hora (1 h) para se eletrolisar completamente 100 mL de uma solução de nitrato
de zinco com uma corrente de 3A. A partir desta informação, calcule a normalidade desta solução.

Resolução: Estudo da eletrólise do nitrato de zinco: Zn(NO3 )2(aq) → Zn+2
(aq) + 2 NO3 (aq)
Semirreação que ocorre no cátodo: Zn+2 −
(aq) + 2 e → Zn(s)

Cálculo da massa de zinco:


96500 C
65,41 g de Zn − − − − − − 2 mol de elétrons x
1 mol de elétrons
mZn − − − − − − − − − − 3 x 3600 C

65,41 x 3 x 3600 706428


mZn = = = 3,66 g
2 x 96500 193000
mZn 3,66 3,66
Cálculo da normalidade (N): N = eqg = 65,41 = 3,27 = 1,12 egg. L−1
Zn x Vsolução x 0,100
2

Questão 1265 - (IME) O alumínio pode ser produzido industrialmente pela eletrólise do cloreto de alumínio fundido, o
qual é obtido a partir do minério bauxita, cujo principal componente é o óxido de alumínio. Com base nas informações
acima, calcule quantos dias são necessários para produzir 1,00 tonelada de alumínio puro, operando-se uma cuba
eletrolítica com cloreto de alumínio fundido, na qual se faz passar uma corrente elétrica constante de 10,0 kA.

Resolução: Estudando a eletrólise ígnea do cloreto de alumínio: AlCl3(s) → Al+3 −


(l) + 3 Cl(l)
Semirreação que ocorre no cátodo: Al+3 −
(l) + 3e → Al(s)
Semirreação que ocorre no ânodo: 2 Cl− (l) → Cl2(g) + 2e

Multiplicando a primeira semirreação por 2 e multiplicando a segunda semirreação por três:

2 Al+3 −
(l) + 6e → 2Al(s)

6 Cl(l) → 3 Cl2(g) + 6e− +
2 Al+3 −
(l) + 6 Cl(l) → 2Al(s) + 3 Cl2(g)

688
1600 FÍSICO-QUÍMICA APLICADA EXERCÍCIOS COMENTADOS - IME – ITA – OLIMPÍADA

Observação: A eletrólise ígnea do cloreto de alumínio produz alumínio metálico e gás cloro.

Calculo da carga (Q) para a produção de alumínio metálico:

27,00 g de Alumínio ------------------- (3 mol de elétrons) x 96500 C/mol de elétrons


1,0 x 106 g de Alumínio --------------- Q
Q = 1,07 x 1010 C

1,07 x 1010
Cálculo do tempo em segundos (t), através da definição de corrente elétrica: 10000 = tempo

1,07 x 1010
tempo = = 1,07 x 106 s
10000

Este tempo em segundos corresponde 12,41 dias.

Questão 1266 – (IME) Em uma célula a combustível, reações de oxidação e redução originam a uma corrente que
pode ser aproveitada, por exemplo, para suprir a potência necessária para alimentar um motor elétrico. Considere um
sistema formado por uma célula a combustível que utiliza hidrogênio e oxigênio, acoplada ao motor de um veículo
elétrico. Sabendo que o sistema opera sem perdas, que a potência do motor é de 30 kW e que o comportamento do
gás (H2) é ideal, calcule a pressão em um tanque de 100 litros de hidrogênio, mantido a 27°C, de forma que esse
veículo percorra um trajeto de 100 km a uma velocidade média de 90 km.h-1. Dados a 27°C:
H2(g) → 2 H+(aq) + 2 e- E = 0,00 V
O2(g) + 4 H+(aq) + 4 e- → 2 H2O(l) E = + 1,23 V

Resolução: Informação para a resolução do problema, consideração T = 27°C:


H2(g) → 2 H+(aq) + 2 e- E = 0,00 V
O2(g) + 4 H+(aq) + 4 e- → 2 H2O(l) E = + 1,23 V

Cálculo da força eletromotriz da pilha (E°), multiplicando a primeira semirreação por dois, temos:
2 H2(g) → 4 H+(aq) + 4 e- ∆G° = −4 x F x (+0,00)
O2(g) + 4 H+(aq) + 4 e- → 2 H2O(l) ∆G° = −4 x F x (+1,23) +

2 H2(g) + O2(g) → 2 H2O(l)


−4 x F x E° = −4 x F x (+0,00) + [−4 x F x (+1,23)]
E° = + 1,23 V
Q
Cálculo da carga gerada através desta potência de 3000 W: P = V x i = V x
t

distância (d)
Sendo o tempo a razão da distância pela velocidade, temos: t = tempo (t)

100 km 3600 s
t= = 1,11 h x ( ) = 4000 s
km 1h
90
h
Q
Então: P = V x
t

Q
3000 = 1,23 x
4000

689
1600 FÍSICO-QUÍMICA APLICADA EXERCÍCIOS COMENTADOS - IME – ITA – OLIMPÍADA

Q = 9,76 x 107 C
Através da estequiometria, temos:
2 mol de H2 ---------- 4 mol de elétrons x (96500 C/mol de elétrons)
nH2 --------------------- 9,76 x 107 C

2 x 9,76 x 107 2 x 9,76 x 107


nH2 = = = 505,70 mol
4 x 96500 4 x 96500
nRT
Considerando que o gás hidrogênio apresenta comportamento ideal, temos: p= V
=
505,70 x 0,08206 x (27+273)
= 124,49 atm
100

Questão 1267 – (IME) Construiu-se uma célula eletrolítica de eletrodos de platina, tendo como eletrólito uma solução
aquosa de iodeto de potássio. A célula operou durante um certo intervalo de tempo sob corrente constante de 0,2 A.
Ao final da operação, o eletrólito foi completamente transferido para um outro recipiente e titulado com solução 0,1
mol.L-1 de tiossulfato de sódio. Sabendo-se que foram consumidos 25 mL da solução de tiossulfato na titulação,
determine o tempo durante o qual a célula operou. Dados para a resolução do problema: Constante de Faraday =
96500 C.
S4O6-2 + 2e- → 2 S2O3-2 E° = + 0,08 V
I2 + 2e- → 2I- E° = + 0,54 V
Resolução: Determinação da reação global invertendo a semirreação do tiossulfato, temos:

I2 + 2e- → 2 I- ∆G° = −2 x F x (+0,54)

2 S2O2-3 → 2e- + S4O2-6 ∆G° = −2 x F x (−0,08) +

I2 + 2 S2O2-3 → 2I- + S4O2-6


− 2 x F x E° = − 2 x F x (+0,54) + [−2 x F x (−0,08)]
− 2 x F x E° = − 2 x F x (+0,54) + [−2 x F x (−0,08)]
E° = +0,54 − 0,08 = +0,46 V
O sentido da reação foi escolhido de modo a reação fosse espontânea, ΔG° < 0, apresentando um potencial positivo,
E° > 0.
Cálculo da quantidade do número de mol (n) de tiossulfato utilizado, a partir da quantidade de matéria:
n
[tiossulfato] = tiossulfato
Vsolução

ntiossulfato = [tiossulfato] x Vsolução

ntiossulfato = 0,10 x 25 x 10−3 = 2,50 x 10−3 mol

Cálculo do número de mol de iodeto (I-): I2 + 2 S2O2-3 → 2 I- + S4O2-6


2 mol de S2O2-3 --------------------- 2 mol de I-
2,5 x 10-3 mol de S2O2-3 ----------- nI-
nI- = 2,5 x 10-3 mol de I-

Cálculo da carga (Q):


2 mol de I- -------------------- 2 mol de elétrons- x (96500 C / mol de elétrons)
2,5 x 10-3 mol de I- ---------- Q

690
1600 FÍSICO-QUÍMICA APLICADA EXERCÍCIOS COMENTADOS - IME – ITA – OLIMPÍADA

Q = 241,25 C
Q
Cálculo do tempo, a partir da equação da corrente elétrica: i = tempo
241,25
tempo = = 1206,25 s
0,20

Questão 1268– (IME) Deseja-se depositar uma camada de 0,85 g de níquel metálico no catodo de uma célula
eletrolítica, mediante a passagem de uma corrente elétrica de 5 A através de uma solução aquosa de nitrato de níquel.
Assinale a opção que apresenta o tempo necessário para esta deposição, em minutos.
a) 4,3
b) 4,7
c) 5,9
d) 9,3
e) 17,0

Resolução: Alternativa D.

Reação de dissociação do nitrato de níquel: Ni(NO3 )2(aq) → Ni+2 −


(aq) + 2 NO3(aq)

Semirreação que ocorre no cátodo: Ni+2 −


(aq) + 2 e → Ni(s)

Quantidade de carga (Q):

96500 C
58,70 g − − − − − − 2 mol de elétrons x
mol de elétrons
0,85 g − − − − − − Q

0,85 x 96500 x 2
Q= = 2794,72 C
58,70
Q 2794,72 C 1 min
Cálculo do tempo: tempo = i
= 5,0 A
= 558,94 s x 60 s
= 9,31 min

Questão 1269 – (U. S. NATIONAL CHEMISTRY OLYMPIAD) Alumínio metálico é obtido comercialmente por
eletrólise Al2O3, misturado com criolita (Na3AlF6).
a) Quantos mol de elétrons devem passar através da célula de alumínio para produzir 5,00 kg?
b) Determine a corrente elétrica em âmperes produzido em a de alumínio, em 10 horas.

Resolução:

Item a) Cálculo do número de mol de elétrons produzidos no cátodo:

Semirreação de redução do alumínio: Al+3(aq) + 3e- → Al(s)


Cálculo da carga (Q):
1 mol de Al ---------- 27,0 g de Al ---------------- 3 mol de elétrons
5000 g de Al --------------- n
n = 555,56 mol de elétrons

Item b) Cálculo da corrente (i):


3600𝑠
Convertendo o tempo para segundos: tempo = 10 h 𝑥 1ℎ
= 36000 𝑠

691
1600 FÍSICO-QUÍMICA APLICADA EXERCÍCIOS COMENTADOS - IME – ITA – OLIMPÍADA

Cálculo da carga (Q): Q = 555,56 mol de elétrons x (96500 C/mol de elétrons) = 5,36 x 107 C
Q 5,36 x 107
Cálculo da corrente (i): i = t
= 36000
= 1489,20 A

Questão 1270 – (IME) Calcular a massa mínima de reagentes para uma pilha seca que deve gerar 0,0l00 A durante
10 horas.
As semirreações são:
Zn → Zn+2 + 2e-
2 MnO2 + NH +4 + 2e- → Mn2O3 + NH3 + OH-

Resolução: Determinação da reação global da pilha:


Zn → Zn+2 + 2e-
2 MnO2 + NH +4 + 2e- → Mn2O3 + NH3 + OH- +
Zn + 2 MnO2 + NH +4 + 2e- → Zn+2 + 2e- + Mn2O3 + NH3 + OH-
Zn + 2 MnO2 + NH +4 → Zn+2 + Mn2O3 + NH3 + OH-

3600 s
Convertendo o tempo para segundos: tempo = 10 horas x = 36000 𝑠
1h

Cálculo da massa mínima de Zinco (Zn):


1 mol de Zn ---------- 65,4 g de Zn ------------------ 2 mol de elétrons x 96500 C/mol de elétrons
mZn --------------- (0,0100 x 36000) C
mZn = 0,122 g de Zn

Cálculo da massa mínima de dióxido de manganês (MnO2):


2mol x 86,90 g/mol de MnO2 --------------------- 2 mol de elétrons x (96500 C/mol de elétrons)
2mMnO2 ------------------------------------------------ (0,0100 x 36000) C

mMnO2 = 0,326 g de MnO2

Questão 1271 – (U. S. NATIONAL CHEMISTRY OLYMPIAD) Uma célula galvânica é baseada nas seguintes
semirreações:

Cr+3(aq) + 3e- → Cr(s) E° = - 0,744 V

Ni+2(aq) + 2e- → Ni(s) E° = - 0,236 V


a) Escreva a equação balanceada para a reação total.
b) Qual eletrodo tem a sua massa aumentada. Justifique sua resposta.
c) Calcule o valor E°célula.
d) Determine o valor ΔG° para a reação a 25°C.
e) Calcule o valor da constante de equilíbrio para a reação a 25°C.
f) Determine a voltagem da célula a 25°C se [Cr+3] e [Ni+2] são ambos alterados para 0,010 mol.L-1.

Resolução:

Item a) Analisando a primeira semirreação – Cromo - Invertendo a semirreação de cromo e multiplicando por dois,
temos:
2 Cr(s) → 2 Cr+3 (aq) + 6e- ΔG°1 = -6 x F x (+ 0,744)
Analisando a segunda semirreação – Níquel - Multiplicando a semirreação por três, temos:

692
1600 FÍSICO-QUÍMICA APLICADA EXERCÍCIOS COMENTADOS - IME – ITA – OLIMPÍADA

3 Ni+2(aq) + 6e- → 3 Ni(s) ΔG°2 = -6 x F x (- 0,236)


Determinação da reação global:
2 Cr(s) → 2 Cr+3 (aq) + 6e- ΔG°1 = -6 x F x (+ 0,744)
3 Ni+2(aq) + 6e- → 3 Ni(s) ΔG°2 = -6 x F x (- 0,236) +
2 Cr(s) + 3 Ni (aq) → 2 Cr (aq) + 3 Ni(s)
+2 +3 (Reação Global)
Item b) O eletrodo que apresenta a sua massa aumentada é o eletrodo de níquel (Ni).

Item c) Cálculo do potencial padrão da reação global (E°): ΔG°T = ΔG°1 + ΔG°2

-6.F.(ET°) = -6.F.(+ 0,744) + [-6.F.(- 0,236)]


E° = + 0,508 V

Item d) Cálculo da variação da energia livre de Gibbs (ΔG°):

96500 C
∆G° = −n x F x E° = −6 mol de elétrons x x (+0,508 V) = −294132 J (−294,13 kJ)
mol de elétrons

Item e) Cálculo da constante de equilíbrio (K) - Primeira maneira resolução: ΔG°Termodinâmico = ΔG°Eletroquímico

− n x F x E° − 6 x 96500 x 0,508 6 x 96500 x 0,508


lnK = = = = 118,72
−RxT − 8,314 x (25 + 273) 8,314 x 298

K = e(+118,72)
Item f) Cálculo do potencial (E), tilizando a Equação de Nernst, a partir da equação global:
Equação global da pilha: 2 Cr(s) + 3 Ni+2(aq) → 2 Cr+3 (aq) + 3 Ni(s)
RxT
E = E° − x lnQ
nxF
+3 2
RxT [Cr(aq) ] 8,314 x (25 + 273) (10−2 )2
E = E° − x ln { 3 } = 0,508 − x ln { }
nxF 6 x 96500 (10−2 )3
[Ni+2
(aq) ]

8,314 x 298 10−4 2477,572


E = 0,508 − x ln { −6 } = 0,508 − x ln (100)
579000 10 579000

E = 0,508 − 4,28 x 10−3 x 4,60 = + 0,488 V


Questão 1272 – Através das semirreações apresentadas abaixo, calcule E°, E e ∆G°, sabendo que [Al+3] = 0,020
mol.L-1 e [Ni+2] = 0,80 mol.L-1.
Al+3(aq) + 3e- → Al(s) E° = - 1,66 V

Ni+2(aq) + 2e- → Ni(s) E° = - 0,25 V


Resolução: Cálculo da ddp padrão, invertendo a primeira semirreação e também multiplicando por dois, além de
multiplicar a segunda semirreação por três:
2 Al(s) → 2 Al+3(aq) + 6e- ΔG°1 = -6 x F x (+ 1,66)
3 Ni+2(aq) + 6e- → 3 Ni(s) ΔG°2 = -6 x F x (- 0,25) +
2 Al(s) + 3 Ni (aq) → 2 Al (aq) + 3 Ni(s)
+2 +3 (Reação Global)
- 6 x F x E° = - 6 x F x (+ 1,66) + [- 6 x F x (-0,25)]

693
1600 FÍSICO-QUÍMICA APLICADA EXERCÍCIOS COMENTADOS - IME – ITA – OLIMPÍADA

- 6 x F x E° = - 6 x F x (+ 1,66) + [- 6 x F x (-0,25)]
E° = + 1,66 – 0,25 = + 1,41 V
RxT
Cálculo da ddp fora das condições padrões (E), a partir da equação de Nernst: E = E° − n x F x lnQ

2
RxT [Al+3
(aq) ]
E = E° − x ln { 3}
nxF [Ni+2 ] (aq)

8,314 x (25 + 273) (0,020)2


E = + 1,41 − x ln { }
6 x 96500 (0,80)3
2477,572 0,0004
E = + 1,41 − x ln { }
579000 0,512
E = + 1,41 − [4,28 x 10−3 x ln(0,00078)] = + 1,41 − [4,28 x 10−3 x (−7,16)] = + 1,41 + 0,031
= +1,44 V
Cálculo da variação da energia de Gibbs (∆G): ∆G = ∆G° + RTlnQ
2
[Al+3
(aq) ]
∆G = − n x F x E° + RTln { 3}
[Ni+2
(aq) ]

(0,020)²
∆G = − 6 x 96500 x 1,41 + 8,314 x (25 + 273) x ln { }
(0,80)3

∆G = − 816390 + 2477,572 x (−7,15) = − 816390 − 17714,64 = −834104,64 J (−834,10 kJ)

Observação: Mesmo para concentrações das quantidades de matéria fornecido pelo problema, o processo continua
sendo espontâneo, uma vez que ∆G < 0.

694
1600 FÍSICO-QUÍMICA APLICADA EXERCÍCIOS COMENTADOS - IME – ITA – OLIMPÍADA

Questão 1273 – Considere as seguintes semirreações de redução apresentadas abaixo:


Sn+2(aq) (0,01 mol.L-1) + 2e- → Sn(s) E° = - 0,14 V
Cu+2(aq) (0,50 mol.L-1) + 2e- → Cu(s) E° = +0,52 V
A partir dessas informações apresentadas, calcule E°, E, ∆G°, ∆G e Keq.

Resolução: Dados do problema:


Sn+2(aq) (0,01 mol.L-1) + 2e- → Sn(s) E° = - 0,14 V
Cu+(aq) (0,50 mol.L-1) + 1e- → Cu(s) E° = + 0,52 V

Cálculo do potencial padrão (E°), invertendo a primeira semirreação do estanho e multiplicando por dois (x2) a segunda
semirreação do cobre:

Sn(s) → Sn+2(aq) + 2e- ∆G° = −2 x F x (+0,14)


2 Cu (aq) + 2e → 2 Cu(s) ∆G° = −2 x F x (+0,52)
+1 - +

Sn(s) + 2 Cu+1(aq) → Sn+2(aq) + 2 Cu(s)


−2 x F x E° = −2 x F x (+0,14) + [−2 x F x (+0,52)]
−2 x F x E° = −2 x F x (+0,14) + [−2 x F x (+0,52)]
E° = +0,14 + 0,52 =+0,66 V
Cálculo da variação da energia livre de Gibbs padrão (∆G°): ∆G° = −n x F x E°
96500 C
∆G° = −2 mol de elétrons x x (+0,66) = −127380 J (−127,38 kJ)
mol de elétrons
RxT
Cálculo do potencial padrão fora das condições padrões (E), a partir da equação de Nernst: E = E° − n x F x lnQ

RxT [Sn+2
(aq) ]
E = E° − x ln { 2}
nxF [Cu+1 ](aq)

8,314 x (25 + 273) 0,01


E = + 0,66 − x ln { }
2 x 96500 (0,50)2
E = + 0,66 − 1,28 x 10−2 x ln
⏟ (0,04)
(−3,22)

E = + 0,66 + 0,0412 = + 0,701 V


Cálculo da variação da energia livre de Gibbs padrão (∆G°): ∆G = ∆G° + R x T x lnQ

[Sn+2
(aq) ] 0,01
∆G = ∆G° + R x T x ln 2 = −127380 + 8,314 x (25 + 273) x ln { }
[Cu+1 (0,50)2
(aq) ]

J kJ
∆G = −127380 + 2477,572 x (−3,22) = − 135357,78 (−135,36 )
mol mol
Questão 1274 – (ITA) Para uma célula eletrolítica passa uma corrente constante igual a 0,965 amperes. Num dos
eletrodos, a reação que ocorre é a seguinte: Cr2 O−2 +
7 + 14H + 6e → 2Cr
− +3
+ 7 H2 O. Qual o tempo certo
durante o qual essa corrente deve passar para que sejam produzidos 0,400 mol de íons Cr +3 ?

Resolução: Através da estequiometria proveniente da equação química, temos:

695
1600 FÍSICO-QUÍMICA APLICADA EXERCÍCIOS COMENTADOS - IME – ITA – OLIMPÍADA

96500 C
2 mol de Cr +3 − − − − − − − 6 mol de elétrons x
mol de elétrons
0,400 mol de Cr +3 − − − − − Q
0,400 x 6 x 96500
Q= = 115800 C
2
115800
Cálculo do tempo: 0,95 x tempo = 115800 → tempo = 0,95
= 120000 s (33,33 horas)

Questão 1275 – Considere os seguintes potenciais apresentados a seguir: X2+(aq) / X(s) é igual a + 0,34 V, e o par
X+(aq) / X(s) é + 0,55 V. A partir desta informação, calcule E°, ∆G° e Keq para o seguinte processo eletroquímico X2+(aq)
/ X+(aq).
Resolução: Arrumando as semirreações apresentadas pelos pares no enunciado do exercício.
+2 96500 C
X (aq) + 2e− → X (s) ∆G° = −2 mol de elétrons x mol de elétrons x (+0,34)

+1 96500 C
X (aq) + 1e− → X (s) ∆G° = −1 mol de elétrons x mol de elétrons x (+0,55)
+2 +1
Para a determinação do potencial da semirreação X (aq) + 1e− → X (aq) será necessário inverter a segunda
semirreação. Realizando este processo, temos:
+2 96500 C
X (aq) + 2e− → X (s) ∆G° = −2 mol de elétrons x mol de elétrons x (+0,34)
96500 C
X (s) → X +1
(aq) + 1e

∆G° = −1 mol de elétrons x x (−0,55) +
mol de elétrons

+2 +1 96500 C
X (aq) + 1e− → X (aq) ∆G° = −1 mol de elétrons x mol de elétrons x 𝐸°

Determinação do potencial padrão (E°):


96500 C
−1 mol de elétrons x x E°
mol de elétrons
96500 C
= −2 mol de elétrons x x (+0,34)
mol de elétrons
96500 C
+ [−1 mol de elétrons x x (−0,55)]
mol de elétrons
−1 x E° = −2 x (+0,34) + [−1 x (−0,55)]
E° = +0,13 V
+2 +1
Cálculo da variação da energia livre de Gibbs padrão (X (aq) + 1e− → X (aq) ):

96500 C
∆G0 = −n x F x E° = −1 mol de elétrons x x (+0,13) = −12545 J
mol de elétrons

Este processo eletroquímico é espontâneo, pelo fato da variação da energia livre de Gibbs for negativo.

Cálculo da constante de equilíbrio (Keq): ∆G0 = −12545 J

−R x T x lnK eq = −12545 J

−8,314 x (25 + 273) x lnK eq = −12545 J

696
1600 FÍSICO-QUÍMICA APLICADA EXERCÍCIOS COMENTADOS - IME – ITA – OLIMPÍADA

12545
lnK eq = = 5,06
8,314 x 298

K eq = 𝑒 (+5,06)

Questão 1276 – (OLIMPÍADA MINEIRA DE QUÍMICA) As semirreações para as reações químicas que ocorrem em
uma pilha seca (pilha de Leclanché) e seus respectivos potenciais padrão de redução são:
Zn+2(aq) + 2e- → Zn(s) Eo = - 0,76 V
2 NH4 (aq) + 2 MnO4(s) + 2e → Mn2O3(s) + H2O(l) + 2 NH3(aq) Eo = + 0,74 V
+ -

Responda as questões que se seguem:


a) Escreva a equação química da reação global que ocorre no interior de uma pilha seca.
b) Calcule a diferença de potencial eletroquímico (∆E) da pilha de Leclanché.

Resolução:
Item a) Equação química da reação global de funcionamento da pilha, invertendo a semirreação do Zinco:
Zn(s) → Zn+2(aq) + 2e-
ΔG° = - 2 x F x (+ 0,76)

Semirreação de redução:
2 NH4+(aq) + 2 MnO4(s) + 2e- → Mn2O3(s) + H2O(l) + 2 NH3(aq)
ΔG° = - 2 x F x (+ 0,74)

Equação global da pilha:


Zn(s) → Zn+2(aq) + 2e-
2 NH4+(aq) + 2 MnO4(s) + 2e- → Mn2O3(s) + H2O(l) + 2 NH3(aq) +
Zn(s) + 2 NH4+(aq) + 2 MnO4(s) + 2e- → Zn+2(aq) + Mn2O3(s) + H2O(l) + 2 NH3(aq) + 2e-

Item b) Cálculo do potencial da pilha:

Zn(s) → Zn+2(aq) + 2e- ΔGI° = - 2 x F x (+ 0,76)


2 NH4+(aq) + 2 MnO4(s) + 2e- → Mn2O3(s) + H2O(l) + 2 NH3(aq) ΔGII° = - 2 x F x (+ 0,74)

ΔG°T = ΔGI° + ΔGII°


- 2.F.E° = - 2.F.(+ 0,76) + {- 2.F.(+ 0,74)}
E° = + 0,76 + 0,74 = + 1,50 V

Questão 1277 - (IME) Os eletrodos de uma bateria de chumbo são Pb e PbO2. A reação global de descarga é Pb +
PbO2 + 2 H2SO4 → 2 PbSO4 + 2 H2O. Admita que o “coeficiente de uso” seja de 25,0%. Este coeficiente representa
a fração do Pb e PbO2 presentes na bateria que são realmente usados nas reações dos eletrodos. Calcule:
a) a massa mínima de chumbo em quilogramas (incluindo todas as formas em que se encontra esse elemento) que
deve existir numa bateria para que ela possa fornecer uma carga de 38,6 x 104 C;
b) o valor aproximado da variação da energia livre da reação, sendo de 2,00 V a voltagem média da bateria quando
fora de uso.
Resolução: Observamos que tanto no processo anódico (Pb/PbSO4) quanto no catódico (PbO2 | PbSO4) são
processados 2 elétrons.
2 mol de Pb ---------------- 2 mol de elétrons x 96500 C/mol de elétrons
nPb --------------------------- 38,6 x 104 C
2 x 38,6 x 104
nPb = = 4 mol
2 x 96500

Como há um “coeficiente de uso” de 25% são necessários o seguinte número de mol:

697
1600 FÍSICO-QUÍMICA APLICADA EXERCÍCIOS COMENTADOS - IME – ITA – OLIMPÍADA

4 mol ---------- 25%


n ---------------- 100%
n = 16 mol
Item a) Cálculo da massa de chumbo: mPb = nPb x < MM >Pb = 16 x 207 = 3312 g

Item b) Cálculo da variação da energia livre de Gibbs:

96500 C
∆G° = −n x F x E° = −2 mol de elétrons x x 2 = − 386000 J
mol de elétrons

Questão 1278 - (OLIMPÍADA BRASILEIRA DE QUÍMICA) Durante a titulação de uma solução de triiodeto (I3-) com
uma solução de tiossulfato de sódio (Na2S2O3) ocorrem as seguintes reações, com seus potenciais padrão de redução:
I3- + 2e- → 3 I- E° = + 0,54 V
S4O6-2 + 2e- → 2 S2O3-2 E° = + 0,08 V
a) Escreva a reação global da redução do triiodeto pelo íon tiossulfato e calcule o potencial de redução padrão
desta reação.
b) O íon triiodeto (I3-), em solução aquosa, permanece em equilíbrio com o iodo molecular(I2) e íon iodeto (I-),
de acordo com a equação I.
I- + I2 → I3-
Os dados termodinâmicos, a temperatura de 25°C, para essas três espécies encontram-se na tabela abaixo.

Espécie ∆H°f (kJ.mol-1) ∆G°f (kJ.mol-1) ∆S°f (J.mol-1)


I- 55,2 - 51,6 111,3
I2 22,5 16,4 137,2
I3
- 51,5 - 51,4 239,3
Calcule o valor da constante de equilíbrio, K, para a reação apresentada na equação I, utilizando os dados da tabela.

Resolução:

Item a) Invertendo a segunda reação química, temos:

I3- + 2e- → 3 I- E° = + 0,54 V ΔG° = -2.F.(+0,54)


2 S2O3-2 → S4O6-2 + 2e- E° = + 0,08 V ΔG° = -2.F.(-0,08) +

I3- + 2 S2O3-2 → 3 I- + S4O6-2 ΔG°TOTAL = -2.F.(+0,54) + [-2.F.(-0,08)]


ΔG°TOTAL = -2 x F x (+ 0,54) + [-2 x F x (- 0,08)]
-2.F.E° = -2.F.(+0,54) + [-2.F.(-0,08)]
E° = + 0,54 – 0,08 = + 0,46V
Item b) Cálculo da constante de equilíbrio (Keq): ∆G° = −R x T x lnK eq

{1 𝑥 51400} − {1 𝑥 (51600) + 1 𝑥 (16400)} = −8,314 x 298 x lnK eq

51400 − 68000 = −8,314 x 298 x lnK eq

−16600 = −2477,572 x lnK eq

16600
lnK eq = = 6,70
2477,572

698
1600 FÍSICO-QUÍMICA APLICADA EXERCÍCIOS COMENTADOS - IME – ITA – OLIMPÍADA

K eq = 𝑒 6,70

Questão 1279 - (U. S. NATIONAL CHEMISTRY OLYMPIAD) Alumínio metálico é obtido comercialmente por eletrólise
Al2O3, misturado com criolita (Na3AlF6).
a) Quantos mol de elétrons devem passar através da célula de alumínio para produzir 5,00 kg?
b) Determine a corrente elétrica em âmperes produzido em a de alumínio, em 10 horas.

Resolução:
Item a) Cálculo do número de mol de elétrons produzidos no cátodo: Al+3(aq) + 3e- → Al(s). Cálculo da carga (Q):

27,0 g de Mg ---------------- 3 mol de elétrons


5000 g de Mg --------------- n
n = 555,56 mol de elétrons

3600 s
Item b) Cálculo do tempo: tempo = 10 horas x 1h
= 36000 s

Cálculo da carga (Q): Q = 555,56 mol x 96500 C/mol = Q = 5,36 x 107 C

5,36 x 107 C
Cálculo da corrente: i = 36000 s
= 1488,89 𝐴

Questão 1280 – (U. S. NATIONAL CHEMISTRY OLYMPIAD) The corrosion of iron is na electrochemical process that
involves the standard reduction potentials given here at 25°C.
Fe+2(aq) + 2e- → Fe(s) E° = - 0,44 V
O2(g) + 4 H+(aq) + 4e- → 2 H2O(l) E° = + 1,23 V
a) Calculate the voltage for the standard cell based on the corrosion reaction, 2 Fe (s) + O2(g) + 4 H+(aq) → 2 Fe+3(aq) +
2 H2O(l).
b) Calculate the voltage if the reaction in part a occurs at pH = 4,00 but all other concentrations are maintained as
they were in the standard cell.
c) For the reaction Fe(OH)2(s) + 2 e- → Fe(s) + 2 OH-(aq), E° = - 0,88 V. Use this information with one of the given
standard potentials to calculate the Kps of Fe(OH)2.

Resolução: Item a) Invertendo a primeira semirreação e multiplicando por dois:


2 Fe (s) → 2 Fe2+(aq) + 4e- E0 = + 0,44 V ΔG1° = -4.F.(+0,44)
Mantendo a segunda semirreação, temos:
O2(g) + 4 H+(aq) + 4e- → 2 H2O(l) E° = + 1,23 V ΔG2° = -4.F.(+1,23)
Equação global para o processo eletroquímico: 2 Fe (s) + O2(g) + 4 H+(aq) → 2 Fe2+(aq) + 2 H2O(l)

Cálculo do potencial-padrão:
ΔGT° = -4.F.E°
ΔGT° = ΔG1° + ΔG2°
-4.F.E° = -4.F.(+ 0,44) + -4.F.(+ 1,23)
E° = + 0,44 + 1,23 = + 1,67 V
Item b) Utilizando a equação de Nernst, temos: 2 Fe (s) + O2(g) + 4 H+(aq) → 2 Fe2+(aq) + 2 H2O(l)

699
1600 FÍSICO-QUÍMICA APLICADA EXERCÍCIOS COMENTADOS - IME – ITA – OLIMPÍADA

2
0,0592 0,0592 [Fe+2 2
(aq) ] x a H2 O
E = E° − x logQ = E° − x log { 4 }
n n [H + ] x a2 x P
(aq) Fe O2

0,0592 (1)2 x (1)2


E = + 1,67 − x log { −4 4 }
4 [10 ] x (1)2 x 1
0,0592
E = + 1,67 − x log10+16
4
0,0592
E = + 1,67 − x 16 = + 1,67 − (0,0592 x 4) = + 1,67 − (0,0592 x 4) = +1,43 𝑉
4
Item c) Fe(OH)2(s) + 2 e- → Fe(s) + 2 OH-(aq) E° = - 0,88 V

Fe+2(aq) + 2e- → Fe(s) E° = - 0,44 V


Invertendo a segunda semirreação, temos:
Fe(OH)2(s) + 2 e- → Fe(s) + 2 OH-(aq) E° = - 0,88 V ΔG° = -2.F.(-0,88)
Fe(s) → Fe+2(aq) + 2e- E° = + 0,44 V ΔG° = -2.F.(+0,44) +

Fe(OH)2(s) → Fe+2(aq) + 2 OH-(aq)


ΔGT° = ΔG1° + ΔG2°
-2.F.E° = -2.F.(- 0,88) + -2.F.(+ 0,44)
E° = - 0,44 V
Cálculo da constante de produto de solubilidade: ∆G° = −R x T x lnK ps = −n x F x E°
96500 C
−8,314 x 298 x lnK ps = −2 mol de elétrons x x (−0,44)
mol de elétrons
−2477,572 x lnK ps = 84920
− 84920
lnK ps = = −34,27
2477,572
K ps = 𝑒 (−34,27)

Questão 1281 – (OLIMPÍADA NORTE – NORDESTE DE QUÍMICA) Em princípio, uma bateria poderia ser
desenvolvida a partir de alumínio metálico e cloro gasoso.
a) Escreva a equação balanceada que ocorre em uma bateria cujas semirreações são: Al +3(aq) / Al(s) e Cl2(g) / 2
Cl-(aq).
b) Diga que semirreação ocorre no ânodo e que semirreação ocorre no cátodo.
c) Calcule o potencial padrão para esta bateria.
d) Se a bateria produz uma corrente de 0,75 A, quanto tempo ela irá operar se o eletrodo de alumínio contiver
30,0 g do metal? (Considere que há quantidade suficiente de cloro).
Dados:
Al+3(aq) + 3e- → Al(s) E° = - 1,66 V
Cl2(g) + 2e → 2 Cl (aq)
- - E° = + 1,36 V

Resolução:

Item a) Analisando a semirreação de redução do Al3+(aq) / Al(s): Invertendo a semirreação de alumínio e multiplicado
por dois, temos:

700
1600 FÍSICO-QUÍMICA APLICADA EXERCÍCIOS COMENTADOS - IME – ITA – OLIMPÍADA

2 Al(s) → 2Al+3(aq) + 6e- E° = + 1,66 V ∆G10 = −6 x F x (+1,66)

Analisando a semirreação de redução do Cl2(g) / 2 Cl-(aq): Multiplicando a semirreação por três, temos:

3 Cl2(g) + 6e- → 6 Cl-(aq) E° = + 1,36 V ∆G20 = −6 x F x (+1,36)

Somando as semirreações, temos:


2 Al(s) → 2Al+3(aq) + 6e- E° = + 1,66 V ∆G10 = −6 x F x (+1,66)
3 Cl2(g) + 6e- → 6 Cl-(aq) E° = + 1,36 V ∆G20 = −6 x F x (+1,36) +
0
2 Al(s) + 3 Cl2(g) + 6e- → 6 Cl-(aq) + 2 Al+3(aq) + 6e- ∆G𝑡𝑜𝑡𝑎𝑙 = −6 x F x E°

Item b)
Semirreação que ocorre no ânodo: Al(s) → Al+3(aq) + 3e-
Semirreação que ocorre no cátodo: Cl2(g) + 2e- → 2Cl-(aq)

Item c) Cálculo do potencial padrão da bateria: ΔG°total = ΔGI° + ΔGII°

- 6.F.E° = - 6.F.(+ 1,66) + {- 6.F.(+ 1,36)}


E° = + 1,66 + 1,36 = + 3,02 V

Item d) Semirreação de redução do alumínio: Al+3(aq) + 3e- → Al(s). Cálculo da carga Q:

1 mol de Al ---------- 27,0 g de Al --------------- 3 mol de elétrons x (96500 C/mol de elétrons)


30,0 g de Al -------------- Q
Q = 321666,67 C

Q 321666,67
Cálculo do tempo, a partir da equação da corrente elétrica (i): tempo = i
= 0,75
=
428888,89 s (119,13 horas)

Questão 1282 – (ITA) Um tanque de estocagem de produtos químicos foi revestido internamente com níquel puro
para resistir ao efeito corrosivo de uma solução aquosa ácida contida em seu interior. Para manter o líquido aquecido,
foi acoplado junto ao tanque um conjunto de resistores elétricos alimentados por um gerador de corrente contínua.
Entretanto, uma falha no isolamento elétrico do circuito dos resistores promoveu a eletrificação do tanque,
ocasionando um fluxo de corrente residual de intensidade suficiente para desencadear o processo de corrosão
eletrolítica do revestimento metálico. Admitindo-se que a superfície do tanque é constituída por uma monocamada de
níquel com densidade atômica igual a 1,61 x 1019 átomos.m─2 e que a área superficial do tanque exposta à solução
ácida é de 5,0 m2, calcule:
a) a massa, expressa em gramas, de átomos de níquel que constituem a monocamada atômica do revestimento
metálico.
b) o tempo necessário, expresso em segundos, para que a massa de níquel da monocamada atômica seja consumida
no processo de dissolução anódica pela passagem da densidade de corrente de corrosão de 7,0 μA.cm-2.

Resolução:

átomos
Item a) Número de átomos de níquel (nNi): nNi = 1,61 x 1019 x 5,0 m2 = 8,05 x 1019 átomos

1 mol de Ni ---------- 58,7 g ---------- 6,02 x1023 átomos de Ni


mNi ----------- 8,05 x1019 átomos de Ni
Massa de Níquel = 7,85 x 10-3 g

701
1600 FÍSICO-QUÍMICA APLICADA EXERCÍCIOS COMENTADOS - IME – ITA – OLIMPÍADA

Item b) Ni+2(aq) + 2e- → Ni(s)

Sabendo que a densidade de corrente é definida como sendo a razão da corrente pela área, temos:

i A 10000 cm² A
j= = 7,0μ x = 7,0 x 10−2
A cm² 1 m² m²
A
Para 5,0 m² de área, temos: j = 7,0 x 10−2 m² x 5,0 m2 = 35,0 x 10−2 A

Cálculo do tempo de operação da célula eletroquímica: Ni+2(aq) + 2e- → Ni(s)

58,69 g de Ni ---------------------------- 2 mol de e- x 96500 C/mol de e-


7,85 x 10-3 g de Ni ---------------------- Q = (35 x 10-2 x tempo)
tempo = 73,75 g
Questão 1283 – (CYPRUS NATIONAL COMPETITION FOR THE INTERNATIONAL CHEMISTRY OLYMPIAD)
a) Calcular o decréscimo de massa do eletrodo de zinco, se uma corrente de 0,15 A, passa através da célula galvânica
abaixo, durante 1,5 h: Zn(s) / Zn2+(aq) // Cu2+(aq) / Cu(s)
b) Calcule o potencial padrão de redução da célula abaixo, a 25°C, sendo a concentração de Zn2+ igual a 0,1 mol/L e
a de Ag+ igual a 0,01 mol/L (E° = 1,56 V)
Zn(s) / Zn2+(aq) // Ag+(aq) / Ag(s)
c) Dar a equação química da reação que ocorre espontaneamente na célula galvânica que consiste das seguintes
meia-células:
MnO2 + 4H+ + 2e- → Mn2+ + 2 H2O E° = + 1,21 V
Ag+ + e- → Ag E° = + 0,80 V
3600 s
Resolução: Item a) Conversão do tempo de horas para segundos: tempo = 1,50 h x 1h
= 5400 s

Cálculo do decréscimo de massa do eletrodo de Zinco, a partir da semirreação de redução do zinco:


Zn2+(aq) + 2e- → Zn(s)

65,4 g de Zn ---------------- 2 mol de elétrons x 96500 C/mol de elétrons


mZn --------------------------- Q = (0,15 A x 5400 s)
0,15 x 5400 x 65,4
mZn = = 0,274 g
2 x 96500

Item b) Cálculo do potencial, a partir da utilização da equação de Nersnt. Equação global da pilha: Zn(s) + 2 Ag+(aq) →
Zn2+(aq) + 2 Ag(s)

RxT RxT [Zn+2 2


(aq) ] x a Ag
E = E° − x lnQ = E° − x ln 4
nxF nxF [Ag + ] x a (aq) Zn

8,314 x 298 0,1


E = 1,56 − x ln
2 x 96500 (0,01)2
2477,572
E = 1,56 − x ln1000 = 1,56 − 1,28 x 10−2 x 6,91 = +1,47 V
193000

702
1600 FÍSICO-QUÍMICA APLICADA EXERCÍCIOS COMENTADOS - IME – ITA – OLIMPÍADA

Item c)

MnO2 + 4H+ + 2e- → Mn2+ + 2 H2O E° = + 1,21 V


Ag+ + e- → Ag E° = + 0,80 V
Invertendo a semirreação de redução da prata e multiplicando por dois, temos:
2 Ag(s) → 2 Ag+(aq) + 2e- ΔGI° = - 2.F.(- 0,80)
2 MnO2(s) + 4H+(aq) + 2e- → Mn+2(aq) + 2 H2O(l) ΔGII° = - 2.F.(+ 1,21) +
2 Ag(s) + 2 MnO2(s) + 4H+(aq) + 2e → 2 Ag (aq) + Mn (aq) + 2 H2O(l) + 2e ΔGT° = - 2.F.E°
- + +2 -

ΔG°T = ΔGI° + ΔGII°


- 2.F.E° = - 2.F.(- 0,80) + {- 2.F.(+ 1,21)}
E° = - 0,80 + 1,21 = + 0,41V

Questão 1284 – Uma solução de cloreto de alumínio apresenta coeficiente de atividade médio igual a 0,96, pela lei
de Debye Huckell. A partir desta informação, calcule a massa de cloreto de alumínio a partir de 800 gramas de água
pura e também o valor das atividades dos íons alumínio e cloreto.

Resolução: AlCl3(aq) → Al+3 −


(aq) + 3 Cl(aq)

Cálculo da força iônica (I), a partir da utilização da lei de Debye-Huckell:

logγ±
x = −|z1 x z2 | x A x √I

log (0,96) = −|3 x (−1)| x 0,509 x √I

−1,16 x 10−2 = −3 x 0,509 x √I

√I = 1,16 x 10−2
2
(√I) = (1,16 x 10−2 )2

I = 1,35 x 10−4

Relação estequiométrica com a moladidade através da reação de dissociação do cloreto de alumínio: AlCl3(aq) →
Al+3 −
(aq) + 3 Cl(aq)

Chamando de W a concentração molal da solução, temos:

1 mol de AlCl3(aq) − − − − − 1 mol de Al+3 −


(aq) − − − − − − 3 mol de Cl(aq)
W mol de AlCl3(aq) − − − − − W mol de Al+3 −
(aq) − − − − − 3W mol de Cl(aq)

1
Cálculo da massa de cloreto de alumínio, a partir da força iônica: I = 2 x {zi2 x WAl+3 + zi2 x 3 x WCl− }

1
I= x {(3)2 x W + (−1)² x 3W}
2
1 1
I= x {9W + 3W} = x 12 x W = 6W
2 2
mAlCl3
1,35 x 10−4 = 6 x (kg)
<MM>AlCl3 x msolvente

703
1600 FÍSICO-QUÍMICA APLICADA EXERCÍCIOS COMENTADOS - IME – ITA – OLIMPÍADA

mAlCl3
1,35 x 10−4 = 6 x
133,5 x 0,800

1,35 x 10−4 x 133,5 x 0,800


mAlCl3 = = 2,40 𝑥 10−3 𝑔
6
mAlCl
3 2,40 x 10−3
<MM>AlCl 𝑚𝑜𝑙
Cálculo da molalidade (W): W = (kg)
3
= 133,5
= 2,25 x 10−5
msolvente 0,800 𝑘𝑔

Analisando o íon (cátion) alumínio: 𝑎𝐴𝑙+3 = γ± +3


x 𝑥 [𝐴𝑙 ] = 0,96 𝑥 2,25 x 10
−5
= 2,16 𝑥 10−5

Analisando o íon (ânion) cloreto: 𝑎𝐶𝑙− = γ± −


x 𝑥 [𝐶𝑙 ]

𝑎𝐶𝑙− = 0,96 𝑥 3 x 2,25 x 10−4 = 6,48 𝑥 10−4

Questão 1285 – Calcule as atividades dos cátions e ânions de uma solução aquosa de cloreto de cálcio, que
apresenta molalidade igual a 3 x 10-4 mol.kg-1 obedecendo à lei de Debye-Huckel.

Resolução: Ionização do cloreto de cálcio: CaCl2(aq) → Ca+2 −


(aq) + 2 Cl(aq)

1 mol de CaCl2(aq) → 1 mol de Ca+2 −


(aq) + 2 mol de Cl(aq)
mol mol mol
3,0 x 10−4 − − − 3,0 x 10−4 − − 6,0 x 10−4
kg kg kg
1
Cálculo da força iônica (I): I = 2 x {zi2 x WAl+3 + zi2 x 3 x WCl− }

1
I= x {(+2)2 x 3,0 𝑥 10−4 + (−1)² x 6,0 𝑥 10−4 }
2
1
I= x {12,0 𝑥 10−4 + 6,0 𝑥 10−4 } = 9,0 𝑥 10−4
2

Cálculo do coeficiente médio de atividade, a partir da utilização da lei de Debye-Huckell: logƔ± =


−|z1 x z2 | x A x √I

logƔ± = −|2 x (−1)| x 0,509 x √9,0 𝑥 10−4

logƔ± = −2 x 0,509 x 3,0 𝑥 10−2

logƔ± = −3,054 x 10−2


−2 )
Ɣ± = 10(−3,054 x 10 = 0,932

Cálculo das atividades dos íons:

Analisando o íon (cátion) cálcio: aCa+2 = 0,932 x 3,0 x 10−4 = 2,80 x 10−4

Analisando o íon (ânion) cloreto: aCl− = 0,932 x 6,0 x 10−4 = 5,59 x 10−4

704
1600 FÍSICO-QUÍMICA APLICADA EXERCÍCIOS COMENTADOS - IME – ITA – OLIMPÍADA

Questão 1286 – (IME – MODIFICADA) Dada a reação global para a descarga de um acumulador de chumbo, que
+
pode ser representada pela equação: 𝐏𝐛(𝐬) + 𝐏𝐛𝐎𝟐(𝐬) + 𝟒𝐇(𝐚𝐪) → 𝟐 𝐏𝐛𝐒𝐎𝟒 + 𝟐 𝐇𝟐 𝐎(𝐥) , com G° = - 88,5
kcal, pede-se:
a) a f.e.m (E) da célula;
b) a constante de equilíbrio (Keq);
c) a massa de chumbo consumida durante uma descarga de 1 hora, com corrente média de 1,0 A.
+
Resolução: Equação eletroquímica global: Pb(s) + PbO2(s) + 4H(aq) → 2 PbSO4 + 2 H2 O(l),

Item a) Cálculo da força eletromotriz padrão (E°): ∆G° = −n x F x E°

96500 𝐶
−88500 = −4 mol de elétrons x x E°
𝑚𝑜𝑙 𝑑𝑒 𝑒𝑙é𝑡𝑟𝑜𝑛𝑠

−88500 −88500
E° = = = + 0,229 V
− 4 𝑥 96500 − 386000

Item b) Cálculo da constante de equilíbrio (Keq): ∆G° = −R x T x lnK eq

−88500 = −8,314 x (25 + 273) x lnK eq

−88500
lnK eq = = +35,72
−8,314 x 298

K eq = e(+35,72)

Item c) Cálculo da massa de chumbo (Pb): Semirreação que ocorre no cátodo: Pb2+(aq) + 2e- → Pb(s)

96500 C
207 g de Pb − − − − − − 2 mol de elétrons x
1 mol de elétrons
mPb − − − − − − − − − 1 x 3600 C

207 x 1 x 3600 745200


mPb = = = 3,86 g
2 x 96500 193000

Questão 1287 – Tem-se duas cubas eletrolíticas ligadas em série, contendo respectivamente soluções de nitrato de
prata e sulfato de cobre II. Após este tempo por causa da passagem de uma corrente elétrica houve a interrupção do
circuito. Em uma das cubas eletrolíticas houve a deposição de 4,32 g de prata. A partir desta informação, determine
a massa de cobre depositada na outra cuba eletrolítica.

Resolução: Analisando a solução de nitrato de prata: AgNO3(aq) → Ag + −


(aq) + NO3(aq)
Semirreação que ocorre no cátodo: Ag + −
(aq) + 1𝑒 → Ag (s)

96500 C
108 g − − − − 1 mol de e− x
mol de e−
4,32 g − − − − Q

4,32 x 1 x 96500
Q= = 3860 C
108

Analisando a solução de sulfato de cobre II: CuSO4(aq) → Cu+2 −2


(aq) + SO4(aq)
Semirreação que ocorre no cátodo: Cu+2 −
(aq) + 2𝑒 → Cu(s)

705
1600 FÍSICO-QUÍMICA APLICADA EXERCÍCIOS COMENTADOS - IME – ITA – OLIMPÍADA

96500 C
63,55 g − − − − 2 mol de e− x
mol de e−
mCu − − − − − 3860 C

63,55 x 3860 245303


mCu = = = 1,27 g
2 x 96500 193000

Questão 1288 – Uma determinada corrente elétrica passa durante seis minutos através de um voltâmetro contendo
ácido sulfúrico diluído, liberando 40 x 10-3 dm³ de hidrogênio gasoso medido a 15°C e pressão igual a 748 mmHg. A
partir destas informações, determine a corrente elétrica produzida por este processo.

Resolução: Analisando a eletrólise aquosa do ácido sulfúrico:


Reação de dissociação do ácido sulfúrico: H2SO4(aq) → 2 H+(aq) + SO4-2(aq)
Reação de ionização da água: H2O(aq) → 2 H+(aq) + OH-(aq)
O único que vai descarregar no cátodo, trata-se do hidrogênio: 2 H+(aq) + 2e- → H2(g)
Quem descarrega com maior facilidade no ânodo (pólo positivo), SO4-2(aq) ou OH-(aq)?
2 OH-(aq) → ½ O2(g) + 2e- + H2O(l)
Somando as semirreações mencionadas acima, temos:
H2SO4(aq) → 2 H+(aq) + SO4-2(aq)
2 H2O(aq) → 2 H+(aq) + 2 OH-(aq)
2 H+(aq) + 2e- → H2(g)
2 OH-(aq) → ½ O2(g) + 2e- + H2O(l) +

H2SO4(aq) + H2O(l) → 2 H+(aq) + SO4-2(aq) + H2(g) + ½ O2(g)


Através da eletrólise do ácido sulfúrico, observa-se que há sim a produção de gás hidrogênio.
Cálculo do número de mol de gás hidrogênio, utilizando a equação dos gases ideais, temos:
96500 C
1 mol de H2(g) − − − − − − − − − 2 mol de e− x
mol de e−
748
(760) 𝑥 40 𝑥 10−3
−−−−−−Q
0,08206 𝑥 (15 + 273)

748
(760) x 40 x 10−3 5774560
Q = 2 x 96500 x { }= = 321,50 C
0,08206 x 288 17961,29

Q 321,50
Cálculo da corrente (i): i = tempo = 360
= 0,893 A

706
1600 FÍSICO-QUÍMICA APLICADA EXERCÍCIOS COMENTADOS - IME – ITA – OLIMPÍADA

Questão 1289 – (OLIMPÍADA DE QUÍMICA DO RGS) Sobre a pilha representada abaixo, pede-se:

Potenciais de redução: EoCu = + 0,34 V; EoAg = + 0,80 V.


a) Seu cátodo e seu ânodo.
b) Quem se oxida e quem se reduz.
c) Qual eletrodo terá sua massa aumentada?
d) Qual das soluções irá diluir-se?
e) A reação catódica.
f) A reação anódica.
g) Sua ddp.
h) A reação global.
i) A energia de Gibbs padrão.
j) A constante de equilíbrio.

Resolução: Semirreação de redução:

Cu+2 −
(aq) + 2e → Cu(s) E° = +0,34 V
Ag +
(aq)

+ 1e → Ag (s) E° = +0,80 V
Invertendo a primeira semirreação e multiplicando por dois a segunda semirreação, temos:

Cu(s) → Cu+2
(aq) + 2e

E° = - 0,34 V ∆G10 = −2 x F x (−0,34)

2Ag + −
(aq) + 2e → 2Ag (s) E° = +0,80 V ∆G20 = −2 x F x (+0,80) +

Cu(s) + 2Ag + +2
(aq) → Cu(aq) + 2Ag (s)

∆G30 = ∆G10 + ∆G20

− 2 x F x E° = −2 x F x (−0,34) + [−2 x F x (+0,80)]

E° = −0,34 + 0,80 = +0,46 V

a) Ânodo: cobre (Cu(s) → Cu+2 − + −


(aq) + 2e ) e câtodo = prata (2Ag (aq) + 2e → 2Ag (s) )

b) Agente oxidante = Ag +
(aq) e agente redutor = Cu(s)

c) O eletrodo que apresenta a massa aumentada é o eletrodo de prata.

d) A solução que irá diluir é a solução de nitrato de prata [AgNO3(aq)].

707
1600 FÍSICO-QUÍMICA APLICADA EXERCÍCIOS COMENTADOS - IME – ITA – OLIMPÍADA

e) Reação catódica: Cu(s) → Cu+2


(aq) + 2e

f) Reação Anódica: 2Ag + −


(aq) + 2e → 2Ag (s)

g) E° = +0,46 V

h) Reação global: Cu(s) + 2Ag + +2


(aq) → Cu(aq) + 2Ag (s)

i) Cálculo da variação da energia livre de Gibbs padrão:

96500 C
∆G° = −n x F x E° = −2 mol de elétrons x x (+0,46) = −88780 J
mol de elétrons

j) Cálculo da constante de equilíbrio: ∆G° = −R x T x lnK eq = ⏟


−n x F x E°
−88780

−8,314 x 298 x lnK eq = −88780


−88780
lnK eq = = +35,83
− 2477,572
K eq = 𝑒 (+35,83)

Questão 1290 – (ITA) Descreva como o hidróxido de sódio é obtido em escala industrial. Suas descrição deve incluir
as matérias-primas utilizadas, as equações das reações químicas envolvidas no processo, as condições de operação
e o aproveitamento de eventuais subprodutos obtidos no processo.
Resolução: Para a produção de hidróxido de sódio (soda cáustica) as matérias-primas utilizadas são cloreto de sódio
e água.

⏞ Na+
Reação de dissociação do cloreto de sódio: NaCl(aq) → −
(aq) + Cl(aq)

+ −
Reação de ionização da água: H2 O(l) → H(aq) + OH(aq)

Quem descarrega com melhor facilidade no cátodo, Na+ + + −


(aq) ou H(aq) ? 2H(aq) + 2e → H2(g)

Quem descarrega com melhor facilidade no ânodo, Cl− − −


(aq) ou OH(aq) ? 2Cl(aq) → Cl2 (𝑔) + 2𝑒

Multiplicando a primeira e a segunda semirreação por dois e somando todas as reações, temos:

2 NaCl(aq) → 2 Na+ −
(aq) + 2 Cl(aq)

+ −
2H2 O(l) → 2H(aq) + 2OH(aq)
+
2H(aq) + 2e− → H2(g)

2Cl−
(aq) → Cl2 (𝑔) + 2𝑒

+

+ −
2 NaCl(aq) + 2H2 O(l) → 2
⏟Na(aq) + 2OH(aq) + H2(g) + Cl2 (g)
2 NaOH(aq)

Observação: Esta é a forma eletroquímica para a produção de hidróxido de sódio (soda cáustica) a partir do cloreto
de sódio, havendo a liberação dos gases H2(g) e Cl2(g).
Questão 1291 – (ITA) Descreva os procedimentos utilizados na determinação do potencial de um eletrodo de cobre
Cu(s) | Cu2+(aq). De sua descrição devem constar:

708
1600 FÍSICO-QUÍMICA APLICADA EXERCÍCIOS COMENTADOS - IME – ITA – OLIMPÍADA

a) A listagem de todo o material (soluções, medidores etc.) necessário para realizar a medição do potencial do eletrodo
em questão.
b) O desenho esquemático do elemento galvânico montado para realizar a medição em questão. Deixe claro nesse
desenho quais são os pólos positivo e negativo e qual dos eletrodos será o anodo e qual será o catodo, quando
corrente elétrica circular por esse elemento galvânico. Neste último caso, escreva as equações químicas que
representam as reações anódicas e catódicas, respectivamente.
c) A explicação de como um aumento do valor das grandezas seguintes afeta o potencial do eletrodo de cobre
(Aumenta? Diminui? Não altera?): área do eletrodo, concentração de cobre no condutor metálico, concentração de
íons cobre no condutor eletrolítico e temperatura.
Resolução:
Item a) Duas cubas de vidro, uma lâmina de cobre e uma lâmina de platina, fio condutor (por exemplo, fio de cobre),
ponte salina (tubo de vidro recurvado contendo solução eletrolítica saturada, com as extremidades fechadas por
algodão), tubo de vidro com entrada para hidrogênio gasoso, soluções apresentando [H+] = 1,0 mol x L-1 e [Cu2+] = 1,0
mol x L-1, voltímetro (o valor obtido no voltímetro será positivo para metais que se reduzem no processo). Temperatura:
25º C. Pressão do H2(g): 1 atm.
Item b)

Semirreação anódica: H2 → 2 H+ + 2 e-
Semirreação catódica: Cu2+ + 2 e- → Cu
Item c) Área do eletrodo: não altera
Concentração do cobre no condutor metálico: não altera
0,0592 [H+ ]
Concentrações de íons cobre: aumenta, pois E = E° − n
x log [Cu+2 ]

RxT
Aumento da temperatura: diminui, pois E = E° − n x F x lnQ

Questão 1292 - (U. S. NATIONAL CHEMISTRY OLYMPIAD)

A) For which reaction is ∆H (enthalpy change) most nearly equal to ∆E (internal energy change)?
(A) H2(g) + ½ O2(g) → H2O(g)
(B) Cl2(g) + F2(g) → 2ClF(g)
(C) H2O(l) → H2O(g)
(D) 2SO3(g) → 2SO2(g) + O2(g)

B) What is the standard cell potential for the reaction, 2 Cr(s) + 3 Sn2+(aq) → 3 Sn(s) + 2 Cr3+(aq) given the E˚ values
shown?
Cr3+(aq) + 3e- → Cr(s) - 0,744 V
Sn2+(aq) + 2e- → Sn(s) - 0,141 V
(A) 0,945 V
(B) 0,603 V

709
1600 FÍSICO-QUÍMICA APLICADA EXERCÍCIOS COMENTADOS - IME – ITA – OLIMPÍADA

(C) -0,603 V
(D) -0,945 V

C) A CuSO4 solution is electrolyzed for 20 minutes with a current of 2,0 ampere. What is the maximum mass of copper
that could be deposited?
(A) 0,20 g
(B) 0,40 g
(C) 0,79 g
(D) 1,6 g

D) When an aqueous solution of potassium fluoride is electrolyzed, which of the following occurs?
(A) O2 and H+ are produced at one electrode and H2 and OH- are formed at the other.
(B) O2 and OH- are produced at one electrode and H2 and H+ are formed at the other.
(C) Metallic K is formed at one electrode and O2 and H+ are formed at the other.
(D) Metallic K is produced at one electrode and elemental F2 is produced at the other.

E) The standard potential for the reaction Cl2(g) + 2Br-(aq) → Br2(l) + 2Cl-(aq) is 0,283 volts. What is the equilibrium constant
for this reaction at 25˚C?
(A) 1,6 × 10-5
(B) 22
(C) 6,1 × 104
(D) 3,8 × 109

Resolução:

A) Alternativa B.
Sabendo que a relação termodinâmica entre a energia interna (∆E) e a entalpia é dada por:
∆E = ∆H + pV
Considerando que o provesso reacional apresenta comportamento ideal, temos:
∆E = ∆H + ∆n x R x T
A alternativa que apresenta a variação da energia interna igual a entalpia trata-se da alternativa B, pelo fato da
variação do número de mol ser igual a zero.
∆E = ∆H + (nprodutos − nreagentes ) x R x T
∆E = ∆H + (2 − 2) x R x T
∆E = ∆H

B) Alternativa B.

Invertendo e multiplicando por dois a semirreação do cromo: 2Cr(s) → 2Cr3+(aq) + 6e–


∆G° = −6 x F x (+0,744)
Multiplicando por três a semirreação do estanho: 3Sn2+(aq) + 6e– → 3Sn(s)
∆G° = −6 x F x (−0,141)
Somando as duas semirreações:
2Cr(s) → 2Cr3+(aq) + 6e– ∆G° = −6 x F x (+0,744)
3Sn2+(aq) + 6e– → 3Sn(s) ∆G° = −6 x F x (−0,141)

2Cr(s + 3Sn2+(aq) → 2Cr3+(aq) + 3Sn(s)


−6 x F x E° = −6 x F x (+0,744) + [−6 x F x (−0,141)]
Eo = + 0,744 + (−0,141) = + 0,603 V

710
1600 FÍSICO-QUÍMICA APLICADA EXERCÍCIOS COMENTADOS - IME – ITA – OLIMPÍADA

C) Alternativa C.
Reação que ocorre no cátodo: Cu+2 −
(aq) + 2e → Cu(s)

Cálculo da massa de cobre depositada:


96500 C
63,55 g de Cu − − − − − − − 2 mol de e− x
1 mol de e−
mCu − − − − − − − − − − − (20 x 60 x 2,0) C

152520
mCu = = 0,790 g
193000

D) Alternativa A.

Analisando a eletrólise aquosa do fluoreto de potássio (KF).


Reação de dissociação do fluoreto de potássio: KF(aq) → K + −
(aq) + F(aq)
+ −
Reação de ionização da água: H2 O(l) → H(aq) + OH(aq)

Quem descarrega com melhor facilidade no cátodo, K + + + −


(aq) ou H(aq) ? 2H(aq) + 2e → H2(g)

− − − 1
Quem descarrega com melhor facilidade no ânodo, F(aq) ou OH(aq) ? 2OH(aq) → H2 O(l) + 2 O2(g) + 2e−

Multiplicando a segunda semirreação por dois e somando todas as reações, temos:

KF(aq) → K + −
(aq) + F(aq)
+ −
2H2 O(l) → 2H(aq) + 2OH(aq)
+
2H(aq) + 2e− → H2(g)
− 1
2OH(aq) → H2 O(l) + 2 O2(g) + 2e− +
1
KF(aq) + H2 O(l) → K + −
(aq) + F(aq) + H2(g) + O2(g)
2
O2 e H+ são formados em um eletrodo e H2 e OH- estão sendo formados em um outro eletrodo.
0
E) Sabendo que a relação termodinâmica entre as energias livre de Gibbs é dada por: ∆Gtermodinâmico =
0
∆Geletroquímico

− R x T x lnKeq = − n x F x E 0
96500 C
− 8,314 x (25 + 273) x lnKeq = − 2 mol de e− x x (+0,283)
1 mol de e−
−54619
lnKeq = = 22,04
−2477,572

K eq = e(22,04) = 3,73 x 109

711
1600 FÍSICO-QUÍMICA APLICADA EXERCÍCIOS COMENTADOS - IME – ITA – OLIMPÍADA

Questão 1293 – (U. S. NATIONAL CHEMISTRY OLYMPIAD)


A) What is the density of propane, C3H8, at 25°C and 740. mmHg?
(A) 0,509 g.L–1 (B) 0,570 g.L–1 (C) 1,75 g.L–1 (D) 1,96 g.L–1
B) Gadolinium-153, which is used to detect osteoporosis, has a half-life of 242 days. Which value is closest to the
percentage of the Gd-153 left in a patient's system after 2 years (730 days)?
(A) 33.0 % (B) 25.0 % (C) 12.5 % (D) 6.25 %
C) A certain reaction is exothermic by 220 kJ and does 10 kJ of work. What is the change in the internal energy of the
system at constant temperature?
(A) +230 kJ (B) +210 kJ (C) –210 kJ (D) –230 kJ
D)
Al3+(aq) + 3e– → Al(s) Eo = – 1,66 V
Cu2+(aq) + 2e– → Cu(s) Eo = + 0,34 V
What voltage is produced under standard conditions by combining the half-reactions with these Standard Electrode
Potentials?
(A) 1,32 V (B) 2,00 V (C) 2,30 V (D) 4,34 V
Resolução:
A) Alternativa A.
Sabendo que a relação entre a densidade e a pressão é dada pela seguinte equação química:
740
p x<MM> ( ) 𝑥 44 𝑔
d= RxT
. Substituindo os valores na equação: d = 0,08206
760
𝑥 (25+273)
= 1,75 𝐿

B) Alternativa C.
Sabendo que para o processo radioativo a cinética é de primeira ordem. Cálculo da constante radioativa:
ln 2
𝑡1 =
2 λ
ln 2 0,693
λ= = = 2,86 𝑥 10−3 𝑑𝑖𝑎𝑠 −1
𝑡1 242
2

[𝐴] [𝐴]
Cálculo do percentual [𝐴] 𝐹𝑖𝑛𝑎𝑙 : ([𝐴] 𝐹𝑖𝑛𝑎𝑙 ) = −λ x tempo
𝑛𝑖𝑐𝑖𝑎𝑙 𝑛𝑖𝑐𝑖𝑎𝑙

[A]Final 365 dias


ln ( ) = −2,86 x 10−3 x 2 anos x
[A]nicial 1 ano
[A]Final
ln ( ) = −2,09
[A]nicial
[A]Final
= 𝑒 (−2,09) = 0,1237 (12,37%)
[A]nicial
C) Alternativa D.
Pela primeira lei da termodinâmica, temos: ∆U = Q + W

712
1600 FÍSICO-QUÍMICA APLICADA EXERCÍCIOS COMENTADOS - IME – ITA – OLIMPÍADA

∆U = −220 + (−10) = −230 kJ


D) Alternativa B.

Invertendo e multiplicando por dois a primeira semirreação: 2Al(s) → 2Al3+(aq) + 6e–; ∆G° = −6 x F x (+1,66)
Multiplicando por três a segunda semirreação: 3Cu2+(aq) + 6e– → 3Cu(s); ∆G° = −6 x F x (+0,34)
Somando as duas semirreações:
2Al(s) → 2Al3+(aq) + 6e– ∆G° = −6 x F x (+1,66)
3Cu2+(aq) + 6e– → 3Cu(s) ∆G° = −6 x F x (+0,34)

2Al(s) + 3Cu2+(aq) + 6e– → 2Al3+(aq) + 3Cu(s) + 6e–


−6 x F x E° = −6 x F x (+1,66) + [−6 x F x (+0,34)]
Eo = + 1,66 + 0,34 = + 2,00 V
Questão 1294 – (CONCURSO PARA DOCENTE – IFRJ - MODIFICADA) Amálgama dentário do mercúrio com uma
liga contendo prata, estanho e cobre, além de zinco ou outros elementos em menores proporções. As utilizações
desse tipo de amálgama na restauração de dentes apresentam duas desvantagens: a cor e o efeito de dor, causado
quando acidentalmente se morde um talher de alumínio ou um papel confeccionado com alumínio. Assim, responda
ao que se questiona:
a) O exemplo descrito refere-se a uma pilha galvânica ou a uma célula eletrolítica? Justifique, identificando as
reações catódica e anódica, bem como a reação global, balanceada da pilha.
b) Calcule a força eletromotriz (fem).
c) Determine o valor da Energia Livre de Gibbs (ΔG°), considerando as seguintes informações:
Al+3(aq) + 3e- → Al(s) ΔE° = - 1,66 V
Hg+2(aq) + 2e- → 2 Hg(amálgama de Ag) ΔE° = + 0,85 V
F = 96485,33 C.mol-1
d) Determine a constante de equilíbrio (Keq).

Resolução:
Item a) O exemplo descrito refere-se a uma pilha galvânica, pelo fato deste efeito ocorrer de forma espontânea.
Invertendo a primeira semirreação e multiplicando por dois: 2 Al(s) → 2 Al+3(aq) + 6e-

∆G° = −6 x F x (+1,66)
Multiplicando a segunda semirreação por três: 3 Hg+2(aq) + 6e- → 3 Hg(amálgama de Ag) ∆G° = −6 x F x (+0,85)
Somando as duas semirreações apresentadas acima: 2 Al(s) → 2 Al+3(aq) + 6e- ∆G° = −6 x F x (+1,66)
+
2 Al(s) + 3 Hg (aq) → 2 Al+3(aq) + 3 Hg(amálgama de Ag)
+2 ∆G° = −6 x F x E°
Reação de oxidação: 2 Al(s) → 2 Al+3(aq) (Ânodo)
Reação de redução: 3 Hg+2(aq) → 3 Hg (Cátodo)

Item b) Cálculo da força eletromotriz:

−6 x F x E° = −6 x F x (+ 1,66) + [−6 x F x (+ 0,85)]


−6 x F x E° = −6 x F x (+ 1,66) + [−6 x F x (+ 0,85)]

E° = +1,66 + 0,85 = + 2,51 V

713
1600 FÍSICO-QUÍMICA APLICADA EXERCÍCIOS COMENTADOS - IME – ITA – OLIMPÍADA

Item c) Cálculo da energia livre de Gibbs padrão (ΔG°): ∆G° = −6 x F x E°

96500 C
∆G° = −6 mol de elétrons x x (+2,51) = −1453290 J (−1453,29 kJ)
1 mol de elétrons

Item d) Cálculo da constante de equilíbrio (Keq): ∆G° = −6 x F x E° = −R x T x ln K eq

− 1453290 = − 8,314 x (25 + 273) x ln K eq

− 1453290
ln K eq = = 586,58
− 2477,572
K eq = e+586,58

Questão 1295 – (OLIMPÍADA BRASILEIRA DE QUÍMICA) 100 ml de uma solução 0,05 mol/L de CrO42- , necessita
ser reduzida completamente a Cr3+ . Se usarmos a eletrólise com uma corrente de 2,5 A. Determine o tempo em
segundos necessários para se processar tal reação.

Resolução: Cálculo do número de mol de cromo:


1000 ml da solução ------------------ 0,05 mol
100 ml da solução -------------------- nCr
nCr = 0,005 mol
Semirreação de rediução do cromo: Cr6+ + 3e- → Cr3+
1 mol de Cr3+ ------------- 3 mol de elétrons x 96500 C/mol de elétrons
0,005 mol ----------------- Q = 2,50 x tempo
0,005 x 3 x 96500
Tempo = = 579 s
2,50

Questão 1296 – (OLIMPÍADA NORTE – NORDESTE DE QUÍMICA) O citocromo, molécula que representaremos
como CyFe+2, reage com o ar que respiramos para fornecer energia necessária para sintetizar adenosina trifosfato
(ATP). O corpo usa ATP como fonte de energia para outras reações. Com base nos potenciais de redução:
CyFe+3(aq) + e- → CyFe+2(aq) E0 = + 0,22 V
O + 4 H + 4e → 2H2O
2 + - E = + 0,82 V
0

Responda as seguintes questões:


a) Considerando que esta reação ocorra em uma célula eletroquímica, indique os eletrodos (catodo e anodo)
onde cada semirreação deve ocorrer.
b) Escreva a equação geral balanceada e determine o potencial E0 gerado nessa reação.
c) Explique se esse processo é exotérmica ou endotérmica.
d) Explique porque essa transformação é do tipo oxido-redução, e identificando os agentes oxidante e redutor.

Resolução: Determinação da reação global e o valor do potencial E°. Invertendo e multiplicando por quatro a primeira
semirreação:

4 CyFe+2(aq) → 4 CyFe+3(aq) + 4 e- ∆G° = −4 x F x (−0,22)


O2 + 4 H+ + 4e- → 2 H2O ∆G° = −4 x F x (+0,82) +
4 CyFe+2(aq) + O2 + 4 H+→ 4 CyFe+3(aq) + 2 H2O

− 𝟒 𝐱 𝐅 x E° = − 𝟒 𝐱 𝐅 x (−0,22) + [−𝟒 𝐱 𝐅 x (+0,82)]


E° = −0,22 + 0,82 = + 0,60 V

714
1600 FÍSICO-QUÍMICA APLICADA EXERCÍCIOS COMENTADOS - IME – ITA – OLIMPÍADA

a) Ânodo: 4 CyFe+2(aq) → 4 CyFe+3(aq) + 4 e-


Cátodo: O2 + 4 H+ + 4e- → 2 H2O
b) Equação química para o funcionamento da pilha: 4 CyFe+2(aq) + O2 + 4 H+→ 4 CyFe+3(aq) + 2 H2O

Cálculo do potencial: E° = + 0,60 V

c) Como o oxigênio molecular se apresenta na reação química como agente oxidante, o processo é exotérmico.

d) A reação é do classificada como redox (óxido-redução), pois há variação do número de oxidação, em que o
CyFe+2(aq) apresenta variação de +2 para +3 (reação de oxidação), o que caracteriza um agente redutor e o O 2 como
agente oxidante.

Questão 1297 – (MESTRE JOÃO ROBERTO DA PACIÊNCIA NABUCO) Numa célula galvânica de zinco e chumbo,
as concentrações das soluções em cada semicélula são 65 g.L-1 de íons Zn+2 e 20,7 g.L-1 de Pb+2 à 25°C. determinar
a ddp da pilha nessa situação. Informação para a resolução do problema:

Zn2+(aq) + 2e- → Zn(s) Eo = - 0,76 V


Pb2+(aq) + 2e- → Pb(s) Eo = - 0,13 V

Resolução: Cálculo da concentração da quantidade de matéria para cada íon:


g
65 mol
[Zn+2 ] = L
g =1 L
65
mol
g
20,7 L mol
+2
[Pb ] = g = 0,1 L
207
mol

Cálculo do potencial padrão (E°), invertendo a semirreação de zinco:

Zn(s) → Zn2+(aq) + 2e- ∆G° = −2 x F x (+ 0,76)


Pb2+(aq) + 2e- → Pb(s) ∆G° = −2 x F x (− 0,13) +
Zn(s) + Pb2+(aq) → Pb(s) + Zn2+(aq) ∆G° = ?

−2 x F x E° = −2 x F x (+ 0,76) − 2 x F x (− 0,13)

−2 x F x E° = −2 x F x (+ 0,76) − 2 x F x (− 0,13)

E° = + 0,76 − 0,13 = + 0,63 V

RxT [Zn+2 ] x a
Cálculo da ddp fora das condições padrões: E = E° − n x F x ln [[Pb+2 ] x aPb ]
Zn

8,314 x (25 + 273) 1 2477,572


E = +0,63 − x ln [ ] = + 0,63 − x ln10 = +0,627 V
2 x 96500 0,10 193000

Questão 1298 – (OLIMPÍADA CEARENSE DO ENSINO SUPERIOR DE QUÍMICA) Suponha que seja possível
construir uma semicélula galvânica usando a semirreação Cu2+(aq) + 1e− → Cu+(aq) e um eletrodo inerte. Na semicélula
construída, as atividades em solução do Cu+ e Cu2+ são de 0,01 e 0,50, respectivamente. Dados:
Cu2+(aq) + 2e− ⇌ Cu(s) E° = 0,339 V
Cu+(aq) + 1e− ⇌ Cu(s) E° = 0,521 V

715
1600 FÍSICO-QUÍMICA APLICADA EXERCÍCIOS COMENTADOS - IME – ITA – OLIMPÍADA

Se for utilizada sob pressão de 1,00 bar e a 25°C, a semicélula apresentará potencial de:
a) 0,06 V
b) 0,18 V
c) 0,26 V
d) 0,43 V
e) 0,86 V

Resolução: Alternativa C.
Determinação do potencial padrão, invertendo a segunda semirreação:

Cu2+(aq) + 2e− ⇌ Cu(s) ∆G0 = −2 x F x (+0,339)


Cu(s) ⇌ Cu+(aq) + 1e− ∆G0 = −1 x F x (−0,521) +

Cu2+(aq) + 1e− ⇌ Cu+(aq) −1 x F x E° = −2 x F x (+0,339) + [−1 x F x (−0,521)]


−1 x E° = −0,678 + 0,521

E° = + 0,157 V

RxT RxT 𝑎1 +
Aplicando a equação de Nernst, temos: E = E° − n x F x lnQ = E° − n x F x ln {𝑎1𝐶𝑢 }
𝐶𝑢+2

8,314 x (25 + 273) 0,010


E = + 0,157 − x ln { } = + 0,157 + 0,100 = + 0,257 V
1 x 96500 0,50
Questão 1299 – (U. S. NATIONAL CHEMISTRY OLYMPIAD) Alumínio é um metal de alta redução. O padrão de
redução para o Al+3(aq) e Fe+3(aq) são:
Al3+(aq) + 3e- → Al(s) E0 = - 1,66 V
Fe3+(aq) + 3e- → Fe (s) E0 = - 0,04 V

E a entropia padrão S° para estas espécies são:

Espécies S° (J.mol-1.K-1)
Al(s) 28
Fe(s) 27
Al+3(aq) - 293,3
Fe+3(aq) - 313,8

a) Calcule ΔG° para a redução do Fe+3(aq) pelo Al a 25°C.


b) Calcule ΔS° para a redução do Fe+3(aq) pelo Al a 25°C.
c) Calcule ΔH° para a redução do Fe+3(aq) pelo Al a 25°C.

Resolução:
Item a) Cálculo da energia livre de Gibbs (ΔG°), mas primeiramente será necessário calcular a ddp da pilha galvânica.

Invertendo a semirreação do alumínio: Al(s) → Al+3(aq) + 3e- ∆G° = −3 x F x (+1,66)


Semirreação do ferro: Fe+3(aq) + 3e- → Fe(s) ∆G° = −3 x F x (−0,04)
Somando as semirreações, temos:
Al(s) → Al+3(aq) + 3e- ∆G° = −3 x F x (+1,66)

716
1600 FÍSICO-QUÍMICA APLICADA EXERCÍCIOS COMENTADOS - IME – ITA – OLIMPÍADA

Fe+3(aq) + 3e- → Fe(s) ∆G° = −3 x F x (−0,04) +

Al(s) + Fe+3(aq) → Fe(s) + Al+3(aq)


−3 x F x E° = −3 x F x (+1,66) + [−3 x F x (−0,04)]
E° = +1,66 − 0,04 = +1,62 V
Cálculo da variação da energia livre de Gibbs padrão: ∆G° = −3 x F x E°
96500 C
∆G° = −3 mol de elétrons x x (+ 1,62) = −468990 J (≅ −469 kJ)
1 mol de elétrons

0 0 0
Item b) Cálculo da entropia padrão da reação (ΔS°): ∆Sreação = ∑ Sprodutos − ∑ Sreagentes

0 0 0 0 0
∆Sreação = {1 x SAl+3 + 1 x SFe } − {1 x SFe+3 + 1 x SAl }

J
= {1 x (−293,3) + 1 x 27} − {1 x (−313,8) + 1 x 28} = +19,50
mol x K
0 0 0
Item c) Cálculo da entalpia padrão da reação (ΔH°): ∆Greação = ∆Hreação − T x ∆Sreação

0
−468990 = ∆Hreação − (25 + 273) x 19,50

0
J kJ
∆Hreação = −468990 + (298 x 19,50) = −463179 (≅ −463,2 )
mol mol
Questão 1300 - Calcule os seguintes itens apresentados a seguir:

a) Considere que uma corrente com uma amperagem desconhecida atue sobre uma solução aquosa de sulfato
de cobre II, sendo eletrolisada a cobre metálico depositado. Se uma corrente foi utilizada durante cinco horas e que
404 miligramas de cobre foram obtidos, determine o valor da corrente elétrica.
b) Determine a carga necessária para que 2,0 gramas de cobre sejam depositados a partir de uma solução de
sulfato de cobre, com a presença de uma corrente de 5,0 A.

Resolução: Item a)

1 mol de Cu --------- 63,55 g de cobre ----------- 2 mol x (96500 C/mol)


404 x 10-3 g --------------- Q
Q = 1226,94 C
No enunciado do problema, o tempo corresponde a cinco horas. Convertendo para segundos, temos:
3600 segundos
Tempo = 5 horas x ( ) = 18000 s
1h
q 1226,94 C
Cálculo da corrente elétrica: i = t
= 18000 s
= 6,82 x 10−2 A

Item b)
1 mol de Cu --------- 63,55 g de cobre ------------------------ 2 mol x (96500 C/mol)
2,0 g de cobre ------------------------- Q
Q = 6073,96 C
Questão 1301 – Considere os potenciais padrão para a pilha galvânica composta por ferro e prata, apresentada pelas
seguintes semirreações:
Fe2+(aq) + 2e- → Fe(s) ε° = - 0,44 V
Ag+(aq) + 1e- → Ag(s) ε° = + 0,80 V
a) determine a reação global e calcule sua respectiva ddp;

717
1600 FÍSICO-QUÍMICA APLICADA EXERCÍCIOS COMENTADOS - IME – ITA – OLIMPÍADA

b) dê os pólos positivo e negativo;


c) dê a representação da pilha pelas recomendações da IUPAC.

Resolução: Item a) Invertendo a primeira semirreação e multiplicando a segunda semirreação por dois:

Fe(s) → Fe2+(aq) + 2e- ∆G° = −2 x F x (+0,44)


2 Ag+(aq) + 2e- → 2 Ag(s) ∆G° = −2 x F x (+0,80) +

Fe(s) + 2 Ag+(aq) → Fe2+(aq) + 2 Ag(s)

−2 x F x E° = −2 x F x (+0,44) + [−2 x F x (+ 0,80)]

E° = + 0,44 + 0,80 = + 1,24V

Item b)

Fe(s) → Fe2+(aq) + 2e- (reação de oxidação / ânodo / pólo negativo)

2 Ag+(aq) + 2e- → 2 Ag(s) (reação de redução / cátodo / pólo positivo)

Item c) Notação da pilha, segundo as recomendações da IUPAC: Fe(s) |Fe+2 +


(aq) ‖Ag (aq) |Ag (s)

Questão 1302 – (OLIMPÍADA BRASILEIRA DE QUÍMICA) Considere a equação química que representa a oxidação
de cloreto por permanganato, em meio ácido: MnO−2 + −
4 + H + Cl → Mn
+2
+ Cl2 + H2 O. Após o
−2
balanceamento desta equação, o coeficiente estequiométrico para o íon 𝑀𝑛𝑂4
é 2 e para o H+ é:
a) 4
b) 8
c) 10
d) 14
e) 16

Resolução: Alternativa E.
A variação do número de oxidação do manganês varia de +7 para +2: Mn+7 → Mn+2, caracterizando uma
semirreação de redução. A variação do número de oxidação do manganês varia de -1 para 0: Cl− → Cl2 ,
caracterizando uma semirreação de oxidação [∆ = 0 – (-1) = 1 x 2 = 2]. Realizando o balanceamento, a equação
química redox fica da seguinte maneira:
𝟐 MnO−2 + −
4 + 𝟏𝟔 H + 10 Cl → 2 Mn
+2
+ 5 Cl2 + 8 H2 O

Questão 1303 – (OLIMPÍADA BRASILEIRA DE QUÍMICA) “O Brasil possui um dos mais eficientes ciclos de
reciclagem de alumínio do mundo. De acordo com a Associação Brasileira do Alumínio (Abal), o índice supera os 35%
ante cerca de 29% da média mundial. Segundo o consultor de Marketing da Alcoa, Eduardo Lima, vem crescendo
ano-a-ano no mundo o uso de alumínio primário reciclado em relação ao metal primário, passando de 17% em 1960
para 33% em 2004. A estimativa para 2020 é a de que o porcentual pule para 40%.” (fonte:
http://sustentar.net/2013/sem-categoria/reciclagem-de-aluminio-deve-saltar-para-40) A reciclagem do alumínio é de
grande importância ambiental e energética, uma vez que a sua reciclagem economiza cerca de 95% de energia
elétrica.
a) Equacione a reação da eletrólise ígnea do óxido de alumínio. Indique os produtos obtidos no cátodo e no ânodo
da cuba eletrolítica.
b) Determine a massa de alumínio produzida em uma cuba eletrolítica com corrente constante de 8000 A durante
150 horas.

Resolução:

718
1600 FÍSICO-QUÍMICA APLICADA EXERCÍCIOS COMENTADOS - IME – ITA – OLIMPÍADA

Item a) Reação de dissociação do óxido de alumínio: Al2 O3(s) → 2Al+3 −2


(l) + 3O(l)
Reação que ocorre no cátodo: Al+3 −
(l) + 3e → Al(s)
0
1
Reação que ocorre no ânodo: O−2
(l) → 2 O2(g) + 2e

Determinação da reação global, multiplicando a primeira semirreação por 2 e multiplicando a segunda semirreação
por três:

Reação que ocorre no cátodo: 2Al+3 − 0


(l) + 6e → 2Al(s)
3
Reação que ocorre no ânodo: 3O−2
(l) → 2 O2(g) + 6e

Item b) Somado as semirreações, temos:

2Al+3 − 0
(l) + 6e → 2Al(s)
3
3O−2
(l) → O2(g) + 6e

+
2
−2 3
2Al+3 0
(l) + 3O(l) → 2Al(s) + O2(g)
2
Produto formado no cátodo: alumínio metálico
Produto formado no ânodo: oxigênio gasoso

Cálculo da massa de alumínio metálico produzido:

96500 C
54 g de de Al(s) − − − − 6 mol de e− x
1 mol de e−
3600 s
mAl(s) − − − − − − − − 8000 x 150 h x
1h
8000 x 150 x 3600 x 54
mAl(s) = = 402901,55 g (≅ 403 kg)
6 x 96500

Questão 1304 – (U. S. NATIONAL CHEMISTRY OLYMPIAD) When aluminum oxide is electrolyzed in the industrial
process for the production of aluminum metal, aluminum is produced at one electrode and oxygen gas is produced at
the other. For a given quantity of electricity, what is the ratio of moles of aluminum to moles of oxygen gas?
a) 1:1
b) 2:1
c) 2:3
d) 4:3

Resolução: Alternativa D.
O processo ocorre através de uma eletrólise ígnea.

Processo de fundição do óxido de alumínio: Al2 O3(s) → 2 Al+3 −2


(l) + 3 O(l)

Semirreação que ocorre no cátodo: Al+3 −


(l) + 3e → Al(s)

Semirreação que ocorre no ânodo: 2O−2


(l) → O2(g) + 4e

Multiplicando a primeira semirreação por quatro: 4 Al+3 −


(l) + 12e → 4 Al(s)

Multiplicando a segunda semirreação por três: 6O−2


(l) → 3 O2(g) + 12e

Somando as semirreações, temos:

4 Al+3 −
(l) + 12e → 4 Al(s)

719
1600 FÍSICO-QUÍMICA APLICADA EXERCÍCIOS COMENTADOS - IME – ITA – OLIMPÍADA

6O−2
(l) → 3 O2(g) + 12e

+

4 Al+3 −2
(l) + 6O(l) → 𝟒 Al(s) + 𝟑 O2(g)

Questão 1305 – (OLIMPÍADA CEARENSE DO ENSINO SUPERIOR DE QUÍMICA) Dadas as concentrações não-
padrão para a reação abaixo: Zn(s) + Cu+2 −1 +2 −1
(aq) (0,100 mol. L ) → Zn(aq) (0,02 mol. L ) + Cu(s) . Sabendo que
os coeficientes de atividade iônica para Cu2+ e Zn2+, nessas condições são 0,485 e 0,675 e que E°(Cu2+/Cu) = + 0,34
V e E°(Zn2+/Zn) = – 0,76 V, pode-se afirmar que E para esta pilha de Daniell é:
a) 1,10 V
b) 1,16 V
c) 1,04 V
d) 1,00 V
e) 0,96 V

Resolução: Alternativa A.
Determinação da reação global e da ddp, invertendo a semirreação do zinco:

Cu+2 −1 −
(aq) (0,100 mol. L ) + 2e → Cu(s) ∆G0 = −2 x F x (+0,34)

Zn(s) → Zn+2 −1
(aq) (0,02 mol. L ) + 2e

∆G0 = −2 x F x (+0,76) +

Cu+2 +2
(aq) + Zn(s) → Zn(aq) + Cu(s)

− 2 x F x E° = −2 x F x (+0,34) + [−2 x F x (+0,76)]


E° = + 0,34 + 0,76 = + 1,10 V
RxT
Cálculo do potencial fora das condições padrões: E = E° − x lnQ
nxF

RxT 𝑎1 +2 𝑥 𝑎1𝐶𝑢
E = E° − x ln { 𝑍𝑛 }
nxF 𝑎1𝐶𝑢+2 𝑥 𝑎1𝑍𝑛

Atividade dos sólidos igual a 1.

RxT 𝑎1𝑍𝑛+2 𝑥 1
E = E° − x ln { 1 }
nxF 𝑎𝐶𝑢+2 𝑥 1
𝑎𝑍𝑛+2

8,314 x (25 + 273) ⏞


0,675 x 0,02
E = + 1,10 − x ln = + 1,10 + 0,0164 = +1,11 V
1 x 96500 0,485
⏟ x 0,100
{ 𝑎𝐶𝑢+2 }
Questão 1306 – (ITA) Para niquelar uma peça de cobre, usou-se uma solução de sulfato de níquel (II), e aparelhagem
conveniente para eletrodeposição. Terminada a niquelação, verificou-se que haviam passado pelo circuito 1,0 x 10-3
mol de elétrons. Conclui-se, então, que a quantidade de níquel depositada sobre a peça de cobre é:
a) 5,0 x 10-4 mol, isto é, 29,35 x 10-3 g
b) 1,0 x 10-3 mol, isto é, 58,71 x 10-3 g
c) 2,0 x 10-3 mol, isto é, 117,42 x 10-3 g
d) 2,5 x 10-4 mol, isto é, 14,67 x 10-3 g
e) 1,0 x 10-3 mol, isto é, 29,35 x 10-3 g

Resolução: Alternativa A.

720
1600 FÍSICO-QUÍMICA APLICADA EXERCÍCIOS COMENTADOS - IME – ITA – OLIMPÍADA

A questão trata da eletrólise do níquel (Ni), a partir de uma solução de sulfato de níquel (II), NiSO4.

NiSO4(s) → Ni+2(aq) + SO4-2(aq)


Ni+2(aq) + 2 elétrons → Ni(s)

A questão pergunta sobre a eletrodeposição do níquel, ou seja, sobre a reação catódica: Ni+2(aq) + 2 e- → Ni(s).

Cálculo do massa de níquel depositada:

58,70 gramas de Ni ------------------------ 2 mol de elétrons de Ni


mNi ------------------------------------------- 1,0 x 10-3 mol de elétrons
mNi = 2,93 x 10-2 g

Cálculo do número de mol de zinco depositada:


58,70 gramas de Ni ------------------------ 1 mol de Ni
2,93 x 10-2 g -------------------------------- nNi
nNi = 5,00 x 10-4 mol

Questão 1307 – (ITA) Considere os dois eletrodos (I e II) seguintes e seus respectivos potenciais na escala do
eletrodo de hidrogênio (E0) e nas condições-padrão:
I. 2 F–(aq)  2e– (CM) + F2(g); E° = + 2,87 V
II. Mn2+(aq) + 4H2O(l)  5e– (CM) + 8H+(aq) + MnO4–(aq); E° = + 1,51 V
A força eletromotriz de um elemento galvânico construído com os dois elementos acima é de
a) -1,81 V
b) -1,13 V
c) 0,68 V
d) 1,36 V
e) 4,38 V

Resolução: Alternativa D.
Como o flúor apresenta maior potencial de redução, ele irá reduzir, enquanto o manganês irá oxidar. Analisando a
primeira semirreação, invertendo e multiplicando por dois, temos:
10 F–(aq)  10e– (CM) + 5 F2(g) E° = + 2,87 V ∆G° = −10 x F x (+2,87)
Analisando a segunda semirreação, invertendo e multiplicando por dois, temos:
10e– (CM) + 16 H+(aq) + 2 MnO4–(aq)  2 Mn2+(aq) + 8 H2O(l)
E° = - 1,51 V ∆G° = −10 x F x (−1,51)
Cálculo do potencial padrão: − 10 x F x E° = −10 x F x (+2,87) + [−10 X F x (−1,51)]
E° = + 2,87 − 1,51 = +1,36 V

Questão 1308 – (ITA) Por uma célula eletrolítica passou uma carga correspondente a 0,20 Faraday. Num dos
eletrodos ocorreu a reação seguinte: MnO4- + 8 H+ + 5e- → Mn+2 + 4 H2O. A quantidade de água produzida neste
eletrodo, em virtude desta reação de eletrodo, é:
a) (0,20 x 4) mol
b) (0,20) x (4/5) mol
c) (0,20) x (5/4) mol

721
1600 FÍSICO-QUÍMICA APLICADA EXERCÍCIOS COMENTADOS - IME – ITA – OLIMPÍADA

d) (0,20 x 5) mol
e) (0,20 x 4 x 5) mol

Resolução: Alternativa B.

MnO4- + 8 H+ + 5e - → Mn+2 + 4 H2O


4 mol de H2O ---------- 5 mol de elétron x {1 F / mol de elétron}
nH2O --------------------- 0,20 F
4 x 0,20
nH2 O = ( ) mol
5

Questão 1309 – (ITA) Uma tubulação de aço enterrada em solo de baixa resistividade elétrica é protegida
catodicamente contra corrosão, pela aplicação de corrente elétrica proveniente de um gerador de corrente contínua.
Considere os seguintes parâmetros:

I. Área da tubulação a ser protegida: 480 m²


II. Densidade de corrente de proteção: 10 mA.m-2

Considere que a polaridade do sistema de proteção catódica seja invertida pelo período de 1 hora. Assinale a opção
correta que expressa a massa, em gramas, de ferro consumida no processo de corrosão., calculada em função de
íons Fe+2(aq). Admita que a corrente total fornecida pelo gerador será consumida no processo de corrosão da tubulação.
a) 1 x 10-3
b) 6 x 10-2
c) 3 x 10-1
d) 5
e) 20

Resolução: Alternativa D.
i
Cálculo da corrente, em amperes, a partir da seguinte equação matemática: j = , onde:
A
J = densidade de corrente, I = corrente elétrica e A = área.
i=JxA
i = 10 x 10-3 x 480 = 4,80 A
Semirreação do cátodo: Fe+2(aq) + 2e- → Fe(s)
1 mol de Fe ---------- 56 g ---------- 2 mol de elétron x (96500 C/mol de elétron)
mFe ------------ (4,80 x 3600) C
mFe = 5,01 g de ferro metálico

722
1600 FÍSICO-QUÍMICA APLICADA EXERCÍCIOS COMENTADOS - IME – ITA – OLIMPÍADA

Questão 1310 – (ITA) A corrente elétrica que passou através dos fios conectores de cobre do circuito durante a
eletrólise foi igual a 1,6 x 10-2 amperes. Qual das opções abaixo contém a conclusão correta sobre o número de
elétrons que passou, por segundo, através da secção (X --- Y) do fio de cobre, conforme assinalado na figura?

a) 1,6 x 10-2
b) 1,0 x 10 12
c) 1,0 x 1017
d) 6,0 x 1020
e) 9,7 x 1021
Resolução: Alternativa C.
Sabendo que a corrente elétrica é a razão entre a carga (q) pelo tempo (t), representada pela seguinte equação: i =
Q
tempo
.
Sendo a carga, o produto do número de elétrons (n) pela carga de um elétron (e- = 1,60 x 10-19C), logo, a equação
Q n x e−
pode ser escrita da corrente elétrica pode ser expressa da seguinte maneira: i = tempo = tempo

Q n x e−
Cálculo do número de elétrons (n): i = tempo = tempo

n x 1,60 x 10−19
1,60 x 10−2 =
1,0

n = 10−17 elétrons

Questão 1311 – (ITA) Por uma célula eletrolítica passa uma corrente de 0,965 A, num dos eletrodos a reação que
ocorre é a seguinte: Cr2O7 –2 + 14 H+ + 6e – → 2 Cr+3 + 7 H2O. O tempo certo durante o qual essa corrente deve passar
para que sejam produzidos 0,4 mol de íons crômio é igual a:
a) {(1/0,4) x (2/6) x 1.105}s
b) {(1/0,4) x (2 x 6) x 1.105}s
c) {(0,4) x (2) x 1.105}s
d) {(0,4) x (6) x 1.105}s
e) {(0,4) x (6/2) x 1.105}s

Resolução: Alternativa E.
Equação química global da célula eletrolítica: Cr2O7 –2 + 14 H+ + 6e – → 2 Cr+3 + 7 H2O
Cálculo do tempo, através da equação química balanceada:
2 mol de Cr+3 -------------- 6 mol de e- x 96500 C/mol de e-

723
1600 FÍSICO-QUÍMICA APLICADA EXERCÍCIOS COMENTADOS - IME – ITA – OLIMPÍADA

0,4 mol de Cr+3 ----------- Q = 0,965 x t

965
2x x tempo = 0,40 x 6 x 96500
1000

2 x tempo = 0,40 x 6 x 105

6
tempo = {0,40 x x 105 } s
2

Questão 1312 - (ITA) A deposição eletrolítica de 2,975 g de um metal de massa atômica 119 requerou 9650 coulombs.
O número de oxidação do metal é:
a) +1
b) +2
c) +3
d) +4
e) Nenhuma das respostas acima está certa.

Resolução: Alternativa D.
O elemento químico que apresenta massa atômica 119 corresponde ao estanho (Sn).
Cálculo do número de oxidação do estanho. Semirreação de redução do Estanho:
Sna+(aq) + ae- → Sn(s).

Cálculo do tempo de operação da célula eletroquímica:


119 g de Sn ---------------------------- (a mol de elétrons x 96500 C/mol de elétrons)
2,975 g de Sn -------------------------- 9650 C
9650 x 119
a= = +4; Sn+4
2,975 x 96500

Questão 1313 – (ITA) Deseja-se depositar uma camada de 0,85 gramas de níquel metálico no catodo de uma célula
eletrolítica, mediante a passagem de uma corrente de 5 A através de uma solução aquosa de nitrato de níquel.
Assinale a opção que apresenta o tempo necessário para esta deposição, em minutos.
a) 4,3
b) 4,7
c) 5,9
d) 9,3
e) 17,0

Resolução: Alternativa D.
Ni(NO3)2(aq) → Ni+2(aq) + 2 NO3- (aq)

Semirreação de redução do níquel: Ni+2(aq) + 2e- → Ni(s).

Cálculo do tempo de operação da célula eletroquímica:


58,69 g de Ni -------------------------- 2 mol de e- x 96500 C/mol de e-
0,85 g de Ni --------------------------- 5 x tempo

724
1600 FÍSICO-QUÍMICA APLICADA EXERCÍCIOS COMENTADOS - IME – ITA – OLIMPÍADA

164050 1 minuto
tempo = = 559,04 segundos x = 9,32 min
293,45 60 segundos

Questão 1314 - (ITA) Na reação iônica Ni(s) + Cu+2 +2


(aq) → Ni(aq) + Cu(s) :
a) O níquel é o oxidante, porque ele é oxidado
b) O níquel é o redutor, porque ele é oxidado
c) O íon cúprico é o oxidante, porque ele é oxidado
d) O íon cúprico é o redutor, porque ele é reduzido
e) Não há oxidante nem redutor, porque não se trata de uma equação de oxirredução.

Resolução: Alternativa B.
Através da reação de oxirredução, o níquel é o agente redutor, proveniente de uma reação de oxidação. Já o íon
cúprico é o agente oxidante, proveniente de uma reação de redução.
Questão 1315 – (OLIMPÍADA BRASILEIRA DE QUÍMICA) Considere uma pilha formada por duas lâminas metálicas,
uma de zinco e outra de cobre imersos em suas respectivas soluções de Zn+2 e Cu+2 separados por uma ponte salina,
conforme figura ao lado. Nessa pilha, é ligada uma lâmpada entre os eletrodos e após certo tempo de funcionamento,
observa-se que a lâmina de zinco sofre uma diminuição de massa e a de cobre um aumento.

Com relação a esta pilha é correto afirmar que:


a) O cobre sofre oxidação
b) O íon Cu+2 é o agente redutor
c) O eletrodo de zinco é o pólo (-)
d) No cátodo ocorre reação de oxidação
e) O sentido do fluxo de elétrons é o eletrodo de cobre para o de zinco passando pelo circuito externo

Resolução: Alternativa C.
Observando a figura da pilha, o funcionamento ocorre pela seguinte reação de oxirredução: Zn(s) + Cu+2(aq) → Zn+2(aq)
+ Cu(s)
Através da reação de oxirredução, temos:
➢ A semirreação de zinco é a reação de oxidação, sendo o Zinco metálico o agente redutor.
➢ A semirreação de cobre é a reação de redução, sendo o cobre aquoso, Cu+2(aq), o agente oxidante.
➢ O sentido do fluxo de elétrons sai do Zinco (ânodo) para o Cobre (cátodo).
➢ O eletrodo de zinco é o pólo negativo.
➢ O eletrodo de cobre é o pólo positivo.

725
1600 FÍSICO-QUÍMICA APLICADA EXERCÍCIOS COMENTADOS - IME – ITA – OLIMPÍADA

Questão 1316 – (GRADUAÇÃO CEFETES) Qual a opção que apresenta o tempo requerido para se obter 7,10 g da
substância produzida no ânodo durante a eletrólise de cloreto de magnésio fundido, usando uma corrente de 7,72 A?
a) 1250 s
b) 3250 s
c) 6500 s
d) 1625 s
e) 2500 s

Resolução: Alternativa E.
Cloreto de magnésio na presença de água, irá ocorrer a dissociação da seguinte forma: MgCl2(aq) → Mg+2(aq) + 2 Cl-
(aq)

Reação que ocorre no ânodo: 2 Cl−


(aq) → Cl2(g) + 2 e

Cálculo do tempo durante o processo de eletrólise aquosa do ânodo (Cl-):


2 mol de Cl- ---------- 71,0 g de Cl- -------------- 2 mol de elétrons x (96500 C/mol de elétrons)
7,10 g de Cl- ---------------- Q = (7,72 x tempo)
Tempo = 2500 s

Questão 1317 - (OLIMPÍADA DE QUÍMICA DO RIO DE JANEIRO) Uma solução de sulfato de níquel II foi eletrolisada
durante 1,50 horas entre eletrodos inertes. Se forem depositados 35,00 g de níquel, qual o valor da corrente elétrica?
a) 10,7 A
b) 12,3 A
c) 15,1 A
d) 21,3 A
e) 23,1 A

Resolução: Alternativa D.
Dissociação do sulfato de níquel: NiSO4(s) → Ni+2(aq) + SO4-2(aq)
Reação que ocorre no cátodo: Ni+2(aq) + 2 e- → Ni(s)
3600 s
Cálculo da carga Q: Tempo = 1,50 horas x 1h
= 5400 s

1 mol de Ni ---------- 58,70 g de Ni ------------ 2 mol de elétrons x 96500 C/mol de elétrons


35,00 g de Ni ------------ Q
Q = 115076,66 C

115076
Cálculo da corrente elétrica (i): i = 5400
= 21,31 A

Questão 1318 – (OLIMPÍADA DE QUÍMICA DO RIO DE JANEIRO) Qual a massa de zinco metálico depositada após
a eletrólise de uma solução aquosa de sulfato de zinco que durou aproximadamente 3,50 horas, sob corrente de 6,00
A?
a) 63,2 g
b) 51,2 g
c) 25,6 g
d) 7,12 mg
e) 427 mg

Resolução: Alternativa C.
Reação de dissociação do sulfato de zinco: ZnSO4(s) → Zn+2(aq) + SO4-2(aq)

726
1600 FÍSICO-QUÍMICA APLICADA EXERCÍCIOS COMENTADOS - IME – ITA – OLIMPÍADA

Reação que ocorre no cátodo: Zn+2(aq) + 2 e- → Zn(s)


3600 s
Tempo = 3,50 horas x = 12600 s
1h
Cálculo da carga Q: Q = 12600 x 6,0 = 75600 C
Cálculo da massa de zinco depositada:
65,50 g de Zn ------------- 2 mol de elétrons x (96500 C/mol de elétrons)
mZn ------------------------- 75600 C
mZn = 25,66 g

Questão 1319 – (OLIMPÍADA BRASILEIRA DE QUÍMICA) Um eletrodo de vanádio é oxidado eletroliticamente. A


massa do eletrodo diminui de 114 mg após passagem de 650 coulombs de corrente. Qual o número de oxidação do
vanádio no produto?
a) +1
b) +2
c) +3
d) +4
e) +5

Resolução: Alternativa C.

Semirreação de redução do Vanádio: Va+(aq) + ae- → V(s).

Cálculo do tempo de operação da célula eletroquímica:


51 g de V ------------------------------ (a mol de elétrons x 96500 C/mol de elétrons)
114 x 10-3 g de V -------------------- 650 C
650 x 51 33150
a= −3
= = +3,01 → V +3
114 x 10 x 96500 11001

Questão 1320 – (OLIMPÍADA BRASILEIRA DE QUÍMICA) A reação básica que ocorre em uma cela na qual Al2O3 e
sais de alumínio são eletrolisados é: Al+3(aq) + 3e- → Al(s). Se a cela opera a 5,0 V e 1,0 x 105 A, quantos gramas de
alumínio metálico serão depositados em 8 horas de operação da cela?
a) 27 kg
b) 85 kg
c) 180 kg
d) 270 kg
e) 540 kg

Resolução: Alternativa D.
Semirreação de redução do alumínio: Al+3(aq) + 3e- → Al(s)
3600 s
Tempo = 8,0 horas x = 28800 s
1h
Cálculo da massa de alumínio (mAl):
27,0 g de Al ---------- 3 mol de elétrons x 96500 C/mol de elétrons
mAl --------------------- Q = (1,0 x 105 x 28800) C
mAl = 268601,04 g (268,60 kg)

727
1600 FÍSICO-QUÍMICA APLICADA EXERCÍCIOS COMENTADOS - IME – ITA – OLIMPÍADA

Questão 1321 – (OLIMPÍADA BRASILEIRA DE QUÍMICA) A corrente necessária para, no período de 100 horas,
produzir 1 kg de magnésio a partir de cloreto de magnésio fundido situa-se entre:
a) 5,0 e 10,0 A
b) 10,0 e 15,0 A
c) 15,0 e 20,0 A
d) 20,0 e 25,0 A
e) 25,0 e 30,0 A

Resolução: Alternativa D.
Analisando a eletrólise ígnea do cloreto de magnésio: MgCl2(s) → Mg+2(l) + 2 Cl-(l)
Semirreação que ocorre no cátodo: Mg+2(aq) + 2 e- → Mg(s)
Semirreação que ocorre no ânodo: 2 Cl-(l) → Cl2(g) + 2e-
Somando as duas semirreações, temos:
Mg+2(l) + 2 e- → Mg(s)
2 Cl-(l) → Cl2(g) + 2e- +
Mg+2(l) + 2 Cl-(l) → Mg(s) + Cl2(g)

Na eletrólise ígnea do cloreto de magnésio há a produção de magnésio metálico e cloro gasoso.


Semirreação no cátodo do magnésio: Mg+2(aq) + 2e- → Mg(s)
3600 s
Cálculo do tempo: Tempo = 100 horas x 1 horas = 360000 s

Cálculo da carga (Q):


24,0 g de Mg --------------- 2 mol de elétrons x 96500 C/mol de elétrons
1000 g de Mg -------------- Q
Q = 8,042 x 106 C
Q 8,046 x 106
Cálculo da corrente (i): i = = = 22,34 A
tempo 360000

Questão 1322 – (OLIMPÍADA PORTUGUESA DE QUÍMICA) Tendo em conta os potenciais de redução indicados:
E° (Fe+2/Fe) = - 0,44 V
E° (Fe+3/Fe+2) = + 0,77 V
O potencial do par Fe+3/Fe é:
a) + 0,037 V
b) – 0,037 V
c) + 0,330 V
d) + 0,110 V

Resolução: Alternativa B.
Havendo uma diferença de elétrons, será necessário o uso da energia livre de Gibbs. Com isso, podemos converter
os potenciais E° a ΔG° pela seguinte expressão matemática: ∆G° = −n x F x E°
Através desta equação eletroquímica, o cálculo do potencial é calculado diretamente. Este procedimento indireto é de
suma importância, pois embora a constante de FARADAY seja cancelado, os números de mol de elétrons (n) em geral
não são cancelados.
Analisando a semirreação de redução do Fe+2 (Fe+2/Fe):

Fe+2(aq) + 2e- → Fe(s) E°= - 0,44 V ∆G° = −2 x 96500 x (−0,44)


Analisando a semirreação de redução do Fe+3 (Fe+3/ Fe+2):

Fe+3(aq) + 1e- → Fe+2 E°= + 0,77 V ∆G° = −1 x 96500 x (+0,77)

728
1600 FÍSICO-QUÍMICA APLICADA EXERCÍCIOS COMENTADOS - IME – ITA – OLIMPÍADA

Somando as semirreações 1 e 2, temos:

Fe+2(aq) + 2e- → Fe(s) E°= - 0,44 V ∆G° = −2 x 96500 x (−0,44)


Fe+3(aq) + 1e- → Fe+2 E°= + 0,77 V ∆G° = −1 x 96500 x (+0,77)
Fe+3(aq) + 3e- → Fe(s) ∆G° = −3 x 96500 x E°

0
∆GTOTAL = ∆G10 + ∆G20
−3 x 96500 x E° = −2 x 96500 x (−0,44) + [−1 x 96500 x (+0,77)]
−3 x 96500 x E° = −2 x 96500 x (−0,44) + [−1 x 96500 x (+0,77)]
−3 x E° = −2 x (−0,44) + [−1 x (+0,77)]
−3 x E° = + 0,88 − 0,77
−3 x E° = + 0,11
0,11
E° = − = −0,0367 V
3

Questão 1323 – (IME) Uma empresa de galvanoplastia produz peças especiais recobertas com zinco. Sabendo que
cada peça recebe 7 g de Zn, que é utilizada uma corrente elétrica de 0,7 A e que a massa molar do zinco é igual a 65
g.mol-1, qual o tempo necessário para o recobrimento dessa peça especial?
a) 4 h e 45 min
b) 6 h e 30 min
c) 8 h e 15 min
d) 10 h e 30 min
e) 12 h e 45 min

Resolução: Alternativa C.

Semirreação de redução do Zinco que ocorre no cátodo: Zn2+(aq) + 2e- → Zn(s).

Cálculo do tempo de operação da célula eletroquímica:


65 g de Zn --------------------------- (2 mol de elétrons x 96500 C.mol-1 de elétrons)
7 g ------------------------------------ (q = 0,7 x tempo)
tempo = 29506,19 s

Conversão de segundos para horas:


1h
Tempo = 29506,19 s x
3600 s
Tempo = 8,25 h

Tempo = 8,0h + 0,25 h

60 min
Tempo = 8,0h + [0,25 h x ] = 8,0h + 15 min
1h

729
1600 FÍSICO-QUÍMICA APLICADA EXERCÍCIOS COMENTADOS - IME – ITA – OLIMPÍADA

Questão 1324 – (IME) Realiza-se a eletrólise de uma solução aquosa diluída de ácido sulfúrico com eletrodos inertes
durante 10 minutos. Determine a corrente elétrica média aplicada, sabendo-se que foram produzidos no cátodo 300
mL de hidrogênio, coletados a uma pressão total de 0,54 atm sobre a água, à temperatura de 300 K. Considere os
seguintes dados:
• Pressão de vapor da água a 300K = 0,060 atm;
• Constante de Faraday = 1F = 96500 C.mol-1;
• Constante universal dos gases perfeitos = 0,08 atm. L.mol-1.K.
a) 2,20 A
b) 1,93 A
c) 1,08 A
d) 0,97 A
e) 0,48 A

Resolução: Alternativa B.

Analisando a eletrólise aquosa do ácido sulfúrico: H2SO4(aq) → 2 H+(aq) + SO4-2(aq)


Reação de ionização da água: H2O(aq) → 2 H+(aq) + OH-(aq)
O único que vai descarregar no cátodo, trata-se do hidrogênio: 2 H+(aq) + 2e- → H2(g)
Quem descarrega com maior facilidade no ânodo (pólo positivo), SO4-2(aq) ou OH-(aq)? Resposta: 2 OH-(aq) → ½ O2(g) +
2e- + H2O(l)
Somando as semirreações mencionadas acima, temos:
H2SO4(aq) → 2 H+(aq) + SO4-2(aq)
2 H2O(aq) → 2 H+(aq) + 2 OH-(aq)
2 H+(aq) + 2e- → H2(g)
2 OH-(aq) → ½ O2(g) + 2e- + H2O(l) +

H2SO4(aq) + 2 H2O(l) → 2 H+(aq) + SO4-2(aq) + H2(g) + ½ O2(g) + H2O(l)


2 H2O(l) → H2(g) + ½ O2(g) + H2O(l)
Através da eletrólise do ácido sulfúrico, observa-se que há sim a produção de gás hidrogênio, logo:
p x V (0,54 − 0,06) x 0,300 0,48 x 0,300
nH2 = = = = 0,06 mol
RxT 0,08 x 300 0,08 x 300
Cálculo da carga (Q):
1 mol de H2 ------------------ 2 mol de elétrons x 96500 C.(mol de elétrons)-1
0,06 mol de H2 -------------- Q
Q = 1158 C
Q 1158
Cálculo da corrente (i): i = tempo = 10 x 60 = 1,93 A

730
1600 FÍSICO-QUÍMICA APLICADA EXERCÍCIOS COMENTADOS - IME – ITA – OLIMPÍADA

Questão 1325 – (ITA) Considere os eletrodos representados pelas semi-equações químicas seguintes e seus
respectivos potenciais na escala do eletrodo de hidrogênio (E) e nas condições-padrão:
I. In+(aq) + e- (CM) → In(s) EI = – 0,14 V
II. In2+(aq) + e- (CM) → In+(aq) EII = – 0,40 V
III. In3+(aq) + 2e- (CM) → In+(aq) EIII = – 0,44 V
IV. In3+(aq) + e- (CM) → In2+(aq) EIV = – 0,49 V
Assinale a opção que contém o valor CORRETO do potencial-padrão do eletrodo representado pela semi-equação:
In3+(aq) + 3e- (CM) → In(s).
a) – 0,30 V
b) – 0,34 V
c) – 0,58 V
d) – 1,03 V
e) – 1,47 V

Resolução: Alternativa B.
A semirreação de redução em questão pode ser determinada a partir das semirreações de redução I e III. Logo:

Analisando a semirreação I: In+(aq) + e- (CM) → In(s) EI = – 0,14 V ∆G° = −1 x F x (−0,14)

Analisando a semirreação III: In+3(aq) + 2e- (CM) → In+(aq) EI = – 0,44 V ∆G° = −2 x F x (−0,44)
Somando as semirreações:
In+(aq) + e-(CM) → In(s) ∆G° = −1 x F x (−0,14)
In+3(aq) + 2e-(CM) → In+(aq) ∆G° = −2 x F x (−0,44) +
In+3(aq) + 3e- (CM) → In(s) ∆G° = −3 x F x E°

Cálculo do potencial-padrão: GT = GI + G III


−3 x F x E° = −1 x F x (−0,14) + [−2 x F x (−0,44)]
−3 x F x E° = −1 x F x (−0,14) + [−2 x F x (−0,44)]
−3 x E° = +0,14 + 0,88
−3 x E° = +1,02
− 1,02
E° = = −0,34 𝑉
3
Observação: O potencial-padrão (E°) deste eletrodo pode ser calculado considerando a sua relação com a variação
de energia livre de Gibbs (G€) através da seguinte expressão termodinâmica, G° = – nFE°, onde: n é o número de
mols de elétrons recebidos na redução; F a constante de Faraday e E° o potencial-padrão da semi-célula de redução.
Questão 1326 – (ITA) Para determinar o valor da constante de Faraday empregou-se uma célula eletrolítica
construída pela imersão de duas chapas de prata em uma solução aquosa de nitrato de prata. O conjunto é ligado a
uma fonte de corrente contínua em série com um amperímetro. Durante certo intervalo de tempo t verificou-se que
pelo circuito passou uma corrente elétrica constante de valor i. Neste período de tempo t foi depositado no cátodo
uma massa m de prata, cuja massa molar é representada por M. Admite-se que a única reação eletroquímica que
ocorre no cátodo é a redução dos cátions de prata a prata metálica. Denominando a constante de Avogadro de NA e
a área do cátodo imersa na solução de S, a constante de Faraday (F) calculada a partir deste experimento é igual a:
ixtxM
a) F= m
b) F = i x t x NA
ixtxm
c) F = MxS

731
1600 FÍSICO-QUÍMICA APLICADA EXERCÍCIOS COMENTADOS - IME – ITA – OLIMPÍADA

ixt
d) F = SxN
A
ixm
e) F= M

Resolução: Alternativa A.
A questão trata-se de uma célula eletrolítica, com duas placas de prata na presença de uma solução de nitrato de
prata. Semirreação que ocorre no cátodo: Ag+(aq) + 1 e- → Ag(s)
1 mol de Ag ---------- MMAg ---------- 1 mol de e- x (F/mol de elétrons)
mAg ---------- Q

Q x (< MM >Ag ) = F x mAg

F x mAg
Q=
< MM >Ag

Fxm
Sabendo que a carga (Q) é o produto da corrente com o tempo, Q = i x t, logo: Q = <MM>Ag
Ag

F x mAg
ixt =
< MM >Ag

Fxm
Isolando F: i x t = <MM>Ag
Ag

i x t x < MM >Ag
F=
mAg

Questão 1327 – (ITA) Considere a reação química representada pela equação abaixo e sua respectiva força
eletromotriz nas condições padrão: O2(g) + 4 H+(aq) + 4 Br-(aq) → 2 Br2(g) + 2 H2O(l), ΔE° = + 0,20 V. Agora, considere
que um recipiente contenha todas as espécies químicas dessa equação, de forma que todas as concentrações sejam
iguais as da condição padrão, exceto a de H+. Assinale a opção que indica a faixa de pH na qual a reação química
ocorrerá espontaneamente:
a) 2,8 < pH < 3,4
b) 3,8 < pH < 4,4
c) 4,8 < pH < 5,4
d) 5,8 < pH < 6,4
e) 6,8 < pH < 7,4

Resolução: Alternativa A.
0,0592
Utilizando a equação de Nernst, temos: E = E° − n
x logQ

2
0,0592 PBr 2
x a2H2 O
E = E° − x log [ ]
n PO2 x [H + ]4 x [Br − ]2

Levando em consideração que nas condições padrões, a pressão de um gás é igual 1 atm, a concentração da
quantidade de matéria igual a 1 mol.L-1 e a atividade de um líquido, por convenção é unitário (a = 1), a equação de
0,0592 (1)² x (1)²
Nernst fica da seguinte maneira: 0 = 0,20 − 4
x log [1 x [H+]4 x (1)² ]

0,0592 1
0 = 0,20 − x log [ + 4 ]
4 [H ]

732
1600 FÍSICO-QUÍMICA APLICADA EXERCÍCIOS COMENTADOS - IME – ITA – OLIMPÍADA

1
(−0,20) x 4 = − 0,0592 x log [ ]
[H + ]4

0,80 1
= log [ + 4 ]
0,0592 [H ]

0,80 1
= log [ + 4 ]
0,0592 [H ]

1
13,51 = log [ ]
[H + ]4

13,51 = 4 x pH

13,51
pH = = 3,38
4

Questão 1328 – (OLIMPÍADA BRASILEIRA DE QUÍMICA) Se a quantidade de elétrons, assim como, a quantidade
de cada uma das espécies químicas que intervém numa reação de uma pilha, são multiplicadas por dois, então, o
potencial da pilha:
a) Aumenta para o dobro
b) Diminui para a metade
c) Eleva-se ao quadrado
d) Fica reduzido a raiz quadrada
e) Não varia

Resolução: Alternativa E.
Multiplicando a semirreação por qualquer número, podendo ser por dois, três, etc..., o potencial da pilha não sofre
nenhuma variação. Podemos dar como exemplo o funcionamento de uma pilha que apresenta as seguintes
semirreações:
Primeira semirreação: Fe3+(aq) + e- → Fe2+(aq) E0 = + 0,77 V
Segunda semirreação: Sn4+(aq) + 2e- → Sn2+(aq) E0 = + 0,15 V
Multiplicando a primeira semirreação por dois:
2 Fe3+(aq) + 2e- → 2 Fe2+(aq) E0 = + 0,77 V ΔG1° = -2 x F x (+0,77)
Invertendo o sentido da segunda semirreação:
Sn2+(aq) → 2e- + Sn4+(aq) E0 = - 0,15 V ΔG2° = -2 x F x (-0,15)
Equação global para o processo eletroquímico: 2 Fe3+(aq) + Sn2+(aq) → 2 Fe2+(aq) + Sn4+(aq)

Cálculo do potencial (E°): ΔGT° = -2.F.E°

ΔGT° = ΔG1° + ΔG2°


-2 x F x E° = -2 x F x (+ 0,77) + -2 x F x (- 0,15)
E° = 0,77 – 0,15 = + 0,62 V

733
1600 FÍSICO-QUÍMICA APLICADA EXERCÍCIOS COMENTADOS - IME – ITA – OLIMPÍADA

Questão 1329 - (OLIMPÍADA ALAGOANA DE QUÍMICA - MODIFICADA) O trabalho produzido por uma pilha é
proporcional à diferença de potencial (ddp) nela desenvolvida quando se une uma meia-pilha onde a reação eletrolítica
de redução ocorre espontaneamente (cátodo) com outra meia pilha onde a reação eletrolítica de oxidação, ocorre
espontaneamente (ânodo). Como exemplo temos:
Ag(s) → Ag+ + e- E° = - 0,80 V
Cu(s) → Cu2+ + 2e- E° = - 0,34 V
A ddp da pilha é de:
a) 0,28 V
b) 0,33 V
c) 0,41 V
d) 0,46 V
e) 0,56 V

Resolução: Alternativa D.
Analisando a semirreação da Prata: 2 Ag+ + 2e- → 2 Ag(s) E° = + 0,80 V ∆G° = −2 x F x (+0,80)
Analisando a semirreação do Cobre: Cu(s) → Cu2+ + 2e- E° = - 0,34 V ∆G° = −2 x F x (−0,34)
Somando as semirreações, temos:
2 Ag+ + 2e- → 2 Ag(s) ∆G° = −2 x F x (+0,80)
Cu(s) → Cu2+ + 2e- ∆G° = −2 x F x (−0,34) +
2 Ag+ + Cu(s) → Cu2+ + 2 Ag(s) ∆G° = −2 x F x E°

Cálculo do potencial-padrão (E°): −2 x F x E° = −2 x F x (+0,80) + [−2 x F x (−0,34)]

E° = +0,80 − 0,34 = +0,46 V

Questão 1330 – (OLIMPÍADA BRASILEIRA DE QUÍMICA) Que produtos são formados durante a eletrólise de uma
solução concentrada de cloreto de sódio?
I. Cl2(g)
II. NaOH(aq)
III. H2(g)

a) Somente I
b) Somente II
c) Somente I e II
d) Somente I e III
e) I, II e III

Resolução: Alternativa E.
Aplicando a eletrólise aquosa do cloreto de sódio:
NaCl(s) → Na+(aq) + Cl-(aq)
H2O(l) → H+(aq) + OH-(aq)
Quem descarrega com mais facilidade no cátodo (pólo negativo), Na+ ou H+?
2 H+(aq) + 2e- → H2(g)
Quem descarrega com mais facilidade no ânodo (pólo positivo), Cl- ou OH-?

734
1600 FÍSICO-QUÍMICA APLICADA EXERCÍCIOS COMENTADOS - IME – ITA – OLIMPÍADA

2 Cl-(aq) → Cl2(g) + 2e-


Somando todas as semirreações, temos:
2 NaCl(s) → 2 Na+(aq) + 2 Cl-(aq)
2 H2O(l) → 2 H+(aq) + 2 OH-(aq)
2 H+(aq) + 2e- → H2(g)
2 Cl-(aq) → Cl2(g) + 2e- +

2 NaCl(s) + 2 H2O(l) → 2 Na+(aq) + 2 OH-(aq) + H2(g) + Cl2(g)


A eletrólise aquosa do cloreto de sódio produz hidróxido de sódio, gás hidrogênio e gás cloro.
Questão 1331 – Numa semicélula, se a pressão for aumentada de 1 atm para 100 atm mantendo a concentração de
íons hidrônio, qual o potencial que será medido?
a) – 0,059 V
b) 0,059
c) 5,09 V
d) 0,259 V

Resolução: Alternativa A.
Através da equação de redução do hidrogênio, 2 H+(aq) + 2 e- → H2(g), temos:
0,0592
Utilizando a equação de Nernst: E = E° − x log Q
n

Considerando que a pressão do e gás hidrogênio seja igual a 1 atm, a concentração de H+ igual a 1 mol.L-1 e o
0,0592 PH
potencial padrão de redução do hidrogênio é igual a zero, temos: E = E° − n
x log [H+2]²

0,0592 1
E = 0,00 − x log
2 (1)2
0,0592
E = 0,00 − x log 1 = 0,00 V
2
Para uma pressão igual a 100 atm, a concentração de H+ seja igual a 1 mol.L-1 e o potencial padrão de redução do
0,0592 PH
hidrogênio é igual a zero, temos: E = E° − n
x log [H+2]²

0,0592 100
E = 0,00 − x log
2 (1)2
0,0592 0,0592
E = 0,00 − x log 100 = − x 2 = − 0,0592 V
2 2
Questão 1332 – (GRILLO) Dentre as alternativas apresentadas logo abaixo, representado pelos elementos químicos
I, II e III, assinale a alternativa que apresenta os produtos que coincide tanto na eletrólise aquosa quanto na ígnea,
para o cloreto de magnésio.
I. Mg(s)
II. H2(g)
III. Cl2(g)
a) Somente I
b) Somente II
c) Somente III
d) Somente I e III

735
1600 FÍSICO-QUÍMICA APLICADA EXERCÍCIOS COMENTADOS - IME – ITA – OLIMPÍADA

e) I, II e III

Resolução: Alternativa C.
Aplicando a eletrólise aquosa do cloreto de magnésio:
MgCl2(s) → Mg+2(aq) + 2 Cl-(aq)
H2O(l) → H+(aq) + OH-(aq)
Quem descarrega com mais facilidade no cátodo (pólo negativo), Mg+2 ou H+?
2 H+(aq) + 2e- → H2(g)
Quem descarrega com mais facilidade no ânodo (pólo positivo), Cl- ou OH-?
2 Cl-(aq) → Cl2(g) + 2e-
Somando todas as semirreações, temos:
MgCl2(s) → 2 Mg+2(aq) + 2 Cl-(aq)
2 H2O(l) → 2 H+(aq) + 2 OH-(aq)
2 H+(aq) + 2e- → H2(g)
2 Cl-(aq) → Cl2(g) + 2e- +

MgCl2(s) + 2 H2O(l) → 2 Mg+2(aq) + 2 OH-(aq) + H2(g) + Cl2(g)


Observa-se que na eletrólise aquosa do cloreto de magnésio há a produção de hidróxido de magnésio, gás hidrogênio
e gás cloro.
Analisando a eletrólise ígnea do cloreto de magnésio:
MgCl2(s) → Mg+2(l) + 2 Cl-(l)
Mg+2(l) + 2e- → Mg(s)
2 Cl-(l) → Cl2(g) + 2e- +

MgCl2(s) → Mg(s) + Cl2(g)


Observa-se que na eletrólise ígnea do cloreto de magnésio há a produção de magnésio metálico e gás hidrogênio.
O produto formado tanto na eletrólise ígnea quanto na aquosa, trata-se do gás cloro.
Questão 1333 – (ITA) A 25°C, o potencial da pilha descrita abaixo é de 0,56 V. Sendo E°(Cu+2/Cu) = + 0,34 V,
assinale a opção que indica aproximadamente o valor do pH da solução.
+ +2
Pt (s) |H2(g,1 bar) , H(aq,x mol.L−1 ‖ Cu(aq,1 mol.L−1 ) |Cu(s)

a) 6,5
b) 5,7
c) 3.7
d) 2,0
e) 1,5

Resolução: Alternativa C.
Analisando a semirreação do cobre: Cu+2(aq) + 2e- → Cu(s)

736
1600 FÍSICO-QUÍMICA APLICADA EXERCÍCIOS COMENTADOS - IME – ITA – OLIMPÍADA

EI = + 0,34 V ∆𝐺° = −2 x F x (+0,34)


Analisando a semirreação do hidrogênio: H2(g) → 2 H+(aq) + 2e-

EI = + 0,00 V ∆G° = −2 x F x (+0,00)


Somando as semirreações, temos:
Cu+2(aq) + 2e- → Cu(s) ∆𝐺° = −2 𝑥 𝐹 𝑥 (+0,34)
H2(g) → 2 H+(aq) + 2e- ∆𝐺° = −2 𝑥 𝐹 𝑥 (+0,00) +
Cu+2(aq) + H2(g) → Cu(s) + 2 H+(aq) ∆𝐺° = −2 𝑥 𝐹 𝑥 𝐸°
Cálculo do potencial-padrão: −2 x F x E° = −2 x F x (0,34) + [ −2 x F x (0,00)]
E° = + 0,34 V

0,0592
Cálculo do pH, a partir da utilização da equação de Nersnt: E = E° − n
x log Q

0,0592 aCu x [H + ]²
0,56 = 0,34 − x log
2 [Cu+2 ] x PH2

0,0592 1 x [H + ]²
0,56 − 0,34 = − x log ( )
2 1x1
0,0592
0,22 = − x log[H + ]²
2
0,0592
0,22 = − x 2 x log[H + ]
2
+]
0,22 = 0,0592 x −log[H

pH

0,22 = 0,0592 x pH
0,22
pH = = 3,72
0,0592
Questão 1334 – (ITA) É de 0,76 V a força eletromotriz padrão, E°, de uma célula eletroquímica, conforme a reação:
Zn(s) + 2 H+(aq) → Zn+2(aq) + H2(g). Na concentração da espécie de Zn+2 igual a 1,0 mol.L-1 e pressão de H2 de 1,0 bar,
a 25°C, foi verificado que a força eletromotriz da célula eletroquímica é de 0,64 V. Nestas condições, assinale a
concentração de íons H+ em mol.L-1.
a) 1,0 x 10-12
b) 4,2 x 10-4
c) 1,0 x 10-4
d) 1,0 x 10-2
e) 2,0 x 10-2

Resolução: Alternativa D.
0,0592
Cálculo da concentração do íon hidrogeniônico, a partir da utilização da equação de Nernst: E = E° − n
x log Q

0,0592 [Zn+2 ] x PH2


0,64 = 0,76 − x log
2 [H + ]2 x aZn
Sabendo que a pressão de hidrogênio é igual a 1 bar, [Zn+2] = 1 mol.L-1 e a atividade do zinco (aZn) igual a 1, temos:

737
1600 FÍSICO-QUÍMICA APLICADA EXERCÍCIOS COMENTADOS - IME – ITA – OLIMPÍADA

0,0592 [Zn+2 ] x PH2


0,64 = 0,76 − x log
2 [H + ]2 x aZn
0,0592 1x1
0,64 − 0,76 = − x log + 2
2 [H ] x 1
0,0592 1
−0,12 = − x log + 2
2 [H ]
− 0,12
= − log[H + ]
−0,0592
log[H + ] = −2,03
[H + ] = 10−2,03 mol. L−1

Questão 1335 – (IME) Considere que a reação abaixo ocorra em uma pilha: 2 Fe+3 + Cu → Cu+2 + 2 Fe+2. Assinale
a alternativa que indica o valor correto do potencial padrão dessa pilha. Dados:

Fe+2 → Fe+3 + e- E° = - 0,77V


Cu+2 + 2e- → Cu E° = + 0,34V
a) + 1,20 V
b) – 0,43 V
c) + 1,88 V
d) – 1,20 V
e) + 0,43 V

Resolução: Alternativa E.

Invertendo a primeira semirreação e multiplicando por dois, temos: 2Fe+3 + 2e- → 2Fe+2 E° = + 0,77V
∆G° = - 2.F.(+0,77)

Invertendo a segunda semirreação, temos: Cu → Cu+2 + 2e- E° = - 0,34V ∆G° = -


2.F.(-0,34)

Somando as duas semirreações, temos:

2Fe+3 + 2e- → 2Fe+2 E° = + 0,77V ∆G° = - 2.F.(+0,77)

Cu → Cu+2 + 2e- E° = - 0,34V ∆G° = - 2.F.(-0,34) +

2Fe+3 + Cu → 2Fe+2 + Cu+2 ∆G°TOTAL = - 2.F.(+0,77) + [- 2.F.(-0,34)]

Cálculo da ddp padrão: - 2.F.E° = - 2.F.(+0,77) + - 2.F.(-0,34)

E° = + 0,77V – 0,34V = + 0,43 V

Questão 1336 – (OLIMPÍADA PERUANA DE QUÍMICA) Determine el potencial estándar a 25°C para la óxido-
reducción 2PH3(g) → 2P(s) + 3H2(g) Eo = ?, dadas las siguientes reducciones a la misma temperatura:
P(s) + 3H+(ac) + 3e- → PH3(g) Eo = - 0,063 V
2H (ac) + 2e → H2(g)
+ - E = 0,000 V
o

a) Eo = 0,0063 V
b) Eo = - 0,0063 V
c) Eo = - 0,0126 V

738
1600 FÍSICO-QUÍMICA APLICADA EXERCÍCIOS COMENTADOS - IME – ITA – OLIMPÍADA

d) Eo = 0,0126 V

Resolução: Alternativa A.

Invertendo e multiplicando por dois a primeira semirreação: 2 PH3(g) → 2 P(s) + 6 H+(ac) + 6e- ∆G° =
−6 x F x (+0,063)

Multiplicando a segunda semirreação por três: 6H+(ac) + 6e- → 3 H2(g) ∆G° =


−6 x F x (+0,000)

Somando as semirreações:

2 PH3(g) → 2 P(s) + 6 H+(ac) + 6e- ∆G° = −6 x F x (+0,063)


6H+(ac) + 6e- → 3 H2(g) ∆G° = −6 x F x (+0,000) +

2 PH3(g) + 6H+(ac) → 2 P(s) + 6 H+(ac) + 3 H2(g)

−6 x F x E° = −6 x F x (+0,063) + [−6 x F x (+0,000)]

E° = + 0,063 + 0,000 = + 0,063 V

Questão 1337 – (ITA - MODIFICADA) Por uma célula eletrolítica passou uma carga correspondente a 0,20 Faraday.
Num dos eletrodos ocorreu a reação seguinte: MnO4- + 8H+ + 5e - → Mn +2 + 4H2O. Calcule a quantidade de água
(mol) produzida neste eletrodo, em virtude desta reação de eletrodo.
Resolução: Através da relação estequiométrica:

5F − − − − − − − 4 mol de H2 O
0,20F − − − − − − nH2 O

0,20 𝑥 4
nH2 O = = 0,16 mol
5

Questão 1338 – (OLIMPÍADA ALAGOANA DE QUÍMICA - MODIFICADA) Considere uma pilha de níquel/zinco e as
semi-reações representadas a seguir, com seus respectivos potenciais de redução:
Ni+2 + 2e- → Ni Eo = - 0,25 V
Zn + 2e → Zn
+2 - Eo = - 0,76 V
A diferença de potencial da pilha é:
a) 0,21 V
b) 0,31 V
c) 0,41 V
d) 0,50 V
e) 0,51 V

Resolução: Alternativa E.
Analisando a semirreação do Níquel: Ni+2(aq) + 2e- → Ni(s) E° = - 0,25 V ∆G° = −2 x F x (−0,25) =
+ 0,50 𝑥 𝐹
Analisando a semirreação do Zinco: Zn+2 + 2e- → Zn Eo = - 0,76 V ∆G° =
−2 x F x (−0,76) = +1,52 𝑥 𝐹
Somando as semirreações, temos:
Ni+2(aq) + 2e- → Ni(s) ∆G° = −2 x F x (−0,25) = + 0,50 𝑥 𝐹
Zn → Zn+2 + 2e- ∆G° = −2 x F x (+0,76) = − 1,52 𝑥 𝐹 +

739
1600 FÍSICO-QUÍMICA APLICADA EXERCÍCIOS COMENTADOS - IME – ITA – OLIMPÍADA

Ni+2(aq) + Zn(s) → Ni(s) + Zn+2(aq) ∆G° = −2 x F x 𝐸 0

Cálculo do potencial-padrão: −2 x F x E 0 = + 0,50 x F − 1,52 x F


−2 x E 0 = + 0,50 − 1,52
−1,02
E0 = = + 0,51 V
−2

Questão 1339 – (ITA) Durante uma eletrólise, a única reação que ocorreu no catodo foi a deposição de certo metal.
Observou-se que a deposição de 8,81 gramas de metal correspondeu à passagem de 0,300mols de elétrons pelo
circuito. Qual das opções a seguir contém o metal que pode ter sido depositado? Dados: Massas atômicas: Ni = 58,71;
Zn = 65,37; Ag = 107,87; Sn = 118,69; Pb = 207,19.
a) Ni
b) Zn
c) Ag
d) Sn
e) Pb

Resolução: Alternativa A.
Dos metais apresentados no enunciado do problema, os elementos níquel (Ni), zinco (Zn), estanho (Sn) e chumbo
(Pb) apresentam número de oxidação igual a +2. Diante de uma escolha, vamos analisar e calcular a massa molar do
metal com número de oxidação +2.
+2
Semirreação de redução no câtodo: X (aq) + 2e− → X (s) , onde X representa o elemento metálico com número de
oxidação igual a +2.
< MM >X − − − − 2 mol de e−
8,81 g − − − − − 0,300 mol de e−
8,81 𝑥 2 17,62 𝑔
< MM >X = = = 58,73 (Níquel, Ni)
0,300 0,300 𝑚𝑜𝑙
Questão 1340 – (ITA) Uma fonte de corrente contínua fornece corrente elétrica a um sistema composto por duas
células eletrolíticas, ligadas em série através de um fio condutor. Cada célula é dotada de eletrodos inertes. Uma das
células contém somente uma solução aquosa 0,3 mol.L-1 de NiSO4 e a outra apenas uma solução aquosa 0,2 mol.L-1
de AuCl3. Se durante todo o período de eletrólise as únicas reações que ocorrem no catodos são as deposições dos
metais, qual das opções corresponde ao valor da relação: massa de níquel depositado / massa de ouro depositado?
a) 0,19
b) 0,45
c) 1,0
d) 2,2
e) 5,0

Resolução: Alternativa B.

Analisando a solução de sulfato de níquel II. Ni+2 −


(aq) + 2e → Ni(s)

Analisando a solução de cloreto de ouro: Au+3 −


(aq) + 3e → Au(s)

Multiplicando a primeira semirreação por três e a segunda semirreação por dois:

3Ni+2 −
(aq) + 6e → 3Ni(s)

2Au+3 −
(aq) + 6e → 2Au(s)

Com isso, observa-se que são produzidos 2 mol de ouro e 3 mol de níquel.

740
1600 FÍSICO-QUÍMICA APLICADA EXERCÍCIOS COMENTADOS - IME – ITA – OLIMPÍADA

𝑚 3 𝑥 58,7 𝑔 176,1 𝑔
Cálculo: 𝑚 𝑁𝑖 = 2 𝑥 197 𝑔
= 394 𝑔
= 0,447 (≅ 0,45)
𝐴𝑢

Questão 1341 – (Graduação CEFETES – Coletânea de provas) Temos representado no esquema abaixo, três
cubas eletrolíticas em série. Calcule as massas dos metais Ag, Cu e Au depositadas, respectivamente, considerando
o sistema submetido a uma corrente de 3 A, durante 30 minutos e 15 segundos.

a) 0,1 g; 0,03 g e 0,062 g


b) 6,1 g; 3,6 g e 11,12 g
c) 6,1 g; 1,8 g e 3,7 g
d) 0,1 g; 0,06 g e 0,062 g
e) 6,1 g; 3,6 g e 5,6 g

Resolução: Alternativa C.
Para a resolução do problema, é importante ressaltar que a corrente elétrica é constante por todo o processo,
conforme pode ser observado na figura fornecida pelo problema.

Q
Cálculo da quantidade de carga (Q): i =
tempo

60 𝑠
Q = i x tempo = 3,0 x (30min x + 15 𝑠) = 3,0 x (1800 + 15) = 3,0 x 1815 = 5445 C
1 𝑚𝑖𝑛
Analisando a solução de nitrato de prata (AgNO3): AgNO3(aq) → Ag + −
(aq) + NO3(aq)

Semirreação que ocorren no cátodo: Ag + −


(aq) + 1e → Ag (s)

96500 𝐶
108 g − − − − − 1 mol de e− 𝑥
1 mol de elétrons
mAg − − − − − 5445 C
108 𝑥 5445 17,62 588060
mAg = = = = 𝟔, 𝟎𝟗 𝐠
96500 96500 96500
Analisando a solução de sulfato de cobre II (CuSO4): CuSO4(aq) → Cu+2 −2
(aq) + SO4(aq)

Semirreação que ocorren no cátodo: Cu+2 −


(aq) + 2e → Cu(s)

96500 𝐶
63,55 g − − − − − 2 mol de e− 𝑥
1 mol de elétrons

741
1600 FÍSICO-QUÍMICA APLICADA EXERCÍCIOS COMENTADOS - IME – ITA – OLIMPÍADA

mCu − − − − − − 5445 C

63,55 𝑥 5445 346029,75


mCu = = = 𝟏, 𝟕𝟗 𝐠
2 𝑥 96500 193000
Analisando a solução de cloreto de ouro III (AuCl3): AuCl3(aq) → Au+3 −
(aq) + 3Cl(aq)

Semirreação que ocorren no cátodo: Au+3 −


(aq) + 3e → Au(s)

96500 𝐶
197 g − − − − − − 3 mol de e− 𝑥
1 mol de elétrons
mAu − − − − − − 5445 C

197 𝑥 5445 1072665


mAu = = = 3,70 𝑔
3 𝑥 96500 289500
Questão 1342 - (ITA) Uma cuba eletrolítica com eletrodos de cobre e contendo solução aquosa de Cu(NO3)2 é ligada
em série com outra provida de eletrodos de prata e contendo solução aquosa de AgNO3. Esse conjunto de cubas em
série é ligado a uma fonte durante certo intervalo de tempo. Nesse intervalo de tempo, um dos eletrodos de cobre
teve um incremento de massa de 0,64 gramas. O incremento de massa em um dos eletrodos da outra célula deve ter
sido:

a) 0,32g
b) 0,54g
c) 0,64g
d) 1,08g
e) 2,16 g

Resolução: Alternativa E.

Analisando a solução de nitrato de cobre II (AgNO3): Cu(𝑁𝑂3 )2(𝑎𝑞) → Cu+2 −


(aq) + 2 NO3(aq)

Semirreação que ocorren no cátodo: Cu+2 −


(aq) + 2e → Cu(s)

96500 𝐶
63,55 g − − − − − − 2 mol de e− 𝑥
1 mol de elétrons
0,64 𝑔 − − − − − − Q

0,64 𝑥 2 𝑥 96500 123520


𝑄= = = 1943,67 𝐶
63,55 63,55
Analisando a solução de nitrato de prata (AgNO3): AgNO3(aq) → Ag + −
(aq) + NO3(aq)

742
1600 FÍSICO-QUÍMICA APLICADA EXERCÍCIOS COMENTADOS - IME – ITA – OLIMPÍADA

Semirreação que ocorren no cátodo: Ag + −


(aq) + 1e → Ag (s)

96500 𝐶
108 g − − − − − 1 mol de e− 𝑥
1 mol de elétrons
mAg − − − − − 1943,67 C

108 𝑥 1943,67 209916,36


mAg = = = 2,17 𝑔
96500 96500

Questão 1343 - (OLIMPÍADA PARAENSE DE QUÍMICA) Quanto será a fem da célula de Daniell a 298K para
concentrações de CuSO4 e ZnSO4 iguais a 0,50 mol.L-1 e 0,10 mol.L-1, respectivamente. Qual deveria ser a fem se
fossem utilizadas atividades em vez de concentrações? (Os valores de γ ± para o CuSO4 e o ZnSO4 em suas
respectivas concentrações são 0,068 e 0,15, respectivamente:

f) 1,125 V; 1,115 V
g) 0,115 V; 0,215 V
h) 1,515V; 1,575V
i) 0,115V; 1,125V
j) 0,15V; 0,10V

Resolução: Alternativa A.

Semirreação do cobre: Cu+2(aq) + 2e- → Cu(s) E° = +0,34 V


Semirreação do zinco: Zn+2 + 2e- → Zn Eo = - 0,76 V
Cálculo da força eletromotriz padrão (E°). Invertendo a segunda semirreação, temos:

Zn → Zn+2 + 2e- Eo = + 0,76 V ∆G° = −2 x F x (+0,76)


Cu+2(aq) + 2e- → Cu(s) E° = + 0,34 V ∆G° = −2 x F x (+0,34) +

Zn + Cu+2(aq) → Zn+2 + Cu(s) ∆G° = −2 x F x E°

−2 x F x E° = −2 x F x (+0,76) + [−2 x F x (+0,34)]

E° = 0,76 + 0,34 = 1,10 V

Cálculo da força eletromotriz levando em consideração que as concentração sejam iguais a [Zn+2
(aq) ] =
0,10 mol. L−1 e [Cu+2 −1
(aq) ] = 0,50 mol. L .

Aplicando a equação de Nernst:


RxT
E = E0 − x lnQ
nxF

RxT a𝐶𝑢 x [Zn+2 ] 8,314 x 298 1 x 0,10


E = E0 − x ln { +2 } = 1,10 − x ln { }
nxF a𝑍𝑛 x [Cu ] 2 x 96500 1 x 0,50

2477,572 0,10
E = 1,10 − x ln { }
193000 0,50

E = 1,10 − 1,28 𝑥 10−2 x ln(0,20) = 1,10 − 1,28 𝑥 10−2 x (−1,61)

743
1600 FÍSICO-QUÍMICA APLICADA EXERCÍCIOS COMENTADOS - IME – ITA – OLIMPÍADA

E = 1,10 + 0,0206 = 1,121 V

Analisando em função das atividades químicas:

aZn+2 = γ± x [Zn+2 ] = 0,15 x 0,10 = 0,015

aCu+2 = γ± x [Cu+2 ] = 0,068 x 0,50 = 0,034


RxT
Aplicando a equação de Nernst: E = E 0 − n x F x lnQ

RxT a𝐶𝑢 x 𝑎𝑍𝑛+2 8,314 x 298 1 x 0,015


E = E0 − x ln { } = 1,10 − x ln { }
nxF a𝑍𝑛 x 𝑎𝐶𝑢+2 2 x 96500 1 x 0,034

2477,572 0,015
E = 1,10 − x ln { }
193000 0,034

E = 1,10 − 1,28 𝑥 10−2 x ln(0,441) = 1,10 − 1,28 𝑥 10−2 x (−0,819) = 1,10 + 0,0105 = +1,105 V

Questão 1344 – Calcule os seguintes itens apresentados a seguir:

a) Considere que uma corrente com uma amperagem desconhecida atue sobre uma solução aquosa de sulfato
de cobre II, sendo eletrolisada a cobre metálico depositado. Se uma corrente foi utilizada durante cinco horas e que
404 miligramas de cobre foram obtidos, determine o valor da corrente elétrica.
b) Determine a carga necessária para que 2,0 gramas de cobre sejam depositados a partir de uma solução de
sulfato de cobre, com a presença de uma corrente de 5,0 A.

Resolução:

Item a)

1 mol de Cu --------- 63,55 g de cobre ----------- 2 mol de elétrons x (96500 C/mol de elétrons)
404 x 10-3 g de cobre -------------- Q
Q = 1226,94 C

No enunciado do problema, o tempo corresponde a cinco horas, convertendo para segundos, temos: Tempo =
3600 segundos
5 horas x ( 1h
) = 18000 s
q 1226,94 C
Cálculo da corrente elétrica: i = = = 6,82 x 10−2 A
t 18000 s

Item b)

1 mol de Cu --------- 63,55 g de cobre ----------- 2 mol x (96500 C/mol)


2,0 g de cobre ------------ Q
Q = 6073,96 C

744
1600 FÍSICO-QUÍMICA APLICADA EXERCÍCIOS COMENTADOS - IME – ITA – OLIMPÍADA

Questão 1345 - Considere uma pilha galvânica operando a uma T = 250C e pressão igual a 1 atm. A descrição da
pilha está fundamentada a partir da seguinte notação apresentada a seguir:

PtSn2+(aq) (0,1 mol.L-1), Sn4+(aq) (0,05 mol.L-1)  Fe3+(aq) (0,01 mol.L-1), Fe2+(aq) (0,2 mol.L-1)Pt

a) Escreva a reação global de oxirredução e calcule a ddp padrão da pilha.


b) Calcule a variação da energia livre de Gibbs padrão da pilha para a reação global.
c) Calcule o ΔG da reação no momento em que as concentrações das espécies químicas são iguais às
indicadas na notação da pilha. A pilha funciona nessas condições?
d) Calcule a constante de equilíbrio, K, quando a pilha parar de gerar corrente elétrica. Informações para a
resolução do problema:
Fe3+(aq) + e- → Fe2+(aq) E0 = + 0,77 V
Sn (aq) + 2e → Sn (aq) E = + 0,15 V
4+ - 2+ 0

Resolução: Dados do problema:


Fe3+(aq) + e- → Fe2+(aq) E0 = + 0,77 V

Sn4+(aq) + 2e- → Sn2+(aq) E0 = + 0,15 V

Invertendo a semirreação de estanho: Sn2+(aq) → Sn4+(aq) + 2e- E0 = - 0,15 V

Multiplicando por dois a semirreação do Ferro: 2 Fe3+(aq) + 2e- → 2Fe2+(aq) E0 = + 0,77 V

Somando as duas semirreações, temos:

Sn2+(aq) → Sn4+(aq) + 2e- E0 = - 0,15 V


2 Fe3+(aq) + 2e- → 2 Fe2+(aq) E0 = + 0,77 V +

Sn2+(aq) + 2 Fe3+(aq) → Sn4+(aq) + 2 Fe2+(aq) E0 = - 0,15 V + 0,77 V = + 0,62 V

Item a) A reação global que caracteriza o funcionamento desta pilha galvânica é a seguinte: Sn2+(aq) + 2 Fe3+(aq) →
Sn4+(aq) + 2 Fe2+(aq)

Item b) Cálculo da variação da energia Livre de Gibbs padrão (∆G°): ∆G° = −n x F x E°

96500 C
∆G° = −2 mol de elétrons x x (+0,62 V) = −119660 J
1 mol de elétron
2
[Fe+2 ] x [Sn+4 ]
Item c) Cálculo da variação da energia Livre de Gibbs (∆G): ∆G = ∆G° + R x T x ln {[Fe+3 ]2 x [Sn+2 ]}

[0,20]2 x [0,05]
∆G = −119600 + 8,314 x (25 + 273) x ln { }
[0,01]2 x [0,1]

[0,20]2 x [0,05]
∆G = −119600 + 8,314 x 298 x ln { } = −112192,06 J
[0,01]2 x [0,1]

Sim, a pilha galvânica funciona de forma espontânea, pois a variação da energia livre de Gibbs é negativa (∆G < 0).

Item d) Quando a pilha parar de gerar corrente elétrica, a partir da variação da energia livre de Gibbs padrão, a
constante de equilíbrio será a seguinte: ∆G° = −R x T x lnK eq

−112192,06 = −8,314 x (25 + 273) x lnK eq

745
1600 FÍSICO-QUÍMICA APLICADA EXERCÍCIOS COMENTADOS - IME – ITA – OLIMPÍADA

lnK eq = 45,28

K eq = e45,28

Questão 1346 – Considere a reação global de oxirredução, a 25°C:

O2(g) + 4H+(aq) + 4 Br-(aq) → 2 H2O(l) + 2 Br2(l)


a) Em condições-padrão a reação acima é espontânea? Justifique com cálculos.
b) Qual é o novo valor de E no momento em que o pH da solução é 3,6? Nesse pH, o processo será espontâneo?
Justifique sua resposta.
c) Calcule a constante de equilíbrio (Keq) da reação.

Dados das semirreações:

Br2(l) + 2e- → 2 Br-(aq) Eo = + 1,07 V

O2(g) + 4H+(aq) + 4e- → 2H2O(l) Eo = + 1,23 V


Resolução:

Dados do problema: O2(g) + 4H+(aq) + 4 Br-(aq) → 2 H2O(l) + 2 Br2(l)

Semirreações de redução:

Br2(l) + 2e- → 2 Br-(aq) Eo = + 1,07 V

O2(g) + 4H+(aq) + 4e- → 2H2O(l) Eo = + 1,23 V


Item a) Invertendo a primeira semirreação e multiplicando por dois:
4Br-(aq) → 2 Br2(l) + 4e- ∆G0 = −4 x F x (−1,07)

O2(g) + 4H+(aq) + 4e- → 2H2O(l) ∆G0 = −4 x F x (+1,23) +

4Br-(aq) + O2(g) + 4H+(aq) → 2 Br2(l) + 2H2O(l)


−4 x F x E° = −4 x F x (−1,07) + [−4 x F x (+1,23)]
−4 x F x E° = −4 x F x (−1,07) + [−4 x F x (+1,23)]
E° = −1,07 + 1,23 = + 0,16 V
Observando a ddp padrão do processo eletroquímico, E° = + 0,16V (E° > 0, positiva), o processo fica sendo
espontâneo.
Cálculo da variação da energia livre de Gibbs padrão (∆G°): ∆G° = −n x F x E°
C
∆G° = −4 mol x (96500 ) x (+0,16 V) = −61760 J
mol

Utem b) Cálculo da concentração de íons H+, a partir do pH = 3,6: pH = - log[H+]


3,6 = - log[H+]
[H+] = 10(-3,6) mol.L-1
0,0592
Utilizando a equação de Nernst, para a determinação do potencial E: E = E° − ( n
) x logQ

746
1600 FÍSICO-QUÍMICA APLICADA EXERCÍCIOS COMENTADOS - IME – ITA – OLIMPÍADA

0,0592 a2H2 O x a2Br2


E = E° − ( ) x log { 2 }
n PO2 [Br − ]+4 x [H + ]+4

0,0592 [1]2 x [1]2


E = +0,16 − ( ) x log { }
4 1 x [10(−3,6) ]4 x [1]4

0,0592 1
E = +0,16 − ( ) x log { (−3,6) 4 }
4 [10 ]

0,0592
E = +0,16 − ( ) x log10(+14,4)
4
0,0592
E = +0,16 − ( ) x log10(+14,4) = + 0,16 − 0,213 = −0,053 V
4

Observa-se que para um pH = 3,60, o processo passa a ser não espontâneo, uma vez que E < 0.

Item c) Cálculo da constante de equilíbrio, (Keq): ∆G° = −R x T x lnK eq

−61760 = −8,314 x (25 + 273) x lnK eq

−61760 = −8,314 x 298 x lnK eq

lnK eq = +24,93

K eq = e(+24,93) = 6,70 x 10+10

Questão 1347 – A pilha voltaica apresentada a seguir, conforme recomendação da IUPAC,

Cd(s)  Cd2+(aq, X mol.L-1) Ni2+(aq, 1,00 mol.L-1) Ni(s), apresenta força eletromotriz igual a 0,36 V a 25°C.
a) Escreva a reação global da pilha voltaica.
b) Calcule o valor de E0 da pilha.
c) Determine a concentração X de íons Cd2+;
d) Determine o valor da energia livre de Gibbs, ∆G.

Informações para a resolução do problema:

Ni2+(aq) + 2e- → Ni(s) E° = - 0,28 V

Cd2+(aq) + 2e- → Cd(s) E° = - 0,40 V


Resolução: Dados para a resolução do problema:

Ni2+(aq) + 2e- → Ni(s) E° = - 0,28 V

Cd2+(aq) + 2e- → Cd(s) E° = - 0,40 V

Cd(s)  Cd2+(aq, X mol.L-1) Ni2+(aq, 1,00 mol.L-1) Ni(s)


Item a) Invertendo a semirreação do cádmio, temos:

Ni2+(aq) + 2e- → Ni(s) ∆G° = −2 x F x (−0,28)

Cd(s) → Cd2+(aq) + 2e- ∆G° = −2 x F x (+0,40) +

747
1600 FÍSICO-QUÍMICA APLICADA EXERCÍCIOS COMENTADOS - IME – ITA – OLIMPÍADA

Ni2+(aq) + Cd(s) → Ni(s) + Cd2+(aq) −2 x F x E° = −2 x F x (−0,28) + [−2 x F x (+0,40)]


−2 x F x E° = −2 x F x (−0,28) + [−2 x F x (+0,40)]

−2 x F x E° = −2 x F x (−0,28) + [−2 x F x (+0,40)]

E° = −0,28 + 0,40 = + 0,12 V

Item b) O potencial padrão da referida pilha voltaica é igual a E° = + 0,12V.

Item c) Cálculo da concentração molar de íons Cádmio (X), a partir da utilização da equação de Nernst:

RxT RxT [Cd+2 ] x aNi


E = E° − x lnQ = E° − x ln { }
nxF nxF aCd x [Ni+2 ]
8,314 x (25 + 273) X. 1
+ 0,36 = + 0,12 − . ln { }
2 x 96500 1.1
8,314 x 298 X
+ 0,36 = + 0,12 − x ln { }
2 x 96500 1
0,24
ln 𝑋 =
−1,28 x 10−2
X = e(−18,75) = 7,19 x 10−9 mol. L−1 .
Item d) Cálculo da variação da energia livre de Gibbs (∆G): ∆G = ∆G° + R x T x lnQ

[Sn+4 ]². a2H2 O


∆G = (−n. F. E°) + R. T. ln { +2 + 4
}
⏟[Sn ]² . PO2 . [H ]
Q
7,19 x 10−9 x 1
= −2 x 96500 x (+0,12) + 8,314 x 298 x ln { } = − 69614,47 J. mol−1
1x1

Questão 1348 –

a) Determine a constante de equilíbrio para a seguinte equação química global: 2 Cu+(aq) → Cu(s) + Cu+2(aq). A 25°C,
considerando as seguintes semirreações: E°Cu+/Cu = + 0,52 V; E°Cu+2/ Cu+ = + 0,16 V.
b) Determine o volume de gás hidrogênio produzido, a 37°C e pressão igual a 720 mmHg, através de uma corrente
de 3,86 A, que atravessa uma solução aquosa de ácido sulfúrico, durante uma hora.

Resolução:

Item a) Desenvolvendo as semirreações, temos:

Cu+(aq) + e- → Cu(s) ∆G° = −1 x F x (+0,52)


Cu+2(aq) + e- → Cu+(s) ∆G° = −1x F x (+0,16)
Para o cálculo do potencial global fornecido pelo problema, 2 Cu+(aq) → Cu(s) + Cu+2(aq), temos:

Invertendo a segunda semirreação, temos:

Cu+(aq) + e- → Cu(s) ∆G° = −1 x F x (+0,52)

Cu+(s) → Cu+2(aq) + e- ∆G° = −1x F x (−0,16) +

748
1600 FÍSICO-QUÍMICA APLICADA EXERCÍCIOS COMENTADOS - IME – ITA – OLIMPÍADA

2 Cu+(aq) → Cu(s) + Cu+2(aq) ∆G° = −1 x F x E°

−1 x F x E° = −1 x F x (+0,52) + [−1 x F x (−0,16)]

−1 x F x E° = −1 x F x (+0,52) + [−1 x F x (−0,16)]

E° = +0,52 − 0,16 = +0,36 V

Cálculo da constante de equilíbrio (Keq): ∆G° = −R x Tx lnK eq = −n x F x E°

−R x T x lnK eq = −n x F x E°

n x F x E° 1 x 96500 x (+0,36) 96500 x 0,36


lnK eq = = =
RxT 8,314 x (25 + 273) 8,314 x 298

lnK eq = 14,02

K eq = e(+14,02) = 1,23 x 106

Item b) Sabendo que a semirreação de hidrogênio é dada por: 2 H+(aq) + 2e- → H2(g)

Cálculo do número de mol de gás hidrogênio produzido:

1 mol de H2(g) ---------- 2 mole- x (96500 C/ mole-)


nH2(g) --------------------- (3,86 x 3600) C
nH2(g) = 0,072 mol

Cálculo do volume de gás produzido (VH2), considerando com comportamento ideal:

nRT 0,072 x 0,08206 x (37 + 273) 0,072 x 0,08206 x 310 x 760


VH2 = = = = 1,93 L
p 720 720
(760)

Questão 1349 – Os potenciais-padrão de redução de duas semirreações a uma temperatura igual a 25°C são
fornecidas a seguir:

O2(g) + 4H+(aq) + 4e- → 2H2O(l) Eo = + 1,229 V


2H+(aq) + 2e- → H2(g) Eo = + 0,00 V
a) Escreva as semirreações que ocorrem no cátodo e no ânodo, além da reação global da célula galvânica.
b) Determine o valor da constante de equilíbrio da reação na célula galvânica, a 25°C.
c) Cálculo da variação da energia livre de Gibbs na temperatura mencionada e diga se o processo é espontâneo
ou não.
d) Calcule a variação do potencial da célula galvânica (∆E) a 90°C, sabendo que o ar atmosférico apresenta
20% de O2 em volume, e que a pressão de H2 é controlada para reagir completamente com o O2.

Resolução: Item a) Determinação da equação global da pilha, invertendo e multiplicando por dois a semirreação de
hidrogênio:

O2(g) + 4H+(aq) + 4e- → 2H2O(l) ∆G° = −4 x F x (+1,229)


2 H2(g) → 4H+(aq) + 4e- ∆G° = −4 x F x (+0,00) +

2 H2(g) + O2(g) ⇌ 2H2O(l) ∆G° = −4 x F x E°

749
1600 FÍSICO-QUÍMICA APLICADA EXERCÍCIOS COMENTADOS - IME – ITA – OLIMPÍADA

−4 x F x E° = −4 x F x (+1,229) + [−4 x F x (+0,00)]


−4 x F x E° = −4 x F x (+1,229) + [−4 x F x (+0,00)] = + 1,229 V
Equação global da pilha: 2 H2(g) + O2(g) → 2H2O(l)
Reação no ânodo: 2 H2(g) → 4H+(aq) + 4e-
Reação no cátodo: O2(g) + 4H+(aq) + 4e- → 2H2O(l)
Item b) Cálculo da constante de equilíbrio (Keq): −4 mol x 96500 x (+1,229) = −8,314 x (25 + 273) x lnK eq

− 474394 = −2477,57 x lnK eq

−474394
lnK eq = = +191,47
−2477,57

K eq = e(+191,47) = 1,43 x 1083

Item a) Cálculo da variação da energia livre de Gibbs padrão: ∆G° = −n x F x E°

∆G° = −4 x 96500 x (+1,229) = −474,39 kJ

O processo é espontâneo, pois a variação da energia livre de Gibbs é negativa.


RxT
Item b) Utilizando a equação de Nersnt, temos: E = E° − n x F x lnQ

RxT a2H O R. T 1
E = E° − x ln { 2 2 } = E° − x ln { 2 }
nxF PH2 x PO2 n. F PH2 x PO2

Equação química: 2 H2(g) + O2(g) ⇌ 2H2O(l)

PO2 = 0,20 x (1 atm) = 0,20 atm

PH2 = 2 x 0,20 x 1 atm = 0,40 atm

8,314 x (90 + 273) 1


E = +1,229 − x ln { }
4 x 96500 (0,40)2 x (0,20)
8,314 x 393 1
E = +1,229 − x ln { } = +1,229 − 8,46 x 10−3 x ln(31,25) = +1,20 V
386000 0,032

750
1600 FÍSICO-QUÍMICA APLICADA EXERCÍCIOS COMENTADOS - IME – ITA – OLIMPÍADA

Questão 1350 – Calcule a constante de dissociação da água, a 37C, a partir das seguintes semirreações fornecidas
abaixo:

O2(g) + 2H2O(l) + 4e- → 4OH-(aq) E = + 0,418 V a T = 37°C

O2(g) + 4H+(aq) + 4 e- → 2H2O(l) E = + 1,27 V a T = 37oC

Resolução: Cálculo da ddp do processo eletroquímico apresentado, invertendo a segunda reação química, temos:

O2(g) + 2H2O(l) + 4e- → 4OH-(aq) ∆G° = − 4 x F x (+ 0,418)

2H2O(l) → O2(g) + 4H+(aq) + 4e- ∆G° = − 4 x F x (−1,27) +

4H2O(l) → 4H+(aq) + 4OH-(aq)

−4 x F x E° = −4 x F x (+ 0,418) + [−4 x F x (−1,27)]


E° = −0,852 V
Dividindo a reação química por quatro, temos:

H2O(l) ⎯→ H+(aq) + OH-(aq) E = - 0,852 V

Sabendo que a energia livre de Gibbs padrão eletroquímica (∆G°) é dada por: ∆G° = - n.F.E° (Equação 1) e ∆G° = -
R.T.lnK (Equação 2)

Igualando as equações (1) e (2) termodinâmicas, temos: −n. F. E° = −R x T x lnK W

(−1) x 96500 x (−0,852) = −8,314 x (37 + 273) x lnK W

− 82218
LnK W = = − 31,90
2577,34

K W = e(−31,90) = 1,40 x 10−14

751
1600 FÍSICO-QUÍMICA APLICADA EXERCÍCIOS COMENTADOS - IME – ITA – OLIMPÍADA

CAPÍTULO X

TESTE PARA PRATICAR

PROFESSOR ALEXANDRE
VARGAS GRILLO

752
1600 FÍSICO-QUÍMICA APLICADA EXERCÍCIOS COMENTADOS - IME – ITA – OLIMPÍADA

TESTE 01 - (OLIMPÍADA BRASILEIRA DE QUÍMICA) A ebulioscopia é um fenômeno intrínseco das soluções e pode
ser utilizada na determinação da massa molar de substâncias desconhecidas. Massas de substâncias moleculares
são dissolvidas em solventes como benzeno, hexano ou tetracloreto de carbono, e em função do efeito coligativo a
massa molar é determinada. Em um ensaio de laboratório, um técnico dissolveu 2,00 g de uma substância
desconhecida em 63 mL de CC4. Considerando os dados abaixo e a temperatura de ebulição da solução de 77C,
assinale a massa molar aproximada da substância dissolvida. Dados: densidade da solução a 20°C: d = 1,59 kg.L–1;
temperatura de ebulição: Te = 349,5 K e constante ebulioscópica: Ke = 5,00 K.kg.mol–1.
a) 200 g.mol–1
b) 50 g.mol–1
c) 120 g.mol–1
d) 100 g.mol–1
e) 180 g.mol–1

TESTE 02 - (GRILLO) O sulfato de bário é um pó branco, inodoro, insolúvel com uma estrutura compactada possuindo
alta densidade absoluta. Considere que foram utilizados 125 g deste material sendo dissolvildo em um quilo de água,
formando uma solução de volume total igual a 350 mililitros. A partir desta informação, calcule os diferentes tipos de
concentrações apresentados abaixo:
a) concentração comum, em g.L-1;
b) a concentração da quantidade de matéria;
c) a densidade da solução;
d) a molalidade;
e) a normalidade.

TESTE 03 - (OLIMPÍADA BRASILEIRA DE QUÍMICA / OLIMPÍADA NORTE-NORDESTE DE UÍMICA) Durante anos,


doenças graves, como o escorbuto, tinham causa desconhecida. Os marinheiros, principalmente, eram atacados por
ela, quando ficavam muito tempo embarcados. A descoberta que frutas cítricas poderiam diminuir e até mesmo evitar
o escorbuto, proporcionaram melhores condições de vida. A fórmula da vitamina C é C 6H8O6 e a dose mínima
recomendada é de 60 mg diárias, quantidade presente em uma laranja pequena. Determine o número de quantidade
de matéria necessário para a dose mínima necessária:
a) 3,4 x 10-4
b) 1,1 x 10-4
c) 2,9 x 10-4
d) 6,0 x 10-4
e) 1,8 x 10-4

TESTE 04 – An certificially produced radioactive isotope decomposes according to the first order law with a half-life
period of 25 min. In what time will 75% of the sample be decomposed?

TESTE 05 – (OLIMPÍADA BRASILEIRA DE QUÍMICA) Durante algum tempo os químicos acreditavam que bastava
conhecer a entalpia da reação para conhecer sua espontaneidade. Porém, há vários exemplos de transformações
químicas que contradizem essa ideia. Para corrigir essa distorção, introduziu-se um outro parâmetro para avaliar a
espontaneidade de uma reação, a entropia. O enunciado de Kelvin é: “É impossível remover energia cinética de um
sistema a uma certa temperatura e converter essa energia integralmente em trabalho mecânico sem que haja uma
modificação no sistema ou em suas vizinhanças”. XXIII Olimpíada Brasileira de Química OBQ 2017 5 Sobre a entropia
é correto afirmar que:
a) A segunda Lei da Termodinâmica afirma que a entropia do universo diminui numa transformação espontânea.
b) Se a entropia de um sistema diminui, a transformação será necessariamente não espontânea.
c) A entropia padrão de uma substância pura é zero nas condições padrão.
d) Uma reação endotérmica e com diminuição de entropia do sistema é espontânea.
e) Numa transformação espontânea, a entropia do universo irá aumentar.

TESTE 05 – Crossley, Kienle and Benbrook (1940) obtained the following values for the rate constant of the reaction.
C6H5N2Cl ⇄ C6H5Cl + N2.

T(°C) 5 25 35 50

753
1600 FÍSICO-QUÍMICA APLICADA EXERCÍCIOS COMENTADOS - IME – ITA – OLIMPÍADA

k (sec-1) 1,5 x 10-6 4,1 x 10-5 2,0 x 10-4 1,4 x 10-3

Determine the parameters A and Eat.

TESTE 06 – Find the total pressure exerted by 4,0 g of C2H6 and 30 g of CO2 contained in a 10,0 liter vessel at 50°C.

TESTE 07 – (OLIMPÍADA CEARENSE DO ENSINO SUPERIOR DE QUÍMICA) Considerando que o zinco metálico
reage com soluções de ácidos, como por exemplo, o ácido clorídrico em água, usada para preparar o hidrogênio em
laboratório, de acordo com a seguinte reação: Zn(s) + 2HCl(aq) → ZnCl2(aq) + H2(g). Ao dissolver 12,05 g de zinco, qual
o volume em mililitros de ácido clorídrico 2,0 mol L-1 necessários para converter completamente o zinco a cloreto de
zinco? Assinale a alternativa correta.
a) 184 mL
b) 368 mL
c) 200 mL
d) 18,4 mL
e) 120 mL

TESTE 08 – (OLIMPÍADA CEARENSE DO ENSINO SUPERIOR DE QUÍMICA) O composto butilítio, LiC4H9 é muito
reativo, sendo usado na obtenção de novas substâncias. Pode-se determinar a quantidade de butilítio, numa mistura
pela adição de ácido clorídrico, ocorrendo a seguinte reação: LiC4H9(s) + HCl(aq) → LiCl(aq) + C4H10(g). Se existem 5,453
g de butilítio dissolvidos em benzeno, ao adicionar excesso de ácido clorídrico (aq), ocorre desprendimento de 0,725
g de butano. Assinale a alternativa que contém a porcentagem do butilítio na amostra original.
a) 64,05%
b) 58,12%
c) 14,56%
d) 12,40%
e) 17,56%

TESTE 09 – (OLIMPÍADA CEARENSE DO ENSINO SUPERIOR DE QUÍMICA) O composto XCl2(NH3)2 pode ser
formado pela reação de XCl4 com NH3. Suponha que 3,571 g de XCl4 reagem com excesso de NH3 produzindo Cl2 e
3,180 g de XCl2(NH3)2. Qual é o elemento X?

a) Au
b) Pd
c) Pt
d) Ag
e) Os

TESTE 10 – (OLIMPÍADA CEARENSE DO ENSINO SUPERIOR DE QUÍMICA) O isótopo 34 15𝑃 é utilizado para
localizar tumores no cérebro e em estudos de formação de ossos e dentes. Uma bancada de laboratório foi
contaminada com 100 mg desse isótopo, que possui meia-vida de 14,3 dias. O tempo mínimo, expresso em dias, para
que a radioatividade caia a 0,1% do seu valor original é igual a:
a) 86
b) 114
c) 124
d) 157
e) 143

TESTE 11– (OLIMPÍADA CEARENSE DO ENSINO SUPERIOR DE QUÍMICA) O metal (M) é obtido a partir do sal
MCl2 fundido. Esse sal foi submetido à eletrólise por 4,65 h a uma corrente de 0,84 A, produzindo 4,00 g do metal M.
O metal M é:
a) Fe
b) Cr
c) Co
d) Zn

754
1600 FÍSICO-QUÍMICA APLICADA EXERCÍCIOS COMENTADOS - IME – ITA – OLIMPÍADA

e) Mn

TESTE 12 – (OLIMPÍADA BRASILEIRA DE QUÍMICA) Um acidente em um laboratório provocou a intoxicação de


um grupo de pessoas por inalação de um gás. Um analista coletou uma amostra desse gás e a introduziu em um
recipiente inelástico de 1 dm3 , à temperatura de 27°C. A amostra de gás contida no recipiente pesou 1,14 g e a
pressão medida no recipiente foi de 1 atm. Assim, pode-se afirmar que este gás é:
a) CO
b) C2H2
c) H2S
d) NO
e) NO2

TESTE 13 – (OLIMPÍADA BRASILEIRA DE QUÍMICA JÚNIOR) A queima de um pedaço de fita de magnésio ocorre
segundo a equação da reação química mostrada abaixo. Nesse processo, ocorre liberação de energia na forma de
calor e de uma intensa luminosidade. 2 Mg + O2 → 2 MgO. Utilizando-se essa reação, qual é a massa de magnésio
necessária para se produzir 2,0 g de óxido magnésio?

TESTE 14 – (GRILLO) Sabendo que a densidade do ácido bórico é igual a 1,23 g.cm-3 e sua concentração em massa
corresponde a 20%, calcule a concentração mássica e também a concentração da quantidade de matéria.

TESTE 15 – (ITA) Estima-se que a exposição a 16 mg m-3 de vapor de mercúrio por um período de 10 min seja letal
para um ser humano. Um termômetro de mercúrio foi quebrado e todo o seu conteúdo foi espalhado em uma sala
fechada de 10m de largura, 10 m de profundidade e 3 m de altura, mantida a 25°C. Calcule a concentração de vapor
de mercúrio na sala após o estabelecimento do equilíbrio 𝐻𝑔(𝑙) = 𝐻𝑔(𝑔) , sabendo que a pressão de vapor do
mercúrio a 25°C é 3 x 10–6 atm, e verifique se a concentração de vapor do mercúrio na sala será letal para um ser
humano que permaneça seu interior por 10 min.

TESTE 16 – (OLÍMPIADA PARAENSE DE QUÍMICA ) 64 g de um composto A dissolvido em 546 g de benzeno


(C6H6), dão uma solução cuja pressão de vapor é igual a 70 mm de Hg, a 20ºC. A pressão de vapor do benzeno, a 20
ºC, é igual a 75 mm de Hg. Calcule a massa molar do composto A é:
c) 42 g/mol
d) 64 g/mol
e) 96 g/mol
f) 128 g/mol
g) 192 g/mol

TESTE 17 – (IFRJ – TURMA QIM241) A uma determinada temperatura alta, em torno de 250-300°C, a reação do
ciclopropano em propeno apresenta velocidade específica igual a 2,19 x 10-4 (min-1; L.mol-1.min-1). Considere que a
quantidade de matéria inicial do hidrocarboneto cíclico seja igual a 1,50 mol.L-1, a mesma variação de temperatura
apresentada. Determine a concentração da quantidade de matéria final e o tempo de meia-vida após 20 horas,
considerando:
a) que o processo seja de primeira ordem;
b) que o processo seja de segunda ordem.

TESTE 18 – (GRILLO) Deduzir a equação da velocidade da decomposição térmica do composto gasoso A3(g), através
da seguinte equação química balanceada: A3(g) → 3/2 B2(g), sugerindo o seguinte mecanismo proposto a seguir:
𝑘𝑎
A3(g) → B2(g) + C
𝑘𝑏
B2(g) + C → A3(g)
𝑘𝑐
C + A3(g) → 2 B2(g)
Considere que a condição do estado estacionário seja a espécie intermediária gasosa C.

TESTE 19 – (GRILLO) A solução de etanol apresenta densidade igual a 2,53 g.mL-1 e concentração da quantidade
de matéria igual a 8,0 mol.L-1. Calcule a molalidade e também a concentração mássica (g.L-1).

755
1600 FÍSICO-QUÍMICA APLICADA EXERCÍCIOS COMENTADOS - IME – ITA – OLIMPÍADA

TESTE 20 – (IIT-JEE) 0,50 mol de cloreto de bário é misturado com 0,20 mol de fosfato de sódio. A partir desta
informação, calcule o número de mol de fosfato de bário.

TESTE 21 – (OLIMPÍADA BRASILEIRA DE QUÍMICA) O iodoetano é um líquido incolor usado como precursor em
reações orgânicas de alquilação na indústria farmacêutica e na produção representada de defensivos agrícolas. A
decomposição do iodoetano é representada pela equação abaixo: C2H5I(g) → C2H4(g) + HI(g).

T(K) 660 680 720 760


k 7,2 x 10-4 2,2 x 10-3 1,70 x 10-2 0,110

a) Determine a energia de ativação de Arrhenius para a reação citada e o valor do fator A.

b) Determine o valor da constante de velocidade da reação à temperatura de 400°C.

c) Explique como um catalisador influencia na variação da entalpia e na constante de equilíbrio de uma reação
química.

d) Indique a geometria de cada carbono tanto para os reagentes quanto para os produtos.

e) Faça uma previsão comparativa das polaridades do iodoetano, do eteno e do iodeto de hidrogênio.

Teste 22 – (OLIMPÍADA BRASILEIRA DE QUÍMICA) A formação de um sólido a partir de líquidos e/ou de gases é
uma das evidências de que ocorreu uma reação química. Para demostrar tal evidência, um recipiente de 2,5 L
contendo amônia gasosa a 0,78 atm e 18,5°C foi conectado em outro recipiente de 1,4 L com cloreto de hidrogênio
gasoso a 0,93 atm e 18,5°C, respectivamente. Sabe-se que a combinação desses gases leva a formação de cloreto
de amônio sólido, logo qual a quantidade aproximada de massa formada desse composto, o gás que sobrou nos
recipientes conectados e a sua pressão?

TESTE 23 – (ITA) Contribuíram de forma direta para o desenvolvimento do conceito de pressão atmosférica:
a) Friedrich August Kekulé e John Dalton
b) MichaeI Faraday e Fritz Haber
c) Galileu Galilei e Evangelista Torriceili
d) Jõns Jacob Berzelius e Eduard Buchner
e) Robert Bunsen e Henry Louis Le Chatelier

TESTE 25 – (IME) Atualmente está havendo uma mobilização mundial para minimizar o efeito estufa, na atmosfera
terrestre, causado pelo aumento da concentração de CO2 e de outros gases. Um das tentativas é o aumento de
catalisadores heterogêneos e homogêneos que possam ser utilizados no controle da poluição ambiental. A eficácia
dos catalisadores está sendo avaliada através de reações simples, tal como a oxidação do etano a 770K, a etileno,
em presença de V2O5 depositado em SiO2. Durante a reação, forma obtidos os produtos CH3CHO, CO e CO2. A partir
dos resultados mostrados na tabela abaixo, responda:
% de produtos
Referência Catalisador CH3CHO CH2CH2 CO CO2
I 100% SiO2 0 0 0 0
II 0,3%V2 /O5SiO2 4 16 3 77
III 1,4%V2O5/SiO2 1 10 2 87
IV 5,6% V2O5/SiO2 9 80 5 6
V 9,8% V2O5/SiO2 15 60 3 22
VI 100% V2O5 0 28 32 40
a) Em relação ao efeito estufa, qual o catalisador que poderá ser mais propício para o controle da poluição
ambiental? Justifique.
b) Por que o V2O5 apresenta propriedades catalíticas?

756
1600 FÍSICO-QUÍMICA APLICADA EXERCÍCIOS COMENTADOS - IME – ITA – OLIMPÍADA

TESTE 26 – (ITA) O número de oxidação do halogênio nas espécies químicas HF, HBrO3, HIO3, ClO3 e ClO4- é,
respectivamente:
a) -1, -4, -6, -6, -7
b) -1, +5, +5, +6, +7
c) +1, +2, +3, +3, +4
d) +1, -3, -5, -6, -7
e) -1, +3, +6, +6, +7

TESTE 27 – (ITA) Assinale a opção relativa aos números de oxidação corretos do átomo de cloro nos compostos
KClO2, Ca(ClO)2, Mg(ClO3)2 e Ba(ClO4)2, respectivamente.
a) –1, –1, –1 e –1
b) +3, +1, +2 e +3
c) +3, +2, +4 e +6
d) +3, +1, +5 e +6
e) +3, +1, +5 e +7

TESTE 28 – (ITA) O kitassato também é usado com vantagem na:


a) filtração com sucção
b) preparação de soluções me geral
c) separação de dois líquidos imiscíveis
d) determinação de densidade de líquidos
e) centrifugação de emulsões

TESTE 29 – (ITA) “Cal viva” é essecialmente:


a) Ca
b) CaO
c) Ca(OH)2
d) CaCO3
e) Nenhuma das afirmativas acima

TESTE 30 – (ITA) A dissolução de um sólido iônico em uma certa quantidade de água faz com que, em relação à
água pura e nas mesmas condições de temperatura e pressão, a solução apresente:
I - Pressão osmótica menor.
II - Pressão de vapor menor.
III - Temperatura de início de fusão menor.
IV - Temperatura de início de ebulição menor.
V - Densidade maior.
Das afirmações acima estão certas:
a) Apenas I, II e V
b) Apenas II, III e IV
c) Apenas II, III e V
d) Apenas I, II, III e V
e) Apenas I, II, IV e V

TESTE 31 – (ITA) Gás natural, um recurso energético cada vez mais importante no mundo, tem sido apontado como
um combustível “limpo” porque sua combustão produziria menos poluentes atmosféricos do que outros combustíveis
fósseis, por exemplo, os derivados de petróleo. Dentre as opções a seguir, contendo afirmações relacionadas com a
combustão do gás natural, assinale a única que contém a afirmação correta. A queima do gás natural:
a) não produz dióxido de carbono;
b) não produz monóxido de carbono, mesmo em condições que favorecem combustão incompleta;
c) produz apenas água, se as condições de combustão forem adequadamente ajustadas;
d) praticamente não produz dióxido de enxofre, se o gás for purificado previamente;
e) praticamente produz nitrogênio, se as condições de combustão forem rigorosamente ajustadas.

TESTE 32 – (ITA) Metano, também conhecido como gás dos pântanos, pode ser:

757
1600 FÍSICO-QUÍMICA APLICADA EXERCÍCIOS COMENTADOS - IME – ITA – OLIMPÍADA

I. Obtido por craqueamento do petróleo.


II. Obtido por hidrogenação do carvão mineral.
III. Formado por decomposição biológica em fundos de lagos.
IV. Extraído de reservas naturais, à semelhança do que acontece com o petróleo.
V. Formado na fermentação dos detritos domésticos, estocados em lixões e aterros sanitários.
Das afirmações acima estão corretas:
a) Apenas I e II
b) Apenas III e IV
c) Apenas I, II e IV
d) Apenas I, III, IV e V
e) Todas

TESTE 33 – (ITA) Joseph Black (1728-1799), médico, químico e físico escocês, conceituou o calor específico. Esta
conceituação teve importantes aplicações práticas, dentre elas a máquina a vapor, desenvolvida pelo engenheiro
escocês James Watt (1736-1819). Que componente do motor a vapor desenvolvido por Watt revolucionou seu uso e
aplicação?
a) Boiler ou fervedor
b) Bomba de recalque
c) Caldeira
d) Condensador
e) Turbina a vapor

TESTE 34 - (ITA) Amostras de massas iguais de duas substâncias, I e II, foram submetidas independentemente a um
processo de aquecimento em atmosfera inerte e a pressão constante. O gráfico abaixo mostra a variação da
temperatura em função do calor trocado entre cada uma das amostras e a vizinhança. Dados: Hf e Hv representam
as variações de entalpia de fusão e de vaporização, respectivamente, e Cp é o calor específico.
Temperaura

II

Calor trocado
Assinale a opção errada em relação à comparação das grandezas termodinâmicas.
a) Hf(I) < Hf(II)
b) Hv(I) < Hv(II)
c) cp,I(s) < Cp,II(s)
d) Cp,II(g) < Cp,I(g)
e) Cp,II(l) < cp,I(l)

TESTE 35 – A lei de Boyle-Mariotte (PV = constnate) não é obedecida no caso da mistura gasosa em equilíbrio
químico: N2(g) + 3H2(g) ⇌ 2 NH3(g) .

a) NH3 não é uma substância simples


b) o equilíbrio químico independe da pressão
c) a massa em gramas não permanece constante
d) o número de mol varia com a pressão
e) a lei somente se aplica para valores baixos de temperatura

758
1600 FÍSICO-QUÍMICA APLICADA EXERCÍCIOS COMENTADOS - IME – ITA – OLIMPÍADA

TESTE 36 – Uma massa de ar atmosférica com massa igual a 2,0 kg passa por um ciclo ilustrado logo a seguir. A
partir do ciclo apresentado, determine o trabalho líquido neste determinado ciclo.

TESTE 37 – (IME) Uma forma de sintetizar óxido nítrico em meio aquoso é reagir nitrito de sódio com sulfato ferroso
e ácido sulfúrico, produzindo, além do óxido nítrico, sulfato férrico e bissulfato de sódio. Partindo de 75,0g de nitrito
de sódio, 150,0 g de ácido sulfúrico e 152,0 g de sulfato ferroso e tendo a reação 90% de rendimento, determine a
massa de óxido nítrico obtida.

TESTE 39 – (GRILLO) A temperatura de aproximadamente igual a 25℃, a massa de nitrogênio gasoso é igual a 1,80
mg. Esta quantidade de gás nitrogênio é dissolvida na presença de 200 gramas de água caso a pressão do gás
nitrogênio seja igual a 740 mmHg. A partir destas informações apresentadas, determine o valor da constante de Henry.

TESTE 40 – (U. S. NATIONAL CHEMISTRY OLYMPIAD)


A - How many oxygen atoms are in 1,00 g of the mineral troegerite, (UO2)3(AsO4)2•12 H2O (M = 1304,0 g/mol)?
(A) 6,47 × 1021
(B) 8,31 × 1021
(C) 1,20 × 1022
(D) 1,39 × 1022

B – Phosphoric acid can be manufactured according to the following reaction: Ca3(PO4)2 + 3 SiO2 + 5 C + 5 O2 + 3
H2O → 3 CaSiO3 + 5 CO2 + 2 H3PO4. If equal masses of calcium phosphate (M = 310,0) and silica (M = 60,0) are
reacted with excess carbon, oxygen, and water to produce 1,00 x 103 kg phosphoric acid (M = 98,0), what mass of
calcium phosphate was used, assuming 100% yield?
(A) 610 kg
(B) 800 kg
(C) 920 kg
(D) 1580 kg

TESTE 41 – (ENADE) As moedas de R$ 0,05 (cinco centavos) são feitas de aço revestido de cobre e, com o passar
do tempo, é possível observar que elas são oxidadas a uma substância de coloração esverdeada. Esse é mais um
caso típico de oxidação atmosférica em ambiente úmido.
O2(g) + 4H+(aq) + 4e- → 2 H2O(l) E° = + 1,23 – (0,059).pH
Cu+2(aq) + 2e- → Cu(s) E° = + 0,34 V
Considerando as semirreações de redução representadas acima, a oxidação atmosférica das moedas em meio neutro
(pH = 7,0) é:
a) Espontânea, pois E° = + 0,89 V
b) Espontânea, pois E° = - 0,89 V
c) Espontânea, pois E° = + 0,48 V
d) Não espontânea, pois E° = - 0,89 V
e) Não espontânea, pois E° = + 0,48 V

TESTE 42 – (GRILLO) Calcule a fração molar do dióxido de carbono na presença de gordura, sabendo que a
constante da lei de Henry é igual a 8,0 x 104 torr e que a pressão parcial do referido gás é igual a 200 kPa.

TESTE 43 – (OLIMPÍADA PARAENSE DE QUÍMICA) Em geral, reação química não ocorre toda vez que acontece
uma colisão entre espécies potencialmente reativas. A reação ocorre quando as espécies reativas possuem um
mínimo de energia no momento da colisão. É uma barreira que as espécies que colidem devem suplantar para produzir
os produtos. Esse mínimo de energia denomina-se energia de:

759
1600 FÍSICO-QUÍMICA APLICADA EXERCÍCIOS COMENTADOS - IME – ITA – OLIMPÍADA

a) reação
b) ativação
c) dissociação
d) ionização
e) combustão

TESTE 44 – (OLIMPÍADA BRASILEIRA DE QUÍMICA) Um profissional da área química deseja preparar uma solução
de ácido bórico (H3BO3) 0,5 mol/L para ser utilizada como fungicida. Para preparar tal solução ele dispõe de 2,5 g de
ácido. O volume, em mL, de solução com a concentração desejada que pode ser preparado utilizando toda a massa
disponível é, aproximadamente:
a) 20
b) 41
c) 81
d) 161
e) 200

TESTE 45 – (IME) A reação entre os gases A e B para obtenção dos gases C e D é homogênea e pode ser estudada
cineticamente segundo a variação da pressão da mistura gasosa durante a reação. Determine a expressão da
velocidade da reação a partir da análise dos dados obtidos em experiências conduzidas a 800°C.
CONCENTRAÇÃO MOLAR
VARIAÇÃO DA PRESSÃO
EXPERIÊNCIA A B
em Hg/min
I 6 x 10-3 1 x 10-3 20
II 6 x 10-3 2 x 10-3 40
III 6 x 10-3 3 x 10-3 60
IV 1 x 10 -3 6 x 10 -3 3
V 2 x 10-3 6 x 10-3 12
VI 3 x 10-3 6 x 10-3 27

TESTE 46 – (OLIMPÍADA BRASILEIRA DE QUÍMICA) Uma solução sarturada de nitrato de potássio constituída,
além do sal, por 100 g de água, está a temperatura de 70°C. essa solução é resfriada a 40°C, ocorrendo precipitação
de parte do sal dissolvido. Com base nesses dadso e no gráfico apresentado abaixo:
Gráfico da solubilidade do nitrato de potássio em função da temperatura.

Pode-se afirmar que a massa de sal que precipitou foi de aproximadamente:


a) 20 g
b) 40 g
c) 60 g
d) 80 g
e) 100 g

760
1600 FÍSICO-QUÍMICA APLICADA EXERCÍCIOS COMENTADOS - IME – ITA – OLIMPÍADA

TESTE 47 –

A - (U. S. NATIONAL CHEMISTRY OLYMPIAD) A particular reaction rate increases by a factor of five when the
temperature is increased from 5°C to 27°C. What is the activation energy of the reaction?
a) 6,10 kJ.mol–1
b) 18,9 kJ.mol–1
c) 50,7 kJ.mol–1
d) 157 kJ.mol–1

B - (U. S. NATIONAL CHEMISTRY OLYMPIAD) The radioisotope N-13, which has a half-life of 10 minutes, is used
to image organs in the body. If an injected sample has an activity of 40 microcuries (40 µCi), what is its activity after
25 minutes in the body?
a) 0,75 µCi
b) 3,5 µCi
c) 7,1 µCi
d) 12 µCi

TESTE 48 – (IME) Analise as afirmativas abaixo e indique se as mesmas são falsas ou verdadeiras, justificando cada
caso.
a) Sólidos iônicos são bons condutores de eletricidade.
b) Compostos apolares são solúveis em água.
c) Caso não sofresse hibridização, o boro formaria a molécula BF.
d) A estrutura geométrica da molécula de hexafluoreto de enxofre é tetraédrica.

TESTE 49 – (IME) Faça o balanceamento da seguinte equação, cuja reação química ocorre em meio ácido:
Cr2 O7 + Fe+2 → Cr +3 + Fe+3 .

TESTE 50 – (IME) São dadas as equações químicas, não ajustadas, a seguir:


I) KClO3 + H2SO4 → HClO4 + ClO2 + K2SO4 + H2O
II) KMnO4 + HCl → KCl + MnCl2 + H2O + Cl2
Para cada uma dessas equações, determine:
a) os seus coeficientes, considerando os menores números inteiros possíveis;
b) o agente redutor;
c) o agente oxidante.

TESTE 51 – (ITA) Uma amostra de uma certa substância foi aquecida em um recipiente aberto e em contato com o
ar. A curva abaixo representa, em termos percentuais, a fração de massa remanescente no recipiente em função da
temperatura.

Das substâncias abaixo, qual poderia apresentar tal comportamento?


a) Uréia.
b) Sulfeto férrico.
c) Nitrato de cálcio.
d) Nitrato de alumínio.
e) Carbonato de sódio.

761
1600 FÍSICO-QUÍMICA APLICADA EXERCÍCIOS COMENTADOS - IME – ITA – OLIMPÍADA

TESTE 52 – (ITA) Assinale a opção correta para a propriedade físico-química cujo valor diminui com o aumento de
forças intermoleculares.

a) Tensão superficial
b) Viscosidade
c) Temperatura de ebulição
d) Temperatura de solidificação
e) Pressão de vapor

TESTE 53 – (ITA) Descreva como se poderia determinar, experimentalmente, a pressão de vapor do etanol na
temperatura de 30C. Sua descrição deve incluir um esquema da aparelhagem que poderia ser utilizada.

TESTE 54 – (ITA) O ferro é obtido nas usinas siderúrgicas:


a) por eletrólise da hematita fundida
b) deixando hematita fundida reagir com carbono fundido
c) por ustulação de pirita
d) aquecendo hematita numa corrente de CO2 e O2
e) por nenhum dos processos anteriores.

TESTE 55 - (ITA) Durante a utilização de um extintor de incêndio de dióxido de carbono, verifica-se formação de um
aerossol esbranquiçado e também que a temperatura do gás ejetado é consideravelmente menor do que a
temperatura ambiente. Considerando que o dióxido de carbono seja puro, assinale a opção que indica a(s)
substância(s) que torna(m) o aerossol visível a olho nu.
a) Água no estado líquido
b) Dióxido de carbono no estado líquido
c) Dióxido de carbono no estado gasoso
d) Dióxido de carbono no estado gasoso e água no estado líquido
e) Dióxido de carbono no estado gasoso e água no estado gasoso

TESTE 56 – (GRILLO) Considere a reação entre o carbeto de cálcio com a água, formando acetileno e hidróxido de
cálcio, conforme a seguinte equação química não balanceada: CaC2(s) + H2O(l) → C2H2(g) + Ca(OH)2(s). A partir de 5,00
gramas de carbeto de cálcio que apresenta 95% de pureza, calcule a massa de hidróxido de cálcio formado.

TESTE 57 – (ITA) Um bom catalisador para a reação N2 (g) + 3H2 (g) ⇄ 2NH3 (g) que no sentido 1 é exotérmica,
a) deverá ser um bom inibidor para a reação de decomposição térmica do NH3 (g).
b) formará maior quantidade de NH3 (g), no equilíbrio, somente se a temperatura for aumentada.
c) deverá ser capaz de converter integralmente em NH3 misturas estequiométricas dos reagentes, qualquer que
seja o valor da constante de equilíbrio.
d) formará maior quantidade de NH3, no equilíbrio, somente se a pressão for reduzida.
e) Nenhuma das respostas anteriores.

TESTE 58 – (ITA) Quais dos elementos abaixo foram isolados pela primeira vez em 1807 por H. Davy, usando um
fenômeno descoberto poucos anos antes por A. Volta?
a) Na e K
b) Cl2 e Br2
c) Pb e Sn
d) N2 e O2
e) Al e Sb

TESTE 59 – (ITA) Nas condições ambientes, qual das substâncias abaixo é um gás incolor e inodoro quando puro e
que, se muito comprimido, pode explodir?
a) SH2
b) NH3
c) C2H4

762
1600 FÍSICO-QUÍMICA APLICADA EXERCÍCIOS COMENTADOS - IME – ITA – OLIMPÍADA

d) C2H2
e) LiH

TESTE 60 – (ITA) Na temperatura ambiente, hidróxido de potássio reage com o cloreto de amônio sólido, com a
liberação de um gás. Assinale a alternativa correta para o gás liberado nesta reação.
a) Cl2
b) H2
c) HCl
d) NH3
e) O2

TESTE 61 – (ITA) Assinale a opção que nomeia o cientista responsável pela descoberta do oxigênio.
a) Dalton
b) Mendeleev
c) Gay-Lussac
d) Lavoisier
e) Proust

TESTE 62 – (ITA) Assinale a afirmação falsa. Na comparação entre Na e Na+ se constata que são diferentes:
a) suas propriedades químicas;
b) o número de elétrons que possuem;
c) os seus raiois atômico e iônico, respectivamente;
d) o número de prótons que possuem;
e) seu comportamento químico frente a água.

TESTE 63 – (ITA) Das substâncias abaixo, qual contém o fósforo mais facilmente assimilável pelos vegetais e
animais?
a) Trifluorfosfato de cálcio
b) Fluorfosfato de cálcio
c) Fosfatos de metais pesados
d) Fosfatos ácidos de cálcio
e) Fosfogênio

TESTE 64 – (ITA) “Ácido é uma substância capaz de receber 1 par de elétrons”. A definição acima corresponde à
proposta de:
a) Arrhenius
b) Bronsted
c) Lavoisier
d) Lewis
e) Ostwald

TESTE 65 – (ITA) Num recipiente mantido a pressão e temperatura ambiente, foram introduzidos 1,00 mol de etanol,
x mol de ácido acético, um pouco de um catalisador adequado e um solvente inerte para que o volume final da mistura
homogênea líquida fosse 5,0 litros. Nestas condições se estabelece o equilíbrio correspondente à equação química:
C2H5OH (SOLU) + CH3COOH (SOLU) ⇌ CH3COOC2H5 (SOLU) + H2O (SOLV). A constante deste equilíbrio é 4,0 na
temperatura ambiente. Uma vez atingido o equilíbrio, verifica-se que o sistema contém 0,50 mol de acetato de etila.
Destas informações podemos concluir que a quantidade x inicialmente posta de ácido acético é:
a) 0,25
b) 0,38
c) 0,50
d) 0,63
e) 0,75

TESTE 66 – (ITA) Descreva um método de preparação do ácido nítrico economicamente viável e utilizado pelas
indústrias químicas modernas para a produção em grande escala. Utilize equações balanceadas para representar as
reações químicas que ocorrem com o emprego do método proposto.

763
1600 FÍSICO-QUÍMICA APLICADA EXERCÍCIOS COMENTADOS - IME – ITA – OLIMPÍADA

TESTE 67 – (ITA) Uma solução constituída por dois componentes A e B apresentando comportamento ideal conforme
Lei de raoult, está em equilíbrio com seu vapor. Utilizando a notação: XA e XB para as respectivas frações em mol das
substâncias A e B na solução líquida; pA e pB para as respectivas pressões de vapor de A e B no vapor em equilíbrio
com a solução líquida, e p0A e p0B para as respectivas pressões de vapor A e puro e B puro numa mesma temperatura.
Assinale a opção que apresenta a relação correta para a pressão de vapor de A (pA) em equilíbrio com a solução
líquida.
a) pA = p0A.(1 – XA)
b) pA = p0B.(1 – XB)
c) pA = p0B.(1 – XA)
d) pA = p0A.(1 – XA)
e) pA = p0B.(1 – XB)

TESTE 68 – (ITA) A figura mostra a variação da massa específica de uma substância pura com a temperatura à
pressão de 1 bar. Então, é correto afirmar que Tx pode representar a temperatura de
a) ebulição da água
b) ebulição do benzeno
c) fusão da água
d) fusão do benzeno
e) fusão do dióxido de carbono

TESTE 69 – (ITA) Um recipiente de paredes adiabáticas e de volume constante contém duas amostras de água pura
separadas por uma parede também adiabática e de volume desprezível. Uma das amostras consiste em 54 g de água
a 25°C e, a outra, em 126 g a 75°C. Considere que a parede que separa as amostras é retirada e que as amostras
de água se misturam até atingir o equilíbrio. Sobre esse processo são feitas as seguintes afirmações:
II. A temperatura de mistura no equilíbrio é de 323 K.
III. A variação de entalpia no processo é nula.
IV. A variação de energia interna no processo é nula.
V. A variação de entropia no processo é nula.
Assinale a opção que apresenta(s) a(s) afirmação(ões) correta(s) sobre a mistura das amostras de água.
a) Apenas I
b) Apenas I e II
c) Apenas II e III
d) Apenas III e IV
e) Apenas IV

764
1600 FÍSICO-QUÍMICA APLICADA EXERCÍCIOS COMENTADOS - IME – ITA – OLIMPÍADA

TESTE 70 – (ITA) Em uma amostra de água do mar dissolve-se um pouco de sacarose. Em relação à consequência
deste acréscimo de sacarose, são feitas as seguintes afirmações:
I. A pressão de vapor da água diminui.
II. A pressão osmótica da solução aumenta.
III. A condutividade elétrica da solução permanece praticamente a mesma.
IV. A temperatura precisará descer mais para que possa começar a solidificação.
V. O grau de dissociação dos sais presentes na água do mar permanecerá praticamente o mesmo.
Das afirmações, estão CORRETAS:
a) Apenas I, II e III.
b) Apenas II, III e IV.
c) Apenas III, IV e V.
d) Apenas II, III, IV e V.
e) Todas.

TESTE 71 – (OLIMPÍADA DE QUÍMICA DO RIO DE JANEIRO) O álcool presente no "bafo" é convertido em ácido
acético, conforme mostra a equação abaixo:

__ C2H5OH + __ K2Cr2O7 + __ H2SO4 → __ C2H4O2 + __ Cr2(SO4)3 + __ K2SO4 + __ H2O

Os coeficientes para o balanceamento da equação são, respectivamente,

a) 3 / 2 / 3 / 5 / 9 / 8 / 10
b) 3 / 3 / 8 / 9 / 4 / 1 / 12
c) 3 / 2 / 2 / 8 / 8 / 3 / 10
d) 3 / 2 / 8 / 3 / 2 / 3 / 12
e) 3 / 2 / 8 / 3 / 2 / 2 / 11

TESTE 72 – (ITA) Um dos sistemas propelentes usados em foguetes consiste de uma mistura de hidrazina (N2H4) e
peróxido de hidrogênio (H2O2). Sabendo que o ponto triplo da hidrazina corresponde à temperatura de 2,0oC e à
pressão de 3,4 mmHg, que o ponto crítico corresponde à temperatura de 380oC e à pressão de 145 atm e que na
pressão de 1 atm as temperaturas de fusão e de ebulição são iguais a 1,0 e 113,5oC, respectivamente, pedem-se:
a) Um esboço do diagrama de fases da hidrazina para o intervalo de pressão e temperatura considerados neste
enunciado;
b) A indicação, no diagrama esboçado no item a), de todos os pontos indicados no enunciado e das fases presentes
em cada região do diagrama;
c) A equação química completa e balanceada que descreve a reação de combustão entre hidrazina e peróxido de
hidrogênio, quando estes são misturados numa temperatura de 25oC e pressão de 1 atm. Nesta equação, indique os
estados físicos de cada substância;
d) O cálculo da variação de entalpia da reação mencionada em c).
Dados eventualmente necessários: variação de entalpia de formação (∆Hof), na temperatura de 25oC e pressão de 1
atm, referem-se a: N2H4(g): ∆Hof = 95,4 kJ.mol–1; N2H4(l): ∆Hof = 50,6 kJ.mol–1; H2O2(l): ∆Hof = –187,8 kJ.mol–1; H2O(g):
∆Hof = –241,8 kJ.mol–1.

TESTE 73 – (AUSTRALIAN SCIENCE OLYMPIAD EXAM) For a spontaneous reaction to occur at constant
temperature and pressure, the Gibbs free energy (ΔG) must be negative. The Gibbs free energy combines two
thermodynamic parameters into the Gibbs equation: ΔG = ΔH - TΔS where, ΔH is the change in enthalpy and ΔS is
the change in entropy. What conditions of ΔH and ΔS for a chemical reaction will always give a spontaneous reaction?
a) ΔH positive, ΔS positive
b) ΔH positive, ΔS negative
c) ΔH negative, ΔS positive
d) ΔH negative, ΔS negative
e) none of the above

TESTE 74 – (ITA - 1950) Determinar pelo método de óxido-redução, com explicação detalhada, os coeficientes da
seguinte equação química: KMnO4 + HCl → KCl + MnCl2 + H2 O + Cl2

765
1600 FÍSICO-QUÍMICA APLICADA EXERCÍCIOS COMENTADOS - IME – ITA – OLIMPÍADA

TESTE 75 – (ITA) Considere:


I. H2 - C = C - CH2OH
II. H3 - C - CO - CH = CH2
III. H3C - O - CH2 - CH = CH2
Os compostos I, II e III são, respectivamente, exemplos de:
a) hidrocarboneto, éter e cetona
b) álcool, cetona e éter
c) hidrocarboneto, cetona e éster
d) álcool, éster e hidrocarboneto
e) hidrocarboneto, éster e éter

TESTE 76 – (ITA) Aquecendo juntos, ácido benzóico e etanol podemos esperar a formação de:
a) Sal e água
b) Éter e água
c) Éster e água
d) Aldeído e água
e) Cetona e água

TESTE 77 – (ITA) Abaixo é esboçado um aparelho de Kipp. Ele foi projetado para:

a) Lavar e secar gases com auxílio de sólidos.


b) Lavar e secar gases com auxílio de líquidos.
c) Obter gases por reações de craqueamento térmico.
d) Obter gases por reação entre líquidos.
e) Obter gases por reação entre sólidos e líquidos.

TESTE 78 – (ITA) A combustão do gás amoníaco produz uma mistura gasosa incolor. Depois de resfriada essa mistura
se torna castanha, quando posta em contato com oxigênio. A equação química que representa a combustão é:
a) 2 NH3 + 3/2 O2 → N2 + 3 H2O
b) 2 NH4OH → 2 NO + 5 N2
c) NH3 + C → HCN + H2
d) 2 NH3 + 5/2 O2 → 2 NO + 3 H2O
e) 2 NH3 + 7/2 O2 → 2 NO2 + 3 H2O

TESTE 79 – (ITA) Em 1803, John Dalton propôs um modelo de teoria atômica. Considere que sobre a base conceitual
desse modelo sejam feitas as seguintes afirmações:
I- O átomo apresenta a configuração de uma esfera rígida.
II- Os átomos caracterizam os elementos químicos e somente os átomos de um mesmo elemento são idênticos em
todos os aspectos.
III- As transformações químicas consistem de combinação, separação e/ou rearranjo de átomos.
IV- Compostos químicos são formados de átomos de dois ou mais elementos unidos em uma razão fixa.
Qual das opções abaixo se refere a todas afirmações corretas?
a) I e IV
b) II e III
c) II e IV

766
1600 FÍSICO-QUÍMICA APLICADA EXERCÍCIOS COMENTADOS - IME – ITA – OLIMPÍADA

d) II, III e IV
e) I, II, III e IV

TESTE 80 – (ITA) A figura a seguir apresenta esboços de curvas representativas da dependência da velocidade de
reações químicas com a temperatura. Na Figura A é mostrado como a velocidade de uma reação de combustão de
explosivos depende da temperatura. Na Figura B é mostrado como a velocidade de uma reação catalisada por
enzimas depende da temperatura. Justifique, para cada uma das Figuras, o efeito da temperatura sobre a velocidade
das respectivas reações químicas.

TESTE 81 - (ITA) A equação química não balanceada e incompleta: x MnO2 + yH+ + z Cl- → r Mn2+ + sS + t Cl2,
se completa quando:

a) x = 1; y = 2; z = 4; r = 2; sS = 2H2O; t = 2
b) x = 2; y = 4; z = 4; r =2; sS = 2H2O; t = 2
c) x = 2; y = 2; z = 2; r = 1; sS = 2OH-; t = 1
d) x = 1; y = 4; z = 4; r = 1; sS = 2H2O; t = 2
e) x = 1; y = 4; z = 2; r = 1; sS = 2H2O; t = 1

TESTE 82 – (ITA) Ao misturar solução aquosa de permanganato de potássio com solução aquosa de ácido clorídrico
ocorre reação que é representada , a seguir de forma não-balanceada: 2 MnO −4 + x H+ + y Cl- → 2 Mn2+ + z Cl2+ w
H2O. O balanceamento dessa equação pode ser feito pelo método das variações dos números de oxidação. Quais
são essas variações para cada átomo e quais são os coeficientes das substâncias que balanceiam a equação ?
Variação do número Coeficientes
Mn O H Cl x y z w
a) -5 0 0 +1 16 10 5 8
b) +7 -2 +1 -1 16 5 5 16
c) +2 -2 +1 0 8 5 5 8
d) -4 0 0 +4 8 5 10 8
e) 0 +1 -1 0 8 10 5 8

TESTE 83 – (ITA - MODIFICADA) Uma solução aquosa 0,84 molar em ácido nítrico tem densidade de 1,03 g/cm3.
Calcule a quantidade de ácido nítrico presente em 50,0 cm3 dessa solução.

TESTE 84 - (IME) O dispositivo a seguir utiliza a radiação solar para quantificar variações em propriedades
termodinâmicas. Este dispositivo é composto por uma lente convergente e por um porta-amostras. A lente possui área
útil de 80,0 cm2, absortividade (α) de 20% e transmissividade (τ) de 80%. O porta-amostras possui absortividade de
100% e volume variável, operando à pressão constante de 1,0 atm.

767
1600 FÍSICO-QUÍMICA APLICADA EXERCÍCIOS COMENTADOS - IME – ITA – OLIMPÍADA

Em um procedimento experimental, injetou-se 0,100 mol de uma substância pura líquida no porta-amostras do
dispositivo. Em seguida, mediu-se um tempo de 15,0 min para a vaporização total da amostra, durante o qual a
irradiação solar permaneceu constante e igual a 750 W/m2. Nesse processo, a temperatura do porta-amostras
estabilizou-se em 351 K. No experimento, o calor sensível da amostra e a radiação emitida pelo porta-amostras são
desprezíveis. Pode-se concluir que na vaporização total da substância, as variações de entalpia molar padrão e de
entropia molar padrão são, respectivamente:
a) 4,32 kJ/mol e 12,3 J/(mol K)
b) 5,40 kJ/mol e 15,4 J/(mol K)
c) 43,2 kJ/mol e 123 J/(mol K)
d) 54,0 kJ/mol e 154 J/(mol K)
e) 31,6 kJ/mol e 90,0 J/(mol K)

TESTE 85 – (IME) Alguns elementos apresentam irregularidades na sua distribuição eletrônica já que as
configurações d5, d10, f7 e f14 são muito estáveis. Por exemplo, o Cu (Z=29), em vez de apresentar a distribuição 1s 2
2s2 2p6 3s2 3p6 4s2 3d9, apresenta 1s2 2s2 2p6 3s2 3p6 4s1 3d10. Determine os 4 números quânticos do elétron mais
externo da prata (Z = 47) sabendo que o mesmo tipo de irregularidade ocorre para este elemento.

TESTE 86 - (ITA) “Ácido é uma substância capaz de receber 1 par de elétrons”. A definição acima corresponde a
proposta de:
a) Arrhenius
b) Bronsted
c) Lavoisier
d) Lewis
e) Ostwald

TESTE 87 – (IME) Considere a reação de decomposição da nitramida em solução aquosa: NH2 NO2(aq) →
N2 O(g) + H2 O(l) . Sabendo-se que a lei de velocidade, determinada experimentalmente, é dada pela expressão v =
[NH2 NO2 ]
kx [H3 O+ ]
, foram propostos três possíveis mecanismos para a reação:

MECANISMO I:
NH2 NO2 + H2 O ⇌ NHNO−2 + H3 O
+
(equilíbrio rápido)
− −
NHNO2 → N2 O + OH (etapa lenta)
H3 O+ + OH − → 2 H2 O (etapa rápida)

MECANISMO II: NH2 NO2 → N2 O + H2 O (etapa elementar)

MECANISMO III:
NH2 NO2 + H3 O+ ⇌ NH3 NO+
2 + H2 O (equilíbrio rápido)
NH3 NO+
2 ⇌ N2 O + H3 O
+
(etapa lenta)

Com base nas informações acima, determine se cada mecanismo proposto é compatível com a expressão da
velocidade experimental, fundamentando suas respostas.

TESTE 88 – (ITA) O que é uma reação de óxido-redução?

768
1600 FÍSICO-QUÍMICA APLICADA EXERCÍCIOS COMENTADOS - IME – ITA – OLIMPÍADA

TESTE 89 – (IME) Escreva as fórmulas das substâncias estáveis, nas CNTP, formadas apenas pelos elementos 11A
23, D 34 e E 20, especificando os tipos de ligações químicas envolvidas.
17 10
TESTE 90 – (PREPARATÓRIO - OLIMPÍADA CEARENSE DO ENSINO SUPERIOR DE QUÍMICA) Um estudante
em um laboratório de Química, para neutralizar 10,0 mL de uma solução de hidróxido de sódio, gastou 8,0 mL de uma
solução de ácido clorídrico 1,0 mol L-1. Qual a concentração da base em mol.L-1 e em g.L-1?

a) 8,0 mol.L-1; 320 g.L-1


b) 0,4 mol.L-1; 40 g.L-1
c) 0,8 mol.L-1; 32 g.L-1
d) 0,8 mol.L-1; 320 g.L-1
e) 8,0 mol.L-1; 32,0 g.L-1

TESTE 91 – (OLIMPÍADA CEARENSE DO ENSINO SUPERIOR DE QUÍMICA) Uma solução foi preparada pela
dissolução de 0,373 g de KCl em 50 g de água. Admite-se, normalmente, que sais como o KCl se dissociam
completamente quando dissolvidos em água. Se a água da solução congela a -0,345°C, qual o percentual real de
dissociação do KCℓ nessa solução? Dado: Kc = 1,853 °C.mol-1.kg.
a) 93,0%
b) 88,0%
c) 91,0%
d) 90,0%
e) 99,0%

TESTE 92 – (ITA) Enunciar a Lei de Hess da termoquímica (princípio dos estados inicial e final).

TESTE 93 – (IME) Dê a configuração eletrônica no estado fundamental do elemento com número atômico Z = 79.
Determine o período e o grupo da Tabela Periódica a que pertence o elemento.

TESTE 94 – (ITA) Uma mistura gasosa é colocada a reagir dentro de um cilindro provido de um pistão móvel, sem
atrito e sem massa, o qual é mantido à temperatura constante. As reações que ocorrem dentro do cilindro podem ser
genericamente representadas pelas seguintes equações:
I. A(g) + 2B(g)  3C(g)
II. C(g)  C(l)
O que ocorre com o valor das grandezas abaixo (Aumenta? Diminui? Não altera?), quando o volume do cilindro é
duplicado? Justifique suas respostas.
a) Quantidade, em mols, da espécie B.
b) Quantidade, em mols, da espécie C líquida.
c) Constante de equilíbrio da equação I.
d) Razão [C]3/[B]2.

TESTE 95 – (ITA) Considere a equação: 1 H2O2 + a Fe++ + b H+ → c Fe3+ + d H2O. Qual afirmação a respeito
das substâncias e da reação é certa?
a) O número de oxidação do oxigênio na água oxigenada é -2.
b) Os coeficiente da equação que faltam são: a = 1; b = 2; c = 1; d = 2.
c) A água oxigenada é oxidante porque cedeu elétrons ao Fe2+.
d) O H+ é o redutor.
e) Nenhuma das afirmações acima está certa.

TESTE 96 – (ITA) A observação experimental de que 1,00 g de oxigênio pode combinar seja com 12,53 g de mercúrio,
seja com 25,06 g de mercúrio, foi generalizada por:
a) Lavoisier, na lei da conservação das massas.
b) Guldberg e Waage, na lei da ação das massas.
c) Proust, na lei das proporções definidas.
d) Dalton, na lei das proporções múltiplas.
e) Richter e Wenzel, na lei das proporções recíprocas.

769
1600 FÍSICO-QUÍMICA APLICADA EXERCÍCIOS COMENTADOS - IME – ITA – OLIMPÍADA

TESTE 97 – (ITA) Qual dos elementos abaixo entrou na lista dos elementos conhecidos em virtude de sua ocorrência
na América Esponhola:
a) Estanho
b) Chumbo
c) Cobre
d) Prata
e) Platina

TESTE 98 – (ITA) A reação não-balanceada e incompleta ocorre em meio ácido: (Cr2O7)-2 + (C2O4)-2 → Cr3+ + CO2.
A soma dos coeficientes estequiométricos da reação completa e balanceada é igual a
a) 11
b) 22
c) 33
d) 44
e) 55

TESTE 99 – (ITA - MODIFICADA) Dado o seguinte mecanismo reacional, constituído de duas etapas elementares (I
e II).

Escreva a expressão para a taxa de variação temporal da concentração do:


a) reagente A.
b) intermediário M.
c) produto C.
d) a equação da velocidade de taxa.

TESTE 100 – (ITA) Um reator com 200 L de capacidade, possui uma mistura de dióxido de nitrogênio e monóxido de
carbono a 400K, cujo comportamento pode ser considerado ideal. Os gases reagem entre si para formar dióxido de
carbono e monóxido de nitrogênio. A pressão total no reator é igual a 32,8 atm e, no início da reação, a pressão parcial
do monóxido de carbono é três vezes maior que a do dióxido de nitrogênio. As massas iniciais de dióxido de nitrogênio
e de monóxido de carbono são, respectivamente,
a) 1,5 kg e 4,2 kg
b) 1,5 kg e 4,5 kg
c) 1,5 kg e 6.6 kg
d) 2,3 kg e 4,2 kg
e) 2,3 kg e 6,6 kg

TESTE 100 – (ITA) Na elaboração das primeiras classificações periódicas, um dos critérios mais importantes para
agrupar elementos numa mesma coluna foi observar:
a) O último subnível eletrônico ser igualmente ocupado.
b) Mesma(s) valência(s) na combinação com elementos de referência.
c) Mesma estrutura cristalina dos próprios elementos.
d) Número atômico crescente.
e) Número de massa crescente.

TESTE 101 – (IME) Um sistema A transfere, naturalmente, uma determinada quantidade de energia, na forma de
calor, para um sistema B, que envolve totalmente A. Assinale a única alternativa correta.
a) A entropia do Universo decrescerá.
b) A entropia do sistema A crescerá.
c) O aumento da entropia do sistema B será maior do que o decréscimo da entropia do sistema A.
d) O aumento da entropia do sistema B será menor do que o decréscimo da entropia do sistema A.
e) O aumento da entropia do sistema B será necessariamente igual ao decréscimo da entropia do sistema A.

770
1600 FÍSICO-QUÍMICA APLICADA EXERCÍCIOS COMENTADOS - IME – ITA – OLIMPÍADA

TESTE 102 – (IME) Há mais de dois séculos, surgiu a expressão “ compostos orgânicos” para designar as substâncias
produzidas por organismos vivos, animais ou vegetais. Atualmente, a química orgânica estuda as substâncias que
possuem átomos de carbono, embora nem todas as substâncias que contenham carbono estejam no universo da
química orgânica. Em tais substâncias orgânicas, os átomos de carbono apresentam hibridização sp, sp 2 ou sp3
conforme as ligações. No metanol, metanal, triclorometano e etino os carbonos apresentam, respectivamente,
hibridização:
a) sp, sp2, sp3 , sp3
b) sp2, sp3 , sp , sp3
c) sp3, sp2 , sp , sp2
d) sp, sp3 , sp2 , sp
e) sp3, sp2 , sp3 , sp

TESTE 103 – (IME) Quantos isômeros existem para o dicloro fenol ?


a) 3
b) 4
c) 5
d) 6
e) 7

TESTE 104 – (ITA) De que maneira influi o catalisador de uma reação química quanto:
a) a velocidade da reação;
b) as massas de reagentes e produtos existentes no equilíbrio?

TESTE 105 – (ITA) A que pressão o N2 terá densidade de 2,0 g.L-1 se a temperatura é de 177°C?

TESTE 106 – (ITA) 22,4 litros de vapor de água, medidos a 100°C e 760 mmHg contém:
a) 6,023 x 1023 moléculas
b) 18 g de água
c) menos de 18g de água
d) mais de 18g de água
e) Nenhuma das respostas anteriores

TESTE 107 – (IME) Dispõe-se de uma amostra de polônio-218 cuja meia-vida é de 3 minutos. Calcule o tempo
necessário para que a amostra se reduza a 20% do seu peso.

TESTE 108 – (ITA) A fórmula do dihidrogenofosfato de amônio é:


a) (NH4)2HPO4
b) (NH3)2H2PO4
c) (NH4)HPO4
d) (NH4)H2PO4
e) Nenhuma das respostas anteriores

TESTE 109 – (ITA) O que é o volume molar?

TESTE 110 – (ITA) Quais são os principais fatores que influem na velocidade de uma reação química?

TESTE 111 – (ITA) Por que motivo em certas reações industriais se utilizam catalisadores? Sem estes, essas reações
se processariam?

TESTE 112 – (ITA) A que temperatura deve ser aquecido um frasco aberto afim de que 1/5 (um quinto) do gás que
ele encontra a 7°C se escape?

TESTE 113 – (IME) A eritromicina é uma substância antibacteriana do grupo dos macrolídeos muito utilizada no
tratamento de diversas infecções. Dada a estrutura da eritromicina abaixo, assinale a alternativa que corresponde às
funções orgânicas presentes.

771
1600 FÍSICO-QUÍMICA APLICADA EXERCÍCIOS COMENTADOS - IME – ITA – OLIMPÍADA

a) Álcool, nitrila, amida, ácido carboxílico.


b) Álcool, cetona, éter, aldeído, amina.
c) Amina, éter, éster, ácido carboxílico, álcool.
d) Éter, éster, cetona, amina, álcool.
e) Aldeído, éster, cetona, amida, éter.

Questão 114 – (IME) Uma amostra de IBr, de massa 8,28 g, é aquecida a 227ºC em um recipiente de 0,250 L,
decompondo-se parcialmente em iodo e bromo. Sabendo que, ao atingir o equilíbrio, em fase gasosa, a pressão
parcial do bromo é de 3,08 atm, calcule o valor da constante de equilíbrio.

TESTE 115 – (IME - MODIFICADA) Uma solução 6,5 mol x L-1 de etanol em água tem massa específica 0,95 g/cm3.
Calcule a molalidade, concentração comum e a fração molar de etanol dessa solução.

TESTE 116 – (IME) As reações eletrolíticas são, sempre, reações de oxidação e redução? Por que?

TESTE 117 – (IME) Uma mistura de 0,1 g de hidrogênio e 6,4 g de oxigênio é armazenada a 760 mmHg e a 27°C.
Calcule:
a) as frações molares dos gases na mistura;
b) a pressão parcial dos gases na mistura;
c) o volume total da mistura considerando comportamento ideal.

TESTE 118 – (IME) Uma solução de ureia (60,06 g.mol-1) com a concentração de 0,450 g / 22,5 g de água, apresenta
uma elevação de 0,170°C no ponto de ebulição da água. Calcule a massa molecular de um composto que na
concentração de 1,84 g / 45,0 g água apresente a mesma elevação do ponto de ebulição.

TESTE 119 – (IME) Calcule a variação de entalpia (em J) no processo de decomposição de 600 mg de nitroglicerina
(C3H5N3O9) que produz nitrogênio, dióxido de carbono e oxigênio gasosos, além de água líquida. Dados: ΔH0f
(C3H5N3O9(ℓ)) = − 354 kJ/mol; ΔH0f (H2O(ℓ)) = − 286 kJ/mol; ΔH0f (CO2(g)) = − 394 kJ/mol.

TESTE 120 – (GRILLO) Em um determinado gás ideal há uma absorção na quantidade de calor na ordem de 8000
cal e o trabalho realizado pelo sistema é na ordem de 10000 calorias. Sabendo que a temperatura inicial é de 1000 K
cal
e que a capacidade calorífica a pressão constante é de 14 , calcule para este processo descrito a temperatura
mol x K
cal
final. Informação para a resolução do problema: R = 2 mol x K.

772
1600 FÍSICO-QUÍMICA APLICADA EXERCÍCIOS COMENTADOS - IME – ITA – OLIMPÍADA

TESTE 121 – (IME) Se A, B e C são elementos de números atômicos 9, 17 e 19, respectivamente, dar:
a) Os tipos de ligação entre AB e AC.
b) A valência de A em AC e de B em BC.
c) Em que estados (sólido, líquido ou gasoso), cada um dos três compostos deveriam existir, em condições normais
de temperatura e pressão?

TESTE 122 – (IME) Considere a equação de dissociação do composto A, que ocorre a uma determinada temperatura:
2𝐴(𝑔) ⇌ 2𝐵(𝑔) + 𝐶(𝑔). Desenvolva a expressão para o cálculo da pressão total dos gases, que se comportam
idealmente, em função do grau de dissociação () nas condições de equilíbrio.

TESTE 123 – (IME - ADAPTADA) Em uma pilha Ni0 / Ni2+ // Ag+1 / Ag0, os metais estão mergulhados em soluções
aquosas 1,0 mol.L-1 de seus respectivos sulfatos, a 25°C. Determine:
a) a equação global da pilha;
b) o sentido do fluxo de elétrons;
c) o valor da força eletromotriz (fem) da pilha;
d) o valor da energia livre de Gibbs padrão;
e) o valor da constante de equilíbrio.
Dados:
Ni2+(aq) + 2e- → Ni0(s) (E0redução = - 0,25 V)
Ag+(aq) + 1e- → Ag0(s) (E0redução = + 0,80 V)

Teste 124 – Deduza a função de estado entalpia.

TESTE 125 – Considere a seguinte reação de decomposição do óxido de alumínio, que ocorre em um sistema
fechado, a 25C. A seguir são mostrados os parâmetros termodinâmicos para a reação de decomposição do referido
óxido: Al2O3(s)  2 Al(s) + 3/2 O2(g), para a qual  = + 90,83 kJ.mol-1 e G = + 58,54 kJ.mol-1.
a) Determine pressão do O2(g) em equilíbrio com Al2O3(s) a 25C.
b) Explique, através do princípio de Le Chatelier, pelo menos duas formas possíveis para que aumente a
produção de oxigênio.

TESTE 126 – (GRILLO) Considere a reação química para a produção de enxofre, em sua forma ortorrômbica, a partir
do sulfeto de hidrogênio, conforme esquematizada na equação química apresentada a seguir: SO2(g) + H2S(g)  3
S(ortorrômbico) + 2 H2O(g). Considere a tabela termodinâmica apresentada a seguir:
a) Explique se o processo é endotérmico ou exotérmico.
b) Calcule a constante de quilpíbrio químico a 25°C.
c) Aumentando a temperatura, o que irá acontecer com o estado do equilíbrio químico e também com o seu
valor.
d) Calcule a constante de quilíbrio a 150°C.

TESTE 127 – Considere a seguinte equação iônica, 2 Fe(s) + O2(g) + 4 H+(aq) → 2 Fe+2(aq) + 2 H2O(l), E° = + 1,67 V. Com
[Fe+2] = 10-3 mol.L-1; PO2 = 0,1 atm e pH = 3, determine o potencial da célula à 25°C.

TESTE 128 – (IME) Dada a reação em fase gasosa N2O4(g) ⇌ 2 NO2(g) com Kp = 0,140. Determinar o grau de
dissociação e as frações molares de equilíbrio, sabendo-se que a pressão total dos gases no equilíbrio é de 1,0 atm.

TESTE 129 – (ITA) Que massa de ozônio comporta um frasco nas mesmas condições de pressão e temperatura
comporta 4,0 g de oxigênio?
a) 2,0 g
b) 4,0 g
c) 6,0 g
d) 16,0 g
e) 48, 0 g

773
1600 FÍSICO-QUÍMICA APLICADA EXERCÍCIOS COMENTADOS - IME – ITA – OLIMPÍADA

TESTE 130 – (ITA) Escreva a reação de combustão completa de um hidrocarboneto genérico (CαHβ) com ar
atmosférico. Considere a presença do nitrogênio gasoso no balanço estequiométrico da reação e expresse os
coeficientes estequiométricos dessa reação em função de α e β.

TESTE 131 – (ITA) Assinale a opção que contém a substância cuja combustão, nas condições-padrão, libera maior
quantidade de energia.
a) Benzeno
b) Ciclohexano
c) Ciclohexanona
d) Ciclohexeno
e) n-Hexano

TESTE 132 - (ITA - MODIFICADA) A 20C uma solução aquosa de hidróxido de sódio tem uma densidade de 1,04
g/cm3 e é 0,946 molar em NaOH. Calcule a quantidade e a massa de hidróxido de sódio presentes em 50,0 cm3 dessa
solução.

TESTE 133 – (ITA) A 45C a densidade da água pura é 0,99 g/cm3. Partindo desta informação, calcule [H2O], isto é,
o número de moles de água por litro de água pura nesta temperatura. Deixe bem claro e raciocínio utilizado nos
cálculos.

TESTE 134 – (ITA) O abaixamento da temperatura de solidificação de uma solução diluída é uma propriedade
coligativa se o sólido que aparece a solidificação for formado:
a) apenas pelo soluto
b) apenas pelo solvente
c) tanto pelo soluto como pelo solvente
d) pela mistura heterogênea do soluto e do solvente
e) pela mistura homogênea do soluto e do solvente

TESTE 135 – (ITA) A figura representa a curva de aquecimento de uma amostra, em que S, L e G significam,
respectivamente, sólido, líquido e gasoso. Com base nas informações da figura é CORRETO afirmar que a amostra
consiste em uma:

a) substância pura.
b) mistura coloidal.
c) mistura heterogênea.
d) mistura homogênea azeotrópica.
e) mistura homogênea eutética.

TESTE 136 – (GRILLO) Um técnico de laboratório misturou duas soluções, uma de cabonato de sódio e outra de
ácido sulfúrico diluído. A primeira solução, carbonato de sódio, apresenta as seguintes características: 0,25 mol.L-1 e
volume igual a 33 mL. Já a solução de ácido sulfúrico apresenta concentração igual a 225 g.L-1 e volume de 20 mL. A
partir desta informação, o técnico deseja obter uma concentração molar para o sal formado de 2,0 mol.L -1 e também
as seguintes características para o gás produzido: 1,50 atmosferas a 127°C. Diante disso, determine:
a) Dê a nomenclatura do sal formado;
b) O volume de sal produzido, em mililitros;

774
1600 FÍSICO-QUÍMICA APLICADA EXERCÍCIOS COMENTADOS - IME – ITA – OLIMPÍADA

c) O volume de gás produzido em centímetros cúbicos.

TESTE 137 – (IME) Uma célula eletrolítica industrial para produzir alumínio utiliza uma corrente de 6700 ampéres.
Calcule a quantidade de alumínio produzida por dia, em quilogramas, admitindo uma eficiência de 90% de processo.

TESTE 138 – Faça a demonstração e organize o raciocínio para o seguinte equilíbrio químico: N2O4(g) ⇄ 2 NO2(g),
em que a constante de equilíbrio em função das pressões parciais é dada pela seguinte expressão:
4α2
K p = {1−α2 } x ptotal .

TESTE 139 – (GRILLO) Cinco mols de um gás perfeito expande isotermicamente, contra uma pressão oposta de 200
kPa, de 40 dm³ para 120 dm³. Calcule os seguintes parâmetros: Q, ∆H, ∆U e W.

TESTE 140 – (OLIMPÍADA DE QUÍMICA DO RIO DE JANEIRO) Suponha que 0,157 g de um certo gás coletado
sobre água ocupa um volume de 135 mL a 30°C e 752,82 mmHg. Considerando o comportamento ideal, determine a
massa molecular do gás. (A 30 °C a pressão de vapor da água é 31,82 mmHg)

a) 29,17 g.mol-1
b) 29,45 g.mol-1
c) 29,90 g.mol-1
d) 30,45 g.mol-1
e) 31,00 g.mol-1

TESTE 141 – A velocidade de formação do produto gasoso C na equação química representada por 2A + B → 2C +
3D é igual a 1,0 kmol/dm³ x min. Dar a velocidade da reação e as velocidades de formação ou de consumo de A, B
e D.

TESTE 142 – (GRILLO) Calcule o abaixamento relativo e o abaixamento da pressão máxima de vapor quando 5,0
gramas de ureia, CO(NH2)2 são dissolvidos em 500 gramas de água, sabendo que a pressão máxima de vapor da
água pura é igual a 32 torr a 28°C.

TESTE 143 – (MESTRE JOÃO ROBERTO DA PACIÊNCIA NABUCO) Calcular a pressão máxima de vapor de uma
solução cloreto de magnésio contendo 19 gramas do mesmo em 360 gramas de água, a 28°C. A pressão máxima de
vapor da água a esta temperatura é igual a 32,0 mmHg e a solução de cloreto de magnésio se encontra 80%
dissociado.

TESTE 144 – (GRILLO) Um volume de 20 m³ de ar atmosférico a uma pressão de 10 atmosferas e 227°C é colocado
em um reator de 200 m³ e que já continha ar sob pressão de 2 atmosferas e 127°C. Calcule a pressão do sistema
reacional (pressão total) quando a temperatura se estabilizar em 327°C?

TESTE 145 – Calcule a variação de entalpia para 1500 gramas de gás oxigênio, em um intervalo que vai de 1000C a
2000C. Informação para a resolução do problema: Calor específico do gás oxigênio, Cp(O2) = 29,96 + 4,18 x 10-3T.

TESTE 146 – (ITA) Uma cuba eletrolítica contendo uma solução de nitrato de prata é atravessada por uma corrente
elétrica de intensidade constante durante 32 minutos e 10 segundos. Observa-se, decorrido este tempo, que o cátodo
apresenta um aumento de massa de 1,08 g. Calcular a intensidade da corrente.

775
1600 FÍSICO-QUÍMICA APLICADA EXERCÍCIOS COMENTADOS - IME – ITA – OLIMPÍADA

TESTE 147 – (ITA) Considere as seguintes semirreações de oxirredução e seus respectivos potenciais padrão na
escala do eletrodo padrão de hidrogênio (EPH):
I. 2 CO2 + 12 H+ + 12e- = C2H5OH + 3 H2O E°I = + 0,085 V
II. O2 + 4 H + 4e = 2 H2O
+ - E°II = + 1,229 V

Assinale a opção que apresenta a afirmação errada sobre uma célula eletroquímica em que a semirreação I ocorre
no ânodo e a semirreação II, no cátodo.
a) A reação global é exotérmica.
b) Trata-se de uma célula a combustível.
c) O potencial da célula é de 1,144 V.
d) O trabalho máximo que pode ser obtido é, em módulo, de 4171 kJ por mol de etanol.
e) A célula converte energia livre da reação de combustão do etanol em trabalho elétrico.

TESTE 148 – (ITA) 12 g de pirita pura, queima com a quantidade estequiométrica de O2 fornecida pelo ar (composição
do ar: 80% de N2 e 20% de O2 em volume) segundo a equação: 4 FeS2 + 11 O2 → 2 Fe2O3 + 8 SO2. Qual o volume
gasoso resultante, medido nas condições normais de temperatura e pressão?
a) 1,3 litros
b) 54,2 litros
c) 29,1 litros
d) 34,0 litros
e) Nenhuma das respostas anteriores

TESTE 149 – (ITA) A dissolução de um sólido iônico em uma certa quantidade de água faz com que, em relação à
água pura e nas mesmas condições de temperatura e pressão, a solução apresente:
I - Pressão osmótica menor.
II - Pressão de vapor menor.
III - Temperatura de início de fusão menor.
IV - Temperatura de início de ebulição menor.
V - Densidade maior.
Das afirmações acima estão certas:
a) Apenas I, II e V.
b) Apenas II, III e IV.
c) Apenas II, III e V.
d) Apenas I, II, III e V.
e) Apenas I, II, IV e V.

TESTE 150 – (ITA) PERGUNTA - Por que afirmação I DA QUESTÃO ANTERIOR (TESTE 149) está certa ou está
errada?

TESTE 151 – (ITA) Considere os seguintes potenciais de eletrodo em relação ao eletrodo padrão de hidrogênio nas
° °
condições-padrão (E ° ): EM 3+ /M2+ = +0,80 V e EM2+ /M0 = −0,20 V. Assinale a opção que apresenta o valor,
°
em V, de EM 3+ /M0 .

a) – 0,33
b) – 0,13
c) +0,13
d) +0,33
e) +1,00

TESTE 152 – (ITA) Esta questão refere-se ao balanceamento estequiométrico da equação.

x MnO− +
4 + y H + 5 H2 O2 → z Mn
+2
+ w H2 O + 5 O2 . Qual das afirmações é falsa?

a) x deve ser igual a z


1
b) x deve ser igual a 3 de y

776
1600 FÍSICO-QUÍMICA APLICADA EXERCÍCIOS COMENTADOS - IME – ITA – OLIMPÍADA

1
c) x deve ser igual a de w
4
d) x deve ser igual a w menos y
e) x deve ser igual a 2z menos y

TESTE 153 – (IIT) The Van’t Hoff ‘s fator for 0,10 mol.L-1, Ba(NO3)2 solution is 2,74. The degree of dissociation is:
a) 91,3%
b) 87%
c) 100%
d) 74%

TESTE 154 – (OLIMPÍADA BRASILEIRA DE QUÍMICA) Em um processo industrial, a obtenção do zinco a partir da
blenda, um minério constituído de sulfeto de zinco, ocorre por meio da reação, não balanceada, que é representada
a seguir: ZnS(s) + O2(g) + CO(g) → Zn(s) + SO2(g) + CO2(g). Considerando que a reação ocorra com rendimento de
80%, qual quantidade de zinco pode ser obtida a partir de 2,425 kg do minério blenda que é 80% (m/m)?
a) 1,041 kg
b) 1,303 kg
c) 1,627 kg
d) 1,940 kg

TESTE 155 – O potencial-padrão do par Cu2+(aq) / Cu é + 0,340 V, e o par Cu+(aq) / Cu é + 0,552 V. Estime Eo (Cu2+(aq)
/ Cu+(aq)) e a sua respectiva constante de equilíbrio.

TESTE 156 – (UNIVERSITY OF WATERLOO DEPARTMENT OF CHEMISTRY) For the following reactions, the given
enthalpy changes are expressed per mole of product formed.
H2(g) + 1/2 O2(g) → H2O(g), ∆H° = −241.8 kJ.mol−1
C(s) + O2(g) → CO2(g), ∆H° = −393,5 kJ.mol−1
2 C(s) + 3 H2(g) → C2H6(g), ∆H° = −84,68 kJ.mol−1
Given the thermochemical equations above, what is the standard enthalpy change for the reaction below? The answers
are expressed in kJ per mole of C2H6.
C2H6(g) + 7/2 O2(g) → 2 CO2(g) + 3 H2O(g)
a) −985.3 kJ mol−1
b) −1428 kJ mol−1
c) −720.0 kJ mol−1
d) −866.2 kJ mol−1
e) −550.6 kJ mol−1

TESTE 157 – Kp = 1,78 atm at 250°C for the decomposition reaction PCl5(g) ⇄ PCl3(g) + Cl2(g). Calculate the percentage
of PCl5that dissociates if 0,0500 moles of PCl5 is placed in a closed vessel at 250°C and 2,00 atm pressure.

TESTE 158 – (OLIMPÍADA BRASILEIRA DE QUÍMICA) Atualmente, muitos suplementos alimentares contêm
substâncias que beneficiam naturalmente a produção do óxido nítrico no organismo. Como fármaco, a produção de
óxido nítrico se inicia com a reação entre dióxido de enxofre, ácido nítrico e água, originando, além desse gás, o ácido
sulfúrico. Como produto final, o óxido nítrico é comercializado em cilindros de 32 litros, diluído em nitrogênio com uma
concentração máxima de 0,08 % em massa e chega a fornecer cerca de 4.800 litros de gás a 25ºC e 1 atmosfera.
a) Escreva a equação química da reação de produção do NO.
b) Qual é a massa aproximada de NO contida no cilindro à qual se refere o enunciado da questão?
c) Determine a densidade do óxido nítrico em relação ao ar e ao dióxido de enxofre.
d) A densidade de um gás X, em relação ao dióxido de enxofre, é 2. Nas mesmas condições de temperatura e
pressão, determine a massa molecular de X.
e) Em um recipiente fechado foram colocados 2 mols de NO(g), 4 mols de SO2(g) e 4 mols de H2(g) sem que
pudessem reagir entre si. Tendo conhecimento que o volume total ocupado foi de 22,0 L e que a temperatura foi
mantida a 0ºC, calcule as frações molares e a pressão total exercida pela mistura.

777
1600 FÍSICO-QUÍMICA APLICADA EXERCÍCIOS COMENTADOS - IME – ITA – OLIMPÍADA

TESTE 159 – (OLIMPÍADA BRASILEIRA DE QUÍMICA) Visto que o petróleo é um combustível não renovável e que
contribui para a poluição do meio ambiente, várias indústrias e centros de pesquisas têm se mobilizado na busca por
novas fontes de energia combustível. É nesse contexto que surge o hidrogênio combustível, considerado como o
combustível do futuro, por ser renovável, inesgotável e principalmente por não liberar gases tóxicos para a atmosfera.
Abaixo têm-se algumas vantagens do combustível hidrogênio:
- Utilização de motores elétricos no lugar de motores a combustão, minimizando a poluição do meio ambiente;
- Seu processo de geração de energia é descentralizado, não sendo necessária a construção de hidrelétricas;
- A geração de energia por meio de pilhas à combustível é mais eficiente do que a obtida pelos processos tradicionais.
A reação abaixo mostra a possibilidade de obtenção de hidrogênio a partir do monóxido de carbono:
CO(g) + H2O(g) ⇌ CO2(g) + H2(g) ΔH0 = - 41,5 kmol-1 ; Kc = 0,625 a 1240 K
Analisando os dados da reação acima, afirma-se:
I - Um aumento da pressão total sobre o sistema não altera o estado de equilíbrio;
II- Uma diminuição da temperatura favorece o aumento na produção de gás hidrogênio;
III- O valor de Kp > Kc nas condições dadas;
IV- A concentração final de cada componente do sistema, em equilíbrio, quando se misturam um mol de cada um dos
reagentes com dois mols de cada um dos produtos, na temperatura da experiência, considerando um balão
volumétrico de 1 L é: [CO] = [H2O] = 0,46 mol.L-1 e [CO2] = [H2] = 2,54 mol.L-1.
Estão corretas as seguintes afirmações:
a) I, II e IV apenas
b) I e II apenas
c) II e IV apenas
d) III e IV apenas
e) I e III apenas

TESTE 160 – (OLIMPÍADA DE QUÍMICA DO RIO DE JANEIRO) O gás oxigênio pode ser obtido em laboratório pela
decomposição do peróxido de hidrogênio ou do clorato de potássio, que forma, além do gás oxigênio, o cloreto de
potássio. Para se obter um mol de gás oxigênio, utiliza-se certa massa de clorato de potássio (m1) ou de peróxido de
hidrogênio (m2). A razão m1/m2 é, aproximadamente:
a) 0,3
b) 0,7
c) 1,2
d) 1,6
e) 3,0

TESTE 161 – O potencial normal do eletrodo Cu+2 / Cu é +0,337 V a 298K. Quando a esta temperatura se estabelece
[Cu+2 ] L
o equilíbrio da seguinte reação: Cu(s) + Cu+2 +
(aq) ⇌ 2 Cu(aq) e a relação [Cu+ ]²
= 1,70 x 106 mol. Calcule o
potencial normal do eletrodo Cu+ / Cu a 298 K.

TESTE 162 – (OLIMPÍADA PORTUGUESA DE QUÍMICA) A Quarta porta abre para um corredor mal iluminado, ao
fundo do qual há uma porta entreaberta. Os nossos amigos percorrem o corredor e entram de rompante numa sala
que, afinal, é uma sala de aulas da sua Escola, onde está a decorrer a prova Global de Química do 10º ano! O tempo
está a acabar e ainda têm de resolver este problema:
"Considere o equilíbrio em fase gasosa: 2SO2(g) + O2(g) → 2 SO3(g) (reacção exotérmica).
Num recipiente fechado, com a capacidade de 10,0 dm3, introduziram-se a uma dada temperatura 6,0 mol de dióxido
de enxofre e 3,0 mol de oxigénio molecular. Atingido o equilíbrio, verificou-se que havia no reactor 4,5 mol de trióxido
de enxofre.
1) Calcule a concentração das diferentes espécies químicas presentes no equilíbrio.
2) Indique, justificando, qual o sentido em que evoluirá o sistema por:
a) aumento da temperatura do reactor;
b) aumento da pressão total do sistema por diminuição do volume do reactor;
c) aumento da pressão por adição de um gás inerte;
d) adição de um catalisador. "

778
1600 FÍSICO-QUÍMICA APLICADA EXERCÍCIOS COMENTADOS - IME – ITA – OLIMPÍADA

TESTE 163 – Um reator contém um dispositivo que se trata de um êmbolo apresentando um gás ideal. O gás
comprime de forma reversível e isotermicamente até o volume reduzir a 1/7 do volume inicial. Calcule o trabalho
realizado pelo gás sobre o êmbolo.

TESTE 164 - (U. S. NATIONAL CHEMISTRY OLYMPIAD) For the reaction A → B that is first-order in A, the rate
constant is 2,08×10–2 s–1. How long would it take for [A] to change from 0,100 M to 0,0450 M?
a) 0,0166 s
b) 16,7 s
c) 38,4 s
d) 107 s

TESTE 165 – (U. S. NATIONAL CHEMISTRY OLYMPIAD) What is the coefficient of I2(s) when the reaction below is
balanced with smallest whole number coefficients? __ Cr2O72–(aq) + __ I–(aq) + __ H+(aq) → __ I2(s) + __ Cr3+(aq) + __
H2O(l).
a) 2
b) 3
c) 4
d) 6

TESTE 166 – (U. S. NATIONAL CHEMISTRY OLYMPIAD) What is the oxidation number of Re in Mg(ReO4)2?
a) +4
b) +5
c) +6
d) +7

TESTE 167 – (U. S. NATIONAL CHEMISTRY OLYMPIAD) It takes 126,5 minutes using a current of 5,15 A to deposit
all of the nickel from 225 mL of a solution containing Ni2+. What was the original concentration of Ni2+ in the solution?
a) 3,60 M
b) 1,80 M
c) 0,900 M
d) 1,50 x 10‒2 M

TESTE 168 – (OLIMPÍADA BRASILEIRA DE QUÍMICA) Ouro metálico dissolve em água régia, uma mistura de
ácido clorídrico e ácido nítrico concentrado e, na química do ouro, as seguintes reações são importantes:
Au3+(aq) + 3e- → Au(s) E° red = + 1,498 V
AuCl4 (aq) + 3e → Au(s) + 4 Cl (aq)
- - - E° red = + 1,002 V
Utilizando as semirreações acima e a semirreação:
NO3-(aq) + 4H+(aq) + 3e- → NO(g) + 2 H2O(l) E° red = + 0,96 V
Responda às questões (a), (b), (c) e (d).
a) Dê a equação equilibrada da reação entre o ouro e o ácido nítrico, para formar Au3+ e NO(g) e calcule a fem-
padrão (E°) associada a esta reação. Esta reação é espontânea?
b) Dê a equação da reação entre o ouro e o ácido clorídrico, formando AuCl 4- e H2(g) e calcule a fem-padrão
(E°) associada a esta reação. Esta reação, em condições-padrão, é espontânea?
c) Dê a reação entre o ouro e a água régia para dar AuCl4- e NO(g) e calcule a fem-padrão (E°) associada a esta
reação. Esta reação é espontânea?
d) Utilizando a equação de Nerst, explique a razão da água régia ser capaz de dissolver o ouro.

TESTE 169 – Assuming CO2 to be an ideal gas, calculate the work done by 10 g of CO2 in expanding isothermally and
reversibly from a volume of 5 liters to 10 liters at 27°C. What are q, ∆U and ∆H?

TESTE 170 – For a certain ideal gas Cp = 8,60 cal.mol-1 x K-1. What will be the final volume and temperature when 2
moles of the gas at 20°C and 15 atm are allowed to expand adiabatically abd reversibly to 5 atm pressure?

TESTE 171 – Find W, Q, ∆U and ∆H for the process given in TEST 170 420.

779
1600 FÍSICO-QUÍMICA APLICADA EXERCÍCIOS COMENTADOS - IME – ITA – OLIMPÍADA

TESTE 172 – Ten grams of N2, at 17°C are compressed adiabatically and reversibly from 8 to 5 liters. Calculate the
final temperature and the work done of gas. What are ∆𝑈 and ∆𝐻 for the process?

TESTE 173 – (OLIMPÍADA ARGENTINA DE QUÍMICA)


I. A decomposição do N2O5 (em NO2 e O2) é uma reação de primeira ordem e a 35°C o valor de sua
constante de velocidade é 0,0086 min-1.
a) Calcule o tempo de meia-vida.
b) Se partimos de 4 mol de N2O5 e foram transcorridos 321,6 minutos desde o início da reação de
decomposição, calcule a quantidade de N2O5 que fica sem se decompor ao fim desse período de tempo.
II. Por sua vez, a decomposição do NO2 pode ser representada pela equação:
2 NO2(g) → 2 NO(g) + O2(g)
e para esta, k = 4,87 x 10-3 L.mol-1.s-1 a 65°C e a energia de ativação tem um valor de 1,039 x 105 J.mol-1. Tendo em
conta esta informação:
c) Indique qual é a ordem total da reação de decomposição do NO2.
d) Calcule a constante de velocidade da reação a 100°C.

TESTE 174 – (OLIMPÍADA NORTE NORDESTE DE QUÍMICA) O metanol é um líquido incolor, inflamável e possui
um odor suave na temperatura ambiente, apresentando fórmula molecular CH3OH e massa molecular 32,04 g.mol-1.
Mas seu manuseio requer cuidados por se tratar de uma substância tóxica. É uma das matérias-primas mais
consumidas na indústria química e pode ser obtido pela mistura gasosa de CO(g) e H2(g), de acordo com a equação:
CO(g) + 2 H2(g) → CH3OH(g).
Em um experimento sobre equilíbrio químico, foram transferidos 2,00 mol de CO(g) e 5,00 mol de H2(g) para um frasco
de 10,0 litros. Em seguida, o frasco foi fechado e aquecido a 350°C até ser atingido o equilíbrio, que continha 0,35
mol de CO(g). Com base nas informações acima, determinar:
a) A quantidade de mol de hidrogênio remanescente e de metanol formado.
b) A fração molar de cada componente no equilíbrio e a constante de equilíbrio em função das frações molares.
c) A pressão total no frasco quando foi atingido o equilíbrio e a constante de equilíbrio em função das pressões
parciais.
d) A variação da energia de Gibbs padrão da reação.
e) A constante de equilíbrio em função das concentrações molares.

TESTE 175 – (GRILLO) Calcule o volume em mililitros de uma solução de permanganato de potássio 0,10 mol.L -1
necessário para reagir completamente com 0,010 mol do íon oxalato segundo a seguinte equação iônica não
balanceada.
−2 + +2
MnO−
4(aq) + C2 O4(aq) + H(aq) → Mn(aq) + CO2(g) + H2 O(l)

TESTE 176 – (GRILLO) A partir da equação química não balanceada apresentada a seguir: KMnO4(aq) +
KCl(aq) + H2 SO4(aq) → MnSO4(aq) + K 2 SO4(aq) + H2 O(l) + Cl2 (g) , calcule a massa de cloro gasoso e
também de sulfato de potássio a partir de 200 g de permanganato de potássio.

TESTE 177 – (ITA) Qual das opções abaixo apresenta o material com maior concentração de carbono?
a) Negro de fumo
b) Carvão
c) Alcatrão
d) Piche
e) Óleo diesel

TESTE 178 - (ITA) Qual das opções a seguir apresenta o gráfico que mostra, esquematicamente, a variação da
condutividade elétrica de um metal sólido com a temperatura?

780
1600 FÍSICO-QUÍMICA APLICADA EXERCÍCIOS COMENTADOS - IME – ITA – OLIMPÍADA

TESTE 179 – (ITA) Dois substratos de vidro, do tipo comumente utilizado na fabricação de janelas, foram limpos e
secos. Nas condições ambiente, depositaram-se cuidadosamente uma gota (0,05 mL) de mercúrio sobre um dos
substratos e uma gota (0,05 mL) de água sobre o outro substrato. Considere os líquidos puros.
a) Desenhe o formato da gota de líquido depositada sobre cada um dos substratos.
b) Justifique a razão de eventuais diferenças nos formatos das gotas dos líquidos depositadas sobre cada um
dos substratos de vidro.
c) Qual a influência do volume do líquido no formato das gotas depositadas sobre os substratos?

TESTE 180 – (OLIMPÍADA BRASILEIRA DE QUÍMICA) Dentre os principais fatores que influenciam na produção
agropecuária, podemos citar: o clima, o material genético, o manejo de pragas, as doenças e plantas daninhas e o
manejo do solo, com especial ênfase no manejo químico como base para a nutrição das plantas. Em razão da
produção de alimentos em escala cada vez maior, os nutrientes do solo que dão vida às plantas vão se esgotando.
Para supri-los, produtos químicos conhecidos como fertilizantes são incorporados à terra em quantidades crescentes.
Para correção da acidez do solo, o procedimento de rotina é a calagem através da incorporação de um óxido básico.
É correto afirmar que esse óxido básico pode ser:
a) MgO2
b) CaO
c) SO2
d) NaO
e) CO

TESTE 181 - (OLIMPÍADA DE QUÍMICA DO RIO DE JANEIRO - MODIFICADA) O DNA é o carreador primário da
informação genética em organismos vivos. O DNA perde a sua atividade pelo desenrolamento da sua estrutura de
dupla hélice. Este é um processo de primeira ordem com energia de ativação de 400 kJ.mol -1. A 37°C (temperatura
fisiológica), a meia-vida para este desenrolamento é de 1045 minutos. Estime o tempo de meia-vida, em minutos, para
o desenrolamento à uma temperatura crítica de 44°C no corpo humano.

TESTE 182 – (GRILLO) Considere as seguintes equações termoquímicas abaixo, todas elas sendo processadas a
uma temperatura de 25°C:
Equação Química I: Fe2O3(s) + 3 C(grafite) → 2 Fe(s) + 3 CO(g) ΔH° = + 117,3 kcal
Equação Química II: C(grafite) + O2(g) → CO2(g) ΔH° = - 94,0 kcal
Equação Química III: CO(g) + ½ O2(g) → CO2(g) ΔH° = - 67,6 kcal
A partir destes dados apresentados, calcule a entalpia padrão de formação do trióxido de diferro no estado sólido.

TESTE 183 – (GRILLO) Durante o experimento que são realizados em células galvânicas, é possível a determinação
do potencial hidrogeniônico (pH) de uma determinada solução desconhecida. As semirreações de redução estão
apresentadas logo a seguir:
Zn2+(aq) + 2e- → Zn(s) Eº = - 0,76 V
H+(aq) + e- → ½ H2(g) Eº = + 0,00 V
A partir desta informação, responda os seguintes itens:
a) Escreva a reação global da célula.
b) Calcule o valor da constante de equilíbrio para a reação global, a 25oC.
c) Calcule o valor de ∆G° para a reação global determinada no item a.

781
1600 FÍSICO-QUÍMICA APLICADA EXERCÍCIOS COMENTADOS - IME – ITA – OLIMPÍADA

d) Calcule o pH da solução desconhecida. Considere que a concentração de Zn2+(aq) e a pressão parcial de H2(g)
foram mantidas nas condições padrões, a 25oC e que o valor de E, nessas condições, é 0,65 V.

TESTE 184 – (IME) A reação de Sabatier-Sanderens consiste na hidrogenação catalítica de alcenos ou de alcinos
com níquel para a obtenção de alcanos. Considerando a reação de hidrogenação do acetileno, um engenheiro químico
obteve os resultados abaixo:
Tempo (min) [Acetileno] (mol.L-1) [Hidrogênio] (mol.L-1) [Etano] (mol.L-1)
0 50 60 0
4 38 36 12
6 35 30 15
10 30 20 20
A partir dessas informações, determine:
a) A velocidade média da reação no período de 4 (quatro) a 6 (seis) minutos;
b) A relação entre a velocidade média de consumo de acetileno e a velocidade média de consumo do
hidrogênio;
c) O efeito do aumento da temperatura de reação na constante de velocidade, considerando a equação de
Arrhenius.

TESTE 185 – (GRILLO) A um recipiente de volume não variável, se adiciona certa quantidade de pentacloreto de
fósforo se decompõe por aquecimento até atingir o equilíbrio químico, conforme a equação a seguir: PCl5(g) ⇌ PCl3(g)
+ Cl2(g). A 200°C, a constante de equilíbrio em função das pressões parciais seja igual a Kp = 4,0. A partir das
informações apresentadas, determine o valor da densidade da mistura gasosa após o equilíbrio, considerando que a
pressão da mistura reacional do sistema seja igual a uma atmosfera.

TESTE 186 – Determine a expressão matemática do trabalho quando o gás sofre uma transformação isotérmica e
reversível.

TESTE 187 – Deduza a equação que relaciona a dependência entre o volume e a temperatura para uma
transformação adiabática reversível.

TESTE 188 – Deduza a equação que relaciona a dependência entre a pressão e o volume para uma transformação
adiabática reversível.

TESTE 189 – As substâncias X e Y reagem para formar W e Z, sabendo que a reação é de segunda ordem em X. A
constante de velocidade em 300 K é 0,500 L.mol-1.min-1. Qual é a meia-vida de X (em min) quando 4,0 x 10-2 mol.L-1
de X é misturado com excesso de Y?
a) 10 min
b) 20 min
c) 30 min
d) 40 min
e) 50 min

TESTE 190 – (MONBUKAGAKUSHO, COLLEGE OF TECHNOLOGY STUDENTS E UNDERGRADUATED


STUDENTS) The density of the aqueous solution in which 11,0 g of sodium hydroxide is dissolved in 87,0 g of water
is 1,142 x 10³ kg.m-3. Answer the following questions:
(a) Calculate the mole fraction of the water.
1) 0,063
2) 0,937
3) 0,036
4) 0,964
5) 0,056

782
1600 FÍSICO-QUÍMICA APLICADA EXERCÍCIOS COMENTADOS - IME – ITA – OLIMPÍADA

TESTE 191 – (MONBUKAGAKUSHO, COLLEGE OF TECHNOLOGY STUDENTS E UNDERGRADUATED


STUDENTS) Calculate the concentration of the sodium hydroxide solution.
1) 1,17 mol.L-1
2) 2,78 mol.L-1
3) 3,71 mol.L-1
4) 4,67 mol.L-1
5) 5,89 mol.L-1

TESTE 192 – Uma reação em fase gasosa do tipo A(g) + B(g) → C(g) em equilíbrio dentro de um recipiente de volume
constante apresenta o seguinte comportamento:
Experimento Pressão total Temperatura
1 2,5 atm 27ºC
2 4,0 atm 177ºC
Sobre este equilíbrio, é correto afirmar que:
a) O valor de Kc diminui com o aumento de temperatura, configurando uma reação exotérmica no sentido
direto.
b) O valor de Kc aumenta com o aumento de temperatura, configurando uma reação exotérmica no sentido
direto.
c) O valor de Kc diminui com o aumento de temperatura, configurando uma reação endotérmica no sentido
direto.
d) O valor de Kc aumenta com o aumento de temperatura, configurando uma reação endotérmica no sentido
direto.
e) O valor de Kc não varia com o aumento de temperatura.

TESTE 193 – Um motor térmico funciona segundo o ciclo de Carnot. A temperatura da fonte quente é de
aproximadamente igual a 323°C e a da fonte fria vale 25°C. A partir desta informação, determine o rendimento desse
motor.

TESTE 194 – (OLIMPÍADA DE QUÍMICA DO RIO DE JANEIRO) Marque a sentença correta:


a) a energia interna de um gás ideal depende da temperatura, mas não depende da pressão;
b) a entropia mede a idade do universo;
c) durante reações químicas espontâneas há um aumento de entropia (não existe exceção);
d) a entropia de substâncias elementares é igual a zero por definição;
e) as reações químicas espontâneas ocorrem sempre com liberação de energia.

TESTE 195 – (OLIMPÍADA BRASILEIRA DE QUÍMICA) Assinale a opção que apresenta valores de ΔH e ΔS para
uma reação que ocorre espontaneamente a qualquer temperatura:
a) ΔH < 0 e ΔS < 0
b) ΔH > 0 e ΔS < 0
c) ΔH > 0 e ΔS = 0
d) ΔH = 0 e ΔS < 0
e) ΔH < 0 e ΔS > 0

TESTE 196 - (ITA) Após inalar ar na superfície, uma pessoa mergulha até uma profundidade de 200 m, em apneia,
sem exalar. Desconsiderando as trocas gasosas que ocorrem nos alvéolos pulmonares, calcule a pressão parcial do
nitrogênio e do oxigênio do ar contido no pulmão do mergulhador.

TESTE 197 – (ITA) A figura a seguir representa o resultado de dois experimentos diferentes (I) e (II) realizados para
uma mesma reação química genérica (reagentes → produtos). As áreas hachuradas sob as curvas representam o
número de partículas reagentes com energia cinética igual ou maior que a energia de ativação da reação (Eat).

783
1600 FÍSICO-QUÍMICA APLICADA EXERCÍCIOS COMENTADOS - IME – ITA – OLIMPÍADA

Baseado nas informações apresentadas nesta figura, é CORRETO afirmar que:


a) a constante de equilíbrio da reação nas condições do experimento I é igual à da reação
nas condições do experimento II
b) a velocidade medida para a reação nas condições do experimento I é maior que a
medida nas condições do experimento II
c) a temperatura do experimento I é menor que a temperatura do experimento II
d) a constante de velocidade medida nas condições do experimento I é igual à medida
nas condições do experimento II
e) a energia cinética média das partículas, medida nas condições do experimento I, é maior que a medida
nas condições do experimento II

TESTE 198– (ITA) Certa reação química exotérmica ocorre, em dada temperatura e pressão, em duas etapas
representadas pela seguinte seqüência de equações químicas:
A+B→E+F+G
E+F+G→C+D
Represente, em um único gráfico, como varia a energia potencial do sistema em transformação (ordenada) com a
coordenada da reação (abscissa), mostrando claramente a variação de entalpia da reação, a energia de ativação
envolvida em cada uma das etapas da reação e qual destas apresenta a menor energia de ativação. Neste mesmo
gráfico, mostre como a energia potencial do sistema em transformação varia com a coordenada da reação, quando
um catalisador é adicionado ao sistema reagente. Considere que somente a etapa mais lenta da reação é influenciada
pela presença do catalisador.

TESTE 199 – (GRILLO) Considere o seguinte processo químico que se observa na produção de sulfato de potássio,
sulfato de manganês II, sulfato de ferro III e água, conforme a equação química não-balanceada apresentada a seguir:
KMnO4 + FeSO4 + H2SO4 → K2SO4 + MnSO4 + Fe2(SO4)3 + H2O. Considere que foram colocados para reagir um
quilograma para cada reagente. A partir desta informação, determine a massa de todos os produtos formados.

TESTE 200 – Determine a constante de equilíbrio para a reação global apresentada a seguir a 250C,
2 Fe3+(aq) + 2I-(aq) → 2Fe2+(aq) + I2(g).

Informações para a resolução do problema:


Fe3+(aq) + e- → Fe2+(aq) E0 = + 0,77V
I2(g) + 2e- → 2I-(aq) E0 = + 0,54V

TESTE 201 – Responda aos seguintes itens apresentados abaixo.

a) Calcule o potencial para a seguinte célula eletroquímica: Cu/Cu2+(aq) (0,005 mol.L1) // Zn2+(aq) (0,002 mol.L-
1) / Zn0 a 470C, considerando que os eletrodos se comportam idealmente. Informações para a resolução do problema:

E0 [Cu2+/Cu] = + 0,34V
E0 [Zn2+/Zn] = - 0,76V
b) Calcule a intensidade da corrente elétrica que deve ser utilizada para depositar 4,25 x 10-5 kg de cobre, de
uma solução de sulfato de cobre, no tempo, no tempo de 4 minutos e 40 segundos.

Teste 202 – (GRILLO) Considere a reação famosa de síntese do gás amoníaco, apresentada a partir da equação

784
1600 FÍSICO-QUÍMICA APLICADA EXERCÍCIOS COMENTADOS - IME – ITA – OLIMPÍADA

química a seguir: N2(g) + 3 H2(g) ⇌ 2 NH3(g). A partir das informações termodinâmicas apresentadas abaixo, calcule o
que se pede nos seguintes itens:
a) A variação da entalpia padrão;
b) A variação de energia livre de Gibbs;
c) A constante de equilíbrio a 25°C e 1 atm;
d) A constante de equilíbrio a 400°C e 1 atm.
Informação para a resolução do problema: calor de formação do gás amoníaco (kJ.mol-1): - 46,1; entropia-padrão
(J.mol-1.K-1): N2(g) = 191; H2(g) = 130 e NH3(g) = 192.

TESTE 203 – Realize o balanceamento por oxirredução das seguintes equações abaixo:

a) KMnO4 + HCl → KCl + MnCl2 + Cl2 + H2 O


b) SnCl2 + FeCl3 → SnCl4 + FeCl2
c) Cl2 + NaOH → NaCl + NaClO3 + H2 O
d) P4 + HNO3 + H2 O → NO + H3 PO4
e) C + HNO3 → CO2 + NO2 + H2 O
f) Ag + HNO3 → AgNO3 + H2 O + NO
g) Br2 + KOH → KBrO3 + KBr + H2 O
h) KIO3 + KI + HCl → I2 + KCl + H2 O
i) HBrO3 + SO2 + H2 O → Br2 + H2 SO4
j) H2 C2 O4 + KMnO4 → CO2 + MnO + K 2 O + H2 O
k) MnO2 + HBr → MnBr2 + Br2 + H2 O
l) MnO− 4 + Fe
+2
+ H + → Mn+2 + Fe+3 + H2 O
m) HgS + HNO3 → Hg(NO3 )2 + S + NO + H2 O
n) Ag 2 S + Al + NaOH + H2 O → Ag + H2 S + NaAlO2
o) Ca3 (PO4 )2 + SiO2 + C → CaSiO3 + CO + P4
p) CaC2 O4 + KMnO4 + H2 SO4 → CaSO4 + KSO4 + MnSO4 + H2 O + CO2
q) H2 𝑆 + Br2 + H2 O → H2 SO4 + HBr
r) Na2 Cr2 O7 + SO2 + H2 O → Cr(OH)SO4 + Na2 SO4
s) HNO3 + I2 → HIO3 + NO2 + H2 O

TESTE 204 – (GRILLO) Calcule a quantidade de calor necessária para a transformação de 40 g de gelo a -10°C em
vapor a 120°C. Informações para a resolução do problema: Calor específico do gelo = 0,50 cal.g -1.°C-1; Calor
específico da água = 1,0 cal.g-1.°C-1; Calor específico do vapor d´água = 0,50 cal.g-1.°C-1; Calor de fusão do gelo = 80
cal.g-1; Calor de evaporação da água = 540 cal.g-1.

TESTE 205 – (GRILLO) Uma amostra de amônia com uma massa de aproximadamente igual a 3,50 gramas a 27°C
se expande reversivelmente e adiabaticamente de 1500 mL até 4,50 L. Calcule a pressão final e também da
temperatura final, sabendo que o coeficiente de Poisson é igual a 1,35.

TESTE 206 – (GRILLO) Considere um mol de um gás monoatômico que apresenta pressão constante igual a 200
kPa. Sabendo que a temperatura inicial é igual a 100°C e a temperatura final sendo igual a 25°C, calcule W; Q; ∆U
e ∆H.

TESTE 207 – (GRILLO) Considere uma reação de decomposição do tipo: A(g) → B(g) + C(g), com velocidade específica
igual a 2,28 x 10-4 h-1, a uma temperatura de 127°C.
a) Explique de forma clara e sucinta, qual a ordem de reação para esta reação gasosa;
b) Calcule o tempo de meia-vida;
c) Determine a pressão após cinco horas, considerando que a pressão inicial seja igual a 625 torr.

TESTE 208 – (GRILLO) Considere um gás ideal monoatômico que apresenta 1,50 mol. Esta massa gasosa é levada
de um volume de 3000 L para 1200 L, através de um processo compressível e isotérmico a temperatura de 300 K. A
partir das informações apresentadas, calcule.
a) O trabalho realizado.
b) Qual é a variação de energia interna durante o processo?

785
1600 FÍSICO-QUÍMICA APLICADA EXERCÍCIOS COMENTADOS - IME – ITA – OLIMPÍADA

c) Qual é a variação de entalpia durante o processo?


d) Qual o calor envolvido no processo? O sistema absorve ou libera calor?

TESTE 209 – (CONCURSO PARA DOCENTE – IFRN) Numa aula de físico-química, um grupo de alunos foi orientado
a montar uma pilha em solução aquosa. O grupo decidiu fazer isso usando eletrodos de prata (Ag+ + e- → Ag, E° = +
0,7994 V) e de níquel (Ni+2 + 2e- → Ni, E° = - 0,25 V). Sobre a pilha a ser montada pelos alunos e, com base nos
conhecimentos de eletroquímica, faça o que se pede a seguir.
a) Escreva a equação global da pilha.
b) Determine o potencial-padrão E° da pilha e indique qual eletrodo será o cátodo e qual será o ânodo.
c) Determine o potencial E da pilha, quando as concentrações dos íons prata e níquel foram 0,20 mol.L -1 e
0,01 mol.L , respectivamente. Dado: log 2 = 0,301.
-1

TESTE 210 – (OLIMPÍADA BRASILEIRA DE QUÍMICA) A reação de dióxido de enxofre com cloro, produz cloreto de
tionila e óxido de dicloro, conforme a equação: SO2(g) + 2 Cl2(g) → SOCl2(g) + Cl2O(g).
a) Escreva as configurações eletrônicas de todos os elementos químicos envolvidos nesta reação e
determine, o grupo e o período da tabela periódica em que cada um deles está localizado.
b) Usando as configurações do item (a) indique o conjunto de número quânticos do último elétron de cada
um destes elementos.
c) Qual destes elementos apresenta a menor energia de ionização? Justifique.
d) Quem apresenta menor raio, o íon sulfeto ou o átomo de enxofre? Justifique.
e) Quantos gramas de Cl2 são necessários para a produção de 595 g de SOCl2?
f) Partindo-se de 10 g de SO2 e 20 g de Cl2, que massa de SOCl2 pode ser obtida, considerando-se um
rendimento de 80%?
g) A reação acima, requer 164,6 kJ por mol de SOCl2 produzido. Usando esta informação e a tabela abaixo,
calcule o calor de formação do SOCl2.
Composto ∆H°f (kJ/mol)
Cl2O 80,3
SO2 - 296,8

TESTE 211 – (U. S. NATIONAL CHEMISTRY OLYMPIAD) A galvanic cell is based on the half-reactions:
Cr3+ + 3e– → Cr E˚ = –0,744 V
Ni2+ + 2e– → Ni E˚ = –0,236 V
a) Write the balanced equation for the overall cell reaction.
b) State which electrode increases in mass as the cell operates. Explain your answer.
c) Calculate E˚cell
d) Determine the value of ∆G˚ for the cell reaction at 25˚C.
e) Calculate the value of K for the cell reaction at 25˚C.
f) Find the voltage of the cell at 25˚C if [Cr3+] and [Ni2+] are both changed to 0,010 M.

TESTE 212 – (OLIMPÍADA DE QUÍMICA DO RIO DE JANEIRO) Foram colocados 112 g de SO3 num recipiente com
a capacidade de 5,00 L. A temperatura foi elevada até 500ºC e mantida constante neste valor. Após atingir o equilíbrio,
constatou-se que a pressão total do sistema era de 23,5 atm. Considerando o equilíbrio estabelecido como 2SO3(g) ⇄
2 SO2(g) + O2(g). Determine:
a) a pressão inicial do sistema.
b) a constante de equilíbrio em termos de pressão, Kp.
c) a constante de equilíbrio em termos de concentração, Kc.
d) em que sentido o equilíbrio se deslocará se metade da concentração de O2 do equilíbrio for removida.

786
1600 FÍSICO-QUÍMICA APLICADA EXERCÍCIOS COMENTADOS - IME – ITA – OLIMPÍADA

TESTE 213 – (OLIMPÍADA NORTE-NORDESTE DE QUÍMICA) Em um mistura dos gases N2O4 e NO2 em equilíbrio,
representado pela equação química abaixo, à temperatura de 0ºC e pressão de 1 atm, as pressões parciais desses
gases são, respectivamente, 0,8 atm e 0,2 atm.
N2O4(g) ⇄ 2 NO2(g).
a) Calcule a constante de equilíbrio expressa em pressões, Kp.
b) Calcule a constante de equilíbrio expressa em concentrações, Kc.
c) Calcule o ΔHº para o equilíbrio acima, a partir dos dados de entalpias padrões de reação a 273,15 K, dados a
seguir:
N2(g) + 2 O2(g) → 2 NO2(g) ∆H° = + 78,31 kJ.mol-1
N2(g) + 2 O2(g) → N2O4(g) ∆H° = + 9,67 kJ.mol-1
d) Um aumento da temperatura levará a uma maior ou menor dissociação de N2O4?

TESTE 214 – (OLIMPÍADA BRASILEIRA DE QUÍMICA) O zinco (do alemão Zink; Zn) é um elemento químico
essencial para o nosso organismo, pois é responsável por inúmeras funções, como a síntese de proteínas, o
funcionamento de alguns hormônios, o bom funcionamento do sistema imunológico e também do reprodutor. O zinco
metálico pode ser obtido a partir de óxido de zinco, ZnO, pela reação a alta temperatura com o monóxido de carbono,
CO. ZnO(s) + CO(g) → Zn(s) + CO2(g)
O monóxido de carbono é obtido a partir de carbono. 2C(s) + O2(g) → 2CO(g)
Qual é a quantidade máxima de zinco (em gramas) que pode ser obtido a partir de uma amostra de 75,0 g de óxido
de zinco com pureza de 87 % e 10,0 g de carbono?
a) 52,4
b) 35,3
c) 54,4
d) 36,6
e) 65,3

TESTE 215 – (OLIMPÍADA DE QUÍMICA DO RIO DE JANEIRO) O ácido sulfúrico é um dos responsáveis pela
formação da chuva ácida. O primeiro equilíbrio envolvido na formação desse ácido na água da chuva é representado
pela equação: SO2(g) + O2(g) ⇌ SO3(g). O equilíbrio foi estabelecido em determinadas condições e está representado
no gráfico abaixo, com as concentrações no eixo das ordenadas, em mol/L, e o tempo na abscissa, em segundos.

Pela análise do gráfico, o valor numérico da constante de equilíbrio para esse sistema é de:

a) 0,66
b) 0,75
c) 1,33
d) 1,50
e) 3,00

TESTE 216 – (OLIMPÍADA DE QUÍMICA DO RIO DE JANEIRO) Em um projeto de feira de ciências, alunos montaram
diversas pilhas e determinaram as diferenças de potencial (ddp) geradas. Todas as pilhas seguiram o mesmo padrão
de montagem, ilustrado na figura abaixo, onde cada metal foi mergulhado numa solução de um sal do mesmo, em
concentração de 1,0 mol/L. A temperatura em que os experimentos foram realizados foi de 25 oC. Ao final de cada
montagem, era indicado que metal funcionava como catodo, que metal funcionava como anodo e a ddp gerada. Com
base nos dados fornecidos, a opção correta referente às pilhas montadas é:

787
1600 FÍSICO-QUÍMICA APLICADA EXERCÍCIOS COMENTADOS - IME – ITA – OLIMPÍADA

a) Catodo = alumínio; Anodo = prata; ddp = + 2,47V


b) Catodo = cobre; Anodo = alumínio; ddp = + 1,33V
c) Catodo = alumínio; Anodo = zinco; ddp = + 2,43V
d) Catodo = chumbo; Anodo = zinco; ddp = + 0,89V
e) Catodo = prata; Anodo = cobre; ddp = + 0,46V

TESTE 217 – (OLIMPÍADA MINEIRA DE QUÍMICA) Muitas vezes verifica-se que a velocidade de uma reação é
proporcional às concentrações dos reagentes elevadas a certas potências. Por exemplo, a velocidade de uma reação
pode ser proporcional à concentração, em mol.L⁻¹, do reagente A elevada à primeira potência, de modo que
escrevemos
v = k x [A]
A constante k é a constante de velocidade da reação, e não depende das concentrações, mas apenas da temperatura.
Uma equação desse tipo, determinada experimentalmente, é uma lei de velocidade da reação. No caso particular do
exemplo dado, quando a velocidade da reação depende apenas da concentração de um reagente elevada à primeira
potência, dizemos que a reação é de primeira ordem. A aplicação prática da lei de velocidade é que, uma vez
conhecida a lei de velocidade e o valor da constante de velocidade, podemos prever a velocidade da reação a partir
da composição da mistura de reação. Além disso, podemos prever a composição da mistura de reação em qualquer
instante do tempo. No caso de uma reação de primeira ordem, a dependência da concentração do reagente A com o
tempo é dada pela seguinte equação
[A] = [A]0 x e−kt
em que [A]0 é a concentração inicial de A (quando t = 0). Com base nas informações acima, identifique entre as
alternativas aquela que contém o gráfico que representa corretamente como varia a concentração do reagente A ao
longo do tempo:

788
1600 FÍSICO-QUÍMICA APLICADA EXERCÍCIOS COMENTADOS - IME – ITA – OLIMPÍADA

TESTE 218 – (OLIMPÍADA MINEIRA DE QUÍMICA) Tem-se um recipiente com N2 puro e outro com He puro. Volumes
e pressões iniciais estão assinalados no esquema seguinte.

Abrindo-se a torneira que separa os gases e, mantendo-se a temperatura, a pressão interna (em atm) se estabiliza
no valor de:
a) 6,00
b) 4,50
c) 3,00
d) 2,60

TESTE 219 – (OLIMPÍADA MINEIRA DE QUÍMICA) Sabe-se que o princípio de Le Chatelier afirma que “quando uma
perturbação externa é imposta a um sistema químico em equilíbrio, este se deslocará de forma a minimizar tal
perturbação”. Em relação ao processo químico representado a seguir, são feitas algumas afirmações. 2 NO 2(g) ⇌
N2O4(g) ∆H < 0
I) Aumentando-se a concentração de NO2, o equilíbrio é deslocado no sentido dos produtos.
II) Aumentando-se a concentração de N2O4, o equilíbrio é deslocado no sentido dos reagentes.
III) Aumentando-se a temperatura, a reação direta será favorecida.
IV) Aumentando-se a pressão total do sistema, a reação inversa será favorecida.
V) O valor da constante de equilíbrio não se altera com a temperatura.
O número de afirmações corretas é:
a) 1
b) 2
c) 3
d) 4

TESTE 220 – (OLIMPÍADA MINEIRA DE QUÍMICA) Para a equação abaixo, está faltando o balanceamento. Faça-o
usando o método que julgar mais adequado ou mais simples.

H2O(ℓ) + HCℓO4(ℓ) + P4O10(s) ⇌ H3PO4(ℓ) + Cℓ2O7(s)

Após feito o balanceamento, escolha a opção que corresponde aos coeficientes de cada uma das substâncias, na
ordem em que aparecem na equação:

a) 3 - 1 - 2 - 6 -2
b) 3 - 1 - 1- 2 - 1
c) 10 - 2 - 1 - 8 - 1
d) 5 - 2 - 1- 4 -1

TESTE 221 – (OLIMPÍADA MINEIRA DE QUÍMICA) A primeira teoria sobre a natureza química dos ácidos e das
bases foi proposta por Arrhenius, em 1887. Segundo Arrhenius, ácidos quando dissolvidos em água aumentavam a
concentração de íons H+ e as bases quando dissolvidas em água aumentavam a concentração de íons hidroxila.
Alguns anos mais tarde (1923), Brönsted e Lowry expandiram os conceitos de Arrhenius e definiram ácidos como
espécies doadoras de prótons e bases, espécies aceptoras de prótons. Aplicando-se o conceito ácido-base de
Brönsted-Lowry às reações abaixo equacionadas, verifica-se que:
1a NH3 + H2O ⇌ NH4+ + OH-
2a HNO3 + H2O ⇌ NO3- + H3O+
3 a HNO3 + H2SO4 ⇌ H2NO3+ + HSO4-
As espécies que funcionaram tanto como ácido quanto como base de Brönsted-Lowry foram:

789
1600 FÍSICO-QUÍMICA APLICADA EXERCÍCIOS COMENTADOS - IME – ITA – OLIMPÍADA

a) NH3 e H2O
b) H2O e HNO3
c) HNO3 e H2SO4
d) NH4+ e H2NO3+

TESTE 222 – (MESTRE JOÃO ROBERTO DA PACIÊNCIA NABUCO) Calcular a pressão máxima de vapor de uma
solução de cloreto de magnésio contendo 19 gramas do mesmo em 360 gramas de água, a 28°C. A pressão máxima
de vapor da água nesta temperatura é igual a 32,0 mmHg e a solução de cloreto de magnésio se encontra 80%
dissociado.

TESTE 223 – (MESTRE JOÃO ROBERTO DA PACIÊNCIA NABUCO - MODIFICADA) Considere o seguinte
equilíbrio químico descrito a seguir: 2 SO2(g) + O2(g) + calor ⇌ 2 SO3(g), onde os reagentes e produtos são
gasosos. Indique a concentração em equilíbrio, quando:
a) aumentando a temperatura;
b) adicionando um catalisador;
c) diminuindo a concentração de O2(g);
d) introduzindo trióxido de enxofre em excesso no recipiente;
e) aumentando a pressão total;
f) diminuindo a temperatura;
g) aumentando a concentração de dióxido de enxofre.

TESTE 224 – (GRILLO) Uma amostra de um gás ideal com 2,0 mol e diatômico apresenta as seguintes características
em seu estado inicial: pinicial = 105 kPa e Tinicial = 276 K, sendo aquecida reversivelmente até 390 K a volume constante.
A partir das informações apresentadas, calcule a pressão final, a energia interna, entalpia, quantidade de calor,
trabalho e o volume inicial.
TESTE 225 – (OLIMPÍADA DE QUÍMICA DO RIO DE JANEIRO) O etilenoglicol (etan-1,2-diol) é um líquido usado
como aditivo na água do sistema de arrefecimento de veículos para evitar que o fluido do radiador ferva em dias
quentes ou congele em dias frios. Qual volume do anticongelante etilenoglicol deve ser adicionado ao radiador de um
carro contendo 5 L de água para que sua temperatura de congelamento seja de 10ºC inferior à da água pura? Dados:
densidade da água: 1 g/mL; densidade do etilenoglicol: 1,11 g/mL; constante crioscópica da água: 1,86ºC.kg/mol;
temperatura de congelamento da água pura: 0ºC.
a) 0,3 L
b) 0,8 L
c) 1,2 L
d) 1,5 L
e) 1,8 L

TESTE 226 – (OLIMPÍADA DE QUÍMICA DO RIO DE JANEIRO) Para determinar a concentração molal de uma
solução aquosa de nitrato de magnésio, um técnico transferiu 200 mL da solução para um béquer e o resfriou até o
congelamento da solução. O experimento foi realizado quatro vezes por razões estatísticas. O técnico definiu a
temperatura de início de congelamento da solução como sendo –2,80oC. Considerando os dados fornecidos no
problema, determine a concentração encontrada pelo técnico. Dados: Temperatura de congelamento (H2O) = 0,0 oC;
Constante crioscópica (H2O) = 1,86oC⋅kg⋅mol–1; Grau de dissociação do nitrato de magnésio = 100%.
a) 0,5 molal
b) 1,0 molal
c) 1,5 molal
d) 2,0 molal
e) 2,5 molal

790
1600 FÍSICO-QUÍMICA APLICADA EXERCÍCIOS COMENTADOS - IME – ITA – OLIMPÍADA

TESTE 227 – (OLIMPÍADA DE QUÍMICA DO RIO DE JANEIRO) É possível determinar o calor latente de vaporização,
LV, de um determinado solvente a partir de sua constante ebulioscópica, Ke (Ke = (R.T2.LV–1)/1000), onde T é a
temperatura de ebulição do solvente puro, em Kelvin, e R é a constante geral dos gases, 2,0 cal.K–1.mol–1. Em um
experimento realizado com este intuito, determinou-se o ponto de ebulição de 200 mL de água pura (d = 1,0 g.mL–1),
encontrando-se 100,0oC. Posteriormente, repetiu-se o experimento, adicionando 70,85 g de nitrato de cálcio aos 200
mL de água. O grau de dissociação do sal foi de 90% e o novo ponto de ebulição foi determinado como 103,14 oC.
Determine o calor latente de vaporização da água.
a) 103 cal.g–1
b) 192 cal.g–1
c) 287 cal.g–1
d) 536 cal.g–1
e) 580 cal.g–1

TESTE 228 – (OLIMPÍADA DE QUÍMICA DO RIO DE JANEIRO) O isótopo 131 do iodo é largamente empregado na
medicina. A radiação emitida por este radioisótopo é utilizada em terapias no tratamento, por exemplo, de
hipertiroidismo, do cancro da tiroideia e doença de Plummer. Seu período de meia vida é de 193 horas. A partir de
uma massa inicial de 1,0 g, quanto tempo levaria para que a massa deste radioisótopo se reduzisse a 0,3 g?
a) 1,74 h
b) 193 h
c) 396 h
d) 336 h
e) 2,89 h

TESTE 229 – (OLIMPÍADA DE QUÍMICA DO RIO DE JANEIRO) Em tese, toda reação química está sujeita à condição
de equilíbrio, na qual:
a) a energia de ativação da reação direta é igual a zero.
b) as reações direta e inversa são totalmente inibidas.
c) o sistema reacional está deslocado para o sentido dos produtos.
d) as concentrações dos reagentes e dos produtos se igualam.
e) as velocidades das reações direta e inversa são equivalentes.

TESTE 230 – (OLIMPÍADA NORTE – NORDESTE DE QUÍMICA) Uma célula eletroquímica é constituída por:
Eletrodo A - fio de platina que mergulha numa solução que contém íons Fe+3 e Fe+2, ambos com concentração 1
mol/L.
Eletrodo B - vareta de tálio mergulhada numa solução 1 mol/L de Tl+1.
Sabendo que os potenciais-padrão referentes às semirreações:
Tl+1 + e- → Tl
Fe+3 + e- → Fe+2
são, respectivamente, - 0,34 V e + 0,77 V, faça o que se pede:
a) Mostre as semirreações que ocorrem no cátodo e no ânodo.
b) Escreva a equação da reação que ocorre na célula.
c) Calcule o potencial da célula.
d) O que ocorrerá com o potencial da célula se diminuirmos a concentração de Tl+1 no eletrodo B?

TESTE 231 – (OLIMPÍADA DE QUÍMICA DO DISTRITO FEDERAL) Reações de oxirredução fazem parte do nosso
dia-a-dia. As pilhas, por exemplo, convertem energia química em elétrica através de reações redox espontâneas. A
concentração das espécies envolvidas nas semi-reações da pilha afetam o potencial da mesma, que pode ser medido
através da equação de Nernst para reações do tipo:

0,0592 [C]c x [D]d


aA + bB ⇌ cC + dD E = E0 − n
x log [A]a x [B]b
em que E é o potencial real, E0 o potencial padrão e n o número de mols de elétrons transferidos na reação.
onsiderando a reação não-balanceada abaixo, resolva os itens que se seguem.
MnO4-(aq) + Br-(aq) + H+(aq) ⇌ Mn2+(aq) + Br2(l) + H2O(l) E0 = 0,42 V
a) Faça o balanceamento de carga e massa para a reação acima.
b) Dê os agentes oxidante e o redutor para esta reação.

791
1600 FÍSICO-QUÍMICA APLICADA EXERCÍCIOS COMENTADOS - IME – ITA – OLIMPÍADA

c) Escreva a equação que representa o potencial da reação em função da concentração das espécies
presentes em solução.
Teste 232 – (OLIMPÍADA BRASILEIRA DE QUÍMICA JÚNIOR) Médicos veterinários substituíram parte do bico
danificado de uma ave por uma prótese metálica feita por uma impressora 3D. O material usado é leve, forte, tem
baixa densidade, apresenta excelente resistência à corrosão e baixa condutividade térmica e elétrica Adaptado de:
http://noticias.uol.com.br/album/2016/01/04/bichosque-foram-noticia. htm?abrefoto=137
A prótese é feita de:
a) Au
b) Fe
c) Pb
d) Ti

TESTE 233 - A pressão osmótica de uma solução contendo 32 g.L-1 de um soluto é 8,20 atm a 27°C. Qual a pressão
osmótica a 47oC de uma solução contendo 10 g.L-1 do mesmo soluto no mesmo solvente?

TESTE 234 – (OLIMPÍADA DE QUÍMICA DO DISTRITO FEDERAL) O eletrólito de uma bateria de carro é uma
solução aquosa 3,95 M de ácido sulfúrico (H2SO4). Sabendo que a densidade desta solução é 1,30 g.mL-1, a
molalidade dessa solução, em molal, é:
a) 3,04
b) 3,95
c) 4,33
d) 4,87
e) 5,14

TESTE 235 – (GRILLO) Considere uma solução de ácido sulfúrico que apresenta densidade de 1,75 g/cm³ e 68% em
massa. A partir das informações obtidas, calcule a concentração da quantidade de matéria, a normalidade e a
molalidade.

TESTE 236 – (ENADE) Uma indústria produz superfosfato com 92% de pureza, a partir de fosfato de cálcio e ácido
sulfúrico concentrado, conforme equação química a seguir. Ca3 (PO4 )2 + 2 H2 SO4 → 2 CaSO4 + Ca(H2 PO4 )2.
Suponha que, em um teste, foram adicionados a um misturador 500 kg de fostato de cálcio e 260 kg de ácido sulfúrico,
produzindo 280 kg de superfosfato Ca(H2 PO4 )2. Assinale a opção que apresenta o reagente limitante e sua
concentração em kmol.
a) Fosfato de cálcio, 1,48 kmol
b) Fosfato de cálcio, 2,65 kmol
c) Ácido sulfúrico, 1,20 kmol
d) Ácido sulfúrico, 1,48 kmol
e) Ácido sulfúrico, 2,65 kmol

TESTE 237 – (MESTRE JOÃO ROBERTO DA PACIÊNCIA NABUCO) Uma massa de 2,55 g de alumínio são
dissolvidos em 450 mL de uma solução aquosa 0,50 mol.L-1 em ácido clorídrico. Determine o volume de hidrogênio a
0,50 atm e 37°C.

TESTE 238 - (MESTRE JOÃO ROBERTO DA PACIÊNCIA NABUCO) Uma determinada amostra constituída de 250
g de cloreto de potássio é dissolvida em 725 g de água, formando uma solução com um volume de 525 ml a 47ºC.
Para o cloreto de potássio nesta solução, calcule:
a) Concentração em g.L-1.
b) Concentração da quantidade de matéria.
c) Molalidade.
d) Fração molar do soluto e do solvente.
e) Densidade da solução.
f) Título (massa/massa).

792
1600 FÍSICO-QUÍMICA APLICADA EXERCÍCIOS COMENTADOS - IME – ITA – OLIMPÍADA

TESTE 239 – (OLIMPÍADA DE QUÍMICA DO RIO DE JANEIRO) Uma certa reação gasosa apresenta cinética de
primeira ordem, apresentando um tempo de meia-vida igual a 25 minutos. Sabendo que o logaritmo neperiano de 2 é
igual a 0,693, assinale a alternativa aproximada da velocidade específica desta reação.
a) 4,62 x 10-3 s-1
b) 6,4 x 10-3 s-1
c) 1,65 h-1
d) 3,78 min-1
e) Nenhuma das respostas anteriores.

TESTE 240 – (OLIMPÍADA BRASILEIRA DE QUÍMICA) Uma das formas de obtenção de cobre metálico, essencial
para uso em eletricidade, é passar gás amônia sobre óxido de cobre(II) aquecido. Nessa reação são obtidos cobre
metálico, nitrogênio e vapor d’água. Considerando que 100 g do óxido de cobre(II) tem 95 % de pureza e que a reação
tem um rendimento de 75 %, assinale a opção CORRETA.
a) 1,73 × 1023 moléculas de água foram obtidas;
b) 10,72 L de gás nitrogênio, nas CNTP, foram obtidos
c) 0,89 mol de cobre metálico foi obtido
d) 2,43 × 1023 átomos de cobre metálico foram obtidos
e) 128,75 g de produtos foram obtidos

TESTE 241 – (GRILLO) O monóxido de carbono e o gás hidrogênio reagem de acordo com a seguinte equação
química: CO(g) + 3H2(g) → CH4(g) + H2O(g). Considere que 1,0 mol de CO e 3,00 mol de H2 reagem num reator com
capacidade de 10,0 litros, a 727oC e quando atingido o equilíbrio contém 0,387 mol de H2O. Calcule a constante de
equilíbrio e a pressão total da mistura reacional?

TESTE 242 – O metanol é largamente empregado como combustível dos carros de corrida é obtido pela reação: C(s)
+ ½ O2(g) + 2H2(g) → CH3OH(g).
a) Escreva a expressão da constante de equilíbrio para esta reação.
b) Calcule Kp (25oC) para a reação acima.
c) Comente a relação existente entre o sinal do ΔGo e a magnitude de Kp.
Informação para a resolução do problema: ΔGfo (CH3OH(g)) = -166,27 kJ mol-1.

TESTE 243 – (MESTRE JOÃO ROBERTO DA PACIÊNCIA NABUCO) Uma massa de 0,820 g de gelatina foi
dissolvida em água suficiente para preparar 150 mL de solução. A pressão osmótica da solução encontrada foi 67,20
mmHg, a 27°C. Determine a massa molecular aparente desta gelatina.

TESTE 244 – (MESTRE JOÃO ROBERTO DA PACIÊNCIA NABUCO) Sabendo-se que a capacidade calorífica a
J
pressão constante da água é de 73,5 mol x K e que a combustão do metano apresenta H0 = -890 kJ.mol-1, calcule a
massa em gramas de metano que deve ser queimada para aquecer 1000 gramas de água de 20oC para uma
temperatura de 90oC.

TESTE 245 – (MESTRE JOÃO ROBERTO DA PACIÊNCIA NABUCO) The van der Waals constant b, for real gases,
may be used to determine the size of an atom or a molecule in the gaseous state. Fishbane et al. determine the value
of b to the gas nitrogen as being 39,4 x 10-6 m3.mol-1. What is the hypothetical radius to one molecule of N2?

TESTE 246 – (OLIMPÍADA MINEIRA DE QUÍMICA) Uma função de estado é aquela que não depende do caminho
de uma transformação, mas apenas dos estados inicial e final do sistema. É o caso, por exemplo, da entalpia (H) e
da energia livre de Gibbs (G). Para as funções de estado, pode-se usar a lei de Hess, em que as equações químicas
podem ser tratadas como equações matemáticas, ou seja, podem ser somadas, subtraídas ou multiplicadas, sendo
que as operações aplicadas às equações químicas devem ser também aplicadas aos respectivos valores das funções
de estado. Esse não é o caso do potencial padrão de redução (E0 ). No entanto, o potencial padrão de redução está
relacionado a uma função de estado, a energia livre de Gibbs, segundo a equação: 𝐆 = −𝐧 𝐱 𝐅 𝐱 𝐄°
em que n representa o número de elétrons, F é a constante de Faraday (96.485 C mol-1). Com base nas informações
acima e dadas as seguintes semi-equações
Fe2+(aq) + 2e− → Fe(s) E0 = − 0,44 V
Fe (aq) + 3e → Fe(s)
3+ − E0 = − 0,040 V

793
1600 FÍSICO-QUÍMICA APLICADA EXERCÍCIOS COMENTADOS - IME – ITA – OLIMPÍADA

qual é o valor do potencial padrão de redução para a semi-equação representada a seguir?


Fe3+(aq) + e− → Fe2+(aq)
a) − 0,48 V
b) − 0,40 V
c) + 0,40 V
d) + 0,76 V

TESTE 247 – (U. S. NATIONAL CHEMISTRY OLYMPIAD) What names apply to chemical species corresponding to
locations 1 and 2 on this reaction coordinate diagram?

Location 1 Location 2
a) activated complex activated complex
b) reaction intermediate activated complex
c) activated complex reaction intermediate
d) reaction intermediate reaction intermediate

TESTE 248 – (GRILLO) A capacidade calorífica a pressão constante de um determinado sistema gasoso ideal é
𝐶
apresentado da seguinte forma: 𝐶𝑝 = 𝐴 + 𝐵𝑇 + , onde A, B e C são constantes empíricas características para
𝑇
cada gás a ser estudado. Escreva de forma correta a expressão da entalpia em função da temperatura absoluta.

TESTE 249 – (OLIMPÍADA DE QUÍMICA DO RIO DE JANEIRO) O gás hidrogênio pode ser um combustível
alternativo em relação a combustíveis fósseis. Uma alternativa é produzir gás hidrogênio por eletrólise da água, para
depois utilizá-lo na síntese do ácido fórmico, a partir da hidrogenação catalítica do gás carbônico. A figura abaixo
ilustra como seria o ciclo de produção e consumo do ácido fórmico, conforme indica o texto.

A reação de síntese do ácido fórmico, por hidrogenação catalítica, pode ser representada pelo equilíbrio em fase
gasosa: H2(g) + CO2(g) + calor ⇌ HCOOH(g)

Visando melhorar o rendimento do processo de produção do ácido fórmico, assinale a alternativa que sugere as
melhores modificações que deveriam ser adotadas.
a) Aumento de temperatura e diminuição de pressão
b) Diminuição de temperatura e aumento de pressão
c) Aumento de temperatura e aumento de pressão
d) Diminuição de temperatura e diminuição de pressão
e) Colocação de um catalisador mantendo-se as demais condições de temperatura e pressão

TESTE 250 – (IME) Calcular a quantidade de calor resultante na transformação de 1 (um) grama de álcool etílico em
ácido acético, sabendo que o calor de combustão do álcool é 7607 cal/g e o do ácido é 3496 cal/g.

794
1600 FÍSICO-QUÍMICA APLICADA EXERCÍCIOS COMENTADOS - IME – ITA – OLIMPÍADA

a) 1826
b) 2507
c) 3571
d) 10583
e) N.R.A.

795
1600 FÍSICO-QUÍMICA APLICADA EXERCÍCIOS COMENTADOS - IME – ITA – OLIMPÍADA

CAPÍTULO XI

TESTE PARA PRATICAR -


GABARITO

PROFESSOR ALEXANDRE
VARGAS GRILLO

796
1600 FÍSICO-QUÍMICA APLICADA EXERCÍCIOS COMENTADOS - IME – ITA – OLIMPÍADA

TESTE 01 - (OLIMPÍADA BRASILEIRA DE QUÍMICA) Resolução: Alternativa A.

O efeito coligativo a ser estudado é o efeito coligativo definido como ebulioscopia.


g
Cálculo da massa de solvente utilizada: mCCl4 = d x V = 1590 L x 63 x 10−3 L = 100,17 g

solução solvente puro


Cálculo da massa molecular da amostra desconhecida: ∆Teb = Teb − Teb = K eb x W x i

Levando em consideração que a solução seja molecular, i = 1.

solução solvente puro msoluto


∆Teb = Teb − Teb = K eb x kg
x1
< MM >soluto x msolvente

2,0
(77 + 273) − 349,50 = 5,0 x x1
< MM >soluto x 0,100

< MM >soluto = 200 g. mol−1

TESTE 02 - (GRILLO) Resolução: Fórmula molecular do sulfato de bário: BaSO4


m 125 g g
a) concentração comum, em g.L-1: C = V soluto = 0,35 L = 357,14 L
solução

msoluto 125
nsoluto <MM>soluto mol
b) a concentração da quantidade de matéria: [BaSO4 ] = = = 233
= 1,53
Vsolução Vsolução 0,35 L

msolução msoluto + msolvente 125 g +1000 g 1125 g


c) a densidade da solução: d = Vsolução
= Vsolução
= 0,35 L
= 350
= 3,21 mL

msoluto 125
nsoluto <MM>soluto mol
d) a molalidade: W = (𝑘𝑔) = (𝑘𝑔) = 233
= 0,54
𝑚𝑠𝑜𝑙𝑣𝑒𝑛𝑡𝑒 𝑚𝑠𝑜𝑙𝑣𝑒𝑛𝑡𝑒 1,0 kg

msoluto 125 eqg


e) a normalidade: N = Eqg 𝑥 Vsolução
= 233 = 3,06 L
soluto 𝑥 0,35
2

TESTE 03 - (OLIMPÍADA BRASILEIRA DE QUÍMICA / OLIMPÍADA NORTE-NORDESTE DE UÍMICA) Resolução:


Alternativa A.

mC H8O6 60 x 10−3 g 60 x 10−3


Cálculo da quantidade de matéria (n): n = <MM>6 = g = 176
mol =
C6 H8 O6 (6 x 12+8 x 1+6 x 16)
mol
3,41 x 10−4 mol

TESTE 04 – Resolução: Equação química hipotética com cinética de primeira ordem: A(g) → B(g) + C(g)
Segundo o problema há decomposição do reagente A de 75%, logo: [A]Final = 0,25 x [A]Inicial
[A]Final
ln ( ) = −k x t
[A]Inicial

0,25 x [A]Inicial
ln ( ) = −k x t
[A]Inicial

ln(0,25) = −k x t

Cálculo da velocidade específica (k), a partir do tempo de meia-vida com cinética de primeira ordem:

797
1600 FÍSICO-QUÍMICA APLICADA EXERCÍCIOS COMENTADOS - IME – ITA – OLIMPÍADA

ln2
t 1⁄ =
2 k
ln2
25 min =
k
ln 2 0,693
k= = = 0,02772 min−1
25 min 25 min

Cálculo do tempo (t): ln(0,25) = −k x t

−ln(0,25) −ln(0,25) + 1,39


t= = = = 50,14 min
k 0,02772 0,02772

TESTE 05 – (OLIMPÍADA BRASILEIRA DE QUÍMICA) Resolução: A alternativa correta é a letra E. Sempre para um
processo espontâneo (dG < 0) a função de estado entropia irá aumenta (dS > 0).

TESTE 05 – Resolução: Pegando os valores que estão no extremo da tabela, temos:


k1 (5 + 273) = 1,50 x 10-6 s-1
k2 (50 + 273) = 1,40 x 10-3 s-1

k Eat 1 1
Aplicando a equação de Arrhenius para a determinação da energia de ativação (Eat): ln (k1 ) = R
x [T − T ]
2 2 1
1,50 x 10−6 Eat 1 1
ln ( −3 )= x [( )−( )]
1,40 x 10 8,314 50 + 273 5 + 273

1 1
ln(1,07 x 10−3 ) x 8,314 = Eat x [ − ]
323 278

(−6,84) x 8,314 = Eat x [3,096 x 10−3 − 3,60 x 10−3 ]

−6,84 x 8,314 J kJ
Eat = −4
= 112832,86 (112,83 )
− 5,04 x 10 mol mol
𝐸
(− 𝑎𝑡 )
Cálculo do fator de frequência (A) a 5°C: 𝑘 = 𝐴 𝑥 𝑒 𝑅𝑥𝑇
E
(− at )
ln k = ln A + ln e R x T
Eat
ln k = ln A −
RxT
112832,86
ln A = ln(1,50 x 10−6 ) +
8,314 x (5 + 273)
112832,86
ln A = −13,41 +
8,314 x 278

ln A = −13,41 + 48,82 = + 35,41

A = e(+35,41) = 2,39 x 1015 s−1

TESTE 06 – Resolução: Cálculo do número de mol de cada composto gasoso:


mCO2 30
nCO2 = = = 0,75 mol
< MM >CO2 44
mC2 H6 4,0
nC2 H6 = = = 0,133 mol
< MM >C2 H6 30

798
1600 FÍSICO-QUÍMICA APLICADA EXERCÍCIOS COMENTADOS - IME – ITA – OLIMPÍADA

Cálculo do número de mol total (nTOTAL): nTOTAL = nCO2 + nC2 H6 = 0,75 mol + 0,133 mol = 0,883 mol

nxRxT 0,883 x 0,08206 x (50+273) 0,883 x 0,08206 x 323


Cálculo da pressão total da mistura gasosa: pTotal = V
= 10
= 10
=
2,34 atm

TESTE 07 – (OLIMPÍADA CEARENSE DO ENSINO SUPERIOR DE QUÍMICA) Resolução: Alternativa A.

Analisando a estequiometria: Zn(s) + 2HCl(aq) → ZnCl2(aq) + H2(g)


1 mol de Zn(s) ---------------- 2 mol de HCl(aq)
n1 ------------------------------ n2
2 x nmetal = nZn
mmetal
2x = nHCl
< MM >metal
mmetal
2x = [HCl] x Vsolução
< MM >metal

12,05
2x = 2 x Vsolução
65,39

Vsolução = 0,1843 L (184,3 mL)

TESTE 08 – (OLIMPÍADA CEARENSE DO ENSINO SUPERIOR DE QUÍMICA) Resolução: Alternativa C.

Analisando a estequiometria: LiC4H9(s) + HCl(aq) → LiCl(aq) + C4H10(g)

1 mol de LiC4H9(s) ---------- 1 mol de C4H10(g)


n1 ------------------------------ n2
n1 = n2
m1 m2
=
< MM >1 < MM >2

m1 0,725
=
(6,94 + 48 + 9) (48 + 10)

m1 0,725
=
63,94 58

63,94 x 0,725
m1 = = 0,799 g
58
0,799
Cálculo do grau de pureza, (%)pureza : (%)pureza = = 0,1465 (14,65%)
5,453

TESTE 09 – (OLIMPÍADA CEARENSE DO ENSINO SUPERIOR DE QUÍMICA) Resolução: Alternativa C.

Cálculo da massa molar do composto X, a partir da reação descrita pelo problema: XCl4 + 2 NH3 → XCl2 (NH3 ) +
Cl2

1 mol de XCl4 ------------------ 1 mol de XCl2(NH3)2


(<MM>X + 4 x 35,5) ---------- (<MM>X + 2 x 35,5 + 2 x 14 + 6 x 1)
3,571 g -------------------------- 3,180 g

799
1600 FÍSICO-QUÍMICA APLICADA EXERCÍCIOS COMENTADOS - IME – ITA – OLIMPÍADA

3,18 x (< MM >X + 142) = 3,571 x (< MM >X + 105)


− 76,605 g
< MM >X = = 195,92
− 0,391 mol

TESTE 10 – (OLIMPÍADA CEARENSE DO ENSINO SUPERIOR DE QUÍMICA) Resolução: Alternativa E.

Tempo de meia−vida
32 32
15Pinicial → 15Pfinal

ln 2
Cálculo da constante radioativa (λ), a partir do tempo de meia-vida (t1/2): 14,30 = λ
ln 2
λ= = 4,85 𝑥 10−2 𝑑𝑖𝑎𝑠 −1
14,30

[ 32P ]
Cálculo do tempo (t): ln ( 3215P final ) = −λ x tempo
[ 15 inicial ]

0,001 𝑥 [ 32
15Pinicial ]
ln ( 32 ) = −4,85 𝑥 10−2 x tempo
[ 15Pinicial ]

ln(0,001) = −4,85 x 10−2 x tempo

ln(0,001) −6,91
tempo = = = 142,47 dias
−4,85 x 10−2 −4,85 x 10−2

TESTE 11– (OLIMPÍADA CEARENSE DO ENSINO SUPERIOR DE QUÍMICA) Resolução: Alternativa E. [MCl2]:

+2
MCl2(l) → M(l) + 2 Cl−
(aq)

+2
Reação que ocorre no cátodo: M(l) + 2e− → M(s)

96500 C
<MM>M ---------- 2 mol de elétrons x 1 mol de elétrons

3600 s
4,0 g -------------- Q = 4,65 h x x 0,84
1h

4,0 x 2 x 96500 g
< MM >M = = 54,90
4,65 x 3600 x 0,84 mol

TESTE 12 – (OLIMPÍADA BRASILEIRA DE QUÍMICA) Resolução: Alternativa A.


m
Considerando que o gás apresenta comportamento ideal, temos: pV = <MM> RT

m 1,14 g
< MM > = RT = x 0,08206 x (27 + 273) = 1,14 x 0,08206 x 300 = 28,06
pV 1x1 mol

Para esta massa molecular o gás é o monóxido de carbono.

TESTE 13 – (OLIMPÍADA BRASILEIRA DE QUÍMICA JÚNIOR) Resolução: Equação química: 2 Mg(s) + O2(g) → 2
MgO(g)

2 mol de Mg ----- 2 mol de MgO


2 x 24 g ----------- 2 x (24 + 16) g
mMg ---------------- 2,0 g

800
1600 FÍSICO-QUÍMICA APLICADA EXERCÍCIOS COMENTADOS - IME – ITA – OLIMPÍADA

2 x 2 x 24 96
mMg = = = 1,2 g
2 x 40 80

TESTE 14 – (GRILLO) Resolução: Ácido bórico: H3BO3 (62 g.mol-1)


g
Cálculo da concentração comum (g.L-1): C = 10 x d x (%) = 10 x 1,23 x 20 = 246 L

C 246
Cálculo da concentração da quantidade de matéria, [H3BO3]: comum (g.L-1): [H3 BO3 ] = <MM> = 62
=
H3 BO3
mol
3,97
L

TESTE 15 – (ITA) Resolução: Cálculo da pressão de vapor, a partir da equação apresentada a seguir: 3,0 x 10−6 =
[Hg] x 0,08206 x (25 + 273)

3,0 x 10−6 = [Hg] x 0,08206 x 298

3,0 x 10−6 mol


[Hg] = = 1,23 x 10−7
0,08206 x 298 L

Sabendo que a massa atômica do mercúrio é igual a 200,59 g.mol-1, a concentração mássica deste elemento é igual a:
mol C
1,23 x 10−7 =
L <MA>Hg

mol C
1,23 x 10−7 =
L 200,59

mol g
C = 1,23 x 10−7 x 200,59
L mol
g
C = 2,46 x 10−5
L
𝑔 1000 𝑚𝑔 1𝐿 1 𝑑𝑚³
Convertendo a concentração mássica para mg.m-3, temos: 𝐶 = 2,46 𝑥 10−5 𝑥 𝑥 𝑥
𝐿 1𝑔 1 𝑑𝑚³ 10−3 𝑚³

𝑚𝑔
𝐶 = 24,6
𝑚³

Para esta concentração mássica a concentração será letal, pois é superior a 16 mg.m-3.

TESTE 16 – (OLÍMPIADA PARAENSE DE QUÍMICA ) Resolução: Alternativa ?


∆p <MM>solvente
Sabendo que o processo coligativo a ser analisado é o tonoscópico, temos: p0 = 1000
xWxi

75 − 70 78 64
= x x1
75 1000 < MM >soluto x 0,546

5 𝑥 1000 𝑥 0,546 𝑥 < MM >soluto = 75 x 78 x 64

374400 g
< MM >soluto = = 137,14
2730 mol

TESTE 17 – (IFRJ – TURMA QIM241) Resolução: Equação química: C3H6(g) (ciclopropano) → C3H6(g) (propeno)

Dados do problema:[C3 H6 ]Inicial = 1,50 mol. L−1 e k = 2,19 x 10−4

801
1600 FÍSICO-QUÍMICA APLICADA EXERCÍCIOS COMENTADOS - IME – ITA – OLIMPÍADA

[C H ]
a) Considerando a cinética de primeira ordem: ln ([C 3H 6] Final ) = −k x t
3 6 Inicial

[C3 H6 ]Final 60 min


ln ( ) = −2,19 x 10−4 x 20 h x
1,50 1h

[C3 H6 ]Final
ln ( ) = −0,2628
1,50

mol
[C3 H6 ]Final = e(−0,2628) x 1,50 = 1,15
L
ln2 0,693
Cálculo do tempo de meia-vida: t1/2 = k
= 2,19 x 10−4 = 3164,38 min

1 1
b) Considerando a cinética de segunda ordem: [C = 1,50 + 2,19 x 10−4 x 1200
3 6 ]Final
H
mol
[C3 H6 ]Final = 1,07
L
1 1 1
Cálculo do tempo de meia-vida: 𝑡1⁄ = 𝑘 𝑥 [𝐶 = 2,19 𝑥 10−4 𝑥 1,50 = 0,2628 = 3,80 𝑚𝑖𝑛
2 3 𝐻6 ]0

TESTE 18 – (GRILLO) Resolução: Através do mecanismo apresentado, observa-se que as duas primeiras reações
químicas representa uma reação reversível, que pode ser representado da seguinte forma: A3(g) ⇌ B2(g) + C

Diante deste processo reversível trata-se da etapa rápida. Para a terceira reação química trata-se da etapa lenta, ou
seja, a etapa determinante.

Equação da velocidade: vreação = k c x [C] x [A3 ]

Conforme o enunciado do problema, será considerado que o estado estacionário seja o composto C.
d[C]
= +k a x [A3 ] − k b x [B2 ] x [C] − k c x [C] x [A3 ]
dt
𝑑[𝐶]
Como a concentração da quantidade de matéria do estado estacionário é muito pequena, = 0, logo:
𝑑𝑡

d[C]
= +k a x [A3 ] − k b x [B2 ] x [C] − k c x [C] x [A3 ]
dt

+ k a x [A3 ] = k b x [B2 ] x [C] + k c x [C] x [A3 ]

+ k a x [A3 ] = [C] x {k b x [B2 ] + k c x [A3 ]}

k a x [A3 ]
[C] =
{k b x [B2 ] + k c x [A3 ]}

Multiplicando a reação química por dois, temos: 2 A3(g) → 3 B2(g)


1 d[A3 ]
vreação = − x = k c x [C] x [A3 ]
2 dt

1 d[A3 ]
− x = k c x [C] x [A3 ]
2 dt

802
1600 FÍSICO-QUÍMICA APLICADA EXERCÍCIOS COMENTADOS - IME – ITA – OLIMPÍADA

1 d[A3 ]
Substitutindo a equação da concentração da quantidade de matéria do intermediário, temos: − 2 x dt
=
k c x [C] x [A3 ]

1 d[A3 ] k a x [A3 ]
− x = kc x x [A3 ]
2 dt {k b x [B2 ] + k c x [A3 ]}

d[A3 ] 2 x k a x k c x [A3 ]²
− =
dt {k b x [B2 ] + k c x [A3 ]}

TESTE 19 – (GRILLO) Resolução:


Fórmula molecular do etanol: C2H6O (46 g.mol-1)
Informação do problema: 1 mL de solução / 2,53 g de solução

Cálculo da massa da solução:


1 mL de solução --------------- 2,50 g de solução
1000 mL de solução ----------- msolução
msolução = 2500 g de solução
n
Cálculo da massa de etanol, a partir da concentração da quantidade de matéria, [Etanol]: [Etanol] = V etanol
solução

metanol = [Etanol] x Vsolução x < MM >etanol = 8,0 x 46 x 1 = 368 g

Massa da solução: msolução = metanol + msolvente

2500 g = 368 g + msolvente

msolvente = 2132 g (2,13 kg)

netanol 8,0 mol


Cálculo da molalidade (W): W = (kg) = 2,13 = 3,75 L
msolvente

m 368,0 g
Cálculo da concentração mássica (C): C = V etanol = 1,0
= 368 L
solução

TESTE 20 – (IIT-JEE) Resolução: Equação química: 3 BaCl2 + 2 Na3PO4 → 6 NaCl + Ba3(PO4)2

Determinação do reagente limitante:


0,50
nBaCl2 = = 0,167 mol
3,0
0,20
nNa3 PO4 = = 0,10 mol
2,0

Como nNa3 PO4 < nBaCl2 , o fostato de sódio é o reagente limitante. Através da estequiometria, temos:
2 mol de Na3PO4 -------------- 1 mol de Ba3(PO4)2
0,20 mol de Na3PO4 ---------- 𝑛𝐵𝑎3 (𝑃𝑂4 )2
nBa3 (PO4 )2 = 0,10 mol

TESTE 21 – (OLIMPÍADA BRASILEIRA DE QUÍMICA) Resolução:


Item a) A partir dos valores que estão no extremo da tabela apresentada, temos:
k1 (660 K) = 7,20 x 10-4
k2 (760 K) = 0,110

803
1600 FÍSICO-QUÍMICA APLICADA EXERCÍCIOS COMENTADOS - IME – ITA – OLIMPÍADA

7,20 x 10−4
Aplicando a equação de Arrhenius para a determinação da energia de ativação (Eat): ln ( ) =
0,110
Eat 1 1
x[ − ]
8,314 760 660

ln(6,54 x 10−3 ) x 8,314 = Eat x (− 1,99 x 10−4 )

−5,03 x 8,314 J 𝑘𝐽
Eat = −4
= 210147,84 (210,15 )
(− 1,99 x 10 ) mol 𝑚𝑜𝑙
E
(− at )
Cálculo do fator de frequência (A): k = A x e RxT
E
(− at )
ln k = ln A + ln e R x T

Eat
ln k = ln A −
RxT
210147,84
Para T = 760 K: ln A = ln(0,110) + 8,314 x 760

ln A = −2,21 + 33,26 = + 31,05

A = e(+31,05) = 3,06 x 1013

Item b) Informação do problema: k1 (660 K) = 7,20 x 10-4 e k2 (673 K) = ? Cálculo da constante cinética à 400°C (673
K):

k1 Eat 1 1
ln ( )= x[ − ]
k3 R T2 T1

7,20 x 10−4 210147,84 1 1


ln ( )= x[ − ]
k3 8,314 673 660

7,20 x 10−4
ln ( ) = 25276,38 x (− 2,93 x 10−5 )
k3

7,20 x 10−4
k3 = = 1,51 x 10−3
0,477

Item c) A utilização de um catalisador para um processo é de aumentar a velocidade da reação química, diminuindo
a energia de ativação, não havendo influência na variação de entalpia e nam da constante de equilíbrio (Keq).

Item d) Equação química: C2H5I(g) → C2H4(g) + HI(g)

C2H5I(g) → Para o iodoetano a configuração geométrica é tetraédrica (109° 28’) com hibridização sp³.

C2H4(g) → Para o eteno a configuração geométrica é trigonal plana com hibridização sp².
Item e) Iodeto de hidrogênio (HI) → ligação covalente polar caracterizando uma molécula polar.

Iodoetano (C2H5I) → Sua molécula apresenta uma parte polar, pois apresenta o iodo em sua estrutura e apolar
entre os átomos de carbono.

Eteno (C2H4) → ligação covalente polar caracterizando uma molécula apolar.

804
1600 FÍSICO-QUÍMICA APLICADA EXERCÍCIOS COMENTADOS - IME – ITA – OLIMPÍADA

Teste 22 – (OLIMPÍADA BRASILEIRA DE QUÍMICA) Resolução: Analisando o recipiente da amônia, calculando o


número de mol: 0,78 x 2,50 = nNH3 x 0,08206 x (18,5 + 273)
1,95
nNH3 = = 8,15 x 10−2 mol
23,92

Analisando o recipiente de cloreto de hidrogênio, calculando o número de mol a partir da equação dos gases ideais:

0,93 x 1,40 = nHCl x 0,08206 x (18,5 + 273)

1,302
nHCl = = 5,44 x 10−2 mol
23,92

Analisando o número de mol de cada reagente, o cloreto de hidrogênio é o reagente limitante.

Equação química: NH3(g) + HCl(g) → NH4Cl(s)

Cálculo da massa de cloreto de amônio: NH3(g) + HCl(g) → NH4Cl(s)


1 mol de HCl ----------------- 1 mol de NH4Cl
5,44 x 10-2 mol --------------- nNH4Cl
nNH4Cl = 5,44 x 10-2 mol

Cálculo da massa de cloreto de amônio: nNH4 Cl = 5,44 x 10−2 mol

mNH4 Cl
= 5,44 x 10−2
(14 + 4 + 35,5)

mNH4 Cl = 2,89 g

Como o HCl é o gás minoritário, a amônia está em excesso. Logo: nexcesso


NH3 = 8,15 x 10−2 − 5,44 x 10−2 =
0,0271 mol

Cálculo da pressão da amônia: p x (2,50 + 1,40) = 0,0271 x 0,08206 x (18,5 + 273)

0,648
p= = 0,166 atm
3,90

TESTE 23 – (ITA) Resolução: Alternativa C.

TESTE 24 – (IFRJ – TURMA QIM 241) Resolução: Dados do problema: 𝑘1 (110℃) = 5 𝑥 𝑘2 (60℃)

k Eat 1 1
Cálculo da energia de ativação, a partir da equação de Arrhenius: ln (k1 ) = R
x {T − T }
2 2 1

5 x k2 Eat 1 1
ln ( )= x{ − }
k2 8,314 (60 + 273) (110 + 273)

Eat 1 1
ln(5) = x{ − }
8,314 333 383

Eat
ln(5) = x {0,0030 − 2,61 x 10−2 }
8,314
Eat
ln(5) = x 3,92 x 10−4
8,314

805
1600 FÍSICO-QUÍMICA APLICADA EXERCÍCIOS COMENTADOS - IME – ITA – OLIMPÍADA

8,314 x 1,64 J
Eat = −4
= 34146,78
3,92 x 10 mol

TESTE 25 – (IME) Resolução:


a) Com 5,6 % de V2O5, forma-se a menor quantidade de CO2, (6%) quando se forma etileno. Como é o CO2
que provoca o efeito-estufa, a redução da quantidade de gás diminui esse efeito.
b) V2O5 diminui a energia de ativação dos reagentes, e com isso acelera a velocidade da reação.

TESTE 26 – (ITA) Resolução: Alternativa B. Os halogênios estão destacados em negrito a seguir: HF, HBrO3, HIO3,
ClO3 e ClO4-

Ácido fluorídrico: O flúor apresenta número de oxidação igual a -1;

Ácido Brômico: +1 + x - 6 = 0 → x = +5

Ácido Iódico: +1 + x - 6 = 0 → x = +5

ClO3: x – 6 = 0 → x = +6

Íon perclórico: x – 8 = -1 → x = +7

TESTE 27 – (ITA) Resolução: Alternativa E.

Destacando em negrito o cloro nos compostos a seguir: KClO2, Ca(ClO)2, Mg(ClO3)2 e Ba(ClO4)2

KClO2: +1 + x - 4 = 0 → x = +3
Ca(ClO)2: +2 + 2x - 4 = 0 → x = +1
Mg(ClO3)2: +2 + 2x - 12 = 0 → x = +5
Ba(ClO4)2: +2 + 2x - 16 = 0 → x = +7

TESTE 28 – (ITA) Resolução: Alternativa A.

O kitassato é um aparelho (vidraria) muito utilizado em laboratório de química, em qualquer área. Seu objetivo é junto
com o funil de Bunchner em processos de filtração sob sucção a vácuo.

TESTE 29 – (ITA) Resolução: Alternativa B. A cal viva é o óxido de cálcio. Este composto binário é classificado
como um óxido básico.

TESTE 30 – (ITA) Resolução: Alternativa C.


O problema retrata sobre as propriedades coligativas. Então com a adição de um soluto não-volátil em uma quantidade
de solvente, a temperatura de ebulição aumenta, a temperatura de congelamento diminui, a pressão de vapor
aumenta. Com isso, a alternativa correta é a alternativa C.

TESTE 31 – (ITA) Resolução: Alternativa D.

TESTE 32 – (ITA) Resolução: Alternativa D.

TESTE 33 – (ITA) Resolução: Alternativa D.


O componente essencial da máquina de James Watt era o condensador de vapor, que minimizava as perdas de calor.

TESTE 34 - (ITA) Resolução: Alternativa B.


De acordo com o gráfico apresentado pela questão podemos concluir da seguinte maneira:

806
1600 FÍSICO-QUÍMICA APLICADA EXERCÍCIOS COMENTADOS - IME – ITA – OLIMPÍADA

Para que este processo ocorra, é necessário fornecer uma maior quantidade de energia para passar a substância I
do estado líquido para o gasoso (ebulição) do que para passar a substância II do estado líquido para o gasoso
(ebulição), logo: HV(I) > HV(II).

TESTE 35 – Resolução: Alternativa D.

O número de mol varia com a pressão segundo o princípio de Le Chatelier. Quanto maior a pressão do sistema, o
equilíbrio será deslocado para a esquerda, ou seja, aumentando a pressão parcial tanto do nitrogênio quanto do
hidrogênio.

TESTE 36 – Resolução: No trajeto 1→ 2: este trajeto é isovolumétrico, então o trabalho o trabalho realizado é igual
a zero (W = 0).

W1/2 = 0

No trajeto 2 → 3: este trajeto caracteriza-se por uma expansão isobárica.

W2/3 = −p x ∆V = −100000 Pa x (10 𝑚3 − 2 𝑚3 ) = −800000 (−800 𝑘𝐽)

Trajeto 3 → 1: Este trajeto caracteriza-se como um processo isotérmico (temperatura constante).

Cálculo da massa molecular do ar atmosférico, levando em consideração que o ar atmosférico apresenta como
composição igual a 20% de oxigênio molecular e 80% de nitrogênio:

< MM > ar x (%)ar = < MM >O2 x (%)O2 + < MM >N2 x (%)N2


atmosférico

< MM > ar x 1,0 = 32,0 x 0,20 + 28 x 0,80


atmosférico

g
< MM > ar = 28,80
atmosférico mol
m ar
2000
Cálculo do número de mol de ar atmosférico: n ar
atmosférico
= <MM> = 28,80 = 69,44 mol
atmosférico ar
atmosférico

100000
pxV ( ) x 10000
Cálculo da temperatura no ponto 3, aplicando a equação dos gases ideais: T3 = = 101325
=
nxR 69,44 x 0,08206
9869,23
5,70
= 1731 𝐾

V
Cálculo do trabalho no trajeto 3 → 1: W3/1 = −n x R x T x ln (V1 )
3

807
1600 FÍSICO-QUÍMICA APLICADA EXERCÍCIOS COMENTADOS - IME – ITA – OLIMPÍADA

2 m3
W3/1 = −69,44 x 8,314 x 1731 x ln ( ) = +1608,39 𝑘𝐽
10 m3

Cálculo do trabalho total: Wtotal = W1/2 + W2/3 + W3/1

Wtotal = 0 − 800 kJ + 1608,39 kJ

Wtotal = +808,39 kJ

TESTE 37 – (IME) Resolução: As fórmulas moleculares dos participantes químicos estão apresentados na tabela a
seguir:

Nitrito de sódio NaNO2


Óxido nítrico ou NO
monóxido de
nitrogênio
Sulfato ferroso ou FeSO4
sulfato de ferro II
Ácido sulfúrico H2SO4
Sulfato férrico ou Fe2(SO4)3
sulfato de ferro III
Bissulfato de sódio NaHSO4
Água H2O

Para a realização do balanceamento desta reação química o método mais adequado é do tipo redox.

A reação química balanceada pelo método oxidação-redução (redox) apresenta o nitrito de sódio como agente
oxidante e o sulfato ferroso como agente redutor. A equação química vai ficar da seguinte maneira: 2 NaNO2 + 2
FeSO4 + 3 H2SO4 → 2 NO + Fe2(SO4)3 + 2 NaHSO4 + 2 H2O

Determinação do reagente limitante, a partir do cálculo do número de mol:


75 1,08
Cálculo do número de mol de nitrito de sódio: nNaNO2 = = = 0,94 mol
69 2

152 1,0
Cálculo do número de mol de sulfato ferroso: nFeSO4 = = = 0,50mol (Reagente limitante)
152 2

150 1,53
Cálculo do número de mol de dióxido de manganês: nH2 SO4 = 98
= 3
= 0,51 mol

Cálculo da massa de monóxido de nitrogênio, a partir do regente limitante (sulfato ferroso):

2 mol de sulfato ferroso − − − −2 mol de monóxido de nitrogênio


2 x 152 g − − − − − 2 x 30 g x 0,90
152 g − − − − − − − mNO

mNO = 27,0 gramas de monóxido de nitrogênio.


TESTE 39 – (GRILLO) Resolução:
1,80 𝑥 10−3
Cálculo do número de mol de N2: 𝑛𝑁2 = 28
= 6,43 𝑥 10−3 𝑚𝑜𝑙

200
Cálculo do número de mol de água: nH2 O = 18
= 11,11 mol

Cálculo do número de mol total: nTOTAL = nN2 + nH2 O

808
1600 FÍSICO-QUÍMICA APLICADA EXERCÍCIOS COMENTADOS - IME – ITA – OLIMPÍADA

nTOTAL = 6,43 x 10−5 mol + 11,11 mol = 11,11 mol

Cálculo da constante de Henry (H): PN2 = H x X N2

PN2 PN2 740 mmHg


H= = nN2 = = 1,28 x 108 mmHg
X N2 6,43 x 10−5
nTOTAL 11,11

TESTE 40 – (U. S. NATIONAL CHEMISTRY OLYMPIAD)


A - Resolução: Alternativa C.
6,02 x 1023
1304 g − − − − − 26 x
1 mol
1,0 g − − − − − − X

26 x 6,02 x 1023 x 1
X= 1304
= 0,12 x 1023 (1,20 x 1022 átomos de oxigênio)

B – Resolução: Alternativa D.

Informação do problema: a massa do fosfato e do óxido são iguais a m.

Equação química balanceada:

Ca3(PO4)2 + 3 SiO2 + 5 C + 5 O2 + 3 H2O → 3 CaSiO3 + 5 CO2 + 2 H3PO4

Determinação do reagente limitante:


𝑚
nfosfato de cálcio = (REAGENTE LIMITANTE)
3 𝑥 310

𝑚
nóxido = 3 𝑥 60

Pela estequiometria temos a seguinte relação:

1 mol de Ca3 (PO4 )2 − − − − − 2 mol de H3 PO4


310 g − − − − − − − − − − − 2 x 98 g
mCa3 (PO4 )2 − − − − − − − − − 1000 kg

310 x 1000
mCa3 (PO4 )2 = = 1581,63 kg
196

809
1600 FÍSICO-QUÍMICA APLICADA EXERCÍCIOS COMENTADOS - IME – ITA – OLIMPÍADA

TESTE 41 – (ENADE) Resolução: Alternativa C.


Cálculo do potencial padrão para a primeira semirreação: E° = + 1,23 − (0,059) x pH
E° = + 1,23 − (0,059) x 7,0
E° = + 0,817 V

Cálculo do potencial padrão para a reação global, invertendo a segunda semirreação e multiplicando por dois:

O2(g) + 4H+(aq) + 4e- → 2 H2O(l) E° = + 0,817 V


2 Cu(s) → 2 Cu (aq) + 4e
+2 - E° = - 0,34 V +
O2(g) + 4H+(aq) + 2 Cu(s) → 2 H2O(l) + 2 Cu+2(aq) E° = + 0,477 V

Para este processo eletroquímico, caracteriza-se um processo espontâneo, com potencial padrão igual a E° = + 0,477
V.
0,0075 torr
TESTE 42 – (GRILLO) Resolução: Conversão da pressão – Pascal → torr: 200000 Pa x 1 Pa
= 1500 torr

Cálculo da fração molar do dióxido de carbono, a partir da lei de Henry.

PCO2 = X CO2 x H, onde h é definido como a constante de Henry.

PCO2 1500 torr


X CO2 = = = 0,01875
H 8,0 x 104 torr

TESTE 43 – (OLIMPÍADA PARAENSE DE QUÍMICA) Resolução: Alternativa B.


A energia de ativação (Eat) é a energia mínima necessária fornecida para os reagentes para que consigam ultrapassar
a barreira do complexo ativado.

TESTE 44 – (OLIMPÍADA BRASILEIRA DE QUÍMICA) Resolução: Alternativa C.


mH3 BO3
[H3 BO3 ] =
< MM >H3 BO3 x Vsolução

2,50
0,50 =
61,0 x Vsolução

2,50
Vsolução = = 8,20 x 10−2 L (82 mL)
0,50 x 61

TESTE 45 – (IME) Resolução: Equação química hipotética: A(g) + B(g) → C(g) + D(g)
a
3,0 x 10−3 27
Cálculo da ordem de reação em relação ao reagente A, entre o experimento IV e VI: (1,0 x 10−3 ) = 3
→ (3)a =9
→a=2

b
3,0 x 10−3 60
Cálculo da ordem de reação em relação ao reagente B, entre o experimento I e III: (1,0 x 10−3 ) = 20 → 3b = 3 →
b=1

Equação da velocidade da reação química: vreação = k x [A]² x [B]¹


Ordem global: 2 + 1 = 3

TESTE 46 – (OLIMPÍADA BRASILEIRA DE QUÍMICA) Resolução: Alternativa D.


140 g de KNO 60 g de KNO
Observando o gráfico, a 70°C há 100 g de H O3 e a 40°C há 100 g de H O3 .
2 2

810
1600 FÍSICO-QUÍMICA APLICADA EXERCÍCIOS COMENTADOS - IME – ITA – OLIMPÍADA

Quantidade de precipitação de nitrato de potássio: 140 g – 60 g = 80 g

TESTE 47 –

A - (U. S. NATIONAL CHEMISTRY OLYMPIAD) Resolução: Alternativa C.


Para a resolução deste problema será necessário aplicar a equação de Arrhenius.
Dados do problema: k 2 = 5 x k1 ; T1 = 277 K e T2 = 300 K.

k1 Eat 1 1
ln ( )= x{ − }
5 x k1 8,314 300 277

1 Eat
ln ( ) = x {− 2,64 x 10−4 }
5 8,314

1
ln ( ) x 8,314 −1,64 x 8,314 J kJ
Eat = 5 = = 50702,80 (50,70 )
−4
{− 2,64 x 10 } {− 2,64 x 10 }−4 mol mol

B - (U. S. NATIONAL CHEMISTRY OLYMPIAD) Resolução: Alternativa C.


Para um processo radioativo, a cinética química é de primeira ordem. Cálculo da constante radioativa (λ), a partir do
ln 2
tempo de meia-vida: 𝑡1/2 = λ
ln 2 0,693
λ= = = 0,0693 𝑚𝑖𝑛−1
𝑡1/2 10 𝑚𝑖𝑛

(N−13)
Cálculo do tempo: ln [(N−13) Final ] = − λ x tempo
Inicial

(N − 13)Final
ln [ ] = − 0,0693 x 25
40 μCi

(N − 13)Final
ln [ ] = − 1,7325
40 μCi

(N − 13)Final = 40 μCi x e⏟−1,7325


0,177

(N − 13)Final = 7,08 μCi

TESTE 48 – (IME) Resolução:


a) Falsa. Os compostos orgânicos são bons condutores de eletricidade quando fundidos.
b) Falsa. Compostos apolares são insolúveis em água, pois esta é um solvente polar.
c) Verdadeira. O boro tem um elétron desemparelhado no subnível p da última camada que seria utilizado para
a formação de uma ligação com o flúor.
d) Falsa. Bipiramidal quadrática.

TESTE 49 – (IME) Resolução:


Semirreação de Redução: Cromo (+7 para +3) → ∆ = 4 x 2 = 8 (há dois átomos de cromo nos reagentes).
Semirreação de Oxidação: Ferro (+2 para +3) → ∆ = 1 x 1 = 1 (há somente um átomo de ferro nos reagentes).

Na presença de um meio ácido, temos: Cr2 O7 + Fe+2 + H + → Cr +3 + Fe+3 + H2 O

Agente redutor: Fe+2

Agente oxidante: Cr2 O7

811
1600 FÍSICO-QUÍMICA APLICADA EXERCÍCIOS COMENTADOS - IME – ITA – OLIMPÍADA

Equação química balnceada pelo método REDOX: Cr2 O7 + 8 Fe+2 + 14 H + → 2 Cr +3 + 8 Fe+3 + 7 H2 O

TESTE 50 – (IME) Resolução: Analisando a primeira equação química – I:

O clorato de potássio tanto vai oxidar quanto vai reduzir, logo o clorato de potássio é o agente oxidante e o agente
redutor.

Equação química balanceada pelo método REDOX: 6 KClO3 + 3 H2 SO4 → 2 HClO4 + 4 ClO2 + 3 K 2 SO4 +
2 H2 O
Coeficiente estequiométricos: 6; 3; 2; 4; 3 e 2.

Analisando a segunda equação química – II: KMnO4 + HCl → KCl + MnCl2 + H2 O + Cl2
1
Semirreação de oxidação: Cl− → Cl2 + 𝑒 −
2
Semirreação de oxidação: Mn+7 + 5𝑒 − → Mn+2

Equação química balanceada por REDOX: 2 KMnO4 + 16 HCl → 2 KCl + 2 MnCl2 + 8 H2 O + 5 Cl2

a) Coeficientes estequiométricos: 2; 16; 2; 2; 8 e 5.


b) Agente redutor: ácido clorídrico (HCl).
c) Agente oxidante: permanganato de potássio (KMnO4).

TESTE 51 – (ITA) Resolução: Alternativa A.


Dentre os cinco compostos listados há quatro compostos iônicos, os quais apresentam altos pontos de fusão e de
ebulição, apresentando assim uma grande dificuldade de passar para o estado de vapor, enquanto que a uréia por
ser um composto molecular apresenta pontos de fusão e ebulição bem inferiores aos dos compostos iônicos, podendo
então evaporar com relativa facilidade. Tendo isto em vista, podemos concluir que o composto em questão, após a
análise do gráfico e levando em conta o exposto acima, é a uréia.

TESTE 52 – (ITA) Resolução: Alternativa E.


Quanto mais intensas as atrações intermoleculares, tanto mais difícil é a vaporização do líquido. Por outro lado, quanto
maior a temperatura de ebulição, tanto menor é a pressão de vapor, pois a temperatura de ebulição é a temperatura
em que a pressão de vapor do líquido iguala a pressão sobre ele.

TESTE 53 – (ITA) Resolução: Numa câmara barométrica do tipo Torricelli, mantida a 30ºC, introduz-se, por meio de
um conta-gotas, o etanol líquido a 30ºC. Este é menos denso que o mercúrio e, por isso, sobe para a câmara
barométrica vazia, onde se vaporiza. O vapor de etanol abaixa a coluna de mercúrio porque exerce pressão sobre
ela. Vai-se introduzindo gota a gota o etanol até a câmara ficar saturada de vapor, isto é, uma nova gota de etanol
introduzida não se vaporizará, permanecendo líquida na superfície do mercúrio da câmara barométrica. Dizemos
então que os vapores do etanol são saturantes a 30ºC e o abaixamento da coluna de mercúrio será igual à pressão
(máxima) de vapor do etanol naquela temperatura.

812
1600 FÍSICO-QUÍMICA APLICADA EXERCÍCIOS COMENTADOS - IME – ITA – OLIMPÍADA

TESTE 54 – (ITA) Resolução: Alternativa E.


O ferro metálico é obtido através de uma reação redox (redução-oxidação) da hematita na presença de monóxido de
carbono no estado gasoso em um alto forno, conforme pode ser observado através da equação química balanceada
a seguir: Fe2 O3(s) + 3 CO(g) → 2 Fe(s) + 3 CO2(g).

TESTE 55 - (ITA) Resolução: Alternativa A.


Quando o gás carbônico sai do recipiente ele absorve calor da água da atmosfera e esta condensa. O colóide formado
por água no estado líquido e gás carbônico é visível a olho nu.

TESTE 56 – (GRILLO) Resolução: Equação química: CaC2(s) + 2 H2O(l) → C2H2(g) + Ca(OH)2(s)

95
Cálculo da massa de carbeto de cálcio puro: mCaC2 = 5,00 g x (100) = 4,75 g

CaC2(s) + 2 H2O(l) → C2H2(g) + Ca(OH)2(s)


1 mol de CaC2 ---------------- 1 mol de Ca(OH)2
nCaC2 ----------------------------- nCa(OH)2
nCaC2 = nCa(OH)2
mCaC2 mCa(OH)2
=
< MM >CaC2 < MM >Ca(OH)2

4,75 g mCa(OH)2
−1
=
(40 + 24)g. mol (40 + 32 + 2)g. mol−1

mCa(OH)2 = 5,49 g

TESTE 57 – (ITA) Resolução: Alternativa E.


A partir do momento em que um catalisador é colocado em um determinado processo, a velocidade vai aumentar, a
energia de ativação vai diminuir e o complexo ativado também vai diminuir. O catalisador inserido em um sistema não
altera a natureza e nem o valor da variação da entalpia e também não altera o valor da constante de equilíbrio.

TESTE 58 – (ITA) Resolução: Alternativa A.

TESTE 59 – (ITA) Resolução: Alternativa D.

TESTE 60 – (ITA) Resolução: Alternativa D.


Fórmulas moleculares:
Hidróxido de potássio: KOH
Cloreto de amônio: NH4Cl

Equação química entre o hidróxido e o sal: KOH + NH4 Cl → KCl + ⏟ 4 OH . O gás formado é a amônia.
NH
𝐍𝐇𝟑(𝐠) + H2 O(l)

TESTE 61 – (ITA) Resolução: Alternativa D.


O gás oxigênio foi descoberto em 1771 pelo sueco Scheele, que lhe deu o nome de ar vital. Priestley, em 1774,
estudou os compostos binários definidos como óxidos (compostos que apresentam o oxigênio como o elemento mais
eletronegativo) e, Antoine Laurent Lavoisier (1743-1794) deu o nome de oxigênio que significa gerador de ácidos.

TESTE 62 – (ITA) Resolução: Alternativa A.


a) Falsa, pois se trata do mesmo elemento químico – sódio.
b) Verdadeiro. Os números de elétrons serão distintos, pois o sódio está na sua forma de cátion.
c) Verdadeiro. O Raio iônico co cátio sódio é menor que no seu estado fundamental.
d) Verdadeiro. O número de pótons para ambos será igual a 11.

813
1600 FÍSICO-QUÍMICA APLICADA EXERCÍCIOS COMENTADOS - IME – ITA – OLIMPÍADA

e) Verdadeiro. Podemos dar como exemplo o hidróxido de sódio. Na presença de ´gua vai haver a liberação do cátion
Na+.

TESTE 63 – (ITA) Resolução: Alternativa D.

TESTE 64 – (ITA) Resolução: Alternativa D.


Esta afirmativa pertence ao americano Gilbert Newton Lewis, que em 1923 desenvolvveu uma teoria mais completa
que Bronsted-Lowry sobre ácidos e bases.

TESTE 65 – (ITA) Resolução: Alternativa D.

Base de cálculo:
1,0 mol de álcool + x (mol) de ácido no estado inicial.
0,50 mol de éster no estado de equilíbrio.

Para a resolução deste problema será necessário utilizar a tabela de equilíbrio químico.

CH3 COOH(aq) C2 H5 OH(aq) ⇄ CH3 COOC2 H5(aq) H2 O(l)


Início X 1,0 0 0
Reage 0,50 0,50 0,50 0,50
Equilíbrio (X – 0,50) / 5,0 0,50/5,0 0,50/5,0 0,50/5,0
0,50 0,50
[éster] x [água] ( )x( )
Substitutindo os valores na expressão da constante de equilíbrio: K c = [ácido] x [álcool]
= 5,0
X−0,50
5,0
0,50
( )x( )
5,0 5,0
(0,50)²
4,0 =
(X − 0,50) x 0,50

0,50
4,0 =
(X − 0,50)

Resolvendo a expressão anterior, X = 0,625 mol.

TESTE 66 – (ITA) Resolução: Equações químicas para a produção de ácido nítrico.

catalisador
5
Produção de monóxido de nitrogênio a partir da amônia: 2 NH3(g) + 2 O2(g) ⏞
→ 2 NO(g) + 3 H2 O(l)

1
Produção de dióxido de nitrogênio a partir do monóxido de nitrogênio: 2 NO + 2 O2(g) → 2 NO2 (g)

Produção de ácido nítrico a partir do dióxido de nitrogênio: 3 NO2 (g) + H2 O(l) → 2 HNO3(aq) + NO(g)

TESTE 67 – (ITA) Resolução: Alternativa D.

A adição de um soluto não volátil à um líquido provoca a diminuição da pressão de vapor, processo esse conhecido
como efeito coligativo tonoscópico. A nova pressão é uma fração da anterior que depende da quantidade em matéria
de soluto. Logo podemos escrever a pressão de A como: pA = pºA . XA

TESTE 68 – (ITA) Resolução: Alternativa C.


O gráfico apresenta um processo/comportamento anômalo na fusão, em que a densidade aumenta até uma
determinada temperatura indicada no gráficomo TX, e depois volta a cair, como esperado. As substâncias
apresentadas, a única que apresenta tal comportamento é a água, cuja densidade máxima na fase líquida ocorre em
torno de 4 °C.

814
1600 FÍSICO-QUÍMICA APLICADA EXERCÍCIOS COMENTADOS - IME – ITA – OLIMPÍADA

TESTE 69 – (ITA) Resolução: Alternativa C.


Afirmativa I: Incorreta.
Amostra I: QI = 54 x c x (T – 25)
Amostra II: QII = 126 x c x (75 - T)

Igualando as quantidades de calor entre os sistemas, temos: QI = QII


54 x c x (T – 25) = 126 x c x (75 - T)
T = 60°C (333K)

Analisando a afirmativa II e III: Trata-se de uma afirmativa correta.


Como as trocas de calores são internas e adiabáticas, a quantidade de calor será igual a zero.
Qp = 0 ↔ ∆H = 0 (A pressão constante) e QV = 0 ↔ ∆U = 0 (A volume constante).

Analisando a afirmativa IV: Trata-se de uma afirmativa Incorreta.


∆Suniverso = ∆Ssistema + ∆Svizinhança
A variação de entropia do universo é sempre será positiva, logo: ∆𝑆𝑠𝑖𝑠𝑡𝑒𝑚𝑎 > 0.
Como o processo ocorre sem trocas de calor, processo adiabático, a entropia das vizinhanças será igual a zero
(∆𝑆𝑣𝑖𝑧𝑖𝑛ℎ𝑎𝑛ç𝑎 = 0). Então, a entropia do sistema será maior que zero, ∆𝑆𝑠𝑖𝑠𝑡𝑒𝑚𝑎 > 0.

TESTE 70 – (ITA) Resolução: Alternativa E.

TESTE 71 – (OLIMPÍADA DE QUÍMICA DO RIO DE JANEIRO) Resolução: Alternativa E.


Equação química balanceada pelo método das tentativas:
6 C2H5OH + 4 K2Cr2O7 + 16 H2SO4 → 6 C2H4O2 + 4 Cr2(SO4)3 + 4 K2SO4 + 22 H2O
Dividindo a equação por dois: 3 C2H5OH + 2 K2Cr2O7 + 8 H2SO4 → 3 C2H4O2 + 2 Cr2(SO4)3 + 2 K2SO4 + 11 H2O

TESTE 72 – (ITA) Resolução: Os itens a) e b) estão indicados no diagrama apresentado a seguir:

Item c) Equação química balanceada: N2H4(l) + 2 H2O2(l) → N2(g) + 4 H2O(g).


Item d) Cálculo da variação de entalpia padrão:
∆H°reação = 𝞷.Σ Hprodutos - 𝞷.Σ Hreagentes, onde 𝞷 representa os coeficientes estequiométricos da reação química.
∆H°reação = 𝞷.ΣHprodutos - 𝞷.ΣHreagentes
∆H°reação = [1 x H0N2 + 4 x H0H2O] – [1 x H0N2H4 + 2 x H0H2O]
∆H°reação = [1 x 0 + 4 x (- 241,8)] – [1 x 50,6 + 2 x (- 187,8)] = - 642,2 kJ.mol-1

TESTE 73 – (AUSTRALIAN SCIENCE OLYMPIAD EXAM) Resolução: Alternativa C.


Para que o processo seja espontâneo a partir da equação de Gibbs, a variação da entalpia deverá ser exotérmica (ΔH
< 0) e a variação da entropia deverá ser positiva (ΔS > 0).

TESTE 74 – (ITA - 1950) Resolução:


Agente redutor: HCl (ácido clorídrico)
Agente Oxidante: KMnO4 (permanganato de potássio)
Equação química balanceada: 2 KMnO4 + 16 HCl → 2 KCl + 2 MnCl2 + 8 H2 O + 5 Cl2

815
1600 FÍSICO-QUÍMICA APLICADA EXERCÍCIOS COMENTADOS - IME – ITA – OLIMPÍADA

TESTE 75 – (ITA) Resolução: Alternativa B.


I. H2 - C = C - CH2OH → Função orgânica: álcool
II. H3 - C - CO - CH = CH2 → Função orgânica: cetona
III. H3C - O - CH2 - CH = CH2 → Função orgânica: éter

TESTE 76 – (ITA) Resolução: Alternativa C.


Uma reação entre um ácido e um álcool resulta em um éster e água. Esta reação é definida como reação de
esterificação.
Reação química: Ácido benzoico + etanol → etanoato de benzila + água

TESTE 77 – (ITA) Resolução: alternativa D.

TESTE 78 – (ITA) Resolução: Alternativa D.

a) A combustão da amônia forma gás nitrogênio, que se trata de um gás incolor;


b) Esta reação não é de combustão e sim de decomposição;
c) Não se trata de uma reação de combustão;
d) Por mais que apresenta uma reação de combustão, o monóxido de nitrogênio (óxido nítrico) não apresenta
cor castanha.
e) Verdadeiro.

TESTE 79 – (ITA) Resolução: Alternativa E.


Segundo Dalton o átomo seria: Uma partícula esférica, maciça e indivisível, concordando com a afirmativa I.
Os átomos de um mesmo elemento químico apresentam todas as propriedades iguais, assim sendo a afirmativa II
também está correta. Os átomos reagem entre si, através de reorganização e combinação, em proporções fixas,
estando assim, III e IV corretas.

TESTE 80 – (ITA) Resolução: Na figura A, a velocidade aumenta pouco até que se atinja o complexo ativado. A partir
desse instante, a velocidade da reação aumenta muito. Na figura B, a velocidade da reação cai muito, a partir do
instante em que o calor desnatura a proteína e desativa o catalisador.

TESTE 81 - (ITA) Resolução: Alternativa E.

Semirreação que ocorre na oxidação: 2 Cl− → Cl2 + 2e−

Semirreação que ocorre na redução: Mn+4 + 2e− → Mn+2

Equação química pelo método REDOX: 2 MnO2 + 8 H + + 4 Cl− → 2 Mn+2 + 4 H2 O + 2 Cl2

Dividindo a equação química por dois: MnO2 + 4 H + + 2 Cl− → Mn+2 + 2 H2 O + Cl2

816
1600 FÍSICO-QUÍMICA APLICADA EXERCÍCIOS COMENTADOS - IME – ITA – OLIMPÍADA

TESTE 82 – (ITA) Resolução: Alternativa A.


1
Semirreação que ocorre na oxidação: Cl− → 2 Cl2 + 1e−
Semirreação que ocorre na redução: Mn+7 + 5e− → Mn+2
Equação química pelo método REDOX: 2 MnO− + −
4 + 16 H + 10 Cl → 2 Mn
+2
+ 5 Cl2 + 8 H2 O

g
TESTE 83 – (ITA - MODIFICADA) Resolução: Fórmula molecular do ácido nítrico: HNO3 (63 mol)

nHNO3
Cálculo do número de mol a partir da concentração da quantidade de matéria: [HNO3 ] = V
solução

nHNO3 = [HNO3 ] x Vsolução = 0,84 x 50 x 10−3 = 4,20 x 10−2 mol

mHNO3
Cálculo da massa de ácido nítrico: nHNO3 = <MM>
HNO3
mHNO3 = 4,20 x 10−2 x 63 = 2,65 g

TESTE 84 - (IME) Resolução: Alternativa C.

I = 750 x 80 x 10-4 W = 6,0 W

Considerando que o problema afirma que há uma transmissividade igual a τ = 80%, temos:

P = 6 x W x 0,80 = 4,80 W
P = 4,80 x 15 x 60 J = 4320 J

A potência calculada foi para 0,100 mol de substância pura líquida. Então, para um mol iremos ter uma quantidade de
calor 10 vzes maior. Logo:
Q = 43200 J (43,20 kJ/mol)
J
𝑄𝑟𝑒𝑣𝑒𝑟𝑠í𝑣𝑒𝑙 43200 J
Sendo a entropia definida como ∆𝑆 = 𝑇
, seu valor será igual a: ∆S = 351 K
mol
= 123,08 mol x K

TESTE 85 – (IME) Resolução: Distribuição eletrônica da prata (Ag): 1s2 2s2 2p6 3s2 3p6 4s2 3d10 4p6 5s1 4d10.

O elétron mais externo da distribuição eletrônica está no subnível 5s¹. Então, os números quânticos são deste subnível
são os seguintes:

Número Quântico Principal (n): n = 5; número Quântico Secundário (l): l = 0; número Quântico Magnético (m): m = 0
e número Quântico Spin (s): s = -1/2.

TESTE 86 - (ITA) Resolução: Alternativa D.


A definição de Lewia é mais completa que Brosnted-lowry e de Arrhenius. Base é todo composto que recebe um par
de elétrons e ácido, pela definição de Lewis, é todo composto que doa um par de elétrons.

TESTE 87 – (IME) Resolução: Primeiramente analisaremos o mecanismo II que é bastante direto. Como se trata de
uma reação elementar, a equação da velocidade é dada pela seguinte expressão: vreação = k x [NH2 NO2 ]1 .
Diante da equação fornecida pelo problema, o mecanismo II não condiz com a lei de velocidade sugerida pelo
problema.

Analisando o mecanismo III: Apresentando as equações químicas apresentadas pelo enunciado,


NH2 NO2 + H3 O+ ⇌ NH3 NO+ +
2 + H2 O (equilíbrio rápido) e NH3 NO2 → N2 O + H3 O
+
(etapa lenta), o
+
[NH3 NO2 ] é o intermediário, por isso realizando o estudo do estado estacionário em função do próprio intermediário
temos:

817
1600 FÍSICO-QUÍMICA APLICADA EXERCÍCIOS COMENTADOS - IME – ITA – OLIMPÍADA

d[NH3 NO+
2]
= + k a x [NH2 NO2 ] x [H3 O+ ] − k b x [NH3 NO+ +
2 ] x [H2 O] − k c x [NH3 NO2 ]
dt

Sabendo que a concentração da quantidade de matéria do intermediário é pequena e sua respectiva derivada é igual
a zero, temos:
d[NH3 NO+ 2]
= + k a x [NH2 NO2 ] x [H3 O+ ] − k b x [NH3 NO+ +
2 ] x [H2 O] − k c x [NH3 NO2 ]
dt

0 = + k a x [NH2 NO2 ] x [H3 O+ ] − k b x [NH3 NO+ +


2 ] x [H2 O] − k c x [NH3 NO2 ]

ka x [NH2 NO2 ] x [H3 O+ ]


Isolando a concentração da quantidade de matéria do intermediário: [NH3 NO+
2] = kb x [H2 O]+kc
Equação da velocidade do mecanismo III: vreação = k 𝑐 x [NH3 NO+ 1
2] .

Substituindo a expressão da concentração da quantidade de matéria do intermediário na equação da velocidade de


reação, temos:

vreação = k 𝑐 x [NH3 NO+


2]
1

k a x [NH2 NO2 ] x [H3 O+ ]


vreação = k 𝑐 x
k b x [H2 O] + k c

Levando em consideração que a concentração da quantidade de matéria do [H3 O+ ] é baixa e a atividade da água
líquida é igual a 1, a equação da velocidade da reação pode ser reescrita da seguinte maneira: vreação =
ka x [NH2 NO2 ] x [H3 O+ ]
k𝑐 x kb x [H2 O]+kc

k a x k 𝑐 x [NH2 NO2 ]
vreação =
kb + kc
Portanto para esta lei de velocidade não condiz com o enunciado.

Analisando o mecanismo I: A equação da taxa de velocidade para o mecanismo I é dada pela seguinte equação:
vreação = k 𝑐 x [NHNO− 1 −
2 ] . Observando as etapas e concluindo que a concentração de [NHNO2 ] é o intermediário
e realizando o seu estado estacionário, temos:

d[NHNO−
2]
= + k a x [NH2 NO2 ] x [H2 O] − k b x [NHNO− + −
2 ] x [H3 O ] − k c x [NHNO2 ]
dt
d[NHNO−
2]
Analisando o etado estacionário do intermediário igual a zero, dt
= 0:

d[NHNO−
2]
= + k a x [NH2 NO2 ] x [H2 O] − k b x [NHNO− + −
2 ] x [H3 O ] − k c x [NHNO2 ]
dt

0 = + k a x [NH2 NO2 ] x [H2 O] − k b x [NHNO− + −


2 ] x [H3 O ] − k c x [NHNO2 ]

kb x [NH2 NO2 ] x [H2O]


Isolando a concentração da quantidade de matéria do intermediário: [NHNO−
2] = kb x [H3 O+ ]+kc

Levando em consideração que a concentração da quantidade de matéria do [H3 O+ ] é baixa e a atividade da água
líquida é igual a 1, a equação da velocidade da reação pode ser reescrita da seguinte maneira: vreação =
kb x [NH2 NO2 ] x [H2 O]
k𝑐 x kb x [H3 O+ ]+kc
kb x kc
vreação = x [NH2 NO2 ] = k ′ x [NH2 NO2 ]
kb + kc

k′

818
1600 FÍSICO-QUÍMICA APLICADA EXERCÍCIOS COMENTADOS - IME – ITA – OLIMPÍADA

Portanto para esta lei de velocidade condiz com o enunciado.

TESTE 88 – (ITA) Resposta: Reação de óxido – redução são reações em que há a transferência de elétrons. A
oxidação aumenta o número de oxidação, diminuindo o número de elétrons da espécie química. A redução diminui o
número de oxidação, aumentando o número de elétrons da espécie química.

TESTE 89 – (IME) Resolução: Inicialmente será necessário realizar a distribuição e possível identificação dos
elementos químicos.
11A → 2 ) 8 ) 1 → 11Na = [Ne] 3s (Metal alcalino)
23 23 1

17D → 2 ) 8 ) 7 → 17Cl = [Ne] 3s 3p (Halogênio)


34 34 2 5

10E → 2 ) 8 → 10Ne (Gás nobre)


20 20

A = Na → ligação metálica
D2 = Cl2 → ligação covalente apolar
AD = NaCl → ligação iônica
E → Gás nobre
O neônio é estável nas condições normais de temperatura e pressão não formando compostos.

TESTE 90 – (PREPARATÓRIO - OLIMPÍADA CEARENSE DO ENSINO SUPERIOR DE QUÍMICA) Resolução:


Alternativa C.

HCl(aq) + NaOH(aq) → NaCl(aq) + H2O(l)

Através da estequiomteria, temos:

1 mol de HCl -------- 1 mol de NaOH


nHCl -------------------- nNaOH
nHCl = nNaOH

[HCl] x V1 = [NaOH] x V2

1,0 𝑥 8,0 = [NaOH] x 10


8,0 𝑚𝑜𝑙
[NaOH] = = 0,80
10,0 𝐿
g
Cálculo da concentração comum: C = [NaOH] x < MM >NaOH = 0,80 x 40 = 32 L

TESTE 91 – (OLIMPÍADA CEARENSE DO ENSINO SUPERIOR DE QUÍMICA) Resolução: O resultado mais


próximo é a alternativa B.

O efeito coligativo a ser estudado e analisado é o da crioscopia. O objetivo do problema é determinar o grau de
dissociação (α).

KCl(aq) → K + −
(aq) + Cl(aq)

nsoluto
A partir da propriedade coligativa crioscópica, temos: ∆TC = K c x (kg) x {1 + (n − 1). α}
msolvente

0,373
|−0,345 − 0| = 1,853 x x {1 + (2 − 1). α}
(39 + 35,5) x 50 x 10−3

0,345 0,373
= x (1 + α)
1,853 74,5 x 50 x 10−3

0,373
0,186 = x {1 + α}
3,725

819
1600 FÍSICO-QUÍMICA APLICADA EXERCÍCIOS COMENTADOS - IME – ITA – OLIMPÍADA

0,186 𝑥 3,725
−1=α
0,373

1,86 − 1 = α

α = 0,86 (86%)

TESTE 92 – (ITA) Resolução: Esta lei foi enunciada em 1840 pelo químico suíço Germain Henry Ivanovitch Hess,
nascido em Genebra em sete de agosto de 1802. Este químico estabeleceu que a energia pode ser trocada de uma
forma em outra. Trata-se de um princípio especial à partir da primeira lei da termodinâmica, Eu, Alexandre Vargas
Grillo, autor da obra, chamo esta lei de Hess como um verdadeiro quebra-cabeça, pois partindo de valores já
determinados/calculados de calores de reaçãoo de formação para diversos compostos é possível calcular a entalpia
de uma determinada reação química a partir de outras reações, uma vez que a entalpia é uma função de estado.
“A variação da entalpia envolvida numa reação química depende apenas dos estados físicos de reagentes e produtos,
não importando o caminho da reação”.

TESTE 93 – (IME) Resolução: Realizaremos a distribuição eletrônica por subníveis.

Pertinente deixar como observação que na prova do IME não há disponível a tabela periódica.
Configuração eletrônica: [Xe] 6s2 4f14 5d9
O elemento pertence ao 6° período e no grupo 11 da tabela periódica.

Realizando a distribuição eletrônica por subníveis, temos:

79X → 1s² 2s² 2p6 3s² 3p6 4s² 3d10 4p6 5s² 4d10 5p6 6s² 4f14 5d9

79X → [Xe] 6s² 4f14 5d9

Mas, a configuração mais estável para esse elemento seria a seguinte distribuição eletrônica representada a seguir:

79X → [Xe] 6s1 4f14 5d10

Observando a distribuição eletrônica, conclui-se que se trata de um metal de transição. Este elemento está no sexto
período e no grupo 1B (grupo 11), referindo-se ao elemento químico ouro (Au).

TESTE 94 – (ITA) Resolução:


Item a) A partir do aumento do volume do cilindro, automaticamente a pressão diminui. Somente a segunda reação
química irá sofrer esta influencia, com isso, o equilíbrio químico para a reação II será deslocado para a esquerda,
aumentando o número de mol da espécie química C(g). Com o aumento da espécie química C(g) na primeira reação
química, proveniente da segunda reação química, o equilíbrio será deslocado para a esquerda, aumentando o número
de mol de A(g) e B(g).
Item b) Através da reação química II, aumentando o volume do recipiente (diminuindo a pressão do sistema reacional),
o equilíbrio será deslocado para a esquerda, aumentando o número de mol de C(g) e diminuindo o número de mol de
C(l).
[C]3
Item c) Expressão da constante de equilíbrio para a equação química I: K c = [A] x [B]2. Não muda, pois a constante
de equilíbrio só varia com a temperatura.
[C]3
Item d) Desmenbrando a equação do equilíbrio químico, temos: K c = [A] x [B]2
[C]3
= K c x [A]
[B]2
[C]3
(aumentando, ↑) = K c (constante) x [A] (aumentando, ↑)
[B]2

820
1600 FÍSICO-QUÍMICA APLICADA EXERCÍCIOS COMENTADOS - IME – ITA – OLIMPÍADA

TESTE 95 – (ITA) Resolução: Alternativa E.


H2O2 é o agente oxidante e o Fe+2 é o agente redutor.
Equação química: H2O2 + 2 Fe++ + 2 H+ → 2 Fe3+ + 2 H2O
a) Falso. O número de oxidação do oxigênio no peróxido de oxigênio é igual a -1.
b) Falso. Os coeficientes são os seguintes: a = 2, b = 2 e c = 2.
c) Falso. É o agente oxidante pelo fato de receber elétrons.
d) Falso. O agente redutor é o Fe+2.

TESTE 96 – (ITA) Resolução: Alternativa D.


A lei das proporções múltiplas foi enunciada em 1803 e dizia o seguinte: “Toda substância possui uma proporção
constante, em massa, na sua composição, e a proporção na qual reagem e se formam é constante.” (Joseph Louis
Proust).

TESTE 97 – (ITA) Resolução: Alternativa A.

TESTE 98 – (ITA) Resolução: Alternativa C.

Para a resolução do balanceamento desta equação química, será necessário utilizar o método íon-elétron e sabendo
que o meio é ácido.

Semirreação de oxidação: 3(C2O4)-2 → 6CO2 + 6e-


Semirreação de redução: 6e- + (Cr2O7)-2 + 14H+ → 2Cr3+ + 7H2O
Somando as duas semirreações: (Cr2O7)-2 + (C2O4)-2 + 14H+ → 2Cr3+ + 6CO2 + 7H2O
Soma dos coeficientes estequiométricos: 14 + 1 + 3 + 2 + 6 + 7 = 33

TESTE 99 – (ITA - MODIFICADA) Resolução:

d[A]
a) Em função do reagente A: dt
= −k1 x [A] + k −1 x [M] − k 2 x [M] x [A]

d[M]
b) Em função do intermediário M: dt
= +k1 x [A] − k −1 x [M] − k 2 x [M] x [A]

d[C]
c) Em função do produto C: dt
= + k 2 x [M] x [A]

d) Determinação da equação de taxa (𝑣𝑟𝑒𝑎çã𝑜 ): A etapa lenta está representada pela seguinte equação química:
k2
⏞C
M + A→

Equação da velocidade: vreação = k 2 x [M] x [A]

Como a concentração da quantidade de matéria do intermediário é muito pequena, consequentemente sua derivada
será igual a zero.

d[M]
= +k1 x [A] − k −1 x [M] − k 2 x [M] x [A]
⏟dt
=0

+k1 x [A] − k −1 x [M] − k 2 x [M] x [A] = 0

+k1 x [A] = +k −1 x [M] + k 2 x [M] x [A]

+k1 x [A] = [M] x {k −1 + k 2 x [A]}

821
1600 FÍSICO-QUÍMICA APLICADA EXERCÍCIOS COMENTADOS - IME – ITA – OLIMPÍADA

+k1 x [A]
[M] =
{k −1 + k 2 x [A]}

Equação da velocidade: vreação = k 2 x [M] x [A]

Substituindo a concentração da quantidade de matéria do intermediário na equação da velocidade, temos: vreação =


k 2 x [M] x [A]

+k1 x [A]
vreação = k 2 x x [A]
{k −1 + k 2 x [A]}

k1 x k 2 x [A]²
vreação =
{k −1 + k 2 x [A]}

TESTE 100 – (ITA)

Resolução: Alternativa D.

Equação química reversível: NO2(g) + CO(g) ⇌ CO2(g) + NO(g)

Base de cálculo: pressão parcial de dióxido de nitrogênio igual a X e pressão parcial do monóxido de carbono igual a
3X no estado inicial.

Para a resolução do problema será necessário utilizar a tabela de equilíbrio químico:

NO2(g) CO(g) ⇄ CO2(g) NO(g)


Início X 3X 0 0
Reage Y Y Y Y
Equilíbrio X - Y 3X – Y Y Y

Sabendo que a pressão total do sistema é igual ao somatório das pressões parcias, temos: Ptotal = PNO2(g) +
PCO(g) + PCO2(g) + PNO(g)

32,8 atm = X − Y + 3X − Y + Y + Y

32,8
X= = 8,2 atm
4
mNO
Cálculo da massa de dióxido de nitrogênio: PNO2 x V = <MM> 2 xRxT
NO2

PNO2 x V x < MM >NO2 8,2 𝑥 200 𝑥 46 75440


mNO2 = = = = 2298,32 g (≅ 2,30 kg)
𝑅𝑥𝑇 0,08206 𝑥 400 32,824
PCO x V x <MM>CO 3 𝑥 8,2 𝑥 200 𝑥 28
Cálculo da massa de monóxido de carbono: m𝐶𝑂 = 𝑅𝑥𝑇
= 0,08206 𝑥 400
= 4196,93 g (≅
4,20 kg)

822
1600 FÍSICO-QUÍMICA APLICADA EXERCÍCIOS COMENTADOS - IME – ITA – OLIMPÍADA

TESTE 100 – (ITA) Resolução: Alternativa B. Os critérios usados para agrupar os elementos em uma mesma coluna
foram suas propriedades químicas semelhantes. Entre essas propriedades está a valência constante na combinação
com elementos de referência.

TESTE 101 – (IME) Resolução: Alternativa C.

A partir do conhecimento da segunda lei da termodinâmica, a entropia do universo é maior que zero, ou seja:
∆𝑆𝑢𝑛𝑖𝑣𝑒𝑟𝑠𝑜 > 0.
∆Suniverso = ∆SA + ∆SB

∆Suniverso = ∆SA + ∆SB > 0

∆SA + ∆SB > 0

A temperatura do sistema A vai diminuir, então ∆SA < 0. E como a temperatura do sistema B vai aumentar,
então ∆SB < 0. Portanto, como ∆SA + ∆SB > 0, logo |∆SB | > |∆SA |. Isso significa que a entropia do sistema B
aumenta mais que a entropia do sistema A diminui.

TESTE 102 – (IME) Resolução: Alternativa E.

A tabela construída para apresentar a resposta apresenta a estrutura da molécula orgânica e sua respectiva
hibridização.

H3C - OH sp3
HCOH sp2
CHCl3 sp3
C2H2 sp

TESTE 103 – (IME) Resolução: Alternativa D.

2 isômeros

3 isômeros

1 isômero

O número total de compostos é igual a seis (6) isômeros.

TESTE 104 – (ITA) Resposta:

a) A utilização de um catalisador em um processo químico aumenta a velocidade de uma reação química.


b) A utilização do catalisador em um processo químico não influencia no equilíbrio química, não influenciando nas
massas dos reagentes e produtos.
m
TESTE 105 – (ITA) Resolução: Utilizando a equação dos gases ideais, temos: p x < MM > = V
x RT
450
d x R x T 2,0 x 0,08206 x ⏞
(177 + 273)
p= = = 2,64 atm
< MM > 28,0

823
1600 FÍSICO-QUÍMICA APLICADA EXERCÍCIOS COMENTADOS - IME – ITA – OLIMPÍADA

TESTE 106 – (ITA) Resolução: Alternativa C.


Cálculo da massa de vapor de água, aplicando a equação dos gases ideais: Massa de vapor de água =
p x V x<MM> 1,0 x 22,4 x 18,0
RxT
= 0,08206 x (100+273) = 13,17 g

TESTE 107 – (IME) Resolução: Informação do problema: mPo Po


final = 0,80 x minicial

ln 2 0,693
Cálculo da constante radioativa (λ): λ = t = 3,0
= 0,231 min−1
1/2
mPo
Cálculo do tempo: ln ( final
) = −λ x tempo
mPo
inicial

0,80 x mPo
inicial
ln ( ) = −0,231 x tempo
mPo
inicial

tempo = 0,965 minutos (57,92 s)

TESTE 108 – (ITA) Resolução: Alternativa D.


dois H+ PO−3
4 NH+
4
⏞ ⏞
Nomenclatura: di fosfato de ⏞
hidrogeno ⏞ amônio – Fórmula molecular: (NH4)H2PO4
𝑉
TESTE 109 – (ITA) Resposta: O volume molar é a razão do volume pelo número de mol, ou seja, . Esta definição é
𝑛
muito utilizada a partir da equação dos gases ideais, em que as condições são as seguintes: 1 atm e 0°C (273 K). A
unidade mais usual do volume molar é L x mol-1.
V
Equação dos gases ideais: 𝑝 𝑥 (⏟) = RT
𝑛
𝑉𝑚𝑜𝑙𝑎𝑟

TESTE 110 – (ITA) Resposta: Os principais fatores que influenciam a velocidade de um processo químico são os
seguintes: Temperatura; Pressão; Concentração da quantidade de matéria dos reagentes; Superfície de contato;
Catalisador; Inibidor e Luminosidade.

TESTE 111 – (ITA) Resposta: A necessidade da utilização de catalisadores específicos em um processo químico é
para aumentar a velocidade de uma reação química e na diminuição da energia de ativação. Sem a utilização dos
catalisadores, as reações seriam sim processadas, mas de maneira mais lenta, em um maior tempo.

TESTE 112 – (ITA) Resolução: Situação inicial: pinicial ; Vinicial ; Tinicial = 7 + 273 = 280 K; ninicial .
ninicial 4 𝑥 ninicial
Situação final: pfinal ; Vfinal ; Tfinal = ? ; nfinal = ninicial − 5
= 5
.

Cálculo da temperatura final, levando em consideração que o processo seja isobárico (pressão constante) e
isovolumétrico (volume constante):
4 x ninicial
ninicial x 280 = x Tfinal
5
4 x ninicial
ninicial x 280 = x Tfinal
5
280 x 5
Tfinal = = 350 K (77°C)
4

TESTE 113 – (IME) Resolução: Alternativa D.

As funções orgânicas presentes nesta estrutura são as seguntes: cetona, álcool, amina, éter e éster.

824
1600 FÍSICO-QUÍMICA APLICADA EXERCÍCIOS COMENTADOS - IME – ITA – OLIMPÍADA

Questão 114 – (IME) Resolução: Cálculo da pressão inicial de brometo de iodo no estado inicial, aplicando a equação
dos gases ideais:
8,28
n x R x T ( 207 ) x 0,08206 x (227 + 273)
pIBr = = = 6,56 atm
V 0,250
Tabela de equilíbrio químico:
2 IBr(g) ⇄ I2(g) Br2 (g)
Início 6,56 atm 0 0
Reage 2 x 3,08 atm 3,08 atm 3,08 atm
Equilíbrio 6,56 – 6,16 – 0,40 atm 3,08 atm 3,08 atm

PI2 x PBr2 (3,08)2


Kp = 2 = = 59,29
PIBr (0,40)2

TESTE 115 – (IME - MODIFICADA) Resolução: Dados do problema:


1 litro de solução apresenta 6,50 mol de etanol (C2 H5 OH)
1 cm3 de solução apresenta 0,95 g de solução
𝑚
Cálculo da massa de etanol, a partir do número de mol: 𝑛 = <𝑀𝑀>
m = n x < MM > = 6,50 x 46 = 299 g

Cálculo da massa da solução a partir de 1,0 litro (1000 cm³):


1 cm3 − − − − − − 0,95 g
1 000 cm3 − − − − msolução
msolução = 950 g

msolução = msoluto + msolvente


msolvente = 950g − 299 g = 651 g

nsoluto 6,50 mol mol


Cálculo da molalidade (W): W = (kg) = = 9,98 x 10−3
msolvente 0,651 kg kg
C
Cálculo da concentração comum a partir da concentração da quantidade de matéria: [Etanol] =
<MM>
g
C = [Etanol] x < MM > = 6,50 x 46 = 299
L
m 651
Cálculo do número de mol de solvente (água): nsolvente = <MM> = 18
= 36,17 mol

nsoluto 6,50 6,50


Cálculo da fração molar do soluto (etanol): X etanol = n + nsolvente
= 6,50 + 36,17 = 42,67 = 0,152
soluto

825
1600 FÍSICO-QUÍMICA APLICADA EXERCÍCIOS COMENTADOS - IME – ITA – OLIMPÍADA

TESTE 116 – (IME) Resolução: As reações definidas como eletrolíticas são sim sempre reações definidas como
oxidação que significa perda de elétrons, que sempre ocorre no ânodo, e redução que significa ganho de elétrons, o
que sempre ocorre no cátodo.

TESTE 117 – (IME) Resolução:


0,10
Cálculo do número de mol do gás hidrogênio: nH2 = 2,0 = 0,05 mol

6,4
Cálculo do número de mol do gás oxigênio: nO2 = 32
= 0,20 mol

Cálculo do número de mol total: ntotal = nH2 + nO2 = 0,05 mol + 0,20 mol = 0,25 mol

Item a) Cálculo das frações molares:


nH2 0,05 mol
X H2 = = = 0,20
ntotal 0,25 mol
nO2 0,20 mol
X O2 = = = 0,80
ntotal 0,25 mol

Item b) Cálculo das pressões parciais:


PH2 = X H2 x Ptotal = 0,20 x 760 mmHg
PO2 = X O2 x Ptotal = 0,80 x 760 mmHg

0,25 x 0,08206 x (27+273)


Item c) Cálculo do volume total, a partir do comportamento ideal dos gases: Vtotal = 1,0
=
6,15 L

TESTE 118 – (IME) Resolução: O efeito coligativo a ser estudado é o efeito coligativo ebulioscópico: ∆Teb =
água
K eb x W x i
Soluto: ureia; Solvente: água
O fator de Van´t Hoff é igual a um (i = 1) pelo fato da solução ser molecular.

água nsoluto água mureia


∆Teb = K eb x (kg)
x i = K eb x (kg)
xi
msolvente < MM >ureia 𝑥 msolvente

água 0,450
Cálculo da constante ebulioscópica da água: 0,170 = K eb x 60,06 𝑥 22,50 x 10−3 x 1,0

água 0,230 °C
K eb = = 0,511
0,450 kg x mol

Cálculo da massa molecular do composto desconhecido, a partir do próprio efeito coligativo ebulioscópico:

água
∆Teb = K eb xWxi
1,84
0,170 = 0,511 x x 1,0
45,0 x 10−3 x < MM >soluto

0,940 g
< MM >soluto = −3
= 122,87
7,65 x 10 mol

TESTE 119 – (IME) Resolução: Equação química: 4 C3H5N3O9(ℓ) → 6 N2(g) + 12 CO2(g) + O2(g) + 10 H2O(l)
0 0 0 0
Cálculo da entalpia da reação (∆Hreação ): ∆Hreação = ∑ Hprodutos − ∑ Hreagentes
0
0
∆Hreação = [6 x HN 0
2
+ 12 x HCO 2
+ 1 x HO0 2 + 10 x HH
0
2O
] − [4 x HC03 H5 N3 O9 ]

826
1600 FÍSICO-QUÍMICA APLICADA EXERCÍCIOS COMENTADOS - IME – ITA – OLIMPÍADA

0
∆Hreação = [6 x 0 + 12 x (−394) + 1 x 0 + 10 x (−286)] − [4 x (−354)] = −4728 − 2860 + 1416
kJ
= −6172
mol
mC H5N3 O9 0,600
Cálculo do número de mol de nitroglicerina: nC3 H5 N3 O9 = <MM>3 = 227
= 2,64 x 10−3 mol
C3 H5 N3 O9

Cálculo da quantidade de calor para esta quantidade de mol calculada acima:


1 mol de nitroglicerina --------- (-6172) kJ
2,64 x 10-3 mol ------------------ ∆H
∆H = - 16,31 kJ

TESTE 120 – (GRILLO) Resolução: Cálculo da variação da energia interna: ∆U = Q + W = + 8000 cal +
(−10000) = −2000 cal

Base de cálculo: considerando que para todo processo o número de mol seja igual a 1 mol.
Cálculo da temperatura inicial, a partir da variação da energia interna: ∆U = n x CV x ∆T

−2000 = 1 x (Cp − R) x (Tfinal − Tinicial )


−2000 = 1 x (14 − 2) x (Tfinal − 1000)
−2000 = 12 x (Tfinal − 1000)
Tfinal = 833,33 K

TESTE 121 – (IME) Resolução: Item a) Primeiramente será necessário realizar a distribuição eletrônica por níveis
dos elementos químicos.

9A → 2 ) 7 → elemento químico flúor (F).


17B → 2 ) 8 ) 7 → elemento químico = cloro (Cl).
19 → 2 ) 8 ) 8 ) 1 → elemento químico = potássio (K).
C
AB → ligação química covalente;
AC → ligação química iônica.

Item b) Na camada de valência do elemento químico A, no composto AC, há um elétron, formando A+1. Já em relação
a camada de valência do elemento químico B, no composto BC, há sete elétrons em sua camada de valência,
formando B-1.

Item c) Composto AB → estado gasoso; Composto AC → estado sólido; Composto BC → estado sólido.
Observação: É importante ressaltar que o Instituto Militar de Engenharia – IME não disponibiliza a tabela periódica em
sua prova.

TESTE 122 – (IME) Resolução: Equação química: 2A(g) ⇌ 2B(g) + C(g)

Base de cálculo: Po no estado inicial para o composto gasoso A(g).

Tabela de equilíbrio Químico:


2 A(g) 2 B(g) C(g)
Início P0 0 0

Reage P0α P0α P0α/2
Equilíbrio P0 - P0α P0α P0α/2
p 𝛼
Cálculo da pressão total: pTotal = ⏟
p0 − p0 𝛼 + p⏟ 0
0𝛼 + ⏟
2
pA pB pC

p0 𝛼 2p0 + p0 𝛼 p0 x (2 + α)
pTotal = p0 + = =
2 2 2
827
1600 FÍSICO-QUÍMICA APLICADA EXERCÍCIOS COMENTADOS - IME – ITA – OLIMPÍADA

p0 x (2+α)
Isolando a pressão inicial na equação química: pTotal = 2

2 𝑥 pTotal
p0 =
(2 + α)

Determinação das pressões parciais para os compostos gasosos. Analisando o composto A: pA = p0 − p0 𝛼

2 x pTotal 2 x pTotal
pA = − xα
(2 + α) (2 + α)

2 x pTotal
pA = x (1 − α)
(2 + α)

Analisando o composto B: pB = p0 𝛼

2𝛼 𝑥 pTotal
pB =
(2 + α)
p0 𝛼 2 𝑥 pTotal 𝛼 𝛼.pTotal
Analisando o composto C: pC = 2
= (2+α)
𝑥 2
= (2+α)

Expressão da constante de equilíbrio em função das pressões parciais dos componentes gasosos: 2 A (g) ⇌ 2 B(g) +
C(g)
PB2 x PC
Kp =
PA2
2α x pTotal 2 α.p Total )
P2B x PC ( (2+α)
) x ( (2+α)
Substituindo as pressões parciais na constante de equilíbrio: K p = P2A
= 2 =
2 x pTotal
( (2+α) x (1−α))

4α² x p2
Total xα x pTotal
(2+α)² (2+α) α³ x pTotal
4 x p2
= (2+α) x (1−α)²
Total x (1−α)²
(2+α)²
α³ x pTotal
Kp =
(2 + α) x (1 − α)²

α³ x p
Isolando a pressão total do sistema reacional na equação anterior: K p = (2+α) x (1−α)²
Total

Kp 𝑥 (2+α) x (1−α)²
pTotal = α³
.

828
1600 FÍSICO-QUÍMICA APLICADA EXERCÍCIOS COMENTADOS - IME – ITA – OLIMPÍADA

TESTE 123 – (IME - ADAPTADA) Resolução:


Item a)
Invertendo a semirreação do níquel: Ni°(s) → Ni2+(aq) + 2e- ΔG1° = - 2.F.(+ 0,25)
Multiplicando a semirreação da prata por 2: 2 Ag (aq) + 2e → 2 Ag°(s) ΔG2° = - 2.F.(+ 0,80)
+ -

Equação global:
Ni°(s) → Ni2+(aq) + 2e-
2 Ag+(aq) + 2e- → 2 Ag°(s) +
Ni°(s) + 2 Ag+(aq) → Ag°(s) + Ni2+(aq) (Equação global da pilha) ΔG°Total = - 2.F.E°

Item b) O sentido do fluxo de elétrons sai do ânodo (eletrodo de Níquel) para o cátodo (eletrodo de prata).

Item c)

ΔG°Total = ΔG1°Ni + ΔG2°Ag


- 2.F.E° = - 2.F.(+ 0,25) + {- 2.F.(+ 0,80)}
E° = + 0,25 + 0,80
E° = + 1,05 V
96500 C
Item d) ∆G0 = −2 mol de elétrons x mol de elétrons x 1,05 V = −202650 J

Item e) ∆G0 = −R x T x lnK eq

−202650 = −8,314 x 298 x lnK eq

−202650
lnK eq = = 81,79
−2477,572

K eq = 𝑒 +81,79

Teste 124 – Resolução: A partir da primeira lei da termodinâmica, temos: dU = dQ + dW

dU = dQ – [p.dV + V.dp]
dU = dQ – p.dV - V.dp (Equação 1)

A partir da definição da função de estado entalpia, temos: dH = dU + pV


dH = dU + p.dV + V.dp (Equação 2)

Substituindo a equação 1 na equação 2, temos: dH = dU + p.dV + V.dp

dH = dQ – p.dV - V.dp + p.dV + V.dp


dH = dQp .

Logo, a função entalpia é igual a quantidade de calor â pressão constante.

TESTE 125 – Resolução:


Item a) Utilizando a equação de Gibbs padrão, temos: +58540 = −8,314 x (25 + 273) x ln K p
+58540
ln K p =
−8,314 x 298
1/2
ln 𝑝𝑂2 = −23,63

1/2
𝑝𝑂2 = 𝑒 −23,63 = 5,48 𝑥 10−11

829
1600 FÍSICO-QUÍMICA APLICADA EXERCÍCIOS COMENTADOS - IME – ITA – OLIMPÍADA

𝑃𝑂2 = 3,0 𝑥 10−21 𝑎𝑡𝑚

Item b) Para maior produção de oxigênio molecular, utilizando o princípio de Le Chatelier, temos:
Diminuindo a pressão total do sistema do sistema reacional, o equilíbrio é deslocado para a direita da reação direta;
Aumentando a temperatura o equilíbrio é deslocado para a direita, uma vez que se trata de uma reação endotérmica.

TESTE 126 – (GRILLO) Resolução:


0
Item a) Cálculo da variação de entalpia padrão: ∆Hreação = ∑ Hprodutos − ∑ Hprodutos
0
kJ
∆Hreação = {3 x 0 + 2 x (−241,8)} − {1 x (−296,9) + 1 x (−20,1)} = −166,6
mol
0
Item b) Cálculo da variação da entropia padrão: ∆Sreação = ∑ Sprodutos − ∑ Sprodutos
0
J
∆Sreação = {3 x (31,9) + 2 x (188,8)} − {1 x (248,5) + 1 x (207,6)} = +17,2
mol. K
0 0 0
Item c) Cálculo da energia livre de Gibbs padrão: ∆Greação = ∆Hreação − T. ∆Sreação
0
J
∆Greação = −166600 − (25 + 273) x 17,2 = −171725,6
mol
0
Cálculo da constante de equilíbrio a 25°C: ∆Greação = −R x T x lnK
−171725,6 = −8,314 x (25 + 273) x lnK 25°C

−171725,6
lnK 25°C = = 69,31
− 2477,572

K 25°C = e(69,31) = 1,26 x 1030

K25°C
Item d) Cálculo da constante de equilíbrio a 150°C (𝐾 150°𝐶 ), aplicando a equação de Van´t Hoff: ln (K150°C ) =
∆H0 1 1
x[ − ]
R (150+273) (25+273)
30
1,26 x 10 − 166600 1 1
ln ( 150°C )= x[ − ]
K 8,314 423 298

1,26 x 1030
ln ( ) = −20038,49 x 9,91 x 10−4
K150°C

1,26 x 1030
ln ( ) = +19,87
K150°C

1,26 x 1030
= e(+19,87)
K150°C

1,26 x 1030 = 4,26 x 108 x K150°C

1,26 x 1030
K150°C = = 2,96 x 1021
4,26 x 108
2
RxT [Fe+2 ] x a2H2O
TESTE 127 – Resolução: Utilizando a equação de Nernst: E = E 0
− nxF x ln a x PO2 x [H+ ]4
Fe

Cálculo do potencial fora dos padrões:

830
1600 FÍSICO-QUÍMICA APLICADA EXERCÍCIOS COMENTADOS - IME – ITA – OLIMPÍADA

8,314 x (25 + 273) [10−3 ]2 x (1)²


E = +1,67 − x ln
4 x 96500 1 x 0,10 x [10−3 ]4

8,314 x 298 10−6


E = +1,67 − x ln −13 = +1,67 − 6,42 x 10−3 x ln 10+7 = +1,57 V
386000 10

TESTE 128 – (IME) Resolução: Base de cálculo: n (mol) inicial de tetróxido de dinitrogênio.
Tabela de equilíbrio químico:
N2O4(g) ⇄ 2 NO2(g)
Início n 0
Reage nα 2nα
Equilíbrio n-nα 2nα

Cálculo do número de mol total (ntotal): ntotal = n − nα + 2nα = n + nα

Expressão das pressões parciais dos compostos gasosos:

nN2 O4 n − nα n(1 − α) (1 − α)
PN2 O4 = x Ptotal = x Ptotal = x Ptotal = xP
ntotal n + nα n(1 + α) (1 + α) total

nNO2 2nα 2nα 2α


PNO2 = x Ptotal = x Ptotal = x Ptotal = xP
ntotal n + nα n(1 + α) (1 + α) total

P2NO2
Expressão da constante de equilíbrio em função das pressões parciais: K p = P . Substituindo os valores das
N2 O4
pressões parciais na expressão da constante de equilíbrio em função das pressões parciais (Kp), temos a seguinte
relação:
2
2α 4α²
2
PNO2 [ x P total ] x P2 4α2 x Ptotal
(1 + α) (1 + α)2 total
Kp = = = =
PN2 O4 (1 − α) (1 − α) 1 − α2
[ x Ptotal ] x Ptotal
(1 + α) (1 + α)
4α2 𝑥 1,0
Substituindo os valores numéricos na equação da constante de equilíbrio: = 0,140
1−𝛼 2

Resolvendo a equação: 𝛼 = 0,183


1−0,183 2 𝑥 0,183
Determinação das frações molares: X N2 O4 = 1+0,183 e X NO2 = 1+0,183
.

TESTE 129 – (ITA) Resolução: Alternativa C.


mO
Equação dos gases ideais para o gás ozônio: pV = 483 RT (Equação 1)
mO2
Equação dos gases ideais para o gás oxigênio: pV = 32
RT (Equação 2)
mO
3 RT
pV
Dividindo a primeira equação pela segunda equação, temos: = 48
4,0
pV RT
32
4,0 mO3
=
32 48
48
m O3 = = 6,0 g
8

831
1600 FÍSICO-QUÍMICA APLICADA EXERCÍCIOS COMENTADOS - IME – ITA – OLIMPÍADA

TESTE 130 – (ITA) Resolução: Para a combustão completa do hidrocarboneto genérico CαHβ será realizado o
balanceamento por tentativa.
β β
Cα Hβ + (α + ) O2 → α CO2 + H2 O
4 2

Considerando que o ar atmosférico apresenta composição de 20% de O2 e 80% de N2, temos:


β
(α + ) mol de O2 − − − − − 20%
4
nN2 − − − − − − − − − − − − 80%
nN2 = 4α + β

Logo a equação química com a presença do nitrogênio atmosférico vai ficar da seguinte maneira:
β β
Cα Hβ + (α + ) O2 + (4α + β)N2 → α CO2 + H2 O + (4α + β)N2
⏟ 4 2
Ar atmosférico

TESTE 131 – (ITA) Resolução: Alternativa E.


15
Item a) Analisando a reação de combustão do benzeno: C6 H6 + 2
O2 → 6 CO2 + 3 H2 O

Item b) Analisando a reação de combustão do ciclohexano: C6 H12 + 9 O2 → 6 CO2 + 6 H2 O

Item c) Analisando a reação de combustão da ciclohexanona: C6 H10 O + 8 O2 → 6 CO2 + 8 H2 O


17
Item d) Analisando a reação de combustão do ciclohexano: C6 H10 + O2 → 6 CO2 + 5 H2 O
2

19
Item e) Analisando a reação de combustão do hexano: C6 H14 + O2 → 6 CO2 + 7 H2 O
2
Quanto maior o número de moléculas de di[óxido de carbono e água, maior será a variação da entalpia. Diante disso,
o hexano é o molécula (hidrocarboneto) que libera maior quantidade de energia.

TESTE 132 - (ITA - MODIFICADA) Resolução: Cálculo do número de mol a partir da concentração da quantidade de
n
matéria: [NaOH] = NaOH
Vsolução
nNaOH = [NaOH] x Vsolução = 0,946 x 50 x 10−3 = 4,73 x 10−2 mol

m
Cálculo da massa de hidróxido de sódio, a partir do número de mol: nNaOH = <MM>
NaOH
NaOH
mNaOH = 4,73 x 10−2 x 40 = 1,892 g

TESTE 133 – (ITA) Resolução: Primeiro será necessário calcular a massa de água para um litro de solução.

0,99 g de H2 O − − − − − 10−3 L
m H2 O − − − − − − − − − 1 L
mH2 O = 990 g
990
nH2O mol
Cálculo da concentração da quantidade de matéria para um litro de solução: [H2 O] = V = 1
18
= 55 L
solução

TESTE 134 – (ITA) Resolução: Alternativa B.


O abaixamento da temperatura de solidificação ou também conhecido como crioscopia ocorre a partir do momento
em que um determinado soluto não volátil é adicionado em um solvente, modificando as propriedades deste solvente.

TESTE 135 – (ITA) Resolução: Alternativa E.

832
1600 FÍSICO-QUÍMICA APLICADA EXERCÍCIOS COMENTADOS - IME – ITA – OLIMPÍADA

O gráfico apresentado pela questão apresenta temperatura constante durante a transição sólido-líquido, o que
caracteriza uma mistura eutética, pois apresenta fusão constante e ebulição variando.

TESTE 136 – (GRILLO) Resolução:

Item a) Equação química: Na2CO3(aq) + H2SO4(aq) → Na2SO4(aq) + H2O(l) + CO2(g). O sal formado pela reação entre o
carbonato de sódio e o ácido sulfúrico é o sulfato de sódio.

Item b) Equação química: Na2CO3(aq) + H2SO4(aq) → Na2SO4(aq) + H2O(l) + CO2(g)

Determinação do reagente limitante:

Analisando o carbonato de sódio: nNa2 CO3 = [Na2 CO3 ] x Vsolução = 0,25 x 33 x 10−3 =
−3
8,25 x10 mol (Reagente limitante)

CH SO4
Analisando o ácido sulfúrico: nH2 SO4 = [H2 SO4 ] x Vsolução = (<MM>
2
) x Vsolução
H2 SO4
225
nH2 SO4 = ( ) x 20 x 10−3 = 4,60 x 10−2 mol
98

Na2CO3(aq) + H2SO4(aq) → Na2SO4(aq) + H2O(l) + CO2(g)

1 mol de Na2CO3(aq) ------------ 1 mol de Na2SO4(aq)


nNa2CO3(aq) -------------------------- nNa2SO4(aq)
nNa2CO3(aq) = nNa2SO4(aq)

[Na2 CO3 ] x Vsolução = [Na2 CO3 ] x Vsolução


0,25 mol. L−1 x 33 x 10−3 = 2,0 x Vsolução
Vsolução = 4,125 x 10−3 mL

Item c) Cálculo do número de mol de dióxido de carbon, a partir da solução de carbonato de sódio:
Na2CO3(aq) + H2SO4(aq) → Na2SO4(aq) + H2O(l) + CO2(g)
1 mol de Na2CO3(aq) -------------------------------- 1 mol de CO2(g)
nNa2CO3(aq) --------------------------------------------- nCO2(g)
nNa2CO3(aq) = nCO2(g)

[Na2 CO3 ] x Vsolução = nCO2


0,25 mol. L−1 x 33 x 10−3 = nCO2
nCO2 = 8,25 x 10−3 mL
nCO2 x R x T [8,25 x 10−3 x 0,08206 x (127+273)] 0,27
Cálculo do volume: VCO2 = = = = 0,18 L (180 cm³)
p 1,50 1,50

TESTE 137 – (IME) Resolução: Semirreação que ocorre no cátodo: Al+3 −


(aq) + 3 e → Al(s)
96500 𝐶
1 mol de Al − − − 27 g − − − 3 mol de elétrons x 𝑥 0,90
1 𝑚𝑜𝑙 𝑑𝑒 𝑒𝑙é𝑡𝑟𝑜𝑛𝑠
mAl − − − (6700 x 24 x 3600) C

27 𝑥 6700 𝑥 24 𝑥 3600
mAl = = 59987,56 𝑔 (≅ 60,0 kg)
3 𝑥 96500 𝑥 0,90

TESTE 138 – Resolução: Base de cálculo: n (mol) inicial de tetróxido de dinitrogênio.

Tabela de equilíbrio químico:


N2O4(g) ⇄ 2 NO2(g)

833
1600 FÍSICO-QUÍMICA APLICADA EXERCÍCIOS COMENTADOS - IME – ITA – OLIMPÍADA

Início n 0
Reage nα 2nα
Equilíbrio n-nα 2nα

Cálculo do número de mol total (ntotal): ntotal = n − nα + 2nα = n + nα

Expressão das pressões parciais:

nN2 O4 n − nα n(1 − α) (1 − α)
PN2 O4 = x Ptotal = x Ptotal = x Ptotal = xP
ntotal n + nα n(1 + α) (1 + α) total
nNO2 2nα 2nα 2α
PNO2 = x Ptotal = xP = xP = xP
ntotal n + nα total n(1 + α) total (1 + α) total

Substituindo os valores das pressões parciais na expressão da constante de equilíbrio em função das pressões
parciais (Kp), temos a seguinte relação:
2
2α 4α²
2
PNO [ x Ptotal ] x 𝑃2 4α2 𝑥 Ptotal
2 (1 + α) (1 + α)2 𝑡𝑜𝑡𝑎𝑙
Kp = = = =
PN2 O4 (1 − α) (1 − α) 1 − 𝛼2
[ x Ptotal ] x Ptotal
(1 + α) (1 + α)

TESTE 139 – (GRILLO) Resolução: Cálculo do trabalho realizado: W = −p x ∆V


W = −200000 x (120 − 40) = −16000 𝐽 (− 16 𝑘𝐽)

Sabendo que o processo é isotérmico (temperatura constante, TA = TB), tanto a variação da energia interna quanto a
variação da entalpia são iguais a zero, ∆U = 0 e ∆H = 0.

Pela primeira lei da termodinâmica, ∆U = Q + W.

0=Q+W
Q = − W = − (−16000) = +16000 J

TESTE 140 – (OLIMPÍADA DE QUÍMICA DO RIO DE JANEIRO) Resolução: Alternativa D.


seco
Cálculo da pressão do gás desconhecido seco em atmosfera (atm): Pgás = 752,82 mmHg − 31,82 mmHg =
721 mmHg
760 mmHg
= 0,949 atm

Cálculo da massa molecular do gás desconhecido, a partir da equação dos gases ideais:

massa x R x T 0,157 x 0,08206 x (30 + 273) g


< MM >= = = 30,47
pxV 0,949 x 0,135 mol

TESTE 141 – Resolução: Equação química hipotética: 2A(g) + B(g) → 2C(g) + 3D(g).

Definindo Vreação = velocidade da reação; VA = velocidade em relação ao reagente A; VB = velocidade em relação ao


reagente B; VC = velocidade em relação ao reagente C e VD = velocidade em relação ao reagente D.

Relação matemática:

1 d[A] d[B] 1 d[C] 1 d[D]


vreação = − x =− =+ x =+ x
2 dt dt 2 dt 3 dt

834
1600 FÍSICO-QUÍMICA APLICADA EXERCÍCIOS COMENTADOS - IME – ITA – OLIMPÍADA

1 d[A] d[B] 1 d[C] 1 d[D]


vreação = − x =− =+ x =+ x
2 dt dt 2 ⏟dt 3 dt
kmol
1,0
dm3 x min

1 d[A] d[B] 1 kmol 1 d[D]


vreação = − x =− = + x 1,0 3
=+ x
2 dt dt 2 dm x min 3 dt
kmol d[A] kmol d[B] kmol d[D] kmol
vreação = 0,50 ; = −1,00 ; = −0,50 e = +1,50
dm3 x min dt dm3 x min dt dm3 x min dt dm3 x min

TESTE 142 – (GRILLO) Resolução: Soluto: cloreto de magnésio e Solvente: água. O efeito coligativo a ser estudado
é o tonoscópico.
∆P ∆P <MM>solvente nsoluto
Cálculo da pressão do abaixamento relativo ( ): = x (kg) xi
P° P° 1000 m solvente

∆P 18 5
Substituindo os valores na equação: P°
= 1000 x 60 𝑥 0,500 x 1,0

∆P 90
= = 0,0030
P° 30000
∆P
Cálculo do abaixamento da pressão máxima de vapor: P°
= 0,0030

∆P
= 0,0030
32 torr

∆P = 32 torr x 0,0030 = 0,096 torr

TESTE 143 – (MESTRE JOÃO ROBERTO DA PACIÊNCIA NABUCO) Resolução: Equação Química:
H2 O
⏞ Mg +2
MgCl2(aq) → −
(aq) + 2 Cl(aq)

Cálculo do fator de van’t Hoff: i = 1 + (n − 1). α

i = 1 + (3 − 1) x 0,80 = 2,60

Soluto: cloreto de magnésio


Solvente: água
P−P°
O efeito coligativo a ser estudado é o tonoscópico. Cálculo da pressão máxima da solução (p): P°
=
<MM>solvente nsoluto
1000
x (kg) xi
m solvente

32 − P 18 19
Substituindo os valores na equação: 32
= 1000 x 95 𝑥 0,360 x 2,60

32−P
= 0,026
32

P = 32 − (32 x 0,026) = 31,17 mmHg

TESTE 144 – (GRILLO) Resolução: Cálculo do número de de ar atmosférico (1), aplicando a equação dos gases
pxV 10 x 20000 200000
ideais: nar1 = R x T = 0,08206 x (227+273) = 41,03 = 4874,48 mol

835
1600 FÍSICO-QUÍMICA APLICADA EXERCÍCIOS COMENTADOS - IME – ITA – OLIMPÍADA

pxV
Cálculo do número de de ar atmosférico (2), aplicando a equação dos gases ideais: nar2 = R x T =
2 x 200000
0,08206 x (127+273)
= 12136,20 mol

Cálculo do número de mol total: ntotal = nar1 + nar2 = 4874,48 mol + 12136,20 mol = 17060,69 mol

ntotal x R x T 17060,69 x 0,08206 x (327+273)


Cálculo da pressão do sistema reacional: ptotal = Vtotal
= (20000+200000)
=
17060,69 x 0,08206 x 600
220000
= 3,82 atm
TESTE 145 – Resolução: A partir da definição de entalpia, temos: dH = n x cp x Dt

473
1500
∆H = ∫ ( ) x (29,96 + 4,18 x 10−3 T). dT
373 32

473
∆H = 46,875 x ∫ (29,96 + 4,18 x 10−3 T). dT
373

473

∆H = 46,875 x {[29,96 x T]473
373 + 4,18 x 10
−3
x( ) ]}
2 373
(473)² (373)²
= 46,875 x 29,96 x (473 − 373) + 4,18 x 10−3 x [ − ] = 140, 61 kJ
2 2

TESTE 146 – (ITA) Resolução:


Solução de nitrato de prata: AgNO3(aq) → Ag + −
(aq) + NO3(aq)
Semirreação que ocorre no cátodo: Ag + −
(aq) + 1 e → Ag (s)
60 s
Determinação do tempo para segundos: Tempo = 32 minutos x 1 minuto + 10 s = 1930 s
Cálculo da carga em coulombs:
96500 C
1 mol de Ag (s) − − − 108 g − − − − 1 mol de elétrons x
mol de elétrons
1,08 g − − − − Q
Q = 965 C
965
Cálculo da corrente elétrica (i): i = = 0,50 A.
1930

TESTE 147 – (ITA) Resolução: Alternativa D.


Invertendo a primeira semirreação: C2H5OH + 3 H2O = 2 CO2 + 12 H+ + 12e- ∆G°I = - 12.F.(-
0,085)

Multiplicando por três a segunda semirreação, temos: 3O2 + 12 H+ + 12e- = 6 H2O ∆G°II = - 12.F.(+
1,229)

Somando as duas semirreações, temos:


C2H5OH + 3 H2O = 2 CO2 + 12 H+ + 12e- ∆G°I = - 12.F.(- 0,085)
3O2 + 12 H+ + 12e- = 6 H2O ∆G°II = - 12.F.(+ 1,229) +
C2H5OH + 3O2 = 2 CO2 + 3 H2O ∆G°III = ?

∆G°III = - 12.F.(- 0,085) + [- 12.F.(+ 1,229)]


- 12.F.E° = - 12.F.(- 0,085) + [- 12.F.(+ 1,229)]
E° = - 0,085 + 1,229 = + 1,144 V

Analisando os itens, temos:

836
1600 FÍSICO-QUÍMICA APLICADA EXERCÍCIOS COMENTADOS - IME – ITA – OLIMPÍADA

a) Correto. Trata-se sim uma reação exotérmica, por se tratar de uma reação de combustão do etanol.
b) Correto. Trata-se sim de uma célula combustível, pelo fato do combustível que representa o etanol estar
sendo queimado na presença do comburente (oxigênio molecular).
c) Correto. Conforme pode ser visualizado acima, o potencial é igual a + 1,144 volts.
d) Incorreto. Sabendo que a variação da energia livre de Gibbs padrão é o trabalho máximo,
∆G° = -Wmáximo, temos que:
∆G° = - n.F.E°
∆G° = - 12 mol de e- x (96500 C/mol e-) x (+ 1,144)
∆G° = - 1324,75 kJ. Este valor corresponde para cada mol de etanol.
e) Correto. A célula converte sim energia livre da reação de combustão em energia elétrica, o que caracteriza
uma célula à combustível.

TESTE 148 – (ITA) Resolução: Alternativa C.

Equação química: 4 FeS2(s) + 11 O2(g) → 2 Fe2O3(s) + 8 SO2(g)

Cálculo do número de mol de gás nitrogênio, a partir da composição do ar atmosférico:


11 mol de O2 − − − − 20%
nN2 − − − − − − − − 80%
nN2 = 44 mol

Equação química com a presença do gás nitrogênio: 4 FeS2(s) + 11 O2(g) + 44 O2(g) → 2 Fe2O3(s) + 8 SO2(g) + 44 O2(g)
mFeS2 12
Cálculo do número de mol de pirita: nFeS2 = = = 0,10 𝑚𝑜𝑙
<MM>FeS2 120

Pela relação estequiométrica a partir da equação química:


4 mol de FeS2 − − − − − − − − − (8 + 44) mol de gases
0,10 mol de FeS2 − − − − − − − − ngases
0,10 x 52
ngases = = 1,3 mol
4
L L
Cálculo do volume dos gases produzidos nas CNTP: Vgases = ngases x 22,4 = 1,30 mol x 22,4 =
mol mol
29,12 L

TESTE 149 – (ITA) Resolução: Alternativa C.


I) Falso. Como se trata de um soluto iônico haverá um fator de correção definido como fator de van’t Hoff, o que com
isso aumenta a pressão osmótica.
II) Verdadeiro. Com a adição de um soluto não volátil, a pressão de vapor da solução diminui.
III) Verdadeiro. Com a adição de um soluto não-volátil, a temperatura de fusão (solidificação) da solução vai diminuir.
IV) Falso. Com a adição de um soluto não volátil , a temperatura de ebulição de uma solução aumenta.
V) A densidade de um solto iônico aumenta.

TESTE 150 – (ITA) PERGUNTA - Resolução: A afirmativa I está errada. Se tratando de um sólido iônico, a equação
matemática da pressão osmótica fica da seguinte maneira:

concentração concentração
π = [da quantidade] x R x T x i = [da quantidade] x R x T x [1
⏟ + (n − 1). α]
de matéria de matéria fator de van′ t Hoff
(FATOR DE CORREÇÃO)

Este fator de correção aumenta a pressão osmótica.

TESTE 151 – (ITA) Resolução: Alternativa C.


Ajustando e organizando as semirreações de reduções apresentadas pela questão:

837
1600 FÍSICO-QUÍMICA APLICADA EXERCÍCIOS COMENTADOS - IME – ITA – OLIMPÍADA

+3 +2
M(aq) + 1e− → M(aq) ∆G° = −1 x F x (+ 0,80)
+2 0
M(aq) + 2e− → M(aq) ∆G° = −2 x F x (− 0,20)

Somando as semirreações apresentadas acima, temos:


+3 +2
M(aq) + 1e− → M(aq) ∆G° = −1 x F x (+ 0,80)
+2 0
M(aq) + 2e− → M(aq) ∆G° = −2 x F x (− 0,20)
+3 0
M(aq) + 3e− → M(aq) ∆G° = −1 x F x (+ 0,80) + [−2 x F x (− 0,20)]
Determinação do potencial padrão (E°): −3 x F x E ° = −1 x F x (+ 0,80) + [−2 x F x (− 0,20)]
−3 x E ° = −0,80 + 0,40
−0,80 + 0,40 −0,40
E° = = = +0,133 V
−3 −3

TESTE 152 – (ITA) Resolução: Alternativa E.


Realizando o balanceamento eplo métdo redox (redução-oxidação), o permanganato é o agente oxidante e o peróxido
de hidrogênio (água oxigenada) é o agente redutor. Então, o balanceamento fica da seguinte maneira, conforme pode
ser observado pela equação abaixo: 2 MnO− +
4 + 6 H + 5 H2 O2 → 2 Mn
+2
+ 8 H2 O + 5 O2
Logo, os coeficientes estequiométricos são os seguintes: x = 2; y = 6; z = 2 e w = 8.

TESTE 153 – (IIT) Resolução: Alternativa B.

Equação Química: Ba(NO3 )2(aq) → Ba+2 −


(aq) + 2NO3(aq)

Aplicando a equação de Van’t Hoff: i = 1 + (n − 1). α

Cálculo do grau de dissociação: 2,74 = 1 + (3 − 1). α

1,74
α= = 0,87 (87%)
2

TESTE 154 – (OLIMPÍADA BRASILEIRA DE QUÍMICA) Resolução: Alternativa A.

Equação química: ZnS(s) + 3/2 O2(g) + CO(g) → Zn(s) + SO2(g) + CO2(g)

1 mol de ZnS(s) -------------------- 1 mol de Zn(s)


96,4 g − − − − − − − − − − − 65,4 x 0,80
2,425 kg x 0,80 − − − − − − − mZn

2,425 x 0,80 x 65,4 0,80


mZn = = 1,042 kg
96,4

TESTE 155 – Resolução: Organizando as semirreações apresentadas pelo exercício:


Cu+2 −
(aq) + 2e → Cu(s) E 0 = +0,340 V

Cu+1 −
(aq) + 1e → Cu(s) E 0 = +0,552 V

Permanecendo a primeira semirreação: Cu+2 −


(aq) + 2e → Cu(s) ∆G0 = −2 x F x (+ 0,340)

Invertendo a segunda semirreação: Cu(s) → Cu+


(aq) + 1e

∆G0 = −1 x F x (− 0,552)

Somando as duas semirreações, temos:

Cu+2 − 0
(aq) + 2e → Cu(s) ∆G = −2 x F x (+ 0,340)

838
1600 FÍSICO-QUÍMICA APLICADA EXERCÍCIOS COMENTADOS - IME – ITA – OLIMPÍADA

Cu(s) → Cu+
(aq) + 1e

∆G0 = −1 x F x (− 0,552) +
Cu+2 − + 0
(aq) + 1e → Cu(aq) ∆G = −1 x F x E° = −2 x F x (+ 0,340) + [−1 x F x (− 0,552)]
−1 x F x E° = −2 x F x (+ 0,340) + [−1 x F x (− 0,552)]

−0,680 + 0,552
E° = = + 0,128 V
−1
0 0
Cálculo da constante de equilíbrio: ∆Geletroquímico = ∆Gtermoquímico

− 1 x 96500 x 0,128 = − 8,314 x (25 + 273) x lnK

− 1 x 96500 x 0,128 12352


lnK = = = + 4,98
− 8,314 x 298 2477,572

K = 𝑒 +4,98 = 145,47

TESTE 156 – (UNIVERSITY OF WATERLOO DEPARTMENT OF CHEMISTRY) Resolução: Alternativa B.


Aplicando a lei de Hess.
Multiplicando a primeira equação termoquímica por três: 3 H2(g) + 3/2 O2(g) → 3 H2O(g) ∆H° = 3 x (−241,8) kJ.mol−1
Multiplicando a segunda equação termoquímica por dois: 2 C(s) + 2 O2(g) → 2 CO2(g) ∆H° = 2 x (−393,5) kJ.mol−1
Invertendo a terceira equação termoquímica: C2H6(g) → 2 C(s) + 3 H2(g) ∆H° = + 84,68 kJ.mol−1

Somando as três equações termoquímicas:

3 H2(g) + 3/2 O2(g) → 3 H2O(g) ∆H° = 3 x (−241,8) kJ.mol−1


2 C(s) + 2 O2(g) → 2 CO2(g) ∆H° = 2 x (−393,5) kJ.mol−1
C2H6(g) → 2 C(s) + 3 H2(g) ∆H° = + 84,68 kJ.mol−1 +
C2H6(g) + 7/2 O2(g) → 2 CO2(g) + 3 H2O(g) ∆H° = 3 x (−241,8) + 2 x (−393,5) + 84,68 = - 1427,72 kJ.mol-1

TESTE 157 – Resolução: Equação química: PCl5(g)  PCl3(g) + Cl2(g)


Base de cálculo: n mol inicial de PCl5(g)
Tabela de equilíbrio químico:

PCl5(g) (mol)  PCl3(g) (mol) Cl2(g) (mol)

Início n 0 0
Reage nα nα nα
Equilíbrio n - nα nα nα

Cálculo do número de mol total do sistema: ntotal = n – nα + nα + nα = n + nα = n x (1 + α)

Determinação das pressões parciais para cada composto gasoso:


n x (1 − α)
PPCl5 = x ptotal
n x (1 + α)
nxα
PPCl3 = PCl2 = x ptotal
n x (1 + α)
PPCl3 x PCl2
Expressão da constante de equilíbrio (Kp): K p = PPCl5
2
α² x Ptotal (1 + α)
Kp = x
(1 + α)² (1 − α) x Ptotal
α² x Ptotal 1
Kp = x
(1 + α) (1 − α)

839
1600 FÍSICO-QUÍMICA APLICADA EXERCÍCIOS COMENTADOS - IME – ITA – OLIMPÍADA

α² x Ptotal
Kp =
1 − α²
α² x 2,0
Substituindo os valores na equação acima: 1,78 = 1−α²

Resolvendo a equação acima, o grau de dissociação é igual a 0,686, o que corresponde a 68,60%.

TESTE 158 – (OLIMPÍADA BRASILEIRA DE QUÍMICA) Resolução:

Item a) Variação do número de oxidação:

S +4 → S +6 + 2e−

N +5 + 3e− → N +2

SO2 → agente redutor

HNO3 → agente oxidante

Equação química devidamente balanceada pelo método REDOX: 3 SO2 + 2 HNO3 + 2 H2 O → 2 NO +


3 H2 SO4

Item b) Para a determinação da massa molecular do monóxido de nitrogênio será necessário considerar como gás
ideal:

PV 30 𝑥 1 𝑥 4800
mNO = = = 5888 g (≅ 5,89 kg)
RT 0,08206 𝑥 (25 + 273)
p x <MM>NO
dNO 30
Item c) Densidade do óxido nítrico em relação ao dióxido de enxofre: = RT
p x <MM>SO = = 0,469
dSO2 2 64
RT
p x <MM>NO
dNO 30
Densidade do óxido nítrico em relação ao ar atmosférico: dar
= RT
p x <MM>ar = 28,84 = 1,04
RT
< MM >ar = 0,21 x < MM >O2 + 0,79 x < MM >N2

g
< MM >ar = 0,21 x 32 + 0,79 x 28 = 28,84
mol
dX
Item d) Densidade do composto X em relação ao dióxido de enxofre: d =2
SO2

p x < MM >X
RT < MM >X
2= =
p x < MM >SO2 64
RT
g
< MM >X = 128
mol

Item e) Cálculo do número de mol total: ntotal = nSO2 + nH2 + nNO = 4 + 4 + 2 = 10 mol

nNO 2
X NO = = = 0,20
ntotal 10

nSO2 4
X SO2 = = = 0,40
ntotal 10

840
1600 FÍSICO-QUÍMICA APLICADA EXERCÍCIOS COMENTADOS - IME – ITA – OLIMPÍADA

nH2 4
X H2 = = = 0,40
ntotal 10
ntotal x R x T
Cálculo da pressão total: ptotal = V

10 x 0,08206 x 273
ptotal = = 10,18 atm
22

TESTE 159 – (OLIMPÍADA BRASILEIRA DE QUÍMICA) Resolução: Alternativa B.


Para a obtenção de hidrogênio molecular será necessário aplicar o princípio de Le Chatelier. Então, temos:
✓ Aumento da pressão parcial ou até mesmo da concentração da quantidade de matéria do monóxido de
carbono;
✓ Aumentando a pressão parcial ou até mesmo a concentração da quantidade de matéria da água no estado
gasoso;
✓ Diminuindo a temperatura do sistema;
✓ Aumentando ou até mesmo diminuindo a pressão do sistema, o equilíbrio não será afetado;
✓ Para a obtenção de hidrogênio molecular, Kc = Kp, pois: K p = K c x (R x T)2−2 → K p = K c.

Analisando a afirmativa IV: Será necessário utilizar a tabela de equilíbrio.

CO(g) H2O(g) ⇌ CO2(g) H2(g)


Início 1 1 2 2
Reage X X X X
1−X 1−X 2−X 2−X
Equilíbrio
1 1 1 1
[CO ] x [H2 ]
Cálculo do valor de X: K c = [CO]2x [H
2 O]

(2 − X) x (2 − X)
0,625 =
(1 − X) x (1 − X)

(2 − X)²
0,625 =
(1 − X)²

0,375X 2 − 5,25X + 3,375 = 0

Resolvendo a equação acima, temos: X1 = 0,68 e X2 = 13,32.

Levando em consideração o valor de X1, as concentrações das quantidades de matéria das espécies gasosas são as
seguintes:

1 − X 1 − 0,68 mol
[CO] = [H2 O] = = = 0,32
1 1 L
2 − X 2 − 0,68 mol
[H2 ] = [CO2 ] = = = 1,32
1 1 L

TESTE 160 – (OLIMPÍADA DE QUÍMICA DO RIO DE JANEIRO) Resolução: Alternativa C.


3
Decomposição do clorato de potássio: KClO3(s) → KCl(s) + 2 O2(g)
3
1 mol de KClO3(s) − − − − − mol de O2(g)
2

841
1600 FÍSICO-QUÍMICA APLICADA EXERCÍCIOS COMENTADOS - IME – ITA – OLIMPÍADA

3
122,5 g de KClO3(s) − − − − − x 32g
2
m1 − − − − − − − − − − − − 32 g (1 mol)
122,5 x 2
m1 = = 81,67 g
3
1
Decomposição do peróxido de hidrogênio: H2 O2(aq) → H2 O(l) + 2 O2(g)

1
1 mol de H2 O2(aq) − − − − − mol de O2(g)
2
1
34 g de H2 O2(aq) − − − − − − x 32g
2
m2 − − − − − − − − − − − − 32 g (1 mol)
m2 = 68 g
m m 81,67 g
Cálculo da razão m2 : m2 = 68 g
= 1,20
1 1

TESTE 161 – Resolução: Para a resolução deste exercício será necessário calcular o potencial padrão (E°),
igualando a energia livre de Gibbs termodinâmico com a energia livre de Gibbs eletroquímica.
° °
∆𝐺𝑡𝑒𝑟𝑚𝑜𝑑𝑖𝑛â𝑚𝑖𝑐𝑜 = ∆𝐺𝑒𝑙𝑒𝑡𝑟𝑜𝑞𝑢í𝑚𝑖𝑐𝑜

− R x T x lnK eq = − n x F x E°
[Cu+ ]2
− 8,314 x 298 x ln = − 2 x 96500 x E°
[Cu+2 ]
1
− 8,314 x 298 x ln = − 2 x 96500 x E°
⏟ 1,70 x 106
− 14,35
−8,314 x 298 x (−14,35)
E° = = −0,184 V
−193000
° ° °
Ereação = Ecátodo − Eânodo = +0,337 − (−0,184) = +0,521 V

TESTE 162 – (OLIMPÍADA PORTUGUESA DE QUÍMICA) Resolução: Para a resolução deste problema será
necessário utilizar a tabela de equilíbrio.
Base de cálculo: 6,0 mol no estado inicial de SO2, 3,0 mol de O2 no estado inicial e 4,5 mol de SO3 no equilíbrio.
2 SO2(g) O2 (g) ⇌ 2 SO3(g)
Início 6,0 3,0 0
Reage 2X X 2X
Equilíbrio 6 − 2X 3 − X 4,50

Cálculo do valor de X, a partir do trióxido de enxofre no equilíbrio: 2X = 4,50


4,50
X= = 2,25 mol
2

Cálculo das concentrações das diferentes espécies químicas presentes na reação química:

6 − 2X 6 − 2 x 2,25 mol
[SO2 ] = = = 0,15
10 10 L

842
1600 FÍSICO-QUÍMICA APLICADA EXERCÍCIOS COMENTADOS - IME – ITA – OLIMPÍADA

3 − X 3 − 2,25 mol
[O2 ] = = = 0,075
10 10 L
4,50 mol
[SO3 ] = = 0,45
10 L

2)

a) Aumentando a temperatura do sistema reacional, o equilíbrio será deslocado para a esquerda.


b) Aumentando a pressão do sistema reacional, o equilíbrio é deslocado para a direita.
c) Com a adição do gás inerte, não vai haver nenhuma influência no deslocamento do equilíbrio químico.
d) A adição do catalisador não influencia no deslocamento do equilíbrio químico.
1
TESTE 163 – Resolução: Dados do problema: Vfinal = 7 x Vinicial
V
Trabalho realizado de forma isotérmica e reversível: W = −n x R x T x ln (V final )
inicial
1
x Vinicial 1
W = −n x R x T x ln (7 ) = − n x R x T x ln ( ) = −n x R x T x ln(7)−1
Vinicial 7

= + n x R x T x ln(7)

TESTE 164 - (U. S. NATIONAL CHEMISTRY OLYMPIAD) Resolução: Alternativa C.


[A]final
ln ( ) = −k x tempo
[A]inicial

0,0450
ln ( ) = −2,08 x 10−2 x tempo
0,100

ln(0,450)
tempo = = 38,38 s
−2,08 x 10−2

TESTE 165 – (U. S. NATIONAL CHEMISTRY OLYMPIAD) Resolução: Alternativa D.


Semirreação que ocorre na oxidação: 2 I − → I2 + 2 e−
Semirreação que ocorre na redução: Cr +6 + 3e− → Cr +3
Equação química: 2 Cr2 O−2 − +
7 + 12 I + 28 H → 6 I2 + 4 Cr
+3
+ 14 H2 O

TESTE 166 – (U. S. NATIONAL CHEMISTRY OLYMPIAD) Resolução: Alternativa D.


MgRe2 O8
O número de oxidação do magnésio é igual a +2 e do átomo de oxigênio igual a -2, diante disso: + 2 + 2x − 16 =
0
2x − 14 = 0
x = +7

TESTE 167 – (U. S. NATIONAL CHEMISTRY OLYMPIAD) Resolução: Alternativa C.

Semirreação que ocorre no cátodo: Ni+2 −


(aq) + 2 e → Ni(s)

minutos
Cálculo da carga a partir da equação da corrente elétrica: Q = 126,50 1 𝑚𝑖𝑛𝑢𝑡𝑜 x 60 s x 5,15 = 39088,50 C

Cálculo da massa de níquel:

96500 C
1 mol de Ni − − − 58,69 g − − − 2 mol de elétrons x
mol de elétrons
mNi − − − 39088,50 C

843
1600 FÍSICO-QUÍMICA APLICADA EXERCÍCIOS COMENTADOS - IME – ITA – OLIMPÍADA

39088,50 x 58,69
mNi = = 11,89 g
2 x 96500
mNi 11,89 mol
Cálculo da concentração da quantidade de matéria: [Ni+2 ] = = = 0,900
<MM>Ni x Vsolução 58,69 x 225 x 10−3 L

TESTE 168 – (OLIMPÍADA BRASILEIRA DE QUÍMICA) Resolução:


Item a) A partir das semirreações, temos:
Au3+(aq) + 3e- → Au(s) E° red = + 1,498 V
NO3-(aq) + 4H+(aq) + 3e- → NO(g) + 2 H2O(l) E° red = + 0,96 V

Para a formação de Au+3(aq) e NO(g), será necessário inverter a primeira semirreação. Logo:
Au(s) → Au3+(aq) + 3e- E° red = - 1,498 V ∆G° = - 3.F.(-1,498)
NO3-(aq) + 4H+(aq) + 3e- → NO(g) + 2 H2O(l) E° red = + 0,96 V ∆G° = - 3.F.(+ 0,96) +
Au(s) + NO3 (aq) + 4H (aq) → Au (aq) NO(g) + 2 H2O(l)
- + 3+ ∆G°TOTAL = - 3.F.(-1,498) + [- 3.F.(+ 0,96)]

∆G°TOTAL = - 3.F.(-1,498) + [- 3.F.(+ 0,96)]


- 3.F.E°TOTAL = - 3.F.(-1,498) + [- 3.F.(+ 0,96)]
E°TOTAL = -1,498) + 0,96 = - 0,538 V (Não se trata de um processo espontâneo, pois E° < 0).

Item b) Para a formação de AuCl-(aq) e H2(g), temos:


P
ara esta reação, temos que ter a semirreação de redução de formação de gás hidrogênio.
2H+(aq) + 2e- → H2(g) E° red = + 0,00 V
Logo:
Invertendo a semirreação do AuCl-(aq) e multiplicando por dois, temos:
2 Au(s) + 8 Cl-(aq) → 2AuCl4-(aq) + 6e- E° red = - 1,002 V ∆G° = - 6.F.(-1,002)

Multiplicando a semirreação de redução do hidrogênio por três, temos:


6H+(aq) + 6e- → 3H2(g) E° red = + 0,00 V ∆G° = - 6.F.(0,00)

Para a formação de AuCl-(aq) e H2(g), temos:


2Au(s) + 8 Cl-(aq) → 2AuCl4-(aq) + 6e- E° red = - 1,002 V ∆G° = - 6.F.(-1,002)
6H+(aq) + 6e- → 3H2(g) E° red = + 0,00 V ∆G° = - 6.F.(0,00) +
2Au(s) + 8 Cl-(aq) + 6H+(aq) → 2AuCl4-(aq) + 3H2(g) ∆G°TOTAL = - 6.F.(-1,002) + [- 6.F.(+ 0,00)]

∆G°TOTAL = - 6.F.(-1,002) + [- 6.F.(+ 0,00)]


- 6.F.E°TOTAL = - 6.F.(-1,002) + [- 6.F.(+ 0,00)]
E°TOTAL = - 1,002 V
Não se trata de um processo espontâneo, pois E° < 0.

Item c)
Invertendo a semirreação do AuCl-(aq), temos:
Au(s) + 4 Cl-(aq) → AuCl4-(aq) + 3e- E° red = - 1,002 V ∆G° = - 3.F.(-1,002)
NO3 (aq) + 4H (aq) + 3e → NO(g) + 2 H2O(l)
- + - E° red = + 0,96 V ∆G° = - 3.F.(+ 0,96) +
Au(s) + NO3-(aq) + 4H+(aq) + 4 Cl-(aq) → AuCl4-(aq) + NO(g) + 2 H2O(l)

∆G°TOTAL = - 3.F.(-1,002) + [- 3.F.(+ 0,96)]


- 3.F.E°TOTAL = - 3.F.(-1,002) + [- 3.F.(+ 0,96)]
E°TOTAL = - 0,042 V
Não se trata de um processo espontâneo, pois E° < 0.

844
1600 FÍSICO-QUÍMICA APLICADA EXERCÍCIOS COMENTADOS - IME – ITA – OLIMPÍADA

Item d) Trabalhando com os ácidos, tanto clorídrico quanto nítrico concentrados, o coeficiente reacional, Q, vai
apresentar um alto valor, apresentando log Q menor que zero, log Q < 0, o que vai ocasionar em uma ddp espontânea
(E° > 0).

TESTE 169 – Resolução: Como o processo é isotérmico (temperatura constante), ∆U = 0 e ∆H = 0.

V 10 g cal
Cálculo do trabalho realizado: Wrev = −n x R x T x ln (V final ) = − 44 g.mol−1 x 1,987 mol x K x (27 +
inicial
10 L
273) x ln ( 5 L ) = −93,58 cal

nxRxT
TESTE 170 – Resolução: Cálculo do volume inicial, considerando comportamento ideal: Vinicial = pinicial
=
2,0 x 0,08206 x (20 + 273)
15
= 3,20 L

Cálculo da capacidade calorífica a volume constante: Cp − CV = R


cal
CV = Cp − R = 8,60 − 1,987 = 6,61
mol x K
C 8,60
Cálculo da constante de Poisson (𝛾): 𝛾 = Cp = 6,61 = 1,30
V

γ γ
Cálculo do volume final: Pinicial x Vinicial = Pfinal x Vfinal

1,30
15 x (3,20)1,30 = 5 x Vfinal

1,30
15 x (3,20)1,30
Vfinal = √ = 7,45 L
5

γ−1 γ−1
Cálculo da temperatura final: Tinicial x Vinicial = Tfinal x Vfinal

3,20 𝐿 1,30−1,0
Tfinal = ( ) x 293 = 227,39 K
7,45 𝐿

TESTE 171 – Resolução: Como o processo é adiabático, a quantidade de calor é igual a zero, Q = 0.

Cálculo da variação da energia interna (∆U): ∆U = n x CV x ∆T = 2 x 6,61 x (227,39 − 293) = −867,36 cal

Cálculo da variação da energia interna (∆H): ∆H = n x CP x ∆T = 2 x 8,60 x (227,39 − 293) =


−1128,49 cal

Como o processo é adiabático, então ∆U = W, logo: ∆U = W = −867,36 cal.


5R
TESTE 172 – Resolução: Sendo o gás diatômico, CV = 2
. Então a capacidade calorífica a pressão constante é
7R
igual a 2.

7R
C
Cálculo do coeficiente de Poisson: 𝛾 = Cp = 2
5R = 1,40.
V
2
γ−1 γ−1
Cálculo da temperatura final: Tinicial x Vinicial = Tfinal x Vfinal
8 L 1,40−1,0
Tfinal = ( ) x 290 = 350 K
5L
10 5
Cálculo da variação da energia interna (∆U): ∆H = n x CP x ∆T = 28 x 2 x 8,314 x (350 − 290) = +445,39 J

845
1600 FÍSICO-QUÍMICA APLICADA EXERCÍCIOS COMENTADOS - IME – ITA – OLIMPÍADA

10 7
Cálculo da variação da entalpia (∆H): ∆H = n x CP x ∆T = 28 x 2 x 8,314 x (350 − 290) = +623,55 J

Como o processo é adiabático, então ∆U = W, logo: ∆U = W = +445,39 J

TESTE 173 – (OLIMPÍADA ARGENTINA DE QUÍMICA) Resolução:


ln 2 0,693
Item a) Cálculo do tempo de meia-vida: t 1 = = = 80,60 min
2 k 0,0086
Item b) Partindo da equação de primeira ordem que relaciona a concentração versus o tempo, temos:
[N O ]
ln [N 2O 5] final = −k x t
2 5 inicial
N O
2 5
nfinal
ln NVO = −k x t
2 5
ninicial
V
N O
2 5
nfinal
ln = −0,0086 x 321,6
4,0
N O
n 2 5
ln final = −2,76
4,0
N O
2 5
nfinal = 4,0 𝑥 𝑒 −2,75 = 0,253 𝑚𝑜𝑙

Item c) A cinética da decomposição do dióxido de nitrogênio é de segunda ordem, o que pode ser observado pela
unidade da velocidade específica, L.mol-1.s-1.

Item d) Aplicando a Equação de Arrhenius, temos:

4,87 𝑥 10−3 1,039 𝑥 105 1 1


ln 100°𝐶
= 𝑥 ( − )
𝑘 8,314 100 + 273 65 + 273

L
k100°C = 0,156
mol x s

TESTE 174 – (OLIMPÍADA NORTE NORDESTE DE QUÍMICA) Resolução: Equação química: CO(g) + 2 H2(g) →
CH3OH(g)

Base de cálculo: 2 mol de CO e 5 mol de H2 na situação inicial e 0,35 mol de CO no equilíbrio.

Tabela de equilíbrio químico:

CO(g) (mol) 2 H2(g) (mol) CH3OH(g) (mol)


Início 2,00 5,00 0
Reage X 2X X
Equilíbrio 2-X 5 – 2X X

Cálculo da incógnita X, a partir do monóxido de carbono: 2,00 – X = 0,35


X = 1,65 mol

Cálculo das espécies químicas no equilíbrio químico:


nCO = 0,35 mol
nH2 = 5 – 2X = 5 – 2.(1,65) = 1,70 mol
nCH3OH = 1,65 mol

846
1600 FÍSICO-QUÍMICA APLICADA EXERCÍCIOS COMENTADOS - IME – ITA – OLIMPÍADA

Cálculo do número de mol total (nT): nT = nCO + nH2 + nCH3OH = 3,70 mol
Item a) A quantidade em mol de gás hidrogênio é igual a 1,70 mol e de metanol igual a 1,65 mol.

Item b) Cálculo das frações molares de cada espécie gasosa:

1,70 mol
X H2 = = 0,46
3,70 mol

0,35 mol
X 𝐶𝑂 = = 9,50 x 10−2
3,70 mol

1,65 mol
X 𝐶H3 OH = = 0,45
3,70 mol
XCH3OH 0,45
Cálculo da constante de equilíbrio em função das frações molares (Kx): K X = 2 = 9,50 x 10−2 x (0,46)2 =
XCO x (XH2 )
22,48

Item c) Cálculo da pressão, considerando que os gases apresentam comportamento ideal:

n x R x T 3,70 x 0,08206 x (350 + 273)


ptotal = = = 18,91 atm
V 10

Determinação das pressões parciais das espécies gasosas:


PH2 = 0,46 x 18,91 atm = 8,69 atm
PCO = 9,50 x 10−2 x 18,91 atm = 1,79 atm
PCH3 OH = 0,45 x 18,91 atm = 8,43 atm

PCH3OH 8,43
Cálculo da constante de equilíbrio em função das pressões parciais (Kp): K p = 2 = =
PCO x (PH2 ) 1,79 x (8,69)2
6,24 𝑥 10−2

Item d) Cálculo da energia livre de Gibbs padrão: ∆G° = −R x T x lnK p = −8,314 x (350 +
kJ
273) x ln(6,24 x 10−2 ) = +14,37
mol

Item e) Cálculo da constante de equilíbrio em função das concentrações das quantidades de matéria:

K p = K c x (R x T)∆n = 6,24 𝑥 10−2 x (0,08206 x 623)1−3 = 2,38 𝑥 10−5

TESTE 175 – (GRILLO) Resolução: Para a resolução da estequiometria será necessário primeiramente balancear a
equação iônica. O agente redutor é o íon oxalato e o agente oxidante é o íon permanganato. Diante disso a equação
iônica balanceada vai ficar da seguinte maneira:
−2 + +2
2 MnO−
4(aq) + 5 C2 O4(aq) + 16 H(aq) → 2 Mn(aq) + 10 CO2(g) + 8 H2 O(l)

Realizando a estequiometria, temos:


−2
2 mol de MnO− 4(aq) − − − − 5 mol de C2 O4(aq)
n1 − − − − − − − − − − − n2

5 x n1 = 2 x n2

5 x 0,10 x V = 2 x 0,010

847
1600 FÍSICO-QUÍMICA APLICADA EXERCÍCIOS COMENTADOS - IME – ITA – OLIMPÍADA

2 x 0,010 0,020
V= = = 0,040 L (40 mL)
5 x 0,10 0,50

TESTE 176 – (GRILLO) Resolução: Para a resolução do problema que envolve a estequiometria será necessário
primeiramente balancear a equação redox. O agente redutor é o cloreto de potássio e o agente oxidante é o
permanganato de potássio. Diante disso a equação redox balanceada fica da seguinte forma:

2 KMnO4(aq) + 10 KCl(aq) + 8 H2 SO4(aq) → 2 MnSO4(aq) + 6 K 2 SO4(aq) + 8 H2 O(l) + 5 Cl2 (g)

A partir da relação estequiométrica, temos:

2 KMnO4(aq) + 10 KCl(aq) + 8 H2 SO4(aq) → 2 MnSO4(aq) + 6 K 2 SO4(aq) + 8 H2 O(l) + 5 Cl2 (g)

2 mol de KMnO4(aq) − − − − − 6 mol de K 2 SO4(aq) − − − − − − − 5 mol de Cl2 (g)


2 x 158 g − − − − − − − − − 6 x 174 g − − − − − − − − − − − − 5 x 71 g
200 g − − − − − − − − − − − mK2 SO4 − − − − − − − − − − − − mCl2
200 x 6 x 174 208800
mK2 SO4 = = = 660,76 g
2 x 158 316
200 x 5 x 71 71000
mCl2 = = = 224,68 g
2 x 158 316

TESTE 177 – (ITA) Resolução: Alternativa A.


O Negro de fumo é um material particulado sólido constituído quase que totalmente de carbono grafite, e origina-se
da combustão incompleta do hidrocarboneto metano (CH4). Já os demais materiais apresentados, tais como o carvão,
alcatrão, piche e óleo diesel, são materiais oriundos de outros elementos químicos com composição bem definida, em
quantidades significativas.

TESTE 178 - (ITA) Qual das opções a seguir apresenta o gráfico que mostra, esquematicamente, a variação da
condutividade elétrica de um metal sólido com a temperatura?

Resolução: Alternativa C.
Ao aumentar a temperatura de um determinado material metálico, a energia cinética média dos cátions que compõem
o material metálico (íons positivos) aumenta, com isso há uma dificuldade na movimentação dos elétrons livres,
responsáveis por conduzir eletricidade. Portanto, o aumento da temperatura diminui a condutividade elétrica de um
metal, como mostrado na figura acima, representado no terceiro gráfico.

TESTE 179 – (ITA) Resolução:


a) A gota de mercúrio forma uma esfera (uma “bolinha”) sobre o plano, enquanto que a gota de água se espalha sobre
a superfície do vidro, formando uma calota esférica, conforme pode ser observado na figura a seguir.

mercúrio água
b) A diferença entre os formatos deve-se à força de atração entre os átomos de mercúrio ser muito grande (ligações
metálicas). Assim, o formato da amostra de mercúrio tende sempre à minimização do volume ocupado. No caso da
água, peso das moléculas é compatível com a força de atração entre as mesmas, provocando a forma de calota
esférica.
c) No caso do mercúrio, o maior volume da gota leva ao “achatamento” da esfera, e esta passa a ter tendência a
fraturar em gotas menores. No caso da água, há apenas o aumento da superfície de contato entre a gota e a superfície
plana (vidro).

848
1600 FÍSICO-QUÍMICA APLICADA EXERCÍCIOS COMENTADOS - IME – ITA – OLIMPÍADA

TESTE 180 – (OLIMPÍADA BRASILEIRA DE QUÍMICA) Resolução: Alternativa B.


Óxidos básicos são compostos binários que reagindo com ácidos resultam em sais e agua e reagindo com a água
resultam no hidróxido. O único óxido básico presente é o óxido de cálcio.
Equação química: CaO + H2 O → Ca(OH)2

TESTE 181 - (OLIMPÍADA DE QUÍMICA DO RIO DE JANEIRO - MODIFICADA) Resolução: Cálculo da velocidade
ln2 0,693
específica à T = 37°C, a partir do tempo de meia-vida: k 37°C = t = 1045 = 6,63 x 10−4 min−1
1/2
k37°C Eat 1
Cálculo da velocidade específica à T = 44°C, utilizando a equação de Arrhenius: ln (k44°C ) = R
x (44+273 −
1
37+273
)

6,63 x 10−4 min−1 400000 1 1


ln ( 44°C )= x( − )
k 8,314 44 + 273 37 + 273

6,63 x 10−4 min−1


ln ( ) = −3,43
k 44°C

k 44°C = 2,05 x 10−2 min−1

k 37°C Eat 1 1
ln ( 44°C )= x( − )
k R 44 + 273 37 + 273

ln2 0,693
Cálculo do tempo de meia-vida para T = 44°C: t1/2 = k44°C = 2,05 x 10−2 = 33,80 min

TESTE 182 – (GRILLO) Resolução: Para a resolução deste problema será necessário utilizar a Lei de Hess. O
problema pede o valor da entalpia padrão para a reação de formação do Fe2O3(s).

2 Fe(s) + 3/2 O2(g) → Fe2O3(s) ΔH°formação = ?

Para a determinação da entalpia padrão de formação do Fe2O3, será necessário inverter a primeira reação, multiplicar
a segunda reação química por três e invertendo e multiplicando a reação química por três, temos:

Equação química I: 2 Fe(s) + 3 CO(g) → Fe2O3(s) + 3 C(grafite) ΔH° = - 117,3 kcal

Equação química II: 3 C(grafite) + 3 O2(g) → 3 CO2(g) ΔH° = 3 x (- 94,0) kcal

Equação química III: 3 CO2(g) → 3 CO(g) + 3/2 O2(g) ΔH° = 3 x (- 67,6) kcal

Somando as três equações químicas, temos:

Equação Química I: 2 Fe(s) + 3 CO(g) → Fe2O3(s) + 3 C(grafite) ΔH° = - 117,3 kcal


Equação Química II: 3 C(grafite) + 3 O2(g) → 3 CO2(g) ΔH° = 3 x (- 94,0) kcal
Equação Química III: 3 CO2(g) → 3 CO(g) + 3/2 O2(g) ΔH° = 3 x (- 67,6) kcal +
Equação química global: 2 Fe(s) + 3/2 O2(g) → Fe2O3(s) ΔH°TOTAL = - 117,3 kcal + [3 x (- 94,0) kcal] + [3
x (- 67,6) kcal] = - 602,10 kcal

TESTE 183 – (GRILLO) Resolução: Invertendo a primeira semirreação e multiplicando a segunda semirreação por
dois, temos:
Zn(s) → Zn+2(aq) + 2e- ∆G° = - 2.F.(+ 0,76)
2H (aq) + 2e → H2(g)
+ - ∆G° = - 2.F.(0,00) +
Zn(s) + 2H+(aq) → Zn+2(aq) + H2(g) ∆G° = -2.F.E°

849
1600 FÍSICO-QUÍMICA APLICADA EXERCÍCIOS COMENTADOS - IME – ITA – OLIMPÍADA

Item a) Equação global para o processo eletroquímico (pilha): Zn(s) + 2H+(aq) → Zn+2(aq) + H2(g)
-2.F.E°= - 2.F.(+ 0,76) + [- 2.F.(0,00)]
E° = + 0,76 V

Item b) Cálculo da constante de equilíbrio (Keq): −R x T x lnK eq = −n x F x E°


96500 C
−8,314 x (25 + 273) x lnK eq = −2 mol x x (+0,76 V)
mol

lnK eq = + 59,20

K eq = e(+59,20)

Item c) Cálculo da variação da energia livre de Gibbs padrão: ∆𝐺 ° = −n x F x E° =


96500 C
−2 mol de elétrons x mol de elétrons x (+0,76) = −146680 J

0,0592
Item d) Utilizando a equação de Nernst, temos: E = E° − n
x log Q

0,0592 [Zn+2 ] x pH2 ]


0,65 = 0,76 − x log { }
n aZn x [H + ]2

0,0592 1𝑥1
0,65 = 0,76 − x log { }
2 1 x [H + ]2

0,0592
−0,11 = − x log[H + ]−2
2
0,0592
−0,11 = − x (−2) x log[H + ]
2
0,11
pH = = 1,86
0,0592

TESTE 184 – (IME) Resolução: Equação química: C2H2(g) + H2(g) → C2H6(g).

Item a) A relação da velocidade média da reação com a velocidade dos participantes da reação química:

d[C2 H2 ] 1 d[H2 ] d[C2 H6 ]


Vreação = − =− x =+
dt 2 dt dt

d[C2 H2 ] 35 − 38 𝑚𝑜𝑙
Vreação = − = = 1,50
dt 6−4 𝐿 𝑥 𝑚𝑖𝑛

Iem b) A relação da velocidade média da reação com a velocidade dos participantes da reação química:

d[C2 H2 ] 1 d[H2 ] d[C2 H6 ]


Vreação = − =− x =+
⏟ dt ⏟ 2 dt ⏟ dt
VC2H2 VH2 V C 2 H6

1 d[H2 ] d[C2 H6 ]
− x =+
⏟2 dt ⏟ dt
VH2 V C 2 H6

850
1600 FÍSICO-QUÍMICA APLICADA EXERCÍCIOS COMENTADOS - IME – ITA – OLIMPÍADA

1
x V = VC2 H6
2 H2
VC2 H6 1
=
VH2 2

Item c) A equação de Arrhenius é descrita a partir da seguinte equação matemática, k = A. e-(Eat/RT), onde:
k = velocidade específica; A = fator pré-exponencial; Eat = energia de ativação; R = constante dos gases;
T = temperatura absoluta.
Eat
(− )
Analisando k1, T1, temos: k1 = A x e R x T1
E
Aplicando o logaritmo neperiano, temos: ln k1 = ln A − R xatT
1
Eat
Analisando para k2, T2, temos: ln k 2 = ln A −
R x T2
Eat E
Igualando as equações acima, temos: ln 𝑘1 + R x T1
= ln k 2 + R xatT
2
𝑘1 Eat 1 1
ln = 𝑥 { − }
k2 R T2 T1
E
(− at )
Analisando a equação na maneira gráfica: k = A x e R x T
Eat 1
Aplicando o logaritmo neperiano, temos: ln
⏟ 𝑘 = ln
⏟ A−⏟R
𝑥⏟
𝑇
. O gráfico plotado fica da seguinte forma:
𝑦 𝑎
𝑏 𝑥

∆y Eat
Através da equação de Arrhenius, o coeficiente angular será: b = − =
∆x R
Conclusão: Aumentando a temperatura, a constante cinética também vai aumentar.

TESTE 185 – (GRILLO) Resolução: Equação química: PCl5(g) ⇌ PCl3(g) + Cl2(g)


Base de cálculo: n mol Inicial de PCl5(g).
Tabela de equilíbrio químico:

PCl5(g)  PCl3(g) Cl2(g)


Início n 0 0
Reage nα nα nα
Equilíbrio n - nα nα nα

Cálculo do número de mol total (nT): nT = nPCl5(g) + nPCl3(g) + nCl2(g) = n – nα + nα + nα = n + nα

Determinação das expressões das pressões parciais, sabendo que a pressão total da mistura reacional é igual a 1.
n − nα n(1 − α) (1 − α)
PPCl5 = x pT = x1 =
n + nα n(1 + α) (1 + α)

851
1600 FÍSICO-QUÍMICA APLICADA EXERCÍCIOS COMENTADOS - IME – ITA – OLIMPÍADA

nα nα α
PPCl5 = PCl2 = x pT = x1=
n + nα n(1 + α) (1 + α)

Cálculo do grau de dissociação a partir da constante de equilíbrio em função das pressões parciais (K p): 𝐾𝑝 =
PPCl5 x PCl2
PPCl5
α α
𝑥 α2
(1 + α) (1 + α)
Kp = =
(1 − α) 1 − α2
(1 + α)
α2
Sabendo que a constante de equilíbrio é igual a 4,0, temos: 4,0 = 1−α2
4 − 4α2 = α2
5α² = 4
4
α² =
5
4
α = √ = √0,80 = 0,8944 (89,44 %)
5

m
Cálculo da densidade, considerando comportamento ideal dos gases: ptotal x < MM > = x (1 + α) x R x T

V
d
ptotal x < MM > 1 x 208,5 g
d= = = 2,83
(1 + α) x R x T (1 + 0,8944) x 0,08206 x (200 + 273) L

TESTE 186 – Resolução: A partir da definição do trabalho termodinâmico, dW = −p x dV (Equação A) e


nxRxT
considerando que o gás apresenta comportamento ideal: p = V

nxRxT
dW = − x dV
V
V2
1
W = −n x R x T x ∫ x dV
V1 V

V2
W = − n x R x T x ln ( )
V1

nxRxT
Colocando em fução da pressão, temos: V = p

nxRxT
p2
W = − n x R x T x ln ( )
nxRxT
p1

p1
W = − n x R x T x ln ( )
p2

852
1600 FÍSICO-QUÍMICA APLICADA EXERCÍCIOS COMENTADOS - IME – ITA – OLIMPÍADA

TESTE 187 – Resolução: O processo sendo adiabático, a quantidade de calor é igual a zero. A partir da primeira lei
da termodinâmica, temos:

∆U = Q + W

∆U = 0 + W

∆U = W

n x CV x dT = −p x dV (Equação A)
𝑛𝑥𝑅𝑥𝑇
Considerando que o gás apresenta comportamento ideal: p = 𝑉

nxRxT
Substituindo a equação dos gases na Equação A: n x CV x dT = − V
x dV
RxT
CV x dT = − V x dV

1 R 1
T
x dT = −C x V
x dV
V

T2 V2
1 R 1
∫ x dT = − x∫ x dV
T1 T CV V1 V

T R V
ln (T2 ) = − C x ln (V2 ) (Equação B)
1 V 1

A partir da relação entre as capacidades caloríficas, temos: CP − CV = 𝑅


C C 𝑅
Dividindo a equação por CV : CP − CV = C
⏟V V V
𝛾
𝑅
γ−1= (Equação C)
CV
T V
Substituindo a equação C na equação B, temos: ln (T2 ) = −(γ − 1) x ln (V2 )
1 1
T2 V2 −(γ−1)
( )=( )
T1 V1

T2 V1 γ−1
( )=( )
T1 V2

TESTE 188 – Resolução: A partir da relação termodinâmica do teste 187, é possível chegar a relação entre o volume
e a pressão para uma transformação adiabática reversível. Considerando que o gás apresenta comportamento ideal:
𝑝𝑥𝑉
𝑇 = 𝑛 𝑥 𝑅, temos:

p2 x V2 γ−1
n x R = (V1 )
p1 x V1 V2
nxR
γ
p2 x V2 V1 x V1−1
=
p1 x V1 V2γ x V2−1

γ γ
p1 x V1 x V1 x V1−1 = p2 x V2 x V2 x V2−1

853
1600 FÍSICO-QUÍMICA APLICADA EXERCÍCIOS COMENTADOS - IME – ITA – OLIMPÍADA

γ γ
p1 x V1 = p2 x V2

TESTE 189 – Resolução: Alternativa E.


Observa-se que se trata de uma reação de pseudo segunda ordem, pois um dos reagentes encontra-se em excesso.
Logo, a equação da velocidade ficará da seguinte maneira: vreação = k x [X]2 . Cálculo do tempo de meia-vida para
1 1
uma cinética de segunda ordem: t1/2 = k x [X] = 0,500 x 0,04 = 50 min
inicial

TESTE 190 – (MONBUKAGAKUSHO, COLLEGE OF TECHNOLOGY STUDENTS E UNDERGRADUATED


STUDENTS) Resolução: número 2.
87,0
Cálculo do número de mol de água: nH2O = 18
= 4,83 mol
13,0
Cálculo do número de mol de hidróxido de sódio: nNaOH = = 0,325 mol
40

Cálculo do número de mol total: nH2O + nNaOH = 4,83 mol + 0,325 mol

nH2O 4,83 mol


Cálculo da fração molar da água: X H2O = n = 4,83 mol+0,325 mol = 0,937
H2O +nNaOH

TESTE 191 – (MONBUKAGAKUSHO, COLLEGE OF TECHNOLOGY STUDENTS E UNDERGRADUATED


STUDENTS) Resolução: número 3.
Os dados estão relacionados no teste 190.

Primeira resolução do problema:


13𝑔 13𝑔
Cálculo do título em massa/massa: = = 13%
13𝑔 +87𝑔 100𝑔

𝑘𝑔 1000 𝑔 1 𝑚³ 𝑔
Conversão da unidade da densidade: 𝑑 = 1,142 𝑚³ 𝑥 1 𝑘𝑔
𝑥 1000 𝑑𝑚³
= 1,142 𝐿

𝐶
Cálculo da concentração da quantidade de matéria da solução de hidróxido de sódio: [𝑁𝑎𝑂𝐻] =
<𝑀𝑀>𝑁𝑎𝑂𝐻

10 x 1,142 x 13 mol
[NaOH] = = 3,71
40 L

Segunda maneira de resolução:

1000 g
1 m3 de solução − − − 1142 kg x
1 kg
Vsolução − − − − − − − 100 g (13g + 87g)

100
Vsolução = 1142000 = 8,76 x 10−5 m3 , este volume equivale a 8,76 x 10−2 dm³.

13 mol
Cálculo da quantidade de matéria da solução de hidróxido de sódio: [NaOH] = 40 x 8,76 x 10−5 m³ = 3,71 L

TESTE 192 – Resolução: Alternativa D.


Observando O quadro em relação à pressão, há um aumento do primeiro experimento (2,5 atm) para o segundo
experimento (4,0 atm). Com este aumento da pressão do sistema reacional, o equilíbrio é deslocado para a diretia do
sentido direto.
Já em relação a temperatura há também um aumento entre o primeiro experimento (27°C) para o segundo
experimento (127°C), isso caracteriza um processo endotérmico, e, aumentando a temperatura o equilíbrio é

854
1600 FÍSICO-QUÍMICA APLICADA EXERCÍCIOS COMENTADOS - IME – ITA – OLIMPÍADA

deslocado para a direita do sentido direto. Com o aumento da temperatura e deslocando para a direita, o valor da
constante de equilíbrio em função das concentrações das quantidades de matéria também aumenta.

TESTE 193 – Resolução: Sabendo que o rendimento de Carnot, designado pela letra R, é dado pela seguinte
T
equação matemática: R = 1 − T fria . Substituindo os valores:
quente
(25 + 273)
R=1−
(323 + 273)
298
R=1−
596
𝑅 = 1 − 0,50 = 0,50 (50%)

TESTE 194 – (OLIMPÍADA DE QUÍMICA DO RIO DE JANEIRO) Resolução: Alternativa A.


a) Afimativa correta. A energia interna é definida como sendo dU = n x Cv x dT, onde a energia interna é
função direta da temperatura.
b) Afirmativa errada. A entropia não mede a idade do universo, mas sim o grau de desordem de um sistema
termodinâmico.
c) Afirmativa errada. Equação de Gibbs: ΔG = ΔH – T.ΔS. Para ΔG < 0 (processo espontâneo), dependendo
da variação de entalpia, caso seja negativa (ΔH < 0), a função entropia do universo passa a ser negativo, dependendo
da temperatura T aplicada.
d) Afirmativa errada. Por convenção, a entalpia padrão das substâncias em seu estado físico mais estável é
igual a zero.
e) Afirmativa errada. Esta afirmativa exposta pelo problema não é uma condição para que o processo seja
espontâneo. Evidentemente caso a variação de entalpia seja negativa (ΔH < 0), ou seja, processo exotérmico, o
processo pode até ser espontâneo, conforme a equação de Gibbs (ΔG = ΔH – T.ΔS), caso a relação T x ΔS também
seja positiva.

TESTE 195 – (OLIMPÍADA BRASILEIRA DE QUÍMICA) Resolução: Alternativa E.


Através da equação de Gibbs, ΔG = ΔH – T x ΔS, o processo será espontâneo (ΔG < 0) quando apresentar ΔH < 0 e
ΔS > 0.

TESTE 196 - (ITA) Resolução: O objetivo da questão é determinar a pressão parcial dos compostos gasosos,
sabendo que levaremos em consideração que o oxigênio apresenta 21% de oxigênio molecular e o nitrogênio
molecular apresenta uma composição de 79%.

Diante desta informação, as frações molares das espécies gasosas são as seguintes: X O2 = 0,21 e XN2 = 0,79.

Cálculo da pressão total: Sabendo que a cada dez metros corresponde em média uma atmosfera (1 atm), temos que:
200 m
ptotal = 1 atm + ( 10 m ) x 1 atm = 21 atm

Cálculo das pressões parciais:

PO2 = X O2 x ptotal = 0,21 x 21 = 16,59 atm


PN2 = X N2 x ptotal = 0,79 x 21 = 4,41 atm

TESTE 197 – (ITA) Resolução: Alternativa C.

Alternativa a): Incorreta, pois a constante de equilíbrio depende do conhecimento da concentração da quantidade de
matéria dos produtos e também dos reagentes da reação, o que não pode ser deduzido do gráfico acima, que
apresenta apenas uma relação entre a energia cinética das partículas e a quantidade de partículas com determinados
níveis de energia em ambos os experimentos.
Alternativa b): Incorreta, pois o experimento II apresenta uma maior quantidade de partículas com energia maior ou
igual à energia de ativação da reação, o que implica em um maior número de colisões efetivas e portanto uma maior
velocidade de reação.

855
1600 FÍSICO-QUÍMICA APLICADA EXERCÍCIOS COMENTADOS - IME – ITA – OLIMPÍADA

Alternativa c) Correta, pois uma maior quantidade de partículas com nível de energia cinética alto no experimento II
significa maior temperatura a que aquelas partículas estão submetidas.
Alternativa d) Incorreta, conforme explicado na alternativa b.
Alternativa e) Incorreta, pois o experimento II apresenta uma quantidade de partículas com alto nível de energia
cinética bem maior do que o experimento I, o qual apresenta a maior parte de suas partículas em condições de baixo
nível de energia.

TESTE 198– (ITA) Resolução:

TESTE 199 – (GRILLO) Resolução: Realizando o balanceamento pelo método de oxirredução, temos:
2 KMnO4 + 10 FeSO4 + 8 H2SO4 → K2SO4 + 2 MnSO4 + 5 Fe2(SO4)3 + 8 H2O, em que o permanganato de potássio é
o agente oxidante e o sulfato de ferro II é o agente redutor.

Cálculo do número de mol para cada reagente, para a determinação do reagente limitante:

1000 g 6,33 mol


nKMnO4 = = = 3,16 mol
158 g/mol 2
1000 g 6,58 mol
nFeSO4 = = ≅ 0,66 mol (Reagente Limitante)
152 g/mol 10
1000 g 10,20 mol
nH2 SO4 = = ≅ 1,27 mol
98 g/mol 8

10 mol de FeSO4 --------- 1 mol de K2SO4 ------- 2 mol de MnSO4 ----- 5 mol de Fe2(SO4)3 ----- 8 mol de H2O
1520 g -------------------- 174 g ------- 302 g ------ 2000g --------- 128 g
1000 g -------------------- mK2SO4 ----- mMnSO4 ----- mFe2(SO4)3 ----- mH2O

As massas dos produtos são os seguintes valores apresentados a seguir: mK2SO4 = 114,47 g; mMnSO4 = 198,68 g;
mFe2(SO4)3 = 1315,79 g e mH2O = 84,21 g.

TESTE 200 – Resolução: As semirreações são as seguintes:


Fe3+(aq) + e- → Fe2+(aq) E0 = + 0,77V
I2(g) + 2e → 2I (aq)
- - E = + 0,54V
0

Multiplicando a primeira semirreação e invertendo a segunda semirreação, temos:

2 Fe3+(aq) + 2e- → 2 Fe2+(aq) ΔG° = -2 x F x (+ 0,77)


2I (aq) → I2(g) + 2e
- - ΔG° = -2 x F x (- 0,54) +
2 Fe3+(aq) + 2I-(aq) → 2 Fe2+(aq) + I2(g) ΔG° = -2 x F x (+ 0,77) + [-2 x F x (- 0,54)]

Cálculo da ddp para o seguinte funcionamento da pilha: ΔG° = -2 x F x (+ 0,77) + [-2 x F x (- 0,54)]
-2.F.E° = -2.F.(+ 0,77) + [-2.F.(- 0,54)]
E° = + 0,77 – 0,54 = + 0,23 V

Cálculo da energia livre de Gibbs padrão e da constante de equilíbrio (Keq) à 25°C:

856
1600 FÍSICO-QUÍMICA APLICADA EXERCÍCIOS COMENTADOS - IME – ITA – OLIMPÍADA

96500 C
∆G° = −n x F x E° = −2 mol de e− x ( ) x (+0,23 V) = −44390 J
mol de e−

∆G° = −R x Tx lnK eq

−8,314 x (25 + 273) x lnK eq = −44390

44390 44390
lnK eq = = = 17,92
8,314 𝑥 298 2477,572

K eq = e(+17,92) = 6,06 x 10+7


TESTE 201 – Resolução: Item a) Cálculo do potencial da pilha a 47°C:

Organizando as semirreações, temos:


Cu+2(aq) + 2e- → Cu(s) E° = + 0,34 V
Zn+2(aq) + 2e- → Zn(s) E° = - 0,76 V

A partir da representação da pilha fornecida pelo enunciado do problema, temos:

Invertendo a semirreação do cobre:


Cu(s) → Cu+2(aq) + 2e- E° = - 0,34 V
Zn+2(aq) + 2e- → Zn(s) E° = - 0,76 V +
Cu(s) + Zn (aq) → Cu (aq) + Zn(s) E° = - 1,10 V
+2 +2

RxT aZn x [Cu+]


Cálculo do potencial da pilha (E), utilizando a equação de Nersnt: E = E° − ( n x F ) x ln {a }
x [Zn+2 ]
Cu

Sabendo que as atividade tanto do cobre quanto do zinco são iguais a um, aCu = aZn = 1, temos:
8,314 x (47 + 273) 1 x 0,005 5
E = −1,10 − ( ) x ln { } = −1,10 − (1,38 x 10−2 ) x ln { } = −1,11 V
2 x 96500 1 x 0,002 2

Item b) Reação de semirreação do cobre: Cu+2(aq) + 2e- → Cu(s). Cálculo da carga (Q):
96500 𝐶
1 mol de Cu ---------- 63,55 g --------- 2 mol de e- x 𝑚𝑜𝑙 𝑑𝑒 𝑒 −
0,0425 g --------- Q
Q = 129,07 C

60 segundos
Cálculo do tempo em segundos: Tempo = 4 minutos x ( 1 minuto
)+ 40 s = 280 s
Q 129,07 C
Cálculo da corrente: i = tempo
= 280 segundos
= 0,46 A

Teste 202 – (GRILLO) Resolução:

Item a) Cálculo da variação da entalpia-padrão:


0 0 0
∆H 0 = [2 x HNH 3
] − {1 x HN 2
+ 3 x HH 2
} = [2 x (−46,1)] − {1 x 0 + 3 x 0} = −92,2 kJ. mol−1.

Observando o valor da entalpia da reação de síntese de formação da amônia, conclui-se que o processo é exotérmico.
0 0 0
Item b) Cálculo da variação da entropia-padrão: ∆S 0 = [2 x SNH3
] − {1 x SN 2
+ 3 x SH 2
}
J
∆S 0 = [2 x (192)] − {1 x (191) + 3 x (130)} = −197
mol x K

Cálculo da variação da energia livre de Gibbs: ∆G0 = −92200 − T x (−197)

857
1600 FÍSICO-QUÍMICA APLICADA EXERCÍCIOS COMENTADOS - IME – ITA – OLIMPÍADA

∆G0 = −92200 + 197 x T


J
Para T = 25°C, a energia livre de Gibbs será igual a: ∆G0 = −92200 + 197 x (25 + 273) = −33494 mol
Observando o valor da energia livre de Gibbs, o processo de síntese da amônia é espontâneo.

Item c) O cálculo do equilíbrio químico será realizado através da equação termodinâmica da variação da energia livre
de Gibbs.

∆G0 = −R x T x lnK p

−33494 = −8,314 𝑥 (25 + 273) x lnK p

−33494
lnK p = − 2477,572 = 13,52

K p = e(+13,52) = 744151,56

K298
p
K
∆H° 1 1
Item d) Cálculo da constante de equilíbrio a 400°C a 1 atm: ln ( )= x[ − ]
K673
p
K R 673 298
1,36 x 106 92200 1 1
ln ( K ) =− x[ − ]
K673
p 8,314 673 298

1,36 x 106
ln ( K673 K ) = 20,73
p

1,36 x 106
= e(20,73)
K673
p
K

K 673
p
K
= 1,36 x 10−3

858
1600 FÍSICO-QUÍMICA APLICADA EXERCÍCIOS COMENTADOS - IME – ITA – OLIMPÍADA

TESTE 203 –

Resolução:
a) 2 KMnO4 + 16 HCl → 2 KCl + 2 MnCl2 + 5 Cl2 + 8 H2 O
b) SnCl2 + 2 FeCl3 → SnCl4 + 2 FeCl2
c) 3 Cl2 + 6 NaOH → 5 NaCl + NaClO3 + 3 H2 O
d) 3 P4 + 20 HNO3 + 8 H2 O → 20 NO + 12 H3 PO4
e) C + 4 HNO3 → CO2 + 4 NO2 + 2 H2 O
f) 3 Ag + 4 HNO3 → 3 AgNO3 + 2 H2 O + NO
g) 3 Br2 + 6 KOH → KBrO3 + 5 KBr + 3 H2 O
h) KIO3 + 5 KI + 6 HCl → 3 I2 + 6 KCl + 3 H2 O
i) 2 HBrO3 + 5 SO2 + 4 H2 O → Br2 + 5 H2 SO4
j) 5 H2 C2 O4 + 2 KMnO4 → 10 CO2 + 2 MnO + K 2 O + 5 H2 O
k) MnO2 + 4 HBr → MnBr2 + Br2 + 2 H2 O
l) MnO− 4 + 5 Fe
+2
+ 8 H + → Mn+2 + 5 Fe+3 + 4 H2 O
m) 3 HgS + 8 HNO3 → 3 Hg(NO3 )2 + 3 S + 2 NO + 4 H2 O
n) 3 Ag 2 S + 2 Al + 2 NaOH + 3 H2 O → 6 Ag + 3 H2 S + 2 NaAlO2
o) 2 Ca3 (PO4 )2 + 6 SiO2 + 10 C → 6 CaSiO3 + 10 CO + P4
p) 5 CaC2 O4 + 2 KMnO4 + 8 H2 SO4 → 5 CaSO4 + KSO4 + 2 MnSO4 + 8 H2 O + 10 CO2
q) H2 S + 4 Br2 + 4 H2 O → H2 SO4 + 8 HBr
r) Na2 Cr2 O7 + 3 SO2 + H2 O → 2 Cr(OH)SO4 + Na2 SO4
s) 10 HNO3 + I2 → 2 HIO3 + 10 NO2 + 4 H2 O

TESTE 204 – (GRILLO) Resolução: Cálculo da quantidade de calor para o aquecimento do gelo: Q1 =
m x cgelo x ∆T
Q1 = 40 x 0,50 x [0 − (−10)] = 200 cal

Cálculo da quantidade de fusão do gelo: Q 2 = m x Lfusão = 40 x 80 = 3200 cal

Cálculo da quantidade de calor para o aquecimento da água no estado líquido: Q 3 = m x cágua x ∆T =


40 x 1,0 x (100 − 0) = 4000 cal

Cálculo da quantidade de calor para o vaporização da água: Q 4 = m x Lvaporização = 40 x 540 = 216000 cal

Cálculo da quantidade de calor para o aquecimento do vapor d´água: Q 5 = m x cvapor x ∆T =


40 x 0,50 x (120 − 100) = 400 cal

Cálculo da quantidade de calor total (QTOTAL): Q TOTAL = Q1 + Q 2 + Q 3 + Q 4 + Q 5 = 200 + 3200 + 4000 +


21600 + 400 = 29400 cal (29,4 kcal)

TESTE 205 – (GRILLO) Resolução: Cálculo da pressão inicial considerando que a amônia apresenta comportamento
ideal:
3,50
n x R x T ( 17 ) x 0,08206 x (27 + 273)
pinicial = = = 3,38 atm
Vinicial 1,50

Cálculo da temperatura final, levando em consideração que o processo seja adiabático (Q=0).
γ−1
Tfinal V
( ) = inicial
γ−1
Tinicial V final
γ−1
Vinicial
Tfinal = Tinicial x γ−1
Vfinal
1,50 1,35−1,0
Tfinal = 300 x ( ) = 204,23 K
4,50

859
1600 FÍSICO-QUÍMICA APLICADA EXERCÍCIOS COMENTADOS - IME – ITA – OLIMPÍADA

γ γ
Cálculo da pressão final: Pinicial x Vinicial = Pfinal x Vfinal

3,38 x (1,50)1,35 = Pfinal x (4,50)1,35

1,50 1,35
Pfinal = 3,38 x ( ) = 0,77 atm
4,50

TESTE 206 – (GRILLO) Resolução: Cálculo do volume inicial, a partir da equação dos gases ideais:
200000
101325
x Vinicial = 1 x 0,08206 x (100 + 273)
Vinicial = 15,51 litros (15,51 x 10−3 m³)
200000
Cálculo do volume final, a partir da equação dos gases ideais: 101325 x Vfinal = 1 x 0,08206 x (25 + 273)
Vinicial = 12,39 litros (12,39 x 10−3 m³)

Cálculo do trabalho realizado: W = − 200000 Pa x (12,39 x 10−3 m3 − 15,51 x 10−3 m3 ) = +624 J


3
Cálculo da Energia Interna: ∆U = n x CV x ∆T = 1 mol x 2 x 8,314 x (298 − 373) = −935,32 J

5
Cálculo da Entalpia: ∆H = n x CP x ∆T = 1 mol x x 8,314 x (298 − 373) = −1558,87 J
2

Como o processo ocorre a pressão constante, ∆H = Q p = −1558,87 J


TESTE 207 – (GRILLO) Resolução:

a) Para esta equação química hipotética a cinética é de primeira ordem. Isso é observado pela unidade da constante
cinética, (tempo)−1 .

ln 2 0,693
b) Cálculo do tempo de meia-vida: t1/2 = k
= 2,28 x 10−4 x h−1 = 3030,47 h

pfinal
c) ln ( A
) = −k x t
pinicial
A

pfinal
A
ln ( ) = − 2,28 x 10−4 x 5
625
−4
pfinal
A = 625 x e−(2,28 x 10 x 5)

−4
pfinal
A = 625 x e−(2,28 x 10 x 5)

pfinal
A = 625 x e−(0,00114) = 624,29 torr

TESTE 208 – (GRILLO) Resolução: Informações do problema: n = 1,50 mol; Pinicial = 1,00 atm; Vinicial = 3000 L; Vfinal
= 1200 L e Tconstante = 300 K.

Item a) Cálculo do trabalho realizado, levando em consideração que o processo ocorre de maneira compressível e
isotérmico.

V2 1200
W = −n x R x T x ln ( ) = −1,50 mol x 8,314 x 300 x ln ( ) = + 3427,03 J
V1 3000

860
1600 FÍSICO-QUÍMICA APLICADA EXERCÍCIOS COMENTADOS - IME – ITA – OLIMPÍADA

Item b) Cálculo da variação da energia interna: como o processo ocorre a temperatura constante, a energia interna é
igual a zero.

Item c) Cálculo da variação de entalpia: como o processo é isotérmico, a entalpia do processo é igual a zero.

Item d) Cálculo da quantidade de calor, a partir da primeira lei da termodinâmica.

∆U = Q + W
0=Q+W
Q = − W = − (+ 3428,12) = − 3427,03 J

Como a quantidade de calor é negativo, o sistema libera calor para a vizinhança.

TESTE 209 – (CONCURSO PARA DOCENTE – IFRN) Resolução:

Item a) Determinação da reação global e da ddp:


Multiplicando por dois a semirreação da prata: 2 Ag+(aq) + 2e- → 2 Ag(s) E = + 0,7994 V
Invertendo a semirreação do níquel: Ni(s) → Ni2+(aq) + 2e- E = + 0,25 V
Somando as duas semirreações, temos:
2 Ag+(aq) + 2e- → 2 Ag(s) E = + 0,7994 V
Ni(s) → Ni2+(aq) + 2e- E = + 0,25 V +
2Ag+(aq) + Ni(s) → 2 Ag(s) + Ni2+(aq) E = + 1,05 V

Item b) O potencial padrão E° da pilha é igual a + 1,05V.

Semirreação no cátodo: 2 Ag+(aq) + 2e- → 2 Ag(s)


Semirreação no ânodo: Ni(s) → Ni2+(aq) + 2e-
0,0592
Item c) Utilizando a equação de Nerst, temos: E = E° − x log Q
n

0,0592 [Ni+2 ] 0,0592 0,01


E = E° − x log {[Ag+]2 }. Substituindo os valores na equação: E = +1,05 − x log {(0,20)2 }
n 2

0,0592 0,0592
E = +1,05 − x log(0,25) = +1,05 − x (−0,60) = +1,05 − 0,0178 = +1,03 V
2 2

TESTE 210 – (OLIMPÍADA BRASILEIRA DE QUÍMICA) Resolução:

Item a) Distribuição eletrônica por subnível e por nível para cada elemento químico.
16S → 1s² 2s² 2p 3s² 3p
6 4

16S → 2 ) 8 ) 6

8O → 1s² 2s² 2p4


8O → 2 ) 6

17Cl → 1s² 2s² 2p6 3s² 3p5


17Cl → 2 ) 8 ) 7

Item b) Distribuição eletrônica por subnível e por nível para cada elemento químico.
16S → 1s² 2s² 2p 3s² 3p
6 4

16S → 2 ) 8 ) 6
O último elétron está no 3p4: n = 3; l = 1; ms = -1 e spin = +1/2.

8O → 1s² 2s² 2p4


8O → 2 ) 6

861
1600 FÍSICO-QUÍMICA APLICADA EXERCÍCIOS COMENTADOS - IME – ITA – OLIMPÍADA

O último elétron está no 2p4: n = 2; l = 1; ms = -1 e spin = +1/2.

17Cl → 1s² 2s² 2p6 3s² 3p5


17Cl → 2 ) 8 ) 7
O último elétron está no 3p5: n = 3; l = 1; ms = 0 e spin = +1/2.

Item c) A ordem crescente de raio atômico é a seguinte: RO < RCl < RS.
O elemento que vai apresentar a menor energia de ionização é o que apresenta o maior raio, ou seja, o enxofre (S).

Item d) O átomo que estiver na sua forma de ânion terá maior raio em comparação ao átomo na sua forma estável,
logo: RS-2 > RS.

Item e) Equação química: SO2(g) + 2 Cl2(g) → SOCl2(g) + Cl2O(g)

2 mol de Cl2 --------- 1 mol de SOCl2


(2 x 71) g ------------ 119 g
mCl2 ------------------- 595 g
84490
mCl2 = = 710 g
119

Item f) Equação química balanceada: SO2(g) + 2 Cl2(g) → SOCl2(g) + Cl2O(g)

Primeiramente será necessário a determinação do reagente limitante e do reagente em excesso:


10 g
nSO2 = = 0,156 mol
64 g/mol
20 g 0,282 mol
nCl2 = = = 0,141 mol (Reagente limitante)
71 g/mol 2

Pela estequiometria, temos:


2 mol de Cl2 --------------- 1 mol de SOCl2
(2 x 71) g ------------------ 119 g x (0,80)
20 g ------------------------ mSOCl2
1904
mSOCl2 = 142 = 13,41 g

Item g) Equação química: SO2(g) + 2 Cl2(g) → SOCl2(g) + Cl2O(g).


0 0 0 0 0
∆Hreação = {1 x HSOCl2
+ 1 x HCl2O
} − {1 x HSO2
+ 2 x HCl2
}

0
164,6 = {1 x HSOCl2
+ 1 x 80,3} − {1 x (−296,8)}

0
164,6 = HSOCl2
+ 377,4

0 kJ
HSOCl = −212,5
2 mol

TESTE 211 – (U. S. NATIONAL CHEMISTRY OLYMPIAD) Resolução: Invertendo e multiplicando por dois a primeira
semirreação e multiplicando por três a segunda semirreação:
+3
2Cr(s) → 2Cr(aq) + 6e− ∆G° = −6 x F x (+0,744)
+2 −
3 Ni(aq) + 6e → 3Ni(s) ∆G° = −6 x F x (−0,236)
2Cr(s) + 3 Ni+2 +3
(aq) → 2Cr(aq) + 3Ni(s) ∆G° = −6 x F x (+0,744) + [−6 x F x (−0,236)]

−6 x F x E° = −6 x F x (+0,744) + [−6 x F x (−0,236)]

862
1600 FÍSICO-QUÍMICA APLICADA EXERCÍCIOS COMENTADOS - IME – ITA – OLIMPÍADA

E° = +0,744 − 0,236 = +0,508 V

Item a) Equação química: 2Cr(s) + 3 Ni+2 +3


(aq) → 2Cr(aq) + 3Ni(s)

Item b) O eletrodo que apresenta a massa aumentada é o de níquel.

Item c) E° = +0,744 − 0,236 = +0,508 V

Item d) ∆G° = −n x F x E° = −6 x 96500 x 0,508 = −294132 J

Item e) Cálculo da constante de equilíbrio: −R x T x lnK = −294132 J

−294132
lnK = = +118,72
−8,314 x 298

K = 𝑒 +118,72
Item f) A partir da reação global, 2Cr(s) + 3Ni+2 +3
(aq) → 2Cr(aq) + 3Ni(s)

+3 2
RxT RxT [Cr(aq) ] x a3Ni
E = E° − x lnQ = E° − x ln { 3 }
nxF nxF
[Ni+2
(aq) ] x a 2
Cr

8,314 x 298 (0,01)2 x (1)3


E = +0,508 − x ln { }
6 x 96500 (0,01)3 x (1)2

8,314 x 298
E = +0,508 − x ln(100)
⏟ = + 0,488 V
6 x 96500 4,605

TESTE 212 – (OLIMPÍADA DE QUÍMICA DO RIO DE JANEIRO) Resolução:


112
( ) x 0,08206 x (500+273)
a) Cálculo da pressão inicial, considerando comportamento ideal: pinicial = 80
5,0
=
17,76 atm

b) Base de cálculo: 17,76 atm de pressão inicial para o trióxido de enxofre.

Tabela de equilíbrio químico:

2 SO3(g) ⇄ 2 SO2(g) O2(g)


Início 17,76 atm 0 0
Reage 2X 2X X
Equilíbrio 17,76 - 2X 2X X

Cálculo do valor de X: pTOTAL = 17,76 – 2X + 2X + X

23,5 atm = 17,76 atm – 2X + 2X + X

X = 23,5 atm – 17,76 atm = 5,74 atm

As pressões parciais dos constituintes gasosos são os seguintes:

PSO3 = 17,76 − 2X = 17,76 − 2 x (5,74 atm) = 6,28 atm

863
1600 FÍSICO-QUÍMICA APLICADA EXERCÍCIOS COMENTADOS - IME – ITA – OLIMPÍADA

PSO2 = 2X = 2 x (5,74 atm) = 11,48 atm

PSO3 = X = 5,74 atm


p2SO2 x pO2 (11,48)2 x 5,74
Cálculo da constante de equilíbrio em função das pressões parciais: K p = p2SO
= (6,28)2
=
3
131,79 x 5,74
39,44
= 19,18

Item c) Cálculo da constante de equilíbrio em função da concentração da quantidade de matéria (Kc): K p =


K c x (R x T)∆n
Kp 19,18 19,18
Kc = ∆n
= 3−2
= = 0,302
(R x T) (0,08206 x 773) 63,43

Item d) O equilíbrio será deslocado para a direita, no sentido da reação direta.

TESTE 213 – (OLIMPÍADA NORTE-NORDESTE DE QUÍMICA) Resolução:

Item a) Cálculo da constante de equilíbrio em função das pressões parciais: N2O4(g) ⇄ 2 NO2(g)

P2NO2
Expressão da consente de equilíbrio: K p =
PN2O4

(0,20)2 0,04
Cálculo da constante de equilíbrio em função das pressões parciais: K p = = = 0,05
0,8 0,8

Item b) Cálculo da constante de equilíbrio em função das concentrações das quantidades de matéria:

K p = K c x (R x T)∆n

Kp 0,05 0,05
Kc = ∆n
= = = 0,0022
(R x T) 0,08206 𝑥 273,15 22,41

Item c) Aplicando a lei de Hess:


Permanecendo a primeira equação: N2(g) + 2 O2(g) → 2 NO2(g) ∆H° = + 78,31 kJ.mol-1
Invertendo a segunda equação: N2O4(g) → N2(g) + 2 O2(g) ∆H° = + 9,67 kJ.mol-1

Somando as duas equações químicas:


N2(g) + 2 O2(g) → 2 NO2(g) ∆H° = + 78,31 kJ.mol-1
N2O4(g) → N2(g) + 2 O2(g) ∆H° = - 9,67 kJ.mol-1 +
N2O4(g) → 2 NO2(g) ∆H° = +78,31 - 9,67 = + 68,67 kJ.mol-1
Item d) Aumentando a temperatura do sistema reacional o equilíbrio é deslocado para a direita, uma vez que a
reação química é endotérmica.

TESTE 214 – (OLIMPÍADA BRASILEIRA DE QUÍMICA) Resolução: Alternativa A.


Cálculo da massa de monóxido de carbono, a partir da segunda equação química: 2 mol de C − − −
− 2 mol de CO
24 g − − − − 56 g
10g − − − − mCO
10 x 56
mCO = = 23,33 g
24

Determinação do reagente limitante:

75 x 0,87
nZnO = = 0,801 mol (Reagente limitante)
65,38 + 16

864
1600 FÍSICO-QUÍMICA APLICADA EXERCÍCIOS COMENTADOS - IME – ITA – OLIMPÍADA

23,33
nCO = = 0,833 mol
12 + 16

Pela relação estequiométrica da primeira equação química: 1 mol de ZnO − − − − 1 mol de Zn

(65,38 + 16) g − − − − 65,38 g


75 x 0,87g − − − − − − mZn

75 x 0,87 x 65,38
mZn = = 52,42 g
(65,38 + 16)

TESTE 215 – (OLIMPÍADA DE QUÍMICA DO RIO DE JANEIRO) Resolução: Alternativa C.


Equação química: 2 SO2(g) + O2(g) ⇌ 2 SO3(g)

[SO3 ]²
Expressão da constante de equilíbrio: K c = [SO 2
2 ] x [O2 ]

(4)² 16
Cálculo da constante de equilíbrio em função das concentrações das quantidades de matéria: K c = (2)2 x 3 = 12 =
1,33

TESTE 216 – (OLIMPÍADA DE QUÍMICA DO RIO DE JANEIRO) Resolução: Alternativa E.

Item a) Analisando o item A:

Invertendo a semirreação do alumínio e multiplicando por três a semirreação da prata:


Al(s) → Al+3
(aq) + 3e

∆G0 = −3 x F x (+ 1,67)
+3
3 Ag (aq) + 3e− → 3Ag (s) ∆G0 = −3 x F x (+ 0,80) +
Al(s) + 3 Ag +3 +3
(aq) → Al(aq) + 3Ag (s)

−3 x F x E° = −3 x F x (+ 1,67) − 3 x F x (+ 0,80)
−3 x F x E° = −3 x F x (+ 1,67) − 3 x F x (+ 0,80)
E° = 1,67 + 0,80 = +2,47 V

Ânodo = Alumínio
Câtodo: Prata

Item b) Analisando o item B:

Invertendo a semirreação do alumínio e multiplicando por dois:


2 Al(s) → 2 Al+3
(aq) + 6e

∆G0 = −6 x F x (+ 1,67)

Multiplicando a semirreação do cobre e multiplicando por três:


3 Cu+2 −
(aq) + 6e → 2 Cu(s) ∆G0 = −6 x F x (+0,34)
2 Al(s) → 2 Al+3(aq) + 6e

∆G0 = −6 x F x (+ 1,67)
3 Cu+2 −
(aq) + 6e → 2 Cu(s) ∆G0 = −6 x F x (+0,34) +
2 Al(s) + 3 Cu+2 +3
(aq) → 2 Al(aq) + 2 Cu(s)

−6 x F x E° = −6 x F x (+ 1,67) − 6 x F x (+0,34)
−6 x F x E° = −6 x F x (+ 1,67) − 6 x F x (+0,34)
E° = 1,67 + 0,34 = +2,01 V

865
1600 FÍSICO-QUÍMICA APLICADA EXERCÍCIOS COMENTADOS - IME – ITA – OLIMPÍADA

Ânodo: Alumínio
Câtodo: Cobre

Item c) Analisando o item C:

Invertendo a semirreação do alumínio e multiplicando por dois:


2 Al(s) → 2 Al+3
(aq) + 6e

∆G0 = −6 x F x (+ 1,67)

Multiplicando a semirreação do zinco e multiplicando por três:


3 Zn+2 −
(aq) + 6e → 2 Zn(s) ∆G0 = −6 x F x (− 0,76)
+3
2 Al(s) → 2 Al(aq) + 6e− ∆G0 = −6 x F x (+ 1,67)
+2
3 Zn(aq) + 6e− → 2 Zn(s) ∆G0 = −6 x F x (− 0,76) +
2 Al(s) + 3 Zn+2 +3
(aq) → 2 Al(aq) + 2 Zn(s)

−6 x F x E° = −6 x F x (+ 1,67) − 6 x F x (−0,76)
−6 x F x E° = −6 x F x (+ 1,67) − 6 x F x (−0,76)
E° = 1,67 − 0,76 = +0,90 V

Ânodo: Alumínio
Câtodo: Zinco

Item d) Analisando o item D:

Invertendo a semirreação do zinco:


+2
Zn(s) → 𝑍𝑛(aq) + 2e− ∆G0 = −2 x F x (+ 0,76)
+2
Zn(s) → 𝑍𝑛(aq) + 2e− ∆G0 = −2 x F x (+ 0,76)
Pb+2 −
(aq) + 2e → Pb(s) ∆G0 = −2 x F x (− 0,13) +
Zn(s) + Pb+2 +2
(aq) → Zn(aq) + Pb(s)

−6 x F x E° = −6 x F x (+ 1,67) − 6 x F x (−0,76)
−2 x F x E° = −2 x F x (+ 0,76) − 2 x F x (− 0,13)
E° = +0,76 − 0,13 = +0,63 V

Ânodo: Zinco
Câtodo: Chumbo

Item e) Analisando o item E:

Multiplicando a semirreação da prata por dois: 2Ag +1 −


(aq) + 2e → 2Ag (s) ∆G0 = −2 x F x (+ 0,80)
+2
Invertendo a semirreação de cobre: Cu(s) → Cu(aq) + 2e− ∆G0 = −2 x F x (−0,34)
Somando as semirreações:

2Ag +1 −
(aq) + 2e → 2Ag (s) ∆G0 = −2 x F x (+ 0,80)
Cu(s) → Cu+2
(aq) + 2e

∆G0 = −2 x F x (− 0,34) +
2Ag +1
(aq) + Cu(s) → Cu+2
(aq) + 2Ag (s)

−2 x F x E° = −2 x F x (+ 0,80) − 2 x F x (−0,34)
E° = +0,80 − 0,34 = +0,46 V

Ânodo: Cobre
Câtodo: Prata

866
1600 FÍSICO-QUÍMICA APLICADA EXERCÍCIOS COMENTADOS - IME – ITA – OLIMPÍADA

TESTE 217 – (OLIMPÍADA MINEIRA DE QUÍMICA) Resolução: Alternativa D.

TESTE 218 – (OLIMPÍADA MINEIRA DE QUÍMICA) Resolução: Alternativa D.


3
Cálculo do número de mol de gás nitrogênio: nN2 = R x T

10
Cálculo do número de mol de gás Hélio: nN2 = R x T
Cálculo do número de mol total: ntotal = nN2 + nHe

3 10 13
ntotal = + =
RxT RxT RxT

Cálculo da pressão total: ptotal x V = ntotal x R x T


13
ptotal x (3 + 2) = xRxT
RxT
13
ptotal = = 2,60 atm
5

TESTE 219 – (OLIMPÍADA MINEIRA DE QUÍMICA) Resolução: Alternativa B.

I. Verdadeiro. Aumentando a concentração da quantidade de matéria do dióxido de nitrogênio, o equilíbrio é deslocado


para a direita.
II. Verdadeiro. Aumentando a concentração da quantidade de matéria do tetróxido de dinitrogênio, o equilíbrio é
deslocado para a esquerda.
III. Falso. Aumentando a temperatura do sistema reacional o equilíbrio é deslocado para a esquerda (reação inversa).
IV. Falso. Aumentando a pressão total do sistema reacional, o equilíbrio é deslocado para a direita (reação direta).
V. Falso. A constante de equilíbrio muda com a temperatura, segundo a equação de Van’t Hoff.

TESTE 220 – (OLIMPÍADA MINEIRA DE QUÍMICA) Resolução: Alternativa D.

A equação química balanceada pelo método das tentativas fica da segunte maneira: 5 H2O(ℓ) + 2 HCℓO4(ℓ) + P4O10(s)
⇌ 4 H3PO4(ℓ) + Cℓ2O7(s)

TESTE 221 – (OLIMPÍADA MINEIRA DE QUÍMICA) Resolução: Alternativa B.


Analisando a primeira equação química: NH3 + H2O ⇌ NH4+ + OH-
NH3 = base de Brönsted – Lowry;
H2O = ácido de Brönsted – Lowry ;
NH4+ = ácido de Brönsted – Lowry;
OH- = base de Brönsted – Lowry.
Analisando a segunda equação química: HNO3 + H2O ⇌ NO3- + H3O+
HNO3 = ácido de Brönsted – Lowry;
H2O = base de Brönsted – Lowry;
NO3- = base de Brönsted – Lowry;
H3O+ = ácido de Brönsted – Lowry.
Analisando a terceira equação química: HNO3 + H2SO4 ⇌ H2NO3+ + HSO4-
HNO3 = base de Brönsted – Lowry;
H2SO4 = ácido de Brönsted – Lowry;
H2NO3+ = ácido de Brönsted – Lowry;
HSO4- = base de Brönsted – Lowry.

867
1600 FÍSICO-QUÍMICA APLICADA EXERCÍCIOS COMENTADOS - IME – ITA – OLIMPÍADA

TESTE 222 – (MESTRE JOÃO ROBERTO DA PACIÊNCIA NABUCO) Resolução: O efeito coligativo a ser estudado
é o tonoscópico. Sulfeto de magnésio: MgCl2(aq) → Mg +2 −
(aq) + 2 Cl(aq)

Se tratando de uma solução iônico, será necessário utilizar a relação de correção de van´t Hoff, considerando que o
grau de dissociação seja de 80% (α).
p0 − p
= K t x W x [1 + (n − 1). α]
p0
msoluto
( )
<MM>soluto 19 mol
Cálculo da molalidade: W = (kg) = 95 x 0,360 = 0,556 kg
msolvente

32 − p 18
= x W⏟ x [1 + (3 − 1) 𝑥 0,80]
32 1000 0,556

32 − p
= 0,0260
32

p = 32 − 32 x 0,0260 = 31,17 mmHg

TESTE 223 – (MESTRE JOÃO ROBERTO DA PACIÊNCIA NABUCO - MODIFICADA) Resolução:


Item a) aumentando a temperatura do sistema reacional, o equilíbrio será deslocado para a direita;
Item b) Adicionando um catalisador não haverá nenhuma influência no equilíbrio químico;
Item c) diminuindo a concentração de oxigênio molecular, o equilíbrio será deslocado para a esquerda;
Item d) aumentando a concentração ou até mesmo a pressão parcial do SO3(g), o equilíbrio será deslocado para a
esquerda;
Item e) aumentando a pressão total do sistema reacional, o equilíbrio será deslocado para a direita;
Item f) Diminuindo a temperatura do sistema reacional, o equilíbrio será deslocado para a esquerda;
Item g) aumentando a concentração de trióxido de enxofre o equilíbrio será deslocado para a esquerda.

TESTE 224 – (GRILLO) Resolução: Cálculo do volume inicial a partir da utilização da equação dos gases ideais:
nxRxT 2,0 x 0,08206 x 276
Vinicial = = 105000 = 43,71 L
pinicial ( )
101325

Como o processo ocorre a volume constante, o trabalho realizado é igual a zero (W = 0). Logo, pela primeira lei da
termodinâmica, temos:

∆U = Q + W
∆U = Q + 0
∆U = Q V

Cálculo da energia interna e também da quantidade de calor a volume constante: ∆U = Q V = n x CV x ∆T =


5
2 x 2 x 8,314 x (390 − 276) = + 4738,98 J

7
Cálculo da entalpia: ∆H = n x Cp x ∆T = 2 x 2 x 8,314 x (390 − 276) = + 6634,57 J
P P
Cálculo da pressão final, a partir da equação combinada dos gases ideais: Tinicial = Tfinal
inicial final
105 kPa Pfinal
=
276 K 390 K
105 x 390
Pfinal = = 148,37 kPa
276

868
1600 FÍSICO-QUÍMICA APLICADA EXERCÍCIOS COMENTADOS - IME – ITA – OLIMPÍADA

TESTE 225 – (OLIMPÍADA DE QUÍMICA DO RIO DE JANEIRO) Resolução: Alternativa D.

Soluto: água (H2O)


Solvente: etilenoglicol (C2H6O2; 62 g.mol-1) com fator de van´t Hoff igual a um (i=1)

O efeito coligativo a ser estudado é a da crioscopia: |∆Tc | = K c x W x i


massa
Cálculo da massa de solvente (água), a partir da densidade: d =
volume

g massa
1,0 =
mL 5000 mL

Massa = 5000 g
msoluto
Cálculo da massa de soluto: |−10 − 0| = 1,86 x x1
62 x 5,0

10 x 62 x 5,0 3100
msoluto = = = 1666,67 g
1,86 1,86
massa
Cálculo do volume do etilenoglicol, a partir da densidade: d = volume

1666,67
V= = 1515,15 mL (1,51 L)
1,1

TESTE 226 – (OLIMPÍADA DE QUÍMICA DO RIO DE JANEIRO) Resolução: Alternativa A.

Mg(NO3 )2(aq) → Mg +2 −
(aq) + 2 NO3(aq)

solvente
solução puro
Tc − Tc = K c x W x {1 + (n − 1). α}

|−2,80 − 0| = 1,86 x W x {1 + (3 − 1) x 1}

2,80 2,86 mol


W= = = 0,502 (molal)
1,86 x 2 5,58 kg

TESTE 227 – (OLIMPÍADA DE QUÍMICA DO RIO DE JANEIRO) Resolução: Alternativa D.

∆Teb = K eb x W x i

Ca(NO3 )2(aq) → Ca+2 −


(aq) + 2 NO3(aq)

g massa
Cálculo da massa do solvente, a partir da densidade: 1 mL = 200 mL

Massa de solvente = 200 g (0,20 𝑘𝑔)

solução
solvente
puro R X T2 msoluto
Teb − Teb = x x {1 + (n − 1). α}
1000 X LV < MM >soluto x m(kg)
solvente

2 x (100+273)2 70,85
Substituindo os valores na equação acima: 103,14 − 100 = 1000 X LV
x 164 x 0,20 x {1 + (3 − 1) x 0,90}

869
1600 FÍSICO-QUÍMICA APLICADA EXERCÍCIOS COMENTADOS - IME – ITA – OLIMPÍADA

2 x (373)2 70,85
3,14 = x x 2,80
1000 X LV 164 x 0,20

3140 x LV = 278258 x 2,16 x 2,80

278258 x 2,16 x 2,80 cal


LV = = 535,96
3140 g

TESTE 228 – (OLIMPÍADA DE QUÍMICA DO RIO DE JANEIRO) Resolução: Alternativa D.


Cálculo da constante cinética (k), a partir da equação do tempo de meia-vida de cinética de primeira ordem: k =
0,693
193 h
= 3,59 x 10−3 h−1
m
Cálculo do tempo (t): ln (m final ) = − k x tempo
inicial
0,30
ln ( ) = − 3,59 x 10−3 x tempo
1,0
ln(0,30) −1,20
tempo = = = 334,26 h
− 3,59 x 10−3 − 3,59 x 10−3

TESTE 229 – (OLIMPÍADA DE QUÍMICA DO RIO DE JANEIRO) Resolução: Alternativa E.


O equilíbrio químico se estabelece quando a velocidade da reação direta for igual a velocidade da reação inversa.

TESTE 230 – (OLIMPÍADA NORTE – NORDESTE DE QUÍMICA) Resolução: Determinação dos três primeiros
itens da questão. Para o funcionamento pleno da pilha será necessário:

Invertendo a primeira semirreação: Tl(s) → Tl+


(aq) + e

∆G0 = −1 x F x (+ 0,34)
Permanecendo a segunda semirreação: Fe+3 − +2
(aq) + e → Fe(aq) ∆G0 = −1 x F x (+ 0,77)

Somando as duas semirreações:

Tl(s) → Tl+
(aq) + e

∆G0 = −1 x F x (+ 0,34)
Fe+3 −
(aq) + e → Fe(aq)
+2
∆G0 = −1 x F x (+ 0,77) +
Tl(s) + Fe+3
(aq) → Tl+
(aq) + Fe+2
(aq) ∆G0 = −1 x F x (+ 0,34) + [−1 x F x (+ 0,77)]

−1 x F x E 0 = −1 x F x (+ 0,34) + [−1 x F x (+ 0,77)]


E 0 = + 0,34 + 0,77 = + 1,11 V

Item a) As semirreações são as seguintes:

Semirreação que ocorre no ânodo: Tl(s) → Tl+


(aq) + e

Semirreação que ocorre no cátodo: Fe+3 − +2


(aq) + e → Fe(aq)

Item b) Reação global: Tl(s) + Fe+3 + +2


(aq) → Tl(aq) + Fe(aq)

0 0
Item c) Cálculo do potencial padrão: E 0 = Ecátodo − Eânodo = +0,34 + 0,77 = + 1,11 V

Item d) Caso a concentração de [Tl+


(aq) ] diminua, o potencial vai aumentar. Este processo pode ser observado pela
equação de Nernst.

0,0592
E = E0 − x lnQ
n

870
1600 FÍSICO-QUÍMICA APLICADA EXERCÍCIOS COMENTADOS - IME – ITA – OLIMPÍADA

0,0592
0
[Tl+ +2
(aq) ] 𝑥 [Fe(aq) ]
E=E − x ln [ ]
n 𝑎 𝑥 [Fe+3 ]
𝑇𝑙 (aq)

Conforme a equação acima, diminuindo a concentração de [Tl+


(aq) ] e mantendo as demais concentrações constantes,
ou seja, 1 mol/L, o potencial vai diminuir.

TESTE 231 – (OLIMPÍADA DE QUÍMICA DO DISTRITO FEDERAL) Resolução:

Item a) Semirreação para a redução: Mn+7 − +2


(aq) + 5e → Mn(aq)
− 0
Semirreação para a oxidação: 2 Br(aq) → Br2(𝑙) + 2e−

Item b) Agente oxidante: MnO−


4

Agente redutor: Br(aq)
2
0,0592 [Mn+2 ] x a5Br x a8H2O
Item c) Equação de Nernst: E = 0,42 − 4 x log [MnO − ]2
x [Br− ]10 x [H+ ]16
4

Teste 232 – (OLIMPÍADA BRASILEIRA DE QUÍMICA JÚNIOR) Resolução: Alternativa D.

O titânio é um elemento que se adapta muito bem em tecidos ósseos e por isso é muito usado em implantes dentários
e na substituição de articulações. Suas principais características estão listadas logo a seguir:
✓ resistente quanto o aço e menos denso;
✓ suas ligas são usadas na indústria aeroespacial pela leveza e capacidade e por suportar altíssimas
temperaturas;
✓ resistente à corrosão é usado
✓ aplicação em joalheria, objetos de decoração e cozinha;
✓ na fabricação de moedas e medalhas comemorativas;
✓ na produção de carcaças de relógios.
m

TESTE 233 - Resolução: Analisando a primeira situação: π1 = { <MM>


} x R x T1 x i
Vsolução

32
Substituindo os dados do problema na equação acima temos: 8,20 = { }x R x (27 + 273) x i
<MM>

8,20 x < MM > = 32 x R x (27 + 273) x i

8,20 x < MM > = 9600 x R x i (Equação A)

C
Analisando a situação II: π2 = {<MM>} x R x T2 x i

C
Substituindo os dados do problema na equação acima temos: π2 = {<MM>} x R x T2 x i

π2 x < MM > = 10 x R x (47 + 273)x i (Equação B)


8,20 x<MM> 9600 x R x i
Dividindo a equação (A) pela equação (B): π x<MM> = 10 x R x (47+273)x i
2

8,20 x < MM >


=3
π2 x < MM >

8,20
π2 = = 2,73 atm
3

871
1600 FÍSICO-QUÍMICA APLICADA EXERCÍCIOS COMENTADOS - IME – ITA – OLIMPÍADA

TESTE 234 – (OLIMPÍADA DE QUÍMICA DO DISTRITO FEDERAL) Resolução: Alternativa C.

Cálculo da massa da solução para um litro de solução:


1,30 g de solução − − − − 10−3 L
𝑚solução − − − − − − − − 1,0 L
msolução = 1300 g

Cálculo da massa de ácido sulfúrico, a partir da concentração da quantidade de matéria: [H2 SO4 ] =
mH2SO4
<MM>H2 SO4 x Vsolução
mH2 SO4
3,95 =
98 x 1,0

mH2 SO4 = 387,10 g

Cálculo da massa do solvente: msolução = mH2 SO4 + msolvente

1300 g = 387,10 g + msolvente

msolvente = 912,9 g (0,912 kg)

nsoluto 3,95 mol


Cálculo da molalidade (W): W = = = 4,33
msolvente 0,912 kg

TESTE 235 – (GRILLO) Resolução: Cálculo da concentração comum: C = 10 x d x (%) = 10 x 1,75 x 68 =


𝑔
1190 𝐿

C 1190 mol
Cálculo da concentração da quantidade de matéria: [H2 SO4 ] = <MM> = 98
= 12,14 L
soluto

Cálculo da normalidade: N = [H2 SO4 ]. x, onde x corresponde ao número de hidrogênio não-ionizáveis (2).
eqg
N = 12,14 x 2 = 24,28
L
Cálculo da molalidade:
Informação do problema: 68% em massa de ácido sulfúrico.
68 gramas de ácido sulfúrico/100 gramas de solução
msolvente = msolução − msoluto

msolvente = 100 g − 68 g = 32 g
68
nH2 SO4 ( ) mol
Cálculo da molalidade (W): W = 98
= 32 x 10−3 = 21,68
msolvente
(kg) kg

TESTE 236 – (ENADE) Resolução: Alternativa C.

Determinação do reagente limitante e do reagente em excesso:


500000
nsal = = 1612,90 mol
310
260000 2653,06
nácido = 98
= 2
= 1326,53 mol (Reagente limitante)

Cálculo da massa de superfosfato, a partir da relação estequiométrica: Ca3 (PO4 )2 + 2 H2 SO4 → 2 CaSO4 +
Ca(H2 PO4 )2

2 mol de H2 SO4 − − − − − − − 1 mol de Ca(H2 PO4 )2

872
1600 FÍSICO-QUÍMICA APLICADA EXERCÍCIOS COMENTADOS - IME – ITA – OLIMPÍADA

2653,06 mol de H2 SO4 − − − − nsal

2653,06
nsal = = 1326,53 x 0,92 = 1,22 kmol
2,0

TESTE 237 – (MESTRE JOÃO ROBERTO DA PACIÊNCIA NABUCO) Resolução: Equação química: Al(s) +
3
3HCl(aq) → AlCl3(aq) + 2 H2(g)

Determinação do reagente limitante:

2,55
nAl = = 0,0944 mol
27
0,225
nHCl = [HCl] x Vsolução = 0,50 x 0,45 = 0,225 mol ∴ = 0,075 mol (Reagente limitante)
3

Cálculo do volume de gás hidrogênio:


3
3 mol de HCl(aq) − − − − mol de H2
2
nHCl − − − − − − − − − nH2
3
3 x nH2 = x nHCl
2 pH 2 x V H 3
Considerando que o gás apresenta comportamento ideal: 3 x RxT
2
= 2 x [HCl] x Vsolução

Substituindo os valores na equação acima, temos:


0,50 x VH2 3
3x = x 0,50 x 0,45
0,08206 x (37 + 273) 2

VH2 = 5,72 L

TESTE 238 - (MESTRE JOÃO ROBERTO DA PACIÊNCIA NABUCO) Resolução:


m 250 g
Item a) Cálculo da concentração mássica ou concentração comum: C = V soluto = 0,525 = 476,19 L
solução
250
nKCl ( ) mol
Item b) Cálculo da concentração da quantidade de matéria: [KCl] = V = 39+35,5
0,525
= 6,39 L
solução
250
nKCl ( ) mol
Item c) Cálculo da molalidade: W = (kg) 39+35,5
= 0,725 = 4,63 kg
msolvente
250
Item d) Cálculo do número de mol do cloreto de potássio: nKCl = 74,5 = 3,35 mol

725
Cálculo do número de mol da água: nH2 O = 18
= 40,28 mol

nKCl 3,35
X KCl = = = 0,0768
nKCl + nH2 O 3,35 + 40,28

nH2 O 40,28
X H2 O = = = 0,923
nH2 O + nKCl 40,28 + 3,35

msolução 250 + 725 g


Item e) Cálculo da densidade: d = Vsolução
= 525
= 1,86 mL
msoluto 250
Item f) Cálculo do t´tulo em massa (massa/massa): (%)massa/massa = = = 0,256 (25,60%)
msolução 250 + 725

873
1600 FÍSICO-QUÍMICA APLICADA EXERCÍCIOS COMENTADOS - IME – ITA – OLIMPÍADA

TESTE 239 – (OLIMPÍADA DE QUÍMICA DO RIO DE JANEIRO) Resolução: Alternativa C.


Sabendo que o tempo de meia vida para um processo que apresenta cinética de primeira ordem é dado por: t1/2 =
ln2
k
.
ln2
Sabendo que o tempo de meia-vida é igual a 25 minutos, temos: 25 min = k
ln2 0,693
k= = = 2,77 x 10−2 min−1
25 25 min
60 min
k = 2,77 x 10−2 min−1 x ( ) = 1,66 h−1
1h

TESTE 240 – (OLIMPÍADA BRASILEIRA DE QUÍMICA) Resolução: Alternativa C.

Equação química: 2 NH3(g) + 3 CuO(s) → 3 Cu(s) + N2(g) + 3 H2 O(g)


Cálculo da massa de óxido de cobre II puro: mCuO = 100 x 0,95 = 95 g
mCuO 95
Cálculo do número de mol de óxido de cobre II: nCuO = <MM> = 79,55 = 1,19 mol
CuO
Item a)
3 mol de CuO(s) − − − − − 3 mol de H2 O(g) x 0,75 x 6,02 x 1023 moléculas de vapor
1,19 mol de CuO(s) − − − − X
𝑋 = 5,35 x 1023 moléculas

Item b)
3 mol de CuO(s) − − − − − 1 mol de N2(g) x 0,75 x 22,4 L
1,19 mol de CuO(s) − − − − VN2
VN2 = 6,66 L

Item c)
3 mol de CuO(s) − − − − − 3 mol de Cu x 0,75
1,19 mol de CuO(s) − − − − nCu
nCu = 0,892 mol

6,02 x 1023 moléculas


Item d) nCu = 0,892 mol x 1 mol
= 5,35 x 1023 moléculas

Item e) 2 NH3(g) + 3 CuO(s) → 3 Cu(s) + N2(g) + 3 H2 O(g)


3 x 79,55 g − − − − − − 3 x 63,55 − − − 1 x 28 − − − 3 x 18
95 g − − − − − − − − − mCu − − − − − m𝐍2 − − − − mH2 O
mCu = 75,89 x 0,75 = 56,92 g
m𝐍2 = 11,14 x 0,75 = 8,36 g
mH2 O = 21,49 x 0,75 = 16,12 g
mCu + m𝐍2 + mH2 O = 56,92g + 8,36g + 16,12g = 81,4 g

TESTE 241 – (GRILLO) Resolução: Tabela de equilíbrio químico:

CO(g) 3H2(𝑔) ⇄ CH4(g) H2 O(l)


Início 1,0 3,0 0 0
Reage X = 0,387 3X = 1,164 X = 0,387 X = 0,387
Equilíbrio 1,0 - X 3,0 – 3X X 0,387

Cálculo da concentração da quantidade de matéria para as espécies gasosas:

1,0 − X 1,0 − 0,387 mol


[CO(g) ] = = = 0,0387
10 10 L

874
1600 FÍSICO-QUÍMICA APLICADA EXERCÍCIOS COMENTADOS - IME – ITA – OLIMPÍADA

3,0 − 3X 3,0 − 3 x 0,387 mol


[H2(𝑔) ] = = = 0,1839
10 10 L
X 0,387 mol
[CH4(𝑔) ] = = = 0,0389
10 10 L
X 0,387 mol
[H2 O(l) ] = = = 0,0389
10 10 L
[CH4(g) ] x [H2 O(l) ]
Cálculo da constante de equilíbrio em função das quantidades de matéria: K c = =
[CO(g) ] x [H2(g) ]³
0,0389 𝑥 0,0389
0,0613 x [1,839]3
= 3,70

Cálculo da pressão da mistura reacional, considerando que o comportamento seja ideal:

(0,613 + 0,389 + 0,389 + 1,839) x 0,08206 x (273 + 727)


ptotal = = 26,50 atm
10

TESTE 242 – Resolução:


PCH3OH
Item a) Expressão da constante de equilíbrio em função das pressãoes parcias: K p = 1/2
PO x P2H2 x aC
2

0
Item b) Cálculo da constante de equilíbrio em função da energia livre de Gibbs padrão: ∆Greação = −R x T x ln K 𝑝
− 8,314 x (25 + 273) x ln K p = − 166270

8,314 x 298 x ln K p = 166270

166270 166270
( )
ln K p = = e 2477,572 = e67,11
2477,572

K p = 1,40 x 1029

Item c) O sinal negativo da variação da energia livre de Gibbs padrão apresenta que o processo de formação do
metanol é espontâneo e consequentemente, conforme os cálculos apresentados no item b, a constante de equilíbrio
apresenta um valor maior que um (K 𝑝 > 1), o que significa que o equilíbrio está deslocado para a direita.

TESTE 243 – (MESTRE JOÃO ROBERTO DA PACIÊNCIA NABUCO) Resolução: Considerando que a gelatina seja
um soluto molecular (i = 1), a pressão osmótica será calculada da seguinte maneira: π = [gelatina] x R x T x i
mgelatina
π= xRxTxi
< MM >gelatina x Vsolução

67,20 0,820
= x 0,08206 x (27 + 273) x 1,0
760 < MM >gelatina x 0,150
15341,94 g
< MM >gelatina = = 1522,02
10,08 mol

TESTE 244 – (MESTRE JOÃO ROBERTO DA PACIÊNCIA NABUCO) Resolução: Cálculo da quantidade de calor
para o aquecimento da quantidade de água: Q = mH2 O x Cp x ∆T

Q = 1000 x 73,50 x (90 − 20) = 5145000 J

875
1600 FÍSICO-QUÍMICA APLICADA EXERCÍCIOS COMENTADOS - IME – ITA – OLIMPÍADA

Cálculo do número de mol de gás metano:


1 mol de CH4 − − − − 890000 J
nCH4 − − − − − − − − 5145000 J

5145000 J
nCH4 = = 5,78 mol
890000 J
mCH
Cálculo da massa de gás metano: nCH4 = <MM>4 = 5,78 mol
CH4

mCH4 = 5,78 x 16 = 92,48 g

TESTE 245 – (MESTRE JOÃO ROBERTO DA PACIÊNCIA NABUCO) Resolução: Considerando que a partícula
gasosa apresenta comportamento ideal e que sua configuração é esférica, temos:
𝑏
𝑉 = 𝑁 , onde b é a constant de van der Waals e NA é a constant de Avogadro. Substituindo os valores na equação
𝐴
b
matemática, temos: V = N
A

4 x π x R³ b
=
3 NA

4 x π x R³ 39,4 x 10−6
=
3 6,02 x 1023

3 x 39,4 x 10−6
R3 =
4 x 3,14 x 6,02 x 1023

R3 = 1,56 x 10−29
3
R = √1,56 x 10−29 = 2,50 𝑥 10−10 (0,250 nm)

TESTE 246 – (OLIMPÍADA MINEIRA DE QUÍMICA) Resolução: Alternativa D.

Invertendo a primeira semirreação e somando com a segunda semirreação.

Fe(s) → Fe2+(aq) + 2e− E0 = + 0,44 V ∆G° = −2 x F x (+0,44)


Fe3+(aq) + 3e− → Fe(s) E0 = − 0,040 V ∆G° = −3 x F x (−0,040)
Fe (aq) + 1e → Fe (aq)
3+ − 2+ ∆G° = −1 x F x E°
∆G° = −1 x F x E° = −2 x F x (+0,44) − 3 x F x (−0,040)
−1 x E° = −0,88 + 0,120

E° = +0,76 V

TESTE 247 – (U. S. NATIONAL CHEMISTRY OLYMPIAD) Resolução: Alternativa B.

876
1600 FÍSICO-QUÍMICA APLICADA EXERCÍCIOS COMENTADOS - IME – ITA – OLIMPÍADA

Conforme as áreas destacadas na figura abaixo, a região 1 refere-se a reação intermediária e a região 2 é o complexo
ativado.

H T
TESTE 248 – (GRILLO) Resolução: Para 1 mol de um sistema gasoso, temos: ∫H 2 dH = ∫T 2 1 x Cp x dT
1 1

T2
C
H2 − H1 = ∫
⏟ 1 x (A + BT + ) x dT
T1 T
∆H

T
T T2 2 T
∆H = A x T|T21 + B x | + C x ln T|T21
2 T
1

T22 T12
∆H = A x (T2 − T1 ) + B x [ − ] + C x (lnT2 − lnT1 )
2 2

T22 T12 T2
∆H = A x (T2 − T1 ) + B x [ − ] + C x ln
2 2 T1

TESTE 249 – (OLIMPÍADA DE QUÍMICA DO RIO DE JANEIRO) Resolução: Alternativa B.

Para melhorar o rendimento para produção de ácido fórmico será necessário utilizar o princípio de Le Chatelier. Então,
para que o equilíbrio seja deslocado para a direita, será necessário:

✓ Aumentar a pressão do sistema reacional;


✓ Aumentar a temperatura do sistema reacional, uma vez que o processo é endotérmico;
✓ Aumentar a pressão parcial de hidrogênio ou de dióxido de carbono.

TESTE 250 – (IME) Resolução: Alternativa C.


Equação química da combustão do álcool etílico: C2 H6 O(l) + 3 O2(g) → 2 CO2(g) + 3 H2 O(l) ∆H =
cal
−7607 g
Equação química da combustão do ácido acético: C2 H6 O2 (l) + 2 O2(g) → 2 CO2(g) + 2 H2 O(l) ∆H =
cal
−3496 g
Para a resolução deste problema será necessário utilizar a lei de Hess, invertendo a segunda equação química:
cal
2 CO2(g) + 2 H2 O(l) → C2 H6 O2 (l) + 2 O2(g) ∆H = +3496 g
cal
C2 H6 O(l) + 3 O2(g) → 2 CO2(g) + 3 H2 O(l) ∆H = −7607 g
+
cal 𝑐𝑎𝑙 cal
C2 H6 O(l) + O2(g) → C2 H6 O2 (l) + 3 H2 O(l) ∆H = +3496 g − 7607 𝑔
= −3571 g

877
1600 FÍSICO-QUÍMICA APLICADA EXERCÍCIOS COMENTADOS - IME – ITA – OLIMPÍADA

CAPÍTULO XII

REFERÊNCIA
BIBLIOGRÁFICA

878
1600 FÍSICO-QUÍMICA APLICADA EXERCÍCIOS COMENTADOS - IME – ITA – OLIMPÍADA

I. Físico-Química: Uma aplicação aos materiais. Rupen Adamian, Ericksson Rocha e Almendra. Rio de Janeiro:

COPPE – UFRJ, 2002.

II. Curso de Termodinâmica Aplicada às Máquinas. Abrahão Izecksohn; Imprensa Nacional Rio de Janeiro – 1943.

III. Introduction to the Thermodynamics of Materials Fourth Edition. David R. Gaskell. Taylor & Francis, New York –

London, 2003.

IV. Curso de Física Básica 2 – Fluidos. H. Moysés Nussenzveig. São Paulo: Edgard Blucher, 1981.

V. Principles of Physical Chemistry Four Edition. Samuel H. Maron; Carl F. Prutton. Editora Collier Macmillian Student

Editions, New York, 1965.

VI. Química Geral – Volume 2. Linus Pauling. AO LIVRO TÉCNICO SA – Indústria e Comércio. Rio de Janeiro, 1982.

VII. Química Geral – Volume 2. Brady, J. E.; Humiston, G. E. tradução de Cristina Maria Pereira dos Santos e Roberto

Barros Faria. Segunda edição. Rio de Janeiro. Livros técnicos e científicos Editora, 1986.

VIII. Físico-Química – Volume 1, Levine, I. N. Tradução e revisão técnica Edilson Clemente da Silva, Oswaldo

Esteves Barcia – Rio de Janeiro: LTC 2012.

IX. Termodinâmica e Física da Estrutura da Matéria – Segunda Edição. Rui Manuel A. Dilão. Escolar Editora, 2014.

X. Antonio Braz de Pádua; Cléia Guiotti de Pádua; João Lucas Correia Silva; Ricardo Spagnuolo Martins; Felipe

Barreiro Postali; Luiz Augusto Calvo Tiritan.Termodinâmica clássica ou termodinâmica do equilíbrio: aspectos

conceituais básicos. Ciências Exatas e da Terra, Londrina, v. 29, n. 1, p. 57-84, jan./jun. 2008.

XI. Atkins, P, & Jones, L. Princípios de Química – Questionando a vida moderna e o meio ambiente – Quinta Edição.

Editora bookman, 2012.

XII. Atkins, P. W.; Paula de, J. Físico-Química”, 8ªedição, volume um. Livros Técnicos e Científicos - LTC, Rio de

Janeiro, 2008.

XIII. Atkins, P. W.; Paula de, J. Físico-Química”, 10ªedição, volume um. Livros Técnicos e Científicos - LTC, Rio

de Janeiro, 2018.

XIV. Crockford, H. D. e Samuel B. Knight; tradução e revisão de Horácio Macedo, Livre docente da UFRJ – Instituto

de Química. Fundamentos de Físico-Química. Rio de Janeiro, Livros Técnicos e Científicos, 1977.

XV. Moore, W. J. Físico-Química - tradução da quarta edição americana: Tibor Rabockai [e outros]. Supervisão Ivo

Jordan. Editora da Universidade de São Paulo, 1976.

XVI. Castellan, G. W. Physical Chemistry, segunda edição. Addison – Wesley Publishing Company, 1964.

879
1600 FÍSICO-QUÍMICA APLICADA EXERCÍCIOS COMENTADOS - IME – ITA – OLIMPÍADA

XVII. Feltre, R. & Yoshinaga, S. Físico-Química – Volume 1 – teoria e Exercícios, Primeira edição. Editora moderna,

1968.

XVIII. Nabuco, João Roberto da Paciência & Barros. Físico-Química. Rio de Janeiro, Primeira edição. Editora Ao

livro técnico, 1979.

XIX. Nabuco, João Roberto da Paciência & Barros. Química Geral. Rio de Janeiro, Primeira edição. Editora Ao

livro técnico, 1979.

XX. Mahan, Bruce M. Departamento de Química, Universidade da Califórnia, Berkeley, Segunda edição, revisada.

Editora Edgard Bluncher LTDA, United States of America, 1978.

XXI. Feltre, R. & Yoshinaga, S. Físico-Química – Volume 3 – teoria e Exercícios, Primeira edição. Editora moderna,

1968.

XXII. Wood, J. H.; Keenan, C. W.; Bull, W. E.; Bowman, N. S. Fundamentals of college chemistry. A Harper

International Edition, 1964.

XXIII. Luiz, A. M. Termodinâmica – Teoria & Problemas. Livros Técnicos e Científicos - LTC, Rio de Janeiro, 2012.

XXIV. Warren, B. A. The First law of thermodynamics in a salty ocen. Progress in Oceanography, page 149 – 167,

70 (2006).

XXV. Bergman, T. L. ; Lavine, A. S.; Incropera, F. P.; Dewitt, D. P. Fundamentos da Transferência de Calor e

Massa; Rio de Janeiro: LTC, 2014.

XXVI. Braga Filho, W.. Transmissão de Calor; São Paulo: Thomson, 2006.

XXVII. Modell, M.; Reid, R. C. Thermodynamics and its applications. New Jersey: Prendice-Hall, 1974.

XXVIII. Smith, J. M.; Van Ness, H. C.; Abbott, M. M. Introdução à termodinâmica da engenharia química. 5.ed. Rio

de Janeiro: Livros Técnicos e Científicos, 2000.

XXIX. Pádua, A. B.; Pádua, C. G.; SILVA, J. L. C. A história da termodinâmica, uma ciência fundamental. Londrina:

EDUEL, 2008.

XXX. Ebbing, D. D., Química Geral. 5ª Ed. Rio de Janeiro, Livros Técnicos e Científicos, V. 1, 1998. 569 p.

XXXI. Kotz, J. C.; Treichel, P.J. Química e Reações Químicas. 3ª ed. Rio de Janeiro, Livros Técnicos e Científicos,

Volumes 1 e 2.

XXXII. Himmelblau, D.M. & Riggs, J. B. Engenharia Química – Princípios e cálculos. LTC; 2014.

XXXIII. Russel, J. B.; Química Geral – Segunda Edição. Pearson Makron Books, 1994.

880
1600 FÍSICO-QUÍMICA APLICADA EXERCÍCIOS COMENTADOS - IME – ITA – OLIMPÍADA

XXXIV. Rogado, J. A Grandeza quantidade de matéria e sua unidade, o mol: Algumas considerações sobre

dificuldades de ensino e aprendizagem. Ciência & Educação, v.10, n.1, p. 63-73, 2004.

XXXV. Levenspiel, O. Engenharia das reações químicas. São Paulo, Edgard Blücher Ltda., 2000.

XXXVI. Fogler, H.S.; Elements of Chemical Reaction Engineering, Pearson Education Inc, New Jersey - USA, 4th Ed.

(2006).

881
1600 FÍSICO-QUÍMICA APLICADA EXERCÍCIOS COMENTADOS - IME – ITA – OLIMPÍADA

CAPÍTULO XIII

APÊNDICE

882
1600 FÍSICO-QUÍMICA APLICADA EXERCÍCIOS COMENTADOS - IME – ITA – OLIMPÍADA

APÊNDICE A – CONVERSÃO DE UNIDADE

ALGUMAS CONSTANTES NECESSÁRIAS


a) Unidade de massa atômica (u): 1u = 1,6606 x 10-24 g
b) Número de Avogadro (N): 6,0221415 x 1023 mol-1
c) Constante de Boltzmann (k): 1,3806 x 10-23 m².kg.s-2.K-1
d) Constante de Faraday (F): 96485 C.mol-1
e) Constante de Planck (h): 6,626 x 10-34 J.s-1
f) Elétron-volt (1 eV): 1,60218 x 10-19 J
g) Massa do elétron (me-): 9,11 x 10-31 kg
h) Massa do prótron (mp+): 1,67 x 10-27 kg
i) Massa do nêutron (mn): 1,67 x 10-27 kg
j) Aceleração da gravidade (g): 9,81 m.s-2
k) Volume molar (Vm): 22,4 L.mol-1
l) Carga eletrônica (e-): 1,60218 x 10-19 C
m) Valor do pi (π): 3,1415
n) Raio de Bohr: 5,29 x 10-18 J

ALGUNS FATORES DE CONVERSÃO IMPORTANTES COM BASE NO SI - VOLUME


a) 1 litro = 10-3 metros cúbicos
b) 1 litro = 1000 centímetros cúbicos
c) 1 mililitro = 10-3 litros
d) 1 litro = 1 decimetro cúbico
e) 1 litro = 1000 mililitro

ALGUNS FATORES DE CONVERSÃO IMPORTANTES COM BASE NO SI - PRESSÃO


a) 1 atmosfera = 760 mmHg
b) 1 atmosfera = 760 torr
c) 1 atmosfera = 101325 Pa
d) 1 atmosfera = 1,01325 bar
e) 1 atm = 14,70 psia (libras por polegada quadrada)
f) 1 torr = 1 mmHg

ALGUNS FATORES DE CONVERSÃO IMPORTANTES COM BASE NO SI - COMPRIMENTO


a) 1 quilômetro = 1000 metros
b) 1 metro = 100 cm
c) 1 centímetro = 10-2 metros
d) 1 metro = 39,37 polegadas
e) 1 jarda = 0,9144 metros
f) 1 milha = 1,609 quilômetros
g) 1 polegada = 2,54 centímetros
h) 1 mícron = 1,0 x 10-6 metros
i) 1 Ângstrom = 1,0 x 10-10 metros
j) 1 nanômetro = 1,0 x 10-9 metros

883
1600 FÍSICO-QUÍMICA APLICADA EXERCÍCIOS COMENTADOS - IME – ITA – OLIMPÍADA

k) 1 picômetro = 1,0 x 10-12 metros

ALGUNS FATORES DE CONVERSÃO IMPORTANTES COM BASE NO SI - MASSA


a) 1 quilograma = 1000 gramas
b) 1 grama = 10-3 quilogramas
c) 1 grama = 1000 miligramas
d) 1 miligrama = 10-3 gramas
e) 1 libra = 453,59 gramas
f) 1 tonelada = 1000 quilogramas
g) 1 tonelada = 106 gramas
h) 1u = 1,6606 x 10-24 gramas
i) 1 tonelada = 2000 libras

ALGUNS FATORES DE CONVERSÃO IMPORTANTES COM BASE NO SI - ENERGIA


a) 1 caloria = 4,18 J
b) 1 caloria = 4,13 x 10-2 atm.L
c) 1 Joule = 1,0 x 107 ergs
d) 1 elétron-volts = 1,6022 x 10-19 J
e) 1 elétron-volts = 96,485 kJ.mol-1
f) 1 atm.L = 101,325 J

APÊNDICE B – TABELA DE CONSTANTES QUÍMICAS

Zero Absoluto 0K - 273,15℃


Aceleração da gravidade g 9,81 m.s-2
Número de Avogadro N0 6,02 x 1023
Constante de Faraday F aproximadamente 96500 C.mol-1
Constante de Coulomb K 8,998 x 109Nm2/C2
Carga do Elétron q 1,602 x 10-19 C
Constante Gravitacional G 6,673 x 10-11 Nm2/kg2
Massa do Elétron me 9,109 x 10-31kg
Massa do Próton mp 1,673 x 10-27kg
Constante de Planck h 6,626 x 10-34 Js
Velocidade da luz no vácuo c 2,997 x 108 m/s

884
1600 FÍSICO-QUÍMICA APLICADA EXERCÍCIOS COMENTADOS - IME – ITA – OLIMPÍADA

APÊNDICE C – CONSTANTE DOS GASES

R = 0,08206 atm.L.mol-1.K-1

R = 0,08206 atm.dm³.mol-1.K-1

R = 82,06 atm.cm³.mol-1.K-1

R = 8,31451 Pa.m³.mol-1.K-1

R = 8,31451 kPa.m³.kmol-1.K-1

R = 8,314 J.mol-1.K-1

R = 1,98722 cal.mol-1.K-1

R = 1,987 Btu.lbmol-1.°R-1

R = 10,73 psia. ft³. lbmol-1.°R-1

R = 62,36 torr.L.mol-1.K-1

R = 62,36 mmHg.L.mol-1.K-1

R = 0,7302 ft³.atm.lbmol-1. °R-1

APÊNDICE D – CONSTANTE DE VAN DER WAALS (a e b)

Espécies a (atm.L².mol-2) b (10-2L.mol-1)

Argônio 1,363 3,219

Eteno 4,530 5,714

Etano 5,562 6,380

Benzeno 18,24 11,54

Metano 2,283 4,278

Cloro 6,579 5,622

Monóxido de carbono 1,505 3,985

Dióxido de carbono 3,640 4,267

Hidrogênio 0,2476 2,661

H2O 5,536 3,049

Sulfeto de hidrogênio 4,490 4,287

Hélio 0,03457 2,370

Criptônio 2,349 3,978

885
1600 FÍSICO-QUÍMICA APLICADA EXERCÍCIOS COMENTADOS - IME – ITA – OLIMPÍADA

Nitrogênio 1,408 3,913

Neônio 0,2135 1,709

Amônia 4,225 3,707

Oxigênio 1,378 3,183

Dióxido de enxofre 6,803 5,636

Xenônio 4,250 5,105

Fonte: Atkins, P. W.; Paula de, J. Físico-Química”, 8ªedição, volume um. Livros Técnicos e Científicos - LTC, Rio de
Janeiro, 2008.

APÊNDICE E – DISTINÇÃO ENTRE PILHA GALVÂNICA E ELETRÓLISE

Propriedades Pilhas Galvânicas Processo de Eletrólise


Espontaneidade Processo espontâneo Processo não espontâneo
Quanto ao uso de ponte salina Utiliza Não utiliza
Fluxo de elétrons Sai do ânodo para o cátodo Sai do ânodo para o cátodo
Quanto a Polaridade Ânodo = Polo negativo Ânodo = Polo positivo
Diferença de potencial Cátodo = polo positivo (E > 0) Cátodo = polo negativo (E < 0)

APÊNDICE F – TABELA DE POTENCIAL-PADRÃO (E° DE REDUÇÃO)

REAÇÃO DE REDUÇÃO E° (VOLTS)


Li+ −
(aq) + 1e → Li(s) -3,05
K+ −
(aq) + 1e → K (s) -2,93
Ba+2 −
(aq) + 2e → Ba (s) -2,90
+2
Sr(aq) −
+ 2e → Sr(s) -2,89
Ca+2
(aq)

+ 2e → Ca(s) -2,87
+
Na(aq) + 1e− → Na(s) -2,71
Mg +2
(aq) + 2e− → Mg (s) -2,37
Be+2
(aq)

+ 2e → Be(s) -1,85
Al+3
(aq) + 3e− → Al(s) -1,66
Mn+2 + 2e− → Mn(s)
(aq) -1,18

2H2 O(l) + 2e− → H2 (g) + 2OH(aq) -0,83
Zn+2
(aq) + 2e− → Zn(s) -0,76
+3
Cr(aq) + 3e− → Cr(s) -0,74
Fe+2
(aq)

+ 2e → Fe(s) -0,44
Cd+2
(aq) + 2e− → Cd(s) -0,40
PbSO4(s) + 2e− → Pb(s) + SO−2
4(aq) -0,31
Co+2
(aq)

+ 2e → Co(s) -0,28
Ni+2
(aq)

+ 2e → Ni(s) -0,25

886
1600 FÍSICO-QUÍMICA APLICADA EXERCÍCIOS COMENTADOS - IME – ITA – OLIMPÍADA

Sn+2 −
(aq) + 2e → Sn(s) -0,14
Pb+2 −
(aq) + 2e → Pb(s) -0,13
+
𝟐𝐇(𝐚𝐪) + 𝟐𝐞− → 𝐇𝟐(𝐠) 0,00
Sn+4 − +2
(aq) + 2e → Sn(aq) +0,13
Cu+2 − +
(aq) + 1e → Cu(aq) +0,15
SO−2 + −
4(aq) + 4H(aq) + 2e → SO2(g) + 2H2 O(l) +0,20
AgCl(s) + 1e− → Ag (s) + Cl− (aq) +0,22
+2 −
Cu(aq) + 2e → Cu(s) +0,34

O2(g) + 2H2 O(l) + 4e− → 4OH(aq) +0,40

I2(s) + 2e− → 2I(aq) +0,53
MnO−2 −
4(aq) + 2H2 O(l) + 3e → MnO2(s) + 4OH(aq)
− +0,59
+
O2(g) + 2H(aq) + 2e− → H2 O(aq) +0,68
Fe+3 − +2
(aq) + 1e → Fe(aq) +0,77
Ag + −
(aq) + 1e → Ag (s) +0,80
Hg +2 −
2(aq) + 2e → 2Hg (l) +0,85
2Hg +2 − +2
(aq) + 2e → Hg 2(aq) +0,92
+
NO− −
3(aq) + 4H(aq) + 3e → NO(g) + 2H2 O(l) +0,96

Br2(l) + 2e− → 2Br(aq) +1,07
+
O2(g) + 4 H(aq) + 4e− → 2H2 O(l) +1,23
MnO2(s) + 4H(aq) +
+ 2e− → Mn+2 (aq) + 2H2 O(l) +1,23
−2 +
Cr2 O7(aq) + 14H(aq) + 6e− → Cr(aq) +3
+ 7H2 O(l) +1,33


Cl2(g) + 2e → 2 Cl(aq) +1,36
Au+3 −
(aq) + 3e → Au(s) +1,50
MnO− + − +2
4(aq) + 8H(aq) + 5e → Mn(aq) + H2 O(l) +1,51
Ce+4 − +3
(aq) + 1e → Ce(aq) +1,61
+
H2 O2(g) + 2H(aq) + 2e− → 2 H2 O(l) +1,77
Co+3 − +2
(aq) + 1e → Co(aq) +1,82
+
O3(g) + 2H(aq) + 2e− → O2 + H2 O(l) +2,07

F2(g) + 2e− → 2 F(aq) +2,87

Fonte: Atkins, P. W.; Paula de, J. Físico-Química”, 8ªedição, volume um. Livros Técnicos e Científicos - LTC, Rio de
Janeiro, 2008.

Observação: Os valores dos potenciais-padrão foram determinados a partir das condições padrões, ou seja,
concentração da quantidade da matéria igual a um (1 mol.L-1), caso esteja estudando para fases gasosas, a pressão
é de 1 atm e temperatura igual a 25°C.

APÊNDICE G – CAPACIDADE CALORÍFICA MÉDIA À PRESSÃO CONSTANTE (𝑐𝑃 )

𝐽
SUBSTÂNCIA 𝑐𝑃 (𝑚𝑜𝑙 𝑥 𝐾 )

Al(s) 24,2

887
1600 FÍSICO-QUÍMICA APLICADA EXERCÍCIOS COMENTADOS - IME – ITA – OLIMPÍADA

Nb(s) 24,6

Nb(l) 41,7

Nb2O5(s) 131,6

Al2O3(s) 77,2

Al2O3(l) 144,9

Fe(s) 25,1

Fe(l) 41,4

Fe2O3(s) 103,7

Ar(g) 20,8

Cl2(g) 33,9

H2(g) 29,0

N2(g) 29,1

O2(g) 29,4

Fonte: Atkins, P. W.; Paula de, J. Físico-Química”, 8ªedição, volume um. Livros Técnicos e Científicos - LTC, Rio de
Janeiro, 2008.

888
1600 FÍSICO-QUÍMICA APLICADA EXERCÍCIOS COMENTADOS - IME – ITA – OLIMPÍADA

Apêndice H - TABELA DE CONSTANTES CRIOSCÓPICAS E EBULIOSCÓPICAS

SOLVENTE CONSTANTE CRIOSCÓPICA CONSTANTE EBULIOSCÓPICA

Água - 1,86 0,51


Benzeno - 5,12 2,53
Ácido acético - 3,90 2,93
Fenol - 7,40 3,04
Naftaleno - 6,94 5,80
Ácido fórmico - 2,77 -
Acetona - 1,71
Álcool comum 1,22 1,22
Cânfora - 40 -
Dissulfeto de carbono - 3,80 2,37
Tetracloreto de carbono - 30 4,95

APÊNDICE I – TABELA PERIÓDICA DOS ELEMENTOS QUÍMICOS

Fonte: http://iupac.org/cms/wp-content/uploads/2015/07/IUPAC_Periodic_Table-28Nov16.jpg (Acesso dia


01/06/2020)

889

Você também pode gostar